You are on page 1of 566

Incluso para a vida Biologia A

Pr-Vestibular da UFSC

1
UNIDADE 1

GENTICA MENDELIANA

BREVE HISTRICO SOBRE MENDEL

Em 1865, Johann Gregor Mendel, monge em um mos-
teiro na atual Repblica Checa, divulgou seus resulta-
dos a sociedade de naturalistas da cidade de Brno. Em
dez anos de trabalho, dedicou-se ao cruzamento de
plantas e observou aps vrias geraes as diferenas
surgidas. Mendel, no teve seus trabalhos reconheci-
dos, por 20 anos. A partir da, Mendel teve seus traba-
lhos reconhecidos. No entanto, os bons resultados
obtidos por Mendel foi devido a escolha de seu materi-
al biolgico, as ervilhas (Pisum sativum), o a qual
apresenta as seguintes caracterstica favorveis ao
estudo da gentica:
facilidade de cultivo;
caracteres marcantes;
ciclo vital curto;
Autofecundao natural; a estrutura da flor herma-
frodita no permite a entrada de plen, o que leva a
planta a ser "pura", isto , as caractersticas no variam
de uma gerao para outra.

CONCEITOS BSICOS DE GENTICA

GENE
um segmento de DNA responsvel pela determina-
o de um carter hereditrio (unidade de transmisso
hereditria).

CROMOSSOMOS HOMLOGOS
So cromossomos que apresentam a mesma forma, o
mesmo tamanho, a mesma posio do centrmero,
sendo um de origem materna e outro paterna.

LOCO/LOCUS
Espao fsico ocupado pelo gene no cromossomo (en-
dereo).

GENES ALELOS
So genes que determinam um mesmo carter. Eles se
situam no mesmo loco em cromossomos homlogos.

HOMOZIGOTO OU PURO
Indivduo que apresenta, no par de genes para certo
carter, dois alelos iguais, sendo um proveniente do pai
e outro da me. Exemplo: AA ou aa

HETEROZIGO OU HBRIDO
So pares de genes que determinam uma caracterstica,
mas apresentam manifestaes diferentes.Exemplo: Aa

DOMINANTE
So genes que se manifestam tanto em homozigose,
quanto em heterozigose. Estes tipos de genes, sempre
so simbolizados pela letra maiscula do alfabeto,
como, por exemplo: cor amarela das sementes das
ervilhas VV ou Vv.

RECESSIVO
Os genes recessivos so simbolizados pela letra mins-
cula do alfabeto e s manifestam-se quando esto em
homozigose, como, por exemplo: cor verde das semen-
tes das ervilhas vv.

FENTIPO
a aparncia de um indivduo, como: a cor dos cabe-
los, cor dos olhos, os grupos sanguneos so exemplos
de fentipos.

GENTIPO
O termo gentipo pode ser aplicado tanto ao conjunto
total de genes de um indivduo como a cada par de
genes em particular. Os filhos herdam dos pais o gen-
tipo que tem a potencialidade de expressar os respecti-
vos fentipos. Um gentipo pode expressar diferentes
fentipos, dependendo de sua interao com o meio.
Portanto: Gentipo + Meio = Fentipo.

1 LEI DE MENDEL
As caractersticas hereditrias so determinadas por
um par de fatores (genes), os quais se separam na for-
mao dos gametas.



CODOMINNCIA GNICA
So casos na gentica onde no ocorre dominncia de
um alelo sobre outro, sendo o fentipo resultante equi-
valente soma dos seus alelos (genes). Um caso cls-
sico de codominncia ocorre na determinao das
cores das ptalas das flores da espcie Mirabilis jalapa
(Planta Maravilha).





Please purchase PDF Split-Merge on www.verypdf.com to remove this watermark.
Incluso para a vida Biologia A

Pr-Vestibular da UFSC

2
A Planta maravilha apresenta trs cores:







Exerccios de Sala #

1. (UFSC) Considerando uma certa caracterstica
biolgica, determinada pelo par de genes alelos A e a,
sendo A dominante sobre a, podemos afirmar corre-
tamente que:
01. Dois indivduos, um com gentipo AA e outro com
gentipo Aa, tem fentipos iguais com relao a es-
te carter biolgico.
02. Do cruzamento Aa x Aa resultam descendentes de
dois gentipos.
04. Do cruzamento Aa x aa resultam descendentes de
dois fentipos, em propores iguais.
08. Os genitores de um indivduo aa podem ter fenti-
pos diferentes entre si.
16. Um indivduo com gentipo Aa produz dois tipos
de gametas, em propores iguais.

2. (UDESC) Em uma planta conhecida como maravi-
lha (Mirabilis jalapa), h trs tipos possveis de colo-
rao de ptalas das flores: branca, vermelha e rosa. O
cruzamento de plantas de flores brancas com plantas
de flores vermelhas resulta em uma descendncia com
todas as plantas com flores rosa (F1). Do cruzamento
dessa F1 obtm-se plantas dos trs tipos. Com base na
informao acima, assinale a alternativa correta.
a) As plantas de flores brancas e vermelhas so certa-
mente homozigotas.
b) As plantas de flores rosa podem ser homozigotas ou
heterozigotas.
c) Flores brancas, vermelhas e rosa correspondem ao
gentipo das plantas.
d) A proporo encontrada na descendncia da F1 de
1 planta de flor rosa, 2 plantas de flores brancas, 1
planta de flor vermelha.
e) Existem trs alelos envolvidos na segregao desse
carter, que exibem uma relao tpica de codomi-
nncia.

Tarefa Mnima #

3. (UNESP) Os vrios tipos de diabete so heredit-
rios, embora o distrbio possa aparecer em crianas
cujos pais so normais. Em algumas dessas formas, os
sintomas podem ser evitados por meio de injees
dirias de insulina. A administrao de insulina aos
diabticos evitar que eles tenham filhos com este
distrbio?
a) No, pois o gentipo dos filhos no alterado pela
insulina.
b) No, pois tanto o gentipo como o fentipo dos
filhos so alterados pela insulina.
c) Sim, pois a insulina incorporada nas clulas e ter
ao nos filhos.
d) Sim, pois a insulina incorporada no sangue fazen-
do com que os filhos no apresentem o distrbio.
e) Depende do tipo de diabete, pois nesses casos o
gentipo pode ser alterado evitando a manifestao
da doena nos filhos.

4. (UFSC) Um experimentador cruzou duas linhagens
puras de uma planta denominada boca-de-leo; uma
constituda de plantas com flores brancas e outra com
flores vermelhas. A descendncia originada (F1) apre-
sentou apenas plantas com flores cor-de-rosa. Da auto-
fecundao das plantas da F1, foram obtidas plantas
com flores exclusivamente brancas, vermelhas ou cor-
de-rosa.
Assinale a(s) proposio(es) verdadeira(s), conside-
rando que neste experimento:

01. Ocorreu a segregao de trs fentipos: o branco, o
vermelho e o cor-de-rosa.
02. A proporo genotpica esperada nas plantas de F2
: 1 planta com flores cor-de-rosa: 2 plantas bran-
cas: 1 planta vermelha.
04. As linhagens puras, que deram origem ao experi-
mento, certamente apresentam gentipos homozi-
gotos.
08. Os indivduos de F1 eram, certamente, heterozigo-
tos.G
16. A F2 esperada ser constituda de 50% de indiv-
duos homozigotos e 50% de indivduos heterozigo-
tos.

5. (PUC-PR) Quando duas populaes da espcie
vegetal 'Zea mays' (milho), uma homozigota para o
alelo dominante (AA) e uma homozigota para um alelo
recessivo (aa), so cruzadas, toda a descendncia da
primeira gerao (F1) assemelha-se ao tipo parental
dominante (Aa), embora seja heterozigota. Porm,
quando a gerao F1 se intercruza, a proporo fenot-
FENTIPOS GENTIPOS
BRANCO BB
VERMELHO VV
ROSA VB
Please purchase PDF Split-Merge on www.verypdf.com to remove this watermark.
Incluso para a vida Biologia A

Pr-Vestibular da UFSC

3
pica mendeliana 3:1 aparecer na gerao F2, pois os
gentipos sero:
a) 1/2 AA e 1/2 aa
b) 1/4 AA, 1/2 Aa e 1/4 aa
c) 1/3 AA e 1/4 aa
d) 1/4 Aa, 1/2 AA e 1/4 aa
e) impossvel determinar os gentipos utilizando os
dados acima.

UNIDADE 2

GENEALOGIAS OU HEREDOGRAMAS

a representao grfica atravs smbolos geomtricos
de uma famlia ou de vrias geraes.


Exemplo de genealogia:




2 LEI DE MENDEL
A primeira Lei de Mendel consiste no estudo da trans-
misso de uma caracterstica para os descendentes. No
diibridismo, Mendel analisou duas caractersticas dife-
rentes, simultaneamente, para saber se os "fatores"
(genes) de caractersticas diversas so independentes
um do outro. Cruzando-se ervilhas de sementes amare-
las e superfcie lisa com ervilhas de sementes verdes
com superfcie rugosa, Mendel obteve o seguinte resul-
tado:



Exerccios de Sala #

1. (UFSC) A sensibilidade gustativa ao PTC (Feniltio-
carbamida) uma caracterstica condicionada por um
gene autossmico em humanos. Considerando a genea-
logia abaixo e descartando a hiptese de mutao,
assinale a(s) proposio(es) verdadeiras.

01. O alelo que condiciona o fentipo sensvel domi-
nante sobre o alelo que condiciona o insensvel.
02. Os indivduos I - 1 e I - 2 so necessariamente
heterozigotos.
04. Os indivduos II - 2, II - 3 e III - 2 so necessaria-
mente homozigticos.
08. II - 5 no tem qualquer possibilidade de ser homo-
zigoto.
16. III - 1 no pode ser heterozigoto.
32. III - 2 e III - 3 tero a possibilidade de produzir um
descendente insensvel ao PTC somente se III 3
for heterozigoto.

2. (UNESP) O diagrama representa o padro de heran-
a de uma doena gentica que afeta uma determinada
espcie de animal silvestre, observado a partir de cru-
zamentos controlados realizados em cativeiro.
Please purchase PDF Split-Merge on www.verypdf.com to remove this watermark.
Incluso para a vida Biologia A

Pr-Vestibular da UFSC 4

A partir da anlise da ocorrncia da doena entre os
indivduos nascidos dos diferentes cruzamentos, foram
feitas as afirmaes seguintes.
I. Trata-se de uma doena autossmica recessiva.
II. Os indivduos I-1 e I-3 so obrigatoriamente homo-
zigotos dominantes.
III. No h nenhuma possibilidade de que um filhote
nascido do cruzamento entre os indivduos II-5 e II-
6 apresente a doena.
IV. O indivduo III-1 s deve ser cruzado com o indi-
vduo II-5, uma vez que so nulas as possibilidades
de que desse cruzamento resulte um filhote que a-
presente a doena.
verdadeiro o que se afirma em:
a) I, apenas.
b) II e III, apenas.
c) I, II e III, apenas.
d) I e IV, apenas.
e) III e IV, apenas.
Tarefa Mnima #
3. (MACKENZIE)

Se os indivduos 7 e 11 se casarem, a probabilidade
desse casal ter uma filha com o mesmo fentipo do av
materno de:
a) 1/2
b) 1/4
c) 1/8
d) 1/3
e) 2/3
4. (UFT) Na espcie humana existem vrias caracters-
ticas cuja herana provm de um par de alelos com
relao de dominncia completa. Na forma do lobo da
orelha o alelo dominante responsvel pelo lobo solto
e o alelo recessivo pelo lobo preso. A capacidade de
enrolar a lngua tambm determinada por um par de
alelos situados em outros cromossomos autossmicos,
onde o alelo dominante determina essa capacidade. A
probabilidade de nascer um descendente com o lobo da
orelha preso e a capacidade de enrolar a lngua de um
casal onde ambos so heterozigotos para as duas carac-
tersticas :
a) 12/16
b) 9/16
c) 4/16
d) 3/16
e) 1/16

5. (FATEC) Na espcie humana, a habilidade para o
uso da mo direita condicionada pelo gene dominan-
te E, sendo a habilidade para o uso da mo esquerda
devida a seu alelo recessivo e. A sensibilidade fenil-
tiocarbamida (PTC) condicionada pelo gene domi-
nante I, e a insensibilidade a essa substncia devida a
seu alelo recessivo i. Esses dois pares de alelos apre-
sentam segregao independente. Um homem canhoto
e sensvel ao PTC, cujo pai era insensvel, casa-se com
uma mulher destra, sensvel, cuja me era canhota e
insensvel. A probabilidade de esse casal vir a ter uma
criana canhota e sensvel ao PTC de:
a) 3/4.
b) 3/8.
c) 1/4.
d) 3/16.
e) 1/8.

UNIDADE 3

POLIALELIA OU ALELOS MLIPLOS

So casos estudados pela gentica em que constatam-
se a existncia de trs ou mais genes alelos envolvidos
na determinao de uma caracterstica. Um exemplo
deste fenmeno o que acontece com os genes que
condicionam a cor da pelagem dos coelhos, os quais
podem ser:

- C Selvagen, que sofreu mutaes, produzindo o gene
c
ch
chinchila, c
h
Himalaia e c
a
albino.

C> c
ch
> c
h
> c
a


SISTEMA ABO

uma caso de alelos mltiplos em humanos.
FENTIPOS GENTIPOS POSSVEIS
AGUTI / SELVAGEM CC,Cc
ch
,Cc
h
,Cc
a
CHINCHILA c
ch
c
ch
,c
ch
c
h
,c
ch
c
a
HIMALAIA c
h
c
h
,c
h
c
a

ALBINO c
a
c
a
TIPO AGLUTINOGNIOS AGLUTININAS
A A Anti-B
B B Anti-A
AB A e B No possui
O No possui Anti-A e Anti-B
Please purchase PDF Split-Merge on www.verypdf.com to remove this watermark.
Incluso para a vida Biologia A

Pr-Vestibular da UFSC 5
TRANSFUSES POSSVEIS

O Sistema ABO um caso de alelos mltiplos pois
existem trs alelos, os quais produzem o aglutinognio
A (I
A
), B (I
B
) e o que no produz ( i ), sendo que
este ltimo recessivo em relao aos dois primeiros,
que entre si no possuem dominncia, ou seja:
I
A
= I
B
> i
FENTIPOS GENTIPOS
A I
A
I
A
- I
A
i
B I
B
I
B
I
B
i
AB I
A
I
B

O ii
FATOR RH
O fator Rh um caso de dominncia simples, sendo
que o gene que determina a produo do aglutinog-
nio Rh (Rh+) dominante quando comparado com o
que impede (Rh-) a sua produo.
R > r

FENTIPO GENTIPO
Rh
+
RR ou Rr
Rh
-
rr

ERITROBLASTOSE FETAL OU DOENA HE-
MOLTICA DO RECM-NASCIDO (DHRN)
Na obstetrcia o conhecimento e a identificao do
fator Rh, permitiram elucidar o mecanismo de herana
de uma doena denominada DHRN (doena hemoltica
do recm-nascido), conhecida como eritroblastose
fetal. Essa enfermidade o resultado de uma incompa-
tibilidade entre a me e o feto, e ocorre quando a me
Rh
-
e se torna sensibilizada com o sangue Rh
+
do feto
da primeira gestao.

Em uma prxima gestao, a me passar para a circu-
lao do segundo feto os anticorpos denominados anti-
Rh, os quais atuaro destruindo as hemcias (hemli-
se). A hemoglobina liberada pela destruio das hem-
cias transformada em uma substncia de cor amare
lada denominada bilirrubina que em quantidade exces-
siva pode provocar graves sequelas (retardo mental,
paralisia e surdez, etc.).

SISTEM MN
Aps a descoberta do Sistema ABO, comeou-se a
investigar a existncia de outros antgenos que pudes-
sem caracterizar as hemcias humanas. Em 1927,
Landsteiner e Levine descobriram a existncia dos
anticorpos anti-M e anti-N em trabalhos de imunizao
de coelhos por hemcias humanas.
Grupo M ................... L
M
L
M
Grupo N .................... L
N
L
N
Grupo MN ................. L
M
L
N

o caso tpico de ausncia de dominncia na espcie
humana. Apesar da possibilidade de ocorrer reao
antgeno-anticorpo no sistema MN, sua importncia
em transfuses de sangue no to grande, a no ser
que estas sejam frequentes, situao em que a pessoa
fica sensibilizada.

Exerccios de Sala #

1. (UFSC) O padro de pelagem em coelhos condi-
cionado por uma srie allica, constituda por 4 alelos:
C - padro aguti - c
ch
- padro chinchila - c
h
- padro
Himalaia e c
a
- padro albino. O alelo C dominante
sobre todos os demais; o alelo Chinchila dominante
sobre o Himalaia e , finalmente, o Himalaia domi-
nante em relao ao albino.
Baseado nessas informaes, assinale a(s) proposi-
o(es) verdadeira(s):

01. A descendncia de um cruzamento entre os coelhos
aguti e chinchila poder ter indivduos aguti, chin-
chila e albino.
02. Do cruzamento entre indivduos com padro hima-
laia, podero surgir indivduos himalaia e albino.
04. O cruzamento entre coelhos albino originar, sem-
pre, indivduos fenotipicamente semelhantes aos
pais.
08. Coelhos aguti, chinchila e himalaia podero ser
homozigotos ou heterozigotos.
16. Todo coelho albino ser homozigoto.

2. (UFSC) A herana dos tipos sanguneos do siste-
ma ABO constitui um exemplo de alelos mltiplos
(polialelia) na espcie humana.
Com relao ao sistema ABO correto afirmar que:

01. O tipo O muito frequente e, por este motivo, o
alelo responsvel por sua expresso dominante
sobre os demais.
02. Os indivduos classificam-se em um dos quatro
gentipos possveis: grupo A, grupo B, grupo AB e
grupo O.
04. Existem trs alelos: o Ia, o Ib e o i.
08. Os alelos Ia e Ib so co-dominantes.
16. Se um indivduo do grupo A for heterozigoto, ele
produzir gametas portadores de Ia ou de i.
Please purchase PDF Split-Merge on www.verypdf.com to remove this watermark.
Incluso para a vida Biologia A

Pr-Vestibular da UFSC 6
32. Os indivduos de tipo sanguneo O possuem agluti-
nognios em suas hemcias, porm no possuem
aglutininas no plasma.
64. em alguns cruzamentos, entre indivduos do grupo
A com indivduos do grupo B, possvel nascerem
indivduos do grupo O.
Tarefa Mnima #
3. (UNESP) Em um acidente de carro, trs jovens
sofreram graves ferimentos e foram levados a um hos-
pital, onde foi constatada a necessidade de transfuso
de sangue devido a forte hemorragia nos trs acidenta-
dos. O hospital possua em seu estoque 1 litro de san-
gue do tipo AB, 4 litros do tipo B, 6 litros do tipo A e
10 litros do tipo O. Ao se fazer a tipagem sangunea
dos jovens, verificou-se que o sangue de Carlos era do
tipo O, o de Roberto do tipo AB e o de Marcos do tipo
A. Considerando apenas o sistema ABO, os jovens
para os quais havia maior e menor disponibilidade de
sangue em estoque eram, respectivamente:
a) Carlos e Marcos.
b) Marcos e Roberto.
c) Marcos e Carlos.
d) Roberto e Carlos.
e) Roberto e Marcos.
4. (UEPG) Os grupos sanguneos, que foram desco-
bertos h pouco mais de cem anos, so determinados
geneticamente como um carter mendeliano. A respei-
to dessa temtica, assinale o que for correto.
01. De acordo com o sistema de grupos sanguneos
ABO, so possveis oito gentipos diferentes.
02. Em relao ao sistema sanguneo ABO, no cruza-
mento A com B podem ocorrer descendentes sem
anticorpos (aglutininas) no plasma.
04. Nas transfuses de sangue, o aglutinognio presen-
te nas hemcias (antgeno) do doador deve ser com-
patvel com a aglutinina presente no plasma (anti-
corpo) do receptor.
08. Existem diferentes grupos sanguneos na espcie
humana, reunidos no sistema ABO. Quando gotas de
sangue de pessoas distintas so misturadas sobre
uma lmina de vidro, pode haver ou no aglutinao
das hemcias. A aglutinao caracterstica da rea-
o antgeno-anticorpo.
16. Um homem do grupo sanguneo AB e uma mulher
cujos avs paternos e maternos pertencem ao grupo
sanguneo O podero ter apenas filhos do grupo O.
5. (UFSC)
Com relao ao fenmeno descrito e suas consequn-
cias, correto afirmar que:
01. A me tem que ser Rh negativo.
02. O pai tem que ser Rh positivo.
04. A criana , obrigatoriamente, homozigota.
08. A me , obrigatoriamente, homozigota.
16. O pai pode ser heterozigoto.
32. A criana Rh negativo.
64. O pai pode ser homozigoto.

UNIDADE 4

ANEUPLOIDIAS HUMANAS

As aneuploidias so alteraes que envolvem a dimi-
nuio ou acrscimo de um ou mais cromossomos nas
clulas de um indivduo. Tais alteraes podem ocorrer
nos cromossomos sexuais ou nos autossomos.

EXEMPLOS


HERANA LIGADA AO SEXO
As heranas ligadas ao sexo, os genes esto localiza-
dos na regio no homloga do cromossomo X.

HEMOFILIA
Caracteriza-se pela ausncia de coagulao do sangue,
quando exposto ao ar. Entre os homens, a hemofilia
ocorre com uma incidncia de 1 : 10.000. (Nas mulhe-
res de 1: 100.000.000)
X
H
= Coagulao normal
Genes
X
h
= Hemofilia

X
H
> X
h


TIPO CARITIPO SEXO CARACT.

KLINEFEL-
TER

44A+XXY

M.
Estatura elevada;
Membros alongados;
Ginecomastia;
Esterilidade.

TURNER

44A+X0

F.
Pequena estatura;
Pescoo alado;
Trax largo;
Esterilidade.

PATAU

TRISS. DO 13

M. & F.
Deformidades faciais;
Polidactilia;
Malformao cerebral;
Retardamento mental .

EDWARDS

TRISS. DO 18

M. & F.
Deformidades faciais;
Orelhas baixas;
Dedos cerrados;
Retardamento mental .

DOWN

TRISS. DO 21

M. & F.
Feies orientais;
Lngua protusa;
Mos pequenas;
Hipotonia da mandbula;
Retardamento mental.

Please purchase PDF Split-Merge on www.verypdf.com to remove this watermark.
Incluso para a vida Biologia A

Pr-Vestibular da UFSC 7

DALTONISMO
uma anomalia caracterizada pela incapacidade de
distinguir duas ou mais cores. Foi descrita em 1798 por
John Dalton que era portador da mesma.
X
D
= viso normal
Genes
X
d
= daltonismo

X
D
> X
d


Exerccios de Sala #
1. (UFPR) Analisando a figura adiante, que representa
um caritipo humano, correto afirmar que se trata do
caritipo de um indivduo:

01. Do sexo masculino.
02. Do sexo feminino.
04. Com Sndrome de Down.
08. Com Sndrome de Patau.
16. Com Sndrome de Edwards.
32. Com caritipo normal.
64. Com uma anomalia numrica de autossomos.
2. (UFSC) As anomalias cromossmicas so bastante
frequentes na populao humana. Um exemplo disso
que aproximadamente uma a cada 600 crianas no
mundo nasce com sndrome de Down. Na grande mai-
oria dos casos, isso se deve presena de um cromos-
somo 21 extranumerrio. Quando bem assistidas, pes-
soas com sndrome de Down alcanam importantes
marcos no desenvolvimento e podem estudar, trabalhar
e ter uma vida semelhante a dos demais cidados.
Sobre as anomalias do nmero de cromossomos,
correto afirmar que:

01. Podem ocorrer tanto na espermatognese quanto na
ovulognese.
02. Ocorrem mais em meninas do que em meninos.
04. Ocorrem somente em filhos e filhas de mulheres de
idade avanada.
08. Esto intimamente ligadas separao incorreta
dos cromossomos na meiose.
16. Ocorrem ao acaso, devido a um erro na gametog-
nese.
32. Ocorrem preferencialmente em populaes de
menor renda, com menor escolaridade e pouca as-
sistncia mdica.
64. Podem acontecer devido a erros na duplicao do
DNA.

Tarefa Mnima #

3. (UFSC) Assinale a NICA proposio correta.
Em um indivduo daltnico, do sexo masculino, o gene
para o daltonismo encontra-se:
01. Em todas as clulas somticas.
02. Em todas as clulas gamticas
04. Apenas nas clulas do globo ocular.
08. Apenas nas clulas-me dos gametas.
16. Apenas nos gametas com cromossomo y.

4. (UFSC) Na espcie humana, o daltonismo uma
anomalia herdvel, relacionada com a viso das cores.
O gene para o daltonismo recessivo ligado ao cro-
mossomo X. correto afirmar, em relao ao dalto-
nismo.

01. Uma mulher daltnica deve ter pai e me daltni-
cos.
02. Uma mulher normal pode ser filha de pai daltnico
e me normal.
04. Um homem daltnico sempre tem pai tambm
daltnico.
08. Um homem normal pode transmitir o gene do dal-
tonismo para seus filhos homens.
16. Um homem daltnico pode ter me normal.

5. (UFSC) A hemofilia uma doena hereditria em
que h um retardo no tempo de coagulao do sangue,
e decorre do no funcionamento de um dos fatores
bioqumicos de coagulao.
Com relao a essa doena, assinale a(s) proposi-
o(es) correta(s).

01. No possvel a existncia de mulheres hemofli-
cas.
02. condicionada por um gene que se localiza no
cromossomo X, em uma regio sem homologia no
cromossomo Y.
04. Entre as mulheres, possvel encontrar um mxi-
mo de trs fentipos e dois gentipos.
08. Entre os homens, possvel ocorrer apenas um
gentipo; por isso, h uma maior incidncia dessa
doena entre eles.
Please purchase PDF Split-Merge on www.verypdf.com to remove this watermark.
Incluso para a vida Biologia A

Pr-Vestibular da UFSC

8
16. Entre os descendentes de um homem hemoflico e
de uma mulher normal, no portadora, espera-se que
50% deles sejam normais e 50% sejam hemoflicos.
32. um exemplo de herana ligada ao sexo, em que
os indivduos afetados tm graves hemorragias,
mesmo no caso de pequenos ferimentos.

UNIDADE 5

HISTOLOGIA ANIMAL

A histologia o ramo da cincia que estuda os tecidos,
os quais so definidos como sendo agrupamentos de
clulas que trabalham com a mesma finalidade. Os
tecidos humanos esto classificados em quatro grandes
grupos:
x Tecido epitelial;
x Tecido conjuntivo;
x Tecido muscular;
x Tecido nervoso.

TECIDOS EPITELIAIS
Os tecidos epiteliais surgem por diferenciao das
clulas da ectoderme e apresentam como funes, o
revestimento externo dos animais e das cavidades
internas de alguns rgos. Podem apresentar funo
secretora e de absoro de substncias, alm de capta-
rem estmulos do meio. As clulas que formam os
tecidos epiteliais se caracterizam por apresentarem:
clulas muito unidas (justapostas);
pouca ou nenhuma substncia intersticial (intercelu-
lar);
ausncia de vasos sanguneos;
pequena variabilidade celular.

TECIDO EPITELIAL DE REVESTIMENTO

Classificao:

Tipo
Forma da
clula
Exemplos Principais
Simples .
Pavimentoso Achatada
Paredes dos vrios vasos san-
guneos (endotlio), pleuras.
Cbico Cbica Ductos da maioria das glndulas
Colunar Colunar
Maior parte do trato digestrio,
vescula biliar.
Pseudoestratificado
Cavidade nasal, traqueia, brn-
quios e epiddimos.
Estratificado .
Pavimentoso Achatada Epiderme, boca e vagina.
Cbico Cbica
Ductos das glndulas sudorpa-
ras.
Colunar Colunar Conjuntiva do olho.
Transio Transitria
Revestimento dos clices renais,
pelve renal, ureter, bexiga e
parte da uretra.





TECIDO EPITELIAL GLANDULAR

Classificao das glndulas

Excrinas: Eliminam os produtos na superfcie do
epitlio atravs de dutos ou canais.
Ex: Sudorparas, mamrias, lacrimais, sebceas, sali-
vares.

Endcrinas: Eliminam os produtos que so coletados
pelos vasos sanguneos (hormnios), so desprovidas
de dutos ou canais.
Ex: Hipfise, tireide, paratireides, adrenais.

Anficrinas ou mistas: Apresentam determinadas regi-
es excrinas e outras endcrinas.

Ex: Pncreas: Suco pancretico & Insulina
Ovrios: vulos & Progesterona
Testculos: Espermatozides & Testosterona

Exerccios de Sala #

1. (UFSC) As glndulas podem ser classificadas como
endcrinas, que liberam seus produtos de sntese no
meio interno; excrinas, que liberam seus produtos de
sntese no meio externo, e glndulas mistas, que libe-
ram alguns de seus produtos de sntese no meio exter-
no e outros no meio interno. Associe as colunas, base-
ado no exposto.


1 endcrina
2 excrina
3 mista
I sebcea
II pncreas
III salivar
IV lacrimal
V suprarenal
VI mamria
VII tireide
Assinale a(s) proposio(es) correta(s).
01. 2 IV
02. 2 VII
04. 1 VI
08. 3 II
16. 2 III
32. 1 I
64. 1 V

2. (UFSC) Tecido epitelial, ou simplesmente epitlio,
aquele que reveste todas as superfcies internas ou
externas do corpo, alm de formar as glndulas. Com
relao a esse tecido, correto afirmar que:

01. Os epitlios de revestimento caracterizam-se por
apresentar clulas justapostas, de forma prismti-
ca, cbica ou achatada, praticamente sem material
intercelular.
02. Os epitlios de revestimento no so vasculariza-
dos, recebendo alimento por difuso a partir de
capilares existentes no tecido conjuntivo sobre o
qual repousa.
04. Os epitlios de revestimento conferem proteo
contra atritos e invaso de microorganismos, ser-
Please purchase PDF Split-Merge on www.verypdf.com to remove this watermark.
Incluso para a vida Biologia A

Pr-Vestibular da UFSC

9
vindo tambm para a absoro de alimento e oxi-
gnio.
08. Os epitlios glandulares apresentam clulas especi-
alizadas em produzir secrees e, no caso das
glndulas endcrinas, apresentam ductos por onde
seus produtos so eliminados para o exterior do
corpo.
16. A epiderme humana pluriestratificada e querati-
nizada e apresenta-se bastante espessa nas reas de
muito atrito, como a sola dos ps.
32. As clulas do epitlio intestinal apresentam clios
que auxiliam no movimento e deslocamento das
substncias que transitam pelo intestino.
64. Nas clulas do epitlio intestinal existe o comple-
xo unitivo, constitudo pela znula de ocluso, z-
nula de adeso e desmossomo, que funciona como
eficiente barreira passagem de substncias inde-
sejveis.

Tarefa Mnima #

3. (UFV) Com relao ao tecido epitelial, analise os
itens I, II e III e assinale a alternativa correta:
I. Possui clulas justapostas, com pouca ou nenhuma
substncia intercelular.
II. Desempenha as funes de proteo, revestimento e
secreo.
III. rico em vasos sanguneos, por onde chegam o
oxignio e os nutrientes para suas clulas.
a) Somente I e III so verdadeiros.
b) Somente II e III so verdadeiros.
c) Somente I e II so verdadeiros.
d) Somente um deles verdadeiro.
e) Todos so verdadeiros.

4. (PUC-PR) A propsito dos tecidos epiteliais,
correto afirmar:
a) Na pele, nas mucosas e nas membranas que envol-
vem os rgos do sistema nervoso, encontramos e-
pitlios de revestimento.
b) O tecido epitelial de revestimento caracteriza-se por
apresentar clulas separadas entre si por grande
quantidade de material intercelular.
c) As principais funes dos tecidos epiteliais so:
revestimento, absoro e sustentao.
d) A camada de revestimento mais interna dos vasos
sanguneos chamada de mesotlio.
e) Os epitlios so ricamente vascularizados no meio
da substncia intercelular.

5. (PUC-RJ) O tecido epitelial tem como funo fazer
o revestimento de todos os rgos do corpo. Neste
sentido, pode-se afirmar que:
a) ricamente vascularizado.
b) Suas clulas so anucleadas.
c) Suas clulas encontram-se justapostas.
d) Apresenta junes celulares como as sinapses.
e) Possui grande quantidade de substncia intercelular.


UNIDADE 6

TECIDOS CONJUNTIVOS

Caractersticas Gerais:
Origem embrionria mesodrmica;
Altamente vascularizado (menos cartilaginoso);
Grande quantidade de substncia intercelular (apre-
senta consistncia varivel, como por exemplo: gelati-
nosa, flexvel, rgida e lquida) ;
Clulas com grande diversidade morfolgica.

TECIDO CONJUNTIVO PROPRIAMENTE DITO
O tecido conjuntivo propriamente dito (TCPD) en-
contrado abaixo dos epitlios e envolvendo os rgos.
Este tipo de tecido envolve nervos, msculos, vasos
sanguneos e preenche os espaos entre dois rgos
diferentes alm de nutrir os tecidos que no possuem
vasos sanguneos, como por exemplo: o tecido epiteli-
al.

TECIDO CONJUNTIVO ADIPOSO
Este tipo de tecido conjuntivo caracteriza-se por apre-
sentar clulas (adipcitos) que armazenam lipdios
(gordura) em seus citoplasmas. A gordura armazenada
nos adipcitos encontram-se em constante renovao,
podendo atuar como reservatrio energtico, isolante
trmico e, tambm contra choques mecnicos.


TECIDO CONJUNTIVO CARTILAGINOSO
O tecido cartilaginoso tambm conhecido como carti-
lagem caracteriza-se pela presena de fibras colgenas
e elsticas. Estas duas fibras proporcionam ao tecido
cartilaginoso uma consistncia firme e flexvel, permi-
tindo sustentar diversas partes do corpo, proporcionan-
do, ao mesmo tempo, uma certa flexibilidade de mo-
vimento.

Tipos de Cartilagem:
Hialina: anis da traqueia, nariz, laringe, brn-
quios, extremidades de ossos (fibra colgena em
moderada quantidade);
Elstica: pavilho do ouvido, epiglote, laringe
(fibras elsticas predominantes e colgenas);
Fibrosa (fibrocartilagens): discos intervertebrais e
meniscos, ossos pubianos (fibra colgena abundan-
te).

TECIDO CONJUNTIVO SSEO
Tecido conjuntivo rgido e resistente que forma o
esqueleto da maioria dos vertebrados. A rigidez do
Please purchase PDF Split-Merge on www.verypdf.com to remove this watermark.
Incluso para a vida Biologia A

Pr-Vestibular da UFSC

10
tecido sseo resultado da interao entre o compo-
nente orgnico (fibras colgenas) e o componente
mineral (sais minerais) da substncia intercelular
(matriz ssea).

Tipos celulares:
Osteoblastos: clulas jovens, quando entram
em atividade, secretam a substncia interce-
lular (parte orgnica), quando adultas sero
denominadas ostecitos ;
Ostecitos: clulas adultas situadas no inte-
rior dos osteoplastos (lacunas). Mantm os
constituintes da matriz e metabolismo sseo;
Osteoclastos: permitem a regenerao do
osso e relacionam-se com a reabsoro da
matriz e a renovao ssea.

Exerccios de Sala #

1. (UFV) Das caractersticas a seguir, aquela que
comum a todos os tipos de tecido conjuntivo :
a) Possuir grande quantidade de substncia intercelu-
lar.
b) Apresentar grande quantidade de fibras elsticas.
c) Possuir substncia intercelular no estado lquido.
d) Apresentar calcificao ainda no perodo embrion-
rio.
e) Apresentar quantidades moderadas de fibras colge-
nas.

2. (PUC-RS) Algumas leses na pele deixam cicatrizes
bem visveis, que podem permanecer durante toda a
vida do indivduo. Qual dos tecidos a seguir o res-
ponsvel pelo processo de cicatrizao?
a) Cartilaginoso.
b) Conjuntivo.
c) Epitelial.
d) Muscular.
e) Nervoso.

Tarefa Mnima #

3. (MACK) A respeito do tecido cartilaginoso, cor-
reto afirmar que:
a) Apresenta vasos sanguneos para sua oxigenao.
b) Possui pouca substncia intercelular.
c) Aparece apenas nas articulaes.
d) Pode apresentar fibras proticas como o colgeno
entre suas clulas.
e) Se origina a partir do tecido sseo.

4. As fibras colgenas so constitudas de colgeno, a
protena mais abundante no nosso corpo, e conferem
resistncia ao tecido em que esto presentes. Esse
tecido o
a) nervoso.
b) conjuntivo.
c) epitelial.
d) muscular cardaco.
e) muscular esqueltico.

5. (UFSC) Considere o esquema a seguir e, aps, assi-
nale a(s) proposio (es) correta(s).

01. A um tipo de tecido muito resistente trao e
forma os tendes que fixam os msculos aos ossos.
02. B uma variedade de tecido conjuntivo denomina-
do sustentao.
04. C representa os msculos.
08. D constitudo por uma parte lquida, por elemen-
tos figurados e por clulas alongadas.
16. O tecido conjuntivo um tecido de conexo de
outros tecidos.

UNIDADE 7

TECIDO CONJUNTIVO HEMATOPOITICO
Tecido conjuntivo responsvel pela produo
dos elementos figurados (clulas) do sangue.

Tipos Ocorre Produo
Mieloide Medula ssea vermelha
Hemcias
Plaquetas
Leuccitos granul-
citos
Linfoide
Gnglios linfticos =
linfonodos
Timo, bao
Tonsilas, etc
Moncitos, linfci-
tos (leuccitos agra-
nulcitos)


Please purchase PDF Split-Merge on www.verypdf.com to remove this watermark.
Incluso para a vida Biologia A

Pr-Vestibular da UFSC

11
TECIDO CONJUNTIVO SANGUNEO
Tecido conjuntivo que apresenta como particularidade
o fato de possuir sua substncia intercelular em estado
lquido (plasma).

Constituio


Plasma: soluo amarelada e translcida;
gua
Sais
Protenas (Fibrinognio, Globulinas, Al-
bumina)

Elementos Figurados:

Hemcias
Anucleadas
Formadas na medula vermelha
Transportam CO2 e O2 ( r 5 mi-
lhes/mm3)
Leuccitos
Formados no bao, timo, linfonodos
e medula ssea.
Defesa atravs da fagocitose e da
produo de anticorpos (r 7 a 10.000
/ mm3)
Plaquetas ou trombcitos
Formadas na medula ssea
Coagulao sangunea (r
300.000/mm3)

COAGULAO SANGUNEA


Exerccios de Sala #

1. (CFT) Em relao ao sangue, incorreto dizer que:
a) Trata-se de um tecido conjuntivo.
b) Seu componente lquido denominado plasma.
c) Os trombcitos no fazem parte do contedo celular
do sangue.
d) Entre as clulas sangneas, destacam-se os eritrci-
tos e os leuccitos.
e) Os eritrcitos so, tambm, conhecidos como glbu-
los vermelhos.

2. (CFT-PR) Nosso corpo formado por quatrilhes
de clulas vivas que necessitam ao mesmo tempo de
gua, alimentos, ar, entre outras substncias. O sangue
o veculo que transporta as substncias necessrias
vida das clulas. Sobre as diferentes funes do sangue
correto afirmar que:
a) Os leuccitos transportam nutrientes e hormnios.
b) O plasma responsvel pelo transporte de oxignio.
c) As plaquetas ajudam na coagulao do sangue.
d) As hemcias so responsveis pela defesa do orga-
nismo.
e) Os glbulos vermelhos regulam a manuteno da
temperatura.

Tarefa Mnima #

3. (PUC-MG) Talvez voc j tenha feito exames de
sangue por solicitao mdica para saber como est
sua sade. Clulas sanguneas apresentam funes
especficas ou no, e a alterao na quantidade delas
nos indica determinados desequilbrios na sade. A
relao est incorreta em:
a) Menor quantidade de leuccitos leucemia.
b) Menor quantidade de plaquetas deficincia de
coagulao.
c) Menor quantidade de hemcias anemia.
d) maior quantidade de eosinfilos processo alrgi-
co.

4. (MACK) A respeito do tecido cartilaginoso, cor-
reto afirmar que:
a) Apresenta vasos sanguneos para sua oxigenao.
b) Possui pouca substncia intercelular.
c) Aparece apenas nas articulaes.
d) Pode apresentar fibras proticas como o colgeno
entre suas clulas.
e) Se origina a partir do tecido sseo.

5. (UDESC) Observe as trs afirmativas sobre o tecido
hematopoitico e o sangue:
I - O tecido hematopoitico possui como funo a
produo de clulas do sangue.
II - Os glbulos vermelhos so produzidos na medula
ssea e, posteriormente, passam para a corrente
sangunea.
III - Os anticorpos so produzidos pelas plaquetas.


Please purchase PDF Split-Merge on www.verypdf.com to remove this watermark.
Incluso para a vida Biologia A

Pr-Vestibular da UFSC

12
Assinale a alternativa correta.
a) I, II e III so verdadeiras.
b) I e II so verdadeiras.
c) II e III so verdadeiras.
d) I e III so verdadeiras.
e) apenas II verdadeira.
UNIDADE 8

TECIDOS MUSCULARES

Os tecidos musculares so responsveis pelo movi-
mento dos animais e pela contrao dos vrios rgos
que formam os organismos. Constitudo por clulas
alongadas (fibras musculares ou micitos) e especiali-
zadas em contrao, este tecido caracteriza-se por
apresentar filamentos contrteis de constituio protei-
ca, denominados de actina e miosina.

TECIDO MUSCULAR LISO
Est presente em alguns rgos internos (tubo
digestrio, bexiga, tero etc) e tambm na parede dos
vasos sanguneos (artrias). As fibras musculares lisas
so uninucleadas e os filamentos de actina e miosina
esto dispostos aleatoriamente, sem formar padro
estriado como nos demais tecidos musculares. A
contrao dos msculos lisos lenta e involuntria.

TECIDO MUSCULAR ESTRIADO CARDACO
A musculatura estriada cardaca ou miocrdio encon-
trado formando o corao. As clulas deste tipo de
musculatura so longas, ramificadas e com as mem-
branas intimamente unidas, atravs de estruturas espe-
ciais denominadas, discos intercalares. Estas estruturas
possuem como funo, aumentar a coeso entre as
clulas, permitindo que o estmulo necessrio contra-
o passe rapidamente de uma clula para outra. Alm
disso, a musculatura estriada cardaca apresenta con-
traes rpidas, ritmadas e involuntrias.



TECIDO MUSCULAR ESTRIADO ESQUELTI-
CO
Tecido que forma os msculos ligados estrutura
ssea, permitindo a movimentao do corpo. A mus-
culatura estriada esqueltica formada por clulas
cilndricas, multinucleadas com estrias longitudinais e
transversais a contrao voluntria, dependente da
vontade do indivduo.


CONTRAO MUSCULAR
As fibras musculares apresentam inmeras miofibrilas
contrteis, entre as quais, existem muitas mitocndrias
com funo energtica. As miofibrilas so constitudas
por dois tipos de protenas: a actina e a miosina, res-
ponsveis pela contrao dos msculos. Na musculatu-
ra estriada, as miofibrilas organizam-se em feixes,
conferindo a este tipo de musculatura uma caractersti-
ca estriada. Atravs da observao microscpica de um
msculo estriado, possvel verificar que as miofibri-
las apresentam, alternadamente, faixas claras e escu-
ras,. Ainda com o auxlio desta figura, observa-se que
as faixas claras so denominadas de faixas I e possuem
na regio central uma estria mais escura, conhecida
como estria Z. A regio compreendida entre duas
estrias Z recebe a denominao de sarcmero. Quando
o tecido muscular se contrai, os filamentos de actina
deslizam sobre os filamentos mais grossos de miosina.
Quando isso ocorre, a faixa I diminui de tamanho,
podendo, inclusive, desaparecer. Sendo assim, as estri-
as Z se aproximam, proporcionando um encurtamento
do sarcmero e, consequentemente, a contrao mus-
cular.


Exerccios de Sala #

1. (UEL) Considere os tipos de fibras musculares e as
aes a seguir:

I. cardaca
II. estriada
III. lisa

a) Contrao involuntria e lenta.
b) Contrao voluntria, em geral vigorosa.
c) Contrao involuntria e rpida.

Assinale a alternativa que associa corretamente os
tipos de fibras musculares com a sua respectiva ao.
Please purchase PDF Split-Merge on www.verypdf.com to remove this watermark.
Incluso para a vida Biologia A

Pr-Vestibular da UFSC

13
a) Ia, IIb, IIIc
b) Ia, IIc, IIIb
c) Ib, IIc, IIIa
d) Ic, IIa, IIIb
e) Ic, IIb, IIIa

2. (MACK) As afirmaes a seguir, referem-se aos
trs tipos de tecido muscular humano.
I - Todos apresentam as miofibrilas, que so estruturas
proticas com capacidade de contrao.
II - Como consequncia da contratilidade, esses tecidos
apresentam clulas com grande quantidade de mito-
cndrias.
III - Actina e miosina so as protenas responsveis
pela contrao desses tecidos, num processo que ne-
cessita da presena de ons clcio.

Assinale:
a) Se todas estiverem corretas.
b) Se apenas I e II estiverem corretas.
c) Se apenas I e III estiverem corretas.
d) Se apenas II e III estiverem corretas.
e) Se apenas III estiver correta.

Tarefa Mnima #

3. (UFPR) Com base nos estudos histolgicos, cor-
reto afirmar que:

01. A epiderme humana formada por tecido epitelial
de revestimento estratificado do tipo pavimentoso
queratinizado.
02. Os glbulos vermelhos do sangue humano so
anucleados.
04. As fibras musculares estriadas esquelticas so
muito pequenas, fusiformes e uninucleadas.
08. Os tendes so formados por tecido conjuntivo
denso.
16. O tecido cartilaginoso altamente irrigado por
vasos sanguneos.
32. O tecido sseo uma variedade de tecido conjunti-
vo em que a substncia intercelular apresenta ele-
vada quantidade de sais de clcio.

4. (UFV) Os msculos so responsveis por diversos
movimentos do corpo humano. Considerando que os
msculos podem ser diferenciados quanto funo que
exercem, assinale a alternativa incorreta:
a) O msculo cardaco se contrai a fim de bombear o
sangue para o corpo.
b) O diafragma o principal msculo respiratrio.
c) O movimento peristltico produzido pelo msculo
estriado.
d) O msculo estriado esqueltico tem controle volun-
trio.
e) O msculo cardaco tem controle involuntrio.

5. (UFV) Preocupados com a boa forma fsica, os
frequentadores de uma academia de ginstica discuti-
am sobre alguns aspectos da musculatura corporal.
Nessa discusso, as seguintes afirmativas foram feitas:

I - O tecido muscular estriado esqueltico constitui a
maior parte da musculatura do corpo humano.
II - O tecido muscular liso responsvel direto pelo
desenvolvimento dos glteos e coxas.
III - O tecido muscular estriado cardaco, por ser de
contrao involuntria, no se altera com o uso de
esterides anabolizantes.

Analisando as afirmativas, pode-se afirmar que:
a) Apenas II e III esto corretas.
b) Apenas I est correta.
c) Apenas II est correta.
d) I, II e III esto corretas.
e) Apenas I e II esto corretas.

UNIDADE 9

TECIDO NERVOSO

CLULAS DO TECIDO NERVOSO
Neurnios: So clulas grandes que apresentam um
corpo celular de onde partem dois tipos de prolonga-
mentos, os axnios e os dendritos. O corpo celular
possui um ncleo grande com nuclolo evidente. O
citoplasma apresenta grande nmero de mitocndrias e
o retculo rugoso bem desenvolvido, visvel ao mi-
croscpio como manchas denominadas Corpsculos de
Nissl. Atua recebendo estmulos de outros neurnios
durante a transmisso do impulso nervoso. Os dendri-
tos (gr. dendron = rvore) so ramificaes que possu-
em a funo de captar estmulos.O axnio (gr. axon =
eixo) o maior prolongamento do neurnio, cuja por-
o final ramificada. A funo do mesmo atuar na
transmisso dos estmulos nervosos.



Neurglias (Clulas da Glia): so clulas relacio-
nadas com a sustentao e nutrio dos neurnios,
produo de mielina e fagocitose. Existem trs tipos de
clulas:
- Astrcitos: maiores clulas da neurglia, com gran-
de nmero de ramificaes. Preenchem os locais lesa-
dos dos neurnios no processo de cicatrizao do teci-
do nervoso.
- Oligodentrcitos: so menores, com poucas e curtas
ramificaes, associadas ao corpo celular ou ao axnio
formando uma bainha de mielina.
- Micrglia: menores clulas da neurglia, com mui-
tas ramificaes curtas, com numerosas salincias.
Responsveis pela fagocitose no tecido nervoso.
Please purchase PDF Split-Merge on www.verypdf.com to remove this watermark.
Incluso para a vida Biologia A

Pr-Vestibular da UFSC

14


CONDUO DO IMPULSO NERVOSO
O impulso nervoso causado por um estmulo no
neurnio, provocando modificaes eltricas e qumi-
cas que so transmitidas ao longo dos neurnios sem-
pre no sentido: dendritocorpo celularaxnio. A
membrana do axnio em repouso apresenta carga el-
trica positiva do lado externo e a carga negativa do
lado interno; diz-se, ento, que o axnio est polariza-
do. Essa diferena mantida atravs da bomba de Na e
K. Ao receber um estmulo, a membrana do neurnio
torna-se mais permevel ao Na, invertendo-se as car-
gas ao redor da membrana.

SINAPSES NERVOSAS
So os locais onde as extremidades entre neurnios
vizinhos se encontram e os estmulos passam de um
neurnio para o seguinte por meio de substncias
qumicas especficas denominadas mediadores
qumicos ou neurotransmissores. O contato fsico entre
os neurnios no existe realmente.Os
neurotransmissores so liberados e migram atravs
(dentro de vesculas) do espao entre os mesmos
transmitindo assim o impulso nervoso de um neurnio
para o outro.




Exerccios de Sala #

1. (Cesgranrio) A observao do desenho a seguir nos
permite concluir que, na passagem do impulso nervoso
pelas sinapses, ocorre:

a) A liberao de mediadores qumicos ou de neuror-
mnios.
b) O contato direto do axnio de uma clula com os
dendritos de outra clula.
c) O fenmeno da bomba de sdio e potssio entre as
clulas.
d) A troca de cargas eltricas ao nvel das sinapses.
e) O envolvimento da bainha de mielina, que atua
como um isolante eltrico.

2. (PUC-MG) A sinapse :
a) Um tipo de fibra muscular envolvida no processo de
contrao cardaca.
b) Uma clula sangunea envolvida na liberao de
tromboplastina para o processo de coagulao.
c) Um tipo de reproduo sexuada, que envolve a for-
mao de gametas, realizada por protozorios cilia-
dos.
d) Uma regio de contato entre a extremidade do ax-
nio de um neurnio e a superfcie de outras clulas.
e) Um fenmeno que explica o fluxo de seiva bruta em
espermatfitas.

Tarefa Mnima #

3. (Cesgranrio)

Observando o esquema anterior, que representa um
neurnio em repouso, podemos afirmar que, nestas
condies:
a) Se a membrana do neurnio for atingida por um
estmulo, as quantidades de ons Na+ e K+ dentro e
fora da membrana se igualam.
b) Devido diferena de cargas entre as faces externa
e interna, o neurnio est polarizado.
c) A ocorrncia do impulso nervoso depende de est-
mulos de natureza eltrica.
d) A quantidade de ons K+ menor na parte interna
do neurnio devido sua sada por osmose.
e) as concentraes dos ons Na+ e K+ se fazem sem
gasto de energia, sendo exemplo de transporte ativo.
Please purchase PDF Split-Merge on www.verypdf.com to remove this watermark.
Incluso para a vida Biologia A

Pr-Vestibular da UFSC

15

4. (UFU) O esquema a seguir representa o reflexo
patelar, que uma resposta involuntria a um estmulo
sensorial.


Com relao a este reflexo, analise as afirmativas a
seguir.
I - Neste reflexo, participam apenas dois tipos de neu-
rnios: 1) o sensitivo, que leva o impulso at a me-
dula espinhal; 2) o motor, que traz o impulso medu-
lar at o msculo da coxa, fazendo-a contrair-se.
II - Em exame de reflexo patelar, ao bater-se com um
martelo no joelho, os axnios dos neurnios sensi-
tivos so excitados e, imediatamente, os dendritos
conduzem o impulso at medula espinhal.
III - Se a raiz ventral do nervo espinhal for seccionada
(veja em A), a pessoa sente a batida no joelho, mas
no move a perna.
Assinale a alternativa que apresenta somente afirmati-
vas corretas.
a) II e III
b) I e II
c) I e III
d) I, II e III

5. (UFPEL) O tecido nervoso um dos quatro tipos de
tecidos presentes no corpo humano, ele fundamental
na coordenao das funes dos diferentes rgos. As
clulas responsveis pelas suas funes so os neur-
nios (figura 1).


Com base nos textos e em seus conhecimentos, in-
correto afirmar que:

a) Geralmente o sentido da propagao do impulso
nervoso A para B, e por isso a estrutura 1 espe-
cializada na transmisso do impulso nervoso para
um outro neurnio ou para outros tipos celulares.
b) Tanto a estrutura representada pelo nmero 1 quanto
2 so ramificaes do neurnio, sendo que geral-
mente a 2 nica e mais longa.
c) A estrutura nmero 3 pode ser formada pela clula
de Schwann. Ela desempenha um papel protetor,
isolante e facilita a transmisso do impulso nervo-
so.
d) A estrutura nmero 4 est no centro metablico do
neurnio, onde tambm se encontra a maioria das
organelas celulares.
e) Considerando o sistema nervoso central, a regio
nmero 5 est presente na substncia cinzenta e au-
sente na branca.

UNIDADE 10

EMBRIOLOGIA

A Embriologia o ramo da cincia que estuda o de-
senvolvimento embrionrio dos animais desde a for-
mao da clula-ovo (zigoto) at o nascimento.

Fases do desenvolvimento embrionrio

Segmentao

Gastrulao

Organognese


FASE DE SEGMENTAO
Aps a fecundao, a clula-ovo ou zigoto sofre muitas
divises celulares denominadas clivagens resultando
na formao das primeiras clulas embrionrias cha-
madas de blastmeros. Aps algumas horas o embrio
apresenta a forma de uma amora: a mrula. Uma
vez formada a mrula, esta invadida por um lquido
que promove a migrao dos blastmeros para a peri-
feria, formando-se, a blstula ou blastocisto (no caso
dos mamferos). Esta estrutura embriolgica apresenta
uma cavidade central cheia de lquido denominada
blastocele e uma camada celular perifrica denominada
blastoderme.


FASE DE GASTRULAO
A gastrulao o processo no qual a blstula evolui
para gstrula. Este estgio embrionrio caracteriza-se
pela formao dos primeiros folhetos germinativos ou
embrionrios, a ectoderme e a endoderme. A gastru-
lao, inicia-se pela invaginao do polo vegetativo
para o interior da blastocele, originando dois folhetos
embrionrios, a ectoderme mais externa e a endoderme
Please purchase PDF Split-Merge on www.verypdf.com to remove this watermark.
Incluso para a vida Biologia A

Pr-Vestibular da UFSC

16
mais interna, revestindo uma cavidade denominada
arquntero ou intestino primitivo. O arquntero se
comunica com meio externo atravs de uma abertura
denominada blastporo. Este blastporo poder futu-
ramente originar a boca, sendo os grupos animais de-
nominados de protostmios. Entretanto, se o blastpo-
ro originar o nus, os grupos animais sero classifica-
dos como deuterostmios (ex: equinodermos e corda-
dos).
A gstrula, que a princpio didrmica ou di-
blstica, evolui para uma estrutura tridrmica ou tri-
blstica denominada nurula. Durante a formao da
nurula ocorre a formao do terceiro folheto: a meso-
derme. Na maioria dos animais, a mesoderme forma
uma cavidade denominada celoma.
Enquanto a mesoderme se diferencia, a ecto-
derme forma na regio superior da gstrula, um apro-
fundamento, o qual dar origem ao tubo neural. Nessa
etapa o teto do arquntero inicia um processo de multi-
plicao celular que dar origem ao eixo de sustenta-
o do embrio: a notocorda.
















ORGANOGNESE

ECTODERME

MESODERME

ENDODERME

x a epiderme e
seus anexos
(exemplo: plos,
cabelos e unhas)
x o revestimento
bucal, nasal,
anal e o esmalte
dos dentes
x o sistema
nervoso (cre-
bro, medula,
nervos e gn-
glios nervosos)
xo celoma
xa derme (camada
situada abaixo da
pele)
xos msculos
estriados, carda-
cos e lisos
xo sistema circu-
latrio (corao,
vasos sanguneos
e sangue)
xo esqueleto
(crnio, coluna
vertebral e ossos
dos membros)
xo sistema uroge-
nital (rins, bexiga,
uretra, gnadas e
dutos genitais
xo tubo diges-
trio, com
exceo da
mucosa bucal
e anal, as
quais so de
origem ecto-
drmica.
xas glndulas
anexas do
sistema diges-
trio, como o
fgado e o
pncreas.
xo revestimen-
to do sistema
respiratrio e
da bexiga
urinria

ANEXOS EMBRIONRIOS
So estruturas que se desenvolvem junto ao
embrio sendo fundamentais para o seu desenvolvi-
mento. So eles:
Saco ou Vescula Vitelnica: Estrutura que possui
a funo de armazenar substncias nutritivas (vitelo)
que sero consumidas pelo embrio . Nos mamferos
placentrios tal estrutura apenas vestigial.
mnio ou Bolsa Amnitica: Membrana que forma
uma bolsa contendo em seu interior o liquido amniti-
co o qual protege o embrio contra choques mecnicos
e tambm evita a sua desidratao.
Alantoide: Este anexo se apresenta especialmente
bem desenvolvido nos animais ovparos.O mesmo
responsvel pelas trocas gasosas entre o embrio e o
meio externo,pela transferncia de Clcio da casca
para o esqueleto do embrio e tambm promove a
eliminao de excretas (Ac.rico).Nos mamferos,
devido a presena da placenta, o alantoide um anexo
muito pouco desenvolvido.
Crion: O Crion uma fina membrana que
cobre o embrio e os demais anexos embrionrios. Tal
estrutura possui funo protetora e nos mamferos
tambm responsvel pela fixao da placenta (vilosi-
dades corinicas) na parede uterina.
Placenta: A placenta um anexo embrionrio
exclusivo dos mamferos e apresenta vrias funes,
como:
* Transferncia de nutrientes;
* Passagem de anticorpos;
* Trocas gasosas;
* Remoo de excretas;
Please purchase PDF Split-Merge on www.verypdf.com to remove this watermark.
Incluso para a vida Biologia A

Pr-Vestibular da UFSC

17
* Produo e passagem de hormnios.
Pela placenta, temos a circulao de sangue ma-
terno e fetal, porm convm salientar, que os mesmos
no se misturam. Ao final dos meses de gestao, com
queda da produo de progesterona por parte da pla-
centa tem incio as contraes uterinas e o trabalho de
parto.
Cordo Umbilical: Anexo que une o feto
placenta.No interior do cordo umbilical existem vasos
sanguneos (2 artrias e 1 veia) por onde uma srie de
substncias so transportadas.

Exerccios de Sala #

1. (UFSC) Baseado nos esquemas abaixo, que corres-
pondem a diferentes fases do desenvolvimento embri-
onrio de um animal, assinale a(s) proposio(es)
verdadeiras(s).

01. A ordem correta em que as fases ocorrem durante o
processo de desenvolvimento c - d - b - a.
02. Em a, j podemos observar a presena dos trs
folhetos embrionrios.
04. A fase representada em b denomina-se gstrula.
08. Em c, temos representadas quatro clulas denomi-
nadas blastmeros.
16. O mesoderma j est presente em d.
32. Os esquemas apresentados referem-se ao desenvol-
vimento embrionrio de um cordado.

2. (UNIFESP) O tratamento da leucemia por meio dos
transplantes de medula ssea tem por princpio a trans-
ferncia de clulas-tronco da medula de um indivduo
sadio para o indivduo afetado. Tal procedimento fun-
damenta-se no fato de que essas clulas tronco:
a) Podem ser usadas para a clonagem de clulas sadias
do paciente;
b) No sero afetadas pela doena, j que foram dife-
renciadas em outra pessoa;
c) Secretam substncias que inibem o crescimento
celular;
d) Podem dar origem a linfcitos T que, por sua vez,
ingerem os leuccitos em excesso;
e) Podem dar origem a todos os diferentes tipos de
clulas sanguneas.

Tarefa Mnima #

3. (UEPG) A respeito do desenvolvimento embrion-
rio, assinale o que for correto.

01. As divises que ocorrem durante a segmentao
denominam-se clivagens, e as clulas que se for-
mam so chamadas mrulas.
02. Na gastrulao, forma-se o blastporo. Os animais
em que o blastporo d origem ao nus so chama-
dos de protostmios, e os animais em que o blast-
poro d origem boca so chamados de deuteros-
tmios.
04. Ao longo do desenvolvimento embrionrio, as
clulas passam por um processo de diferenciao
celular em que alguns genes so "ativados", pas-
sando a coordenar as funes celulares. Surgem
dessa maneira os tipos celulares, que se organizam
em tecidos.
08. De um modo geral, em praticamente todos os ani-
mais podem ser observadas trs fases consecutivas
de desenvolvimento embrionrio: segmentao,
gastrulao e organognese.
16. Na organognese ocorre diferenciao dos rgos a
partir dos folhetos embrionrios formados logo a-
ps a gastrulao.

4. (UFSC) Pesquisas recentes revelam que a cocana
atravessa a barreira placentria, indo afetar o desen-
volvimento normal de bebs. A droga ataca, princi-
palmente, o sistema nervoso provocando, posterior-
mente, dificuldades na aprendizagem e na integrao
da criana com os pais e com o meio. A placenta um
dos anexos embrionrios presentes em vertebrados.

Sobre esses anexos correto afirmar que:

01. A vescula vitelnica possui funo de armazenar
substncias nutritivas (vitelo).
02. O mnio atenua abalos e traumatismos, sofridos
pela me, que possam atingir o embrio.
04. O alantide possui, exclusivamente, funo prote-
tora.
08. O cordo umbilical liga o feto a placenta.
16. A placenta, dentre outras funes, responsvel
pela nutrio e serve como barreira contra infec-
es.

Please purchase PDF Split-Merge on www.verypdf.com to remove this watermark.
Incluso para a vida Biologia A

Pr-Vestibular da UFSC

18
5. (UFSC) A figura a seguir mostra o corte transversal
de um embrio e anexos embrionrios.

Em relao figura, correto afirmar que:
01. A seta 1 indica o principal local de produo da
gonadotrofina corinica, hormnio que quando es-
t presente na urina sinal inequvoco de gravi-
dez.
02. A seta 2 indica a bolsa amnitica, que tem por
funo hidratar e proteger o feto contra eventuais
choques mecnicos.
04. A figura representa um embrio de mamfero.
08. A figura representa o embrio de uma ave.
16. A placenta (indicada pela seta 3) responsvel pela
intensa troca de substncias entre me e filho. Es-
ta troca ocorre porque h passagem do sangue da
me para o filho e vice-versa.
32. A figura difere da representao de um embrio de
rpteis por possuir mnio e crio, anexos inexis-
tentes nesses animais.

UNIDADE 11

FISIOLOGIA HUMANA

SISTEMA DIGESTRIO
O sistema digestrio humano constitudo
pelos seguintes rgos: boca, faringe, esfago, estma-
go, intestino e nus.


DIGESTO NA BOCA
Na boca, o alimento sofre inicialmente a ao mecni-
ca, ou seja, mastigado e misturado saliva que
produzida pelas glndulas salivares (partidas, subma-
xilares e sublinguais). A saliva uma secreo consti-
tuda, principalmente, por gua, substncias bacterici-
das e por enzimas, como a ptialina (amilase salivar),
que atua na digesto do amido, convertendo-o em
molculas menores (maltoses). Aps a mastigao o
alimento deglutido. Na deglutio, o alimento passa
para o esfago e atravs de fortes contraes (involun-
trias) da musculatura (lisa), o bolo alimentar alcana,
atravs dos movimentos peristlticos, a abertura do
estmago.



Peristaltismo

DIGESTO NO ESTMAGO
No estmago, o bolo alimentar misturado ao cido
clordrico, que alm de atuar na funo anti-sptica,
responsvel pela converso do pepsinognio (enzima
inativa) em pepsina (enzima ativa). A pepsina uma
protease sendo portanto responsvel pela quebra das
ligaes qumicas existentes entre os aminocidos que
formam as protenas. A produo de suco gstrico
(cido clordrico e enzimas) estimulada pelo horm-
nio gastrina, o qual secretado pelas clulas localiza-
das na parte final do estmago. A massa cida e pasto-
sa formada no interior do estmago recebe o nome de
quimo.



Please purchase PDF Split-Merge on www.verypdf.com to remove this watermark.
Incluso para a vida Biologia A

Pr-Vestibular da UFSC 19
DIGESTO NO DUODENO
O quimo passa para o intestino delgado que mede
cerca de 6 metros de comprimento e ao longo do seu
interior ocorre a principal parte da digesto e da absor-
o do alimento pelo organismo. O intestino delgado
dividido em duas regies, o duodeno e o jejuno-leo. O
duodeno possui cerca de 25 centmetros e forma a
parte inicial do intestino delgado. No duodeno so
lanadas as secrees provenientes do fgado e do
pncreas.O Pncreas secreta o suco pancretico que
alcalino (pH entre 7,5 e 8,8) que, junto com a bile,
neutralizam a acidez do quimo. O suco pancretico
possui algumas enzimas como tripsina e quimiotripsi-
na, as quais continuam o processo digestivo das prote-
nas iniciado no estmago. O suco pancretico tambm
apresenta outras enzimas como: a amilase pancretica,
que atua a quebra de molculas de amido, as nucleases,
que fragmentam cidos nuclicos (DNA e RNA) e a
lpase pancretica, que metaboliza molculas de gordu-
ra. importante salientar que a bile produzida no
fgado e armazenada na vescula biliar, no dotada de
enzimas digestivas, apenas sais biliares, que atuam na
emulso das gorduras, ou seja, transformam placas
de gordura em pequenas gotculas facilitando a ao da
lpase. As clulas localizadas na parede intestinal
secretam o suco entrico (intestinal). Este apresenta as
seguintes enzimas:
maltase, a que converte a maltose em glicose;
lactose: converte a lactose em glicose e galactose;
lpase entrica: converte gorduras em cidos gra-
xos, glicerol e monoglicerdeos;
peptidases: convertendo peptdeos em aminoci-
dos.



O FIM DA DIGESTO
As molculas de glicose, aminocidos, nucleotdeos e
cidos graxos so absorvidos pela parede intestinal,
que apresenta uma srie de dobras, denominadas mi-
crovilosidades, que aumentam consideravelmente a
superfcie de contato com os diversos nutrientes, favo-
recendo a absoro.


O INTESTINO GROSSO
O intestino grosso um tubo muscular medindo cerca
de 1,5 metro de comprimento e 7 centmetros de di-
metro.O intestino grosso dividido, didaticamente, em
trs partes: ceco, clon e reto. O clon (parte mais
longa) sub-dividido em clon ascendente (que sobe
em direo ao fgado), clon descendente (desce pelo
lado esquerdo) e clon transverso (que atravessa a
parte superior do abdome). No intestino grosso ocorre
a reabsoro da gua e sais minerais alm da formao
das fezes. As fezes so formadas por gua e restos no
digeridos de alimento, os quais atravs do reto alcan-
am o nus e so eliminadas para o meio externo (e-
gesto).



Exerccios de Sala #

1. (UFSC)Os seres vivos necessitam de um supri-
mento de energia capaz de manter sua atividade meta-
blica. Essa energia extrada dos alimentos, que
podem ser produzidos pelos prprios organismos, no
caso dos auttrofos, ou obtidos a partir de uma fonte
orgnica externa, no caso dos hetertrofos. As substn-
cias orgnicas, tais como protenas, carboidratos e
lipdios, devem ser desdobradas em compostos mais
simples e mais solveis, de tal maneira que possam ser
assimiladas pelo organismo. A esse processo de trans-
formao dos alimentos em compostos relativamente
mais simples, absorvveis e utilizveis denominamos
digesto.
W. R. Paulino. Biologia Atual, Ed. tica, 1996. p. 296.

Com relao a esse assunto, assinale a(s) proposi-
o(es) verdadeira(s).
01. A mastigao, a deglutio e os movimentos peris-
tlticos constituem a digesto qumica.
Please purchase PDF Split-Merge on www.verypdf.com to remove this watermark.
Incluso para a vida Biologia A

Pr-Vestibular da UFSC

20
02. A gua e os sais minerais so absorvidos, pelo
tubo digestivo, sem transformao qumica.
04. A digesto do amido rpida e ocorre em dois
momentos: na boca, pela ao da amilase salivar e
no estmago, sob a ao das peptidases.
08. A bile no tem enzimas, mas apresenta sais bilia-
res, que emulsificam os lipdios, transformando-os
em gotculas menores que facilitam a digesto das
gorduras
16. Os nutrientes digeridos so absorvidos principal-
mente no intestino delgado, onde as clulas epiteli-
ais das vilosidades apresentam expanses digitifor-
mes as microvilosidades , que aumentam, consi-
deravelmente, a superfcie de absoro dos nutrien-
tes.
32. Pessoas, que tiveram sua vescula biliar extirpada,
no apresentam dificuldade em digerir lipdios e,
por isso, podem fazer uma dieta rica em gorduras.

2. (PUC-RJ) As condies de acidez dos sucos presen-
tes no sistema digestrio humano variam de acordo
com as diferentes partes do tubo digestrio. Assim em
relao ao pH podemos afirmar que:
a) Na boca cido e l ocorre principalmente a diges-
to de amido.
b) Na boca neutro e l ocorre principalmente a diges-
to de gordura.
c) No estmago cido e l ocorre principalmente a
digesto de protenas.
d) No intestino neutro e l no ocorre nenhum tipo de
digesto enzimtica.
e) No estmago bsico e l ocorre principalmente a
digesto de protenas.

Tarefa Mnima #

3. (PUC-MG) A figura a seguir representa alguns
rgos do trato digestivo.

Assinale a afirmativa incorreta.

a) I uma glndula que transforma e acumula metab-
litos e tambm neutraliza substncias txicas.
b) II responsvel por armazenar e concentrar a bile e
secret-la quando necessrio.
c) III responsvel pela digesto do alimento, secreo
de hormnios e absoro de gua e sais.
d) IV uma glndula que secreta enzimas digestivas e
hormnios que controlam a glicemia.

4. (FUVEST) Ao comermos um sanduche de po,
manteiga e bife, a digesto do
a) bife inicia-se na boca, a do po, no estmago, sendo
papel do fgado produzir a bile que facilita a digesto
das gorduras da manteiga.
b) bife inicia-se na boca, a do po, no estmago, sendo
papel do fgado produzir a bile, que contm enzimas
que digerem gorduras da manteiga.
c) po inicia-se na boca, a do bife, no estmago, sendo
papel do fgado produzir a bile que facilita a digesto
das gorduras da manteiga.
d) po inicia-se na boca, a do bife, no estmago, sendo
papel do fgado produzir a bile, que contm enzimas
que completam a digesto do po, do bife e das gor-
duras da manteiga.
e) po e a do bife iniciam-se no estmago, sendo as
gorduras da manteiga digeridas pela bile produzida
no fgado.

5. (UDESC) O alimento, no sistema digestrio huma-
no, percorre os seguintes rgos antes de chegar ao
intestino delgado:
a) Faringe - laringe - diafragma estmago.
b) Boca - faringe - esfago estmago.
c) Boca - traquia - fgado - intestino grosso.
d) Faringe - esfago - pncreas fgado.
e) Esfago - vescula biliar - fgado estmago.

UNIDADE 12

SISTEMA RESPIRATRIO
Os rgos que formam o sistema respiratrio so: as
fossas nasais, faringe, laringe, traqueia, brnquios,
bronquolos e alvolos pulmonares, onde ocorrem as
trocas gasosas.


Nas fossas nasais ocorrem muitos pelos curtos, estes
tem como funo reter partculas em suspenso do ar e
tambm microrganismos. Alm disso, as fossas nasais
apresentam um grande nmero de vasos sanguneos o
que proporciona um aquecimento do ar. Da regio das
fossas nasais, o ar vai para a faringe, que tambm
comum ao sistema digestrio. Da faringe o ar segue
Please purchase PDF Split-Merge on www.verypdf.com to remove this watermark.
Incluso para a vida Biologia A

Pr-Vestibular da UFSC

21
para traqueia. Na regio anterior da traqueia fica a
laringe, que possui as pregas vocais. A traqueia bifur-
ca-se e forma os brnquios. Estes ramificam-se in-
meras vezes no interior dos pulmes e assumem um
aspecto semelhantes aos galhos de uma rvore. A rami-
ficao dos brnquios origina os bronquolos, os quais
terminaro em uma diminuta estrutura em forma de
cacho denominada alvolo pulmonar.

VENTILAO PULMONAR
A entrada de ar pelas vias respiratrias deve-se a con-
trao do msculo diafragma e dos msculos intercos-
tais. O diafragma ao se contrair, juntamente com os
intercostais, provoca um aumento de volume da caixa
torcica, fazendo com que a presso interna diminua,
tornando-se menor que a presso do ar atmosfrico.
esta diferena que faz com que o ar penetre nos pul-
mes. Na expirao, ocorre exatamente o inverso, ou
seja, a musculatura envolvida relaxa e isso provoca a
reduo do volume torcico. Com isso, o ar sai dos
pulmes.


TRANSPORTE GASOSO E HEMATOSE
O mecanismo de transporte dos gases ocorre nos al-
volos pulmonares, onde a concentrao de oxignio
superior a dos capilares sanguneos que envolvem os
mesmos. Pelo mecanismo de difuso, o oxignio passa
dos alvolos para o sangue, onde uma pequena parcela
fica dissolvida no plasma, mas a maior parte entra nos
glbulos vermelhos, indo se combinar com as molcu-
las de hemoglobina. Dessa maneira, combinado com a
hemoglobina, o oxignio transportado aos tecidos.
Nos tecidos, o oxignio passa do sangue para as clu-
las. Essa difuso ocorre porque a concentrao de
oxignio no interior da clula reduzida, devido ao
contnuo consumo desse gs no processo de respirao
celular. A respirao celular alm de consumir oxig-
nio tambm produz gs carbnico que transferido
das clulas para o sangue. Ao entrar nos glbulos ver-
melhos o gs carbnico reage com a gua e, com a
participao de uma enzima (anidrase carbnica),
transforma-se em cido carbnico. Em seguida, o cido
carbnico dissocia-se em ons bicarbonato, que so
transferidos para o plasma por difuso. A maior parte
do gs carbnico transportado, portanto, na forma on
bicarbonato, dissolvido no plasma, mas convm salien-
tar, que a hemoglobina tambm transporta uma certa
parcela (cerca de 15% - 23%) e, tambm, encontramos
CO2 diretamente dissolvido no plasma (cerca de 7%).

Exerccios de Sala #

1. (UFSC) O esquema a seguir apresenta um modelo
simplificado de nosso sistema respiratrio.

Assinale a(s) proposio(es) correta(s) sobre o mes-
mo e suas relaes com os demais sistemas orgnicos.
01. Separadas pelo palato ("cu da boca"), as fossas
nasais e a boca servem de entrada para o ar inspi-
rado.
02. A traqueia um tubo formado por anis osteocarti-
laginosos que lhe do rigidez e boa sustentao.
04. A hematose ocorre nos alvolos, com a troca do
oxignio atmosfrico pelo gs carbnico sangu-
neo.
08. Pessoas portadoras de fenda palatina produzem
sons anasalados pois, quando falam, o ar sai tanto
pela boca como pelo nariz.
16. O esquema apresenta apenas o pulmo direito visto
ser ele o principal, tendo o esquerdo funo se-
cundria.
32. Em caso de obstruo das vias areas (engasgo)
por balas ou outros objetos estranhos, em especial
se ocorrer nos brnquios, deve-se bater nas costas
da pessoa engasgada para expulsar o objeto estra-
nho.

2. (PUC-PR) Nos seres humanos a oxigenao do
sangue ocorre, com maior intensidade, ao nvel dos
alvolos pulmonares, no interior dos pulmes.
Os movimentos respiratrios que facilitam a entrada do
ar nos pulmes e, consequentemente, a sua sada, ocor-
rem pela ao
a) da traquia e dos brnquios.
b) do diafragma e da pleura.
c) do mediastino e dos msculos peitorais.
d) dos msculos intercostais e do diafragma.
e) da faringe e da laringe.

Tarefa Mnima #

3. (UFV) Observe o esquema representado abaixo, de
parte do sistema respiratrio humano, e assinale a
alternativa incorreta.
Please purchase PDF Split-Merge on www.verypdf.com to remove this watermark.
Incluso para a vida Biologia A

Pr-Vestibular da UFSC

22

a) O ar chega aos pulmes pelo esfago, indicado por
I.
b) O diafragma, indicado por V, auxilia nos movimen-
tos respiratrios.
c) Os pulmes e brnquios esto indicados por III e IV,
respectivamente.
d) Embora no esteja indicada, a laringe se localiza
acima da traqueia.
e) Os bronquolos, indicados por II, conduzem ar aos
alvolos.

4. (PUC-RIO) A respirao a troca de gases do
organismo com o ambiente. Nela o ar entra e sai dos
pulmes graas contrao do diafragma. Considere
as seguintes etapas do processo respiratrio no homem:
I. Durante a inspirao, o diafragma se contrai e desce
aumentando o volume da caixa torcica.
II. Quando a presso interna na caixa torcica diminui
e se torna menor que a presso do ar atmosfrico, o
ar penetra nos pulmes.
III. Durante a expirao, o volume torcico aumenta, e
a presso interna se torna menor que a presso do
ar atmosfrico.
IV. Quando o diafragma relaxa, ele reduz o volume
torcico e empurra o ar usado para fora dos pul-
mes.
Assinale as opes corretas:
a) I e II.
b) II, III e IV.
c) I, II e III.
d) I, II e IV.
e) Todas.

5. (FUVEST) Assinale a alternativa que indica o com-
portamento da caixa torcica, dos msculos intercos-
tais e do diafragma durante a expirao humana.
a) A caixa torcica aumenta de volume, os msculos
intercostais contraem-se e o diafragma abaixa.
b) A caixa torcica aumenta de volume, os msculos
intercostais contraem-se e o diafragma levanta.
c) A caixa torcica diminui de volume, os msculos
intercostais contraem-se e o diafragma levanta.
d) A caixa torcica diminui de volume, os msculos
intercostais relaxam-se e o diafragma levanta.
e) A caixa torcica diminui de volume, os msculos
intercostais relaxam-se e o diafragma abaixa.




UNIDADE 13

SISTEMA CIRCULATRIO
CORAO HUMANO
Localizado no centro da caixa torcica e atrs do osso
esterno, o corao humano constitudo por quatro
cmaras: dois trios (esquerdo e direito) e dois ventr-
culos (tambm esquerdo e direito) que se contraem e
relaxam de forma rtmica, impulsionando o sangue
para todas as partes do corpo. A contrao cardaca
recebe a denominao de sstole e o relaxamento de
distole.
Os dois trios no possuem comunicao entre si e
localizam-se na regio superior do corao. Recebem
sangue trazido ao corao pelas veias. O trio esquerdo
comunica-se apenas com o ventrculo esquerdo e o
trio direito com o ventrculo direito. Esta comunica-
o entre o trio e o ventrculo ocorre por meio de
orifcios protegidos por vlvulas. No lado direito, loca-
liza-se a vlvula tricspide e no lado esquerdo a vlvu-
la bicspide ou mitral.


PEQUENA E GRANDE CIRCULAO
A circulao humana do tipo dupla, ou seja, o sangue
passa duas vezes pelo corao. Essas duas passagens
determinam a diviso da circulao em dois tipos:
grande (sistmica) e pequena (pulmonar) circulao. O
sangue arterial sai do ventrculo esquerdo pela artria
aorta e se ramifica pelo corpo. Estas ramificaes tor-
nam-se cada vez menores e mais finas, formando as
arterolas e, finalmente, os capilares sanguneos. nos
capilares que ocorrem as trocas gasosas, de nutrientes e
excretas. Neste momento, o gs carbnico e as excretas
saem das clulas e penetram no sangue, transformando
o sangue arterial em sangue venoso. No retorno as
ramificaes dos capilares unem-se formando vasos
denominados vnulas e veias. As veias iro transportar
o sangue venoso at o trio direito.
O sangue venoso penetra no trio direito e
passa para o ventrculo direito. Em seguida, o sangue
bombeado e, atravs da artria pulmonar, chega aos
pulmes, onde ocorrer a troca gasosa. O sangue veno-
so rico em gs carbnico recebe oxignio dos alvolos,
transformando-se em sangue arterial. A circulao que
transporta sangue venoso aos pulmes e retorna com
Please purchase PDF Split-Merge on www.verypdf.com to remove this watermark.
Incluso para a vida Biologia A

Pr-Vestibular da UFSC

23
sangue arterial ao corao denominada pequena cir-
culao ou circulao pulmonar.


SISTEMA EXCRETOR

OS RINS
A maior parte da excreo realizada pelos rins. Os
dois rins esto localizados na regio dorsal da cavidade
abdominal, um de cada lado da coluna vertebral. Estes
rgos so responsveis pela eliminao de ureia e
tambm pelo controle da concentrao de gua e sais
no corpo. De cada rim parte um ureter, estes canais
transportam a urina at bexiga urinria, a urina ser
lanada para o exterior atravs da uretra.



OS NFRONS
Os rins so abastecidos com sangue atravs das artrias
renais, as quais se ramificam muitas vezes em seu
interior, formando inmeras arterolas. Cada arterola
alcana um nfron, que a unidade excretora do rim.
Cada nfron composto de duas partes: o corpsculo
de renal e os tbulos renais. O corpsculo de renal
constitudo por um emaranhado de pequenos vasos
provenientes das ramificaes da arterola aferente,
denominado glomrulo renal, que, por sua vez, encon-
tra-se envolvido por uma cpsula chamada, cpsula
renal. Esta cpsula proveniente da dilatao do tbu-
lo renal, que se caracteriza por apresentar um longo
tbulo contorcido denominado tbulo proximal, que,
por sua vez, desemboca numa estrutura em forma de
U, denominada ala nfrica, a partir da qual se distende
o tbulo distal. A unio de vrios tbulos distais, de
vrios nfrons, forma um tbulo coletor.



A FORMAO DA URINA
Durante a etapa de filtrao, a presso sangunea ex-
pulsa, do glomrulo renal para a cpsula renal, subs-
tncias como: gua, pequenas molculas de sais mine-
rais, aminocidos, vitaminas, cidos graxos, ureia e
cido rico. No entanto, muitas dessas substncias so
teis ao organismo e devem ser reabsorvidas pelo
sangue enquanto outras, como a ureia, devero ser
expulsas para o meio externo atravs da urina. Sendo
assim, o filtrado produzido na cpsula renal passa para
o interior do tbulo contorcido proximal, onde ter
incio o processo de reabsoro. A arterola eferente
que surge, a partir da cpsula renal, tem como funo
capturar os nutrientes desejveis ao organismo que se
encontram no filtrado. Nesta regio cerca de 85% da
gua presente no filtrado retorna ao sangue atravs dos
capilares por osmose. As partculas de soluto (glicose,
aminocidos e os ons sdio) retornam aos capilares
sanguneos atravs do mecanismo de transporte ativo
com gasto de energia. Ao longo da ala nfrica, mais
gua ser reabsorvida por osmose e ons sdio por
transporte ativo e no tbulo distal, ocorre novamente
reabsoro ativa dos sais. A permeabilidade dos tbu-
los renais gua regulada pelo hormnio antidiurti-
co (ADH), que produzido pelo hipotlamo e lanado
no sangue pela neuro-hipfise. Sendo assim, o rim, que
controla a concentrao de gua e de sais minerais,
rgo muito importante para o bem-estar do orga-
nismo. Ao sair do tbulo coletor, a composio da
urina : 95% de gua; 2% de ureia; 1% de cloreto de
sdio e 2% de outros sais e produtos nitrogenados,
como o cido rico, a amnia e a creatina.

Please purchase PDF Split-Merge on www.verypdf.com to remove this watermark.
Incluso para a vida Biologia A

Pr-Vestibular da UFSC

24


Exerccios de Sala #

1. (UFSC) Segundo o ministrio da Sade, o corao
a primeira causa de morte no Pas, logo em seguida
est a violncia (homicdio, suicdio, acidente de
trnsito) e o cncer.
Com relao ao sistema cardiovascular assinale a(s)
proposio(es) verdadeira(s).
01. Os principais vasos responsveis pela irrigao do
msculo cardaco so as artrias coronrias ligadas
aorta.
02. O infarto do miocrdio ocorre quando uma parte da
musculatura cardaca, por ficar sem irrigao, faz o
msculo entrar em falncia.
04. A hipertenso, o diabetes, o fumo e a obesidade so
fatores de risco para doenas cardiovasculares.
08. Alimentao adequada, bem como, atividade fsica
e check-up regulares diminuem o risco do infarto.
16. A contrao do msculo cardaco denominado
sstole e o perodo de relaxamento, distole.
32. Nas pessoas hipertensas o corao trabalha mais,
j que precisa impulsionar o sangue atravs de va-
sos endurecidos e, por isso, mais resistentes.

2. (UFSC) Com relao ao Sistema Cardiovascular e
com base no esquema a seguir, cujas setas indicam o
trajeto do sangue no corpo, assinale a(s) proposi-
o(es) correta(s).

01. As cavidades I e II representam os ventrculos e as
cavidades III e IV representam os trios (ou aur-
culas).
02. O sangue que leva o oxignio para as clulas mus-
culares do corao (miocrdio) atravs das artrias
coronrias impulsionado pela cavidade IV.
04. Os vasos sangneos representados pelas setas B e
C correspondem s veias e os vasos sangneos
representados pelas setas A e D correspondem s
artrias.
08. O trajeto que o sangue faz da cavidade III at a
cavidade II corresponde circulao sistmica,
tambm chamada grande circulao.
16. Nas cavidades I e III circula sangue arterial, ao
passo que nas cavidades II e IV circula sangue ve-
noso.
32. Quando as cavidades III e IV esto em distole, as
cavidades I e II esto em sstole, e vice-versa.
64. Entre as cavidades I e II localiza-se a vlvula bi-
cspide (ou mitral) e entre as cavidades III e IV
localiza-se a vlvula tricspide.

Tarefa Mnima #

3. (MACK) Um estudante observou que um determi-
nado vaso sanguneo apresentava paredes espessas e
que o sangue que circulava em seu interior era de um
vermelho escuro. Podemos afirmar corretamente que
o vaso em questo era a:
a) Veia pulmonar, que leva sangue venoso do corao
para o pulmo.
b) Veia cava, que traz sangue venoso do corpo em
direo ao corao.
c) Veia pulmonar, que leva sangue arterial do pulmo
para o corao.
d) artria pulmonar, que leva sangue venoso do cora-
o para o pulmo.
e) artria pulmonar, que leva sangue arterial do pulmo
para o corao.

4. (UEL) A ingesto de lcool inibe a liberao de
ADH (hormnio antidiurtico) pela hipfise. Assim
sendo, espera-se que um homem alcoolizado:
a) Produza grande quantidade de urina concentrada.
b) Produza grande quantidade de urina diluda.
c) Produza pequena quantidade de urina concentrada.
d) Produza pequena quantidade de urina diluda.
e) Cesse completamente a produo de urina.

5. (MACK) O esquema adiante, representa o aparelho
excretor humano. As setas A e B indicam o sentido do
fluxo sanguneo.

Os nmeros 1, 2 e 3 indicam, respectivamente:
Please purchase PDF Split-Merge on www.verypdf.com to remove this watermark.
Incluso para a vida Biologia A

Pr-Vestibular da UFSC

25
a) Artria aorta, ureter e veia cava.
b) Veia cava, ureter e artria aorta.
c) Veia cava, uretra e artria aorta.
d) Artria aorta, uretra e veia cava.
e) Artria aorta, uretra e veia porta.

UNIDADE 14

SISTEMA ENDCRINO
As glndulas que formam o sistema endcrino so: a
hipfise, a tireoide, as paratireoides, o pncreas, as
adrenais, alm dos ovrios e testculos.


A HIPFISE
Pequena glndula localizada no interior da caixa crani-
ana (base do crebro) est anatomicamente dividida em
trs regies: adeno-hipfise, neuro-hipfise e lobo
intermedirio.


ADENO-HIPFISE
Controlada por uma regio do crebro, denominada
hipotlamo, a adeno-hipfise libera no sangue uma
srie de hormnios. Estes hormnios secretados pela
adeno-hipfise recebem a denominao de hormnios
trficos, e tm como funo, controlar as outras gln-
dulas. So eles:

- HORMNIO TIREOTRFICO (TSH) - Esti-
mula a tireide;
- HORMNIO ADRENOCORTICOTRFICO
(ACTH) - Controla o crtex das supra-renais;
- PROLACTINA (LTH) - Estimula a produo e a
secreo de leite;
- HORMNIO FOLCULO-ESTIMULANTE
(FSH) - Provoca o crescimento dos folculos nos
ovrios e a formao de espermatozides nos test-
culos;
- HORMNIO LUTEINIZANTE (LH) - Respon-
svel pela ovulao, pela formao do corpo lteo
nos ovrios e a produo de testosterona nos test-
culos.

Obs: A adeno-hipfise tambm secreta o HORM-
NIO DO CRESCIMENTO, denominado somatotr-
fico ou GH. Este hormnio tem como funo, aumen-
tar a estatura dos jovens em fase de desenvolvimento.
A deficincia desse hormnio provoca o nanismo, j a
hiperfuno, provoca o gigantismo.

NEURO-HIPFISE
A neuro-hipfise uma expanso anatmica do pr-
prio hipotlamo e os hormnios que esta glndula
secreta: ocitocina e antidiurtico (ADH), so produzi-
dos pelos neurnios localizados nessa regio especfica
do crebro. A ocitocina, responsvel pelas contraes
uterinas durante o parto, este hormnio tambm atua
na liberao do leite durante a amamentao, atravs
da estimulao da musculatura que expulsam o lquido
quando o beb suga o seio.
O hormnio antidiurtico nos tbulos renais,
aumentando a permeabilidade gua, provocando,
assim, uma maior reabsoro desse lquido e contro-
lando a quantidade de urina eliminada.

LOBO INTERMEDIRIO
Esta regio da hipfise responsvel pela secreo do
hormnio melanotrfico, que estimula a produo de
melanina. No homem, o lobo intermedirio da hipfise
muito reduzido.

A TIREOIDE
Esta glndula, localizada no pescoo, responsvel
pela secreo dos hormnios tetraiodotironi-
na(tiroxina) e triiodotironina. Ambos possuem tomos
de Iodo em suas molculas. A tireoide controla o me-
tabolismo geral do organismo.A hiperfuno da tireoi-
de causa o hipertireoidismo (glndula funciona acima
do nvel normal), Os indivduos portadores de hiperti-
reoidismo, em geral, so nervosos, tensos, os batimen-
tos cardacos so acelerados, apresentam intolerncia
ao calor, transpirao excessiva e insnia, entre outros
sintomas. A hipofuno da tireoide causa o hipotireoi-
dismo (glndula funciona abaixo do nvel normal). Os
indivduos portadores de hipotireoidismo, em geral,
so apticos, sonolentos, apresentam batidas cardacas
fracas e, algumas vezes, inchao em vrias partes do
corpo. Caso este quadro ocorra na infncia, surge uma
deficincia mental, denominada cretinismo. Esta pato-
logia pode ser eliminada pela adio de pequenas
quantidades de iodo ao sal de cozinha. Alm disso, a
tireoide ainda secreta pequenas quantidades de calcito-
nina, que participa no controle do clcio no organismo.

Please purchase PDF Split-Merge on www.verypdf.com to remove this watermark.
Incluso para a vida Biologia A

Pr-Vestibular da UFSC 26

AS PARATIREOIDES
Tambm localizadas no pescoo encontram-se em
nmero de quatro. Estas glndulas produzem o horm-
nio paratormnio (PTH), que responsvel pelo con-
trole da taxa de clcio no organismo. Quando ocorre
uma reduo na concentrao de clcio no sangue, este
hormnio passa a promover a retirada de clcio dos
ossos, lanando, em seguida, no sangue. Alm disso,
estimula a absoro de clcio no intestino e a reabsor-
o pelos tbulos renais. A calcitonina, secretada pela
tireoide, atua de forma oposta. Sendo assim, estes dois
hormnios, ajudam a controlar a taxa de clcio no
sangue, que importante para diversas funes do
organismo, como por exemplo: a coagulao sangunea
e a contrao muscular.
O PNCREAS
Localizado no abdmen, prximo ao estmago, o pn-
creas apresenta uma regio excrina, produtora de suco
pancretico e uma endcrina, representada por cente-
nas de milhares de pequenos grupos de clulas, as
chamadas Ilhotas de Langerhans. Estas clulas que
formam as Ilhotas de Langerhans produzem dois hor-
mnios:
Clulas ALFA: produzem o hormnio glucagon
(atua no desdobramento do glicognio em glicose);
Clulas BETA: Responsveis pela produo do
hormnio insulina (atua na membrana celular, tornan-
do-as permeveis glicose),

AS ADRENAIS
Estas glndulas esto localizadas na parte superior de
cada rim e se caracterizam por apresentar duas regies
distintas: o crtex e a medula.
O crtex a regio mais superficial da glndula e
responsvel pela produo dos seguintes hormnios:
Cortisol: Produz glicose a partir de protenas e
gorduras, alm de diminuir o consumo de glicose nas
clulas;
Aldosterona; Promove um aumento da reabsoro
de sdio nos tbulos renais e, conseqentemente, de
cloro e gua.
Corticossexuais: Compreendem os andrgenos,
com efeitos masculinizantes e pequenas quantidades de
progesterona e estrgenos (hormnios femininos), os
exercem efeitos pouco significativos na fisiologia
normal.
A medula a regio mais interna da glndula e
responsvel pela produo dos seguintes hormnios:
Adrenalina e Noradrenalina: Em situaes de
perigo, a medula estimulada por uma parte do siste-
ma nervoso, e passa a liberar uma grande de horm-
nios, principalmente adrenalina. Como resultado, ,
aumenta o ritmo respiratrio e circulatrio, alm da
elevao da presso arterial. Estes eventos, entre ou-
tros, proporcionados pela adrenalina e noradrenalina,
faz com que o organismo consiga enfrentar situaes
de crticas e perigosas.


Exerccios de Sala #

1. (PUC-PR) A produo do hormnio luteinizante
estimula as clulas intersticiais ou de Leydig a liberar
um hormnio que, por sua vez, responsvel pela
manuteno dos caracteres sexuais. Assinale a opo
que corresponde ao descrito no texto:
a) A hipfise produz o hormnio luteinizante e estimu-
la o testculo a produzir testosterona.
b) O testculo produz hormnio luteinizante e estimula
a hipfise a produzir o estrgeno.
c) O hormnio luteinizante estimula o testculo a pro-
duzir o estrgeno, estimulando a hipfise.
d) O hormnio luteinizante estimula o ovrio a produ-
zir a progesterona, estimulando a hipfise.
e) O hipotlamo produz o hormnio luteinizante esti-
mulando a hipfise a produzir testosterona.

2. (PUC-Campinas) Considere a frase a seguir.
"Filhotes de ces e gatos, alimentados exclusivamente
com carne crua, podem apresentar desequilbrio nutri-
cional havendo comprometimento da estrutura ssea."
Essa frase desencadeou uma discusso da qual resulta-
ram as seguintes afirmaes:
I. A carne crua apresenta um nvel muito baixo de
clcio o qual inibe o funcionamento das glndulas
paratireides.
II. As paratireides secretam paratormnio que provo-
ca a retirada de clcio dos ossos.
Please purchase PDF Split-Merge on www.verypdf.com to remove this watermark.
Incluso para a vida Biologia A

Pr-Vestibular da UFSC

27
III. As paratireides podem ser inibidas de liberarem
paratormnio, quando o sangue apresenta elevado
nvel de clcio circulante.
correto o que se afirma somente em:
a) I
b) II
c) III
d) I e II
e) II e III

Tarefa Mnima #

3. (UNESP) Um paciente adulto procurou um endo-
crinologista porque estava com baixo peso, metabolis-
mo basal muito alto, nervosismo e globo ocular salien-
te (exoftalmia). A disfuno hormonal que poderia ser
responsvel pelo quadro apresentado pelo paciente
envolve
a) o pncreas.
b) a paratireide.
c) a adrenal.
d) a tiride.
e) a supra-renal.

4. (UFPE) A associao entre adrenalina (epinefrina) e
as emoes tornou-se to popular que este hormnio
passou a ser sinnimo de esportes radicais, situaes
de risco e sentimentos fortes. Identifique abaixo as
propriedades da adrenalina.
a) Mobiliza as reservas energticas, de sorte a baixar
os nveis de glicose na corrente sangunea.
b) Aumenta os batimentos cardacos e diminui os mo-
vimentos respiratrios.
c) secretado pelo crtex da glndula adrenal e pelas
terminaes do sistema nervoso simptico.
d) Reduz o dimetro dos brnquios pelo relaxamento
de sua musculatura.
e) Aumenta a presso arterial sistlica.

5. (UFPR) Numa situao de perigo, um animal fica
em estado de alerta (defesa). O comportamento apre-
sentado depende de uma srie de reaes que envol-
vem diversos sistemas orgnicos. Com relao a esse
estado, correto afirmar que:
01. O sistema nervoso ter papel decisivo no preparo
do animal para enfrentar o perigo ou realizar a fu-
ga.
02. Haver liberao de adrenalina, ocorrendo aumento
da presso arterial e maior irrigao dos msculos e
do crebro.
04. A frequncia respiratria aumentar, pois o animal
necessitar de mais oxignio para o seu metabolis-
mo.
08. A reao imediata do animal frente ao perigo de-
pender diretamente do sistema linftico.
16. A frequncia cardaca aumentar para melhorar a
irrigao sangunea dos tecidos.



UNIDADE 15

SISTEMA NERVOSO
Anatomicamente, o sistema nervoso dos vertebrados
pode ser dividido em:

SISTEMA NERVOSO CENTRAL (SNC): cons-
titudo pelo encfalo e pela medula raquidiana ou espi-
nhal;

SISTEMA NERVOSO PERIFRICO (SNP):
constitudo por uma rede de nervos que se espalham ao
longo de todo organismo.


O ENCFALO
Esta parte do sistema nervoso constituda por quatro
regies, denominadas: crebro, cerebelo, ponte e
bulbo.

Crebro: centro da inteligncia, da memria e da
linguagem. A massa cerebral ocupa quase toda caixa
craniana, na sua regio mais externa (crtex cerebral)
apresenta uma srie de dobras, as quais aumentam
consideravelmente a rea superficial do crebro. O
crtex formado por milhares de corpos celulares de
neurnios, o que acaba conferindo a esta regio, uma
cor cinzenta. J a sua camada inferior formada pelos
prolongamentos dos neurnios (axnios),e apresenta
uma cor branca.

Cerebelo: rgo que atua em conjunto com o c-
rebro, coordenando os movimentos do corpo, o equil-
brio e o tnus muscular.

Ponte: estrutura situada acima do bulbo e abaixo
do crebro. A ponte atua na conduo do impulso
nervoso para o cerebelo, e tambm serve como uma
Please purchase PDF Split-Merge on www.verypdf.com to remove this watermark.
Incluso para a vida Biologia A

Pr-Vestibular da UFSC

28
rea de passagem para as fibras nervosas que ligam o
crebro medula.

Bulbo: estrutura situada acima da medula espinhal,
o bulbo recebe informaes de muitos dos nossos r-
gos internos, controlando suas funes, como, por
exemplo: o batimento cardaco, a presso sangunea, a
respirao, salivao, tosse, o ato de engolir, piscar dos
olhos, etc.

A MEDULA RAQUIDIANA (ESPINHAL)
A medula raquidiana uma estrutura cilndrica, com
cerca de 1 cm de dimetro, que se distribui ao longo
dos ossos (vrtebras) que formam a coluna cervical.
Ao contrrio do crebro, a medula espinhal apresenta a
substncia branca disposta externamente e a substncia
cinzenta disposta internamente. Da substncia branca
partem prolongamentos de neurnios motores e sensi-
tivos.

Obs: Arco Reflexo: Os reflexos so respostas invo-
luntrias a um estmulo sensorial. Por exemplo, ao
aproximarmos a mo sobre um metal muito quente, o
estmulo trmico captado por terminaes nervosas
sensitivas e transformado em impulso nervoso o qual
ser transmitido at a medula, onde neurnios associa-
tivos estimulam os neurnios motores. Estes, por sua
vez, transmitem o impulso nervoso at os msculos do
brao, que se contraem, determinando a retirada da
mo da fonte de calor. Cabe salientar que nos reflexos
no ocorre a participao do crebro nas respostas
aos estmulos.


O SISTEMA NERVOSO PERIFRICO (SNP)
Este sistema constitudo por gnglios e nervos. Os
nervos cranianos (12 pares), partem da regio encefli-
ca, e os raquianos (31 pares), partem da medula raqui-
diana.O sistema nervoso perifrico apresenta neurnios
sensoriais que recebem as informaes dos rgos do
sentido e dos rgos internos. Alm deles, tambm
existem os neurnios motores que levam mensagens do
sistema nervoso central para os msculos e para as
glndulas.

O SISTEMA NERVOSO AUTNOMO (SNA)
Constitudo por nervos que levam impulsos nervosos
para a musculatura lisa, as glndulas e ao miocrdio.
Sendo assim, o sistema nervoso autnomo tem o papel
de controlar, de forma involuntria, a secreo glandu-
lar, a digesto, a excreo, os batimentos cardacos,
etc. A grande maioria dos rgos controlados pelo
sistema autnomo recebe dois tipos de nervos, os quais
funcionam de forma antagnica, ou seja, enquanto um
dos nervos estimula um determinado rgo, o outro
inibe o funcionamento do mesmo rgo. Desta forma,
o sistema nervoso autnomo dividido em: sistema
nervoso simptico e parassimptico.
O efeito de cada um destes sistemas, simptico e pa-
rassimptico, difere de rgo para rgo, como, por
exemplo, o corao, que estimulado pelo simptico e
inibido pelo parassimptico; j com a musculatura do
tubo digestrio ocorre o contrrio, o simptico diminui
o peristaltismo enquanto o parassimptico aumenta.


Exerccios de Sala #

1. (UFSC) "Empresas criam programas para detectar e
ajudar os funcionrios viciados em substncias qumi-
cas". Com essa manchete, a revista "Veja", de
4/7/2001, divulga uma matria sobre "As Drogas no
trabalho". Com relao ao tipo, uso e consequncias
das drogas, assinale a(s) proposio(es) correta(s).

01. As drogas que usualmente chamamos de "drogas
psicotrpicas" so aquelas que agem sobre o siste-
ma nervoso do indivduo, modificando sua maneira
de sentir, pensar ou agir.
02. As anfetaminas, muitas vezes utilizadas pelos ca-
minhoneiros, para permanecerem mais tempo acor-
dados, so poderosos estimulantes, cujo consumo
constante acaba provocando tolerncia, o que leva
o usurio a um aumento das dosagens.
04. O uso contnuo da maconha traz dificuldades de
aprendizagem e de memorizao, alm de ocasio-
nar, como o fumo, problemas respiratrios.
08. A cocana e o crack so drogas que tm alto poder
de dependncia e, quando consumidos em grandes
quantidades, podem provocar a morte por parada
cardaca.
16. O consumo de bebidas alcolicas produz uma
sensao de bem-estar, sem comprometer a sade
das pessoas.
Please purchase PDF Split-Merge on www.verypdf.com to remove this watermark.
Incluso para a vida Biologia A

Pr-Vestibular da UFSC

29
32. A herona e outras drogas injetveis, alm de cau-
saem dependncia qumica, tambm representam
risco de contgio pelo vrus HIV.

2. (CESGRANRIO) comum ouvir expresses como
estas: "Meu corao disparou", "Fiquei to nervoso
que comecei a suar", "Senti a boca seca".
Estas reaes so caractersticas de um estado emocio-
nal alterado, e so controladas sob a ao do(s)

a) sistema nervoso autnomo.
b) sistema nervoso somtico.
c) hormnios da tireide.
d) nervos do cerebelo.
e) centro nervoso medular.

Tarefa Mnima #
3. (CESGRANRIO) Os anestsicos, largamente usa-
dos pela medicina, tornam regies ou todo o organismo
insensvel dor porque atuam
a) nos axnios, aumentando a polarizao das clulas.
b) nas sinapses, impedindo a transmisso do impulso
nervoso.
c) nos dendritos, invertendo o sentido do impulso ner-
voso.
d) no corpo celular dos neurnios, bloqueando o meta-
bolismo.
e) na membrana das clulas, aumentando a bomba de
sdio.

4. (FUVEST) A figura representa um arco-reflexo: o
calor da chama de uma vela provoca a retrao do
brao e o afastamento da mo da fonte de calor. Imagi-
ne duas situaes: em A seria seccionada a raiz dorsal
do nervo e em B, a raiz ventral.

Considere as seguintes possibilidades relacionadas
transmisso dos impulsos nervosos neste arco-reflexo:
I - A pessoa sente a queimadura, mas no afasta a mo
da fonte de calor.
II - A pessoa no sente a queimadura e no afasta a
mo da fonte de calor.
III - A pessoa no sente a queimadura, mas afasta a
mo da fonte de calor.

Indique quais dessas possibilidades aconteceriam na
situao A e na situao B, respectivamente.
a) A - I; B - II.
b) A - I; B - III.
c) A - II; B - I.
d) A - II; B - III.
e) A - III; B - II.

5. (PUC-SP) O esquema abaixo representa um arco-
reflexo simples. O conhecimento sobre reflexos medu-
lares deve-se a trabalhos pioneiros feitos, no incio
deste sculo, pelo fisiologista ingls C.S. Sherrington.

No esquema, 1, 2, 3 e 4 indicam, respectivamente:
a) Neurnio aferente, sinapse, neurnio sensorial e
rgo receptor.
b) Sinapse, neurnio aferente, neurnio motor e rgo
efetuador.
c) Neurnio motor, sinapse, neurnio aferente e rgo
receptor.
d) Neurnio aferente, sinapse, neurnio motor e rgo
efetuador.
e) Neurnio motor, neurnio aferente, sinapse e rgo
receptor.

UNIDADE 16

BOTNICA

SISTEMTICA VEGETAL

AS BRIFITAS
As brifitas so plantas de pequeno porte representadas
na natureza principalmente pelos musgos. Os musgos
possuem rizoides, cauloides e filoides e so desprovi-
dos de vasos condutores de seiva o que justifica o seu
porte fsico reduzido. Como no produzem flores, a
reproduo das brifitas dependente da gua existen-
te no meio. O ciclo reprodutivo desses vegetais ocorre
por metagnese (alternncia de geraes). A etapa
duradoura e haploide denominada etapa gametoftica
(gametfito). A etapa efmera e diploide denominada
etapa esporoftica (esporfito).









Please purchase PDF Split-Merge on www.verypdf.com to remove this watermark.
Incluso para a vida Biologia A

Pr-Vestibular da UFSC

30
REPRODUO DAS BRIFITAS



AS PTERIDFITAS
As pteridfitas so plantas consideradas mais evolu-
das que as brifitas, pois, ao contrrio destas, j possu-
em vasos condutores de seiva. Os representantes mais
conhecidos das pteridfitas so as samambaias e as
avencas. Assim como as brifitas, no produzem flores,
tendo ento seu processo reprodutivo dependente da
gua. As pteridfitas tambm possuem ciclo de vida
com alternncia de geraes, sendo duradoura a etapa
esporoftica (esporfito) e efmera a etapa gametoftica
(gametfito).

Reproduo das Pteridfitas:

Exerccios de Sala #

1. (UFRS) Brifitas e pteridfitas apresentam vrias
caractersticas em comum, mas tambm diferem em
muitos aspectos. Assinale a caracterstica que pertence
a apenas um desses grupos de plantas.

a) Crescer preferencialmente em solos midos e som-
breados.
b) Necessitar de gua para reproduzir-se.
c) No ter flores, sementes e frutos.
d) Ser criptgama.
e) Ser portadora de tecidos de transporte.

2. (FATEC) Analise a descrio abaixo:
"Grupo de plantas de pequeno porte, encontradas em
locais midos e sombreados, que crescem no solo ou
sobre os troncos das rvores. H poucas espcies dul-
ccolas e nenhuma marinha. Este grupo de plantas
apresenta rizides e no possui vasos condutores".

Aps a anlise do texto, assinale a alternativa que
apresenta o nome do grupo das plantas com as caracte-
rsticas apresentadas.
a) Brifitas.
b) Angiospermas.
c) Gimnospermas.
d) Dicotiledneas.
e) Pteridfitas.

Tarefa Mnima #

3. (PUC-RS) Responder questo preenchendo com V
(verdadeiro) ou F (falso) os parnteses correspondentes
s afirmativas sobre os musgos.
( ) Pertencem ao grupo das brifitas.
( ) So seres vivos heterotrficos absortivos.
( ) So desprovidos de traquedeos.
( ) Preferem solos secos e frios.
( ) So parentes das hepticas.
A sequncia correta resultante do preenchimento dos
parnteses, de cima para baixo,
a) F - F - V - V - V
b) F - V - F - V - F
c) V - F - V - F - V
d) V - V - F - V - V
e) V - V - V - F - F

4. (UFPEL) Os vegetais vasculares que possuem raiz,
caule e folhas, mas no so dotados de flores, frutos e
sementes so:
a) Algas, como as cianofceas, a alface-do-mar e as
algas pardas.
b) Pteridfitas, como as samambaias, avencas e xaxins.
c) Angiospermas, como as gramneas, o eucalipto e os
cactos.
d) Gimnospermas, como os ciprestes, os pinheiros e o
'Ginkgo biloba'.
e) Fungos, como a orelha-de-pau, os cogumelos e as
leveduras.

Please purchase PDF Split-Merge on www.verypdf.com to remove this watermark.
Incluso para a vida Biologia A

Pr-Vestibular da UFSC

31
5. (UECE) As plantas, assim como todos os demais
seres vivos, possuem ancestrais aquticos e desta for-
ma sua histria evolutiva encontra-se relacionada
ocupao progressiva do ambiente terrestre. Para que
isso pudesse acontecer algumas caractersticas foram
selecionadas e dentre elas podemos destacar:
I - Sistema vascular;
II - Esporfito dominante;
III Filides;
IV - Esporfito no ramificado.

So caractersticas prprias de pteridfitas e brifitas,
respectivamente:

a) I e II; III e IV
b) I e III; II e IV
c) II e IV; I e III
d) III e IV; I e II

UNIDADE 17

AS GIMNOSPERMAS

As gimnospermas so representadas pelos pinheiros,
sequias e ciprestes. So plantas tipicamente terrestres
e vivem preferencialmente em ambientes frios e tem-
perados. As gimnospermas apresentam razes, caules,
folhas e so as primeiras plantas dentro da escala evo-
lutiva e apresentarem flores e sementes. As gimnos-
permas no possuem o fruto envolvendo as sementes e,
por este motivo, a denominao de gimnospermas
(gymno = nu; sperma = semente), sementes nuas.

REPRODUO DAS GIMNOSPERMAS



AS ANGIOSPERMAS
As angiospermas possuem razes, caules, folhas, se-
mentes e so os nicos vegetais a desenvolverem fru-
tos. Por este motivo, a denominao de angiospermas
(angion = bolsa; sperma = semente), ou seja, plantas
com sementes no interior de uma bolsa, o fruto, como,
por exemplo: a laranjeira, o limoeiro, o abacateiro, o
coqueiro, a jabuticabeira, etc.




CLASSIFICAO DAS ANGIOSPERMAS
As angiospermas podem ser classificadas de acordo
com o nmero de cotildones presentes na semente.



Exerccios de Sala #

1. (UFSC) Atualmente a Terra dominada pelo grupo
vegetal das Angiospermas, com cerca de 250.000 es-
pcies espalhadas por todo o mundo. A maior parte dos
alimentos de origem vegetal derivada de plantas
desse grupo.
Com respeito s Angiospermas, correto afirmar que:

01. So os nicos vegetais que produzem sementes.
02. As monocotiledneas so uma diviso deste grupo,
cujos representantes apresentam raiz axial ou pivo-
tante, flores tetrmeras, sementes com dois cotil-
dones e crescimento acentuado em espessura.
04. Suas flores originam estruturas chamadas frutos
que auxiliam na disperso de suas sementes.
08. Em algumas espcies, o fruto pode se desenvolver
sem que ocorra o processo de fecundao, origi-
nando os chamados frutos partenocrpicos.
16. Suas flores podem ser polinizadas por algumas
aves, mamferos e insetos.
32. Alguns de seus frutos so comestveis; como por
exemplo, o chuchu e o tomate.

2. (UEPG) No que respeita s plantas monocotiled-
neas e dicotiledneas e suas principais caractersticas,
assinale o que for correto.
01. Os feixes lbero-lenhosos das monocotiledneas
so espalhados, e os das dicotiledneas so dispos-
tos em crculo.
02. Os elementos florais das monocotiledneas so
geralmente mltiplos de 3, e os das dicotiledneas
so geralmente mltiplos de 4 ou 5.
Please purchase PDF Split-Merge on www.verypdf.com to remove this watermark.
Incluso para a vida Biologia A

Pr-Vestibular da UFSC

32
04. A raiz das monocotiledneas pivotante, e a das
dicotiledneas fasciculada.
08. As nervuras das folhas monocotiledneas so reti-
culadas, e as das folhas dicotiledneas so parale-
las.
16. O milho uma planta monocotilednea, e o feijo
uma planta dicotilednea.

Tarefa Mnima #

3. (UFSC) Os principais grupos vegetais (Brifitas,
Pteridfitas, Gimnospermas e Angiospermas) apresen-
tam em comum um ciclo de vida que ocorre atravs de
alternncia de geraes (metagnese), em que uma
gerao haplide alterna-se com outra diplide. Com
relao a este ciclo e considerando o esquema a seguir,
assinale a(s) proposio(es) correta(s).

01. O esquema representa um ciclo de vida haplodi-
plobionte (ou haplonte-diplonte) tpico dos princi-
pais grupos de vegetais.
02. Os eventos que ocorrem em I e III do esquema
correspondem, respectivamente, meiose e mito-
se.
04. Neste ciclo, o esporfito forma o gametfito por
reproduo assexuada e o gametfito forma o espo-
rfito por reproduo sexuada.
08. Nas gimnospermas e angiospermas, o esporfito
originado pela fuso dos gametas masculino e
feminino que so, respectivamente, o androceu e o
gineceu.
16. Os eventos II e IV do esquema correspondem,
respectivamente, fecundao e germinao.
32. Nas brifitas e pteridfitas, a fase gametoftica
duradoura e evidente e a fase esporoftica, ao con-
trrio, reduzida e pouco evidente.

4. (UFSC) H mais de 250 milhes de anos, as gim-
nospermas, originadas das pteridfitas, dominaram as
paisagens terrestres durante o Trissico e o Jurssico,
juntamente com os dinossauros. Hoje, esse grupo vege-
tal est restrito a alguns locais da Terra, conhecidos
como Florestas de Conferas, como as ainda existentes
no sul do Brasil.

Com relao a esse grupo de plantas, correto afirmar
que:

01. Sua madeira utilizada na indstria de papel e
celulose, na indstria de mveis e na construo de
casas.
02. Algumas espcies tm carter ornamental, como os
ciprestes e os populares pinheiros de Natal.
04. No Brasil, comum o consumo do pinho como
alimento, que a semente do pinheiro-do-paran.
08. Suas plantas produzem sementes nuas, ou seja, no
h a formao de frutos.
16. So plantas avasculares, com flores perfeitas.
32. Todas as espcies do grupo so diicas.

5. (UEL) Considerando-se brifitas, pteridfitas, gim-
nospermas e angiospermas, fizeram-se as seguintes
afirmaes:

I. O gametfito , comparativamente, mais desenvolvi-
do nas brifitas.
II. O gametfito atinge o mximo de reduo nas gim-
nospermas e nas angiospermas.
III. O esporfito das pteridfitas sempre parasita do
gametfito, enquanto que nos outros grupos ele
auttrofo.
IV. Os esporfitos dos quatro grupos de plantas so
haplides, originando esporos por mitose.

So corretas somente:
a) I e II
b) I e III
c) II e III
d) II e IV
e) III e IV

UNIDADE 18

HISTOLOGIA VEGETAL

o ramo da botnica que estuda os tecidos de origem
vegetal. Os mesmos so classificados em dois tipos
bsicos:

Tecidos Embrionrios ou Meristemas.

Tecidos Permanentes ou Adultos.

OS MERISTEMAS
So tecidos constitudos por clulas embrionrias que
por mitose originam todos os demais tecidos definiti-
vos. Os meristemas so subdivididos em dois tipos
diferentes: o meristema primrio o meristema secun-
drio.

O MERISTEMA PRIMRIO
As clulas do meristema primrio, ao se diferenciarem,
formaro estruturas que iro determinar o crescimento
em comprimento (longitudinal) de todos os vegetais e
um pequeno crescimento em espessura.

Classificao:
Dermatognio: Tecido embrionrio responsvel
pela formao da epiderme vegetal.
Periblema: Tecido responsvel pela formao dos
tecidos do crtex (casca) do caule ou raiz.
Pleroma: Tecido responsvel pela formao dos
tecidos que constituem o cilindro central do caule e
raiz.
Please purchase PDF Split-Merge on www.verypdf.com to remove this watermark.
Incluso para a vida Biologia A

Pr-Vestibular da UFSC

33
OBS: Na raiz, existe tambm o caliptrognio, respon-
svel pela formao da coifa ou caliptra, estrutura que
no se encontra no caule.


O MERISTEMA SECUNDRIO
Tecido embrionrio que determina o crescimento em
espessura da maioria das Gimnospermas e Dicotiled-
neas.

Classificao:

Felognio: O Felognio responsvel pela forma-
o do sber, que tecido definitivo de revestimento, e
tambm origina os tecidos da Feloderme.
Cmbio: A atividade do cmbio produz os vasos do
xilema para o lado de dentro e floema para o lado de
fora do cilindro central, proporcionando o crescimento
em espessura em dicotiledneas e gimnospermas.


Os tecidos permanentes:

A EPIDERME
Tecido que reveste, protege e possibilita trocas entre o
meio ambiente e os tecidos internos do corpo do vege-
tal. As clulas epidrmicas possuem as seguintes pecu-
liaridades: forma fsica achatada e justaposta (muito
unidas); normalmente sem cloroplastos, consequente-
mente, forma uma pelcula incolor.



ANEXOS EPIDRMICOS
Cutcula: A cutcula formada por um lipdio
(cutina) que impermeabiliza (caule e folhas.) o vegetal.
bastante espessa em vegetais de clima quente e seco
onde impede o excesso de transpirao.


Pelos ou tricomas: So anexos epidrmicos, que
formam prolongamentos em direo ao meio. Depen-
dendo da planta os pelos podem ter funo secretora,
protetora, ou ainda absorvente.

Estmatos: So anexos epidrmicos que permitem
as trocas gasosas entre o interior e exterior do vegetal.
O estmato estruturado por duas clulas (clulas
guardis) em forma de rim que limitam uma abertura
(ostolo). Esta abertura comunica-se com espaos exis-
tentes abaixo da epiderme. Normalmente, os estmatos
so encontrados na face inferior das folhas.



Acleos: um anexo com forma pontiaguda e
resistente devido ao depsito de lignina. Protegem o
caule de certos vegetais.


O SBER
Tecido constitudo por clulas mortas e que reveste o
caule e raiz de dicotiledneas e gimnospermas. O sber
Please purchase PDF Split-Merge on www.verypdf.com to remove this watermark.
Incluso para a vida Biologia A

Pr-Vestibular da UFSC

34
apresenta um conjunto de clulas pluriestratificadas e
justapostas. O sber possui as seguintes funes:
Substitui a epiderme em caule e razes de gimnosper-
mas e dicotiledneas;
Protegem o vegetal contra a umidade, variaes brus-
cas de temperatura, ataque de fungos, insetos e outros
agentes.


O COLNQUIMA
Conjunto de clulas vivas com cloroplastos e que se
caracterizam por apresentarem uma parede celular com
reforos celulsicos. O colnquima atua na sustentao
e determina resistncia e flexibilidade nas regies mais
jovens do vegetal.


O ESCLERNQUIMA
Conjunto de clulas mortas por deposio de lignina
em sua parede. Esta impregnao da parede por lignina
determina a morte celular, proporcionando, alta resis-
tncia para alguns rgos da planta,

Exerccios de Sala #

1. (PUC-Campinas) A presena de diversos tipos de
plos nos vegetais lhes proporciona uma melhor adap-
tao ao meio ambiente. So processos relacionados
com a presena de plos vegetais, exceto:
a) Proteger contra ataques de animais.
b) Facilidade de disperso de frutos e sementes.
c) Aumento no poder de absoro de gua e sais.
d) Facilitar a perda de gua em excesso, acumulada
nos parnquimas.

2. (PUC-MG) O sber
a) um tecido de conduo encontrado em vegetais
superiores com crescimento primrio e secundrio.
b) um tecido com funo de proteo encontrado em
vegetais superiores apenas com crescimento secun-
drio.
c) uma estrutura utilizada para armazenamento de
amido primrio, resultante da atividade da perider-
me.
d) um pigmento que responsvel pela colorao das
flores.
e) um tecido de revestimento que permite o aumento
ou decrscimo na transpirao da planta.

Tarefa Mnima #

3. (UEL) Os tecidos nos quais encontram-se estmatos
e lenticelas em uma rvore so, respectivamente,
a) epiderme e sber.
b) xilema e colnquima.
c) floema e xilema.
d) sber e floema.
e) colnquima e epiderme.
4. (UFMG) Todas as alternativas contm adaptaes
evolutivas que permitiram a sobrevivncia dos vegetais
fora do ambiente aqutico, exceto:
a) Epiderme impregnada de cutina.
b) Presena de parede celular.
c) Presena de raiz.
d) Tecidos condutores: xilema e floema.
e) Troncos recobertos de sber.

5. (UDESC) Os meristemas dos vegetais so tambm
chamados tecidos de crescimento, porque suas clulas
a) possuem em seus citoplasmas um grande nmero de
vacolos.
b) apresentam uma grande capacidade de multiplica-
o.
c) atingem grandes tamanhos.
d) so as mais frequentes na composio dos caules.
e) produzem hormnios de crescimento.

UNIDADE 19

O XILEMA
Tecido responsvel pelo transporte da seiva bruta ou
mineral da raiz at as folhas. As clulas do xilema so
mortas devido impregnao de lignina na parede
celular.

Please purchase PDF Split-Merge on www.verypdf.com to remove this watermark.
Incluso para a vida Biologia A

Pr-Vestibular da UFSC 35
O FLOEMA
Tecido envolvido com o transporte da seiva elaborada
ou orgnica das folhas a todas as regies vivas do
vegetal. Os vasos do floema so formados por clulas
vivas.


O PARNQUIMA CLOROFILIANO
Tecido que apresenta clulas ricas em cloroplastos,
portanto relacionadas ao processo de fotossntese.
Localiza-se abaixo da epiderme das folhas e dos caules
verdes, e ainda em frutos verdes. O parnquima cloro-
filiano constitudo por dois tipos de clulas que lhe
confere a subdiviso em:
Palidico: constitudo por clulas volumosas e
justapostas. So clulas ricas em cloroplastos. Suas
clulas esto localizadas imediatamente abaixo da
epiderme.
Lacunoso: constitudo por clulas menos volu-
mosas e com menor quantidade de cloropastos. Entre
as clulas h espaos (lacunas) para a circulao dos
gases para a fotossntese. Fica localizado abaixo do
parnquima palidico.

O PARNQUIMA DE RESERVA OU MILFERO
Tecido que apresenta clulas vivas e concentradas
principalmente em certos caules, razes, frutos e se-
mentes. Os plastos e vacolos das suas clulas armaze-
nam substncias tais como: protenas, glicdios (ami-
do), lipdios, vitaminas, sacarose, etc. O parnquima de
reserva pode ser comparado ao tecido adiposo dos
animais.


O PARNQUIMA AQUFERO
Tecido formado por clulas que armazenam grande
quantidade de gua. Este tipo de parnquima encon-
trado principalmente em certos caules de plantas xer-
fitas (ex:cactos).

O PARNQUIMA AERFERO
Tecido formado por clulas que circundam espaos em
que no interior armazenado ar. um parnquima
encontrado em plantas aquticas (flutuadoras)




Exerccios de Sala #

1. (UFSC) Pouca vantagem representaria, para animais
e plantas, serem multicelulares, se todas as clulas
fossem iguais. [...] Os rgos das plantas, [...] so
formados por tecidos.
(FROTA-PESSOA, O. "Os caminhos da vida I. Estrutura
e ao". So Paulo: Scipione, 2001, p. 157).
Com relao a esse assunto correto afirmar que:
01. As razes, a epiderme e os estmatos so exemplos
de rgos das plantas.
02. Na epiderme existem clulas meristemticas com
funo de aerao da planta.
04. O xilema e o floema compem o sistema de tecidos
vasculares das plantas.
08. Os diversos tipos de parnquimas exercem funes
de respirao, fotossntese e aerao, entre outras.
16. Os frutos, outro tipo de rgo nas plantas, so
formados basicamente de clulas meristemticas.
32. O colnquima e o esclernquima so tecidos de
conduo, compostos de parnquimas vivos.
64. Pelo xilema circula a seiva bruta, rica em gua e
sais minerais.
Please purchase PDF Split-Merge on www.verypdf.com to remove this watermark.
Incluso para a vida Biologia A

Pr-Vestibular da UFSC

36

2. (UFSC) As plantas so seres vivos pluricelulares e
organizados que apresentam diferentes tecidos.Com
relao aos tecidos vegetais, assinale a(s) proposi-
o(es) correta(s).
01. Em plantas vasculares, o tecido condutor especiali-
zado na conduo da seiva bruta o floema e, na
conduo da seiva elaborada, o xilema.
02. Os tecidos meristemticos so formados por clu-
las diferenciadas que, por desdiferenciao destas
clulas, originam todos os demais tecidos da planta.
04. O colnquima e o esclernquima constituem os
tecidos de sustentao do vegetal.
08. Os tecidos parenquimticos executam numerosas
tarefas, tais como o preenchimento de espaos, a re-
alizao da fotossntese e o armazenamento de subs-
tncias.

Tarefa Mnima #

3. (PUC-MG) O desenho representa o corte de uma
folha indicando tecidos e/ou estruturas foliares.

Assinale a funo que no ocorre em nenhuma das
estruturas representadas.
a) Transpirao.
b) Transporte de seivas.
c) Fotossntese.
d) Troca gasosa.
e) Respirao.

4. (UFMG) Observe os esquemas de tecidos, numera-
dos de 1 a 5. Indique a alternativa que contm os n-
meros relacionados apenas a tecidos vegetais.

a) 1 e 4.
b) 1 e 5.
c) 2 e 3.
d) 2 e 4.
e) 3 e 5.

5. (UFMG) O esquema a seguir refere-se a um corte
transversal de uma folha de vegetal em que estruturas
histolgicas foram indicadas pelos nmeros de 1 a 5.
Em relao a esse esquema, incorreto afirmar-se
que;

a) 1 uma estrutura de proteo.
b) 2 um epitlio com capacidade de renovao.
c) 3 o principal tecido fotossinttico.
d) 4 contm estrutura responsvel pela conduo de
seiva.
e) 5 depende do turgor das clulas para seu funciona-
mento.















Please purchase PDF Split-Merge on www.verypdf.com to remove this watermark.
Incluso para a Vida Biologia B

Pr-Vestibular da UFSC 1
UNIDADE 1

CITOQUMICA

COMPONENTES INORGNICOS - GUA E SAIS
MINERAIS
A gua exerce um papel fundamental e essencial nos
organismos, sendo inclusive o componente qumico
encontrado em maior quantidade nas clulas e,
consequentemente, nos seres vivos. No entanto, observa-se
que a quantidade de gua nas clulas varia de espcie para
espcie e, tambm, de acordo com a idade e o tipo de
atividade funcional. Por exemplo, nos celenterados (gua
viva), a concentrao de gua em suas clulas pode chegar a
mais de 90%, enquanto nas sementes de alguns vegetais a
quantidade de gua , aproximadamente, 5%. J as clulas
nervosas, que so super ativas, a concentrao de gua pode
chegar a 80%, ao contrrio das clulas sseas, que
apresentam uma concentrao em torno de 40%. Alm disso,
num beb, a gua responsvel por mais de 85% do peso
corporal, ao passo que, numa pessoa idosa, corresponde
cerca de 70 a 75% do seu peso corporal.

FUNES DA GUA

REAES DE HIDRLISE
A gua um reagente indispensvel para a maioria das
reaes qumicas que ocorrem nos seres vivos. Por exemplo,
todo o oxignio existente na atmosfera, tambm
proveniente da quebra de molculas de gua.
Por outro lado, a exemplo do que ocorre nos processos de
digesto, as protenas obtidas nos alimentos como carne e
leite, tero que ser fragmentadas em pequenas molculas
(aminocidos), para que as clulas possam absorv-las.
Neste tipo de reao, na qual ocorre a quebra de grandes
molculas em pequenas molculas, tambm h participao
da molcula de gua, e essa reao recebe o nome espao de
reao de hidrlise (hydro: gua / lise: quebra).

REGULADOR TRMICO
A gua contribui para conservar a temperatura constante dos
animais, no deixando com que ocorram variaes bruscas,
o que poderia levar o organismo morte.
Essa caracterstica deve-se ao fato da gua
apresentar um alto calor especfico. Outra caracterstica
muito importante da gua, como regulador trmico para os
seres vivos, com relao ao seu elevado grau de
vaporizao, quando comparada com outros lquidos. Por
exemplo, para evaporar 1g de gua, necessria uma
quantidade de calor dez vezes maior do que seria para elevar
1g de outro lquido, como a amnia. A evaporao da gua
evita o superaquecimento dos animais e vegetais. As plantas
s no se superaquecem durante um dia de muito sol, porque
o intenso calor que estes vegetais absorvem acaba sendo
perdido quando a gua evapora de sua superfcie. Fato
semelhante tambm ocorre com os animais, quando o suor,
presente na superfcie do corpo, devido a uma atividade
fsica ou a um dia muito quente, promove o resfriamento.
Alm disso, a gua protege os organismos contra os
efeitos do congelamento, devido ao seu alto calor de fuso.
Portanto, para que a gua se torne gelo, esta ter que ser
exposta, por um longo tempo, a temperaturas muito baixas.

AGENTE PASSIVO NA OSMOSE
A osmose a passagem de molculas de gua de uma
soluo menos concentrada (meio hipotnico) para outra
mais concentrada (meio hipertnico).



SOLVENTE UNIVERSAL
A gua um excelente solvente, pois capaz de dissolver
vrias substncias qumicas polares, ou seja, com cargas
positivas e negativas, como por exemplo, os sais minerais,
os acares, os aminocidos, as protenas e os cidos
nucleicos. As substncias qumicas que se dissolvem na
gua so denominadas substncias hidrfilas (hydro: gua /
philos: amigo), enquanto os lipdios (gorduras e ceras), por
apresentarem baixa polaridade (so substncias no-polares),
so insolveis em gua, sendo denominadas substncias
hidrfobas (hydro: gua / phobos: medo).
A polaridade da gua responsvel pela sua caracterstica de
solvente universal, pois as suas molculas associam-se,
atravs das pontes de hidrognio, tanto aos ons positivos,
quanto aos ons negativos.

VECULO DE TRANSPORTE
Devido caracterstica de solvente universal, a gua o
principal veculo transportador, pois ao dissolver muitas
substncias qumicas, permite que a clula realize a ingesto,
a digesto e a absoro, alm de facilitar a entrada e a sada
de muitos compostos atravs da membrana celular.

SAIS MINERAIS
Como a gua, os sais minerais so fundamentais para o bom
funcionamento e sobrevivncia do organismo. A
quantidade de sais minerais tambm varia de espcie para
espcie. Algumas espcies apresentam concentraes ao
redor de 1%, e outras podem apresentar at 5%. Os sais
minerais podem ser encontrados nos seres vivos sob duas
formas bsicas: imobilizados e pouco solvel, constituindo,
por exemplo, as carapaas, as cascas dos ovos e os
esqueletos dos animais; dissolvidos em gua, portanto
dissociados em ons, sendo considerados de mxima
importncia para o bom funcionamento da clula e,
consequentemente, do organismo.

Please purchase PDF Split-Merge on www.verypdf.com to remove this watermark.
Incluso para a Vida Biologia B

Pr-Vestibular da UFSC
2

Exerccios de Sala #

1. (UFSC) A maior parte dos seres vivos composta de
gua. No corpo humano, a porcentagem de gua pode variar
de 20%, nos ossos, a 85%, nas clulas nervosas; nas
medusas (animais marinhos), a porcentagem de gua chega a
mais de 95%. Assinale as afirmativas que indicam
corretamente a importncia da gua nos seres vivos.
01. A maioria dos elementos qumicos, presentes nos seres
vivos, necessitam de um meio aquoso para se
dissolverem e reagirem uns com os outros.
02. A gua atua no transporte e remoo dos produtos do
metabolismo.
04. A grande capacidade da gua de absorver calor protege o
material vivo contra sbitas mudanas trmicas.
08. A gua atua como lubrificante, estando presente nos
lquidos corporais, entre um rgo e outro.
D como resposta a soma dos nmeros associados s
alternativas corretas.

2. Qual das alternativas abaixo no est relacionada
economia de gua ou proteo contra desidratao?
a) Pele queratinizada.
b) Reduo da taxa de transpirao.
c) Folhas vegetais com cutcula espessa.
d) Embrio envolvido pela vescula amnitica (bolsa
dgua).
e) Alta taxa de mico.

3. (UFRN) Elementos que fazem parte da constituio das
molculas de ATP, clorofila e hemoglobina so,
respectivamente:
a) magnsio, ferro e fsforo. d) magnsio, fsforo e ferro.
b) ferro, magnsio e fsforo. e) fsforo, ferro e magnsio.
c) fsforo, magnsio e ferro.
SAIS
MINERAIS
IMPORTNCIA FONTES
Clcio x Importante componente na constituio dos ossos e dos dentes;
x de fundamental importncia para que inicie a coagulao do
sangue;
x necessrio para o funcionamento do impulso nervoso e da
contrao dos msculos.
encontrado nos vegetais verdes, no leite e
nos laticnios.
Cloro x Importante no balano de lquidos do corpo;
x Importante on negativo no lquido extracelular;
x Componente importante na manuteno do pH.
Sal de cozinha.
Cobre x Importante componente de muitas enzimas;
x Essencial para a sntese (produo) de hemoglobina.
encontrado nos feijo, ovos, peixe e no
trigo integral.
Cobalto x Constituinte da vitamina B12;
x Essencial para a produo das hemcias.
encontrado na carne e nos laticnios.
Enxofre x Constituinte de muitas protenas, tambm essencial para a
atividade normal do metabolismo.
encontrado na carne e nos vegetais.

Fsforo x Como o clcio, o fsforo tambm um importante componente
dos ossos e dos dentes;
x Essencial para o armanezamento e transferncia de energia no
interior das clulas, sendo componente da molcula de ATP;
x Tambm o componente das molculas de DNA e RNA.
encontrado no leite, laticnios, carnes e
cereais.
Ferro x Constituinte importe da hemoglobina, da mioglobina e de
enzimas respiratrias.
x fundamental para a respirao celular.
encontrado na carne, no fgado, na gema
do ovo, nos legumes e nos vegetais verdes.
Flor x Tambm constituinte dos ossos e dos dentes, protegendo-os
contra as cries.
gua fluorada.
Iodo x Componente dos hormnios da tireide, os quais, por sua vez,
estimulam o metabolismo.
encontrado no sal de cozinha, laticnios e
frutos do mar.
Magnsio x Componente de muitas coenzimas;
x necessrio para que ocorra o funcionamento normal dos
nervos e msculos.
encontrado nos cereais e vegetais verdes.
Mangans x Importante para que ocorra a ativao de diversas enzimas. encontrado no cereais, gema de ovo e
vegetais verdes.
Potssio x Importante on positivo no interior das clulas;
x Participa na contrao muscular e na atividade das clulas
nervosas.
encontrado na carne, leite e, em muitas
frutas.
Sdio x Fundamental para a conduo do impulso nervoso;
x Importante no balano de lquidos do corpo;
x Importante on negativo no lquido extracelular.
encontrado no sal de cozinha.
Zinco x Constituinte de vrias enzimas, como por exemplo, no processo
de digesto.
encontrado em vrios alimentos.
Please purchase PDF Split-Merge on www.verypdf.com to remove this watermark.
Incluso para a Vida Biologia B

Pr-Vestibular da UFSC 3
UNIDADE 2

COMPOSTOS ORGNICOS DA CLULA

Os compostos orgnicos presentes nos seres vivos so:

GLICDIOS (Carboidratos)
So molculas orgnicas que tambm recebem a
denominao de carboidratos, hidratos de carbono,
sacardeos e acares. Ao longo das molculas de
carboidratos, sempre est presente um grupo aldedo ou um
grupo cetona e, nos demais carbonos, o grupamento
hidroxila.

FUNES DOS GLICDIOS
x Energtica: Os glicdios constituem a primeira e a
principal substncia a ser convertida em energia calorfica
na forma de ATP (trifosfato de adenosina) nas clulas,
atravs dos processos de respirao celular e pela
fermentao.
x Estrutural ou plstica: Alguns carboidratos, em
determinadas clulas proporcionam rigidez, consistncia e
elasticidade, como por exemplo, a pectina, a hemicelulose e
a celulose encontrados na parede das clulas vegetais e a
quitina, constituindo o exoesqueleto dos artrpodos.

CLASSIFICAO DOS GLICDIOS
Os glicdios apresentam a seguinte formula geral: C
n
(H
2
O)
n
e so classificados em trs grupos:
x Monossacardeos: so glicdios que possuem um baixo
nmero de tomos de carbono em sua molcula.
x Oligossacardeos: so glicdios que se formam a partir da
unio de dois at dez monossacardeos, podendo ser
denominado de OSDEOS. O grupo mais importante de
oligossacardeos so os denominados dissacardeos,
formados pela unio de apenas dois monossacardeos ou
oses, como por exemplo, a maltose constituda por duas
molculas de glicose; a lactose, constituda por uma
molcula de glicose mais uma de galactose e a sacarose que
constituda por uma molcula de glicose mais uma de
frutose.
x Polissacardeos: So glicdios que possuem mais de dez
monossacardeos ligados em cadeia, como a celulose que
constituda por mais de 1000 glicoses; o amido, principal
reserva energtica dos vegetais, constitudo por mais de
1400 glicoses e o glicognio, principal reserva energtica
dos animais e fungo constitudo por mais de 30000
glicoses.

LIPDIOS
So molculas orgnicas encontradas em grande quantidade
nos seres vivos, constituindo, aproximadamente, 5% da
matria viva. Os lipdios so formados pela associao de
uma molcula de lcool, ligada a molculas de cidos
graxos, os quais so longas cadeias de carbono e hidrognio,
apresentando numa das extremidades um grupo cido
(COOH). Esta associao de alcoois com cidos graxos
resulta em molculas denominadas steres.

FUNES DOS LIPDIOS
x Estrutural: Os lipdios, juntamente com as protenas,
conferem s clulas rigidez, consistncia e elasticidade.
x Reserva Energtica: Os lipdios constituem a
segunda fonte de energia calorfica para as clulas,
produzindo quatro vezes mais energia do que uma molcula
de glicdio. Entretanto, as clulas oxidam muito mais
facilmente os glicdios do que os lipidos.
Nos animais, os lipdios so armazenados em clulas
especiais denominadas adiposas ou adipcitos. Essas clulas
localizam-se no panculo adiposo (camada de tecido que se
dispe abaixo da pele), ao redor de certas vsceras e na
medula ssea amarela.
x Isolamento trmico: O panculo adiposo constitui
um revestimento natural dos animais homeotrmicos contra
o frio, mantendo o calor do corpo e, consequentemente,
atuando como um isolante trmico.
x Isolamento eltrico: Os lipdios no conduzem
cargas eltricas, portanto suas molculas so denominadas
apolares, como por exemplo, a camada de fosfolipdios da
membrana plasmtica e a bainha de mielina das clulas
nervosas, as quais so responsveis pela diferena de
potencial (DDP), entre o meio interno e o meio externo da
clula.

CLASSIFICAO DOS LIPDIOS
Os lipdios so classificados em simples e complexos.
x Lipdios simples: So lipdios que apresentam na
molcula de lcool e nos cidos graxos somente os tomos
de carbono (C), de hidrognio (H) e oxignio (O), como por
exemplo:
- os glicerdeos, formados por um glicerol (glicerina),
mais trs cidos graxos. So os leos e gorduras, que se
diferenciam apenas com relao ao ponto de fuso.
Enquanto os leos so lquidos, temperatura ambiente, e de
origem vegetal, as gorduras so slidas e de origem animal;
- os cerdeos so lipdios semelhantes aos glicerdeos,
possuindo o cido graxo em sua molcula, diferindo-se
apenas pelo tipo de lcool que possui at dezesseis carbonos
na cadeia. As ceras de abelha e a carnaba so exemplos de
cerdeos.
- os esterdeos so lipdios com alcois policiclicos
com steres de cidos graxos. A progesterona, a testosterona
e o estrgeno que so hormnios sexuais, o colesterol, a
vitamina A, D e a cortisona.
x Lipdios complexos: So lipdios que apresentam,
alm dos tomos de carbono, hidrognio e oxignio, outros
tomos, como por exemplo, o nitrognio, fsforo e o
enxofre. Os fosfolipdios da membrana plasmtica de todas
as clulas, a esfingomielina que forma a bainha de mielina
em determinados neurnios e a lecitina, presente na gema do
ovo, so exemplos de lipdios complexos.

AS PROTENAS
O termo protena utilizado quando a molcula apresenta
mais de setenta aminocidos, os quais esto unidos uns aos
outros, atravs das ligaes peptdicas, formando longas
cadeias. Como j foi citado, a enorme variedade de
protenas existentes nos seres vivos decorrente do fato de
que os 20 aminocidos apresentam-se formando muitas
sequncias e combinaes diferentes, possibilitando uma
variedade incrvel de molculas proteicas. As protenas
ainda diferem umas das outras pela quantidade de
aminocidos, pelos tipos de aminocidos e pela sequncia
em que estes aminocidos esto unidos. Por exemplo, a
bactria Escherichia coli, contm aproximadamente 800
Please purchase PDF Split-Merge on www.verypdf.com to remove this watermark.
Incluso para a Vida Biologia B

Pr-Vestibular da UFSC
4
espcies distintas de protenas e, em cada uma das clulas
humanas, j foram identificados mais de trs mil tipos de
protenas que atuam em todos os processos vitais da clula,
desde a produo de energia at a sntese e destruio de
substncias intracelulares.

OS AMINOCIDOS
Os aminocidos so unidades que constituem as protenas.
Estas molculas se caracterizam por apresentar cadeias de
carbono contendo, obrigatoriamente, o tomo hidrognio,
oxignio e nitrognio, podendo, algumas vezes, aparecer
tambm o tomo de enxofre. Os vegetais produzem todos os
aminocidos de que necessitam a partir da fotossntese e do
nitrato (NO
3
-), retirado do meio ambiente. J os animais,
no so capazes de produzir todos os aminocidos que
necessitam, tendo de obt-los atravs da alimentao. Na
espcie humana, so necessrios 20 tipos diferentes de
aminocidos, os quais, em combinaes diferentes, so
capazes de originar milhares de protenas. O organismo
humano, contudo, capaz de produzir apenas onze dos vinte
aminocidos, sendo necessrio obter atravs da alimentao
(carne, leite, queijo, peixe e ovos), os outros nove
aminocidos que lhe faltam. Todos os aminocidos obtidos
atravs da alimentao so denominados aminocidos
essenciais, enquanto que aqueles sintetizados pelo prprio
organismo so conhecidos como aminocidos no-essenciais
ou naturais.

LIGAES ENTRE OS AMINOCIDOS
A ligao qumica que se estabelece entre dois aminocidos
denominada de ligao peptdica. Esta ligao ocorre
devido a unio de um tomo de hidrognio (H), proveniente
do grupo amina (NH
2
), com a hidroxila (OH), proveniente
do grupo carboxila (COOH), resultando a unio dos
aminocidos e a formao de uma molcula de gua. As
molculas resultantes da unio de aminocidos so
denominadas peptdeos. Portanto, dois aminocidos formam
um dipeptdeo, ao passo que trs aminocidos formaro um
tripeptdeo e, quatro, um tetrapeptdeo,, sucessivamente. Os
termos oligopeptdeo (oligo; pouco) e polipeptdeo (poli;
muito), tambm so utilizados para denominar molculas
formadas por poucos aminocidos e muitos aminocidos,
respectivamente.

ESTRUTURA DAS PROTENAS
A sequncia de aminocidos na protena denominada
estrutura primria. Essa sequncia o fator responsvel
pela atividade biolgica da protena e, atualmente, j se
conhece a sequncia de muitos aminocidos que formam
algumas protenas, como por exemplo, a hemoglobina, que
constituda por 574 aminocidos; da insulina (um hormnio
secretado pelo pncreas), da ocitocina (hormnio
responsvel pelas contraes do parto), da mioglobina
(protena do msculo), entre outros. interessante ressaltar
que a simples troca de um destes aminocidos poder causar
consequncias graves, como o caso da anemia falciforme,
provocada pela substituio de um aminocido, o cido
glutmico, por outro aminocido, a valina, numa certa regio
da molcula.
No entanto, as protenas no so simples fios
esticados, geralmente as cadeias polipeptdicas esto
enroladas em forma helicoidal, lembrando um fio de
telefone. Esse enrolamento denominado estrutura
secundria da protena, a qual consequncia da atrao
entre tomos de aminocidos que esto prximos um dos
outros. Esta cadeia polipeptdica helicoidal costuma ainda
dobrar-se sobre si mesma, assumindo o aspecto de um
novelo e constituindo a estrutura terciria da protena.
Alm disso, muitas protenas so constitudas por mais de
uma cadeia polipeptdica, como por exemplo, a hemoglobina
do nosso sangue e a clorofila dos vegetais, que so
constitudos por quatro cadeias polipeptdicas. Esta estrutura
extremamente complexa de algumas molculas proteicas
denominada de estrutura quaternria.

Estrutura das protenas
A letra A representa a estrutura primria.
A letra B representa a estrutura secundria.
A letra C representa a estrutura terciria.
A letra D representa a estrutura quaternria.

FUNES DAS PROTENAS
x Funo estrutural ou plstica: so protena que
participam da estrutura dos tecidos, conferindo-lhes,
consistncia, rigidez e elasticidade, como por exemplo,
colgeno, protena de alta resistncia encontrada na pele,
nas cartilagens, nos ossos e nos tendes; a actina e a
miosina, protenas contrteis encontradas em grande
quantidade nos msculos, participando do mecanismo de
contrao muscular; a queratina, protena impermeabilizante
encontrada na pele, cabelos e unhas, protege contra a
dessecao; os gens, constitudos pelo cido
desoxirribonucleico, apresentam em sua constituio
qumica, molculas proteicas; o fibrinognio, protena
encontrada no sangue e relacionada com o mecanismo de
coagulao e a albumina, protena encontrada na maioria
dos tecidos animais e nos ovos das aves. No sangue,
constituinte do plasma (parte lquida), o qual desempenha
importante papel na regulao osmtica.
x Funo hormonal: vrios hormnios so de
constituio proteica, como por exemplo, a insulina.
x Funo de defesa: as protenas que realizam a
defesa do nosso organismo so denominados de anticorpos,
os quais iro atuar sobre os corpos estranhos conhecidos
como antgenos, causadores de vrias doenas.
x Funo energtica: o organismo obtm energia
para a realizao de vrias atividades a partir das molculas
de glicdios, lipdios e das protenas. Os aminocidos que
constituem as molculas de protenas passam a fazer parte
da respirao celular, sendo convertidos em ATP.
x Funo enzimtica: todas as enzimas so
protenas, as quais so fundamentais na acelerao das
reaes bioqumicas que ocorrem no organismo, como por
exemplo, as lipases, que transformam os lipdios em
unidades menores denominados cidos graxos, para serem
posteriormente utilizados pelas clulas.
x Conduo dos gases: a hemoglobina, presente nos
animais vertebrados, e a hemocianina, nos invertebrados,
so dois tipos de protenas responsveis pelo transporte do
gs carbnico e do oxignio no organismo dos animais.
Please purchase PDF Split-Merge on www.verypdf.com to remove this watermark.
Incluso para a Vida Biologia B

Pr-Vestibular da UFSC 5
UNIDADE 3

CIDOS NUCLEICOS: DNA e RNA

Somente em 1953, com os trabalhos de Watson e Crick foi
possvel compreender as vrias propriedades da molcula de
DNA, como por exemplo, a sua capacidade de
autoduplicao e de produzir molculas de RNA, alm de,
tambm, terem esclarecido a arquitetura do DNA.
Existem dois tipos de cidos nucleicos, o DNA (cido
desoxirribonucleico) e o RNA (cido ribonucleico). Os
cidos nucleicos so macromolculas constitudas por
milhares de unidades ligadas entre si, denominados
nucleotdeos. Cada nucleotdeo sempre composto por um
uma base nitrogenada, grupo fosfato e uma pentose.
Com relao pentose, as molculas de DNA so
constitudas pela desoxirribose e as molculas de RNA
constitudas pela ribose. Com relao s bases nitrogenadas
dos cidos nucleicos, existem 5 tipos diferentes: guanina;
adenina; citosina; timina e uracila. As duas primeiras
bases nitrogenadas so denominadas pricas e so
constitudas por um anel simples de carbono, ao passo que
as trs ltimas so denominadas pirimdicas e so
constitudas por um anel duplo de carbono.

ESTRUTURA
DA MOLCULA
DNA RNA
Bases nitrogenadas
Pentose
Tipo de filamento

A MOLCULA DE DNA
A molcula de DNA tambm denominada de ADN (cido
Desoxirribonucleico). Essa molcula formada por milhares
de nucleotdeos, os quais se encontram dispostos ao longo
de duas cadeias enroladas uma sobre a outra e de forma
helicoidal. Nas molculas de DNA, sempre a base prica de
um filamento liga-se por pontes de hidrognio base
pirimdica do outro filamento, como por exemplo: a adenina,
que prica, liga-se, por duas pontes de hidrognio,
timina, que pirimdica, ou vice-versa. J a guanina, que
tambm prica, liga-se citosina, que pirimdica, ou
vice-versa, atravs de trs pontes de hidrognio.


DUPLICAO DA MOLCULA DE DNA
Como se observa na figura acima, a estrutura da molcula de
DNA helicoidal e para que ocorra a sua duplicao
necessria a participao de algumas enzimas, para que
possa haver o desemparelhamento e o desenrolamento da
hlice dupla, abrindo-se a molcula. No processo de
duplicao da molcula de DNA, as pontes de hidrognio
entre as bases nitrogenadas se rompem e os dois filamentos
que constituam a hlice dupla comeam a se separar.
medida que as bases nitrogenadas vo se separando, os
nucleotdeos que se encontram dispersos na cariolinfa ou
lquido nuclear vo se unindo a cada um dos filamentos
rompidos, atravs da ao de uma enzima denominada DNA
polimerase, dando origem a um filamento complementar.
Entretanto, a unio dos nucleotdeos sempre ocorre entre
adenina com timina ou timina com guanina e, ainda, a
citosina com a guanina ou a guanina com a citosina.
Observa-se, entretanto, que cada um dos dois novos
filamentos formados apresentam um filamento original e um
filamento novo. Assim, dizemos que as molculas de DNA
apresentam uma duplicao semiconservativa.

A MOLCULA DE RNA
A molcula de RNA apresenta-se constituda por apenas um
nico filamento. Com relao pentose, a molcula de RNA
possui a ribose, e as bases nitrogenadas so: adenina,
citosina, guanina e uracila, no existindo no RNA a timina.
As molculas de DNA, alm de possurem a capacidade de
autoduplicao, tambm so capazes de produzir as
molculas de RNA, as quais recebero as mensagens do
DNA e passaro a comandar a sntese de uma determinada
protena no citoplasma celular.

A SNTESE DA MOLCULA DE RNA
Para que ocorra a formao da molcula de RNA
necessrio que haja o rompimento das pontes de hidrognio
que unem as bases nitrogenadas da molcula de DNA. Esse
rompimento pode ser total ou parcial, uma vez que pode
ocorrer a cpia de apenas uma parte da molcula de DNA.
Em seguida, ocorre a unio dos nucleotdeos pertencentes
molcula de RNA, num processo semelhante ao que ocorre
na duplicao do DNA. No entanto, onde havia a unio da
timina com a adenina na molcula de DNA, no RNA, a
timina no se faz presente e sempre ser a uracila que se
ligar a adenina. Alm disso, a unio dos nucleotdeos da
molcula de RNA s ocorre sobre uma das fitas do DNA,
denominada de fita-molde ou fita-ativa. Aps a formao
das molculas de RNA, estes migram para o citoplasma,
onde realizaro a sntese das protenas.



Formao das molculas de RNA
m
. Observe que para a
formao do RNA a uracila se liga a adenina, formando uma
fita nica.





Esquema da molcula
de DNA, a qual
formada por milhares de
nucleotdeos dispostos
ao longo de duas
cadeias enroladas, uma
sobre a outra, de forma
helicoidal. A letra A e
B representam as
bases nitrogenadas e a
letra C, a pentose
mais o grupo fosfrico.

Please purchase PDF Split-Merge on www.verypdf.com to remove this watermark.
Incluso para a Vida Biologia B

Pr-Vestibular da UFSC
6
UNIDADE 4

A SNTESE DE PROTENAS

Quando realizamos uma anlise bioqumica das clulas,
observamos que em todas elas existe uma variedade muito
grande de protenas, responsveis, inclusive, pelas
propriedades especficas de cada clula, como, por exemplo,
a contrao dos msculos, revestimento, transporte de gases,
etc. Portanto, as caractersticas de qualquer organismo vivo
dependem basicamente dos tipos de protenas que o
constituem.
As molculas de DNA possuem todas as instrues
para que ocorra a sntese (formao) das protenas. No
entanto, essas instrues no so transferidas diretamente da
molcula de DNA para a molcula proteica que est sendo
sintetizada. Existe, entretanto, a participao de uma
molcula intermediria, a qual atua como mensageira dessas
instrues. Na verdade, essa molcula mensageira um tipo
especial de RNA, denominado de RNA mensageiro (RNA
m
).
Este RNA
m
transportar a mensagem contida na molcula de
DNA e orientar a sntese de protenas. Sendo assim,
podemos dividir a sntese de protenas em duas etapas:
- a primeira a transcrio do cdigo gentico, que
consiste no processo de transferncia das instrues contidas
no DNA para o RNA;
- a segunda a traduo do cdigo gentico, que
consiste na sntese das protenas, orientando os tipos de
aminocidos que sero utilizados e, tambm, a posio que
estes devero ocupar na molcula proteica.
No captulo sobre as protenas, verificamos que na
constituio qumica das molculas proticas podem existir
at 20 tipos diferentes de aminocidos (lembre-se que as
protenas so formadas por muitos aminocidos). Se cada
uma destas letras (A), (T), (C) e (G) da molcula de DNA
fosse especfica para formao de um determinado tipo de
aminocido, consequentemente, todas as protenas
possuiriam apenas quatro tipos diferentes de aminocidos.
No entanto, os pesquisadores concluram que para uma
protena possuir os possveis 20 aminocidos em sua
estrutura molecular seria necessrio um cdigo trplice, ou
seja, que as bases nitrogenadas se agrupassem trs a trs,
possibilitando a codificao dos vinte aminocidos. Sendo
assim, cada sequncia de trs bases nitrogenadas da
molcula de DNA codifica a posio de um determinado
tipo de aminocido numa protena. Esta sequncia de trs
bases nitrogenadas recebe a denominao cdon.
Durante a sntese de protenas, observa-se a presena
de trs tipos de molculas de RNA, como: o RNA
mensageiro (RNA
m
), o RNA transportador (RNA
t
) e o RNA
ribossmico (RNA
r
). O RNA
m
formado a partir da
molcula de DNA, num processo denominado de transcrio
do cdigo gentico. Durante este processo, verifica-se a
presena da enzima RNA-polimerase unindo as bases
nitrogenadas adenina com a uracila e citosina com a guanina
e, em seguida, o destacamento da fita-molde da molcula de
DNA que lhe deu origem.
O RNA transportador (RNA
t
) tambm produzido
no ncleo, a partir de algumas molculas de DNA. Logo
aps a sua formao no ncleo da clula, o RNA
t
migra para
o citoplasma e passa a capturar aminocidos, transportando-
os, em seguida, para o RNA mensageiro (RNA
m
), o qual se
encontra associado aos ribossomos. O RNA
t
se caracteriza
por possuir poucos nucleotdeos e, sua molcula, apresenta-
se dobrada sobre si mesma. Alm disso, tambm possui uma
regio especfica para cada aminocido que ser
transportado at o RNA mensageiro (RNA
m
) e, de uma outra
regio, constituda por uma trinca de nucleotdeos,
denominada de anticdon. atravs do anticdon que o
RNA
t
reconhece o local no qual o RNA
m
dever colocar o
aminocido por ele transportado.
O RNA ribossmico (RNA
r
) tambm origina-se no
ncleo a partir do DNA, mas estes migram para o citoplasma
e associam-se a determinadas protenas, originando os
ribossomos. Esses ribossomos so responsveis pelo
acoplamento do RNA
t
, que transporta os aminocidos ao
RNA
m
, o qual possui o cdigo para a formao de uma
determinada protena.

Please purchase PDF Split-Merge on www.verypdf.com to remove this watermark.
Incluso para a Vida Biologia B

Pr-Vestibular da UFSC 7
AS VITAMINAS
As vitaminas so substncias orgnicas essenciais para o organismo e so obtidas somente atravs da alimentao. As principais
vitaminas, a sua utilizao pelo organismo, os sintomas de sua deficincia e a sua obteno, esto resumidos na Tabela 1.

VITAMINA UTILIZAO PELO
ORGANISMO
SINTOMAS DA
DEFICINCIA
FONTES
Tipo A necessria para o bom
funcionamento dos olhos, evita a
cegueira noturna e previne
resfriados.
cegueira noturna,
xeroftalmia e cegueira
total.
cenoura, abbora, milho, pssego
(vegetais amarelos em geral),
gema do ovo e fgado.
Tipo B
3
(niacina) mantm o estado normal de
resistncia e elasticidade do
tecido nervoso e muscular e o
bom funcionamento do sistema
digestivo.
nervosismo, problemas
digestivos, cansao.
carnes magras, ovos, figado e
leite.
Tipo B
1
(tiamina) mantm o estado normal do
tecido muscular e nervoso e
previne o beribri
beribri (fraqueza
muscular), perda de
apetite e nervosismo.
cereais, feijo, figado, carne de
porco, vegetais de folha
Tipo B
2
(Riboflavina) mantm a tonalidade saudvel da
pele e atua na coordenao
motora.
ruptura da mucosa bucal,
dos lbios e da lngua.
couve, repolho, espinafre
(vegetais de folha), carnes
magras e ovos.
Tipo B
6
(piridoxina) mantm a pele saudvel. doenas da pele,
distrbios nervosos e
cansao.
levedo de cerveja, cereais,
fgado, carnes magras.
Tipo C previne infeces, mantm a
integridade dos vasos e o
escorbuto (tendncia
hemorragia).
cansao, fadiga, insnia,
nas crianas ocorre
sangramento das
gengivas, dentes
alterados e escorbuto.
frutas ctricas como o limo, a
acerola e a laranja.
Tipo D mantm os ossos e os dentes em
bons estado e previne o
raquitismo.
ossos fracos, problemas
nos dentes e raquitismo.
gema de ovo e leo de fgado de
bacalhau.
Tipo E previne o aborto, promove a
fertilidade.
esterilidade e aborto. carnes, laticnios, alface .
Tipo K atua na coagulao do sangue e
previne as hemorragias.
Hemorragias. vegetais verdes, tomate e
castanha.

Exerccios de Sala #

1. (UFMS) Os organismos animais conseguem sintetizar a
maioria dos aminocidos. As reaes de sntese ocorrem
nas clulas do parnquima heptico. Porm, alguns
aminocidos no so sintetizados pelos animais. Em relao
a essas molculas, correto afirmar:
01. Os aminocidos naturais so aqueles produzidos no
organismo.
02. Os aminocidos essenciais so aqueles que devem ser
obtidos atravs da alimentao.
04. Nas protenas da carne, do leite e dos ovos encontram-
se todos os aminocidos essenciais, sendo, por isso,
considerados alimentos completos.
08. Os aminocidos so unidades dos cidos nucleicos.
16. Um elevado nmero de aminocidos pode se originar
por hidrlise de uma protena.

2. As protenas so substncias orgnicas de mltiplas
funes nos seres vivos, conforme sua forma na natureza,
elas podem ser classificadas como fibrosas ou globulares.
Somos diferentes uns dos outros, pois produzimos protenas
diferentes com funes plsticas, metablicas,
imunolgicas, energticas e outras.

01. As protenas so resultado de ligaes peptdicas entre
os aminocidos
02. A principal funo das protenas energtica, pois
liberam muitas calorias ao serem decompostas.
04. O colgeno uma protena fibrosa encontrada na pele
com a funo plstica.
08. As protenas pouco importam para o metabolismo, alm
de produzirem energia.
16. Quando dois aminocidos se ligam peptidicamente
sempre liberam uma molcula de gua.
32. Por no serem solveis na gua, as protenas no podem
ser transportadas pelo tecido sanguneo.
64. Os anticorpos so protenas que possuem propriedades
imunolgicas, ou seja, a capacidade de destruir corpos
estranhos ao nosso.

3. Ao mandar uma nave espacial a Marte, a NASA
produziu um rob Pathfinder para coletar amostras no
solo do planeta. Todos podemos observar ao vivo pela
lnternet ou pela TV a cabo as fotos do planeta vermelho.
Uma das substncias que o rob procurava eram as
enzimas, se estas fossem encontradas, seria a prova de que
pelo menos um dia teria existido vida em marte. Quanto ao
rob Pathfinder, sumiu!!!!

Please purchase PDF Split-Merge on www.verypdf.com to remove this watermark.
Incluso para a Vida Biologia B

Pr-Vestibular da UFSC
8
01. Toda a protena uma enzima, mas nem toda a enzima
uma protena.
02. As enzimas possuem a funo de diminuir a energia
necessria para que a reao ocorra.
04. As enzimas podem ser chamadas de catalisadores
biolgicos pois aceleram a velocidade das reaes
metablicas.
08. O pH nunca influencia a atividade enzimtica. pois as
enzimas tanto trabalham em meio cido como em meio
alcalino.
16. Ao elevar a temperatura de uma reao a velocidade da
atividade enzimtica duplica, triplica ou quadruplica at
um ponto timo. que depende do ser vivo.
32. Ao passar a temperatura do ponto timo, as enzimas
podem comear a perder suas propriedades catalticas e
dependendo da elevao da temperatura, as enzimas
podem ficar inativas ou at desnaturar.
64. Sempre que aumentamos o substrato numa reao, no
importa a quantidade, a velocidade da reao enzimtica
sempre aumenta.

4. Acreditamos que as principais causas de mortes por
doenas cardacas entre os humanos esto relacionadas com
a m alimentao que fazemos ao longo de nossas vidas.
Assinale a soma das alternativas corretas para os lipdeos e
suas funes nos seres vivos.
01. Os lipdeos so molculas de lcool ligadas a cidos
graxos.
02. Estas substncias podem possuir a funo energtica
quando se quebram as longas cadeias de carbono,
liberando energia para a formao de ATP.
04. Os fosfolipdios possuem a funo plstica ou
estrutural, pois formam a membrana citoplasmtica
lipoproteica.
08. As clulas preferem queimar os lipdeos antes dos
glicdios, assim elas gastam menos energia.
16. Por serem pssimos condutores de calor, os lipdios so
timos reguladores trmicos dos seres vivos.
32. Alguns lipdios podem assumir propriedades catalticas
e assim serem denominados de enzimas.
64. Por serem solveis na gua, os lipdios so facilmente
transportados pelo sangue.

5. Hoje sabemos que um Gene uma parte do ADN
(cido Desoxirribonucleico), tambm conhecido por DNA,
que decodifica uma protena, ou seja, possui a receita da
sequncia correta do encadeamento polipeptdico entre
aminocidos. O ADN resultado de uma sequncia pr-
determinada

de bases nitrogenadas, compostas por (T)
timina, (A) adenina, (C) citosina e (G) guanina. Na
transcrio, o ADN, que no pode sair do ncleo, serve de
molde para a fabricao do ARN (Acido Ribonucleico,
tambm conhecido como RNA), o qual constitudo de
uma sequncia de bases nitrogenadas (A, C, G) mais a (U)
uracila, no possuiindo (T). Os ribossomos ligados ao
retculo endoplasmtico ou soltos no citoplasma ligam-se
ao ARNm (mensageiro) do citoplasma e, transportando
aminocidos especficos, chegam aos ribossomos vrios
tipos de ARNt (Transportador). Para que os aminocidos se
liguem peptidicamente necessrio que a trinca de bases do
ARNm combine com a trinca de bases do ARNt. A trinca
de bases do ARNt denominada de anti-cdon, j a trinca
do ARNm e do ADN chamada de cdon.
Com as informaes do texto acima, assinale a soma das
alternativas corretas:
01. Podemos compreender atravs do texto acima, que um
gene ocupa toda a dupla tira do ADN.
02. Podemos afirmar, de acordo com o texto, que o ADN
no est envolvido com o material gentico transmitido
de pais para filhos.
04. Se uma protena possui 310 aminocidos, podemos
concluir que o ARNm que decodifica tal protena ir
possuir 310 cdons.
08. Para a protena da alternativa (04), o mesmo ARNm
ter 930 bases nitrogenadas.
16. Se um cdon do ARNm for (A-A-C), o anti-cdon do
ARNt que combina com este ser, (U-U-G).
32. Para o mesmo cdon do ARNm acima, existe um cdon
de ADN (T-T-G) do qual ele foi transcrito.
64. Outro ARNt que combinaria com o ARNm da
alternativa (16) teria um anti-cdon (T-T-G).

6. (UFMS) Os cidos nucleicos so as molculas mestras
da vida. Elas so responsveis pela sntese de todas as
enzimas que controlam, de alguma forma, a atividade
celular. Relacione os cidos nucleicos com suas
caractersticas.

I DNA
II RNA

A acar da molcula = desoxirribose
B acar da molcula = ribose
C presena de timina
D presena de uracila
E cadeia dupla
F cadeia simples
G capacidade de autoduplicao

Est(o) correta(s) a(s) associao(es):
01. I A 16. I F
02. II B 32. II E
04. II G 64. II D
08. I C

UNIDADE 5

CITOLOGIA

A clula a unidade bsica estrutural e funcional da vida de
todos os seres vivos e o ramo da biologia que estuda as
clulas a citologia.
A maioria das clulas apresenta medidas microscpicas,
mas algumas podem ser macroscpicas, como: a gema do
ovo, fibra de algodo, o Paramecium sp, alvolo da laranja,
entre outras.

PADRES DE ORGANIZAO DAS CLULAS
Todas as clulas seguem um padro inicial, apresentando
uma membrana celular delimitando um lquido interno do
meio externo. No entanto, internamente as clulas podem
ser: procariticas ou eucariticas.
As clulas procariticas no contm
compartimentos internos delimitados por membranas. O
Please purchase PDF Split-Merge on www.verypdf.com to remove this watermark.
Incluso para a Vida Biologia B

Pr-Vestibular da UFSC 9
DNA encontra-se disperso no citoplasma, no protegido por
membrana nuclear (carioteca), como se observa nas
bactrias e cianobactrias. J as clulas eucariticas so
clulas que apresentam todo um sistema de compartimentos
constitudo por membranas, como, por exemplo: os
protistas, fungos, plantas e animais.

FORMAS DAS CLULAS
As clulas diferem muito em suas formas, como pode ser
observado no corpo humano, por exemplo. A forma da
clula depende da funo exercida por ela.

Exerccios de Sala #

1. Sobre Citologia, some as afirmaes corretas:
01. Citologia uma cincia biolgica que estuda a unidade
estrutural e funcional dos seres vivos.
02. Todas as clulas so macroscpicas.
04. A maioria das clulas est abaixo de 100 micrmetros
de dimetro, logo, so microscpicas.
08. Algumas clulas so macroscpicas, tais como: Gema
do ovo, vulo humano, alvolo da laranja, alga nitela.
16. Um micrmetro a milsima parte de um milmetro e
um () ngstron a dcima-milionsima parte de um
milmetro.
32. Os aparelhos mais usados para visualizar clulas e suas
estruturas so os microscpios ptico e eletrnico.
64. As clulas so iguais em forma, consequentemente as
funes tambm so iguais.

2. (UFSC) Os procariontes, representados pelas bactrias e
pelas algas azuis:
01. Apresentam tecidos.
02. No apresentam ncleo verdadeiro.
04. So organismos inferiores na escala dos seres vivos.
08. So unicelulares.
16. Apresentam DNA como material gentico.
32. So parasitas obrigatrios.
64. No realizam meiose.

3. (UFSC) Bizzozero classificou as clulas em lbeis,
estveis e permanentes. Esta classificao permite que se
afirme:
01. O tecido nervoso e o floema, por serem altamente
especializados, apresentam clulas permanentes.
02. Tecidos de revestimento, como o epitelial, so formados
por clulas lbeis.
04. Tecidos de sustentao, como muscular e colnquima,
apresentam capacidade de regenerao, apresentam
clulas lbeis.
08. O meristema primrio, responsvel pela origem dos
demais tecidos vegetais, constitudo por clulas
estveis.
16. Os parnquimas vegetais e o tecido conjuntivo animal,
responsveis pelo preenchimento, so constitudos por
clulas estveis.
32. Os glbulos brancos, especializados na defesa em
alguns organismos animais, so clulas permanentes.

4. Some as alternativas com estruturas celulares exclusivas
de clulas vegetais:
01. Membrana plasmtica, citoplasma, ncleo.
02. Parede celulsica, vacolo de suco celular.
04. Complexo de Golgi, lisossomo, centrolos.
08. Plastos ou plastdios, membrana celulsica.
16. Ncleo, vacolo de suco celular e parede celulsica.
32. Citoplasma, retculo endoplasmtico, centrolos.

Tarefa Complementar #

5. (UFPR) A Escherichia coli no possui membrana
nuclear, tem as enzimas do processo respiratrio em dobras
da membrana plasmtica e no possui as estruturas
celulares mais conhecidas. Isso faz com que seja
classificada como:
a) clula procariota. d) metazorio.
b) clula eucariota. e) vrus.
c) protozorio.

6. (UFRGS) As clulas, segundo Bizzozero, so
classificadas em lbeis, estveis e permanentes. As lbeis
so pouco diferenciadas e, aps cumprirem suas funes,
so substitudas. As estveis so mais diferenciadas e com
capacidade de reproduo e de regenerao. As
permanentes, altamente diferenciadas e com funes muito
especializadas, no se reproduzem e nem se regeneram.
A partir dos dados descritos, indicar a alternativa correta
que exemplifica, respectivamente, clulas lbeis, estveis e
permanentes:
a) epiteliais; neurnios e hepticas.
b) hepticas; epiteliais e sseas.
c) neurnios; epiteliais e sseas.
d) sanguneas; sseas e nervosas.
e) sseas, hepticas e nervosas.

7. (UFRN) Das estruturas citadas abaixo, a nica que no
caracteriza uma clula vegetal :
a) lisossomo. d) vacolos de suco
celular.
b) parede celular. e) plastos.
c) plasmodesmos.

UNIDADE 6

FUNES E ESTRUTURAS DAS CLULAS
EUCARITICAS

MEMBRANA PLASMTICA
Presente nas clulas vegetais e animais, a membrana
plasmtica tambm chamada de membrana celular,
membrana citoplasmtica ou plasmalema. A membrana se
caracteriza por ser uma pelcula delgada e elstica que
envolve todas as clulas, revestindo e separando o
citoplasma e as organelas celulares do meio externo.
Funcionando como uma barreira seletiva facilitando ou
dificultando a entrada de substncias que interessam
clula, a membrana possui uma permeabilidade seletiva.
Medindo em mdia 75 , a membrana s pode ser
visualizada ao microscpio eletrnico. Em 1972, S. J.
Singer e G. L. Nicholson elaboraram o atual modelo para a
estrutura da membrana: o modelo do mosaico fluido. Este
modelo prope que a membrana composta por trs tipos
de molculas, lipdeos (fosfolipdeos e colesterol),
Please purchase PDF Split-Merge on www.verypdf.com to remove this watermark.
Incluso para a Vida Biologia B

Pr-Vestibular da UFSC
10
protenas (globulares) e uma pequena frao de glicdios,
que nas clulas animais podem estar aderido aos dois
primeiros, formando os glicolipdios e as glicoprotenas,
que juntas formam o glicoclice.


PAREDE CELULAR
Junto membrana citoplasmtica das clulas vegetais existe
um reforo externo formado por celulose (polissacardeo
composto por 4.000 molculas de glicose), sendo tambm
denominada parede celulsica. Esta parede porosa,
permitindo a livre passagem de gua.

TRANSPORTES ATRAVS DA MEMBRANA
CITOPLASMTICA
Os transportes atravs da membrana podem acontecer sem
que ocorra gasto de energia (ATP) ou com gasto de energia.
No primeiro processo, o transporte denominado
TRANSPORTE PASSIVO e, no segundo caso, em que h
gasto de energia, denominado TRANSPORTE ATIVO.

TIPOS DE TRANSPORTES PASSIVOS

DIFUSO - o deslocamento de substncias do local de
maior concentrao para o local de menor concentrao e
podem ser do seguinte tipo:
- Difuso Simples - quando a substncia passa pela
bicamada de fosfolipdios da membrana plasmtica. Passam
facilmente substncias lipossolveis, tais como: oxignio,
monxido e dixido de carbono.
- Difuso Facilitada substncias polares como a maioria
dos aminocidos, acares, ons no se difundem pela
bicamada lipdica. Esses elementos tero que atravessar a
membrana com o auxlio de protenas porosas ou protenas
carregadoras. As protenas integrais camada lipdica
formam canais, os quais se abrem para a passagem dos
referidos elementos.
- Osmose - a passagem de solventes do meio menos
concentrado em soluto (hipotnico) para o meio mais
concentrado (hipertnico). Este mecanismo ocorre onde h
uma membrana com poros que s permitem a passagem de
molculas de solvente. A Osmose pode ser facilmente
observada ao colocarmos hemcias e clulas vegetais em
solues com concentrao de diferentes solutos. As
hemcias quando colocadas em soluo hipertnica
murcham, perdem H
2
O e sofrem plasmlise. Ao serem
colocadas em solues hipotnicas, ganham H
2
O e podem
sofrer plasmoptise (ou hemlise no caso das hemcias).
Clulas de vegetais se caracterizam por apresentarem
parede celulsica. Essa estrutura porosa, portanto
permevel aos solventes e aos solutos. No entanto, a
membrana plasmtica que seleciona as substncias que
devero penetrar nas clulas. Sendo assim, clulas vegetais,
quando colocadas em soluo hipertnica, observa-se uma
ntida perda de volume do vacolo e do citoplasma,
ocasionando um afastamento da membrana plasmtica da
parede celulsica. Em soluo hipotnica, a clula vegetal
ao ganhar solvente, passa a apresentar um aumento do
vacolo e do citoplasma, fazendo com que a membrana
plasmtica encoste-se parede celulsica. Ao contrrio do
que se observa nas hemcias, a clula vegetal no sofre
plasmoptise.



TRANSPORTES ATIVOS
Nestes tipos de transporte, as substncias deslocam-se do
meio de menor concentrao para o meio de maior
concentrao. Nestes casos, a clula ou a membrana
plasmtica gasta energia (ATP) para movimentar a
substncia. Alguns exemplos seriam:
- Bomba de Sdio e Potssio (Na
+
e K
+
) - constituda
por protenas que removem o Na
+
para o lado externo
da clula e devolvem o K
+
para o interior das clulas.
- Protenas que acoplam o Ca
++
na membrana
plasmtica e no retculo endoplasmtico. A
concentrao de Ca
++
fora da clula muito alta e
tende a penetrar nas clulas. H protenas
transportadoras que transportam o Ca
++
contra um
gradiente inico e de concentrao.
- As clulas intestinais absorvem ativamente
aminocidos e Na
+
atravs da mesma protena
transportadora da membrana plasmtica.

ENDOCITOSE
Protenas, cidos nucleicos e polissacardeos, so grandes
molculas polares que no conseguem atravessar a
membrana, mesmo gastando energia. Algumas clulas
especializaram-se em transportar estes tipos de molculas
atravs da Fagocitose, Pinocitose, Exocitose.

FAGOCITOSE
Macromolculas fusionam-se membrana plasmtica e so
englobadas. Durante esse processo, observa-se que a
membrana invagina-se e forma uma vescula (estrutura que
lembra uma bolsa) com as macromolculas em seu
interior, as quais sofrero a ao de enzimas digestivas. As
substncias aproveitveis pela clula sero assimiladas e as
substncias no desejveis sero eliminadas.

PINOCITOSE
Na pinocitose, as macromolculas so sempre dissolvidas
em um solvente. Como na fagocitose a membrana invagina-
se, forma uma vescula e as substncias aproveitveis pela
clula sero assimiladas e as substncias no desejveis
sero eliminadas. Esse processo observado nos capilares
Osmose em hemcias e clulas vegetais
Please purchase PDF Split-Merge on www.verypdf.com to remove this watermark.
Incluso para a Vida Biologia B

Pr-Vestibular da UFSC 11
sanguneos e tecido adjacente, os quais captam e
eliminarem rapidamente os seus fludos.

EXOCITOSE
a eliminao de grandes substncias pelas clulas. As
substncias a serem eliminadas so envolvidas pela
membrana, a qual se fusiona originando um canal, o qual
possibilita a eliminao dos produtos no aproveitveis pela
clula, durante a fagocitose e a pinocitose.

UNIDADE 7

CITOPLASMA

O espao compreendido entre o ncleo e a membrana
plasmtica preenchido por uma massa coloidal, na qual se
encontram suspensas as estruturas intracelulares, e
denominado citoplasma. O hialoplasma a parte do
citoplasma mais prxima ao ncleo. nessa regio que se
encontram a maioria dos organoides (Retculo
Endoplasmtico, Ribossomo, Complexo de Golgi,
Lisossomo, Centrolos, Mitocndrias). O coloide formado
por milhares de micelas proteicas suspensas na gua e
apresenta um movimento constante (movimento
Browniano), o qual impede a precipitao da suspenso
coloidal. A maior ou menor concentrao destas protenas
proporciona o surgimento de um plasma-gel, (mais
protenas, mais gelatinoso) e plasma-sol, (menos protenas,
mais fludo). O plasma sol mais interno, da a
denominao de endoplasma; o plasma-gel mais externo-
ectoplasma.
No hialoplasma tambm existem inmeras fibras
formadas de actina e miosina, protenas contrteis que
formam o citoesqueleto das clulas. Estas protenas esto
constantemente em contrao e relaxamento provocando
um deslocamento contnuo do citoplasma como um todo.
Neste fluxo de citoplasma os organoides tambm se
deslocam ativa ou passivamente promovendo a CICLOSE.

RETCULO ENDOPLASMTICO
As clulas eucariticas possuem uma continuao da
carioteca (membrana nuclear) formada por um sistema de
membranas duplas de lipoprotenas em forma de rede de
tbulos ou sacos achatados. Seu aspecto e distribuio
variam de clula para clula, mas abundante nas clulas
secretoras e ausente nas hemcias adultas de mamferos.
Esta estrutura s pode ser observada ao
microscpio eletrnico, devido a sua espessura, que
semelhante ao da membrana plasmtica, em torno de 75 .
O retculo endoplasmtico pode ser de dois tipos:
- Reticulo Endoplasmtico Rugoso - tambm conhecido
como granular ou ergastoplasma, possui ribossomos
aderidos e participa da sntese de protenas;
- Reticulo Endoplasmtico Liso - no possui
ribossomos, participando da sntese de lipdeos e
polissacardeos.
Alm de transportar substncias no interior da clula,
o retculo facilita as reaes enzimticas. O retculo liso
tambm promove a sntese triglicerdeos, fosfolipdios e
esteroides, os quais so bem desenvolvidos em clulas das
suprarrenais e das gnadas que secretam hormnios
esteroides. Alm disso, armazenam temporariamente
diferentes tipos de substncias como: enzimas, protenas e
lipdeos e regulam a presso osmtica por armazenar
substncias como o Ca
++
, modificando a concentrao
deste elemento no hialoplasma.


RIBOSSOMOS
Podem ser encontrados livres, presos ao retculo
endoplasmtico ou ligados entre si por uma fita de RNA,
sendo, neste caso, denominados de polissomos ou
polirribossomos. Os ribossomos so constitudos por duas
subunidades de RNA ribossmico e protenas e
fundamentais na sntese de protenas, estando, portanto,
presente em todas as clulas. Na forma de polissomos,
fabricam protenas para a prpria clula e quando h
necessidade de produzir protenas para fora da clula, estas
so produzidas junto ao retculo endoplasmtico rugoso.

COMPLEXO GOLGIENSE
Conjunto de vesculas ou sculos achatados lipoproteico
proveniente de expanses do retculo endoplasmtico liso
presente apenas nas clulas eucariticas. Pode ser
visualizado ao microscpio ptico e possuem como
funes:
- Armazenamento de protenas sintetizadas pelo retculo
endoplasmtico rugoso, as quais so armazenadas nas
vesculas do complexo golgiense. A partir destas
vesculas brotam outras vesculas menores repletas de
enzimas denominadas lisossomos primrios;
- Sntese de mucopolissacardeos, os quais so
polimerizados no complexo golgiense e transformado
em polissacardeos, os quais associam-se as protenas
oriundas do retculo endoplasmtico rugoso formando
as glicoprotenas, as quais posteriormente sero
eliminadas pela clula;
- Sntese de lipdeos, como o colesterol, hormnios
sexuais, cortisol, estrgeno, progesterona, testosterona;
- Nas clulas vegetais observa-se entre as membranas o
dictiossomos, os quais so bolsas do complexo
golgiense que polimerizam monossacardeos formando
a celulose, constituindo a parede celulsica das clulas
vegetais.

LISOSSOMOS
So pequenas bolsas de membrana lipoproteica contendo no
seu interior enzimas digestivas e presente apenas nas
clulas eucariticas. Estas vesculas surgem a partir de
dobras do retculo endoplasmtico e do Complexo
golgiense, sendo observado somente microscpio
eletrnico. Apresentam as seguintes funes:
HETEROFAGIA - nos casos de fagocitose e pinocitose,
formando-se dentro da clula o fagossomo e o pinossomo,
Please purchase PDF Split-Merge on www.verypdf.com to remove this watermark.
Incluso para a Vida Biologia B

Pr-Vestibular da UFSC
12
respectivamente. Juntam-se ao fagossomo e pinossomo
inmeros lisossomos que iro constituir o lisossomo
secundrio ou vacolo digestivo. As enzimas passam ento
a interagir com os produtos englobados e as substncias
aproveitveis pela clula sero eliminadas pelo vacolo,
sobrando em seu interior apenas resduos da digesto,
denominados de corpo residual ou vescula de
clasmocitose, que ser eliminada pela clula, atravs da
exocitose.
AUTOFAGIA - quando uma estrutura intracelular deixa de
exercer sua funo, ocorre a formao de um vacolo como
fagossomo, originando lisossomos que provocaro a
digesto.
AUTLISE - com a morte celular observa-se o aumento
significativo de lisossomos que rompem a sua membrana
liberando sobre toda a clula suas enzimas. So exemplos
da autlise a involuo da cauda do girino. A destruio das
clulas cardacas no enfarte do miocrdio ou outra regio
enfartada.

PEROXISSOMOS
So vesculas que contm enzimas, como a peroxidase e a
catalase, as quais so produzidas no retculo
endoplasmtico rugoso e participam do metabolismo dos
perxidos (gua oxigenada) que so formados pelo
metabolismo celular. So encontrados tanto em clulas
animais, como vegetais e no homem so muito
desenvolvidos nos hepatcitos, pois estas clulas tm um
importante papel na metabolizao de molculas txicas,
como o lcool.

MITOCNDRIAS
So organelas que apresentam forma de gros ou bastes,
sendo observadas em clulas aerbias e responsveis pela
respirao celular (processo responsvel pela produo de
energia). As mitocndrias podem ser visualizadas ao
microscpio ptico, porm seus detalhes, somente ao
microscpio eletrnico.
Constitudas por uma dupla membrana lipoproteica, a parte
interna forma dobras ou septos chamados cristas
mitocondriais, que contm enzimas aceptoras de eltrons,
fundamentais para que ocorra o processo de respirao
celular. Entre estas cristas h uma soluo coloidal
semelhante citoplasma, a matriz mitocondrial, a qual
apresenta diversas enzimas respiratrias, DNA e RNA
prprios, alm de ribossomos. Por isso, as mitocndrias
possuem autonomia reprodutiva e so considerada
organismos primitivos que passaram a viver de forma
simbitica com as clulas eucariticas atuais. Ao conjunto
de mitocndrias d-se o nome de condrioma e quanto maior
for o metabolismo celular, maior ser a quantidade de
mitocndria para produzir energia.

CENTROLOS
Estrutura no membranosa do centro celular ou
centrossomo. Ao redor dos centrolos partem fibrilas que
constituem o ster, o qual se caracteriza por ser um
conjunto de fibrilas proteicas de tubulina que apresentam
capacidade de encurtamento e alongamento proporcionando
a migrao dos cromossomos para os plos durante a
diviso celular e a formao clios e flagelos.
Os centrolos so duas formaes cilndricas dispostas
perpendicularmente, em que cada cilindro formado por 9
grupos de 3 micro-tbulos dispostos concentricamente.
Estes microtbulos so de constituio proteica, os quais se
autoduplicam, podendo originar outro par de centrolos.

PLASTOS
So organoides encontrados em clulas vegetais, algas e
certos protozorios com funo de armazenamento de
substncias de reserva e pigmentos fotossintetizantes.
Podem ser visualizados ao microscpio ptico, podendo
assumir diversas formas: espiralizada, estrelada, lenticular,
esfrica.
Tipos de plastos:

- LEUCOPLASTOS: No possuem pigmentos, mas
acumulam substncias de reserva nutricional.
- CROMOPLASTOS: So os plastos que sob luminosidade
sintetizam pigmentos.

ESTRUTURA DOS CLOROPLASTOS - delimitados por
duas membranas lipoproteicas, sendo a externa lisa e a
interna formando dobras denominadas lamelas. Em
determinados locais estas lamelas sofrem vrios
dobramentos sobrepostos, sendo o conjunto destes
dobramentos, em todo o cloroplasto, denominado grana. A
unidade do grana que corresponde a uma nica pilha de
dobras o granum e cada dobra de membrana um
tilacoide. No interior das membranas do tilacoide so
encontradas milhares de molculas de clorofila,
responsveis pela fotossntese.

Exerccios de Sala #

1. (UFSC) Observe a lista dos componentes celulares
abaixo.
COMPONENTES CELULARES
A
B
C
D
E





Membrana
Plasmtica
Parede Celular
Cloroplasto
Lisossomos
Vacolo
F
G
H
I




Carioteca
Retculo
Endoplasmtico
Complexo de Golgi
Ribossomos

Assinale a(s) proposio(es) que estabelece(m) a(s)
relao(es) correta(s) entre as clulas dos diferentes
grupos de seres vivos e os componentes celulares.
01. Clula animal: B
Clula bacteriana: A, C
02. Clula animal: I
Clula vegetal: G, E
04. Clula animal: F
Clula bacteriana: D, G
08. Clula animal: A
Clula vegetal: G
Clula bacteriana: I
16. Clula bacteriana: E, F
Clula vegetal: H
32. Clula animal: C
Clula bacteriana: E, D
64. Clula animal: B, F
Clula bacteriana: D

Please purchase PDF Split-Merge on www.verypdf.com to remove this watermark.
Incluso para a Vida Biologia B

Pr-Vestibular da UFSC 13
2. (UFSC) Para sobreviver, as clulas precisam obter do
ambiente que as cerca, nutrientes como o acar,
aminocidos, sais, ons, cidos graxos, etc. Existem vrios
processos pelos quais as clulas obtm substncias do meio.
Assinale a(s) proposio(es) que correlacionam
corretamente o processo e sua(s) caracterstica(s):

1) difuso. 2) transporte ativo.

a) gasto de ATP.
b) movimento contra o gradiente de concentrao.
c) movimento a favor do gradiente de concentrao.
d) presena de catalisadores.
e) passagem de substncias atravs da membrana
plasmtica.

01. 1-C 08. 1-E 64. 2-E
02. 2-A 16. 2-D
04. 2-B 32. 1-A

3. (UFSC) A membrana plasmtica uma membrana
Semipermevel, no havendo condies, norma-mente,
para o extravasamento dos coloides citoplasmticos para
fora da clula. Sob esse aspecto, a membrana j comea a
selecionar o que deve entrar na clula ou dela sair.
Considerando os diferentes processos de passagem atravs
da membrana plasmtica, correto afirmar que
01. a osmose a passagem de molculas de gua, sempre
no sentido do meio mais concentrado para o menos
concentrado.
02. na difuso facilitada, participam molculas especiais,
de natureza lipdica e h gasto de energia.
04. no transporte ativo, enzimas agem como
transportadoras de molculas, tais como o acar, ou
ons.
08. a fagocitose um tipo de endocitose, onde ocorre o
englobamento de partculas slidas.
16. a pinocitose outro tipo de endocitose, ocorrendo,
neste caso, o englobamento de pequenas pores de
substncias lquidas.
32. pela exocitose, substncias inteis clula so
eliminadas com o auxlio dos centrolos.

4. (UFSC) Estudos preliminares em mineiros da regio
carbonfera de Cricima tm apresentado resultados
preocupantes com relao pneumoconiose, que uma
afeco pulmonar, provocada pela inalao de poeira do
carvo e de outros minrios. Essa uma doena ligada
leso da membrana lisossmica. Com relao aos
lisossomos, assinale a(s) alternativa(s) correta(s).
01. So estruturas nucleares.
02. Origina-se a partir do Complexo de Golgi.
04. So ricos em enzimas.
08. So os responsveis pela digesto intracelular.
16. Em clulas vegetais auxiliam o processo fotossinttico.
32. Ao unirem-se aos fagossomos formam vacolos
digestivos.

5. (UFSC) Em relao ocorrncia, origem, estrutura e
funo das organelas citoplasmticas, assinale a(s)
proposio(es) verdadeira(s).
01. Os vacolos pulsteis ocorrem em alguns Protistas e participam da
manuteno do equilbrio homeosttico.
02. O Complexo de Golgi existe em abundncia nas clulas
secretoras e participa da sntese de aminocidos.
04. As mitocndrias so formadas de enzimas oxidantes e
participam do processo de desintoxicao celular.
08. Os lisossomos originam-se do ergastoplasma (RER) e
do Complexo de Golgi e participam do processo de
respirao celular.
16. Os vacolos do suco celular so exclusivos das clulas
vegetais, sendo pequenos e numerosos nas clulas
jovens e geralmente nico na clula adulta.
32-. Os plastos so organelas citoplasmticas que ocorrem
em todos os vegetais e em todos os Protistas.
64. Os centrolos coordenam o processo de diviso
cromossmica.

Tarefa Complementar #

6. (UFSC) Os lisossomos so organoides membranosos,
com formato esfrico, que contm enzimas digestivas. Em
relao a essa estrutura citoplasmtica, assinale a(s)
proposio (es) correta(s).
01. Os lisossomos desempenham, entre outras, funes de
defesa celular.
02. As enzimas lisossmicas so fabricadas no retculo
endoplasmtico liso, passando em seguida para o
sistema de Golgi, que as empacota e as libera sob a
forma de lisossomos secundrios.
04. A funo heterofgica dos lisossomos refere-se
digesto de substncias que so absorvidas pela clula
por fagocitose ou pinocitose.
08. O lisossomo secundrio formado pela fuso do
vacolo alimentar, que contm o alimento englobado
por pinocitose ou fagocitose, com o lisossomo
primrio, que contm as enzimas digestivas.
16. Juntamente com as mitocndrias, os lisossomos so
responsveis por uma reciclagem de molculas e
organoides inativos.
32. Em girinos, o fenmeno de reabsoro da cauda
comparado a um suicdio celular j que, com o
rompimento dos lisossomos, ocorre uma autodigesto
das molculas e dos organoides que constituem as
clulas daquela estrutura.

7. (UFSC) Os cientistas Gerald Schatten e Peter Sutovsky,
..., descobriram que as mitocndrias (...) de
espermatozides so destrudas aps fertilizarem os
vulos.
Texto extrado da Revista Cincia Hoje, 27(158): maro/
2000. P. 12.

Com relao ao assunto acima exposto, correto afirmar
que:
01. As mitocndrias so organelas responsveis pela
produo de energia nas clulas.
02. As mitocndrias dependem do DNA do ncleo das
clulas a que pertencem para se multiplicarem.
04. O fenmeno acima descrito explica por que os
mamferos s herdam o DNA mitocondrial do lado
materno.
08. As mitocndrias, por conterem seu prprio DNA,
tambm participam da lise nas clulas e nos tecidos.
16. As mitocndrias dos vegetais possuem clorofila em sua
constituio.
Please purchase PDF Split-Merge on www.verypdf.com to remove this watermark.
Incluso para a Vida Biologia B

Pr-Vestibular da UFSC
14
32. As mitocndrias exercem uma funo denominada
respirao celular, que produz ao final o ATP,
molcula altamente energtica.
64. Nas clulas eucariticas primitivas, as mitocndrias
esto ausentes.

UNIDADE 8

NCLEO

Estrutura celular identificada por Robert Brown em 1831,
em clulas da epiderme de orqudeas, o ncleo detm em
sua constituio as informaes genticas que sero
transmitidas s clulas filhas ao se reproduzirem. Alm
disso, o ncleo o centro de controle de todas as atividades
que acorrem nas clulas.

IMPORTNCIA DO NCLEO
x do ncleo que parte todo o comando do metabolismo
celular, pois nenhuma protena sintetizada sem a
participao das molculas de DNA presente no ncleo,
determinando assim, os aspectos metablicos e
morfolgicos de uma clula;
x Em clulas-ovo ou zigotos, das quais o ncleo foi
removido, observa-se que as clulas no podem se dividir,
sempre acabam morrendo;
x Quando o ncleo removido de organismos unicelulares,
estes sobrevivem muito pouco e acabam morrendo. No
entanto, se o ncleo de outra bactria for transplantado, este
organismo se mantm vivo.
x Clulas anucleadas, como as hemcias de mamferos,
apresentam um perodo de vida curto, por volta de 120 dias.

VARIAES NO NMERO E NA FORMA DO
NCLEO CELULAR
A maioria das clulas apresenta um nico ncleo, mas os
protozorios ciliados, por exemplo, possuem dois ncleos:
um com pequeno tamanho, denominado microncleo, e
outro maior, o macroncleo.
Algumas clulas so multinucleadas, como as
clulas musculares estriadas esquelticas e outras, no
apresentam ncleo, como as hemcias. Geralmente, o
ncleo esfrico e mantm uma relao direta com o
formato da clula. Contudo, alguns leuccitos (glbulos
brancos), possuem formato riniforme (forma de rins) e as
algumas clulas musculares apresentam formato fusiforme.

COMPONENTES DO NCLEO
Um ncleo eucarionte apresenta as seguintes estruturas:

1 - CARIOTECA
A carioteca, tambm denominada de membrana nuclear ou
cariomembrana, um envoltrio constitudo por duas
membranas lipoproteicas, visveis apenas ao microscpio
eletrnico. Uma membrana da carioteca est em contato
direto com o citoplasma, apresentando ribossomos aderidos
superfcie e formando o retculo endoplasmtico rugoso.
A outra membrana fica em contato com o interior do ncleo
envolvendo o material gentico e, entre estas duas
membranas existe um espao denominado perinuclear.
Alm disso, a carioteca perfurada por milhares de poros,
atravs dos quais determinadas substncias entram e saem
do ncleo.

2 - CARIOLINFA
A cariolinfa, tambm denominada de carioplasma ou suco
nuclear, o lquido que preenche o espao interno do
ncleo e que mantm suspensas as estruturas intranucleares,
como a cromatina e os nuclolos. Alm disso, a cariolinfa
apresenta como principais caractersticas o pH variando
entre 7,6 a 7,8, alguns lipdios, sais minerais, enzimas,
protenas, glicdios e gua, logo, uma composio qumica
semelhante ao hialoplasma.

3 - CROMATINA
A cromatina observada quando a clula no se encontra
em processo de diviso celular, numa fase denominada de
intrfase. Nesta fase, a cromatina se apresenta como sendo
um conjunto de filamentos formado por uma longa
molcula de DNA associada a molculas proteicas
chamadas de histonas. Cada um destes filamentos recebe o
nome de cromonema e quando a clula entrar no processo
de diviso celular, estes filamentos iro se condensar e se
espiralizar, transformando-se em cromossomos.
Ao usar corantes bsicos numa clula em intrfase,
possvel observar ao microscpio ptico que certas regies
deste conjunto de filamentos (cromatina) coram mais
intensamente do que outras. Estas regies mais coradas
correspondem a pores dos filamentos mais enroladas e
mais condensados e denominam-se de heterocromatina. As
regies menos coradas correspondem a regies distendidas
dos filamentos e so chamadas de eucromatina.

4 - NUCLOLOS
Durante a intrfase, perodo de no diviso celular,
possvel observar corpos esfricos ou ovais, constitudos de
RNA associado a protenas e, tambm, de um pouco de
DNA, o qual proveniente de um cromossomo denominado
cromossomo organizador do nuclolo.
No nuclolo so produzidas molculas de RNA
ribossmico, que se associam a protenas para formar as
subunidades que constituem os ribossomos, as quais iro
migrar para o citoplasma e se tornaro ativas na sntese de
protenas.

Exerccios de Sala #

1. (UFSC) O ncleo uma estrutura que coordena e
comanda todas as funes celulares.
Assinale a(s) proposio(es) que apresenta(m) relaes
corretas entre as estruturas nucleares, sua ocorrncia e
caractersticas qumicas ou funcionais.
01. Ao observarmos o ncleo interfsico em microscpio
ptico, verificamos a total compactao da cromatina,
que passa a chamar-se cromossomo.
02. A membrana nuclear apresenta poros ou annuli,
atravs dos quais ocorrem importantes trocas de
macromolculas entre ncleo e citoplasma.
04. A carioteca corresponde ao fluido onde esto
mergulhados os cromossomos e as estruturas que
formam o nuclolo.
08. O nuclolo, mergulhado no nucleoplasma, est sempre
presente nas clulas eucariticas, podendo haver mais
de um por ncleo.
Please purchase PDF Split-Merge on www.verypdf.com to remove this watermark.
Incluso para a Vida Biologia B

Pr-Vestibular da UFSC 15
16. O nuclolo uma regio de intensa sntese de RNA
ribossmico (RNAr).
32. A cromatina formada por uma nica e longa molcula
de RNA, associada a vrias molculas de
glicoprotenas.

2. Sobre ncleo eucaritico, em intrfase, some as
alternativas corretas:
01. Todas as clulas apresentam ncleo eucaritico.
02. Est presente em todas as fases da clula, inclusive
durante a diviso da celular.
04. A maioria das clulas apresenta apenas um ncleo, mas
h clulas multinucleadas como as musculares estriadas
esquelticas.
08. As hemcias so clulas anucleadas e, por este motivo,
apresentam vida curta.
16. Todas as clulas apresentam ncleo esfrico.
32. As estruturas do ncleo eucaritico em intrfase so:
carioteca, cariolinfa, nuclolos e cromatina.

3. Sobre nuclolos, some as alternativas corretas:
01. Um ncleo pode apresentar mais de um nuclolo.
02. O nuclolo verdadeiro pode ser tambm denominado
plasmossomo.
04. O plasmossomo constitudo por RNA ribossmico,
enzimas, nucleoprotenas e sais.
08. O cariossomo (nuclolo falso) um adensamento maior
de heterocromatina, logo, com DNA.
16. O plasmossomo apresenta reao de Feulgen positiva,
pois apresenta DNA.

CROMOSSOMOS
Como j foi salientada, a fase da vida de uma clula que
no se encontra em diviso denominada de intrfase.
Nesta fase, pode-se observar no interior do ncleo um
conjunto de filamentos denominado cromatina. Cada um
destes filamentos, durante a diviso celular, sofre um
processo de espiralizao e condensao, tornando-se mais
espesso e, consequentemente, transformando-se em
cromossomos. Sendo assim, os cromossomos tambm
apresentam a mesma constituio qumica da cromatina, ou
seja, so filamentos formados por uma longa molcula de
DNA associada a molculas proteicas (histonas). Alm
disso, um cromossomo pode ser definido, geneticamente,
como sendo uma srie linear de genes (segmentos de
DNA), os quais controlam todo o metabolismo celular,
atravs da produo de protenas.

CLASSIFICAO DOS CROMOSSOMOS
xCromossomos metacntricos: quando o
centrmero se localiza na regio central do cromossomo,
ficando os dois braos com o mesmo tamanho.
xCromossomos submetacntricos: quando o
centrmero fica um pouco deslocado da regio central do
cromossomo, ficando os dois braos com tamanhos
desiguais.
xCromossomos acrocntricos: quando o
centrmero se localiza muito prximo de uma das
extremidades do cromossomo, ficando um dos braos com
um tamanho bem menor do que o outro.
xCromossomos telocntricos: quando o centrmero
se localiza numa das extremidades, ficando o cromossomo
com apenas um nico brao.

CROMTIDE
Em determinados momentos da diviso celular, os
cromossomos se encontram com o formato de uma letra
X e passam a ser denominados de cromossomos
duplicados. Esta duplicao ocorre na fase de intrfase e
cada lado longitudinal dos cromossomos duplicados, passa
a ser chamados de cromtides-irms, ou simplesmente de
cromtides

CROMOSSOMOS HOMLOGOS
As clulas da espcie humana possuem 46 cromossomos,
dos quais, 23 so provenientes do espermatozoide e 23 do
vulo. Estes 46 cromossomos apresentam-se em pares, com
as seguintes caractersticas:
- mesmo tamanho;
- mesma posio do centrmero;
- mesmo loci gnico (localizao dos genes);
- um de origem paterna, outro de origem materna.

CROMOSSOMOS AUTOSSOMOS E
HETEROSSOMOS
Dos 23 pares de cromossomos encontrados nas clulas da
espcie humana, 22 pares determinam caractersticas no
sexuais e so denominados de cromossomos autossomos.
Estes cromossomos so representados pela letra A,
quando a clula for haploide (por exemplo: os gametas) ou
por 2A, quando a clula for diploide como, por exemplo,
as clulas que formam o nosso corpo, conhecidas como
somticas.
Enquanto 22 pares de cromossomos so autossomos,
um nico par, que pode ser XX ou XY, chamado de
cromossomo sexual ou heterossomo ou alossomo e
manifesta as caractersticas sexuais do indivduo. O par de
cromossomos XX manifesta todas as caractersticas
femininas e o par XY, as masculinas.

GENOMA E CARITIPO
O genoma o conjunto haploide de cromossomos que uma
clula possui. Sendo assim, nas clulas gamticas existe um
nico genoma e nas clulas que formam o nosso corpo, as
denominadas somticas, apresentam dois genomas.
O caritipo constitui uma anlise dos cromossomos de uma
clula diploide, com relao: ao tamanho, posio do
centrmero e quantidade. A organizao destes
cromossomos aos pares e em ordem decrescente, ou seja,
do par maior para o menor, recebe o nome de idiograma.

Tarefa Mnima #

4. (UFC) O caritipo consiste na montagem fotogrfica, em
sequncia, de cada um dos tipos cromossmicos. Ele nos
permite saber qual o nmero e qual a forma dos
cromossomos de uma espcie, bem como estabelece o seu
padro cromossmico normal. A partir da anlise da figura
abaixo, e em relao a esse estudo, correto afirmar que:
Please purchase PDF Split-Merge on www.verypdf.com to remove this watermark.
Incluso para a Vida Biologia B

Pr-Vestibular da UFSC
16


01. O caritipo o quadro cromossmico das clulas
haploides de cada espcie.
02. Na espcie humana, os cromossomos so classificados
em 7 grupos, compreendendo 22 pares de cromossomos
autossmicos, e mais um par de cromossomos sexuais
que, no homem, XY e, na mulher, XX.
04. Para a obteno do caritipo, so utilizadas clulas de
leuccitos em anfase meitica.
08. Em fetos, normalmente a cariotipagem s deve ser feita
quando h real suspeita de algum tipo de alterao
cromossmica, j que as tcnicas de coleta de material
apresentam risco de aborto.
16. A partir da anlise de caritipos, informaes valiosas
podem ser obtidas, tais como a existncia de
cromossomos extras ou de quebras cromossmicas,
auxiliando no diagnstico de certas anomalias genticas.
32. A Sndrome de Down, ou trissomia do cromossomo 16,
e o daltonismo so exemplos de doenas de origem
gentica que podem ser diagnosticadas atravs do
exame cariotpico.

5. Some as alternativas corretas:
01. A cromatina sexual um dos dois cromossomos sexuais
XX que no se desespiralizou totalmente durante a
intrfase.
02. A cromatina sexual est presente somente em machos,
pois os cromossomos sexuais so XX.
04. Um indivduo com Sndrome de Klinefelter apresenta
uma cromatina sexual.
08. Mulheres com Sndrome de Turner apresentam uma
cromatina sexual.
16. Mulheres que apresentam trs cromossomos sexuais
XXX apresentam duas cromatinas sexuais.
32. As mulheres normais apresentam uma s cromatina
sexual.

6. (UFSC) Em uma determinada espcie animal, o nmero
total de cromossomos, por clula somtica, igual a 48.
Baseado nisso, assinale a(s) proposio(es) verdadeira(s).
01. O nmero haploide dessa espcie 48.
02. Nas clulas sexuais, o nmero de cromossomos igual a
12.
04. Em caso de poliploidia, o nmero de cromossomos, por
clula, fica abaixo do nmero diploide.
08. Caso ocorra trissomia em um dos pares cromossmicos,
o nmero de cromossomos passar para 49.
16. Uma clula tetraploide conter 96 cromossomos.
32. Os gametas dessa espcie contero 24 cromossomos.

7. Sobre cromossomos, some as afirmaes corretas:
01. Representam-se cromossomos autossomos numa clula
diploide por 2A e numa clula haploide por A.
02. Os cromossomos heterossomos representam-se pelas
letras XX no sexo feminino e por XY no sexo
masculino, em uma clula diploide.
04. Ao conjunto de cromossomos de uma clula haploide,
chamamos de genoma.
08. Uma clula diploide apresenta dois genomas.
16. A anlise diploide dos cromossomos de uma espcie
quanto ao nmero, tamanho, posio do centrmero, o
caritipo.
32. Idiograma a montagem dos cromossomos de uma
espcie, dos maiores para os menores.

UNIDADE 9.
23
DIVISO CELULAR MITOSE

Desde a formao da clula-ovo ou zigoto, todas as nossas
clulas esto em constante diviso celular, com exceo das
clulas nervosas e musculares estriadas que so altamente
especializadas e acabaram por perder a capacidade
proliferativa.
No entanto, antes das clulas se dividirem, se
encontram em uma etapa denominada intrfase, a qual
constituda por 3 fases:

Fase G
1
: Esta fase se caracteriza pelo fato de a clula
apresentar suas atividades funcionais normais. Os
cromossomos sofrem uma desespiralizao e a clula
aumenta consideravelmente o seu volume.
Fase S: Nesta fase ocorre a duplicao do material
gentico, fenmeno extremamente importante para a
formao de duas clulas idnticas ao final da mitose, pois,
para que haja a formao de 2 clulas com o mesmo
nmero de cromossomos da clula-me, necessrio que
ocorra a duplicao do DNA.
Fase G
2
: A clula novamente aumenta o seu volume e se
prepara para sofrer a uma diviso. O grfico abaixo
representa a variao da concentrao do material gentico
(DNA) nos 3 perodos da intrfase (G
1
, S e G
2
) e, tambm,
ao longo das etapas da diviso de uma clula.

MITOSE
As clulas que formam os rgos dos animais so
denominadas somticas. Estas clulas sofrem divises do
tipo mitose, a qual se caracteriza por originar ao final duas
clulas-filhas com mesmo nmero de cromossomos da
clula me. Nos animais pluricelulares, este tipo de diviso
tem como objetivo principal multiplicao celular e,
consequentemente, o crescimento do ser. Em alguns seres
vivos, a mitose tem por finalidade formar gametas
masculinos e femininos, como ocorre, por exemplo, nos
vegetais.

ETAPAS DA MITOSE
Quando uma clula sofre diviso celular, observa-se
profunda alterao ao nvel do citoplasma e do ncleo.
Estas alteraes podem ser divididas didaticamente em
quatro fases distintas propostas por Fleming. Estas etapas
so:
xPRFASE xMETFASE
xANFASE xTELFASE

Please purchase PDF Split-Merge on www.verypdf.com to remove this watermark.
Incluso para a Vida Biologia B

Pr-Vestibular da UFSC 17
CARACTERSTICAS DA PRFASE
No incio da prfase ocorre a espiralizao e a condensao
dos filamentos de cromatina, os quais se tornam
cromossomos. Os nuclolos diminuem de tamanho e
acabam por desaparecer. No citoplasma, o centro celular
(fibras do ster mais centrolo) duplica-se e, em seguida,
cada centro-celular migra para plos opostos.
No final da prfase, os cromossomos encontram-se
espalhados pela clula, mas unidos aos centros celulares
atravs de fibras de constituio proteica, originando as
fibras do fuso acromtico ou o aparelho mittico.

Esquemas representando a prfase da mitose

CARACTERSTICAS DA METFASE
Nesta fase, os cromossomos esto ocupando a regio
mediana (equatorial) da clula. No centrmero de cada
cromossomo existem duas regies denominadas
cinetcoros, onde h formao de fibras de protenas em
cada cromtide. Essas fibras proteicas ligam-se s fibras
do fuso. na metfase que ocorre o momento de melhor
visualizao dos cromossomos, pois estes se encontram
no mximo de sua condensao.

Esquema representando a metfase da mitose. Nesta fase
os cromossomos se encontram na regio central formando
a placa equatorial.

CARACTERSTICAS DA ANFASE
A anfase se caracteriza pelo rompimento dos
cromossomos duplicados ao nvel do centrmero e cada
cromtide (lado longitudinal), passar a migrar para plos
opostos e cada uma das cromtides passar a ser um novo
cromossomo. A este fenmeno de rompimento dos
cromossomos, denomina-se cromocinese. Nesta fase, se a
mitose estiver ocorrendo com uma clula diploide (2n),
devido a migrao das cromtides para plos opostos, a
clula ser, por um rpido momento, tetraploide (4n).
Caso a mitose esteja ocorrendo com uma clula haplpide
(n), esta passar a ser uma clula diploide.


Esquemas representando a anfase da mitose.

CARACTERSTICAS DA TELFASE
Nesta fase, os cromossomos se encontram nos plos
opostos devido migrao que ocorreu na anfase. Ao
redor dos cromossomos, localizados nos plos opostos,
observa-se a formao da carioteca (cariocinese). Todos
os cromossomos iniciam a sua descondensao, voltando
a ser longos filamentos e os nuclolos reaparecem. Na
regio central da clula a membrana plasmtica sofre uma
citocinese (diviso do citoplasma), originando duas
clulas-filhas.

Esquema representando a telfase da mitose. Nesta fase ocorre a
formao da carioteca, envolvendo os cromossomos no plo
superior e inferior, e a citocinese.

MITOSE EM CLULAS VEGETAIS
As clulas vegetais, quando sofrem mitose, tambm
passam pelas fases de prfase, metfase, anfase e
telfase. No entanto, observam-se algumas diferenas
entre a mitose das clulas vegetais, com relao s clulas
animais, como por exemplo:
- Nas clulas vegetais no h centrolos e fibras do ster,
sendo denominado, portanto, de mitose anastral e
acntrica.
- A citocinese nas clulas vegetais do tipo centrfuga
(ocorre do centro para a periferia) e nas clulas animais
centrpeta (ocorre da periferia em direo ao centro da
clula).

Exerccios de Sala #

1. Some as alternativas corretas sobre ciclo celular:
01. Clulas que entram em diviso celular (lbeis e estveis)
passam por um perodo de preparao para a diviso, a
intrfase.
02. No G1, a clula apresenta um incremento metablico no
citoplasma, logo, aumenta o volume celular.
04. O intervalo S caracteriza-se pela duplicao do DNA,
logo, dos cromonemas.
08. O intervalo G2 caracteriza-se novamente pelo
metabolismo citoplasmtico com sntese de protenas.
Please purchase PDF Split-Merge on www.verypdf.com to remove this watermark.
Incluso para a Vida Biologia B

Pr-Vestibular da UFSC
18
16. Ao final do intervalo S a clula apresenta duas vezes
mais material gentico, podendo, na diviso celular,
originar duas clulas-filhas com igual material gentico
da clula-me.
32. Durante a mitose, a clula divide o nmero de
cromossomos metade.

2. (F.Objetivo-SP) Durante o processo mittico de diviso
celular ocorrem os seguintes eventos:
I - Incio da condensao cromossmica.
II - Diviso dos centrmeros e separao das cromtides.
III - Acontece a citocinese.
IV - Cromossomos alinhados no plano equatorial da clula.

A sequncia correta de tais eventos :
a) I --- II---III---IV d) II --- I --- III --- IV
b) I---IV--- II --- III e) III--- II ---I ---IV
c) I --- III --- II ---IV

3. (FURN) Uma clula com 20 cromossomos, extrada da
regio meristemtica da raiz do milho, ao sofrer mitose
dever formar:
a) 2 clulas com 20 cromossomos cada
b) 4 clulas com 20 cromossomos cada
c) 2 clulas com 10 cromossomos cada
d) 4 clulas com 5 cromossomos cada
e) 4 clulas com 10 cromossomos cada

MEIOSE
A meiose um tipo de diviso celular que ocorre nos
animais para a formao de gametas e nos vegetais para a
formao de esporos. As clulas que se dividem por meiose
so diploides e sofrem duas divises consecutivas. A
primeira diviso denominada reducional, pois reduz o
nmero de cromossomos metade, e a segunda diviso do
tipo equacional, pois equaciona o nmero de clulas. O
resultado final da meiose a formao de 4 clulas
haploides e, geralmente, uma diferente da outra,
caracterstica extremamente importante para que ocorra a
variabilidade das espcies.

Esquema representando a diviso meitica. Este tipo de diviso
constituda por duas etapas a meiose I (Reducional) e a meiose II
(Equacional), originando 4 clulas haploides.

A meiose constituda pelas mesmas fases da mitose. No
entanto alguns eventos interessantes ocorrem durante a
prfase da meiose I. Estes eventos so divididos em 5
subfases:

xLeptteno xZigteno xPaquteno
xDiplteno xDiacinese

x Em Leptteno, os filamentos de cromatina esto iniciando
a espiralizao e transformando-se em cromossomos. o
incio da prfase.
x Em Zigteno, ocorre a sinapse cromossmica, ou seja, os
cromossomos homlogos ficam emparelhados e cada locus
(local onde o gene est situado no cromossomo) se dispe
exatamente paralelo ao locus correspondente do
cromossomo homlogo.
x Em Paquteno, as cromtides homlogas esto muito
prximas, observando-se a formao das ttrades
bivalentes. Em seguida, estas se entrelaam, podendo
ocorrer quebras ao longo das cromtides dos cromossomos
homlogos, seguidos por soldaduras. No entanto, estas
soldaduras ocorrem de modo trocado, ou seja, uma
cromtide solda-se ao fragmento de seu homlogo e vice-
versa. Este fenmeno denomina-se crossing-over ou
permuta gnica.
x Em Diplteno, os cromossomos comeam a se separar,
surgindo entre as cromtides homlogas a formao de
figuras com o formato de uma letra X, denominadas de
quiasmas. Os quiasmas indicam quantas permutaes
ocorreram.
x Em diacinese, os cromossomos homlogos continuam a
separar e os quiasmas deixam de existir, a terminalizao
dos quiasmas. Nesta fase ainda ocorre a desintegrao da
carioteca, marcando o final da prfase I.

Esquema representando as 5 subfases da prfase I.

METFASE I
Nesta fase, os pares de cromossomos se organizam na
regio central, formando a placa equatorial. Os centrmeros
dos cromossomos homlogos se ligam s fibras proteicas
provenientes dos centrolos localizados nos plos opostos.
Alm disso, nesta fase que os cromossomos se encontram
mais condensados.

Cromossomos na regio central, formando a placa equatorial.

ANFASE I
Na anfase I no ocorre diviso dos centrmeros, como se
observa na anfase da mitose. Cada par de cromossomos
homlogos migra em direo a um dos plos da clula, por
encurtamento das fibras do fuso.
Please purchase PDF Split-Merge on www.verypdf.com to remove this watermark.
Incluso para a Vida Biologia B

Pr-Vestibular da UFSC 19

Esquema representando os cromossomos duplicados migrando
para plos opostos.

TELFASE I
Nesta fase, observa-se a reorganizao da carioteca e os
cromossomos comeam a se desespiralizar, ocorrendo, em
seguida, a diviso do citoplasma (citocinese). No entanto,
algumas vezes estes eventos da telfase I no ocorrem e a
clula inicia diretamente a segunda diviso meitica.

Esquema representando o fim da prfase I. Nesta fase ocorre a
reorganizao da carioteca, a citocinese e a formao de duas
clulas haploides.

PRFASE II
Nesta fase, os cromossomos encontram-se espalhados pela
clula em total desordem. Como na prfase da mitose, os
centrolos localizam-se nos plos opostos (superior e
inferior) e todos os cromossomos esto unidos aos
centrolos atravs das fibras do fuso ou cromossmicas.
Alm disso, ocorre a desintegrao da carioteca.

Esquema representando as duas clulas haplides em prfase II.

METFASE II
Os cromossomos encontram-se alinhados na regio
mediana da clula, formando a placa equatorial. Nesta fase,
como ocorre na metfase da mitose, os cromossomos esto
no mximo de sua condensao, sendo o momento de
melhor visualizao.

Esquema representando os cromossomos na regio central,
formando a placa equatorial.
ANFASE II
Nesta fase, os cromossomos se rompem ao nvel do
centrmero e cada cromtide migra para plos opostos,
devido ao encurtamento das fibras do fuso.

Esquema representando a migrao das cromtides.

TELFASE II
Os cromossomos nos plos opostos iniciam a sua
desespiralizao e a carioteca em cada plo comea a sua
reorganizao (fenmeno denominado de cariocinese),
envolvendo todos os cromossomos. Alm disso, todas as
organelas celulares separam-se em iguais quantidades para
os dois lados da clula e a membrana plasmtica sofre uma
diviso (fenmeno denominado citocinese), formando-se,
consequentemente, quatro clulas haploides.


Esquema demonstrando a formao de 4 clulas haploides
ao final da telfase II.

Tarefa Mnima #

4. (UFSC) A mitose e a meiose so dois tipos de diviso
celular. Com relao a esses processos, assinale a(s)
proposio(es) verdadeira(s).
01. A mitose uma diviso do tipo equacional.
02. A meiose ocorre na linhagem germinativa, quando da
produo dos gametas.
04. A meiose ocorre em quatro etapas sucessivas.
08. O nmero de cromossomos das clulas resultantes de
ambos os processos igual ao das clulas que lhes
deram origem, porm somente as clulas que sofreram
meiose apresentam recombinao gentica.
16. A mitose ocorre nas clulas somticas.
32. Ambos os processos ocorrem em todos os seres.
64. Em alguns organismos a mitose utilizada como forma
de reproduo.

5. Sobre as fases da meiose, some as afirmaes corretas.
01. Durante a meiose I, ocorre: prfase I, metfase I,
anfase I e telfase I.
02. Durante a prfase I, ocorrem eventos com os
cromossomos descritos pelas subfases: leptteno
zigtenopaqutenodipltenodiacinese.
04. No leptpeno, os cromossomos iniciam a espiralizao.
08. Os cromossomos homlogos emparelham-se durante o
diacinese.
Please purchase PDF Split-Merge on www.verypdf.com to remove this watermark.
Incluso para a Vida Biologia B

Pr-Vestibular da UFSC
20
16. Durante o paquteno, as cromtides no-irms,
homlogas, tocam-se. Podem quebrar-se e soldar-se no
cromossomo homlogo, efetuando o crossing-over.
32. Pela repulso dos centrmeros dos cromossomos
homlogos, as cromtides homlogas formam um X, o
quiasma, no diplteno.

6. A Mitose e a Meiose so importantes processos
biolgicos, pois permitem que o nmero de cromossomos
de uma clula permanea igual, ou seja, reduzido, para
possibilitar sua restaurao numrica aps a fecundao.
Com relao aos eventos e aos resultados destes dois
processos, correto afirmar que:
01. Ao contrrio da Mitose, que ocorre em todas as clulas,
a Meiose restringe-se quelas da linha germinativa, que
produziro gametas.
02. Nos dois processos, ocorre a compactao da cromatina,
fenmeno este que, alm de facilitar a diviso correta
dos cromossomos, impede que o material gentico seja
atacado por enzimas, presentes no citoplasma, que
destroem o DNA.
04. Uma mutao que ocorra em uma das cromtides de
uma clula somtica ser transmitida a todas as suas
clulas-filhas, atravs da diviso mittica.
08. A Mitose o sistema de reproduo dos organismos
nos quais no existe a presena de sexo nem a formao
de clulas germinativas.
16. Se considerarmos, em uma mesma espcie, duas
clulas-filhas, uma originada por Mitose e a outra por
Meiose, a primeira conter metade do nmero de
cromossomos e o dobro da quantidade de DNA da segunda.
32. Na Meiose, existe a possibilidade de ocorrer o
fenmeno de recombinao, que a troca de segmentos
entre quaisquer dois cromossomos, gerando, com isso,
alta variabilidade gentica para os indivduos
envolvidos.
64. A Meiose compreende duas etapas de diviso
cromossmica, sendo que, aps a primeira, o nmero
de cromossomos das clulas-filhas metade do das
clulas-mes.


UNIDADE 10

CLASSIFICAO GERAL DOS SERES VIVOS

Atualmente, os seres vivos esto agrupados em cinco reinos, de acordo com algumas caractersticas anatmicas e fisiolgicas,
como demonstrado no quadro abaixo.


REGRAS DE NOMENCLATURA
Para facilitar o estudo dos seres vivos necessrio que se
faa uma organizao em grupos, de acordo com alguns
critrios, como: semelhanas fisiolgicas, anatmicas,
bioqumicas e DNA.
Tambm preciso dar nomes comuns aos diversos seres
vivos e, que, estes, ainda tenham o mesmo nome em
qualquer lugar da Terra.
Em 1735 um botnico sueco chamado Kal von Lin,
estabeleceu a espcie como unidade bsica de classificao.
Mais tarde, outros cientistas estabeleceram mais duas novas
divises: FILO e FAMLIA. Sendo assim, a sequncia
taxonmica atual : REINO - FILO - CLASSE - ORDEM
FAMLIA - GNERO e ESPCIE.


Please purchase PDF Split-Merge on www.verypdf.com to remove this watermark.
Incluso para a Vida Biologia B

Pr-Vestibular da UFSC 21
Sendo assim:
- ESPCIES com muitas semelhanas podem ser
reunidas em outro grupo taxonmico, o GNERO;
- GNEROS afins formam uma FAMLIA, que
podem ser reunidas para formar uma ORDEM e
assim, sucessivamente, at serem classificados e
formarem um REINO.
Para definir essas semelhanas existem algumas regras,
muitas delas, inclusive, estabelecidas por Lineu, como:
Todo nome deve ser escrito em latim ou latinizado;
Do REINO at o GNERO, os nomes devero ser escritos
com a primeira letra maiscula;
A ESPCIE binominal e o primeiro nome igual ao do
GNERO, sendo que o segundo nome dever ser escrito
com letra minscula, como, por exemplo: Felis domesticus.
A forma da escrita ter que ser destacado do texto, em
itlico ou sublinhado (Homo sapiens);

Exerccios de sala #

1. (UFSC) Considerando todos os seres vivos, esto
descritos e catalogados quase dois milhes de espcies. Mas
esse nmero est longe do total real: segundo algumas
estimativas, pelo menos 50 milhes de espcies ainda no
teriam sido descritas. O sistema de classificao usado hoje
distribui os seres vivos em cinco grandes reinos: Monera,
Protista, Fungi, Animalia e Plantae.
CINCIA HOJE, vol. 24, 142, p. 6.
Com relao a este assunto, correto afirmar que:
01. Os reinos Animalia e Plantae tambm so conhecidos,
respectivamente, como Metazoa e Metaphyta.
02. Poucos representantes do reino Fungi so clorofilados.
04. Os reinos Monera e Protista incluem seres unicelulares
procariontes e eucariontes, respectivamente.
08. Todos os organismos pertencentes ao reino Animalia so
uni ou pluricelulares e eucariontes.
16. Mofos, leveduras e cogumelos so exemplos de
organismos integrantes do reino Fungi.
32. Os representantes do reino Plantae so pluricelulares e
eucariontes, e sintetizam seu alimento.

2. Com base em seus conhecimentos sobre a classificao
dos seres vivos, some as alternativas corretas.
01. Na moderna classificao, os seres vivos foram
agrupados em cinco reinos biolgicos, denominados:
Monera, Protista, Fungi, Metaphyta e Metazoa.
02. Um organismo eucarionte, pluricelular e auttrofo
poderia ser uma gramnea.
04. As categorias taxonmicas colocadas ordenadamente,
em graus hierrquicos, so: reino, filo ou diviso, classe,
famlia, ordem, gnero e espcie.
08. O nome cientfico do mexilho ou marisco em
determinada categoria taxonmica Mytilidae e, em
outra Perna perna. Podemos dizer, com base nas regras
de nomenclatura, que Mytilidae o nome da famlia e
Perna perna da espcie.
16. Os fungos Penicillium roquefortii e Penicillium
camembertii, so utilizados na produo de queijos. Pela
anlise dos nomes cientficos, podemos concluir que
esses seres no pertencem a mesma espcie, mas
pertencem ao mesmo gnero e famlia.

UNIDADE 11

VRUS

Os vrus se diferem de todos os seres vivos, devido s
seguintes caractersticas:
- No apresentam um metabolismo prprio;
- No possuem uma organizao celular e se
cristalizam, quando no esto parasitando uma
clula.
- S se reproduzem no interior de clulas vivas,
sendo, portanto, parasitas intracelulares
obrigatrios, sendo responsveis por vrias
doenas.
At hoje, a classificao dos vrus gera muita discusso,
pois alguns pesquisadores os consideram como sendo
partculas ou fragmentos de clulas e, outros, classificam os
vrus como seres extremamente simples. Contudo, uma
tendncia considerar os vrus como sendo seres vivos.

ESTRUTURA DOS VRUS
Os vrus so constitudos por uma cpsula de protena,
denominada capsdeo, que envolve o material gentico, o
qual pode ser o cido desoxirribonucleico (vrus de DNA)
ou cido ribonucleico (vrus de RNA), nunca ocorrendo os
dois tipos de cidos nucleicos no mesmo vrus. Os
envoltrios proteicos podem apresentar vrias formas, como,
por exemplo, esfrica, helicoidal, hexagonal, cilndrica, etc.
Alguns vrus so formados apenas pela cpsula
proteica (capsdeo) e o material gentico (DNA ou RNA).
Outros, por sua vez, apresentam um envoltrio
externo envolvendo a cpsula proteica, sendo estes vrus
denominados de envelopados ou capsulados, como, por
exemplo: o vrus da AIDS. Existem vrus que infectam
apenas bactrias, outros infectam os fungos, outros as
plantas e animais.
O vrus mais estudado at hoje o bacterifago, o
qual apresenta uma cpsula proteica de formato hexagonal
envolvendo a molcula de DNA e uma regio denominada
de cauda, constituda por um eixo cilndrico e fibras
proteicas na extremidade, com funo de fixar o vrus
bactria.

Estrutura do bacterifago

Please purchase PDF Split-Merge on www.verypdf.com to remove this watermark.
Incluso para a Vida Biologia B

Pr-Vestibular da UFSC
22

PRINCIPAIS VIROSES

VIROSE TRANSMISSO LOCAL DA INFECO PROFILAXIA SINTOMAS
CAXUMBA Objetos
contaminados,
gotculas de saliva
O vrus multiplica-se nas
glndulas partidas,
podendo localizar-se em
outros rgos como ovrios
e testculos
Vacinao Inchao abaixo e em frente
das orelhas e, se atingir os
testculos ou os ovrios, pode
tornar a pessoa estril
FEBRE
AMARELA
Picada do mosquito
Aedes aegypti
O vrus, atravs da picada
do mosquito, localiza-se no
fgado, na medula ssea e
no bao
Vacinao e combate
aos mosquitos Aedes
aegypti
Febre alta, vmitos, calafrios
e pele amarelada, podendo
ser fatal
GRIPE Gotculas de
secreo
O vrus instala-se nas vias
respiratrias
Nenhum Febre, dores de cabea e
musculares, obstruo nasal e
tosse
HIDROFOBIA
(raiva)
Saliva introduzida
pela mordida de
animais
O vrus instala-se no
sistema nervoso
Vacinao em animais
e aplicao de soro e
vacina em pessoas
mordidas
Febre, mal estar, delrios e
morte
HEPATITE Gotculas de saliva,
objetos
contaminados
O vrus instala-se no fgado,
destruindo as clulas
Saneamento, cuidado
com alimentos e
injeo de
gamaglobulina
febre, nuseas, ictercia
POLIOMIELITE Alimentos e objetos
contaminados
O vrus penetra pela boca,
multiplica-se no intestino e,
em seguida instala-se no
sistema nervoso central
destruindo os neurnios
Vacinao Paralisia dos membros
RUBOLA Saliva, contato
direto
Penetra pelas vias
respiratrias e se dissemina
pelo sangue
Aplicao de
imunoglobulina
Febre, erupes cutneas. No
feto provoca morte ou
deficincias congnitas
SARAMPO Saliva e secrees O vrus penetra pelas vias
respiratrias e se dissemina
atravs do sangue
Vacinao Febre, vermelhido por todo
corpo, podendo ser fatal em
crianas

Exerccios de sala #

1. (UFSC-Modificado) Os hospitais esto sendo fechados e
as pessoas esto morrendo. Um curto e incisivo relato de
uma rea da cidade de Canto proporcionou uma das
primeiras descries sobre o caos na provncia de
Guangdong, no sul da China, atingida por uma doena
misteriosa, agora conhecida como sndrome respiratria
aguda grave (SARS, na sigla em ingls).
(SCIENTIFIC AMERICAN BRASIL. So Paulo: Duetto editorial, ano 2,
v.13, p. 12, jun. 2003).
A SARS veio se juntar a uma srie de outras doenas
parasitrias que atingem o homem.
Com relao a essas doenas que podem atingir as
populaes humanas, correto afirmar que:
01. A tuberculose, causada por um vrus, foi responsvel por
muitas mortes no passado. No entanto, atualmente, os
casos fatais, provocados por essa doena no mundo, so
raros.
02. O combate ao mosquito Aedes aegypti e a vacinao da
populao so medidas eficazes no combate
esquistossomose.
04. Nas doenas causadas por vrus, esses eventualmente se
utilizam da maquinaria de sntese protica da clula
hospedeira para a construo de suas prprias protenas.

08. Em seu ciclo de vida, muitos parasitas se utilizam de
hospedeiros intermedirios para alcanarem seu
hospedeiro definitivo.
16. SARS, AIDS e Dengue so ocasionadas por vrus.
32. Como medida profiltica para todas as doenas
parasitrias conhecidas, a Organizao Mundial de
Sade (OMS) recomenda que os governos dos pases
atingidos utilizem campanhas de vacinao.

2. (UFSC) A febre amarela, antes restrita a regies
afastadas, um mal que comea a ameaar, cada vez mais,
os centros urbanos. Sobre a febre amarela, correto afirmar
que:
01. Apresenta, dentre outros sintomas, febre alta e vmito.
02. causada por vermes.
04. O contgio d-se pela ingesto de alimentos
contaminados.
08. Vacinas contra esse mal ainda no foram desenvolvidas.
16. Uma das formas de preveno a eliminao do
mosquito que transmite essa doena.
32. uma doena que pode causar a morte

3. Durante o ano de 1994, foram veiculadas campanhas de
vacinao de animais com o objetivo de controlar a raiva.
Em relao a essa doena, correto afirmar que:
01. Um dos sintomas apresentados por animais raivosos a
ingesto exagerada de gua.
Please purchase PDF Split-Merge on www.verypdf.com to remove this watermark.
Incluso para a Vida Biologia B

Pr-Vestibular da UFSC 23
02. Pode ser transmitida atravs da mordedura de ces e
gatos raivosos.
04. Apresenta sintomas brandos, raramente levando morte
seus portadores.
08. Todos os demais mamferos so imunes a essa doena.
16. Seu agente causador um vrus.

4. Apesar das campanhas divulgadas em todos os veculos
de comunicao, a dengue tem se espalhado para reas onde
antes no ocorria. Sobre a dengue, assinale a(s)
proposio(es) correta(s).
01. Como preveno, existem vacinas que atuam
imunizando totalmente as pessoas.
02. uma doena infecciosa.
04. Sua forma de transmisso se d pela picada de um tipo
de mosquito.
08. Tem como sintomas febre alta, moleza, dores musculares
e de cabea, entre outros.
16. J foi descartada a eliminao do mosquito transmissor
da doena, como forma de combat-la, pois um mtodo
ineficaz.
32. Em alguns casos podem ocorrer hemorragias fatais.

5. A hepatite uma doena que se caracteriza por inchao
no fgado, acompanhado de prostrao e febre, urina escura,
fezes descoradas, alm de outros sintomas. Em relao a
essa doena correto afirmar que:
01. Existe uma forma contagiosa de origem viral.
02. Existe uma forma no contagiosa, provocada pelo
excesso de bebida alcoolica.
04. No existe vacina para nenhum tipo.
08. A ictercia (olhos e pele amarelados) outro sintoma
marcante.
16. Durante o seu tratamento recomendada uma
alimentao rica em acar, a fim de reduzir o esforo do
rgo atingido.
32. O uso de seringas descartveis uma forma de preveni-
la.

UNIDADE 12

MONERA

Os organismos que fazem parte do Reino Monera se
caracterizam por serem unicelulares e procariontes, ou seja,
no possuem a carioteca (membrana que envolve o material
gentico). Alm disso, os moneras no apresentam organelas
membranosas, como acorre com a maioria das clulas dos
seres vivos, como, por exemplo, o retculo endoplasmtico,
o complexo de Golgi, as mitocndrias e os plastos.

ESTRUTURA DAS BACTRIAS
As bactrias so encontradas nos mais variados ambientes,
sendo amplamente disseminadas em nosso planeta, podendo
ser encontradas no solo, na gua, nos animais, nas plantas,
nos objetos, parasitando organismos ou vivendo em
mutualismo, sendo, portanto hetertrofas. No entanto,
algumas espcies, podem produzir o seu prprio alimento
atravs da quimiossntese ou por um tipo especial de
fotossntese denominado fotossntese bacteriana, as quais
possuem o pigmento bacterioclorofila. As bactrias so
portadoras de uma parede celular rgida, semelhante a dos
vegetais, mas de composio qumica diferente. No interior
da clula bacteriana, encontra-se a membrana plasmtica,
que lipoproteica, o citoplasma e o DNA bacteriano, o qual
est ligado a uma invaginao da membrana plasmtica,
denominada mesossomo.

Estruturas da bactria

TIPOS MORFOLGICOS
cocos: quando a bactria apresenta um formato esfrico;
bacilos: quando a bactria apresenta um formato em
basto;
espirilos: quando a bactria apresenta um formato em
espiral;
vibries: quando a bactria apresenta um formato em
vrgula.
Os cocos, no entanto, podem formar colnias,
originado as bactrias que dependendo do seu arranjo
recebem as seguintes denominaes: diplococos,
estreptococos, estafilococos, ttrades e sarcina.
Mais raramente, os bacilos tambm podem formar colnias,
ocorrendo aos pares, formando os diplobacilos, ou em
fileira, formando os estreptobacilos.

REPRODUO DAS BACTRIAS
A maioria das bactrias se reproduz atravs de um tipo de
reproduo assexuada denominado cissiparidade ou diviso
binria ou bipartio. Neste tipo de diviso o cromossomo
sofre uma duplicao e forma-se mais um mesossomo. Em
seguida, ocorre a cotocinese (diviso do citoplasma),
originando duas clulas-filhas idnticas. As bactrias
possuem um alto poder de reproduo e, em algumas horas,
sob condies favorveis podem originar milhares de
descendentes, todos geneticamente idnticos entre si. Esse
conjunto de seres geneticamente idnticos denominado
clone.
No entanto, alteraes genticas tambm ocorrem
nas bactrias, as quais podem ser por mutao ou por
transmisso de material gentico de uma bactria para outra.
Essa transmisso de material gentico pode ocorrer atravs
de trs mecanismos distintos: conjugao, transformao e
transduo.



Please purchase PDF Split-Merge on www.verypdf.com to remove this watermark.
Incluso para a Vida Biologia B

Pr-Vestibular da UFSC
24

ALGUMAS DOENAS CAUSADAS POR BACTRIAS

DOENA BACTRIA TRANSMISSO
Coqueluche Bordetella pertussis vias respiratrias
Ttano Clostridium tetani contaminao em ferimentos
Tifo Rickettsia prowazeki picada de artrpodes
Hansenase Mycobacterium leprae contato direto
Gonorreia ou blenorreia Neisseria gonorrheae contato sexual
Sfilis Treponema pallidum contato sexual
Clera Vibrio cholerae contaminao gua e alimentos
Pneumonia Diplococos pneumoniae vias respiratrias
Tuberculose Mycobacterium tuberculosis vias respiratrias
Difteria Corinebacterium dipheteriae vias respiratria
Peste bubnica Pasteurella pestis do rato ao homem por picada de pulga
Resfriado Hemophilus influenzae vias respiratrias
Febre tifoide Salmonela typhosa contaminao de gua e alimentos
Meningite Neisseria meningitidis vias respiratrias

CIANOBACTRIAS
As cianobactrias se caracterizam por serem unicelulares,
mas a maioria forma colnias filamentosas constitudas por
vrios indivduos e envolvidas por uma espessa camada de
muco. Sendo procariontes, no apresentam carioteca e os
seus pigmentos responsveis pela fotossntese encontram-se
espalhados pelo citoplasma, ao contrrio dos vegetais
superiores, que possuem os pigmentos no interior de
estruturas membranosas denominadas plastos. Algumas
cianobactrias so seres extremamente importantes, pois so
capazes de fixar o nitrognio diretamente da atmosfera. Este
fato permite com que as cianofceas sobrevivam em
ambientes extremamente ridos, onde outros grupos
biolgicos normalmente no se desenvolvem.

REPRODUO DAS CIANOBACTRIAS
As cianobactrias se reproduzem assexuadamente por
cissiparidade e em espcies que formam colnias a
reproduo feita por hormognios. O hormognio um
pequeno pedao da colnia, geralmente apical (situado na
extremidade da colnia) e que possui algumas poucas
clulas. Este pedao se destaca da colnia, passando a
formar novas cianofceas, tambm coloniais. As do gnero
Nostoc, Anabaena e Oscillatoria so exemplos de
cianobactrias.

Exerccios de sala #

1. (UFSC) O controle das infeces que ocorrem em
hospitais uma tarefa rdua e contnua [...] A microbiota
(conjunto de organismos encontrados no corpo humano) do
paciente pode se tornar patognica, principalmente naqueles
que esto com o sistema imunolgico comprometido,...
(Trecho do texto: Infeco hospitalar: a soluo em suas
mos, extrado da Revista Cincia Hoje, 29 (173), de julho
de 2001, p. 80).
Com relao a esse assunto, suas causas e medidas de
reduo, correto afirmar que:
01. A maioria dos casos de infeco hospitalar causada por
agentes bacterianos.
02. Os traumatismos e a contaminao por vermes acarretam
o surgimento desses tipos de infeco.
04. Entre os pacientes com maior risco de contrarem
infeco hospitalar esto os soropositivos para o HIV,
uma vez que eles apresentam maior chance de terem seus
sistemas imunolgicos comprometidos.
08. O uso indiscriminado de antibiticos tambm pode levar
a uma maior incidncia de infeco hospitalar, pois ele
facilita o surgimento de microorganismos resistentes.
16. Entre os procedimentos que podem aumentar as chances
da ocorrncia da infeco hospitalar, esto aqueles que
requerem a entubao endotraqueal e/ou a nasogstrica,
j que exigem tempo prolongado de respirao mecnica
e riscos de contaminao secundria (atravs das mos).
32. uma simples medida a lavagem adequada das mos
antes do contato com o paciente pode reduzir
drasticamente o ndice de infeco hospitalar.

2. Febre alta, vmitos e rigidez na nuca so sintomas
associados meningite. Em Santa Catarina, em 1988, essa
doena teve ocorrncia expressiva com cerca de 200 casos
registrados oficialmente, na sua maioria crianas. Em
relao a essa doena transmissvel, correto afirmar que:
01. Existem vrios microorganismos que podem causar a
doena, sendo os mais comuns os meningococos A, B e
C, os pneumococos e alguns vrus.
02. Existe vacina para todos os tipos de meningite.
04. Transmite-se de pessoa para pessoa atravs de contato
direto (pela fala, tosse, espirro ou beijo).
08. A sua preveno, em pessoas que tiveram contato com
um indivduo afetado, deve ser feita com uso de
antibiticos.
16. caracterizada por inflamao do sistema nervoso
central.

3. (UFPR) Aps os atos terroristas de 11 de setembro de
2001, perpetrados contra a cidade de Nova Iorque, os
Estados Unidos passaram a viver sobressaltados com a
perspectiva de uma guerra biolgica, em que poderia ser
utilizado o Bacillus anthracis, causador do antraz. Sobre as
preocupaes com essa perspectiva e os fatos que as
fundamentam, correto afirmar:
01. O Bacillus anthracis tem a propriedade de esporulao,
o que permite sua sobrevida prolongada.
Please purchase PDF Split-Merge on www.verypdf.com to remove this watermark.
Incluso para a Vida Biologia B

Pr-Vestibular da UFSC 25
02. Os esporos invisveis da bactria, ao serem inalados, so
responsveis pelo antraz pulmonar, forma rapidamente
fatal da doena.
04. As preocupaes dos Estados Unidos devem-se ao fato
de a bactria ser resistente penicilina, o que dificulta o
tratamento do antraz.
08. Embora a populao esteja vacinada, a proteo
conferida insuficiente.
16. As preocupaes so infundadas, pois o antraz uma
doena de animais, que no atinge o homem e no pode,
portanto, ser utilizada em guerra biolgica.
32. Os esporos da bactria penetram no organismo atravs
da pele ou membranas mucosas.

Tarefa Mnima #

4. Considere os seguintes componentes celulares:

1 - membrana plasmtica 5 - ribossomos
2 carioteca 6 - retculo endoplasmtico
3 cromossomos 7 - mitocndrias
4 hialoplasma 8 cloroplastos

Dentre as alternativas seguintes, assinale a que tiver a
sequncia representativa de estruturas ausentes em bactrias:

a) 1 - 2 - 7 - 8 b) 2 - 6 - 7 8 c) 2 - 3 - 5 6
d) 3 - 6 - 7 8 e) 5 - 6 - 7 - 8

5. (UFSC) Pegue todas as espcies de mamferos, aves,
rpteis, anfbios, peixes e insetos conhecidos da Amaznia.
Agora triture tudo e tente encaixar o que sobrou dentro de
um pacotinho de acar. S assim, talvez, seja possvel ter
uma idia ainda que muito distante da biodiversidade de
microrganismos que podem ser encontrados em um nico
grama de solo: um milho de espcies de bactrias, segundo
um estudo publicado na revista Science.
Com relao s bactrias, assinale a(s) proposio(es)
correta(s).
01. As bactrias encontradas em grandes quantidades no
solo so responsveis por todas as doenas microbianas
em humanos.
02. O ciclo do nitrognio depende de alguns desses seres
microscpicos.
04. A ciclagem de nutrientes e da energia nos ecossistemas
est diretamente relacionada ao metabolismo
bacteriano.
08. A diversidade bacteriana decorrente de sucessivas
mutaes e da passagem de material gentico entre
bactrias geneticamente diferentes.
16. As bactrias, juntamente com as algas verdes
microscpicas, compreendem o reino Monera.

PROTISTAS
Os Protozorios pertencem ao Reino Protista e apresentam
as seguintes caractersticas:
So eucariontes, unicelulares e desprovidos de clorofila;
Com formas variadas: esfricas, ovais, alongadas, amorfas,
etc.;
Livres, fixos, simbiontes, comensais ou parasitas;
Hetertrofos;
Reproduo em geral assexuada por cissiparidade;
Hbitat variado, desde que haja umidade.

ESTRUTURA DOS PROTOZORIOS
Sendo unicelulares e eucariontes, os protozorios so
formados por membrana, citoplasma e ncleo e por vrias
organelas, as quais so classificadas de acordo com a
funo:
de digesto: citstoma, citofaringe, vacolo digestivo,
vacolo excretor, citopgeo;
de regulao osmtica: vacolo pulstil ou contrtil;
de sustentao: carapaa ou teca, cpsula central;
de coordenao: estigma ou mancha ocelar motorium;
de locomoo e resposta: flagelos, clios, pseudpodos.

Esquema geral de um protozorio ciliado, evidenciando as
estruturas relacionadas com a nutrio

CLASSIFICAO
Fazem parte do Reino Protista as algas e os protozorios.
Estes ltimos que esto sendo estudados neste captulo da
Biologia pertencem ao Filo Protozoa, o qual dividido em 4
Classes, dependendo da estrutura de locomoo:
- CLASSE FLAGELATA OU MASTIGOPHORA (G.
Mastix = chicote; phoros = portador) - Compreende
protozorios que se locomovem por flagelo.
- CLASSE SARCODINA OU RHIZOPODA (G. sarx =
carne; eidos = forma) - protozorios que se locomovem por
pseudpodos.
- CLASSE CILIATA (L. Cilium = clio) - Compreende
protozorios que se locomovem por clios.
- CLASSE SPOROZOA (G. sporo = semente; zon =
animal) - Compreende protozorios sem estrutura de
locomoo.

PRINCIPAIS PROTEOSES

DOENA DE CHAGAS
Causado pelo flagelado Trypanossoma cruzi, a transmisso
ocorre pelo percevejo Triatoma infestans, conhecido
popularmente como barbeiro ou chupana. O barbeiro torna-
se vetor quando adquire o protozorio ao picar animais
silvestres, como o gamb ou o tatu. Durante a noite, ao picar
o indivduo, o Trypanossoma que se encontra nas fezes deste
barbeiro, penetra na pele do indivduo aps este se coar. O
protozorio, atravs do sangue chega ao fgado e ao corao,
provocando febre prolongada, anorexia e insuficincia
cardaca, por aumento do volume do corao.
Profilaxia: Para erradicar a doena necessrio o uso de
inseticidas ou como o barbeiro vive em casas de pau-a-
pique, a construo de casa de alvenaria.

Please purchase PDF Split-Merge on www.verypdf.com to remove this watermark.
Incluso para a Vida Biologia B

Pr-Vestibular da UFSC
26
MALRIA OU IMPALUDISMO OU FEBRE PALUSTRE
Causada por um esporozorio denominado Plasmodium, a
transmisso ocorre pela picada da fmea do mosquito
Anopheles, muito encontrado em climas tropicais ou por
transfuses ou seringas contaminadas pelo protozorio. Esta
doena provoca febres em perodos constantes, dependendo
da espcie. Segundo a Organizao Mundial de Sade
milhes de pessoas em todo o mundo so infectadas com o
Plasmodium, o que a coloca como um das doenas mais
importantes da atualidade. Existem trs espcies diferentes:
Plasmodium vivax que causa a febre ter benigna,
caracterizada por produzir febre de 48 em 48 horas.
Plasmodium malarie que causa a febre quart benigna,
caracterizada por produzir febre de 72 em 72 horas.
Plasmodium falciparum que causa a febre maligna,
caracterizada por produzir febre de em intervalos
irregulares.
Profilaxia: Evitar a procriao do vetor Anopheles e tomar
cuidado nas transfuses e procedimentos cirrgicos.

Exerccios de sala #

6. Sabemos que a maioria dos protozorios de vida livre e
que podem ser encontrados na gua doce na terra e no mar,
mas infelizmente muitos causam doenas muito graves nos
animais, plantas e at nos seres humanos. A estes ltimos
chamamos de parasitas, e no homem eles podem ser
encontrados em rgos como o corao, no sangue e em
outros locais. Muitos so transmitidos diretamente pelos
alimentos e gua contaminada, outros so transmitidos
indiretamente por hospedeiros intermedirios. Assinale a
somatria correta para os protozorios e as doenas que eles
podem causar:
01. O filo Protozoa pode ser classificado em quatro Classes,
os mastigforos, os rizpodos, os ciliata e os esporozoa.
O principal critrio utilizado para diferenciar os
componentes destas quatro classes a estrutura de
locomoo.
02. O plasmdio causador da malria reproduz-se
assexuadamente dentro das hemceas humanas e
reproduz-se dentro do estmago do mosquito do gnero
Anopheles.
04. A leucorreia causada pelo protozorio Trycomonas
vaaginalis e pode ser transmitido s mulheres atravs de
banheiros pblicos, toalhas, roupas ntimas e outros.
08. A amebase pode causar graves distrbios
gastrintestinais e a melhor forma de profilaxia
(preveno) fazer um bom saneamento bsico, pois
este protozorio transmitido pelas fezes humanas,
contaminando a gua e o solo.
16. O agente causador da doena de chagas (Trypanossoma
cruzy) transmitido diretamente por um inseto
hematfago conhecido popularmente como mutuca. Este
aps picar vtimas, injeta sua saliva onde encontramos o
protozorio.

7. (UFSC) Assinale a(s) proposio(es) que indica(m)
corretamente doena(s) causada(s) por protozorio.
01. Raiva 16. Giardase
02. Doena de Chagas 32. Dengue
04. Toxoplasmose 64. Febre amarela
08. Esquistossomose
8. Tamanho no documento. Biologicamente, pode-se
dizer que nem sempre a dimenso do indivduo
proporcional ao grau de periculosidade que o mesmo
oferece.
Baseando-se nessa premissa, correto afirmar que (mais de
uma alternativa pode estar correta):
01. A pequena fmea Anopheles capaz de injetar centenas
de plasmdios no sangue humano, causando a temvel
malria.
02. As minsculas fezes do Triatoma podem conter dezenas
de tripanossomos causadores da doena de Chagas.
04. O microscpico flagelado Leishmania pode ocasionar
leses cutneas e orofarngeas deformantes em
moradores de regies florestais.
08. A quase invisvel ameba Paramecium capaz de
parasitar e colonizar o intestino grosso do homem,
provocando ulceraes e diarreia.
16. O insignificante protozorio Giardia, causador da
toxoplasmose, pode ser eliminado com as fezes na
forma de cistos, que pode sobreviver meses em solo
mido.

Tarefa Mnima #

9. (UFSC) Em maro de 2005 foi constatado um surto da
Doena de Chagas na regio litornea de Santa Catarina,
atingindo 25 pessoas e resultando em 3 mortes. Este fato,
totalmente inesperado para uma rea no endmica da
doena, dificultou inicialmente o diagnstico por parte dos
profissionais de sade e chamou a ateno dos meios de
comunicao, tendo grande repercusso em todo o pas. A
constatao da infeco natural pelo Trypanosoma cruzi em
um gamb e em vrios exemplares de triatomneos
confirmou a existncia de um ciclo de transmisso do
parasita naquela regio.
Sobre a origem, transmisso, aspectos clnicos, diagnstico e
tratamento da Doena de Chagas, correto afirmar que:

01. Em geral, a doena tem duas etapas distintas no homem:
a fase inicial, aguda, caracterizada por elevada
parasitemia e estado febril, seguida de uma fase crnica,
caracterizada pela diminuio do nmero de parasitas
circulantes.
02. Os hospedeiros intermedirios do Trypanosoma cruzi
podem ser tanto vertebrados como invertebrados.
04. Uma vez instalado no hospedeiro vertebrado, o parasita
invade os tecidos penetrando nas clulas, estabelecendo-
se no citoplasma e se multiplicando, o que provoca a
seguir o rompimento do contedo celular, com
consequente liberao dos novos indivduos para o meio
extracelular e a corrente sangunea.
08. As formas mais comuns de transmisso da doena so o
contato com fluidos orgnicos de doentes e ingesto de
alimento contaminado.
16. O tratamento mais eficaz da Doena de Chagas baseia-
se na aplicao de antibiticos potentes.





Please purchase PDF Split-Merge on www.verypdf.com to remove this watermark.
Incluso para a Vida Biologia B

Pr-Vestibular da UFSC 27
UNIDADE 13

REINO FUNGI

Os fungos so seres eucariontes (com carioteca),
unicelulares ou pluricelulares e sem pigmentos
fotossintetizantes e encontrados nos mais variados
ambientes, mas desenvolvem-se melhor em lugares midos
e ricos em matria orgnica. Possuem uma parede celular de
quitina e a sua reproduo normalmente envolve a
participao de esporos, como ocorre entre os vegetais. No
entanto, os fungos se distinguem dos vegetais por no serem
auttrofos e por armazenarem glicognio como substncia
energtica, enquanto que os vegetais armazenam o amido.
Alm disso, os fungos so hetertrofos por absoro, pois
possuem uma digesto extracorprea, eliminando para o
ambiente enzimas que digerem o alimento, os quais, em
seguida, so absorvidos. J os animais, tambm so
hetertrofos, mas por ingesto.

ESTRUTURA DOS FUNGOS
A grande maioria dos fungos pluricelular, sendo que as
clulas se caracterizam por serem longas e filamentosas e
recebem a denominao de hifas. O conjunto de hifas forma
o miclio, o qual por sua vez, no origina um tecido
verdadeiro. Poucas espcies de fungos so unicelulares ou
formados por poucas clulas, como o caso das leveduras
(Saccharomyces cervisiae).
O corpo da maioria dos fungos apresenta um
miclio vegetativo e um miclio de reproduo, tambm
denominado de corpo de frutificao. O miclio vegetativo
apresenta as hifas esto mergulhadas no meio onde o fungo
se encontra em busca de nutrientes, enquanto o miclio de
reproduo se encontra acima do substrato.

CLASSIFICAO DOS FUNGOS

FICOMICETOS
Caracterizam-se por apresentarem hifas do tipo cenocticas,
ou seja, sem paredes transversais. Estes fungos podem ser
aquticos ou terrestres, tendo-se com exemplo de
ficomicetos o bolor do po.










Bolor do po Rhizopus nigricans. Os esporngios maduros so
constitudos por muitos esporos que sob ao do vento
disseminam-se, originando novos fungos.

ASCOMICETOS
Os ascomicetos se caracterizam por apresentarem hifas em
forma de saco, em cujo interior so produzidos esporos
denominados ascsporos. No grupo dos ascomicetos esto
includos os fungos do gnero Saccharomyces, Penicillium e
Aspergillus, entre outros.

BASIDIOMICETOS
Os basidiomicetos so conhecidos como cogumelos-de-
chapu, orelhas-de-pau, entre outros. O corpo de frutificao
denominado de basidiocarpo e assemelha-se a um chapu,
o qual constitudo por vrias hifas frteis denominadas
basdios. Estes basdios, no entanto, produzem os
basidisporos, que quando expulsos podem originar novos
miclios. Entre os basidiomicetos, encontramos os fungos
comestveis (Agaricus cumpestris), os cogumelos
alucingenos (Amanita muscaria), os causadores do
ferrugem do caf, do trigo, entre outros.

Exerccios de sala #

1. (UFSC) O mofo que ataca os alimentos, os cogumelos
comestveis e o fermento de fazer o po so formados por
organismos que pertencem ao reino Fungi. Com relao a
esse grupo assinale a(s) proposio(es) verdadeira(s).
01. So organismos eucariontes, unicelulares ou
pluricelulares, autotrficos facultativos.
02. O material nutritivo de reserva o glicognio.
04. Em funo da nutrio hetertrofa, esses seres podem
viver em mutualismo, em saprobiose ou em parasitismo.
08. Alguns fungos so utilizados na obteno de
medicamentos.
16. Nutrem-se por digesto extracorprea, isto , liberam
enzimas digestivas no ambiente, que fragmentam
macromolculas em molculas menores, permitindo sua
absoro pelo organismo.
32. Na alimentao humana so utilizados, por exemplo, na
fabricao de queijos, como o roquefort e o gorgonzola.
64. Reproduzem-se, apenas, assexuadamente por meio de
esporos, formados em estruturas denominados
esporngios, ascos e basdios.

2. As doenas causadas pelos fungos so conhecidas como
micoses. Em relao a esse tipo de doena, correto
afirmar:
01. Podem ocorrer com plantas, animais e, inclusive, no
homem.
02. No homem, as micoses mais comuns ocorrem na pele.
04. O p-de-atleta e rachaduras entre os dedos so exemplos
de micoses.
08. As micoses desenvolvem-se com mais facilidade em
regies frias e secas.
16. Podem ocorrer em mucosas como, por exemplo, o
sapinho, comum em crianas.

Tarefa Mnima #

3. Os fungos so importantes para o homem em todos os
processos abaixo, exceto:
a) fermentao como na produo de bebidas alcoolicas.
b) fabricao de antibiticos, como a penicilina.
c) alimentao como os cogumelos comestveis.
d) decomposio de organismos mortos.
e) purificao do ar atravs da fotossntese.

4. Qual das opes abaixo marca uma caracterstica comum
a todos os fungos?
Please purchase PDF Split-Merge on www.verypdf.com to remove this watermark.
Incluso para a Vida Biologia B

Pr-Vestibular da UFSC
28
a) Presena de celulose como constituinte bsico da parede
celular.
b) Ausncia de pigmentos fotossintetizantes.
c) Ausncia de formao de gametas.
d) Adaptao ao parasitismo.
e) Talo do tipo miclio.

REINO ANIMAL
No Reino Animal ou Metazoa esto includos os seres
pluricelulares, hetertrofos e eucariontes.
PARAZORIOS So animais que no possuem tecido
organizado, sendo representado pelo grupo dos porferos.
EUMETAZORIOS So animais que possuem tecido
organizado e compreende todos os outros grupos animais.

PORFEROS
Os porferos ou espongirios, so animais classificados
como parazorios, por no apresentarem tecidos
organizados. So aquticos, sendo a maioria marinha, no
possuem rgos de locomoo e, por isso, so ssseis,
vivendo fixos nas rochas marinhas ou no fundo do mar.

ESTRUTURA GERAL DOS PORFEROS
A superfcie do corpo dos porferos constituda por
milhares de minsculos poros por onde a gua penetra,
trazendo o alimento (plnctons) e o oxignio. O interior do
corpo desses animais recebe o nome de espongiocele ou
trio e a abertura localizada na extremidade superior o
sculo.
As esponjas possuem um tegumento interno e outro
externo, separados por substncia gelatinosa, a espongina,
que em muitos casos, possuem estruturas de sustentao, as
espculas, as quais podem ser calcrias ou de slica.
Como no possuem tecidos, consequentemente, no
podemos mencionar sistema digestivo, excretor, nervoso,
etc.
No entanto, esses animais apresentam grupos de
clulas que executam importantes funes para a
manuteno de suas vidas, como os COANCITOS que se
localizam na cavidade atrial e tem como funo a nutrio
do animal e a remoo do oxignio da gua.
Outras clulas tambm aparecem, como:
Porcito clulas formadora dos poros;
Pinaccitos clulas com funo de revestimento;
Amebcitos transporte de alimentos e regenerao;
Esclercitos clulas formadoras de espculas.

TIPOS ANATMICOS
Os celenterados possuem como principais tipos
morfolgicos:
- Plipos - vivem fixos em substratos;
- Medusas - so de vida livre.

REPRODUO ASSEXUADA
Brotamento - forma comum de reproduo assexuada nas
esponjas, onde, forma-se um broto que pode permanecer
grudado, formando colnias, ou se soltar, formando um
indivduo isolado.

REPRODUO SEXUADA
Por no apresentarem tecidos, no podem formar gnadas
(rgos). Existem espcies dioicas (sexos separados) e
espcies monoicas (hermafroditas), neste ltimo caso, a
produo dos gametas masculinos e femininos, ocorrem em
pocas diferentes, para evitar a autofecundao. Os gametas
formam-se a partir de amebcitos, transformando-se em
espermatozoides, que so liberados e penetram em outra
esponja, portanto uma fecundao interna forma uma larva
ciliada (anfiblstula), que nada, saindo do corpo esponja e se
fixando em um substrato rgido.

UNIDADE 14

CNIDRIOS OU CELENTERADOS

So os primeiros animais com simetria radial e os nicos
que fazem metagnese ou alternncia de geraes. No
possuem aparelho respiratrio, circulatrio ou excretor. No
entanto, so os primeiros a possurem, sistema nervoso, o
qual do tipo difuso. Todos os cnidrios so aquticos e
predominantemente marinhos. Possuem uma boca e uma
cavidade gstrica.

ESTRUTURA GERAL DOS CNIDRIOS
Os cnidrios possuem duas camadas: a epiderme e a
gastroderme e entre elas situa-se a mesogleia.
Na epiderme existem milhares de clulas denominadas
cnidoblastos ou cnidcitos, as quais liberam substncias
urticantes (por isso o nome cnidrio) e paralisantes, com
funo de proteo e captura de alimentos. Estas substncias
urticantes esto no interior de um longo filamento, o qual
selocaliza dentro de uma cpsula denominada nematocisto.

REPRODUO ASSEXUADA
Podem formar colnias por brotamento ou realizarem a
estrobilizao.

REPRODUO SEXUADA
A reproduo sexuada tpica dos cnidrios a metagnese
ou alternncia de gerao. Este tipo de reproduo se
caracteriza por apresentar reproduo sexuada, seguida de
reproduo assexuada do tipo estrobilizao.

Metagnese ou alternncia de gerao

CLASSIFICAO
SCYPHOZOA - Predominam as formas de medusas. A fase
do plipo passageira. O exemplo tpico a Aurelia aurita.
HYDROZOA - Existem espcies com forma de plipos,
como, tambm de medusas. No entanto, a forma
predominante a de plipo. a nica classe que possui
representantes de gua doce.
Os principais exemplos so: Obelina - forma colnias e a
Hidra - de gua doce, que s possui forma de plipos;
Please purchase PDF Split-Merge on www.verypdf.com to remove this watermark.
Incluso para a Vida Biologia B

Pr-Vestibular da UFSC 29
Esponja
Broto
Indivduo unido
Esponja desagregada
Esponja reconstituda
Espermatozide
Clula
transportadora
vulo
Ovo
A B C

ANTHOZOA - Encontra-se exclusivamente na forma de
plipos. O exemplo tpico a anmona. Muitos plipos
desta classe formam colnias e originam, devido
calcificao, recifes de corais.

Exerccios de sala #

21. (UFSC) O filo porfera representado pelas esponjas. Na figura, as letras A, B e C referem-se aos
aspectos reprodutivos destes animais.









01. A representa um tipo de reproduo assexuada.
02. B representa um tipo de reproduo sexuada.
04. C representa, pela presena de clulas sexuais, a reproduo sexuada.
08. A denominado brotamento.
16. Para a formao do ovo, em C, deve ocorrer a fecundao.
32. Em A e B, os organismos produzidos por estes mecanismos possuem diferenas genticas em relao ao indivduo que lhe deu
origem.
64. O fenmeno apresentado em C possibilita o aumento da variabilidade entre as esponjas.

22. Relacione as colunas:
( 1 ) coancitos ( ) cavidade central das esponjas.
( 2 ) trio ( ) clula de defesa dos celenterados.
( 3 ) mesoglia ( ) digesto intracelular dos porferos.
( 4 ) cnidoblastos ( ) camada mdia da estrutura dos
porferos.
a) 2 1 4 3 c) 3 1 4 - 2
b) 2 4 - 1 3 d) 3 4 1 - 2

Tarefa Mnima #

23. (FMSAmaro-SP) Em Hydrozoa, ocorre o fenmeno de
Alternncia de Geraes com as formas plipo e medusa
que correspondem, respectivamente, s formas de
reproduo:
a) ambas assexuadas.
b) assexuada e sexuada.
c) ambas sexuadas.
d) sexuada e assexuada.
e) ambas, simultaneamente, sexuada e assexuada.

PLATELMINTOS VERMES ACHATADOS

Surge pela primeira vez, entre os grupos animais, as
seguintes caractersticas.
simetria bilateral;
presena de mesoderme (triblsticos), mas so acelomados;
olhos simples (ocelos);
sistema excretor atravs de clulas denominadas clulas-
flama ou solencitos;
sistema nervoso com gnglios cerebroides a partir dos
quais partem cordes nervosos para todo o corpo, indicando
um incio de cefalizao.

CLASSIFICAO
Este filo possui representantes de vida livre e parasita de
interesse mdico e veterinrio. So trs classes de
platelmintos:

CLASSE TURBELLARIA
So de vida livre;
Possuem tubo digestivo incompleto;
O corpo no segmentado;
As planrias so monoicas (hermafrodita), mas realizam
fecundao cruzada, trocando espermatozoides. Tambm
podem se reproduzir assexuadamente por lacerao;
Presena de ocelos.

CLASSE TREMATODA
So endoparasitas;
Possuem tubo digestivo incompleto;
O corpo no segmentado;
Possuem ventosas para fixao no hospedeiro;
O principal representante o esquistossoma. O macho
possui canal ginecforo o qual aloja a fmea. Outro exemplo
a Fascola heptica, parasita de carneiros.

CLASSE CESTODA
So endoparasitas;
Podem atingir mais de 8 metros de comprimento;
No possuem aparelho digestivo;
O corpo segmentado;
Os principais representantes so: Taenia solium e a Taenia
saginata.
Larva chama-se cisticerco.

PLATELMINTES DE INTERESSE MDICO
Schistosoma mansoni
o verme causador da esquistossomose (barriga
dgua). Vivem no interior dos vasos abdominais, onde
Please purchase PDF Split-Merge on www.verypdf.com to remove this watermark.
Incluso para a Vida Biologia B

Pr-Vestibular da UFSC
30
ocorre o acasalamento e a fecundao. As fezes dos
indivduos parasitados, contendo os ovos, ao atingirem a
gua doce, originam uma larva ciliada, denominada
miracdio. Esta larva, ao encontrar um caramujo
Biomphalaria, o penetra e transforma-se em sacos de
centenas de clulas reprodutoras, os esporocistos, que
originaro as larvas do tipo cercrias, com ventosas.
Estas larvas abandonam o caramujo, penetrando,
posteriormente, na pele das pessoas para fechar o ciclo de
vida.

Taenia solium e Taenia saginata
Vive no intestino humano e, sendo hermafrodita, formam
dentro das progltides os ovos, os quais so eliminados com
as fezes, contaminando o solo. O porco (no caso da Taenia
solium) ou o boi (no caso da Taenia saginata), ao ingerirem
os ovos, tero na musculatura formas da larva da tnia
denominadas de cisticerco e popularmente de pipoquinha ou
canjiquinhas. O homem, ao ingerir os cisticercos, ao comer a
carne malcozida ou crua, passa a adquirir a tenase. No
entanto, se o homem ingerir diretamente o ovo da Taenia
solium, passar a ter uma doena denominada cisticercose.

UNIDADE 15

ASQUELMINTOS VERMES
CILNDRICOS

A maioria dos asquelmintos de vida livre. No entanto, a
nica classe de interesse mdico a dos nematelmintes,
pois composta por vermes parasitas. Estes animais so
triblsticos, pseudocelomados e possuem o corpo protegido
por uma cutcula resistente, em funo da qual foi dado o
nome do filo (askos = saco, envoltrio; helminis = vermes).
Principais caractersticas do grupo:
o primeiro grupo animal a possuir sistema digestivo
completo, ou seja, com boca e nus;
o sistema nervoso ganglionar e ventral;
a excreo realizada por clulas em forma de H, os tubos
em H.
no possuem sistema respiratrio, sendo realizada
diretamente pela epiderme (cutnea).
no possuem sistema circulatrio;
no so segmentados;
Possuem sexos separados, sendo a fecundao interna.

NEMATELMINTES DE INTERESSE MDICO

Ascharis lumbricoides
Vivem no intestino do homem delgado e causam a
ascaridase, conhecida como lombriga, sendo, portanto um
endoparasita. No intestino, ocorre reproduo sexuada e os
ovos eliminados juntos com as fezes contaminam a gua e
os alimentos. Ao serem ingeridos surge uma larva no
intestino, migrando, depois, para a corrente sangunea,
fgado, corao, pulmo, traqueia, retornando ao sistema
digestivo.

Please purchase PDF Split-Merge on www.verypdf.com to remove this watermark.
Incluso para a Vida Biologia B

Pr-Vestibular da UFSC 31
Ancylostoma duodenalis e Necator americanus
Estes vermes possuem o ciclo semelhante ao do Ascaris lumbricoides, no entanto, o contgio ocorre diretamente atravs da pele,
geralmente dos ps. Ao cair na circulao passa por vrios rgos, como o ascaris, chegando at os pulmes, traqueias, indo se
instalar no intestino. O ancilostoma possui dentes e o necator lminas, ambas estruturas servem para penetrar na pele e ao
chegarem ao sistema digestivo, raspar a parede intestinal, para alimentarem-se de sangue, fazendo assim com que a pessoa fique
com aspecto anmico. Devido a este fato a doena causada o amarelo.

Exerccios de sala #

1. (UFSCar) A esquistossomose hoje uma das principais
doenas endmicas em pases subdesenvolvidos da regio
neotropical, incluindo o Brasil. Santa Catarina constitui o
limite extremo meridional da distribuio desse problema de
sade pblica no pas. Focos de caramujos infectados tm
sido encontrados no municpio de So Francisco do Sul.
Assinale a(s) proposio(es) correta(s):
01. Caramujos do gnero Biomphalaria so vetores dessa
doena no Brasil.
02. Miracdios e cercrias so estgios larvais do parasita.
04. As cercrias penetram no homem atravs da pele,
migram pela circulao sangunea e alojam-se,
preferencialmente, no sistema porta-heptico.
08. Os ovos do verme adulto passam atravs da parede do
intestino humano e so eliminados atravs das fezes
humanas.
16. Medidas de preveno da doena incluem melhoria dos
hbitos e condies de higiene da populao humana.

2. Uma criana de regio rural de Santa Catarina, aps
consulta mdica, foi diagnosticada como portadora de
Ascaridase. Com relao a esta parasitose, correto
afirmar:
01. No existem medicamentos que permitam sua cura.
02. provocada por um nematdeo popularmente conhecido
como lombriga.
04. Um de seus sintomas a anemia.
08. Um outro sintoma o edema dos membros inferiores.
16. Uma maneira de evit-la o tratamento de gua e
esgotos.

3. (UFSC) Sobre os vermes do gnero Taenia, assinale a(s)
proposio(es) correta(s).
01. A Taenia solium um asquelminto do grupo trematoda.
02. A tenase causada pela ingesto de cisticercos em carne
bovina ou suna.
04. A Taenia solium no causa a cisticercose humana.
08. A higiene pessoal e o saneamento bsico so importantes
medidas na preveno das parasitoses causadas por
Taenia sp.
16. A Taenia solium adulta hermafrodita, apresenta o
corpo segmentado, esclex com 4 ventosas e uma coroa
de acleos. Os ltimos segmentos so cheios de ovos,
que ficam encistados no tecido muscular de sunos.
32. A cisticercose humana ocasionada pela presena da
larva da Taenia solium e adquirida pela ingesto de
ovos do parasito liberados nas fezes de pessoas
infectadas.

4. Na classe cestoda, os principais representantes
endoparasitas humanos so as Taenia solium e Taenia
saginata. Assinale a alternativa verdadeira, considerando as
proposies seguintes:
I - A Taenia saginata transmitida pela carne de gado com
pipoca e apresenta ventosa com ganchos no seu esclex.
II - A Taenia solium transmitida atravs da carne de porco
com pipoca e apresenta pseudoventosas no esclex.
III - Ambas, a Taenia solium e Taenia saginata, possuem
estrobilizao e no apresentam aparelho digestivo.
a) Apenas III est correta.
b) Apenas II est correta.
c) Apenas I est correta.
d) As proposies I e II esto corretas.
e) As proposies I e III esto corretas.

MOLUSCOS

CARACTERSTICAS GERAIS DOS MOLUSCOS
Os moluscos so animais de corpo mole e a maioria das
espcies possui o corpo dividido em cabea, p e massa
visceral. Podem conter rgos recobertos ou no por concha,
a qual produzida pela cavidade denominada manto ou
palio. Possuem simetria bilateral, so protostmios,
triblsticos e celomados.
Existem espcies marinhas, terrestres e dulccolas.
So de vida livre e raramente parasitas. O sistema excretor
constitudo por nefrdeos e o sistema circulatrio aberto
ou lacunar, com exceo dos polvos e das lulas, os quais,
alm de possurem hemoglobina (como nos vertebrados), o
sistema circulatrio fechado. Muitos tambm apresentam
uma lngua com dentculos denominada de rdula, utilizada
para raspar o alimento no substrato. A maioria dos moluscos
hermafrodita, com fecundao cruzada, mas existem
espcies marinhas dioicas e com fecundao externa, como
ocorre com as ostras e os mexilhes.

CLASIFICAO

CLASSE PELICPODA OU BIVALVIA
So exclusivamente aquticos. O significado do nome da
classe derivada do seu p em forma de machado, porm
so conhecidos pela presena de duas valvas, sendo assim
chamados de bivalves. No possuem diferenciao da
cabea e no possuem rdula. So diicos, com fecundao
externa e desenvolvimento indireto, sendo a larva ciliada
denominada de trocfora e vliger. Os exemplos mais
conhecidos so o marisco, a ostra, o berbigo.
]
CLASSE CEFALPODA
Possuem ps que saem da cabea. So os moluscos mais
complexos, com um crebro e olhos bem desenvolvidos.
Possuem concha interna, so diicos, com fecundao
interna e desenvolvimento externo.

CLASSE GASTRPODA
So os moluscos que possuem os ps na barriga. O exemplo
tpico o caracol, univalve e a lesma, que no possui
concha. A cabea possui dois pares de tentculos, o primeiro
olfativo e o segundo portador de olhos.


Please purchase PDF Split-Merge on www.verypdf.com to remove this watermark.
Incluso para a Vida Biologia B

Pr-Vestibular da UFSC
32
CLASSE ESCAFPODA
Conhecidos como dentlios, por possurem sua forma
semelhante a um dente. Vivem enterrados na areia.

CLASSE ANFINEURA
O corpo semelhante a dos gastrpodes e possuem oito
placas calcreas. Encontrado em costes rochosos.

Quton

ANELDEOS

CARACTERSTICAS GERAIS DOS ANELDEOS
Os aneldeos so vermes mais evoludos e se caracterizam
por apresentarem um corpo segmentado (metameria), so
celomados, protostmios, com simetria bilateral. O sistema
digestivo completo e possuindo papo responsvel pela
digesto fsica a e moela, que realiza a digesto mecnica.
Alm disso, o intestino possui um dobramento que
aumenta a superfcie de absoro, denominado tiflossole. O
sistema nervoso do tipo ganglionar e ventral, o sistema
circulatrio fechado e a excreo realizada por
nefrdeos, os quais so encontrados em nmero de dois, em
cada anel que forma o corpo dos aneldeos.
A respirao cutnea, sendo que as espcies
marinhas apresentam respirao branquial.

CLASSIFICAO

CLASSE POLIQUETA
Os poliquetos so aneldeos marinhos que se caracterizam
por possurem vrias cerdas no corpo.

CLASSE OLIGOQUETA
Possuem poucas cerdas que, a olho nu, nem sempre podem
ser observadas. So geralmente terrestres e o exemplo tpico
a minhoca.


CLASSE ACHAETA ou HIRUDINEA
A maioria de gua doce e se caracterizam por possurem
duas ventosas para fixao e outra para sugar o sangue. As
espcies mais conhecidas so as sanguessugas.



Exerccios de sala #

5. Mexilho, marisco e ostra-de-pobre so alguns dos
nomes populares do molusco Perna perna, de grande
ocorrncia no litoral de Santa Catarina. As populaes locais
consomem o molusco na sua alimentao e h interesse no
seu cultivo comercial, devido ao seu alto teor nutritivo. Os
moluscos apresentam, como caractersticas gerais:
01. Corpo mole, geralmente protegido por concha.
02. Presena de tentculos em algumas espcies.
04. Sistema circulatrio aberto ou lacunar.
08. Concha univalva e bivalva.
16. Vida livre ou ssseis.
32. Aquticos ou terrestres.

6. Vem aumentando consideravelmente o nmero de
pessoas envolvidas com a minhocultura. Assim, as
minhocas recuperam sua devida importncia na produo
agrcola. Sobre esses animais, correto afirmar que:
01. Solos que os possuem so compactados e com
dificuldades de penetrao de gua.
02. Possuem corpo dividido em anis, da serem
denominados aneldeos.
04. So hermafroditas sem, contudo, realizarem a
autofecundao.
08. Promovem a formao do hmus, com a transformao
da matria morta que cai no solo.
16. Escavam galerias que proporcionam um maior
arejamento do solo.
7. (PUC-SP) Os aneldeos so animais com o corpo
formado por muitos segmentos ou metmeros, e que
apresentam como caracterstica obrigatria:
a) habitat aqutico.
b) sistema excretor com um par de nefrdios por segmento.
c) respirao branquial.
d) hermafroditismo.
e) um par de cerdas por segmento.

8. Quanto reproduo, as minhocas podem ser
classificadas como:
a) diicas com fecundao cruzada, desenvolvimento direto.
b) diicas com fecundao cruzada e desenvolvimento
indireto.
c) monoicas com autofecundao cruzada e
desenvolvimento indireto.
d) monicas com fecundao cruzada e desenvolvimento
direto.
e) monicas com fecundao cruzada e desenvolvimento
indireto.

UNIDADE 16

ARTROPODA

CARACTERSTICAS GERAIS DOS ARTRPODOS
Os artrpodos se caracterizam por apresentarem apndices
(patas) articulados para locomoo e um exoesqueleto
quitinoso. Devido ao exoesqueleto rgido, o crescimento
destes animais descontnuo, sendo necessrio, para que o
animal cresa a realizao de mudas ou ecdise.
Please purchase PDF Split-Merge on www.verypdf.com to remove this watermark.
Incluso para a Vida Biologia B

Pr-Vestibular da UFSC 33
Nesses, animais a circulao aberta ou lacunar possuindo
hemocianina como pigmento respiratrio. O sistema nervoso
ganglionar e ventral, com ocelos ou olhos e antenas com
funo tctil ou quimioreceptora, alm de rgos gustativos
e auditivos. So dioicos com dimorfismo sexual, a
fecundao interna e o desenvolvimento externo
caracterstica da grande maioria das espcies.

Quadro com os diferentes grupos de artrpodos e suas caractersticas peculiares


A CLASSE INSECTA representada pelos insetos,
apresentam respirao do tipo traqueal, a qual se caracteriza
por ser constituda por tubos (semelhantes a traqueias) que
se abrem ao longo do corpo do animal (na regio do
abdome) e se comunicam diretamente com os vrios tecidos.
Consequentemente, nesses animais o sistema circulatrio
no participa do transporte dos gases. O sistema excretor so
projees do intestino, denominado tbulos de Malpighi. Os
insetos possuem sexos separados, fecundao interna e
podem ser classificados, de acordo com o desenvolvimento,
em:

AMETBOLO
Que no sofre metamorfose. Exemplo: traa.

HEMIMETBOLO
A metamorfose ocorre de maneira gradual, como por
exemplo: o gafanhoto.

HOLOMETBOLO
Com metamorfose completa, como ocorre nas borboletas e
nas moscas. Nestes animais, do ovo, eclode surge uma larva
ou lagarta, totalmente diferente do adulto.

A CLASSE CRUSTCEA representada pelos camares,
siris, lagostas, barata-da-praia, tatu-de-jardim e as cracas,
possuem como sistema excretor as glndulas verdes ou
antenais.

CLASSE ARACHNIDA representada pelas aranhas,
escorpies, carrapatos e caros. As aranhas so ovparas e os
escorpies so ovovivparos. Alguns possuem apndices
parecidos com patas denominados palpos, que servem para
prender as presas ou como rgo sexual e, tambm, as
quilceras, rgos inoculadores de veneno. Algumas aranhas
possuem fiandeiras para tecer as teias.

CLASSE CHILOPODA - compreende os quilpodos
(centopeias e lacraias)
CLASSE DIPLOPODA - compreende o piolho-de-cobra e
embus.

Piolho-de-cobra




CARACTERSTICAS GERAIS
Todos os representantes do Filo Equinodermata so
exclusivamente marinhos e se caracterizam por
apresentarem espinhos na pele com pequenas estruturas em
forma de pina, as pedicilrias, que tm a funo: defesa e
limpeza do corpo.
Tambm possuem um conjunto de canais
denominado sistema ambulacrrio, o qual realiza
importantes funes como: participao na locomoo,
obteno de alimento, respirao e excreo. Seu sistema
nervoso pouco desenvolvido e, apesar dos equinodermos
apresentarem um aspecto simples, so animais
deuterostmios, caracterstica embrionria que torna estes
animais muito prximos do Filo dos Cordados.
EQUINODERMAS
Please purchase PDF Split-Merge on www.verypdf.com to remove this watermark.
Incluso para a Vida Biologia B

Pr-Vestibular da UFSC
34

Esquema representando o sistema ambulacral

CLASSIFICAO

CLASSE DOS ASTEROIDES - Possuem o corpo em forma
de estrela, sendo que seu corpo tem de 5 a 50 braos que
partem de um anel central. O exemplo mais conhecido a
estrela-do-mar.

CLASSE DOS EQUINOIDES - O exemplo tpico o
ourio-do-mar; possui o corpo desprovido de braos e com
lanterna de Aristteles (aparelho triturador do alimento).

CLASSE DOS OFIUROIDES - Possuem corpo achatado do
qual saem cinco tentculos em forma de serpentes, por isso a
denominao serpente-do-mar. So os nicos que no
possuem nus.

CLASSE DOS CRINOIDES - Nesta Classe esto os lrios-
do-mar. Estes animais so fixos e seus tentculos do
aspecto de flor. O nus e a boca ficam lado a lado.

CLASSE DOS HOLOTUROIDES - Possuem corpo
alongado e cilndrico, com tentculos ao redor da boca que
so modificaes dos ps ambulacrais. O mais conhecido o
pepino-do-mar.

Exerccios de sala #

1. (UFSC) As curvas abaixo ilustram a diferena de
crescimento entre os animais. Com base na figura e no
assunto crescimento em animais, CORRETO afirmar que:













01. As curvas A e B representam, respectivamente, tipos de
crescimento descontnuo e contnuo.
02. A curva A pode representar o crescimento de um
vertebrado.
04. A curva B pode representar o crescimento de um
artrpode.
08. Os intervalos assinalados pelas letras C e D representam
momentos de crescimento nulo.
16. Em C e D, os animais poderiam estar sofrendo muda.
32. Os animais, cujos crescimentos so representados pela
curva A, apresentam exoesqueleto.

2. Os artrpodos apresentam uma enorme biodiversidade,
especialmente da classe dos insetos, a qual supera sozinha
em nmero de espcies todos os outros filos do reino
Animal somados.
Assinale a somatria das alternativas corretas com relao
aos artrpodos e suas adaptaes:
01. Os insetos possuem um exoesqueleto quitinoso muito
resistente contra choques, raios solares e desidratao.
02. O crescimento de um inseto lento e progressivo
semelhante ao dos humanos.
04. Os insetos possuem um par de antenas com capacidade
de percepo olfativa e qumica do ambiente.
08. A maioria dos insetos excreta seus resduos metablicos
na forma de cido rico evitando a perda de gua durante
a excreo.
16. As aranhas como os insetos que possuem quatro pares de
patas e dois pares de antenas.
32. Muitos insetos possuem respirao branquial quando
adultos como a barata.
64. Os insetos so homeotermos, ou seja, possuem uma
temperatura constante que independe da temperatura
ambiental.

3. Nesta questo faz-se referncia a vrios filos de animais
invertebrados. Com base no s em caractersticas
morfolgicas, mas tambm em sistemtica, correto afirmar
que:
01. O camaro, o caranguejo e o siri so moluscos
comestveis.
02. Os equinodermos so exclusivamente marinhos.
04. Os aneldeos so animais de corpo cilndrico, no-
segmentado.
08. Devido existncia de um exoesqueleto quitinoso, os
artrpodes sofrem o fenmeno das mudas ou ecdises
durante o seu crescimento.
16. Os insetos pertencem ao filo artrpoda e so os nicos
invertebrados capazes de voar.

4. Assinale a(s) alternativa(s) que associa(m) corretamente
o grupo a algumas de suas caractersticas:
01. Equinodermas - lanterna-de-aristteles e madreporito
02. Moluscos - rdula e manto
04. Porferos - cnidoblastos e nematocistos
08. Celenterados - coancitos e espculas
16. Aneldeos - clitelo e parapdios
32. Artrpodes - tubos de Malpighi e ecdises
64. Asquelmintos - exoesqueleto quitinoso e clulas-flama

5. (Mackenzie-SP) Os artrpodes constituem um grupo de
seres vivos bem varivel com relao sua organizao e
processos metablicos. Uma caracterstica comum a todos
eles :
a) a fecundao interna com desenvolvimento indireto.
b) o sistema digestivo completo.
c) a presena de pigmento respiratrio no sangue.
d) a presena de antenas.
e) a excreo por tbulos de Malpighi.





A
B

C

D
TAMANHO
TEMPO
Please purchase PDF Split-Merge on www.verypdf.com to remove this watermark.
Incluso para a Vida Biologia B

Pr-Vestibular da UFSC 35
UNIDADE 17

CORDADOS

CARACTERSTICAS GERAIS DOS CORDADOS
Os animais do Filo Cordado se caracterizam por possurem:

SISTEMA NERVOSO DORSAL - nos filos anteriores os
cordes nervosos eram duplos e ventrais. Os cordados so os
nicos nos quais o sistema nervoso tem origem a partir do
tubo neural, localizado na regio dorsal do animal.

NOTOCORDA - estrutura de sustentao, situada na linha
mediana dorsal, ocorrendo durante o processo embrionrio
ou permanecendo por toda a vida, em alguns; serve de
molde para o tecido sseo que vai formar a coluna vertebral.

FENDAS BRANQUIAIS - esta estrutura ocorre pelo menos
na fase embrionria; em seres aquticos permanece por toda
a vida.

CLASSIFICAO DOS CORDADOS
Acranios ou protocordata.
No possuem crnio e vrtebras. So divididos em:

SUBFILO UROCORDATA Tambm chamados de
Tunicados, este grupo possui notocorda na fase embrionria,
no estando presente na fase adulta. So animais marinhos,
ssseis e o mais conhecido a Ascdia.

Ascdia adulta e sua larva

SUBFILO CEFALOCORDATA
A notocorda persiste no adulto, desde a cauda at a cabea.
representado pelo anfixo, um pequeno animal que parece
um peixe sem nadadeiras. Vive quase sempre enterrado na
areia do mar. muito estudado pelos embriologistas, por
possuir notocorda, tubo neural e fendas branquiais.


Anfioxo

SUBFILO VERTEBRATA

CARACTERSTICAS GERAIS DOS VERTEBRADOS
Os vertebrados se caracterizam por apresentarem um
endoesqueleto cartilaginoso ou sseo, um eixo principal
(coluna vertebral) e um crnio na regio anterior. Os
vertebrados, como pode ser observado no quadro de
classificao, so divididos em dois grupos:

Agnata (sem mandbula);
Gnatostomata (com mandbula).

GRUPO AGNATA
Os agnatos ou ciclostomados (lampreias e feiticeiras) no
possuem mandbula, escamas e nadadeiras pares. O corao,
como nos peixes, possuiu duas cavidades, um trio e um
ventrculo.

Lampreia

GRUPO GNATOSTOMA
Este grupo subdividido em duas superclasses:
Piscies (peixes); Tetrpodas.

SUPERCLASSE DOS PEIXES

CARACTERSTICAS GERAIS DOS PEIXES
Os peixes possuem o corpo recoberto por escamas e
movimentam-se por nadadeiras pares e mpares. So
pecilotrmicos, apresentam respirao branquial e circulao
simples com um corao constitudo por duas cavidades.
Esta superclasse subdividida em duas classes:
Condrcties peixes com esqueleto cartilaginosos;
Ostecties peixes com esqueleto sseos.
Estas duas classes diferenciam-se por vrios fatores.

CHONDRICTHYES OU CARTILAGINOSOS
esqueleto cartilaginoso;
boca ventral e transversal;
no possuem bexiga natatria;
no possuem oprculo (com exceo das quimeras);
corpo coberto por escamas de origem dermoepidrmica,
tambm denominadas escamas placoides ou dentculos
drmicos.

OSTEICHTHYES OU SSEOS
esqueleto sseo;
boca anterior;
possuem bexiga natatria;
possuem oprculo protegendo as brnquias;
corpo geralmente coberto por escamas de origem drmica,
que podem ser de trs tipos: cicloide, ctenoide e ganoide.
Existem ostecties de pele lisa, sem escamas.

Please purchase PDF Split-Merge on www.verypdf.com to remove this watermark.
Incluso para a Vida Biologia B

Pr-Vestibular da UFSC
36
LINGUADO
TUBARO
ATUM
RAIA
CAVALO-
MARINHO
Exerccios de sala #

1. (UFSC) - Segundo estimativas recentes, o grupo dos
Peixes est representado por mais de 20 mil espcies, sendo
60% marinhas. Embora no parea, um grupo muito
diversificado, com variadas adaptaes, mltiplas formas e
tamanhos...
Csar, Cezar e Bedaque Cincias, Editora Saraiva, 1999.
p. 85.
Com relao a esse grupo animal, assinale a(s)
proposio(es) correta(s).















01. Os exemplos esquematizados na figura mostram que os
peixes se diferenciam, com relao ao formato do corpo
e disposio das nadadeiras.
02. Os peixes so vertebrados aquticos, homeotermos de
respirao branquial ou cutnea.
04. Os peixes cartilaginosos apresentam bocas terminais e as
brnquias protegidas por oprculos.
08. O tubaro e a raia representam o grupo dos peixes
cartilaginosos.
16. A linha lateral, visvel nas figuras do atum e do linguado,
um rgo sensorial, atravs do qual o peixe pode
perceber a direo e a velocidade da correnteza da gua.
32. O cavalo-marinho, como a maioria dos peixes, ovparo;
nesse animal, o macho, de modo bastante peculiar,
carrega seus ovos em bolsas incubadoras.

2. Tainhas (osteicties) e caes (condricties) fazem parte da
superclasse dos peixes. Sobre esses animais, assinale a
alternativa correta.
a) Nos condricties a boca ventral.
b) Os osteicties possuem esqueleto cartilaginoso.
c) Apenas os condricties possuem nadadeira caudal.
d) Os osteicties no apresentam oprculo.
e) Os osteicties possuem vlvula espiral.

SUPERCLASSE DOS TETRPODAS

CLASSE ANPHIBIA (ANFBIOS)

CARACTERSTICAS GERAIS DOS ANFBIOS
Representam um importante passo na histria evolutiva dos
vertebrados, pois foram os primeiros a conquistarem o
ambiente terrestre. No entanto, ainda dependem do ambiente
aqutico, principalmente, para a reproduo. A sua pele
lisa, sem escamas, como mucosas, adaptadas a respirao
cutnea. Nesses animais a respirao branquial na fase
larval e cutnea e pulmonar na fase adulta. O corao possui
trs cavidades, sendo do tipo dupla, incompleta e fechada,
fatos que os torna pecilotrmicos. A classe dos anfbios
dividida em trs ordens:

ORDEM ANURA
O nome significa sem cauda, so os sapos, rs e pererecas.
Alguns sapos possuem junto cabea glndulas paratoides
que produzem um lquido venenoso que no pode ser
espirrado ou inoculado, mas quando um predador tenta
com-lo, ao apertar esta glndula, libera o veneno.

ORDEM APODA/GIMNOFIONA
Possuem semelhana com as cobras. Sendo o principal
exemplo, a cobra-cega.

ORDEM URODELA
So os anfbios com cauda e patas, possuindo um aspecto de
lagarto, sendo o exemplo tpico a salamandra.

UNIDADE 18

CLASSE REPTILIA (RPTEIS)

CARACTERSTICAS GERAIS DOS RPTEIS
A pele dos rpteis impermevel e seca, recoberta por
placas crneas drmicas (crocodilianos), escamas ou
carapaas (tartarugas). Sua respirao pulmonar e
circulao fechada, dupla e incompleta, apesar de possuir
quatro cavidades, pois o septo que separa os dois ventrculos
permite a mistura do sangue arterial com o venoso, com
exceo dos crocodilianos. Estes so os principais fatores
que os tornam pecilotrmicos. Ocorre fecundao interna,
sendo que a maioria ovpara, com ovos ricos em vitelo,
amnitas e com casca. Porm algumas espcies so
ovovivparas. Ambos possuem desenvolvimento direto.
Esta classe possui as seguintes ordens:

QUELONIA - Corpo recoberto por carapaa compreende as
tartarugas (marinhas), cgados (duccolas) e jabutis
(terrestres).
Please purchase PDF Split-Merge on www.verypdf.com to remove this watermark.
Incluso para a Vida Biologia B

Pr-Vestibular da UFSC 37
SQUAMATA
So os rpteis revestidos por escamas. Subdividem-se em
lacertlios (lagartos e lagartixas) e ofdios (cobras), estes
ltimos, apresentam vrias espcies peonhentas, que podem
ser identificadas por possurem: cabea triangular, cauda que
se afina bruscamente, olhos pequenos, movimentos
vagarosos, atitude de ataque, a maioria ovovivparas.
So exemplos de peonhentas: jararaca, surucucu, coral.
Sendo exemplos de no-peonhentas: sucuri, jibia.

CROCODILIA
Possuem escamas e placas sseas drmicas revestindo o
corpo. So animais semi-aquticos e podem ser encontrados
na gua doce e no mar. Esto divididos em trs grupos: o
dos jacars, o dos crocodilos e o dos gaviais.

Exerccios de sala #

1. Um dos gneros de ofdios encontrados em Santa
Catarina o Bothrops sendo um exemplo comum a jararaca.
Leia as proposies abaixo e assinale as corretas sobre as
espcies do referido gnero.
01. So peonhentas.
02. Possuem olhos com pupila vertical.
04. A fosseta loreal est ausente.
08. Apresentam dentes inoculadores.
16. Costumam viver em locais extremamente frios e midos.
32. No existe um soro desenvolvido para neutralizar o
veneno.

2. O ovo terrestre foi uma grande inveno dos
vertebrados, que assim puderam conquistar o ambiente
terrestre. Essa conquista ocorreu pela primeira vez com:
a) aves c) anfbios e) mamferos
b) rpteis d) peixes


CLASSE DAS AVES

CARACTERSTICAS GERAIS DAS AVES
So vertebrados bpedes, homeotrmicos, que
apresentam respirao pulmonar, circulao dupla, fechada e
corao com quatro cavidades, alm de hipoderme, que at
ento no existia, o que lhes proporciona a homeotermia. Os
membros anteriores so transformados em asas.

Estrutura geral das aves

As aves so dotadas de vrias estruturas que facilitam o vo,
como, por exemplo:
ossos pneumticos;
musculatura peitoral bem desenvolvida;
sacos areos;
presena da glndula uropigiana que impermeabiliza as
penas, impedindo que se encharquem. Estas so as nicas
glndulas presentes na pele das aves;
osso externo em forma de quilha.
Na traqueia, h um rgo sonoro, a siringe.
So dioicos com dimorfismo sexual, fecundao
interna e desenvolvimento direto, no interior dos ovos.
Possuem sistema digestivo completo, com papo e moela.

CLASSE MAMMALIA (MAMFEROS)

CARACTERSTICAS GERAIS DOS MAMFEROS
presena de glndulas mamrias;
respirao exclusivamente pulmonar;
pele com diversos fneros (cornos, unhas, plos) e
glndulas (mamrias, sebceas, sudorparas);
corao com quatro cavidades, e circulao dupla e
completa;
so os nicos com placenta e cordo umbilical;
presena de diafragma;
so todos dioicos, havendo ntido dimorfismo sexual, com
desenvolvimento interno e direto.

A classe dos mamferos compreende trs grupos:
MONOTREMADOS OU PROTRIOS - Os nicos
mamferos ovparos; possuem glndulas mamrias, cloaca; o
exemplo tpico o ornitorrinco da Austrlia.
Ornitorrinco quidna

MARSUPIAIS OU METATRIOS - So dotados de uma
bolsa, chamada de marspio, na regio do ventre, que serve
de abrigo para o embrio, pois o tero destes animais
pouco desenvolvido, ento o desenvolvimento do embrio se
d no marspio, sendo o exemplo tpico o canguru.

PLACENTRIOS OU EUTRIOS - Possuem o tero
bem desenvolvido, nos quais se encontram diversas ordens,
como por exemplo:

CARNVORA - Possuem dentes caninos bem
desenvolvidos, adaptados a rasgar e a perfurar a carne de
outros animais que usam como alimento.
Exemplo: cachorro, leo, urso, gato, leo-marinho, foca.

CHIROPTEPTERA - Braos modificados em asas.
Exemplo: morcego.

CETACEA - Mamferos aquticos, representados pelas
baleias e golfinhos. Possuem membros anteriores
transformados em nadadeiras, membros posteriores ausentes
e cauda propulsora bem desenvolvida. So excelentes
nadadores. Narinas no alto da cabea, por onde sai o
esguicho da baleia e do golfinho. Esse esguicho se

Please purchase PDF Split-Merge on www.verypdf.com to remove this watermark.
Incluso para a Vida Biologia B

Pr-Vestibular da UFSC
38
forma no momento em que esses animais expiram: o ar
quente, contendo vapores de gua, expelido e condensa ao
entrar em contato com o ar externo mais frio.

PRIMATAS - Mos e ps com cinco dedos distintos, cabea
em ngulo reto com o pescoo.
Exemplo: homem.

Exerccios de sala #

3. (UFSC) Com relao ao Reino Animal, assinale a(s)
proposio(es) correta(s).
01. Formado por animais ssseis e geralmente hermafroditas,
a maioria das espcies do filo Porferos so aquticas,
apesar de existirem algumas espcies terrestres.
02. A lombriga e a solitria (tnia), parasitas do intestino
humano, pertencem aos filos Platelmintos e
Nematelmintos, respectivamente.
04. Animais cujo corpo formado por numerosos anis
repetidos (metmeros) pertencem ao filo Aneldeos, do
qual a minhoca o representante mais conhecido.
08. O filo Cnidrios formado, basicamente, por dois tipos
morfolgicos de indivduos, que so: plipos, cujo
principal representante a gua-viva, e medusas,
representadas pelos corais.
16. No filo Moluscos existem indivduos com concha
externa, como o caso das ostras e mexilhes, e tambm
indivduos sem ela, como o caso da lula e do polvo.
32. De todo o reino animal, o filo Artrpodes o que
apresenta o maior nmero de espcies.
64. No filo Cordados, somente a classe Mamferos apresenta
circulao dupla e completa.

4. (UFSC) O Reino Animal apresenta grande variedade de
organismos, com cerca de um milho de espcies
catalogadas. Sobre os principais grupos animais, assinale
a(s) proposio(es) correta(s).
01. Todas as espcies do grupo Equinodermos so
exclusivamente marinhas.
02. A tnia e a lombriga, vermes que causam doenas ao ser
humano, pertencem ao grupo dos Nematdeos.
04. Apesar de terem organizao corporal muito simples, os
Porferos apresentam trs folhetos germinativos.
08. Espcies que apresentam exoesqueleto podem ser
observadas nos grupos Moluscos, Artrpodes e
Equinodermas.
16. Todos os Cordados possuem vrtebras.
32. Os Aneldeos so parasitas obrigatrios.
64. Anmonas, guas-vivas e corais so representantes dos
Cnidrios.

UNIDADE 19

ECOLOGIA

A palavra Ecologia deriva do grego Oikos = casa Logos =
estudo. A ecologia estuda as relaes entre os seres vivos e o
meio ambiente.
O termo ecologia foi usado pela primeira vez por
Ernest Haeckel (bilogo alemo) em 1870, para definir as
interaes do seres vivos entre si e com o meio ambiente.
Para sistematizar o estudo da ecologia, utilizamos os
seguintes nveis de organizao:

CLULAS TECIDOS RGOS SISTEMAS
INDIVDUO POPULAO COMUNIDADE
ECOSSISTEMAS - BIOSFERA

CONCEITOS ECOLGICOS
Os principais conceitos ecolgicos so:
ESPCIE - Indivduos semelhantes que se reproduzem
entre si, gerando descendentes frteis e apresentando o
mesmo nmero de cromossomos.
POPULAO - conjunto de indivduos da mesma espcie,
que vive na mesma rea e no mesmo tempo, mantendo, entre
si, uma interao. O tamanho de uma populao deve
manter-se, mais ou menos constante ao longo do tempo. As
populaes apresentam um POTENCIAL BITICO, ou
seja, uma capacidade de aumentar o nmero indivduos em
exageradamente, desde que as condies sejam timas. No
entanto, caso uma populao cresa muito, pode ocorrer a
extino de uma ou mais populaes menores. Sendo assim,
existem fatores que regulam o tamanho das populaes,
como os outros seres vivos (parasitas e predadores) e os
aspectos fsicos e qumicos do ambiente. Alm disso, o
crescimento da populao tambm depende da taxa de
natalidade e da taxa de imigrao, j os fatores que a
diminuem so: a taxa de mortalidade e a taxa de emigrao.
HABITAT - o lugar onde a populao interage com os
componentes biticos e abiticos.
FATORES BITICOS - Os fatores biticos constituem os
seres vivos do ecossistema.
FATORES ABITICOS - So todos os fatores fsicos e
qumicos do ecossistema.
FATORES FSICOS - Radiao solar, temperatura, luz,
umidade, ventos.
FATORES QUMICOS - Nutrientes orgnicos ou
inorgnicos presentes no solo ou guas. Na atmosfera os
principais fatores abiticos so: CO
2
; CO; O
2
; O
3
; H
2
O; N
2.

NICHO ECOLGICO - o comportamento da
populao. o modo pelo qual a populao se adapta a seu
meio. Qual o alimento da populao, de quem os indivduos
servem de alimento e seu comportamento reprodutivo.
COMUNIDADE OU BIOCENOSE OU BIOTA - o
conjunto de vrias populaes que ocupam uma determinada
rea.
BIOSFERA - a camada da terra ocupada pelos seres
vivos.

Exerccios de sala #
1. (UFSC) Faa a associao entre os termos da coluna da
esquerda com os conceitos ou exemplos apresentados na
coluna da direita. Aps, marque a(s) proposio(es)
correta(s).
Please purchase PDF Split-Merge on www.verypdf.com to remove this watermark.
Incluso para a Vida Biologia B

Pr-Vestibular da UFSC 39
A -Bioma
B -Hbitat
C-Nicho
Ecolgico

I - Local fsico onde vive uma
espcie.
II - Lugar ocupado pela espcie no
ecossistema.
III - Mata Atlntica.
IV - Comunidades clmax dos
ecossistemas terrestres.
V - Beira de uma lagoa.
VI - Teia alimentar.
VII -Consumidor primrio.

01. C VI 08. A IV 64.A III
02. B l 16.B - V
04. C II 32. B VII

2. (UFSC) O aterro na Baa Sul na Ilha de Santa Catarina -
no municpio de Florianpolis - para construo da Via Ex-
pressa Sul, que ligar o Centro da Cidade regio do
Aeroporto Herclio Luz, demandar o deslocamento do uma
grande quantidade de areia do fundo da referida baa.
Com relao s conseqncias ecolgicas da efetivao de
tal aterro, correto afirmar que:
01. As comunidades que vivem no local a ser aterrado no
sero afetadas.
02. As comunidades que vivem no local de onde ser
extrada a areia necessria, certamente, sero afetadas.
04. Das populaes afetadas nesse processo, podemos citar
os moluscos bivalves que vivem enterrados.
08. As espcies planctnicas sero as que sofrero maior
impacto, chegando a serem extintas desse ecossistema.
16. A mudana na configurao do leito da baa influenciar
o meio bitico.

3. (UFPR) Atualmente a biologia tem a preocupao de
estudar os seres vivos no isoladamente, mas em conjunto
com o meio ambiente. De acordo com esta proposta,
correto afirmar que:
01. Ecologia a parte da biologia que estuda as interaes
dos seres vivos uns com os outros e com o meio
ambiente.
02. Populao um conjunto de indivduos de diferentes
espcies, os quais ocupam uma determinada rea.
04. Ecossistema o conjunto de relaes entre os seres vivos
e o mundo fsico.
08. Hbitat o conjunto dos hbitos ou atividades de uma
determinada espcie.
16. Biosfera constitui a poro do planeta habitada pelos
seres vivos.

CADEIA ALIMENTAR

A cadeia alimentar uma sequncia ordenada, na qual um
ser vivo serve de alimento para outro. Com exceo dos
organismos denominados auttrofos, os outros seres
necessitam capturar seus alimentos para que ocorram os
processos vitais para a manuteno da vida.

CLASSIFICAO DOS SERES NUMA CADEIA
ALIMENTAR

SERES AUTTROFOS - So aqueles que a partir dos
elementos abiticos produzem seu prprio alimento. Se
caracterizam por realizarem fotossntese ou quimiossntese
e so chamados de produtores. Dentre os produtores, os
mais importantes na manuteno dos ecossistemas so os
fotossintetizantes, como os vegetais terrestres, os aquticos e
as algas (principalmente marinhas).

SERES HETERTROFOS - So todos aqueles que no
conseguem produzir seu prprio alimento como acontece
com os auttrofos. Necessitam, portanto, se alimentar direta
ou indiretamente dos produtores. Os seres hetertrofos so
de dois tipos: os consumidores ou decompositores.
- Consumidores primrios - so aqueles que se alimentam
diretamente dos produtores. Se o hbito alimentar for
exclusivamente de vegetais, sero chamados de herbvoros.
J os consumidores secundrios so aqueles que se
alimentam dos consumidores primrios e os tercirios,
seriam aqueles que se alimentam dos secundrios, podendo
estes dois ltimos seres denominados de carnvoros.
(alimenta-se exclusivamente de carne) ou onvoros (com
hbito alimentar bem variado).

DECOMPOSITORES Tambm denominados de
saprfitas ou saprvoros, so seres que usam os produtores
e os consumidores como alimento, depois de mortos. Estes
organismos possuem importante papel para os ecossistemas
por deixarem no ambiente subprodutos da decomposio,
como compostos inorgnicos. Este processo recebe a
denominao de ciclagem da matria e realizada,
principalmente, por bactrias e os fungos.

Esquema de uma teia alimentar

TEIA ALIMENTAR Num ecossistema, vrias cadeias
alimentares se intercruzam formando o que se denomina de
teia ou rede alimentar. Enquanto a cadeia uma sequncia
linear de seres vivos onde um ser vivo serve de alimento
para outro, a teia ou rede alimentar o conjunto de vrias
cadeias alimentares do ecossistema. Numa cadeia ou teia
alimentar observam-se diversos nveis, os quais so
denominados de: nveis trficos ou alimentares.

NVEL TRFICO - o conjunto de todos os seres vivos
que apresentam, ao longo da cadeia ou teia alimentar, o
mesmo hbito alimentar ou mesma forma de nutrio.
Sempre o primeiro nvel trfico ocupado pelos
produtores, sendo o segundo ocupado pelos consumidores
primrios que poderiam ser os herbvoros ou os onvoros.
Os carnvoros que se alimentam dos herbvoros, ocuparias o
terceiro nvel trfico na cadeia alimentar e, assim,
Please purchase PDF Split-Merge on www.verypdf.com to remove this watermark.
Incluso para a Vida Biologia B

Pr-Vestibular da UFSC
40
sucessivamente. Observe que os seres que podem ocupar
desde o segundo at o ltimo nvel trfico so os omnvoros
e, o ser humano, um exemplo.
Os decompositores ocupam um nvel trfico
parte, pois reaproveitam os alimentos, dejetos e seres
mortos de todos os nveis trficos.

PIRMIDES ECOLGICAS
So representaes grficas dos diferentes tipos de
cadeias alimentares. Existem trs tipos de pirmides
ecolgicas: nmero, biomassa e energia. g , rg


Exerccios de sala #

4. (UFSC) Observe bem a figura abaixo e assinale a(s)
proposio(es) correta(s):













SOARES, Jos Luis. Biologia. So Paulo: Scipione, 1991, v. nico, p. 300.

01. Fungos e bactrias so representantes dos
seres decompositores.
02. A figura representa uma cadeia alimentar com trs nveis
trficos.
04. O fluxo de energia cclico, sendo renovado pelos
organismos decompositores.
08. A ona e o gavio representam os produtores.
16. O mocho, o lagarto e a cobra so classificados, nessa
figura, como consumidores tercirios ou de 3
a
ordem.
32. Na cadeia: verdura veado ona existe um
decrscimo energtico entre os nveis trficos.
64. Uma grande parte da energia obtida pelo
coelho, ao comer a verdura, gasta em seu
processo de respirao celular.

5. As relaes entre vrios nveis trficos, entre os
organismos presentes num lago, esto representadas na
seguinte pirmide (mais de uma alternativa pode estar
correta).

No que diz respeito a essa situao, correto afirmar que:

a) Os nveis I e II so ocupados por organismos produtores.
b) O nvel IV poderia ser ocupado por um peixe predador ou
uma ave.
c) A energia contida no nvel I menor do que a contida no
nvel III.
d) O nmero de indivduo que compem o nvel III maior
do que o nmero de indivduos no nvel II.

UNIDADE 20

RELACES ECOLGICAS ENTRE
OS SERES VIVOS

As relaes entre as espcies de uma mesma comunidade
so denominadas cenoses ou interaes ecolgicas. Essas
relaes podem ser:
Harmnicas - Associaes em que no existe nenhum
prejudicado.
Desarmnicas - Associaes em que um indivduo tira
proveito e o outro prejudicando.
Intraespecficas - Ocorre entre indivduos da mesma
espcie.
Interespecficas - Ocorre entre indivduos de espcies
diferentes.

RELAES HARMNICAS

COLNIAS - So associaes harmnicas intraespecficas,
onde grande nmero de indivduos passam a viver juntos. A
desagregao de um indivduo da colnia provoca a sua
morte.

COLNIAS HOMOTPICAS - Quando todos os indivduos
so iguais. Exemplo: colnia de corais.

COLNIAS HETEROROTPICAS - Quando os indivduos
so diferentes entre si e, cada indivduo, realiza um
determinado tipo de trabalho, como, por exemplo: as
caravelas do gnero Physalia.

SOCIEDADE - So associaes harmnicas intra-
especficas, onde os indivduos vivem juntos, no entanto,
no possuem nenhuma relao anatmica. Na sociedade,
todos os indivduos cooperam para o bem comum,
observando-se neste tipo de relao uma ntida diviso de
trabalho, como, por exemplo: a sociedade das abelhas, das
formigas e dos cupins.

SIMBIOSE - O termo simbiose, criado em 1879 por De
Bary, designa toda e qualquer associao permanente entre
indivduos de espcies diferentes que, normalmente, exerce
Please purchase PDF Split-Merge on www.verypdf.com to remove this watermark.
Incluso para a Vida Biologia B

Pr-Vestibular da UFSC 41
influncia recproca no metabolismo. Sendo assim, no
vlida a utilizao do termo simbiose para designar,
somente, as relaes do tipo harm6onicas como ocorre entre
as algas e os fungos que formam liquens.

MUTUALISMO - uma associao harmnica
interespecfica em que os dois indivduos apresentam
relaes benficas entre si. Neste tipo de relao, a ausncia
de um dos indivduos acarreta a morte do outro. Os
exemplos mais comuns de mutualismo so:
- Os liquens-associao entre algas e fungos.
- Os Cupins e os protozorios
- As Micorrizas associaes ente fungos e razes

PROTOCOOPERAO - um mutualismo no-
obrigatrio, onde cada indivduo envolvido na relao pode
viver sem a presena do outro. Estes indivduos se
relacionam para tornar a obteno de alimentos ou a
proteo mais fcil.

Protocooperao entre a anmona-do-mar e o paguro, que vive no
interior de conchas vazias de gastrpodes.

COMENSALISMO E INQUILINISMO
uma associao harmnica interespecfica em que apenas
um dos indivduos tira proveito, mas sem prejudicar o outro.
No inquilinismo esta associado a proteo, enquanto que no
comensalismo a associao alimentar.
Um exemplo de inquilinismo o caso do Fierasfer, um
pequeno peixe que vive dentro do corpo do pepino-do-mar.
Para alimentar-se, o Fierasfer sai do pepino-do-mar e depois
volta.
Um curioso exemplo de comensalismo a associao do
tubaro com o peixe-piloto. Os peixes-pilotos vivem ao
redor do tubaro, alimentando-se dos restos de comida que
escapam de sua boca.


Comensalismo entre tubaro e peixo-piloto
Inquilinismo entre o pepino do mar e o Fierasfer
RELAES DESARMNICAS

AMENSALISMO OU ANTIBIOSE - uma associao
desarmnica onde o produto da secreo mata o outro ser ou
apenas inibe o seu desenvolvimento. O exemplo clssico
do fungo (mofo) Penicilium notatum, que produz a
penicilina que inibe o crescimento de colnias de bactrias.
Outro exemplo o das mars-vermelhas. Sob certas
condies ambientais (aumento de alimento, temperatura),
certas algas marinhas do grupo dos dinoflagelados, crescem
exageradamente, consumindo muito O
2
da gua e liberando
substncias txicas que provocam a morte de animais
marinhos.

SINFILIA OU ESCLAVAGISMO - a relao em que
certas formigas mantm os pulges em cativeiro.
Os pulges sugam a seiva elaborada de vegetais eliminando
com suas fezes substncias aucaradas, muito apreciadas
pelas formigas.

PREDATISMO - uma associao desarmnica
interespecfica na qual distinguimos dois tipos de indivduo:
o predador e a presa. O predador geralmente maior como
uma prole menor, enquanto que a presa, geralmente
menor, mas a prole apresenta um maior nmero de
indivduos.

PARASITISMO - uma associao desarmnica
interespecfica na qual um indivduo (parasita) vive sobre ou
dentro do hospedeiro, tirando proveito e prejudicando o
hospedeiro. O parasita normalmente age lentamente,
procurando no levar o hospedeiro a morte.
Exemplos: Carrapato, piolho, pulga, sanguessuga, cip-
chumbo, infeces bacterianas, virais, doena-de-chagas,
malria.

COMPETIO INTRAESPECFICA - uma relao
desarmnica entre indivduos de uma mesma espcie. Em
um ecossistema, a disputa pelo mesmo nicho ecolgico pode
levar extino de populaes menos aptas e
sobrevivncia das mais aptas.

CANIBALISMO - Relao desarmnica intraespecfica
onde um ser da mesma espcie serve de alimento para outro.
Exemplos: Certos aracndeos (viva-negra) e certos insetos.

Exerccios de sala #

1. (UFSC) Em um ecossistema h muitos tipos de interao
entre os componentes das diversas espcies. Algumas
interaes so mutuamente proveitosas, outras so
mutuamente prejudiciais e outras, ainda, beneficiam apenas
uma das espcies, prejudicando ou no a outra. Dessa
forma, as interaes podem ser classificadas como
harmnicas ou desarmnicas.
Em relao a esse assunto, assinale a(s) proposio(es)
correta(s).
01. A interao das plantas epfitas (bromlias, por exemplo)
e suas plantas hospedeiras um tipo de parasitismo, j
que a rvore hospedeira prejudicada em seu
desenvolvimento.
02. Quando o caranguejo paguru (tambm conhecido como
ermito) ocupa a concha vazia de um caramujo, ocorre
Please purchase PDF Split-Merge on www.verypdf.com to remove this watermark.
Incluso para a Vida Biologia B

Pr-Vestibular da UFSC
42
um caso de favorecimento mtuo entre as duas espcies
envolvidas.
04. Quando algas e fungos se associam, formando os
liquens, ambos se favorecem, e tal relao um
exemplo de simbiose.
08. As sociedades e as colnias representam relaes
harmnicas que so estabelecidas, necessariamente,
entre indivduos de uma mesma espcie.
16. O predatismo um tipo de relao desarmnica, em que
apenas o predador leva vantagem, j que o resultado
final da interao a morte da presa.
32. A ocorrncia de vermes tipo Ascaris e Taenia, no
interior do homem, um caso de endoparasitismo,
enquanto insetos hematfagos, como a pulga e o
mosquito, so exemplos de ectoparasitas.

2. (UFSC) Entre os seres vivos que habitam determinado
ambiente, podem ser observadas interaes biolgicas com
diferentes tipos de relaes. Estas relaes podem ser
harmnicas ou desarmnicas, entre espcies diferentes ou
entre indivduos da mesma espcie.
Sobre estas relaes, assinale a(s) proposio(es)
correta(s).
01. Relaes interespecficas so aquelas estabelecidas entre
indivduos de mesma espcie e relaes intraespecficas
so aquelas estabelecidas entre indivduos de espcies
diferentes.
02. O predatismo e o parasitismo so exemplos de relaes
desarmnicas.
04. Colnia uma associao entre indivduos da mesma
espcie, que se mantm ligados anatomicamente
formando uma unidade estrutural.
08. O mutualismo um tipo de relao desarmnica
interespecfica.
16. A bactria Mycobacterium tuberculosis um
ectoparasita que causa a tuberculose no ser humano.
32. Apesar de o predatismo ser uma relao interespecfica
desarmnica, ele pode ser benfica e importante para o
controle da populao de presas e a manuteno do
equilbrio do ecossistema.

UNIDADE 21

CICLOS BIOGEOQUMICOS

o segmento da ecologia que se ocupa em estudar as trocas
de substncias entre os componentes biticos e abiticos nos
ecossistemas, necessrios para a manuteno da vida.

CICLO DA GUA
A gua nos seres vivos o componente mais abundante (no
homem, cerca de 65% e na medusa, so aproximadamente
95%). Alm de atuar como solvente e reagente de vrias
reaes qumicas uma das matrias-primas da fotossntese.
Em nosso planeta 3/4 gua, sendo que 97% desta gua
salgada.

AS DIVISES DO CICLO DA GUA
O ciclo da gua pode ser dividido em: pequeno ciclo e
grande ciclo.

O PEQUENO CICLO Neste ciclo, a gua dos oceanos,
lagos, rios e subterrneas sofrem evaporao pela ao do
calor ambiental e passa para forma gasosa, originando as
nuvens. Nas camadas mais elevadas ocorre condensao,
retornando crosta terrestre na forma de chuva.

O GRANDE CICLO Neste ciclo, a gua absorvida
pelos seres vivos e participa de seu metabolismo, sendo
posteriormente devolvida ao meio ambiente, atravs da
transpirao e das excretas.

CICLO DO CARBONO E DO OXIGNIO
O carbono fundamental na formao dos compostos
orgnicos, como: as protenas, os lipdios, os carboidratos e
os cidos nuclicos. Atravs da fotossntese, os seres
fotossintetizantes retiram o CO
2
da atmosfera que passam a
fazer parte das molculas orgnicas. Sendo assim, parte do
carbono retirado do ar passa a fazer parte da biomassa dos
seres fotossintetizantes, podendo, eventualmente, ser
transferida aos animais herbvoros e depois aos outros seres
de uma cadeia alimentar. Dessa forma, o carbono que
fixado pela fotossntese vai sendo repassado aos diferentes
nveis trficos, retornando gradativamente atmosfera,
atravs da respirao dos seres vivos e da ao dos
decompositores.
O ciclo do oxignio e do carbono esto
intimamente relacionados, em que um depende da ao do
outro. O oxignio indispensvel para que ocorra um
fenmeno biolgico denominado respirao celular, o qual
se caracteriza pela produo de molculas de ATP. Esta
molcula considerada o combustvel das clulas que
constituem todos os seres vivos, sendo formada a partir de
molculas orgnicas ricas em tomos de carbono como a
glicose. No entanto, para que ocorra a combusto e a queima
dos combustveis fsseis imprescindvel a presena do gs
oxignio. A concentrao de oxignio na atmosfera de,
aproximadamente, 21% e vem mantendo-se constante a
milhares de anos, devido a fotossntese realizada pelos
vegetais, principalmente as microalgas.


CICLO DO NITROGNIO
O nitrognio importante para os seres vivos, pois
fundamental na formao das protenas. Apesar de existir
grande quantidade de nitrognio livre na atmosfera (79%),
poucos so os organismos que conseguem fix-lo. Entre
estes organismos, temos:
- Certas bactrias do gnero Nitrobacter encontradas no
solo;
- As Rhizobiu, vivem em mutualismo nos ndulos
radiculares das plantas da famlia das leguminosas;
- Algumas cianofceas, como a Nostoc.
Please purchase PDF Split-Merge on www.verypdf.com to remove this watermark.
Incluso para a Vida Biologia B

Pr-Vestibular da UFSC 43
Os vegetais fixam o nitrgnio na forma de nitratos e os
animais obtm o nitrognio alimentando-se dos vegetais.


Exerccios de sala #

1. (UFSC) Preste ateno nos seguintes dados fornecidos
pelo PNUMA (Programa das Naes Unidas para o Meio
Ambiente):
Em 25 anos, metade da populao mundial pode enfrentar
problemas em obter gua suficiente para consumo e
irrigao.
Um tero do mundo composto por reas em que o
consumo de gua supera a oferta.
No toa, as Naes Unidas declararam 2003 o Ano
Internacional da gua Doce. Nas ltimas dcadas, a
escassez de gua passou da esfera acadmica para a
cotidiana.
(05/06/2003 disponvel em: www.folha.com.br!)
Esses dados mostram que existe uma grande preocupao
mundial com os recursos hdricos potveis. Com respeito
gua doce e suas fontes de obteno (so) correta(s) a(s)
proposio(es):
01. A obteno de gua doce potvel, a partir da
dessalinizao da gua do mar, um processo rpido e
econmico.
02. A preservao das matas ciliares e das matas do fundo
dos vales medida importante para a manuteno da
qualidade da gua dos rios.
04. Os lenis freticos, devido sua profundidade, no so
alcanados por contaminantes lanados no solo.
08. Rios e lagos constituem-se na principal fonte de gua
doce para consumo das populaes humanas.
16. A distribuio de gua doce no mundo muito
homognea.
32. Muitos rios, de grandes cidades brasileiras, poderiam
estar sendo usados como fonte de captao de gua
potvel. Isto, no entanto, no ocorre, devido ao
lanamento direto de esgoto e lixo domstico nesses rios.
64. No Brasil, graas fiscalizao rigorosa e observncia
das leis ambientais, no existem problemas de
contaminao dos rios por efluentes originados de
indstrias txteis, de papel e de produtos qumicos.

2. (UFSC) O esquema abaixo representa, de forma
simplificada, os ciclos do carbono e do oxignio. Assinale
a(s) proposio(es) correta(s).























01.I e II representam, respectivamente, o O
2
e o CO
2
.
02.O oxignio se encontra no meio abitico como integrante
do ar atmosfrico, ou no meio bitico, como constituinte
das molculas orgnicas dos seres vivos.
04.Praticamente, todo o oxignio livre da atmosfera e da
hidrosfera tem origem biolgica, no processo de
fotossntese.
08.A necessidade de O
2
para a respirao explica o
aparecimento dos animais antes dos vegetais na Terra.
16.Alguns fatores, como excessivas combustes sobre a
superfcie da terra, tm determinado o aumento gradativo
da taxa de CO
2
na atmosfera.
32. A manuteno das taxas de oxignio e gs carbnico, no
ambiente, depende de dois processos opostos: a
fotossntese e a respirao.
3. (UFSC) Durante 4 horas, um volume estimado em 1,34
milhes de litros de leo vazou de um duto da refinaria de
Duque de Caxias, causando o maior desastre ecolgico j
ocorrido na Bahia da Guanabara. (...) A mancha de leo se
estendia (..), projetando-se dos manguezais de Duque de
Caxas...
Trecho do artigo: 500 anos de degradao, Revista Cincia Hoje, 27
(158) 2000. p. 42-43.
Assinale a(s) proposio(es) verdadeira(s) sobre a ao
do petrleo e suas conseqencias com relao ao meio
ambiente e aos seres vivos que nele habitam.
01. No controle desse poluente, podem ser utilizados
detergentes, pois no so nocivos aos organismos
marinhos, e servem para dispersar e emulsionar o leo.
02. A fina camada de leo sobrenadante, dispersa na
superfcie da gua, reduz a capacidade da absoro de
luz na gua afetando, significatvamente, a atividade
fotossintetizante das algas.
04. A utilizao de certas bactrias decompositoras promove
a degradao do petrleo, e representa uma das medidas
adequadas para a recuperao desse ambiente.
08. A maior parte do petrleo ficou concentrada na zona
costeira, principalmente nos manguezais, destruindo esse
berrio de vida, alm de afetar a vida, por exemplo,
dos pescadores e catadores de caranguejo da regio.
I
II
Fotossn-
tese
respirao
animais
combuste
s
plantas e
fitoplncto
n

Please purchase PDF Split-Merge on www.verypdf.com to remove this watermark.
Incluso para a Vida Biologia B

Pr-Vestibular da UFSC
44
16. Os problemas provocados pelo desastre ecolgico, na
baia da Guanabara, foram minimizados, pois os tcnicos
e bilogos se uniram, removendo o poluente em poucas
horas.

Tarefa Mnima #

4. (Fuvest-SP) A maior parte do nitrognio que compe as
molculas orgnicas ingressa nos ecossistemas pela ao de:
a) algas marinhas. d) fungos.
b) animais. e) plantas terrestres.
c) bactrias.

5. (PUC-SP) Em agricultura, amplamente utilizado o
plano de rotao de culturas, em que diferentes espcies
vegetais so sucessivamente cultivadas em um mesmo
terreno. Nesse processo, muitas vezes so utilizadas as
leguminosas, pois estas plantas se associam a:
a) bactrias e enriquecem o solo de compostos sulfurosos.
b) bactrias e enriquecem o solo de compostos nitrogenados.
c) fungos e enriquecem o solo de compostos nitrogenados.
d) nemtodos e enriquecem o solo de compostos fosforados.
e) bactrias que tornam suas razes fasciculadas, atenuando o
efeito da eroso.

SUCESSO ECOLGICA

a substituio de uma comunidade por outra num
ecossistema, at atingir o clmax. Imaginemos uma regio
desabitada, onde a sobrevivncia de animais e vegetais
desfavorvel: o terreno muito rido, a iluminao direta
provoca temperaturas elevadas e a fixao dos vegetais
muito difcil. Se algas cianofceas ou liquens forem
transportados pelo vento, conseguiro se estabelecer na
regio, uma vez que realizam a fotossntese e fixam o
nitrognio, sendo, por isso, capazes de viver apenas com
gua, ar e uns poucos sais. Formam, assim, uma
comunidade pioneira ou ecese. A comunidade pioneira
modifica, aos poucos, as condies iniciais da regio,
humificando o solo, permitindo a instalao de plantas mais
exigentes. As sementes de capim, por exemplo, tm
condies de germinar e se desenvolver no local. O capim,
por sua vez, acarreta novas modificaes ambientais, como
os arbustos e samambaias. Assim, a partir da comunidade
pioneira, outras vo se estabelecendo formando uma
sequncia de comunidades intermedirias, chamadas
sries ou seras, at chegar comunidade final, denominada
clmax. O tipo de comunidade clmax varia de acordo com
as condies climticas da regio. A comunidade clmax
no tem, obrigatoriamente, existncia eterna. Pode ser
destruda por mudanas climticas, catstrofes ou, o que
mais comum, pela ao do homem. Aps a extino da
comunidade clmax, pode se insta-lar uma sucesso
secundria. o que vem acontecendo na maior parte das
florestas mundiais.

Exerccios de sala #
6. (UDESC) Durante o primeiro semestre deste ano,
assistimos aos incndios ocorridos na floresta amaznica,
em Roraima. De propores espantosas, acarretaram sria
destruio aquela floresta, sendo necessria, inclusive, a
ajuda internacional para seu combate. Nesses casos, aps
debelado o agente destruidor, vemos que nas regies
afetadas inicia-se um processo denominado sucesso
ecolgica. Com relao a esse assunto correto afirmar
que:
a) Vrias sucesses que ocorram em um mesmo territrio
culminaro sempre com o mesmo tipo de comunidade
clmax.
b) Em curto prazo, sucedem-se comunidades
intermedirias, aparecendo primeiramente rvores de
grande porte.
c) Os animais de pequeno porte surgem apenas ao final da
sucesso.
d) A comunidade clmax, fim da sucesso, permanece
inalterada, sendo praticamente de existncia eterna.
e) Como primeira etapa da sucesso ecolgica temos a
ecese, ou seja, a instalao de seres pioneiros, tais como
os lquens.

7. Considere os dados abaixo:
I - aumento da diversidade e do nmero de espcies
hetertrofas.
II - diversidade baixa, com predominncia de organismos
auttrofos.
III - diversidade alta e estvel.
Em uma sucesso ecolgica, a sequncia observada :

a. I, II, III c. I, III, II e. III, I, II
b. II, I, III d. II, III, I

Exerccios de sala #

8. (FCC-SP) Nos rios, onde se lana grande quantidade de
esgoto, muitas vezes os peixes morrem porque:
a) h excesso de nutrientes orgnicos.
b) o suprimento de oxignio decresce.
c) o fitoplncton prolifera.
d) os decompositores competem com os seres aerbicos.
e) os consumidores comem os peixes.

9. (Vunesp-SP) Nos rios so lanados, geralmente, grande
quantidade de esgotos, provocando, em alguns casos, a
morte de muitos peixes. Assinale a alternativa que melhor
explica a morte desses animais:
a) Aumento da quantidade de oxignio e diminuio na
quantidade de bactrias anaerbicas.
b) Aumento na quantidade de bactrias aerbicas e
consequente aumento na quantidade de oxignio.
c) Diminuio na quantidade de oxignio e aumento na
quantidade de bactrias anaerbicas.
d) Diminuio no nmero de indivduos herbvoros que no
eliminam grande parte de fitoplncton.
e) Diminuio da quantidade de alimentos com consequente
mortandade dos peixes a longo prazo.

10. (Fuvest-SP) A eutrofizao marinha por nitratos e
fosfatos tem provocado proliferao excessiva das
populaes de algas, fenmeno conhecido como florao
das guas. A alta mortalidade de peixes que acompanha
este fenmeno deve-se (ao):
a) acmulo de nitratos e fosfatos ao longo da cadeia
alimentar.
b) competio entre algas e peixes por espao fsico.
c) competio entre algas e peixes por alimentos.
Please purchase PDF Split-Merge on www.verypdf.com to remove this watermark.
Incluso para a Vida Biologia B

Pr-Vestibular da UFSC 45
d) liberao excessiva de uria pelas algas.
e) diminuio de oxignio na gua, causada pela
decomposio das algas.

UNIDADE 22



TEORIAS SOBRE A ORIGEM DA VIDA
Desde os tempos mais remotos, vrias teorias e hipteses
foram levantadas no intuito de explicar como surgiu a vida e
de onde surgiu o ser humano. Dentre as inmeras hipteses
formuladas, as que mais se destacaram foram:

CRIACIONISMO
Deus criou todos os seres como so atualmente h milhares
de anos atrs, ou seja, acreditava-se na imutabilidade das
espcies.

PANSPERMISMO OU COSMOGNICA
A vida originou-se em outro planeta e chegou a Terra por
meio de esporos, os quais originaram os primeiros seres
primitivos. Essa hiptese no explica como surgiu o
primeiro ser vivo, s transfere o problema.

ABIOGNESE OU GERAO ESPONTNEA
Pela gerao espontnea seria possvel criar seres vivos a
partir de elementos do meio, como calor, umidade e lodo. Os
principais defensores desta hiptese foi Aristteles (sc. IV
a.C.), Jean Baptiste Van Helmont (1577-1644), mdico e
qumico, que inclusive tinha receita para obteno de
camundongos: "Juntar camisa suja com germes de trigo num
canto escuro do poro." Outro abiogenista foi Anton Van
Leeuwenhoek (1632-1723) que desenvolveu o microscpio
ptico e descobriu que as bactrias se multiplicavam numa
gota d'gua, reforando a teoria da abiognese.

BIOGNESE
Um ser vivo s surge de outro ser vivo por reproduo. O
Italiano Francesco Redi (1627 - 1697) tentou provar que a
Abiognese no estava correta, atravs de um simples
experimento:
- Utilizando 2 frascos com carne em putrefao, Redi
colocou uma tela junto ao gargalo do frasco 1, de
maneira que as moscas no pudessem alcanar a carne
em decomposio.
- O frasco 2, manteve-se descoberto, proporcionando
um contato direto das moscas com a carne em
putrefao.
Redi pde observar que no frasco 2, com gargalo
destampado, surgiam centenas de larvas, comprovando que
os vermes no surgiam da carne em decomposio, como
acreditavam os abiogenistas. E assim, um ser vivo s pode
surgir de outro ser vivo por reproduo. No entanto, nesta
poca a abiognese ainda estava muito enraizada e o
experimento de Redi no obteve a ateno merecida.
O Padre, tambm italiano, Lzzarro Spallanzani (1765),
criticou energicamente a abiognese, demonstrando que em
soluo nutritiva esterilizada, no apareciam novos seres
vivos, mas Needham (abiogenista) contra-atacou dizendo
que ao submeter a infuso temperaturas elevadas e
mantendo-as fechadas, destrua-se o princpio ativo.

Louis Pasteur
Somente no final do sculo passado Louis Pasteur (1822 -
1895) preparou um caldo nutritivo de carne com extrato de
frutos e pasteurizou-o, aquecendo e resfriando bruscamente.
O bico do frasco utilizado foi retorcido (como um pescoo
de cisne) de modo a no entrar em contato com o ar. Dessa
forma, o caldo manteve-se estril at que o bico do balo foi
quebrado, passando, a partir deste momento, a apresentar
seres vivos. Pasteur provou, dessa forma, que um ser vivo s
surge de outro ser vivo por reproduo, colocando um ponto
final na teoria da gerao espontnea.

Exerccios de sala #

1. A curiosidade de saber como a vida surgiu na face da
Terra uma constante na maioria das pessoas. Assinale
a(s) proposio(es) que relaciona(m) corretamente as
colunas:
I - Teoria da gerao espontnea.
II - Teoria Cosmozica.
III - Teoria Heterotrfica.
IV - Teoria Criacionista.
A. Os primeiros seres vivos surgiram na Terra e devem ter
sido incapazes de sintetizar seu prprio alimento.
B. Os seres vivos originaram-se a partir de elementos do
meio.
C. Organismos extraterrestres instalaram-se na Terra e aqui
implantaram a vida.
D. Os seres vivos originaram-se por obra de um ser divino.

01. I B 04.III A
02. II C 08.IV D

2. (UFSCar) O meio ambiente cria a necessidade de uma
determinada estrutura em um organismo. Este se esfora
para responder a essa necessidade. Como resposta a esse
esforo, h uma modificao na estrutura do organismo. Tal
modificao transmitida aos descendentes.
O texto sintetiza as principais ideias relacionadas ao:

a) fixismo. c) mendelismo. e) lamarckismo.
b) darwinismo. d) criacionismo.

A TEORIA HETEROTRFICA SOBRE A ORIGEM DOS
PRIMEIROS SERES VIVOS
H dcadas, Alexander I Oparin, membro da academia de
cincias da Rssia, emitiu uma teoria para explicar a origem
do primeiro ser vivo na Terra. Segundo esta teoria, a Terra
era incandescente e aos poucos a crosta terrestre foi se
resfriando. A atmosfera primitiva, com tempestades
contnuas, descargas eltricas e raios ultra-violetas, fez com
que os quatro elementos fundamentais (CH
4
- metano; NH
3
-
amnia; H
2
O- gua e H
2
hidrognio livre ) cassem sobre a
crosta terrestre. Esses elementos sofrendo a ao das altas
temperaturas e descargas eltricas transformaram-se em
aminocidos. Devido inconstncia das guas, esses
aminocidos ficaram sobre rochas quentes, reagindo por
desidratao, formando proteinoides que foram carregados
para o mar, formando protenas, resultando numa sopa
nutritiva. Essas molculas proteicas, combinadas e isoladas
do meio, formaram os coacervados, no entanto, estes no
so considerados os primeiro seres vivos. Essa teoria foi
EVOLUO
Please purchase PDF Split-Merge on www.verypdf.com to remove this watermark.
Incluso para a Vida Biologia B

Pr-Vestibular da UFSC
46
denominada heterotrfica, naturalista ou teoria da evoluo
gradual dos sistemas qumicos.
Em 1953, Stanley Miller testou em laboratrio a
teoria heterotrfica, repetindo as condies desfavorveis da
Terra propostas por Oparin, chegando, no final do
experimento, a obter aminocidos.
Representao da Terra primitiva e o experimento
de Stanley-Miller para comprovar a teoria proposta por
Oparin


OS PRIMEIROS SERES VIVOS
O primeiro ser vivo que surgiu na Terra deve ter ser sido um
organismo hetertrofo, ou seja, incapaz de produzir seu
prprio alimento. Supe-se, ainda, que os primeiros seres
hetertrofos eram muito simples e obtinham seu alimento do
mar primitivo, o qual era considerado uma grande sopa
nutritiva. S depois de muito tempo que surgiram os seres
auttrofos apresentando clorofila e capazes de sintetizar o
seu prprio alimento. Esses seres, realizando fotossntese,
acabavam liberando oxignio para a atmosfera,
proporcionando condies para o surgimento de seres
aerbicos.
Muitas pessoas pensam que os primeiros seres que
surgiram na Terra eram auttrofos. No entanto, estes
organismos apresentam molculas muito complexas e
enzimas muito especializadas, caractersticas que no
poderiam estar presentes nos primeiros seres vivos.

AS PRIMEIRAS CLULAS
Acredita-se que os primeiros seres vivos tenham surgido h
cerca de 3,5 bilhes de anos, de modo semelhante ao
proposto anteriormente. Essas clulas eram estrutural e
funcionalmente muito simples, sendo formadas por uma
membrana plasmtica delimitando um citoplasma, no qual
estavam presentes as molculas de cidos nucleicos. Clulas
com esta organizao so denominadas procariticas.
Atualmente, organismos procariontes descendentes
dessas primeiras clulas so as bactrias e as cianobactrias.

AS CLULAS EUCARITICAS
A partir dos procariontes anaerbios ancestrais teriam
derivado os organismos com estrutura celular mais
complexa: os eucariontes. O surgimento dos eucariontes
deve ter ocorrido h cerca de 1,5 bilhes de anos.
A maioria dos organismos apresenta clulas
eucariticas. Podem ser unicelulares como os protozorios
ou pluricelulares como as plantas, os animais e os fungos.
Segundo a hiptese de Robertson, as primeiras clulas
eucariticas teriam surgido a partir das clulas procariticas
que passaram a desenvolver evaginaes da membrana
plasmtica, tornando-se maiores e mais complexas. Esses
dobramentos teriam dado origem s vrias estruturas
citoplasmticas delimitada por membrana e, tambm a
membrana que separa o material gentico do citoplasma,
formando a carioteca e um ncleo individualizado.

Exerccios de sala #

3. Em 1953, Miller submeteu ao de descargas eltricas
de alta voltagem uma mistura de vapor de gua, amnia
(NH
3
), metano (CH
4
) e hidrognio. Obteve, como resultado,
entre outros compostos, os aminocidos glicina, alanina,
cido asprtico e cido aminobutrico.
Com base nesse experimento, pode-se afirmar que:

01. Ficou demonstrada a hiptese da gerao espontnea.
02. No se podem produzir protenas artificialmente; elas
provm necessariamente dos seres vivos.
04. Formam-se molculas orgnicas complexas em
condies semelhantes s da atmosfera primitiva.
08. A vida tem origem sobrenatural, que no pode ser
descrita em termos fsicos nem qumicos.
16. Compostos orgnicos podem se formar em condies
abiticas.

4. Considere estes eventos relativos origem da vida:
I - Aparecimento do processo de fermentao.
II - Formao de coacervados.
III - Aparecimento dos processos de fotossntese e
respirao aerbica.
IV - Estabelecimento do equilbrio entre hetertrofos e
auttrofos.
A ordem lgica em que esses eventos ocorrem :
a) III - II IV I d) II - III - IV - I
b) I - II - IV - III e) IV - III - II - I
c) II - I - III - IV

5. Considerando a hiptese heterotrfica e os processos
energticos de fermentao, respirao aerbica e
fotossntese, pode-se dizer que:
a) a fotossntese foi o primeiro processo a ser utilizado
pelos seres vivos na obteno de energia para sua
sobrevivncia.
b) existiam nos oceanos primitivos molculas complexas,
indicativas da existncia de realizao de respirao
aerbica nos seres primitivos.
c) os primeiros seres vivos utilizavam o processo de
fermentao para obter a energia indispensvel sua
sobrevivncia.
d) os primeiros seres vivos auttrofos realizavam a
fermentao para obter a energia necessria sua
sobrevivncia.
e) a respirao aerbica foi o primeiro processo de
obteno de energia realizado pelos seres vivos nos
oceanos primitivos.

UNIDADE 23

TEORIAS SOBRE A EVOLUO DAS
ESPCIES

LAMARCKISMO
A teoria de Lamarck mais conhecida como lei do "uso e
desuso", que se fundamenta em dois princpios bsicos:
Please purchase PDF Split-Merge on www.verypdf.com to remove this watermark.
Incluso para a Vida Biologia B

Pr-Vestibular da UFSC 47
1) Os rgos, quando muito usados, se hipertrofiam e,
quando no usados, se atrofiam (uso e desuso).
2) Segundo Lamarck, a atrofia e a hipertrofia seriam
hereditrias e, um exemplo clssico desta teoria para
Lamarck, o que se observa com os msculos estriados. O
uso intenso dos msculos proporciona a hipertrofia e a
paralisia atrofia. O outro exemplo era o tamanho do
pescoo das girafas que cresceram devido ao esforo
realizado por estes animais para capturarem as folhas na
copa das rvores.
Um dos indivduos que combateu o lamarckismo foi August
Weissman, o qual realizou um experimento que consistia em
cortar o rabo dos camundongos por vrias geraes e, em
nenhuma gerao, os camundongos nasceram sem rabo ou
com rabo atrofiado.

Segundo Lamarck, o pescoo longo foi obtido devido
necessidade de obteno de alimentos nas copas e, depois,
transmitido aos descendentes.

DARWINISMO
Charles Darwin, aps ter analisados vrias amostras
coletadas ao longo de sua viagem ao redor do mundo e
realizado um estudo do livro de Thomas R. Malthus, lanou
comunidade cientfica, em 1859, o seu livro A ORIGEM
DAS ESPCIES, revolucionando as explicaes sobre como
devem ter surgido as milhares de espcies diferentes que
encontramos em nosso planeta.
Entre as suas teorias, destacam-se:
- Em uma populao h sempre indivduos mais
aptos e menos aptos;
- A populao cresce em P.G. e o alimento cresce em
P.A. (T. Malthus);
- Devido indisponibilidade de alimento no meio,
haveria uma luta pela sobrevivncia (Seleo
Natural), sobrevivendo o mais apto.

AS FALHAS DO DARWINISMO
Darwin no conseguiu explicar como surgiam as variaes
entre os indivduos de uma mesma espcie, pois ainda no se
conhecia nada sobre gentica. Sendo assim, Darwin utilizou
as explicaes de Lamarck para esclarecer estas variaes;
Usou a teoria de T. Malthus (alimento x populao), hoje
no admitida como teoria que comprove a seleo natural;
A luta pela vida no se explica de maneira to simples
como de uma presa fugindo do predador, mas sim, a luta do
indivduo contra as adversidades do meio ambiente.
Um exemplo clssico de adaptao s adversidades
do meio, foi o que ocorreu com as mariposas de Manchester.
Antes da revoluo industrial as mariposas claras tinham
mais chances de sobrevivncia, devido camuflagem que
estas mantinham com os liquens, quando comparado com as
escuras, que eram constantemente predadas pelos pssaros.
Aps a revoluo industrial, com aumento de
fuligem nos troncos das rvores e o desaparecimento dos
liquens, as mariposas brancas passaram a ser mais visveis
pelos pssaros e, consequentemente, s mais predadas. As
ideias de Darwin que continuam a ser aceitas so "a luta pela
vida" e a "seleo natural".

TEORIA SINTTICA DA EVOLUO OU
NEODARWINISMO
Com base nos atuais conhecimentos sobre gentica, sabe-se
que as variaes surgem em uma espcie por alterao no
material gentico (DNA), sendo transmitidas de gerao
para gerao. Estas variaes surgem sem causa aparente ou
intencional do organismo e so denominadas de mutaes.
Alm disso, outro fator importante que leva a
variabilidade entre os indivduos da mesma espcie o
crossing-over que acontece durante a formao dos
gametas. A teoria sinttica, tambm se baseia na seleo
natural, no isolamento geogrfico e reprodutivo. Todas
estas teorias tentam responder, atualmente, grande
diversidade de espcies presentes em nosso planeta.

LEMBRE-SE
As variaes de uma espcie dependem das mutaes;
as mutaes acontecem ao acaso;
a luta pela vida feita entre indivduos e o meio;
a luta pela vida resulta na seleo natural dos mais aptos;
o isolamento geogrfico e sexual impede que as
caractersticas se misturem, agindo, assim, na formao de
novas espcies.

RESISTNCIAS DE INSETICIDAS
Quando se aplica um inseticida em uma populao de
insetos sensveis ao de um D.D.T., por exemplo, existe
no meio desta populao insetos mais resistentes. A
capacidade de resistncia a um inseticida dada,
geneticamente, devido a uma mutao favorvel. Quando se
aplica o D.D.T., observa-se no comeo, uma sensvel
reduo dos insetos, devido a morte dos insetos no
resistentes. No entanto, insetos resistentes sobrevivem e
continuam a se reproduzir transmitindo as caractersticas
favorveis e resistentes ao D.D.T., aos descendentes.
Portanto, no so os insetos que se tornam resistentes ao
D.D.T., e sim, houve uma seleo de linhagens resistentes
ao D.D.T.

UNIDADE 24

EVIDNCIAS DA EVOLUO

FSSEIS
Os ancestrais dos atuais seres vivos deixaram restos e
impresses em rochas em todo o mundo. Tais fsseis podem
ser datados atravs da determinao dos materiais
radioativos neles contidos. A paleontologia o ramo da
cincia que estuda os fsseis.
Ex: Mamutes da Sibria conservados em blocos de gelo.

Please purchase PDF Split-Merge on www.verypdf.com to remove this watermark.
Incluso para a Vida Biologia B

Pr-Vestibular da UFSC
48
ANATOMIA COMPARADA
Os estudos de anatomia comparada revelam que os rgos
homlogos tm a mesma origem embrionria, mas nem
sempre a mesma funo (ex.: brao do homem e asa do
morcego). J os rgos anlogos tm origem embrionria
diferente, embora possuam a mesma funo (ex.: asa de
inseto e asa de morcego). Os conceitos de homologia e
analogia permitem compreender os processos de
convergncia adaptativa e irradiao adaptativa. A
convergncia ocorre quando grupos diferentes se adaptam
s mesmas condies ambientais; o que leva seleo de
rgos anlogos.
A irradiao ocorre quando no decurso da evoluo,
organismos de um mesmo grupo sofrem ao de condies
ambientais diferentes, desta forma seus rgos homlogos se
tornaro diferentes.

BIOQUMICA
Diferentes organismos apresentam protenas comuns,
enquanto outros apresentam diferentes sequncias de
aminocidos, sugerindo desta forma, um grau de parentesco
evolutivo entre eles.

RGOS VESTIGIAIS
rgos que embora sem funo atual, permanecem
vestigialmente, indicando sua existncia anterior em sua
forma completa.
Ex: Apndice intestinal vermiforme. Membros anteriores
atrofiados das baleias.

Exerccios de sala #

1. (UFSC) O conhecimento do processo evolutivo
fundamental para a compreenso da vida. O estudo de
fsseis uma importante evidncia de que a evoluo dos
organismos ocorreu. Com relao a esse estudo, correto
afirmar que:
01. Fsseis so restos ou vestgios de seres que viveram no
passado.
02. Os tipos de fsseis encontrados em determinada camada
de solo refletem a flora e a fauna existentes no local, por
ocasio da formao das rochas.
04. A partir de uma parte do corpo, de uma pegada ou de
uma impresso corporal, possvel deduzir o tamanho e
a forma dos organismos que as deixaram.
08. O mtodo do carbono 14 auxilia na determinao da
idade de um fssil.
16. No foram encontrados, at o momento, registros fsseis
no sul do Brasil.

2. (UFSC) Ao formular sua teoria para explicar a evoluo
dos organismos, o ingls Charles Darwin baseou-se em
fatos, tais como:
01. Em uma espcie, os indivduos no so exatamente
iguais, havendo diferenas que tornam alguns mais
atraentes, mais fortes, etc.
02. Populaes crescem mais depressa do que a quantidade
de alimentos necessria para supri-las.
04. Caracteres adquiridos so passados s descendncias.
08. Uso demasiado de uma estrutura leva hipertrofia da
mesma.
16. Mutaes so muito frequentes
3. (UFSC) "Modernamente, o mutacionismo sofreu alguns
acrscimos, foi aperfeioado em certos detalhes e se
constituiu na nova Teoria Sinttica da Evoluo, que a
teoria da atualidade para explicar como as espcies se
transformaram no tempo e originaram a imensa variedade
dos seres que hoje conhecemos."
(Texto extrado do livro "Biologia - volume nico", de Jos Lus Soares,
1997, p. 286).
Com relao Teoria Sinttica da Evoluo, correto
afirmar que:
01. Considera a seleo natural como fonte de variabilidade
gentica.
02. As mutaes adaptativas ocorrem ao acaso, no
admitindo a procura intencional da evoluo.
04. O isolamento (geogrfico e sexual) um fator
importante para a evoluo.
08. A seleo natural no preserva necessariamente os mais
aptos.
16. Os primeiros seres vivos surgiram por gerao
espontnea.

4. (UFSC) Jean-Baptiste Antoine de Monet (1744-1829),
tambm chamado Jean-Baptiste Lamarck, e Charles Darwin
(1809-1882) deram importante contribuio para o
pensamento evolucionista. Sobre suas ideias, correto
afirmar que:
01. Lamarck acreditava que a adaptao dos seres vivos ao
ambiente era resultado de modificaes lentas e graduais
ao longo de inmeras geraes.
02. De acordo com Darwin, os indivduos sofrem mutaes
com o propsito de melhor se adaptarem ao meio em que
vivem, e assim deixarem descendentes mais bem
adaptados.
04. De acordo com Lamarck, o uso frequente e repetido de
um rgo o fortalece, enquanto o desuso de tal rgo o
enfraquece, processo que atualmente conhecido como
evoluo divergente.
08. Darwin apresentou as observaes de fsseis das ilhas
Galpagos em defesa de suas ideias.

5. (UFSC) Existem vrias provas da evoluo e dentre elas
podemos citar as embriolgicas. Sobre o tema, correto
afirmar que:
01. As nadadeiras dos golfinhos, assim como brao e mo
humanos, so ditos rgos homlogos e so herdados de
um ancestral comum.
02. As nadadeiras dos golfinhos e as asas das aves tm a
mesma origem embrionria e diferentes funes,
decorrentes da adaptao a diferentes modos de vida,
processo conhecido como divergncia evolutiva.
04. As nadadeiras dos golfinhos e as nadadeiras das tainhas
so rgos de diferentes origens embrionrias e tm a
mesma funo, o que chamado de convergncia
evolutiva.
08. As asas dos insetos e as asas das aves so ditos rgos
homlogos, pois tm a mesma origem embrionria.
16. As nadadeiras dos golfinhos, as asas dos morcegos e os
braos e as mos dos humanos tm origem embrionria
diferente.
32. As baleias, os golfinhos, os peixes-boi e as focas
pertencem ordem dos cetceos, pois possuem rgos
Please purchase PDF Split-Merge on www.verypdf.com to remove this watermark.
Incluso para a Vida Biologia B

Pr-Vestibular da UFSC 49
anlogos e sinrgicos em comum, como as nadadeiras e a
bexiga natatria.

6. (UFSC) Em 2009 comemora-se 200 anos do nascimento
de Charles Darwin e 150 anos da publicao do livro A
Origem das Espcies, obra que coroou um extenso trabalho
de Darwin e foi um marco no estudo da Biologia.
Sobre evoluo, assinale a(s) proposio(es) correta(s).
01. O francs Jean Baptiste Lamarck contribuiu de maneira
significativa com o trabalho de Darwin, pois ele j
defendia a ideia de que a modificao das espcies era
devido seleo natural.
02. Uma das bases da teoria de Darwin foi a Lei da Herana
dos Caracteres Adquiridos, que, embora com mais de
200 anos, se mantm atual at os dias de hoje.
04. Segundo a hiptese da seleo natural, grandes
modificaes nos indivduos so transmitidas de pai
para filho.
08. No livro A Origem das Espcies, Darwin formulou a
hiptese de que o ambiente selecionava os mais aptos
(seleo natural), que tinham mais chances de
sobreviver e deixar descendentes.
16. Os fsseis, a semelhana embriolgica entre as espcies
e a existncia de estruturas vestigiais desprovidas de
funo nos animais so evidncias da evoluo.
32. Segundo Darwin, o aparecimento de novas mutaes
influenciado pelo meio ambiente.

Please purchase PDF Split-Merge on www.verypdf.com to remove this watermark.
Incluso para a Vida Espanhol

Pr-Vestibular da UFSC
1
UNIDADE 1

ACENTUACIN GRFICA Y SEPARACIN
SILBICA

CLASIFICACIN TNICA

Sobresdrjula Esdrjula Grave o Llana Aguda
L GI CA MEN TE
Antes de la
antepenltima
slaba
Antepenlti
ma slaba
Penltima
slaba
ltima
slaba
m7, 6, 5, 4 3 2 1

Se acentan grficamente las
palabras
Ejemplos

agudas
terminadas en vocal
(a, e, i, o , u)
o en consonante
n o s
cancin
corts
llevar
and

graves o
llanas
terminadas en
consonante que NO
SEA n o s
gil
carcter mbar
csped

esdrjulas

todas
nade
acadmico
clebre filsofo

sobresdrjulas

todas
djatelo
escrbeselo
arrglemelo
dgaselo

ACENTO DIACRTICO

Palabra Funcin
Gramatical
Ejemplo
el Artculo
definido
El hombre que vino ayer es
mi jefe.
l Pronombre
personal
l no recibi mi recado.
mi Adjetivo posesivo Perd mi billetera.
Nota musical En mi menor, por favor.
m Pronombre
personal
No pensaste en m cuando
tomaste esta decisin.
tu Adjetivo posesivo Tu libro es muy bueno.
t Pronombre
personal
No me dijeron que t ya
habas llegado.
de Preposicin Me encanta la torta de
chocolate.
Nombre de letra Diana se escribe con de
mayscula.
d Imperativo del
verbo dar
D um poo de atencin.
se Pronombre reflex. Juana se acuesta tarde.
s Imperativo del
verbo ser
S justo, hombre!
Presente del
indicativo del
verbo saber
No s si vale la pena ir a ver
esta pieza.
si Conjuncin Si llueve, no voy al club.
Nota musical Cntalo, en si mayor.
s

Pronombre
reflexivo
Pablo slo piensa en s
mismo.
s Adverbio de
afirmacin
- Aceptas el trabajo?
- S.
mas Conjuncin Quise comprarlo mas me
sala muy caro.
ms Adverbio de
cantidad
Ms vale pjaro en mano
que cien volando.
te Pronombre reflex. Te invito a cenar.
Nombre de letra Esta es la te de Tadeo.
t Sustantivo Para m, t de menta, por
favor.
aun Adverbio (hasta;
incluso)
Aun los mas chicos
entienden eso.
an Conjuncin
(todava)
An lo espero.
solo Sustantivo Fue sublime el solo de
violn.
Adjetivo Por qu te sientes tan solo?
slo Advrbio
(solamente)
Slo faltan seis das para las
vacaciones.

Ejercicios de Sala #
1. Indica la(s) proposicin(es) cuyo(s) trminos destacados
est(n) usado(s) correctamente.
01. No se que me pasa.
02. Se el len est muerto, no hay peligro.
04. Puedes venir con nosotros, si quieres.
08. Quieres acompaarme? - S, cmo no!
16. La fiera no est muerta. S qued dormida.
32. No se te olvide llamarme.

2. Seala la proposicin correcta. Las palabras polen -
colibr - sembrando son clasificadas:
a) aguda llana grave
b) llana aguda llana
c) grave aguda esdrjula
d) aguda esdrjula llana
e) llana llana grave

3. Seala la alternativa incorrectamente acentuada.
a) ciudad sali rumor rfaga
b) cay poca rbol carcter
c) frances cicln poca realidad
d) huracn probe salio agitacin
e) lampara bailar emergncia ptio

Tarea Complementaria #
Texto: Mecedes Sosa: No necesito hombres(UDESC)



Desde 1978, cuando muri Pocho, su segunda pareja,
eligi estar sola. "Hubo otras relaciones", aclara, pero dice
que as se siente bien. Otra vez arriba de los escenarios
haciendo lo que ms le gusta, cantar, La Negra habla de
hombres, de amores, de sus padres, de su adis al rock y de su
pasin por los autos y la velocidad.
Please purchase PDF Split-Merge on www.verypdf.com to remove this watermark.
Incluso para a Vida Espanhol

Pr-Vestibular da UFSC
2
Hoy, qu quiere que le diga... Para andar con un tipo 20
aos ms joven haciendo papelones, prefiero este presente,
asegura.
- Ay, ay, ay! Mercedes Sosa se toma el abdomen, se queja,
pide que llamemos a Mara, su asistente personal y ama de
llaves. Mara aparece empujando una puerta y corre hasta un
silln menos mullido que el mito que lo ocupa expandiendo
toda su entereza.
- Dgame, seora. -Me duele ac, Mara
- Y... Le cay mal lo que comi anoche.
- No. Para m son calambres en el alma.
Silencio. Enseguida La Negra hace un guio, como
satisfecha por la ocurrencia. Y Mara sonre, y todos remos.
Antes del potico calambre, la protagonista vena haciendo
memoria y balance. Hablaba de su hijo Fabin y de Gustavo,
el hijo de Pocho Mazzitelli, su ltima pareja conocida. De los
cuatro viviendo bajo un mismo techo. Y s, dice, esos
calambres que me agarran cada vez que pienso cunto dara
porque ellos volvieran a ser chicos y mos. Se trata de
recuerdos, entonces llegamos a la muerte de Pocho y el
relato, cronolgico, conduce al exilio europeo. Por todo
equipaje, tres valijas, una cartera de mano y un grabador que
era como su casita.
- En Madrid, tena un Ford Fiesta, despus me compr un
Peugeot 505 ltimo modelo. Uno de los primeros que entr a
Espaa. Despus, pero dos aos despus, el auto empez a
conocerse all... Mercedes Sosa, su hijo, Pocho, el exilio
Cmo llegamos a hablar de autos?
- Mi pasin siempre fueron los autos. Soy medio fierrera,
sabe?
- Me gustan y adoro la velocidad. Tengo un Audi A6, que
ahora est en venta, y un Volvo S80. Los conoce?
- Y le gustaba correr?
- (Pone cara de a ver si nos entendemos) No me gustaba,
me gusta...

4. De acuerdo al texto podemos afirmar que, despus de la
muerte de su segundo compaero, Mercedes Sosa decidi:
a) casarse en 1978.
b) buscar otro compaero.
c) quedarse sola.
d) estar con otro.
e) dejar de ser viuda.

5. De acuerdo con el primer prrafo del texto se puede
afirmar que:
a) Mercedes Sosa diz que teve outros relacionamentos.
b) Mercedes Sosa diz que no teve nenhum relacionamento.
c) Mercedes Sosa diz que sempre viveu sozinha.
d) Mercedes Sosa diz teve e tem outros relacionamentos.
e) Mercedes Sosa diz que continua procurando outro
relacionamento.

6. Traduzca la siguiente frase, indicando la opcin correcta
para: En Madrid, tena un Ford Fiesta.
a) Em Madrid, teve um Ford Fiesta
b) Em Madrid, tinha um Ford Fiesta
c) Em Madrid, teria um Ford Fiesta
d) Em Madrid, tive um Ford Fiesta
e) Em Madrid, terei um Ford Fiesta

7. Segn el texto, se puede afirmar que a Mercedes Sosa:
a) tena miedo de correr.
b) no le gustaba la velocidad.
c) le gustaba correr.
d) odiaba la velocidad.
e) no le gustaba correr.

8. Seale la opcin que se relaciona directamente con el
trecho del texto abajo.
Mi pasin siempre fueron los autos. Soy medio fierrera,
sabe?
Me gustan y adoro la velocidad. Tengo un Audi A6, que
ahora est en venta, y un Volvo S80. Los conoce?

a) Ela tem dois carros e um deles, o Volvo S80, est venda.
b) Ela gosta de carros e atualmente tem dois, mas um est
venda.
c) Ela gosta de carros; tinha dois e j vendeu um.
d) Ela gostava de carros e hoje s tem dois, pois um est
venda.
e) Ela adora carros, mas no gosta de velocidade, e
atualmente tem dois.

UNIDADE 2

LOS PRONOMBRES PERSONALES



















(*) = Pronombres reflexivos. Indican que la accin est
siendo realizada pelos propios sujetos.

a) Verbo conjugado prclisis.

Interesa a m.
Me interesa.
Juan dice la verdad.
Juan la dice.

Coment a l.
Le coment.
Mara necesita el libro.
Mara lo necesita.

b) Verbo no conjugado nclisis.

Estudiar las teoras.
Estudiarlas.
Haciendo los deberes.
Hacindolos.

SECUENCIA DE PRONOMBRES
La (s) ________________
Me
Lo (s) ________________

La (s) ________________
Podemos tener: Te
Lo (s) ________________

Pronombre
Sujeto
Pronombre
Complemento
Formas
tonas
Formas
Tnicas


Singular

1
a
yo me* mi, conmigo
2
a
t te* ti, contigo

3
a


l, ella,
usted

se*
lo, la, le
s, consigo,
el, ella,
usted



Plural




1
a

nosotros,
nosotras

nos*
nosotros,
nosotras

2
a

vosotros,
vosotras

os*
vosotros,
vosotras

3
a


ellos, ellas,
ustedes

se*
los, las, les
s, consigo,
ellos, ellas,
ustedes

Please purchase PDF Split-Merge on www.verypdf.com to remove this watermark.
Incluso para a Vida Espanhol

Pr-Vestibular da UFSC
3

La (s) ________________
Se Le (s) ________________
Lo (s) ________________

Ejemplos: Carla cont a m la verdad.
Carla me la cont.

Repetir a ti el mensaje.
Repetrtelo.
Ejercicios de Sala #
1. Complete la siguiente frase:______ me dijo que ______
viajara el jueves. Entonces ______ vendremos a verlo antes.
a) T ella nosotros
b) Ella usted nosotros
c) Ella t nosotros
d) l t ellas
e) l usted vosotros

2. Sealar la(s) alternativas correcta(s):Cuando mi amigo
lleg, _____ dije que no podra acompaar_____.
a) lo, lo
b) el, lo
c) lo, el
d) el, le
e) le, lo

3. Indica los pronombres que completan correctamente la
siguiente oracin:Cuando los mdicos ________ preguntan
yo _______ respondo.
a) me os
b) te te
c) le lo
d) me les
e) os los

Tarea Complementaria #
Texto B (UFSC)




La estacin Allende es una de esas estaciones de campo con
unos cuantos paisanos, un jefe en mangas de camisa, una
volanta y unos tarros de leche.
Me irritaron dos hechos: la ausencia de Mara y la
presencia de un chofer.
Apenas descend, se me acerc y me pregunt:
Usted es el seor Castel?
No respond serenamente. No soy el seor Castel.
En seguida pens que iba a ser difcil esperar en la estacin
el tren de vuelta; podra tardar medio da o cosa as. Resolv,
con malhumor, reconocer mi identidad.
S agregu, casi inmediatamente, soy el seor Castel.
El chofer me mir con asombro.
Tome le dije, entregndole mi valija y mi caja de
pintura.
Caminamos hasta el auto.
La seora Mara ha tenido una indisposicin me
explic el hombre.
Una indisposicin!, murmur con sorna. Cmo conoca
esos subterfugios! Nuevamente me acometi la idea de
volverme a Buenos Aires, pero ahora, adems de la espera del
tren haba otro hecho: la necesidad de convencer al chofer de
que yo no era, efectivamente, Castel o, quiz, la necesidad de
convencerlo de que, si bien era el seor Castel, no era loco.
Medit rpidamente en las diferentes posibilidades que se me
presentaban y llegu a la conclusin de que, en cualquier
caso, sera difcil convencer al chofer. Decid dejarme
arrastrar a la estancia. Adems, qu pasara en caso de
volverme? Era fcil de prever porque sera la repeticin de
muchas situaciones anteriores: me quedara con mi rabia,
aumentada por la imposibilidad de descargarla en Mara,
sufrira horriblemente por no verla, no podra trabajar, y todo
en honor a una hipottica mortificacin de Mara. Y digo
hipottica porque jams pude comprobar si verdaderamente
la mortificaban esa clase de represalias.
Hunter tena cierto parecido con Allende (creo haber dicho
ya que son primos); era alto, moreno, ms bien flaco; pero de
mirada escurridiza. Este hombre es un ablico y un
hipcrita, pens. Este pensamiento me alegr (al menos as
lo cre en ese instante).
Me recibi con una cortesa irnica y me present a una
mujer flaca que fumaba con una boquilla largusima. Tena
acento parisiense, se llamaba Mim Allende, era malvada y
miope.
Pero dnde diablos se habra metido Mara? Estara
indispuesta de verdad, entonces? Yo estaba tan ansioso que
me haba olvidado casi de la presencia de esos entes. Pero al
recordar de pronto mi situacin, me di bruscamente vuelta, en
direcin a Hunter, para controlarlo. Es un mtodo que da
excelentes resultados con individuos de este gnero.
Hunter estaba escrutndome con ojos irnicos, que trat de
cambiar instantneamente.
Mara tuvo una indisposicin y se ha recostado dijo.
Pero creo que bajar pronto.

4. Seala la(s) proposicin(es) que refleje(n) algn(as) de
la(s) idea(s) presentes en el texto.
01. El narrador se encuentra entre amigos a los que ha ido a
visitar.
02. Mara, sin duda, no desea verlo.
04. El narrador slo desea ver a Mara.
08. Mara sufri un accidente pero se recuperar pronto.
16. El narrador no se encuentra a gusto.

5. En un primer momento, el narrador niega ser el seor
Castel porque:
01. Est burlndose del chofer.
02. Mara no fue a esperarlo y est alterado.
04. Desea hacerle una sorpresa a Mara.
08. Est ofendido porque slo lo espera un chofer
desconocido.

6. Qu ocurrira, segn el texto, si el narrador regresase a la
ciudad?
01. El narrador vivira situaciones ya vividas anteriormente.
02. Mara, seguramente, no sentira su ausencia.
04. El narrador no podra dedicarse a su trabajo.
08. Con seguridad, su actitud le provocara honda tristeza a
Mara.
16. Seguramente, Mara nunca se lo perdonara.


Please purchase PDF Split-Merge on www.verypdf.com to remove this watermark.
Incluso para a Vida Espanhol

Pr-Vestibular da UFSC
4
UNIDADE 3

LOS POSESIVOS Y LOS DEMOSTRATIVOS

Pronombre Personal Adjetivos Posesivos Ejemplos Pronombres Posesivos
yo mi mis amigo(a) + (s)
sntesis
mo ma
mos mas
t tu tus contador(a)
exmen(es)
tuyo tuya
tuyos - tuyas
l / ella / usted su sus raz(ces)
tab(es)
suyo suya
suyos suyas
nosotros / nosotras nuestro nuestra
nuestros nuestras
pas ciudad
estados provincias
nuestro nuestra
nuestros nuestras
vosotros / vosotras vuestro vuestra
vuestros vuestras
orgullo confianza
triunfos virtudes
vuestro vuestra
vuestros vuestras
ellos /ellas / ustedes su sus derecho(s)
ley(es)
suyo suya
suyos suyas


LUGAR
SINGULAR
Masculino - Femenino
PLURAL
Masculino - Femenino

PROXIMIDAD
AQU / AC este esta estos estas CERCA
AH ese esa esos esas MS O MENOS
ALL / ALL aquel aquella aquellos aquellas LEJOS


LUGAR
SINGULAR
Masculino - Femenino
PLURAL
Masculino - Femenino
FORMAS
NEUTRAS

PROXIMIDAD
AQU / AC ste sta stos stas esto CERCA
AH se sa sos sas eso MS O MENOS
ALL / ALL aqul aqulla aqullos aqullas aquello LEJOS

Ejercicios de Sala #

1. Todos los vecinos trajeron el apoyo _________ a
________ vigilancia y cuidado de ________ nios.
a) su / nuestra / sus
b) nuestro / suya / sus
c) suyo / nuestra / vuestros
d) suyo / suya / vuestros
e) nuestro / vuestro / suyos

2. Elije la alternativa que completa el siguiente enunciado:
T te quejas de mis ronquidos, yo de ____________
a) tu insensibilidad
b) su intolerancia
c) suya inpertinencia
d) tuya incomprensin
e) vuestra intransigencia

3. Identifique la alternativa que completa correctamente las
frases abajo:
I Desde ________ da no lo he visto ms. (lejos)
II En _________ taller se trabaja mucho. (cerca)
III - Qu calle es __________? (cerca)
IV - Quin es ___________ chica? (mas o menos cerca)
a) aquello ste sta sa
b) aquel ese esa esta
c) aquellos se estas sa
d) aqul eses sa esas
e) aquel este sta esa



4. Cul de las siguientes frases est incorrecta?:
a) No quiero ms se!
b) Este mes terminaremos.
c) Es so lo que les incomoda?
d) No eran aquellas palabras.
e) Yo, t, se y aqul.

Tarea Complementaria #

Texto: Cosas de la seduccin(UDESC)

1



5

Dicen que las flores se visten de colores
deslumbrantes para enamorar a las aves y a los
insectos que cumplirn la tarea de cooperar en su
reproduccin. As, seducido por la hermosura de los
ptalos y a la vista del polen, el colibr con su aleteo
nervioso, se detiene en el aire, bebe su dulce nctar y
luego emprende el vuelo hacia otra flor sembrando el
mundo de belleza.
5. De acuerdo con el texto se puede afirmar que:
a) Aves e insectos sembran el mundo de belleza con sus
hermosos colores.
b) El colibr se viste de colores para llevar el plen a las
flores.
c) El colibr coopera con la reproduccin de las flores a
travs de la polinizacin.
d) Slo el colibr es responsable por la polinizacin de las
flores.
e) Otras aves e insectos no contribuyen para la polinizacin
de las flores.
Please purchase PDF Split-Merge on www.verypdf.com to remove this watermark.
Incluso para a Vida Espanhol

Pr-Vestibular da UFSC
5
6. el colibr con su aleteo(lnea 5), es interpretado por:
a) el colibr es alterado por volar.
b) el colibr con sus aletas movidas vuelve.
c) el colibr mueve sus alas que le dan aliento.
d) por las alteraciones de las alas, el colibr tiene aletas.
e) el colibr mueve sus alas con rapidez.

7. La traduccin correcta para se visten (lnea 1) es:
a) se viram
b) se olham
c) eles se vem
d) se visitam
e) se vestem

8. La traduccin correcta para ...el vuelo hacia otra flor
sembrando el mundo de belleza.(lneas 7-8) es:
a) voar para outra flor e fazer o mundo asombrar-se de
beleza.
b) o vo em direo a outra flor semeando o mundo de
beleza.
c) voltar para outra flor semblando o mundo de beleza.
d) o vo fazia outra flor semeando o mundo de beleza.
e) o vo outra flor para sombrear o mundo de beleza.

UNIDADE 4

LOS ARTCULOS

DETERMINANTES
NM./GN. Masculino Femenino
Singular EL (O) LA (A)
Plural LOS (Os) LAS (As)

INDETERMINANTES
NM./GN. Masculino Femenino
Singular UN UNA
Plural UNOS UNAS

USO DO ARTIGO NEUTRO LO

lo + adjetivo + que
intensifica / enfatiza el valor del adjetivo.

lo + adverbio + que
intensifica / enfatiza el valor del adverbio.

lo + participio + que
intensifica / enfatiza el valor del participio.

lo + posesivo el posesivo puede desempear un papel
afectivo o generalizador.

lo + que se refiere a algo o un lugar no especificado pero
conocido por el hablante y por el oyente.

lo + de se refiere a algo o un lugar no especificado pero
conocido por el hablante y por el oyente.

LAS CONTRACCIONES
Ellas son:
al (ao) = a + el
del (do) = de + el

No se produce contraccin de preposicin con artculo si este
ltimo pertenece al nombre de aquello a lo cual se hace
referencia.

La noticia se extrajo de El Mercurio. (y no del Mercurio)
El aumento en la temperatura atmosfrica se debe a El Nio.
(y no al Nio)

Ejercicios de Sala #

1. ___ bueno sera que Carmen Paz fuese ___ de las
mejores alumnas. Cules artculos estn correctos?
a) Un las
b) Lo la
c) Lo una
d) Uno unas
e) Lo el

2. En la frase Me lastima el alma ver gente pobre, la
palabra alma es femenina, pero est acompaada del
artculo masculino el para que no ocurra la eufona. Seala
la(s) proposicin(es) abajo donde ocurre el mismo fenmeno.
01. el rbol
02. el agua
04. el hambre
08. el hada
16. el puente
32. el guila

3. Complete la frase con el o lo. _____ que importa es
que _____ jazz es _____ estilo de msica que ms me gusta.
a) lo el el
b) lo el lo
c) lo lo lo
d) el el el
e) el lo lo

4. Seala la proposicin cuya secuencia complete
correctamente las siguientes frases:
I _______ jueves pasado fuimos _______ teatro.
II Ayer salimos _____ cine _____ diez de la noche.
III ______ 31 de diciembre celebramos _____ noche vieja.

a) lo del del las lo la
b) el al del a las el la
c) los al del las en el lo
d) los el al a las en el la
e) el el al a las en lo el

5. Elija la alternativa que rellena correctamente las lagunas
en: Yo me acercaba _______ calle _______ puerto.
a) al / de lo
b) a la / del
c) a el / de
d) al / del
e) a la / de lo

Tarea Complementaria #

Texto: El biocombustible se quema (UFSC)

Please purchase PDF Split-Merge on www.verypdf.com to remove this watermark.
Incluso para a Vida Espanhol

Pr-Vestibular da UFSC
6
La crisis alimentaria siembra dudas sobre el papel del
biocarburante en la seguridad energtica y ambiental.

"Un crimen contra la humanidad". Palabras gruesas que
parecen destinadas a los nazis, el Gulag, la Camboya de Pol
Pot o Srebrenica. Pero que las Naciones Unidas y el
Gobierno de India el segundo pas ms poblado del mundo
asocian ahora a los biocombustibles por su incidencia sobre
la crisis alimentaria, los precios de los cereales y el hambre
que acecha a millones de personas en todo el mundo. La
demostracin palpable de que el debate ha calado est en los
autobuses madrileos: unos 400 autocares de la Comunidad
de Madrid circulan ya con carburantes que utilizan en su
fabricacin cereales o aceites vegetales. Al lado de la
flamante pegatina "funciona con biodisel" , en algunos de
esos vehculos poda leerse esta semana una pintada
siniestra: "Asesinos".
Mimados por los subsidios y la legislacin en Europa y
en Estados Unidos, los biocarburantes han crecido en los
ltimos aos a la misma velocidad que ahora pierden lustre y
apoyos por todos lados. Han dejado de ser la quintaesencia
de lo polticamente correcto. La ONU los ha puesto en el
disparadero y las crticas arrecian desde el Fondo Monetario
Internacional y la OCDE, foros donde dominan los pases
ricos hasta el Banco Mundial y la FAO, las instituciones
multilaterales centradas en el mundo en desarrollo.

6. Sobre el primer prrafo del texto 2 se puede afirmar que:
01. el biocombustible es severamente criticado por causar
estragos idnticos a los de la ltima guerra mundial.
02. la India repudia el uso del biocombustible, mientras que
la ONU ve en l la solucin para la crisis alimentaria.
04. un organismo internacional y una nacin densamente
poblada relacionan el biocombustible con la caresta de
los alimentos.
08. la propaganda en pro del biocombustible en los autobuses
de Espaa es reforzada por una agresiva campaa de los
medios de comunicacin.
16. el uso descontrolado de los biocombustibles puede causar
daos anlogos a los de otras tragedias mundiales.

7. Sobre el segundo prrafo del texto 2 se puede afirmar que:
01. el combustible biolgico ha pasado recientemente de
hroe a villano, tanto por parte de los pases ricos como de
los pobres.
02. en Estados Unidos el biocombustible recibi y sigue
recibiendo un apoyo unnime e incondicional.
04. la ONU es a favor, pero instituciones como la FAO y el
Banco Mundial son contra el uso del combustible
biolgico.
08. no hace mucho los biocombustibles eran vistos como
paradigma de lo polticamente correcto, pero actualmente
ya no lo son.
16. el descrdito reciente del combustible biolgico se debe
a la campaa en su contra promovida por los exportadores
de petrleo.

8. El trmino subrayado en la frase: La crisis alimentaria
siembra dudas sobre el papel del biocarburante... en
portugus tiene sentido de:
01. evita
02. suscita
04. descarta
08. gera
16. elimina
32. levanta

9. Seala la definicin o explicacin correcta.
01. Crisis alimentaria: su plural es crises alimentarias.
02. Cereales: semillas como trigo, maz y cebada.
04. Carburante: sinnimo de combustible.
08. Calar: prohibicin de hablar.
16. Biodisel: especie de combustible biolgico.
32. Autocares: sinnimo de autobuses.

UNIDADE 5
EL SUSTANTIVO

GNERO
En el caso de las palabras masculinas
terminadas en o a
Ej: El nio es bonito La nia es bonita

terminadas en consonante + a
Ejs: ingls inglesa / doctor doctora

NMERO
En el caso de las palabras singulares
terminadas en a, o, e + s
Ejs: roja rojas / taco tacos / mate mates
sof sofs / caf cafs / domin domins

terminadas en las vocales tnicas
, + es (forma culta)
Ejs: aj ajes / man manes /

, + s (forma popular)
Ejs: champ champs / men mens

terminadas en las vocales atonas
i, u + s
tribu tribus / mapamundi mapamundis / yanqui yanquis

terminadas en consonante + es
Ejs: pared paredes / reloj relojes /
cordel cordeles / ratn ratones /
dolor dolores / rey reyes

terminadas en consonante s o x,
antecedidos por vocal tona se mantienen
invariables.
Ejs: el martes los martes / el trax los trax
la crisis las crisis / el atlas los atlas / la tesis las tesis
el anlisis los anlisis / la caries las caries
el virus los virus / el cumpleaos los cumpleaos
el bceps los bceps / el frceps los frceps

terminadas en consonante z c +es
Ejs: la luz las luces / capaz capaces

Ejercicios de Sala #

1. Cul(es) de la(s) siguiente(s) alternativa(s) est(n)
incorrecta(s):
01. disfraz - disfrazes
02. crisis - crisis
04. guaran - guarans
08. bamb - bambes
16. capitn - capitanes

Please purchase PDF Split-Merge on www.verypdf.com to remove this watermark.
Incluso para a Vida Espanhol

Pr-Vestibular da UFSC
7
2. Indique los plurales de los vocablos siguientes: origen,
rgimen, ley, paz.
a) origens, rgimens, leis, pases
b) origenos, regimenos, leys, pazes
c) origes, regimes, leis, paz
d) orgenes, regmenes, leyes, paces
e) origens, regimentos, legales, pazes

3. Seale la(s) alternativa(s) que presente(n) la(s) forma(s)
plural(es) correcta(s).
01. rey reyes
02. ferrocarril ferrocarriles
04. carcter carcteres
08. rgimen regmenes
16. lpiz lpizes

Tarea Complementaria #
Texto: Agra y Taj Mahal


1



5




10




15




20




25




30

Agra est sucia, totalmente dejada y parece
existir nicamente para comer de las migas del
Taj Mahal. Infectada con una obsesin por
exprimir al mximo a cada turista que circula por
sus calles, no hace ms que espantarlos y
repelerlos.
Los vendedores, taxistas y dueos de hoteles
que habitan la jungla que es Agra son, sin duda,
de los ms persistentes e irritantes del planeta.
Saben que el Taj Mahal les garantiza un flujo
constante de turistas y convencidos de que no
tienen nada que perder- hacen del asedio un arte.
Algo de razn tienen: ver de cerca uno de los
monumentos ms maravillosos e imponentes del
mundo bien vale el martirio.
El Taj Mahal fue construido durante el siglo
XVII por el emperador Shah Jahan como
mausoleo para su segunda esposa, que muri
durante el parto de su dcimo cuarto hijo. Su
construccin llev ms de veinte aos y requiri
de la mano de obra de veinte mil personas. Hasta
se trajeron especialistas de Europa.
Todo el Taj Mahal es simtrico. Las cuatro
caras del monumento son iguales y mide de alto
lo mismo que de ancho. A los costados del
mausoleo hay dos edificios iguales: uno de ellos
es una mezquita; el otro fue construido solamente
para conservar el equilibrio (no se puede rezar en
su interior porque no mira a la Mecca).
Shah Jahan fue eventualmente derrocado por
uno de sus hijos, quien lo encarcel en el fuerte
de Agra. Cuenta la leyenda que el emperador
pas sus ltimos aos mirando al Taj Mahal
desde la ventana de su celda.

4. Indica la proposicin correcta segn el texto.
a) Agra es una persona sucia.
b) Agra es una ciudad.
c) Agra es un animal infectado.
d) Agra es un monumento maravilloso.
e) Agra es una jungla.
5. Indica la interpretacin correcta de: ...hacen del asedio un
arte (lnea 12).
a) Una de las principales expresiones de arte es el asedio.
b) Se dedican a asediar a las personas de las ms variadas
formas.
c) Hacen que las personas se sientan asediadas por el arte del
lugar.
d) Asediar a las personas es prohibido, por eso lo disfrazan
con arte.
e) A los vendedores, turistas y dueos de hoteles no les gusta
asediar a las personas.

6. Seala la opcin correcta para la traduccin de ancho
(lnea 25).
a) Profundidade
b) Altura
c) Comprimento
d) Espessura
e) Largura

7. Seala la opcin correcta. En la lnea 27, el otro se
refiere a:
a) un costado del mausoleo.
b) un hombre.
c) una mezquita.
d) un edificio.
e) un mausoleo.

8. Marque (V) si la proposicin es verdadera, o (F) si la
proposicin es falsa.
( ) Agra es un lugar acogedor.
( ) Los vendedores, taxistas y dueos de hoteles tienen miedo
de perder clientes.
( ) Vale la pena ir a Agra.
( ) El Taj Mahal es un monumento maravilloso.
( ) El emperador Shah Jahan pas sus ltimos aos en el Taj
Mahal.
La secuencia correcta, de arriba hacia abajo, es:
a) F F V V V
b) V F V V F
c) V F V F F
d) F F V V F
e) F V V V F

UNIDADE 6

EL ADJETIVO

A) Grado positivo: Es el modo normal del adjetivo.

Palabra
Propuesta
Palabra
Opuesta
Palabra
Propuesta
Palabra
Opuesta
claro oscuro blando duro
bueno malo gordo flaco
limpio sucio largo corto
joven viejo vaco lleno
seco mojado abierto cerrado
ancho estrecho menor mayor
grande pequeo alto bajo
peor mejor antiguo nuevo


Please purchase PDF Split-Merge on www.verypdf.com to remove this watermark.
Incluso para a Vida Espanhol

Pr-Vestibular da UFSC
8
B) Grado comparativo: Establece una comparacin de
igualdad, inferioridad o superioridad de calidad de un ser en
relacin a otro.

- Comparativo de superioridad: ms + adjetivo + que
Ejs.: Emilio es ms estudioso que Deyanira.

- Comparativo de inferioridad: menos + adjetivo + que
Ejs.: La radio Atlntida es menos popular que la Band

- Comparativo de igualdad: tan + adjetivo + como
Ejs.: La Pepsi-Cola es tan buena como la Coca-Cola.
C) Grado superlativo: Expresa el grado ms intenso de
cualidad (caracterstica) de un ser. Se dividen en dos tipos.

- Superlativo Absoluto: Se forma agregando los sufijos
simo(a), rrimo(a) al adjetivo, anteponindole adverbios
como muy, sumamente, extraordinariamente, etc. o
prefijos como re, requete, archi
Adj. Formas de Superlativo Absoluto
caro carsimo / carsima muy caro / sper caro
feo fesimo / fesima muy feo /
extremadamente feo
bajo bajsimo / bajsima muy bajo /
sumamente baja
lento lentsimo / lentsima muy lento / requetelento
buena buensima / buensimo muy buena / rebuena
vieja viejsima / viejsimo muy vieja / archivieja

- Superlativo Relativo: Indica la superioridad de una
determinada caracterstica o cualidad de un ser en relacin a
otro ser que pertenece al mismo universo que el primero. Por
ejemplo:

Este chiquillo es el alumno ms alto de la clase.
Esa ropa es la ms ridcula que haya visto en toda mi vida.

FORMAS APOCOPADAS DE ADJETIVOS
Adjetivo Apcope Ejemplo
bueno buen Enrique era un buen director.
malo mal Hoy hace un mal da para
jugar a la pelota.
alguno algn Me gustara hacer algn viaje
por mar.
ninguno ningn No me gusta ningn equipo de
ftbol.
Santo san Todo mundo conoce a San
Antonio, pero no a Santo
Toribio.
cualquiera cualquier* Cualquier pantaln de
mezclilla y cualquier remera
te servir.
grande gran* Fue un gran comienzo para
quien luego seria una gran
actriz.
primero primer Este ser el primer
aniversario de la escuela.
tercero tercer Yo vivo en el tercer piso.
tanto tan* Esto no es tan importante para
m, como lo es para ti..
cuanto cuan* No sabe cuan contenta me
siento de que nos hayamos
conocido.
(*) = Estas palabras son apocopadas tanto delante de
sustantivos masculinos como femeninos.

Ejercicios de Sala #

1. Elige las frases con uso adecuado del adjetivo en espaol:
01. Un bueno ejemplo de crtica al delito oficial es el relato de
Bartolom de las Casas.
02. La fe de los paganos no es ms pequea que la de los
fieles cristianos.
04. Cristbal Coln descubri un continente ms grande que
el europeo.
08. El indio era considerado como un buen salvaje.
16. Fray Bartolom, grande defensor de los indios, era
espaol.
32. El cristiano era un malo patrn.

2. Rellena los espacios con la(s) alternativa(s) correcta(s):
Los cristianos eran _________ que los indios.
Los indios eran __________ que los cristianos.
01.muy malos / muy buenos.
02. malos / buenos.
04. mejores / peores.
08. ms malos / ms buenos.
16. mellores / peores.
32. peores / mejores.

Tarea Complementaria #
Texto: Respetar la Naturaleza (UFSC)


A algunos les parecer extrao que se haga entrar en la
moral el respeto a los reinos mineral, vegetal y animal.
Pero debe recordarse que estos reinos constituyen la morada
humana, el escenario de nuestra vida.
El poeta mexicano Enrique Gonzlez dice: ... Y
quitars, piadoso, tu sandalia para no herir las piedras del
camino. Dante, el gran poeta italiano, supone que, al
romper la rama de un rbol, el tronco le reclama y le grita:
Por qu me rompes?. Este smbolo nos ayuda a
entender cmo el hombre de conciencia moral
plenamente cultivada siente horror por las mutilaciones y
destrozos.
En verdad, el espritu de maldad asoma ya cuando
enturbiamos una fuente de agua clara, o echamos
inmundicias a los ros o desechos txicos al mar; o cuando
arrancamos ramas de los rboles por slo ejercitar las
fuerzas; o cuando contribuimos a ensuciar el aire que todos
necesitamos; o cuando matamos animales fuera de los casos
en que nos sirven de alimento; o cuando torturamos por
crueldad a los animales domsticos, o bien nos negamos a
adoptar prcticas que los alivien un poco en su trabajo.
Este respeto al mundo natural que habitamos, a las cosas
de la tierra, va creando en nuestro espritu una conciencia
de la importancia que tiene para nosotros la preservacin
de la ecologa, esto es, la relacin que existe entre los
organismos vivos y el medio ambiente. Al mismo tiempo, este
respeto despierta un hbito de contemplacin amorosa que
Please purchase PDF Split-Merge on www.verypdf.com to remove this watermark.
Incluso para a Vida Espanhol

Pr-Vestibular da UFSC
9
contribuye a nuestra felicidad y que, de paso, desarrolla
nuestro espritu de observacin y nuestra inteligencia.
Pero no debemos quedarnos con los ojos fijos en la
tierra. Tambin debemos levantarlos a los espacios celestes.
Debemos interesarnos por el cielo que nos cubre, su rgimen
de nubes, lluvias y vientos, sus estrellas nocturnas. Cuando un
hombre que tiene un jardn ignora los nombres de sus plantas
y sus rboles, sentimos que hay en l algo de salvaje; que no
se ha preocupado por labrar la estatua moral que tiene el
deber de sacar de s mismo. Igual diremos del que ignora
las estrellas de su cielo y los nombres de sus constelaciones.
El cuidado del ambiente y la preservacin del equilibrio
ecolgico de la naturaleza son indispensables para nuestra
supervivencia. Y el amor a la morada humana es una garanta
moral, es una prenda de que la persona ha alcanzado un
apreciable nivel del bien: aquel en que se confunden el bien y
la belleza, la obediencia al mandamiento moral y el deleite
en la contemplacin esttica. Este punto es el ms alto que
puede alcanzar, en este mundo, el ser humano.
REYES, Alfonso. Cartilla Moral. Mxico: Secretara de Educacin
Pblica, 1992. p. 29-30.

3. Seala la(s) proposicin(es) correta(s), segn el texto 1.
01. El Planeta Tierra y sus recursos son nuestros. Podemos
usarlos y abusar como nos plazca.
02. Podemos usar los recursos que la Naturaleza nos
proporciona, pero sin alterar ni descuidar el equilibrio
ecolgico.
04. Todo ser humano debe conocer los nombres de todas las
plantas y de todos los astros del universo.
08. El ser humano con un mnimo de sensibilidad usa los
recursos de la Naturaleza y contempla sus bellezas.
16. El uso de insecticidas es desaconsejado porque causa
serios problemas al medio ambiente.

4. De acuerdo con el texto 1, Cmo reacciona el rbol, segn
Dante, cuando es mutilado? Seala la(s) proposicin(es)
correcta(s).
01. Pregunta el porqu de semejante accin.
02. La rama pregunta al tronco por qu la abandona.
04. El rbol que sufre la vejacin gratuita se marchita y muere.
08. El rbol maltratado protesta, apelando a la consciencia
moral del agresor.
16. Reacciona, vengndose del agresor.

5. El reclamo Por qu me rompes? puede ser sustituido,
sin alterar el sentido, por:
01. No me rompas!
02. No rompeme!
04. No moleste!
08. No me mutilar!
16. A que no me rompes!
32. Por qu me mutilas?

6. Seala la(s) proposicin(es) que completa(n)
correctamente la frase de acuerdo con el texto 1.
Una conducta ecolgicamente sana prohbe matar a los
animales, ...
01. pues ellos tienen los mismos derechos que nosotros.
02. excepto en casos de entretenimiento o deporte.
04. y quien los mate sufrir igual pena.
08. a no ser en caso de necesidad.
16. excepto cuando se haga para exportacin.
32. a no ser en caso de precisin.
7. Seala la(s) proposicin(es) cuyas expresiones completan
correctamente la frase:
La gente, _______, contamina el medio ambiente, no por
maldad, ____ por ignorancia.
01. aveces si no
02. a veces sino
04. muchas vezes pero
08. con frecuencia sino
16. a las vezes se no


UNIDADE 7

CONJUGACIN VERBAL

MODO INDICATIVO DO VERBO: CANTAR
Pronombres
Personales
Pretrito
Indefinido
Pretrito
Imperfecto
Presente Futuro
Imperfecto
Condicional
Imperfecto
Yo cant cantaba canto cantar cantara
T cantaste cantabas cantas cantars cantaras
l / ella / usted cant cantaba canta cantar cantara
Nosotros / nosotras cantamos cantbamos cantamos cantaremos cantaramos
Vosotros / vosotras cantasteis cantabais cantis cantaris cantarais
Ellos / ellas / ustedes cantaron cantaban cantan cantarn cantaran
MODO SUBJUNTIVO
Pronombres Personales Pretrito Imperfecto Presente Futuro Imperfecto
Yo cantara - cantase cante cantare
T cantaras - cantases cantes cantares
l / ella / usted cantara - cantase cante cantare
Nosotros / nosotras cantramos - cantsemos cantemos cantaremos
Vosotros / vosotras cantarais - cantaseis cantis cantareis
Ellos / ellas / ustedes cantaran - cantasen canten cantaren
Please purchase PDF Split-Merge on www.verypdf.com to remove this watermark.
Incluso para a Vida Espanhol

Pr-Vestibular da UFSC
10


MODO IMPERATIVO
Afirmativo Negativo
canta t no cantes t
cante l / ella / usted no cante l / ella / usted
cantemos nosotros / as no estemos nosotros / as
cantad vosotros / as no cantis vosotros / as
canten ellos / ellas / ustedes no canten ellos / ellas /ustedes
Infinitivo cantar Gerundio cantando Participio cantado


MODO INDICATIVO DEL VERBO HABER + Participio
Pronombres
Personales
Pretrito
Anterior
Pretrito
Pluscuamperfecto
Pretrito
Perfecto
Futuro
Perfecto
Condicional
Perfecto
Yo hube haba he habr habra
T hubiste habas has habrs habras
l / ella / usted hubo haba ha / hay* habr habra
Nosotros / nosotras hubimos habamos hemos habremos habramos
Vosotros / vosotras hubisteis habais habis habris habrais
Ellos / ellas / ustedes hubieron haban han habrn habran

Ejercicios de Sala #
1. Rellenar las lagunas con el tiempo verbal apropiado:
Pretrito indefinido y Presente del Indicativo: Antao me
______ mucho con los chistes, pero hoy no lo _______ ms.
a) redo - hacemos
b) rio har
c) remos haremos
d) re hago
e) reir hago

2. En la frase:Recorr buena parte de esa fulgurante
cornisa el verbo est:
a) En presente del indicativo
b) En pretrito indefinido del indicativo
c) En pretrito perfecto del indicativo
d) En potencial simple o imperfecto
e) En pretrito imperfecto del subjuntivo

Tarea Complementaria #

Texto: El Maestro (UFSC)






1



5




10










Lo fue mo en clase de retrica, y era bajo, rechoncho,
con gafas idnticas a las que lleva Schubert en sus
retratos, avanzando por los claustros a un paso corto y
pausado, breviario en mano o descansada sta en los
bolsillos del manteo, el bonete derribado bien atrs
sobre la cabeza grande, de pelo gris y fuerte. Casi
siempre silencioso, o si emparejado con otro profesor
acompasando la voz, que tena un tanto recia y
campanuda, las ms veces solo en su celda, donde haba
algunos libros profanos mezclados a los religiosos, y
desde la cual vea en la primavera cubrirse de hoja
verde y fruto oscuro un mural que escalaba la pared del
patinillo lbrego adonde abra su ventana.
Un da intent en clase leernos unos versos,
15




20




25




30




35

trasluciendo su voz el entusiasmo emocionado, y debi
serle duro comprender las burlas, veladas primero,
descubiertas y malignas despus, de los alumnos
porque admiraba la poesa y su arte, con resabio
acadmico como es natural. Fue l quien intent
hacerme recitar alguna vez, aunque un pudor ms fuerte
que mi complacencia enfriaba mi elocucin; l quien me
hizo escribir mis primeros versos, corrigindolos luego y
dndome como precepto esttico el que en mis temas
literarios hubiera siempre un asidero plstico.
Me puso a la cabeza de la clase, distincin que ya
tempranamente comenc a pagar con cierta
impopularidad entre mis compaeros, y antes de los
exmenes, como comprendiese mi timidez y desconfianza
en m mismo, me dijo: Ve a la capilla y reza. Eso te
dar valor.
Ya en la universidad, egostamente, dej de
frecuentarlo. Una maana de otoo y hondo, en mi
camino hacia la temprana clase primera, vi un pobre
entierro solitario doblar la esquina, el muro de ladrillos
rojos, por m olvidado, del colegio: era el suyo. Fue el
corazn quien sin aprenderlo de otros me lo dijo. Debi
morir solo. No s si pudo sostener en algo los ltimos
das de su vida.
CERNUDA, Luis. Ocnos, seguido de Variaciones sobre tema
mexicano.Sevilla: Ayuntamiento de Sevilla, 2002. p. 67 - 69.

3. Elige la(s) afirmacin(es) que corresponda(n) al contenido
del primer prrafo del texto acerca del maestro.
01.Tena libros profanos entre los religiosos.
02. Era un hombre delgado, no llevaba gafas y caminaba
despacio.
04. Raramente estaba solo en su celda.
08. Sola hablar poco.
16. Vea desde su ventana la torre de la iglesia.

4. Seala la(s) proposicin(es) correcta(s). Segn lo
presentado en el segundo y tercer prrafos, entre los recuerdos
de la relacin del narrador con su maestro se cuentan:
01. el intento del narrador de publicar sus poesas.
02. los paseos de los dos por los claustros.
04. el consejo al narrador de rezar en la capilla.
Please purchase PDF Split-Merge on www.verypdf.com to remove this watermark.
Incluso para a Vida Espanhol

Pr-Vestibular da UFSC
11
08. el estmulo al narrador para escribir versos.
16. la correccin de los primeros versos del narrador.

5. Seala la(s) proposicin(es) correcta(s). Me puso a la
cabeza de la clase (lnea 19). Poner a alguien a la cabeza
de algo es un giro que, en el texto, equivale a:
01. poner a alguien para representar a los dems.
02. censurar la conducta.
04. elegir a alguien democrticamente.
08. elegir como lder.
16. sealar a alguien como cabecilla del grupo.

6. Seala la(s) proposicin(es) correcta(s).
El relato acerca del maestro nos presenta a un hombre con
algunas de las siguientes caractersticas:
01. caminaba rpidamente.
02. vivi y muri solitario.
04. caminaba siempre con una Biblia en la mano.
08. era un hombre rudo.
16. era canoso.

UNIDADE 8

LOS HETEROGENRICOS,
HETEROSEMNTICOS Y HETEROTNICOS

Heterogenricos: Son vocablos que presentan variacin de
gnero entre el espaol y el portugus.
Portugus
(femenino)
Espaol
(masculino)
Portugus
(masculino)
Espaol
(femenino)
A aguardente El
aguardiente
O a, o be La a, la be
A
aprendizage
m
El
aprendizaje
O aquecedor La
estufa
A rvore El rbol O aspirador
de p
La aspiradora
de polvo
A cor El color O baralho La baraja
A coragem El coraje O crcere La crcel
A ctis El cutis O costume La costumbre
A desordem El desorden O
computador
La
computadora
A dor El dolor O cume La cumbre
A equipe El equipo O joelho La rodilla
A estante El estante O labor La labor
A estria El estreno O legume La legumbre
A faca El cuchillo O leite La leche
A fraude El fraude O lume La lumbre
A
homenagem
El homenaje O machado La hacha
A linguagem El lenguaje O mel La miel
A macieira El manzano O nariz La nariz
A mensagem El mensaje O paradoxo La paradoja
A origem El origen O protesto La protesta
A paisagem El paisaje O rdio La radio
A passagem El pasaje O sal La sal
A ponte El puente O sangue La sangre
A segunda... El lunes O sinal La seal
A testemunha El testigo O sorriso La sonrisa
A vertigem El vrtigo O visto La visa
A viagem El viaje O natal La navidad

Heterosemnticos: Son vocablos que, a pesar de tener
semejanza grfica tienen significados distintos en espaol y
en portugus.

apellido: sobrenome
apurado: apressado
arrestar: prender
berro: agrio
borrar: apagar
brincar: saltar
calzada: pista da rua
carro: carroa
cachorro: filhote
cena: jantar
clausurar: interditar
coche: carro
competncia: concorrncia
contestar: responder
copo: floco
cuello: pescoo
distinto: diferente
embarazada: grvida
enojar: aborrecer, irritar
escoba: vassoura
escenario: palco
escritrio: escrivaninha
exquisito: gostoso, refinado
jubilacin: aposentadoria
ladrillo: tijolo
largo: comprido
lograr: conseguir
oficina: escritorio
palco: camarote
pronto: logo, brevemente
rato: momento
rojo: vermelho
rubio: loiro
salsa: molho
sitio: local, posto
stano: poro
taller: oficina
taza: xcara, vaso sanitrio
tirar: lanar
todavia: ainda
vaso: copo
zurdo: canhoto

Heterotnicos: Son vocablos que, aunque tengan formas
parecidas, se diferencian por la posicin de la slaba tnica.

academia:academia
acrobacia: acrobacia
acrobata: acrbata
lcool: alcohol
alergia: alergia
algum: alguien
anedota: ancdota
anemia: anemia
anestesia: anestesia
atmosfera: atmsfera
asfixia: asfixia
atrofia: atrofia
bigamia: bigamia
burocracia: burocracia
burocrata: burcrata
canibal: canbal
centigrama: centgramo
crebro: cerebro
canguru: canguro
demagogia: demagogia
democracia: democracia
democrata: demcrata
diplomacia: diplomacia
dspar: dispar
elogio: elogio
epidemia: epidemia
euforia: euforia
filantropo: filntropo
fobia: fobia
gacho: gaucho
hemorragia: hemorragia
heri: hroe
hidrognio: hidrgeno
imbecil: imbcil
impar: impar
limite: lmite
magia: magia
medocre: mediocre
microfone: micrfono
mope: miope
neurastenia: neurastenia
nvel: nivel
nostalgia: nostalgia
oceano: ocano
oxignio: oxgeno
pntano: pantano
periferia: periferia
polcia: polica
prottipo: prototipo
quiromancia: quiromancia
regime: rgimen
rptil: reptil
siderurgia: siderurgia
taquicardia: taquicardia
terapia: terapia
traquia: trquea
telefone: telfono
vertigem: vrtigo

Ejercicios de Sala #
1. Seale la alternativa correcta en relacin a la acentuacin.
a) Slo, prpio, patria, ftbol
b) Rgimen, burcratas, limite, palcio
c) Demcrata, peripecia, nvel, clarn
Please purchase PDF Split-Merge on www.verypdf.com to remove this watermark.
Incluso para a Vida Espanhol

Pr-Vestibular da UFSC
12
d) Das, tirana, aqu, da
e) Crcel, relj, prottipo, fro

2. La frase El presidente Allende muere en su sitio equivale
a:
a) el presidente Allende vive en su sitio.
b) el primer mandatario sucumbe en su puesto.
c) al jefe de gobierno lo matan en su finca.
d) el presidente Allende fallece sitiado.
e) al presidente lo quemaron en el palacio.

Tarea Complementaria #
Texto: Texto A (UFSC)

He necesitado andar todo el camino que dejo recorrido,
para llegar al punto en que nuestro drama comienza.
Es intil detenerse en el carcter, objeto y fin de la
Revolucin de la Independencia. En toda la Amrica fueron
los mismos, nacidos del mismo origen, a saber: el movimiento
de las ideas europeas. La Amrica obraba as porque as
obraban todos los pueblos. Los libros, los acontecimientos,
todo llevaba a la Amrica a asociarse a la impulsin que a la
Francia haban dado Norteamrica y sus propios escritores;
a la Espaa, la Francia y sus libros.
Pero lo que necesito notar para mi objeto es que la
revolucin, excepto en su smbolo exterior, independencia del
Rey, era slo interesante e inteligible para las ciudades
argentinas, extraa y sin prestigio para las campaas.
En las ciudades haba libros, ideas, espritu municipal,
juzgados, derechos, leyes, educacin: todos los puntos de
contacto y de mancomunidad que tenemos con los europeos;
haba una base de organizacin, incompleta, atrasada, si se
quiere; pero precisamente porque era incompleta, porque no
estaba a la altura de lo que ya se saba que poda llegar a ser,
se adoptaba la revolucin con entusiasmo.
Para las campaas, la revolucin era un problema;
sustraerse a la autoridad del Rey era agradable, por cuanto
era sustraerse a la autoridad. La campaa pastora no poda
mirar la cuestin bajo otro aspecto. Libertad, responsabilidad
del poder, todas las cuestiones que la revolucin se propona
resolver eran extraas a su manera de vivir, a sus
necesidades. Pero la revolucin le era til en este sentido:
que iba a dar objeto y ocupacin a ese exceso de vida que
hemos indicado, y que iba a aadir un nuevo centro de
reunin, mayor que el tan circunscrito a que acudan
diariamente los varones en toda la extensin de las
campaas.

3. Segn el texto, se puede afirmar que:
01. Norteamrica influy con sus ideas a Francia.
02. Espaa influy la revolucin francesa.
04. Espaa influy la independencia norteamericana.
08. Los escritores norteamericanos influyeron con sus ideas a
los franceses.
16. Francia y sus libros influyeron a los espaoles.

4. Segn el texto, la revolucin era interesante y comprensible:
01. Para todo el mundo.
02. Para las ciudades argentinas y no para el campo.
04. Para todos en lo que se refiere a su smbolo exterior.
08. Para todos, en lo que se refiere a la independencia del Rey.
16. Para las ciudades argentinas y no para las campaas
polticas

5. La proposicin: La Amrica obraba as porque as obraban
todos los pueblos, se traduce al portugus:
01. Amrica chegavam deste modo, porque assim chegavam
a todos os povos.
02. A Amrica atuava assim, porque assim atuavam todos os
povos.
04. A Amrica pensava assim, porque assim pensavam todos os
povos.
08. A Amrica se comportava assim, porque assim se
comportavam todos os povos.

UNIDADE 9

PRONOMBRES EXCLAMATIVOS, INTERROGATIVOS E
INDEFINIDOS
Todos deben ser acentuados, inclusive en las frases
interrogativas indirectas.

Qu da es hoy?
No s qu regalarle a mi mam.
Qu calor hace!

Cuando usamos la frmula que + presente del subjuntivo,
para expresar deseos, que no debe ser acentuado
grficamente.

Que vuelvas pronto!
Que te vaya bien!

Oraciones interrogativas indirectas
A pesar de no tener punto interrogativos al comienzo y al
final, formulan preguntas introducidas por un verbo o
locucin de entendimiento como: saber, entender, decir,
preguntar, mirar, ver, etc..

Como dijimos, no aparecen los puntos de interrogacin,
pero los pronombres interrogativos van siempre acentuados.
No me pregunt qu quera.
No me preguntes dnde met las llaves pues no lo s.
No s cunto tiempo llevaremos para llegar.

USO DE LOS EXCLAMATIVOS

- Cuando queremos dar mayor intensidad a lo que estamos
diciendo.

Quin tuviera un montn de dinero para comprarse ese yate!
Qu horror! Cmo llueve!
Cunto me cuesta aguantarlo!

LOS INDEFINIDOS

Son palabras que indican imprecisin, indeterminacin.
Ellos implican una nocin cuantitativa, a pesar de que esta
exprese u n nmero indeterminado de objetos, sentimientos,
unidades, grado de intensidad de algo, etc.





Please purchase PDF Split-Merge on www.verypdf.com to remove this watermark.
Incluso para a Vida Espanhol

Pr-Vestibular da UFSC
13

Invariables Variables
alguien
quienquiera
quienesquiera
alguno(s)
alguna(s)
nadie
cualquiera
cualesquiera
poco(s)
poca(s)
algo tal(es)
demasiado(s)
demasiada(s)
nada
ninguno
ninguna*
mucho(s)
mucha(s)
ms - menos bastante(s) todo(s) / toda(s)
uno varios / varias uno(s) / una(s)
dems otro(s) / otra(s)
cierto(s)
cierta(s)
cada tanto(s) / tanta(s)
mismo(s)
misma(s)
semejante**
diverso
diversa***
propio
propia****

(*) = Ninguno / ninguna: Estas formas admiten plural, pero
su uso es poco comn.
(**) = Semejante: Con sentido de tal
- Cmo puedes decirme semejante disparate?
(***) = Diverso / diversa: Con sentido de varios / varias
- Haba productos de diverso tipo.
(****) = Propio / propia: Con sentido de mismo /
misma El propio presidente los recibi.

Ejercicios de Sala #
1. Indica el conectivo correcto en la respuesta a la pregunta:
Por qu no me has trado lo que te ped?
a) Porque no fue posible.
b) Por qu no quiero.
c) Para que es muy difcil.
d) Porqu no encontr el libro.
e) Puesto que pides algo imposible.

2. Contesta correctamente la pregunta del angustiado
fumador: Cuntos cachimbos podr an fumar?
01. Quin sabe!
02. Nadie lo s.
04. No sabamos.
08. Ninguin puede saberlo.
16. No lo s, ni me importa.

Tarea Complementaria #

Texto: El caf viens (UFSC)

1



5




10
Esteban poda recordar el momento exacto en
que se dio cuenta que su hermana era una sombra
fatdica. Fue cuando gan su primer sueldo. Decidi
que se reservara cincuenta centavos para cumplir
un sueo que acariciaba desde la infancia: tomar un
caf viens. Haba visto, a travs de las ventanas del
Hotel Francs, a los mozos que pasaban con las
bandejas suspendidas sobre sus cabezas, llevando
unos tesoros: altas copas de cristal coronadas por
torres de crema batida y decoradas con una hermosa




15




20




25




30




35




40




45

guinda glaceada. El da de su primer sueldo pas
delante del establecimiento muchas veces antes de
atreverse a entrar. Por ltimo cruz con timidez el
umbral, con la boina en la mano, y avanz hacia el
lujoso comedor, entre lmparas de lgrimas y
muebles de estilo, con la sensacin de que todo el
mundo lo miraba, que mil ojos juzgaban su traje
demasiado estrecho y sus zapatos viejos. Se sent en
la punta de la silla, las orejas calientes, y le hizo el
pedido al mozo con un hilo de voz. Esper con
impaciencia, espiando por los espejos el ir y venir de
la gente, saboreando de antemano aquel placer
tantas veces imaginado. Y lleg su caf viens,
mucho ms impresionante de lo imaginado,
soberbio, delicioso, acompaado por tres galletitas
de miel. Lo contempl fascinado por un largo rato.
Finalmente se atrevi a tomar la cucharilla de
mango largo y con un suspiro de dicha, la hundi en
la crema. Tena la boca hecha agua. Estaba
dispuesto a hacer durar ese instante lo ms posible,
estirarlo hasta el infinito. Comenz a revolver
viendo cmo se mezclaba el lquido oscuro del vaso
con la espuma de la crema. Revolvi, revolvi,
revolvi... Y, de pronto, la punta de la cucharilla
golpe el cristal, abriendo un orificio por donde
salt el caf a presin. Le cay en la ropa. Esteban,
horrorizado, vio todo el contenido del vaso
desparramarse sobre su nico traje, ante la mirada
divertida de los ocupantes de otras mesas. Se par,
plido de frustracin, y sali del Hotel Francs con
cincuenta centavos menos, dejando a su paso un
reguero de caf viens sobre las mullidas alfombras.
Lleg a su casa chorreado, furioso, descompuesto.
Cuando Frula se enter de lo que haba sucedido,
coment cidamente: eso te pasa por gastar el
dinero de las medicinas de mam en tus caprichos.
Dios te castig. En ese momento Esteban vio con
claridad los mecanismos que usaba su hermana para
dominarlo, la forma en que consegua hacerlo
sentirse culpable y comprendi que deba ponerse a
salvo.

3. De acuerdo con la idea global del texto es correcto afirmar
que:
01. Esteban sola gastar el dinero de su madre en los cafs, por
eso a Frula le molestaba lo que l haca.
02. El caf se desparram ante la mirada de los frecuenta-
dores del hotel.
04. Despus del incidente en el Hotel Francs, Esteban
percibi que la relacin con su hermana era nefasta.
08. Esteban jams se olvid del triste episodio en el hotel,
porque fue cuando se dio cuenta de cmo era su hermana.

4. De las siguientes afirmaciones: se sent en la punta de la
silla e le hizo el pedido al mozo con un hilo de voz, se
deduce que:
01. Esteban no frecuentaba muy a menudo ambientes como
aquel, por eso se senta bastante incmodo.
02. Los cafs eran sitios muy frecuentados y ruidosos por lo
que haba que estar siempre atento.
04. A pesar de la experiencia en observar desde la calle el ir y
venir de los mozos, Esteban an no se senta a gusto en
aquel sitio.
Please purchase PDF Split-Merge on www.verypdf.com to remove this watermark.
Incluso para a Vida Espanhol

Pr-Vestibular da UFSC
14
08. El espacio estaba siempre lleno de personas elegantes y
por eso era importante dar a conocer su buena educacin.
16. La actitud de Esteban demostraba su nerviosismo e
inhibicin.

5. Seala la(s) proposicin(es) donde la palabra destacada
podra ser sustituida correctamente como sinnimo de hundir
o hundirse.
01. El ciclista se detuvo en los ltimos tres kms.
02. Los turistas inundan en enero las costas brasileas.
04. El edificio se derrumb porque estaba mal construido.
08. El Imperio Romano se desplom en pocos aos.
16. El velero naufrag a causa de una tormenta.

6. Seala la(s) proposicin(es) que sustituye(n) corectamente
la expresin subrayada:
Lleg a su casa chorreado, furioso, descompuesto
01. mesurado
02. airoso
04. estropeado
08. atascado
16. estreido
32. desfigurado

7. Seala la(s) proposicin(es) que guarda(n) relacin entre s.
01. copa, taza, vaso, cliz.
02. silln, sof, taburete, silla.
04. cama, sbana, manta, jabn.
08. taller, camarero, mostrador, mozo.
16. traje, falda, calcetines, corbata.
32. sueldo, paga, salario, ingresos.

8. Seala la(s) proposicin(es) correcta(s).
Tomar un caf viens (Viens de Viena)
01. Albano de Albania
02. Griego de Grecia
04. Irano de Irn
08. Bosniense de Bosnia
16. Guatemaltino de Guatemala
32. Quiteo de Quito

UNIDADE 10
LOS ADVERBIOS

Los adverbios son palabras invariables que califican un
verbo, un adjetivo, otro adverbio o un pronombre.

Tipo de
Adverbio
Formas
Simples
Locuciones
Adverbiales
de lugar aqu, ac, ah, all,
all, encima, debajo,
arriba, abajo, dentro,
adentro, fuera, afuera,
adelante, delante,
detrs, cerca, lejos,
alrededor, aparte
all arriba, aqu
abajo, por todas
partes, a la derecha, a
la izquierda, fuera de,
a lo lejos, en el
centro, ms encima,
etc.
de tiempo ayer, anoche, hoy,
maana, ahora,
entonces, antes,
despus, pronto,
temprano, tarde,
todava, an, mientras,
por la maana, por la
tarde, por la noche,
pasado maana, hoy
da, dentro de poco,
en breve, de aqu en
adelante, de vez en
siempre, recin, ya,
nunca, jams,
primero, luego,
enseguida / en
seguida.
cuando, en el futuro,
etc.
de modo bien, mal, peor, mejor,
as, tal, despacio,
deprisa, casi, como
y los terminados en
-mente: especialmente,
gilmente,
difcilmente, etc.
de repente, de nuevo,
a la francesa, a lo
grande, a
regaadientes, a
ciegas, a menudo, a
golpes, a cntaros, a
hurtadillas, etc.

de
cantidad
bastante, muy, mucho,
demasiado, cuanto,
cuan, nada, poco,
tanto, tan, menos, ms,
algo, casi, todo, slo,
apenas, medio,
adems, incluso,
tambin.
al menos, poco a
poco, solamente, al
por mayor, al por
menor, poco ms,
poco menos, etc.
de
afirmacin
s, tambin, claro,
cierto, seguro.
por cierto, sin duda,
por supuesto, desde
luego, como no,
seguramente,
verdaderamente, etc.
de
negacin
no, tampoco, nunca,
jams, siquiera.
de ningn modo, ni
con mucho, ni por
asomo, etc.
de
duda
quiz, quizs, tal vez,
acaso.
quiz, quizs, tal vez,
acaso

Ejercicios de Sala #
1. Complete la frase con muy o mucho(s):
Hay ____ que hacer, pues la alfabetizacin es ______ difcil
en pases subdesarrollados, donde hay ______ analfabetos.
a) mucho muchos mucho
b) mucho mucho muy
c) muy muy mucho
d) muy mucho muchos
e) mucho muy muchos

2. _____ hemos llegado _____ ______ _____. Los
adverbios que completan correctamente esta frase son:
a) anoche mucho ms tarde
b) maana mucho ms tarde
c) pasado maana muy ms tarde
d) ahora muy ms tarde
e) maana muy ms tarde

3. Los trminos subrayados en la frase Tampoco es casual
que recin despus de dar este paso pueden traducirse
respectivamente por:
a) To pouco cedo
b) Tambm no recentemente
c) Tambm ultimamente
d) Portanto recente
e) Assim de novo





Please purchase PDF Split-Merge on www.verypdf.com to remove this watermark.
Incluso para a Vida Espanhol

Pr-Vestibular da UFSC
15
Tarea Complementaria #

Texto: Planeta Marte

1



5




10




15




20




25




30




35




40
En Marte se encontr agua. Decan las noticias
del da de ayer. La NASA y ESA (Agencia Espacial
Europea) encontraron pruebas de agua lquida en
la superficie del planeta Marte. Y los pasquines: La
Prensa, BBC, CNN, El Pas, dieron el noticin. Los
tres hermanos grandes de la OTAN convocaron
una reunin extraoficial. Y en el Parlamento de esa
institucin se escuch un dilogo.
USA Los marcianos tienen armas de
destruccin masiva. Reino Unido Son un peligro
para la Paz Mundial. Israel Son terroristas
galcticos.
Y gritan al unsono. Debemos conducirlos por
buen camino y llevarlos a la democracia.
Y preguntan a su Dios todo poderoso Padre!
Tenemos el mundo en nuestras garras, ya no nos es
suficiente. Queremos colonizar Marte, hay agua,
bosques y atmsfera, pero esos enanos nos
distraen, los odiamos y nos fascinan. Padre, qu
debemos hacer?
Los enanos les fascinan y odian porque an no
son miembros del mercado. Para convertirlos en
batracios y primates versin marciana, hay que
enviar productos de nuestras transnacionales.
Ustedes deben estudiarlos, cortejarlos,
engatusarlos, hipnotizarlos, hacerlos dependientes
y de esta manera habrn comprado y cambiado
tambin sus nimas, Djales que escuchen Shakira,
Jessica Simpson, Britney Spears, djales que
coman hamburguesas, tomen Coca-Cola y que sus
pastas dentfricas contengan ms porcentaje de
flor. Maniplenlos. Como han hecho siempre.
En Marte, los marcianos viven bajo la
superficie del planeta y de vez en cuando dejan que
su orina salga a la superficie, para que el sol la
evapore. Conocen muy bien a los tres hermanos,
han aprendido de los indios, palestinos, africanos y
dicen que no van a cambiar sus tapabarros por
vaqueros, ni usarn camisas Lacaste, ni gafas de
marca mayor. Dicen que ellos nacieron desnudos y
que el resto es falsa cultura. Avisaron al
escarabajo estelar que ellos quieren vivir en paz.
Por que si no Y digo: El que avisa de antemano
no es traidor.

4. Indica la proposicin correcta para el tema principal del
texto.
a) La bsqueda de agua en unin con los habitantes de Marte.
b) Traer los marcianos para la superficie del planeta para que
sientan el aire fresco.
c) La tentativa de dominio de Marte a travs de la imposicin
consumista ilusoria.
d) Respetar los deseos de los habitantes desnudos para
conquistar el agua.
e) Vivir en paz, pues ya avisaron el escarabajo antes que el
agua evapore.

5. Conocen muy bien a los tres hermanos (lneas 21-22),
se refiere a:
a) Shakira, Jessica Simpson, Britney Spear
b) NASA, ESA, OTAN
c) Marcianos
d) Indios, palestinos, africanos
e) Hamburguesas, coca-cola, pasta dentfrica

6. Para convertirlos entambin sus nimas (lneas 14-17),
quiere decir que:
a) Estarn ms animados despus de las tcnicas aplicadas
para el cambio de vida, y la conquista no dar trabajo.
b) Sabrn que la llegada ser de conquista rpida y favorable,
con honestidad, con buenos tratos para la integracin.
c) Sern conquistados con facilidad, y les gustar el cambio
para otra realidad de convivencia, pero podrn quedarse
malhumorados.
d) Estn dispuestos a ayudar con la influencia social, y slo
traer beneficios para los enanos, comprarn agua para
beber.
e) Usarn formas para la conquista de los marcianos, y al
final creen que ellos sern ms felices, hasta comprarn
sus humores.

7. Seala la proposicin correcta. A qu se niegan los
habitantes de Marte?
a) Vestirse con taparrabos
b) Vivir en Marte
c) Vivir bajo la superficie del planeta
d) Sacar los taparrabos y vestirse
e) Vivir en paz

8. La palabra desnudos (lnea 24), puede ser cambiada, sin
alterar el sentido de la frase por:
a) Con nudos
b) Sin nudos
c) Sin ropas
d) Pelados
e) Sin comida

UNIDADE 11

LAS PREPOSICIONES

Las preposiciones son palabras invariables que unen
trminos de la oracin, estableciendo una relacin entre ellos,
que puede ser de complementacin o de explicacin.

A
Direccin y movimiento = Vamos a pasear.
Nmero, medida o precio = Se vende a cien
reales.
Finalidad u objeto = Escribo a todos.
Modo, medio o instrumento = Habla a gritos.
Indica seres personificados (personas o
animales) = Vi a Juan en el cine.
ANTE
Significa delante de o en presencia de =
Ante la duda, piensa bien.
Tiene el sentido de respecto a = No quiero
Please purchase PDF Split-Merge on www.verypdf.com to remove this watermark.
Incluso para a Vida Espanhol

Pr-Vestibular da UFSC
16
comentarios ante este imprevisto.
BAJO
Posicin inferior = El gato se meti bajo el
(debajo del) auto.
Dominio o subordinacin = Ests bajo su
poder.
Dependencia = Ests bajo los cuidados de tus
padres.

CON

Rasgo personal = Con lo estudioso que es
seguro que le va bien.
Indica compaa = Despus voy con mi mam.
Medio o instrumento = Jugar con la baraja
espaola.
Modo o manera = Cantar con alegra.
Caracterstica de algo = Guantes con lana por
dentro.
CONTRA
Situacin adversa: Est todo contra l.
Oposicin: Luch contra el campen.
Resistencia fsica: Se apoy contra la pared.
DE
Materia, asunto o tema = Clase de espaol.
Causa o forma = Ella est muerta de nervios.
Uso o empleo = Mquina de escribir.
Punto de partida o procedencia = Salimos de
Tucumn el viernes.
Posesin = Aquel auto es de Rubn.
Definicin verbal = No es fcil de creer.
DESDE
Periodo de tiempo = Desde el ao pasado que
no lo veo.
Posicin o lugar = No daba para ver muy bien
desde aqu.
DURANTE
Duracin de tiempo = Trabaj durante muchos
aos como cerrajero.
Duracin de accin = Nadie dijo nada durante
la procesin.
EN
Medio de transporte = Viajar en coche.
Localizacin = Vivo en Lima.
Situacin o circunstancia = De nuevo ests en
problemas.
Tempo = Te casaste en 1995 o antes?
Modo = Estoy hablando en serio.




ENTRE
Situacin intermediaria entre personas o cosas
= Entre ellos dos hay un atleta.
Relacin = Entre t y yo no deben haber
secretos.
Participacin y/o cooperacin = Lo construimos
entre todos los nietos.
Intervalo de tiempo = Vivi en Caracas entre
1986 y 1992.
Comparacin = Entre esta marca y otra, hay
diferencias.
EXCEPTO Ausencia o exclusin = Todos vinieron excepto
Nacho.
HACIA Tiempo aproximado = Llegaremos hacia el
medioda.
Direccin aproximada = Este verano pienso ir
hacia el sur para pasar las vacaciones.



HASTA
Indica trmino o lmite de:
Tiempo = Est bien, pero slo hasta las ocho
de la noche.
Lugar = Hoy caminaremos hasta la cascada.
Accin = Bailamos hasta que nos cansamos.
Cantidad = Pago hasta US $1000 al contado.
INCLUSO Equivale a at mesmoen portugus = Incluso
a l tambin le gust.
MEDIANTE Significa por medio de = Venci mediante
mucho esfuerzo y dedicacin.

PARA
Finalidad, utilidad, aptitud = Agua para beber.
Direccin del movimiento = Va para su casa.
Implicacin personal = Para ti todo es igual,
todo te da lo mismo.
Comparacin o desproporcin = Hay poca
comida para tanta gente.
Equivale a la expresin por qu = Para qu
viniste a verme hoy?


POR

Explica razones o motivos = Por eso son tan
diferentes.
Lugar de trnsito o paso = Vamos por la acera.
Multiplicacin = Dos por dos son cuatro?
Proporcin = Diez por ciento.
Valor = Lo compr por cuarenta mil pesos.
Tiempo = Sali por ms de una hora
Medio de comunicacin = Llam por telfono
pero no dej ningn recado.
Objetivo o finalidad = Por verte hara
cualquier cosa.

PRO
Equivale a decir a favor de = Una
contribucin pro desvalidos.
Equivale a decir con la intencin de = Vamos
trabajar en pro de construir un pas mas justo.

SALVO
Sinnimo de excepto = Se salvaron todos,
salvo el capitn.

SEGN
Referencia = Segn Juan y Miguel no hay
porque preocuparse.
Adverbio de tiempo o de modo = Te sentirs
mejor segn pasen los das / te sanes.

SIN
Privacin o carencia = Est sin plata
Condicin para la accin = Nos van a pagar sin
que tengamos que trabajar el mes completo.


SOBRE
Significa encima de (sobreposicin) = Est
sobre la mesa
Significa a respecto de (sobreposicin) = Hoy
vamos a hablar sobre los hijos.


TRAS
Posterioridad: Espacial = Corrimos tras el tren.
Temporal = Da tras da espero tu retorno.
Personal = Uno tras otro los vio pasar.
Puede significar adems de = Tras llegar
tarde, se emborracha.

Ejercicios de Sala #
1. Estamos mirando hacia la ventana. La preposicin
subrayada corresponde a la locucin:
a) por encima de
b) alrededor de
c) en direccin de
d) por detrs de
e) acima de

2. Disert con brillo __________ el tribunal examinador
Qu preposicin completa la frase con coherencia?
a) bajo
b) ante
c) perante
d) desde
e) a



Please purchase PDF Split-Merge on www.verypdf.com to remove this watermark.
Incluso para a Vida Espanhol

Pr-Vestibular da UFSC
17
Tarea Complementaria #

Texto: Zoolgico para semillas del mundo (Alejandro
Nadal) (UFSC)

1



5




10




15




20




25




30




35




40




45
Ayer fue inaugurado el proyecto La Bveda Global de
Semillas, que consiste en la construccin de depsitos
subterrneos en la isla de Svalbard, a mil kilmetros del
Polo Norte. All se conservarn hasta 4.5 millones de
muestras de semillas. stas estarn almacenadas a
temperaturas de menos 18 grados centgrados y podran
permanecer viables miles de aos. Dichas muestras
provendrn de cultivos y ecosistemas de todo el mundo,
desde los suelos de Madagascar hasta los valles
interandinos de Per y Ecuador, pasando por las
mesetas de Etiopa y el altiplano mexicano.
El objetivo central del proyecto es mantener una
reserva de semillas para el caso de que una catstrofe
global amenace la seguridad alimentaria de la
humanidad. Y para lograr dicho fin busca preservar
muestras de variedades de los cultivos ms importantes
del mundo.
La Bveda Global de Semillas est a 70 metros de
profundidad en una montaa de roca slida y se accede
a las tres naves del depsito por un tnel subterrneo de
100 metros de largo. A decir de sus constructores, La
Bveda puede soportar una catstrofe por guerra
nuclear, el impacto de un avin o los terremotos que
pueden presentarse en esa apartada regin.
El punto de partida del proyecto es que la diversidad
gentica puede peligrar por guerras, desastres
naturales, falta de financiamiento adecuado o mal
manejo de las semillas. El director de la construccin
de la bveda Cary Fowler afirma que este instrumento
permitira reconstituir la agricultura mundial en caso de
una catstrofe. Se referir a la dramtica situacin por
la que atraviesa la agricultura en el mundo?
El proyecto ha sido inaugurado precisamente cuando
la agricultura sustentable atraviesa su peor crisis. Las
corporaciones trasnacionales, los gobiernos de muchos
pases y los organismos internacionales responsables
para el sector agropecuario han declarado una guerra
sin cuartel a los millones de campesinos de subsistencia
y a los agricultores que utilizan los principios de la
agroecologa como base de sus estrategias de
produccin. No sorprende que el proyecto est apoyado
por las fundaciones Rockefeller, Bill y Melinda Gates, el
Banco Mundial y empresas como DuPont (Pioneer) y
Syngenta. Estos socios del gobierno noruego no tienen
buena reputacin en el tema de la agricultura
sustentable.

3. Seala la(s) proposicin(es) que expresa(n) la idea del
texto.
01. El proyecto en cuestin propone aplicar a las semillas
tcnicas usadas en los zoolgicos.
02. Se trata de un recurso para salvar a las especies animales
en peligro de extincin.
04. Se trata de un proyecto para salvar los cultivos del planeta
en caso de extincin.
08. La Bveda es una moderna Arca de No que salvar a los
elegidos para que pueblen el mundo, en caso de catstrofe
nuclear.
16. Curiosamente, el proyecto es apoyado por empresas que
provocan los peligros que se pretende conjurar.

4. Seala la explicacin adecuada de los vocablos a la
izquierda, tomados del texto.
01. seguridad alimentaria (lnea 11) seguro de vida.
02. 100 metros de largo (lnea 17) medicin de la
longitud.
04. mal manejo (lnea 22) accin criminal.
08. no tienen buena reputacin (lnea 34) carecer de
prestigio.
16. guerra sin cuartel (lnea 30) guerra con armas
nucleares.

5. Qu pregunta(s) encuentra(n) respuesta en el texto?
01. Qu especies animales se conservarn en el Zoolgico de
semillas?
02. Quines son los patrocinadores del proyecto?
04. Durante cunto tiempo funcionar el proyecto?
08. Hasta cuntas muestras de semillas sern conservadas en
La Bveda?
16. Qu lugar se ha elegido para la construccin de La
Bveda?
32. Habr en La Bveda semillas de cultivos brasileos?

6. As como consta en el tercer prrafo, donde leemos que los
constructores hicieron la Bveda, podemos decir tambin
que:
01. los FONTANEROS arreglan los grifos.
02. los ALBAILES hacen abanicos.
04. las AZAFATAS auxilian en los tribunales.
08. los SASTRES hacen trajes.
16. los CERRAJEROS hacen martillos.
32. los BEDELES cuidan del orden fuera de las aulas.

UNIDADE 12

LAS CONJUNCIONES
Las conjunciones son palabras que sirven para unir dos
trminos o dos oraciones dentro de una misma oracin.

Conjunciones coordinadas / coordinantes
Copulativas
y - e


- La conjuncin y del espaol equivale a la
conjuncin e del portugus.

Slo est faltando vino y bebidas.
Nios y nias.

- Cuando la conjuncin y precede a una
palabra que comienza con i o hi (seguida
por consonante) ella debe ser sustituida por la
letra e.

Es una reunin de padres e hijos.
Saludables e inteligentes.

Please purchase PDF Split-Merge on www.verypdf.com to remove this watermark.
Incluso para a Vida Espanhol

Pr-Vestibular da UFSC
18
- Se usa la conjuncin y antes del diptongo
hie.
Lo que necesitas es un shampoo con extracto de
miel y hierbas.
ni - La conjuncin ni del espaol corresponde a
la conjuncin nem del portugus.

Disclpeme pero yo no como ni mariscos, ni
algas.
No me preguntes a m, que yo ni los conozco
todava.

Disyuntivas
o - u - La conjuncin o del espaol corresponde a
la conjuncin ou del portugus y cuando
antecede a palabras que comienzan por o o
hu, ella debe ser sustituida por u .

Decidmoslo al cara o sello.
Su recuperacin es cuestin de das u horas.

- Cuando la conjuncin o aparece entre
nmeros debe ser sustituida por la
(acentuada) para que no se le confunda con
el nmero 0.

7 8. 524 529? 0,013 0,016


Adversativas
mas - pero - Ambas corresponden a la conjuncin ms
del portugus. En espaol la forma peroes
mucho ms usual en la lengua hablada que la
forma mas.

Me gustara salir con ustedes, mas estoy lleno
de trabajo.
Bueno es el cilantro, pero no tanto.
sino - Corresponde al mas sim, exceto ou seno, a
no ser del portugus. Siempre se emplea
despus de una negacin.

No come carnes sino pescado. (exceto)
Nunca fui al sur sino en verano. (seno, exceto)
Esa mujer no sabe hacer otra cosa sino
quejarse. (a no ser)
No era solo l sino todos los que estaban en la
fiesta. (ms)

- No debe ser confundida con la expresin si
no que indica una condicin.

Si no vienes a verme, me enojar.
aunque - Corresponde al ainda que, a pesar de, mesmo
que del portugus.

Aunque se lo dijimos muchas veces, l no quiso
hacernos caso.
Ir a verte, aunque est lloviendo.
Aunque viejo, todava conserva su vitalidad.
sin
embargo

- Corresponde a las formas embora, contudo,
todava, entretanto del portugus. Puede
aparecer entre comas o entre punto y coma y
no
obstante
coma.

No les gusta, sin embargo, lo aceptan porque es
un pariente.
Ella podra haber entrado en la universidad; no
obstante, no quiso.

Conjunciones subordinadas / subordinantes
Temporales
mientras - Corresponde al enquanto del portugus.
Expresa simultaneidad de acciones. Su sinnimo
en espaol es entretanto o la locucin
mientras tanto, algo ms equivalentes al
portugus enquanto isso.
Anoche te reste mientras dormas.
Puedo preparar unos tragos mientras tanto?
Entretanto los chiquillos se estaban agrupando.
en cuanto






- Corresponde al to logo, assim que del
portugus. En espaol pueden ser sustitui-das por
tan pronto, ni bien.

En cuanto te llame, me avisas para hablar un
poco con l.
Te escribir en cuanto llegue a la casa.
Luego en cuanto termine, te lo presto para que
lo copies.
- El en cuanto del espaol tambin significa
no que se refere a en portugus.

En cuanto a lo que ya conversamos, hay
alguna duda?
Nadie puede decir nada en cuanto a su
conducta.
apenas










- El apenas del espaol puede significar casi
no y tan slo, respectivamente quase no y
s o somente en portugus.

Est tan viejito que apenas anda.
Se senta tan extrao que casi no coma.
Apenas compr queso y mortadela.
Demor casi tres horas y trajo dos camisetas
apenas.

- Puede significar tambin to logo, mal o
nem bem en portugus.

Apenas supe la noticia, me vine corriendo.
Apenas consigui el empleo y ya quera un
aumento.

Consecutivas
luego - Corresponde al portanto, por conseguinte del
portugus.
Como no quisiste entrar en nuestro equipo,
luego te arrepentirs.
Pienso, luego existo.

conque - Equivale al ento, de maneira que del
portugus.
Ya me lo dijeron todo, conque no me vengas a
engaar.
Conque te saliste con la tuya.
as que - En portugus significa de modo que, de
Please purchase PDF Split-Merge on www.verypdf.com to remove this watermark.
Incluso para a Vida Espanhol

Pr-Vestibular da UFSC
19
maneira que, ento.
Se cort el agua, as que no podrn ducharse.

- No debe ser confundido con assim que del
portugus, que significa to logo.
(Port.) Assim que voc chegar do colgio,
arruma teu quarto.
(Esp.) Tan pronto llegues del colegio, arregla
tu pieza.
Condicionales
si - Equivale a la conjuncin condicional se del
portugus. Pero mientras que en el portugus se
usa el verbo en el futuro del subjuntivo, en
espaol se usa en el presente del indicativo.
(Port.) Se puder, amanh vou estudar fsica.
(Esp.) Si puedo, maana voy a estudiar fsica
siempre que - Puede ser temporal o condicional, dependiendo
del contexto.

Siempre que me suelto el pelo, me dicen que me
veo ms madura. (Temporal con sentido de
cuando o de todas las veces)
Te ir bien, siempre que te organices con
anticipacin.. (Condicional con el sentido de
si)

Ejercicios de Sala #

1. Cul de las alternativas completa correctamente la laguna
del siguiente enunciado?: Esa gente sufra tanto en vida que
acept el sacrificio ____________ fuera para alcanzar algo
ms all de la muerte.
a) antes
b) aunque
c) luego
d) sin embargo
e) todava
2. Indique la(s) opcin(es) correcta(s): Angel _________
musa vienen de fuera, dolor _________ imagen de dentro;
_________ nada sern sin el trabajo incansable del poeta.
01. e y mientras
02. e e pero
04. y e no obstante
08. y u todava
16. e o sino
32. y e sin embargo
64. y y mas

Tarea Complementaria #

Texto: Educacin para la inclusin social (UFSC)

1



5
Pese a los grandes avances que se han registrado en
materia de educacin en Amrica Latina y el Caribe en
la ltima dcada, an hay 4,2 millones de nios y nias
en edad escolar que no asisten a la escuela.
Pero, quines son estos excluidos del siglo XXI en




10




15




20




25




30




35




40




45




50




55




60
una regin que parece haber consolidado su democracia
y logrado una mayor estabilidad econmica que algunos
pases de Norteamrica y Europa?
Ciertamente no los vamos a encontrar en los
promedios nacionales y menos an en los regionales. Un
promedio de escolarizacin primaria del 95 por ciento,
muy cercano al de los pases desarrollados, no nos alerta
sobre los varios millones de nios y nias que no asisten
a la escuela. Sin embargo, las brechas entre los ms
ricos y los ms pobres son alarmantes, especialmente en
la educacin secundaria. En Guatemala y El Salvador,
por ejemplo, solo el 20 por ciento de la poblacin ms
pobre tiene acceso al nivel secundario, frente a un 60 por
ciento de los que tienen mayores ingresos.
Uno de los principales grupos excluidos, que
constituye ms del 30 por ciento de la poblacin de la
regin, est conformado por los pueblos indgenas y los
afrodescendientes. Otro son las nias. Aun en los pases
cuyos promedios en educacin primaria indican paridad,
se evidencia un rezago de las nias en las zonas rurales e
indgenas.
Tener alguna discapacidad es otro factor de
exclusin: solo entre el 20 y el 30 por ciento de los nios
con discapacidades asisten a la escuela en nuestros
pases. Igualmente, la escuela est aun cerrada para la
mayora de los nios, nias y adolescentes que viven con
VIH y SIDA.
Invertir en educacin influye en el desarrollo
humano y tiene un gran retorno para el individuo y para
la sociedad. Cuanto ms se invierte en educacin, tanto
menos se gasta en curar enfermedades que son
prevenibles y en reducir la violencia.
Pese a las evidencias, an invertimos poco en
educacin. En casi todos los pases de la regin, la
educacin primaria y la secundaria media son gratuitas.
Sabemos, sin embargo, que en la prctica esto no es as.
Aun cuando la matrcula es gratuita, los otros costos
asociados a la educacin pueden ser demasiado altos
para las familias pobres.
A pesar de los problemas, los pases de la regin han
avanzado mucho y han encontrado soluciones exitosas
con potencial de reproduccin. Por ello, Unicef
promueve espacios de intercambio entre los gobiernos de
la regin. As lo haremos hoy en Buenos Aires ante los
ministros, ministras y altos responsables de la infancia y
de la adolescencia reunidos en la XII Conferencia
Iberoamericana.
Todos los Objetivos de Desarrollo del Milenio sern
ms fcilmente alcanzables si aseguramos ms y mejor
educacin a todos y cada uno de los nios, nias y
adolescentes de nuestros pases. En este sentido, la
educacin es un derecho en s mismo y tambin la llave
para la realizacin de los dems derechos universales.
La generacin de los bicentenarios merece un
mundo ms inclusivo y ms equitativo. Este mundo
comienza en la escuela.

3. Seala la(s) proposicin(es) correcta(s). De acuerdo con el
texto 1, se puede afirmar que:
01. la educacin no es un gran problema en pases
latinoamericanos, ya que el promedio de escolarizacin
primaria del 95 por ciento los acerca de los pases
desarrollados.
Please purchase PDF Split-Merge on www.verypdf.com to remove this watermark.
Incluso para a Vida Espanhol

Pr-Vestibular da UFSC
20
02. acerca de la educacin, el texto argumenta sobre grupos
que se encuentran excluidos en Amrica Latina y el Caribe,
en especial: los discapacitados, las nias, los indgenas, los
afrodescendientes y los adolescentes que viven con VIH y
SIDA.
04. la inversin en la educacin es un beneficio no slo
individual como social; es un camino para la disminucin
de la violencia.
08. los promedios nacionales representan eficientes
indicadores acerca de la realidad educacional de los pases
de Amrica Latina y el Caribe.
16. muchos pases latinoamericanos, como Guatemala y El
Salvador, han encontrado soluciones para el problema de la
exclusin en la educacin de la poblacin pobre, lo que
hace con que Unicef promueva espacios de intercambio
entre los gobiernos de la regin.

4. Acerca de las informaciones presentes en los prrafos 7
o
, 8
o

e 9
o
del texto 1, seala la(s) proposicin(es) correcta(s).
01. Los pases de Amrica Latina y Caribe todava invierten
poco en educacin. Sin embargo, la gratuidad de la
enseanza primaria y secundaria garantizan la inclusin de
alumnos de todos los niveles sociales.
02. La educacin es la llave para la conquista de los derechos
universales, adems de ser un derecho a todos los nios,
nias y adolescentes de los pases de Amrica Latina y el
Caribe.
04. Buenos Aires ha avanzado mucho en relacin a la
educacin. Por ello, sus ministros, ministras y altos
responsables de la infancia y la adolescencia presentarn a
los dems pases sus soluciones exitosas.
08. No cobrar la matrcula no es suficiente para considerar la
educacin gratuita. Hay que llevar en cuenta todos los
dems costos.
16. La propuesta de Unicef es permitir a los gobiernos de
Amrica Latina y el Caribe intercambiar las soluciones
exitosas encontradas por los pases de la regin.




5. Seala la(s) proposicin(es) correcta(s). En el primer
prrafo del texto 1, la expresin pese a puede ser sustituida,
sin alterar el significado de la frase, por:
01. A causa de
02. En cuanto a
04. A pesar de
08. En contra de
16. Por lo tanto

6. Seala la(s) proposicin(es) correcta(s). Las expresiones
pero, an y as, destacadas en el texto 1, podran ser
respectivamente sustituidas, sin perjuicio de significado, por:
01. ms, aunque, de este modo
02. mas, aunque, en cuanto
04. sin embargo, incluso, mientras
08. sin embargo, todava, de este modo
16. mas, todava, de esta forma

7. Con base en el fragmento abajo, seala la(s)
proposicin(es) correcta(s).
A pesar de los problemas, los pases de la regin (1)han
avanzado mucho y han encontrado soluciones exitosas con
potencial de reproduccin. Por (2)ello, Unicef promueve
espacios de intercambio entre los gobiernos de la regin.
(5)As (3)lo (4)haremos hoy en Buenos Aires ante
los ministros, ministras y altos responsables de la infancia y y
de la adolescencia reunidos en la XII Conferencia
Iberoamericana.
01. (1)han avanzado verbo avanzar, conjugado en la 3.
persona del plural del pretrito perfecto compuesto de
indicativo.
02. El pronombre complemento (3) lo sustituye el hecho
citado anteriormente de Unicef promover espacios de
intercambio entre los gobiernos de la regin.
04. El verbo (4) haremos puede ser sustituido sin perjuicio
de significado por la forma vamos a hacer.
08. (2) ello puede ser sustituido por eso sin cambiar el
significado de la frase.
16. La frase (5) As lo haremos hoy en Buenos Aires ante los
ministros,... pasada a la 1. persona del singular se escribira
de la siguiente manera: As lo harei hoy en Buenos Aires
ante los ministros,...


Please purchase PDF Split-Merge on www.verypdf.com to remove this watermark.
Incluso para a Vida Fsica A

Pr-Vestibular da UFSC 1
UNIDADE 1
CINEMTICA INTRODUO

Mvel
Chamamos de mvel o objeto que est em movimento. Os
mveis podem ser classificados em:
Ponto Material ou Partcula: o mvel ser considerado
uma partcula quando suas dimenses puderem ser
desconsideradas no estudo de um movimento.
Corpo Extenso: o mvel ser um corpo extenso quando
suas dimenses no forem desprezadas.
Ateno:
1) No se pode desconsiderar a massa de uma partcula.
2) Todo mvel que realizar movimento de rotao dever
ser considerado um corpo extenso.

Movimento e Repouso
Um mvel estar em movimento ou repouso dependendo
do referencial adotado.
Exemplo: Um motorista de nibus enquanto dirige est em
movimento em relao estrada, mas est em repouso em
relao ao seu assento.

Trajetria
a linha geomtrica que representa o caminho descrito por
uma partcula em movimento em relao a um dado
referencial.
A trajetria relativa, isto , depende do referencial
adotado.

Posio em uma trajetria (Espao)
Representado pela letra s, espao o valor algbrico da
distncia, medida sobre a trajetria, entre a posio
ocupada por um mvel at a origem (O: ponto de
referncia)

Na figura, o espao ocupado pelo mvel representado pela
esfera s = 3 m.

Deslocamento ( s
&
A )
a distncia entre a posio inicial e a posio final do
mvel, sem se preocupar com a trajetria. uma grandeza
vetorial.
0
s s s = A



Considerando, na figura acima, que a posio inicial do
mvel foi 0
0
= s e a posio final foi m s 5 = , o
deslocamento escalar calculado:
m s s s s s 5 0 5
0
= A = A = A



Distncia Percorrida (d)
a medida da distncia, sobre a trajetria, percorrida pelo
corpo. uma grandeza escalar.


Suponha que o mvel da figura acima partiu da posio
0
0
= s , deslocou-se at a posio m s 6
1
= e retornou
para a posio final m s 3
2
= . Neste caso, o deslocamento
foi:
m s s s s s 3 0 3
0
= A = A = A
Para determinar a distncia percorrida, deve-se somar os
deslocamentos a favor (
ida
s A ) e contra (
volta
s A ) a
trajetria:
volta ida
s s d A + A =

No exemplo acima, o mvel deslocou-se por 6m a favor e
3m contra a trajetria. Portanto, a distncia percorrida foi
de 9m.

Velocidade Escalar Mdia (V
m
)
o quociente entre a distncia percorrida e o tempo gasto
para percorr-la.
t
d
V
m
A
=

Velocidade Mdia ou Velocidade Vetorial Mdia (
m
V
&
)
o quociente entre o deslocamento e o tempo gasto para
realiz-lo.
t
s
V
m
A
A
=
&
&


*Unidades de Velocidade:

SI CGS Usual
s
m

s
cm

h
km




Acelerao Mdia (a
m
)
o quociente entre a variao de velocidade de um mvel
( v A ) pelo intervalo de tempo correspondente ( t A ).
t
v
a
m
A
A
=
*Unidade de acelerao (SI):
2
s
m



-3 -2 -1 0 1 2 3 4 5 6 ...
(m)
-3 -2 1 0 1 2 3 4 5 6 ...
(m) (m)
-3 -2 -1 0 1 2 3 4 5 6 ...
(m)
s
m

h
km

x 3,6
3,6
Please purchase PDF Split-Merge on www.verypdf.com to remove this watermark.
Fsica A Incluso para a Vida

Pr-Vestibular da UFSC 2
Exerccios de Sala #

1. A respeito dos conceitos de ponto material e corpo
extenso, assinale a alternativa correta:
a) Um ponto material um corpo de tamanho muito
pequeno.
b) Um corpo extenso um corpo de tamanho muito
grande.
c) Ponto material um corpo de massa desprezvel em
comparao com a de um homem.
d) Ponto material um corpo de tamanho e massa
desprezveis em comparao com o tamanho e a massa
de um homem.
e) Quando estudamos o movimento de rotao de um
corpo, ele no pode ser considerado ponto material.

2. (PUC-PR) Um automvel percorre certo trecho com
velocidade escalar mdia de 40 km/h e depois volta pelo
mesmo trecho com velocidade escalar mdia de 60 km/h.
Sua velocidade escalar mdia no trajeto de ida e volta foi,
em km/h, igual a:
a) 48
b) zero
c) 40
d) 50
e) 60

Tarefa Mnima #

3. (UFAL) Uma pessoa percorreu, caminhando a p,
6,0km em 20 minutos. A sua velocidade escalar mdia, em
unidades do Sistema Internacional, foi de:
a) 2,0
b) 4,0
c) 5,0
d) 8,0
e) 10

4. (UFV) Um aluno, sentado na carteira da sala, observa os
colegas, tambm sentados nas respectivas carteiras, bem
como um mosquito que voa perseguindo o professor que
fiscaliza a prova da turma. Das alternativas abaixo, a nica
que retrata uma anlise correta do aluno :
a) A velocidade de todos os meus colegas nula para todo
observador na superfcie da Terra.
b) Eu estou em repouso em relao aos meus colegas, mas
ns estamos em movimento em relao a todo
observador na superfcie da Terra.
c) Como no h repouso absoluto, no h nenhum
referencial em relao ao qual ns, estudantes,
estejamos em repouso.
d) A velocidade do mosquito a mesma, tanto em relao
aos meus colegas, quanto em relao ao professor.
e) Mesmo para o professor, que no para de andar pela
sala, seria possvel achar um referencial em relao ao
qual ele estivesse em repouso.





5. (FEI) Um automvel percorre 300km. Na primeira
metade deste percurso sua velocidade de 75km/h, e na
segunda metade, sua velocidade o dobro da velocidade
na primeira metade. Quanto tempo ele levar para realizar
todo o percurso?
a) 2,5 h
b) 3,0 h
c) 3,5 h
d) 4,0 h
e) 2,0 h

6. (UFRJ) Dois trens, um de carga e outro de passageiros,
movem-se nos mesmos trilhos retilneos, em sentidos
opostos, um aproximando-se do outro, ambos com
movimentos uniformes. O trem de carga, de 50m de
comprimento, tem uma velocidade de mdulo igual a 10
m/s e o de passageiros, uma velocidade de mdulo igual a
v. O trem de carga deve entrar num desvio para que o de
passageiros possa prosseguir viagem nos mesmos trilhos,
como ilustra a figura. No instante focalizado, as distncias
das dianteiras dos trens ao desvio valem 200m e 400m,
respectivamente.

Calcule o valor mximo de v para que no haja coliso.

Tarefa Complementar #

7. (UFPE) A imprensa pernambucana, em reportagem
sobre os riscos que correm os adeptos da "direo
perigosa", observou que uma pessoa leva cerca de 4,0s
para completar uma ligao de um telefone celular ou
colocar um CD no aparelho de som de seu carro. Qual a
distncia percorrida por um carro que se desloca a 72 km/h
durante este intervalo de tempo no qual o motorista no
deu a devida ateno ao trnsito?
a) 40 m c) 80 m e) 97 m
b) 60 m d) 85 m

8. A figura mostra, em determinado instante, dois carros A
e B em movimento retilneo uniforme.

O carro A, com velocidade escalar 20m/s, colide com o B
no cruzamento C. Desprezando as dimenses dos
automveis, a velocidade escalar de B :
a) 12 m/s c) 8 m/s e) 4 m/s
b) 10 m/s d) 6 m/s
Please purchase PDF Split-Merge on www.verypdf.com to remove this watermark.
Incluso para a Vida Fsica A

Pr-Vestibular da UFSC 3
9. (UFSC) Descendo um rio em sua canoa, sem remar,
dois pescadores levam 300 segundos para atingir o seu
ponto de pesca, na mesma margem do rio e em trajetria
retilnea. Partindo da mesma posio e remando, sendo a
velocidade da canoa em relao ao rio igual a 2,0 m/s, eles
atingem o seu ponto de pesca em 100 segundos. Aps a
pescaria, remando contra a correnteza do rio, eles gastam
600 segundos para retornar ao ponto de partida.

Considerando que a velocidade da correnteza V
CR

constante, assinale a(s) proposio(es) correta(s):
01. Quando os pescadores remaram rio acima, a velocidade
da canoa em relao margem foi igual a 4,00 m/s.
02. No possvel calcular a velocidade com que os
pescadores retornaram ao ponto de partida, porque a
velocidade da correnteza no conhecida.
04. Quando os pescadores remaram rio acima, a velocidade
da canoa, em relao ao rio, foi de 1,50m/s.
08. A velocidade da correnteza do rio 1,00m/s.
16. O ponto de pesca fica a 300 metros do ponto de partida.
32. No possvel determinar a distncia do ponto de
partida at o ponto de pesca.
64. Como a velocidade da canoa foi de 2,0 m/s, quando os
pescadores remaram rio abaixo, ento, a distncia do
ponto de partida ao ponto de pesca 200m.


10. (UFSC) Um trem A, de 150 metros de comprimento,
deslocando-se do sul para o norte, comea a atravessar
uma ponte frrea de pista dupla, no mesmo instante em que
um outro trem B, de 500 metros de comprimento, que se
desloca do norte para o sul, inicia a travessia da ponte. O
maquinista do trem A observa que o mesmo se desloca
com velocidade constante de 36 km/h, enquanto o
maquinista do trem B verifica que o seu trem est a uma
velocidade constante de 72 km/h, ambas as velocidades
medidas em relao ao solo. Um observador, situado em
uma das extremidades da ponte, observa que os trens
completam a travessia da ponte ao mesmo tempo.
Assinale a(s) proposio(es) correta(s):
01. Como o trem B tem o dobro da velocidade do trem A,
ele leva a metade do tempo para atravessar a ponte
independentemente do comprimento dela.
02. A velocidade do trem A, em relao ao trem B, de
108 km/h.
04. No podemos calcular o comprimento da ponte, pois
no foi fornecido o tempo gasto pelos trens para
atravess-la.
08. O comprimento da ponte 200 metros.
16. Os trens atravessam a ponte em 35 segundos.
32. A velocidade do trem B, em relao ao trem A, de
108 km/h.
64. O comprimento da ponte 125 metros e os trens a
atravessam em 15 segundos.


UNIDADE 2

MOVIMENTO RETILNEO
UNIFORME - MRU

o movimento em linha reta com velocidade de mdulo
constante.
0 . =
A
A
= = =
t
s
V const v
m


Funo horria das posies:

0
0
0
.
s s s
v v t s s
t t t
A
= = =
A /

0
. s s v t = +


Exerccios de Sala #

1. (FATEC) A tabela fornece, em vrios instantes, a
posio s de um automvel em relao ao km zero da
estrada em que se movimenta.
t (h) 0,0 2,0 4,0 6,0 8,0 10,0
s (km) 200 170 140 110 80 50
A funo horria que nos fornece a posio do automvel,
com as unidades fornecidas, :
a) s = 200 + 30t
b) s = 200 - 30t
c) s = 200 + 15t
d) s = 200 - 15t
e) s = 200 - 15t
2


2. (PUC-PR) Um automvel parte de Curitiba com destino
a Cascavel com velocidade de 60 km/h. 20 minutos depois,
parte outro automvel de Curitiba com o mesmo destino,
velocidade 80 km/h.
Depois de quanto tempo, contado a partir da partida do
mvel A, o 2 automvel alcanar o 1?
a) 60 min c) 80 min e) 56 min
b) 70 min d) 90 min

Tarefa Mnima #

3. (Mack) Uma partcula descreve um movimento retilneo
uniforme, segundo um referencial inercial. A equao
horria da posio, com dados no S.I., x=-2+5t. Neste
caso, podemos afirmar que a velocidade escalar da
partcula :
a) - 2m/s e o movimento retrgrado.
b) - 2m/s e o movimento progressivo.
c) 5m/s e o movimento progressivo
d) 5m/s e o movimento retrgrado.
e) - 2,5m/s e o movimento retrgrado.




Please purchase PDF Split-Merge on www.verypdf.com to remove this watermark.
Fsica A Incluso para a Vida

Pr-Vestibular da UFSC 4
4. (UFRJ) Nas Olimpadas de 2004, em Atenas, o
maratonista brasileiro Vanderlei Cordeiro de Lima liderava
a prova quando foi interceptado por um fantico. A
gravao cronometrada do episdio indica que ele perdeu
20 segundos desde o instante em que foi interceptado at o
instante em que retomou o curso normal da prova.
Suponha que, no momento do incidente, Vanderlei
corresse a 5,0 m/s e que, sem ser interrompido, mantivesse
constante sua velocidade. Calcule a distncia que nosso
atleta teria percorrido durante o tempo perdido.

5. (UNESP) Num caminho-tanque em movimento, uma
torneira mal fechada goteja razo de 2 gotas por segundo.
Determine a velocidade do caminho, sabendo que a
distncia entre marcas sucessivas deixadas pelas gotas no
asfalto de 2,5 metros.

6. (Unitau) Uma motocicleta com velocidade constante de
20m/s ultrapassa um trem de comprimento 100m e
velocidade 15m/s. A durao da ultrapassagem :
a) 5s. c) 20s. e) 30s.
b) 15s. d) 25s.

7. (Unitau) Uma motocicleta com velocidade constante de
20m/s ultrapassa um trem de comprimento 100m e
velocidade 15m/s. O deslocamento da motocicleta durante
a ultrapassagem :
a) 400m. c) 200m. e) 100m.
b) 300m. d) 150m.

Tarefa Complementar #

8. (Mack) Na ltima volta de um grande prmio
automobilstico, os dois primeiros pilotos que finalizaram a
prova descreveram o trecho da reta de chegada com a
mesma velocidade constante de 288 km/h. Sabendo que o
primeiro colocado recebeu a bandeirada final cerca de 2,0s
antes do segundo colocado, a distncia que os separava
neste trecho derradeiro era de:
a) 80 m. c) 160 m. e) 576 m.
b) 144 m. d) 288 m.

9. (PUC-SP) Duas bolas de dimenses desprezveis se
aproximam uma da outra, executando movimentos
retilneos e uniformes (veja a figura). Sabendo-se que as
bolas possuem velocidades de 2m/s e 3m/s e que, no
instante t=0, a distncia entre elas de 15m, podemos
afirmar que o instante da coliso :

a) 1 s
b) 2 s
c) 3 s
d) 4 s
e) 5 s

10. (UFPE) Um automvel faz o percurso Recife-Gravat
a uma velocidade mdia de 50 km/h. O retorno, pela
mesma estrada, realizado a uma velocidade mdia de 80
km/h. Quanto, em percentual, o tempo gasto na ida
superior ao tempo gasto no retorno?




UNIDADE 3

MOVIMENTO RETILNEO
UNIFORMEMENTE VARIADO - MRUV

Um movimento no qual o mvel mantm sua acelerao
escalar constante e no nula, denominado movimento
uniformemente variado. Em conseqncia, a acelerao
escalar instantnea (a) e a acelerao escalar mdia (a
m
)
so iguais.
0 . =
A
A
= = =
t
v
a const a
m


Equao horria das velocidades:
t a v v .
0
+ =

Equao horria das posies:
2
0 0
.
.
2
a t
s s v t = + +

Equao de Torricelli:
2 2
0
2. . v v a s = + A

Exerccios de Sala #

1. (UNESP) Um veculo est rodando velocidade de 36
km/h numa estrada reta e horizontal, quando o motorista
aciona o freio. Supondo que a velocidade do veculo se
reduz uniformemente razo de 4 m/s em cada segundo a
partir do momento em que o freio foi acionado, determine;
a) o tempo decorrido entre o instante do acionamento do
freio e o instante em que o veculo pra.
b) a distncia percorrida pelo veculo nesse intervalo de
tempo.

2. (PUC-Campinas) A funo horria da posio s de um
mvel dada por s=20+4t-3t
2
, com unidades do Sistema
Internacional. Nesse mesmo sistema, a funo horria da
velocidade do mvel ;
a) -16 - 3t c) 4 - 6t e) 4 - 1,5t
b) -6t d) 4 - 3t

Tarefa Mnima #

3. (UERJ) Ao perceber o sinal vermelho, um motorista,
cujo carro trafegava a 80 km/h, pisa no freio e pra em 10
s. A desacelerao mdia do veculo, em km/h
2
, equivale,
aproximadamente a:
a) 1,4 10
3
c) 1,8 10
4
b) 8,0 10
3
d) 2,9 10
4

4. (PUC-RS) Um jogador de tnis recebe uma bola com
velocidade de 20,0m/s e a rebate na mesma direo e em
sentido contrrio com velocidade de 30,0m/s. Se a bola
permanecer 0,100s em contato com a raquete, o mdulo da
sua acelerao mdia ser de:
a) 100m/s
2
c) 300 m/s
2
e) 600 m/s
2

b) 200 m/s
2
d) 500 m/s
2

Please purchase PDF Split-Merge on www.verypdf.com to remove this watermark.
Incluso para a Vida Fsica A

Pr-Vestibular da UFSC 5
5. (UFSCar) Em um piso horizontal um menino d um
empurro em seu caminhozinho de plstico. Assim que o
contato entre o caminhozinho e a mo do menino
desfeito, observa-se que em um tempo de 6 s o brinquedo
foi capaz de percorrer uma distncia de 9 m at cessar o
movimento. Se a resistncia oferecida ao movimento do
caminhozinho se manteve constante, a velocidade inicial
obtida aps o empurro, em m/s, foi de:
a) 1,5. c) 4,5. e) 9,0.
b) 3,0. d) 6,0.

6. (PUC-Rio) Um carro viajando em uma estrada retilnea
e plana com uma velocidade constante V
1
=72km/h passa
por outro que est em repouso no instante t = 0 s. O
segundo carro acelera para alcanar o primeiro com
acelerao a
2
=2,0m/s
2
. O tempo que o segundo carro leva
para atingir a mesma velocidade do primeiro :
a) 1,0 s. c) 5,0 s. e) 20,0 s.
b) 2,0 s. d) 10,0 s.

7. (PUC-PR) Um automvel trafega em uma estrada
retilnea. No instante t = 0s, os freios so acionados,
causando uma acelerao constante at anular a
velocidade, como mostra a figura.
A tabela mostra a velocidade em determinados instantes.

Com base nestas informaes so feitas algumas
afirmativas a respeito do movimento:
I - O automvel apresenta uma acelerao no sentido do
deslocamento.
II - O deslocamento do veculo nos primeiros 2s 34m.
III - A acelerao do veculo -1,5m/s
2
.
IV - A velocidade varia de modo inversamente
proporcional ao tempo decorrido.
V - A velocidade do veculo se anula no instante 7,5s.

Est correta ou esto corretas:
a) somente I. c) somente III. e) II e V.
b) I e II. d) IV e V.

Tarefa Complementar #

8. (PUC-RS) Um "motoboy" muito apressado, deslocando-
se a 30m/s, freou para no colidir com um automvel a sua
frente. Durante a frenagem, sua moto percorreu 30m de
distncia em linha reta, tendo sua velocidade
uniformemente reduzida at parar, sem bater no
automvel. O mdulo da acelerao mdia da moto, em
m/s
2
, enquanto percorria a distncia de 30m, foi de:
a) 10 c) 30 e) 108
b) 15 d) 45

9. (UFSCar) Uma partcula se move em uma reta com
acelerao constante. Sabe-se que no intervalo de tempo de
10s ela passa duas vezes pelo mesmo ponto dessa reta, com
velocidades de mesmo mdulo, v=4,0m/s, em sentidos
opostos. O mdulo do deslocamento e o espao percorrido
pela partcula nesse intervalo de tempo so,
respectivamente:
a) 0,0m e 10m. c) 10m e 5,0m. e) 20m e 20m.
b) 0,0m e 20m. d) 10m e 10m.
10. (UFSC) No momento em que acende a luz verde de
um semforo, uma moto e um carro iniciam seus
movimentos, com aceleraes constantes e de mesma
direo e sentido. A variao de velocidade da moto de
0,5m/s e a do carro de 1,0m/s, em cada segundo, at
atingirem as velocidades de 30m/s e 20m/s,
respectivamente, quando ento seguem o percurso em
movimento retilneo uniforme.
Considerando a situao descrita, assinale a(s)
proposio(es) correta(s).
01. A velocidade mdia da moto, nos primeiros 80s, de
20,5m/s.
02. Aps 60s em movimento, o carro est 200m frente da
moto.
04. A moto ultrapassa o carro a 1200 m do semforo.
08. A ultrapassagem do carro pela moto ocorre 75s aps
ambos arrancarem no semforo.
16. O movimento da moto acelerado e o do carro
retilneo uniforme, 50s aps iniciarem seus
movimentos.
32. 40 s aps o incio de seus movimentos, o carro e a moto
tm a mesma velocidade.



UNIDADE 4
GRFICOS CINEMTICOS

MOVIMENTO UNIFORME (MU)

Posio X tempo tg u = |v|

Mov. Progressivo (v > 0)


Mov. Retrgrado (v < 0)





s
0
u
t
s
u
t
s
Please purchase PDF Split-Merge on www.verypdf.com to remove this watermark.
Fsica A Incluso para a Vida

Pr-Vestibular da UFSC 6
Velocidade X tempo rea = |s|

Mov. Progressivo (v > 0)




Mov. Retrgrado (v < 0)




MOVIMENTO UNIFORMEMENTE VARIADO
(MUV)


Posio X tempo
Razes: instantes nos quais o mvel passa pela origem
Vrtice: mudana de sentido (v = 0)








Velocidade X tempo rea = |s|
tg u = |a|





Acelerao X tempo rea = |v|




Exerccios de Sala #

1. (Mack) Um mvel se desloca sobre uma reta conforme
o diagrama a seguir. O instante em que a posio do mvel
de +20m :


a) 6 s
b) 8 s
c) 10 s
d) 12 s
e) 14 s
rea
v
u
t
v

rea
t
v

rea
t
v
s
s
0

t
t
s
0

s

rea
v
u
t
v

rea
t
a

rea
t
a
a > 0
a < 0
a > 0
a < 0
a > 0
a < 0
Please purchase PDF Split-Merge on www.verypdf.com to remove this watermark.
Incluso para a Vida Fsica A

Pr-Vestibular da UFSC 7
2. (UFPE) O grfico a seguir mostra a velocidade de um
objeto em funo do tempo, em movimento ao longo do
eixo x. Sabendo-se que, no instante t = 0, a posio do
objeto x = - 10 m, determine a equao x(t) para a posio
do objeto em funo do tempo.

a) x(t) = -10 + 20t - 0,5t
2
b) x(t) = -10 + 20t + 0,5t
2
c) x(t) = -10 + 20t - 5t
2
d) x(t) = -10 - 20t + 5t
2
e) x(t) = -10 - 20t - 0,5t
2

Tarefa Mnima #

3. Duas partculas A e B se movem numa mesma trajetria
e o grfico a seguir indica suas posies (s) em funo do
tempo (t). Pelo grfico podemos afirmar que as partculas:

a) Movem-se no mesmo sentido.
b) Movem-se em sentidos opostos.
c) No instante t=0, encontram-se a 40m uma da outra.
d) Movem-se com a mesma velocidade.
e) No se encontram.

4. (PUC-Campinas) Um caminho C de 25m de
comprimento e um automvel A de 5,0m de comprimento
esto em movimento em uma estrada. As posies dos
mveis, marcadas pelo para-choque dianteiro dos veculos,
esto indicadas no grfico para um trecho do movimento.
Em determinado intervalo de tempo o automvel
ultrapassa o caminho.

Durante a ultrapassagem completa do caminho, o
automvel percorre uma distncia, em metros, igual a
a) 5 c) 18 e) 60
b) 15 d) 20



5. (Unifesp) Em um teste, um automvel colocado em
movimento retilneo uniformemente acelerado a partir do
repouso at atingir a velocidade mxima. Um tcnico
constri o grfico onde se registra a posio x do veculo
em funo de sua velocidade v. Atravs desse grfico,
pode-se afirmar que a acelerao do veculo :


a) 1,5 m/s
2

b) 2,0 m/s
2
.
c) 2,5 m/s
2
.
d) 3,0 m/s
2
.
e) 3,5 m/s
2
.

6. (PUC-SP) O grfico representa a variao da velocidade
com o tempo de um mvel em movimento retilneo
uniformemente variado.

A velocidade inicial do mvel e o seu deslocamento
escalar de 0 a 5,0 s valem, respectivamente:
a) - 4,0 m/s e - 5,0 m d) - 4,0 m/s e 5,0 m
b) - 6,0 m/s e - 5,0 m e) - 6,0 m/s e 25 m
c) 4,0 m/s e 25 m

7. (FUVEST) Dois veculos A e B se deslocam em
trajetrias retilneas e paralelas uma outra. No instante
t=0s eles se encontram lado a lado. O grfico adiante
representa as velocidades dos dois veculos, em funo do
tempo, a partir desse instante e durante os 1200s seguintes.
Os dois veculos estaro novamente lado a lado, pela
primeira vez, no instante:

a) 400 s.
b) 500 s.
c) 600 s.
d) 800 s.
e) 1200 s.

Tarefa Complementar #

8. (UNESP) O grfico na figura descreve o movimento de
um caminho de coleta de lixo em uma rua reta e plana,
durante 15s de trabalho.

a) Calcule a distncia total percorrida neste intervalo de
tempo.
b) Calcule a velocidade mdia do veculo.

Please purchase PDF Split-Merge on www.verypdf.com to remove this watermark.
Fsica A Incluso para a Vida

Pr-Vestibular da UFSC 8
9. (Fatec) Um objeto se desloca em uma trajetria
retilnea. O grfico a seguir descreve as posies do objeto
em funo do tempo.

Analise as seguintes afirmaes a respeito desse
movimento:
I - Entre t = 0s e t = 4s o objeto executou um movimento
retilneo uniformemente acelerado.
II - Entre t = 4s e t = 6s o objeto se deslocou 50m.
III - Entre t = 4s e t = 9s o objeto se deslocou com uma
velocidade mdia de 2m/s.

Deve-se afirmar que apenas:
a) I correta.
b) II correta.
c) III correta.
d) I e II so corretas.
e) II e III so corretas.

10. (UFSC) Um ratinho se afasta de sua toca em busca de
alimento, percorrendo uma trajetria retilnea. No instante
t=11s, um gato pula sobre o caminho do ratinho e ambos
disparam a correr: o ratinho retornando sobre a mesma
trajetria em busca da segurana da toca e o gato atrs do
ratinho. O grfico da figura representa as posies do
ratinho e do gato, em funo do tempo, considerando que
no instante t=0 o ratinho partiu da posio d=0, isto , da
sua toca.

Assinale a(s) proposio(es) correta(s) sobre o
movimento do ratinho e do gato:
01. O ratinho chega 1,0 segundo antes do gato que,
portanto, no consegue alcan-lo.
02. O ratinho se deslocou com velocidade constante entre
os instantes t=5,0s e t=7,0s.
04. O movimento do ratinho foi sempre retilneo e
uniforme, tanto na ida como na volta.
08. O gato se encontrava a 5,0 metros do ratinho quando
comeou a persegui-lo.
16. O ratinho parou duas vezes no seu trajeto de ida e de
volta at a toca.
32. O gato percorre uma distncia maior que a do ratinho,
em menor tempo, por isso o alcana antes que ele possa
chegar toca.

UNIDADE 5
QUEDA LIVRE E LANAMENTO
VERTICAL
Consideraes:
1) Como a acelerao da gravidade nas proximidades da
Terra considerada constante, nosso movimento ser
uniformemente variado (MUV).
2) Em um mesmo lugar da Terra todos os corpos caem
livremente com a mesma acelerao, independentemente
do seu peso, forma ou tamanho. Isto , naquele lugar da
Terra o valor de g o mesmo para qualquer corpo em
queda livre.
3) Quando lanamos um corpo verticalmente para cima,
quando este alcanar a altura mxima sua velocidade ser
nula (V = 0).

Queda Livre


h g v
t g v
t g
H
A =
=
=
. . 2
.
2
.
2
2


Lanamento vertical (para baixo):
No lanamento para baixo, o movimento semelhante
queda livre, porm a velocidade inicial no nula (v
0
0).
2
.
.
2
0
t g
t v H + =
t g v v .
0
+ = h g v v A + = . . 2
2
0
2

Lanamento vertical (para cima):

h g v v
t g v v
t g
t v H
A =
=
=
. . 2
.
2
.
.
2
0
2
0
2
0


Exerccios de Sala #

1. Querendo determinar a altura de um edifcio, um
estudante deixou cair uma pedra do terrao e ela levou 3s
para chegar ao cho. (g=10m/s
2
)
a) Qual a altura que ele obteve para o edifcio?
b) Qual a velocidade da pedra ao chegar ao cho?

2. Uma bola lanada para cima com velocidade de 20m/s
(g = 10m/s
2
). Indique a afirmativa errada (despreze a
resistncia do ar):
a) a bola atinge uma altura de 20 m.
b) no ponto mais alto a velocidade da bola nula.
c) no ponto mais alto a acelerao da bola nula.
d) a bola retorna ao ponto de partida com velocidade de 20
m/s.
e) a bola volta ao ponto de partida depois de 4s.
V = 0
g
&

0
v
&

V
0
= 0
g
&

Please purchase PDF Split-Merge on www.verypdf.com to remove this watermark.
Incluso para a Vida Fsica A

Pr-Vestibular da UFSC 9
Tarefa Mnima #

3. Foi veiculada na televiso uma propaganda de uma
marca de biscoitos com a seguinte cena: um jovem casal
estava num mirante sobre um rio e algum deixava cair l
de cima um biscoito. Passados alguns instantes, o rapaz se
atira do mesmo lugar de onde caiu o biscoito e consegue
agarr-lo no ar. Em ambos os casos, a queda livre, as
velocidades iniciais so nulas, a altura de queda a mesma
e a resistncia do ar nula. Para Galileu Galilei, a situao
fsica desse comercial seria interpretada como:
a) impossvel, porque a altura da queda no era grande o
suficiente.
b) possvel, porque o corpo mais pesado cai com maior
velocidade.
c) possvel, porque o tempo de queda de cada corpo
depende de sua forma.
d) impossvel, porque a acelerao da gravidade no
depende da massa dos corpos.
e) impossvel, porque o corpo mais pesado cai mais
devagar devido resistncia do ar.

4. (UFPE) Uma esfera de ao de 300g e uma esfera de
plstico de 60g, de mesmo dimetro, so abandonadas
simultaneamente do alto de uma torre de 60m de altura.
Qual a razo entre os tempos que levaro as esferas at
atingirem o solo? (Despreze a resistncia do ar).
a) 5,0 c) 1,0 e) 0,2
b) 3,0 d) 0,5

5. Um paraquedista, quando a 120m do solo, deixa cair
uma bomba. Esta leva 4s para atingir o solo. Qual a
velocidade de descida do paraquedista? ( g = 10m/s
2
) .
a) 1 m/s c) 5 m/s e) 10 m/s
b) 2 m/s d) 8 m/s

6. (UNESP) Para deslocar tijolos comum vermos em
obras de construo civil um operrio no solo, lanando
tijolos para outro que se encontra postado no piso superior.
Considerando o lanamento vertical, a resistncia do ar
nula, a acelerao da gravidade igual a 10m/s
2
e a distncia
entre a mo do lanador e a do receptor 3,2m, a velocidade
com que cada tijolo deve ser lanado para que chegue s
mos do receptor com velocidade nula deve ser de:
a) 5,2 m/s. c) 7,2 m/s. e) 9,0 m/s.
b) 6,0 m/s. d) 8,0 m/s.

7. (UFRJ) De um ponto
localizado a uma altura h do
solo, lana-se uma pedra
verticalmente para cima. A
figura a seguir representa, em
grfico cartesiano, como a
velocidade escalar da pedra
varia em funo do tempo,
entre o instante do lanamento
(t = 0) e o instante em que chega ao solo (t = 3s).
a) Em que instante a pedra retoma ao ponto de partida?
Justifique sua resposta.
b) Calcule de que altura h a pedra foi lanada.




Tarefa Complementar #

8. (PUC-PR) Em um planeta isento de atmosfera e onde a
acelerao gravitacional em suas proximidades pode ser
considerada constante igual a 5m/s
2
, um pequeno objeto
abandonado em queda livre de determinada altura,
atingindo o solo aps 8 segundos. Com essas informaes,
analise as afirmaes:

I - A cada segundo que passa, a velocidade do objeto
aumenta em 5m/s durante a queda.
II - A cada segundo que passa, o deslocamento vertical do
objeto igual a 5 metros.
III - A cada segundo que passa, a acelerao do objeto
aumenta em 4m/s
2
durante a queda.
IV - A velocidade do objeto ao atingir o solo igual a 40
m/s.
a) Somente a afirmao I est correta.
b) Somente as afirmaes I e II esto corretas.
c) Todas esto corretas.
d) Somente as afirmaes I e IV esto corretas.
e) Somente as afirmaes II e III esto corretas.

9. (Cesgranrio) O Beach Park, localizado em Fortaleza-
CE, o maior parque aqutico da Amrica Latina situado
na beira do mar. Uma de suas principais atraes um
tobogua chamado "Insano". Descendo esse tobogua, uma
pessoa atinge sua parte mais baixa com velocidade de 28
m/s. Considerando a acelerao da gravidade g = 9,8m/s
2
e
desprezando os atritos, conclui-se que a altura do
tobogua, em metros, de:
a) 40,0 c) 36,8 e) 28,0
b) 38,0 d) 32,4

10. (UFSC) Uma pequena bola
lanada verticalmente para
cima, sob a ao somente da
fora peso, em um local onde a
acelerao da gravidade igual
a 10 m/s
2
. O grfico representa
a posio da bola em funo do
tempo.
Assinale a(s) proposio(es) correta(s).

01. No instante 2,0s a bola atingiu a altura mxima e a
acelerao atuante sobre ela nula.
02. No instante 2,0s a velocidade da bola nula, mas a
acelerao e a fora resultante que atua sobre ela
apresentam valores diferentes de zero.
04. A velocidade inicial da bola igual a 20 m/s.
08. A fora resultante e a acelerao permanecem
invariveis durante todo o movimento.
16. No instante 2,0s a velocidade da bola e a fora
resultante sobre ela so nulas.
32. O movimento pode ser descrito pela funo
d = 20t - 5t
2
.
64. A acelerao varivel e atinge o seu valor mximo no
instante t = 4,0s.






Please purchase PDF Split-Merge on www.verypdf.com to remove this watermark.
Fsica A Incluso para a Vida

Pr-Vestibular da UFSC 10
UNIDADE 6

LANAMENTO HORIZONTAL

Equaes do Lanamento Horizontal
Na Vertical Na Horizontal
0
0
=
y
v g a
y
=
2
.
2
1
t g h =
t g v
y
. =
h g v
y
A = . . 2
2



0 0
v v
x
= (constante)
0 =
x
a
t v D .
0
=




















LANAMENTO OBLQUO
Assim como no lanamento horizontal, o lanamento
oblquo o movimento descrito pela soma de dois
movimentos, um na direo vertical e outro na direo
horizontal. Desprezando a resistncia do ar, o movimento
na vertical um lanamento vertical e, na direo
horizontal, um movimento retilneo uniforme.
A rigor, no h diferena entre o lanamento
horizontal e o lanamento oblquo; o que muda so apenas
as condies iniciais, que agora dependem do ngulo de
inclinao da velocidade inicial em relao horizontal.
Em ambos os casos os projteis descrevem trajetrias
parablicas.















Equaes do Lanamento oblquo
Na Vertical Na Horizontal
u sen v v
y
.
0 0
=
g a
y
= (negativa)
2
0
.
2
1
t g t v h
y
=
t g v v
y y
.
0
=
h g v v
y y
A = . . 2
2
0
2



u cos .
0 0
v v
x
= (constante)
0 =
x
a
t v D
x
.
0
=


Exerccios de Sala #

1. A figura mostra a trajetria de um projtil disparado
horizontalmente de um canho. Despreze os atritos com o
ar e adote g = 10 m/s
2
. Calcule:

245m
2800m
a) tempo de queda do
projtil (t).

b) a intensidade da
velocidade com que o
projtil abandona o
canho.


2. (UFSC) Uma jogadora de basquete joga uma bola com
velocidade de mdulo 8 m/s, formando um ngulo de 60
com a horizontal, para cima. O arremesso to perfeito
que a atleta faz a cesta sem que a bola toque no aro.
Desprezando a resistncia do ar, assinale a(s)
proposio(es) verdadeira(s):

01. O tempo gasto pela bola para alcanar o ponto mais
alto da sua trajetria de 0,5s.
02. O mdulo da velocidade da bola, no ponto mais alto
da sua trajetria, igual a 4m/s.
04. A acelerao da bola constante em mdulo, direo e
sentido desde o lanamento at a bola atingir a cesta.
08. A altura que a bola atinge acima do ponto de
lanamento de 1,8 m.
16. A trajetria descrita pela bola desde o lanamento at
atingir a cesta uma parbola.

3. (ITA) Um avio est a 8,0km de altura e voa
horizontalmente a 700km/h, patrulhando as costas
brasileiras. Em dado instante, ele observa um submarino
inimigo parado na superfcie. Desprezando as foras de
resistncia do ar e adotando g = 10m/s
2
podemos afirmar
que o tempo de que dispe o submarino para se deslocar
aps o avio ter soltado uma bomba de:
a) 108 s. b) 20 s
c) 30 s. d) 40 s
e) No possvel determin-lo se no for conhecida a
distncia inicial entre o avio e o submarino.




y
v
&
x
v
&
x
v v
& &
=
0

y
v
&
x
v
&
v
&
v
&
0
0
&
&
=
y
v
D
(Alcance)
H
ox
v
&

oy
v
&

o
v
&

0
&
&
=
y
v

mnima
v
&

D
Hmxi
y
v
&

x
v
&

v
&

u
Please purchase PDF Split-Merge on www.verypdf.com to remove this watermark.
Incluso para a Vida Fsica A

Pr-Vestibular da UFSC 11
Au
Ax
C
R
Tarefa Mnima #

4. (Cesgranrio) Para bombardear um alvo, um avio em
voo horizontal, a uma altitude de 2,0km, solta uma bomba
quando a sua distncia horizontal at o alvo de 4,0km.
Admite-se que a resistncia do ar seja desprezvel. Para
atingir o mesmo alvo, se o avio voasse com a mesma
velocidade, mas agora a uma altitude de apenas 0,50km,
ele teria que soltar a bomba a uma distncia horizontal do
alvo igual a:
a) 0,25 km. c) 1,0 km. e) 2,0 km.
b) 0,50 km. d) 1,5 km.

5. (Fei) Uma esfera de ao de massa 200g desliza sobre
uma mesa plana com velocidade igual a 2m/s. A mesa est
a 1,8m do solo. A que distncia da mesa a esfera ir tocar o
solo? Obs: despreze o atrito. Considere g = 10m/s
2

a) 1,25m c) 0,75m e) 1,2m
b) 0,5m d)1,0m

Tarefa Complementar #

6. (PUC-CAMP) Um projtil lanado segundo um
ngulo de 30 com a horizontal, com uma velocidade de
200m/s. Supondo a acelerao da gravidade igual a 10m/s
2

e desprezando a resistncia do ar, o intervalo de tempo
entre as passagens do projtil pelos pontos de altura 480m
acima do ponto de lanamento, em segundos, : (DADOS:
sen 30 = 0,50; cos 30 = 0,87)
a) 2,0 c) 6,0 e) 12
b) 4,0 d) 8.0

7. (PUC-SP) Suponha que em uma partida de futebol, o
goleiro, ao bater o tiro de meta, chuta a bola, imprimindo-
lhe uma velocidade
0
v
&
cujo vetor forma, com a horizontal,
um ngulo o. Desprezando a resistncia do ar, so feitas as
afirmaes abaixo.

I - No ponto mais alto da trajetria, a velocidade vetorial
da bola nula.
II - A velocidade inicial
0
v
&
pode ser decomposta segundo
as direes horizontal e vertical.
III - No ponto mais alto da trajetria, nulo o valor da
acelerao da gravidade.
IV - No ponto mais alto da trajetria, nulo o valor da
componente vertical da velocidade.

Esto corretas:
a) I, II e III c) II e IV e) I e II
b) I, III e IV d) III e IV


UNIDADE 7
MOVIMENTO CIRCULAR UNIFORME
MCU

COMPONENTES DA ACELERAO
ACELERAO CENTRPETA:








T
a
&
: varia o mdulo do vetor velocidade.
C
a
&
: varia a direo do vetor velocidade.

R T R
a a a
& & &
+ =
2 2 2
R T R
a a a + =

Para calcular o mdulo da acelerao centrpeta,
utilizaremos a seguinte frmula:
R
v
a
C
2
=
onde R o raio da trajetria.

MOVIMENTO CIRCULAR UNIFORME
Um objeto realiza um movimento circular uniforme (que
passaremos a abreviar MCU) quando o movimento se
realizar sobre uma circunferncia (a
C
= 0) e o mdulo da
velocidade do objeto no variar (a
T
= 0).

PERODO E FREQUNCIA
Perodo (T): tempo necessrio para o mvel completar
uma volta
Frequncia (f): nmero de voltas que o mvel realiza em
uma unidade de tempo
tempo
voltas de n
= f

Comparando o nmero de voltas com o tempo, temos:

N de voltas Tempo

f
T
1
=

1 T
f 1

No SI, a unidade de perodo o segundo (s) e de
frequncia o hertz (Hz) ou rotaes por segundo (rps).

DESLOCAMENTO ANGULAR
Num MCU, o deslocamento angular corresponde ao
ngulo varrido pelo mvel quando realiza um
deslocamento. Veja na figura:





A medida do deslocamento
angular dada por:

R
x A
= Au
[rad]
tangente
T
a
&
C
a
&
R
a
&
Please purchase PDF Split-Merge on www.verypdf.com to remove this watermark.
Fsica A Incluso para a Vida

Pr-Vestibular da UFSC 12
VELOCIDADE ANGULAR MDIA

Corresponde ao ngulo descrito na unidade de tempo.
t A
A
=
u
e
[rad/s]
Para 1 volta completa, temos t u . 2 = A e T t = A :
T
t
e
. 2
=

Como
f
T
=
1
, temos que:
f . . 2t e =

RELAO ENTRE VELOCIDADE ESCALAR E
VELOCIDADE ANGULAR

Para 1 volta completa, temos R x . . 2t = A e T t = A :
f R
T
R
t
x
V . . . 2
. . 2
t
t
= =
A
A
=

Como
e
t
t = =
T
f
. 2
. . 2
, R V . e =

TRANSMISSO DE MOVIMENTO CIRCULAR

Um movimento circular pode ser transmitido de uma roda
(polia) para outra atravs de dois procedimentos bsicos:
ligao das polias por uma correia ou corrente ou pelo
contato entre elas (Ex.: engrenagens). Veja as figuras:








B B A A
B B A A
B A
R f R f
R R
V V
. .
. .
=
=
=
e e


Exerccios de Sala #

1.(UFSC) Obtenha a soma dos valores numricos
associados s opes corretas:
Em relao a um corpo que executa um movimento
circular uniforme, podemos dizer que:

01. Por existir uma variao na direo do vetor
velocidade, o corpo possuir uma acelerao centrpeta.
02. A acelerao centrpeta um vetor perpendicular
velocidade e dirigida para o centro da trajetria.
04. O vetor velocidade tem mdulo constante, mas a sua
direo varia continuamente.
08. A acelerao centrpeta inversamente proporcional ao
quadrado do raio da circunferncia.
16. O tempo gasto para efetuar uma volta completa
denominado frequncia (em Hz) do movimento.
32. A velocidade angular ser dada por 2t dividido por T
(perodo) e se refere ao ngulo descrito na unidade de
tempo.
2. A figura abaixo mostra uma bicicleta em movimento
retilneo e uniforme, cuja roda maior tem raio de 0,5m e a
menor 0,25m. A roda menor gira com frequncia de 4,0
Hz.

Determine:
a) a frequncia da roda maior.
b) a velocidade escalar da bicicleta.

Tarefa Mnima #

3.(EUM) Qual das seguintes propriedades caracteriza o
movimento de um satlite artificial em torno da Terra,
admitindo que o movimento seja circular uniforme?
a) Velocidade constante em mdulo e direo.
b) Acelerao constante, paralela ao vetor velocidade.
c) Acelerao radial constante em mdulo.
d) Acelerao constante com um componente paralelo ao
vetor velocidade e o outro perpendicular a ela.
e) Acelerao nula.

4. (UCS) Para calcular a velocidade angular de uma
partcula que descreve um movimento circular uniforme,
basta conhecer:
a) a acelerao centrpeta.
b) o perodo de revoluo.
c) a velocidade escalar linear.
d) o raio do crculo descrito.
e) o dimetro do crculo descrito.

5. (FCC) Uma partcula executa um movimento uniforme
sobre uma circunferncia de raio 20cm. Ela percorre
metade da circunferncia em 2,0s. A frequncia, em Hz, e
o perodo, em s, valem, respectivamente:
a) 4,0 e 0,25 c) 1,0 e 1,0 e) 0,25 e 4,0
b) 2,0 e 0,50 d) 0,50 e 2,0

6.(FEI) Determine a velocidade angular do ponteiro dos
segundos de um relgio analgico.
a) 60 rad/s c) 30t rad/s e) t/30 rad/s
b) 60t rad/s d) t/60 rad/s

Tarefa Complementar #

7. (UFMA) Num movimento circular uniforme,
quadruplicando o raio e dobrando a velocidade, o mdulo
da acelerao centrpeta:
a) metade da anterior.
b) No se altera.
c) o dobro da anterior.
d) a quarta parte da anterior.

8. (UECE) A figura mostra um disco que gira em torno do
centro O. A velocidade do ponto X 50cm/s e a do ponto
Y de 10cm/s.

A distncia XY vale
20cm. Pode-se afirmar
que o valor da velocidade
angular do disco, em
radianos por segundo, :
a) 2,0 c) 10,0
b) 5,0 d) 20,0
A B
A B
RA
RA
RB
RB
Please purchase PDF Split-Merge on www.verypdf.com to remove this watermark.
Incluso para a Vida Fsica A

Pr-Vestibular da UFSC 13
UNIDADE 8
DINMICA
Dinmica a parte da Mecnica que estuda os movimentos
dos corpos, analisando as causas que explicam como um
corpo em repouso pode entrar em movimento, como
possvel modificar o movimento de um corpo ou como um
corpo em movimento pode ser levado ao repouso. Essas
causas so, como veremos, as foras.
FORA

uma interao entre dois corpos. a causa da
acelerao de um corpo. Sem ela, no possvel alterar a
velocidade de um objeto.
A fora tem intensidade, direo e sentido, ou seja,
ela uma grandeza vetorial.
Quanto sua natureza, uma fora pode ser de
contato (por exemplo, a fora feita por uma criana para
puxar um carrinho de brinquedo atravs de um barbante)
ou de campo, quando pode existir fora mesmo a distncia,
sem que haja contato entre os corpos (foras gravitacional,
eltrica e magntica).

1
a
Lei de Newton ou Princpio da Inrcia
Esta lei explica os estados de movimento dos objetos para
os quais a fora resultante zero.
Quando a fora resultante que atua em um objeto nula
(F
R
= 0), dizemos que este objeto se encontra em
equilbrio.

=
(MRU) dinmico equilbrio
(repouso) esttico equilbrio
0
& &
R
F

2
a
Lei de Newton ou Princpio Fundamental da
Dinmica
Quando a fora resultante que atua em um determinado
objeto for diferente de zero, este objeto estar sujeito a
uma acelerao que diretamente proporcional fora
resultante. A resultante
R
F
&
das foras que atuam em um
corpo de massa m produz uma acelerao a
&
tal que:
a m F
R
&
&
. =
R
F
&
e a
&
so vetores que possuem a mesma direo, o
mesmo sentido e intensidade proporcionais.
No SI, a unidade de fora o Newton (N).

Fora Peso: a fora de atrao que a Terra exerce nos
corpos.
Quando um corpo est em movimento, sob ao exclusiva
de seu peso, ele adquire uma acelerao chamada
acelerao da gravidade.
De acordo com a 2 Lei de Newton: P = mg

3
a
Lei de Newton ou Princpio da Ao e Reao
As foras sempre existem aos pares.
Quando um corpo A aplica uma fora
A
F
&
num corpo B,
este aplica em A uma fora
B
F
&
. As foras (
A
F
&
e
B
F
&
) tm a
mesma intensidade, a mesma direo e sentidos opostos.
Uma das foras chamada de Ao e a outra de Reao.


Exerccios de Sala #

1. (ACAFE) Um carro segue por uma estrada com vrias
malas sobre o seu teto. Numa curva fechada para a
esquerda, uma das malas que no estava bem presa
atirada para a direita do motorista. Tal fato explicado:
a) Pela lei da gravidade.
b) Pela conservao da energia.
c) Pelo princpio da inrcia.
d) Pelo princpio da ao e reao.
e) Pelo princpio de Pascal.

2. (UFSC) A figura abaixo mostra o bloco A de 6kg em
contato com o bloco B de 4kg, ambos em movimento
sobre uma superfcie horizontal sem atrito, sob a ao da
fora horizontal F
&
, de mdulo 50N. O mdulo, em
newtons, da resultante das
foras que atuam sobre o
bloco A :

3. (UFMG) Um homem que pesa 80 kgf est sobre uma
balana de mola dentro de um elevador em movimento
vertical. Se o elevador est descendo, a balana acusa um
valor maior ou menor do que 80kgf? Justifique sua
resposta.

Tarefa Mnima #

4. (FCMSCSP) No necessria a existncia de uma fora
resultante atuando:
a) Quando se passa do estado de repouso ao de movimento
uniforme.
b) Para manter o corpo em movimento retilneo e uniforme.
c) Para manter um corpo em movimento circular e
uniforme.
d) Para mudar a direo de um objeto sem alterar o mdulo
de sua velocidade.
e) Em nenhum dos casos anteriores.

5. (FUVEST) Adote: g = 10 m/s
2
Um homem tenta levantar uma caixa de 5kg que est sobre
uma mesa aplicando uma fora vertical de 10N. Nesta
situao, o valor da fora que a mesa aplica na caixa :

a) 0N
b) 5N
c) 10N
d) 40N
e) 50N

6.(UNIMEP) Um corpo A de massa m
A
= 1,6kg est unido
por um fio a um outro B de massa m
B
= 0,40kg. No
instante inicial, o corpo A tinha uma velocidade de mdulo
5,0 m/s e se movia para a direita, conforme sugere a figura
abaixo. Desprezando os atritos, aps 5s, qual o mdulo e o
sentido da velocidade do corpo A?

A
B g
&
V0 = 5,0 m/s
Please purchase PDF Split-Merge on www.verypdf.com to remove this watermark.
Fsica A Incluso para a Vida

Pr-Vestibular da UFSC 14
7.(UFRGS) Um elevador comea a subir, a partir do andar
trreo, com acelerao de mdulo 5,0 m/s
2
. O peso
aparente de um homem de 60kg no interior do elevador,
supondo g = 10 m/s
2
, igual a:
a) 60 N c) 300 N e) 900 N
b) 200 N d) 600 N

8. No esquema, desprezam-se todos os atritos e a inrcia
da polia. O fio suposto ideal, isto , sem peso e
inextensvel. Os blocos
A, B e C tm massas
iguais a m e a acelerao
da gravidade vale g.
Determine a intensidade
da fora que A exerce
em B.
Aplicao numrica:
m = 3,0 kg e g = 10 m/s
2



9.(FCMSCSP) Uma balana de mola colocada em um
elevador que est descendo com movimento retardado e
acelerao de mdulo igual a 0,2g, no qual g o mdulo
da acelerao da gravidade local. Uma pessoa de massa 70
kg est sobre a balana. Sendo g = 10m/s
2
, a balana
indicar:
a) 70 N c) 140 N e) 210 N
b) 700 N d) 840 N

Tarefa Complementar #

10. (UFSC) A figura representa um automvel A,
rebocando um trailer B, em uma estrada plana e
horizontal. A massa do automvel e a massa do trailer so,
respectivamente, iguais a 1.500kg e 500 kg. Inicialmente, o
conjunto parte do repouso, atingindo a velocidade de 90
km/h em 20 segundos. Desprezam-se os efeitos da fora de
resistncia do ar sobre o veculo e o reboque.

Em relao situao descrita, assinale a(s)
proposio(es) correta(s).

01. A intensidade da fora transmitida ao trailer a mesma
da fora resultante sobre o conjunto.
02. At atingirem a velocidade de 90 km/h, o automvel e
seu reboque tero percorrido 250m.
04. O trailer exerce uma fora de 625N sobre o automvel.
08. A fora resultante sobre o conjunto igual a 2500N.
16. A acelerao do conjunto igual a 1,25m/s
2
.
32. No havendo nenhuma fora que se oponha ao
movimento do trailer, o automvel no necessita fazer
nenhuma fora adicional para aceler-lo.
64. A fora que o automvel faz sobre o trailer no pode
ter a mesma intensidade da fora que o trailer faz sobre
o automvel porque, neste caso, o sistema
permaneceria em repouso.



UNIDADE 9
ATRITO E PLANO INCLINADO

ATRITO

Considere um corpo de peso P em repouso sobre uma
superfcie horizontal. Vamos aplicar ao corpo uma fora
F
&
que tende a desloc-lo na direo horizontal. As
superfcies em contato apresentam rugosidades que se
opem ao deslocamento do corpo.
F
Rugosidades
F
AT

Esta fora que aparece no sentido contrrio ao movimento
ou tendncia de movimento do corpo em relao
superfcie denominada fora de atrito (
AT
F
&
).
O Atrito Esttico atua sobre corpos em repouso
sujeitos a uma fora no suficiente para coloc-los em
movimento. Como o corpo permanece em repouso, de
acordo com a Primeira Lei de Newton, a resultante das
foras que nele atuam igual a zero. Nesse caso, a fora de
atrito esttico sempre ser igual fora motriz.





F F
AT
& &
=

Fora de Destaque o mximo valor suportado
pelo atrito esttico. Se a fora motriz for maior que a fora
de destaque, o corpo entra em movimento e o atrito deixa
de ser esttico. Portanto, enquanto o corpo est em
repouso, a fora de atrito esttico tem o mesmo valor da
fora motriz e no pode superar a fora de destaque, logo:
destaque ATest
F F < < 0
N F
e destaque
. =

e
= coeficiente de atrito esttico
N = Fora de reao normal superfcie.

O Atrito Cintico ou Dinmico ocorre quando o
corpo se encontra em movimento e constante,
independente de sua velocidade ou tipo de movimento.
N F
d ATdin
. =

d
= coeficiente de atrito dinmico

Ateno!
mais fcil manter um corpo em movimento do que
inici-lo. Por qu? Porque o coeficiente de atrito esttico
maior do que o dinmico.

d e
>


A
B
C
V = 0
AT
F
&

F
&

Please purchase PDF Split-Merge on www.verypdf.com to remove this watermark.
Incluso para a Vida Fsica A

Pr-Vestibular da UFSC 15
PLANO INCLINADO
Considere um corpo deslizando num plano inclinado, sem
atrito, e formando um ngulo u com a horizontal.
Sobre o corpo atuam as foras peso P e a reao
normal N. comum decompor o peso P em duas foras
componentes:
PN: normal ao plano inclinado e equilibrada pela reao
normal N;
PT: paralela ao plano inclinado.











u u
u u
cos . cos
sen . sen
P P
P
P
P P
P
P
N
N
T
T
= =
= =

Exerccios de Sala #

1. Um bloco de massa m = 1,0kg est em repouso sobre
um plano horizontal no qual existe atrito. Sabe-se que o
coeficiente de atrito esttico entre o bloco e o plano vale
0,5, e o coeficiente de atrito dinmico vale 0,4. Adote g =
10m/s
2
. Aplica-se sobre o bloco uma fora horizontal
constante de intensidade F.




Para cada valor de F na tabela a seguir, marque:
a) o valor da intensidade da fora de atrito;
b) o tipo de atrito: esttico (E) ou dinmico (D);
c) o mdulo da acelerao do bloco.

F F
AT
Tipo a
2,0
4,0
5,0
6,0
7,0
Obs: as unidades esto no SI.

2. (MACK) A figura mostra um corpo de massa 50kg
sobre um plano inclinado sem atrito, que forma um ngulo
u com a horizontal. A intensidade da fora F
&
que far o
corpo subir o plano inclinado com acelerao constante de
2,0 m/s
2
:
Dados: g = 10 m/s
2
; sen u = 0,6
a) 400 N
b) 300 N
c) 200 N
d) 100 N
e) 50N



Tarefa Mnima #

3. (UDESC) Uma fora horizontal F
&
comprime um bloco
de peso 10N contra uma parede vertical.
O coeficiente de atrito esttico entre o bloco e a parede
0,20. Qual o menor valor da intensidade da fora F
&
para o
bloco permanecer em equilbrio?

4. (AMAN) Um automvel se move em uma estrada
horizontal, com velocidade constante de 30m/s. Num dado
instante o carro freado e, at parar, desliza sobre a estrada
numa distncia de 75m. Determinar o coeficiente de atrito
entre os pneus e a estrada. Usar g = 10m/s
2
.
a) 0,2 c) 0,4 e) 0,6
b) 0,3 d) 0,5

5. (VUNESP) No sistema a seguir, A tem massa m
A
=10kg.
B tem massa m
B
=15kg. o=45. Qual ser o coeficiente de
atrito entre as superfcies em contacto, do corpo A com o
plano, para que o corpo se desloque com movimento
uniforme?

Observaes: g = 10m/s
2
;
o peso da corda, o atrito no
eixo da roldana e a massa
da roldana so
desprezveis.


6. (MACK) A ilustrao a seguir refere-se a uma certa
tarefa na qual o bloco B dez vezes mais pesado que o bloco
A dever descer pelo plano inclinado com velocidade
constante. Considerando que o fio e a polia so ideais, o
coeficiente de atrito cintico entre o bloco B e o plano
dever ser:
Dados:
sen o = 0,6
cos o = 0,8
a) 0,500
b) 0,750
c) 0,875
d) 1,33
e) 1,50


7. (FATEC) O corpo A, de massa
10kg, apoiado sobre uma superfcie
horizontal, est parado, prestes a
deslizar, preso por um fio ao corpo B,
de massa 2,0kg.
Considerando-se o fio e a roldana ideais e adotando
g=10m/s
2
, o coeficiente de atrito esttico entre o corpo A e
a superfcie vale:
a) 2,0 c) 0,20 e) 0,50
b) 0,10 d) 0,40

Tarefa Complementar #

8.(UFSC) Uma prensa utilizada para sustentar um bloco
apoiado em uma parede vertical, como ilustrado na Figura
1. O bloco e a parede so slidos e indeformveis. A
prensa exerce uma fora de 10
4
N sobre o bloco, na direo
perpendicular s superfcies em contato. A massa do bloco
de 50kg e o coeficiente de atrito esttico entre o bloco e a
parede 0,35. Em seguida, mais blocos de mesma massa
N
&

T
P
&

N
P
&

P
&

u
u
F
&

AT
F
&

u
F
&

Please purchase PDF Split-Merge on www.verypdf.com to remove this watermark.
Fsica A Incluso para a Vida

Pr-Vestibular da UFSC 16
so colocados em cima do primeiro, como mostrado na
Figura 2, porm a fora que a prensa exerce permanece
inalterada.

Em relao situao descrita, assinale a(s)
proposio(es) correta(s).
01. Com a fora aplicada possvel sustentar um total de
sete blocos iguais ao primeiro.
02. A fora que a parede exerce sobre o primeiro bloco
igual a 10
4
N e a fora de atrito esttico entre a parede e
o bloco igual a 3500N.
04. A fora necessria para sustentar apenas um bloco
igual a 175N.
08. A fora de atrito esttico entre a parede e os blocos
acima do primeiro nula.
16. Se o coeficiente de atrito esttico entre a parede e o
bloco for nulo, a prensa no sustentar o primeiro bloco
contra a parede por maior que seja a fora aplicada F
&
.
32. Quanto mais polidas forem as superfcies em contato da
parede e do bloco, menor ser o coeficiente de atrito e,
portanto, menor ser o nmero de blocos que a fora
aplicada poder sustentar.
64. Como o peso de cada bloco de 500N, a fora F
&

aplicada pela prensa poder sustentar 20 blocos.

9. (UFSC) No que diz respeito ao atrito, correto afirmar:
01. uma coisa extremamente intil em qualquer
circunstncia prtica.
02. um dos fatores que mais contribuem para o desgaste
de diversos tipos de equipamentos e utenslios, como
engrenagens mecnicas, solas de sapatos, pneus, etc.
04. Se o atrito no existisse teramos muita dificuldade para
executar determinadas tarefas como, por exemplo,
caminhar.
08. A fora de atrito, a que um dado corpo se acha
submetido, proporcional fora normal que a
superfcie exerce sobre o corpo.
16. O coeficiente de atrito cintico proporcional
velocidade adquirida por um corpo, e a sua unidade S.
I. o newton.metro/segundo (Nm/s).
32. O coeficiente de atrito cintico sempre
numericamente superior ao coeficiente de atrito
esttico.

10. (UFSC) Um homem empurra uma mesa com uma
fora horizontal F
&
, da esquerda para a direita,
movimentando-a neste sentido. Um livro solto sobre a
mesa permanece em repouso em relao a ela.

Considerando a situao descrita, assinale a(s)
proposio(es) correta(s).
01. Se a mesa deslizar com velocidade constante, a fora de
atrito sobre o livro no ser nula.
02. Como o livro est em repouso em relao mesa, a
fora de atrito que age sobre ele igual, em mdulo,
fora F
&
.
04. Se a mesa deslizar com acelerao constante, atuaro
sobre o livro somente as foras peso, normal e a fora
F
&
.
08. Se a mesa deslizar com acelerao constante, a fora de
atrito que atua sobre o livro ser responsvel pela
acelerao do livro.
16. Se a mesa deslizar com velocidade constante, atuaro
somente as foras peso e normal sobre o livro.
32. Se a mesa deslizar com acelerao constante, o sentido
da fora de atrito que age sobre o livro ser da esquerda
para a direita.

UNIDADE 10

COMPONENTES DA FORA RESULTANTE

O Princpio Fundamental da Dinmica estabelece
que, para produzir uma acelerao a num ponto material,
deve ser aplicada nesse ponto uma fora resultante F tal
que F= ma.
Nessas condies, se um ponto material descreve
uma curva, existe acelerao centrpeta e, portanto,
existem foras com componentes normais trajetria. A
resultante das foras componentes normais trajetria
recebe o nome de resultante centrpeta ou fora centrpeta
Fc.
Se o mdulo da velocidade de um ponto material
varia, existe acelerao tangencial e, portanto, foras com
componentes tangentes trajetria. A resultante destas
foras componentes recebe o nome de resultante
tangencial ou fora tangencial F
T
.
Considere um ponto material em movimento
curvilneo sob ao de vrias foras que, quando
decompostas, resultam em Fc e F
T
conforme a figura. Para
calcular o valor da fora centrpeta e da fora tangencial
temos, respectivamente, que:

R
v
m F
C
2
=
e a m F
T
. =











T C R
F F F
& & &
+ =
2 2 2
T C R
F F F + =

No caso de o movimento curvilneo ser
uniforme, a resultante tangencial nula, pois o mdulo da
velocidade no varia. A resultante de todas as foras a
resultante centrpeta.

Esquerda Direita
tangente
T
F
&

C
F
&

R
F
&

normal
Please purchase PDF Split-Merge on www.verypdf.com to remove this watermark.
Incluso para a Vida Fsica A

Pr-Vestibular da UFSC 17
Exerccios de Sala #

1. (UNIMEP) Determinar a
inclinao que deve ter uma
estrada, em uma curva de 400
m de raio, para que um carro,
com velocidade de mdulo 40
m/s, no derrape,
independentemente do coeficiente de atrito. Adote g = 10
m/s
2
.

2. Um pndulo constitudo por um fio ideal de
comprimento 0,50m e esfera pendular de massa 3,0kg.
Quando a esfera pendular realiza uma oscilao circular e
passa pelo ponto mais baixo (fio vertical), sua velocidade
tem mdulo igual a 2,0m/s. Adote g = 10m/s
2
. Pede-se:
a) a intensidade da resultante centrpeta, quando a esfera
passa pelo ponto mais baixo;
b) a intensidade da fora tensora no fio nessa posio.

Tarefa Mnima #

3. (ACAFE) O barco viking um entretenimento
encontrado em diversos parques de diverso. Analisando o
movimento de ida e volta do barco somente no pice do
movimento, observa-se que o movimento de um pndulo
simples. Em relao ao exposto, a alternativa verdadeira :
a) as foras que atual sobre o passageiro so a fora
centrpeta, a fora peso e a fora normal.
b) O mdulo da fora normal que o assento exerce sobre o
passageiro maior no ponto mais baixo da trajetria.
c) O mdulo da fora-peso do passageiro maior no
ponto mais baixo da trajetria.
d) O mdulo da fora-peso do passageiro sempre igual
ao mdulo da fora normal que o assento exerce sobre
ele.
e) A fora resultante sobre o passageiro sempre a fora
centrpeta.

4. (UFRGS) Uma moto descreve uma circunferncia
vertical no globo da morte de raio 4m (g = 10 m/s
2
). A
massa total da moto 150kg. A velocidade da moto no
ponto mais alto 12m/s. A fora que a moto exerce no
globo, em N, :
a) 1500 c) 3900 e) n. d. a.
b) 2400 d) 4000

5. (UFPR) Qual a velocidade mxima que um carro pode
fazer uma curva horizontal de 25m de raio, se o coeficiente
de atrito esttico entre os pneus e a estrada 0,8? (Use g =
10 m/s
2
)

6. (FUVEST) A figura a seguir mostra, num plano
vertical, parte dos trilhos do percurso circular de uma
"montanha russa" de um parque de diverses. A velocidade
mnima que o carrinho deve ter, ao passar pelo ponto mais
alto da trajetria, para no desgrudar dos trilhos vale, em
metros por segundo,
a)
20

b)
40

c)
80

d)
160

e)
320



7. (UFMG) Observe o desenho.
Esse desenho representa um
trecho de uma montanha russa.
Um carrinho passa pelo ponto P e
no cai.
Pode-se afirmar que, no ponto P,

a) a fora centrfuga que atua no carrinho o empurra
sempre para frente.
b) a fora centrpeta que atua no carrinho equilibra o seu
peso.
c) a fora centrpeta que atua no carrinho mantm sua
trajetria circular.
d) a soma das foras que o trilho faz sobre o carrinho
equilibra seu peso.
e) o peso do carrinho nulo nesse ponto.

Tarefa Complementar #

8. (UFSC) Deseja-se construir um brinquedo para um
parque de diverses, que consiste em um cilindro sem
assoalho que gira em torno de um eixo vertical, com
velocidade angular e = 2 rad/s, no qual as pessoas ficariam
pressionadas contra a parede interior sem escorregar para
baixo, conforme a figura. Considerando-se que o
coeficiente de atrito esttico entre a parede e as costas das
pessoas seja = 0,5, qual o raio mnimo, em m, que dever
ter o cilindro para que as pessoas no escorreguem? (Use
g = 10 m/s
2
)
W


9. (UFSC) Um piloto executa um looping com seu avio
manobra acrobtica em que a aeronave descreve um arco
de circunferncia no plano vertical que atinge, no ponto
mais baixo da trajetria, ao completar a manobra, a
velocidade mxima de 540km/h. O raio da trajetria
igual a 450m e a massa do piloto 70 kg. Nessas manobras
acrobticas devemos considerar que a maior acelerao
que o organismo humano pode suportar 9g (g =
acelerao da gravidade).
Com base nos dados
fornecidos, assinale a(s)
proposio(es) correta(s).

01. Se o raio de trajetria fosse menor do que 250m, o
piloto seria submetido a uma acelerao centrpeta
mxima maior do que 9g (nove vezes a acelerao da
gravidade).
02. A fora centrpeta sobre o piloto, na parte mais baixa
da trajetria, cinco vezes maior do que o seu peso.
Please purchase PDF Split-Merge on www.verypdf.com to remove this watermark.
Fsica A Incluso para a Vida

Pr-Vestibular da UFSC 18
04. O piloto submetido a uma acelerao centrpeta
mxima igual a 5g (cinco vezes a acelerao da
gravidade).
08. A velocidade mnima para que o avio complete a
volta, no topo da trajetria, igual a 270km/h.
16. A fora que o avio faz sobre o piloto, na parte mais
baixa da trajetria, igual a 4200N.
32. A fora que o piloto faz sobre o avio igual ao seu
peso, em toda a trajetria.
64. O piloto submetido a uma acelerao centrpeta
mxima no topo da trajetria, quando a fora de
sustentao do avio mnima.

10. (UFSC) Um avio descreve uma curva em trajetria
circular com velocidade escalar constante, num plano
horizontal, conforme est representado na figura, onde F
a fora de sustentao perpendicular s asas; P a fora
peso; o o ngulo de inclinao das asas em relao ao
plano horizontal; R o raio de trajetria. So conhecidos
os valores: o=45; R=1000metros; massa do
avio=10000kg.

Assinale a(s) proposio(es) correta(s), considerando,
para efeito de clculos, apenas as foras indicadas na
figura.
01. Se o avio realiza movimento circular uniforme, a
resultante das foras que atuam sobre ele nula.
02. Se o avio descreve uma trajetria curvilnea, a
resultante das foras externas que atuam sobre ele ,
necessariamente, diferente de zero.
04. A fora centrpeta , em cada ponto da trajetria, a
resultante das foras externas que atuam no avio, na
direo do raio da trajetria.
08. A fora centrpeta sobre o avio tem intensidade igual a
100000N.
16. A velocidade do avio tem valor igual a 360km/h.
32. A fora resultante que atua sobre o avio no depende
do ngulo de inclinao das asas em relao ao plano
horizontal.

UNIDADE 11

TRABALHO E POTNCIA

TRABALHO

a quantidade de energia transformada ou transferida
atravs da aplicao de uma fora.
Matematicamente, o trabalho definido da seguinte
maneira:
d F. = t








Observe, na ilustrao anterior, que o
deslocamento se d na direo horizontal. Desta forma, a
componente F
Y
no influencia no movimento, portanto
no realiza trabalho. Assim, o trabalho ser:
d F
X
. = t
Como u cos . F F
X
= , temos que:

u t cos . .d F = [J]

<
>
resistente
motor
0
0
t
t


Trabalho da Fora Peso

Considere que um objeto deva se deslocar entre
os pontos A e B na figura abaixo: A fora peso realiza
trabalho apenas na direo vertical (altura).







t
peso
= P.h
Se o objeto desce: trabalho motor (+)
Se o objeto sobe: trabalho resistente (-)

Mtodo Grfico







2. Potncia

Potncia a rapidez com que se realiza um trabalho.
t
P
M
A
=
t
[W]
Outras unidades: 1 HP = 746 W
1 CV = 735 W

Mtodo Grfico










RENDIMENTO

a relao entre a potncia til (P
U
) e a potncia total (P
T
)
de um sistema mecnico.
T
U
P
P
= q

Ateno!
- Rendimento uma grandeza adimensional;
- Ser sempre menor do que 1 e maior do que 0;
0 s q < 1
- Pode ser expresso em porcentagem.
q
%
= q.100%
d
&

F
&

X
F
&

Y
F
&

u
A
B
C
h
d
F
A
0
t = A
t = A
t
P
A
0
Please purchase PDF Split-Merge on www.verypdf.com to remove this watermark.
Incluso para a Vida Fsica A

Pr-Vestibular da UFSC 19
Exerccios de Sala #

1.(ACAFE) Um bloco de 10 kg puxado por uma fora de
200N que forma um ngulo de 60 com a horizontal. O
bloco desloca-se 20m sobre uma superfcie horizontal, sem
atrito. Determine o trabalho total realizado sobre o bloco.
a) 200 J d) 1400 J
b) 600 J e) 2000 J
c) 1000 J

2.(FEI) Uma fora F paralela trajetria de seu ponto de
aplicao varia com o deslocamento de acordo com a
figura a seguir. Qual o trabalho realizado pela fora F no
deslocamento de 1 a 5 m?
a) 100J
b) 20J
c) 1 2J
d) 15J
e) 10J

3.(UEL) Um operrio ergue, do cho at uma prateleira a
2,0m de altura, uma saca de soja de massa 60kg, gastando
2,5s na operao. A potncia mdia despendida pelo
operrio, em watts, no mnimo, (Dados: g = 10m/s
2)

a) 2,4.10
2
c) 3,5.10
2
e) 6,0.10
2

b) 2,9.10
2
d) 4,8.10
2

Tarefa Mnima #

4.(UEL) O trabalho realizado por F
&
, no deslocamento de
x = 0 at x = 4,0m, em joules, vale:
a) zero
b) 10
c) 20
d) 30
e) 40


5.(FEI) Um corpo de massa 5kg retirado de um ponto A
e levado para um ponto B, distante 40m na horizontal e
30m na vertical, traadas a partir do ponto A. Qual o
mdulo do trabalho realizado pela fora peso?
a) 2500 J c) 900 J e) 1500 J
b)2000 J d) 500 J

6.(VUNESP) Um motor de potncia til igual a 125W,
funcionando como elevador, eleva a 10m de altura, com
velocidade constante, um corpo de peso igual a 50N, no
tempo de:
a) 0,4 s c) 12,5 s e) 4,0 s
b) 2,5 s d) 5,0 s

7.(UFRJ) Uma pessoa caminha sobre um plano horizontal.
O trabalho realizado pelo peso desta pessoa
a) sempre positivo.
b) sempre negativo.
c) sempre igual a zero.
d) positivo, se o sentido do deslocamento for da esquerda
para a direita.
e) negativo, se o sentido do deslocamento for da direita
para a esquerda.



8.(UEL) Um guindaste ergue um fardo, de peso 1,0.10
3
N,
do cho at 4,0 m de altura, em 8,0s. A potncia mdia do
motor do guindaste, nessa operao, em watts, vale:
a) 1,0 . 10
2
d) 5,0 . 10
2

b) 2,0 . 10
2
e) 2,0 . 10
3

c) 2,5 . 10
2


9.(FGV) Um veculo de massa 1500kg gasta uma
quantidade de combustvel equivalente a 7,5. 10
6
J para
subir um morro de 100m e chegar at o topo. O rendimento
do motor do veculo para essa subida ser de:
a) 75 % d) 50 %
b) 40 % e) 20 %
c) 60 %

Tarefa Complementar #

10.(UFSC) Um homem ergue um bloco de 100 newtons a
uma altura de 2,0 metros em 4,0 segundos com velocidade
constante. Qual a potncia em watts desenvolvida pelo
homem?

11.(UFSC) Um homem empurra uma caixa ladeira abaixo.
Assinale a(s) proposio(es) que relaciona(m) a(s)
fora(s) que realiza(m) trabalho(s) positivo(s).
01. Fora-peso da caixa.
02. Fora normal sobre a caixa
04. Fora de atrito cintico.
08. Fora do homem sobre a caixa.
16. Fora de resistncia do ar sobre a caixa.

12.(UFSC) Em relao ao conceito de trabalho, correto afirmar
que:
01. Quando atuam somente foras conservativas em um
corpo, a energia cintica deste no se altera.
02. Em relao posio de equilbrio de uma mola, o
trabalho realizado para comprimi-la, por uma distncia
x, igual ao trabalho para distend-la por x.
04. A fora centrpeta realiza um trabalho positivo em um
corpo em movimento circular uniforme, pois a direo
e o sentido da velocidade variam continuamente nesta
trajetria.
08. Se um operrio arrasta um caixote em um plano
horizontal entre dois pontos A e B, o trabalho efetuado
pela fora de atrito que atua no caixote ser o mesmo,
quer o caixote seja arrastado em uma trajetria em
ziguezague ou ao longo da trajetria mais curta entre A
e B.
16. Quando uma pessoa sobe uma montanha, o trabalho
efetuado sobre ela pela fora gravitacional, entre a base
e o topo, o mesmo, quer o caminho seguido seja
ngreme e curto, quer seja menos ngreme e mais longo.
32. O trabalho realizado sobre um corpo por uma fora
conservativa nulo quando a trajetria descrita pelo
corpo um percurso fechado.









Please purchase PDF Split-Merge on www.verypdf.com to remove this watermark.
Fsica A Incluso para a Vida

Pr-Vestibular da UFSC 20
UNIDADE 12

ENERGIA

ENERGIA CINTICA

Podemos calcular a energia cintica de um corpo de massa
m que se movimenta com uma velocidade v da seguinte
forma:
2
.
2
v m
E
c
=

TEOREMA DA ENERGIA CINTICA

O trabalho da resultante das foras agentes em um corpo
em determinado deslocamento mede a variao de energia
cintica ocorrida nesse deslocamento.
t = AE
c

ENERGIA POTENCIAL GRAVITACIONAL

Chamamos de energia potencial gravitacional a energia
armazenada em um sistema devido sua posio em um
campo de gravidade, em outras palavras, sua altura em
relao referncia.



E
P
= m.g.h



ENERGIA POTENCIAL ELSTICA

Energia potencial elstica a energia armazenada em um
corpo elstico deformado. Para calcular essa energia
calculamos o trabalho da fora elstica para, a partir da
posio de equilbrio, produzir uma deformao x na mola
de constante elstica K.
2
.
2
x K
E
pe
=
ENERGIA MECNICA

a soma da energia cintica com a energia potencial de
um sistema fsico.
E
M
= E
c
+ E
p


SISTEMAS CONSERVATIVOS E CONSERVAO
DA ENERGIA MECNICA

Foras conservativas so aquelas as quais est associada
uma energia potencial, como o peso e a fora elstica.
Quando um corpo est sob ao de uma fora conservativa
que realiza trabalho resistente, a energia cintica diminui,
mas em compensao ocorre um aumento de energia
potencial. Quando a fora conservativa realiza trabalho
motor, a energia cintica aumenta, o que corresponde a
uma diminuio equivalente de energia potencial. Quando,
em um sistema de corpos, as foras que realizam trabalho
so todas conservativas, o sistema chamado sistema
conservativo.
Foras dissipativas so aquelas que, quando
realizam trabalho, este sempre resistente, em qualquer
deslocamento. Como consequncia, a energia mecnica de
um sistema, sob ao de foras dissipativas, diminui.
Conservao da Energia Mecnica
A energia mecnica de um sistema permanece
constante quando este se movimenta sob ao de foras
conservativas e eventualmente de outras foras que
realizam trabalho nulo.

Exerccios de Sala #

1. (UDESC) Um homem, cuja massa igual a 80,0 kg,
sobe uma escada com velocidade escalar constante. Sabe-
se que a escada possui 20
degraus e a altura de cada
degrau de 15,0 cm.
DETERMINE a energia
gasta pelo homem para subir
toda a escada.
Dado: g = 10,0m/s
2

2. (MACK) Um pequeno bloco de massa m abandonado
do ponto A e desliza ao longo de um trilho sem atrito,
como mostra a figura a seguir. Para que a fora que o trilho
exerce sobre o bloco no ponto D seja igual ao seu peso,
supondo ser R o raio do arco de circunferncia de dimetro
BD, a altura h deve ser igual a:

a) 2R.
b) 2,5R.
c) 3R.
d) 3,5R.
e) 4R.

Tarefa Mnima #

3. (UFRS) Uma pedra de 4kg de massa colocada em um
ponto A, 10m acima do solo. A pedra deixada cair
livremente at um ponto B, a 4m de altura.
Quais so, respectivamente, a energia potencial no ponto
A, a energia potencial no ponto B, e o trabalho realizado
sobre a pedra pela fora peso? (Use g=10m/s
2
e considere
o solo como nvel zero para energia potencial).
a) 40 J, 16 J e 24 J. d) 400 J, 160 J e 560 J.
b) 40 J, 16 J e 56 J. e) 400 J, 240 J e 560 J.
c) 400 J, 160 J e 240 J.


4. (FATEC) Um objeto de massa 400g desce, a partir do
repouso no ponto A, por uma rampa, em forma de um
quadrante de circunferncia de raio R=1,0m. Na base B,
choca-se com uma mola de constante elstica k=200N/m.
Desprezando a ao de foras dissipativas em todo o
movimento e adotando g=10m/s
2
, a mxima deformao
da mola de:

a) 40cm
b) 20cm
c) 10cm
d) 4,0cm
e) 2,0cm

h g !
m
m
v
&

Please purchase PDF Split-Merge on www.verypdf.com to remove this watermark.
Incluso para a Vida Fsica A

Pr-Vestibular da UFSC 21
5.(UFPE) Um bloco solto no
ponto A e desliza sem atrito
sobre a superfcie indicada na
figura a seguir. Com relao ao
bloco, podemos afirmar:
a) A energia cintica no ponto B menor que no ponto C.
b) A energia cintica no ponto A maior que no ponto B.
c) A energia potencial no ponto A menor que a energia
cintica no ponto B.
d) A energia total do bloco varia ao longo da trajetria
ABC.
e) A energia total do bloco ao longo da trajetria ABC
constante.

Tarefa Complementar #

6. (UFSC) Um corpo parte do repouso deslizando do topo
de um plano inclinado, de uma altura de 2,7m em relao
ao plano horizontal (veja figura a seguir). Devido ao atrito,
ele perde 1/3 de sua energia mecnica inicial no percurso
do topo at a base do
plano inclinado.
Calcule ento a
velocidade, em m/s,
com que o corpo
chega na base.

7. (UFSC) Nos trilhos de uma montanha-russa, um
carrinho com seus ocupantes solto, a partir do repouso,
de uma posio A situada a uma altura h, ganhando
velocidade e percorrendo um crculo vertical de raio R =
6,0 m, conforme mostra a figura. A massa do carrinho com
seus ocupantes igual a 300 kg e se despreza a ao de
foras dissipativas sobre
o conjunto.
Assinale a(s)
proposio(es)
correta(s).
01. Na ausncia de foras dissipativas a energia mecnica
do carrinho se conserva, isto , a soma da energia
potencial gravitacional e da energia cintica tem igual
valor nas posies A, B e C, respectivamente.
02. A energia mecnica mnima para que o carrinho
complete a trajetria, sem cair, igual a 4 500J.
04. A posio A, de onde o carrinho solto para iniciar seu
trajeto, deve situar-se altura mnima h = 15m para
que o carrinho consiga completar a trajetria, passando
pela posio B, sem cair.
08. A velocidade mnima na posio B, ponto mais alto do
crculo vertical da montanha-russa, para que o carrinho
no caia 60 m/s.
16. A posio A, de onde o carrinho solto para iniciar seu
trajeto, deve se situar altura mnima h = 12m para que
o carrinho consiga completar a trajetria passando pela
posio B, sem cair.
32. Podemos considerar a conservao da energia mecnica
porque, na ausncia de foras dissipativas, a nica fora
atuante sobre o sistema a fora peso, que uma fora
conservativa.
64. A energia mecnica do carrinho no ponto C menor do
que no ponto A.

8. (UFSC) A figura mostra um bloco, de massa m = 500g,
mantido encostado em uma mola comprimida de X = 20
cm. A constante elstica da mola K = 400 N/m. A mola
solta e empurra o bloco que, partindo do repouso no ponto
A, atinge o ponto B, onde para. No percurso entre os
pontos A e B, a fora de atrito da superfcie sobre o bloco
dissipa 20% da energia mecnica inicial no ponto A.

Assinale a(s) proposio(es) correta(s):
01. Na situao descrita, no h conservao da energia
mecnica.
02. A energia mecnica do bloco no ponto B igual a 6,4 J.
04. O trabalho realizado pela fora de atrito sobre o bloco,
durante o seu movimento, foi 1,6 J.
08. O ponto B situa-se a 80cm de altura, em relao ao
ponto A.
16. A fora peso no realizou trabalho no deslocamento do
bloco entre os pontos A e B, por isso no houve
conservao da energia mecnica do bloco.
32. A energia mecnica total do bloco no ponto A igual a
8,0 J.
64. A energia potencial elstica do bloco, no ponto A,
totalmente transformada na energia potencial
gravitacional do bloco, no ponto B.

9.(UFSC) Na figura abaixo, a
esfera tem massa igual a 2,0kg e se
encontra presa na extremidade de
uma mola de massa desprezvel e
constante elstica de 500 N/m. A
esfera est, inicialmente, em
repouso, mantida na posio A,
onde a mola no est deformada. A
posio A se situa a 30cm de altura em relao posio
B.
Soltando a esfera, ela desce sob a ao da gravidade. Ao
passar pelo ponto B, a mola se encontra na vertical e
distendida de 10cm. Desprezam-se as dimenses da esfera
e os efeitos da resistncia do ar.
Considerando-se a situao fsica descrita, assinale a(s)
proposio(es) correta(s).
01. A velocidade da esfera no ponto mais baixo da
trajetria, ponto B, igual a 6,0 m/s.
02. Toda a energia potencial gravitacional da esfera, na
posio A, transformada em energia cintica, na
posio B.
04. A velocidade da esfera no ponto B igual a 3,5 m/s.
08. A fora resultante sobre a esfera na posio B igual a
30N.
16. A energia mecnica da esfera, na posio B, igual
sua energia potencial gravitacional na posio A.
32. Parte da energia potencial gravitacional da esfera, na
posio A, convertida em energia potencial elstica,
na posio B.
64. A energia cintica da esfera, na posio B, igual a sua
energia potencial gravitacional, na posio A.


Please purchase PDF Split-Merge on www.verypdf.com to remove this watermark.
Fsica A Incluso para a Vida

Pr-Vestibular da UFSC 22
10. (UFSC) A figura abaixo mostra o esquema (fora de
escala) da trajetria de um avio. O avio sobe com grande
inclinao at o ponto 1, a partir do qual tanto a ao das
turbinas quanto a do ar se cancelam totalmente, e ele passa
a descrever uma trajetria parablica sob a ao nica da
fora peso. Durante a trajetria parablica, objetos soltos
dentro do avio parecem flutuar. O ponto 2 corresponde
altura mxima de 10km.
Assinale a(s) proposio(es) correta(s).

01. A componente horizontal da velocidade constante
entre os pontos 1, 2 e 3.
02. Para justificar por que os objetos flutuam, a fora
gravitacional da Terra sobre os objetos no pode ser
desprezada entre os pontos 1, 2 e 3.
04. Os objetos parecem flutuar porque a fora de atrao
gravitacional da Terra sobre eles desprezvel.
08. A acelerao vertical, em relao ao solo, a 10km de
altura (ponto 2), vale zero.
16. A energia cintica do avio, em relao ao solo, tem o
mesmo valor no ponto 1 e no ponto 3.
32. A energia potencial gravitacional do avio no ponto 1
menor do que no ponto 2.

UNIDADES 13 e 14

QUANTIDADE DE MOVIMENTO,
IMPULSO E COLISES

QUANTIDADE DE MOVIMENTO

A quantidade de movimento (ou Momento Linear) Q
&
de
uma partcula de massa m e velocidade vetorial v
&

(conforme a figura) uma grandeza vetorial, definida
como:


v m Q
&
&
. = [kg.m/s]

Num sistema de partculas, a quantidade de
movimento do sistema igual soma vetorial das
quantidades de movimento de cada partcula do sistema.

IMPULSO DE UMA FORA CONSTANTE

uma grandeza vetorial definida como o produto da fora
aplicada F
&
pelo intervalo de tempo t A que ela atuou:


t F I A = .
& &
[N.s]

TEOREMA DO IMPULSO
O impulso de uma fora constante F
&
, em um intervalo de
tempo t A , igual variao da quantidade de movimento
produzida por essa fora, no intervalo de tempo t A .
Q I
& &
A =


IMPULSO DE UMA FORA VARIVEL

Quando a intensidade de uma fora varia no decorrer do
tempo, ela pode ser representada num grfico da fora em
funo do tempo.

Impulso rea =

PRINCPIO DA CONSERVAO DA
QUANTIDADE DE MOVIMENTO

Ao analisarmos o movimento de um sistema de corpos
precisaremos separar as foras que atuam nos corpos em
dois conjuntos: o das foras internas e o das foras
externas. Uma fora chamada de interna quando ela
exercida por um corpo de sistema sobre outro corpo do
mesmo sistema. Uma fora atuante num corpo do sistema
chamada de externa quando exercida por um corpo que
est fora do sistema.
Quando a resultante das foras externas igual a
zero, dizemos que esse sistema isolado de foras
externas. Exemplos de sistemas isolados: Exploses e
Colises.
Em um sistema isolado, a quantidade de
movimento constante. O enunciado em negrito constitui
o Princpio da Conservao da Quantidade de Movimento.

COLISES

Fases de uma Coliso

Fase de Deformao: inicia quando os corpos entram em
contato e termina quando eles possuem a mesma
velocidade. Nessa fase h transformao de energia
cintica em energia potencial elstica e outros tipos de
energia, como sonora, trmica, etc. (perdas).

Fase de Restituio: comea quando os corpos tm a
mesma velocidade e termina quando eles se separam.
Nesta fase, a energia potencial elstica volta a ser cintica,
com ou sem perda de energia mecnica.



x
y(km)
10
1 3

2
-
-
-
Please purchase PDF Split-Merge on www.verypdf.com to remove this watermark.
Incluso para a Vida Fsica A

Pr-Vestibular da UFSC 23
Coeficiente de Restituio

Considere a coliso unidimensional do exemplo
abaixo:

* Antes da coliso






* Depois da coliso






A velocidade relativa entre os corpos antes da
coliso chamada de velocidade de aproximao, e dada
por:
V
AP
= V
A
- V
B


Aps a coliso, a velocidade relativa entre os
corpos chamada de velocidade de afastamento, e
calculada como:
V
AF
= V
B
V
A


O coeficiente de restituio o nmero que mede
a intensidade de segunda fase, e calculado como:

AP
AF
V
V
e =

Tipos de coliso

Coliso (Perfeitamente) Elstica

e = 1
No h perda de energia mecnica
Duas fases

Coliso Parcialmente Elstica ou Parcialmente
Inelstica

0 < e < 1
H perda de Energia Mecnica
Duas Fases
Coliso (Perfeitamente) Inelstica

e = 0
H a maior perda de energia
Apenas a fase de deformao
Os corpos no se separam depois da coliso

Todos os tipos de coliso so considerados sistemas
isolados de foras externas, por isso, a quantidade de
movimento total do sistema se conserva.







Exerccios de Sala #

1. (UEL) Se os mdulos das quantidades de movimento de
dois corpos so iguais, necessariamente eles possuem:
a) mesma energia cintica.
b) velocidade de mesmo mdulo.
c) mdulos das velocidades proporcionais s suas massas.
d) mesma massa e velocidades de mesmo mdulo.
e) mdulos das velocidades inversamente proporcionais s
suas massas.

2. (UERJ) Uma bola de futebol de massa igual a 300g
atinge uma trave da baliza com velocidade de 5,0 m/s e
volta na mesma direo com velocidade idntica.
O mdulo do impulso aplicado pela trave sobre a bola, em
N.s, corresponde a:
a) 1,5 b) 2,5 c) 3,0 d) 5,0

3. (UFPE) A fora resultante que atua sobre um bloco de
2,5kg, inicialmente em repouso, aumenta uniformemente
de zero at 100N em 0,2s, conforme a figura a seguir. A
velocidade final do bloco, em m/s, :

a) 2,0
b) 4,0
c) 6,0
d) 8,0
e) 10

Tarefa Mnima #

4. (UFSM) Um corpo de massa 2 kgvaria sua velocidade
de 10 m/s para 30 m/s, sob a ao de uma fora constante.
O impulso da fora sobre o corpo , em Ns,
a) 20 c) 40 e) 80
b) 30 d) 60

5. (PUC-Campinas) Um corpo de massa "m" se encontra
em repouso sobre uma superfcie horizontal, sem atrito,
quando submetido ao de uma fora F, constante,
paralela superfcie, que lhe imprime uma acelerao de
2,0m/s
2
. Aps 5,0s de movimento, o mdulo da sua
quantidade de movimento vale 20kg. m/s.
A massa "m" do corpo, em kg, vale:
a) 5,0 b) 2,0 c) 1,0
d) 0,20 e) 0,10

6. (PUC-Campinas) Um garoto de 58kg est sobre um
carrinho de rolim que percorre uma pista em declive. A
componente da fora resultante que age no garoto, na
direo do movimento, tem mdulo representado no
grfico, para um pequeno trecho do movimento. Sabe-se
que a velocidade do garoto no instante t
1
=2,0s 3,0m/s.

Pode-se concluir que velocidade do
garoto em m/s, no instante t
2
=16s, igual a
a) 13 c) 19 e) 163
b) 16 d) 43

A B
V
A V
B
B A B
V
A V
B
Please purchase PDF Split-Merge on www.verypdf.com to remove this watermark.
Fsica A Incluso para a Vida

Pr-Vestibular da UFSC 24
7. (PUC-PR) Dois patinadores, um de massa 100kg e outro
de massa 80kg, esto de mos dadas em repouso sobre uma
pista de gelo, onde o atrito desprezvel. Eles empurram-
se mutuamente e deslizam na mesma direo, porm em
sentidos opostos. O patinador de 100kg adquire uma
velocidade de 4m/s. A velocidade relativa de um dos
patinadores em relao ao outro , em mdulo, igual a:


a) 5 m/s
b) 4 m/s
c) 1 m/s
d) 9 m/s
e) 20 m/s

8. (UERJ) Um peixe de 4kg,
nadando com velocidade de 1,0m/s,
no sentido indicado pela figura,
engole um peixe de 1kg, que estava
em repouso, e continua nadando no mesmo sentido.
A velocidade, em m/s, do peixe maior, imediatamente aps
a ingesto, igual a:
a) 1,0 b) 0,8 c) 0,6 d) 0,4

9. (UFPE) Um bloco de massa m
1
= 100g comprime uma
mola de constante elstica k = 360 N/m, por uma distncia
x = 10,0 cm, como mostra a figura. Em um dado instante,
esse bloco liberado, vindo a colidir em seguida com outro
bloco de massa m
2
= 200g, inicialmente em repouso.
Despreze o atrito entre os blocos e o piso. Considerando a
coliso perfeitamente inelstica, determine a velocidade
final dos blocos, em m/s.

10. (PUC-SP) Dois carros, A e B,
de massas iguais, movem-se em
uma estrada retilnea e horizontal,
em sentidos opostos, com
velocidades de mesmo mdulo.
Aps se chocarem frontalmente,
ambos param imediatamente devido coliso.
Pode-se afirmar que, no sistema, em relao situao
descrita,
a) h conservao da quantidade de movimento do sistema
e da sua energia cintica total.
b) no h conservao da quantidade de movimento do
sistema, mas a energia cintica total se conserva.
c) nem a quantidade de movimento do sistema e nem a
energia cintica total se conservam.
d) a quantidade de movimento do sistema transformada
em energia cintica.
e) h conservao da quantidade de movimento do sistema,
mas no da sua energia cintica total.

Tarefa Complementar #

11.(UFSC) As esferas A e B da
figura tm a mesma massa e
esto presas a fios inextensveis,
de massas desprezveis e de
mesmo comprimento, sendo L a
distncia do ponto de suspenso at o centro de massa das
esferas e igual a 0,80m. Inicialmente, as esferas se
encontram em repouso e mantidas nas posies indicadas.
Soltando-se a esfera A, ela desce, indo colidir de forma
perfeitamente elstica com a esfera B. Desprezam-se os
efeitos da resistncia do ar.
Assinale a(s) proposio(es) correta(s).
01. Considerando o sistema constitudo pelas esferas A e
B, em se tratando de um choque perfeitamente elstico,
podemos afirmar que h conservao da quantidade de
movimento total e da energia cintica total do sistema.
02. No possvel calcular o valor da velocidade da esfera
A no instante em que se colidiu com a esfera B, porque
no houve conservao da energia mecnica durante
seu movimento de descida e tambm porque no
conhecemos a sua massa.
04. A velocidade da esfera A, no ponto mais baixo da
trajetria, imediatamente antes colidir com a esfera B,
4,0m/s.
08. Durante o movimento de descida da esfera A, sua
energia mecnica permanece constante e possvel
afirmar que sua velocidade no ponto mais baixo da
trajetria, imediatamente antes de colidir com a esfera
B, de 3,0m/s.
16. Imediatamente aps a coliso, a esfera B se afasta da
esfera A com velocidade igual a 4,0m/s.
32. Aps a coliso, a esfera A permanece em repouso.
64. Aps a coliso, a esfera A volta com velocidade de
4,0m/s, invertendo o sentido do seu movimento inicial.

12. (UFSC) Na segunda-feira, 12 de junho de 2000, as
pginas esportivas dos jornais nacionais eram dedicadas ao
tenista catarinense Gustavo Kuerten, o "Guga", pela sua
brilhante vitria e conquista do ttulo de bicampeo do
Torneio de Roland Garros. Entre as muitas informaes
sobre a partida final do Torneio, os jornais afirmavam que
o saque mais rpido de Gustavo Kuerten foi de 195km/h.
Em uma partida de tnis, a bola atinge velocidades
superiores a 200km/h.
Consideremos uma partida de tnis com o "Guga" sacando:
lana a bola para o ar e atinge com a raquete, imprimindo-
lhe uma velocidade horizontal de 180km/h (50m/s). Ao ser
atingida pela raquete, a velocidade horizontal inicial da
bola considerada nula. A massa da bola igual a 58
gramas e o tempo de contato com a raquete 0,01s.
Assinale a(s) proposio(es) verdadeira(s):

01. A fora mdia exercida pela raquete sobre a bola
igual a 290N.
02. A fora mdia exercida pela bola sobre a raquete
igual quela exercida pela raquete sobre a bola.
04. O impulso total exercido sobre a bola igual a 2,9N.s.
08. O impulso total exercido pela raquete sobre a bola
igual a variao da quantidade de movimento da bola.
16. Mesmo considerando o rudo da coliso, as pequenas
deformaes permanentes da bola e da raquete e o
aquecimento de ambas, h conservao da energia
mecnica do sistema (bola + raquete), porque a
resultante das foras externas nula durante a coliso.
32. O impulso exercido pela raquete sobre a bola maior
do que aquele exercido pela bola sobre a raquete, tanto
assim que a raquete recua com velocidade de mdulo
muito menor que a da bola.
Please purchase PDF Split-Merge on www.verypdf.com to remove this watermark.
Incluso para a Vida Fsica A

Pr-Vestibular da UFSC 25
13. (UFSC) O air-bag, equipamento utilizado em veculos
para aumentar a segurana dos seus ocupantes em uma
coliso, constitudo por um saco de material plstico que
se infla rapidamente quando ocorre uma desacelerao
violenta do veculo, interpondo-se entre o motorista, ou o
passageiro, e a estrutura do veculo. Consideremos, por
exemplo, as colises frontais de dois veculos iguais, a uma
mesma velocidade, contra um mesmo obstculo rgido, um
com air-bag e outro sem air-bag, e com motoristas de
mesma massa. Os dois motoristas sofrero, durante a
coliso, a mesma variao de velocidade e a mesma
variao da quantidade de movimento. Entretanto, a
coliso do motorista contra o air-bag tem uma durao
maior do que a coliso do motorista diretamente contra a
estrutura do veculo. De forma simples, o air-bag aumenta
o tempo de coliso do motorista do veculo, isto , o
intervalo de tempo transcorrido desde o instante
imediatamente antes da coliso at a sua completa
imobilizao. Em consequncia, a fora mdia exercida
sobre o motorista no veculo com air-bag muito menor
durante a coliso.
Considerando o texto acima, assinale a(s) proposio(es)
correta(s).
01 A variao da quantidade de movimento do motorista
igual variao da quantidade de movimento do
veculo.
02.A variao da quantidade de movimento do motorista
do veculo a mesma, em uma coliso, com ou sem a
proteo do air-bag.
04.O impulso exercido pela estrutura do veculo sobre o
motorista igual variao da quantidade de
movimento do motorista.
08.A coliso do motorista contra o air-bag tem uma
durao maior do que a coliso do motorista
diretamente contra a estrutura do veculo.
16.O impulso exercido sobre o motorista o mesmo, em
uma coliso, com air-bag ou sem air-bag.
32.Tanto a variao da quantidade de movimento do
motorista como o impulso exercido para par-lo so
iguais, com ou sem air-bag; portanto, a fora mdia
exercida sobre ele a mesma, tambm.
64.A grande vantagem do air-bag aumentar o tempo de
coliso, e assim, diminuir a fora mdia atuante sobre o
motorista.

14. (UFSC) Dois astronautas, A e B, se encontram livres
na parte externa de uma estao espacial, sendo
desprezveis as foras de atrao gravitacional sobre eles.
Os astronautas com seus trajes espaciais tm massas m
A
=
100kg e m
B
= 90kg, alm de um tanque de oxignio
transportado pelo astronauta A, de massa 10kg. Ambos
esto em repouso em relao estao espacial, quando o
astronauta A lana o tanque de oxignio para o astronauta
B com uma velocidade de 5,0 m/s. O tanque choca-se com
o astronauta B que o agarra, mantendo-o junto a si,
enquanto se afasta.
Considerando como
referencial a
estao espacial,
assinale a(s)
proposio(es)
correta(s):

01. Considerando que a resultante das foras externas
nula, podemos afirmar que a quantidade de movimento
total do sistema constitudo pelos dois astronautas e o
tanque se conserva.
02. Antes de o tanque ter sido lanado, a quantidade de
movimento total do sistema constitudo pelos dois
astronautas e o tanque era nula.
04. Como vlida a terceira lei de Newton, o astronauta A,
imediatamente aps lanar o tanque para o astronauta
B, afasta-se com velocidade igual a 5,0m/s.
08. Aps o tanque ter sido lanado, a quantidade de
movimento do sistema constitudo pelos dois
astronautas e o tanque permanece nula.
16. Imediatamente aps agarrar o tanque, o astro-nauta B
passa a se deslocar com velocidade de mdulo igual a
0,5 m/s.

15. (UFSC) Durante as festividades comemorativas da
Queda da Bastilha, na Frana, realizadas em 14 de julho de
2005, foram lanados fogos de artifcio em homenagem ao
Brasil. Durante os fogos, suponha que um rojo com
defeito, lanado obliquamente, tenha explodido no ponto
mais alto de sua trajetria, partindo-se em apenas dois
pedaos que, imediatamente aps a exploso, possuam
quantidades de movimento
1
p
&
e
2
p
&
.
Considerando-se que todos os movimentos ocorrem em um
mesmo plano vertical, assinale a(s) proposio(es) que
apresenta(m) o(s) par(es) de vetores
1
p
&
e
2
p
&
fisicamente
possvel(eis).

01.


02.



04.



08.




16.













A B
1
p
&
2
p
&
1
p
&
2
p
&
1
p
&
2
p
&
1
p
&
2
p
&
2
p
&

0
1
= p
Please purchase PDF Split-Merge on www.verypdf.com to remove this watermark.
Fsica A Incluso para a Vida

Pr-Vestibular da UFSC 26
UNIDADE 15

AS LEIS DE KEPLER

PRIMEIRA LEI DE KEPLER
Cada planeta gira em torno do Sol
em trajetria elptica, de modo que o
Sol fica em um dos focos da elipse.
O ponto de maior aproximao com
o Sol se chama PERILIO,
enquanto que o de maior
aproximao se chama AFLIO.

SEGUNDA LEI DE KEPLER
O segmento de reta que liga o Sol
a um planeta descreve uma rea
que proporcional ao tempo de
percurso.
Assim, a velocidade escalar de
um planeta no constante: quanto mais longe do Sol
(Aflio), menor a velocidade.

TERCEIRA LEI DE KEPLER
A distncia entre o perilio e o
aflio chamada de eixo maior
da elipse. Assim, a distncia
mdia R tambm chamada de
semi-eixo maior da elipse.
H casos em que a elipse
muito pouco achatada, sendo praticamente uma
circunferncia. o caso, por exemplo, dos planetas Vnus
e Netuno. Nesses casos, o raio mdio R o prprio raio da
circunferncia. Os clculos de Kepler nos leva concluso
de que:




SATLITES DE UM PLANETA

Mais tarde, usando a lei da gravitao
de Newton (que veremos na prxima
aula) foi possvel demonstrar que as
leis de Kepler valem para qualquer
sistema em que temos um corpo de
massa muito "grande" em torno do
qual giram corpos de massas
"pequenas". o caso, por exemplo, de um planeta e seus
satlites.

Exerccios de Sala #

1. (UERJ) A figura ilustra o movimento de um planeta em
torno do sol.
Se os tempos gastos para o
planeta se deslocar de A para
B, de C para D e de E para F
so iguais, ento as reas A
1
,
A
2
, e A
3
- apresentam a
seguinte relao:
a) A
1
= A
2
= A
3
b) A
1
> A
2
= A
3

c) A
1
< A
2
< A
3
d) A
1
> A
2
> A
3

2. (UNIRIO) Um satlite de telecomunicaes est em sua
rbita ao redor da Terra com perodos T. Uma viagem do
nibus Espacial far a instalao de novos equipamentos
nesse satlite, o que duplicar sua massa em relao ao
valor original. Considerando que permanea com a mesma
rbita, seu novo perodo T' ser:
a) T' = 9T c) T' = T e) T' = 1/9T
b) T' = 3T d) T' = 1/3T

Tarefa Mnima #

3. (UFMG) A figura a seguir representa a rbita elptica de
um cometa em trono do sol.
Com relao aos mdulos das velocidades desse cometa
nos pontos I e J, v
i
e v
j
, e aos mdulos das aceleraes
nesses mesmos pontos, a
i
e a
j
, pode-se afirmar que
a) v
i
< v
j
e a
i
< a
j

b) v
i
< v
j
e a
i
> a
j

c) v
i
= v
j
e a
i
= a
j

d) v
i
> v
j
e a
i
< a
j

e) v
i
> v
j
e a
i
> a
j


4. (UFF) Os eclipses solar e lunar - fenmenos
astronmicos que podem ser observados sem a utilizao
de instrumentos pticos - ocorrem sob determinadas
condies naturais. A poca de ocorrncia, a durao e as
circunstncias desses eclipses dependem da geometria
varivel do sistema Terra-Lua-Sol.
Nos eclipses solar e lunar as fases da Lua so,
respectivamente,
a) minguante e nova. d) nova e cheia.
b) minguante e crescente. e) cheia e cheia.
c) cheia e minguante.

5. (ITA) Estima-se que, em alguns bilhes de anos, o raio
mdio da rbita da Lua estar 50% maior do que
atualmente. Naquela poca, seu perodo, que hoje de 27,3
dias, seria:
a) 14,1 dias. c) 27,3 dias. e) 50,2 dias.
b) 18,2 dias. d) 41,0 dias.

6. (UFMG) Suponha que a massa da lua seja reduzida
metade do seu valor real, sem variar o seu volume.
Suponha ainda que ela continue na mesma rbita em torno
da terra. Nessas condies, o perodo de revoluo da lua,
T(lua), em torno da terra, e a acelerao da gravidade na
lua, g(lua), ficariam:
a) T(lua) aumentado e g(lua) aumentada.
b) T(lua) diminudo e g(lua) diminuda.
c) T(lua) inalterado e g(lua) aumentada.
d) T(lua) inalterado e g(lua) diminuda.
e) T(lua) inalterado e g(lua) inalterada.

7. (UNITAU) Um satlite
artificial S descreve uma rbita
elptica em torno da Terra, sendo
que a Terra est no foco,
conforme a figura adiante. Indique a alternativa correta:
a) A velocidade do satlite sempre constante.
b) A velocidade do satlite cresce medida que o satlite
caminha ao longo da curva ABC.
c) A velocidade do ponto B mxima.
d) A velocidade do ponto D mnima.
e) A velocidade tangencial do satlite sempre nula.
Please purchase PDF Split-Merge on www.verypdf.com to remove this watermark.
Incluso para a Vida Fsica A

Pr-Vestibular da UFSC 27
8. (UFRJ) Um satlite geoestacionrio, portanto com
perodo igual a um dia, descreve ao redor da Terra uma
trajetria circular de raio R. Um outro satlite, tambm em
rbita da Terra, descreve trajetria circular de raio R/2.
Calcule o perodo desse segundo satlite.

Tarefa Complementar #

9. (UFSC) Sobre as leis de Kepler, assinale a(s)
proposio(es) verdadeira(s) para o sistema solar.
01. O valor da velocidade de revoluo da Terra, em torno
do Sol, quando sua trajetria est mais prxima do Sol,
maior do que quando est mais afastada do mesmo.
02. Os planetas mais afastados do Sol tm um perodo de
revoluo, em torno do mesmo, maior que os mais
prximos.
04. Os planetas de maior massa levam mais tempo para dar
uma volta em torno do Sol, devido sua inrcia.
08. O Sol est situado num dos focos da rbita elptica de
um dado planeta.
16. Quanto maior for o perodo de rotao de um dado
planeta, maior ser o seu perodo de revoluo em
torno do Sol.
32. No caso especial da Terra, a rbita exatamente uma
circunferncia.

10. (UFSC) Durante aproximados 20 anos, o astrnomo
dinamarqus Tycho Brahe realizou rigorosas observaes
dos movimentos planetrios, reunindo dados que serviram
de base para o trabalho desenvolvido, aps sua morte, por
seu discpulo, o astrnomo alemo Johannes Kepler (1571-
1630). Kepler, possuidor de grande habilidade matemtica,
analisou cuidadosamente os dados coletados por Tycho
Brahe, ao longo de vrios anos, tendo descoberto trs leis
para o movimento dos planetas. Apresentamos, a seguir, o
enunciado das trs leis de Kepler.
1
a
lei de Kepler: Cada planeta descreve uma rbita elptica
em torno do Sol, da qual o Sol ocupa um dos focos.
2
a
lei de Kepler: O raio-vetor (segmento de reta
imaginrio que liga o Sol ao planeta) varre reas iguais,
em intervalos de tempo iguais.
3
a
lei de Kepler: Os quadrados dos perodos de translao
dos planetas em torno do Sol so proporcionais aos cubos
dos raios mdios de suas rbitas.
Assinale a(s) proposio(es) que apresenta(m)
concluso(es) correta(s) das leis de Kepler:
01. A velocidade mdia de translao de um planeta em
torno do Sol diretamente proporcional ao raio mdio
de sua rbita.
02. O perodo de translao dos planetas em torno do Sol
no depende da massa dos mesmos.
04. Quanto maior o raio mdio da rbita de um planeta em
torno do Sol, maior ser o perodo de seu movimento.
08. A 2
a
lei de Kepler assegura que o mdulo da velocidade
de translao de um planeta em torno do Sol
constante.
16. A velocidade de translao da Terra em sua rbita
aumenta medida que ela se aproxima do Sol e diminui
medida que ela se afasta.
32. Os planetas situados mesma distncia do Sol devem
ter a mesma massa.
64. A razo entre os quadrados dos perodos de translao
dos planetas em torno do Sol e os cubos dos raios
mdios de suas rbitas apresentam um valor constante.
UNIDADE 16

GRAVITAO UNIVERSAL

LEI DE NEWTON PARA A GRAVITAO

Dadas duas partculas de massas m
A
e m
B
, separadas por
uma distncia d, existe entre elas um par de foras de
atrao cujo mdulo dado por:


No qual G uma constante, chamada constante de
gravitao universal e cujo valor no SI :


SATLITE ESTACIONRIO

Chamamos de satlite estacionrio (ou geoestacionrio)
um satlite que gira em torno da Terra de modo que, para
um observador na Terra, o satlite parece estar parado.
Para que isso ocorra, a rbita do satlite deve estar no
plano do Equador, e seu perodo de
translao (T) deve ser igual ao
perodo de rotao da Terra.
T = 24h = 86 400s
Os satlites estacionrios so
utilizados para as transmisses de TV
e telefonia a longas distncias. O
sinal enviado ao satlite e deste para outro ponto da
Terra.

ACELERAO DA GRAVIDADE
Consideremos uma partcula de massa m a
uma distncia d do centro da Terra. Essa
partcula ser atrada pela Terra com uma
fora de intensidade F dada por

No qual M a massa da Terra. Essa fora o peso do
corpo, e assim, podemos escrever
F = P = mg
Onde g a acelerao da gravidade. Assim:

Vemos ento que o valor da acelerao da
gravidade diminui com o aumento de d: quanto mais
afastados da Terra estivermos, menor o valor de g.
Para um ponto situado prximo da superfcie da
Terra, o valor de d aproximadamente igual ao raio R da
Terra. Assim, o valor de g prximo da superfcie (g
s
)
dado por:


Quando fazemos a medida de g obtemos valores
diferentes em diferentes pontos da superfcie da Terra. Isso
ocorre por vrios motivos.
Please purchase PDF Split-Merge on www.verypdf.com to remove this watermark.
Fsica A Incluso para a Vida

Pr-Vestibular da UFSC 28
Um dos motivos que a Terra no esfrica e
nem homognea. Outro motivo a rotao da Terra. Por
causa da mesma, h uma pequena tendncia de os corpos
serem expelidos para fora da Terra (devido inrcia).
Assim, mesmo que a Terra fosse rigorosamente esfrica e
homognea o valor medido de g iria variar com a latitude.
Desse modo, o valor medido de g mximo nos plos e
mnimo no equador.

Exerccios de Sala #
1. (UNESP) A fora gravitacional entre um satlite e a
Terra F. Se a massa desse satlite fosse quadruplicada e a
distncia entre o satlite e o centro da Terra aumentasse
duas vezes, o valor da fora gravitacional seria
a) F/4. c) 3F/4. e) 2F.
b) F/2. d) F.

2. (UFMG) O Pequeno Prncipe, do livro de mesmo nome,
de Antoine de Saint-Exupry, vive em um
asteride pouco maior que esse personagem,
que tem a altura de uma criana terrestre.
Em certo ponto desse asteride, existe uma
rosa, como ilustrado na figura ao lado:
Aps observar essa figura, Jlia formula as
seguintes hipteses:
I - O Pequeno Prncipe no pode ficar de p ao lado da
rosa, porque o mdulo da fora gravitacional menor
que o mdulo do peso do personagem.
II - Se a massa desse asteride for igual da Terra, uma
pedra solta pelo Pequeno Prncipe chegar ao solo antes
de uma que solta na Terra, da mesma altura.
Analisando essas hipteses, podemos concluir que
a) apenas a I est correta.
b) apenas a II est correta.
c) as duas esto corretas.
d) nenhuma das duas est correta.

3. (PUC-MG) Dois corpos A e B, de massas 16M e M,
respectivamente, encontram-se no vcuo e esto separadas
por uma certa distncia. Observa-se que outro corpo, de
massa M, fica em repouso quando colocado no ponto P,
conforme a figura. A razo x/y entre as distncias
indicadas igual a:

a) 2 b) 4
c) 6 d) 8
e) 16

4. (Unicamp) A atrao gravitacional da Lua e a fora
centrfuga do movimento conjunto de rotao da Lua e da
Terra so as principais causas do fenmeno das mars.
Essas foras fazem com que a gua dos oceanos adquira a
forma esquematizada (e exagerada) na figura adiante. A
influncia do Sol no fenmeno das mars bem menor,
mas no desprezvel, porque quando a
atrao do Sol e da Lua se conjugam a
mar se torna mais intensa.
a) Quantas mars altas ocorrem em
um dia em um mesmo local?
b) Como estar a mar no Brasil quando a Lua estiver bem
acima do Japo?
c) Faa um desenho mostrando a Terra, a Lua e o Sol na
situao em que a mar mais intensa. Qual a fase da
Lua nessa situao?
Tarefa Mnima #

5. (ACAFE) A imprensa comentava, antes das Olimpadas
de Sydney, que os atletas teriam uma maior dificuldade em
quebrar alguns recordes olmpicos, como os do arremesso
de peso, do salto em distncia e do salto em altura. Do
ponto de vista da Fsica, o comentrio da imprensa se
baseava:
a) Na alimentao dos atletas em Sydney.
b) No clima australiano.
c) Na longitude de Sydney.
d) Na diferena de fuso-horrio.
e) Na latitude de Sydney.

6. (ACAFE) A distncia do centro da Terra Lua ,
aproximadamente, 60 vezes o raio da Terra. Sendo g
T
o
valor da acelerao da gravidade da Terra na sua
superfcie, a acelerao da gravidade da Terra num ponto
da rbita da Lua ser de, aproximadamente:
a) g
T
/60 c) 60g
T
e) 6g
T

b) g
T
/3600 d) g
T
/6


7. (ACAFE) Certa vez, um mineiro, estando no extremo
sul do Chile, enviou para So Paulo, por meio de um
amigo, uma determinada quantidade de ouro,
cuidadosamente pesada numa balana de molas. Quando o
ouro foi entregue, pesava menos do que antes e o amigo foi
preso por furto. Considerando que os dois locais esto na
mesma altitude, pode-se afirmar que a priso foi:

a) Justa, pois o ouro deveria ter peso maior em So Paulo.
b) Injusta, pois a acelerao da gravidade menor no
extremo sul do Chile do que em So Paulo.
c) Justa, pois a massa de ouro entregue foi menor.
d) Justa, pois o ouro deveria ter o mesmo peso nos dois
locais.
e) Injusta, pois a acelerao da gravidade maior no
extremo sul do Chile do que em So Paulo.

8. (UFC) Considere duas massas puntiformes sob ao de
fora gravitacional mtua. Assinale a alternativa que
contm a melhor representao grfica da variao do
mdulo da fora gravitacional sobre uma das massas, em
funo da distncia entre ambas.


9. (PUC-PR) O movimento planetrio comeou a ser
compreendido matematicamente no incio do sculo XVII,
quando Johannes Kepler enunciou trs leis que descrevem
como os planetas se movimentam ao redor do Sol,
baseando-se em observaes astronmicas feitas por
Tycho Brahe. Cerca de cinquenta anos mais tarde, lsaac
Newton corroborou e complementou as leis de Kepler com
sua lei de gravitao universal.
Assinale a alternativa, dentre as seguintes, que no est de
acordo com as ideias de Kepler e Newton:

Please purchase PDF Split-Merge on www.verypdf.com to remove this watermark.
Incluso para a Vida Fsica A

Pr-Vestibular da UFSC 29
a) A fora gravitacional entre os corpos sempre atrativa.
b) As trajetrias dos planetas so elipses, tendo o Sol como
um dos seus focos.
c) O quadrado do perodo orbital de um planeta
proporcional ao cubo de sua distncia mdia ao Sol.
d) A fora gravitacional entre duas partculas diretamente
proporcional ao produto de suas massas e inversamente
proporcional ao cubo da distncia entre elas.
e) Ao longo de uma rbita, a velocidade do planeta,
quando ele est mais prximo ao Sol (perilio), maior
do que quando ele est mais longe dele (aflio).

10. (UFRN) O turismo chegou ao espao! No dia
30/04/2001, o primeiro turista espacial da histria, o norte-
americano Denis Tito, a um custo de 20 milhes de
dlares, chegou Estao Espacial Internacional, que est
se movendo ao redor da Terra. Ao mostrar o turista
flutuando dentro da estao, um reprter erroneamente
disse: "O turista flutua devido ausncia de gravidade".
A explicao correta para a flutuao do turista :
a) A fora centrpeta anula a fora gravitacional exercida
pela Terra.
b) Na rbita da estao espacial, a fora gravitacional
exercida pela Terra nula.
c) A estao espacial e o turista esto com a mesma
acelerao, em relao Terra.
d) Na rbita da estao espacial, a massa inercial do turista
nula.

11. (Sobral) O grupo Paralamas do Sucesso gravou h
algum tempo uma bela msica chamada "Tendo a Lua".

Tendo a Lua

Hoje joguei tanta coisa fora
Vi o meu passado passar por mim
Cartas e fotografias, gente que foi embora
A casa fica bem melhor assim
O cu de caro tem mais poesia que o de Galileu
E lendo teus bilhetes, eu penso no que fiz
Querendo ver o mais distante e sem saber voar
Desprezando as asas que voc me deu
Tendo a Lua aquela gravidade aonde o homem flutua
Merecia a visita no de militares, mas de bailarinos e de
voc e eu.
(CD Acstico MTV Paralamas do Sucesso, 1999 - EMI)

Do ponto de vista da Fsica, analise a letra da msica e
verifique as afirmaes a seguir, assinalando a verdadeira:

a) Na Lua, um homem pode realmente flutuar, pois no h
gravidade.
b) A gravidade prpria da Lua na sua superfcie cerca de
1/6 da gravidade prpria da Terra na sua superfcie.
Assim, um homem que pesa 900 N na Terra (onde g =
10m/s
2
), na Lua ter peso aproximado de 150N.
c) O homem flutua ao caminhar na Lua porque no satlite a
sua massa diminui.
d) Est errado dizer que na Lua o homem flutua, pois l
no existe atmosfera.
e) A acelerao da gravidade da Lua cerca de 6 vezes
maior que a acelerao da gravidade da Terra,
entretanto, neste satlite da Terra, a massa do homem
no varia, fazendo com que seu peso permanea
sempre constante.

Tarefa Complementar #

12. (UFSC) Um satlite
artificial, de massa m, descreve
uma rbita circular de raio R em
torno da Terra, com velocidade
orbital v
&
de valor constante,
conforme representado
esquematicamente na figura.
(Desprezam-se interaes da
Terra e do satlite com outros
corpos.)
Considerando a Terra como referencial na situao
descrita, assinale a(s) proposio(es) correta(s):

01. O satlite sofre a ao da fora gravitacional exercida
pela Terra, de mdulo igual a
2
R
Mm
G F
G
=
, onde G a
constante de gravitao universal e M a massa da
Terra.
02. Para um observador na Terra, o satlite no possui
acelerao.
04. A fora centrpeta sobre o satlite igual fora
gravitacional que a Terra exerce sobre ele.
08. A acelerao resultante sobre o satlite tem a mesma
direo e sentido da fora gravitacional que atua sobre
ele.
16. A acelerao resultante sobre o satlite independe da
sua massa e igual a
2
R
M
G
, onde G a constante de
gravitao universal e M a massa da Terra.
32. A fora exercida pelo satlite sobre a Terra tem
intensidade menor do que aquela que a Terra exerce
sobre o satlite; tanto que o satlite que orbita em
torno da Terra e no o contrrio.

13. (UFSC) A figura abaixo representa a trajetria de um
planeta em torno do Sol. Esta trajetria elptica e os
segmentos de reta entre os pontos A e B e entre C e D so,
respectivamente, o eixo maior e o eixo menor da elipse.
Esta figura est fora de escala, pois a excentricidade das
rbitas planetrias pequena e as suas trajetrias se
aproximam de circunferncias.







A tabela abaixo apresenta dados astronmicos
aproximados de alguns planetas:

DISTNCIA MDIA
AO SOL
MASSA
RAIO
MDIO
Terra d
TS
m
T
R
T

Saturno 10 d
TS
95 m
T
9 R
T

Urano 20 d
TS
14 m
T
4 R
T

Netuno 30 d
TS
17 m
T
4 R
T

d
TS
: distncia mdia da Terra ao Sol
m
T
: massa da Terra
R
T
: raio da Terra

Assinale a(s) proposio(es) correta(s).
A B
C
D
Sol
m
v
&
M
Please purchase PDF Split-Merge on www.verypdf.com to remove this watermark.
Fsica A Incluso para a Vida

Pr-Vestibular da UFSC 30
01. O mdulo da velocidade de um planeta quando passa
por A maior do que quando passa por B.
02. O perodo de Urano cerca de 2,8 vezes o perodo de
Saturno.
04. O perodo de Netuno de aproximadamente 52 anos.
08. O mdulo da fora mdia que o Sol exerce sobre
Saturno cerca de nove vezes maior que o mdulo da
fora mdia que o Sol exerce sobre a Terra.
16. O mdulo da fora que Urano exerce sobre um corpo
na sua superfcie aproximadamente quatro vezes
maior que o mdulo da fora que a Terra exerce sobre
este corpo na sua superfcie.

UNIDADE 17 E 18

ESTTICA

EQUILBRIO ESTTICO DO PONTO MATERIAL

Considere o ponto O onde esto aplicadas as seguintes
foras:

Para que o ponto O esteja em equilbrio esttico (repouso),
necessrio que a fora resultante que atua sobre este
ponto seja nula ( 0
& &
=
R
F ).

Mtodo do Polgono
Fechado

Para que a fora resultante
seja nula, somam-se os vetores
pelo mtodo da linha
poligonal e a figura encontrada dever ser um polgono
fechado. Para o exemplo acima, teremos:

Teorema de Lamy (Lei dos senos)



Mtodo das Decomposies

F
1x
= F
1
.cos u
0
& &
=
Rx
F

F
1y
= F
1
.sen u
0
& &
=
Ry
F

Equilbrio Esttico do Corpo Extenso
Para os corpos extensos, podem-se ter movimentos de
translao e rotao. Para o movimento de translao, a
condio de equilbrio que a fora resultante aplicada
seja nula ( 0
& &
=
R
F ). Para o movimento de rotao,
necessrio que a soma dos momentos das foras que atuam
neste corpo (torques) seja zero (
_
= 0
O
F
M ).

Momento de uma Fora (ou Torque)

a grandeza relacionada com o movimento de rotao de
um corpo extenso.

Onde:
O plo
d brao de alavanca
r reta suporte da fora F

[N.m]

O momento ser positivo quando o corpo girar no sentido
anti-horrio e negativo quando o corpo girar no sentido
horrio.

Condio de Equilbrio de Rotao

1) Identificar todas as foras que atuam no corpo extenso
(se for para considerar o peso do corpo, ele dever estar
concentrado no centro de massa do objeto que, para corpos
homogneos e simtricos, estar localizado no centro do
corpo);
2) Escolher a posio do plo (Dica: considere o plo num
local por onde passa uma fora que voc no conhece e
no quer calcular);
3) Calcular o momento de cada fora em relao ao plo
escolhido (Cuidado para no mudar o plo de posio);
4) Somar todos os momentos e igualar a zero. A partir da,
voc ter uma equao com uma nica varivel. Isole-a e
calcule o que se pede.

Exerccios de Sala #

1. (FUVEST) Um bloco de peso P suspenso por dois fios
de massa desprezvel, presos a paredes em A e B, como
mostra a figura adiante. Pode-se afirmar que o mdulo da
fora que tenciona o fio preso em B, vale:

a) P/2.
b) P/ 2 .
c) P.
d) 2 P.
e) 2 P.

2. (Mackenzie) No esquema representado, o homem
exerce sobre a corda uma fora de 120 N e o sistema ideal
se encontra em equilbrio. O peso da carga Q :
Please purchase PDF Split-Merge on www.verypdf.com to remove this watermark.
Incluso para a Vida Fsica A

Pr-Vestibular da UFSC 31

a) 120N.
b) 200N.
c) 240N.
d) 316N.
e) 480N.


3. (UDESC) Um paciente, em um
programa de reabilitao de uma leso
de joelho, executa exerccios de
extenso de joelho usando um sapato
de ferro de 15N. Calcule,
JUSTIFICANDO seu raciocnio passo
a passo, at atingir o resultado:
a) A massa do sapato de ferro;
b) A quantidade de torque gerado no joelho pelo sapato de
ferro, nas posies (1) e (2), mostradas na figura, sabendo
que a distncia entre o centro de gravidade do sapato de
ferro e o centro articular do joelho 0,4 metros.

4. (Cesgranrio) Um fio, cujo
limite de resistncia de 25N,
utilizado para manter em
equilbrio, na posio
horizontal, uma haste de metal,
homognea, de comprimento
AB=80cm e peso=15N. A barra
fixa em A, numa parede, atravs de uma articulao,
conforme indica a figura.
A menor distncia x, para a qual o fio manter a haste em
equilbrio, :
a) 16cm c) 30cm e) 40cm
b) 24cm d) 36cm

5. (UFPE) Uma tbua uniforme de 3m de comprimento
usada como gangorra por duas crianas com massas 25kg e
54kg. Elas sentam sobre as extremidades
da tbua de modo que o sistema
fica em equilbrio quando
apoiado em uma pedra distante
1,0m da criana mais pesada.
Qual a massa, em kg, da tbua?
Dado: g = 10 m/s
2


Tarefa Mnima #

6. (Cesgranrio) Na figura a seguir, uma esfera rgida se
encontra em equilbrio, apoiada em uma parede vertical e
presa por um fio ideal e inextensvel. Sendo P o peso da
esfera e 2P a fora mxima que o fio suporta antes de
arrebentar, o ngulo formado entre a parede e o fio de:

a) 30
b) 45
c) 60
d) 70
e) 80

7. (FAAP) Na estrutura representada, a barra homognea
AB pesa 40N e articulada em A.
A carga suspensa pesa 60N. A trao no cabo vale:


a) 133,3 N
b) 33,3 N
c) 166,6 N
d) 66,6 N
e) 199,9 N

8. (Mackenzie) Um corpo, que est sob a ao de 3 foras
coplanares de mesmo mdulo, est em equilbrio. Assinale
a alternativa na qual esta situao possvel.


9. (Unirio)
Na figura ao lado, o corpo
suspenso tem o peso 100N.
Os fios so ideais e tm
pesos desprezveis, o sistema
est em equilbrio esttico
(repouso). A trao na corda
AB, em N, : (Dados:
g=10m/s
2
; sen30=0,5 e cos30= 2 3 ).
a) 20 b) 40 c) 50 d) 80 e) 100

10. (Fatec) Uma pequena esfera de massa igual a 4,0 g,
carregada eletricamente, est suspensa por uma corda. Sob
a ao de uma fora eltrica horizontal, a corda se desloca
at que atinge o equilbrio ao formar um ngulo de 37
com a vertical. Sabendo que cos 37 = 0,80 e sen 37 =
0,60, a intensidade da fora eltrica e a tenso na corda
so, respectivamente:

a) 70 N e 56 N
b) 30 N e 50 N
c) 7,0 N e 5,6 N
d) 3,0 N e 5,0 N
e) 3,0 x 10
-2
N e 5,0 x
10
-2
N

11. (FEI) A barra a seguir homognea da seo
constante e est apoiada nos pontos A e B. Sabendo-se que
a reao no apoio A R
A
=200kN, que F
1
=100kN e
F
2
=500kN, qual o peso da barra?


a) 300 kN
b) 200 kN
c) 100 kN
d) 50 kN
e) 10 kN




Please purchase PDF Split-Merge on www.verypdf.com to remove this watermark.
Fsica A Incluso para a Vida

Pr-Vestibular da UFSC 32
12. (Cesgranrio) Cristiana e Marcelo namoram em um
balano constitudo por um assento horizontal de madeira
de peso desprezvel e preso ao teto por duas cordas
verticais. Cristiana pesa 4,8 10
2
N e Marcelo, 7,0 10
2
N.
Na situao descrita na figura, o balano est parado, e os
centros de gravidade da moa e do rapaz distam 25cm e
40cm, respectivamente, da corda que, em cada caso, est
mais prxima de cada um. Sendo de 1,00m a distncia que
separa as duas cordas, qual a tenso em cada uma delas?


a) Cristiana: 1,6 10
2
N e
Marcelo: 10,2 10
2
N
b) Cristiana: 3,2 10
2
N e
Marcelo: 8,6 10
2
N
c) Cristiana: 4,0 10
2
N e
Marcelo: 7,8 10
2
N
d) Cristiana: 4,8 10
2
N e
Marcelo: 7,0 10
2
N
e) Cristiana: 6,4 10
2
N e
Marcelo: 5,4 10
2
N

13. (PUC-Camp) Trs blocos de massas iguais so
pendurados no teto atravs de dois fios que passam
livremente pelas argolas 1 e 2. Considerando desprezveis
as massas dos fios e as eventuais foras de atrito, o sistema
pode oscilar. Durante a oscilao, a acelerao dos corpos
ser nula quando o ngulo o indicado na figura for:

a) maior que 120
b) igual a 120
c) igual a 90
d) igual a 60
e) menor que 60


14. (UFSM)
Uma barra homognea e
horizontal de 2m de
comprimento e 10kg de
massa tem uma
extremidade apoiada e a outra suspensa por um fio ideal,
conforme a figura. Considerando a acelerao
gravitacional como 10m/s
2
, o mdulo da tenso no fio (T,
em N) :
a) 20 c) 50 e) 200
b) 25 d) 100

15. (UFRJ) A figura mostra uma garrafa mantida em
repouso por dois suportes A e B. Na situao considerada a
garrafa est na horizontal e os suportes exercem sobre ela
foras verticais. O peso da garrafa e seu contedo tem um
mdulo igual a 1,4kgf e seu centro de massa C se situa a
uma distncia horizontal D=18cm do suporte B.

Sabendo que a distncia
horizontal entre os suportes A e
B d=12cm, determine o
sentido da fora que o suporte
A exerce sobre a garrafa e
calcule seu mdulo.





Tarefa Complementar #

16. (UFSC) A figura abaixo mostra as foras de mdulos
Q = 10N, R = 70N, S = 20N e T = 40N que atuam
sobre uma barra homognea, com peso de mdulo 30N e
com 2m de comprimento, que tende a girar em torno do
ponto O. Assinale a(s) proposio(es) verdadeira(s).

01. O momento da fora T em relao ao ponto O igual a
zero.
02. O momento da fora S em relao ao ponto O igual
ao momento da fora R em relao ao ponto O.
04. O momento da fora Q em relao ao ponto O tem
mdulo igual a 20N.m.
08. O momento do peso da barra em relao ao ponto O
igual ao momento da fora R em relao ao ponto O.
16. A barra est em equilbrio de rotao.
32. O momento resultante em relao ao ponto O nulo.

17. (UFSC) O andaime suspenso (figura 1), conhecido
como mquina pesada ou trec trec, indicado para servios
de revestimento externo, colocao de pastilhas, mrmores,
cermicas e servios de pedreiro. Um dispositivo situado
no andaime permite que o pedreiro controle o sistema de
polias para se movimentar verticalmente ao longo de um
prdio. A figura 2 mostra um andaime homogneo
suspenso pelos cabos A, B, C e D, que passam por polias
situadas no topo do edifcio e formam ngulos de 90 com
o estrado do andaime.

Figura 1









Chama-se: o peso do andaime de
A
P
&
, e o seu mdulo de
A
P ; o peso de um pedreiro que est no andaime de
P
P
&
, e
o seu mdulo
P
P
; as tenses exercidas pelos cabos A, B, C
e D no andaime de
A
T
&
,
B
T
&
,
C
T
&
e
D
T
&
, e seus mdulos de
T
A
, T
B
,
C
T
e
D
T
, respectivamente.
Considerando que o segmento de reta auxiliar ST passa
pelo centro do estrado o dividindo em duas partes de
comprimentos iguais e que o andaime no apresenta
qualquer movimento de rotao, assinale a(s)
proposio(es) correta(s).

Cabo D Cabo A Cabo B Cabo C
Estrado
S
T
lado
esquerdo
lado
direito
Figura 2
Please purchase PDF Split-Merge on www.verypdf.com to remove this watermark.
Incluso para a Vida Fsica A

Pr-Vestibular da UFSC 33
01. T
A
+ T
B
+
C
T
+
D
T = P
A
+ P
P
somente se o andaime
estiver em repouso.
02.
A
T
&
+
B
T
&
+
C
T
&
+
D
T
&
= (
A
P
&
+
P
P
&
) se o andaime
estiver descendo e acelerando.
04. T
A
+ T
B
=
C
T +
D
T se o pedreiro estiver sobre o
segmento de reta ST do estrado do andaime e o
andaime estiver em movimento uniforme na vertical.
08.
C
T +
D
T > T
A
+ T
B
somente se o pedreiro estiver
mais prximo da extremidade direita do estrado do
andaime, independentemente do andaime estar em
movimento na vertical.
16. Se o pedreiro estiver mais prximo da extremidade
esquerda do estrado do andaime e o andaime estiver em
repouso, ento T
A
+ T
B
> T
C
+ T
D
.

UNIDADE 19 E 20

HIDROSTTICA I

MASSA ESPECFICA X DENSIDADE
A massa especfica () de uma substncia a razo entre a
massa (m) de uma quantidade da substncia e o volume
(V) correspondente:

Uma unidade muito usual para a massa especfica o
g/cm
3
, mas no SI a unidade o kg/m
3
. A relao entre elas
a seguinte:


Observao:
comum encontrarmos o termo densidade (d) em lugar de
massa especfica (). Usa-se "densidade" para representar
a razo entre a massa e o volume de objetos slidos (ocos
ou macios), e "massa especfica" para fluidos.

PRESSO
Consideremos uma fora aplicada perpendicularmente a
uma superfcie com rea A. Definimos a presso (p)
aplicada pela fora sobre a rea pela seguinte relao:

No SI, a unidade de presso o pascal (Pa) que
corresponde a N/m
2
.
O conceito de presso nos permite entender muitos dos
fenmenos fsicos que nos rodeiam. Por exemplo, para
cortar um pedao de po, utilizamos o lado afiado da faca
(menor rea), pois, para uma mesma fora, quanto menor a
rea, maior a presso produzida.

Presso Hidrosttica Princpio de Stevin
"A diferena entre as presses em dois pontos
considerados no seio de um lquido em equilbrio (presso
no ponto mais profundo e a presso no ponto menos
profundo) vale o produto da massa especifica do lquido
pelo mdulo da acelerao da gravidade do local onde
feita a observao, pela diferena entre as profundidades
consideradas.

p
A
p
B
= g h
A partir do Teorema de Stevin podemos concluir:
A presso aumenta com a profundidade. Para pontos
situados na superfcie livre, a presso correspondente
igual exercida pelo gs ou ar sobre ela. Se a superfcie
livre estiver ao ar atmosfrico, a presso correspondente
ser a presso atmosfrica, p
atm
.
Pontos situados em um mesmo lquido e em uma
mesma horizontal ficam submetidos mesma presso.
A superfcie livre dos lquidos em equilbrio
horizontal.

Presso Atmosfrica e a Experincia de Torricelli

O fsico italiano Evangelista
Torricelli (1608-1647) realizou
uma experincia para
determinar a presso
atmosfrica ao nvel do mar.
Ele usou um tubo de
aproximadamente 1,0m de
comprimento, cheio de
mercrio (Hg) e com a
extremidade tampada.

Depois, colocou o tubo, em p e com a boca tampada para
baixo, dentro de um recipiente que tambm continha
mercrio. Torricelli observou que, aps destampar o tubo,
o nvel do mercrio desceu e se estabilizou na posio
correspondente a 76 cm, restando o vcuo na parte vazia
do tubo.
A presso no ponto A igual presso no ponto B. Assim:
p
B
= p
A
p
ATM
= p
coluna(Hg)

p
ATM
= 76cmHg = 760mmHg = 1,01x10
5
Pa

Exerccios de Sala #

1. (FAAP) A massa de um bloco de granito 6,5t e a
densidade do granito 2.600kg/m
3
. Qual o volume do
bloco?
a) 0,0025 m
3
c) 0,25 m
3
e) 25,00 m
3
b) 0,025 m
3
d) 2,50 m
3

2. (VUNESP) Um tijolo, com as dimenses indicadas,
colocado sobre uma mesa com tampo de borracha,
inicialmente da maneira mostrada em 1 e, posteriormente,
na maneira mostrada em 2.

Please purchase PDF Split-Merge on www.verypdf.com to remove this watermark.
Fsica A Incluso para a Vida

Pr-Vestibular da UFSC 34
Na situao 1, o tijolo exerce sobre a mesa uma fora F
1
e
uma presso p
1
; na situao 2, a fora e a presso exercidas
so F
2
e p
2
.
Nessas condies, pode-se afirmar que:
a) F
1
= F
2
e p
1
= p
2
b) F
1
= F
2
e p
1
> p
2

c) F
1
= F
2
e p
1
< p
2
d) F
1
> F
2
e p
1
> p
2

e) F
1
< F
2
e p
1
< p
2


3. (Unicamp) Um mergulhador persegue um peixe a 5,0m
abaixo da superfcie de um lago. O peixe foge da posio
A e se esconde em uma gruta na posio B, conforme
mostra a figura a seguir. A presso atmosfrica na
superfcie da gua igual a P
0
=1,0.10
5
N/m
2
.
Adote g = 10m/s
2
.

a) Qual a presso sobre o mergulhador?
b) Qual a variao de presso sobre o peixe nas posies A
e B?

Tarefa Mnima #
4. (Cesgranrio) Eva possui duas bolsas A e B, idnticas,
nas quais coloca sempre os mesmos objetos. Com o uso
das bolsas, ela percebeu que a bolsa A marcava o seu
ombro. Curiosa, verificou que a largura da ala da bolsa A
era menor do que a da B. Ento, Eva concluiu que:
a) O peso da bolsa B era maior.
b) A presso exercida pela bolsa B, no seu ombro, era
menor.
c) A presso exercida pela bolsa B, no seu ombro, era
maior.
d) O peso da bolsa A era maior.
e) As presses exercidas pelas bolsas so iguais, mais os
pesos so diferentes.

5. (ENEM) A gasolina vendida por litro, mas em sua
utilizao como combustvel, a massa o que importa. Um
aumento da temperatura do ambiente leva a um aumento
no volume da gasolina. Para diminuir os efeitos prticos
dessa variao, os tanques dos postos de gasolina so
subterrneos. Se os tanques no fossem subterrneos:
I - Voc levaria vantagem ao abastecer o carro na hora
mais quente do dia, pois estaria comprando mais massa
por litro de combustvel.
II - Abastecendo com a temperatura mais baixa, voc
estaria comprando mais massa de combustvel para
cada litro.
III - Se a gasolina fosse vendida por kg em vez de por litro,
o problema comercial decorrente da dilatao da
gasolina estaria resolvido.
Destas consideraes, somente
a) I correta. d) I e II so corretas.
b) II correta. e) II e III so corretas.
c) III correta.

6. (UFSM) Um cliente est h muito tempo, de p, numa
fila de Banco, com os dois ps apoiados no solo,
exercendo, assim, certa presso sobre o mesmo.
Levantando uma perna, de modo que apenas um dos ps
toque o solo, a presso que o cliente exerce fica
multiplicada por:
a) 1/4. c) 1. e) 4.
b) 1/2. d) 2.

7. (Unicamp) Um barril de chopp completo, com bomba e
serpentina, como representado na figura a seguir, foi
comprado para uma festa. A bomba utilizada para
aumentar a presso na parte superior do barril forando
assim o chopp pela serpentina. Considere a densidade do
chopp igual da gua.

a) Calcule a mnima presso aplicada pela bomba para que
comece a sair chopp pela primeira vez no incio da
festa (barril cheio at o topo, serpentina inicialmente
vazia).
b) No final da festa o chopp estar terminando. Qual deve
ser a mnima presso aplicada para o chopp sair pela
sada quando o nvel do lquido estiver a 10 cm do
fundo do barril, com a serpentina cheia?

8. (ITA) Um vaso comunicante em forma de U possui
duas colunas da mesma altura h=42,0cm, preenchidas com
gua at a metade. Em seguida, adiciona- se leo de massa
especfica igual a 0,80g/cm
3
a uma das colunas at a
coluna estar totalmente preenchida, conforme a figura B. A
coluna de leo ter comprimento de:


a) 14,0 cm
b) 16,8 cm
c) 28,0 cm
d) 35,0 cm
e) 37,8 cm
9. (PUC-Camp) O grfico adiante mostra a relao
aproximada entre a presso atmosfrica e a altitude do
lugar, comparada ao nvel do mar.

Em uma cidade a 1.000m de altitude, a presso
atmosfrica, em N/m
2
, vale aproximadamente
Dados: Densidade do Hg = 13,6 10
3
kg/m
3
g = 10 m/s
2
a) 7,0 10
4
d) 1,0 10
5
b) 8,0 10
4
e) 1,1 10
5
c) 9,0 10
4

Please purchase PDF Split-Merge on www.verypdf.com to remove this watermark.
Incluso para a Vida Fsica A

Pr-Vestibular da UFSC 35
Tarefa Complementar #

10. (UFSC) Um recipiente cheio de gua at a borda tem
massa total (gua+recipiente) de 1.200g. Coloca-se dentro
do recipiente uma pedra de massa 120g que, ao afundar,
provoca o extravasamento de parte do lquido. Medindo-se
a massa do recipiente com a gua e a pedra, no seu interior,
encontrou-se 1.290g. Calcule o valor da massa especfica
da pedra em g/cm
3
, sabendo que a massa especfica da
gua 1,0g/cm
3
.

11. (UDESC) O nvel da gua em uma represa est a 15,0
m de altura da base. Sabendo-se que a gua est em
repouso e que a presso atmosfrica na superfcie igual a
1,0 x 10
5
N/m
2
, DETERMINE a presso exercida na base
da represa. Dados: massa especfica da gua = = 1,0 x
10
3
kg/m
3
acelerao da gravidade no local = g = 10,0m/s
2

12. (UFSC) Os alunos de uma escola, situada em uma
cidade A, construram um barmetro para comparar a
presso atmosfrica na sua cidade com a presso
atmosfrica de outra cidade, B.
Vedaram uma garrafa muito bem, com uma rolha e um
tubo de vidro, em forma de U, contendo mercrio.
Montado o barmetro, na cidade A, verificaram que a
altura das colunas de mercrio eram iguais nos dois ramos
do tubo, conforme mostra a Figura 1.
O professor os orientou para transportarem o barmetro
com cuidado at a cidade B, a fim de manter a vedao da
garrafa, e forneceu-lhes a Tabela abaixo, com valores
aproximados da presso atmosfrica em funo da altitude.
Ao chegarem cidade B, verificaram um desnvel de 8,0
cm entre as colunas de mercrio nos dois ramos do tubo de
vidro, conforme mostra a Figura 2.

Considerando a situao descrita e que os valores
numricos das medidas so aproximados, face
simplicidade do barmetro construdo, assinale a(s)
proposio(es) correta(s).
01. Na cidade A, as alturas das colunas de mercrio nos
dois ramos do tubo em U so iguais, porque a presso
no interior da garrafa igual presso atmosfrica
externa.
02. A presso atmosfrica na cidade B 8,0 cmHg menor
do que a presso atmosfrica na cidade A.
04. Sendo a presso atmosfrica na cidade A igual a 76
cmHg, a presso atmosfrica na cidade B igual a 68
cmHg.
08. A presso no interior da garrafa praticamente igual
presso atmosfrica na cidade A, mesmo quando o
barmetro est na cidade B.
16. Estando a cidade A situada ao nvel do mar (altitude
zero), a cidade B est situada a mais de 1000 metros de
altitude.
32. Quando o barmetro est na cidade B, a presso no
interior da garrafa menor do que a presso
atmosfrica local.
64. A cidade B se encontra a uma altitude menor do que a
cidade A.
UNIDADE 21

HIDROSTTICA II

PRINCPIO DE PASCAL

O acrscimo de presso produzido num lquido em
equilbrio se transmite integralmente a todos os pontos do
lquido.


Sendo Ap
1
= Ap
2
e lembrando que Ap = F/A , escrevemos:

Como A
2
> A
1
, temos F
2
> F
1
, ou seja, a intensidade da
fora diretamente proporcional rea do tubo. A prensa
hidrulica uma mquina que multiplica a fora aplicada.

Princpio de Arquimedes

Contam os livros que o sbio grego Arquimedes (282-212
AC) descobriu, enquanto tomava banho, que um corpo
imerso na gua se torna mais leve devido a uma fora
exercida pelo lquido sobre o corpo, vertical e para cima,
que alivia o peso do corpo. Essa fora do lquido sobre o
corpo denominada empuxo .
Portanto, num corpo que se encontra imerso em um
lquido, agem duas foras: a fora peso ( ), devido
interao com o campo gravitacional terrestre, e a fora de
empuxo ( ), devido sua interao com o lquido.

Quando um corpo est totalmente imerso em um lquido,
podemos ter as seguintes condies:
* se ele permanece parado no ponto onde foi colocado, a
intensidade da fora de empuxo igual intensidade da
fora peso (E = P);
* se ele afundar, a intensidade da fora de empuxo menor
do que a intensidade da fora peso (E < P); e
* se ele for levado para a superfcie, intensidade da fora
de empuxo maior do que a intensidade da fora peso (E >
P). Para saber qual das trs situaes ir ocorrer, devemos
enunciar o princpio de Arquimedes:
Todo corpo mergulhado num fluido (lquido ou gs)
sofre, por parte do fluido uma fora vertical para cima,
cuja intensidade igual ao peso do fluido deslocado
pelo corpo.
Seja V
f
o volume de fluido deslocado pelo corpo. Ento a
massa do fluido deslocado dada por:

m
f
= d
f
V
f

A intensidade do empuxo igual ao do peso dessa massa
deslocada:
E = m
f
g = d
f
V
f
g
Please purchase PDF Split-Merge on www.verypdf.com to remove this watermark.
Fsica A Incluso para a Vida

Pr-Vestibular da UFSC 36
Para corpos totalmente imersos, o volume de fluido
deslocado igual ao prprio volume do corpo. Neste caso,
a intensidade do peso do corpo e a do empuxo so dadas
por:

P = d
c
V
c
g e E = d
f
V
c
g

Comparando-se as duas expresses, observamos que:

* Se d
c
> d
f
, o corpo desce em movimento acelerado
(F
R
= P E);
* Se d
c
< d
f
, o corpo sobe em movimento acelerado
(F
R
= E P);
* Se d
c
= d
f
, o corpo encontra-se em equilbrio.

Quando um corpo mais denso que um lquido totalmente
imerso nesse lquido, observamos que o valor do seu peso,
dentro desse lquido, aparentemente menor do que no ar.
A diferena entre o valor do peso real e o valor do peso
aparente corresponde ao empuxo exercido pelo lquido:

P
aparente
= P
real
- E

Flutuao
Para um corpo flutuando em um lquido, temos as
condies a seguir.

1) Ele se encontra em equilbrio:
E = P
2) O volume de lquido que ele desloca menor do que o
seu volume:
V
deslocado
< V
corpo
3) Sua densidade menor do que a densidade do lquido:
d
corpo
< d
lquido
4) O valor do peso aparente do corpo nulo:
P
aparente
= P E = O

A relao entre os volumes imersos e o total do corpo
dada por:

E = P d
liquido
V
imerso
g = d
corpo
V
corpo
g



Exerccios de Sala #
1. (Fei-94) No macaco hidrulico, representado na figura a
seguir, sabe-se que as reas das seces transversais dos
vasos verticais so A
1
= 20cm
2
e A
2
= 0,04m
2
. Qual o
peso mximo que o macaco pode levantar quando fazemos
uma fora de 50N em A
1
?

a) 100 N
b) 1000 N
c) 200 kgf
d) 1000 kgf
e) 10000 kgf

2. (UFPR) Considerando os conceitos de presso e
empuxo, correto afirmar:



01. A presso em um ponto no fundo de um tanque que
contm gua em equilbrio depende da altura da coluna
de gua situada acima desse ponto.
02. Se um objeto flutua na gua com 1/3 do seu volume
submerso, ento sua densidade igual a 1/3 da
densidade da gua.
04. Quando um objeto se encontra em repouso no fundo de
um reservatrio contendo gua, a intensidade do
empuxo menor que a intensidade do peso do objeto.
08. Dadas duas banquetas de mesma massa, uma com trs
pernas e outra com quatro, e cada perna com a mesma
seco reta, a de trs pernas exercer menor presso
sobre o solo.
16. A prensa hidrulica, o freio hidrulico e a direo
hidrulica so exemplos de aplicao do Princpio de
Arquimedes.

3. (Unitau) Um navio de 100 toneladas, aps receber certa
quantidade de sacos de caf, de 60kg cada, passou a ter um
volume submerso V=160m
3
. Quantas sacas de caf
entraram no navio se a densidade da gua 1,0g/cm
3
?


Tarefa Mnima #
4. (UFES) A tubulao da figura a seguir contm lquido
incompressvel que est retido pelo mbolo 1 (de rea igual
a 10,0cm
2
) e pelo mbolo 2 (de rea igual a 40,0cm
2
). Se a
fora F
1
tem mdulo igual a 2,00N, a fora F
2
, que mantm
o sistema em equilbrio, tem mdulo igual a:

a) 0,5 N
b) 2,0 N
c) 8,0 N
d) 500,0 N
e) 800,0 N

5. (UEL) Na prensa hidrulica representada a seguir, os
dimetros dos mbolos so d
1
e d
2
, tais que d
1
=2d
2
.

A relao F
1
/F
2
entre as intensidades das foras exercidas
nos dois mbolos, quando situados no mesmo nvel, vale:
a) 4 d) 1/2
b) 2 e) 1/4
c) 1

6. (Vunesp) Um bloco de madeira, quando posto a
flutuar livremente na gua, cuja massa especfica
1,00g/cm
3
, fica com 44% de seu volume fora d'gua. A
massa especfica mdia dessa madeira, em g/cm
3
, :
a) 0,44 c) 1,00 e) 1,56
b) 0,56 d) 1,44

7. (Fuvest) Icebergs so blocos de gelo flutuantes que se
desprendem das geleiras polares. Se apenas 10% do
volume de um iceberg fica acima da superfcie do mar e se
a massa especfica da gua do mar vale 1,03g/cm
3
,
podemos afirmar que a massa especfica do gelo do
iceberg, em g/cm
3
, vale, aproximadamente:
a) 0,10 c) 0,93 e) 1,00
b) 0,90 d) 0,97
Please purchase PDF Split-Merge on www.verypdf.com to remove this watermark.
Incluso para a Vida Fsica A

Pr-Vestibular da UFSC 37
8. (UDESC) Leia com ateno e analise as afirmativas.

I - Pontos a igual profundidade, em um mesmo lquido em
equilbrio, suportam presses iguais.
II - A presso que um lquido exerce no fundo de um
recipiente depende do volume do lquido nele contido.
III - Um corpo imerso em um lquido sofrer um empuxo
tanto maior quanto maior for profundidade em que
estiver.
IV - Um navio flutua porque o peso da gua deslocada
igual ao seu peso.
Assinale a alternativa correta:
a) Todas as afirmativas esto corretas.
b) Somente est correta a afirmativa I.
c) Somente esto corretas as afirmativas I, II e III.
d) Somente esto corretas as afirmativas I e IV.
e) Somente esto corretas as afirmativas I, III e IV.

Tarefa Complementar #
9. (UFF) Uma prensa
hidrulica, sendo utilizada como
elevador de um carro de peso P,
se encontra em equilbrio,
conforme a figura. As seces
retas dos pistes so indicadas
por S
1
e S
2
, tendo-se S
2
=4S
1
.
A fora exercida sobre o fluido F
1
e a fora exercida pelo
fluido F
2
. A situao descrita obedece
a) ao Princpio de Arquimedes e, pelas leis de Newton,
conclui-se que F
1
=F
2
=P.
b) ao Princpio de Pascal e, pelas leis de ao e reao e
de conservao da energia mecnica, conclui-se que
F
2
=4F
1
=P.
c) ao Princpio de Pascal e, pela lei da conservao da
energia, conclui-se que F
2
=1/4F1=P.
d) apenas s leis de Newton e F
1
=F
2
=P.
e) apenas lei de conservao de energia.

10. (UFSC) Assinale a(s) proposio(es) correta(s):

01. Usando um canudinho seria muito mais fcil tomar um
refrigerante na Lua do que na Terra, porque a fora de
atrao gravitacional na Lua menor.
02. possvel a medida aproximada da altitude pela
variao da presso atmosfrica.
04. Uma pessoa explodiria se fosse retirada da atmosfera
terrestre para o vcuo. A presso interna do corpo seria
muito maior do que a presso externa (nula, no vcuo)
e empurraria as molculas para fora do corpo. Este
um dos motivos pelos quais os astronautas usam roupas
especiais para misses fora do ambiente pressurizado
de suas naves.
08. Para repetir a experincia realizada por Evangelista
Torricelli, comparando a presso atmosfrica com a
presso exercida por uma coluna de mercrio,
necessrio conhecer o dimetro do tubo, pois a presso
exercida por uma coluna lquida depende do seu
volume.
16. Vrios fabricantes, para facilitar a retirada da tampa
dos copos de requeijo e de outros produtos,
introduziram um furo no seu centro, selado com
plstico. Isso facilita tirar a tampa porque, ao retirar o
selo, permitimos que o ar penetre no copo e a presso
atmosfrica atue, tambm, de dentro para fora.
32. Quando se introduz a agulha de uma seringa numa veia
do brao, para se retirar sangue, este passa da veia para
a seringa devido diferena de presso entre o sangue
na veia e o interior da seringa.
64. Sendo correta a informao de que So Joaquim se
situa a uma altitude de 1353m e que Itaja est ao nvel
do mar (altitude = 1 m), podemos concluir que a
presso atmosfrica maior em So Joaquim, j que ela
aumenta com a altitude.

11. (UFSC) A figura representa um navio
flutuando em equilbrio, submetido ao
apenas do seu prprio peso e do empuxo
exercido pela gua. Considerando a
situao descrita, assinale a(s) proposio(es) correta(s):
01. O empuxo exercido sobre o navio maior do que o seu
peso. Caso contrrio, um pequeno acrscimo de carga
provocaria o seu afundamento.
02. O empuxo exercido sobre o navio igual ao seu peso.
04. Um volume de gua igual ao volume submerso do
navio tem o mesmo peso do navio.
08. Mesmo sendo construdo com chapas de ao, a
densidade mdia do navio menor do que a densidade
da gua.
16. Se um dano no navio permitir que gua penetre no seu
interior, enchendo-o, ele afundar totalmente, porque,
cheio de gua sua densidade mdia ser maior do que a
densidade da gua.
32. Sendo o empuxo exercido sobre o navio igual ao seu
peso, a densidade mdia do navio igual densidade
da gua.

12. (UFSC) Um corpo C, de
formato cbico, tem massa
igual a 0,08kg e massa
especfica igual a 800kg/m
3
. Ele

mantido inicialmente
submerso, em repouso, em um
lquido de massa especfica
igual a 1200 kg/m
3
tambm em repouso em um tanque. A
parte superior desse corpo est a uma distncia d = 4m da
superfcie do lquido, como est representado na figura
abaixo.
Em um determinado instante, o corpo solto e, aps certo
intervalo de tempo, aflora superfcie do lquido.
Desprezando qualquer tipo de atrito e desconsiderando a
fora de empuxo do ar sobre o corpo, assinale a(s)
proposio(es) correta(s).
01. O mdulo da fora de empuxo que o lquido exerce no
corpo C, na posio mostrada na figura acima, maior
que o mdulo da fora peso desse corpo.
02. Imediatamente aps ser liberado, o corpo C adquire um
movimento retilneo uniforme vertical para cima.
04. O trabalho realizado pela fora de empuxo que o
lquido exerce sobre o corpo C, no percurso d, igual a
4,8 J.
08. Quando o corpo C estiver flutuando livremente na
superfcie do lquido, ter 1/3 de seu volume submerso.
16. Um outro corpo, de volume igual ao do corpo C,
somente permaneceria em equilbrio quando totalmente
imerso nesse lquido, se o seu peso tivesse mdulo
igual a 1,2 N.

d
C
Please purchase PDF Split-Merge on www.verypdf.com to remove this watermark.
Incluso para a Vida Fsica B

Pr-Vestibular da UFSC

1
UNIDADE 1

TERMOMETRIA

Temperatura
a grandeza fsica que mede o estado de agitao das
partculas de um corpo, caracterizando o seu estado
trmico.

Calor
o nome que a energia trmica recebe quando passa
de um corpo de maior temperatura para um outro de
menor temperatura, ou seja, energia trmica em
trnsito.

Equilbrio Trmico

Dois ou mais corpos esto em equilbrio
trmicos quando possuem a mesma temperatura.

Escalas Termomtricas
Escala Fahrenheit
Escala Kelvin
Escala Celsius

Lembre-se:
Ponto de Gelo temperatura em que a gua
congela (presso normal)
Ponto de Vapor temperatura em que a gua
evapora (presso normal)


Obs.: A escala Kelvin tambm conhecida por escala
absoluta ou escala termodinmica, tem origem no zero
absoluto e no existe temperatura inferior a esta.

Converso entre Escalas

5
273
9
32
5

=
tk tf tc


Variao de Temperatura (T)

T
C
= T
K

9. T
C
= 5. T
F

Exerccios de Sala #

1. Em relao termometria, certo dizer que:

a) - 273 K representa a menor temperatura possvel de
ser atingida por qualquer substncia.
b) a quantidade de calor de uma substncia equivale
sua temperatura.
c) em uma porta de madeira, a maaneta metlica est
sempre mais fria que a porta.
d) a escala Kelvin conhecida como absoluta porque
s admite valores positivos.
e) o estado fsico de uma substncia depende
exclusivamente da temperatura em que ela se
encontra.

2. Um termmetro encerrado dentro de um bulbo de
vidro onde se faz vcuo. Suponha que o vcuo seja
perfeito e que o termmetro esteja marcando a
temperatura ambiente, 25C. Depois de algum tempo,
a temperatura ambiente se eleva a 30C. Observa-se,
ento, que a marcao do termmetro:
a) eleva-se tambm, e tende a atingir o equilbrio
trmico com o ambiente.
b) mantm-se a 25C, qualquer que seja a temperatura
ambiente.
c) tende a reduzir-se continuamente, independente da
temperatura ambiente.
d) vai se elevar, mas nunca atinge o equilbrio trmico
com o ambiente.
e) tende a atingir o valor mnimo da escala do
termmetro.

Tarefa Mnima #

3. Os termmetros so instrumentos utilizados para
efetuarmos medidas de temperaturas. Os mais comuns
se baseiam na variao de volume sofrida por um
lquido considerado ideal, contido num tubo de vidro
cuja dilatao desprezada. Num termmetro em que
se utiliza mercrio, vemos que a coluna desse lquido
"sobe" cerca de 2,7 cm para um aquecimento de 3,6C.
Se a escala termomtrica fosse a Fahrenheit, para um
aquecimento de 3,6F, a coluna de mercrio "subiria":

a) 11,8 cm b) 3,6 cm c) 2,7 cm d) 1,8 cm e) 1,5 cm

4. O grfico a seguir relaciona as escalas
termomtricas Celsius e Fahrenheit.

Um termmetro graduado na escala Celsius indica
uma temperatura de 20C. A Correspondente
indicao de um termmetro graduado na escala
Fahrenheit :

a) 22F c) 68F e) 222F
b) 50F d) 80F

5. Com relao aos conceitos de calor, temperatura e
energia interna, assinale a(s) proposio(es)
correta(s).
01. Associa-se a existncia de calor a qualquer corpo,
pois todo corpo possui calor.
02. Para se admitir a existncia de calor so
Please purchase PDF Split-Merge on www.verypdf.com to remove this watermark.
Incluso para a Vida Fsica B

Pr-Vestibular da UFSC

2
necessrios, pelo menos, dois sistemas.
04. Calor a energia contida em um corpo.
08. Quando as extremidades de uma barra metlica
esto a temperaturas diferentes, a extremidade
submetida temperatura maior contm mais calor
do que a outra.
16. Duas esferas de mesmo material e de massas
diferentes, aps ficarem durante muito tempo em
um forno a 160
o
C, so retiradas deste e
imediatamente colocadas em contato. Logo em
seguida, pode-se afirmar, o calor contido na esfera
de maior massa passa para a de menor massa.
32. Se colocarmos um termmetro, em um dia em que
a temperatura est a 25
o
C, em gua a uma
temperatura mais elevada, a energia interna do
termmetro aumentar.

6. Em um determinado dia, a temperatura mnima em
Belo Horizonte foi de 15 C e a mxima de 27 C. A
diferena entre essas temperaturas, na escala kelvin,
de:
a) 12. b) 21. c) 263. d) 285.

7. Um cientista criou uma escala termomtrica D que
adota como pontos fixos o ponto de ebulio do lcool
(78 C) e o ponto de ebulio do ter (34 C). O
grfico a seguir relaciona esta escala D com a escala
Celsius.

A temperatura de ebulio da gua vale, em D:
a) 44 b) 86 c) 112 d) 120 e) 160

8. Uma escala termomtrica arbitrria X atribui o
valor -20X para a temperatura de fuso do gelo e
120X para a temperatura de ebulio da gua, sob
presso normal. A temperatura em que a escala X d a
mesma indicao que a Celsius :
a) 80 b) 70 c) 50 d) 30 e) 10

9. Um menino ingls mediu sua temperatura com um
termmetro graduado na escala Fahrenheit e encontrou
96,8F. Esse menino est:
a) com temperatura de 38C.
b) com temperatura de 34,6C.
c) com febre alta, mais de 29C.
d) com temperatura menor que 36C.
e) com a temperatura normal de 36C.

10. Analise as seguintes afirmaes sobre conceitos
de termologia:
I) Calor uma forma de energia.
II) Calor o mesmo que temperatura.
III) A grandeza que permite informar se dois corpos
esto em equilbrio trmico a temperatura.


Est(o) correta(s) apenas:
a) I.
b) II.
c) III.
d) I e II.
e) I e III.

UNIDADE 2

DILATAO TRMICA DOS SLIDOS E
LQUIDOS
Dilatao Linear
aquela em que predomina a variao em uma nica
dimenso, ou seja, o comprimento. Para estudarmos a
dilatao linear, consideremos uma barra de
comprimento inicial L
i
, temperatura inicial t
i
.
Aumentando a temperatura da barra t
f
, seu
comprimento passa a L
f
.

Em que AL = L
f
- L
i
a variao de comprimento, isto
, a dilatao linear da barra, na variao de
temperatura At = t
f
- t
i
.

AL = L
i
o At

Dilatao Superficial

aquela em que predomina a variao em duas
dimenses, ou seja, a rea. Consideremos uma placa
de rea inicial A
i
, temperatura inicial t
i
. Aumentando
a temperatura da placa para t
f
, sua rea passa para A
f
.

Em que:
AA = A
f
- A
i
At = t
f
- t
i


AA = A
i
|At

O coeficiente de dilatao superficial para cada
substncia igual ao dobro do coeficiente de dilatao
linear, isto :

| = 2o
Obs.: A escala Kelvin tambm conhecida por escala
absoluta ou escala termodinmica, tem origem no zero
absoluto e no existe temperatura inferior a esta.

Dilatao Volumtrica
aquela em que ocorre quando existe variao das
trs dimenses de um corpo: comprimento, largura e
espessura. Com o aumento da temperatura, o volume
da figura sofre um aumento V, tal que:

AV = V
f
- V
i
At = t
f
- t
i


Please purchase PDF Split-Merge on www.verypdf.com to remove this watermark.
Incluso para a Vida Fsica B

Pr-Vestibular da UFSC

3

AV = V
i.
. At

Em que
V
i
= volume inicial.
V
f
= volume final.
AV = variao de volume (dilatao volumtrica).
Em que o coeficiente de dilatao
volumtrica do material que constitui o corpo.
O coeficiente de dilatao volumtrica
aproximadamente igual ao triplo do coeficiente de
dilatao linear o, isto : = 3o

Dilatao dos Lquidos
Como os lquidos no apresentam forma prpria, s
tem significado o estudo de sua dilatao volumtrica.
Ao estudar a dilatao dos lquidos tem de se levar em
conta a dilatao do recipiente slido que o contm.
De maneira geral, os lquidos dilatam-se
sempre mais que os slidos ao serem igualmente
aquecidos.
No aquecimento de um lquido contido num
recipiente, o lquido ir, ao dilatar-se juntamente com
o recipiente, ocupar parte da dilatao sofrida pelo
recipiente, alm de mostrar uma dilatao prpria,
chamada dilatao aparente.
A dilatao aparente aquela diretamente
observada e a dilatao real aquela que o lquido
sofre realmente.
Consideremos um recipiente totalmente cheio
de um lquido temperatura inicial t
i
.
Aumentando a temperatura do conjunto
(recipiente + lquido) at uma temperatura t
f
, nota-se
um extravasamento do lquido, pois este se dilata mais
que o recipiente.
A dilatao aparente do lquido igual ao
volume que foi extravasado.
A dilatao real do lquido dada pela soma
da dilatao aparente do lquido e da dilatao
volumtrica sofrida pelo recipiente.




AV
real
= AV
ap
+ AV
recip
V
i

real
At = V
i

ap
AT + V
i

recip.
At

real
=
ap
+
recip.
Exerccios de Sala #

1. Voc convidado a projetar uma ponte metlica,
cujo comprimento ser de 2,0 km. Considerando os
efeitos de contrao e expanso trmica para
temperaturas no intervalo de - 40 F a 110 F e que o
coeficiente de dilatao linear do metal de 12 10
-6

C
-1
, qual a mxima variao esperada no
comprimento da ponte? (O coeficiente de dilatao
linear constante no intervalo de temperatura
considerado).
a) 9,3 m b) 2,0 m c) 3,0 m d) 0,93 m e) 6,5 m

2. Uma bobina contendo 2000 m de fio de cobre
medido num dia em que a temperatura era de 35 C,
foi utilizada e o fio medido de novo a 10 C. Esta nova
medio indicou:
a) 1,0 m a menos
b) 1,0 m a mais
c) 2000 m
d) 20 m a menos
e) 20 mm a mais

Tarefa Mnima #

3. Uma barra de metal tem comprimento igual a
10,000 m a uma temperatura de 10,0 C e
comprimento igual a 10,006 m a uma temperatura de
40 C. O coeficiente de dilatao linear do metal
a) 1,5 10
-4
C
-1

b) 6,0 10
-4
C
-1

c) 2,0 10
-5
C
-1

d) 2,0 10
-6
C
-1
e) 3,0 10
-6
C
-1

4. A figura a seguir representa uma lmina bimetlica.
O coeficiente de dilatao linear do metal A a
metade do coeficiente de dilatao linear do metal B.
temperatura ambiente, a lmina est na vertical. Se a
temperatura for aumentada em 200 C, a lmina:

a) continuar na vertical.
b) curvar para a frente.
c) curvar para trs.
d) curvar para a direita.
e) curvar para a esquerda.

5. O grfico a seguir representa a variao, em
milmetros, do comprimento de uma barra metlica, de
tamanho inicial igual a 1 000 m, aquecida em um
forno industrial. Qual o valor do coeficiente de
dilatao trmica linear do material de que feita a
barra, em unidades de 10
-6
/C?
Please purchase PDF Split-Merge on www.verypdf.com to remove this watermark.
Incluso para a Vida Fsica B

Pr-Vestibular da UFSC

4


6. Ao se aquecer de 1 C uma haste metlica de 1 m, o
seu comprimento aumenta de 2.10
-2
mm. O aumento
do comprimento de outra haste do mesmo metal, de
medida inicial 80 cm, quando a aquecemos de 20 C,
:
a) 0,23 mm.
b) 0,32 mm.
c) 0,56 mm.
d) 0,65 mm.
e) 0,76 mm.

7. Uma placa de alumnio tem um grande orifcio
circular no qual foi colocado um pino, tambm de
alumnio, com grande folga. O pino e a placa so
aquecidos de 500 C, simultaneamente.
Podemos afirmar que:
a) a folga ir aumentar, pois o pino ao ser aquecido ir
contrair-se.
b) a folga diminuir, pois ao aquecermos a chapa a
rea do orifcio diminui.
c) a folga diminuir, pois o pino se dilata muito mais
que o orifcio.
d) a folga ir aumentar, pois o dimetro do orifcio
aumenta mais que o dimetro do pino.
e) a folga diminuir, pois o pino se dilata, e a rea do
orifcio no se altera.

8. O coeficiente de dilatao trmica do alumnio
(A) , aproximadamente, duas vezes o coeficiente de
dilatao trmica do ferro (Fe). A figura mostra duas
peas onde um anel feito de um desses metais envolve
um disco feito do outro. temperatura ambiente, os
discos esto presos aos anis.

Se as duas peas forem aquecidas uniformemente,
correto afirmar que:
a) apenas o disco de A se soltar do anel de Fe.
b) apenas o disco de Fe se soltar do anel de A.
c) os dois discos se soltaro dos respectivos anis.
d) os discos no se soltaro dos anis.

9. A figura a seguir ilustra um arame rgido de ao,
cujas extremidades esto distanciadas de "L".


Alterando-se sua temperatura, de 293K para 100C,
pode-se afirmar que a distncia "L":
a) diminui, pois o arame aumenta de comprimento,
fazendo com que suas extremidades fiquem mais
prximas.
b) diminui, pois o arame contrai com a diminuio da
temperatura.
c) aumenta, pois o arame diminui de comprimento,
fazendo com que suas extremidades fiquem mais
afastadas.
d) no varia, pois a dilatao linear do arame
compensada pelo aumento do raio "R".
e) aumenta, pois a rea do crculo de raio "R" aumenta
com a temperatura.

10. O volume de um bloco metlico sofre um
aumento de 0,6% quando sua temperatura varia de 200
C. O coeficiente de dilatao linear mdio desse
metal, em C
-1
, vale:
a) 1,0.10
-5
b) 3,0.10
-5
c) 1,0.10
-4
d) 3,0.10
-4

e) 3,0.10
-3


11. Assinale a(s) proposio(es) correta(s) em
relao a alguns fenmenos que envolvem os
conceitos de temperatura, calor, mudana de estado e
dilatao trmica.
01. A temperatura de um corpo uma grandeza fsica
relacionada densidade do corpo.
02. Uma substncia pura ao receber calor ficar
submetida a variaes de temperatura durante a
fuso e a ebulio.
04. A dilatao trmica um fenmeno especfico dos
lquidos, no ocorrendo com os slidos.
08. Calor uma forma de energia.
16. O calor se propaga no vcuo.

12. Em uma chapa metlica feito um orifcio
circular do mesmo tamanho de uma moeda. O
conjunto (chapa com a moeda no orifcio),
inicialmente a 25 C, levado a um forno e aquecido
at 225 C. Aps o aquecimento, verifica-se que o
orifcio na chapa ficou maior do que a moeda. Dentre
as afirmativas a seguir, indique a que est correta.

a) O coeficiente de dilatao da moeda maior do que
o da chapa metlica.
b) O coeficiente de dilatao da moeda menor do que
o da chapa metlica.
c) O coeficiente de dilatao da moeda igual ao da
chapa metlica, mas o orifcio se dilatou mais
porque a chapa maior que a moeda.
d) O coeficiente de dilatao da moeda igual ao da
chapa metlica, mas o orifcio se dilatou mais
porque o seu interior vazio.
e) Nada se pode afirmar sobre os coeficientes de
dilatao da moeda e da chapa, pois no dado o
tamanho inicial da chapa.





Please purchase PDF Split-Merge on www.verypdf.com to remove this watermark.
Incluso para a Vida Fsica B

Pr-Vestibular da UFSC

5
13. A respeito da dilatao trmica, fenmeno de
expanso e contrao que ocorre nas substncias
quando h variao de sua temperatura, assinale o que
for correto.
01. A variao do volume de uma substncia
proporcional ao produto entre seu volume inicial e
a variao de temperatura.
02. O coeficiente de dilatao uma grandeza
adimensional.
04. Em corpos que tm apenas uma dimenso, ocorre
dilatao linear.
08. Se uma placa que contm um orifcio sofrer um
aumento em sua temperatura, as dimenses do
orifcio aumentaro.

14. Duas substncias A e B tm seus grficos de
densidade temperatura representados a seguir. As
substncias so colocadas a 4C em garrafas de vidro
distintas, ocupando todo o volume das garrafas.
Considere o coeficiente de dilatao do vidro das
garrafas muito menor que o das substncias A e B. As
garrafas so, ento, fechadas e colocadas em um
refrigerador a 0C. Aps um longo perodo de tempo,
pode-se dizer que:


a) a garrafa de A se quebra e a de B no.
b) a garrafa de B se quebra e a de A no.
c) as garrafas de A e B se quebram.
d) as garrafas de A e B no se quebram.
e) os dados fornecidos no so suficientes para se
chegar a uma concluso.

UNIDADE 3

CALORIMETRIA

UNIDADES DE CALOR
O calor uma forma de energia que passa de um corpo
para outro como consequncia da diferena de
temperaturas entre os corpos. Sendo energia, sua
unidade no Sistema Internacional o joule (J). No
entanto, por razes histricas, ainda hoje usamos uma
unidade introduzida na poca em que no se sabia a
natureza do calor. Essa unidade a caloria cujo
smbolo cal.
A relao entre a caloria e o joule :

1 cal = 4,186 joules

CAPACIDADE TRMICA
Suponhamos que uma quantidade de calor Q
seja fornecida a um corpo. Supondo que no haja
mudana de estado, esse calor provocar no corpo,
uma variao de temperatura t que proporcional a
Q, isto , podemos escrever
Q = C (t)
onde C uma constante chamada de capacidade
trmica do corpo.
C =
Q
__________
t
CALOR ESPECFICO
Quando um corpo feito de uma nica substncia, sua
capacidade trmica (C) proporcional sua massa
(m), isto , podemos escrever:

C = m . c (II)

onde c uma constante que depende da substncia e
chamada de calor especfico da substncia.

CALOR SENSVEL
O calor sensvel responsvel pela variao da
temperatura de um corpo.

Q = m . c (t) (III)

Dessa equao tiramos:
c =
Q
--------
m . t

A seguir fornecemos os calores especficos de algumas
substncias:

TABELA - Calores especficos de
algumas substncias
SUBSTNCIA
CALOR
ESPECFICO (c)
(cal/g.C)
SLIDOS
Alumnio 0,215
Cobre 0,092
Ouro 0,031
Ao 0,107
Prata 0,056
Gelo 0,5
LQUIDOS
gua 1,0
lcool etlico 0,58
Mercrio 0,033

TROCAS DE CALOR
Ns dizemos que um conjunto de corpos est
termicamente isolado quando ele no ganha nem
perde calor para o meio externo. Um modo de fazer
isso colocar o conjunto em um recipiente de paredes
isolantes, isto , que no deixam passar o calor. Um
recipiente como esse chamado de calormetro.
Please purchase PDF Split-Merge on www.verypdf.com to remove this watermark.
Incluso para a Vida Fsica B

Pr-Vestibular da UFSC

6
Suponhamos que coloquemos dentro de um
calormetro dois corpos que inicialmente tenham
temperaturas diferentes. Durante algum tempo haver
passagem de calor do corpo mais quente para o corpo
mais frio. Essa passagem de calor pra no momento
em que atingido o equilbrio trmico, isto , quando
os corpos ficam com a mesma temperatura.

Qrec + Qced = 0

MUDANA DE ESTADO FSICO

TIPOS DE MUDANAS
Quando uma substncia passa do estado slido para o
estado lquido, essa transformao chama-se fuso. A
transformao inversa (de lquido para slido) chama
solidificao. A passagem do estado lquido para o
gasoso chama-se vaporizao; a transformao inversa
chamada condensao.


CALOR DE TRANSFORMAO
Quando uma substncia muda de estado de agregao,
absorve (ou cede) uma quantidade de calor que
proporcional massa (m). Assim podemos escrever:

Q = m L

onde L uma constante chamada calor de
transformao. Quando se trata da fuso (ou
solidificao) a constante L chamada de calor de
fuso; quando se trata da ebulio (ou liquefao) a
constante L chama-se calor de vaporizao. A
constante L tambm chamada de calor latente (da o
smbolo L).
Da equao Q = mL tiramos:

Na tabela a seguir fornecemos os valores de L para
algumas substncias.
TABELA 2 - Calores latentes de algumas
substncias
Substncia
Calor de
fuso
(cal/g)
Calor de
vaporizao
(cal/g)
gua 80 540
lcool
etlico
25 204
Ouro 15 557
Prata 21 558
Cobre 32 1.210
CURVA DE AQUECIMENTO
Podemos fazer um grfico da temperatura em funo
da quantidade de calor fornecido

Exerccios de Sala #

1. Adote: calor especfico da gua: 1,0 cal/g.C
Um bloco de massa 2,0 kg, ao receber toda energia
trmica liberada por 1000 gramas de gua que
diminuem a sua temperatura de 1 C, sofre um
acrscimo de temperatura de 10 C. O calor especfico
do bloco, em cal/g.C, :
a) 0,2 b) 0,1 c) 0,15 d) 0,05 e) 0,01

2. Adote: calor especfico da gua: 1,0 cal/gC
Calor de combusto a quantidade de calor liberada
na queima de uma unidade de massa do combustvel.
O calor de combusto do gs de cozinha 6000
kcal/kg. Aproximadamente quantos litros de gua
temperatura de 20 C podem ser aquecidos at a
temperatura de 100 C com um bujo de gs de 13 kg?
Despreze perdas de calor:
a) 1 litro
b) 10 litros
c) 100 litros
d) 1000 litros
e) 6000 litros

Tarefa Mnima #

3. Um frasco contm 20 g de gua a 0 C. Em seu
interior colocado um objeto de 50 g de alumnio a 80
C. Os calores especficos da gua e do alumnio so
respectivamente 1,0 cal/gC e 0,10 cal/gC. Supondo
no haver trocas de calor com o frasco e com o meio
ambiente, a temperatura de equilbrio desta mistura
ser
a) 60 C b) 16 C c) 40 C d) 32 C e) 10 C

4. A temperatura de dois corpos M e N, de massas
iguais a 100 g cada, varia com o calor recebido como
indica o grfico a seguir. Colocando N a 10 C em
contato com M a 80 C e admitindo que a troca de
calor ocorra somente entre eles, a temperatura final de
equilbrio, em C, ser

a) 60 b) 50 c) 40 d) 30 e) 20

Please purchase PDF Split-Merge on www.verypdf.com to remove this watermark.
Incluso para a Vida Fsica B

Pr-Vestibular da UFSC

7
5. Uma fonte trmica, de potncia constante e igual a
20 cal/s, fornece calor a um corpo slido de massa 100
g. A variao de temperatura do corpo em funo do
tempo t dada pelo grfico a seguir.

O calor especfico da substncia que constitui o corpo,
no estado lquido, em cal/gC, vale
a) 0,05 b) 0,10 c) 0,20 d) 0,30 e) 0,40

6. Quando dois corpos de tamanhos diferentes esto
em contato e em equilbrio trmico, e ambos isolados
do meio ambiente, pode-se dizer que:
a) o corpo maior o mais quente.
b) o corpo menor o mais quente.
c) no h troca de calor entre os corpos.
d) o corpo maior cede calor para o corpo menor.
e) o corpo menor cede calor para o corpo maior.

7. Um certo volume de um lquido A, de massa M e
que est inicialmente a 20 C, despejado no interior
de uma garrafa trmica que contm uma massa 2M de
um outro lquido, B, na temperatura de 80 C. Se a
temperatura final da mistura lquida resultante for de
40 C, podemos afirmar que a razo CA/CB entre os
calores especficos das substncias A e B vale:
a) 6 b) 4 c) 3 d) e) 1/3

8. O grfico a seguir representa o calor absorvido por
dois corpos slidos M e N em funo da temperatura.

A capacidade trmica do corpo M, em relao do
corpo N, vale
a) 1,4
b) 5,0
c) 5,5
d) 6,0
e) 7,0

9. A figura a seguir representa a temperatura de um
lquido no-voltil em funo da quantidade de calor
por ele absorvida. Sendo a massa do lquido 100 g e
seu calor especfico 0,6 cal/gC, qual o valor em C da
temperatura T?

10. Analise as seguintes afirmaes sobre conceitos
de termologia:
I) Calor uma forma de energia.
II) Calor o mesmo que temperatura.
III) A grandeza que permite informar se dois corpos
esto em equilbrio trmico a temperatura.
Est(o) correta(s) apenas:
a) I. b) II. c) III. d) I e II. e) I e III.

11. O grfico a seguir representa a quantidade de calor
absorvida por dois objetos A e B ao serem aquecidos,
em funo de suas temperaturas.

Observe o grfico e assinale a(s) proposio(es)
correta(s).
01. A capacidade trmica do objeto A maior que a do
objeto B.
02. A partir do grfico possvel determinar as
capacidades trmicas dos objetos A e B.
04. Pode-se afirmar que o calor especfico do objeto A
maior que o do objeto B.
08. A variao de temperatura do objeto B, por caloria
absorvida, maior que a variao de temperatura
do objeto A, por caloria absorvida.
16. Se a massa do objeto A for de 200 g, seu calor
especfico ser 0,2 cal/gC.

12. Assinale a(s) proposio(es) correta(s) em
relao a alguns fenmenos que envolvem os
conceitos de temperatura, calor, mudana de estado e
dilatao trmica.
01. A temperatura de um corpo uma grandeza fsica
relacionada densidade do corpo.
02. Uma substncia pura ao receber calor ficar
submetida a variaes de temperatura durante a
fuso e a ebulio.
04. A dilatao trmica um fenmeno especfico dos
lquidos, no ocorrendo com os slidos.
08. Calor uma forma de energia.
16. O calor se propaga no vcuo.

13. Determine a quantidade de calor necessria para
transformar 100 g de 8gelo, inicialmente a 0 C, em
100 g de gua a 30 C. Sabe-se que o calor latente de
fuso do gelo 80 cal/g e o calor especfico da gua
1 cal/gC.
a) 6000 cal.
b) 3000 cal.
c) 11000 cal.
d) 8000 cal.
e) 10000 cal.

14. Um aquecedor eltrico mergulhado em um
recipiente com gua a 10 C e, cinco minutos depois, a
gua comea a ferver a 100 C. Se o aquecedor no for
desligado, toda a gua ir evaporar e o aquecedor ser
danificado. Considerando o momento em que a gua
Please purchase PDF Split-Merge on www.verypdf.com to remove this watermark.
Incluso para a Vida Fsica B

Pr-Vestibular da UFSC

8
comea a ferver, a evaporao de toda a gua ocorrer
em um intervalo de aproximadamente
Calor especfico da gua = 1,0 cal/(gC)
Calor de vaporizao da gua = 540 cal/g
Desconsidere perdas de calor para o recipiente, para o
ambiente e para o prprio aquecedor.
a) 5 minutos.
b) 10 minutos.
c) 12 minutos.
d) 15 minutos.
e) 30 minutos.

UNIDADE 4

TRANSMISSO DE CALOR

CONDUO DE CALOR
A conduo um processo pelo qual o calor se
transmite ao longo de um meio material por meio da
transmisso de vibrao de suas molculas. As
molculas mais energticas (de maior temperatura)
transmitem energia para as menos energticas (menor
temperatura).
Existem materiais que conduzem o calor
rapidamente por exemplo, os metais. Tais materiais
so chamados de bons condutores. Podemos perceber
isso fazendo um experimento como o ilustrado na
Fig.1.

Segurando uma barra de metal que tem uma
extremidade sobre uma chama, rapidamente o calor
transmitido para nossa mo. Por outro lado h
materiais nos quais o calor se propaga muito
lentamente. Tais materiais so chamados de isolantes.
Como exemplos, podemos citar a borracha, a l, o
isopor e o amianto.
O fato de a l ser um bom isolante explica por
que no inverno usamos agasalhos de l; ela dificulta a
perda do calor de nosso corpo para o meio externo.

FLUXO DE CALOR
Suponhamos que em um intervalo de tempo passe
uma quantidade de calor Q por uma superfcie S
(Fig.2).

Fig. 2
O fluxo de calor atravs da superfcie S
definido por:



A experincia mostra que o fluxo de calor atravs da
barra dado por:


onde k uma constante cujo valor depende do
material e chamada condutividade trmica do
material.

CONVECO
A conveco de calor a transmisso de calor por
meio do transporte de matria. Ela ocorre no interior
de fluidos (lquidos e gases) como consequncia da
diferena de densidades entre diferentes partes do
fluido. Por exemplo, consideremos o caso ilustrado na
Fig. 1 em que um recipiente contendo gua colocado
sobre uma chama. Pelo aquecimento, a parte inferior
da gua se dilata e fica com densidade menor do que a
parte superior. Com isso, ocorre uma corrente
ascendente e outra descendente. Essas correntes so
chamadas correntes de conveco.

Fig. 1
IRRADIAO
No estudo da eletricidade apresentaremos o conceito
de onda eletromagntica. Por enquanto vamos adiantar
que todos os corpos emitem ondas eletromagnticas
cuja intensidade aumenta com a temperatura. Essas
ondas se propagam no vcuo e dessa maneira que a
luz e o calor so transmitidos do Sol at a Terra.

Fig.7

Entre as ondas eletromagnticas, a principal
responsvel pela transmisso do calor a onda de
infravermelho.
Quando chegamos perto de uma fogueira,
uma lmpada incandescente ou um aquecedor eltrico,
sentimos o calor emitido por eles. Uma parcela desse
calor pode atingir-nos por conduo atravs do ar,
porm essa parcela pequena, pois o ar mau
condutor de calor. A maior parte do calor que
Please purchase PDF Split-Merge on www.verypdf.com to remove this watermark.
Incluso para a Vida Fsica B

Pr-Vestibular da UFSC

9
recebemos dessas fontes vem por irradiao de ondas
eletromagnticas.

Exerccios de Sala #

1. Indique a alternativa que associa corretamente o
tipo predominante de transferncia de calor que ocorre
nos fenmenos, na seguinte sequncia:
- Aquecimento de uma barra de ferro quando sua
extremidade colocada numa chama acesa.
- Aquecimento do corpo humano quando exposto ao
sol.
- Vento que sopra da terra para o mar durante a noite.
a) conveco - conduo - radiao.
b) conveco - radiao - conduo.
c) conduo - conveco - radiao.
d) conduo - radiao - conveco.
e) radiao - conduo - conveco.

2. Sabe-se que o calor especfico da gua maior que
o calor especfico da terra e de seus constituintes
(rocha, areia, etc.). Em face disso, pode-se afirmar
que, nas regies limtrofes entre a terra e o mar:
a) durante o dia, h vento soprando do mar para a terra
e, noite, o vento sopra no sentido oposto.
b) o vento sempre sopra sentido terra-mar.
c) durante o dia, o vento sopra da terra para o mar e
noite o vento sopra do mar para a terra.
d) o vento sempre sopra do mar para a terra.
e) no h vento algum entre a terra e o mar.

Tarefa Mnima#

3. Uma estufa para flores, construda em alvenaria,
com cobertura de vidro, mantm a temperatura interior
bem mais elevada do que a exterior. Das seguintes
afirmaes:
I. O calor entra por conduo e sai muito pouco por
conveco
II. O calor entra por radiao e sai muito pouco por
conveco
III. O calor entra por radiao e sai muito pouco por
conduo
IV. O calor entra por conduo e conveco e s pode
sair por radiao

A(s) alternativa(s) que pode(m) justificar a elevada
temperatura do interior da estufa (so):
a) I, III b) I, II c) IV d) II, III e) II

4. Calor uma forma de energia que transferida
entre dois sistemas quando entre eles existe uma
diferena de temperatura, e a transferncia pode
ocorrer por conduo, conveco ou radiao. A
respeito deste assunto, assinale o que for correto.
01. Na conduo, a transferncia de calor ocorre de
partcula a partcula, dentro de um corpo ou entre
dois corpos em contato.
02. A transferncia de calor em um meio fluido ocorre
por conveco.
04. Na radiao, a transferncia de calor entre dois
sistemas ocorre atravs de ondas eletromagnticas.
08. O fluxo de calor atravs de um corpo
inversamente proporcional sua espessura.

5. Depois de assar um bolo em um forno a gs,
Zulmira observa que ela queima a mo ao tocar no
tabuleiro, mas no a queima ao tocar no bolo.
Considerando-se essa situao, correto afirmar que
isso ocorre porque:
a) a capacidade trmica do tabuleiro maior que a do
bolo.
b) a transferncia de calor entre o tabuleiro e a mo
mais rpida que entre o bolo e a mo.
c) o bolo esfria mais rapidamente que o tabuleiro,
depois de os dois serem retirados do forno.
d) o tabuleiro retm mais calor que o bolo.

6. O uso mais popular de energia solar est associado
ao fornecimento de gua quente para fins domsticos.
Na figura a seguir, ilustrado um aquecedor de gua
constitudo de dois tanques pretos dentro de uma caixa
termicamente isolada e com cobertura de vidro, os
quais absorvem energia solar.

A. Hinrichs e M. Kleinbach. "Energia e meio ambiente". So Paulo:
Thompson, 3 ed., 2004, p. 529 (com adaptaes).

Nesse sistema de aquecimento,
a) os tanques, por serem de cor preta, so maus
absorvedores de calor e reduzem as perdas de
energia.
b) a cobertura de vidro deixa passar a energia luminosa
e reduz a perda de energia trmica utilizada para o
aquecimento.
c) a gua circula devido variao de energia
luminosa existente entre os pontos X e Y.
d) a camada refletiva tem como funo armazenar
energia luminosa.
e) o vidro, por ser bom condutor de calor, permite que
se mantenha constante a temperatura no interior da
caixa.

7. Com relao aos processos de transferncia de
calor, considere as seguintes afirmativas:
1. A conduo e a conveco so processos que
dependem das propriedades do meio material no
qual ocorrem.
2. A conveco um processo de transmisso de calor
que ocorre somente em metais.
3. O processo de radiao est relacionado com a
propagao de ondas eletromagnticas.

Assinale a alternativa correta.
a) Somente a afirmativa 1 verdadeira.
b) Somente a afirmativa 2 verdadeira.
c) Somente a afirmativa 3 verdadeira.
Please purchase PDF Split-Merge on www.verypdf.com to remove this watermark.
Incluso para a Vida Fsica B

Pr-Vestibular da UFSC

10
d) Somente as afirmativas 1 e 3 so verdadeiras.
e) Somente as afirmativas 2 e 3 so verdadeiras.

8. Um resistor R colocado dentro de um recipiente
de parede metlica, no qual feito vcuo e que possui
um termmetro incrustado em sua parede externa. Para
ligar o resistor a uma fonte externa ao recipiente foi
utilizado um fio, com isolamento trmico que impede
transferncia de calor para as paredes do recipiente.
Essa situao encontra-se ilustrada na figura a seguir.

Ligando o resistor, nota-se que a temperatura indicada
pelo termmetro aumenta, mostrando que h
transferncia de calor entre o resistor e o termmetro.
Pode-se afirmar que os processos responsveis por
essa transferncia de calor, na ordem correta, so
a) primeiro conveco e depois radiao.
b) primeiro conveco e depois conduo.
c) primeiro radiao e depois conveco.
d) primeiro radiao e depois conduo.
e) primeiro conduo e depois conveco.

UNIDADE 5

GASES PERFEITOS

Variveis do estado de um gs
Presso resultado dos choques consecutivos das
molculas nas paredes do recipiente.
Volume dado pelo volume do recipiente onde o
gs est contido.
Temperatura Mede a agitao das molculas do
gs.

OBS.: No estudo dos gases deve-se usar a temperatura
absoluta (em Kelvin).
Equao de Clapeyron:
nRT V . p =


Onde : n = m/M

K mol
Cal
K mol
T
K mol
l atm
R
.
2
.
31 , 8
.
.
082 , 0 = ~ =


N1 = N2
Lei geral dos gases perfeitos
2
2 2
1
1 1
T
V . P
T
V P
=

N1 N2
Lei geral dos gases perfeitos
2
.
1
2
2 2
1
1 1
n T
V P
n T
V P
=


Transformaes Gasosas

Isotrmica (Boyle Mariotte)
Caractersticas:
Temperatura permanece constante.
P e V so inversamente proporcionais

Isobrica (Charles)
Caractersticas:
Presso permanece constante.
V e T so diretamente proporcionais.

Isomtrica, Isovolumtrica ou Isocrica (Gay
Lussac)
Caractersticas:
Volume, permanece constante.
P e T so diretamente proporcionais

Adiabtica
Caracterstica: No ocorre troca de calor entre o
sistema e o meio.

Exerccios de Sala #

1. Antes de iniciar uma viagem, um motorista
cuidadoso calibra os pneus de seu carro, que esto
temperatura ambiente de 27 C, com uma presso de
30 lb/pol
2
. Ao final da viagem, para determinar a
temperatura dos pneus, o motorista mede a presso dos
mesmos e descobre que esta aumentou para 32 lb/pol
2
.
Se o volume dos pneus permanece inalterado e se o
gs no interior ideal, o motorista determinou a
temperatura dos pneus como sendo:
a) 17 C b) 27 C c) 37 C d) 47 C e) 57 C

Tarefa Mnima #

2. Quando o balo do capito Stevens comeou sua
ascenso, tinha, no solo, presso de 1 atm, 75000 m
3

de hlio. A 22 km de altura, o volume do hlio era de
1500000 m
3
. Se pudssemos desprezar a variao de
temperatura, a presso (em atm) a esta altura valeria:
a) 1/20 b) 1/5 c) 1/2 d) 1 e) 20

3. Uma amostra de gs perfeito foi submetida s
transformaes indicadas no diagrama PV a seguir.
Nessa sequncia de transformaes, os estados de
maior e de menor temperatura foram, respectivamente:







a) 1 e 2 b) 1 e 3 c) 2 e 3 d) 3 e 4 e) 3 e 5

4. Um gs perfeito est sob presso de 20 atm, na
temperatura de 200 K e apresenta um volume de 40
litros. Se o referido gs tiver sua presso alterada para
40 atm, na mesma temperatura, qual ser o novo
volume?
Please purchase PDF Split-Merge on www.verypdf.com to remove this watermark.
Incluso para a Vida Fsica B

Pr-Vestibular da UFSC

11
5. A respeito do funcionamento da panela de presso,
assinale o que for correto.
01. De acordo com a lei dos gases, as variveis
envolvidas nos processos so: presso, volume e
temperatura.
02. O aumento da presso no interior da panela afeta o
ponto de ebulio da gua.
04. A quantidade de calor doado ao sistema deve ser
constante, para evitar que a panela venha a
explodir.
08. O tempo de cozimento dos alimentos dentro de
uma panela de presso menor porque eles ficam
submetidos a temperaturas superiores a 100 C.

6. Para se realizar uma determinada experincia, -
coloca-se um pouco de gua em uma lata, com uma
abertura na parte superior, destampada, a qual , em
seguida, aquecida, como mostrado na Figura I;
- depois que a gua ferve e o interior da lata fica
totalmente preenchido com vapor, esta tampada e
retirada do fogo;
- logo depois, despeja-se gua fria sobre a lata e
observa-se que ela se contrai bruscamente, como
mostrado na Figura II.

Com base nessas informaes, correto afirmar que,
na situao descrita, a contrao ocorre porque
a) a gua fria provoca uma contrao do metal das
paredes da lata.
b) a lata fica mais frgil ao ser aquecida.
c) a presso atmosfrica esmaga a lata.
d) o vapor frio, no interior da lata, puxa suas paredes
para dentro.

7. Regina estaciona seu carro, movido a gs natural,
ao Sol. Considere que o gs no reservatrio do carro se
comporta como um gs ideal. Assinale a alternativa
cujo grfico melhor representa a presso em funo da
temperatura do gs na situao descrita.



8. Um "freezer" programado para manter a
temperatura em seu interior a -19C. Ao ser instalado,
suponha que a temperatura ambiente seja de 27C.
Considerando que o sistema de fechamento da porta a
mantm hermeticamente fechada, qual ser a presso
no interior do "freezer" quando ele tiver atingido a
temperatura para a qual foi programado?
a) 0,72 atm
b) 0,78 atm
c) 0,85 atm
d) 0,89 atm
e) 0,94 atm

9. Um gs ideal sofre uma compresso adiabtica
durante a qual sua temperatura absoluta passa de T
para 4T. Sendo P a presso inicial, podemos afirmar
que a presso final ser
a) menor do que P.
b) igual a P.
c) igual a 2 P.
d) igual a 4 P.
e) maior do que 4 P.

10. Uma massa de gs perfeito a 17C, que sofre uma
transformao isotrmica, tem seu volume aumentado
de 25%. A presso final do gs, em relao inicial
ser:
a) 20% maior.
b) 20% menor.
c) 25% menor.
d) 80% menor.
e) 80% maior.

11. Um extintor de incndio cilndrico, contendo CO
2

possui um medidor de presso interna que,
inicialmente, indica 200 atm. Com o tempo, parte do
gs escapa, o extintor perde presso e precisa ser
recarregado. Quando a presso interna for igual a 160
atm, a porcentagem da massa inicial de gs que ter
escapado corresponder a:

a) 10% b) 20% c) 40% d) 60% e) 75%

Obs: Considere que a temperatura permanece
constante e o CO
2
nessas condies, comporta-se
como um gs perfeito
1 atm = 10
5
N/m
2

12. Um recipiente rgido contm gs perfeito sob
presso de 3 atm. Sem deixar variar a temperatura, so
retirados 4 mols do gs, fazendo com que a presso se
reduza a 1 atm. O nmero de mols existente
inicialmente no recipiente era:
a) 6 b) 8 c) 10 d) 12 e) 16

13. A quantidade de 2,0mols de um gs perfeito se
expande isotermicamente. Sabendo que no estado
inicial o volume era de 8,20L e a presso de 6,0atm e
que no estado final o volume passou a 24,6L,
determine:
a) a presso final do gs;
b) a temperatura, em C, em que ocorreu a expanso.
Dado:
Constante universal dos gases perfeitos: 0,082atm.L/mol.K

14. Num recipiente indeformvel, provido de vlvula
especial, encontram-se confinados 2 mols de oxignio
(molcula - grama = 32 g) nas C. N. T. P.. Num dado
instante, abre-se a vlvula e permite-se que 8 g do gs
escapem, mantendo-se, contudo a mesma temperatura.
A nova presso do gs :
Please purchase PDF Split-Merge on www.verypdf.com to remove this watermark.
Incluso para a Vida Fsica B

Pr-Vestibular da UFSC

12
Dado: R = 0,082 atm.L/mol . K

a) 15/16 atm
b) 7/8 atm
c) 1/4 atm
d) 7/16 atm
e) 1/8 atm

UNIDADE 6

TERMODINMICA

Trabalho Termodinmico (W)
A equao ( V p W A = . ) s pode ser usada
quando a presso se mantm constante. Quando a
presso varia (durante a variao de volume), o
trabalho do gs deve ser calculado graficamente.
possvel demonstrar que, em qualquer caso, o trabalho
W do gs tem mdulo numericamente igual rea da
regio sombreada no grfico da Fig. A W ~


1) Trabalho positivo =
o gs realiza ou cede
trabalho.

2) Trabalho negativo =
o gs sofre ou recebe
trabalho.


Energia interna de um gs ideal
Verifica-se que a energia interna (U) de um gs ideal
proporcional temperatura absoluta, isto ,
T R n U . .
2
3
=
.
No entanto, para analisarmos os valores energticos
necessrio conhecer a variao da energia interna
( U A ) do gs.

Primeira Lei da Termodinmica
Quando fornecemos a um gs uma quantidade de calor
Q, esse calor pode ser usado de dois modos:

1) uma parte pode ser usada para realizar um trabalho
W.
2) outra parte pode se transformar em energia interna
do gs.
Assim: U W Q A + = onde U A a variao da
energia interna do gs. A equao traduz a Primeira
Lei da Termodinmica que na realidade uma
consequncia do Princpio da Conservao da Energia.
OBS: Isotrmica: W Q =
Adiabtica U = - W
Isocrica: U Q A =
Transformao Cclica
aquela em que o gs sofre diversas transformaes
retornando as suas condies iniciais.
0
P
V
rea = w
A
B
C

Em um ciclo a variao da energia interna zero
( 0 = AU ).

Mquinas trmicas
So dispositivos que
convertem calor em
trabalho e vice-versa:
mquinas a vapor, motores
a exploso, refrigerados,
etc.

MQUINA
Q
1
Q
2
FONTE FRIA
FONTE QUENTE
Calor
recebido
Trabalho
realizado
W
Calor
cedido


2 Lei da Termodinmica: O calor flui
espontaneamente do corpo de maior temperatura para
o de menor temperatura.
No podemos ter uma maquina trmica com
rendimento de 100%.

1
Q
W
n =
ou
1
2
1
Q
Q
n =


Ciclo de Carnot
O ciclo de Carnot o ciclo reversvel constitudo por
dois processos isotrmicos (A-B e C-D) e dois
processos adiabticos (B-C e D-A). Por questes
didticas, a figura representa o ciclo de Carnot para
um gs ideal, e percorrido em um certo sentido,
embora qualquer substncia possa ser levada a
executar um ciclo de Carnot e o sentido possa ser
invertido.


Exerccios de Sala #

1. Sem variar sua massa, um gs ideal sofre uma
transformao a volume constante. correto afirmar
que:
a) a transformao isotrmica.
b) a transformao isobrica.
c) o gs no realiza trabalho.
d) sua presso diminuir ,se a temperatura do gs
aumentar.
e) a variao de temperatura do gs ser a mesma em
qualquer escala termomtrica.



N = 1- T
2
/ T
1
Please purchase PDF Split-Merge on www.verypdf.com to remove this watermark.
Incluso para a Vida Fsica B

Pr-Vestibular da UFSC

13
Tarefa Mnima #

2. O biodiesel resulta da reao qumica desencadeada
por uma mistura de leo vegetal com lcool de cana.
A utilizao do biodiesel etlico como combustvel no
pas permitiria uma reduo sensvel nas emisses de
gases poluentes no ar, bem como uma ampliao da
matriz energtica brasileira. O combustvel testado foi
desenvolvido a partir da transformao qumica do
leo de soja. tambm chamado de B-30 porque
constitudo de uma proporo de 30% de biodiesel e
70% de diesel metropolitano. O primeiro diagnstico
divulgado considerou performances dos veculos
quanto ao desempenho, durabilidade e consumo.

Um carro-teste consome 4,0 kg de biodiesel para
realizar trabalho mecnico. Se a queima de 1 g de
biodiesel libera 5,0 10
3
cal e o rendimento do motor
de 15%, o trabalho mecnico realizado, em joules,
vale, aproximadamente,
Dado: 1 cal = 4,2 joules
a) 7,2 10
5

b) 1,0 10
6

c) 3,0 10
6

d) 9,0 10
6

e) 1,3 10
7

3. Um mol de um gs ideal aquecido, a presso
constante, passando da temperatura Ti = 300 K para a
temperatura Tf = 350 K. O trabalho realizado pelo gs
durante esse processo aproximadamente (o valor da
constante universal dos gases R 8,31 J/(mol.K))
igual a:
a) 104 J. c) 312 J. e) 520 J.
b) 208 J. d) 416 J.

4. A figura a seguir representa o grfico presso
versus volume da expanso isotrmica de um gs
perfeito. correto afirmar que:


a) a curva apresentada uma isobrica
b) a rea sombreada do grfico representa
numericamente o trabalho realizado pelo gs ao se
expandir
c) a rea sombreada numericamente igual ao trabalho
realizado sobre o gs para sua expanso
d) a curva do grfico uma isocrica

4. Um sistema termodinmico realiza o ciclo ABCA
representado a seguir:

O trabalho realizado pelo sistema no ciclo vale, em
joules:
a) 2,5 10
5

b) 4,0 10
5
c) 3,0 10
5

d) 5,0 10
5

e) 2,0 10
5


5. A primeira lei da termodinmica diz respeito :
a) dilatao trmica
b) conservao da massa
c) conservao da quantidade de movimento
d) conservao da energia
e) irreversibilidade do tempo

6. Considere as proposies a seguir sobre
transformaes gasosas.
I. Numa expanso isotrmica de um gs perfeito, sua
presso aumenta.
II. Numa compresso isobrica de um gs perfeito, sua
temperatura absoluta aumenta.
III. Numa expanso adiabtica de um gs perfeito, sua
temperatura absoluta diminui.
Pode-se afirmar que apenas:
a) I correta.
b) II correta.
c) III correta.
d) I e II so corretas.
e) II e III so corretas.

7. Com relao s transformaes sofridas por um gs
perfeito, assinale a alternativa incorreta.
a) Na transformao adiabtica, a variao de energia
cintica das molculas nula
b) Na transformao isobrica, no h variao da
presso do gs.
c) Na transformao isotrmica, a energia cintica
mdia das molculas no se altera.
d) Na transformao adiabtica, no h troca de calor
com o meio exterior.
e) Na transformao isotrmica, h troca de calor com
o meio exterior.

8. Considere uma certa massa de um gs ideal em
equilbrio termodinmico. Numa primeira experincia,
faz-se o gs sofrer uma expanso isotrmica durante a
qual realiza um trabalho W e recebe 150J de calor do
meio externo. Numa segunda experincia, faz-se o gs
sofrer uma expanso adiabtica, a partir das mesmas
condies iniciais, durante a qual ele realiza o mesmo
trabalho W.
Calcule a variao de energia interna U do gs nessa
expanso adiabtica.

Please purchase PDF Split-Merge on www.verypdf.com to remove this watermark.
Incluso para a Vida Fsica B

Pr-Vestibular da UFSC

14
9. Quando um gs ideal sofre uma expanso
isotrmica,
a) a energia recebida pelo gs na forma de calor igual
ao trabalho realizado pelo gs na expanso.
b) no troca energia na forma de calor com o meio
exterior.
c) no troca energia na forma de trabalho com o meio
exterior.
d) a energia recebida pelo gs na forma de calor
igual variao da energia interna do gs.
e) o trabalho realizado pelo gs igual variao da
energia interna do gs.

10. Um gs ideal sofre uma transformao: absorve
50cal de energia na forma de calor e expande-se
realizando um trabalho de 300J. Considerando
1cal=4,2J, a variao da energia interna do gs , em J,
de
a) 250 b) -250 c) 510 d) -90 e) 90

11. A respeito de conceitos relacionados
Termodinmica, assinale a(s) alternativa(s) correta(s).

01. A energia interna de um gs ideal pode ser medida
diretamente.
02. Em algumas situaes, calor adicionado a uma
substncia e no ocorre nenhuma variao de
temperatura. Tais situaes no esto de acordo
com a definio usual de calor como sendo uma
forma de energia em trnsito devido a uma
diferena de temperatura.
04. impossvel a ocorrncia de processos nos quais
no se transfira e nem se retire calor de um sistema
e nos quais a temperatura do sistema sofra
variao.
08. Durante uma transformao isotrmica de um gs
ideal, existe equivalncia entre o calor e o trabalho
trocados entre o sistema e o exterior.
16. A capacidade calorfica de um corpo representa a
quantidade de calor que o corpo pode estocar a
uma certa temperatura.
32. Durante uma transformao cclica de um gs
ideal, existe equivalncia entre o calor e o trabalho
trocados entre o sistema e o exterior.
64. Na passagem de um sistema de um estado inicial 1
para um estado final 2, a variao da energia
interna entre os dois estados depende do processo
que provocou tal passagem.

12. Os estudos cientficos desenvolvidos pelo
engenheiro francs Nicolas Sadi Carnot (1796-1832)
na tentativa de melhorar o rendimento de mquinas
trmicas serviram de base para a formulao da
segunda lei da termodinmica.
Acerca do tema, considere as seguintes afirmativas:
1. O rendimento de uma mquina trmica a razo
entre o trabalho realizado pela mquina num ciclo e o
calor retirado do reservatrio quente nesse ciclo.
2. Os refrigeradores so mquinas trmicas que
transferem calor de um sistema de menor temperatura
para outro a uma temperatura mais elevada.
3. possvel construir uma mquina, que opera em
ciclos, cujo nico efeito seja retirar calor de uma fonte
e transform-lo integralmente em trabalho.

Assinale a alternativa correta.
a) Somente as afirmativas 1 e 3 so verdadeiras.
b) Somente a afirmativa 1 verdadeira.
c) Somente a afirmativa 2 verdadeira.
d) Somente as afirmativas 1 e 2 so verdadeiras.
e) Somente as afirmativas 2 e 3 so verdadeiras.

13. A cada ciclo, uma mquina trmica extrai 45 kJ de
calor da sua fonte quente e descarrega 36 kJ de calor
na sua fonte fria. O rendimento mximo que essa
mquina pode ter de
a) 20%. b) 25%. c) 75%. d) 80%. e) 100%.

14. O uso de combustveis no renovveis, como o
petrleo, tem srias implicaes ambientais e
econmicas. Uma alternativa energtica em estudo
para o litoral brasileiro o uso da diferena de
temperatura da gua na superfcie do mar (fonte
quente) e de guas mais profundas (fonte fria) em uma
mquina trmica para realizar trabalho. (Desconsidere
a salinidade da gua do mar para a anlise das
respostas).

Assinale a(s) proposio(es) correta(s).
01. Supondo que a mquina trmica proposta opere em
um ciclo de Carnot, teremos um rendimento de
100%, pois o ciclo de Carnot corresponde a uma
mquina trmica ideal.
02. Uma mquina com rendimento igual a 20% de
uma mquina ideal, operando entre 7 C e 37 C,
ter um rendimento menor que 10%.
04. Na situao apresentada, a temperatura mais baixa
da gua de aproximadamente 4 C pois, ao
contrrio da maioria dos lquidos, nesta
temperatura a densidade da gua mxima.
08. impossvel obter rendimento de 100% mesmo
em uma mquina trmica ideal, pois o calor no
pode ser transferido espontaneamente da fonte fria
para a fonte quente.
16. No possvel obtermos 100% de rendimento,
mesmo em uma mquina trmica ideal, pois isto
viola o princpio da conservao da energia.

UNIDADE 7

PTICA GEOMTRICA, ESPELHOS
PLANOS E ESFRICOS

Estuda os fenmenos luminosos
Luz: Agente fsico capaz de sensibilizar nossos rgos
visuais (retina). Esta propaga-se atravs de ondas
Please purchase PDF Split-Merge on www.verypdf.com to remove this watermark.
Incluso para a Vida Fsica B

Pr-Vestibular da UFSC

15
eletromagnticas, isto , podem viajar no vcuo
(ausncia de matria).

Raio de Luz


Feixes ou pinceis de luz

Convergente

Divergente

Paralelo

FONTE DE LUZ
Corpo luminoso: (Fonte Primria)- Emite luz prpria.
o Incandescente : Quente
o Luminescente: Fria : Fluorescente e Fosforescente.
o Ex: Sol, lmpada acesa, etc...

Corpo iluminado: (Fonte Secundria)- Reflete luz
recebida de outras fontes.
Ex: Lua, lmpada apagada, etc....

Luz policromtica: possui vrias cores.

Luz monocromtica: possui apenas uma cor

Fenmenos Luminosos

Reflexo da Luz
Reflexo especular

Reflexo difusa


Fenmeno que ocorre devido propagao
retilnea da luz
Princpio da ptica geomtrica

- Princpios de propagao da Luz
Propagao retilnea da luz em meios transparentes e
homogneos, a luz propaga-se em linha reta.

- Cmara escura de orifcio.

- H - Altura do objeto
- h - Altura da imagem na cmara
- D - Distncia do objeto a cmara
- d - Comprimento da cmara.
- Princpio de Reversibilidade da Luz.
"A trajetria da luz independe do sentido da
propagao"

- Princpio da Independncia dos Raios Luminosos.
"Raios de luz que se cruzam no interferem entre si"

ESPELHOS PLANOS

Formao de Imagens em Espelhos planos

1 - Imagens de um ponto


2 - Imagens de um corpo extenso

- Imagem Virtual (Atrs do espelho)

Caractersticas da imagem no espelho plano
1- Imagem virtual (Atrs do espelho)
2- Mesmo tamanho do objeto
3- Imagem e objeto so equidistantes (mesma
distncia) do espelho
Please purchase PDF Split-Merge on www.verypdf.com to remove this watermark.
Incluso para a Vida Fsica B

Pr-Vestibular da UFSC

16
4 - Objeto e imagem so reversos (enantiomorfos)


COMPOSIO DOS ESPELHOS PLANOS
Dois espelhos podem formar vrias imagens e o
nmero de imagens depende do ngulo formado pelos
espelhos.
1
360
=
o
N

Campo visual de um espelho plano

C.V. - Campo Visual

Espelhos Esfricos


Elementos de um espelho esfrico.


Equaes dos Espelhos Esfricos

- R = Raio de curvatura
- f = Distncia focal
- R = 2f
- p = Distncia do objeto ao espelho
- p' = Distncia da imagem ao espelho

- Equao dos Pontos Conjugados (Eq. Gauss)
`
1 1 1
p p f
+ =

p f
f
p
p
o
i
A

= = =
`


Regra de sinais.
P`(+) Imagem real
P`(-) Imagem virtual
f(+) Espelho cncavo ou lente convergente
f(-) Espelho convexo ou lente divergente
A(+) Imagem virtual
A(-) Imagem real
A > 1 Imagem maior que o objeto
A < 1 Imagem menor que o objeto
A = 1 Imagem do mesmo tamanho do objeto

Considerao:
Se a imagem projetada, ela ser REAL. Sendo real
ela ser INVERTIDA.

Elementos de um Espelho Esfrico

- Foco - C/2
- (alfa)= ngulo de abertura
- C = Raio de curvatura (R)
- V - Vertice
- EP- Eixo Principal

Raios Incidentes Notveis


1. Todo raio de luz que incide paralelamente ao EP,
reflete na direo do foco.
2. Todo raio de luz que incide na direo do foco,
reflete paralelamente ao EP.
3. Todo raio de luz que incide na direo do C,
reflete na mesma direo.
4. Todo raio de luz que incide no vrtice do espelho,
reflete simetricamente em relao ao EP.
Condio de Nitidez de Gauss
Imagens ntidas para alfa < 10
Foco: Ponto de encontro dos raios refletidos (ou de
seus prolongamentos) paralelamente ao eixo principal.
Please purchase PDF Split-Merge on www.verypdf.com to remove this watermark.
Incluso para a Vida Fsica B

Pr-Vestibular da UFSC

17
- Cncavo

- Convexo


Exerccios de Sala #

1. A figura adiante mostra uma vista superior de dois
espelhos planos montados verticalmente, um
perpendicular ao outro. Sobre o espelho OA incide um
raio de luz horizontal, no plano do papel, mostrado na
figura. Aps reflexo nos dois espelhos, o raio emerge
formando um ngulo com a normal ao espelho OB.
O ngulo vale:

a) 0 b) 10 c) 20 d) 30 e) 40

Tarefa Mnima #

2. Aproveitando materiais reciclveis, como latas de
alumnio de refrigerantes e caixas de papelo de
sapatos, pode-se construir uma mquina fotogrfica
utilizando uma tcnica chamada "pin hole" (furo de
agulha), que, no lugar de lentes, usa um nico furo de
agulha para captar a imagem num filme fotogrfico.
As mquinas fotogrficas "pin hole" registram um
mundo em imagens com um olhar diferente. Um poste
com 4 m de altura fotografado numa mquina "pin
hole". No filme, a altura da imagem do poste, em
centmetros, :

a) 12 b) 10 c) 8 d) 6 e) 4






3. A velocidade da luz, no vcuo, vale
aproximadamente 3,0.10
8
m/s. Para percorrer a
distncia entre a Lua e a Terra, que de 3,9.10
5
km, a
luz leva:
a) 11,7 s b) 8,2 s c) 4,5 s d) 1,3 s e) 0,77 s

4. Na figura a seguir, F uma fonte de luz extensa e A
um anteparo opaco.

Pode-se afirmar que I, II e III so, respectivamente,
regies de:
a) sombra, sombra e penumbra.
b) sombra, sombra e sombra.
c) penumbra, sombra e penumbra.
d) sombra, penumbra e sombra.
e) penumbra, penumbra e sombra.

5. No vcuo, todas as ondas eletromagnticas.
a) tm a mesma frequncia.
b) tm a mesma intensidade.
c) se propagam com a mesma velocidade.
d) se propagam com velocidades menores que a da luz.
e) so polarizadas.

6. Considere as seguintes afirmativas:
I- A gua pura um meio translcido.
II- O vidro fosco um meio opaco.
III- O ar um meio transparente.
Sobre as afirmativas acima, assinale a alternativa
correta.
a) Apenas a afirmativa I verdadeira.
b) Apenas a afirmativa II verdadeira.
c) Apenas a afirmativa III verdadeira.
d) Apenas as afirmativas I e a III so verdadeiras.
e) Apenas as afirmativas II e a III so verdadeiras.

7. Ana Maria, modelo profissional, costuma fazer
ensaios fotogrficos e participar de desfiles de moda.
Em trabalho recente, ela usou um vestido que
apresentava cor vermelha quando iluminado pela luz
do sol.
Ana Maria ir desfilar novamente usando o mesmo
vestido. Sabendo-se que a passarela onde Ana Maria
vai desfilar ser iluminada agora com luz
monocromtica verde, podemos afirmar que o pblico
perceber seu vestido como sendo

a) verde, pois a cor que incidiu sobre o vestido.
b) preto, porque o vestido s reflete a cor vermelha.
c) de cor entre vermelha e verde devido mistura das
cores.
d) vermelho, pois a cor do vestido independe da
radiao incidente.





Please purchase PDF Split-Merge on www.verypdf.com to remove this watermark.
Incluso para a Vida Fsica B

Pr-Vestibular da UFSC

18
8. Muitas vezes, ao examinar uma vitrina, possvel
observar no s os objetos que se encontram em
exposio atrs do vidro, como tambm a imagem de
si prprio formada pelo vidro, A formao dessa
imagem pode ser explicada pela.
a) reflexo parcial da luz.
b) reflexo total da luz.
c) refrao da luz.
d) transmisso da luz.
e) difrao da luz.

9. Uma cmara escura de orifcio fornece a imagem de
um prdio, o qual se apresenta com altura de 5 cm.
Aumentando-se de 100 m a distncia do prdio
cmara, a imagem se reduz para 4 cm de altura. Qual
a distncia entre o prdio e a cmara, na primeira
posio?
a) 100 m b) 200 m c) 300 m d) 400 m e) 500 m

10. Em um dado instante uma vara de 2,0 m de altura,
vertical, projeta no solo, horizontal, uma sombra de 50
cm de comprimento. Se a sombra de um prdio
prximo, no mesmo instante, tem comprimento de 15
m, qual a altura do prdio?

11. Um lpis encontra-se na frente de um pequeno
espelho plano E, como mostra a figura. O lpis e a
imagem esto corretamente representados na
alternativa:

12. Uma cmara escura de orifcio fornece a imagem
de um prdio, o qual se apresenta com altura de 5cm.
Aumentando-se de 100m a distncia do prdio
cmara, a imagem se reduz para 4cm de altura. Qual
a distncia entre o prdio e a cmara, na primeira
posio?
a) 100 m b) 200 m c) 300 m d) 400 m e) 500 m

13. A luz solar se propaga e atravessa um meio
translcido. Qual das alternativas a seguir representa o
que acontece com a propagao dos raios de luz?



14. A luz solar penetra numa sala atravs de uma
janela de vidro transparente. Abrindo-se a janela, a
intensidade da radiao solar no interior da sala:
a) permanece constante.
b) diminui, graas conveco que a radiao solar
provoca.
c) diminui, porque os raios solares so concentrados
na sala pela janela de vidro.
d) aumenta, porque a luz solar no sofre mais difrao.
e) aumenta, porque parte da luz solar no mais se
reflete na janela.

15. Admita que o sol subitamente "morresse", ou seja,
sua luz deixasse de ser emitida. 24 horas aps este
evento, um eventual sobrevivente, olhando para o cu,
sem nuvens, veria:
a) a Lua e estrelas.
b) somente a Lua.
c) somente estrelas.
d) uma completa escurido.
e) somente os planetas do sistema solar.

16. A figura a seguir mostra um objeto A colocado a
5m de um espelho plano, e um observador O,
colocando a 7m deste mesmo espelho.
Um raio de luz que parte de A e atinge o observador O
por reflexo no espelho percorrer, neste trajeto de A
para O

a) 9m b) 12m c) 15m d) 18m e) 21m

17. Uma pessoa deseja usar um espelho plano
vertical, a partir do cho, para ver-se de corpo inteiro,
desde a cabea at os ps. A altura do espelho:
a) deve ser pelo menos igual altura da pessoa.
b) deve ser pelo menos igual metade da altura da
pessoa.
c) depende da distncia da pessoa ao espelho.
d) depende da altura da pessoa e da sua distncia ao
espelho.

18. Um menino, parado em relao ao solo, v sua
imagem em um espelho plano E colocado parede
traseira de um nibus. Se o nibus se afasta do menino
com velocidade de 2m/s, o mdulo da velocidade da
imagem, em relao ao solo, :

a) 4 m/s b) 3 m/s c) 2 m/s d) 1 m/s

19. Um raio de luz r incide sucessivamente em dois
espelhos planos E1 e E2, que formam entre si um
ngulo de 60, conforme representado no esquema a
seguir. Nesse esquema o ngulo igual a
Please purchase PDF Split-Merge on www.verypdf.com to remove this watermark.
Incluso para a Vida Fsica B

Pr-Vestibular da UFSC

19

a) 80 b) 70 c) 60 d) 50 e) 40

20. Quando colocamos um pequeno objeto real entre
o foco principal e o centro de curvatura de um espelho
esfrico cncavo de Gauss, sua respectiva imagem
conjugada ser:
a) real, invertida e maior que o objeto.
b) real, invertida e menor que o objeto.
c) real, direita e maior que o objeto.
d) virtual, invertida e maior que o objeto.
e) virtual, direita e menor que o objeto.

21. Um objeto colocado a 15 cm de um espelho
cncavo forma uma imagem no infinito. Se for
colocada uma lente de distncia focal 15 cm, distante
30 cm do espelho, aquela imagem formada no infinito
agora estar:
a) ainda no infinito.
b) reduzida e a 15 cm do espelho.
c) reduzida e a 30 cm do espelho.
d) ampliada e a 45 cm do espelho.
e) concentrada em um ponto distante 45 cm do
espelho.

22. Se um espelho forma uma imagem real e
ampliada de um objeto, ento o espelho :
a) convexo e o objeto est alm do foco.
b) convexo e o objeto est entre o foco e o espelho.
c) cncavo e o objeto est entre o foco e o centro do
espelho.
d) cncavo e o objeto est alm do foco.
e) cncavo ou convexo e com o objeto entre o foco e o
centro do espelho.

23. Um objeto real, representado pela seta, colocado
em frente a um espelho podendo ser plano ou esfrico
conforme as figuras.
A imagem fornecida pelo espelho ser virtual:

a) apenas no caso I.
b) apenas no caso II.
c) apenas nos casos I e II.
d) nos casos I e IV e V.
e) nos casos I, II e III.




24. Considere o esquema tico a seguir, onde V o
vrtice do espelho cncavo, C seu centro de curvatura
e) F seu foco principal.

Associe as colunas a seguir:

POSIO DO
OBJETO
( ) esquerda
de C
( ) sobre C
( ) entre C e F
( ) sobre F
( ) entre F e V
CARACTERSTICAS DA
IMAGEM
1. real, maior e invertida
2. imagem imprpria
3. real, menor e invertida
4. real, igual e invertida
5. virtual, maior e direita

A sequncia correta, de cima para baixo, ser:
a) 3, 4, 1, 5, 3.
b) 1, 3, 4, 5, 2.
c) 5, 4, 2, 1, 3.
d) 1, 5, 4, 3, 2.
e) 3, 4, 1, 2, 5.

UNIDADE 8

REFRAO DA LUZ

Refrao da Luz
Variao da velocidade de propagao da luz quando
ocorre mudana de meio. Esta variao quase sempre
vem acompanhada de desvio do raio luminoso.

ndice de Refrao Absoluto de um meio (N):

'
v
c
n =


N meio , c = Velocidade da luz no vcuo , V =
Velocidade da luz no meio;

- Vcuo: c : N(vcuo) = 1
- Ar: V(ar) : N(ar) = (aproximadamente) 1;
- gua: V(gua) : N(gua) > 1;
- Vidro: V(vidro) : N(vidro) > 1 ;

- Concluso: N 1
Obs.: N mede a dificuldade que a luz encontra em
viajar pelo meio.

- ndice de Refrao Relativo:

N
A,B
= N
A
/ N
B
= V
B
/ V
A


Leis da Refrao:

o 1 - Raio Incidente (RI) , Reta Normal (N) e Raio
Refratado (RR) so coplanares;

o 2 - Snell Descartes:
B B A A
sen n sen n u u . . =

Please purchase PDF Split-Merge on www.verypdf.com to remove this watermark.
Incluso para a Vida Fsica B

Pr-Vestibular da UFSC

20
Refrao Atmosfrica
1. A luz, ao entrar na atmosfera terrestre, sofre
pequenas variaes ao passar dentre as diversas
camadas de ar.
2. Pela refringncia ser diretamente proporcional a
densidade, a luz desvia do menos refringente para o
mais refringente, aproximando-se da reta normal;
3. Quando chega perto do cho existe um ar super
aquecido de menor densidade que provoca um desvio
do meio mais refringente para o mais refringente,
provocando, as vezes, a reflexo total. Isso caracteriza
as miragens e as impresses de asfalto molhado que
temos;

Reflexo Total

- Fibras pticas;
- Miragens.


- Condies:
- A luz deve vir do + refringente para o menos
refringente;
- O ngulo de incidncia deve ser maior do que o
ngulo limite(L);

Clculo do ngulo limite (L):



Dioptro Plano
Associao de dois meios com refringncia diferentes,
separadas por uma superfcie plana.

p
p
n
n
, ,
=




Prisma ptico

ai = i - r
a = a1 + a2
A = r + r'
- Legenda:
- A: ngulo de abertura ou Refringncia;
- a1: ngulo desvio (1 Face);
- a2: ngulo desvio (2 Face);
- a : ngulo desvio Total

- Concluso:
a = i + i' - A
- Obs.: Pode existir reflexo total em prismas
pticos.

Exerccios de Sala #

1. Na figura adiante, um raio de luz monocromtico se
propaga pelo meio A , de ndice de refrao 2,0.

Dados: sen 37 = 0,60, sen 53 = 0,80
Devemos concluir que o ndice de refrao do meio B
:
a) 0,5 b) 1,0 c) 1,2 d) 1,5 e) 2,0

Tarefa Mnima #

2. Um raio luminoso incide sobre a superfcie da gua,
conforme a figura a seguir. Qual alternativa representa
o que acontece com o raio?


3.Quando um raio de luz monocromtica, proveniente
de um meio homogneo, transparente e istropo,
identificado por meio A, incide sobre a superfcie de
separao com um meio B, tambm homogneo,
transparente e istropo, passa a se propagar nesse
segundo meio, conforme mostra a figura. Sabendo-se
que o ngulo menor que o ngulo , podemos
afirmar que:
Please purchase PDF Split-Merge on www.verypdf.com to remove this watermark.
Incluso para a Vida Fsica B

Pr-Vestibular da UFSC

21
a) no meio A a
velocidade de
propagao da luz
menor que no meio B.
b) no meio A a
velocidade de
propagao da luz
sempre igual velocidade no meio B.
c) no meio A a velocidade de propagao da luz
maior que no meio B.
d) no meio A a velocidade de propagao da luz
maior que no meio B, somente se o ngulo
limite de incidncia.
e) no meio A a velocidade de propagao da luz
maior que no meio B, somente se o ngulo
limite de refrao.

4. Amanda segura um copo de vidro cheio de gua.
Um raio luminoso monocromtico vindo do ar com
velocidade de aproximadamente atravessa
todo o copo. Sobre este fenmeno, analise as
afirmaes a seguir:
I - Ao entrar no vidro, a velocidade da onda luminosa
passa a ser maior do que .
II - ao entrar na gua, a velocidade da onda luminosa
passa a ser menor do que .
III - Ao sair do copo, a velocidade da onda luminosa
volta a ser de .
IV - Durante todo o fenmeno, a frequncia da onda
luminosa permanece constante.
Assinale a nica alternativa correta:
a) I.
b) Apenas II.
c) Apenas III.
d) II, III e IV.
e) Apenas II e III.

5. Um raio de luz monocromtica, propagando-se num
meio transparente A, cujo ndice de refrao n
A
,
incide na superfcie S de separao com outro meio
transparente B, de ndice de refrao n
B
, e se refrata
como mostra o esquema a seguir.

Sendo i o ngulo de incidncia e r o ngulo de
refrao, analise as afirmaes que seguem.

( ) Se i > r ento n
A
> n
B
.
( ) A reflexo total pode ocorrer desde que a luz
esteja se propagando do meio mais refringente para o
menos refringente.
( ) O ngulo limite L para esse par de meios tal que
senL=n
B
/n
A
.
( ) A lei de Snell-Descartes, da refrao, para a
situao mostrada no esquema expressa por: n
A
sen
i=n
B
sen(r).
( ) Se n
A
> n
B
, a velocidade de propagao da luz
maior no meio A que no B.

6. A figura a seguir mostra um lpis de comprimento
AB, parcialmente imerso na gua e sendo observado
por um estudante. Assinale a(s) proposio(es)
correta(s).

01. O estudante v o lpis "quebrado" na interface ar-
gua, porque o ndice de refrao da gua maior
do que o do ar.
02. O feixe luminoso proveniente do ponto B, ao
passar da gua para o ar se afasta da normal,
sofrendo desvio.
04. O estudante v o lpis "quebrado" na interface ar-
gua, sendo o fenmeno explicado pelas leis da
reflexo.
08. O observador v o lpis "quebrado" na interface
ar-gua porque a luz sofre disperso ao passar do
ar para a gua.
16. O ponto B', visto pelo observador, uma imagem
virtual.

7. Uma fibra ptica uma estrutura cilndrica feita de
vidro, constituda, basicamente, de dois materiais
diferentes, que compem o ncleo e a casca, como
pode ser visto em corte na figura a seguir.

Sua propriedade de guiamento dos feixes de luz est
baseada no mecanismo da reflexo interna total da luz
que ocorre na interface ncleo-casca. Designando por
n(ncleo) e n(casca) os ndices de refrao do ncleo e
da casca, respectivamente, analise as afirmaes a
seguir, que discutem as condies para que ocorra a
reflexo interna total da luz.

I. n(ncleo) > n(casca).
II. Existe um ngulo L, de incidncia na interface
ncleo-casca, tal que sen(L)=n(casca)/n(ncleo).
III. Raios de luz com ngulos de incidncia > L
sofrero reflexo interna total, ficando presos
dentro do ncleo da fibra.
Analisando as afirmaes, podemos dizer que:

a) somente I est correta.
b) somente I e II esto corretas.
c) todas esto corretas.
d) somente I e III esto corretas.
e) nenhuma se aplica ao fenmeno da reflexo interna
total da luz em uma fibra ptica.

8. Um pssaro sobrevoa em linha reta e a baixa
altitude uma piscina em cujo fundo se encontra uma
pedra. Podemos afirmar que
a) com a piscina cheia o pssaro poder ver a pedra
Please purchase PDF Split-Merge on www.verypdf.com to remove this watermark.
Incluso para a Vida Fsica B

Pr-Vestibular da UFSC

22
durante um intervalo de tempo maior do que se a
piscina estivesse vazia.
b) com a piscina cheia ou vazia o pssaro poder ver a
pedra durante o mesmo intervalo de tempo.
c) o pssaro somente poder ver a pedra enquanto
estiver voando sobre a superfcie da gua.
d) o pssaro, ao passar sobre a piscina, ver a pedra
numa posio mais profunda do que aquela em que
ela realmente se encontra.
e) o pssaro nunca poder ver a pedra.

9. Um raio de luz monocromtica passa do meio 1
para o meio 2 e deste para o meio 3. Sua velocidade de
propagao relativa aos meios citados v1, v2 e v3,
respectivamente.

O grfico representa a variao da velocidade de
propagao da luz em funo do tempo ao atravessar
os meios mencionados, considerados homogneos.
Sabendo-se que os ndices de refrao do diamante, do
vidro e do ar obedecem desigualdade n(diam) >
n(vidro) > n(ar), podemos afirmar que os meios 1, 2 e
3 so, respectivamente,

a) diamante, vidro, ar.
b) diamante, ar, vidro.
c) ar, diamante, vidro.
d) ar, vidro, diamante.
e) vidro, diamante, ar.

10. Uma folha de papel, com um texto impresso, est
protegida por uma espessa placa de vidro. O ndice de
refrao do ar 1,0 e o do vidro 1,5. Se a placa tiver
3cm de espessura, a distncia do topo da placa
imagem de uma letra do texto, quando observada na
vertical, :

a) 1 cm
b) 2 cm
c) 3 cm
d) 4 cm

UNIDADE 9

LENTES ESFRICAS

Lentes Esfricas:
Associao de dois meios com refringncias diferentes
separados por duas superfcies curvas ou uma plana e
outra curva.
Basicamente mais comum termos uma lente cujo
ndice de refrao maior que o ndice do meio. Ex:
Lentes de vidros imersas no ar.

- Lentes de bordas Finas (Delgadas):


- Se N(lente) > N(meio)
- Lente Convergente
- Fo>0
- Representao:


- Lentes de Bordas Grossas:

- Se N(lente) > N(meio)
- Lente Divergente
- Fo<0
- Representao:




Lentes Esfricas: Construo de imagens em lentes
convergentes

1 - Objeto antes do Ao;(Olho humano)

Imagem:
- Real;
- Invertida;
- Menor.
2 - Objeto no Ao; (Mquina de Xerox)



Please purchase PDF Split-Merge on www.verypdf.com to remove this watermark.
Incluso para a Vida Fsica B

Pr-Vestibular da UFSC

23
Imagem:
- Real;
- Invertida;
- Igual.

3 - Objeto entre Ao e Fo;

Imagem:
- Real;
- Invertida;
- Maior.

4 - Objeto no Fo;

Imagem:
- Imprpria.

5 - Objeto entre Fo e O;

Imagem:
- Virtual;
- Direita;
- Maior;

Lentes Esfricas: Construo de imagens em lentes
divergentes

- Caso nico;


- Imagem:
- Virtual;
- Direita;
- Menor.

Lentes Esfricas: Frmulas

`
1 1 1
p p f
+ =
p f
f
p
p
o
i
A

= = =
`


Legenda:
- F = Foco Objeto;
- p = Distncia Imagem lente;
- p = Distncia Objeto lente;
- i = Tamanho da imagem;
- o = Tamanho do objeto;
- A = Aumento.

F > 0 = Lente Convergente;
F < 0 = Lente Divergente;
p > 0 - imagem real invertida;
p < 0 - imagem virtual direita;
i > 0 - imagem direita virtual;
i < 0 - imagem invertida real;
A > 0 - Imagem Direita;
A < 0 - Imagem Invertida.

Lentes Esfricas: Vergncia

f
V
1
=

Unidade para vergncia: m
-1
ou di (dioptria)

Defeitos da Viso

Miopia
a condio em que os olhos podem ver objetos
que esto perto, mas no capaz de ver claramente
objetos que esto longe.
Na miopia, o foco das imagens ocorre antes da
retina.
A miopia tem tendncia familiar. Geralmente a miopia
aumenta durante a fase de crescimento.
Tratamento
culos, lentes de contato
ou cirurgia refrativa. A cirurgia
refrativa procura modificar a
curvatura da crnea,
provocando achatamento da
parte central, para que a
imagem se forme na retina

Hipermetropia
Nesta situao o olho geralmente menor que o
normal, dificultando para que o cristalino focalize na
retina os objetos colocados prximo ao olho.
Normalmente, as crianas so moderadamente
hipermtropes, condio que diminui com a idade.
Tratamento
A hipermetropia pode
ser tratada com culos e lentes
de contato.


Astigmatismo
O astigmatismo geralmente causado por
irregularidade da crnea, e seu efeito a distoro da
Please purchase PDF Split-Merge on www.verypdf.com to remove this watermark.
Incluso para a Vida Fsica B

Pr-Vestibular da UFSC

24
imagem.
Tratamento
A correo do
astigmatismo pode ser feita por
culos ou por lentes de contato.
Existem algumas tcnicas
cirurgicas para reduzir grandes
astigmatismos (semelhantes as
da cirurgia de miopia).

Presbiopia
Acompanhando o
envelhecimento, o
cristalino perde aos
poucos a sua
elasticidade. Com isto, o
olho fica sem
capacidade de
acomodar-se, ou seja,
conseguir foco para ler
ou costurar.
Este processo comea a
ser sentido por volta dos
40 anos.
Tratamento
O uso de culos para
perto passa a ser necessrio
em quem antes enxergava
bem.
Os pacientes que j
utilizavam culos passam a
precisar lentes diferentes
para longe e para perto.
Assim, podem usar um
culos bifocal ou multifocal
(para longe e para perto), ou
um pequeno apenas para
leitura.
Exerccios de Sala #
1. Um objeto, colocado entre o centro e o foco de uma
lente convergente, produzir uma imagem:

a) virtual, reduzida e direita
b) real, ampliada e invertida
c) real, reduzida e invertida
d) virtual, ampliada e direita


2. Na figura a seguir, representam-se vrios raios
luminosos que atravessam uma lente convergente. Dos
cinco raios representados, indique aquele que est
representado de maneira incorreta (F e F' so os focos
da lente):

a) 4 b) 5 c) 1 d) 2 e) 3
Exerccios de Sala #

3. Quando um raio de luz monocromtica passa
obliquamente pela superfcie de separao de um meio
para outro mais refringente, o raio aproxima-se da
normal superfcie. Por essa razo, uma lente pode ser
convergente ou divergente, dependendo do ndice de
refrao do meio em que se encontra. As figuras 1 e 2
representam lentes com ndice de refrao n imersas
em meios de ndice de refrao n, sendo N a normal
superfcie curva das lentes.

Considerando essas informaes, conclui-se que:
a) a lente 1 convergente se n
2
< n
1
.
b) a lente 1 convergente se n
2
> n
1
.
c) a lente 2 divergente se n
2
> n
1
.
d) a lente 2 convergente se n
2
< n
1
.
e) as lentes 1 e 2 so convergentes se n
1
= n
2
.

4. Um objeto (O) encontra-se em frente a uma lente.
Que alternativa representa corretamente a formao da
imagem (I)?


4. A glicerina uma substncia transparente,
cujo ndice de refrao praticamente igual
ao do vidro comum. Uma lente, biconvexa,
de vidro totalmente imersa num recipiente
com glicerina. Qual das figuras a seguir
melhor representa a transmisso de um feixe
de luz atravs da lente?








Please purchase PDF Split-Merge on www.verypdf.com to remove this watermark.
Incluso para a Vida Fsica B

Pr-Vestibular da UFSC

25
6. O esquema abaixo mostra a imagem projetada sobre
uma tela, utilizando um nico instrumento ptico
"escondido" pelo retngulo sombreado. O tamanho da
imagem obtida igual a duas vezes o tamanho do
objeto que se encontra a 15cm do instrumento ptico.

Nessas condies, podemos afirmar que o retngulo
esconde:
a) um espelho cncavo, e a distncia da tela ao espelho
de 30cm.
b) uma lente convergente, e a distncia da tela lente
de 45cm.
c) uma lente divergente, e a distncia da tela lente
de 30cm.
d) uma lente convergente, e a distncia da tela lente
de 30cm.
e) um espelho cncavo, e a distncia da tela ao espelho
de 45cm.

7. Um estudante, utilizando uma lente, consegue
projetar a imagem da chama de uma vela em uma
parede branca, dispondo a vela e a lente na frente da
parede conforme a figura.


Assinale a(s) proposio(es) correta(s).

01. Tanto uma lente convergente quanto uma lente
divergente projetam a imagem de um ponto luminoso
real na parede.
02. A lente convergente, necessariamente, porque
somente uma lente convergente fornece uma
imagem real de um objeto luminoso real.
04. A imagem virtual e direita.
08. A imagem real e invertida.
16. A lente divergente, e a imagem virtual para que
possa ser projetada na parede.
32. Se a lente convergente, a imagem projetada na
parede pode ser direita ou invertida.
64. A imagem real, necessariamente, para que possa
ser projetada na parede.

8. Um objeto colocado a uma distncia de 12cm de
uma lente delgada convergente, de 8cm de distncia
focal. A distncia, em centmetros, da imagem
formada em relao lente :
a) 24 b) 20 c) 12 d) 8 e) 4

9. Uma lente convergente de 2,00 dioptrias
(popularmente 2,00 "graus") tem distncia focal de:
a) 500cm b) 200cm c) 100cm d) 50cm e) 20cm

10. Um objeto colocado prximo de uma lente projeta
uma imagem de altura trs vezes maior que ele e
invertida. A distncia entre o objeto e a imagem de
40 cm.

Assinale a(s) proposio(es) correta(s).
01. A distncia entre o objeto e a lente de 20 cm.
02. A distncia focal da lente de 7,5 cm.
04. A lente convergente.
08. Uma lente divergente s pode formar imagens
virtuais.
16. Uma lente convergente pode formar imagens
reais e virtuais.

11. O esquema representa, em escala, uma lente
divergente L, o eixo principal, o objeto O e os raios de
luz r1 e r2 que so utilizados para localizar a imagem
do objeto.


Acompanhe o traado dos raios r
1
e r
2
para localizar a
imagem do objeto e os focos da lente.

( ) O objeto tem 10 cm de comprimento
( ) O objeto est a 15 cm da lente.
( ) A imagem se forma a 20 cm da lente.
( ) A imagem tem 10 cm de comprimento.
( ) A distncia focal da lente 13 cm.

12. As deficincias de viso so compensadas com o
uso de lentes. As figuras a seguir mostram as sees
retas de cinco lentes.

Considerando as representaes acima, correto
afirmar que:
a) as lentes I, III e V podem ser teis para
hipermetropes e as lentes II e IV para mopes.
b) as lentes I, II e V podem ser teis para
hipermetropes e as lentes III e IV para mopes.
c) as lentes I, II e III podem ser teis para
hipermetropes e as lentes IV e V para mopes.
d) as lentes II e V podem ser teis para hipermetropes
e as lentes I, III e IV para mopes.
e) as lentes I e V podem ser teis para hipermetropes e
as lentes II, III e IV para mopes.
13. Aps examinar os olhos de Slvia e de Paula, o
oftalmologista apresenta suas concluses a respeito da
formao de imagens nos olhos de cada uma delas, na
forma de diagramas esquemticos, como mostrado
nestas figuras:
Please purchase PDF Split-Merge on www.verypdf.com to remove this watermark.
Incluso para a Vida Fsica B

Pr-Vestibular da UFSC

26

Com base nas informaes contidas nessas figuras,
correto afirmar que:

a) apenas Slvia precisa corrigir a viso e, para isso,
deve usar lentes divergentes.
b) ambas precisam corrigir a viso e, para isso, Slvia
deve usar lentes convergentes e Paula, lentes
divergentes.
c) apenas Paula precisa corrigir a viso e, para isso,
deve usar lentes convergentes.
d) ambas precisam corrigir a viso e, para isso, Slvia
deve usar lentes divergentes e Paula, lentes
convergentes.

UNIDADE 10
ONDULATRIA I
uma perturbao que se propaga atravs de
um meio.
Uma onda transporta energia, sem o transporte de
matria.

Classificao das ondas:

a) Quanto natureza:
Mecnicas: So aquelas que necessitam de
um meio material para sua propagao.

Eletromagnticas: So aquelas que no
necessitam de meio material para se propagar.

b) Quanto direo de propagao:
Unidimensionais: So aquelas que se
propagam apenas em uma nica direo.

Bidimensionais: So aquelas que se propagam
em duas direes, ou seja, em um plano.

Tridimensionais: So aquelas que se
propagam em todas as direes e sentidos.

c) Quanto direo de vibrao:
Transversais: So aquelas cuja direo de
propagao perpendicular direo de vibrao.

Longitudinais: So aquelas cuja direo de
propagao coincide com a direo de vibrao.


Elementos de onda


Amplitude da onda (A) O maior valor da elongao,
relacionada com a energia transportada pela onda.
Frequncia (f) Nmero de oscilaes executados por
qualquer ponto da corda, por unidade de tempo.
Perodo (T) - Tempo de uma oscilao completa de
qualquer ponto da corda.
Cristas e Vales- Os pontos A e B so denominados cristas e
os ponto c denominado vale.

Comprimento de onda () a distncia entre duas
cristas ou dois vales consecutivos.

Relaes f =
T
1

T
V

= ou
f V . =

Exerccios de Sala #

1. Considere as seguintes afirmaes, sobre o
movimento ondulatrio:
I Uma onda para a qual a direo de propagao
perpendicular direo de vibrao chamada de
onda transversal.
II No vcuo todas as ondas eletromagnticas tm a
mesma frequncia.
III A propagao de uma onda envolve
necessariamente transporte de energia.
IV A velocidade e a frequncia de uma onda no se
alteram quando ela passa de um meio para outro.

Assinale a alternativa correta.

a) Somente as afirmativas I e III so verdadeiras.
b) Somente a afirmativa III verdadeira.
c) Somente as afirmativas II e IV so verdadeiras.
d) Somente as afirmativas I, II e IV so verdadeiras.
e) Todas as afirmativas so falsas.

2. Uma onda transversal peridica, cujo comprimento
de onda 40,0 cm, propaga-se com velocidade de 1,60
m/s ao longo de uma corda. O grfico em papel
quadriculado representa a forma dessa corda em um
dado instante. Quais so a amplitude e o perodo da
onda, respectivamente?

A
crista

crista
n
n

vale vale
Please purchase PDF Split-Merge on www.verypdf.com to remove this watermark.
Incluso para a Vida Fsica B

Pr-Vestibular da UFSC

27
a) 7,5 cm e 0,25 s
b) 15,0 cm e 0,25 s
c) 7,5 cm e 4,00 s-1
d) 6,0 cm e 0,25 s
e) 3,0 cm e 4,00 s

Tarefa Mnima #

3. Um menino na beira de um lago observou uma
rolha que flutuava na superfcie da gua, completando
uma oscilao vertical a cada 2 s, devido ocorrncia
de ondas. Esse menino estimou como sendo 3 m a
distncia entre duas cristas consecutivas. Com essas
observaes, o menino concluiu que a velocidade de
propagao dessas ondas era de:
a) 0,5 m/s.
b) 1,0 m/s.
c) 1,5 m/s.
d) 3,0 m/s.
e) 6,0 m/s.

4. Com relao ao movimento ondulatrio, podemos
afirmar que:
a) a velocidade de propagao da onda no depende do
meio de propagao.
b) a onda mecnica, ao se propagar, carrega consigo as
partculas do meio.
c) o comprimento de onda no se altera quando a onda
muda de meio.
d) a frequncia da onda no se altera quando a onda
muda de meio.
e) as ondas eletromagnticas somente se propagam no
vcuo.

5. Considere as afirmaes a seguir, a respeito da
propagao de ondas em meios elsticos.
I. Em uma onda longitudinal, as partculas do meio no
qual ela se propaga vibram perpendicularmente
direo de propagao.
II. A velocidade de uma onda no se altera quando ela
passa de um meio para outro.
III. A frequncia de uma onda no se altera quando ela
passa de um meio para outro.

Est(o) correta(s):

a) apenas I.
b) apenas II.
c) apenas III.
d) apenas I e II.
e) apenas I e III.

6. So exemplos de ondas os raios X, os raios gama,
as ondas de rdio, as ondas sonoras e as ondas de luz.
Cada um desses cinco tipos de onda difere, de algum
modo, dos demais.
Qual das alternativas apresenta uma afirmao que
diferencia corretamente o tipo de onda referido das
demais ondas acima citadas?
a) Raios X so as nicas ondas que no so visveis.
b) Raios gama so as nicas ondas transversais.
c) Ondas de rdio so as nicas ondas que transportam
energia.
d) Ondas sonoras so as nicas ondas longitudinais.
e) Ondas de luz so as nicas ondas que se propagam
no vcuo com velocidade de 300000 km/s.

7. Uma campainha emite som com frequncia de 1
kHz.
O comprimento de onda dessa onda sonora , em
centmetros, igual a:
a) 1 b) 7 c) 21 d) 34

8. Analise as afirmativas a seguir relativas a diferentes
ondas eletromagnticas e indique qual a correta.
a) No vcuo, a radiao ultravioleta propaga-se com
velocidade maior do que as microondas.
b) No vcuo, a velocidade dos raios X menor que a
velocidade da luz azul.
c) As ondas de rdio tm frequncias maiores que a
luz visvel.
d) Os raios X e raios tm frequncias menores que a
luz visvel.
e) A frequncia da radiao infravermelha menor
que a frequncia da luz verde.

9. A faixa de emisso de rdio em frequncia
modulada, no Brasil, vai de, aproximadamente, 88
MHz a 108 MHz. A razo entre o maior e o menor
comprimento de onda desta faixa :
a) 1,2
b) 15
c) 0,63
d) 0,81
e) Impossvel calcular no sendo dada a velocidade de
propagao da onda

10. Numa experincia clssica, coloca-se dentro de
uma campnula de vidro onde se faz o vcuo, uma
lanterna acesa e um despertador que est despertando.
A luz da lanterna vista, mas o som do despertador
no ouvido. Isso acontece porque:
a) o comprimento de onda da luz menor que o do
som.
b) nossos olhos so mais sensveis que nossos ouvidos.
c) o som no se propaga no vcuo e a luz sim.
d) a velocidade da luz maior que a do som.
e) o vidro da campnula serve de blindagem para o
som mas no para a luz.

UNIDADE 11

ONDULATRIA II

Fenmenos ondulatrios

Reflexo:
Quando uma onda atinge uma superfcie de
separao de dois meios, e retorna ao meio de origem.

Please purchase PDF Split-Merge on www.verypdf.com to remove this watermark.
Incluso para a Vida Fsica B

Pr-Vestibular da UFSC

28
Quando a corda tem a extremidade fixa ocorre
reflexo com inverso de fase.

Quando a corda tem a extremidade mvel ocorre
reflexo sem inverso de fase.

Refrao:
Uma onda ao atingir a superfcie de separao
de dois meios, muda seu meio de propagao,
alterando sua velocidade e o seu comprimento de
onda, mas mantendo constante sua frequncia.


Quando a onda propaga-se em uma corda
menos densa e atinge a superfcie de separao de uma
corda mais densa ocorre o fenmeno da reflexo e da
refrao. A reflexo ocorre com inverso de fase.


Quando a onda propaga-se em uma corda mais densa e
atinge a superfcie de separao de uma corda menos
densa ocorre o fenmeno da reflexo e da refrao. A
reflexo ocorre sem inverso de fase.
Difrao:

o fenmeno que permite uma onda
contornar um obstculo.

Polarizao:
Ocorre quando uma onda, ao passar por um
determinado obstculo, passa a se propagar em uma
direo.

A polarizao s ocorre com ONDAS
TRANSVERSAIS.
Interferncia:
a sobreposio dos efeitos de vrias ondas.
Ou seja, mais de uma onda se encontram no mesmo
tempo, no mesmo lugar.

Interferncia construtiva

Interferncia destrutiva

Ondas estacionrias

So ondas resultantes da superposio de duas
ondas de mesma frequncia, mesma amplitude, mesmo
comprimento de onda, mesma direo e sentidos opostos.


n
l
n
2
=
l
v n
f
n
2
.
=
o n
f n f . =

Exerccio de Sala #

1. Considere as seguintes afirmaes, sobre o
movimento ondulatrio:
I Uma onda para a qual a direo de propagao
perpendicular direo de vibrao chamada de
onda transversal.
II No vcuo todas as ondas eletromagnticas tm a
mesma frequncia.
III A propagao de uma onda envolve necessariamente
transporte de energia.
IV A velocidade e a frequncia de uma onda no se
alteram quando ela passa de um meio para outro.

Assinale a alternativa correta.

a) Somente as afirmativas I e III so verdadeiras.
b) Somente a afirmativa III verdadeira.
c) Somente as afirmativas II e IV so verdadeiras.
d) Somente as afirmativas I, II e IV so verdadeiras.
e) Todas as afirmativas so falsas.

2. Uma onda transversal peridica, cujo
comprimento de onda 40,0 cm, propaga-se
com velocidade de 1,60 m/s ao longo de uma
corda. O grfico em papel quadriculado
representa a forma dessa corda em um dado
instante. Quais so a amplitude e o perodo da
onda, respectivamente?



a) 7,5 cm e 0,25 s
b) 15,0 cm e 0,25 s
c) 7,5 cm e 4,00 s-1
d) 6,0 cm e 0,25 s
e) 3,0 cm e 4,00 s



Please purchase PDF Split-Merge on www.verypdf.com to remove this watermark.
Incluso para a Vida Fsica B

Pr-Vestibular da UFSC

29
Tarefa Mnima #
3. Um candidato, no intuito de relaxar aps se
preparar para as provas do Vestibular 2007, resolve
surfar na praia da Joaquina em dia de timas ondas
para a prtica deste esporte.



Assinale a(s) proposio(es) correta(s).

01. A onda do mar que conduzir o surfista no possui
nenhuma energia.
02. Ao praticar seu esporte, o surfista aproveita parte da
energia disponvel na onda e a transforma em energia
cintica.
04. A lei da conservao da energia permite afirmar que
toda a energia da onda do mar aproveitada pelo
surfista.
08. Se o surfista duplicar sua velocidade, ento a energia
cintica do surfista ser duas vezes maior.
16. Tanto a energia cintica como a energia potencial
gravitacional so formas relevantes para o fenmeno
da prtica do surf numa prancha.
32. Por ser um tipo de onda mecnica, a onda do mar pode
ser til para gerar energia para consumo no dia-a-dia.

4. A figura representa dois pulsos de onda, inicial-
mente separados por 6,0 cm, propagando-se em um
meio com velocidades iguais a 2,0 cm/s, em
sentidos opostos.

Considerando a situao descrita, assinale a(s)
proposio(es) correta(s):

01. Quando os pulsos se encontrarem, haver interferncia
de um sobre o outro e no mais haver propagao dos
mesmos.
02. Decorridos 2,0 segundos, haver sobreposio dos
pulsos e a amplitude ser mxima nesse instante e
igual a 2,0 cm.
04. Decorridos 2,0 segundos, haver sobreposio dos
pulsos e a amplitude ser nula nesse instante.
08. Decorridos 8,0 segundos, os pulsos continuaro com a
mesma velocidade e forma de onda,
independentemente um do outro.
16. Inicialmente as amplitudes dos pulsos so idnticas e
iguais a 2,0 cm.

5. Na Lagoa da Conceio, em Florianpolis, em um
determinado dia, o vento produz ondas peridicas na
gua, de comprimento igual a 10 m, que se propagam
com velocidade de 2,0 m/s. Um barco de 3,0 m de
comprimento, inicialmente ancorado e, aps certo
tempo, navegando, atingido pelas ondas que o fazem
oscilar periodicamente.

Assinale a(s) proposio(es) correta(s).
01. Estando o barco ancorado ele atingido por uma crista
de onda e oscila uma vez a cada 5,0 segundos.
02. Estando o barco ancorado, ele oscila 5 vezes em cada
segundo.
04. Estando o barco navegando com velocidade de 3,0 m/s
na direo de propagao das ondas mas em sentido
contrrio a elas, ele oscila uma vez a cada 2,0
segundos.
08. A frequncia de oscilao do barco no depende da
sua velocidade de navegao, mas somente da
velocidade de propagao das ondas.
16. Se o barco tivesse um comprimento um pouco menor,
a frequncia da sua oscilao seria maior.
32. A frequncia de oscilao do barco no depende do
comprimento das ondas, mas somente da velocidade
das mesmas e do barco.
64. Estando o barco navegando com velocidade de 3,0 m/s
na direo de propagao das ondas e no mesmo
sentido delas, ele oscila uma vez a cada 10 segundos.

6. Dois pulsos, A e B, so produzidos em uma corda
esticada que tem uma das extremidades fixada em uma
parede, conforme mostra a figura abaixo.

Depois de o pulso A ter sofrido reflexo no ponto da
corda fixo na parede, ocorrer interferncia entre os
dois pulsos.

correto afirmar que a interferncia entre esses dois
pulsos :

a) destrutiva e, em seguida, os pulsos seguiro juntos, no
sentido do pulso de maior energia.
b) destrutiva e, em seguida, cada pulso seguir seu
caminho, mantendo suas amplitudes originais.
c) construtiva e, em seguida, os pulsos seguiro juntos, no
sentido do pulso de maior energia.
d) construtiva e, em seguida, cada pulso seguir seu
caminho, mantendo suas amplitudes originais.
e) destrutiva e, em seguida, os pulsos deixaro de existir,
devido absoro de energia durante a interao.

7. Para se estudar as propriedades das ondas num
tanque de gua, faz-se uma rgua de madeira vibrar
regularmente, tocando a superfcie da gua e
produzindo uma srie de cristas e vales que se
deslocam da esquerda para a direita.
Na figura a seguir esto esquematizadas duas barreiras
verticais separadas por uma distncia
aproximadamente igual ao comprimento de onda das
ondas.
Please purchase PDF Split-Merge on www.verypdf.com to remove this watermark.
Incluso para a Vida Fsica B

Pr-Vestibular da UFSC

30


Aps passas pela abertura, a onda apresenta
modificao:

a) em sua forma e em seu comprimento de onda.
b) em sua forma e em sua velocidade.
c) em sua velocidade e em seu comprimento de onda.
d) somente em sua forma.
e) somente em sua velocidade.

8. (Mackenzie-SP) Um menino na beira de um lago
observou uma rolha que flutuava na superfcie da
gua, completando uma oscilao vertical a cada 2 s,
devido ocorrncia de ondas. Esse menino estimou
como sendo 3 m a distncia entre duas cristas
consecutivas. Com essas observaes, o menino
concluiu que a velocidade de propagao dessas ondas
era de:
a) 0,5 m/s.
b) 1,0 m/s.
c) 1,5 m/s.
d) 3,0 m/s.
e) 6,0 m/s.

9. Com relao ao movimento ondulatrio, podemos
afirmar que:
a) a velocidade de propagao da onda no depende do
meio de propagao.
b) a onda mecnica, ao se propagar, carrega consigo as
partculas do meio.
c) o comprimento de onda no se altera quando a onda
muda de meio.
d) a frequncia da onda no se altera quando a onda
muda de meio.
e) as ondas eletromagnticas somente se propagam no
vcuo.

10. A figura abaixo representa uma onda harmnica
que se propaga, para a direita, em uma corda
homognea. No instante representado, considere os
pontos da corda indicados: 1, 2, 3, 4 e 5. Assinale a
afirmativa correta:

a) os pontos 1 e 3 tm velocidade nula.
b) os pontos 2 e 5 tm velocidade mxima.
c) o ponto 4 tem velocidade maior que o ponto 1.
d) o ponto 2 tem velocidade maior que o ponto 3.
e) os pontos 1 e 3 tm velocidade mxima.

11. Considere as afirmaes a seguir, a respeito da
propagao de ondas em meios elsticos.

I. Em uma onda longitudinal, as partculas do meio no
qual ela se propaga vibram perpendicularmente
direo de propagao.
II. A velocidade de uma onda no se altera quando ela
passa de um meio para outro.
III. A frequncia de uma onda no se altera quando ela
passa de um meio para outro.

Est(o) correta(s):
a) apenas I.
b) apenas II.
c) apenas III.
d) apenas I e II.
e) apenas I e III.







Please purchase PDF Split-Merge on www.verypdf.com to remove this watermark.
Incluso para a Vida Fsica C

Pr-Vestibular da UFSC 1
UNIDADE 1
ELETRIZAO E LEI DE COULOMB

CARGA ELTRICA ELEMENTAR



I. Entre dois eltrons existe um par de foras de repulso.
II. Entre dois prtons existe um par de foras de repulso.
III. Entre um prton e um eltron existe um par de foras de
atrao.

Isolantes e Condutores
Os corpos chamados condutores so aqueles em que os
portadores de cargas eltricas tem facilidade de
movimentao.
Os corpos chamados isolantes so aqueles em que
os portadores de cargas tem dificuldade de movimentao.

Eletrizao por Atrito
Atritando-se corpos de materiais diferentes, h passagem de
eltrons de um corpo para o outro, de modo que um dos
corpos fica eletrizado positivamente (perdeu eltrons) e o
outro fica eletrizado negativamente (ganhou eltrons).



Eletrizao por Contato
Quando um corpo eletrizado colocado em contato com um
corpo inicialmente neutro, ocorre uma passagem de eltrons
de um corpo para o outro e assim, os dois corpos ficam com
cargas de mesmo sinal.





Eletrizao por Induo
Quando um corpo eletrizado colocado prximo a um corpo
neutro ocorre a induo eletrosttica, ou seja, as cargas do
condutor neutro so separadas. Para que a eletrizao
acontea necessrio fazer a ligao do condutor neutro
com a terra.




Lei de Coulomb
As foras eltricas obedecem ao princpio da ao e reao
(3 Lei de Newton), isto , tm a mesma intensidade, mesma
direo e sentidos opostos, agindo em corpos diferentes.

"A intensidade da fora entre duas cargas puntiformes ou
pontuais varia com o inverso do quadrado da distncia
entre elas e diretamente proporcional ao produto dos
valores absolutos das cargas.Assim:


2
2 1 0
. .
d
Q Q K
F =


Onde:
|Q
1
| e |Q
2
| so valores absolutos de cargas Q
1
e Q
2
.
d distncia entre as cargas
K constante eletrosttica

.
10 . 9
2
2
9
0
C
m N
K =

Exerccios de Sala #

1. (UFRS) Um basto eletricamente carregado atrai uma
bolinha condutora X, mas repele uma bolinha condutora Y.
As bolinhas X e Y se atraem na ausncia do basto. Sendo
essas foras de atrao e repulso de origem eltrica,
conclui-se que:
a) Y est eletricamente carregada, X est eletricamente
descarregada ou eletricamente carregada com cargas de
sinal contrrio ao das cargas de Y.
b) ambas as bolinhas esto eletricamente descarregadas.
c) X e Y esto eletricamente carregadas com cargas de
mesmo sinal.
d) X est eletricamente carregada com cargas de mesmo
sinal das do basto.
e) Y est eletricamente descarregada e X carregada.

2. Duas cargas eltricas, Q1 = 2C e Q2 = -1,5C, esto
localizadas no vcuo distantes 30cm uma da outra.
Determine a fora de interao entre as cargas. Considere
2 2 9
/ . 10 . 9 C m N k
o
= .

Tarefa Mnima #

3. (SUPRA) Durante as corridas de Frmula 1, em que os
carros desenvolvem altas velocidades, estes sofrem
eletrizao por atrito com o ar, o que acarreta grande risco
de exploso principalmente durante o abastecimento. Essa
eletrizao se d por:
a) perda de eltrons da superfcie, carregando-se
positivamente.
b) perda de prtons da superfcie, carregando-se
negativamente.
c) ganho de eltrons do ar, carregando-se positivamente.
d) ganho de prtons do ar, carregando-se negativamente.
e) perda de eltrons da superfcie, carregando-se
alternadamente de forma positiva e negativa.

4. (PUC-SP) Duas esferas A e B, metlicas e idnticas, esto
carregadas com cargas respectivamente iguais a 16 C e 4
C. Uma terceira esfera C, metlica e idntica a anteriores,
est inicialmente descarregada. Coloca-se C em contato com
A. Em seguida, esse contato desfeito e a esfera C
colocada em contato com B. Supondo-se que no haja troca
de cargas eltricas com o meio exterior, a carga final de C
de:
a) 8 C. b) 6 C. c) 4 C. d) 3 C. e) nula.

5. (UFSC) As esferas, na figura abaixo, esto suspensas por
fios de seda. A carga eltrica da esfera A positiva. As
cargas eltricas do basto isolante B e da esfera C so,
e = 1,6 . 10
-19
C
Please purchase PDF Split-Merge on www.verypdf.com to remove this watermark.
Fsica C Incluso para a Vida

Pr-Vestibular da UFSC
2
respectivamente: (D o valor da soma da(s) alternativa(s)
correta(s) como resposta.)

01. positiva e positiva.
02. positiva e negativa.
04. positiva e neutra.
08. neutra e positiva.
16. negativa e positiva.
32. negativa e negativa.
64. neutra e negativa.

6. (UFPA) Um corpo A, eletricamente positivo, eletriza um
corpo B que inicialmente estava eletricamente neutro, por
induo eletrosttica. Nessas condies, pode-se afirmar que
o corpo B ficou eletricamente:

a) positivo, pois prtons da Terra so absorvidos pelo corpo.
b) positivo, pois eltrons do corpo foram para a Terra.
c) negativo, pois prtons do corpo foram para a Terra.
d) negativo, pois eltrons da Terra so absorvidos pelo
corpo.
e) negativo, pois prtons da Terra so absorvidos pelo corpo.

7. (UFCE) A figura ao lado mostra as esferas metlicas, A e
B, montadas em suportes isolantes. Elas esto em contato, de
modo a formarem um nico condutor descarregado. Um
basto isolante, carregado com carga negativa, -q, trazido
para perto da esfera A, sem toc-la. Em seguida, com o
basto na mesma posio, as duas esferas so separadas.
Sobre a carga final em cada uma das esferas podemos
afirmar:
------
A B

a) a carga final em cada uma das esferas nula.
b) a carga final em cada uma das esferas negativa.
c) a carga final em cada uma das esferas positiva.
d) a carga final positiva na esfera A e negativa na esfera B.
e) a carga final negativa na esfera A e positiva na esfera B.

8. (ACAFE) Com relao fora de natureza eletrosttica,
existente entre duas cargas eltricas puntiformes, podemos
afirmar que:
a) o mdulo da fora inversamente proporcional
distncia entre as cargas;
b) o mdulo da fora independente do meio em que as
cargas se encontram;
c) a fora aumenta, em mdulo, quanto aumenta a distncia
entre as cargas;
d) a fora, em mdulo, mantm-se invarivel se as duas
cargas aumentarem de valor na mesma proporo.
e) o mdulo da fora quadruplicada se ambas as cargas
forem duplicadas, mantendo-se invarivel a distncia
entre as cargas.

9. (UFSC) Obtenha a soma dos valores numricos,
associados s opes corretas:

01. Dois corpos eletrizados com cargas de mesmo mdulo
mesmo sinal se atraem;
02. A. Lei de Coulomb afirma que a fora de atrao
eletrosttica entre duas cargas de mesmo sinal
diretamente proporcional ao inverso da distncia de
separao entre cargas;
04. Um corpo inicialmente neutro, fica eletrizado com carga
positiva quando, por algum processo, so removidos
eltrons do mesmo.
08. Um corpo, inicialmente neutro, fica eletrizado com carga
negativa quando, por algum processo, so adicionados
eltrons ao mesmo;
16. Um corpo est eletrizado positivamente quando tem falta
de eltrons.
32. O eletroscpio de folhas de ouro um dispositivo
destinado a indicar a presena de cargas eltricas em
corpos eletrizados;
64. Qualquer eletroscpio, inclusive o de folhas de ouro,
um dispositivo destinado a armazenar cargas eltricas e
neutraliz-las, por atrito, nas experincias de
eletrosttica.

10. (FESP) Trs esferas condutoras A, B e C tm mesmo
dimetro. A esfera A est inicialmente neutra, e as outras
duas carregadas com q
B
= 6 mC q
C
= 7 mC. Com a esfera
A, toca-se primeiramente B e depois C. As cargas eltricas
de A, B e C, depois dos contatos, so respectivamente:
a) zero, zero, e 13mC. d) 6 mC, 7mC e zero.
b) 7 mC, 3 mC e 5mC. e) todas iguais a 4,3Mc
c) 5mC, 3mC e 5mC.

UNIDADE 2
CAMPO ELTRICO E POTENCIAL
ELTRICO

O CONCEITO DE CAMPO ELTRICO
Suponhamos que, ao ser colocado em um ponto P, uma
carga puntiforme q sofra a ao de uma fora eltrica .
Dizemos ento que no ponto P existe um campo eltrico ,
definido por:
q
F
E =


Observando essa equao vemos que:
1) se q > 0, e tero o mesmo sentido.
2) se q < 0, e tero sentidos opostos.

Campo de uma carga puntiforme


Please purchase PDF Split-Merge on www.verypdf.com to remove this watermark.
Incluso para a Vida Fsica C

Pr-Vestibular da UFSC 3

Se a carga Q for positiva o campo ser representado
por linhas afastando-se da carga; se Q for negativa a linhas
estaro aproximando-se da carga.

Mdulo de Campo Eltrico
2
1
d
Q
k E =

Vimos como calcular o campo eltrico produzido
por uma carga puntiforme. Se tivermos mais de uma carga, o
campo eltrico em um ponto P calculado fazendo a soma
vetorial dos campos produzidos por cada carga.

Linhas de fora
Essas linhas so desenhadas de tal modo que, em cada
ponto, o campo eltrico tangente linha.


OBS: Onde as linhas esto mais prximas o campo mais
intenso e onde elas esto mais afastadas o campo mais
"fraco".

Campo eltrico uniforme








Trabalho sobre uma carga (W)
pB pA AB
E E W =


possvel demonstrar que o trabalho da fora eltrica
atuante em uma carga q dado por:
B
o
A
o
AB
d
q Q K
d
q Q K
W
. . . .
= onde k a constante da lei de
Coulomb e d
A
e d
B
so as distncias dos pontos A e B
carga Q.

Potencial eltrico(V)
O potencial eltrico a razo entre a energia potencial
eltrica e a carga eltrica no determinado ponto do campo
eltrico. Logo:
q
E
V
pA
A
= . A diferena de potencial
B A AB
V V V = , ento :
q
W
V
AB
AB
=

Isso nos demonstra que o potencial de um ponto em
um campo eltrico pode ser definido como sendo:
d
Q K
V
o
P
.
=


OBS: O potencial de uma carga positiva tem o sinal positivo
e o potencial de uma carga negativa tem o sinal negativo.

Algumas propriedades do potencial eltrico:

1. O potencial diminui ao longo de uma linha de fora
2. Uma carga positiva, abandonada numa regio onde h
campo eltrico, desloca-se espontaneamente para pontos
de potenciais decrescentes e uma carga negativa,
abandonada numa regio onde h campo eltrico,
desloca-se espontaneamente para pontos de potenciais
crescentes.

Superfcies Equipotenciais
Todos os pontos dessa superfcie tm o mesmo potencial e
por isso ela chamada de superfcie equipotencial.



O potencial em um campo uniforme dado: V
AB
= E.d

Exerccios de Sala #

1. (UFSCar-SP) Para que o campo eltrico resultante em P
seja o indicado na figura, necessrio que as cargas eltricas
estejam distribudas da seguinte maneira:
a) q
1
. e q
2
positivas e q
3
negativa.
b) q
1
positiva, q
2
e q
3
negativas.
c) q
1
e q
2
negativas e q
3
positiva.
d) q
1
. e q
3
positivas e q
2
negativa.
e) q
2
e q
3
positivas e q
1
negativa


2. (UFSC) Obtenha a soma dos valores numricos
associados s opes corretas:
01. A lei que rege os fenmenos de atrao e repulso de
cargas eltricas denominada Lei de Coulomb.
02. Na natureza, normalmente os corpos se encontram em
equilbrio eletrosttico, pois os tomos se compem de
nmeros idnticos de cargas positivas e negativas.
04. O trabalho realizado sobre uma carga eltrica, para
moviment-la em equilbrio, sobre uma superfcie
equipotencial, diferente de zero.
08. A diferena de potencial entre dois pontos de uma
mesma superfcie equipotencial nula.
16. Nos materiais condutores de eletricidade, os portadores
de carga apresentam grande facilidade de movimento no
interior do material. Nos isolantes, difcil a
movimentao dos portadores de carga.
Please purchase PDF Split-Merge on www.verypdf.com to remove this watermark.
Fsica C Incluso para a Vida

Pr-Vestibular da UFSC
4
Tarefa Mnima #

3. (UFPI) Uma carga de prova q, colocada num ponto de um
campo eltrico E = 2 10
3
N/C, sofre a ao de uma fora F
= 18 10
-5
N. O valor dessa carga, em coulombs, de:

a) 9 10
-8
c) 36 10
-8
e) 36 10
-2
b) 20 10
-8
d) 9 10
-2


4. (UFPA) Com relao s linhas de fora de um campo
eltrico, pode-se afirmar que so linhas imaginrias:
a) tais que a tangente a elas em qualquer ponto tem a mesma
direo do campo eltrico.
b) tais que a perpendicular a elas em qualquer ponto tem a
mesma direo do campo eltrico.
c) que circulam na direo do campo eltrico.
d) que nunca coincidem com a direo do campo eltrico.
e) que sempre coincidem com a direo do campo eltrico.

5. (VUNESP) Na figura, o ponto P est equidistante das
cargas fixas + Q e - Q. Qual dos vetores indica a direo e o
sentido do campo eltrico em P, devido a essas cargas?
a) b) c)
d) e)


6. Um campo eltrico uniforme de mdulo C N x E / 10 20
3
=
mostrado abaixo. Sabendo que o potencial em A e B so
respectivamente, 50V e 30V, podemos afirmar que:





a) O trabalho da fora eltrica para levar uma carga q de A
para B maior na trajetria 2 do que na trajetria 1;
b) A distancia entre A e B vale 20x10
-3
m;
c) A fora eltrica ao transportar uma carga q =6c de A
para B realiza um trabalho de 1,2x10
-4
J;
d) O trabalho da fora eltrica para uma carga q de A para
B maior pela trajetria 1, pois ela menor;
e) No possvel calcular a distancia entre A e B.

7. (UNICAP) Assinale as proposies verdadeiras e faa o
somatrio.
01. Um corpo neutro no pode ser carregado por contato ou
induo.
02. A fora de atrao ou de repulso entre duas cargas
eltricas varia diretamente com a quantidade de carga e
inversamente com o quadrado da distncia que as separa.
04. Potencial e tenso so termos equivalentes. O potencial
tem a dimenso de trabalho por unidade de carga e
medido em watt.
08. O potencial, em qualquer ponto de um campo eltrico,
definido como o trabalho efetuado para deslocar uma
unidade de carga positiva de um ponto fixo de referncia
at esse ponto.
16. Em torno de qualquer sistema de cargas eltricas, h um
campo eltrico.

8. (UFSC) Para entender como funciona a eletroforese do
DNA, um estudante de Biologia colocou ons de diferentes
massas e cargas em um gel que est dentro de uma cuba na
qual h eletrodos em duas das extremidades opostas. Os
eletrodos podem ser considerados como grandes placas
paralelas separadas por 0,2 m. Aps posicionar os ons, o
estudante aplicou entre as placas uma diferena de potencial
de 50J/C que foi posteriormente desligada. O meio onde os
ons se encontram viscoso e a fora resistiva precisa ser
considerada. Os ons se deslocam no sentido da placa
negativamente carregada para a placa positivamente
carregada e ons maiores tendem a se deslocar menos.
(Desconsidere o efeito do gel no campo eltrico). As figuras
mostram esquemas do experimento e do resultado. Observe-
as e assinale a(s) proposio(es) correta(s):

01. Enquanto a diferena de potencial estiver aplicada, a
fora eltrica que atua em um on ser constante,
independentemente de sua posio entre as placas.
02. Pelo sentido do movimento dos ons, podemos afirmar
que eles tm carga negativa.
04. Quanto maior for a carga do on, mais intensa vai ser
a fora eltrica que atua sobre ele.
08. Os ons maiores tm mais dificuldade de se locomover
pelo gel. Por este motivo podemos separar os ons
maiores dos menores.
16. Um on, com carga de mdulo 8,0 x 10
-19
C, que se
deslocou 0,1 m do incio ao fim do experimento, dissipou
2 x 10
-17
J no meio viscoso.

9. (UFSC) Em relao a fenmenos eletrostticos, assinale
a(s) proposio(es) correta(s).

01. Se uma barra de vidro positivamente carregada atrair
um objeto suspenso, este objeto estar carregado
negativamente e se a mesma barra repelir um objeto
suspenso, este segundo objeto estar positivamente
carregado.
02. A carga eltrica conservada, mas no quantizada.
04. A fora eltrica que um pequeno corpo eletricamente
carregado exerce sobre outro se altera ao aproximarmos
dele outros corpos tambm carregados.
08. O potencial eltrico no centro de uma pequena esfera
carregada tem o mesmo valor do potencial eltrico na sua
superfcie.
16. Se uma barra de vidro for eletricamente carregada por
atrito, fica com excesso de carga no local onde foi
atritada.






+
ons
-

-
-
gel
FINAL
INCIO ons

-
-
-
V
gel
-
-
-
Please purchase PDF Split-Merge on www.verypdf.com to remove this watermark.
Incluso para a Vida Fsica C

Pr-Vestibular da UFSC 5
UNIDADE 3
ELETRODINMICA

Corrente Eltrica
Quando temos um movimento ordenado de partculas com
carga eltrica, dizemos que temos uma corrente eltrica.

Sentido da corrente
Nos condutores slidos, o
sentido da corrente eltrica
corresponde ao sentido do
movimento de eltrons, pois
so eles que se deslocam, ou
seja, a corrente do
potencial menor (plo negativo) para o potencial maior (polo
positivo). Este o sentido real da corrente.
No estudo da corrente eltrica, entretanto, adota-se
um sentido convencional, que do deslocamento das cargas
positivas, ou seja, do potencial maior para o menor.

Intensidade de Corrente
A intensidade mdia da corrente (i
m
) nesse intervalo de
tempo definida por:



No Sistema Internacional a unidade de intensidade de
corrente o ampre cujo smbolo A.

Grfico i x t
Na Fig. temos o
grfico de i em funo do
tempo t para o caso em
que i constante. Nesse
caso, a rea da regio
sombreada nos d o
mdulo da carga que
passa pela seo reta do
fio no intervalo de tempo
At.

Resistncia-1 Lei de Ohm
a oposio feita por um condutor passagem da corrente
eltrica. Sendo i a intensidade da corrente que percorre o fio,
definimos a resistncia R do fio pela equao:



No Sistema Internacional, a unidade de
resistncia o ohm, cujo smbolo O.
H condutores que obedecem a lei de Ohm, tais
condutores so chamados hmicos.


Em um condutor que no hmico o grfico de U em
funo de i no retilneo.

Resistividade- 2 Lei de Ohm
A resistncia de um condutor depende de sua forma, de seu
tamanho e de que material feito.
Consideremos o caso de um fio cilndrico, de comprimento
L e cuja seo reta tem rea A. A experincia mostra que a
resistncia R desse fio dada por:
A
L
R = onde uma
constante denominada resistividade do material.

Exerccios de Sala #

1. (UFPA) Para conhecer o valor da resistncia eltrica de
um ferro eltrico existente em sua casa, Joozinho usou um
ampermetro, um voltmetro e uma fonte de tenso conforme
o esquema abaixo. Ele aplicou tenses e obteve correntes,
conforme o grfico abaixo. Assinale a alternativa que
contm o valor da resistncia, em ohms, encontrada por
Joozinho:











a) 50 b) 40 c) 30 d) 20 e) 10

2. (PUC-RJ) Considere duas lmpadas, A e B, idnticas a
no ser pelo fato de que o filamento de B mais grosso que
o filamento de A. Se cada uma estiver sujeita a uma ddp de
110 volts:

a) A ser a mais brilhante, pois tem a maior resistncia.
b) B ser a mais brilhante, pois tem a maior resistncia.
c) A ser a mais brilhante, pois tem a menor resistncia.
d) B ser a mais brilhante, pois tem a menor resistncia.
e)ambas tero o mesmo brilho.

Tarefa Mnima #

3. (PUC-MG) O grfico representa a curva caracterstica
tenso - corrente para um determinado resistor.

t
Q
i
A
A
=
i
V
R =
i (A)
V
(Volt)
1 10 20

2



1

0
,
Please purchase PDF Split-Merge on www.verypdf.com to remove this watermark.
Fsica C Incluso para a Vida

Pr-Vestibular da UFSC
6
Em relao ao resistor, correto afirmar:

a) hmico e sua resistncia vale 4,5 x 10
2
.
b) hmico e sua resistncia vale 1,8 x 10
2
.
c) hmico e sua resistncia vale 2,5 x 10
2
.
d) no hmico e sua resistncia vale 0,40 .
e) no hmico e sua resistncia vale 0,25 .

4. (UFSC) Assinale as afirmativas corretas e some os
valores respectivos:

01. Define-se resistncia de um condutor como a razo entre
a diferena de potencial aplicada a seus extremos e a
corrente que passa atravs dele.
02. A resistncia de um ferro eltrico deve ser grande de
forma a produzir um maior efeito joule.
04. A lei de ohm um caso particular da definio de
resistncia.
08. A resistncia de um fio condutor inversamente
proporcional ao comprimento do fio.
16. A resistncia de um fio condutor diretamente
proporcional ao dimetro do fio.
32. A resistividade independe da forma do material.

5. A resistncia eltrica de um resistor em forma de fio vale
80 O. Calcule o comprimento deste fio, sabendo que, ao se
cortar 2m do mesmo, a resistncia passa a valer 60 O.

6. Um fio metlico de resistncia eltrica R =10 O tem
comprimento l =200 cm e rea de seco transversal A =
4x10
-4
cm
2
. Determine a resistividade do material que
constitui esse fio.

7. (UFSC) O grfico a seguir se refere a dois condutores, A
e B, de metais idnticos e mesmo comprimento.

Na situao mostrada correto afirmar que:
01. Nenhum dos dois condutores obedece Lei de Ohm.
02. Ambos os condutores obedecem Lei de Ohm.
04. O condutor que possui maior rea da sua seo reta
transversal o A.
08. O condutor que possui maior rea da sua seo reta
transversal o B.
16. O condutor que possui maior resistividade o A.
32. O condutor que possui maior resistividade o B.
64. A resistividade de ambos os condutores a mesma, mas
a resistncia do condutor B maior que a resistncia do
condutor A.

8. Aplica-se uma ddp de 200V nas extremidades de um fio
condutor de 10m de comprimento e seco transversal de
rea 2,5mm
2
. Sabe-se que a corrente eltrica que circula no
fio tem intensidade 10A. Calcule a resistividade do material
que constitui o fio.

9. O filamento de tungstnio de uma lmpada tem
resistncia de 20 O a 20
o
C. Sabendo-se que sua seco
transversal mede 1,102x10
-4
mm
2
e que a resistividade do
tungstnio a 20
o
C 5,51 x 10
-2
mm
2
/m determine o
comprimento do filamento.

10. Aplica-se uma ddp de 60V a um resistor cuja resistncia
vale 20 O. Determine a intensidade da corrente que
atravessa.

11. (UFSC) Some os valores das afirmativas corretas:
01. Resistncia a propriedade que os materiais possuem de
se opor passagem da corrente eltrica.
02. Os metais, em geral, so bons condutores porque
possuem muitos eltrons livres.
04. A corrente eltrica aparece em um condutor quando se
aplica uma d.d.p. s extremidades, pois a d.d.p. a fonte
de energia para mover as cargas.
08. A Lei de Ohm garante que a corrente eltrica que
atravessa qualquer condutor proporcional diferena
de potencial aplicada s extremidades deste.
16. Define-se resistncia eltrica como o quociente entre a
diferena de potencial aplicada s extremidades do
condutor e corrente eltrica que o atravessa.
32. A corrente eltrica, ao passar atravs de um fio, gera
calor (Efeito Joule) devido ao fato de que os choques entre
as cargas so parcialmente elsticos.

2.(UFSC) Um tcnico eletricista, para obter as
caractersticas de um determinado resistor, submete o
mesmo a vrios valores de diferena de potencial, obtendo
as intensidades de corrente eltrica correspondentes. Com os
valores obtidos, o tcnico constri o grfico V X i mostrado
abaixo, concluindo que o grfico caracteriza a maioria dos
resistores reais.


















Analise o grfico e assinale a(s) proposio(es) correta(s).

01. A resistncia desse resistor tende a aumentar com o
seu aquecimento, devido ao aumento da corrente.
02. No trecho de 0 a 600 mA, o resistor considerado
hmico, pois o valor da resistncia constante.
04. No trecho de 600 mA at 800 mA, a relao
i
V
R= no
pode ser aplicada, pois o resistor no mais hmico.
Please purchase PDF Split-Merge on www.verypdf.com to remove this watermark.
Incluso para a Vida Fsica C

Pr-Vestibular da UFSC 7
08. Quando passa pelo resistor uma corrente de 800 mA, a
resistncia eltrica do mesmo 5O.
16. Se o tcnico desejar construir um resistor de resistncia
igual a 5 O, utilizando um fio de nquel cromo ( = 1,5 x
10
-6
O.m) com rea da seco reta de 1,5 mm
2
, o
comprimento deste fio dever ter 5 m.
32. Quando a intensidade da corrente aumenta de 200
mA para 400 mA, a potncia dissipada por efeito Joule no
referido resistor duplica.

UNIDADE 4
RESISTORES E POTNCIA ELTRICA

Introduo
Chamamos de resistor todo condutor cuja nica funo
transformar a energia eltrica em energia trmica.


Associao em srie
Neste caso os trs resistores so percorridos pela mesma
corrente, de intensidade i.









A tenso U entre os extremos A e B da associao
igual soma das tenses entre os extremos de cada resistor:
V = V
1
+ V
2
+ V
3

Vemos ento que, se substituirmos a associao de
resistores por um nico resistor de resistncia R
E
(Fig.), este
ser percorrido pela mesma corrente. A resistncia R
E

chamada de resistncia
equivalente associao.

Associao em paralelo

Calculo do resistor equivalente
3 2 1
1 1 1 1
R R R R
E
+ + =
Caso de apenas 2 resistores:
2 1
2 1
.
R R
R R
R
E
+
=

Caso os resistores sejam iguais
n
R
R
E
=


Curto-circuito
Quando dois pontos de um circuito so ligados por um fio de
resistncia desprezvel, dizemos que os dois pontos esto em
curto-circuito.


Potncia
Quando um sistema absorve (ou fornece) uma energia , num
intervalo de tempo t, a potncia mdia absorvida (ou
recebida) nesse intervalo de tempo definida por:
t
E
P
ot
A
=
No Sistema Internacional de Unidades, a unidade
de energia o joule (J), a unidade de tempo o segundo (s)
e a unidade de potncia o watt (W):

Potncia em resistores





Porm, essa potncia pode ser expressa de outros modos,
usando a equao:
2
.i R P =
R
V
P
2
=


Exerccios de Sala #

1. (VUNESP) Num circuito eltrico, dois resistores, cujas
resistncias so R
1
e R
2
, com R
1
> R
2
, esto ligados em srie.
Chamando de i
1
e i
2
, as correntes que os atravessam e de V
1

e V
2
as tenses a que esto submetidos, respectivamente
podemos afirmar que:
a) i
1
= i
2
e V
1
= V
2
d) i
1
> i
2
e V
1
< V
2
b) i
1
= i
2
e V
1
> V
2
e) i
1
< i
2
e V
1
> V
2

c) i
1
> i
2
e V
1
= V
2


2. (UNICAP) No circuito abaixo, sendo de 1,0 A a
intensidade da corrente, designada i
3
, podemos concluir que:
Assinale V para as afirmativas verdadeiras e F para as
afirmativas falsas.

( ) o circuito abaixo um circuito em srie;
( ) o circuito abaixo um circuito em paralelo;
( ) o valor de V 100 volts;
( ) a corrente i
2
vale 2,0 A;
( ) a corrente i
1
vale 3,0 A.



P = V . i (I)
R
E
= R
1
+ R
2
+ R
3

Please purchase PDF Split-Merge on www.verypdf.com to remove this watermark.
Fsica C Incluso para a Vida

Pr-Vestibular da UFSC
8
Tarefa Mnima #

3. (UFSC) Assinale a(s) proposio(es) correta(s).
01. Para a maioria dos metais a resistividade diminui quando
h um aumento na temperatura.
02. A dissipao de energia por efeito Joule num resistor
depende do sentido da corrente e independe da tenso
aplicada sobre ele.
04. Para dois condutores de mesmo material e mesmo
comprimento, sendo que um tem o dobro da rea de
seo do outro, teremos uma mesma intensidade de
corrente se aplicarmos a mesma tenso sobre ambos.
08. Para um condutor hmico um aumento de tenso
corresponde a um aumento proporcional de corrente
eltrica.
16. Ao se estabelecer uma corrente eltrica num fio metlico
submetido a uma certa tenso contnua, teremos prtons
se movendo do plo positivo ao negativo.
32. Os metais geralmente so bons condutores de
eletricidade e de calor.

4. (PUC-RS) A figura
representa um gerador
ideal de tenso, trs
resistores e dois
interruptores (chaves).
Com os interruptores CH1
fechado e CH2 aberto, a
diferena de potencial
entre os pontos B e C
vale:
a) 10 V d) 17 V c) 15 V
b) 12 V e)20V

5. (UFMG) A figura ilustra a forma como trs lmpadas
esto ligadas a uma tomada. A corrente eltrica no ponto P
do fio iP e no ponto Q iQ .









Em um determinado instante, a lmpada L2 se queima.
Pode-se afirmar que
a) a corrente iP se altera e iQ no se altera.
b) a corrente iP no se altera e iQ se altera.
c) as duas correntes se alteram.
d) as duas correntes no se alteram.

6. (PUC-PR) O circuito representado formado pelo
gerador de F.E.M. 60 V, resistncia interna 1W e por
resistores. A corrente no resistor de 9 e a diferena de
potencial entre os pontos A e B so respectivamente:



a) 4A, 4V.
b) 2A, 6V.
c) 4A, 8V.
d) 2A, 2V.
e)3,3A,6,6V.


7. (UNICAP) No circuito abaixo, Va - Vb = 22,4V.


Assinale as afirmativas verdadeiras.
01. A resistncia equivalente O 25 .
02. O valor da resistncia R O 0 , 4 .
04. A potncia dissipada em R 1,0 W.
08. A corrente l
1
0,6 A.
16. A corrente l
2
0,4 A.

8. (UNICAP) No circuito abaixo, tem-se um gerador, de
resistncia interna nula, de 20 V e resistores O = = 5
5 1
r r
e O = = = 10
4 3 2
r r r . Assinale as afirmativas verdadeiras.

01. A potncia entregue ao circuito pelo gerador de 30 W.
02. A potncia dissipada pelo resistor r
2
de 2,5 W.
04. A diferena de potencial entre os pontos A e C vale 10V.
08. A corrente no resistor r
1
de 0,5 A.
16. A corrente no resistor r
5
de 2 A.

9. (UNICAP) Na figura 7, os pontos A e B esto submetidos
a uma ddp de 4 volts. (Utilize esta informao para
responder s trs primeiras proposies desta questo.)
Assinale as afirmativas verdadeiras.

01. A resistncia equivalente da associao O 2 .
02. A ddp entre os pontos C e D 6 volts.
04. A potncia dissipada na associao 6 watts.
08. A resistncia de um condutor independe do seu
comprimento, dependendo apenas do material que o
constitui.
16. Nos condutores hmicos, a relao entre a ddp aplicada e
a corrente corresponde constante.


Please purchase PDF Split-Merge on www.verypdf.com to remove this watermark.
Incluso para a Vida Fsica C

Pr-Vestibular da UFSC 9
10. (UFSC) O circuito eltrico representado na figura possui
cinco resistores: R
1
= 4 O, R
2
= 2 O, R
3
= 4 O, R
4
= 4 O e
R
5
= 4 O e duas fontes de tenso: V
1
= 15V e V
2
= 10V.
Uma chave (ch) est inicialmente na posio N, com o
circuito aberto.

Assinale a(s) proposio(es) correta(s).

01. O circuito eltrico, estando a chave ch posicionada em
A, possui resistncia equivalente igual a 3,0 O.
02. Com a chave ch posicionada em B, a potncia eltrica
dissipada no resistor R
4
igual a 400W.
04. Quando a chave ch for movida da posio N para a
posio B, circular pelo circuito uma corrente eltrica
igual a 5,0 A.
08. Quando a chave ch for movida da posio N para a
posio A, circular pelo circuito uma corrente eltrica
igual a 5,0 A.
16. A diferena de potencial no resistor R
4
igual
diferena de potencial no resistor R
5
, no importando a
posio da chave ch no circuito, porque eles esto
associados em paralelo.

UNIDADE 5
GERADOR ELTRICO

GERADOR REAL

Os geradores fornecem energia s cargas eltricas que
passam por ele.
Nos geradores reais uma parte da energia recebida
pelas cargas perdida dentro do prprio gerador. Dizemos
que o gerador real tem uma resistncia interna (r). Assim, a
tenso V (diferena de potencial) entre os plos do gerador
em geral menor do que a fora eletromotriz: U = V = E ri
onde i a intensidade da corrente que atravessa o gerador.
Na figura damos o smbolo usado para o gerador real.








2) Curva caracterstica



Quando i = 0 temos V = E. Esse caso chamado
gerador em aberto.
O caso V = 0 ocorre quando ligamos os plos A e B
do gerador por um fio de resistncia nula, isto , colocamos
os terminais do gerador em curto-circuito.



Potncia do gerador
O gerador ter as potncias mencionadas abaixo:

U . i = potncia til fornecida pelo gerador = P
u
E . i = potncia total produzida pelo gerador = P
t

R i
2
= potncia dissipada = P
d

Assim:

3) Rendimento do Gerador
Dividindo a potncia til pela potncia total, obtemos o
rendimento (n) do gerador.


Associao de geradores
Os dois principais modos so: srie e paralelo.

Srie
Na Fig. exemplificamos um caso de associao em
srie usando trs pilhas de lanterna.
Essa associao pode ser substituda por um nico
gerador (gerador equivalente) cuja fora eletromotriz (E) e
resistncia interna so dadas por:






P
t
= P
u
+ P
d

Please purchase PDF Split-Merge on www.verypdf.com to remove this watermark.
Fsica C Incluso para a Vida

Pr-Vestibular da UFSC
10
Paralelo
Na Fig. temos um caso de trs pilhas associadas em
paralelo. No caso de associao em paralelo, somente
usamos geradores idnticos.
Neste caso, a associao pode ser substituda por um nico
gerador (gerador equivalente) com a mesma fora
eletromotriz E mas com resistncia interna ) dada por:




Exerccios de Sala #

1. (VUNESP) Um ampermetro ideal A, um resistor de
resistncia R e uma bateria de f.e.m. e resistncia interna
desprezvel esto ligados em srie. Se uma segunda bateria,
idntica primeira, for ligada ao circuito como mostra a
linha tracejada da figura:







a) a diferena de potencial no ampermetro aumentar.
b) a diferena de potencial no ampermetro diminuir.
c) a corrente pelo resistor aumentar.
d) a corrente pelo resistor no se alterar.
e) a corrente pelo resistor diminuir.

2. (UEL) A diferena de potencial obtida nos terminais de
um gerador 12volts. Quando esses terminais so colocados
em curto-circuito, a corrente eltrica fornecida pelo gerador
5,0 ampres. Nessas condies, a resistncia interna do
gerador , em ohms, igual a:
a) 2,4 c) 9,6 e) 60
b) 7,0 d) 17

Tarefa Mnima #

3. Uma bateria tem fora eletromotriz de 12 V. A energia
que ela fornece a cada eltron que a atravessa e a energia
que ela fornece a uma carga de 1C, valem, respectivamente:
a) 1,92x10
-18
J e 12 J d) 3,92x10
-18
J e 15 J
b) 3,6x10
-18
J e 12 J e) 9,22x10
-17
J e 2 J
c) 1,92x10
-16
J e 5 J

4. Uma bateria apresenta ddp de 7,0V quando atravessada
por uma corrente de 10A ddp de 6,0V quando atravessada
por corrente de 20A. A sua fora eletromotriz e resistncia
interna, valem respectivamente:
a) 10 V e 0,5 O d) 10 V e 0,1 O
b) 5 V e 0,2 O e) 8 V e 0,1 O
c) 8 V e 0,5 O

5. Quando uma bateria est em circuito aberto um
voltmetro ideal ligado aos seus terminais marca 12V.
Quando a bateria est fornecendo energia a um resistor R,
estabelece no circuito uma corrente de 1A, e o voltmetro
registra 10V nos terminais da bateria. Determine a f.e.m e a
resistncia interna.
a) 10 V e 4O c) 12 V e 2O e) 15 V e 2O
b) 5 V e 4O d) 8 V e 4O

6. Uma bateria de automvel tem f.e.m. 12V e resistncia
interna 0,5 W. Determine a mxima intensidade de corrente
que se pode obter desta bateria.
a) 10A c) 24A e) 6A
b) 15A d) 12A

7. Tem-se um gerador de fora eletromotriz 6V e resistncia
interna 1,5 W. A leitura de um ampermetro ideal e um
voltmetro ideal ligado aos seus plos, so respectivamente:
a) 3A e 10 V c) 2A e 10 V e) 1A e 5 V
b) 4A e 6 V d) 5A e 15 V

8. Um gerador tem fora eletromotriz 36V e resistncia
interna 4,5 O.
a) Represente, num grfico, a tenso v no gerador em funo
da intensidade da corrente i que o atravessa.
b) Qual a potncia que o gerador lana no circuito externo
sob tenso de 27V?

9. Um gerador de f.e.m. 24V e resistncia interna de 1O
est ligado a um circuito externo. A tenso entre os
terminais do gerador de 20V. A intensidade da corrente
eltrica que o atravessa e as potncias gerada, til e a
dissipada que produz so respectivamente:
a) 3A, 100 W, 70W e 30W
b) 5A, 120 W, 95W e 25W
c) 2A, 87 W, 58W e 29W
d) 1A, 60 W, 48W e 12W
e) 4A, 96 W, 80W e 16W

10. Um gerador apresenta tenso de 20V quando
atravessado por uma corrente de 20A e, tenso de 15v
quando atravessado por corrente de 30A. Calcule sua fora
eletromotriz e sua resistncia interna.
a) 25 V e 0,4O c) 12 V e 2O e) 25 V e 2O
b) 35 V e 0,8O d) 30 V e 0,5O

UNIDADE 6

RECEPTORES ELTRICOS

Chamamos de receptor eltrico a um aparelho que
transforme energia eltrica em outro tipo de energia que
no seja apenas trmica.





Please purchase PDF Split-Merge on www.verypdf.com to remove this watermark.
Incluso para a Vida Fsica C

Pr-Vestibular da UFSC 11
Equao do receptor
Quando o receptor submetido a uma diferena de
potencial (tenso) U, ela se divide em duas parcelas:
1) Uma parcela E, denominada fora contra-
eletromotriz (fcem), correspondente energia eltrica que
ser transformada em outra forma de energia (que no seja
energia trmica).
2) Uma parcela r.i , correspondente dissipao de
energia, isto , correspondente transformao de energia
eltrica em energia trmica.
Assim, para o receptor temos: U = V = E + r.i

Como essa equao de primeiro
grau e o coeficiente de i positivo
(+ r), o grfico de U em funo de i
tem o aspecto da figura, onde a
tangente do ngulo u
numericamente igual ao valor de r.

Potncia do receptor
O receptor tem trs potencias distintas:


U . i = potncia total consumida pelo receptor = P
t

E . i = potncia til do receptor = P
u

r.i
2
= potncia dissipada no interior do receptor = P
d

d U T
P P P + =


Rendimento do receptor
O rendimento do receptor obtido efetuando a diviso entre
a potncia til e a potncia total:




Circuito gerador-receptor
Na figura representamos uma situao
em que uma bateria (gerador) faz
funcionar um motor (receptor) que
usado para levantar um bloco.

Essa situao pode ser representada pelo seguinte esquema:




onde:
E' = fora eletromotriz do gerador
r' = resistncia interna do gerador
E" = fora contra-eletromotriz do receptor
r" = resistncia interna do receptor
Naturalmente devemos ter:
E' > E"
A corrente sai pelo positivo do gerador e entre no plo
positivo do receptor.


Exerccios de Sala #

1. Para o circuito abaixo, determine o sentido e a
intensidade da corrente eltrica.

2. Um receptor tem fora contra eletromotriz igual a 20V e
resistncia interna igual a 5,0O . Ao ser ligado num circuito,
atravessado por uma corrente de intensidade 2,0A
Determine:
a) a ddp nos terminais do receptor;
b) a potncia eltrica fornecida ao receptor;
c) a potncia eltrica que o receptor transforma em outra
forma de energia que no trmica;
d) o rendimento eltrico do receptor.

Tarefa Mnima #

3. Um motor eltrico, de resistncia interna 2O , ligado a
uma ddp de 100V. Constata-se que o motor percorrido por
uma corrente de 5A. Determine a f.c.e.m do motor; a
potncia dissipada internamente e o que acontece se
impedirmos o eixo de girar.
a) 90V, 50W e queima
b) 50V, 20W e queima
c) 70V, 50W e aquece
d) 90V, 30W e queima
e) 80V, 40W e aquece

4. A curva caracterstica de um receptor dada no grfico
abaixo. Determine a f.c.e.m do receptor; a resistncia interna
do receptor; e as potncias fornecidas, til e dissipada pelo
receptor quando ligado num circuito e atravessado por uma
corrente eltrica de intensidade 5,0A.


a) 15V, 3O, e 100W, 50W, 50W
b) 10V, 2O, e 50W, 30W, 20W
c) 10V, 2O, e 100W, 50W, 50W
d) 20V, 1O, e 150W, 90W, 60W
e) 20V, 2O, e 200W, 100W, 100W

5. (ACAFE) Assinale a afirmativa correta:
a) A diferena de potencial entre os terminais de um gerador
no ideal sempre igual sua fora eletromotriz.
b) A fora eletromotriz a relao entre o trabalho do
gerador e a durao do seu funcionamento.
c) A fora contra-eletromotriz e a relao entre o trabalho
til e a corrente eltrica que atravessa o receptor.
d) A resistncia interna de um gerador eltrico ideal nula.
e) Em um receptor eltrico ideal, a diferena de potencial
sempre diferente da fora contra-eletromotriz.

Please purchase PDF Split-Merge on www.verypdf.com to remove this watermark.
Fsica C Incluso para a Vida

Pr-Vestibular da UFSC
12
6. (UFSC) No circuito abaixo representado, temos duas
baterias de foras eletromotrizes c
1
= 9,0 V e c
2
=
3,0 V, cujas resistncias internas valem r
1
= r
2
= 1,0O .
So conhecidos, tambm, os valores das resistncias R
1
=
R
2
= 4,0 O e R
3
= 2,0 O. V
1
,

V
2
e V
3
so voltmetros e
A um ampermetro, todos ideais.

V
1
V
3
R
1
R
2
R
3
A

V
2
c 1
c 2
r
1
r
2


+
+
V
1

Assinale a(s) proposio(es) correta(s):
01. A bateria c
1
est funcionando como um gerador de fora
eletromotriz e a bateria c
2
como um receptor, ou gerador
de fora contra eletro-motriz.
02. A leitura no ampermetro igual a 1,0 A.
04. A leitura no voltmetro V
2
igual a 2,0 V.
08. A leitura no voltmetro V
1
igual a 8,0 V.
16. Em 1,0 h, a bateria de fora eletromotriz c
2
consome 4,0
Wh de energia.
32. A leitura no voltmetro V
3
igual a 4,0 V.
64. A potncia dissipada por efeito Joule, no gerador, igual
1,5 W.

7. A curva caracterstica de um motor representada
abaixo.Calcule a f.c.e.m , a resistncia interna e determine,
em quilowatts-hora (kwh), a energia eltrica que o motor
consome em 10 horas para o motor funcionando nas
condies do ponto P


a) 100V, 100O, e 1,0kWh d) 200V, 200O, e 1,5kWh
b) 100V, 200O, e 1,0kWh e) 400V, 300O, e 2,5kWh
c) 200V, 100O, e 1,5kWh

8. Considere o circuito a seguir. Determine a leitura no
ampermetro, ideal, nos casos (1) a chave ch est na posio
B e (2) a chave ch est na posio C;




a) (1) 3A e (2) 6A
b) (1) 2A e (2) 5A
c) (1) 1A e (2) 4A
d) (1) 3A e (2) 4A
e) (1) 2A e (2) 6A



UNIDADE 7

CAPACITORES

CAPACITNCIA
Suponhamos que um capacitor esteja eletrizado com carga
Q, isto + Q, em uma armadura e carga - Q na outra. Entre
as armaduras existe uma diferena de potencial cujo mdulo
U. Verifica-se que U e Q so diretamente proporcionais,
isto , Q = C. U onde C uma constante de
proporcionalidade denominada capacitncia do capacitor.
No sistema internacional a unidade de capacitncia o
faraday cujo smbolo F.

Verifica-se que a capacitncia depende dos seguintes
fatores:
1) isolante colocado entre as armaduras
2) forma, tamanho e posio relativa entre as armaduras

d
A
C c =

Energia de capacitor
Como Q e U so proporcionais, o grfico da carga em
funo da tenso retilneo e tem o aspecto da Fig.



Quando o capacitor est carregado. Pode-se demonstrar que
essa energia dada pela rea da regio sombreada no
grfico.
Assim a energia pode tambm ser dada por:


ou


Associao de capacitores em srie
Na figura representamos uma situao em que h
trs capacitores associados em srie.

Observe que todas as armaduras ficam com a
mesma carga, em mdulo.
Assinalamos as tenses em cada capacitor (U
1
, U
2
,
U
3
) e a tenso U entre os extremos. Obviamente devemos
ter:



Assim, por exemplo, se tivermos 4 capacitores em srie, a
capacitncia equivalente (C) ser calculada por:
U = U
1
+ U
2
+ U
3

Please purchase PDF Split-Merge on www.verypdf.com to remove this watermark.
Incluso para a Vida Fsica C

Pr-Vestibular da UFSC 13


Se tivermos apenas dois capacitores em srie, temos:



Se tivermos n capacitores iguais associados em srie, tendo
cada um capacitncia C, a capacitncia equivalente ser
calculada por:


Associao de capacitores em paralelo
Na figura representamos trs capacitores associados em
paralelo. Isto significa que os trs esto submetidos mesma
tenso U, fornecida pela bateria. No entanto, se os
capacitores forem diferentes, as cargas em cada um deles
sero diferentes.

Podemos representar o capacitor equivalente
associao, isto , o capacitor que ligado mesma bateria,
ter carga total Q igual carga da associao:



Exerccios de Sala #

1. (PUC-MG) Um condensador de F 5 , 0 conectado aos
terminais de uma bateria de 12 V. correto afirmar que:
a) aps totalmente carregado, sua capacidade passa a ser
F 1 .
b) a tenso em seus terminais aumenta at o mximo de 6 V.
c) enquanto durar a ligao bateria, o condensador se
carregar, razo de 5 10
-7
C/V.
d) quase instantaneamente, armazena-se nele a carga de 6
10
6
C.
e) 30 J de energia eltrica se convertem em calor no
condensador.

2. (PUC-MG) Trs capacitores A,B e C iguais so ligados a
uma fonte de acordo com a figura abaixo.

Assinale a opo que representa um conjunto coerente para
o valor do mdulo das cargas acumuladas nos capacitores A,
B e C, NESSA ORDEM:

a) 100, 100, 100 d) 100, 100, 50
b) 100, 50, 50 e) 50, 50, 100
c) 50, 100, 100
Tarefa Mnima #

3. Um capacitor de capacidade 200 pF est ligado a uma
bateria de 100v. Determinar as cargas das placas e a energia
potencial eltrica acumulada nas placas.
a) 2x10
-8
C e 10
-8
j d) 2x10
-8
C e 10
-5
j
b) 4x10
-8
C e 10
-5
j e) 3x10
-8
C e 10
-4
j
c) 3x10
-8
C e 10
-7
j

4. Um capacitor plano tem placas de rea 20 cm
2
cada,
separados entre si de 10 cm. O capacitor carregado atravs
de uma fonte de tenso de l00V. Supondo que entre as
placas reine o vcuo determine a capacidade eltrica do
capacitor; a quantidade de carga do capacitor e a intensidade
do campo eltrico entre as armaduras.
Dados: c = 8,8 x 10
-12
F/m.
a) 4,36x10
-3
F, 4,36x10
-11
C, e 2000V/m
b) 2,06x10
-3
F, 1,76x10
-11
C, e 3000V/m
c) 1,76x10
-3
F, 1,76x10
-11
C, e 1000V/m
d) 4,36x10
-3
F, 5,36x10
-11
C, e 500V/m
e) 1,76x10
-3
F, 4,76x10
-11
C, e 1200V/m

5. Trs capacitores so associados, conforme figura:

Aplicando-se entre A e, B a ddp de 8V, determine a carga e
a ddp em cada capacitor; a carga da
associao; a capacitncia do capacitor
equivalente; e a energia potencial
eltrica da associao.

a) 60C, 40C, 16C, 136C, 17C, e 544j
b) 80C, 40C, 10C, 136C, 17C, e 544j
c) 50C, 40C, 15C, 136C, 17C, e 544j
d) 60C, 40C, 10C, 136C, 17C, e 544j
e) 80C, 40C, 16C, 136C, 17C, e 544j

6. Determine a carga armazenada pelo capacitor nos
circuitos:

a) a)1,5C, b)5C d) a)2,5C, b)7C
b) a)2,5C, b)5C e) a)0,5C, b)4C
c) a)1,5C, b)7C

7. (ACAFE) Dois capacitores de mesma capacitncia so
associados em paralelo. Pode-se ento afirmar que:

a) a carga do capacitor equivalente igual carga de cada
um dos capacitores.
b) a tenso entre as placas do capacitor equivalente o
dobro da tenso entre as placas de cada capacitor.
C = C
1
+ C
2
+ C
3
Q = Q
1
+ Q
2
+ Q
3


Please purchase PDF Split-Merge on www.verypdf.com to remove this watermark.
Fsica C Incluso para a Vida

Pr-Vestibular da UFSC
14
c) a capacitncia do capacitor equivalente igual
capacitncia de cada capacitor.
d) a capacitncia do capacitor equivalente menor que a
capacitncia de cada um dos capacitores.
e) a energia armazenada no capacitor equivalente o dobro
da energia armazenada em cada um dos capacitores.

8. (ACAFE) Complete corretamente a afirmativa:
Em um capacitor plana e paralelo ___________.

a) as cargas eltricas armazenadas nas placas possuem o
mesmo sinal.
b) Uma placa possui quantidade de carga eltrica diferente
da outra.
c) a capacitncia inversamente proporcional rea das
placas.
d) a capacitncia diretamente proporcional distncia
entre as placa.
e) a capacitncia depende docampo eltrico eltrico que se
encontra entre as placas.

9. (ACAFE) A figura a seguir representa um capacitor de
placas paralelas carregado. Pode-se afirmar que o campo
eltrico entre as placas deste capacitor :

a) maior em Q.
b) menor em R.
c) maior em S do que em R.
d) menor em Q do que em S.
e) igual em R e S.


10. (ACAFE) Dois capacitores
iguais so associados em srie e a combinao ento
carregada. Sejam C a capacitncia, Q a carga e V
D
potencial
de cada capacitor. Os valores correspondentes para a
combinao sero:
a) 2C; Q; 2V d) 2C; Q; V/2
b) C/2; Q; 2V e) 2C; 2Q; V
c) C/ Q/2 V

11. (ACAFE) Um capacitor com ar entre as placas
carregado com uma determinada diferena de potencial. Ao
introduzirmos um dieltrico entre as placas, podemos
afirmar que:
a) a carga nas placas do capacitor aumenta.
b) a capacitncia do capacitor permanece constante.
c) a voltagem entre as placas do capacitor diminui.
d) o valor do campo eltrico entre as placas do capacitor no
se altera.
e) a energia armazenada no capacitor aumenta.

UNIDADE 8

MAGNETISMO

MS
Um fato importante observado que os ms tm, em geral,
dois pontos a partir dos quais parecem se originar as foras.
Quando pegamos, por exemplo, um m em forma de barra
(Fig.) e o aproximamos de pequenos fragmentos de ferro,
observamos que esses fragmentos so atrados por dois
pontos que esto prximos das extremidades. Tais pontos
foram mais tarde chamados de plos (mais adiante veremos
porque).

Inseparabilidade dos plos
Por mais que se quebre um m, cada pedao um novo m
(Fig.). Portanto, no possvel separar o plo norte do plo
sul.







Magnetismo da Terra
A partir dessas observaes, percebemos que a terra se
comporta como se no seu interior houvesse um enorme m
em forma de barra (Fig.).



Porm, os plos desse grande m no coincidem com os
plos geogrficos, embora estejam prximos deles.

Portanto:
- o plo norte da bssola atrado pelo sul magntico, que
est prximo do norte geogrfico;
- o plo sul da bssola atrado pelo norte magntico que
est prximo do sul geogrfico.

O campo magntico
Para visualizar a ao do campo magntico, usado o que
chamamos de linhas de campo. Essas linhas so desenhadas
de modo que, em cada ponto (Fig.), o campo magntico
tangente linha.



Campo magntico uniforme
Quando o m tem a forma de ferradura, as linhas de campo
tm o aspecto mostrado na figura.

Please purchase PDF Split-Merge on www.verypdf.com to remove this watermark.
Incluso para a Vida Fsica C

Pr-Vestibular da UFSC 15
Exerccios de Sala #

1. (PUC-RS) Cargas eltricas podem ter sua trajetria
alterada quando em movimento no interior de um campo
magntico. Esse fenmeno fundamental permite explicar
a) o funcionamento da bssola.
b) o aprisionamento de partculas carregadas pelo campo
magntico da Terra.
c) a construo de um aparelho de raio X.
d) o funcionamento do pra-raios.
e) o funcionamento da clula fotoeltrica.

2. (UFSC) Uma bssola aponta aproximadamente para o
Norte geogrfico porque:
I o Norte geogrfico aproximadamente o Norte
magntico.
II o Norte geogrfico aproximadamente o sul magntico.
III o Sul geogrfico aproximadamente o norte magntico.
IV o Sul geogrfico aproximadamente o sul magntico.

Est(o) correta(s):
a) I e IV. c) II e III. e) Nenhuma.
b) Somente III. d) Somente IV.

Tarefa Mnima #

3. (UFRGS) Um prego de ferro AB, inicialmente no
imantado, aproximado do plo sul (S) de um m
permanente, conforme mostra a figura.


Nessa situao, forma-se um plo ________ e o m e o
prego se _______ .
Assinale a alternativa que preenche de forma correta as duas
lacunas, respectivamente.
a) sul em A atraem d) norte em A atraem
b) sul em A repelem e) norte em B atraem
c) sul em B repelem

4. (UFOP-MG) A figura abaixo mostra os plos norte e sul
de um m e cinco pontos marcados por I, II, III, IV e V.
Para que uma agulha da bssola fique na posio
S N
, ela dever ser colocada no ponto:

a) I b) II c) III d) IV e) V

5. (Mack-SP) As linhas de induo de um campo magntico
so:
a) o lugar geomtrico dos pontos, onde a intensidade do
campo magntico constante.
b) as trajetrias descritas por cargas eltricas num campo
magntico.
c) aquelas que em cada ponto tangenciam o vetor induo
magntica, orientadas no seu sentido.
d) aquelas que partem do plo norte de um m e vo at o
infinito.
e) nenhuma das anteriores.

6. (Osec-SP) Um estudante dispe de duas peas de material
ferromagntico. Uma delas um m permanente.
Desejando saber qual das peas o m, imaginou trs
experimentos, apresentados a seguir.
I - Pendurar as peas, sucessivamente, nas proximidades de
um m permanente e verificar qual pode ser repelida.
II - Aproximar as duas peas e verificar qual atrai a outra.
III - Aproximar as duas peas e verificar qual repele a outra.

Dentre essas experincias, a que permitir ao estudante
determinar qual pea o m :
a)somente a I e a II. d)somente a I.
b)somente a II. e)somente a I e a III.
c)somente a III.

7. (ACAFE) Complete corretamente a afirmativa:
Quando se magnetiza uma barra de ferro, ____________.
a) retiram-se ms elementares da barra.
b) acrescentam-se ms elementares barra.
c) ordenam-se os ms elementares da barra.
d) retiram-se eltrons da barra.
e) retiram-se prtons da barra.

8. (Cescem-SP) A prego de ferro AB, inicialmente no
imantado, aproximado do plo norte N de um m, como
mostra a figura abaixo. A respeito desta situao, so feitas
trs afirmaes:
I - O campo magntico do
m magnetiza o prego
parcialmente.
II - Em A forma-se um plo
norte e em B, um plo
sul.
III - O m atrai o prego.

Destas afirmaes, est(o) correta(s):
a) apenas I. c) apenas I e II. e) I, II e III.
b) apenas III. d)apenas II e III.
9. (PUC-RS) Dois campos magnticos uniformes,
1
B e
2
B ,
cruzam-se perpendicularmente. A direo do campo
resultante dada por uma bssola, conforme a figura. Pode-
se concluir que o mdulo B do campo resultante :

a) B = B
1
. sem 30.
b) B = B
1
. cos 30.
c) B = B
2
. tg 30.
d) B =
2
2
2
1
B B +
e) B = B
1
+ B
2
.

10. (UFSC) No incio do perodo das grandes navegaes
europeias, as tempestades eram muito temidas. Alm da
fragilidade dos navios, corria-se o risco de ter a bssola
danificada no meio do oceano. Sobre esse fato, correto
afirmar que:
01. A agitao do mar podia danificar permanentemente a
bssola.
02. A bssola, assim como os metais (facas e tesouras),
atraa raios que a danificavam.
04. O aquecimento do ar produzido pelos raios podia
desmagnetizar a bssola.
2
B
1
B
N
S
o
30
Please purchase PDF Split-Merge on www.verypdf.com to remove this watermark.
Fsica C Incluso para a Vida

Pr-Vestibular da UFSC
16
08. O campo magntico produzido pelo raio podia
desmagnetizar a bssola.
16. As gotas de chuva eletrizadas pelos relmpagos podiam
danificar a bssola.
32. A forte luz produzida nos relmpagos desmagnetizava
as bssolas, que ficavam geralmente no convs.

UNIDADE 9

ELETROMAGNETISMO

At agora temos considerado situaes em que o campo
magntico produzido por um m. No entanto, em 1820, o
fsico dinamarqus Hans Christian Oersted (1777-1851)
observou que as correntes eltricas tambm produzem
campo magntico.



Campo Magntico de um Condutor Reto
Para obtermos o sentido do campo, usamos a regra da mo
direita.

O mdulo de em um ponto P dado por:
d
i
B
o
t

2
.
=
No qual d a distncia do ponto P ao fio e
o
uma
constante, denominada permeabilidade do vcuo, cujo
valor no Sistema Internacional :
o
= 4t . 10
-7
(T.m)/A

Campo Magntico de Espira Circular




Verifica-se que no centro da espira, a intensidade do campo
magntico dada por:
d
i
B
o
2
.
=

Bobina Chata
Neste caso, a intensidade do campo magntico no centro da
bobina ser dada por:
d
i
N B
o
2
.
=





No qual N o nmero de espiras.


Campo Magntico de um Solenide



A intensidade do campo magntico no interior do solenide
dada por: i
l
N
B
o
. = Onde N o nmero de espiras.

Exerccios de Sala #

1. Um fio condutor, vertical e longo, percorrido por uma
corrente de intensidade i = 2A, conforme a figura abaixo.
Determine a intensidade, a direo e o sentido do vetor
induo magntica num ponto a 10 cm do fio.
Dado: =4t.10
-7
T . m/A.


2. (UFSC) A figura representa um fio infinito, o percorrido
por uma corrente de 15A. Sabendo-se que ambos os
segmentos AB e DE tem comprimento de 0,1m, o raio R do
semicrculo DB de
0,05t m, determine o valor do campo magntico, em (10
-5

N/Am), no ponto C.


Tarefa Mnima #

3. Dois fios longos, retos e paralelos, situados no vcuo, So
percorridos por correntes contrrias, com intensidades 2A e
4A, e separadas entre si de 0,20 m. Calcule a intensidade do
vetor induo magntica resultante no ponto P, indicado na
figura. Dado: =4t.10
-7
T . m/A

a) 12x10
-7
T
b) 20x10
-7
T
c) 220x10
-7
T
d) 120x10
-7
T
e) 50x10
-7
T


4. (Mack-SP) Um fio retilneo muito longo percorrido por
uma corrente eltrica constante i, e o vetor induo
magntica, num ponto P perto do fio, tem mdulo B. Se o
mesmo fio for percorrido por uma corrente eltrica =
constante 2i, o vetor do mdulo do vetor induo magntica
Please purchase PDF Split-Merge on www.verypdf.com to remove this watermark.
Incluso para a Vida Fsica C

Pr-Vestibular da UFSC 17
no mesmo ponto
P :






a) B/4 b) B/2 c) BX d) 2B e) 4B

5. Determine a intensidade do vetor induo magntica
originado pela corrente eltrica, no ponto O, nos seguintes
casos ( =4t.10
-7
T . m/A.):
a)


b)

c)

6. Dois condutores retos paralelos e extensos so
percorridos por corrente de mesma intensidade i =10A
Determine a intensidade do vetor induo magntica , no
ponto P, nos casos indicados abaixo. dado =4t.10
-7
T .
m/A.

















7. Dois condutores retos paralelos e extensos conduzem
correntes de sentidos opostos e intensidade i
1
= i
2
= 100A.
Determine a intensidade do vetor induo magntica no
ponto P.
Dado: =4t.10
-7
T . m/A
a) 2,8x10
-7
T d) 1,0x10
-7
T
b) 3,8x10
-7
T e) 2,2x10
-7
T
c) 1,8x10
-7
T

8. Uma espira condutora circular, de raio R, percorrida por
uma corrente de intensidade i, no sentido horrio. Uma outra
espira circular de raio R/2 concntrica com a precedente e
situada no mesmo plano. Qual deve ser o sentido e qual o
valor da intensidade de uma corrente que (percorrendo essa
segunda espira) anula o campo magntico resultante no
centro O? Justifique.

9. Duas espiras circulares concntricas, de 1 m de raio cada
uma, esto localizadas em anos perpendiculares. Calcule a
intensidade do campo magntico no centro das espiras,
sabendo que cada espira conduz 0,5 A.

10. (UFU-MG) Em um tomo de hidrognio, considerando
o eltron como sendo uma massa puntiforme que gira no
plano da folha em um rgo circular, como mostra a figura,
o vetor campo magntico criado no centro do crculo por
esse eltron representado por:


11. (ACAFE) Complete corretamente a afirmativa.
- Uma carga eltrica puntiforme em movimento
___________.
a) retilneo produz somente campo magntico.
b) retilneo produz somente campo eltrico.
c) retilneo produz campo eltrico e magntico.
d) curvilneo produz somente campo magntico.
e) curvilneo no produz campo eltrico, nem magntico.

UNIDADE 10

FORA MAGNTICA SOBRE CARGAS
ELTRICAS

DEFINIO DO MDULO DA FORA MAGNTICA
.
Usando esse fato, a intensidade de foi definida de modo
que a intensidade da fora magntica dada por:

O sentido de depende do sinal da carga. Na
figura indicamos o sentido de para o caso em que q > 0 e
tambm para uma q < 0. Esse sentido pode ser obtido por
uma regra chamada regra da mo direita, tambm
conhecida como regra do tapa.
0,10
m
0,10
i i P A
)
0,10m 0,10m
i i P B)
Please purchase PDF Split-Merge on www.verypdf.com to remove this watermark.
Fsica C Incluso para a Vida

Pr-Vestibular da UFSC
18



Unidade da intensidade de
No Sistema Internacional de Unidades, a unidade de
intensidade de se chama tesla e seu smbolo T.

OBS: Pelo fato de a fora magntica ser perpendicular
velocidade, ela nunca realiza trabalho.

Movimento quando o campo uniforme

I- Caso em que e tm a mesma direo
J vimos anteriormente que neste caso a fora
magntica nula e, assim, o movimento ser retilneo e
uniforme.

II- Caso em que perpendicular a
Neste caso teremos um movimento circular e
uniforme. Na figura, o campo perpendicular ao plano do
papel e "entrando" nele (Smbolo ).



O raio da trajetria ser: .
Sendo um movimento circular e uniforme, o perodo desse
movimento dado por: .

III- Caso em que e formam ngulo u qualquer
Neste caso podemos decompor a velocidade em
duas componentes, uma componente perpendicular a e
uma componente paralela a .



A trajetria uma hlice cilndrica cujo raio R.

Exerccios de Sala #

1. Uma partcula eletrizada com carga eltrica q = 2,0c
move-se com velocidade v = 3,0 .10
3
m/s em uma regio do
espao onde existe um campo magntico de induo cuja
intensidade de 5,0T, conforme a figura abaixo. Determine
as caractersticas da fora magntica que age na partcula. O
plano de B e V o plano do papel.

2. Em cada um dos casos dados a seguir determine a direo
e o sentido da fora magntica sobre a carga q assinalada.O
sinal da carga est discriminado em cada caso.

a)

b)

c)

d)


Tarefa Mnima #

3. A figura abaixo representa a combinao de um campo
eltrico uniforme , de intensidade 4,0 .10
4
N/C, com um
campo magntico uniforme de induo , de intensidade
2,0.10
-2
T. Determine a velocidade v que uma carga q =
5.10
-6
C deve ter para atravessar a regio sem sofrer
desvios.


V
B
0 > q
0 > q
V
B
0 > q
V
B
V
0 > q
B
Please purchase PDF Split-Merge on www.verypdf.com to remove this watermark.
Incluso para a Vida Fsica C

Pr-Vestibular da UFSC 19
a) 2x10
6
m/s c) 4x10
6
m/s e) 6x10
6
m/s
b) 3x10
6
m/s d) 5x10
6
m/s

4. (UFSC) Assinale as afirmativas corretas e some os
valores respectivos.
01. O fato de um prton, ao atravessar uma certa regio do
espao, ter sua velocidade diminuda poderia ser
explicado pela presena de um campo eltrico nesta
regio.
02. O fato de um eltron, ao atravessar uma certa regio do
espao, no sofrer desvio em sua trajetria nos permite
afirmar que no existe campo magntico nesta regio.
04. A trajetria de uma partcula eletricamente neutra no
alterada pela presena de um campo magntico.
08. A fora magntica que atua numa partcula eletricamente
carregada sempre perpendicular ao campo magntico.
16. A fora magntica que atua numa partcula eletricamente
carregada sempre perpendicular velocidade desta.
32. A velocidade de uma partcula eletricamente carregada
sempre perpendicular ao campo magntico na regio.

5. Uma partcula a, cuja carga
eltrica q = 3,2 x 10
-19
C, move-
se com velocidade de v = 3,0 x 10
5

m/s em uma regio de campo
magntico , de intensidade 2,5 x
10
5
T, conforme a figura.
Determine o mdulo da fora
magntica sobre a partcula.

a) 3,2.10
-8
N d) 4,1.10
-8
N
b) 2,4.10
-8
N e) 5,0.10
-8
N
c) 1,6.10
-8
N

6. (UFSC) As afirmativas abaixo referem-se a fenmenos
magnticos. Assinale a(s) proposio(es) verdadeira(s):
01.Um estudante quebra um m ao meio, obtendo dois
pedaos, ambos com plo sul e plo norte.
02.Um astronauta, ao descer na Lua, constata que no h
campo magntico na mesma, portanto ele poder usar
uma bssola para se orientar.
04. Uma barra imantada se orientar ao ser suspensa
horizontalmente por um fio preso pelo seu centro de
gravidade ao teto de um laboratrio da UFSC.
08. Uma barra no imantada no permanecer fixa na porta
de uma geladeira desmagnetizada, quando nela colocada.
16. Uma das formas de desmagnetizar uma bssola coloc-
la num forno quente.
32. Uma das formas de magnetizar uma bssola coloc-la
numa geladeira desmagnetizada.

7. Um feixe de eltrons lanado no interior de um campo
magntico com velocidade , paralelamente ao campo
magntico uniforme de induo , conforme ilustra a figura.
Podemos afirmar que o feixe:


a) sofrer uma deflexo para cima, mantendo-se no plano da
pgina.
b) sofrer uma deflexo para baixo, mantendo-se no plano
da pgina.
c) sofrer uma deflexo para dentro da pgina.
d) manter sua direo original.
e) sofrer uma deflexo para fora da pgina.

8. Uma carga eltrica q, de massa m move-se inicialmente
com velocidade constante V
0
no vcuo. A partir do instante
t= 0, aplica-se um campo magntico uniforme de induo B,
perpendicular a V
0
. Afirma-se que:

a) A partcula continua em movimento retilneo e uniforme.
b) A partcula passa a descrever uma circunferncia de raio
Bq
mv
r = .
c) A partcula passa a descrever uma hlice cilndrica.
d) A partcula passa a descrever um movimento retilneo
uniformemente variado.
e) Nenhuma das afirmaes anteriores correta.

9. Um eltron penetra em um campo magntico segundo um
ngulo u (ngulo que o vetor velocidade v faz com as linhas
de B). Nestas condies a trajetria do eltron uma:
a) circunferncia c) hiprbole e) parbola
b) linha reta d) hlice

10. (PUC-SP) Um corpsculo carregado com carga de 100
C passa com velocidade de 25 m/s na direo
perpendicular a um campo de induo magntica e fica
sujeito a uma fora de 5 . 10
-4
N. A intensidade desse
campo vale:
a) 0,1 T b) 0,2 T c) 0,3 T d) 1,0 T e) 2,0 T

11. (PUC-SP) Quando uma barra de ferro magnetizada
so:
a) acrescentados eltrons barra.
b) retirados eltrons da barra.
c) acrescentados ms elementares barra.
d) retirados ms elementares da barra.
e) ordenados os ms elementares da barra.

UNIDADE 11

FORA MAGNTICA SOBRE
CONDUTORES

CONDUTOR RETILNEO
Nessa figura representamos uma fila de eltrons movendo-se
com velocidade ; o sentido da corrente convencional (i)
oposto ao movimento dos eltrons. O fio forma ngulo u
com o campo magntico.


Please purchase PDF Split-Merge on www.verypdf.com to remove this watermark.
Fsica C Incluso para a Vida

Pr-Vestibular da UFSC
20
Para obtermos o mdulo da fora magntica sobre o
condutor, basta aplicarmos a equao:

Fora Magntica entre Condutores Retos e Paralelos

Na Figura a seguir representamos dois fios X e Y, retos,
longos e paralelos, percorridos por correntes de intensidades
i
1
e i
2
, de mesmo sentido.



Nessa figura representamos o campo magntico
produzido pela corrente i
1
. A intensidade do campo sobre o
condutor Y :
d
i
B
o
t

2
.
1
1
=
Portanto a fora magntica ( ) sobre o fio Y tem
intensidade F dada por:



OBS: Aplicando a regra da mo direita, percebemos que,
neste caso, as foras entre os fios so de atrao. Quando os
fios so percorridos por correntes de sentidos opostos, as
foras so de repulso.

Exerccios de Sala #

1. Um condutor retilneo, de comprimento 1 = 0,2m,
percorrido por uma corrente eltrica de intensidade i = 2A.
Sabe-se que o condutor est totalmente imerso em um
campo magntico uniforme, cujo vetor induo magntica
tem intensidade B = 0,5T. Sendo 30 o ngulo formado entre
a direo dele e a da corrente eltrica, caracteriza a fora
magntica que atua sobre o condutor.

2. Em um motor eltrico, fios que conduzem uma corrente
de 5A so perpendiculares a um campo de induo
magntica de intensidade 1T. Qual a fora exercida sobre
cada centmetro do fio?

Tarefa Mnima #

3. Uma das maneiras de se
obter o valor de um campo
magntico uniforme
colocar um fio condutor
perpendicularmente s
linhas de induo e medir a
fora que atua sobre o fio
para cada valor da corrente
que o percorre. Em uma destas experincias, utilizando-se
um fio de 0,1m, obtiveram-se dados
que permitiram a construo do
grfico abaixo, onde F a intensidade
da fora magntica e i a corrente eltrica. Determine a
intensidade do vetor campo magntico.
a) 10
-4
T c) 10
-1
T e) 10
-2
T
b) 10
-3
T d) 10
-5
T

4. (PUC-SP) A espira
condutora ABCD
rgida da figura pode
girar livremente em
torno do eixo L. Sendo
percorrida pela corrente
de valor i, a espira, na
posio em que se
encontra, tender a:
a) ser elevada verticalmente.
b) girar no sentido horrio.
c) girar no sentido anti-horrio.
d) permanecer em repouso, sem movimento giratrio.
e) girar de 90 para se alinha com o campo de induo
magntica do m.

5. (UFSC) Obtenha a soma dos valores numricos
associados s opes corretas. Um condutor retilneo,
percorrido por uma corrente eltrica I, colocado entre os
plos de um im como indica a
figura abaixo.

Podemos afirmar que:
01. A fora magntica que age no condutor tem a direo
norte-sul do m e aponta no sentido do plo sul.
02. A fora magntica que age no condutor tem a direo
norte-sul do m e aponta no sentido do plo norte.
04. A fora magntica sobre o condutor aponta para dentro
do plano do papel.
08. A fora magntica sobre o condutor aponta para fora do
plano do papel.
16. A fora magntica que age no condutor tem o mesmo
sentido que a corrente eltrica I.
32. No existe fora magntica atuando no condutor.
64. A fora magntica depende da intensidade da corrente
eltrica I que percorre o condutor.

6. (UFSC) Considere um fio retilneo infinito, no qual passa
uma corrente i. Marque no carto-resposta a soma dos
valores associados s das proposies verdadeiras:

01. Se dobrarmos a corrente i, o campo magntico gerado
pelo fio dobra.
02. Se invertermos o sentido da corrente, inverte-se o
sentido do campo magntico gerado pelo fio.
04. O campo magntico gerado pelo fio cai 1/r
2
, onde r a
distncia ao fio.
F
m
= B . i . L . sen u
Please purchase PDF Split-Merge on www.verypdf.com to remove this watermark.
Incluso para a Vida Fsica C

Pr-Vestibular da UFSC 21
08. Se colocarmos um segundo fio, tambm infinito,
paralelo ao primeiro e pelo qual passa uma corrente no
mesmo sentido de i, no haver fora resultante entre
fios.
16. Se colocarmos um segundo fio, tambm infinito,
paralelo ao primeiro e pelo qual passa corrente no
sentido inverso a i, haver uma fora repulsiva entre os
fios.
32. Caso exista uma partcula carregada, prxima ao fio, ser
sempre diferente de zero a fora que o campo magntico
gerado pelo fio far sobre a partcula.

7. (Santa Ceclia-SP) Um trecho
MN de um fio retilneo com
comprimento de 10 cm,
conduzindo uma corrente eltrica de 10 ampres, est imerso
em uma regio, no vcuo, onde existe um campo de induo
magntica de 1,0 tesla, conforme a figura. A fora que age
no trecho do fio :

a) 1,0 newton, para dentro do papel.
b) 0,5 newton, para fora do papel.
c) 1,0 newton, no sentido do campo.
d) 1,5 newton, no sentido oposto ao do campo.
e) 1,0 newton, para fora do papel.
8. (PUC-SP) Um condutor retilneo de comprimento 0,5 m
percorrido por uma corrente de intensidade 4,0 A. O
condutor est totalmente imerso num campo magntico de
intensidade 10
-3
T, formando com a direo do campo um
ngulo de 30. A intensidade da fora magntica que atua
sobre o condutor :
a) 10
3
N c) 10
-4
N e) nula
b) 2.10
-2
N d) 10
-3
N.

9. Dois condutores retos e extensos, paralelos, so separados
por r = 1m e percorridos por correntes iguais de 1A e de
mesmo sentido. Se ambos esto no vcuo (
0
= 4 .10
-7

T.
m
/A), caracterize a fora magntica entre eles por
centmetro de comprimento.
a) 3,0x10
-9
N c) 2,0x10
-9
N e) 1,5x10
-9
N
b) 2,5x10
-9
N d) 1,0x10
-9
N

10. Dois fios longos, retos e paralelos, situados no vcuo,
so percorridos por correntes contrrias, de intensidades i
1
=
2A e i
2
= 4A. A distncia entre os fios de 0,1 m.
a) Os fios se atraem ou se repelem?
b) Com que fora, para cada metro de comprimento do fio?
c) O que ocorrer se inverter o sentido da corrente i
2
?
Dado: permeabilidade magntica do vcuo:
0
= 4t .10
-7
T .
m
/A.




UNIDADE 12

INDUO ELETROMAGNTICA

FLUXO MAGNTICO



Sendo u o ngulo entre e , definimos o fluxo (|) de
atravs da superfcie, pela equao:
No Sistema Internacional de Unidades, a unidade
de fluxo magntico o weber (Wb).

Fora eletromotriz induzida
Suponhamos que a corrente induzida tenha intensidade i e o
circuito tenha resistncia R. Tudo se passa como se
houvesse no circuito um gerador de fora eletromotriz E,
dada pela equao vista na aula de corrente eltrica:
E = R . i
Essa fora eletromotriz chamada de fora eletromotriz
induzida.

Variaes de Fluxo
Como o fluxo dado por: | = B . A . cos u, percebemos que
o fluxo pode variar de trs maneiras:
1) variando o campo magntico
2) variando a rea A
3) variando o ngulo u (girando o circuito)

Lei de Lenz
Heinrich Lenz (1804-1865), nascido na Estnia, estabeleceu
um modo de obter o sentido da corrente induzida:

A corrente induzida tem um sentido tal que se ope
variao de fluxo.

Lei de Faraday
Suponhamos que o fluxo magntico que atravessa um
circuito sofra uma variao A| num intervalo de tempo At. O
valor mdio da fora eletromotriz induzida nesse intervalo
de tempo dado, em mdulo, por:

No entanto o sinal "menos" serve apenas para
lembrar a lei de Lenz, isto , que a fora eletromotriz
induzida se ope variao de fluxo.

Condutor Retilneo movendo-se sob a Ao de Campo
Magntico Uniforme
Na figura representamos um condutor em forma de U sobre
o qual se move, com velocidade , um condutor reto WZ. O
conjunto est numa regio em que h um campo magntico
uniforme , perpendicular ao plano do circuito. Na posio
da figura, a rea do circuito :

| = B. A . cos u
Please purchase PDF Split-Merge on www.verypdf.com to remove this watermark.
Fsica C Incluso para a Vida

Pr-Vestibular da UFSC
22
Assim, temos:


Transformadores
Transformador de tenso um dispositivo capaz de elevar
ou rebaixar uma ddp.



Sejam N
1
e N
2
os nmeros de espiras no primrio e
secundrio, respectivamente. Pode-se, ento, demonstrar
que:
2
1
2
1
N
N
V
V
=

Onde V
1
e V
2
so tenses no primrio e secundrio
respectivamente.

Exerccios de Sala #

1. O campo Magntico uniforme de induo , em uma
regio, tem intensidade 0,5 T. Calcule a fem induzida em um
condutor retilneo de 10 cm de comprimento, que se desloca
com velocidade de 1 m/s.


2. Um transformador est ligado a uma tomada de 120V.
Seu primrio tem 800 espiras. Calcule o nmero de espiras
do secundrio, sabendo que a ele ligada uma campainha de
6V.

3. (UFLA-MG) A figura a
seguir representa um
transformador que serve para
elevar ou reduzir nveis de
tenso (voltagem). Com
relao indicao do ponteiro
do galvanmetro (G) e a
posio da chave ( C ), pode-se afirmar que:
a) O ponteiro do galvanmetro ir defletir no sentido
horrio enquanto a chave ( C ) permanecer fechada.
b) O ponteiro do galvanmetro ir defletir no sentido anti-
horrio, enquanto a chave ( C ) permanecer fechada.
c) O ponteiro do galvanmetro sofrer deflexes somente
nos instantes em que se fechar ou abrir a chave.
d) Considerando a chave ( C ) fechada no haver deflexo
instantnea do ponteiro no instante de sua abertura.
e) O ponteiro do galvanmetro ficar oscilando enquanto a
chave ( C ) permanecer fechada.

Tarefa Mnima #

4. (PUC-RS) Responder questo com base nas
informaes e
figura abaixo. Uma
bobina est prxima
de um m em
forma de barra
como indica a figura.
Trs situaes podem ocorrer, alternativamente:
I - Somente o m se move.
II - Somente a bobina se move.
III - Os dois se movem, ambos com mesma velocidade em
sentidos contrrios.

De acordo com os dados acima, correto dizer que ser
induzida uma fora eletromotriz nos extremos da bobina:

a) somente na situao I. d) em nenhuma das situaes.
b) somente na situao II. e) em todas as situaes.
c) somente nas situaes I e II.

5. (UFSC) Em um laboratrio de Fsica experimental, um
m deixado cair verticalmente, atravs de um solenide
longo,
feito de fio de cobre esmaltado, tendo
pequena resistncia hmica, em cujas
extremidades temos conectado um
galvanmetro (G).
A situao est ilustrada na figura ao
lado.
Em relao situao descrita,
assinale a(s) proposio (es)
correta(s).


01. A presena do solenide no afeta o movimento de
queda do m.
02. Com o movimento do m, surge uma fora eletromotriz
induzida nas espiras do solenide e o galvanmetro
indica a passagem de corrente.
04. Ao atravessar o solenide, o m fica sob a ao de uma
fora magntica que se ope ao seu movimento, o que
aumenta o tempo que esse m leva para atravessar o
solenide.
08. Ao atravessar o solenide, o m fica sujeito a uma fora
magntica que se adiciona fora peso, diminuindo o
tempo que o m leva para atravessar o solenide.
16. O sentido da corrente induzida no solenide, enquanto o
m est caindo na metade superior do solenide, tem
sentido oposto ao da corrente induzida enquanto o m
est caindo na metade inferior do solenide.
32. O galvanmetro no indica passagem de corrente no
solenide durante o movimento do m em seu interior.
64. Parte da energia mecnica do m convertida em calor,
nas espiras do solenide, por efeito Joule.

|E|=BLv
Please purchase PDF Split-Merge on www.verypdf.com to remove this watermark.
Incluso para a Vida Fsica C

Pr-Vestibular da UFSC 23
N N
S
6. (PUC-RS) O fenmeno da induo eletromagntica
usado para gerar praticamente toda a energia eltrica que
consumimos. Esse fenmeno consiste no aparecimento de
uma fora eletromotriz entre os extremos de um fio condutor
submetido a um:
a) campo eltrico.
b) campo magntico invarivel.
c) campo eletromagntico invarivel.
d) fluxo magntico varivel.
e) fluxo magntico invarivel.

7. (UFSC) Na figura abaixo, o condutor CD tem resistncia
desprezvel e mede 60,0 centmetros de comprimento,
movimentando-se sobre dois trilhos condutores, com
velocidade constante e igual a 80,0 metros por segundo para
a direita. O campo magntico aplicado uniforme,
perpendicular ao plano da pgina e o seu sentido saindo
da figura. Sabendo-se
que a intensidade
(mdulo) de 10,0
teslas, que a
resistncia R vale
20,0 ohms e existe o
aparecimento de uma
fora eletromotriz
induzida, determine o valor da corrente eltrica medida pelo
ampermetro (suposto ideal), em ampres.

8. (UFSC) Duas espiras, uma retangular e outra circular, so
colocadas prximas a um fio retilneo percorrido por uma
corrente
constante I, como
se mostra na
figura abaixo. As
espiras so
submetidas s
foras
1
F
&
e
2
F
&

de maneira a se deslocarem com uma mesma velocidade v
*
,
constante, que as afasta do fio. A rea da espira retangular
o dobro da rea da espira circular.

Assinale a(s) proposio(es) correta(s).
01. Como a corrente no fio permanece constante, no ocorre
variao do fluxo magntico atravs das espiras e,
portanto, nenhuma
corrente induzida
nas mesmas.
02. Como o fluxo
magntico varia
atravs da rea das
espiras, uma
corrente induzida se
estabelece em ambas
as espiras.
04. O sentido da
corrente induzida na espira circular horrio e na espira
retangular anti-horrio.
08. Quanto maior a velocidade com que as espiras se
afastam do fio, maiores so as correntes induzidas nas
espiras.
16. Parte do trabalho realizado pelas foras
1
F
&
e
2
F
&

transformado em calor por efeito Joule nas espiras.
32. As espiras tm reas diferentes, porm tm a mesma
velocidade; assim, o valor da corrente induzida o
mesmo nas duas espiras e, como ambas se afastam do
fio, o sentido das correntes induzidas o mesmo, ou seja,
tem sentido horrio.
64. Como a rea da espira retangular o dobro da rea da
espira circular, a corrente induzida na espira retangular
maior do que a corrente induzida na espira circular.

9. (UFSC) Uma espira retangular de fio condutor posta a
oscilar, no ar, atravessando em seu movimento um campo
magntico uniforme,
perpendicular ao seu
plano de oscilao,
conforme est
representado na figura
abaixo. Ao oscilar, a
espira no sofre
rotao (o plano da
espira sempre
perpendicular ao campo magntico) e atravessa a regio do
campo magntico nos dois sentidos do seu movimento.
Assinale a(s) proposio(es) correta(s).
01. Como a espira recebe energia do campo magntico, ela
levar mais tempo para atingir o repouso do que se
oscilasse na ausncia dos ms.
02. O campo magntico no influencia o movimento da
espira.
04. Parte da energia mecnica ser convertida em calor por
efeito Joule.
08. A espira levar menos tempo para atingir o repouso, pois
ser freada pelo campo magntico.
16. O sentido da corrente induzida enquanto a espira est
entrando na regio do campo magntico, oposto ao
sentido da corrente induzida enquanto a espira est
saindo da regio do campo magntico.
32. Os valores das correntes induzidas no se alteram se
substitumos a espira retangular por uma espira circular,
cujo raio seja a metade do lado maior da espira
retangular.
64. As correntes induzidas que aparecem na espira tm
sempre o mesmo sentido.

10. (UFSC) Pedrinho, aps uma aula de Fsica, resolveu
verificar experimentalmente o que tinha estudado at o
momento. Para tal experimento, ele usou uma bobina com
50 espiras, um m preso a um suporte no condutor e uma
lmpada incandescente de 5 W de potncia. O experimento
consistia em mover o m para dentro e para fora da bobina,
repetidamente.

Ao terminar o experimento, Pedrinho fez algumas
observaes, que esto listadas na forma de proposies.

Assinale a(s) proposio(es) correta(s).

01. O mdulo da fora eletromotriz induzida na bobina
diretamente proporcional variao do fluxo magntico
em funo da distncia.
Please purchase PDF Split-Merge on www.verypdf.com to remove this watermark.
Fsica C Incluso para a Vida

Pr-Vestibular da UFSC
24
02. difcil mover o m dentro da bobina, pois o campo
magntico de cada espira oferece uma resistncia ao
movimento do m. Isto explicado pela Lei de Lenz.
04. Se a corrente na lmpada for de 2 A, a fora
eletromotriz induzida em cada espira da bobina
0,05 V.
08. A frequncia do movimento do m no interior da
bobina no interfere na luminosidade da lmpada.
16. Para haver uma corrente induzida na bobina necessrio
que o circuito esteja fechado.
32. O trabalho realizado para mover o m para dentro
e para fora da bobina transformado integralmente em
energia luminosa na lmpada.

11. (UFSC)
Na transmisso de energia eltrica das usinas at os pontos
de utilizao, no bastam fios e postes. Toda a rede de
distribuio depende fundamentalmente dos
transformadores, que ora elevam a tenso, ora a rebaixam.
Nesse sobe-e-desce, os transformadores no s resolvem
um problema econmico, como melhoram a eficincia do
processo. O esquema abaixo representa esquematicamente
um transformador ideal, composto por dois enrolamentos
(primrio e secundrio) de fios envoltos nos braos de um
quadro metlico (ncleo), e a relao entre as voltagens
no primrio e no secundrio dada por
s
p
s
p
N
N
V
V
= .







Em relao ao exposto, assinale a(s) proposio(es)
correta(s).
01. O princpio bsico de funcionamento de um
transformador o fenmeno conhecido como induo
eletromagntica: quando um circuito fechado
submetido a um campo magntico varivel, aparece no
circuito uma corrente eltrica cuja intensidade
proporcional s variaes do fluxo magntico.
02. No transformador, pequenas intensidades de corrente no
primrio podem criar grandes intensidades de fluxo
magntico, o que ocasionar uma induo
eletromagntica e o aparecimento de uma voltagem no
secundrio.
04. O transformador acima pode ser um transformador de
elevao de tenso. Se ligarmos uma bateria de
automvel de 12 V em seu primrio (com 48 voltas),
iremos obter uma tenso de 220 V em seu secundrio
(com 880 voltas).
08. Podemos usar o transformador invertido, ou seja, se o
ligarmos a uma tomada em nossa residncia (de corrente
alternada) e aplicarmos uma tenso de 220 V em seu
secundrio (com 1000 voltas), obteremos uma tenso de
110 V no seu primrio (com 500 voltas).
16. Ao acoplarmos um transformador a uma tomada e a um
aparelho eltrico, como no h contato eltrico entre os
fios dos enrolamentos primrio e secundrio, o que
impossibilita a passagem da corrente eltrica entre eles,
no haver transformao dos valores da corrente
eltrica, somente da tenso.
32. O fluxo magntico criado pelo campo magntico que
aparece quando o transformador ligado depende da
rea da seco reta do ncleo metlico.















Primrio
Voltagem (V
p
)
N de voltas(N
p
)
Secundrio
Voltagem (V
s
)
N de v
oltas(N )
Please purchase PDF Split-Merge on www.verypdf.com to remove this watermark.
Incluso para a Vida Gabaritos

Pr-Vestibular da UFSC

1
|
.
|

\
|
4
1
,
4
1
10
9
2
3 + x
2
1 4

+
x
x
MATEMTICA A

Unidade 1
1)
2)
3)
4) a) 120 b) 12 c) 240
d) 12
5) b
6) a) 10 b) 16
7) 80
8) e
9) d
10) b
11) d
12) d
13) b
14) 47
15) b
16) 13
17) b
18) b

Unidade 2
1)
2)
3)
4) a) {1, 2, 3, 4, 6, 12}
b) {0, 3, 6, 9, 12, 15,....}
c) {3, 4, 5, 6, 7}
d) {-1, 0, 1, 2}
e) {0, 2, 4, 6, 8, 10,
12,.....}
f) {1, 3, 5, 7, 9, ......}
5) c
6)
3
23

7) a) S = {-10,10} b) S = {-
8, 6} c) S = C d) S = {-
1,1}
8) a
9)
198
127

10) e
11) b
12) 06
13) b
14) c
15) d
16) d
17) e

Unidade 3
1)
2)
3)
4) a) 4 b)
3
1

c)
7
4


d) S = 9 e) S = C f)
5) b
6) e
7) a) (2,1) b) (3,2) c)

8) a) {xe R| x > 7} b)
{xe R| x > 2 }
c) {xe R| x >
6
1
}
9) 08
10) 1
11) 82
12) x > 100km
13) 16
14) 95
15) 39
16) 92
17) 40
18) b

Unidade 4
1)
2)
3)
4) a) {2,3} b) {2,4} c) {2,
1/3} d) {2} e) C
f) {-5, 5} g) {0,5}
5) a
6) a
7) a
8) 5
9) S = {0}
10) a
11) 62
12) x = 3
13) a
14) 15
15) 07
16) a
17) 03
18) 05

Unidade 5
1)
2)
3)
4) e
5) 31
6) a) {x e R| x = 3} b) {x
e R| x > 3} c) {x e R| x s
6, x = 2} d) 9
7) 10
8) c
9) a
10) a) -1 b) 3 c) 2 e
4
11) e
12) b
13) d
14) d
15) 21
16) 33
17) 29
18)


Unidade 6
1)
2)
3)

4)


5) 02
6) a
7) b
8) 02
9) c
10) d
11) e
12) 01
13) c
14) 99
15) e
16) d
17) d
18) 0,2

Unidade 7
1)
2)
3)
4) a)


razes: -1 e 3 vrtice: (1, -
4) Im = { y e R / y > 4 }
b)


razes: -2 e 4 vrtice: (1, -
9) Im = { y e R / y > -9
}
c)

raiz: 1 vrtice: (1, 0) Im
= { y e R / y s 0 }
d)

razes: 0 e 3 vrtice: (3/2, -
9/4) Im = {y e R/ y > -
9/4}
5) 55
6) 27
7) b
8) a
9) 29
10) c
11) 0 e 4
12) e
13) d
14) 01
15) 23
16) c
17) e
18) 08

Unidade 8
1)
2)
3)
4) a) {x e R | x < 2 ou x >
4} b) {x e R | 2 s x s 4}
c) {x e R | - 3 < x < 3}
d) {x e R | -2 s x s 2}
e) {x e R | x < 0 ou x >
6} f) {x e R | x s -1
ou x > 1}
5) a
6) a) ]-4, -1[ ]1, 3[
b) ]-, -4] [-1, 1] [3,
[
c) ]-, -4[ ]3, 4[
d ) ]-, - 1] [0, 1] e) [3,
[
7) a) {x e R| x < - 4 ou 2 s
x s 3 ou x > 4}
b) {x e R| -4 < x < 2 ou 3
< x < 4}
c) {x e R|x < 1 ou 0 s x
< 1}
d) {x e R|x < 1 ou x > 3}
8) d
9) a) {x e R | x = 3} b) 9
c) C d) {3}
10) a) R b) R c) C d)
C
11) e
12) a
13) c
14) a
15) d
16) d
17) d
18) a

Unidade 9
1)
2)
3)
4) a) f(g(x)) = 2x
2
+ 2
b) g(f(x)) = 2x
2
+ 8x + 8
c) f(f(x)) = x + 4
d) g(g(x)) = 8x
4

e) 20
f) 18
g) 8
5) a
6) 81
7) a) f
-1
(x) =

b) f
-1
(x) = 4x 2

c) f
-1
(x) =
8) 01
9) 61
10) 00
11) 99
12) e
13) d
14)
3 1 ) ( )
2 ) ( )
2
7
) ( )
1
1
1
+ + =
+ =

x x f c
x x f b
x
x f a
15) c
16) 05
17) 03
18) x
2
+ 6x + 9

2
1 9 + x
Please purchase PDF Split-Merge on www.verypdf.com to remove this watermark.
Incluso para a Vida Gabaritos

Pr-Vestibular da UFSC

2
3
5
3
5
Unidade 10
1)
2)
3) a) 7 b) 4 c) 3 d)
02 e) 00
4) 02
5) b
6) a) S = { xe R| x > 2 }
b) S = { xe R| x > 3 }
c) S = { xe R| - 2 < x <
2 }
d) S = { xe R| x < - 5 ou
x > 9 }
7) a
8) c
9) 02
10) 01
11) 01
12) 00
13) 03
14) {x e9| x s - 2 ou x > 2}
15) 30
16)

17) a) {-1, 1} b) {0, 1}

Unidade 11
1)
2)
3) a) 9 b) 1 c) 0 d) -
7/26
4) a) 13 b) 6
5) a) 1, 07 b) 1, 71 c)
0, 17 d) 0, 54
6) b
7) b
8) 06
9) b
10) e
11) 3r s t/3
12)
cd
b a
3 2

13) 09
14) 17
15) a
16) a) 1 < x < 3 e x = 2
b) x < - 2 ou 2 < x < 5 e x =
4
17) 14

Unidade 12
1)
2)
3) a
4) a
5) a) { 6}
b) {2, -1}
c) {27}
d) {9}
e) { }
f) 08
6) 05
7) a) { x e R| x > 6} b) { x
e R| 3 < x < 7}
8) 30
9) e
10) 04
11) 31
12) 99
13) 16
14) 25
15) 47
16) 03
17) c

MATEMTICA B

Unidade 1
1) a) 81 b) 81 c) 81
d) 1 e) 0 f) 1 g)
16
1

h)
125
8
i) 18 j) 5 k)
35/12
2) a) 2
15
b) 2
13
3) a) 2
100
+ 1 b) 2
101
c)
2
102
d) 2
200
e) 2
99
f) 2
50

4) a) 5 b) 2 c) 0 d) 1
e) 9/4 f) 0,5
5)a)
2
2 5
b)
3 2
c)
5
25 2
3

d) 5(
2 3 +
)
6) e
7) 15
8) c
9) d
10) e
11) e
12) 31
13) c
14) d
15) e

Unidade 2
1) a) 6 b) 3 c) 5 2

2) e
3) 30
4) x = 2 y = 2 3

5) 14
6) 180 m
7) x = 100 3 y = 100

8) e 9) 31 10) 57

Unidade 3
1) 4 2
2) 75
3) 14
4) d
5) e
6) b
7) b
8) a
9) 2 7
10) b

Unidades 4 e 5
1) a) 120 b) 30
2) a
3) 2
4) b
5) a
6)a) S =
)
`

2
t

b) S =
)
`

2
3
,
2
t t
c) S =
)
`

18
33
,
6
7 t t
d)
)
`

4
7
,
4
t t

7) c
8) c
9) b
10) c
11) b
12) 13
13) c
14) c
15) 04

Unidade 6
1)
3
2 2

2) 00
3) 00
4) 01
5) a) 4)
)
`

3
4
,
3
t t

b)
3 7
0
4 4
, , ,
t t
t

`
)

6) 01
7) 01
8)
2

9) b
10) d

Unidade 7
1) a) 2 b) 2 c) 1
2) a
3) 25
4) e
5) 41
6) 01
7) a
8) 86
9) 12
10) c

Unidades 8 e 9
1) e
2) 13
3) e
4) e
5) 16
6) 08
7) c
8) 03
9) a
10) e
11) a
12) 81

Unidade 10
1) a) 5x + y 7 = 0 b) y = -
5x + 7 c) 5 e 7
2) 23
3) y = x
3
- 2
4) c
5) d
6) y = 3x 2
7) 07
8) 55
9) 90
10) 20

Unidade 11
1) c
2) a
3) c
4)
2
2 5
5) d
6) 04
7) 09
8) d
9) 02
10) 90

Unidade 12
1) a
2) c
3) a
4) a
5) a
6) c
7) 08
8) c
9) a
10) 28

MATEMTICA C

Unidade 1
1)
2)
3)
4) 34,50 cand/vaga
5) 24 e 36
6) x = 15 e y = 5
7) c
8) 48, 72, 96, 144
9) 72, 64, 84
10) 35 anos e 20 anos
11) a
12) d
13) d
14) 04
15) 10
16) a
17) p =
2
1
m =
2
1


18)
6
1
,
6
5
,
5
1


Unidade 2
1)
2)
3)
4) c
5) b
6) 75
7) a) 20 b) 44 c) 20 d)
30
o

8) a
9) 85
10) a
11) 21
12) 80
13) b
14) e
15) 30
16) 130
17) 120

Unidade 3
1)
2)
3)
4)
5)
6) b
7) c
8) c
Please purchase PDF Split-Merge on www.verypdf.com to remove this watermark.
Incluso para a Vida Gabaritos

Pr-Vestibular da UFSC

3
9) a) 10 3 b) 10 c) 10
2
10) c
11) d
12) e
13) quadrado e dodecgono
14) d
15) d
16) a
17) d
18) 40
o
19) 2
20)
R
2
1 5


Unidade 4
1)
2)
3)
4)
5) a) 43 b) 50 c) 75
6) a
7) a
8) 3/5
9) 29
10) a
11) c
12) 50
13) 32
14) 215
15) 20
16) b
17) 2,6; 3,9; 6,5
18) b
19) 20

Unidade 5
1)
2)
3)
4) c
5) a
6) 12
7) 13
8) 15
9) b
10) e
11) c
12) 9t cm
2

13) b
14) 03
15) a
16) d
17) 16
18) 20

Unidade 6
1)
2)
3)
4) a
5) a
6) a
7) e
8) e
9) 23
10) 18
11) d
12) d
13) a

Unidade 7
1)
2)
3) 32dm
3
4) 16
5) c
6) 36
7) a
8) 96
9) 04
10) d
11) 72

Unidade 8
1)
2)
3) 64
4) 68
5) 02
6) 64
7) 02
8) 48
9) a
10) 13
11) 06
12) a) 80 b) 512

Unidade 9
1)
2)
3)
4) 64
5) 48
6) 24
7) b
8) b
9) 03
10) 18
11) 64
12) d
13) c
14) d
15) d
16) a
17) b

Unidade 10
1)
2)
3)
4)
5)
6) 54
7) 09
8) c
9) a
10) e
11) 07
12) d
13) b
14) a
15) 96
16) 64
17) a
18) d
19) d
20) 09

Unidade 11
1)
2)
3)
4) a) 1 b) 4 c) -9/8
5) 07
6) 01
7) 06
8) 54
9) 04
10) a
11) a
12) 61
13) 120
14) d
15) b
16) b
17) 30
18) 60
19) 99
20) 02
21) 35
22) 90
23) 40

Unidade 12
1)
2)
3)
4)
5)
6) a) 2 b) 1/8
7) 03
8) 03
9) c
10) b
11) c
12) d
13) a
14) 16
15) 16
16) 06
17) d
18) 50
19) a
20) 96
16) 32
17) 3h
18) a
19) 09
20) 15


MATEMTICA D

Unidade 1
1) 12,50
2) 80
3) a)18 b) 6 c) 60
d) 0,04 e) 0,04 f) 1/5
4) d
5) R$ 45,20
6) 252
7) 8 dias
8) a) 20 b) 2
c) 240 d) 0,6
e) 0,06 f) 0,0025 g)
70%
9) e
10) 08
11) b
12) a
13) a
14) a
15) a
16) 44
17) 40
18) d
19) 02

Unidade 2
1) a) 90 b) 11
2) a) n=9 b) n=2
3) 12
4) 175.760 000
5) 840
6) 15
7) a) 9 b) 3 c) 8
8) 05
9) c
10) d
11) a
12) 04
13) 02
14) d
15) 08
16) 12
17) e
18) d
19) 60
20) c

Unidade 3
1) a)24 b)720 c)60
d)151.200
2) 240
3) 10
4) 56
5) 24
6) 60
7) 24
8) 210
9) c
10) a
11) 28
12) e
13) a
14) 30
15) 35
16) 12
17) d
18) a

Unidade 4
1) 145
2) 10
3) a)128 b)1024 c)219
4) x=5 ou x=10
5) 19
6) b
7) 13
8) 32
9) 04
10) c
11) c
12) C
13) b
14) d
15) c

Unidade 5
1) x
4
+8x
3
+24x
2
+32x+16
2) 240x


3) T5=81
4) 1
5) 280
6) 01
7) 37
8) a
9) e
10) a
11) e
12) 01
13) c
14) a

Unidade 6
1) 330
2) A=2
3) P(3)=0
4) a) 5 b) 0 c) 38
5) 4
6) 66
Please purchase PDF Split-Merge on www.verypdf.com to remove this watermark.
Incluso para a Vida Gabaritos

Pr-Vestibular da UFSC

4
7) 66
8) d
9) 00
10) d
11) 66
12) d
13) a

Unidade 7
1) e
2) A=11
3) a
4) 23
5) 35
6) b
7) 11
8) 07
9) 04
10) b
11) e
12) e
13) d

Unidade 8
1) a
2) {2,3,7}
3) x=2
4) d
5) d
6) 08
7) c
8) c
9) c
10) 00
11) b
12) e
13) 03

Unidade 9
1) 1)




2) 28
3) 12
4)
2 1 1 1
2 3 2 4 5
5 2 4 1
3 4 3 2 5
8 5 9 9
a b c d ) ) ) )
| | | |
| | | | | |
| | | |
\ . \ . | |
\ . \ .

5) 34
6) 6 e 2
7) 36
8)a)
2 2
2 2
X
| |
=
|

\ .

b)
3 0
4 1
X
| |
=
|
\ .

3 3
5 1
Y
| |
=
|

\ .

9) 12
10) e
11) 12
12) d
13) e
14) b
15) b
16) 9 17
10 12

(

17) a
18) b

Unidade 10
1) a)



b)



c)



d)



e)



f)



2) 92
3) a) 7 b) 14
4) -64
5) 05
6)
|
|
.
|

\
|
0 0
0 0

7) 01
8) 56
19) a) 14 b) 11 c) 15
10) 03
11) 08
12) 05
13) d
14) 01
15) 40
16) 32
17) 80
18)
|
|
.
|

\
|
0 0
0 0

19) 56
20) 19
21) e
22) b

Unidade 11
1) -48
2) 48
3) a)


b)



4) x=6
5) 12
6) 6
7) c
8) 121
9) 36
10) a
11) b
12) a
13) d
14) e
15) d
16) d
17) a
18) a
19) 5 e 1
20) d
21) 05
22) c
23) 05
24)

Unidade 12
1) a) x=2 , y= -1
b) S.P.I
c)


2) a
3) b
4) 03
5) b
6) a
7) 02
8) 02
9) 26
10) a
11) a
12) 09
13) 04
14) 09
15) 18
16) 13
17) c

FSICA A

Unidade 1
1) e
2) a
3) c
4) e
5) b
6) 16
7) c
8) a
9) 28
10) 58
Unidade 2
1) d
2) a
3) c
4) 100 m
5) 5 m/s
6) c
7) a
8) c
9) c
10) 60
Unidade 3
1) a) 2,5m/s b) 12,5m
2) c
3) d
4) d
5) b
6) c
7) d
8) b
9) b
10) 52
Unidade 4
1) c
2) a
3) b
4) e
5) b
6) b
7) d
8) a) 60 m b) 4 m/s
9) c
10) 25

Unidade 5
1) a) 45m b) 30m/s
2) c
3) d
4) c
5) e
6) d
7) a) 2 s b) 30 m
8) d
9) a
10) 46

Unidade 6
1) a) 7s b) 400 m/s
2) 22
3) d
4) e
5) e
6) c
7) c
Unidade 7
1) 39
2) a) 2Hz b) 2t m/s
3) c
4) b
5) e
6) e
7) b
8) a

Unidade 8
1) c
2) 30
3)
4) b
5) e
6) 5 m/s
7) e
8) 10
9) d
10) 30

Unidade 9
1)
2) a
3) 50
4) e
5) 1,1
6) c
7) c
8) 59
9) 46
10) 56

Unidade 10
1) tg u = 0,4
2) a) 24N b) 54N
3) b
4) c
5) 10\2 m/s
6) c
7) c
8) 5m
9) 30
10) 45

Unidade 11
1) e
2) c
3) d
4) c
5) e
6) e
7) c
8) d
9) e
10) 50
11) 09
12) 50



Please purchase PDF Split-Merge on www.verypdf.com to remove this watermark.
Incluso para a Vida Gabaritos

Pr-Vestibular da UFSC

5
Unidade 12
1) 2400J
2) b
3) c
4) b
5) e
6) 06
7) 13
8) 35
9) 60
10) 51

Unidade 13 e 14
1) e
2) c
3) b
4) c
5) b
6) a
7) d
8) b
9) 02
10) e
11) 60
12) 15
13) 94
14) 27
15) 09

Unidade 15
1) a
2) c
3) e
4) d
5) e
6) d
7) b
8) 2 6 h
9) 11
10) 86

Unidade 16
1) d
2) b
3) b
4) a) duas
b) mar alta
c)

5) e
6) b
7) e
8) b
9) d
10) c
11) b
12) 29
13) 03

Unidade 17 e 18
1) d
2) b
3) a) 1,5 kg
b) (1) zero; (2) 6 N.m
4) b
5) 8 kg
6) c
7) a
8) b
9) c
10) e
11) c
12) e
13) b
14) d
15) 2,1 kgf vertical para
baixo
16) 49
17) 28

Unidade 19 e 20
1) d
2) c
3) 1,5.10
5
N/m
2
/Zero
4) b
5) e
6) d
7) a) 4.10
3
N/m
2

b) 7.10
3
N/m
2
8) d
9) c
10) 04
11) 2,5.10
5
N/m
2

12) 15

Unidade 21
1) b
2) 07
3) 1000
4) a
5) a
6) b
7) c
8) d
9) b
10) 54
11) 30
12) 21

FSICA B

Unidade 1
1) D
2) A
3) E
4) C
5) 34
6) A
7) D
8) C
9) E
10) E
11) 10
12) A
13) C
14) A
15) A
16) D
17) C
18) D

Unidade 2
1) B
2) A
3) C
4) E
5)
6) B
7) D
8) B
9) E
10) B
11) 24
12) B
13) 9
14) A
15) B
16) A

Unidade 3
1) D
2) D
3) B
4) D
5) A
6) C
7) B
8) E
9) 50
10) E
11) 27
12) 24
13) C
14) E
15) C
16) D
17) C
18) B
19) D
20) 20
21) D
22) B
23) 10

Unidade 4
1) D
2) A
3) D
4) 15
5) B
6) B
7) D
8) D
9) D
10) A
11) C
12) E

Unidade 5
1) D
2) A
3) C
4) 20L
5) 11
6) C
7) D
8) C
9) E
10) B
11) B
12) A
13) 2/27
14) B
15) E
16) 58
17) A
18) D

Unidade 6
1) C
2) E
3) D
4) B
5) D
6) C
7) A
8) 150J
9) A
10) D
11) 40
12) D
13) A
14) 14
15) 26
16) D
17) D
18) E
19) D

Unidade 7
1) C
2) C
3) D
4) C
5) C
6) C
7) B
8) A
9) D
10) 60
11) A
12) D
13) A
14) E
15) C
16) C
17) A
18) A
19) B
20) A
21) E
22) C
23) D
24) E
25) B
26) 36
27) 16
28) B
29) E
30) B

Unidade 9
1) D
2) E
3) C
4) D
5) F-V-F-V-F
6) 19
7) C
8) A
9) D
10) B
11) A
12) A
13) D
14) 48
15) A

Unidade 10
1) D
2) E
3) A
4) A
5) C
6) D
7) 74
8) A
9) D
10) 30
11) F-F-V-V-F
12) A
13) D
14) D

Unidade 11
1) A
2) A
3) C
4) D
5) C
6) D
Please purchase PDF Split-Merge on www.verypdf.com to remove this watermark.
Incluso para a Vida Gabaritos

Pr-Vestibular da UFSC

6
7) D
8) E
9) A
10) C
11) C

Unidade 12
1) A
2) A
3) 50
4) 28
5) 69
6) D
7) D
8) C
9) D
10) E
11) C
12) B
13) B


FSICA C

Unidade 1
1) a
2) 0,3 N
3) a
4) b
5) 32
6) d
7) d
8) e
9) 60
10) c

Unidade 2
1) b
2) 11
3) a
4) a
5) c
6) c
7) 18
8) 31
9) 24

Unidade 3
1) e
2) d
3) c
4) 37
5) 8cm
6) 2.10
-7

7) 10
8) 5.10
-6

9) 4cm
10) 3A
11) 63
12) 19

Unidade 4
1) b
2) (F) (F) (F) (V) (V)
3) 40
4) c
5) a
6) d
7) 26
8) 18
9) 17
10) 19

Unidade 5
1) d
2) a
3) a
4) a
5) c
6) c
7) b
8) Pu = 54W
9) e
10) d





Unidade 6
1) 24
2) a) 30v b) 60v c) 20v
d) n=67%
3) a
4) c
5) d
6) 27
7) d
8) e

Unidade 7
1) c
2) c
3) a
4) c
5) e
6) d
7) e
8) e
9) e
10) b
11) c

Unidade 8
1) b
2) c
3) d
4) e
5) c
6) d
7) c
8) e
9) d
10) 08

Unidade 9
1) 4.10
-6
T
2) 86
3) d
4) d
5) a) 6,3x10
-5
T, b) 1,6x10
-5
T,
c) 4,7x10
-5

6) a) zero, b) 4x10
-5
t
7) c
8) ant-hor., i/2
9) (2t)
1/2
x10
-7
T
10) a
11) c

Unidade 11
1) 0,1 N
2) 0,05 N
3) e
4) d
5) 68
6) 19
7) e
8) a
9) c
10) a


Unidade 12
1) 0,05 V
2) 40 espiras
3) c
4) e
5) 86
6) d
7) 24
8) 94
9) 28
10) 22
11) 41

QUMICA A

Unidade 1
1) 04
2) d
3) e
4) e
5) 08
6) b
7) 53
8) 39
9) 16
10) 10
11) 57

Unidade 2
1) c
2) c
3) b
4) d
5) 02
6) 08
7) 09
8) 82
9) 03

Unidade 3
1) a
2) 05
3) 62

Unidade 4
1) e
2) e
3) b
4) c
5) d
6) a
7) 19
8) 08
9) 58
10) 47
Unidade 5
1) d
2) c
3) b
4) b
5) 54
6) c
7) 69
8) 04
9) 45
10) 15
11) 40

Unidade 6
1) 29
2) c
3) 19
4) 16



Unidade 7
1)10
2) d
3) b
4) 82
5) a
6) 23
7) 43
8) 12
9) 63
10) 61

Unidade 8
1) d
2) 07
3) e
4) c
5) b
6) 92

Unidade 9
1) 61
2) b
3) 25
4) 26
5) d
6) e
7) d
8) c

Unidade 10
1) 05
2) 24
3) b
4) d
5) e
6) a
7) b

Unidade 11
1) 27
2) b
3) c
4) d

Unidade 12
1) d
2) c
3) 08
4) 31
5) b

Unidade 13
1) d
2) d
3) c
4) e
5) c

Unidade 14
1) d
2) a
3) e
4) 09
5) c
6) 38

Unidade 15
1) a
2) d

Unidade 16
1) b
2) 09
3) c
4) b
8
36
Please purchase PDF Split-Merge on www.verypdf.com to remove this watermark.
Incluso para a Vida Gabaritos

Pr-Vestibular da UFSC

7
Unidade 17
1) b
2) b
3) d
4) e
5) e
6) e

Unidade 18
1) b
2) d
3) e
4) 47
5) b
6) c
7) 42
8) 40
9) 36
10) 38
11) 37


Unidade 19
1) 14
2) e

Unidade 20
1) e
2) 13
3) d
4) d
5) d
6) a
7) 09
8) 59

Unidade 21
1) 23
2) b
3) e
4) 47
5) 14

Unidade 22
1) 06
2) 09
3) 11
4) 09
5) 10
6) 24
7) 13
8) 05
9) 05
10) 08
11) 20
12) 06
13) 15
14) 13
15) 12
16) 18
17) 11
18) 09
19) 15
20) 43

Unidade 23
1) 02
2) 147
3) 448
4) 30
5) 81
6) d
7) 20
8) 686
9) b
10) 43

Unidade 24
1) 07
2) e
3) 18
4) 6
5) 75
6) 60
7) c
8) 42
9) 84
10) 30
11) c
12) 01
13) c
14) 15

QUMICA B

Unidade 1
1) 1

Unidade 2
1) 20
2) 1200
3) 368
4) 200
5) 7,5
6) 80
7) e
8) d
9) a
10) 0,15
11) 0,8
12) 0,25
13) d
14) e
15) 04
16) a
17) a
18) d
19) a
20) a

Unidade 3
1) b
2) b
3) c
4) d
5) a
6) 0,3
7) 0,2

Unidade 4
1) a
2) b
3) d
4) a
5) a
6) b
7) b
8) c
9) d
10) 12
11) 01

Unidade 5
1) 1

Unidade 6
1) b
2) 21
3) e
4) e
5) 25
6) b
7) a
8) 24
9) d
10) c
11) b
12) c
13) b
14) d
15) a

Unidade 7
1) 1

Unidade 8
1) 03
2) 30
3) b
4) 41
5) b
6) 35
7) b
8) d
9) c
10) e
11) e
12) d

Unidade 9
1) c
2) c
3) c
4) a
5) a
6) 09
7) 04
8) 41
9) b
10) a
11) e


Unidade 10
1) b
2) c
3) c
4) b
5) a
6) e
7) d
8) 29
9) b
10) e
11) 11,3
12) 1,7
13) b

Unidade 11
1)d
2) c
3) c
4) e
5) e
6) 29

QUMICA C

Unidade 1
1) 43
2) 33

Unidade 2
1) 02
2) c
3) e
4) e
5) a
6) a
7) c
8) e
9) c
10) d
11) d
12) b
13) b
14) a

Unidade 3
1) 11
2) 33

Unidade 4
1) 21
2) 63
4) e
5) c
6) a
4) a
5) c
6) c
7) d
8) e
9) d
10) a
11) b
12) d
13) e
14) a
15) 21
16) 42
17) 12
18) e
19) b
20) a
21) e
22) b

Unidade 5
1) 15
2) 63

Unidade 7
1) 92
2) c
3) d
4) e
5) c
6) b
7) 98
8) e
9) a
10) a
11) b
12) e
13) a
14) c
15) a
16)09

Unidade 9
1) b
2) 30
3) b
4) d
5) c
6) b
7) d
8) c
9) a
10) b
11) c
12) e
13) c
Please purchase PDF Split-Merge on www.verypdf.com to remove this watermark.
Incluso para a Vida Gabaritos

Pr-Vestibular da UFSC

8
14) 62
15) b
16) d
17) c
18) a
19) a
20) d
21) a
22) c

BIOLOGIA A

Unidade 1
1) 29
2) a
3) a
4) 29
5) b

Unidade 2
1) 35
2) a
3) b
4) d
5) b

Unidade 3
1) 31
2) 92
3) d
4) 14
5) 91

Unidade 4
1) 69
2) 89
3) 01
4) 20
5) 34

Unidade 5
1) 89
2) 87
3) c
4) a
5) c

Unidade 6
1) a
2) b
3) d
4) b
5) 23

Unidade 7
1) c
2) c
3) a
4) e
5) b

Unidade 8
1) e
2) a
3) 43
4) c
5) b

Unidade 9
1) a
2) d
3) c
4) c
5) a

Unidade 10
1) 47
2) e
3) 12
4) 27
5) 06

Unidade 11
1) 26
2) d
3) c
4) c
5) b

Unidade 12
1) 13
2) d
3) a
4) a
5) d

Unidade 13
1) 65
2) 38
3) d
4) b
5) a

Unidade 14
1) a
2) e
3) d
4) e
5) b

Unidade 15
1) 47
2) a
3) b
4) c
5) d

Unidade 16
1) e
2) a
3) c
4) b
5) a

Unidade 17
1) 60
2) 19
3) 21
4) 15
5) a

Unidade 18
1) d
2) b
3) a
4) b
5) b

Unidade 19
1) 76
2) 12
3) b
4) d
5) b

BIOLOGIA B

Unidade 1
1)15
2) e
3) c

Unidade 4
1) 23
2) 85
3) 54
4) 23
5) 60
6) 75

Unidade 5
1) 61
2) 94
3) 19
4) 10
5) a
6) d
7) a

Unidade 7
1) 10
2) 31
3) 28
4) 46
5) 81
6) 45
7) 35
Unidade 8
1) 26
2) 44
3) 15
4) 26
5) 53
6) 56
7) 63

Unidade 9
1) 31
2) b
3) a
4) 83
5) 55
6) 75

Unidades 10
1) 53
2) 27

Unidade11
1) 24
2) 49
3) 18
4) 46
5) 47

Unidade 12
1) 61
2) 28
3) 39
4) b
5) 10
6) 15
7) 22
8) 07
9) 05

Unidade 13
1) 62
2) 23
3) e
4) b

Unidade 14
1) 93
2) b
3) b

Unidade 15
1) 31
2) 22
3) 42
4) a
5) 63
6) 30
7) b
8) d

Unidade 16
1) 30
2) 13
3) 26
4) 51
5) b

Unidade 17
1) 57
2) a

Unidade 18
1) 11
2) b
3) 52
4) 65

Unidades 19
1) 94
2) 22
3) 21
4) 97
5) b

Unidade 20
1) 60
2) 38

Unidade 21
1) 42
2) 55
3) 14
4) c
5) b
6) e
7) b
8) b
9) c
10) e

Unidade 22
1) 15
2) e
3) 20
4) c
5) c

Unidade 24
1) 15
2) 03
3) 06
4) 01
5) 07
6) 24

HISTRIA A

Unidade 1
1) 12
2) c
3) 12
4) 28
5) e
6) b
7) 06
Please purchase PDF Split-Merge on www.verypdf.com to remove this watermark.
Incluso para a Vida Gabaritos

Pr-Vestibular da UFSC

9
8) 21

Unidade 2
1) 22
2) 27
3) a
4) 15
5) a
6) 07
7) 23
8) e
10) b
11) a

Unidade 3
1) 20
2) 15
3) 05
4) 05
5) 20

Unidade 4
1) 59
2) 15
3) 20
4) e
5) 22

Unidade 5
1) 30
2) 30
3) d
4) b
5) e

Unidade 6
1) 26
2) 11
3) 62
4) a
5) d

Unidade 7
1) 79
2) 27
3) a
4) a
5) a

Unidade 8
1) 30
2) 23
3) e
4) c
5) c

Unidade 9
1) 15
2) 30
3) c
4) d
5) a

Unidade 10
1) 25
2) 37
3) e
4) d
5) d
6) d

Unidade 11
1) 15
2) 09
3) a
4) b
5) c

Unidade 12
1) 36
2) 03
3) d
4) c
5) a

HISTRIA B

Unidade 1
01- a
02- 14
03- a
04- e
05- a
06- a

Unidade 2
01- 06
02- 14
03- 16
04- c
05- b
06- d

Unidade 3
1) 06
2) 15
3) b
4) 09
5) d
6) c

Unidade 4
01- 01
02- 46
03- 62
04- d
05- b
06- c


Unidade 5
1) 03
2) 02
3) d
4) d
5) b
6) c
Unidade 4
1) 07
2) 03
3) 10
4) 09
5) 02
6) 14


Unidade 5
1) 03
2) 04
3) 03
4) d
5) d
6) e

Unidade 6
1) 11
2) 08
3) a
4) b
5) a
6) e


Unidade 7
1) 05
2) 02
3) c
4) 01
5) d
6) e

Unidade 8
1)10
2)03
3) 07
4) 03
5) d
6) e

Unidade 9:
1) 01
2) 04
3) 16
4) 16
5) d
6) a

Unidade 10:
1) 04
2) 08
3) 21
4) 06
5) 21
6) 26

Unidade 11:
1) 06
2) 02
3) b
4) e
5) d
6) c

Unidade 12:
1) 11
2) 04
3) a
4) b
5) a
6) c

GEOGRAFIA A

Unidade 1
1) A
2) 28
3) C
4) B
5) B
6) B

Unidade 2
1) 30
2) 29
3) 24
4) D

Unidade 3
1) 14
2) B
3) D
4) D

Unidade 4
1) B
2) 49
3) A
4) B
5) B

Unidade 5
1) 19
2)
3) 30
4) 24

Unidade 6
1) 07
2) D
3) B
4) A

Unidade 7
1) D
2) 24
3) 05
4) D

Unidade 8
1) 28
2) E
3) D
4) A
5) B
6) D

Unidade 9
1) A
2) C
3) E
4) A

Unidade 10
1) E
2) 31
3) B
4) D
5) C

Unidade 11
1) 02
2) D
3) A
4) D
5) 43
6) E
7) C

Unidade 12
1) 01
2) B
3) 06
4) C

Unidade 13
1) C
2) D
3) E
4) C
5) A
6) C
7) C
8) C
9) D
10) E
11)C

GEOGRAFIA B

Unidade 1
1) E
2) Quanto segurana
nacional temos trfico de
drogas e contrabando; em
Please purchase PDF Split-Merge on www.verypdf.com to remove this watermark.
Incluso para a Vida Gabaritos

Pr-Vestibular da UFSC

10
termos socioambientais,
temos expanso agrcola e
desmatamento,
principalmente em Rondnia
e no Acre.
3) a) 1 __ 150.000.000
16 __ x
x = 16 . 150.000.000 : 1
x = 2.400 Km
As abelhas no atingem a
Patagnia e a Cordilheira
dos Andes devido a baixas
temperaturas e pouca
umidade.
b) Dentre os vrios aspectos
positivos, podemos destacar:
pode ser feita em pequenas
reas; ajuda na
complementao da renda
familiar; tem baixos custos,
possibilita o cooperativismo.
4) D
5) C
6) D
7) 19
8) 26

Unidade 2
1) D
2) C
3) E
4) A
5) 23
6) 38

Unidade 3
1)A
2) A
3) E
4) D
5) B
6) A
7) D
8) 18

Unidades 4 e 5
1) B
2) B
3) A
4) C
5) D
6) E
7) 12
8) 13

Unidade 6
1) A
2) a) eroso,assoreamento.
b) gua potvel
abastecimento de
populaes, dessedentao
animal e de cidades, pesca,
produo de energia,
transporte, recreao.
3) a) - Bacia do Paran:
Potencial elevado que
atualmente possui elevado
aproveitamento,
considerando o nmero de
cidades banhadas por esta
bacia.
- Bacia do
Amazonas: Potencial
elevado que atualmente
possui baixo grau de
aproveitamento,
considerando que a rea
possui baixa densidade
demogrfica.
b) - Poluio: Altos ndices
de poluio decorrentes da
descarga direta de esgotos
urbanos;
- Poluio decorrente da
atividade de agropecuria
(agrotxicos, irrigao);
- Elevado ndice de
represamento para a
construo de usinas
hidreltricas:
desconfigurao das
caractersticas naturais.
- Alterao do equilbrio
ecolgico dos rios pela
construo de hidrovias.
4) C
5) B
6) A
7) a) O rio So Francisco
desloca-se em boa parte de
seu percurso e nas reas da
transposio planejada, em
reas de bioma de Caatinga.
Trata-se de um bioma
caracterizado pelo clima
semirido com baixa
pluviosidade e vegetao de
arbustos de porte mdio e
espcies xerfitas. Os solos
so rasos e existe hidrografia
intermitente nas reas
apontadas.
b) Impactos positivos:
aumento da oferta de gua
para irrigao e
dessedentao animal; mais
gua para o abastecimento
urbano.
Impactos negativos: eventual
alterao de flora e fauna da
regio atingida; possvel
aumento da concentrao
fundiria; possibilidade de
salinizao dos solos;
diminuio da vazo do So
Francisco a jusante da rea
de captao.
8) D

Unidade 7 e 8
1) D
2) A
3) C
4) D
5) A
6) A
7) E
8) D

Unidade 9
1) A
2) C
3) C
4) B
5) C
6) D

Unidade 10
1) a) A necessidade de
elevao da renda familiar e
a ampliao do nmero de
mulheres com salrios
superiores ao de seus
companheiros, so aspectos
que justificam a crescente
participao da mulher como
chefes de famlia, a partir de
uma maior insero feminina
no mercado de trabalho.
b) A maior participao da
mulher no mercado de
trabalho do Brasil provocou
uma ampliao significativa
de seu exrcito de reserva,
ou seja, da massa de pessoas
disponveis para o trabalho.
Esse fato acaba gerando uma
maior oferta de mo-de-obra
criando uma tendncia de
reduo dos salrios e
aumento das taxas de
desemprego.
2) A
3) D
4) 74

Gabarito 11 e 12
1) A
2) B
3) A
4) D
5) D
6) E
7) D
8) D
9) C
10) 43

Unidade 13 e 14
1) Nos anos 1950 o Estado
tinha um papel relevante na
industrializao com
fomentos ao setor. No final
da dcada, perodo JK, o
capital privado internacional
comea a entrar no pas e
ocorre a expanso dos
setores de bens de consumo.
O mercado consumidor
restrito aos grandes centros
urbanos e s camadas mais
abastadas. No final da
dcada de 1990, a economia
est flexibilizada ao sabor da
globalizao econmica de
modelo neoliberal com uma
produo industrial
diversificada e com um
mercado consumidor maior e
com mais acesso ao crdito.
2) A
3) a) A regio Nordeste
passou por um processo de
industrializao mais recente
comparativamente ao
Sudeste. Devido a suas
condies locacionais,
infraestruturais, dos
investimentos e da
qualificao de sua mo de
obra, a regio Nordeste
desenvolveu mais o setor de
bens de consumo no
durveis como os setores
alimentcio, caladista, txtil
e construo civil. So
setores relativamente mais
simples e suas cadeias
produtivas tem maior
capacidade de contratar mo
de obra, com vantagens
comparativas para uma
regio carente de atividades.
b) A concentrao histrica
de capital na regio Sudeste,
foi geradora de novas
necessidades de consumo e
diversificao.Isso acaba
transformando a regio na
maior concentrao
industrial do Brasil,
caracterizado por setores os
mais variados, com unidades
de produo que vo desde
as mais simples, bens de
consumo no durveis, como
alimentcia e construo
civil, at as mais complexas
e desenvolvidas como
informtica e aviao,
passando pela indstria
pesada como siderrgicas. A
evoluo tecnolgica e
comercial criou novas
demandas e mudanas
locacionais, favorecendo a
descentralizao da
produo e o surgimento de
tecnopolos que concentram
reas de formao de mo de
obra, pesquisa e produo e
com nveis cada vez maiores
de automao que
modificam a estrutura
funcional, demandando cada
vez menos trabalhadores.
So fatos que exigem novas
poltica pblicas de
qualificao e localizao da
mo de obra com nfase a
novas possibilidades como
servios e terceiro setor.
4) B
5) A
6) E
7) B

Unidade 15 e 16
1) C
2) D
3) Concentrao fundiria ou
concentrao de terras
Uma das causas:
modernizao da
agricultura.
o sistema de sesmarias
adotado na poca colonial.
legislao fundiria que
restringe o acesso terra ao
pequeno agricultor.
Uma das consequncias:
xodo rural.
reduo dos cultivos de
subsistncia.
empobrecimento do
morador do campo.
aumento das desigualdades
de renda no pas.
4) B
5) A
6) C
7) B
8) B
9) A
Please purchase PDF Split-Merge on www.verypdf.com to remove this watermark.
Incluso para a Vida Gabaritos

Pr-Vestibular da UFSC

11
10) D


GRAMTICA

Unidade 1
Flexo dos Substantivos
Gnero: o alvar, a alface, o
cl, o champanha, a libido, a
omelete, a omoplata, o
apendicite, a cal, a
comicho, a dinamite, o d,
a d, a sentinela
Nmero: caroos, poos,
cidados, fornos, socorros,
acares, gizes, carteres,
meles/mis, projteis,
trofus, degraus, escrives,
capites, sacristes, os trax,
papis, alcois, abdomens,
hfens, barezinhos,
lugarezinhos - aguardentes,
os bl-bl-bls, pes-de-l,
os leva-e-traz, os corre-
corres, quintas-feiras,
bananas-ma, peixes-
espada, sambas-enredo,
abaixo-assinados, amores-
perfeitos, beija-flores, bem-
amados, guarda-roupas,
guardas-civis, guarda-sis
Locues adjetivas:
abdominal, apcola, discente,
docente, asinino, passional,
episcopal, lacustre, lcteo,
ocular, capilar, pluvial,
fluvial, digital, galinceo,
blico, insular, senil, renal,
onrico

UNIDADE 2
Exerccios de Sala
1) antropologia; biologia;
etimologia; fonologia;
morfologia; patologia;
zoologia
2) D E A J I G H
B F C
3) E
Tarefa Mnima
1) E
2) E
3) 02 + 04 + 08 + 16
Tarefa Complementar
1) 04
2) D
3) D
4) A

Unidade 3
Exerccios de Sala
1) 01 + 04 + 16 + 32
2) 01 + 02 + 04
3) D
Tarefa Mnima
1) 01 + 04
2) 02 + 08
3) 08 + 16 + 32
Tarefa Complementar
1) A
2) C
3) E

Unidade 4
Exerccios de Sala
1) C
2) 04
3) 02 + 08 + 32
Tarefa Mnima
1) E
2) 02 + 04 + 16
3) D
Tarefa Complementar
1) A
2) 04 + 08
3) D
4) B

Unidade 5
Exerccios de Sala
1 a 4 - Abertas
Tarefa Mnima
1) 02
2 a 5 - Abertas
Tarefa Complementar
1) 08 + 64
2) Aberta
3) 02 + 16 + 32
4) A
5) D

Unidade 6
Exerccios de Sala
1) 3-4-1-5-1-5-4-5-2-1-5-3
2) C
3 e 4 - Abertas
Tarefa Mnima
1) C
2) B
3) B
4) 01 + 32
5) 01 + 04 + 16
Tarefa Complementar
1) 01 + 02 + 32
2) 04 + 08
3) 04 + 16 + 64
4) 01

Unidade 7
Exerccios de Sala
1) adjunto adverbial de
tempo; sujeito simples;
verbo intransitivo;
predicativo; adjunto
adnominal
2) B
3) 7-4-3-10-5-8-6-4-7-3-4-3-
2-4-3-6-9-1
4 a 9 - Aberta
Tarefa Mnima
1) 01 + 08
2) 02 + 04 + 08
3) 02 + 04 + 32
4) 01 + 32
5) B
6) D
Tarefa Complementar
1) 02
2) 01 + 16 + 32
3) C
4) 01 + 16 + 32
5) 01 + 08 + 16

Unidade 8
Exerccios de Sala
1) Aberta
Tarefa Mnima
1) E
2) C
3) C
Tarefa Complementar
1) D
2) C
3) D

Unidade 9
Exerccios de Sala
1) Aberta
2) 16 + 32
3) 02
Tarefa Mnima
1) 01 + 04 + 08
2) 01 + 04
3) Aberta
Tarefa Complementar
1) E
2) A
3) E
4) A
5) 01 + 16

Unidade 10
Exerccios de Sala
1) Aberta
2) admite, precipitemos,
respondais, produzamos,
exijais, pe, vem, cumpri,
intervenhais
3) A
4) 04
5) 01 + 16
Tarefa Mnima
1) B
2) 04
3) 01 + 02
4) D
Tarefa Complementar
1) D
2) D
3) C
4) C
5) 01 + 04 + 08
6) 16

Unidade 11
Exerccios de Sala
1) aspire a, aspiram ao,
aspiro , assisti ao, assisti ,
obedece s, vo , visou o,
visou o, assisto em
Tarefa Mnima
1) 01
2) C
3) C
Tarefa Complementar
1) 04 + 08 + 32
2) 01 + 02 + 32
3) 01 + 16 + 64
4) 04 + 16 + 32


Unidade 12
Exerccios de Sala
1 e 2 Abertas
Tarefa Mnima
1) C
2) E
3) 01 + 02 + 04 + 08
4) A
5) E
6) C
Tarefa Complementar
1) C
2) D
3) D
4) 01 + 04 + 08 + 16
5) D
6) Aberta

Unidade 13
Exerccios de Sala
1) Pronomes pessoais do
caso reto no atuam como
complemento verbal direto.
2) Eu queria encontr-la
ainda hoje.
3) Conhecemo-nos naquela
festa.
4) No, posto que lhes
um complemento indireto e
convidar um VTD.
5) Vou convid-los para
minha formatura.
Tarefa Mnima
1) B
2) 04 + 08 + 16
3) B
Tarefa Complementar
1) 01 + 32
2) A
3) E
4) 01 + 16 + 32

Unidade 14
Exerccios de Sala
1 a 5 Abertas
Tarefa Mnima
1) C
2) E
3 a 5 Abertas
Tarefa Complementar
1) 08
2) C
3) B
4) A

Unidade 15
Exerccios de Sala
1) a. alternativa; b.
conclusiva; c. adversativa; d.
adversativa; e. explicativa; f.
conclusiva; g. adversativa; h.
aditiva alternativa; i.
conclusiva
Tarefa Mnima
1) A
2) D
3) B
4) D
5) A
6) 02 + 04 + 08 + 16 + 32
Tarefa Complementar
1) 02
2) 01 + 02 + 04
3) B
4) C
5) 02 + 08 + 16
6) A
7) B

Unidade 16
Exerccios de Sala
1) a. subjetiva; b. subjetiva;
c. objetiva direta; d.
predicativa; e. objetiva
direta; f. apositiva
2) a. explicativa; b.
restritiva; c. explicativa; d.
restritiva; e. explicaiva
3) Aberta
Tarefa Mnima
1) A
2) 01 + 04 + 16 + 32
Tarefa Complementar
Please purchase PDF Split-Merge on www.verypdf.com to remove this watermark.
Incluso para a Vida Gabaritos

Pr-Vestibular da UFSC

12
1) 16
2) E

Unidade 17
Exerccios de Sala
1) a. consecutiva; b.
concessiva; c. proporcional;
d. final; e. causal; f.
comparativa; g.
conformativa; h. temporal; i.
condicional
Tarefa Mnima
1) C
2) B
3) B
Tarefa Complementar
1) B
2) D
3) C
4) C

Unidade 18
Exerccios de Sala
1) Aberta
Tarefa Mnima
1) 1a; 2b; 3e; 4a; 5d
2) 3 6 1 5 4 2
3) 4 6 3 7 1 5 2
Tarefa Complementar
1) 2 1 4 3
2) 4 1 2 3
3) 3 1 3 4 4 2 1
3 4

Unidade 19
Exerccios de Sala
1) a. conjuno integrante; b.
pronome reflexivo; c.
pronome reflexivo; d.
partcula apassivadora; e.
pronome reflexivo; f.
conjuno subordinativa
(condicional); g. partcula
expletiva; h. pronome
reflexivo
2) a. conjuno
subordinativa (consecutiva);
b. pronome relativo; c.
interjeio; d. conjuno
integrante; e. preposio; f.
substantivo
Tarefa Mnima
1) D
2) D
3) C
4) D
5) E
Tarefa Complementar
1) D
2) D
3) B

Unidade 20
Exerccios de Sala
01) a. inadequado; b. m: c.
belas; d. pretas(os); e.
maduras; f. alta(os); g.
escuras(os); h. clara; i. nos
mercados; j. no mercado; k.
alerta; l. anexas; m. anexo: n.
bastantes; o. bom; p.
necessrio; q. necessria; r.
meia; s. tal quais; t. meio; u.
bastantes
Tarefa Mnima
1) B
2) 01 + 02 + 04 + 08 + 16
3) 01 + 02 + 04 + 08
Tarefa Complementar
1) 01 + 02 + 04 + 08
2) E
3) 02 + 08 + 32

LITERATURA

1) 0
2) 7
3) 20
4) d
5) a
6) a
7) c
8) a
9) e
10) 21
11) e
12) c
13) 37
14) e
15) 25
16) b
17) e
18) 27
19) c
20) e
21) c
22) a
23) d
24) b
25) e
26) 22
27) c
28) e
29) 43
30) 42
31) 27
32) 13
33) c
34) b
35) c
36) a
37) b
38) c
39) b
40) e
41) 93
42) a
43) d
44) c
45) 30
46) d
47) 20
48) d
49) c
50) e
51) e
52) e
53) a
54) 31
55) 21
56) e
57) 26
58) c
59) c
60) 09
61) a
62) c
63) b
64) a
65) 21
66) b
67) b
68) b
69) D

INGLS

Unidade 1
a) d; b) a; c) b
1) a) Does; b) Do; c) Does;
d) Do
2) b
3) a
4) Written on the board
5) a) plans; b)
Knows/wants; c)
spends; d) drink
6) d
7) b
8) a
9) c
10) a
11) d
12) d
13) c

Unidade 2
1) c
2) b
3) b
4) b
5) 29
6) 27
7) b
8) c
9) c
10) c
11) b

Unidade 3
1) d
2) d
3) c
4) b
5) on the board
6) b
7) 17
8) 20

Unidade 4
1) e
2) a
3) d
4) d
5) e
6) e
7) d

Unidade 5
1) on the board
2) 08
3) 37
4) 20
5) 15
6) 14

Unidade 6
1) On the board
2) C

Unidade 7
1) On the board
2) On the board
3) On the board
4) Since; for; since; for;
since; for

Unidade 8
1) a) much; b) many
2) Many; much; many;
much; much
3) b
4) d
5) d
6) d
7) a
8) d
9) b
10) c
11) c
12) c
13) a
14) c


Unidade 9
1) b
2) d
3) c
4) b
5) b
6) 38
7) 11
8) 25
9) d
10) c
11) 25
12) 10
13) c
14) c
15) d
16) e
17) d

Unidade 10
1) 44
2) 37
3) 52
4) 89
5) 10

Unidade 11
1) 49
2) 29
3) 38
4) 11
5) 20
6) 37
7) 17
8) 43
9) 21
10) 42

Unidade 12
1) 14
2) 01
3) 15
4) 14
5) D
6) A
7) C
8) A
9) C
10) a) they; b) it; c) he/she;
d) we; e) they; f) you; g)
she
11) a) him; b) them; c) they;
d) us; e)it; f) she; g)me;
h) her; i)he/it
Please purchase PDF Split-Merge on www.verypdf.com to remove this watermark.
Incluso para a Vida Gabaritos

Pr-Vestibular da UFSC

13
12) a) their/ours; b) yours; c)
my/mine; d) yours/mine;
e) my/hers
13) a) himself; b) myself; c)
yourselves; d)
themselves; e) herself;
f) ourselves

Unidade 13
1) a) which; b) whose; c)
who; d) which; e) who;
f) who; g) which; h)
whose; i)which; j)
whose
2) a) who; b) ok; c) ok; d)
who; e) ok; f) which
3)D
4)A
5)D
6) C
7) a) how; b) what; c)why;
d)which; e)where;
f)how; g)when; h)what;
i)how; j)where
8) B
9) D
10) C
11) 18
12) 49
13) 24
14) 5
15) 02

ESPANHOL

Unidade 1
1) 44
2) B
3) E
4) C
5) A
6) B
7) C
8) B


Unidade 2
1) B
2) E
3) D
4) 20
5) 10
6) 5

Unidade 3
1) C
2) A
3) E
4) C
5) C
6) E
7) E
8) B

Unidade 4
1) C
2) 46
3) A
4) B
5) B
6) 20
7) 9
8) 42
9) 54

Unidade 5
1) 26
2) D
3) 11
4) 14
5) 33
6) 22
7) 18
8) 13

Unidade 6
1) 14
2) 44
3) 10
4) 9
5) 33
6) 40
7) 10

Unidade 7
1) D
2) B
3) 9
4) 28
5) 9
6) 18

Unidade 8
1) D
2) B
3) 25
4) 14
5) 10

Unidade 9
1) A
2) 17
3) 14
4) 21
5) 28
6) 36
7) 51
8) 34

Unidade 10
1) E
2) A
3) B
4) C
5) C
6) E
7) D
8) C

Unidade 11
1) C
2) B
3) 20
4) 10
5) 56
6) 57


Unidade 12
1) B
2) 36
3) 6
4) 26
5) 4
6) 24
7) 15

Please purchase PDF Split-Merge on www.verypdf.com to remove this watermark.
Incluso para a Vida Geografia A

Pr-Vestibular da UFSC
1
UNIDADE 1

A Terra realiza diversos movimentos. Os principais so a
ROTAO e a TRANSLAO. A rotao o
movimento que a Terra faz em torno de seu prprio eixo.
Sua durao de 24 horas e a sua principal consequncia
a sucesso das noites e dos dias. A rotao se d no
sentido de oeste leste, sendo sua velocidade da ordem
de 1660 km por hora nas proximidades do Equador. O
movimento que a Terra faz ao redor do Sol, uma curva
fechada de forma elptica, chamado de translao. A
durao desse movimento de 365 dias e 6 horas. O
planeta encontra-se mais prximo do Sol (perilio) no
incio do ano e mais distante dele (aflio) no meio do ano.
Como o ano convencional dura somente 365 dias e a
translao de 365 dias e 6 horas, essa diferena de 6
horas compensada de 4 em 4 anos, acrescentando um
dia a mais no ms de fevereiro. O ano de 366 dias, com
29 dias em fevereiro, denominado de bissexto. A
principal consequncia do movimento de translao a
sucesso das estaes do ano: primavera, vero, outono e
inverno.










ROTAO


TRANSLAO
COORDENADAS GEOGRFICAS E FUSOS
HORRIOS
O Planeta Terra cortado por linhas imaginrias cuja
funo localizar qualquer ponto em sua superfcie. Com
essa finalidade, determinam-se as coordenadas
geogrficas: a longitude e a latitude. Se voc quiser
localizar algum ponto no globo terrestre, saiba que o
Equador o maior crculo perpendicular ao eixo do
planeta. O Equador divide a Terra em dois hemisfrios:
Sul e Norte. Acima e abaixo do Equador, podemos traar
infinitos crculos paralelos que, medida que se afastam
para o Sul ou para o Norte, diminui de tamanho. A
distncia dos paralelos medida em graus. Portanto,
chamamos de latitude a distncia, em graus, de qualquer
lugar da superfcie terrestre em relao ao Equador.



Entretanto, jamais acharemos algum ponto na Terra se
conhecermos somente sua latitude. Precisamos cruzar as
duas coordenadas: latitude e longitude. Esta a distncia,
tambm medida em graus, de qualquer lugar da superfcie
terrestre em relao ao Meridiano de Greenwich uma
cidade prxima de Londres - que divide o planeta em dois
hemisfrios: ocidental e oriental. Os meridianos so
crculos que vo de plo a plo, tendo por esta razo, o
mesmo comprimento.
A localizao de reas do planeta por meio das
coordenadas geogrficas essencial para as viagens
martimas, a navegao area, sendo muito til para
planejar e efetuar operaes militares. O comandante de
um navio, o piloto de avio e os generais que coordenam
ataques precisam saber seu ponto de destino e, no caso
militar, as regies que sero atingidas.












OS MOVIMENTOS DA TERRA
Please purchase PDF Split-Merge on www.verypdf.com to remove this watermark.
Geografia A Incluso para a Vida

Pr-Vestibular da UFSC
2
OS FUSOS HORRIOS
Como voc aprendeu anteriormente, a rotao da Terra
o movimento responsvel pela sucesso de dias e noites.
Mas tambm a causa das diferenas de horrio entre as
diversas regies do planeta. Em relao ao Meridiano de
Greenwich, qualquer ponto sobre a superfcie terrestre
varia de 0
0
a 180
0
para Oeste ou Leste. Assim, a longitude
completa da Terra de 360
0
que, divididos pelas 24 horas
de durao do dia, do como resultado 15
0
. Dessa
maneira, 15
0
que nosso planeta gira, corresponde a 1 hora.
Portanto, ao dividirmos o mundo em 24 partes (conforme
a durao do dia), cada uma delas significa um fuso
horrio. Em 1895, quando da Conferncia de Geografia
realizada em Londres, foi estipulado que todas as regies
dentro de um mesmo fuso adotariam o mesmo horrio. Na
ocasio, tambm foi estabelecido que o Meridiano de
Greenwich fosse o meridiano de referncia, em funo
do qual todos os relgios do planeta so acertados.


Exerccios de Sala #

1. (UFPR) A relao Sol-Terra faz com que em qualquer
lugar do planeta existam diferenas no tempo atmosfrico.
Essas diferenas tm origem em dois fatores principais, que
so os movimentos de rotao e de translao. Analise as
alternativas a seguir e identifique a incorreta no que se refere
influncia desses movimentos no tempo atmosfrico e nos
climas da Terra.
a) o movimento de rotao que determina os ciclos da
produo agrcola e, portanto, indica quando plantar,
quando colher, quando guardar e quando descansar.
b) O movimento de translao, combinado com a inclinao
do eixo da Terra sempre no mesmo ngulo, faz com que os
hemisfrios Norte e Sul sejam expostos alternadamente de
forma diferente luz, proporcionando assim as estaes do
ano.
c) Se a Terra no tivesse o movimento de rotao, a face
iluminada seria trrida e a face escura gelada, sendo
impossvel vida no planeta.
d) O movimento de translao o que determina a durao do
foto-perodo dirio, sendo que, para o hemisfrio Sul, a
maior durao do dia iluminado ocorre em 22 de
dezembro, quando inicia o vero.
e) O movimento de rotao o responsvel pela exposio do
planeta luz solar, fazendo com que haja certo equilbrio
em relao temperatura, pois gera os dias e as noites.

2. (UFMS) A Terra possui uma inclinao de 2327 em seu
eixo, em relao ao plano da rbita. Tal inclinao,
associada ao seu movimento de rotao e translao,
propicia a incidncia dos raios solares de maneira diferente
sobre o globo terrestre.

Sobre o movimento de translao identificado no esboo
acima, correto afirmar que:

01. O movimento de translao aquele que a Terra
realiza em torno de um eixo imaginrio que a atravessa
de plo a plo;
02. No dia 21 de maro ns temos o equincio de
primavera para o hemisfrio sul e o equincio de
outono para o hemisfrio norte;
04. Nos dias 21 de junho e 21 de dezembro ocorrem os
dias de solstcio, ou seja, quando h mxima
desigualdade na distribuio de luz e calor entre os
hemisfrios;
08.Os dias 21 de maro e 23 de setembro, tambm
conhecidos como equincios, so os dias do ano em
Please purchase PDF Split-Merge on www.verypdf.com to remove this watermark.
Incluso para a Vida Geografia A

Pr-Vestibular da UFSC
3
que os raios solares esto distribuindo de forma
equitativa, luz e calor para os dois hemisfrios;
16. No dia 21 de junho temos o solstcio de vero no
hemisfrio norte e o solstcio de inverno no hemisfrio
sul;
32. No solstcio do inverno, no hemisfrio sul, ocorre o dia
mais longo e a noite mais curta do ano.

Tarefa Mnima #

3. (UFPE) Observe atentamente o mapa a seguir e
identifique os pontos A, B, C, D e E.

1) o ponto E o que apresenta o menor valor de latitude.
2) os pontos A e B esto situados praticamente mesma
distncia longitudinal de Greenwich.
3) o ponto C localiza-se numa faixa de latitudes mdias e
de baixas altitudes.
4) o ponto D est situado numa faixa climtica bastante
diferente daquela onde se localiza o ponto E.
5) o maior valor de latitude encontrado no ponto D.
Esto corretas:
a) 1, 2, 3, 4 e 5
b) 1 e 2 apenas
c) 1, 4 e 5 apenas
d) 3, 4 e 5 apenas
e) 1 e 4 apenas

4. (UCS) O globo terrestre cortado por linhas
imaginrias que tm a funo de possibilitar a localizao
de qualquer ponto em sua superfcie. Essas linhas so
denominadas Coordenadas Geogrficas.
Analise as proposies abaixo.

I. Conhecer a latitude de um ponto no suficiente para
localiz-lo. H a necessidade do cruzamento de duas
coordenadas: a Latitude e a Longitude.
II. O meridiano de zero grau ou de referncia, que passa
pelo subrbio de Greenwich, em Londres, divide a
Terra nos hemisfrios Ocidental e Oriental.
III. A linha do Equador corresponde ao crculo mnimo,
perpendicular ao eixo terrestre, que determina a diviso
do globo em dois hemisfrios, o Meridional e o
Austral.
Considerando essas proposies, em relao s
Coordenadas Geogrficas, certo afirmar que:
a) apenas a I est correta.
b) apenas a I e a II esto corretas.
c) apenas a II e a III esto corretas.
d) apenas a I e a III esto corretas.
e) a I, a II e a III esto corretas.

5. Os Jogos Pan-Americanos so uma verso continental
dos Jogos Olmpicos, incluindo esportes do Programa
Olmpico e outros no disputados em Olimpadas.
Realizados de quatro em quatro anos, sempre um ano antes
dos Jogos Olmpicos, tiveram sua primeira edio em
1951, em Buenos Aires, capital da Argentina. Porm, sua
origem remete a 1932, nos Jogos Olmpicos de Los
Angeles. Inspirados pela realizao, seis anos antes, dos
primeiros Jogos Centro-Americanos, representantes de
pases latino-americanos no Comit Olmpico
Internacional (COI) propuseram a criao de uma
competio que reunisse todos os pases das Amricas,
com o intuito de fortalecer o esporte na regio. No ano de
2007, foram realizados no Brasil, especificamente na
cidade do Rio de Janeiro.
A ideia deu origem ao primeiro Congresso
Esportivo Pan-Americano, realizado em Buenos Aires, em
1940. A princpio, o Congresso definiu que os jogos
inaugurais seriam disputados em 1942, na prpria capital
argentina - planos adiados por causa da Segunda Guerra
Mundial. Disponvel em:
http://www.rio2007.org.br Acesso em: 28 abr. 2007. [Adaptado].

Partindo do princpio que as competies tenham incio s
7 horas e desconsiderando as possveis mudanas de
horrio em funo do estabelecimento do horrio de vero,
julgue a validade das afirmativas relativas s diferenas
entre o horrio da sede do evento (que est no mesmo fuso
horrio de Braslia) e o horrio de algumas das cidades do
mundo.

I. Considerando o horrio de incio das competies, os
moradores da Cidade do Mxico assistiro ao evento s
4 horas.
II. Imigrantes de origem brasileira residentes em Tquio
(Japo), em Paris (Frana) e no Cairo (Egito) assistiro
ao incio das competies a partir das 19 horas, 10
horas e 13 horas, respectivamente.
III. Os habitantes do Hava e de Los Angeles (Estados
Unidos) assistiro ao incio dos jogos, respectivamente,
meia-noite e s 2 horas.
IV. Os moradores de Bogot (Colmbia) e de Lima (Peru)
assistiro ao incio das competies s 5 horas, e os de
Buenos Aires (Argentina), s 6 horas.
Assinale a alternativa correta:
a) Apenas as afirmativas I e II so verdadeiras.
b) Apenas as afirmativas I e III so verdadeiras.
c) Apenas as afirmativas II e III so verdadeiras.
d) Apenas as afirmativas III e IV so verdadeiras.

Please purchase PDF Split-Merge on www.verypdf.com to remove this watermark.
Geografia A Incluso para a Vida

Pr-Vestibular da UFSC
4
6. (ACAFE) O sistema de coordenadas geogrficas da
Terra baseia-se na rede de coordenadas cartesianas e foi
traado considerando-se a Terra como uma esfera perfeita,
no Congresso Internacional de Cartografia de Londres, no
ano de 1895.
Leia as alternativas a seguir e assinale a que estiver
correta:

a) As linhas denominadas de paralelos so crculos
mximos que cortam a Terra em duas partes idnticas,
de plo a plo.
b) Longitude o valor angular do arco do paralelo,
compreendido entre o Meridiano de Greenwich e o
meridiano do lugar de referncia, considerando sempre
o plano do Equador, isto , a distncia, em graus de um
dado ponto da superfcie terrestre Linha de
Greenwich.
c) Os fusos horrios so determinados a partir do
Meridiano de Greenwich. Devido ao sentido do
movimento de rotao da Terra, que feito de Leste
para Oeste, as horas diminuem para a direo Leste de
Greenwich e aumentam para a direo Oeste.
d) So meridianos importantes: a Linha do Equador e os
Trpicos de Cncer e de Capricrnio.
e) O Brasil atravessado por cinco fusos horrios, todos
adiantados em relao hora no Meridiano de
Greenwich (GMT).

UNIDADE 2

O CLIMA DA TERRA

CLIMA
x O clima o comportamento do tempo
atmosfrico ao longo do ano. Se falarmos que o
dia est quente ou o dia est seco estamos
nos referindo ao comportamento da atmosfera
nesses dias.

Zonas trmicas da Terra
O clima de uma regio determinado pela massa de ar dominante e depende da zona trmica em que a regio est localizada.
Assim:
x as zonas polares conhecem climas frios, j que so dominadas por massas de ar frio ao longo de todo o ano.
x a zona equatorial possui clima quente, pois dominada por massas de ar quente, mesmo no inverno.
x as zonas temperadas, onde as estaes do ano so bem definidas, conhecem massas de ar frio no inverno e de ar quente
no vero.

OS FATORES DO CLIMA
O clima de uma determinada regio resultante de uma
srie de fatores:

Latitude quanto mais nos distanciamos do Equador,
portanto quanto maior a latitude, menores so as mdias
anuais de temperatura.

Altitude no alto de uma montanha, sentiremos mais frio,
no mesmo momento e na mesma latitude, do que numa
praia. Portanto, quanto maior a altitude, menor a
temperatura. Sabemos que os raios solares, quando atingem
qualquer ponto da Terra, aquecem sua superfcie que
irradiar o calor para a atmosfera.

Massas de ar ventos que se deslocam, por diferena de
presso, entre as diversas regies do planeta, sempre
carregando as caractersticas de umidade e temperatura da
regio de onde vieram. medida que se deslocam, vo se
alterando pelo contato com outras massas de ar com as quais
trocam calor. As massas de ar podem ser classificadas em
ocenicas, que apresentam muita umidade; e em
continentais, quase sempre secas; as que se originam de
regies tropicais e equatoriais so quentes e as que nasceram
em reas temperadas e polares so frias.

Continentalidade e Maritimidade se uma regio se
encontra prxima a grandes quantidades de gua sofrer
alteraes tanto na umidade relativa do ar como tambm na
temperatura. Assim, nas reas continentais a temperatura
maior do que nas regies prximas ao mar.

Correntes martimas as grandes massas de gua que se
deslocam pelos oceanos, possuindo presso, quantidade de
sal e temperaturas prprias, tambm influenciam o clima.
Exemplo disso a corrente quente do Golfo (Gulf
Stream), que impede o congelamento do mar do Norte. J a
corrente fria de Humboldt ameniza as temperaturas tanto do
norte do Chile como no sudoeste dos Estados Unidos da
Amrica.
Please purchase PDF Split-Merge on www.verypdf.com to remove this watermark.
Incluso para a Vida Geografia A

Pr-Vestibular da UFSC
5


MAPA DAS CORRENTES MARTIMAS

Relevo influi na temperatura e na umidade, pois facilita ou dificulta a
circulao das massas de ar. Nos Estados Unidos, por exemplo, a Serra
Nevada e as Montanhas Rochosas impedem o trnsito das massas de ar vindas
do Pacfico, fazendo com que as chuvas sejam abundantes nas regies
prximas ao mar. Do outro lado das montanhas o clima rido. Em nosso
pas, a Serra do Mar, pela sua disposio longitudinal, facilita a circulao da
massa polar e dificulta o trnsito da massa de ar tropical atlntica.
Vegetao as plantas tiram umidade do solo pela raiz e a enviam
atmosfera pelas folhas, contribuindo para alterar a temperatura. Alm disso, a vegetao impede que os raios solares atinjam
diretamente a superfcie terrestre. Portanto, o desmatamento (a destruio das florestas pela derrubada de rvores) ruim, j que
diminui a umidade do ar e provoca a elevao da temperatura.
Tarefa Mnima #

1. Abaixo esto descritas ocorrncias que constituem
problemas ambientais, que se generalizaram e extrapolaram
grandemente os antigos limites. Leia-as com ateno e
assinale a(s) proposio(es) verdadeira(s).

01. A chuva cida exemplifica um tipo de degradao
ambiental global, prprio das sociedades tradicionais
rurais.
02. O efeito estufa um problema ambiental, ligado ao
desenvolvimento, com efeito planetrio.
04. A destruio das florestas tropicais diminui a rica
biodiversidade, alm de alterar o clima da rea, com
reflexos sobre outros espaos geogrficos.
08. No Terceiro Mundo, no que diz respeito maioria da
populao, concentra-se a poluio da misria,
caracterizada, por exemplo, pela fome, subnutrio,
lixes, ausncia de esgotos e de gua potvel.
16. A diminuio da camada de oznio permite que a nociva
radiao ultra-violeta atinja diretamente a Terra,
aumentando a incidncia de doenas da pele.

2. (UFSC) As figuras abaixo mostram dois fenmenos
ambientais de grandes cidades. Analise-as, e assinale a(s)
proposio(es) correta(s):



01. Na figura 1, que apresenta condies atmosfricas
normais, o ar mais prximo ao solo mais quente do
que o ar em altitude, o que ocasiona movimentos de
conveco e a consequente formao de ventos que
dissipam os gases poluentes.
Please purchase PDF Split-Merge on www.verypdf.com to remove this watermark.
Geografia A Incluso para a Vida

Pr-Vestibular da UFSC
6
02. Nos espaos altamente urbanizados, significativa a
diferena de temperatura entre a regio central, mais
fria, e a periferia, com maior temperatura, fato
comprovado na figura de nmero 3.
04. No inverno, figura 2, a rpida penetrao de ar frio
provoca a inverso das camadas atmosfricas, quando
os gases poluentes permanecem retidos no local, em
virtude da ausncia de ventos, situao responsvel pelo
surgimento de doenas respiratrias.
08. Em So Paulo, na cidade do Mxico e em Tquio, a
inverso trmica, fenmeno mostrado na figura 2,
agravada no s pelo grande volume de poluentes
emitidos a partir da grande frota de veculos, como
tambm pela quantidade de indstrias nessas trs
metrpoles.
16. A poluio, o asfalto, o concreto e a ausncia de reas
verdes so os fatores responsveis pela maior
temperatura nas regies centrais das cidades, fenmeno
conhecido como ilhas de calor, conforme mostrado
na figura 3.

3. (UNEB) Com base nos conhecimentos sobre a dinmica
da atmosfera, pode-se afirmar:

01. A atmosfera absorve diretamente toda a energia solar
que chega Terra.
02. As reas quentes ou de alta presso, receptoras de
massas de ar, recebem o nome de ciclonais.
04. As reas frias ou de baixa presso, dispersoras de massas
de ar, recebem o nome de anticiclonais.
08. A circulao atmosfrica alterada pelo movimento de
rotao que d origem s estaes do ano e pela
existncia das massas continentais.
16. As massas de ar possuem temperatura e umidade de
acordo com a regio de origem.

4. (UFCE) Com relao aos principais tipos de chuvas,
convectivas, frontais e orogrficas, analise as seguintes
assertivas:
I. As chuvas convectivas so provocadas pela ocorrncia de
subidas de ar quente e o resfriamento das camadas
superiores da atmosfera.
II. As chuvas frontais so causadas pelo encontro de uma
massa de ar frio com outra quente e mida.
III. As chuvas orogrficas ocorrem quando as massas de ar
quente e mido se elevam e se resfriam nas encostas das
montanhas.

Com base nas assertivas acima, correto afirmar que:
a) I e III so verdadeiras.
b) I e II so verdadeiras.
c) II e III so verdadeiras.
d) I, II e III so verdadeiras.
e) Apenas I verdadeira.

UNIDADE 3

ORIGEM DOS CONTINENTES

Os continentes, conforme se apresentam nos dias de hoje,
foram na verdade originados de um processo de
fragmentao e afastamento de terras emersas de um nico
aglomerado primordial, processo este que durou centenas de
milhes de anos. Este aglomerado de terras continentais,
chamado Pangeia, existiu h cerca de 200 milhes de anos
atrs.
O afastamento de suas pores continentais foi
gerado provavelmente a partir da atividade tectnica
terrestre que, no perodo referido, encontrava-se em plena
ao e em larga escala. Segundo consta nos estudos
realizados, uma primeira poro continental teria sido
separada das demais na regio setentrional da Pangeia. A
este primeiro grande fragmento deu-se o nome de Laursia,
originada por volta de 130 milhes de anos atrs. Os
territrios que na atualidade formam a frica e a Amrica
do Sul formavam dois fragmentos colados em suas regies
costeiras. Especula-se tal fato, inclusive, pela similaridade
entre tipos de vegetao e terrenos encontrados nos dois
continentes.
AS CAMADAS DA TERRA
Quatro so as principais camadas de nosso planeta:






As quatro pores da Terra
AS ESTRUTURAS GEOLGICAS
A crosta terrestre formada por doze placas tectnicas que
flutuam sobre o magma pastoso. Na fase inicial da Terra,
existiam menos placas. Com o tempo, em razo de se
moverem em vrios sentidos, j que o planeta esfrico, as
placas se encontraram em vrios pontos da crosta terrestre,
dando origem aos terremotos e aos dobramentos do relevo.
Em grego, o termo tectnica quer dizer processo de
construir. Para a cincia geogrfica, so as deformaes da
crosta terrestre geradas pelas presses provenientes do
interior do planeta.
Nas reas de encontro das placas, a crosta terrestre
frgil, principalmente nas regies de contato dos oceanos
com os continentes, o que possibilita a sada de magma,
dando origem aos vulces. Dos choques entre as placas,
surge o atrito que provoca os terremotos. Nos oceanos, as
placas (sima) so pesadas e, por este motivo, tendem a
mergulhar sob as continentais (sial). Esse fenmeno,
conhecido como subduco, gera as fossas martimas,
normalmente nas zonas onde ocorre o encontro das placas.
Como as placas ocenicas se situam debaixo das
continentais, a presso das primeiras sobre estas ltimas
provocam dobras e enrugamentos, provocando, desde a
era mesozoica, os movimentos orogenticos (em grego,
oros significa montanha). Data da o aparecimento das
grandes cadeias montanhosas do planeta Terra, formadas
pelo enrugamento, elevao ou dobramento de partes da
crosta terrestre. Este fenmeno relativamente recente na
histria do nosso planeta, tendo acontecido no fim da era
mesozoica e incio da cenozoica. Por essa razo,
Please purchase PDF Split-Merge on www.verypdf.com to remove this watermark.
Incluso para a Vida Geografia A

Pr-Vestibular da UFSC
7
denominamos dobramento moderno. As mais altas cadeias
de montanhas do planeta, tais como o Himalaia, as Rochosas
e os Andes, so de formao recente, apresentando elevadas
altitudes, pouco desgaste e grande instabilidade fsica, pois
elas esto ainda em processo de formao. Nelas, so
comuns vulces e terremotos.
A Terra, se levarmos em conta a sua origem geolgica,
conhece trs formaes bsicas:
x bacias sedimentares
x escudos cristalinos
x dobramentos modernos

Estruturas geolgicas
Os dobramentos modernos, ou cadeias orognicas recentes,
correspondem s grandes cadeias montanhosas do globo
datadas do perodo Tercirio da Era Cenozica. Sua gnese
explicada pelo movimento das placas tectnicas. Os
principais exemplos desse fenmeno so os Andes, os
Alpes, o Himalaia e as Montanhas Rochosas. Por serem de
formao recente, no foram ainda desgastadas pela eroso e
apresentam altitudes elevadas. O Brasil, por exemplo, no
conhece formaes geradas por dobramentos modernos.

AS FORMAS DO RELEVO
Relevo a forma assumida pela superfcie da crosta
terrestre aps ser modelada pela atuao de agentes
internos e externos. Os fatores fundamentais para a
formao do relevo so os de origem tectnica, tais como a
constituio de cadeias de montanhas e a ocorrncia de
atividades vulcnicas ou terremotos.
As transformaes das formas de terreno na
superfcie terrestre so geradas pelos movimentos das placas
tectnicas e complementadas pela ao de agentes externos
erosivos, como vento, chuva, rios, neve e gelo. Em termos
bem simples, os agentes externos alteram as formas de
terreno modeladas pelos internos. As principais formas de
relevo so as montanhas, os planaltos, as plancies e as
depresses.
As montanhas so elevaes no terreno que
atingem grandes altitudes. Um conjunto de montanhas
recebe as denominaes de serra, cadeia ou cordilheira.
Os planaltos so forma de relevo onde a eroso
predomina sobre a sedimentao, consistindo de terrenos,
por vezes, planos, e, em certos casos, bastante acidentados.
Os planaltos mais elevados do planeta se encontram na sia,
junto Cordilheira do Himalaia, sendo denominados de
telhados do mundo: os planaltos de Pamir e do Tibete.
As plancies so terrenos planos, formados por
acumulao de sedimentos. Elas aparecem,
fundamentalmente, nas baixadas fluviais, lacustres e
litorneas. Embora quase sempre a plancie se diferencia do
planalto pela altitude (plancie baixa e planalto alto), isso
nem sempre verdadeiro. Na realidade, a diferena entre
planalto e plancie a formao geolgica: a plancie
consiste em rochas sedimentares acumuladas; o planalto
produto da eroso e da acumulao de rochas cristalinas e
sedimentares. A maior plancie do mundo a Siberiana,
situada na Repblica da Federao Russa.
As depresses so terrenos com altitudes inferiores
s das regies prximas. H dois tipos de depresses:
relativa e absoluta. A primeira consiste num terreno mais
baixo em relao s reas prximas, mais ainda assim acima
do nvel do mar. O exemplo clssico de depresso relativa
o pntano.

Tarefa Mnima #

1. (UFGO-adaptada) A teoria da tectnica de placas fornece
uma interpretao para a gnese e a dinmica da litosfera,
indicando que a crosta terrestre apresenta-se fragmentada em
grandes blocos, de dimenso e movimento heterogneos. O
esquema a seguir (retirado do livro Geografia Geral e do
Brasil: Paisagem e Territrio, de Demtrio Magnoli e
Regina Arajo, 1997) demonstra a atividade existente na
regio das placas tectnicas entre o oceano Pacfico e a
frica.

Sobre a dinmica das placas tectnicas, some as alternativas
corretas:
01. A inexistncia de registros fsseis de fauna ou flora,
comum s reas dos continentes, um dos fatores que
mais dificultam a comprovao do movimento das
placas.
02. As perturbaes geolgicas que atingem a crosta, como
os terremotos e o vulcanismo, originam-se da presso
exercida pelos movimentos convectivos do manto.
04. As regies mais afastadas das zonas de convergncia ou
afastamento das placas como o caso do territrio
brasileiro apresentam relativa estabilidade tectnica.
08. As reas de contato entre as placas tectnicas se
relacionam com gerao das altas montanhas (Andes,
Himalaia etc.) nos continentes.

2. (UEL) Observe a ilustrao a seguir:

O planeta Terra tem aproximadamente 4,6 bilhes de anos.
Para efeito de estudos, os gelogos dividiram esse tempo em
ons, eras, perodos e pocas. Com base na ilustrao e nos
conhecimentos sobre o tema, analise as afirmativas a seguir:

Please purchase PDF Split-Merge on www.verypdf.com to remove this watermark.
Geografia A Incluso para a Vida

Pr-Vestibular da UFSC
8
I. A era mesozica durou, aproximadamente, 165 milhes de
anos.
II. A durao da era cenozica foi maior que a durao da
era mesozica.
III. As duraes dos perodos trissico, jurssico e cretceo
so proporcionais a 8, 10 e 15, respectivamente.
IV. O perodo tercirio da era cenozica durou
aproximadamente 63.200 anos.
Assinale a alternativa que contm todas as afirmativas
corretas.
a) I e II.
b) I e III.
c) III e IV.
d) I, II e IV.
e) II, III e IV.

3. (PUC-MG) A estrutura geolgica da superfcie terrestre
constitui o embasamento do modelado do relevo, em
contnuo processo de transformao. So grandes estruturas
geolgicas, exceto:
a) Os escudos cristalinos ou macios antigos, resultantes da
solidificao do material magmtico e da ascenso de
suas formaes rochosas at a superfcie.
b) As bacias sedimentares, de formao antiga ou recente,
resultantes da ao destrutiva da eroso sobre os macios
e da posterior deposio do material erodido sobre reas
rebaixadas ou de sedimentao em perodos mais
recentes.
c) Os dobramentos modernos, originados do entrechoque de
placas, formando os episdios mais recentes de
acomodao tectnica.
d) Os crculos de fogo, formadores de reas de elevada
instabilidade tectnica, com elevada incidncia de
atividade vulcnica, terremotos e maremotos.

4. (UFMG) Identifique os itens da esquerda com a sua
localizao direita, assinalando depois a sequncia correta.
Observe que o bloco da direita tem um item a mais.

A sequncia correta :
a) I-C, II-D, III-F, IV-A, V-E
b) I-F, II-E, III-D, IV-C, V-B
c) I-B, II-D, III-C, IV-F, V-A
d) I-A, II-D, III-F, IV-C, V-B

UNIDADE 4

AS ROCHAS E OS MINERAIS

A crosta do nosso planeta composta de rochas e
minerais. O mineral um material slido formado por
um elemento ou uma composio qumica encontrado
em todas as camadas geolgicas da Terra. Exemplos de
minerais so o ouro, o diamante, o talco e o quartzo. A
rocha, por sua vez, um composto de minerais, podendo
conter tambm elementos orgnicos. O basalto, o arenito,
o granito e o mrmore so exemplos de rocha. As rochas
e os minerais so objeto de ampla explorao econmica:
os chamados minerais preciosos, tais como o diamante
e o ouro, so a matria prima das joias que enfeitam as
pessoas; o granito, formado pelos minerais quartzo, mica
e feldspato, utilizado na fabricao de pisos, pias e na
pavimentao de rampas que do acesso s residncias.
As rochas conhecem um processo constante de
transformao denominado de ciclo das rochas pelo
qual rochas antigas tornam-se novos tipos de rocha.

TIPOS DE ROCHAS
As rochas se dividem, quanto a sua origem, em trs
tipos.
As rochas sedimentares surgem pela acumulao
de material orgnico e de detritos ou fragmentos de
outras rochas. As sedimentares podem ser detriticas
(quando sua origem decorre de detritos), orgnicas e
qumicas. O processo de formao das rochas
sedimentares chamado de intemperismo, e causado
por agentes fsicos, qumicos e biolgicos. Quando as
rochas so decompostas por agente fsicos e biolgicos,
ocorre o intemperismo fsico. Quando a decomposio
gerada por um agente qumico, ocorre o intemperismo
qumico.
As rochas magmticas ou gneas so criadas
pela solidificao do magma, quer no interior, quer na
superfcie da crosta terrestre. A solidificao no interior
do planeta sempre lenta; superfcie, rpida. O
granito um exemplo de rocha formada nas profundezas
da Terra; o basalto resultante da solidificao rpida de
magma na superfcie.
As rochas metamrficas so geradas pelas
alteraes de temperatura, presso e profundidade
sofridas pelas rochas magmticas ou sedimentares.
Portanto, qualquer tipo de rocha, quando transformada,
uma rocha metamrfica. O mrmore, por exemplo,
resulta das mudanas do calcrio.

OS SOLOS
Chamamos de solo a superfcie da crosta terrestre,
permanentemente alterada pelo intemperismo e onde
nasce e se desenvolve a vida vegetal. Todo solo
formado por sedimentos provenientes da desintegrao
das rochas, misturados com vrios materiais orgnicos,
frutos da decomposio de plantas e animais.
Os solos, quanto origem, esto divididos em trs tipos:
Os solos orgnicos so resultantes da sedimentao
de material orgnico. As florestas e as plancies cobertas
de vegetao rasteira so exemplos de solos orgnicos.
Em ambos os casos, os elementos vegetais mortos
sofreram decomposio, gerando o humo. Por essa
razo, tais solos so conhecidos como humferos. A
Plancie Polonesa, que se estende da Polnia at a
Rssia, um bom exemplo de solo orgnico, da
decorrendo sua impressionante fertilidade.
Please purchase PDF Split-Merge on www.verypdf.com to remove this watermark.
Incluso para a Vida Geografia A

Pr-Vestibular da UFSC
9
Os solos aluviais so criados por sedimentos
transformados em lugares distantes e transportados pelas
guas dos rios e pelos ventos. As reas ribeirinhas so
exemplos de solos aluviais.
Os solos eluviais so formados pela decomposio
de um certo tipo de rocha e sempre aparecem em locais
onde as rochas matrizes foram transformadas pela
intemperizao. A produo de caf, que por um longo
perodo foi a causa responsvel pela prosperidade de So
Paulo, deu-se em terras roxas derivadas das alteraes
sofridas pelo basalto, rocha de origem magmtica.
Tambm o plantio da cana de acar, produto que
marcou o inicio da colonizao do Nordeste brasileiro,
foi possvel graas a um tipo de solo denominado
massap, fruto das transformaes do calcrio, este de
origem sedimentar.

OS MINRIOS E AS ESTRUTURAS
GELOGICAS
Nos escudos formados durante a era pr-cambriana,
encontramos grandes quantidades de minerais
metlicos, tais como ferro, ouro e bauxita. Por sua vez,
nos escudos da era paleozoica, so abundantes minerais
no-metlicos, como, por exemplo, gesso e cimento. Por
sua vez, os dobramentos modernos so ricos em todos
os tipos de minrio. Nas bacias sedimentares
cavidades cheias de pedaos minerais de rochas
vitimadas pela eroso e preenchidas tambm por restos
orgnicos, encontramos combustveis fosseis. Nos
ambientes aquticos, onde so abundantes os
plnctons*, surge o petrleo. J nas regies onde
existiram florestas, atualmente soterradas, costumam ser
abundantes reservas de carvo.

Exerccios de Sala #

1. (PUC-RS) As rochas, antes de serem trabalhadas pela
eroso, so preparadas por um conjunto de reaes
qumicas ou fenmenos fsicos, para a ao de desgaste.
A essa fase que precede a eroso, denominamos:
a) abraso d) evapotranspirao
b) intemperismo e) estratificao
c) orognese

Tarefa Mnima #

2. (UFBA) Com base no esquema e nos conhecimentos
sobre a dinmica do meio ambiente, pode-se afirmar:
01. O relevo, ao mesmo tempo que recebe influncia do
clima, pode tambm interferir, significativamente, na
temperatura, na presso atmosfrica e na distribuio
espacial das isoietas anuais.
02. O solo representa o elemento sntese de uma
paisagem, porque as pequenas alteraes que
ocorrem nos demais componentes do meio natural
so imediatamente refletidas nesse componente de
natureza estritamente abitica.
04. A organizao espacial das bacias hidrogrficas, nos
seus mais diferentes padres de drenagem, uma
consequncia direta dos regimes e da intensidade das
chuvas que abastecem as redes fluviais nas diversas
latitudes.
08. O relevo terrestre produzido pelo ajuste de
processos internos e externos, sendo os processos
exgenos representados pela orognese e pela
epirognese, responsveis pela esculturao do
modelado.
16. Os domnios estruturais que aparecem no territrio
brasileiro so caracterizados por feies escarpadas,
intercaladas por vales bem encaixados, nos quais os
processos endgenos tiveram, no passado geolgico,
uma intensa atividade.
32. O domnio dos mares de morro, existente no
Sudeste brasileiro, atesta a grande influncia dos
processos exgenos na paisagem regional.
64. Os solos, nas regies intertropicais marcadas pela
ao do intemperismo fsico, so geralmente bem
desenvolvidos e profundos, favorecendo a cobertura
natural.

3. (UnB) O solo e o subsolo so passveis de degradao
ambiental. Analisando os processos que neles ocorrem,
pode-se perceber o grau de vulnerabilidade de um
sistema natural.
Com relao s causas que comprometem a utilizao
sustentvel do solo e do subsolo, julgue os itens
seguintes como verdadeiros ou falsos.

( ) A ao erosiva da gua no solo ocorre quando ela, em
vez de escoar superficialmente, sem qualquer empecilho,
infiltra-se para o subsolo.
( ) Comumente conhecidos como lixes, os grandes
depsitos urbanos de lixo so fonte de contaminao do
subsolo, sendo que a coleta seletiva de lixo ajuda a
reduzir esse problema.
( ) Substncias qumicas provenientes de indstrias
podem poluir o solo, mas, felizmente, o subsolo est a
salvo desses tipos de contaminaes devido sua maior
profundidade.
( ) Os agrotxicos apresentam o risco de contaminar a
gua potvel e de matar microrganismos essenciais ao
solo.
Assinale a alternativa que contenha a sequncia correta:

A) F V F V
B) F F V V
C) V V F V
D) F V F F
E) V F F V
Please purchase PDF Split-Merge on www.verypdf.com to remove this watermark.
Geografia A Incluso para a Vida

Pr-Vestibular da UFSC
10

4. (Fatec-SP) Em numerosas reas do territrio brasileiro
pode-se encontrar o processo de laterizao do solo, isto
:
a) O aumento da acidez do solo que afeta sua fertilidade.
b) A formao de crostas ferruginosas que impossibilita
o uso do solo para a agricultura.
c) A retirada dos nutrientes orgnicos do solo devido a
grande volume de chuvas.
d) A contaminao do solo pelo uso indiscriminado de
fertilizantes qumicos.
e) A formao de sulcos provocados pela eroso em
reas de forte declividade.

5. (UFSM)
O homem tem a capacidade de interferir na
geomorfologia terrestre de forma limitada () nos
fatores externos que sua atuao marcante () O
homem pode mudar o relevo indiretamente ou
diretamente.
(OLIVA, J., & GIANSANTI, R. Espao e Modernidade Temas da
Geografia Mundial. So Paulo: Atual, 1995. p. 305.)

A partir dessa citao, assinale V (verdadeira) ou F
(falsa) em cada afirmativa a seguir.
( ) A expresso eroso acelerada foi criada para
caracterizar a ao indireta do homem nos processos
erosivos. Um exemplo clssico dessa ao o
desmatamento.
( ) A impermeabilizao do solo ocorre, com maior
expresso, nas reas urbanas. Durante as chuvas, as
guas, no podendo infiltrar-se no solo, formam
enormes enxurradas provocando eroso em encostas
e reas no-impermeabilizadas.
( ) Desviando cursos de rios ou represando as guas
desses rios, o homem age diretamente como criador
de formas de relevo. Um exemplo disso a
construo da usina hidreltrica de Itaipu.
A sequncia correta :
a) V - F - V. d) V - F - F.
b) V - V - V. e) F - F - V.
c) F - V - F.
UNIDADE 5

HIDROGRAFIA

Os rios so correntes de gua que se originam
do subsolo ou da atmosfera. O vapor de gua que existe
na atmosfera, quando se condensa, precipita-se na forma
de chuva, aumentando a quantidade de gua dos rios.
Estes podem nascer em lagos, fontes ou at mesmo em
outros rios e desguam no mar, em lagos e, por vezes,
em outros rios. Na superfcie terrestre, as guas seguem
trs caminhos: escorrer, evaporar e infiltrar-se no solo.
As guas que penetram o solo e as que escorrem buscam
as partes mais baixas dos terrenos, outro fator de
formao de rios, de lagos e mares. Quando evapora, a
gua retorna atmosfera como vapor, reiniciando o ciclo
das chuvas.



Perfil Transversal
Todo rio pode ser observado de duas maneiras: no perfil
transversal e no perfil longitudinal. Este ltimo, o
perfil longitudinal, estende-se da nascente at a foz,
estando dividido em trs partes: nascente, curso e foz. A
foz pode apresentar-se de trs formas: em forma de
esturio, onde inexistem ilhas e modelada por somente
um canal, que lembra um funil. A foz em forma de
delta cheia de ilhas e assim o rio desgua atravs de
diversos canais. A denominao desse tipo de foz
decorre da forma em delta criado pelos sedimentos nela
depositados pelo fluxo das mars. H um terceiro tipo de
foz, a mista ou complexa, na qual um lado da foz em
Please purchase PDF Split-Merge on www.verypdf.com to remove this watermark.
Incluso para a Vida Geografia A

Pr-Vestibular da UFSC
11
forma de delta e o outro esturio. No Brasil, um
exemplo desse tipo de foz o delta-esturio do rio
Amazonas.
Se olharmos um rio longitudinalmente, perceberemos
que duas so as suas direes. A primeira, denominada
de jusante, o deslocamento da gua da nascente para a
foz. A segunda, chamada de montante, , a direo do
rio, em seu curso, da foz para a nascente. Se
contemplarmos um rio de maneira transversal, isto ,
quando nos colocamos no meio do fluxo fluvial, olhando
para a jusante, notaremos que ele forma um conjunto, o
vale fluvial, abrangendo as duas vertentes, as duas
margens e o leito fluvial (lugar por onde ele corre).
Quando os rios desguam, eles so chamados de
afluentes. Dessa maneira, todo o rio afluente ou do
mar, ou de um lago e, por vezes, de outro rio, ao
conjunto formado por um rio e seus afluentes dada a
denominao de rede fluvial. A zona ou rea banhada
por uma rede de rios conhecida como bacia fluvial. Os
rios, quando desguam para o oceano, ocorre uma
drenagem exorreica (exo, em grego, significa fora);
quando as guas de um rio permanecem no interior de
guas continentais, por exemplo num lago, a drenagem
do tipo endorreico (em grego, endo quer dizer
dentro).

REDE FLUVIAL
Escorrendo por uma determinada superfcie, os rios
passam por diferentes tipos de terrenos: alguns baixos e
outros altos. Estes ltimos desempenham o papel de
divisores de guas entre dois rios. Nesses divisores,
forma-se uma rede de captao pela qual toda a gua se
encaminha para o mesmo ponto, denominado de
vertente.
Os rios podem ser mais ou menos densos,
conforme o clima da regio por onde ele passa. Em reas
com altos ndices de chuva, os rios so caudalosos e
permanentes, pois possuem um grande volume de gua e
jamais secam. J em zonas ridas ou semi-ridas, os rios
tendem a ser temporrios, secando no perodo da falta de
chuvas. Somente os rios que nascem numa rea chuvosa
conseguem percorrer longas extenses de clima rido ou
semi-rido. Na maioria dos rios, as nascentes, quando da
falta de chuva, chegam a secar e nos perodos chuvosos o
volume de guas aumenta. A variao da quantidade de
um rio ao longo de um ano recebe a denominao de
regime. Normalmente, as nascentes so alimentadas
pelas guas da chuva, sendo, nesse caso, conhecidas
como nascentes pluviais. Outras captam gua
proveniente do derretimento da neve, as nascentes nevais
e algumas dependem de geleiras, as nascentes glaciais.
Em raros casos, as nascentes so mistas, isto ,
alimentadas por chuva e neve, fenmeno muito comum
no Japo.
Rios que descem de planalto tendem a ter um
curso retilneo; os que percorrem reas planas formam
meandros, ou seja, ao se desviarem dos obstculos
naturais existentes nas plancies, esses rios fazem curvas
e o curso das guas mais lento. Os rios so
fundamentais para a economia de qualquer pas, pois
abastecem as cidades com gua potvel, so usados para
irrigar plantaes e geram eletricidade. Os rios que
correm em regies planas so extremamente navegveis,
facilitando o transporte interzonal. Outra contribuio
dos rios para a vida humana o fato deles serem cheios
de peixes, animais de alto ndice proteico e essenciais
para a alimentao humana. Tambm no podemos
esquecer que os rios ajudam o lazer, pois neles podemos
nadar e praticar esportes.

OS LAGOS
Os lagos consistem em depresses de terreno
preenchidas por gua. Estas depresses so causadas ou
por vulces, ou por movimentos das placas tectnicas ou
em funo do deslizamento de geleiras. No nosso pas,
onde a estrutura geolgica bastante antiga, as
depresses so de origem sedimentar, formando as
bacias sedimentares. Os mares fechados, como o
Cspio e o Morto, no passam de grandes lagos, todos os
dois com elevada salinidade. Assim como os rios, os
lagos podem nascer numa fonte, tambm podem ser
alimentados por outros rios e originrios de mares e rios.
O maior lago do mundo o Cspio, situado na fronteira
da Rssia. O lago mais profundo do planeta o Baikal,
localizado na fronteira da Monglia com a Rssia. No
Brasil, o maior lago a Lagoa dos Patos, situado no Rio
Grande do Sul. A bacia lacustre maior importante do
mundo formada pelos lagos Erie, Superior, Huron,
Ontrio e Michigan, formando o complexo denominado
de Grandes Lagos.

AS GUAS DO PLANETA TERRA

O nosso mundo, denominado Terra, talvez devesse ser chamado de Planeta gua, j que de sua rea total, da ordem de
510.000.000 km, 70%, ou seja, 360.000.000 km, consistem em terras imersas, cobertas por guas martimas e fluviais. As
zonas emersas, 30% do territrio do planeta, esto distribudas pelos diversos continentes e ilhas. No Hemisfrio Norte
denominado de hemisfrio das terras ou hemisfrio continental 2/3 das terras so emersas. No Hemisfrio Sul
chamado de hemisfrico ocenico ou hemisfrio da guas , somente 1/3 de sua rea total no coberta pelas guas.





Please purchase PDF Split-Merge on www.verypdf.com to remove this watermark.
Geografia A Incluso para a Vida

Pr-Vestibular da UFSC
12

OCEANOS

Fonte: Estudando a Paisagem, prof. Osvaldo Piffer

Verdadeiramente, s existe, em todo o planeta, um nico
oceano, pois as guas que cobrem a maior parte da
superfcie da Terra formam um conjunto nico, contnuo e
sem separaes. Pacfico, Atlntico, ndico e Glacial
rtico os grandes oceanos do planeta foram
denominaes dadas por razes histricas e no geogrficas.
O oceano Pacfico, cuja rea de 180.000.000 km, ocupa
metade de toda a superfcie ocenica da Terra, estendendo-
se do sul ao norte e separando a sia e a Oceania do
continente americano. Nas Filipinas, localiza-se sua maior
profundidade, a fossa das Marianas, situada a 11.000 m
abaixo da superfcie. Do ponto de vista econmico, o oceano
de maior destaque o Atlntico, cuja extenso de
106.000.000 km diariamente percorrido por navios
militares e transatlnticos carregados de passageiros,
fundamentalmente no Atlntico Norte. Sua maior
profundidade, com 9.000 m, a fossa de Porto Rico, na
Amrica Central. No centro do Atlntico, existem
montanhas submersas conhecidas como Dorsal Atlntica,
cuja a forma lembra um enorme S, e suas reas mais
elevadas formam as ilhas de Ascenso, Canrias, Tristo
da Cunha e Santa Helena. O oceano ndico, com
75.000.000 km, est situado ao sul da sia, a leste da
frica e a oeste da Oceania. A fossa de Java, com 7.700
metros e localizada a noroeste da Austrlia, sua maior
profundidade. Por fim, ao redor do Plo Norte, existe o
oceano Glacial rtico, que ocupa somente 3% da superfcie
total do planeta.

Exerccios de Sala #

1. (UFSE-adaptada) Analise as proposies que seguem:
01. Os rios exercem intenso trabalho como agente
modelador do relevo terrestre.
02. Os rios, ao escavarem seus vales, podem encontrar
rochas de diferentes graus de resistncia, tornando-os
mais ou menos profundos.
04. A existncia de quedas dgua est relacionada
movimentao das placas tectnicas.
08. No baixo curso do rio predominam os processos
erosivos, fato que confirmado pela presena de
inmeros meandros.
16. O desmatamento das margens dos rios responsvel
pelo assoreamento do leito fluvial, o que provoca
grandes cheias.

2. (UFMS) A figura abaixo indica etapas do ciclo da gua
na natureza. Certos aspectos relacionados ao ciclo
hidrolgico, normalmente no indicados nos esquemas,
podem ser deduzidos pela integrao ou transposio e
informaes pertinentes.

Considere as seguintes afirmaes:
I. A dinmica do ciclo hidrolgico independe da energia
solar.
II. Nas cidades, o ciclo da gua sofre interferncia das
construes e da pavimentao, a ponto de ocorrerem
enchentes ou inundaes.
III. Os animais no tm participao no ciclo da gua na
natureza.
IV. A interceptao da gua precipitada e sua taxa de
infiltrao no solo so menores nas reas desmatadas.
Em relao ao ciclo da gua, esto corretas apenas as
afirmaes
a) I e IV. d) II e III.
b) III e IV. e) II e IV.
c) I e II..








Please purchase PDF Split-Merge on www.verypdf.com to remove this watermark.
Incluso para a Vida Geografia A

Pr-Vestibular da UFSC

13
Tarefa Mnima #

3. (UFSC) Observe atentamente o mapa-mndi e assinale
a(s) proposio(es) verdadeira(s).



Fonte: SENE, Eustquio de; MOREIRA, Joo C. Trilhas da geografia:
espao geogrfico mundial e globalizado. So Paulo: Scipione, 2001
(adaptado).

01. O Oceano Pacfico, o maior dos oceanos, banha terras
dos continentes americano e europeu, situadas
integralmente no hemisfrio ocidental.
02. Algumas reas da faixa tropical, situada entre os trpicos
de Cncer e de Capricrnio, so fracamente povoadas,
como o caso da Amaznia, na Amrica do Sul.
04. Pelo mapa, pode-se concluir que, embora as massas
lquidas cubram a maior parte da superfcie terrestre,
sua distribuio desigual nos hemisfrios Norte e Sul.
08. Os principais pases industrializados localizam-se na
zona temperada do Norte, enquanto a maioria dos
pases perifricos, entre os eles o Brasil, situa-se na
faixa tropical.
16. O Brasil localiza-se integralmente no hemisfrio
ocidental e tem suas horas atrasadas em relao
Europa e frica, porque o movimento de rotao da
Terra se faz de Oeste para Leste.

4. (UFSC) Leia o texto abaixo com ateno.

(...) Temos uma disponibilidade no Brasil de 35mil m
3

per capita. Isso significa 17 vezes o que tem a
Alemanha e quase 10 vezes a Frana. Portanto, temos
muita gua. E, talvez, por isso mesmo, nos damos o
direito de deteriorar nossa gua, gastando-a de forma
extremamente irresponsvel.
REBOUAS, Aldo da C. Falta uma poltica nacional de gua.
Cincia Hoje, jun. 1995.

Considerando a importncia da gua como recurso natural,
assinale a(s) proposio(es) verdadeira(s):

01. O Brasil, pelo fato de ter uma das maiores reservas
hdricas superficiais do mundo, e de possuir em seu
subsolo parte do enorme aqufero Guarani, no enfrenta,
em parte alguma do seu territrio, crise de
abastecimento de gua.
02. A gua, esse precioso lquido, ainda aparentemente
abundante, poder ser no futuro alvo de disputas no
mundo, onde uma parcela significativa da populao j
vive em situao de penria crnica pela escassez desse
recurso.
04. O manejo desse lquido precioso exige procedimentos
que preservem suas qualidades, como ocorre no Brasil,
onde a formao de uma conscincia social a respeito
fundamenta-se numa poltica nacional de gesto
racional dos recursos hdricos.
08. A gua, que j era usada na antiguidade como
instrumento de dominao, continua gerando discrdias
e disputas, pois a disponibilidade de gua doce no
mundo reduzida e sua distribuio na superfcie do
planeta, desigual.
16. A escassez de gua, em muitas regies do planeta,
consequncia, no somente da crescente poluio no
meio urbano e rural, como tambm do aumento
contnuo do consumo, decorrente do crescimento
populacional e de atividades econmicas.

UNIDADE 6

A VEGETAO DA PAISAGEM

Chamamos de vegetao o conjunto de plantas e vegetais
existentes nas diversas regies do planeta. Ela sempre a
expresso das condies de solo e de clima onde ocorre. De
fato, a variedade das espcies, a quantidade de plantas, seu
tamanho e sua distribuio dependem do terreno, das
condies climticas e da umidade. Em solos ricos em
humo, portanto altamente orgnicos, a vegetao densa e
rica; nos solos mais pobres, as formaes vegetais so
poucas e rarefeitas. Existe uma relao de troca entre o solo
e a vegetao: o solo rico em materiais orgnicos gera uma
vegetao luxuriante, que fornece mais humo para o terreno.
Por sua vez, o solo com mais humo fica mais rico e
desenvolve uma vegetao que ir fornecer mais humo.
Portanto trata-se de um permanente ciclo interativo. J um
solo pobre base de uma vegetao pobre. Da o perigo do
desmatamento, que danifica os terrenos. Todo esse processo
prova a interdependncia dos elementos do ecossistema.

TIPOS DE VEGETAO
Xerfilas: plantas adaptadas falta de umidade.
Higrfilas ou higrfitas: plantas que necessitam de muita
umidade, ricas em folhagem e sempre perenes.
Tropfilas ou tropfitas: plantas que vivem em estaes
secas e midas.
Aciculifoliadas: plantas que possuem folhas na forma de
agulhas, como, por exemplo, os pinheiros, nos quais a
transpirao menor e h maior preservao de gua .
Latifoliadas: plantas de folhas largas e existentes em
regies muito midas.
Caduciflias: plantas que perdem suas folhas nos perodos
frios e secos do ano.


Please purchase PDF Split-Merge on www.verypdf.com to remove this watermark.
Incluso para a Vida Geografia A

Pr-Vestibular da UFSC

14
Exerccios de Sala #

1. (UFSC) A figura representa as vrias esferas que
constituem o nosso planeta. De acordo com ela, assinale
a(s) proposio(es) correta(s).




01.A figura busca demonstrar a interdependncia das vrias
esferas ou partes do nosso planeta.
02. Os reflexos da ao do homem vo alm e aqum da
superfcie, atingindo camadas da atmosfera e
profundidades da litosfera e da hidrosfera.
04. A importncia da litosfera, constituda de rochas, deve-se
aos solos, minerais, energia e agentes internos.
08. A atmosfera a camada gasosa que envolve o planeta, e,
de suas partes, a mesosfera a mais importante para ns.
16. A hidrosfera a parte formada pelas guas, sendo que as
necessidades humanas esto limitadas s partes mais
superficiais.

Tarefa Mnima #

2. (CEFET-PR) A vegetao original corresponde ao
conjunto de plantas que nascem e crescem naturalmente, isto
, sem a interveno do homem. Das alternativas a seguir,
que descrevem as principais formaes vegetais da Terra,
identifique a incorreta.
a) No domnio das florestas equatoriais e tropicais, o clima
permanentemente quente, e a amplitude trmica anual
no ultrapassa 6 C. As chuvas so abundantes, de
origem convectivas. Nesse ambiente de calor e umidade,
desenvolveu-se a maior variedade vegetal do globo.
b) No domnio dos desertos e semi-desertos, o total anual
de precipitao inferior a 500mm, muito mal
distribuda. Ocorre falta de umidade do ar e grandes
amplitudes trmicas dirias.
c) No domnio das florestas de folhas caducas, as
precipitaes so bem distribudas durante o ano,
primitivamente dominavam grandes extenses de
florestas abertas, de espcies caduciflias, tambm
chamadas decduas.
d) O domnio das estepes ocorre nas regies de clima
tropical mais seco, cujo perodo de chuvas ocorre no
vero. formado por plantas rasteiras, intercaladas por
vegetao arbustiva de pequeno porte, pobre em folhas e
flores.
e) No domnio das florestas de conferas, o clima frio e
mido, dominado principalmente por massas de ar
polar, com veres muito curtos e queda de neve de 3 a 6
meses ao ano. A vegetao uniforme e no renova as
folhagens anualmente, cujas partes areas so adaptadas
precipitao de neve.

3. (PUC-PR) As florestas tropicais, a despeito das leis
ambientais e da criao de parques e reservas, continuam
sendo reduzidas em suas reas, por conta da devastao de
seus recursos naturais.
Confira as caractersticas a seguir que se relacionam com as
florestas tropicais do globo:

1 - Ombrfilas.
2 - Elevada biodiversidade.
3 - Homogeneidade de espcies.
4 - Elevada pluviosidade.
5 - Espcies latifoliadas.
6 - Caduciflias.
7 - Baixo ndice de evapo-transpirao.
8 - Apresenta vrios estratos.

Assinale a alternativa que contm as caractersticas das
florestas tropicais.
a) 2; 3; 4; 7; 8.
b) 1; 2; 4; 5; 8.
c) 2; 4; 6; 7; 8.
d) 1; 3; 5; 6.
e) 3; 4; 5; 7.

5. (UNIRIO) "Manguezal ameaado - A construo de um
aterro s margens da Linha Vermelha pode ameaar uma das
ltimas reas de manguezal da Baa de Guanabara (...)"
("Jornal do Brasil")
Os constantes aterros e os despejos de esgoto residencial e
industrial so as maiores ameaas aos manguezais cariocas.
Quanto importncia desse ecossistema, correto afirmar
que:
a) so verdadeiros berrios da vida marinha, pois muitos
peixes e crustceos tm, nos manguezais, o estgio
inicial de sua cadeia alimentar.
b) servem de proteo s reas de restinga, pois diminuem
os processos de sedimentao marinha.
c) constituem essenciais fornecedores de enxofre para a
atividade petroqumica do Estado do Rio de Janeiro.
d) desempenham a funo de catalisadores de oxignio para
a formao dos bancos de coral.
e) possuem uma vegetao rica em madeira de lei, muito
utilizada na fabricao de mveis.





Please purchase PDF Split-Merge on www.verypdf.com to remove this watermark.
Incluso para a Vida Geografia A

Pr-Vestibular da UFSC

15
UNIDADE 7

ESTUDO DO MEIO AMBIENTE

Com o advento da Revoluo Industrial, a partir de 1750, na
Inglaterra, a natureza passou a ser vista no mais como um
local onde o homem poderia adaptar-se, mas sim como algo
rentvel, que se fosse explorado poderia gerar lucros.

Vejamos, ento, alguns exemplos dessa degradao:

Extrao de madeira para fins comerciais;
Instalao de projetos agropecurios;
Implantao de projetos de minerao;
Construo de usinas hidreltricas;
Propagao do fogo resultante de incndios.

Como consequncias desta agresso natureza, temos:
x destruio da biodiversidade;
x genocdio e etnocdio das naes indgenas;
x eroso e empobrecimento dos solos;
x enchentes e assoreamento dos rios;
x elevao das temperaturas globais;
x desertificao;
x proliferao de pragas e doenas.

O nvel de preocupao com a questo ambiental, tal
como se apresenta hoje, um fato muito recente na histria
da humanidade. Esse tipo de conscincia teve inicio na
dcada de 1970, sendo que at ento se limitava pratica-
mente s preocupaes sanitrias decorrentes da poluio do
ar e das guas e da mortandade de peixes resultantes desta
ltima.
Foi somente a partir da dcada de 1970 (Reunio de
Estocolmo, em 1972, patrocinada pela ONU) que a consci-
ncia ou a percepo global das reais consequncias da
utilizao de energia nuclear, dos grandes desmatamentos e
da queima de combustveis comeou a preocupar a opinio
pblica e os governantes.
Patrocinada pela ONU, uma Segunda Conferncia
mundial - a Rio 92 ou ECO 92 - debateu temas como ener-
gia nuclear, buraco na camada de oznio, efeito estufa e
biodiversidade. De grande relevncia era o acordo sobre
biodiversidade, que propunha, entre outras, duas importantes
medidas: o pagamento de royalties aos pases fornecedores
de matria prima e a transferncia de tecnologia pelos pases
do Primeiro Mundo, para os quais a biorrevoluo j uma
realidade, para os pases em desenvolvimento ou
subdesenvolvidos. Entretanto, o mais importante e
industrializado pas do mundo, -os Estados Unidos, no
assinou o acordo, embora tenha feito promessas de
intensificar o controle sobre o meio ambiente.

GRANDES PROBLEMAS AMBIENTAIS

x A poluio do ar
Com o progresso industrial, as grandes concentraes
urbanas, a queima de hidrocarbonetos, de florestas e de
pastagens, o homem joga na atmosfera milhes de toneladas
de gases e partculas slidas. A qualidade do ar nos centros
urbanos cada dia pior. Problemas respiratrios, visuais e
dermatolgicos se acentuam.
O ambiente urbano um dos mais poludos. Nele
ocorrem com grande intensidade quase todos os tipos de
poluio: sonora, visual, atmosfrica, lixo espalhado pelas
ruas, esgotos a cu aberto, cortios e favelas,
congestionamentos de trnsito etc.
A velocidade do crescimento populacional e urbano
(sobretudo nos pases subdesenvolvidos), ao lado da es-
cassez de recursos legais (leis de proteo ao meio ambi-
ente) e financeiros acarreta verdadeira degradao do ambi-
ente urbano e da qualidade de vida de milhes de pessoas
em todo o mundo.

x O problema do lixo
Nas sociedades de consumo em que vivemos atualmente, o
lixo um dos maiores problemas. Lixo domstico,
hospitalar, industrial, radioativo, todos so problemas na
medida em que as autoridades os ignoram ou os tratam de
maneira irresponsvel, o que ocorre na maioria dos casos.
Aterros sanitrios, coleta diferenciada incinerao e
proteo em containers blindados so algumas das medidas
usuais nos pases que mostram maior preocupao com esse
tipo de problema.
Infelizmente, em alguns pases menos desenvolvidos,
roedores, aves de rapina e seres humanos disputam o lixo
como fonte de alimentao.
Um bom exemplo de reciclagem de lixo urbano ocorre
em Curitiba-PR, onde as usinas de reciclagem da Prefeitura
separam e reaproveitam grande parte do lixo produzido
pelos quase dois milhes de habitantes da capital
paranaense.
Algumas maneiras de reciclar o lixo e gerar energia:
Recuperar materiais inorgnicos para
reaproveitamento:
Processar metais, plsticos e polimeros para
reutilizao em manufatura;
Recuperar hmus e protenas para raes,
fertilizantes e piscicultura.

Poluio sonora
O Homem pode suportar o limite de at 140 dB (o decibel
uma medida usada para avaliar a capacidade auditiva do ser
humano). Aps 140 dB, o tmpano humano pode sofrer
rompimento e a pessoa ser acometida de intensa dor.
O som amplificado das discotecas e danceterias, o
barulho das britadeiras, dos veculos automotores e dos avi-
es tm agredido o homem urbano com uma intensidade
nunca vista. Problemas circulatrios e do sistema nervoso
central, stress e at mesmo a loucura so os principais pro-
blemas causados pelo excesso de som.

Inverso trmica
Inverso trmica o fenmeno natural. Ocorre geralmente
no inverno e caracteriza-se pela no-circulao vertical das
camadas de ar. Em condies normais, enquanto as camadas
de ar aquecido se expandem, as camadas de ar mais frio
(mais densas) descem. Na inverso trmica ocorre ao
contrrio: o ar frio permanece estacionrio, prximo da
superfcie, enquanto o ar quente permanece por cima,
aquecido, nas camadas mais altas da atmosfera. Com a
camada de ar frio por baixo e sem se movimentar, os
Please purchase PDF Split-Merge on www.verypdf.com to remove this watermark.
Incluso para a Vida Geografia A

Pr-Vestibular da UFSC

16
poluentes que esto no ar agravam o fenmeno, causando
problemas respiratrios: ardume nos olhos, ressecamento do
nariz e da garganta, etc.

BURACO NA CAMADA DE OZNIO

A importncia do oznio
O oznio um gs de extrema importncia. Alm de suas
caractersticas qumicas, tem a funo de proteger a
superfcie terrestre das radiaes ultravioletas, o UV-B, que
faz muito mal sade das pessoas, animais e plantas. O
oznio faz, portanto, o papel de escudo protetor, o qual
repele a parte indesejvel da radiao solar.
A radiao UV-B uma pequena parte da radiao do
Sol que composta por vrias cores, principalmente verde,
violeta e vermelho. Alm da cor violeta, h a ultravioleta,
que chama-se UV-B, e justamente esta parte que
prejudicial aos organismos vivos na Terra.

As chuvas cidas
A combinao de gs carbnico (CO
2
) e gua (H
2
0),
presentes na atmosfera produz o cido Carbnico (H
2
C0
3
),
que embora fraco, j torna as chuvas normalmente cidas.
A chuva cida, to prejudicial aos organismos vivos,
nada mais do que a elevao drstica dos teores de acidez
na atmosfera e so causadas pelo lanamento de poluentes
como dixido de enxofre (502), emitido a partir da queima
de combustveis fsseis e do oxignio (02), j presentes na
atmosfera, bem como do dixido de enxofre (NO
2
), que aps
combinarem-se formam o trixido de enxofre (503),
altamente poluente, por exemplo.
A concentrao de trixido de enxofre aumentou na
atmosfera, como resultado da ampliao do uso de combus-
tveis fosseis nos transportes, nas termoeltricas e nas in-
dstrias.
Os pases que mais emitem poluentes para a atmosfera
so aqueles situados no Hemisfrio Norte, portanto,
industrializados, como o Nordeste dos Estados Unidos, alm
do Japo e Europa Ocidental.

A Ilha de calor
As cidades apresentam temperaturas mdias maiores que as
zonas rurais de mesma latitude. Dentro delas, as
temperaturas aumentam da periferia em direo ao centro.
Em casos extremos, a diferena de temperatura entre as
zonas perifricas e o centro pode atingir at 10 C, esse
fenmeno, conhecido como ilha de calor, resulta, muitas ve-
zes, de alteraes humanas sobre o meio ambiente.
O uso de grandes quantidades de combustveis fsseis
em aquecedores, automveis e indstrias transforma a
cidade em uma fonte inesgotvel de calor. Os materiais usa-
dos na construo, como o asfalto e o concreto, servem de
refletores para o calor produzido na cidade e para o calor
solar. De dia, os edifcios funcionam como um labirinto de
reflexo nas camadas mais altas de ar aquecido. noite, a
poluio do ar impede a disperso do calor.






Tarefa Mnima #

1. (UFMA)
(...)
Essa uma possibilidade
Em que poucos podem ter pensado
A gua engolindo e tragando as cidades
Tragdias surgindo por todos os lados.
(...)
Efeito estufa, poluio crescente,
Temperaturas ganham intensidade,
A gua avana sobre os continentes,
Distrbios explodem com ferocidade.
Fogo e gua,
(Tribo de Jah.)

Com base no contedo das estrofes apresentadas, pode-se
firmar que:
a) O efeito estufa perde intensidade com o crescimento
vertical das cidades.
b) O desequilbrio ecolgico, nos centros urbanos, o
principal responsvel pelas enchentes urbanas.
c) O avano da gua sobre os continentes um fenmeno
de origem antrpica.
d) O aumento da temperatura responsvel pela frequncia
dos desastres ecolgicos.
e) O homem mantm o equilbrio ecolgico com os
recursos tcnicos-cientficos.

2. (UFSE-adaptada) O efeito estufa um fenmeno
conhecido desde o final do sculo XIX, quando alguns
cientistas comearam a se preocupar com a interferncia das
atividades humanas no equilbrio trmico da atmosfera.
Sobre o efeito estufa pode-se afirmar que:

01. Os maiores responsveis pelos gases estufa so os pases
pobres do hemisfrio Sul que ainda utilizam a lenha
como fonte de energia.
02. Os pases da Unio Europeia so os que mais resistem
diminuio das emisses de gases estufa, apesar das
fortes presses dos Estados Unidos que j diminuram
cerca de 20% destas emisses.
04. J ficou demonstrado que o principal fator gerador de
gases estufa so as queimadas nas reas florestais,
chamadas com propriedade de pulmes do mundo.
08. Na dcada de 1990, em uma conveno realizada na
cidade de Kioto (Japo), foi fixada como meta prioritria
a reduo de gases estufa pelos pases desenvolvidos.
16. Um dos possveis efeitos deste fenmeno seria o
derretimento de parte da calota polar inundando
inmeras cidades costeiras.

3. (UFSC) A questo ambiental tem-se caracterizado como
uma das grandes preocupaes do mundo moderno. Muitos
dos recursos utilizados na produo industrial so extrados
diretamente da natureza, causando-lhe prejuzos por vezes
incalculveis. Hoje, bastante corrente a reutilizao e/ou
reciclagem de muitos produtos, bem como uma maior
preocupao com medidas anti-poluio, alm de uma
melhoria na educao, quando se refere questo do meio-
ambiente. Tudo isso como medida para que no futuro
Please purchase PDF Split-Merge on www.verypdf.com to remove this watermark.
Incluso para a Vida Geografia A

Pr-Vestibular da UFSC

17
possamos ter um ambiente propcio continuidade das
atividades econmicas e, principalmente, vivel prpria
vida.

Com relao a essa temtica, correto afirmar que:

01. A preocupao com a degradao ambiental legtima e
oportuna, pois muitos recursos necessrios vivncia
humana podero se esgotar em pouco tempo.
02. A inquietao por questes ambientais um exagero,
fruto apenas de discusses de inmeros grupos
ecolgicos radicais.
04. Com um sistema socioeconmico voltado
principalmente produo de mercadorias, visando
basicamente ao lucro, torna-se difcil, sob o capitalismo,
a no degradao dos recursos naturais.
08. No h problemas quanto aos recursos minerais, pois os
estudos garantem, para qualquer caso (gua, minrios,
fontes de energia trmica) reservas suficientes para os
prximos 500 anos.
16. Com a queda no processo de urbanizao, diminuiro,
vertiginosamente, os problemas socioambientais, tanto
nas cidades quanto no meio rural.

4. (ENEM) Quanto mais desenvolvida uma nao, mais
lixo cada um de seus habitantes produz. Alm de o
progresso elevar o volume de lixo, ele tambm modifica a
qualidade do material despejado. Quando a sociedade
progride, ela troca a televiso, o computador, compra mais
brinquedos e aparelhos eletrnicos. Calcula-se que 700
milhes de aparelhos celulares j foram jogados fora em
todo o mundo. O novo lixo contm mais mercrio, chumbo,
alumnio e brio. Abandonado nos lixes, esse material se
deteriora e vaza. As substncias liberadas infiltram-se no
solo e podem chegar aos lenis freticos ou aos rios
prximos, espalhando-se pela gua.
"Anurio Gesto Ambiental" 2007, p. 47-8 (com adaptaes).

A respeito da produo de lixo e de sua relao com o
ambiente, correto afirmar que:

a) As substncias qumicas encontradas no lixo levam,
frequentemente, ao aumento da diversidade de espcies
e, portanto, ao aumento da produtividade agrcola do
solo.
b) O tipo e a quantidade de lixo produzido pela sociedade
independem de polticas de educao que proponham
mudanas no padro de consumo.
c) A produo de lixo inversamente proporcional ao nvel
de desenvolvimento econmico das sociedades.
d) O desenvolvimento sustentvel requer controle e
monitoramento dos efeitos do lixo sobre espcies
existentes em cursos d'gua, solo e vegetao.
e) O desenvolvimento tecnolgico tem elevado a criao de
produtos descartveis, o que evita a gerao de lixo e
resduos qumicos.






UNIDADE 8

ASPECTOS DEMOGRFICOS

Demografia o estudo estatstico das populaes humanas.
O estudo da populao humana sobre a superfcie terrestre
sempre despertou interesse por todos os governantes ou
pessoas detentoras de poder. Na tentativa de explicar a
importncia e a preocupao dos governantes com o au-
mento das populaes, citamos a teoria elaborada por
MALTHUS, em 1798, conhecida como teoria demogrfica,
onde ele afirma que a misria e a pobreza so resultados do
desequilbrio entre os recursos naturais e a populao, pois
dizia que:
A populao, sem limitaes, aumenta na proporo
geomtrica. Os meios de subsistncia aumentam somente
em proporo aritmtica. Um pequeno conhecimento dos
nmeros mostrar a imensidade do primeiro poder em
comparao com o segundo
Logo aps o trmino da Segunda Guerra Mundial
(1939-1945), tornou-se fato novamente a preocupao com
o crescimento populacional. A Teoria Neomalthusiana
relacionava o rpido crescimento demogrfico de muitos
pases subdesenvolvidos, resultantes das altas taxas de
natalidade, que acabava gerando numa grande quantidade de
jovens, o que acarretaria aos pases em questo um nus
muito grande para a populao ativa, dificultando o
desenvolvimento econmico. Defendiam os
neomalthusianos que se ocorresse um aumento na renda per
capita, haveria, consequentemente, crescimento econmico e
desenvolvimento.
No mesmo perodo foi lanada a teoria reformista
que pregava que a pobreza nos pases subdesenvolvidos era
ocasionada pela pssima distribuio da renda. Cabia ao
Estado o papel de redistribuir a riqueza, proporcionando
melhores condies de vida a toda a populao, desse modo,
a taxa de natalidade seria reduzida espontaneamente.

DINMICA POPULACIONAL
D-se o nome de populao ao conjunto de pessoas que
residem em determinado territrio, que pode ser uma cidade,
um estado, um pas ou mesmo o planeta.
Podemos classific-la de acordo com a religio, a
nacionalidade, o local de moradia (rural ou urbana), a
atividade econmica (ativa ou inativa). J seu
comportamento e suas condies de vida, so retratados
atravs de indicadores sociais, tais como:

x taxas de natalidade,
x taxa de mortalidade,
x expectativa de vida,
x ndices de analfabetismo,
x participao na renda, etc.

Populao absoluta
Corresponde ao total de habitantes de determinado lugar
(pas, estado, municpio).



Please purchase PDF Split-Merge on www.verypdf.com to remove this watermark.
Incluso para a Vida Geografia A

Pr-Vestibular da UFSC

18
Populao relativa
Corresponde ao nmero de habitantes de determinado lugar,
por quilmetro quadrado.

ESTRUTURA DA POPULAAO

Composio por idade e sexo
A populao dos pases do mundo pode ser dividida em trs
nveis de idade, o que chamamos de pirmide etria, ficando
assim distribuda:

Adultos 20 a 59 anos
Jovens 0 a19 anos
Velhos 60 anos a mais

Existe tambm outra diviso, na qual a populao
jovem abrange de 0 a 14 anos e os adultos de 15 a 59 anos.
No entanto, o que foi levado em considerao nesta
classificao foi a Populao Economicamente Ativa
(populao que recebe remunerao por seu
trabalho).Observe as pirmides etrias abaixo:





















Na pirmide A nota-se que sua base mais larga,
pois apresenta um elevado nmero de jovens em sua
estrutura etria. Caracteriza um pas subdesenvolvido.
Na pirmide B nota-se que h maior homogeneidade
entre as divises. Acentua-se o nmero de pessoas com mais
de 60 anos (velhos), o que caracteriza a elevada expectativa
de vida.

Crescimento Vegetativo = Taxa de Natalidade Taxa de
Mortalidade

Taxa de natalidade: a relao entre o nmero de
nascimentos e a populao de um pas na proporo de
1/1000.
Taxa de mortalidade: a relao entre o nmero de
bitos e a populao de um pas na proporo de 1/1000.
No Brasil, o crescimento vegetativo est em torno de
18,9%
0
, pois a natalidade est em 26,8%
0
e a mortalidade
em 7,9%
0
, isso quer dizer que a cada ano a populao
brasileira aumenta em torno de 18,9%.

Causas do aumento vegetativo
x Melhoria das condies de higiene,
x Progressos na medicina,
x Modernizao das mquinas agrcolas, as quais, em
tese, podem produzir mais alimentos.

No entanto, a situao aqui relatada refere-se a pases
em transio demogrfica, como o caso do Brasil, visto
que no podemos esquecer que o grande problema de nosso
pas a pssima distribuio de renda e o desemprego que
assola milhes de famlias desse imenso territrio.

Exerccios de Sala #

1. Um estudo sobre a dinmica e a distribuio da
populao de uma determinada rea realizado a partir do
conhecimento e da compreenso dos seus indicadores
demogrficos. Em relao a alguns desses indicadores, some
as alternativas corretas:

01. A densidade demogrfica obtida a partir da diviso da
superfcie territorial de um lugar pela sua populao
absoluta.
02. O crescimento vegetativo calculado com base nas taxas
de natalidade, mortalidade e migrao.
04. O superpovoamento de uma rea no identificado
apenas pela densidade demogrfica mas tambm pelas
condies socioeconmicas existentes.
08. A taxa de mortalidade infantil identifica o nmero de
bitos de crianas menores de um ano.
16. A taxa de fecundidade um indicador populacional que
influencia diretamente o comportamento de um outro
indicador, o da natalidade.

2. (UFPEL) Observe a figura que representa a distribuio
etria da populao mundial



O modelo de transio demogrfica explica as alteraes nas
taxas de natalidade e de mortalidade dos pases, conforme
passam por diferentes estgios da economia industrial.

correto afirmar que:
a) H um equilbrio entre as taxas de natalidade e de
mortalidade no incio da transio demogrfica. O
nmero de nascimentos e de bitos apresenta valores
ainda pouco expressivos, correspondendo fase
denominada de pr-industrial.
Please purchase PDF Split-Merge on www.verypdf.com to remove this watermark.
Incluso para a Vida Geografia A

Pr-Vestibular da UFSC

19
b) Os pases desenvolvidos, que tiveram sua industrializao
entre o fim do sculo XVIII e incio do XIX, apresentam
uma queda na taxa de mortalidade devido aos avanos na
medicina e na tecnologia. Nessa fase, a segunda da
transio demogrfica, ocorre um crescimento
populacional em todas as faixas etrias.
c) H um novo equilbrio entre as taxas de natalidade e de
mortalidade, com valores elevados, quando a transio
demogrfica atinge a quarta fase. Os nascimentos so
altos, mas equilibrados com as altas taxas de
mortalidade. o caso dos pases de industrializao
tardia, como os da Amrica Latina e sia.
d) Alguns pases experimentam uma fase de crescimento
negativo da populao, quando a taxa de natalidade
menor que a de mortalidade. Esse fato prprio de
pases muito pobres, com polticas sociais precrias e
que incentivam as famlias a no terem filhos.
e) Os processos de transio demogrfica apontam para o
envelhecimento da populao dos pases. Os dois
extremos de riqueza apresentam preocupaes: os mais
pobres sofrem com o desemprego em massa, e os mais
ricos sentem a falta de jovens para o trabalho.

Tarefa Mnima #

3. (UFPEL) O relgio da populao mundial, mecanismo
criado pelo censo dos Estados Unidos para contar a
populao da Terra e o ritmo de crescimento demogrfico,
indicou em 25 de fevereiro de 2006 que o mundo atingiu a
marca de 6,5 bilhes de pessoas.
Nas regies mais pobres do Planeta, como frica, ndia e
Oriente Mdio, a populao cresce mais rpido. Nos pases
industrializados da Europa, a populao est diminuindo.
Entretanto, na soma, a Terra est ficando cada vez mais
lotada.

Com relao ao crescimento populacional ou demogrfico,
correto afirmar que:
a) a taxa de migrao o nmero de pessoas que se desloca
de uma rea para outra no mesmo pas.
b) o crescimento natural ou vegetativo corresponde
diferena entre a entrada e a sada de pessoas de uma
determinada rea.
c) a taxa de fecundidade, que representa o nmero de filhos
por mulher, no interfere no crescimento demogrfico.
d) o crescimento vegetativo corresponde diferena entre
nascimentos e bitos em uma dada populao.
e) so populosos os pases cuja populao relativa muito
elevada, e povoados aqueles que possuem grande
populao absoluta.

4. (ENEM) A tabela a seguir apresenta dados relativos a
cinco pases.


Com base nessas informaes, infere-se que:
a) A educao tem relao direta com a sade, visto que
menor a mortalidade de filhos cujas mes possuem maior
nvel de escolaridade, mesmo em pases onde o
saneamento bsico precrio.
b) O nvel de escolaridade das mes tem influncia na sade
dos filhos, desde que, no pas em que eles residam, o
abastecimento de gua favorea, pelo menos, 50% da
populao.
c) A intensificao da educao de jovens e adultos e a
ampliao do saneamento bsico so medidas suficientes
para se reduzir a zero a mortalidade infantil.
d) Mais crianas so acometidas pela diarreia no pas III do
que no pas II.
e) A taxa de mortalidade infantil diretamente proporcional
ao nvel de escolaridade das mes e independe das
condies sanitrias bsicas.

5. (PUC-PR) "O governo francs ir pagar uma licena de
750 euros (cerca de R$ 2.050,00) por ms, durante um ano, a
famlias que decidirem ter um terceiro filho, anunciou ontem
o primeiro-ministro do pas, Dominique de Villepin."
("Folha de S. Paulo", 23.09.2005)

A reportagem acima ilustra uma poltica cada vez mais
comum entre os pases europeus. As alternativas abaixo
contm possveis causas que motivam a adoo de tais
medidas, exceto:
a) As baixas taxas de natalidade de muitos pases europeus.
b) As altas taxas de mortalidade europeias, que resultam na
diminuio da PEA - populao economicamente ativa.
c) A tentativa de evitar que num futuro a mdio prazo a
populao nativa possa tornar-se minoritriadiante da
populao imigrante - cujas taxas decrescimento
vegetativo so bem mais altas.
d) O impacto que a diminuio da mo de obra ativa est
causando ao sistema previdencirio europeu.
e) A difcil tarefa dos dirigentes da Unio Europeia em
administrar a necessidade de manunteno de um fluxo
controlado de movimentos populacionais horizontais ao
mesmo tempo em que tenta reprimir o aumento da
xenofobia.







Please purchase PDF Split-Merge on www.verypdf.com to remove this watermark.
Incluso para a Vida Geografia A

Pr-Vestibular da UFSC

20
6. (PUC-RS) Responda a questo com base nas pirmides
etrias e nas afirmativas, que tratam do tema populao.


I. A pirmide B apresenta uma configurao que demonstra
o aumento das taxas de natalidade nas trs ltimas
dcadas.
II. A pirmide D evidencia uma estrutura estvel, embora
deva ter passado por uma situao de guerra, pois h uma
diminuio considervel no nmero de homens adultos.
III. A pirmide A apresenta uma contradio tpica de pases
pobres: alta natalidade e mortalidade infantil,
acompanhadas de uma elevada expectativa de vida da
populao.
IV. A pirmide C indica elevado ndice de expectativa de
vida e a pirmide A, elevada taxa de mortalidade.

Com base nas pirmides e nas afirmativas, conclui-se que
somente esto corretas:
a) I e II
b) I, II e III
c) I e III
d) II e IV
e) III e IV

UNIDADE 9

MOBILIDADE DA POPULAO MUNDIAL
AS MIGRAES

MIGRAES INTERNAS
Nomadismo: deslocamento de um grupo humano em
busca de alimentos (povos selvagens ou melhores pastagens
para seus rebanhos (pastores nmades) retornando quase
sempre ao ponto de partida.
Transumncia: deslocamento cclico de pessoas entre
dois lugares, geralmente por motivos climticos.
xodo rural: trata-se do deslocamento de pessoas das
reas rurais para as reas urbanas. Caracterstico de pases
subdesenvolvidos.
Migrao pendular (Communtig): Trata-se do des-
locamento dirio entre a periferia e o centro das grandes
cidades, consistindo num vai-vem (manh - para o centro /
tarde - para a periferia).

MIGRAES EXTERNAS
Os movimentos demogrficos ocorrem em todas as partes do
mundo, e as causas so muito conhecidas.
Podemos citar as guerras, as enchentes, a fome e as
calamidades com as quais a humanidade sempre conviveu.



Exerccios de Sala #

1. (PUC-MG) "Para produtos e capital, a globalizao
acabou com as fronteiras. Para pessoas, no."
("poca", 03/07/06, p.38)

A diferena de nvel de vida leva milhares de habitantes de
pases pobres a tentar a busca de novas oportunidades nos
pases ricos, gerando problemas de imigrao. Nesse
contexto, incorreto afirmar:
a) A motivao que impulsiona o processo migratrio, antes
de ser um processo de expulso pela falta de perspectiva
de trabalho no pas de origem, resultante da atrao que
os pases ricos exercem sobre os pases pobres.
b) As restries impostas migrao tm gerado polticas
cada vez mais rgidas de controle de fronteiras,
condicionando o surgimento de inmeros itinerrios de
rotas no convencionais.
c) A acolhida e a rejeio da mo-de-obra do imigrante so
partes de um mesmo processo: pases ricos precisam de
sua fora de trabalho para tarefas menos qualificadas ao
mesmo tempo em que desenvolvem resistncia contra
suas caractersticas culturais/tnicas.
d) As migraes internacionais no foram estancadas pelo
processo de globalizao, pois, a despeito de uma
considervel gerao de trabalho nos pases, fonte de
migrantes, a globalizao tem contribudo, dentre outras
coisas, para a primarizao das economias, concentrao
de renda e ampliao das desigualdades sociais nos
pases perifricos.
2. (UFMG) Considerando-se os reflexos das migraes
internacionais na organizao do espao mundial,
incorreto afirmar que, na atualidade, h
a) Um aumento de aes decorrentes da xenofobia que
caracteriza parcela da populao dos pases receptores de
imigrantes.
b) Um crescimento do contingente de imigrantes ilegais, o
que tem favorecido a criao de leis que dificultam e
criminalizam a presena deles nos pases receptores.
c) Uma plena integrao cultural e socioeconmica, no pas
receptor, das geraes posteriores de imigrantes,
tornadas cidados nacionais.
d) Uma tendncia mudana do perfil tnico, nos pases
receptores, em razo do nmero de imigrantes recebidos
e de seu comportamento demogrfico diferenciado.

Tarefa Mnima #

3. (UFPel) BRASILEIROS EM FUGA

A via mexicana tem sido a alternativa dos imigrantes
que desistiram de obter o visto americano ou no tiveram
coragem de recorrer a falsificaes. Para chegar l, o
viajante pode contratar, ainda no Brasil, os servios dos
traficantes de gente ou ir por conta prpria. Somando-se
gastos com passagem area, transporte at a fronteira,
hospedagem e alimentao, os custos da aventura podem
chegar aos 10 mil reais.
Ricardo Amorim, "Veja", 07/02/2001 [adapt.].

Please purchase PDF Split-Merge on www.verypdf.com to remove this watermark.
Incluso para a Vida Geografia A

Pr-Vestibular da UFSC

21
Analise as seguintes afirmativas, a respeito da dinmica da
populao.

I. Desde os primrdios da histria humana, ocorrem
movimentos migratrios: grupos humanos abandonam o
lugar onde vivem, motivados por perseguies de carter
religioso ou poltico, por causas naturais ou em busca de
melhores condies de vida.
II. Para os Estados Unidos, fluxos migratrios significativos
envolvem os latino-americanos, como os "braceros" (do
Mxico), os "balseros" (do Caribe) e os "brazucos"
(brasileiros que migram, em busca de emprego e
melhores condies de vida).
III. Na atualidade, os movimentos da populao envolvem
aspectos preocupantes, como o trfico de imigrantes e o
trfico de mulheres, as quais, seduzidas com promessas
de altos salrios, acabam envolvidas em redes de
prostituio.
IV. A emigrao de brasileiros cresceu a partir da Revoluo
de 1964 - a chamada dcada perdida; apesar disso, o
Brasil no alterou sua condio histrica de pas receptor
de estrangeiros.
V. A atual fase de migrao de brasileiros - indita e
expressiva em relao ao total da populao -
consequncia do processo de globalizao, que
aumentou a escala geogrfica das relaes
socioeconmicas, consolidando, no Brasil, as reas de
repulso.

Assinale a alternativa com o conjunto de afirmativas
verdadeiras.
a) I, II e III
b) II, III e IV
c) I, II e V
d) II, III e V
e) I, III e IV

UNIDADE 10

GEOGRAFIA URBANA

Processo De Urbanizao
Durante milhares de anos da existncia humana na Terra a
vida foi, em sua grande maioria, ditada pelas atividades
rurais.
Este quadro comea a mudar a partir da Revoluo
Industrial (1750 - Inglaterra), quando os pases, medida
que se industrializavam, rapidamente atraiam multides para
as cidades.

Origem Das Cidades

Naturais ou espontneas
Trata-se de aglomerados urbanos que surgiram sem nenhum
planejamento prvio, surgindo como povoados,
transformando-se em vilas e evoluindo para as cidades.
Exemplos: Curitiba (Arraial de Minerao), Recife
(porto de pescadores).
Cidades artificiais ou planejadas
So cidades que foram projetadas para funes es-
pecficas.
Exemplos: Brasilia (Capital Federal), Palmas (Capital
do Estado de Tocantins).

Funes das cidades
Administrativas: Florianpolis, Braslia,
Washington;
Comerciais: Santos, Campina Grande, Feira de
Santana;
Industriais: Volta Redonda, Baltimore, Detroit, So
Bernardo do Campo;
Universitrias: Oxford, Cambridge;
Religiosas: Meca, Aparecida do Norte, Ftima;
Estaes de Sade: Arax, Poos de Caldas, Caldas
Novas.
Tursticas: Long Beach, Acapulco, Rio de Janeiro.

Problemas urbanos
Abastecimento de gua
Abastecimento de alimentos
Energia Eltrica
Transportes
Poluio
Habitao
Higiene Saneamento
Violncia, etc.

Exerccios de Sala #

1. (UFPE) As cidades no mundo vm apresentando, desde
os meados do sculo XX, e particularmente nas ltimas
dcadas, significativas mudanas na sua forma (morfologia),
fatos que representam, influenciaram ou esto associados a
estas mudanas, so:
01. A disseminao do uso do transporte individual
automotivo, requerendo vias largas e amplas reas de
estacionamento.
02. A presena das grandes estruturas do varejo,
denominadas shopping centers, que passaram a competir
com os tradicionais centros das cidades que, em muitos
casos, entraram em decadncia.
04. O desenvolvimento das vias de contorno (perimetrais),
na circulao urbana das mdias e grandes cidades, vias
estas ligando os subrbios espalhados e revelando a
crescente importncia de outros roteiros de deslocamento
dentro da cidade, para trabalho, compras, servios, etc.,
que no aqueles deslocamentos radiais da populao em
direo ao centro.
08. A formao dos corredores de servios e comrcio, seja
nas vias radiais seja nas de contorno, como espcies de
centros alongados, que competem com o velho centro e
pressupem uso amplo do transporte individual.
16. A criao de seces ou zonas funcionais nas grandes
cidades fora do centro tradicional mas beneficiadas por
acesso com concentrao de servios, como sade,
finanas, seguros, consultorias, imobilirias, escritrios
de firmas em geral, transportadoras, servios etc.

Please purchase PDF Split-Merge on www.verypdf.com to remove this watermark.
Incluso para a Vida Geografia A

Pr-Vestibular da UFSC

22
Tarefa Mnima #

2. (UFG-GO) A urbanizao dos pases subdesenvolvidos
constitui um fenmeno marcante da segunda metade do
sculo XX. As caractersticas desse fenmeno, na Amrica
Latina, expressas na paisagem urbana das metrpoles, so
decorrentes da:
a) Instalao de indstrias de bens de produo nos
arredores das pequenas cidades e prximas s fontes de
matria-prima.
b) Industrializao tardia e da modernizao das atividades
agrcolas, conjugadas concentrao de pessoas nas
grandes cidades.
c) Aglomerao humana e do aumento do poder aquisitivo
da populao, favorecidos pela expanso do capital
financeiro na economia.
d) Inovao tecnolgica e do aumento da produtividade das
indstrias de bens de consumo, para suprirem as
necessidades da vida urbana.
e) Implementao de parque industrial e da regulao, por
meio do planejamento governamental, de deslocamentos
populacionais para as cidades.

3. (UFPR) Na Geografia, o termo polarizar significa atrair,
influenciar, fazer convergir para si. Assim, para que uma
determinada rea possa exercer as funes de polo, precisar
concentrar um nmero considervel de atividades e de
recursos capazes de influenciar processos que ocorrem em
outras reas. Com base no texto e nos conhecimentos de
Geografia, assinale a alternativa incorreta.
a) O poder de polarizao de uma cidade est associado ao
tamanho de sua populao.
b) A implantao de indstrias numa cidade pode ampliar o
poder polarizador dela ao atrair novos investimentos
industriais e criar encadeamentos produtivos com
indstrias de outras cidades.
c) No contexto da globalizao, o poder polarizador das
grandes metrpoles faz com que elas assumam a funo
de elos privilegiados entre as economias nacionais e o
exterior.
d) A polarizao faz com que a populao de alta renda
empregada na indstria e nos servios resida nas
metrpoles, enquanto que a pobreza se localize nas
pequenas e mdias cidades no metropolitanas.
e) A presena de cidades com forte capacidade de
polarizao essencial para a articulao da rede urbana,
motivo pelo qual essa rede menos estruturada nas
regies pouco desenvolvidas.

4. (UFPEL) O processo contemporneo de urbanizao tem
produzido uma srie de espaos caractersticos, que
evidenciam as transformaes sociais. A rede urbana um
sistema de cidades formado pela conexo dos sistemas de
transporte e das comunicaes entre cidades, por onde
ocorre o fluxo de pessoas, mercadorias, informaes e
capitais. assim, nos diferentes pases e regies do mundo,
podem ser observados, por exemplo, espaos como os
caracterizados a seguir.
I. conjunto de cidades conurbadas ligadas pela expanso da
periferia da malha urbana, nas quais se observa um
municpio-ncleo.
II. Unio entre duas ou mais metrpoles, com integrao do
fluxo de pessoas, capitais, informaes, mercadorias e
servios.
III. Cidades que so sedes de importantes empresas com
grande tecnologia, conectadas aos fluxos do espao
mundial.
IV. Cidades ou aglomeraes com mais de 10 milhes de
habitantes.
Com base em seus conhecimentos e nas informaes
anteriores, correto afirmar que os espaos acima se
referem, respectivamente, :
a) conurbao, megalpole, metrpole nacional e cidades
globais.
b) metrpole, regio metropolitana, cidades globais e
megalpole.
c) metrpole, megalpole, cidades globais e megacidades.
d) megalpole, cidades globais, capitais nacionais e
metrpole.
e) regio metropolitana, conurbao, capital regional e
megacidades.

UNIDADE 11

AGROPECURIA

A produo agropecuria atual precisa ser estudada dentro
do funcionamento da economia globalizada. Os produtores
rurais esto se tornando empresrios e produzindo aquilo
que d mais lucro e que tem mercado em todo o mundo.
Assim, existem pases que esto entre os maiores
exportadores de algum gnero agrcola e, ao mesmo tempo,
possuem uma grande quantidade de pessoas passando fome.

2.1-Os diferentes Agrossistemas
Consolidam-se nos tipos de cultivo ou de criao que sero
produzidas as espcies de plantas e/ou raas de animais,
assim como as tcnicas envolvidas na produo agrcola ou
na pecuria, alm de analisar o tamanho das propriedades
rurais e o nvel tecnolgico. As propriedades rurais so
classificadas segundo o nvel tecnolgico aplicado na
pecuria e agricultura. Com isso, os Agrossistemas podem
ser:

Agropecuria Tradicional: a atividade sem uso de
tecnologia moderna. Ainda persiste nas mos de pequenos
produtores, mas tem dificuldade de se manter no mercado
quando disputam os produtores modernizados. A criao de
gado extensiva, ou seja, gado criado solto com poucos
investimentos com a gentica, com a sade animal. As
plantaes so feitas sem utilizao de defensivos agrcolas,
as sementes no so selecionadas, as tcnicas praticadas so
rudimentares como arado de trao animal, com produo
baixa pela falta de modernizao.

Agropecuria Moderna: a atividade com o uso de
tecnologia de ponta. A criao de gado intensiva, ou seja,
Please purchase PDF Split-Merge on www.verypdf.com to remove this watermark.
Incluso para a Vida Geografia A

Pr-Vestibular da UFSC

23
cuidados com a gentica, analisando as vantagens da criao
de uma determinada raa, utilizao de medicamentos, alm
de acompanhamento de um veterinrio.
O cultivo agrcola tambm intensivo, ou seja, alta
produtividade em menos terras cultivadas, isso ocorre
porque a produo estruturada nas mais modernas tcnicas
e mquinas. Nesse tipo de produo realizado,
primeiramente, a correo do solo; so observadas as
previses do tempo para executar o plantio; as sementes so
selecionadas, imunes a pragas e tambm adaptadas ao clima;
h aplicao de fertilizantes, alm do acompanhamento de
um agrnomo; e o trabalho de plantio e colheita realizado
por modernos tratores e colheitadeiras, garantindo alta
produtividade.

2.2 -Plantations
So grandes propriedades rurais monocultoras, ou seja,
cultivam uma nica cultura com produo destinada
exportao. As plantations so heranas do perodo colonial
de vrios pases das Amricas, frica e sia. Utilizando
mo de obra escrava, eram responsveis pela produo de
produtos tropicais muito apreciados na Europa e que
enriqueceram as metrpoles na poca

2.3-Agrossistemas Alternativos
Representa uma forma de produo ecologicamente correta
para amenizar os problemas sociais e ambientais. Nesse
sistema, busca-se a eliminao de agrotxico e chamado
de produo orgnica, atualmente o produto orgnico tem
conseguido um valor mais elevado por seus produtos, o
preo maior devido qualidade dos produtos, pois so
mais saudveis, no h adio de substncias qumicas, pois
o combate s pragas e os fertilizantes so feitos com
controle biolgico, ou seja, agentes que no so prejudiciais
ao organismo e natureza. A produo alternativa pratica a
policultura (cultivo de vrias culturas) e jamais monocultura
(cultivo de uma nica cultura). Os objetivos so alimentos
saudveis e equilibro ambiental, diminuio do xodo rural e
do desemprego.

Exerccios de Sala #

1. Sobre a atividade agropecuria, no Brasil e no mundo,
possvel afirmar, de forma correta, que:
01. Com a modernizao das tcnicas agrcolas e o uso da
biotecnologia, elevaram-se os ndices de produtividade
agrcola e a quantidade de trabalhadores rurais no
mundo.
02. Nos pases subdesenvolvidos, o progresso tcnico-
cientfico atingiu a atividade com mais intensidade nas
regies especializadas na produo para o mercado
externo.
04. O uso de produtos agrcolas transgnicos questionado e
proibido em todo o mundo, por seus efeitos sobre a
sade humana serem ainda desconhecidos.
08. enquanto no mundo desenvolvido aumenta o uso de
mo-de-obra na agricultura, o xodo rural esvazia
permanentemente o campo, nos pases subdesenvolvidos.
16. A atuao do MST contribuiu para a reduo da
concentrao fundiria que marco principal da reforma
agrria que est sendo efetivada no Brasil.

2. (PUC-SP) Na Rodada Doha da Organizao Mundial do
Comrcio (OMC), realizada em Cancn (Mxico) no ano de
2003, o Brasil e mais 19 pases em desenvolvimento
protagonizaram um movimento contra a poltica de
subsdios agrcolas, desenvolvida pelos pases europeus e
pelos EUA, que beneficia os agricultores desses pases
desenvolvidos. A respeito desse desacordo no comrcio
mundial correto afirmar que:

a) A reivindicao do fim dos subsdios pretende fazer valer
no mercado internacional a maior produtividade nos
negcios agropecurios dos pases em desenvolvimento,
o que se deve tecnologia mais avanada empregada no
processo produtivo.
b) Os pases em desenvolvimento optaram por ser
exportadores de commodities (produtos agropecurios,
minrios, madeiras etc.) em funo desse comrcio ser
mais valorizado no mercado internacional, por causa da
escassez de terras agrcolas nos pases desenvolvidos.
c) O combate aos subsdios agrcolas vem de setores cada
vez mais minoritrios no interior dos pases em
desenvolvimento, visto que a maioria deles, o Brasil
inclusive, est abrindo mo dos commodities e
especializando-se em bens industriais, com alto valor
agregado.
d) Os enormes subsdios agrcolas aos agricultores dos
pases desenvolvidos so uma forma de protecionismo
("fechamento") de seus mercados internos, o que
contraria a abertura muitas vezes exigida dos mercados
dos pases em desenvolvimento.
e) A participao modesta (e cada vez menor) dos pases em
desenvolvimento no mercado internacional no est
relacionada s polticas protecionistas dos pases
desenvolvidos, mas sim a grande ineficincia produtiva,
o que os torna isolados no contexto da globalizao.

Tarefa Mnima #

3. (Mackenzie) A biotecnologia utilizada para estimular o
aumento da produtividade no campo tem sido aplicada j h
algum tempo e a avaliao de seus resultados tem sido
controvertida por inmeros problemas surgidos, exceto:
a) Por ser uma tecnologia de baixo custo, pode ser utilizada
pelos pases pobres.
b) Porque as novas variedades so produzidas por grandes
corporaes.
c) Porque a utilizao das novas variedades s possvel
atravs da compra de patentes e pacotes tecnolgicos.
d) Porque ainda no esto completamente definidos os riscos
sade provocados pelo consumo desses produtos.
e) Porque a homogeneidade das espcies cultivadas torna
toda safra vulnervel a uma nica praga ou doena.



Please purchase PDF Split-Merge on www.verypdf.com to remove this watermark.
Incluso para a Vida Geografia A

Pr-Vestibular da UFSC

24
4. As novas biotecnologias e a engenharia gentica
aplicadas agricultura so relativamente recentes e,
portanto, ainda sujeitas a debates, baseados em diferentes
dimenses tericas, dentre as quais cita-se a
a) Geopoltica, pois inmeros pases tropicais do Sul tm
desenvolvido essas pesquisas e garantido lugar de
destaque no mundo cientfico.
b) Ambiental, pois suas bases esto relacionadas ao uso de
material orgnico e no agrotxico e, portanto,
ecologicamente correta.
c) Pedolgica, pois a grande fonte de preocupao destas
pesquisas proteger o solo, considerado elemento
fundamental para qualquer agrossistema.
d) Social, pois so difundidas como instrumentos eficientes
para a diminuio da fome e das carncias alimentares da
populao.
e) Econmica, pois visa reduzir a dependncia alimentar de
boa parte da populao mundial, visto serem estas
tecnologias de domnio pblico.

5. (UNIOESTE) Sobre modernizao agrcola e meio
ambiente correto afirmar que
01. A destruio das florestas e sua substituio por certas
culturas provocam um maior impacto dos agentes
meteorolgicos sobre os solos, acelerando o transporte
do solo das encostas para os vales.
02.Com as chuvas, os rios, com seus leitos assoreados,
transbordam, inundam cidades e destroem plantaes.
04. A mecanizao agrcola em reas de encostas e tem sido
tambm uma grande estimuladora da eroso.
08. A monocultura pode favorecer o desenvolvimento de
grande quantidade de pequenas espcies de animais
invasores, as pragas.
16.Os solos da Amaznia garantem o sucesso de
assentamentos agrcolas em qualquer lugar florestado.
32.O solo um elemento vivo da natureza, sua
contaminao o torna progressivamente sem vida e
menos produtivo.

6. Leia com ateno os textos a seguir sobre Reforma
Agrria.
I. Todos os pases desenvolvidos do mundo promoveram, de
alguma forma, o acesso de famlias ao campo, e nenhum
deles tem a concentrao de terras semelhante a do Brasil.
II. Nos EUA, o trabalhador familiar teve acesso a terra desde
meados do sculo passado, na "conquista do Oeste". O Ato
de Propriedade Rural (Homestead Law - 1862) fixou o
tamanho dos lotes a serem distribudos para os colonos,
que deviam cultiv-los, pelo menos por 5 anos.
III. Os principais argumentos utilizados a favor de uma
ampla e profunda reforma agrria no Brasil destacam o
aumento da oferta de alimentos e a resoluo dos
problemas da fome e do desemprego, resultantes da
estrutura fundiria concentrada.
Em relao s afirmaes feitas,
a) apenas a I est correta.
b) apenas a II est correta.
c) apenas a III est correta.
d) apenas a II e a III esto corretas.
e) todas esto corretas.

7. Considere o grfico apresentado a seguir.


Fonte: FAO/The Economist

A anlise do grfico e seus conhecimentos sobre a
populao mundial permitem afirmar que os atuais ndices
de aumento da produo mundial de alimentos
a) reduzem consideravelmente o problema da fome,
sobretudo nos pases com acentuada vocao agrcola.
b) tornam a teoria Malthusiana definitivamente ultrapassada,
possibilitando populao voltar a crescer aos nveis
anteriores aos de 1950.
c) no eliminam a fome das reas pobres uma vez que os
maiores aumentos na produo ocorreram nas reas
desenvolvidas, beneficiando uma pequena parcela da
populao mundial.
d) refletem o esforo realizado pelos pases de mais
avanada tecnologia, no sentido de promover uma
revoluo agrcola em nvel mundial.
e) revelam a eficincia de programas conjuntos de controle
de natalidade e desenvolvimento agrcola realizados nos
pases subdesenvolvidos.

UNIDADE 12

NOVA ORDEM MUNDIAL

Dos Trs Mundos Oposio Norte/Sul
A regionalizao do espao mundial com base em critrios
sociais sempre est ligada ordem internacional que
prevalece num certo momento, ao equilbrio instvel dos
pases e os grupos de pases, disputa (ou cooperao)
entre as grandes potncias mundiais. Aps 1945 o mundo
dividiu-se em trs "mundos" ou conjuntos de pases: o
primeiro mundo (pases capitalistas desenvolvidos); o
segundo mundo (pases socialistas ou de economia
planificada); e o terceiro mundo (reas perifricas ou
subdesenvolvidas, com frequncia marcadas por disputas
entre capitalismo e socialismo).

O Reforo das disparidades entre o Norte e o Sul
Com a crise do mundo socialista, aumenta a oposio entre
o Norte e o Sul. Isso porque deixa de haver o conflito
LESTE/OESTE, ou seja, entre o socialismo real e o
capitalismo.
Please purchase PDF Split-Merge on www.verypdf.com to remove this watermark.
Incluso para a Vida Geografia A

Pr-Vestibular da UFSC

25
As duas superpotncias das ltimas dcadas tinham um
poderio avassalador e nenhum conflito importante no plano
mundial deixava de ter a participao direta ou indireta
delas.
Nessa poca, a oposio entre o Norte rico e o Sul pobre
nunca transparecia claramente, porque estava sempre
abafada pelo conflito LESTE/OESTE.
O segundo mundo chegou a abranger cerca de 32% da
populao mundial no incio dos anos 80, mas hoje ele
praticamente no existe mais. Assim, colocando-se os
antigos pases socialistas mais pobres ou menos
industrializados (China, Monglia, Camboja, Vietn, Cuba,
etc.) no Sul subdesenvolvido, e os mais industrializados
(Rssia, Hungria, Polnia, Repblica Tcheca, etc.) no
Norte, temos a oposio entre o Norte desenvolvido, com
23% da populao mundial, e o Sul, com 71% desse total
demogrfico. Essa a principal oposio mundial dos anos
90.

OS PASES DESENVOLVIDOS
As principais caractersticas dos pases chamados
desenvolvidos so:
Agricultura intensiva, isto , moderna e racional, com o
emprego de mquinas, tcnicas eficiente de produo e
mo de obra qualificada. Como consequncia, uma
pequena parcela da populao empregada na agricultura
consegue elevada produtividade, geralmente capaz de
sustentar a populao de todo o pas. Nvel cientfico e
tecnolgico elevado, responsvel por um constante
aperfeioamento das atividades humanas.
Meios de transporte e comunicao modernos e
eficientes.
Predomnio da populao urbana sobre a populao
rural.
Baixo crescimento natural da populao.
Elevado nvel de vida da populao, caracterstica
expressa atravs de:
- baixas taxas de mortalidade infantil;
- alta expectativa de vida, ou seja, elevada durao mdia
de vida;
- reduzido nmero de analfabetos;
- boas condies de alimentao e habitao.
Mas h significativas diferenas entre os pases
desenvolvidos. Alguns atingiram um elevado nvel de
desenvolvimento tecnolgico e comandam as principais
empresas mundiais. o caso dos Estados Unidos, da
Alemanha, do Japo e da Frana. Outros, como Espanha,
Portugal, Grcia e Austrlia, apresentam ainda parte
significativa de sua economia assentada no setor
agropecurio, possuem menor grau de desenvolvimento
tecnolgico e, portanto, exercem menos influncia sobre a
economia internacional.

O Subdesenvolvimento
De forma sucinta, podemos definir o subdesenvolvimento
como uma situao econmico-social caracterizada por
dependncia econmica e grandes desigualdades sociais.
Subordinao ou dependncia econmica:
Todos os pases do Sul ou do terceiro mundo so
economicamente dependentes dos pases desenvolvidos.
Tal dependncia manifesta-se de trs maneiras:

I. Endividamento externo normalmente, todos os pases
subdesenvolvidos possuem vultosas dvidas para com
grandes empresas financeiras internacionais.

II. Relaes comerciais desfavorveis geralmente os
pases subdesenvolvidos exportam produtos primrios (no
industrializados), como gneros agrcolas e minrios. As
importaes, por sua vez, consistem basicamente de
produtos manufaturados, material blico e produtos de
tecnologia avanada (avies, computadores, etc.). Esta
relao comercial revela-se terrivelmente desvantajosa,
pois os artigos importados tm valor agregado bem maior
do que os exportados, e ainda se valorizam mais
rapidamente.

III. Forte influncia de empresas estrangeiras nos pases
subdesenvolvidos, boa parte das principais empresas
industriais, comerciais, mineradoras e, s vezes, at
agrcolas, so de propriedade estrangeira, possuindo a
matriz nos pases desenvolvidos. So as chamadas
multinacionais. Uma grande parcela dos lucros dessas
empresas remetida para suas matrizes, o que provoca
descapitalizao no terceiro mundo.

Grandes Desigualdades Sociais
Em todos os pases subdesenvolvidos, a diferena entre
ricos e pobres muito mais acentuada do que nos pases
desenvolvidos. Por exemplo, na Colmbia, 2,6% da
populao possui 40% da renda nacional; no Chile, 2% dos
proprietrios possuem 50% das terras agrcolas. Dessa
forma, a populao de baixa renda acaba sofrendo de srios
problemas de subnutrio, falta de moradias, atendimento
mdico-hospitalar inadequado, insuficincia de escolas, etc.

Exerccios de Sala #

1. (UFSC) Hoje em dia, na Amrica Latina, 15% a 20%
da populao desfruta de um estilo de vida de Primeiro
Mundo: matriculam seus filhos em escolas particulares,
pertencem a country clubes, jogam tnis, fazem ginstica
aerbica, fazem cirurgias plsticas, viajam em carros
luxuosos e comunicam-se mediante computadores e fax.
Moram em condomnios fechados, viajam para o exterior,
seus filhos frequentam universidades no exterior, falam
ingls e so a audincia qual os presidentes latino-
americanos dirigem seus discursos sobre a nova
prosperidade global. O restante da populao levado do
Terceiro para o Quarto Mundo com cortes nos gastos
sociais (sade e educao), corte nos investimentos de
manuteno de servios pblicos como gua e esgoto.
Ingressam no setor informal da economia. o declnio do
Estado de Bem-estar Social e a pauperizao da
populao.
Esse contraste, extrado do livro Hegemonia dos Estados
Unidos no novo milnio de James Petras e Henry
Veltmeyer, mostra a realidade na Amrica Latina.

Sobre o texto correto afirmar que:
Please purchase PDF Split-Merge on www.verypdf.com to remove this watermark.
Incluso para a Vida Geografia A

Pr-Vestibular da UFSC

26
01. A situao socioeconmica das duas realidades acima
descritas histrica, porm exacerbou-se com a entrada
do neoliberalismo, fase atual do capitalismo.
02. A expanso capitalista atual forou a demolio do
Estado do Bem-estar Social, que existiu em maior ou
menor escala em grande nmero de pases, agravando
as diferenas sociais.
04. A constatao acima permite concluir que, na Amrica
Latina, h um crescimento em grande escala do capital,
o que faz aumentar o nmero de emprego e diminuir a
pobreza, os crimes e os sofrimentos humanos.
08. A comparao no texto vem confirmar que a diferena
dos dois conjuntos de pases s diminuir com o
crescimento econmico do Terceiro ou Quarto Mundo,
que superar a evoluo econmica do Primeiro
Mundo.
16. Do texto acima, deduz-se que a situao de misria e
pobreza da maioria latino-americana s ser resolvida
com a cooperao dos Estados imperiais e das elites que
esto no poder.

2. (UFRJ) Pases em desenvolvimento so os que, apesar
de apresentarem vrias caractersticas inerentes aos pases
subdesenvolvidos, encontram-se em um nvel intermedirio
de desenvolvimento. Uma caracterstica desse grupo de
pases :
a) O aumento do setor primrio, tendo em vista que o seu
processo de industrializao clssica tem mecanizado a
lavoura, liberando mo de obra qualificada para o setor
tercirio.
b) O aumento da participao dos setores secundrios e
tercirio da economia, que determina mais um
crescimento econmico do que um desenvolvimento do
pas, visto que o setor secundrio representado por
uma industrializao tardia.
c) Uma industrializao voltada para a fabricao de bens
de produo, j que atingiram um alto grau de
tecnologia nacional e possuem um grande mercado
consumidor interno.
d) A planificao de sua economia, com os meios de
produo nas mos do estado e com nfase s indstrias
de base, como, por exemplo, a siderurgia.
e) O desenvolvimento satisfatrio da vida e a autonomia
de criar sua prpria tecnologia, usando seu prprio
capital para o seu desenvolvimento econmico.

Tarefa Mnima #

3. (UFSCAR) Leia o texto: Durante milnios, a histria do
homem faz-se a partir de momentos divergentes, como uma
soma de aconteceres dispersos, disparatados, desconexos. J
a histria do homem de nossa gerao aquela em que os
momentos convergiram, o acontecer de cada lugar podendo
ser imediatamente comunicado a qualquer outro, graas ao
domnio do tempo e do espao escala planetria. A
instantaneidade da informao globalizada aproxima os
lugares, torna possvel uma tomada de conhecimento
imediata (...) e cria entre lugares e acontecimentos uma
relao unitria escala do mundo.
SANTOS, Milton. A natureza do espao.
A interpretao desse texto permite-nos afirmar que:
a) A sociedade atual, conectada pelas tecnologias de
informao, produz um mundo igualitrio, j que os
lugares esto unidos por relaes globais.
b) O avano da tecnologia informacional, ao alterar a escala
do tempo e do espao, anulou a existncia e o papel dos
lugares.
c) Na sociedade atual cada momento e cada lugar
compreendem eventos que so interdependentes,
includos em um sistema global de informaes.
d) A instantaneidade das informaes no possibilita a
ocorrncia de eventos em lugares e momentos
especficos.
e) A existncia de relaes unitrias globais oferece
oportunidades a todos os lugares e possibilita maior
autonomia a eles.

4. (UFSC) A partir do final da dcada de 1970, a China vem
promovendo um intenso processo de reformulao
econmica. Sobre as mudanas ocorridas a partir desse
perodo, assinale a(s) proposio(es) correta(s).

01. Tais mudanas no alavancaram a economia chinesa,
pois o pas continua tendo deficit na sua balana
comercial, como aqueles registrados no perodo da
Revoluo Cultural.
02. Deng Xiao Ping comeou a transformao pela
agricultura, promovendo o assalariamento rural e
subsidiando o povo para a compra de produtos agrcolas.
04. Apartir da criao das Zonas Econmicas Especiais, a
China vem enfrentando um forte xodo rural.
08. Apoltica de abertura econmica na China significou a
adeso completa a uma economia de mercado nos
moldes capitalistas.
16. Potncia nuclear, a China tem atualmente como principal
ponto de tenso o Japo, pois os chineses pretendem
reintegrar o arquiplago japons ao seu territrio.

5. (UFRS) AChina vem expandindo sua economia e
ampliando suas relaes para alm de suas fronteiras. H
poucos anos, ela ingressou na Organizao Mundial do
Comrcio (OMC), submetendo-se s regras do comrcio
internacional.
Assinale a alternativa correta em relao a essa temtica.
a) A partir do processo de abertura econmica, a maioria da
populao chinesa passou a viver em reas urbanas e a
possuir renda per capita semelhante de vrios pases
desenvolvidos.
b) Entre os principais produtos brasileiros exportados
China, destacam-se aparelhos de tica e preciso e
componentes eletrnicos.
c) A parceria sino-brasileira ampliou-se quando foi acertado
um dos mais importantes projetos na rea tcnico-
cientfica entre os dois pases: o desenvolvimento de
satlites de rastreamento de recursos naturais.
d) O crescimento econmico chins obtido nos ltimos anos
coincidiu com a abertura poltica, j que aumentou o
nmero de partidos polticos e o pas se tornou uma
repblica democrtica.
e) Apesar de a China ser uma das dez maiores economias
do planeta.
Please purchase PDF Split-Merge on www.verypdf.com to remove this watermark.
Incluso para a Vida Geografia A

Pr-Vestibular da UFSC

27

UNIDADE 12

SANTA CATARINA

LOCALIZAO
O Estado de Santa Catarina localiza-se na parte subtropical
do Brasil e representa aproximadamente 1,1% do territrio
brasileiro. Com uma rea de 95.442,90 km
2
, o estado
possui aproximadamente 550km de litoral com o oceano
Atlntico, o territrio catarinense encontra-se entre os
paralelos 25 1941 e 25 23 55 de latitude Sul e entre os
meridianos 48 19 37 e 53 50 00 de longitude Oeste. A
costa catarinense corresponde a 7% do litoral brasileiro.

LIMITES
Ao Norte - Com o Paran, desde as nascentes de rio
Peperi-Guau at a foz do rio Sai-Guau no Oceano
Atlntico.
Ao Sul - Com o Rio Grande do Sul, desde a confluncia
dos rios Peperi-Guau e Uruguai at a foz do Mampituba
no Oceano Atlntico.
Ao Leste Com o Oceano Atlntico.
Ao Oeste - Com a Rep. Federativa da Argentina atravs do
rio Peperi-Guau.

GEOLOGIA
O territrio catarinense possui quatro unidades bem
distintas quando consideramos o aspecto geolgico.
I - Unidade Sedimentar Quaternria - Formada pelos
depsitos dos maiores rios de Santa Catarina, a rea
de formao mais recente.
II - Unidade Cristalina Pr Cambriana - Formada por
planaltos e serras mais prximas ao litoral, as altitudes
encontram-se muito reduzidas devido a grande eroso.
III - Unidade Sedimentar Paleozoica - uma das reas que
apresenta elevados desnveis altimtricos, nela
encontram-se desde a serra de Lages at a depresso
Carbonfera de Cricima.
IV Unidade Basltica Mesozoica a regio que
apresenta os maiores altitudes do estado.
RELEVO




.








O relevo catarinense pode ser dividido em trs partes
principais:
- As plancies costeiras representadas por uma faixa estreita
ao longo do litoral, em geral apresentam altitudes de 400
metros.
- As serras, Geral (sul) e do Mar (nade) so as barreira
quase intransponveis entre a plancie costeira e o planalto.
As altitudes nesta parte variam de 400 a 800 metros.
- O planalto Meridional catarinense decai para oeste, com
altitudes de 800 a 1200 metros em mdia.
- A maior altitude do estado o morro da Igreja no
municpio de Urubici (1840 m).
- No relevo de Santa Catarina, podemos perceber que
temos ainda uma plancie na parte sudoeste do estado, esta
plancie chamada de: Plancie do Rio Uruguai.

CLIMA













Por localizar-se totalmente abaixo do trpico de
capricrnio. Santa Catarina possui um clima tipo
Subtropical.
Na classificao de Koppen, a mais utilizada em
todo o mundo, SC possui dois subtipos climticos:
Cfa - Clima mesotrmico mido com chuvas bem
distribudas, veres quentes e invernos brandos. (Atua em
todo o litoral e na plancie do Rio Uruguai)
Cfb - Clima mesotrmico mido com chuvas bem
distribudas, veres brandos e invernos rigorosos. (Atua em
todo o Planalto Serrano e parte do Oeste do Estado)
Esta classificao (Koppen) funciona como uma
padronizao, assim, qualquer pessoa que entenda a
classificao pode conhecer a base de um clima, apenas por
trs letras.

HIDROGRAFIA

A hidrografia de Santa Catarina pode ser dividida em duas
vertentes:
Vertente do Atlntico ou do Leste, ou ainda do sudeste-
sul
Vertente do Interior ou do Uruguai
- Percebe-se que o divisor de guas constitudo pela Serra
do Mar em conjunto com a Serra Geral.
O regime de praticamente todos os rios brasileiros pluvial
(com exceo do Amazonas), ou seja, eles dependem
exclusivamente das chuvas para existirem.
As quatro reas de maior comprometimento so:
1) Sul do estado: rejeitos de carvo e rede de esgotos.
2) Norte do estado: metais pesados (resduos industriais),
esgotos e agrotxicos.
3) Bacia do Rio do Peixe: indstria do papel, frigorficos,
agrotxicos e esgotos.
4) Bacia do ltaja-Au: Indstrias (corantes) e esgotos.
Please purchase PDF Split-Merge on www.verypdf.com to remove this watermark.
Incluso para a Vida Geografia A

Pr-Vestibular da UFSC

28
VEGETAO

















Santa Catarina possui uma grande diversidade de
paisagens naturais, com diferentes formaes vegetais, isto
se d graas a localizao geogrfica, ao relevo e aos
diferentes tipos de solo que apresenta.
Antes dos colonizadores e imigrantes chegarem, SC
apresentava uma vegetao nativa do tipo florestal. Hoje,
depois de tantos desmatamentos, as florestas recuaram e
deram lugar agricultura, s pastagens, e s cidades.
Mesmo assim, nosso estado ainda possui a maior cobertura
vegetal nativa do sul do Brasil.

AGRICULTURA

Em Santa Catarina predominam os minifndios, que ficam
menores a cada diviso, em geral, por herana. Com uma
baixa produo, os agricultores acabam migrando para a
cidade em busca de melhores condies de vida (xodo
rural).

Na agricultura podemos ter dois tipos de lavoura:

- lavouras temporrias exigem novo plantio aps a
colheita, os principais cultivos so:
-o milho e a soja (principalmente no oeste).
-o arroz (sobretudo em Jaragu do Sul e Joinville), o feijo
(oeste, regio serrana e norte),
-o fumo ( no sul e no vale do ltaja).
-a cebola ( no vale do ltajai, sobretudo em ltuporanga).
-o alho (regio serrana, sobretudo em Curitibanos e Frei
Rogrio).

- lavouras permanentes - no exigem novo plantio aps a
colheita. Os principais cultivos so:
-a banana (Sul, sobretudo em Jacinto Machado);
-a ma (oeste e regio serrana, sobretudo Fraiburgo e So
Joaquim);
-a uva (oeste e sul, com destaque para Urussanga).

Por causa desta diversidade de culturas, SC possui vrias
festas ligadas a produo de suas lavouras. Entre elas
destacam-se:

- Festa do vinho e da uva, em Urussanga;
- Festa do milho, em Santo Amaro da Imperatriz;
- Festa da cebola, em Ituporanga;
- Fenarreco, em Brusque;
- Octoberfest, em Blumenau e ltapiranga;
- Festa das flores, em Joinville;
- Marejada, em ltaja;
- Festa do Pinho, em Lages
- Festa da cachaa, em Luis Alves;
- Festival de dana, em Joinville;
- Fenaostra, em Florianpolis.

PECURIA

Suinocultura
A criao de sunos praticada principalmente no oeste
catarinense, tendo como municpios que se destacam neste
tipo de cultura:
Concrdia, ltapiranga, Chapec, Seara, Videira.

Avicultura
A avicultura catarinense conta com o que h de mais
moderno no setor, tornando o estado o maior produtor de
aves do Brasil.

Obs.: Esto localizados em territrio catarinense os
maiores frigorficos do Brasil, que abatem os animais
(porcos e aves) e industrializam sua carne para consumo
interno e tambm para exportao.

EXTRATIVISMO VEGETAL
A extrao vegetal em Santa Catarina baseia-se
principalmente em:

Madeira em tora e lenha regies: Oeste, Serrana e
Vale do Itaja.
Palmito Joinville e Vale do ltaja
Carvo vegetal produzido em vrias partes do estado,
principalmente no Sul e no Vale do ltaja
Erva mate Planalto Serrano e planalto Norte.

EXTRAO ANIMAL
Em Santa Catarina, a pesca desponta como principal
atividade de extrao animal e praticada em todas as
modalidades: amadora, artesanal e industrial.
- A piscicultura a criao de peixes em aqurios ou
tanques para fins econmicos.
Em Santa Catarina, a pesca praticada com maior
intensidade nos municpios de ltaja, Laguna, Navegantes,
Florianpolis e Gov. Celso Ramos.

A maricultura (criao de ostras e mariscos) catarinense
lder nacional.

EXTRATIVISMO MINERAL
O territrio catarinense possui muitos minrios. Em nosso
territrio exploramos areias, argilas e, principalmente,
fluorita e carvo mineral.

Areia Sul e Vale do ltaja

Argila - em todo o estado, mas, principalmente, no vale do
ltaja, as argilas so usadas sobretudo para fabricao de
cermica, no estado temos a maior fabricante mundial de
Please purchase PDF Split-Merge on www.verypdf.com to remove this watermark.
Incluso para a Vida Geografia A

Pr-Vestibular da UFSC

29
pisos e azulejos, a Cecrisa.

Fluorita - SC o estado de maior reserva e extrao de
fluorita do Brasil, o municpio que mais contribui o
Morro da Fumaa.

Carvo Mineral SC extrai 3/5 de toda a extrao
nacional. No municpio de Cricima localiza-se a
Companhia Carbonfera Catarinense. Entre os outros
municpios que se destacam na extrao de carvo, esto:
Siderpolis, Tubaro e Lauro MuIler.
O carvo mineral muito importante para o estado
de Santa Catarina, pois grande parte de sua produo
utilizada para gerar energia na usina termeltrica Jorge
Lacerda, que fica no municpio de Capivari de Baixo.

INDSTRIA

A influncia dos imigrantes italianos e alemes ntida no
processo de industrializao catarinense.A indstria de
Santa Catarina lder em diversos setores e encontra-se
bem distribuda pelo territrio, sendo que as indstrias do
mesmo setor em geral agrupam-se em uma mesmo regio.
Setores industriais de Santa Catarina:

Eixos Econmicos
0l - Metal Mecnico
02 - Cermico e Minerao
03- Madeireiro, Papel e Celulose
04- Mobilirio
05- Txtil
06- Agro-industrial
07 - Tecnolgico

Em Santa Catarina esto grandes indstrias de destaque
nacional e at internacional em vrios setores.
Cecrisa Eliane Hering
Telca Sadia Perdigo
Portobelo Tigre Embraco
Weg

AS REGIES DE SANTA CATARINA

O estado de Santa Catarina pode ser dividido em seis
grandes regies para ser melhor estudado.















a) Regio Oeste
a regio de ocupao mais recente. A maior parte de sua
populao formada por descendentes de imigrantes
alemes e italianos do estado vizinho, o Rio Grande do Sul.
O principal ramo industrial o de alimentos
(agroindstria), tendo como principais cidades: So Miguel
dOeste, Chapec, Xanxer, Joaaba, Caador e Concrdia.

b) Regio Norte
No incio, sua colonizao foi feita por paulistas;
posteriormente, os alemes e eslavos colonizaram a regio.
Na regio de Joinville e Jaragu do Sul est instalado o
maior porque industrial catarinense. A agricultura
catarinense pouco desenvolvida, com destaque apenas
para a produo de arroz. As cidades mais importantes so:
Canoinhas, So Bento do Sul, Joinville, Jaragu do Sul,
Mafra, Porto Unio e So Francisco do Sul.

c) Regio Serrana
Uma das mais belas regies do estado, com altitudes
superiores a 1300 metros, colonizada principalmente por
tropeiros de So Paulo, a regio dedica-se pecuria de
bovinos e agricultura de milho. As indstrias que mais se
destacam so a de papel, papelo e madeira. Lages,
Curitibanos e Campos Novos so as cidades mais
importantes da regio serrana.
d) Vale do ltaja
Colonizada principalmente por imigrantes alemes, tem
como cidades mais importantes Blumenau, ltaja,
ltuporanga e Brusque.
Blumenau e Brusque so o bero da fiao catarinense,
possuindo um grande nmero de indstrias txteis, Na
arquitetura, destacam-se em Blumenau as casas em estilo
enxaimel (tijolos e madeira).

e) Grande Florianpolis
A cidade de Florianpolis, localizada na ilha de Santa
Catarina, juntamente com outros municpios vizinhos,
forma a grande Florianpolis.
Regio de colonizao mesclada, onde encontramos
portugueses, italianos, e alemes. Possui um centro
comercial bastante diversificado e desenvolvido. A
indstria de software e de telefonia est se desenvolvendo
com razovel fora em Florianpolis.
Fazem parte da grande Fpolis: guas Mornas, Alfredo
Wagner, Angelina. Anitpolis, Antnio Carlos, Biquau,
Canelinha, Florianpolis, Garopaba. Gov. Celso Ramos,
Leoberto Leal, Major Gercino, Nova Trento, Palhoa,
Paulo Lopes, Rancho Queimado, Sto Amaro da Imperatriz,
So Bonifcio, So Joo Batista, So Jos e Tijucas,

f) Sul
Esta regio, colonizada no seu litoral por portugueses e no
interior por italianos e alemes, apresenta hoje um parque
industrial moderno e diversificado, com destaque para a
indstria de cermica.
Tubaro, Laguna, Cricima e Ararangu so as maiores
cidades.

ASPECTOS DEMOGRFICOS CATARINENSES
Populao Absoluta: 5.350.000 hab (censo. 2000)
Densidade Demogrfica: 52 hab/km
2

Mortalidade Infantil: 23,8(1996)
IDH: 0.863
Populao urbana: 73,13% (1996)
Please purchase PDF Split-Merge on www.verypdf.com to remove this watermark.
Incluso para a Vida Geografia A

Pr-Vestibular da UFSC

30
Populao rural:26,87% (1996)
Analfabetismo: 7.37% (1996)
Esperana de vida: 73 anos
PEA: 54, I%(1996)

Distribuio da populao pelos setores da economia
Primrio: 31.8%
Secundrio: 26, 1%
Tercirio: 42,1%

Cidades mais populosas de Santa Catarina
De acordo com o censo de 2000 temos:
Joinville: 430 mil hab
Florianpolis: 341 mil hab
Blumenau: 260 mil hab
So Jos:173 mil hab
Cricima: 170 mil hab
Lages: 156 mil hab
Itaja: 151 mil hab
Chapec:140 mil hab

TRANSPORTES
a) Transporte rodovirio

RODOVIRIO













Classificao das rodovias federais que cortam SC:
- BR 101
- BR 116 Brs Longitudinais (que correm no sentido norte-
sul)
- BR 153
- BR 280 Brs Transversais (que correm no sentido leste-
oeste)
- BR 282
- BR 470
- BR 480 BRs de ligao (que ligam duas cidades, um
porto a uma cidade, uma BR a outra, entre outras ligaes
possveis.)













b) Transporte ferrovirio

PRINCIPAIS TRONCOS FERROVIRIOS













O sistema ferrovirio catarinense, atualmente, encontra-se
em decadncia. Em pleno funcionamento, existem apenas
alguns trechos de antigas ferrovias que recebem
manuteno precria. A estrada de ferro mais conhecida do
territrio catarinense a de So Paulo Rio Grande, que
corta a regio das batalhas da conhecida Guerra do
Contestado.
No estado, ainda funcionam:
- Estrada de ferro Teresa Cristina (sul do estado), que
possui um terminal para transportar carvo para o porto de
lmbituba, para termeltrica Jorge Lacerda que fica no
municpio de Capivari de Baixo.
- Estrada de ferro Dona Francisca (nordeste), utilizada em
alguns trechos pela indstria ou para turismo.

c) Transporte areo
Em Santa Catarina atualmente existem 27 aeroportos,
sendo que 6 deles operam com Linhas regulares, so eles:
Florianpolis, Joinville, Navegantes, Chapec, Cricima e
Lages.

d) Portos
Os portos catarinenses esto integrados ao sistema
nacional. Em Santa Catarina temos quatro portos:
- So Francisco do Sul containers, soja, trigo,
compressores.
- ltaja - frango, produtos txteis, derivados de petrleo.
- lmbituba - carvo mineral, acar, frango, insumos.
- Laguna -- terminal pesqueiro

FONTES DE ENERGIA
No estado de Santa Catarina, as principais fontes para a
produo de energia so:
Carvo mineral retirado na depresso carbonfera de
Cricima e utilizado na termeltrica Jorge Lacerda para a
produo de energia eltrica, que hoje se encontra cargo
da Gerasul e da CELESC.
Rios - com a construo de vrias barragens, o homem
utiliza a fora dgua para a produo de energia eltrica.
No estado existem mais de 10 usinas hidreltricas em
funcionamento, entre elas destacam-se:
- Usina de Bracinho municpio de Schroeder
- Usina de Caveiras municpio de Loges
- Usina de Cedros e Palmeiras - municpio de Rio dos
Cedros
Please purchase PDF Split-Merge on www.verypdf.com to remove this watermark.
Incluso para a Vida Geografia A

Pr-Vestibular da UFSC

31
- Usina Gov. Celso Ramos - municpio de Faxinal dos
Guedes
- Usina Gov. Ivo Silveira municpio de Capinzal
- Usina Pery - municpio de Curitibanos
- Usina Garcia - municpio de Angelina
- Usina Salto municpio de Blumenau
- Usina de lt municpio de It

Ventos - Parque Elico Bom Jardim da Serra
Gs Natural - Vem da Bolvia e passa pelo litoral
catarinense.

Exerccios de Sala #

1. (UDESC) Observe o mapa que representa a vegetao
nativa de Santa Catarina.

Assinale a alternativa que representa os tipos de vegetao
indicados, respectivamente, pelas setas 1 e 2.
a) Florestas Galerias e Mata Atlntica
b) Floresta Equatorial e Subtropical
c) Floresta Araucria e reas de Campos
d) Campos e Mata Atlntica
e) Vegetao Litornea e Floresta Araucria

Tarefa Mnima #

2. (UDESC) Alguns historiadores analisam a indstria
catarinense a partir de alguns eixos tradicionais,
relacionando-os a regies especficas do Estado.
Considerando esse recorte, relacione a coluna de cima, que
contm os eixos da indstria, com a de baixo, de regies a
que eles se relacionam.

(1) Txtil
(2) Eletro-Metal-Mecnico
(3) Madeireiro e Celulose
(4) Cermico

( ) Regio de Lages at Canoinhas
( ) Regio do Vale do Rio Itaja
( ) Regio de Cricima e Tijucas
( ) Regio do litoral Norte, principalmente os municpios de
Joinville e Jaragu do Sul

Assinale a alternativa que contm a sequncia numrica
correta, de cima para baixo.
a) 1 3 2 4
b) 1 3 4 2
c) 4 1 2 3
d) 3 1 4 2
e) 2 3 4 1

3. (UDESC) Observe a tabela abaixo:
ndice de Desenvolvimento Humano Municipal e
populao dos municpios da Microrregio do Mdio Vale
do Itaja



Sobre os municpios da Microrregio do Mdio Vale do
Itaja, assinale a alternativa incorreta.

a) Blumenau da Oktoberfest o municpio com maior
populao e maior IDH municipal da regio.
b) Apina, onde se pratica o turismo de aventura, com
esportes como rafting, rapel e montanhismo, um
municpio pequeno, com o pior IDH municipal da regio.
c) Brusque a segunda cidade em populao na regio,
estando entre as cinco de maior IDH municipal para a
regio.
d) Apina, que contou em sua colonizao com imigrantes
alemes, possui o pior IDH municipal da regio e uma
populao maior que Ascurra, com a qual faz fronteira
ao Norte.
e) Embora os municpios de Doutor Pedrinho e Benedito
Novo possuam o mesmo IDH municipal, o primeiro tem
aproximadamente 1/3 da populao do segundo, o que
faz com que a populao de Doutor Pedrinho tenha uma
qualidade de vida muito superior da populao de
Benedito Novo.

4. (UDESC) Sobre a relao homem-natureza em Santa
Catarina, correto afirmar:
a) As dunas e os mangues do litoral catarinense so
preservados e acabam sendo exemplos da ocupao
planejada.
b) O lixo h muito deixou de ser um problema e uma
ameaa ao meio-ambiente no Estado.
c) As enchentes de 1983 mostraram a Santa Catarina e
tambm ao pas, j naquela poca, as consequncias e os
agravamentos das condies naturais advindas do
crescimento sem planejamentos e preocupaes
ambientais.
d) A reciclagem uma ideia que agrava a produo de lixo e
piora muito a relao homem-natureza.
e) O cultivo do fumo no Estado exemplo da bem-sucedida
relao homem-natureza.
Please purchase PDF Split-Merge on www.verypdf.com to remove this watermark.
Incluso para a Vida Geografia A

Pr-Vestibular da UFSC

32

5. (UDESC) Sobre o relevo catarinense, assinale a
alternativa incorreta.
a) As depresses catarinenses circundam todas as serras,
algumas abaixo do nvel do mar.
b) A maior parte do relevo compe-se de terrenos planos e
altos.
c) Entre os planaltos e as plancies aparecem as serras.
d) Destacam-se dois importantes conjuntos de serras: a Serra
do Mar e a Serra Geral.
e) As plancies so regies predominantes do litoral, tambm
chamadas de plancies costeiras.

6. (UDESC) Leia o excerto abaixo.
O slogan Bela e Santa Catarina foi criado para melhor
vender a imagem do Estado. O maior atrativo como produto
turstico era sua diversidade cultural, amalgamada agora no
Estado de Festa. [...] As festas e eventos culturais, mesmo
como criaes recentes, tm um papel importante na vida
local. No Sul do Brasil, o Estado de Santa Catarina tem sido
palco de eventos [festivos]. [...] Surgem vrias festas
ressaltando os respectivos elementos, permeadas,
evidentemente, com carter mercadolgico.
(SEVERINO, Jos Roberto. Itaja e a identidade aoriana: a maquiagem
do possvel. Itaja: UNIVALI, 1999, p. 11-49.)
Analise as proposies em relao ao texto e realidade de
Santa Catarina.
I. Pode-se afirmar que Santa Catarina um Estado Belo,
europeu e sem problemas sociais.
II. As festas mais recentes, criadas no Estado, procuram
manter um certo vnculo com a cultura local, mas
tentam, sob diferentes formas, aproveitar o que de
vendvel e espetacular pode ser comercializado.
III. A nfase no carter tnico dessas festas pode provocar a
crena e mesmo o preconceito de queo Estado de Santa
Catarina o mais organizado, limpo e dinmico da
Federao, em detrimento dos demais.
IV. Desde a primeira Oktoberfest, realizada no ano de 1984
em Blumenau, comum descreverem aquela cidade
como Alemanha Brasileira.

Assinale a alternativa correta.
a) ( ) Somente a afirmativa III verdadeira.
b) ( ) Somente as afirmativas I e II so verdadeiras.
c)( ) Somente as afirmativas II, III e IV so verdadeiras.
d) ( ) Somente as afirmativas I e II so verdadeiras.
e) ( ) Somente a afirmativa IV verdadeira.

7. (UDESC) A vegetao original do estado de Santa
Catarina foi muito desmatada para abastecer serrarias, para
fabricao de mveis, bem como para dar lugar agricultura
e ao crescimento urbano. Sobre a vegetao original do
Estado, correto afirmar que:
I. A Mata das Araucrias ou Mata dos Pinhais predominava
em grande parte do Estado e era constituda de espcies
como o pinheiro e a erva-mate;
II. A vegetao de mangue era encontrada no litoral, mas
tambm em reas de banhado no interior do Estado, onde
a umidade alta, formando manchas bem pequenas;
III. A Mata Subtropical localizava-se prxima ao Rio
Uruguai e possua espcies como a canela e a grpia;
IV. Na Mata Atlntica, encontravam-se plantas como o ip
amarelo, a peroba e o palmiteiro;
V. Os campos catarinenses predominavam em reas de
planalto.

Assinale a alternativa correta.
a) ( ) Somente as afirmativas I e V so verdadeiras.
b) ( ) Somente as afirmativas II e IV so verdadeiras.
c) ( ) Somente as afirmativas I, III, IV e V so verdadeiras.
d) ( ) Somente a afirmativa III verdadeira.
e) ( ) Todas as afirmativas so verdadeiras.
8. (UDESC) Nas alternativas abaixo encontram-se os nomes
dos parques estaduais catarinenses. Assinale a alternativa
que contm a maior unidade de conservao.
a) Parque Estadual do Rio Canoas
b) Parque Estadual da Serra Furada
c) Parque Estadual da Serra do Tabuleiro
d) Parque Estadual das Araucrias
e) Parque Estadual Fritz Plaumann

9. (UDESC) O turismo rural est se desenvolvendo
bastante, prometendo novas fontes de recursos aos
catarinenses.
Assinale a alternativa que contm o nome da regio do
Estado onde surgiu esta atividade.
a) Microrregio de Itaja
b) Litoral Norte
c) Microrregio de Chapec
d) Planalto Serrano
e) Sul do Estado

10. (UDESC) O conjunto de serras que compem o divisor
de guas do estado catarinense :
a) Serra dos Faxinais e Serra do Irani
b) Serra de Chapec e Serra do Capanema
c) Serra Geral e Serra do Espigo
d) Serra do Tabuleiro e Serra do Mirador
e) Serra do Mar e Serra Geral

11.(UDESC) A hidrografia catarinense divide-se em duas
grandes vertentes. Os rios da Bacia do Sudeste ou Atlntica,
que desguam no mar, e os rios da Bacia do Uruguai. Dentre
os rios da Bacia do Uruguai, dois so importantes, pois suas
guas engrossam o volume do rio Uruguai, que faz a divisa
do Estado com o Rio Grande do Sul. Assinale a alternativa
que contm o nome destes rios.
a) Rio Mampituba e Rio do Peixe
b) Rio do Peixe e Rio Iguau
c) Rio Canoas e Rio Pelotas
d) Rio Pelotas e Rio Tubaro
e) Rio Itapocu e Rio Canoas




Please purchase PDF Split-Merge on www.verypdf.com to remove this watermark.
Incluso para a Vida Geografia B

Pr-Vestibular da UFSC

1
UNIDADE 1

CARACTERSTICAS GERAIS DO
TERRITRIO BRASILEIRO

O TERRITRIO BRASILEIRO
O Brasil, possuindo um territrio de 8.514.876,599
Km
2
, costuma ser considerado um pas continental'.
De fato, com uma das maiores extenses territoriais do
mundo (quinto lugar), inclui-se entre os seis pases que
tm mais de 7 milhes de km
2
. A extenso do territrio
brasileiro corresponde a uma parcela aproximada de
1,66% da superfcie terrestre (cerca de 6% das terras
emersas do globo).
O territrio brasileiro ocupa, atualmente, cerca
de 47% da rea da Amrica do Sul e est localizado em
sua poro centro-oriental. Para ter uma ideia da
imensido do nosso pas, podemos lembrar tambm
que toda a Europa, a ocidental e a oriental (excluindo a
parte europeia da Rssia), onde existem atualmente 39
Estados independentes, possui apenas cerca de 5,2
milhes de km2. Alguns estados do Brasil - como o
Amazonas, o Par, Mato Grosso ou Minas Gerais - tm
cada qual uma rea territorial superior de inmeros
pases europeus reunidos.

A formao territorial do Brasil
A ideia de descobrimento do Brasil foi muito comum,
se ainda no o . Era como se o pas j estivesse
"pronto" e faltasse somente algum, um navegador
portugus, que o encontrasse. Mas, se o Brasil somos
ns, o povo - ou melhor, a sociedade brasileira, com
sua cultura, territrio e instituies -, ento lgico que
ele ainda no existia em 1500. O que havia era um
espao fsico habitado por inmeras sociedades
indgenas, cada uma com um territrio diferente.
Os colonizadores portugueses se apropriaram
de certas reas, normalmente expulsando ou extermi-
nando (ainda, s vezes, escravizando) os ndios que as
ocupavam, e, com o tempo, expandiram o seu territrio
e criaram neste novo mundo uma sociedade diferente,
que um dia se tornou um Estado-Nao independente.
A construo do Brasil, que durou vrios scu-
los, teve dois aspectos principais: a criao de uma
sociedade com cultura (valores e hbitos) e instituies
prprias (especialmente o Estado ou poder pblico em
todos os nveis e esferas); e a formao territorial, isto
, a forma de ocupao da terra e a sua delimitao por
meio de fronteiras.

Aspectos da colonizao
Um aspecto marcante na colonizao de todo o con-
tinente americano - e, por extenso, do Brasil -, com
exceo apenas de partes da Amrica do Norte, foi
servir para o enriquecimento das metrpoles (as naes
europeias). De fato, o que alguns historiadores chamam
de "sentido" da nossa colonizao est nisto: ela foi
organizada para fornecer ao comrcio europeu acar,
tabaco e alguns outros gneros; mais tarde, ouro e
diamantes; depois, algodo e, em seguida, caf. E isso
acarretaria algumas marcas economia e sociedade
brasileiras que, em alguns casos, permanecem at hoje,
por exemplo: povoamento mais intenso na faixa
atlntica, onde se localizam os portos; utilizao dos
melhores solos para a produo de gneros destinados
exportao, e no de alimentos para a populao;
formao de uma sociedade constituda principalmente
por uma minoria de altssimas rendas (que mantm
ligaes econmicas com o exterior) e uma maioria
com baixas rendas, que serve como fora de trabalho
barata; dependncia econmica em relao aos centros
mundiais do capitalismo.
Assim, a colonizao do Brasil teve um
carter de colnia de explorao, o que significa que
ela foi inserida na poltica mercantilista da poca,
servindo como uma das condies indispensveis para
que ocorresse a Primeira Revoluo Industrial, de
meados do sculo XVIII at o final do sculo XIX.
Esse acontecimento marcou a passagem do capitalismo
comercial tpico da poca moderna (sculos XVI a
XVIII), em que o comrcio era o setor chave da
economia, para o capitalismo industrial.

Localizao
O territrio brasileiro cortado por dois crculos
imaginrios o Equador, que passa pela embocadura
do rio Amazonas, e o Trpico de Capricrnio, que
corta os estados do Mato Grosso do Sul, Paran e de
So Paulo. O Brasil tem seu territrio assim
distribudo:
100% No hemisfrio Oeste.
7% No hemisfrio Norte.
93% No hemisfrio Sul.
93% Na Zona Intertropical.
7% Na Zona Temperada Sul (subtropical).


Please purchase PDF Split-Merge on www.verypdf.com to remove this watermark.
Incluso para a Vida Geografia B

Pr-Vestibular da UFSC

2
Fronteiras

Temos 15.719 km de fronteiras, feitas principalmente
por rios e serras; o maior trecho com a Bolvia
3.126 km, e o menor com o Suriname 593 km. O
Chile e o Equador no tm fronteiras com o Brasil. No
Leste, o pas faz 10.959 km de fronteira com o Oceano
Atlntico. Visualize essas e outras caractersticas do
territrio observando o mapa abaixo:


Fusos Horrios
Devido sua grande extenso longitudinal, o territrio
brasileiro atravessado por quatro fusos horrios,
sendo neles a hora atrasada em relao Hora de
Greenwich. Entretanto, em junho de 2008 houve uma
nova padronizao do fuso brasileiro e, desde ento, o
pas estabeleceu apenas dois fusos continentais e um
ocenico. No segundo fuso horrio brasileiro (menos
trs horas em relao a Greenwich), temos a Hora
Oficial do Brasil (Hora de Braslia).


Exerccios de Sala #

1. (PUC-SP) Leia com ateno: "[...] a Amaznia se
destaca pela extraordinria continuidade de suas
florestas, pela ordem de grandeza de sua principal rede
hidrogrfica e pelas sutis variaes de seus
ecossistemas, em nvel regional e de altitude. Trata-se
de um gigantesco domnio de terras baixas florestadas,
disposto em anfiteatro [...]"
(Aziz AB'SBER In: Os Domnios de Natureza no Brasil,
p. 65)

Esse trecho se refere ao domnio morfoclimtico
amaznico. Considerando a classificao dominante (e
atual) do relevo brasileiro, correto dizer que:
a) A Amaznia um imenso segmento territorial de
plancies rebaixadas, produto de deposio de
sedimentos.
b) Embora apresente terras baixas, a Amaznia
constituda de planaltos na sua maior extenso, e
apenas alguns pontos so realmente plancies.
c) H presena dominante de plancies, com pequenos
segmentos de depresses nas margens dos maiores
rios.
d) Planaltos, depresses e plancies, formaes de
origens diferentes, equivalem-se em extenso, e
esto, mais ou menos, na mesma faixa de altitude.
e) Predominam as depresses, com a presena de
plancies descontnuas no sul e ao longo da calha do
Rio Amazonas, e uma formao planltica ao norte.







Please purchase PDF Split-Merge on www.verypdf.com to remove this watermark.
Incluso para a Vida Geografia B

Pr-Vestibular da UFSC

3
2. (UERJ)

Indique dois problemas relacionados segurana
nacional e dois problemas de ordem socioambiental
que ocorrem ao longo da faixa de fronteira indicada no
mapa.

3. (UNICamp) A abelha, no Brasil, um hbrido das
abelhas europeias ('Apis mellifera mellifera', 'Apis
mellifera ligustica', 'Apis mellifera caucasica' e 'Apis
mellifera carnica') com a abelha africana ('Apis
mellifera scutellata'). Essa abelha, africanizada, possui
um comportamento muito semelhante ao da 'Apis
mellifera scutellata', em razo da maior adaptabilidade
desta raa s condies climticas do Pas. Muito
agressiva, porm menos que a africana, a abelha do
Brasil tem grande facilidade de enxamear, alta
produtividade e tolerncia a doenas.(Embrapa Meio-Norte,
http://sistemasdeproducao.cnptia.embrapa.br/FontesHTML/Mel/SPM
el/racas.htm, acessado em 05/09/2008.)

Goudie, A. "The Human Impact on the Natural Environment". 6
a
ed.,
Malden: Blackwell Publishing, 2006, p. 69.

a) Calcule a distncia, em quilmetros, de propagao
da abelha africana entre o ponto de origem e a
cidade de Fortaleza. Por que a propagao da
abelha africana no avanou para a Patagnia
Argentina e a Cordilheira dos Andes?
b) A apicultura uma atividade capaz de causar
impactos positivos, tanto sociais quanto
econmicos, alm de contribuir para a manuteno
e preservao de ecossistemas existentes. Aponte
dois aspectos econmicos positivos trazidos pela
apicultura, em especial para a agricultura familiar.
4. (UFMG) Analise este fluxograma:

Queimadas na Floresta Amaznica



A partir da anlise desse fluxograma e considerando-se
outros conhecimentos sobre o assunto, incorreto
afirmar que
a) a inflamabilidade da floresta decorre de aes
humanas associadas, direta ou indiretamente, a
causas naturais.
b) a reduo da cobertura florestal, ao comprometer a
evapotranspirao, pode, a longo prazo, acarretar
reduo das chuvas.
c) o aumento do nmero e da intensidade das
queimadas na Amaznia pode tornar-se, num ciclo
vicioso, um processo de retroalimentao.
d) o fenmeno El Nio tem relao direta, mas
favorvel, com a reduo das queimadas na
Amaznia brasileira.

UNIDADE 2

OS CLIMAS DO ESPAO BRASILEIRO

CLIMAS DO BRASIL


O CLIMA DO BRASIL
Com a maior parte do seu territrio na Zona
Intertropical do planeta, o Brasil dominado por
climas quentes equatorial e tropical. Somente no Sul
que notamos a presena de clima subtropical, com
ocorrncia de geadas no inverno (podendo at ocorrer
nevadas, pois os meses de inverno so midos no
Planalto Meridional do Paran para o Sul).
Entre os fatores que influenciam o clima, so
destacados como influncia direta:
Please purchase PDF Split-Merge on www.verypdf.com to remove this watermark.
Incluso para a Vida Geografia B

Pr-Vestibular da UFSC

4
- a latitude, pois o Brasil possui uma extenso Norte-Sul superior a 4.300 Km, tendo aproximadamente 92% do
territrio na faixa intertropical e o restante na rea subtropical;
- a altitude, caracterstica de algumas reas, especialmente do sul e do sudeste, onde h reas com altitudes mais
elevadas, o que faz as temperaturas oscilarem com maior frequncia que o restante do pas;
- as massas de ar, pois o Brasil recebe influncias da massa Equatorial continental (eMc), massa Equatorial
atlntica (mEa), massa Tropical continental (mTc), massa Tropical atlntica (mTa) e massa Polar atlntica (mPa)



Fonte: SIMIELLI, M.E. GEOATLAS. So Paulo: tica, 2003.








Please purchase PDF Split-Merge on www.verypdf.com to remove this watermark.
Incluso para a Vida Geografia B

Pr-Vestibular da UFSC

5
Tipos Climticos
- Equatorial: Quente e chuvoso, tpico na regio da
Floresta Amaznica.

FONTE: VESENTINI, J.W.
Geografia do Brasil. So Paulo:
tica, 2001.

- Tropical Atlntico: caracterstico do litoral
brasileiro, sofrendo variaes trmicas medida que se
afasta do Equador.

FONTE: VESENTINI, J.W.
Geografia do Brasil. So Paulo:
tica, 2001.

FONTE: VESENTINI, J.W.
Geografia do Brasil. So Paulo:
tica, 2001.

- Tropical Semi-rido: caracteriza o serto
nordestino, com chuvas irregulares e mal distribudas.


VESENTINI, J.W. Geografia do
Brasil. So Paulo: tica, 2001.

Tropical de Altitude: na regio da Serra da
Mantiqueira e Serra do Mar, entre os estados de SP,
MG e RJ, esse clima tem suas temperaturas amenizadas
pela altitude, aproximando-se das caractersticas
subtropicais do Brasil.
Subtropical: o clima caracterstico da regio ao Sul
do Trpico de Capricrnio (Sul de SP, PR, SC e RS).
Ela sofre ao direta da mPa. o clima mais regular do
Brasil e o nico que apresenta as estaes claramente
definidas, com vero muito quente e invernos
rigorosos.

FONTE: VESENTINI, J.W.
Geografia do Brasil. So Paulo:
tica, 2001.
O CLIMA COMO RECURSO NATURAL
O clima, entendido como manifestao
habitual da atmosfera num determinado ponto, um
dos importantes recursos naturais disposio do
homem e foi considerado matria de interesse comum
da humanidade por deciso da ONU em 1989. um
dos principais fatores responsveis pela repartio dos
animais e vegetais sobre o globo.
Da mesma forma, a gua doce (...) tem sua
distribuio e seus estoques determinados, em grande
parte, pelas condies do clima.
A atividade agrcola e o rendimento das
colheitas dependem, fundamentalmente, da evoluo
do tempo. Se este for desfavorvel, a produo poder
ficar comprometida.
Please purchase PDF Split-Merge on www.verypdf.com to remove this watermark.
Incluso para a Vida Geografia B

Pr-Vestibular da UFSC

6
A gerao de energia hidreltrica (...)
depende fundamentalmente do fluxo hdrico, e este, do
comportamento do tempo. (...)
O vento, j no sculo XVI, era utilizado
como importante fonte de energia, principalmente para
impulsionar as embarcaes que, nessa poca,
comeavam a alargar os horizontes geogrficos. (...)
A radiao solar tambm cada vez mais
aproveitada para a gerao de energia em escala
industrial, com vantagem de ser assim como os ventos
e os rios, uma fonte inesgotvel e limpa. (...)
Captando o calor diretamente do Sol, (...)
reas esto destinadas a se tornarem, quando as
limitaes tecnolgicas houverem sido superadas, as
grandes fornecedoras de energia para o consumo
industrial.
As caractersticas de seu clima preparam-
nas para essa importante funo no mundo futuro.
ROSS, J. L. S. Geografia do Brasil. So Paulo: Edusp, 1998. p. 87-
88.
Exerccios de Sala #

1. (UFRGS) Observe o mapa de climas do Brasil e os
trs climogramas que seguem.




Assinale a correspondncia correta entre as localidades
A, B e C assinaladas no mapa e os climogramas I, II e
III.
a) A (I) B (II) C (III) d) A (II) B (I) C (III)
b) A (II) B (III) C (I) e) A (III) B (II) C (I)
c) A (III) B (I) C (II)

2. (UFRGS) A relao entre eventos meteorolgicos e
as caractersticas de ocupao do territrio resultou em
catstrofes no estado de Santa Catarina em 2008.

Assinale com V (verdadeiro) ou F (falso) os fatores a
seguir, conforme eles estejam ou no relacionados a
essas catstrofes,

( ) Combinao de frentes frias vindas do sul e massas
de ar quentes e midas vindas do norte do pas,
( ) Influncia da corrente martima quente vinda do
sul, conhecida como corrente das Malvinas.
( ) Expanso da ocupao humana nas reas de risco
no btoma Mata Atlntica.
( ) Chuvas torrenciais que geram deslizamentos de
encostas,

A sequncia correta de preenchimento dos parnteses,
de cima para baixo, e
a) V - V - F - V. d) F - V - F - F.
b) F - F - V - F. e) F - V - V - V.
c) V - F - V - V.

3. (G1)

Please purchase PDF Split-Merge on www.verypdf.com to remove this watermark.
Incluso para a Vida Geografia B

Pr-Vestibular da UFSC

7
Sobre a dinmica das massas de ar que atuam no
territrio brasileiro no inverno e no vero, conforme o
mapa acima, assinale a proposio correta.
a) A massa Tropical continental (mTc), quente e
mida, tem como centro de origem o Nordeste da
Amaznia.
b) A massa Equatorial continental (mEc) responsvel
pelo fenmeno conhecido como friagem na
Regio Norte.
c) A massa Tropical atlntica (mTa), quente e seca, tem
como centro de origem o Atlntico Sul. A mTa atua
durante todo o ano na Amaznia ocidental e, no
vero, provoca as chuvas orogrficas em todas as
demais regies brasileiras.
d) A massa Equatorial atlntica (mEa), quente e mida,
tem como centro de origem o Atlntico Sul.
e) A massa Polar atlntica (mPa) tem como centro de
origem o Atlntico Sul. Essa massa de ar
responsvel pela precipitao de neve durante o
inverno nas regies serranas dos estados de Santa
Catarina e do Rio Grande do Sul.








4. (UFPR) Nesta tera-feira (15/09/09), reas de
instabilidade que se deslocam pelo norte da Argentina
devem chegar ao Brasil a partir da tarde e voltam a
provocar pancadas de chuva no oeste e norte do RS, no
centro-oeste de SC, no oeste do PR e no sul de MS,
onde tem-se uma massa de ar quente e mida.

O texto acima refere-se previso do tempo para o dia
15/09/09, realizada pelo Centro de Previso do Tempo
e Estudos Climticos do Instituto Nacional de
Pesquisas Espaciais. Levando em considerao os
dados apresentados, assinale a alternativa correta.
a) A Frente Polar Atlntica, principal rea de
instabilidade da Amrica do Sul meridional,
responsvel pelas chuvas previstas no texto.
b) As reas de instabilidade so geradas por nuvens de
desenvolvimento vertical, por isso a previso de
pancadas de chuva.
c) As pancadas de chuva so tpicas dos climas midos,
muito bem representados pelas regies mencionadas
no texto.
d) O deslocamento da massa de ar tropical em direo a
leste que gera as reas de instabilidade
mencionadas no texto.
e) A massa de ar quente e mida que se encontra sobre
o estado do Mato Grosso do Sul corresponde
massa tropical continental, geradora de chuvas em
pancadas.

UNIDADE 3

GEOLOGIA E RELEVO DO BRASIL

FORMAO GEOLGICA

A formao geolgica no territrio brasileiro muito
antiga (formada nas eras Arqueozoica e Proterozoica) e
constituda por terrenos cristalinos (rochas
magmticas e metamrficas) e sedimentares.
Nos terrenos cristalinos da era
Arqueozoica, destacam-se os dois grandes ressaltos, ou
escudos, conhecidos como Guiano e Brasileiro.
Nesses terrenos, o aproveitamento econmico
pequeno. J nos terrenos Proterozoicos, que afloram
em cerca de 4% do pas (em meio s reas
Arqueozoicas), esto as nossas principais jazidas
minerais.
Os terrenos sedimentares so
predominantes no Brasil, boa parte recobrindo a base
de rochas cristalinas. Esses terrenos so conhecidos
como Bacias Sedimentares, sendo as principais:
Amaznica; Litornea ou Costeira; Pantanal;
Sanfranciscana; Paranaica; Recncavo Baiano e
Maranho-Piau. Nessas reas so encontrados carvo,
petrleo e xisto.
Please purchase PDF Split-Merge on www.verypdf.com to remove this watermark.
Incluso para a Vida Geografia B

Pr-Vestibular da UFSC

8

FONTE: MOREIRA, I. O Espao Geogrfico Geografia Geral e do Brasil. So Paulo: tica, 2003.

UNIDADES DE RELEVO
O relevo brasileiro de formao muito
antiga e bastante erodida. Assim, predomina um
terreno de altitude modesta (com mdia de 900
metros). Ele moldado apenas por agentes externos,
uma vez que a atuao dos agentes internos
praticamente inexistente em nosso territrio.
Esquematicamente, o gegrafo Jurandir Ross dividiu o
relevo brasileiro da seguinte forma:
- Planaltos terrenos irregulares (com acentuados
aclives e declives) e com altitudes superiores a 200
metros;
- Plancies terrenos com certa regularidade (sem
declives e aclives acentuados) onde prevalecem
altitudes que variam de 0 a 200 metros;
- Depresses terrenos com certa regularidade, mas
localizados em altitudes que variam de 200 a 500
metros.
Essa classificao, concluda em 1995, foi
fundamentada com as pesquisas do Projeto
Radambrasil. Segundo essa classificao, o relevo
brasileiro est dividido em 11 Planaltos, 11
Depresses e 6 Plancies.













Please purchase PDF Split-Merge on www.verypdf.com to remove this watermark.
Incluso para a Vida Geografia B

Pr-Vestibular da UFSC

9
UNIDADES DE RELEVO DO BRASIL

FONTE: ROSS,J.L.S.(Org.), Geografia do Brasil. 2.ed. So Paulo:
Edusp, 1998.
Entretanto, existem algumas classificaes mais
tradicionais do relevo brasileiro, como a que foi
realizada na dcada de 1960 pelo renomado gegrafo
AbSaber. Nessa classificao, o relevo do Brasil
dividido da seguinte forma:



Planaltos

- Guiano (ou das Guianas)
- Brasileiro, sendo subdividido em:
Central, Meridional, Nordestino, Serras e
Planaltos do Leste-Sudeste, Maranho-
Piau e Uruguaio-Rio-Grandense.


Plancies

- Plancies e terras baixas amaznicas;
- Plancies e terras baixas costeiras;
- Plancies do Pantanal.

Exerccios de Sala #
1. (UDESC) Sobre o relevo catarinense, assinale a
alternativa incorreta.
a) As depresses catarinenses circundam todas as
serras, algumas abaixo do nvel do mar.
b) A maior parte do relevo compe-se de terrenos
planos e altos.
c) Entre os planaltos e as plancies aparecem as serras.
d) Destacam-se dois importantes conjuntos de serras: a
Serra do Mar e a Serra Geral.
e) As plancies so regies predominantes do litoral,
tambm chamadas de plancies costeiras.

2. (UFG) Segundo os gegrafos Aroldo de Azevedo
(1948) e Aziz Ab' Saber (1956), no Planalto Meridional
do Brasil destaca-se a ocorrncia de solos de terra roxa,
caracterizados por elevada fertilidade natural e por isso
muito utilizados nas atividades agrcolas. O tipo de
rocha, a estrutura geolgica que d origem ao solo de
terra roxa e a atividade agrcola historicamente nele
desenvolvida so, respectivamente:
a) o basalto, que uma rocha gnea extrusiva da Bacia
Sedimentar do Paran, onde se desenvolveu o
cultivo de caf.
b) o arenito, que uma rocha sedimentar marinha da
Bacia Sedimentar do Maranho, onde se
desenvolveu a plantao de arroz.
c) o granito, que uma rocha gnea intrusiva do Escudo
Cristalino do Brasil Central, onde se desenvolveu o
cultivo de feijo.
d) o gnaisse, que uma rocha metamrfica bandeada
do Escudo Cristalino Atlntico, onde se
desenvolveu o plantio de laranja.
e) o diabsio, que uma rocha gnea extrusiva da Bacia
Sedimentar da Amaznia, onde se desenvolveu o
cultivo de pimenta-do-reino.

3. (UFRGS) A coluna da esquerda, a seguir, apresenta
o nome dos dois biomas que ocorrem no Rio Grande
do Sul; a da direita, as unidades de relevo do estado.
Associe adequadamente a coluna da direita da
esquerda.

1 Mata Atlntica ( ) Plancie Costeira
2 Pampa ( ) Depresso perifrica
( ) Planalto Meridional
( ) Escudo Sul-Rio-
Grandense

A sequncia correta de preenchimento dos parnteses,
de cima para baixo,
a) 1 1 2 1. d) 2 1 2 2.
b) 1 1 2 2. e) 2 2 1 2.
c) 1 2 1 1.

4. (UNEMAT) Segundo Ross (1995), o relevo
brasileiro apresenta grande variedade morfolgica,
decorrente, principalmente, da ao de agentes
externos, sobre os agentes internos. Os agentes
externos que mais participam da formao do relevo
so:
a) abalos ssmicos e vulces.
b) as erupes vulcnicas do passado e os raios solares.
c) a eroso e umidade.
d) o clima (temperatura, ventos, chuvas) e os rios.
e) as intempries e a ao antrpica.

Please purchase PDF Split-Merge on www.verypdf.com to remove this watermark.
Incluso para a Vida Geografia B

Pr-Vestibular da UFSC

10
UNIDADE 4 E 5

VEGETAO ORIGINAL E DOMNIOS
MORFOCLIMTICOS BRASILEIROS

No Brasil, por causa da imensido do
territrio, existem vrias paisagens vegetais, ou
biomas. Abordamos como vegetao original, pois esse
o primeiro elemento das paisagens que o ser humano
modifica. As formaes vegetais que prevalecem no
espao brasileiro so:
- Floresta Amaznica: Tambm denominada de
latifoliada equatorial, est localizada no Norte do pas e
abrange cerca de 50% da rea total do pas. uma
floresta heterognea, de mata densa e intricada (com
milhares de espcies vegetais), e perene, ou seja,
sempre verde.
- Floresta Atlntica: Conhecida tambm como
floresta latifoliada tropical,corresponde, mais ou
menos, ao clima tropical mido. Est localizada entre o
litoral e o interior do pas. So espcies tpicas dessa
floresta o pau-brasil, o cedro, a peroba e o jacarand.
Aproximadamente 96% de sua rea original j foi
dizimada pela colonizao.
- Mata de Araucrias: Essa formao vegetal, que
tambm recebe o nome de floresta aciculifoliada,
corresponde, mais ou menos, s reas de clima
subtropical. Nela predominam os pinheiros (Araucria
angustiflia), embora tambm aparea a erva-mate, a
imbuia, diversos tipos de canela, cedros e ips.
Calcula-se que apenas 5% de sua mata original ainda
permanece intocada.
- Cerrado: Essa formao surge em reas de clima
tropical tpico do Planalto Central, com predomnio de
arbustos e vegetao rasteira. Tambm denominado de
savana, aproximadamente 45% de sua vegetao foi
destruda.
- Caatinga: uma vegetao tpica do semi-rido
nordestino. Apresenta vegetao pobre, prevalecendo
as xerfilas e cactceas.
- Complexo Pantanal: Trata-se de uma paisagem
complexa, pois uma mistura de outras formaes
vegetais brasileiras (cerrado, amaznica e floresta
atlntica).
- Campos: um tipo de vegetao rasteira
localizada principalmente no Sul do Brasil.
- Vegetaes Litorneas: So caractersticas das
terras baixas e plancies do litoral. A aparecem os
manguezais, a vegetao de praias, a das dunas e das
restingas.
No lugar da expresso paisagem natural,
alguns gegrafos costumam utilizar a denominao de
Domnio Morfoclimtico (morfo , forma, que, nesse
caso, se refere ao relevo; climtico, relativo ao
clima). No Brasil, podemos reconhecer seis principais
domnios e, entre eles, numerosas faixas de transio
(com elementos de dois ou mais deles, que no so
classificados).
Vegetao Original do espao brasileiro

VESENTINI, J. W. "Sociedade e Espao: Geografia Geral e do
Brasil". So Paulo: tica, 1999. p. 261.


DOMNIOS MORFOCLIMTICOS DO
BRASIL

Os Domnios Morfoclimticos Brasileiros so
regies com diferenas gritantes entre as caractersticas
climticas, botnicas, pedolgicas, hidrolgicas e
fitogeogrficas demarcadas pelo gegrafo brasileiro
Aziz abSaber. Essa classificao relativamente
recente, feita em 1970, divide o extenso territrio
brasileiro em seis partes muito distintas umas das
outras.


















Please purchase PDF Split-Merge on www.verypdf.com to remove this watermark.
Incluso para a Vida Geografia B

Pr-Vestibular da UFSC

11
DOMNIOS MORFOCLIMTICOS (Azis
AbSaber)
FONTE: MOREIRA, I. O Espao Geogrfico Geografia Geral e
do Brasil. So Paulo: tica, 2003.


A ocupao da Amaznia
Dentre os problemas ecolgicos que ocorrem no Brasil,
um dos mais graves refere-se devastao da floresta
Amaznica. Dentre as consequncias desse processo,
esto:

- a perda da biodiversidade e extino de espcies;
- expulso de indgenas e posseiros;
- aumento de gs carbnico na atmosfera, causado
pelas grandes queimadas;
- empobrecimento dos solos da regio.

Calcula-se que a cada ano ocorram, no
mnimo, um desmatamento de 3 milhes de hectares. A
floresta j perdeu 16,3% de sua vegetao primria
desde a dcada de 1970. Para conter esse avano, o
governo iniciou, em 2002, as atividades do Sistema de
Vigilncia da Amaznia (Sivam), com a inaugurao
de centrais regionais de vigilncia de Manaus e Porto
Velho. Ao ser concludo (no custo de 1,4 bilho de
dlares), o projeto contar com 5 satlites, 25 radares,
33 avies e mais de 2,1 mil funcionrios.

Exerccios de Sala #

1. (FATEC) Os cerrados brasileiros so
formados por rvores com aspecto
xeromrfico, com rvores tortuosas e
espaadas, com troncos de cortia espessa
e folhagem coricea e pilosa, muitas vezes
lembrando a caatinga arbustiva densa, da
regio do semirido nordestino.
[Adaptado de: ROSS, J. (org.). "Geografia
do Brasil". So Paulo: Edusp, 1996].

O fator que pode explicar tal semelhana
fisionmica entre os dois tipos de
vegetao :
a) a baixa umidade nos solos do cerrado,
com rvores com menor capacidade de
captar e armazenar gua do ambiente.
b) a baixa fertilidade natural dos solos do
cerrado, em geral muito cidos, pobres em
clcio e nutrientes em geral.
c) a vigncia de um clima tropical seco e de
altitude no cerrado, responsvel por
invernos mais chuvosos e veres mais
quentes e secos.
d) o uso intensivo das queimadas como
fator de manejo e controle do cerrado, para
eliminao de gramneas.
e) o extenso desmatamento do domnio dos
cerrados para a produo de soja e gado,
tornando a regio mais seca.

2. (UEL) No Brasil, a retomada do
crescimento econmico, a partir de 2004, teve como
consequncia o aumento da demanda de carvo vegetal
para o abastecimento das indstrias siderrgicas de
Minas Gerais e, ao mesmo tempo, a diminuio dos
investimentos aplicados no replantio de florestas
destinadas produo desse recurso.

Com base nos conhecimentos sobre o tema, assinale a
alternativa que corretamente identifica a formao
vegetal diretamente afetada pela maior demanda de
carvo vegetal em Minas Gerais.
a) Caatinga.
b) Cerrado.
c) Campos Gerais.
d) Mata Atlntica.
e) Mata de Araucria.

3. (UFG) Segundo uma reportagem do jornal O Globo
(nov. 2009), entre os meses de agosto de 2008 a julho
de 2009 foram desmatados, na Amaznia, 7.008 km
2

de floresta, de acordo com dados do Instituto Nacional
de Pesquisas Espaciais. Apesar de esse nmero
significar uma reduo de 45% em relao ao ano
anterior, o desmatamento ainda origina diversos
prejuzos socioambientais Floresta Amaznica,
causando
Please purchase PDF Split-Merge on www.verypdf.com to remove this watermark.
Incluso para a Vida Geografia B

Pr-Vestibular da UFSC

12
a) diminuio da fertilidade dos solos, comprometendo
a potencialidade agrcola.
b) aumento da poluio do ar, provocando chuvas
cidas que impedem o desenvolvimento da
agricultura.
c) diminuio da fauna, prejudicando as atividades
tursticas.
d) aumento da eroso elica, comprometendo o
calendrio agrcola tradicional das populaes.
e) diminuio dos nveis fluviais, alterando os usos e
as apropriaes econmicas dos rios.

4. (G1) No livro Os Sertes, durante o primeiro
captulo intitulado A Terra, Euclides da Cunha faz
uma descrio das caractersticas paisagsticas do local
em que ocorreu a Guerra de Canudos. A seguir
trazemos um trecho do livro em que so descritas
algumas caractersticas da vegetao do local de
conflito:
Embora esta no
tenha as espcies reduzidas
dos desertos mimosas
tolhias ou eufrbias speras
sobre o tapete das gramneas
murchas e se afigure farta
de vegetais distintos, as suas
rvores, vistas em conjunto,
semelham uma s famlia de
poucos gneros, quase
reduzida a uma espcie
invarivel, divergindo apenas
no tamanho, tendo todas a
mesma conformao, a mesma
aparncia de vegetais
morrendo, quase sem troncos,
em esgalhos logo ao irromper
do cho. que por um efeito
explicvel de adaptao s
condies estreitas do meio
ingrato, evolvendo
penosamente em crculos
estreitos, aquelas mesmo que
tanto se diversificam nas matas
ali se talham por um molde
nico.

A partir desse trecho, podemos deduzir que a paisagem
do local em que ocorreu a Guerra de Canudos
caracterizada pela:
a) presena da Mata Atlntica, vegetao comum em
regies de clima tropical mido.
b)presena da Mata de Araucria, vegetao
caracterstica de regies de clima subtropical.
c) presena da Caatinga, vegetao tpica de regies de
clima tropical semirido.
d) presena da Floresta Amaznica, vegetao muito
presente em regies de clima mediterrneo.
e) presena de Mangues, vegetao litornea de clima
tropical de altitude.


UNIDADE 6

RECURSOS HDRICOS

O Brasil possui a maior reserva mundial de recursos
hdricos. Abriga, em seu territrio, uma das maiores
redes hidrogrficas do planeta metade de toda a gua
disponvel da Amrica do Sul -, alm de extensas
reservas de gua subterrneas. Apesar de todo esse
potencial, o pas no est livre do problema da escassez
de gua.
O uso predatrio dos recursos hdricos,
poluio, assoreamento dos rios e desperdcio so os
principais responsveis pela escassez de gua.

Bacias hidrogrficas brasileiras

O Brasil, dada a sua grande extenso territorial e a
predominncia de climas midos, tem uma extensa
rede hidrogrfica. As bacias hidrogrficas brasileiras
oferecem, em muitos trechos, grandes possibilidades de
navegao. Apesar disso, o transporte hidrovirio
pouco utilizado no pas. Em outros trechos, nossos rios
apresentam um enorme potencial hidreltrico, bastante
explorado no Centro-Sul do pas em decorrncia da
concentrao urbano-industrial, mas sub-utilizado em
outras regies, como a Amaznia.
Tecnicamente, a hidrografia brasileira
apresenta os seguintes aspectos:
- No possui lagos tectnicos, pois as
depresses tornaram-se bacias sedimentares. Em nosso
Please purchase PDF Split-Merge on www.verypdf.com to remove this watermark.
Incluso para a Vida Geografia B

Pr-Vestibular da UFSC

13
territrio, s h lagos de vrzea (temporrios, muito
comuns no Pantanal) e lagoas costeiras, como a dos
Patos (RS) e a Rodrigo de Freitas (RJ), formadas por
restingas.
- Todos os rios brasileiros, com exceo do
Amazonas, possuem regime pluvial. Uma pequena
quantidade da gua do rio Amazonas provm do
derretimento de neve na cordilheira dos Andes,
caracterizando um regime misto (nival e pluvial).
- Todos os rios so exorreicos; mesmo os que
correm para o interior tm como destino final o oceano,
como o Tiet, afluente do rio Paran, que por sua vez
desgua no mar (esturio do Prata).
- H rios temporrios apenas no Serto
nordestino, onde o clima semi-rido. No restante do
pas, os rios so perenes.
- Predominam rios de planalto em reas de
elevado ndice pluviomtrico. A existncia de muitos
desnveis no terreno e o grande volume de gua
possibilitam a produo de hidroeletricidade.
- Com exceo do rio Amazonas, que possui foz
mista (delta e esturio), e do rio Parnaba, que possui
foz em delta, todos os rios brasileiros que desguam
livremente no oceano formam esturios.

As principais bacias hidrogrficas brasileiras so:

- Bacia do rio Amazonas: a maior bacia
hidrogrfica do planeta tem sua vertente delimitada
pelos divisores de gua da cordilheira dos Andes, pelo
planalto das Guianas e pelo planalto Central. Seu rio
principal nasce no Peru, com o nome de Maraon, e
passa a ser denominado Solimes da fronteira brasileira
at o encontro com o rio Negro. A partir da, recebe o
nome de Amazonas. o rio mais extenso (total de
7.100 km) e de maior volume de gua do planeta. Esse
fato explicado pela presena de afluentes de ambos os
lados que, por estarem nos dois hemisfrios (norte e
sul), permitem a dupla captao das cheias de vero.
Os afluentes do rio Amazonas nascem, em sua maioria,
nos escudos dos planaltos das Guianas e Brasileiro,
possuindo, assim, o maior potencial hidreltrico
disponvel do pas. Ao carem na bacia sedimentar, que
plana, tornam-se rios navegveis. O rio Amazonas,
que corre no centro da bacia, totalmente navegvel.

- Bacia do rio Tocantins: esta bacia drena,
aproximadamente, 9,5% do territrio nacional. Seus
principais rios nascem no estado de Gois e no Bico do
Papagaio (TO), onde o Tocantins recebe seu principal
afluente, o rio Araguaia. Em terras paraenses, o
Tocantins desgua no Golfo Amaznico, onde se
localiza a ilha de Maraj. Por apresentar longos trechos
navegveis, essa bacia utilizada para escoar parte da
produo de gros (destaque para a soja) das regies
que banha. A usina hidreltrica de Tucuru, a segunda
maior do pas, foi construda no rio Tocantins e atende
s necessidades de consumo de energia do Projeto
Carajs, no Par.

- Bacia Platina (composta pela bacia do
Paran e bacia do Uruguai): o Brasil tambm
banhado pela segunda maior bacia hidrogrfica do
planeta. Seus trs rios principais Paran, Paraguai e
Uruguai formam o rio da Prata, ao se encontrarem,
em territrio argentino. A bacia do rio Paran apresenta
o maior potencial hidreltrico instalado do pas, alm
de trechos importantes para a navegao, com destaque
para a hidrovia do Tiet. A bacia do Paraguai, que
atravessa o Pantanal Mato-grossense, amplamente
navegvel. J a bacia do Uruguai, com pequeno
potencial hidreltrico e poucos trechos navegveis, tem
importncia econmica apenas regional.

- Bacia do rio So Francisco: uma extensa
bacia hidrogrfica, responsvel pela drenagem de
aproximadamente 7,5% do territrio nacional. O rio
So Francisco, que nasce em Minas Gerais, atravessa o
serto semi-rido mineiro e baiano, possibilitando a
sobrevivncia da populao ribeirinha de baixa renda, a
irrigao em pequenas propriedades e em grandes
projetos agroindustriais e a criao de gado. O So
Francisco um rio bastante aproveitado para a
produo de hidroeletricidade. Ele navegvel em um
longo trecho dos estados de Minas Gerais e Bahia,
desde que a barragem de Trs Marias no lhe retenha
muita gua.

- Bacias secundrias: o Brasil possui trs
conjuntos de bacias secundrias: Atlntico Norte-
Nordeste, Atlntico Leste e Atlntico Sudeste. As
bacias hidrogrficas que os compem no possuem
ligao entre si. Elas foram agrupadas por sua
localizao geogrfica ao longo do litoral. O rio
principal de cada uma delas tem sua prpria vertente,
delimitando, portanto, uma bacia hidrogrfica. Por
exemplo, as bacias do Atlntico Leste so formadas
pelo agrupamento das bacias do Paraba do Sul, Doce,
Jequitinhonha, Pardo, Contas e Paraguau.

Aqufero Guarani, um mar potvel subterrneo
Denominam-se aquferos as reservas de gua
subterrneas que representam uma alternativa
estratgica ao problema de falta de gua. No Brasil, o
principal deles o aqufero Guarani, maior reserva de
gua doce da Amrica do Sul. (...)
Esse verdadeiro mar potvel subterrneo
estende-se por cerca de 1,2 milho de Km
2
, dois teros
dos quais em territrio brasileiro, onde atinge oito
estados do centro-sul do pas (MT, MS, GO , MG, SP,
PR, SC e RS). O restante prolonga-se por Uruguai,
Argentina e Paraguai. (...)
Adaptado de Aureliano Biancarelli para o jornal Folha
de So Paulo, 19 de maio de 1996.







Please purchase PDF Split-Merge on www.verypdf.com to remove this watermark.
Incluso para a Vida Geografia B

Pr-Vestibular da UFSC

14
Fique ligado...


























Exerccios de Sala #

1. (UFSM) Analise os grficos.

Considerando os dados fornecidos pelos grficos,
correto afirmar:
a) O setor agrcola apresenta os maiores volumes de
captao de gua, e a rede de abastecimento
domstico, as maiores perdas.
b) No futuro, a perda de gua por evaporao dever
superar o volume de gua captado para uso
industrial.
c) Independente do setor analisado, o percentual de
perda de gua se manteve mais ou menos constante
ao longo do sculo XX.
d) Durante o perodo analisado, o setor agrcola foi o
que apresentou o menor crescimento na captao de
gua.
e) A captao de gua para consumo industrial s
supera o volume captado para uso domstico a partir
de 1975.

2. (UFC) Os rios so correntes naturais de gua doce,
com canais definidos e fluxos perenes ou intermitentes
que desembocam nos oceanos, lagos ou em outros rios.
Nessa condio, os rios realizam aes de
transformao das paisagens e tm grande importncia
social.
a) Cite os processos associados aos rios a partir dos
quais ocorre a transformao das paisagens
naturais.
b) Aponte trs situaes de uso dos rios pela sociedade.
c) Aponte a maior e mais importante bacia hidrogrfica
do Cear e nomeie os dois maiores audes nela
localizados.

C.I. Bacia hidrogrfica:

3. (UEG) O Brasil dotado de uma vasta rede
hidrogrfica. Muitos de seus rios destacam-se pela
extenso, largura, profundidade e volume de gua
escoado, tornando o pas detentor de uma das maiores
reservas de gua doce do mundo. Apesar desta
realidade, o pas j enfrenta problemas no que concerne
ao abastecimento urbano de gua potvel (como no
caso de So Paulo), alm de conflitos no campo pela
distribuio de gua para as atividades da agricultura e
pecuria. Com base nestas informaes, responda ao
que se pede.
a) Caracterize o potencial das bacias hidrogrficas do
Paran e do Amazonas para o abastecimento de gua
potvel nos centros urbanos, considerando a atual
distribuio geogrfica da populao sobre o territrio
nacional.
b) Cite e explique dois impactos negativos decorrentes
da utilizao dos recursos hdricos da Bacia do Paran.

4. (Mackenzie) "... Bacia hidrogrfica amplamente
navegvel, pois atravessa regies de relevo pouco
acidentado no pantanal mato-grossense e, por essa
mesma razo, apresenta pequeno potencial hidreltrico,
sofrendo um intenso processo de inundao durante as
chuvas de vero, fenmeno responsvel pela
denominao de Pantanal."
Paulo Roberto Moraes, "Geografia Geral e do Brasil".

A bacia hidrogrfica a que se refere o texto a:
a) Bacia Platina, sub-bacia do Rio Paran.
b) Bacia do Uruguai, sub-bacia do Rio Paran.
c) Bacia Platina, sub-bacia do Rio Paraguai.
d) Bacia do Paran, sub-bacia do Rio Uruguai.
e) Bacia do Uruguai, sub-bacia do Rio Cuiab.

Please purchase PDF Split-Merge on www.verypdf.com to remove this watermark.
Incluso para a Vida Geografia B

Pr-Vestibular da UFSC

15
UNIDADE 7 E 8
A DIVISO REGIONAL DO ESPAO
BRASILEIRO E A REGIO SUL DO BRASIL
AS REGIES DO BRASIL

Regio natural a poro territorial que
apresenta um quadro fsico comum relevo, clima,
vegetao, hidrografia. o caso do Pantanal Mato-
Grossense, da Campanha Gacha ou do Serto
Nordestino.
Com mais de 8,5 milhes de km
2
, o Brasil
apresenta diferentes aspectos naturais, humanos e
econmicos em seu territrio. Considerando-se tais
diferenas, reas relativamente homogneas so
agrupadas em regies.
A diviso regional do Instituto Brasileiro de
Geografia e Estatstica (IBGE) identifica cinco regies
no Brasil: Norte, Nordeste, Sudeste, Sul e Centro-
Oeste.
Alm dos fatores naturais, humanos e
econmicos, essa diviso considera os limites dos
estados.


REGIES GEOECONMICAS
Regio geoeconmica a poro territorial
que apresenta um mesmo quadro sociocultural e
econmico, quase sempre como conseqncia de um
quadro fsico comum.
No caso das regies geoeconmicas
brasileiras, so consideras a presena de trs grandes
complexos regionais no pas: Amaznia, Nordeste e
Centro-Sul. Tal diviso leva em conta os aspectos
fsicos, humanos e econmicos, desprezando os limites
estaduais e privilegiando os aspectos geogrficos mais
marcantes na definio de tais reas: o quadro natural
na Amaznia, o aspecto social nordestino e o quadro
econmico no Centro-Sul.



ASPECTOS FSICOS E SCIOECONMICOS
DA REGIO SUL
A Regio Sul do Brasil, definida pelo IBGE,
consta dos Estados do Paran, Santa Catarina e Rio
Grande do Sul. Ela representa 6,8% do territrio
nacional e possui, em um densidade demogrfica de 41
hab./km
2
, 15% do total da populao do pas.
As caractersticas do clima subtropical no
favoreceram o povoamento especulativo nessa regio,
como ocorreu na fachada atlntica do pas. A ocupao
iniciou-se com a criao de gado para a produo de
charque e de couro, mas s se efetivou com a imigrao
europeus do sculo XIX, principalmente de italianos,
Please purchase PDF Split-Merge on www.verypdf.com to remove this watermark.
Incluso para a Vida Geografia B

Pr-Vestibular da UFSC

16
alemes e eslavos, que introduziram formas novas de
aproveitamento econmico do espao: pequena
propriedade, o policultivo, a associao agricultura-
pecuria e a explorao direta da terra.
A produo agropecuria muito diversificada:
trigo e soja plantados no planalto; arroz e l, no RS;
couro e carne na Campanha Gacha; milho, feijo,
batata, fumo e outros produtos.
Entre 1970 e 2000, a participao da regio no
total da produo industrial brasileira subiu de 11%
para 18%. No Paran, o principal parque industrial
localiza-se em torno da capital, com indstrias
alimentcias, madeireiras, indstrias qumicas, de
material eltrico, de transporte e automobilstica. A
produo industrial do Rio Grande do Sul est
concentrada na regio metropolitana de Porto Alegre
secundada pela rea de Caxias do Sul, importante plo
metal-mecnico. Em Santa Catarina, a indstria
alimentcia e de vesturio, que atuam desde o incio de
sculo XX, modernizaram-se graas aos capitais
agroindustriais; indstrias mecnicas e de material
eltrico tambm foram atradas pelo estado. Diferente
dos outros estados do Sul, em Santa Catarina o parque
industrial no se concentra na capital, mas est disperso
pelas reas de colonizao alem, sobretudo Blumenau
e Joinville.
Nos ltimos anos tem ocorrido um maior
desenvolvimento das atividades primrias e
secundrias, em virtude da proximidade com pases do
Mercosul.

Exerccios de Sala #

1. (UFRS) Observe o grfico a seguir, que mostra a
evoluo do nmero de municpios da regio Sul no
perodo de 1970-2000.

Com base no grfico, so feitas as seguintes afirmaes.

I. A regio Sul apresentou na ltima dcada um
crescimento significativo no nmero de municpios dos
Estados que a compem.
II. O Estado do Rio Grande do Sul tem, desde o incio
da dcada de 1980, o maior nmero de municpios entre
os trs Estados.
III. Em 1970, a regio Sul estava constituda por uma
rede de aproximadamente 700 cidades, ao passo que, no
ano 2000, esse mosaico passa a se compor de cerca de
1.000 municpios, comprovando o intenso processo de
fragmentao de seu territrio.
Quais esto corretas?
a) Apenas I. d) Apenas I e III.
b) Apenas II. e) Apenas II e III.
c) Apenas III.

2. (UFRS) Alguns tipos de poluio das guas tm
causas naturais, mas a maioria causada pelas
atividades humanas. O mapa a seguir mostra reas em
que ocorrem problemas que afetam os recursos hdricos
dos Estados do Rio Grande do Sul e de Santa Catarina.


Com base nos dados apresentados no mapa preencha as
lacunasdo texto a seguir.

As reas do mapa em que os recursos hdricos so
contaminados por efluentes com agrotxicos derivados
das lavouras de arroz so as de nmero .......... ; as
contaminadas pelos resduos provenientes de
abatedouros de porcos e aves so as de nmero .........; e
as contaminadas pelos rejeitos oriundos de atividades
mineradoras so as de nmero ..............

A alternativa que preenche corretamente as lacunas do
texto, na ordem em que aparecem,
a) 1, 2 e 3. c) 2, 1 e 3. e) 3, 2 e 1.
b) 1, 3 e 2. d) 2, 3 e 1.

3. (G1) O Complexo Regional do Centro-Sul possui
reas que se individualizam em virtude do seu
desenvolvimento econmico. Associaram-se
incorretamente as unidades desse Complexo s suas
respectivas atividades econmicas em:
a) Poro sul de Gois - cultivo de arroz e de soja.
b) Quadriltero Ferrfero - explorao de minrio de
mangans.
c) Tringulo Mineiro - fabricao de automveis e
produtos qumicos.
d) Norte do Rio de Janeiro e Esprito Santo - extrao
de petrleo.

4. (G1) A regio Sul representa 6,5% do territrio
nacional, possui 15% da populao brasileira e faz
parte do Complexo Regional do Centro-Sul.

Referindo-se a essa regio, correto afirmar que:

a) a grande propriedade e a monocultura transformaram
essa rea em um dos grandes esteios agrcolas do Pas.
Please purchase PDF Split-Merge on www.verypdf.com to remove this watermark.
Incluso para a Vida Geografia B

Pr-Vestibular da UFSC

17
b) o meio ambiente subtropical favoreceu o
povoamento especulativo como ocorreu na fachada
atlntica do Pas.
c) a expanso da fronteira agrcola, nos ltimos anos,
aumentou sua produo agropecuria e sua
participao na economia nacional.
d) a criao de uma significativa rede de transportes e
a proximidade com o Sudeste contriburam para o
desenvolvimento do seu parque industrial.

UNIDADE 9

DEMOGRAFIA BRASILEIRA

Estrutura da Populao Absoluta do Brasil



Em 2009, esperana de vida ao nascer era de 73,17
anos
Em 2009, a esperana de vida ao nascer no Brasil
alcanou os 73,17 anos. Em relao a 2008 houve alta
de 0,31 anos (3 meses e 22 dias) e, entre 1980 e 2009,
alta de 10,60 anos (10 anos, 7 meses e seis dias).
Assim, ao longo de 29 anos, esse indicador teve um
crescimento mdio anual de 4 meses e 12 dias e,
segundo Reviso 2008 da Projeo da Populao do
Brasil, poder chegar a 81,29 anos em 2050. J a
mortalidade infantil caiu de 69,12 para 22,47 bitos por
mil nascidos vivos, desde
1980.

Entre 1980 e 2009, mortalidade infantil cai de 69,12
para 22,47 bitos por mil nascidos vivos
A taxa de mortalidade infantil obteve importantes
redues no perodo: em 1980, correspondia a 69,12
bitos de menores de 1 ano para cada mil nascidos
vivos e, em 2009, passou, para 22,47%0.

Censo 2010: populao do Brasil de 190.732.694
pessoas


Please purchase PDF Split-Merge on www.verypdf.com to remove this watermark.
Incluso para a Vida Geografia B

Pr-Vestibular da UFSC

18




A Densidade Demogrfica do Brasil


FONTE: IBGE. Atlas Geogrfico Escolar. 2ed. IBGE: Rio de
Janeiro, 2004.
Exerccios de Sala #

1. (IBMECRJ) Sobre o crescimento populacional e
as suas relaes com a vida social e econmica,
assinale a alternativa correta:
a) H uma estreita relao entre a diminuio das
taxas de natalidade e o nvel de
desenvolvimento das condies de vida das
populaes.
b) Observa-se que o aumento populacional vem
sendo acompanhado de um aumento
proporcional na produo de alimentos, razo
pela qual se prev o fim da fome ao longo do
sculo XXI.
c) Quanto maior o ndice de crescimento dos
pases desenvolvidos, maiores so os seus nveis
de crescimento populacional, forma encontrada
de permitir uma distribuio mais justa da
riqueza nacional.
d) O fracasso de projetos como "cada famlia, um
filho", faz que pases como a ndia, a China e a
Austrlia experimentem ndices cada vez
maiores de crescimento populacional.
e) Um dos fatores responsveis pelos baixos nveis
de produtividade econmica dos pases
subdesenvolvidos , sem dvida, o reduzido
volume de populao infantil nessas regies.

2. (Mackenzie 2009)


Com base na tabela, e considerando o crescimento
natural da populao brasileira, observe as
afirmaes a seguir e assinale a alternativa correta.

I. Nas dcadas de 1940 e 1960, as taxas de
mortalidade eram elevadas em virtude das
precrias condies mdico-sanitrias, da
escassez de remdios e vacinas e da falta de
infraestrutura nos servios de saneamento
bsico.
II. A diminuio da taxa de mortalidade, entre as
dcadas de 1980 e 2000, ocorreu de forma
gradativa, em virtude da lenta urbanizao,
diante das dificuldades do Brasil em
industrializar-se nesse perodo.
III. A partir da dcada de 1940, o declnio da taxa
de natalidade teve relao direta e, tambm
indireta, com a urbanizao e com a
industrializao.
IV. Os fatores inibidores de natalidade, tpicos do
meio urbano, como acesso a mtodos
anticoncepcionais, entre outros, somente sero
efetivados, a partir da dcada de 2020, quando
se projeta, realmente, um crescimento natural
baixo.

a) Somente I e II esto corretas.
b) Somente II e III esto corretas
c) Somente I e III esto corretas.
d) Somente I e IV esto corretas.
e) I, II, III e IV esto corretas.

Please purchase PDF Split-Merge on www.verypdf.com to remove this watermark.
Incluso para a Vida Geografia B

Pr-Vestibular da UFSC

19
3. (UEG)

De acordo com a anlise dos grficos acima, correto afirmar:
a) nas ltimas dcadas, o crescimento vegetativo e a expectativa de vida no Brasil tm aumentado progressivamente
graas ao avano da medicina.
b) as taxas de natalidade no mundo subdesenvolvido, como no caso do Brasil, tm apresentado elevada porcentagem
em virtude da falta de polticas pblicas de controle da natalidade.
c) as duas ltimas dcadas apresentam, respectivamente, uma reduo do nmero de filhos por mulher e o aumento
do porcentual de idosos, em funo do crescimento da expectativa de vida.
d) a acelerada urbanizao, associada ao processo de industrializao e ao ingresso da mulher no mercado de
trabalho, justificam o aumento da fecundidade no Brasil nas ltimas dcadas.




4.(UERJ) A transio demogrfica que ocorre no Brasil gera diferenas socioespaciais entre as macrorregies do
pas. De acordo com os mapas, as menores propores de populao em idade ativa so encontradas na seguinte
macrorregio brasileira:
a) Sul.
b) Norte.
c) Sudeste.
d) Nordeste



Please purchase PDF Split-Merge on www.verypdf.com to remove this watermark.
Incluso para a Vida Geografia B

Pr-Vestibular da UFSC

20


Em razo da mudana de metodologia, no se pode
comparar o novo IDH com os ndices divulgados em
relatrios anteriores. Mas seguindo a nova
metodologia, em comparao com os dados
recalculados para 2009, o IDH do Brasil mostra uma
evoluo de quatro posies.

Dos trs subndices que compem o IDH,
apenas o de longevidade no passou por alteraes:
continua sendo medido pela expectativa de vida ao
nascer. No subndice de renda, o PIB (Produto Interno
Bruto) per capita foi substitudo pela Renda Nacional
Bruta (RNB) per capita, que contabiliza a renda
conquistada pelos residentes de um pas, incluindo
fluxos internacionais, como remessas vindas do
exterior e ajuda internacional, e excluindo a renda
gerada no pas, mas repatriada ao exterior. Ou seja, a

RNB traz um retrato mais preciso do bem-estar
econmico das pessoas de um pas. No subndice de
educao, houve mudanas nos dois indicadores. Sai a
taxa de analfabetismo, entra a mdia de anos de estudo
da populao adulta; para averiguar as condies da
populao em idade escolar, em vez da taxa bruta de
matrcula passa a ser usado o nmero esperado de anos
de estudos.

Please purchase PDF Split-Merge on www.verypdf.com to remove this watermark.
Incluso para a Vida Geografia B

Pr-Vestibular da UFSC

21



















Distribuio da PEA (Populao
Economicamente Ativa)





FLUXOS MIGRATRIOS NO BRASIL








Please purchase PDF Split-Merge on www.verypdf.com to remove this watermark.
Incluso para a Vida Geografia B

Pr-Vestibular da UFSC

22
Exerccios de Sala #

1. (UNIFESP) Recente pesquisa divulgada pelo IBGE apontou um crescimento da participao de mulheres como
chefes de famlia no Brasil.
a) Aponte e explique uma determinao econmica deste fato.
b) Descreva e explique uma conseqncia para o mercado de trabalho no pas.

2. Analise os mapas dos movimentos migratrios no Brasil, no perodo 1940 - 2000.


Sobre esses movimentos, correto afirmar que a regio
a) Centro-Oeste tornou-se o grande plo de atrao populacional entre 1940 e 1970, fortalecido pela construo de
Braslia.
b) Nordeste acolheu um significativo fluxo migratrio no ano de 2000, em decorrncia de sua expanso econmica.
c) Sul apresentou uma repulso demogrfica entre 1940 e 1970, devido mecanizao das lavouras.
d) Norte recebeu migrantes nordestinos entre 1970 e 1990, fixados na Amaznia Ocidental.


3. (PUCMG) Observe atentamente o grfico e, a
seguir, assinale a afirmativa incorreta.
a) O maior equilbrio entre populao rural e urbana
verificou-se no final dos anos 60.
b) O declnio da populao rural acentuou-se
significativamente a partir de meados dos anos 70.

c) O ritmo de crescimento da populao rural e urbana
promoveu um desequilbrio cada vez mais
acentuado entre elas, a partir da dcada de 70.
d) O ritmo de crescimento da populao total tornou-se
superior ao da populao urbana a partir de meados
da dcada de 90.

4. (UFSC) "Restava ainda a senzala dos tempos do
cativeiro. Uns vinte quartos com o mesmo alpendre
na frente. As negras do meu av, mesmo depois da
abolio, ficaram todas no engenho, no deixaram a
rua, como elas chamavam a senzala".
REGO, Jos Lins do. "Menino de engenho". So Paulo: Jos
Olympio, 2005. p. 83.
Please purchase PDF Split-Merge on www.verypdf.com to remove this watermark.
Incluso para a Vida Geografia B

Pr-Vestibular da UFSC

23


A partir da anlise do exposto no texto e com base nos
seus conhecimentos, assinale a(s) proposio(es)
correta(s).

01. O IBGE faz diferentes levantamentos da populao
brasileira. O mais completo deles o Censo
Demogrfico, realizado de cinco em cinco anos.
02. Os japoneses, que pertencem etnia amarela, no
ano de 2008 comemoraram o centenrio de
imigrao no Brasil.
04. Devido exclusivamente s questes econmicas, a
concentrao de negros superior nas Regies Sul
e Sudeste, em relao s demais regies brasileiras.

16. O elemento branco que participou da formao
tnica do Brasil pertence ao grupo anglo-saxo, o
nico a colonizar o Brasil.
32. Os primeiros escravos negros chegaram ao Brasil
somente com a cafeicultura, ou seja, no sculo XIX.
64. No conjunto dos elementos brancos, os portugueses
constituem o grupo mais numeroso na formao
tnica do Brasil.

UNIDADE 11 E 12

A URBANIZAO DO BRASIL

A urbanizao corresponde transferncia de
populaes originrias das zonas rurais em direo as
cidades. A urbanizao s ocorre quando a populao
das cidades cresce mais que a rural, como resultado da
migrao campo-cidade. O crescimento urbano, por sua
vez, diz respeito ao aumento da populao que vive nas
cidades e resulta apenas do crescimento natural ou
vegetativo da populao urbana.
A acelerao do processo de urbanizao no
Brasil ocorre a partir de 1940. Em 1970, a maior parte
da populao j vivia na zona urbana, o que refletiu a
modernizao econmica e no grande desenvolvimento
industrial, possvel graas a entrada em grande escala
de capital estrangeiro no pas.
Entretanto, ao mesmo tempo em que acelerou
o ritmo desenvolvimento econmico do pas, a
introduo de indstrias baseadas num padro
tecnolgico tpico dos pases desenvolvidos criou
problemas sociais. A modernizao da economia atraiu
mais de trabalhadores do que as novas atividades
conseguiam absorver, resultando em desemprego e
graves problemas sociais nas principais cidades do
Brasil.
Conceitos da Geografia Urbana

xodo Rural O processo de urbanizao brasileiro
apoiou-se essencialmente no xodo rural, ou seja, na
tranferncia de populaes do meio rural para as
cidades. O xodo rural envolve dois movimentos
interligados:
- a repulso da fora de trabalho do campo;
- a atrao da fora de trabalho para as cidades.
Metropolizao (metros= me; polis=cidade)
A palavra metrpole j designa as grandes
cidades que exercem influncia sobre extensas reas
geogrficas vizinhas. Pode ser a principal cidade de
uma regio (metrpole regional) ou de um pas
(metrpole nacional). As principais aglomeraes
urbanas do Brasil recebem o nome de Regies
Metropolitanas. Essas so reas administrativas
formadas pelos maiores municpios do pas e os
municpios a elas conurbados. Atualmente so 23
regies metropolitanas no territrio nacional.
Conurbao Quando os municpios no
apresentam limites fsicos na malha urbana.

Hierarquia Urbana - Esse conceito est
baseado na noo de rede urbana, um conjunto
integrado de cidades que estabelecem relaes
econmicas, sociais e polticas entre si. A hierarquia
urbana brasileira produz duas formas de avaliao:
Modelo Industrial ou tradicional Quanto
maior o centro urbano, mais diversificada sua infra-
estrutura econmica e maiores as possibilidades de
coordenar os principais fluxos de mercadorias e
servios, influenciando as outras cidades de sua rede.
Temos nesse modelo as metrpoles globais, metrpoles
regionais e os centros regionais.
Modelo Informacional A implantao de
modernos sistemas de transporte e de comunicaes
reduziu as distncias e possibilitou a desconcentrao
das atividades econmicas, que se difundiram por todo
o pas e hoje so coordenadas a partir de diretrizes
produzidas nos grandes centros nacionais e
internacionais. Nesse modelo as cidades no se
relacionam apenas com os centros maiores aos quais se
subordinavam na antiga hierarquia urbana, havendo
uma ruptura com a hierarquia urbana tradicional.

Populao urbana sobe de 81,25% para 84,35%
J em 2010, apenas 15,65% da populao
(29.852.986 pessoas) viviam em situao rural, contra
84,35% em situao urbana (160.879.708 pessoas).
Entre os municpios, 67 tinham 100% de sua populao
vivendo em situao urbana e 775 com mais de 90%
nessa situao. Por outro lado, apenas nove tinham
mais de 90% de sua populao vivendo em situao
rural.
Please purchase PDF Split-Merge on www.verypdf.com to remove this watermark.
Incluso para a Vida Geografia B

Pr-Vestibular da UFSC

24
Em 2000, da populao brasileira 81,25%
(137.953.959 pessoas) viviam em situao urbana e
18,75% (31.845.211 pessoas) em situao rural. Entre
os municpios, 56 tinham 100% de sua populao
vivendo em situao urbana e 523 com mais de 90%
nessa situao. Por outro lado, 38 tinham mais de 90%
vivendo em situao rural e o nico municpio do pas
a ter 100% de sua populao em situao rural era
Nova Ramada (RS).

REGIES METROPOLITANAS BRASILEIRA

IBGE mostra a nova dinmica da rede urbana
brasileira
Existem no pas doze grandes redes de
influncia, que interligam at mesmo municpios
situados em diferentes estados. A rede centralizada por
So Paulo, por exemplo, tambm abrange parte de
Minas Gerais, do Mato Grosso do Sul, Mato Grosso,
Rondnia e Acre. O Rio de Janeiro tem projeo no
prprio estado, no Esprito Santo, no sul da Bahia, e na
Zona da Mata mineira.
A rede de Braslia influi no oeste da Bahia, em
alguns municpios de Gois e no noroeste de Minas
Gerais. As outras nove redes de influncia so
centralizadas por Manaus, Belm, Fortaleza, Recife,
Salvador, Goinia, Belo Horizonte, Curitiba e Porto
Alegre.
Foram analisadas informaes fornecidas pela
rede de agncias do IBGE sobre 4.625 municpios, e
registros administrativos do prprio instituto, de rgos
estatais e empresas. A atual configurao da rede
urbana brasileira comparada com estudos feitos pelo
IBGE em 1972, 1987 e 2000.
Entre os diversos dados comparativos
coletados a respeito das 12 redes de influncia, nota-se
que, para fazer compras, a populao brasileira se
desloca cerca de 49 km, em mdia. Na rede de
influncia de Manaus, no entanto, essa distncia de
218 Km. Para freqentar uma universidade, o
Please purchase PDF Split-Merge on www.verypdf.com to remove this watermark.
Incluso para a Vida Geografia B

Pr-Vestibular da UFSC

25
deslocamento mdio, em Mato Grosso de 112 Km,
contra 41 Km na rede de influncia do Rio de Janeiro.
Pacientes percorrem, em mdia no pas, 108 km em
busca de atendimento mdico.

O estudo Regies de Influncia das Cidades
mostra as redes formadas pelos principais centros
urbanos do Pas, baseadas na presena de rgos do
executivo, do judicirio, de grandes empresas e na
oferta de ensino superior, servios de sade e domnios
de internet. Tais redes, s vezes, se sobrepem
diviso territorial oficial, estabelecendo forte influncia
at mesmo entre cidades situadas em diferentes
unidades da federao.
Para definir os centros da rede urbana
brasileira, buscam-se informaes de subordinao
administrativa no setor pblico federal, no caso da
gesto federal, e de localizao das sedes e liais de
empresas, para estabelecer a gesto empresarial. A
oferta de equipamentos e servios informaes de
ligaes areas, de deslocamentos para internaes
hospitalares, das reas de cobertura das emissoras de
televiso, da oferta de ensino superior, da diversidade
de atividades comerciais e de servios, da oferta de
servios bancrios, e da presena de domnios de
Internet complementa a identicao dos centros de
gesto do territrio.
.
Hierarquia das metrpoles e centros tecem
as redes de influncia
Foram identicadas 12 redes de primeiro
nvel. As cidades foram classicadas em cinco nveis,
por sua vez subdivididos em dois ou trs subnveis:

1. Metrpoles Os 12 principais centros urbanos do Pas, com grande porte, fortes relacionamentos entre si e, em
geral, extensa rea de influncia direta. Tm trs subnveis:
a. Grande metrpole nacional So Paulo, o maior conjunto urbano do Pas, com 19,5 milhes de habitantes, em
2007, e no primeiro nvel da gesto territorial; b. Metrpole nacional Rio de Janeiro e Braslia, com populao de
11,8 milhes e 3,2 milhes em 2007, respectivamente, tambm esto no primeiro nvel da gesto territorial. Juntamente
com So Paulo, constituem foco para centros localizados em todo o Pas; c. Metrpole Manaus, Belm, Fortaleza,
Recife, Salvador, Belo Horizonte, Curitiba, Goinia e Porto Alegre, com populao variando de 1,6 (Manaus) a 5,1
milhes (Belo Horizonte), constituem o segundo nvel da gesto territorial. Note-se que Manaus e Goinia, embora
estejam no terceiro nvel da gesto territorial, tm porte e projeo nacional que lhes garantem a incluso neste
conjunto.

2. Capital regional so 70 centros que, como as metrpoles, tambm se relacionam com o estrato superior da rede
urbana. Com capacidade de gesto no nvel imediatamente inferior ao das metrpoles, tm rea de influncia de mbito
regional, sendo referidas como destino, para um conjunto de atividades, por grande nmero de municpios. Este nvel
tambm tem trs subdivises: Capital regional A (11 cidades, com medianas de 955 mil habitantes e 487
relacionamentos); Capital regional B (20 cidades, com medianas de 435 mil habitantes e 406 relacionamentos);
Capital regional C (39 cidades com medianas de 250 mil habitantes e 162 relacionamentos).
Please purchase PDF Split-Merge on www.verypdf.com to remove this watermark.
Incluso para a Vida Geografia B

Pr-Vestibular da UFSC

26
3. Centro sub-regional 169 centros com atividades
de gesto menos complexas, dominantemente entre os
nveis 4 e 5 da gesto territorial; tm rea de atuao
mais reduzida, e seus relacionamentos com centros
externos sua prpria rede do-se, em geral, apenas
com as trs metrpoles nacionais. Com presena mais
adensada nas reas de maior ocupao do Nordeste e
do Centro-Sul, e presena mais esparsa nas Regies
Norte e Centro-Oeste, esto subdivididos em grupos: a.
Centro sub-regional A constitudo por 85 cidades,
com medianas de 95 mil habitantes e 112
relacionamentos; e b. Centro sub-regional B
constitudo por 79 cidades, com medianas de 71 mil
habitantes e 71 relacionamentos.
4. Centro de zona 556 cidades de menor porte e com
atuao restrita sua rea imediata; exercem funes
de gesto elementares. Subdivide-se em: a. Centro de
zona A 192 cidades, com medianas de 45 mil
habitantes e 49 relacionamentos. Predominam os nveis
5 e 6 da gesto territorial (94 e 72 cidades,
respectivamente), com nove cidades no quarto nvel e
16 no classicadas como centros de gesto; e b.
Centro de zona B 364 cidades, com medianas de 23
mil habitantes e 16 relacionamentos. A maior parte,
235, no havia sido classicada como centro de gesto
territorial, e outras 107 estavam no ltimo nvel.
5. Centro local as demais 4 473 cidades cuja
centralidade e atuao no extrapolam os limites do seu
municpio, servindo apenas aos seus habitantes, tm
populao dominantemente inferior a 10 mil habitantes
(mediana de 8 133 habitantes). Em relao aos
elementos da anlise, os 802 centros acima do nvel 5
abrangem 548 centros de gesto do territrio e 254
cidades com centralidade identicada a partir dos
questionrios, que foram includas no conjunto nal.
Destas 254 cidades, a maior parte est classicada
como centro de zona, mas trs centros Bragana
(PA), Itapipoca (CE) e Afogados da Ingazeira (PE)
exercem o papel de centro sub-regional em sua regio.

Os problemas sociais das grandes cidades
Um dos problemas mais graves das grandes
aglomeraes urbanas a habitao. Nas ltimas
dcadas, multiplicaram-se os cortios e as favelas
(80% em regies metropolitanas), onde as condies de
salubridade so precrias e o terreno sujeito a
deslizamentos e enchentes. Em muitos casos, a
alternativa foi procurar reas mais afastadas, o que
resultou na ampliao da rea urbanizada e das
distncias no interior das grandes cidades. Soma-se a
esse fato, o desemprego, a deteriorao dos servios
populares de assistncia mdico-hospitalar, falta de
transportes coletivos e de infra-estrutura urbana, como
pavimentao, luz, gua e coleta de esgotos.
Outro problema comum nas grandes cidades
brasileiras a violncia. Os acidentes de trnsito com
milhares de feridos e mortos a cada ano, tm ndices
bem altos no Brasil. Tal nmero se deve ao descaso das
autoridades, a abusos e impunidade dos motoristas e
desrespeito do/ao pedestre e ciclistas. A violncia
policial, especialmente sobre a populao mais pobre,
tambm freqente no Brasil. Ao mesmo tempo,
cresce cada vez mais o nmero de assaltos e
assassinatos, frutos do crescimento do desemprego, da
ao do trfico de drogas e da falta de assistncia s
famlias pobres e vtimas da violncia.
Para amenizar essas questes urbanas, cada
vez mais ouvimos falar em planejamento urbano,
entretanto, se no houver uma participao democrtica
dos moradores, essas iniciativas no sero suficientes.

Problemas ambientais dos grandes centros
urbanos
As grandes e mdias cidades geralmente so
mais poludas que as pequenas ou que o meio rural.
Isso porque nela se concentram as indstrias, veculos e
pessoas, agravando o problemas do lixo, dos resduos e
Please purchase PDF Split-Merge on www.verypdf.com to remove this watermark.
Incluso para a Vida Geografia B

Pr-Vestibular da UFSC

27
das emisses industriais, do congestionamento, do
barulho, etc. Com isso, dentre os principais problemas
urbanos do Brasil temos:
Poluio Atmosfrica A poluio do ar
causada pela presena de partculas slidas em
suspenso e de gases txicos, como o dixido de
carbono, monxido de carbono, dixido de enxofre,
xidos de nitrognio, etc. Os principais agentes
poluidores so as indstrias, os veculos automotores
(automveis, caminhes, nibus), as usinas
termeltricas, a queima de matas e, s vezes, o
aquecimento domstico e a carvo ou lenha. Refinarias
de petrleo, fbricas de cimento e de produtos
qumicos, usinas termeltricas, siderrgicas e
metalrgicas so as que poluem mais intensamente
atmosfera. Como conseqncia da poluio desses
agentes temos o efeito estufa e as chuvas cidas. Os
efeitos da poluio atmosfrica so ainda agravados
com as inverses trmicas.
A poluio do ar manifesta-se praticamente
em todos os grandes centros urbanos do Brasil, contudo
mais grave nas cidades em que a concentrao
industrial ou de veculos maior. Apenas nos anos
1990, comeou-se a estabelecer limites para as
emisses de gases, com programas de catalisadores nos
carros novos e filtros especiais em chamins de
fbricas.
Carncia de reas verdes Estabeleceu-se
internacionalmente que so necessrios no mnimo
16m2 de rea verde por habitante, no entanto no Brasil
isso raro: em So Paulo, por exemplo, existem apenas
4,5 m2 por habitante. Isso intensifica a poluio do ar e
torna mais restritas as opes de lazer da populao.
Os problema do lixos dos esgotos O volume do lixo
produzido por pessoa muito grande nas sociedades
industrializadas, incluindo o Brasil. Um estudo do
IBGE mostrou que, em mdia, cada morador da rea
urbana no Brasil gera 220 Kg de lixo domiciliar por
ano. Temos uma mdia de 500 Kg de lixo anual por
pessoa se somarmos isso a resduos provenientes de
indstrias, escritrios, restaurantes e hospitais (que
deve receber tratamento de coleta especial e
incinerao). H muitas cidades que no tem onde
colocar o lixo que produzem. Alm disso, cerca de 45
milhes de brasileiros no dispem de coleta de lixo.
Dos resduos recolhidos, 13% so levados para aterros
controlados e 10% para aterros sanitrios.
Os esgotos urbanos, geralmente, so
despejados sem tratamentos em rios que cortam a
cidade, poluindo-os intensamente e transformando-os
muitas vezes em rios ftidos e mortos(sem peixes). O
Atlas de Saneamento, do IBGE, mostra que menos de
20% do total de esgoto gerado no Brasil recebe
tratameto. Em 2000 eram recolhidos 52,8% dos esgotos
domiciliares da cidade e 3,1% dos domiclios rurais.





Exerccios de Sala #

1. (UEG) Invadindo espaos
As cidades que antes serviam para abrigar os cidados,
hoje so o ambiente tpico dos automveis.
Nos pases em desenvolvimento, a ao do
poder pblico em favor do automvel foi e tem sido to
eficaz que fica cada vez mais difcil para os moradores
das cidades viver com um mnimo de conforto sem um
automvel particular. S os que, em razo do seu
padro de renda, no podem almejar ter um carro
sujeitam-se ao ineficiente sistema de transporte
pblico. Neles perdem vrias horas do dia, muitos dias
por ano, alguns anos de vida.
Se as condies fossem outras, se o transporte
pblico fosse mais eficiente, menor seria a parcela de
renda que boa parte da populao precisa reservar para
compra e manuteno de um carro particular, menores
seriam as demandas por investimentos pblicos no
sistema virio, maiores seriam as disponibilidades da
renda pessoal para outras atividades, incluindo lazer, e
maiores seriam os recursos que o poder pblico poderia
destinar para melhorar a qualidade de vida de uma
populao.
OKUBARO, Jorge J. O automvel, um condenado? So Paulo:
Senac, 2001. p. 52-53. (Adaptado).

De acordo com a anlise do texto acima, correto
afirmar:
a) O elevado custo, os problemas de congestionamento
das grandes cidades (nibus, automveis,
caminhes) so os maiores responsveis pela
poluio atmosfrica nos centros urbanos,
ocasionando a reduo na qualidade de vida da
populao.
b) A baixa tarifa do transporte urbano um incentivo
ao trabalhador, independentemente do tempo gasto
para o deslocamento entre a casa e o trabalho, o que
resulta em ganho no oramento no final do ms.
c) A qualidade do transporte coletivo urbano, fruto de
estratgias de planejamento, acaba por estimular a
utilizao do transporte coletivo, diminuindo o
nmero de veculos nos grandes centros urbanos.
d) A crescente preocupao com o planejamento
urbano pelos rgos oficiais do governo tem trazido
melhorias na conduo do trfego e a diminuio
dos custos na infraestrutura viria.

2. (UFG) Leia o texto a seguir.
No fundo do vale o lenol fretico aflora para formar
os rios. Estes tm seus ciclos regulados pelos perodos
de cheia e vazante, e pelos espaos representados pelas
plancies de inundao. Este termo encerra em si sua
funo: abrigar as guas do rio quando do seu natural
extravasamento nas pocas de cheias.
LOPES, Luciana Maria. Tragdia ou descaso. Disponvel em:
<www.opopular.com.br/anteriores/03out2009/opiniao>. Acesso em:
3 out. 2009.

Este texto analisa as recorrentes tragdias na regio Sul
do Brasil, com desmoronamentos, desabamentos de
casas, mortes e centenas de pessoas desabrigadas.
Please purchase PDF Split-Merge on www.verypdf.com to remove this watermark.
Incluso para a Vida Geografia B

Pr-Vestibular da UFSC

28
A explicao geogrfica para essas tragdias pode ser
encontrada no seguinte fato:
a) Desvios dos leitos dos rios que direcionam o fluxo
das guas em um mesmo sentido, tornando as
enchentes inevitveis.
b) Ausncia de planejamento do uso do solo causando
especulao imobiliria e possibilitando a ocupao
de novos espaos sem fiscalizao.
c) Encostas ngremes que impedem a absoro de
quantidade volumosa de gua vertida em direo aos
vales.
d) Altas precipitaes pluviomtricas anuais que
dificilmente so previstas devido ao uso de
equipamentos meteorolgicos obsoletos.
e) Presena de solos profundos porosos que retm
gua, provocando desabamentos de construes.

3. (CESGRANRIO)

Considerando o grfico acima e o contexto social,
poltico e econmico e suas repercusses na
organizao do espao brasileiro, a partir de 1950,
analise as afirmaes a seguir.

I As transformaes ocorridas na estrutura urbana
brasileira foram resultado de um rpido crescimento
da industrializao que caracterizou o pas na
segunda metade do sculo XX.
II Os problemas decorrentes da urbanizao tendem a
se agravar de acordo com a previso do grfico, e
se tornam urgentes polticas de planejamento
urbano e investimentos em infraestrutura urbana.
III A tendncia, no caso brasileiro, de que essa
previso no se realize, j que os investimentos e o
financiamento de melhorias na rea rural tm sido
ao comum nos ltimos governos.
IV A estimativa apresentada no considerou o
retorno de grande parte da populao urbana para o
campo, em virtude de problemas decorrentes da
urbanizao, tais como violncia e desemprego.

Esto corretas apenas as afirmaes
a) I e II.
b) I e IV.
c) II e III.
d) II e IV.
e) III e IV.

4. (UERJ) Hoje, a interao espacial entre
comunidades, no que tange ao deslocamento de
pessoas moradoras em uma delas para visitarem
amigos ou parentes ou estabelecerem contatos
associativos com pessoas residentes em outras, tornou-
se um tanto difcil, devido aos mecanismos de controle
impostos pelos traficantes e rivalidade e aos choques
entre quadrilhas baseadas em favelas diferentes (...).
SOUZA, Marcelo Lopes de. O desafio metropolitano: um estudo
sobre a problemtica scio-espacial nas metrpoles brasileiras. Rio
de Janeiro: Bertrand Brasil, 2000.

O fenmeno descrito no texto, que vem ocorrendo nas
ltimas dcadas, corresponde mais diretamente ao
seguinte processo socioespacial:
a) hierarquizao
b) regionalizao
c) metropolizao
d) territorializao

UNIDADE 13 E 14

INDUSTRIALIZAO NO BRASIL

Estrutura industrial
Existem trs tipos principais de indstria: de
transformao, extrativa e da construo. As indstrias
de transformao, o tipo mais comum e caracterstico
da atividade industrial, podem ser divididas nos
seguintes tipos:


O processo de concentrao industrial no Brasil
As primeiras indstrias a surgir no Brasil
foram as de bens de consumo no-durveis
alimentcias e txteis. Essas atividades, com mquinas
movidas a energia eltrica, surgem no final do sculo
XIX.
A economia agroexportadora cafeeira teve
contribuio decisiva na gerao de capital necessrio
para sustentar o processo de industrializao que
surgia, ao mesmo tempo em que o Estado fomentava
esse a vinda de imigrantes para o trabalho livre.
A partir da dcada de 1930, alm das
indstrias de bens no-durveis, instalaram-se
paulatinamente os setores de bens de consumo
durveis, de bens intermedirios e de bens de capital.
O Estado passou a atuar como agente planejador da
economia, formulando polticas especficas para
favorecer a instalao dos diferentes setores industriais
com uma poltica energtica e de financiamento.
A partir de 1950, por meio dos investimentos
das multinacionais, a industrializao brasileira se
expandiu. Com a participao do capital externo, coube
ao Estado o investimento em infra-estrutura de energia,
transporte e comunicaes, e implantao de indstrias
pesadas (siderrgica, metalrgica, petrleo, eletricidade
Please purchase PDF Split-Merge on www.verypdf.com to remove this watermark.
Incluso para a Vida Geografia B

Pr-Vestibular da UFSC

29
e minerao), a modernizao da agricultura e a
formulao de polticas de desenvolvimento regional.
Esse processo promoveu uma concentrao
industrial na Regio Sudeste do Brasil, o qual
prosseguiu at por volta de 1970. A atividade industrial
aproveitou uma srie de condies favorveis criadas
em So Paulo pelo caf: mo-de-obra, mercado
consumidor, eletricidade, transportes e excelente
sistema bancrio. Minas Gerais, Paran e at Santa
Catarina ganharam com essa concentrao industrial,
ampliando as sua infra-estrutura e atividades
econmicas.

Fatores fundamentais para impulsionar o
desenvolvimento industrial do Brasil:
- a ocorrncia da Primeira e Segunda Guerras
Mundiais
- ampliao e reequipamento do parque industrial
aps a Segunda Guerra Mundial;
- instalao da CSN (Companhia Siderrgica
Nacional) de Volta Redonda, em 1946;
- Plano de Metas (governo de Juscelino Kubitscheck)
para o setor de energia e transporte.

Desconcentrao Industrial
Por volta de 1970, comeou a ocorrer uma relativa
desconcentrao industrial no Brasil, com decrscimo
relativo de So Paulo e crescimento maior em outras
unidades da Federao (Rio Grande do Sul, Santa
Catarina, Bahia, Minas Gerais, Gois, Amazonas, Mato
Grosso e outras). importante ressaltar que no foi
tanto uma regresso da atividade industrial em So
Paulo, mas um maior crescimento em outros estados.
Entre os fatores que contriburam para essa
deseconomia de escala podemos citar:
- custos elevados de impostos;
- terrenos demasiadamente caros;
- congestionamentos freqentes no trnsito;
- elevados custos para moradia, transporte e
alimentao (o que implica maiores salrios);
- incentivos de outros estados e municpios para atrair
empresas (lotes baratos, infra-estrutura, iseno de
impostos, etc).
Graas a sua proximidade geogrfica e a densa
rede de transporte e comunicaes, a Regio Sul foi
beneficiada com o processo de desconcentrao
industrial do Sudeste. O setor secundrio foi o que
mais se desenvolveu ao longo das ltimas dcadas.
Atualmente, vrias empresas nacionais e estrangeiras
tem sido atradas para a regio, interessadas no amplo
mercado dos pases do Cone Sul. O eixo Porto Alegre-
Caxias do Sul e o parque industrial de Curitiba so
destaque nesse processo. Em Santa Catarina as
indstrias tradicionais se modernizaram, atraindo
empresas dos setores mecnicos e eltrico por quase
todo o estado.
No Nordeste houve a expanso das indstrias
de bens intermedirios dos setores qumicos (Recife),
petroqumico (Bahia) e de material eltrico, bem como
a modernizao das indstrias de bens de consumo.
A agroindstria o destaque na Regio Centro-Oeste.
No eixo Campo Grande-Goinia-Braslia, h destaque
para as indstrias madeireira, farmacutica, de borracha
e de papel.
Na Regio Norte, a expanso da Zona Franca
de Manaus foi responsvel pelo crescimento da
atividade industrial, com destaque para as indstrias do
setor de eletroeletrnicos, do setor ptico e as
atividades industriais extrativas ligadas a riqueza
mineral do Par.



O BRASIL NA ECONOMIA GLOBAL
A siderurgia brasileira classificada,
internacionalmente, como uma das mais competitivas
do mundo, considerando sua modernidade e quantidade
de produo. O Brasil o 9 Produtor de ao no
ranking mundial, sendo responsvel por 50,2% da
produo da Amrica Latina e 2,9% do total mundial.
as inovaes tecnolgicas, o desenvolvimento de
Produtos, Processos e mtodos gerenciais so fatores
que influenciam na alta competitividade do parque
siderrgico brasileiro.



O crescimento do comrcio externo do
Brasil tem proporcionado sucessivos supervits na
balana comercial do pas desde 2001, depois de seis
anos de dficits (1995/2000). Trs aspectos explicam
essas mudanas:
Please purchase PDF Split-Merge on www.verypdf.com to remove this watermark.
Incluso para a Vida Geografia B

Pr-Vestibular da UFSC

30
- a desvalorizao do real, que favorece as
exportaes e dificulta as importaes, o que barateia
os produtos nacionais no exterior e encarece os
estrangeiros no Brasil;
- a queda do nvel de atividade econmica
brasileira, que gera recuo ainda maior nas importaes;
- a adoo pelo governo de uma poltica mais
agressiva para abrir novos mercados para as
exportaes brasileiras.
A manuteno de uma faixa de cmbio
adequada, ou seja, o valor pelo qual a moeda brasileira
cotada em relao ao dlar, fundamental para bons
resultados da balana comercial.
O Brasil e os principais blocos econmicos mundiais
Nos ltimos anos, tm-se ampliado as relaes
comerciais do Brasil com a Unio Europia (UE) e
com a sia, ao mesmo tempo em que diminui a
participao relativa dos Estados Unidos nos
intercmbios externos do pas.
O Mercosul (Mercado Comum do Sul)
Criado em 1991, com o Tratado de Assuno, a
formao desse bloco econmico visa estabelecer um
livre comrcio entre Argentina, Brasil, Paraguai e
Uruguai. O Mercosul permitiu um salto no intercmbio
entre os quatro pases membros (que passou de 8 para
28 bilhes de dlares). Em 1996 o Chile e a Bolvia
foram admitidos na qualidade de membros associados.
A instabilidade poltica e econmica da Argentina
prejudicou temporariamente o crescimento do bloco. A
integrao total do bloco depender da capacidade de
ajustarem as tarifas alfandegrias s combinadas pelos
atuais pases membros.
A Integrao do Espao Brasileiro no Mercosul


Barreiras exportao
As polticas protecionistas dos pases desenvolvidos
dificultam as exportaes dos pases em
desenvolvimento. Os chamados subsdios podem
aparecer como emprstimos a juros menores, reduo
de impostos, garantia de preos mnimos para cada
safra e prioridades na compra da produo interna. H
ainda a cobrana de impostos mais elevados sobre os
produtos importados e barreiras no-tarifrias, ou seja,
o aumento nas exigncias de padres de produo e
qualidade para os produtos vindos de fora
Exerccios de Sala #
1. (UDESC) Estabelea uma anlise comparativa entre
o contexto do processo da industrializao brasileira da
dcada de 50 e final da dcada de 90, abordando dois
aspectos: o papel do estado e o mercado consumidor.

2. (UEL) A partir dos anos de 1930, o Brasil
intensificou seu processo de industrializao e, assim, a
indstria superou a agropecuria em termos de
participao no PIB. At os anos de 1980, o Estado
atuou de forma decisiva nesse processo.

Com base nos conhecimentos sobre a participao do
Estado no processo industrializao brasileira entre
1930 e 1980, correto afirmar que o Estado brasileiro:

a) Investiu na chamada indstria de base, construiu
infraestrutura nos setores de energia, transporte e
comunicao e foi responsvel pela criao da
legislao trabalhista.
b) Priorizou o transporte ferrovirio, estatizou as
empresas do setor de bens de consumo, adotou
legislao trabalhista mais rgida em relao quela
que vigorou Vargas.
c) Estatizou a indstria de bens de consumo durveis,
privatizou as empresas estatais de gerao e
distribuio de energia eltrica, petrleo e gs
natural e revogou a legislao trabalhista do perodo
Vargas.
d) Incentivou, por meio de privatizaes, investimentos
no setor de infraestrutura de transportes, tais como
estradas e hidrovias, e abriu o mercado interno
importao reduzindo barreiras alfandegrias.
e) Abriu, por meio de parcerias, o mercado interno ao
investimento especulativo estrangeiro nas reas de
securidade social, telecomunicaes e finanas,
facilitando a remessa de recursos financeiros para o
exterior.

3. (FUVEST)


Com base no mapa acima e em seus conhecimentos,
a) identifique o tipo de indstria predominante na
regio Nordeste, considerando sua capacidade
geradora de emprego.
b) caracterize o parque industrial da regio Sudeste.
Please purchase PDF Split-Merge on www.verypdf.com to remove this watermark.
Incluso para a Vida Geografia B

Pr-Vestibular da UFSC

31

Considere, na sua anlise, a presena da indstria de
ponta de alta tecnologia nessa regio e sua capacidade
geradora de emprego.

4. (G1)
Com base na anlise do mapa e em seus conhecimentos
sobre a infraestrutura de transportes e circulao no
Brasil, considere as seguintes proposies:

I Na Regio Norte do Brasil predomina o uso de
hidrovias, fato que pode ser explicado em parte,
pela presena da Bacia do Rio Amazonas que
possui muitos rios navegveis.
II Existe uma maior densidade de meios de transporte
nas Regies Sudeste e Sul, pelo maior dinamismo
econmico dessas reas em relao s demais
regies do Brasil.
III As ferrovias so hoje em dia o principal meio de
circulao de mercadorias e de pessoas no Brasil,
especialmente nas Regies Nordeste e Centro-
oeste, onde esto mais presentes.

Assinale a alternativa correta.
a) Apenas as proposies II e III so verdadeira.
b) Apenas as proposies I e II so verdadeira.
c) Apenas a proposio I verdadeira.
d) Apenas as proposies I e III so verdadeiras.
e) Apenas a proposio III verdadeira.

UNIDADES 15 e 16

AGROPECURIA NO DO BRASIL

Censo Agro 2006: IBGE revela retrato do Brasil
agrrio
O Censo Agropecurio 2006 revelou que a
concentrao na distribuio de terras permaneceu
praticamente inalterada nos ltimos vinte anos,
embora tenha diminudo em 2.360 municpios. Nos
Censos Agropecurios de 1985, 1995 e 2006, os
estabelecimentos com mais de 1.000 hectares
ocupavam 43% da rea total de estabelecimentos
agropecurios no pas, enquanto aqueles com menos
de 10 hectares ocupavam, apenas, 2,7% da rea total.
Focalizando-se o nmero total de estabelecimentos,
cerca de 47% tinham menos de 10 hectares, enquanto
aqueles com mais de 1.000 hectares representavam
em torno de 1% do total, nos censos analisados.
Em 2006, os cerca de 5,2 milhes de
estabelecimentos agropecurios do pas ocupavam
36,75% do territrio nacional e tinham como
Please purchase PDF Split-Merge on www.verypdf.com to remove this watermark.
Incluso para a Vida Geografia B

Pr-Vestibular da UFSC

32
atividade mais comum a criao de bovinos. A rea
total dos estabelecimentos agropecurios brasileiros
diminuiu em 23,7 milhes de hectares (-6,69%), em
relao ao Censo Agropecurio 1995, uma possvel
causa foi a criao de novas Unidades de
Conservao Ambiental (crescimento de 19,09% de
rea) e demarcao de terras indgenas (crescimento
de 128,2%), totalizando mais de 60 milhes de
hectares. Entre 1995 e 2006, os estabelecimentos
agropecurios registraram reduo de suas reas de
florestas (-11%) e de pastagens naturais (-26,6%), e
aumento nas reas de pastagens plantadas de 1,7
milho de hectares (1,8%), sobretudo na regio Norte
(39,7%), enquanto aquelas dedicadas agricultura
cresceram 19,4%, sendo que o maior aumento ocorreu
no Centro-Oeste (63,9%).
A grande maioria dos produtores
entrevistados eram analfabetos ou sabiam ler e
escrever mas no tinham freqentado a escola (39%),
ou no possuam o ensino fundamental completo
(43%), totalizando mais de 80% de produtores rurais
com baixa escolaridade. Trabalhavam em
estabelecimentos agropecurios, 18,9% da populao
ocupada no pas. 77% dos ocupados tinham laos de
parentesco com o produtor e 35,7% no sabiam ler e
escrever Havia mais de 1 milho de crianas com
menos de 14 anos de idade trabalhando na
agropecuria.
Com crescimento de 88% na produo, a
soja foi a cultura que mais se expandiu na ltima
dcada, sendo que em 46,4% desses estabelecimentos
optou-se por sementes transgnicas. Apenas 1,8% dos
estabelecimentos agropecurios praticavam
agricultura orgnica no pas, sendo que 42,5% destes
produtores ligavam-se a associaes, sindicatos ou a
cooperativas. 6,3% dos estabelecimentos declararam
utilizar irrigao, o que representou um aumento de
39% em relao ao Censo anterior. Mais da metade
dos estabelecimentos onde houve utilizao de
agrotxicos no recebeu orientao tcnica (785 mil
ou 56,3%). Alm disso, 15,7% dos produtores rurais
responsveis por estabelecimentos onde houve
aplicao de agrotxicos no sabem ler e escrever, o
que potencializa o risco de intoxicao e uso
inadequado do produto. O rebanho bovino brasileiro
era de 171,6 milhes de cabeas em dezembro de
2006, sendo que Mato Grosso do Sul reunia 20,4
milhes de cabeas, enquanto Par registrou maior
crescimento (119,6%).
Os estabelecimentos que tm como atividade
principal a cana-de-acar ou a soja ficaram com a
maior participao no valor da produo agropecuria
(ambos 14%), seguidos por aqueles que se dedicam
criao de bovinos (10%). Esses so alguns dos
resultados do 10 Censo Agropecurio - 2006, maior
levantamento sobre a estrutura produtiva do setor
primrio brasileiro, que traz um perfil de
aproximadamente 5,2 milhes de estabelecimentos,
em todos os 5.564 municpios brasileiros.
Os resultados do Censo Agropecurio 2006
mostram que a estrutura agrria brasileira,
caracterizada pela concentrao de terras em grandes
propriedades rurais no se alterou nos ltimos vinte
anos. A manuteno da desigualdade na distribuio
de terras expressa-se na comparao das informaes
nos trs ltimos censos agropecurios: Na
comparao entre 1985, 1995 e 2006, as propriedades
com menos de 10 hectares ocupavam, apenas, 2,7%
(7,8 milhes de hectares) da rea total dos
estabelecimentos rurais, enquanto os estabelecimentos
com mais de 1.000 hectares concentravam mais de
43% (146,6 milhes de hectares) da rea total em
ambos os trs censos agropecurios. Focalizando-se o
nmero total de estabelecimentos, cerca de 47%
tinham menos de 10 hectares, enquanto aqueles com
mais de 1.000 hectares representavam em torno de 1%
do total, nos censos analisados.



Os Critrios da Reforma Agrria
Em 1993 o Congresso Nacional oficializou um critrio
de produtividade, no qual, para ser considerada
produtiva, uma propriedade rural deve apresentar, no
mnimo, um ndice de utilizao de 80% da rea
aproveitvel (j descontadas as reas de reserva
vegetal, mananciais, etc.). Desde ento, as propriedades
com aproveitamento abaixo desse ndice podem ser
desapropriadas para realizar-se a reforma agrria.
A classificao de uma propriedade em produtiva ou
no-produtiva, excluindo-a ou tornando-a legalmente
passvel de desapropriao, d origem a discusses, at
mesmo na Justia. A possibilidade de questionar em
Please purchase PDF Split-Merge on www.verypdf.com to remove this watermark.
Incluso para a Vida Geografia B

Pr-Vestibular da UFSC

33
juzo a classificao feita pelo Incra permite que muitos
latifundirios evitem ou retardem a desapropriao de
suas terras. Os conflitos no campo tornam-se cada vez
mais agudos.

A Pecuria no Brasil
O Brasil o primeiro produtor de gado bovino do
mundo. O pas tambm possui o terceiro rebanho suno
do planeta, sendo precedido pela China e EUA. Na
criao de aves tambm o maior produtor, com um
grande desenvolvimento da produo em alguns
estados (SP, MG e toda Regio Sul).
O espao agrrio brasileiro passou por recentes
transformaes, representadas principalmente pela
modernizao das atividades (intensificao das
produes) e pela ampliao das fronteiras agrcolas
(extensificao das produes).
Os problemas ambientais do meio rural
Na rea rural os problemas ambientais esto
principalmente relacionados com as reas que
passam por um processo de modernizao agrria,
mas no s a essas. Com mecanizao e introduo
de tcnicas ditas modernas, como o uso intenso de
adubos qumicos e agrotxicos. Em reas de
agricultura tradicional, as queimadas, o
desmatamento e o assoreamento dos rios so as
questes mais relevantes.
BRASIL, CELEIRO FUTURO DO MUNDO?
Sob o comando do mercado internacional,
agronegcio se expande e reorganiza os espaos
produtivos no territrio brasileiro

A agropecuria est na origem de uma cadeia
produtiva ligada a um conjunto de atividades
industriais e de servios que se denominam
gronegcio. Se a atividade agropecuria responsvel
por aproximadamente 10% do PIB do Brasil, o
agronegcio responsvel por um tero do PIB, 35%
dos empregos e 40% das exportaes.
Nos ltimos anos, o Brasil se tornou um dos
maiores produtores e exportadores de commodities
agropecurias do mundo. Atualmente lder mundial
na exportao de acar, caf, suco de laranja, soja,
tabaco, carne bovina e frango. Est entre os maiores
produtores de lcool, algodo e milho. Embora o
volume de produo seja crescente, os produtos
exportados so, em grande parte, de baixo valor
agregado. Por isso, a participao do Brasil no mercado
mundial de bens agrcolas de apenas 4%
As vantagens comparativas do Brasil,
especialmente nos aspectos naturais, so enormes.
Vastas reas de relevo pouco acidentado, condies
edficas e climticas favorveis, recursos hdricos
abundantes. Segundo a Organizao das Naes
Unidas para a Agricultura e Alimentao (FAO), o
Brasil possui um estoque de 300 milhes de hectares de
terras a serem utilizadas. Gigantes territoriais como
Rssia, Estados Unidos, ndia, China, Austrlia e
Canad possuem extenses bem menores de terras
disponveis
Existe tambm um contexto internacional
favorvel para o Brasil nos prximos anos. H um
cenrio de aquecimento da demanda por alimentos por
conta do crescimento econmico global e da populao
mundial, especialmente da China, grande importador
do Brasil, e da ndia, um mercado promissor.
Mas nem tudo so flores, pois um leque de
obstculos externos e internos limita a expanso das
exportaes. Os primeiros abrangem, antes de tudo, os
vrios tipos de protecionismo (subsdios, cotas,
barreiras tarifrias e no-tarifrias) utilizados
especialmente pelos pases mais desenvolvidos. Mas
incluem at um possvel aumento da produo agrcola
de vrios pases africanos que ainda possuem
expressivos estoques de terras disponveis.
Internamente, os maiores entraves esto
ligados precariedade da infra-estrutura viria
(rodovias, ferrovias, portos). Pelo menos metade da
produo agropecuria transportada por rodovias e
cerca de 75% delas esto em pssimo estado. Apesar de
melhorias parciais, as ferrovias so quase sempre
precrias e os portos, geralmente ineficientes.
Resultado: o preo dos produtos exportados fica menos
competitivo.
Please purchase PDF Split-Merge on www.verypdf.com to remove this watermark.
Incluso para a Vida Geografia B

Pr-Vestibular da UFSC

34
A asfixiante carga tributria e o baixo
financiamento da produo e investimento em
pesquisas so tambm obstculos importantes. Alm
disso, a valorizao do real perante o dlar encareceu
as exportaes. H, ainda, a questo do controle
fitossanitrio que, nos ltimos anos, foi negligenciado,
o que abriu caminho para o ressurgimento de focos de
febre aftosa. Em 2005, dezenas de pases impuseram
embargo carne brasileira. Mais do que as qualidades e
o crescimento da concorrncia internacional, so as
deficincias internas que sabotam o sonho de
transformao do Brasil no celeiro do mundo.
A evoluo espacial da produo agropecuria
no Brasil apresenta trs grandes movimentos
simultneos: o deslocamento da pecuria e das culturas
de gros e algodo para a rea dos cerrados e regio
amaznica; a intensificao de novas tecnologias nas
regies mais desenvolvidas; o crescimento das reas
conquistadas pela irrigao em pores do semi-
rido nordestino.
A expanso nas reas de cerrado s foi
possvel pelas transformaes tecnolgicas que
permitiram a incorporao produtiva dessas reas.
A criao de infra-estruturas e os incentivos fiscais
do governo foram fatores importantes para essa
evoluo. Nesse processo, foi determinante a ao
da Embrapa (Empresa Brasileira de Pesquisa
Agropecuria), trabalhando conjuntamente com
universidades e centros de pesquisas. Hoje, a regio
dos cerrados, que tem nas regies Centro-Oeste e
Nordeste sua maior extenso, est cada vez mais
especializada na produo de gros (especialmente
soja), algodo e pecuria bovina.
O dinamismo nessas duas regies tem
implicaes econmico-sociais e ambientais. A
expanso da produo em padres modernos e em
regies de baixa densidade demogrfica exerce um
forte efeito econmico sobre as atividades urbanas,
impulsionando a demanda de insumos, mquinas,
equipamentos e servios em geral.
Do ponto de vista ambiental, a expanso da
produo acelerou a destruio do ecossistema do
cerrado, que hoje s possui cerca de 20% de sua
cobertura original. Esse processo de conquista de
novos espaos, especialmente em Mato Grosso,
empurrou as atividades agropecurias do cerrado
para reas florestais das franjas meridional e oriental da
Amaznia, gerando uma extensa rea de terras
degradadas o arco do desmatamento que se
estende do oeste do Maranho ao leste do Acre. O
processo contribuiu para que, hoje, mais de 17% da
vegetao florestal original tenha sido destruda.
Ao mesmo tempo, mudanas estruturais
aconteceram na agropecuria no Centro-Sul. Um dos
melhores exemplos So Paulo, que perdeu
participao na produo de gros, mas apresentou
forte crescimento na produo de cana-de-acar,
laranja e fruticultura, indicando uma opo pelos
cultivos de maior valor agregado.
Na Regio Nordeste, dois fenmenos mais ou
menos recentes modificaram o panorama regional: a
expanso de gros nos cerrados da Bahia, Maranho e
Piau e o desenvolvimento de projetos de irrigao em
regies do semi-rido. No caso da agricultura irrigada,
destacam-se os que se verificam no mdio vale do So
Francisco (plo fruticultor de Petrolina-Juazeiro) e nos
vales do Au (RN), Acara (CE) e Parnaba (PI).
Nessas reas, a atividade de maior expresso
a fruticultura com destaque para uva, manga, mamo.
A produo realizada ao longo de todo o ano, por
conta clima quente e seco, o que permitiu no s o
aumento das exportaes como tambm o
abastecimento regular do mercado interno, antes sujeito
s variaes sazonais da oferta. Aos efeitos positivos
sobre a gerao de renda e emprego nessas reas
somam-se as novas possibilidades de integrao
produtiva com a indstria.
FONTE:MAGNOLI , Demtrio. Disponvel em: <
http://www.clubemundo.com.br/noticia_show.asp?id=1042&prod=1
> . Acessado em 05 de maro de 2009.

Exerccios de Sala #
1. (UFPR) Os ndices mnimos de uso da terra
utilizados atualmente pelo INCRA para que ela cumpra
Please purchase PDF Split-Merge on www.verypdf.com to remove this watermark.
Incluso para a Vida Geografia B

Pr-Vestibular da UFSC

35
sua funo social foram elaborados em 1980, com base
nos indicadores de produtividade das lavouras e dos
rebanhos por hectare, levando-se em conta o nvel
tcnico da agropecuria, segundo os dados do censo
agropecurio de 1975 do IBGE. Hoje eles esto
completamente defasados, pois, por exemplo, no estado
de So Paulo, basta produzir 1.900 kg/hectare de milho
para que a propriedade seja considerada produtiva.
Entretanto, a produtividade mdia do milho nesse
estado na safra de 2005/6 foi de 4.150 kg/ha. E por que
at agora esses ndices no foram atualizados? Porque
assim imveis com baixas produtividades escapam da
desapropriao e da reforma agrria.
(Adaptado de: OLIVEIRA, Ariovaldo Umbelino de. "Me engana que
eu gosto": A no atualizao dos ndices de produtividade da terra no
governo Lula. Rdio Agncia Notcias do Planalto, 26 mar. 2007.)

A respeito desse tema, considere as seguintes
afirmativas:
1. A expanso das reas para agropecuria, elevando os
indicadores de produtividade, mostra que os ndices
de uso da terra no precisam ser atualizados, pois
ainda esto de acordo com as necessidades do
campo.
2. O debate sobre a atualizao dos ndices de
produtividade mostra que, na dinmica da reforma
agrria, convergem aspectos legais, tcnico-
produtivos e sobretudo polticos.
3. A mudana dos indicadores da funo social da terra
ajuda a reforma agrria, pois mostra que esta deve
ser implementada onde no foram alcanados altos
ndices de produtividade.
4. A resistncia atualizao dos ndices de
produtividade revela a manuteno do latifndio,
que teve sua origem na forma de repartio da terra
realizada pelos portugueses aps a conquista e se
prolonga at os nossos dias, como uma estrutura
produtiva que condena o campo subutilizao.

Assinale a alternativa correta.
a) Somente a afirmativa 1 verdadeira.
b) Somente as afirmativas 1 e 3 so verdadeiras.
c) Somente as afirmativas 2 e 4 so verdadeiras.
d) Somente as afirmativas 1, 2 e 3 so verdadeiras.
e) Somente as afirmativas 1, 2 e 4 so verdadeiras.

2. (UFU)


Analise os mapas apresentados e assinale a alternativa correta.
a) A incorporao das reas do Sudeste e do Centro-Oeste na produo de trigo foi possvel graas ao processo de
resfriamento por que passa a Terra.
b) A produo da regio Sul do Brasil diminuiu em decorrncia da entrada no mercado brasileiro de trigo importado da
Europa.
c) As condies ideais de produo de trigo so encontradas no Sul do Brasil, com temperaturas elevadas durante todo o
ano e precipitao anual de 700ml.
d) O desenvolvimento de variedades adaptadas ao clima tropical tem permitido o aumento da produtividade nas reas
do Sudeste e Centro-Oeste do Brasil.

3. (UERJ) Herdeiro do pampa pobre
Mas que pampa essa que eu recebo agora
Com a misso de cultivar razes
Se dessa pampa que me fala a histria
No me deixaram nem sequer matizes?

Passam s mos da minha gerao
Heranas feitas de fortunas rotas
Campos desertos que no geram po
Please purchase PDF Split-Merge on www.verypdf.com to remove this watermark.
Incluso para a Vida Geografia B

Pr-Vestibular da UFSC

36
Onde a ganncia anda de rdeas soltas

Herdei um campo onde o patro rei
Tendo poderes sobre o po e as guas
Onde esquecido vive o peo sem leis
De ps descalos cabresteando mgoas

Se for preciso, eu volto a ser caudilho
Por essa pampa que ficou pra trs
Porque eu no quero deixar pro meu filho
A pampa pobre que herdei de meu pai
Gacho da Fronteira e Vaine Duarte
http://letras.terra.com.br

A regio do pampa, no Rio Grande do Sul, reflete a
realidade rural brasileira e suas mazelas.

Identifique o processo socioespacial que originou a
estrutura agrria descrita no texto. Aponte tambm uma
de suas causas e uma de suas consequncias
socioeconmicas.

4. (Enem) Antes, eram apenas as grandes cidades que
se apresentavam como o imprio da tcnica, objeto de
modificaes, suspenses, acrscimos, cada vez mais
sofisticadas e carregadas de artifcio. Esse mundo
artificial inclui, hoje, o mundo rural.
SANTOS, M. A Natureza do Espaco. So Paulo: Hucitec, 1996.

Considerando a transformao mencionada no texto,
uma consequncia socioespacial que caracteriza o atual
mundo rural brasileiro :
a) a reduo do processo de concentrao de terras.
b) o aumento do aproveitamento de solos menos
frteis.
c) a ampliao do isolamento do espao rural.
d) a estagnao da fronteira agrcola do pas.
e) a diminuio do nvel de emprego formal.

5. (Unicamp) Com relao fruticultura na regio do
Vale do So Francisco no Nordeste brasileiro,
correto afirmar que
a) a regio tem terras frteis e adequadas fruticultura
graas insero de projetos irrigveis, o que
compensa o clima seco e o alto ndice de insolao
durante a maior parte do ano.
b) a regio tem clima mido, com chuvas bem
distribudas ao longo do ano, caracterstica favorvel
fruticultura.
c) a regio importante produtora de frutas, mas no
foi possvel implantar a vitinicultura, apesar de
vrias tentativas, porque a cultura no se adapta ao
clima.
d) os maiores produtores de frutas tropicais da regio e
do pas encontram-se em polos agroindustriais dos
municpios pernambucanos de Juazeiro e Petrolina.


O processo de industrializao envolve a produo e o
consumo de produtos energticos, uma vez que a
indstria a atividade econmica que mais consome
energia. Por isso, est muito subordinada a ela,
particularmente localizao dos recursos energticos,
viabilidade de utilizao econmica de suas fontes e,
sobretudo, contabilizao dos custos aspectos
essenciais do projeto de industrializao do pas.
A energia pode ser definida como capacidade de
produzir trabalho, incluindo-se a a energia muscular
do ser humano at a energia nuclear. A matriz
energtica utilizada no Brasil foi sendo ampliada e
diversificada com o decorrer das atividades industrias,
como demonstra a figura abaixo.

As fontes de energia so divididas em duas grandes
categorias:

Fontes no-renovveis: aquelas cujas reservas podem
se esgotar, como o caso do petrleo, do carvo, do
gs natural, do urnio, etc.

Fontes renovveis: que podem produzir energia sem
se esgotarem, como os rios, mars, vento, sol,
biomassa, etc.

As principais fontes exploradas at hoje so as no-
renovveis. No Brasil, essas fontes de energia esto
espacializadas da seguinte forma:

- Gs Natural
Representando 3% da matriz energtica brasileira, os
grandes produtores so Rio de Janeiro e Bahia, seguido
de longe por Sergipe, Rio Grande do Norte e
Amazonas. O Brasil tambm importa gs natural da
Bolvia atravs de um extenso gasoduto, que vai de
Santa Cruz de La Sierra, na Bolvia, a Guararema, em
So Paulo, e da a regio industrial de Porto Alegre no
Rio Grande do Sul. Nesse estado outro gasoduto
permite a importao do gs natural da Bolvia.
FONTES DE ENERGIA
Please purchase PDF Split-Merge on www.verypdf.com to remove this watermark.
Incluso para a Vida Geografia B

Pr-Vestibular da UFSC

37


- Petrleo

At a dcada de 1970, a limitada produo nacional era
obtida de poos terrestres, a maior parte localizada no
Recncavo Baiano. Hoje, a principal fonte so os
poos submarinos da plataforma continental,
principalmente na regio vizinha ao estado do Rio de
Janeiro (Bacia de Campos). Do total produzido no pas,
mais de 60% so originrios da plataforma continental.
Em seguida ao Rio de Janeiro, esto os estados do Rio
Grande do Norte e da Bahia, respectivamente. Sergipe
e Cear tambm so outros destaques na produo
nacional. As refinarias esto prximas aos centros de
consumo, sendo que a Petrobrs detm 98% dessa
atividade em vrias regies brasileiras.
- Carvo Mineral

A preocupao com o aproveitamento do carvo
mineral recente no Brasil. As reservas esto
depositadas em terrenos sedimentares antigos, na borda
oriental da bacia do Paran, no baixo Amazonas e na
bacia do Parnaba, mas apenas o carvo do sul
explorado. Apenas parte do carvo catarinense
coqueificada e remetida para a Companhia Siderrgica
Nacional, em Volta Redonda (RJ). No Rio Grande do
Sul, cerca de 50% da produo aproveitada para a
fabricao de ao. O carvo muito utilizado na gerao
termeltrica de energia. Na regio sul ela representa
15% do total da regio.

Hidreletricidade
A fonte de energia renovveL mais utilizada no Brasil
a energia de origem hidrulica. Ela abundante no
Brasil, pois temos a rede hidrogrfica mais densa do
mundo, com enorme potencial hidreltrico. Isso explica
porque mais de 90% da potncia instalada nas usinas
provm de geradores hidrulicos, cabendo aos
geradores trmicos apenas 8%.

As regies Sul e Sudeste apresentam grandes
empreendimentos hidreltricos, como por exemplo: o
complexo Urubupung (SP e MS) com as usinas de
Jupi e Ilha Solteira; a usina de Trs Marias (rio So
Francisco; e a usina de Itaipu (maior hidreltrica do
planeta, localizada no rio Paran). No norte destaca-se
a usina de Tucuru, no rio Tocantins, construda para
alimentar a produo de alumnio realizada na regio
de Belm e que fornece energia para outras regies do
Brasil. Na Regio Nordeste destacam-se as usinasdo
rio So Francisco, como Sobradinho, Moxot e Paulo
Afonso, construdas como parte do projeto de
desenvolvimento da regio nas dcadas de 1960 e
1970. O Brasil s aproveita 30% de seu potencial
hidreltrico, sendo um poderoso argumento contra o
uso da energia nuclear
FONTE: SENE, E. GEOGRAFIA GERAL E DO BRASIL.
So Paulo: Editora Scipione, 2003
Os dez maiores consumidores de petrleo
Please purchase PDF Split-Merge on www.verypdf.com to remove this watermark.
Incluso para a Vida Geografia B

Pr-Vestibular da UFSC

38


































OUTRAS FONTES DE ENERGIA

Energia Nuclear
No final da dcada de 1960, o governo brasileiro
comeou a definir o Programa Nuclear Brasileiro,
destinado a implantar no pas a produo de energia
atmica. A usina nuclear decorrente desse programa foi
Angra I, localizada em Angra dos Reis (RJ). Essa usina
entrou em operao em 1984. Pelos riscos que
representa, esse projeto tem sido alvos de crticas, tanto
por parte de ecologistas, de cientistas e da sociedade
civil.

lcool
O lcool pode ser produzido de numerosos vegetais,
como a cana-de-aucar, batata e cevada. Diante da
perspectiva de esgotamento das reservas de petrleo e
de carvo, tem sido valorizado como combustvel
alternativo. Nos anos de 1970, com o aumento dos
preos do petrleo no mercado internacional, foi criado
o Programa Nacional do lcool (Prolcool), que
recebeu incentivos governamentais e desenvolveu
tecnologia prpria, apresentando grande produo em
menos de dez anos. A inviabilidade est no processo





































produtivo, pois exige grandes extenses de terra. Sua
utilizao restringe-se ao setor de transportes.

Xisto Betuminoso
O aproveitamento econmico do xisto betuminoso
consiste em separar o betume da rocha
(hidrocarbonetos) e produzir petrleo a partir dele. As
maiores concentraes esto no municpio de So
Mateus do Sul (PR), onde foi instalada uma usina para
process-lo. Em mdia, o teor de leo na rocha
inferior a 10%. Dificuldades tcnicas e ambientais
restringem a utilizao dessa fonte energtica.

Energia Solar
No Brasil essa fonte de energia muito utilizada para o
aquecimento da gua em habitaes. A tecnologia
existente rudimentar para a gerao de energia,
aproveitando muito pouco da intensa radiao solar que
o territrio brasileiro recebe.

Biogs
Trata-se do gs produzido a partir de matria orgnica
em decomposio (esterco, palha, bagao vegetal ou
lixo) pela ao de certas bactrias. O biodigestor o
aparelho utilizado para o aproveitar o biogs.


Please purchase PDF Split-Merge on www.verypdf.com to remove this watermark.
Incluso para a Vida Geografia B

Pr-Vestibular da UFSC

39
Exerccios de Sala #

1. (UDESC) Sobre as reservas de petrleo da camada
pr-sal, assinale a alternativa incorreta.
a) O campo de Jubarte ser o primeiro, no Brasil, a
produzir leo abaixo da camada de sal. Apesar de
ser na Bacia de Santos que esto sendo depositadas
as expectativas de maior volume de reservas na
camada do pr-sal, no litoral capixaba, ainda na
Bacia de Campos, que o primeiro leo ser retirado
desta formao geolgica.
b) A grande vantagem do Brasil na explorao de
petrleo da camada do pr-sal que a Petrobrs
100% estatal e nica dona do petrleo existente na
referida reserva.
c) O presidente Luiz Incio Lula da Silva quer que o
modelo de explorao da camada pr-sal transforme
o Brasil em um grande exportador de derivados, que
valem mais que exportar o petrleo cru.
d) A Petrobrs realizou, tambm, uma avaliao
regional do potencial petrolfero do pr-sal que se
estende nas bacias do Sul e Sudeste brasileiros. Os
volumes recuperveis estimados de leo e gs para
os reservatrios do pr-sal, se confirmados, elevaro
significativamente a quantidade de leo existente em
bacias brasileiras, colocando o Brasil entre os pases
com grandes reservas de petrleo e gs do mundo.
e) A Petrobrs e scias identificaram indcios de
petrleo em diferentes pontos na camada pr-sal,
que se estende por 800 km desde o litoral do
Esprito Santo ao de Santa Catarina.

2. (UDESC) Aponte as vantagens e as desvantagens
para o Brasil de um eventual aumento da produo de
energia nuclear.

3. (UEL) Analise o grfico a seguir e responda
questo.

Uma discusso frequente na mdia atual diz respeito s
alternativas de gerao de energia para o abastecimento
da populao e dos processos produtivos.
Com base no grfico e nos conhecimentos sobre o
tema, considere as afirmativas a seguir.
I. As fontes no-renovveis representam,
aproximadamente, 54% da matriz energtica brasileira.
II. Gs natural e carvo mineral so, assim como o
petrleo, fontes renovveis, com forte participao na
matriz energtica nacional.
III. Em comparao s termoeltricas e usinas
nucleares, as hidreltricas so menos comprometedoras
para o meio ambiente, entretanto a construo de
barragens provoca graves impactos socioambientais.
IV. As condies brasileiras de clima e relevo originam
um grande potencial hidrulico que, no entanto, ainda
pouco aproveitado.

Assinale a alternativa correta.
a) Somente as afirmativas I e II so corretas.
b) Somente as afirmativas II e IV so corretas.
c) Somente as afirmativas III e IV so corretas.
d) Somente as afirmativas I, II e III so corretas.
e) Somente as afirmativas II, III e IV so corretas.

4. (UERJ)

H 34 anos, os governos do Brasil e da Alemanha
firmavam programa de cooperao que previa a
construo de oito centrais termonucleares, alm de
usinas de enriquecimento de urnio e de
reprocessamento do combustvel nuclear.
Alm das irregularidades apontadas na reportagem, o
atual programa nuclear brasileiro tem como principal
problema:
a) risco de poluio ambiental
b) inviabilidade da tecnologia adotada
c) ausncia de fontes de investimentos
d) indisponibilidade de mo de obra qualificada

Please purchase PDF Split-Merge on www.verypdf.com to remove this watermark.
Incluso para a Vida Gramtica


PR-VESTIBULAR DA UFSC

UNIDADE 1


MORFOLOGIA I (Classes Gramaticais)


1) SUBSTANTIVO
Formao: primitivo ou derivado; simples ou
composto
Classificao: comum ou prprio; concreto ou
abstrato
Substantivo coletivo
Voc lembra o coletivo de...
Abelhas?
Estrelas?
Ilhas?
Artistas?
Ladres?
Avies?
Pintos?
Palavras?
Animais de uma regio?

Flexo dos substantivos
a) Biformes
b) Uniformes (epicenos, comuns de dois gneros,
sobrecomuns)
Coloque o artigo para determinar o gnero dos
seguintes substantivos. Isso coisinha de primrio!!!
Alvar Apendicite
Alface Cal
Cl Comicho
Champanha Dinamite
Libido D (sentimento)
Omelete D (nota musical)
Omoplata Sentinela

Passe os substantivos abaixo para o plural.
ATENO PRONNCIA!!!
Caroo - Degrau -
Poo - Escrivo -
Cidado - Capito -
Forno - Sacristo -
Socorro - O trax -
Acar - Papel -
Giz - lcool -
Carter - Abdmen -
Mel - Hfen -
Projtil - Barzinho -
Trofu - Lugarzinho -

Agora, passe para o plural esses substantivos
compostos:
Aguardente - Samba-enredo -
O bl-bl-bl - Abaixo-assinado -
Po-de-l - Amor-perfeito-
O leva-e-traz - Beija-flor-
O corre-corre - Bem-amado-
Quinta-feira - Guarda-roupa-
Banana-ma - Guarda-civil-
Peixe-espada - Guarda-sol -


2) ARTIGO
Qual a forma correta?
a) O diretor solicitou a presena de ambos alunos.
b) O diretor solicitou a presena de ambos os alunos.

a) Este o homem cujo amigo desapareceu.
b) Este o homem cujo o amigo desapareceu.

3) ADJETIVO
Locuo adjetiva: preposio + substantivo (ou
advrbio), com valor de adjetivo.
Ex.: dia de chuva (chuvoso);
atitudes de anjo (angelical)

Agora com voc. Mais uma coisinha de primrio!!!
D os adjetivos para as seguintes locues:
De abdmen - De cabelo -
De abelha - De chuva -
De aluno - Do rio -
De professor - De dedo -
De asno - De galinha -
De paixo - De guerra -
De bispo - De ilha -
De lago - De velho -
De leite - De rim -
De olho - De sonho -

4) NUMERAL
- Cardinais
- Ordinais
- Multiplicativos
- Fracionrios

5) PRONOME: pessoais, de tratamento, possessivos,
demonstrativos, relativos, indefinidos, interrogativos

6) ADVRBIO: afirmao, negao, dvida, tempo,
lugar, modo, intensidade

7) PREPOSIO
Chegou de nibus. (meio)
Chegou de Pernambuco. (origem)
Chegou com ele. (companhia)

Principais preposies:





8) CONJUNO
- Coordenativas
a) Aditiva (adio): e, nem, mas tambm, mas ainda,
bem como, como tambm, tampouco.
b) Adversativa (oposio): mas, porm, todavia, no
entanto, entretanto, contudo.
a, ante, at, aps, com, contra, de, desde, em, entre,
para, perante, por, sem, sob, sobre, trs

Please purchase PDF Split-Merge on www.verypdf.com to remove this watermark.
Incluso para a Vida Gramtica

Pr-Vestibular da UFSC

2
c) Alternativa (alternncia): ou... ou, ora... ora,
quer... quer, seja... seja.
d) Conclusiva (concluso): por isso, logo, portanto,
pois (Desde que no esteja no incio da orao!),
ento, assim
e) Explicativa (explicao): pois (Sempre no incio
da orao!), porque, que, porquanto
- Subordinativas
a) Causal:. Porque, visto que, j que, uma vez que,
como (equivalendo a porque)
b) Comparativa: Como, que (precedido de mais ou
de menos)
c) Consecutiva: d idia de conseqncia. Que
(precedido de um termo intensivo: to, tal, tanto)
d) Concessiva: Embora, se bem que, ainda que, por
mais que, por menos que, conquanto
e) Condicional: Se, caso, contanto, que, desde que
f) Conformativa: Como, conforme, consoante,
segundo
g) Final: A fim de que, para que, que
h) Proporcional: Que, medida que, proporo
que, quanto mais, quanto menos
i) Temporal: Quando, enquanto, logo que, desde que,
assim que
j) Integrante: que, se

9) INTERJEIO
Indique interjeies que exprimam:
a) alegria
b) advertncia
c) alvio
d) animao
e) deseja
f) dor
g) espanto
h) impacincia

UNIDADE 2

MORFOLOGIA II (Formao de Palavras)

Estrutura do vocbulo
a) Radical
b) Vogal temtica
c) Tema
d) Desinncia
e) Vogal e consoante de ligao
f) Afixos
- Prefixos: antes do radical.
Ex.: adjunto, bisneto, preconceito
- Sufixos: depois do radical.
Ex.: plumagem, rouparia, pedreiro

PROCESSOS DE FORMAO
DE PALAVRAS
1. DERIVAO (= um radical)
1.1 - Prefixao leal o desleal
1.2 - Sufixao leal o lealdade
1.3 - Prefixao e Sufixao leal o deslealdade
1.4 - Parassntese - Colocao simultnea de prefixo e
sufixo de tal modo que a palavra nova s exista com
os dois ao mesmo tempo noite o anoitecer
1.5 - Regressiva- A palavra nova menor que a
original, seja em relao ao nmero de letras, ao de
fonemas ou de ambos.
Ex.: combater o combate; voar o vo
1.6 - Imprpria - A palavra muda de classe
gramatical sem mudar de forma.
Ex.: O cantar do pssaro belo.

Substantivo

2. COMPOSIO (= mais de um radical)
2.1 - Justaposio - sem perda ou alterao de som.
Ex.: sempre + viva o sempre-viva
gira + sol o girassol
2.2 - Aglutinao - com perda ou transformao de
som.
Ex.: gua + ardente o aguardente
plano + alto o planalto

3. OUTROS PROCESSOS
3.1 - Hibridismo - a formao de uma palavra com
radicais de origem diferente.
Ex.: automvel o grego + latim
sociologia o latim + grego
Florianpolis o portugus + grego
3.2 - Onomatopeia - a reproduo de sons da
natureza.
Ex.: tique-taque; trim; paft; bl-bl-bl; cacarejar
3.3 - Abreviao - reduo de uma palavra at o
limite de sua compreenso.
Ex.: moto (de motocicleta), cine (de cinema)
3.4 - Siglonimizao - (Sigla)
VASP, AIDS, Varig, Celesc

Exerccios de Sala #

1. Faa como no modelo:
a) estudo do homem: antropologia
b) estudo da vida: ____________________________
c) estudo da origem: __________________________
d) estudo do som: ____________________________
e) estudo da forma: ___________________________
f) estudo da doena: __________________________
g) estudo do animal: ___________________________

2. Associe a coluna da esquerda com a da direita.
Nome tcnico
1. ( ) brontofobia
2. ( ) acrofobia
3. ( ) claustrofobia
4. ( ) ergofobia
5. ( ) nictofobia
6. ( ) oclofobia
7. ( ) pantofobia
8. ( ) fotofobia
9. ( ) agorafobia
10. ( ) tricofobia
Medo ou averso a
a) espaos fechados
b) luz
c) cabelo
d) trovo
e) altura
f) espaos abertos
g) multides
h) tudo em geral
i) noite
j) trabalho; esforo

Please purchase PDF Split-Merge on www.verypdf.com to remove this watermark.
Incluso para a Vida Gramtica

Pr-Vestibular da UFSC

3
3. Assinale a alternativa correta.
a) A palavra rebater composta, pois tem dois
radicais: <re> e <bater>.
b) Campons e portugus so palavras que
possuem afixos.
c) Boiadeiro e seringueiro pertencem mesma
famlia etimolgica, isto , so cognatos.
d) Na frase Aquele jogadorzinho fez um gol,
finalmente!, o sufixo -inho, em jogadorzinho, expressa
carinho, como em filhinho.
e) O termo sublinhado em No sei o porqu desta
confuso. Constitui uma derivao imprpria.

Tarefa Mnima #

1. Assinale a alternativa correta quanto aos processos
de formao de palavras.
a) Em empobrecer temos o processo de derivao
parassinttica. J em passatempo, ocorreu a
composio por aglutinao.
b) Reagir, universal e desrespeito so palavras
formadas por derivao prefixal.
c) Aguardente, embora, outrora e pernalta so
palavras formadas por justaposio.
d) Um exemplo de derivao imprpria a palavra
cine.
e) As palavras pedreiro, socialista, acidez e frontal
so todas formadas por derivao sufixal.

2. Em qual dos exemplos abaixo est presente um
caso de derivao parassinttica?
a) L vem ele, vitorioso do combate.
b) Ora, v plantar batatas!
c) Comeou o ataque.
d) Assustado, comeou a se distanciar do animal.
e) No vou mais me entristecer, vou cantar.

3. Assinale a(s) alternativa(s) em que todas as
palavras so formadas pelo mesmo processo:
01. Aeromoa, couve-flor, pernalta.
02. Furta-cor, verde-claro, vaivm
04. Boquiaberto, fidalgo, aguardente.
08. Girassol, guarda-civil, pontap.
16. Combate, ataque, salto.
32. Envelhecer, aterrissar, retroagir

Tarefa Complementar #

1. Assinale a(s) alternativa(s) correta(s) e d o valor
total.
01. As palavras leite, leitar, lactente, leito,
aleitamento e eleitorado pertencem mesma
famlia etimolgica, isto , so formadas a partir de
uma nica raiz.
02. Em redondeza, h prefixo e sufixo.
04. Na palavra incoerncia, o prefixo in- significa
privao, negao.
08. Em Quem no entende este esdrxulo dialeto
chamado economs incapaz de deter-se s
pginas de economia dos jornais, as palavras em
negrito so primitivas.

2. Se, a partir da palavra tarde, formamos tardar e
entardecer, essas duas ltimas sero respectivamente:
a) derivadas por sufixao e por prefixao.
b) derivadas por prefixao e parassintetismo.
c) derivadas por prefixao e por prefixao.
d) derivadas por sufixao e parassintetismo.
e) composta por aglutinao e por justaposio.

3. Na frase Ela tem um qu de mistrio, o processo de
formao da palavra destacada chama-se:
a) composio.
b) aglutinao.
c) justaposio.
d) derivao imprpria.
e) parassntese.

4. As palavras expatriar, amoral, aguardente so
formadas, respectivamente, por:
a) derivao parassinttica, derivao prefixal,
composio por aglutinao.
b) derivao sufixal, derivao prefixal, composio
por aglutinao.
c) derivao prefixal, derivao prefixal, composio
por justaposio.
d) derivao parassinttica, derivao sufixal,
composio por aglutinao.
e) derivao prefixal, derivao prefixal, composio
por justaposio.

UNIDADE 3

FONOLOGIA













Fique atento!
a) Vogais
b) Semi-vogais
c) Consoantes

Encontros voclicos
Hiato V + V te-a-tro
Ditongo SV + V (crescente) qua-se
V + SV
(decrescente)
bai-xo
Tritongo SV + V + SV U-ru-guai
Fonema = som produzido pelas letras
Dfono = uma letra a qual representa dois fonemas.
Dgrafo = duas letras que produzem apenas um
fonema.

Sero sempre Dgrafos: SS RR LH NH CH
IM IN OM ON UM UN

Por vezes, sero Dgrafos:XC SC S QU GU
AM AN EM EN
Please purchase PDF Split-Merge on www.verypdf.com to remove this watermark.
Incluso para a Vida Gramtica

Pr-Vestibular da UFSC

4

Ateno!!!
gOIAba, quEIjo, churrasquEIra, carguEIra
no so tritongos.
Separao silbica

Prai-a joi-a mei-a fei-o sei-o sai-a sa-i-a

Mai-o mai- mai-or ru-im sub-li-nhar

Exerccios de Sala #

1. Assinale a(s) alternativa(s) correta(s):
01. A palavra trem possui 4 letras e 4 fonemas.
02. A palavra emprego possui 7 letras e 7 fonemas.
04. As palavras andam e tambm possuem ditongo.
08. Nas palavras goiaba, queijo e Paraguai temos a
presena de tritongo.
16. Em guerrinha temos trs dgrafos.
32. A palavra tadinho possui mais letras que fonemas,
pois "nh" dgrafo.

2. Assinale a(s) alternativa(s) em que, na lngua culta,
haja, em todas as palavras, o mesmo nmero de
fonemas.
01. Nexo, festa, culto.
02. Miopia, classes, caminho.
04. Psiu, desa, quero.
08. Lhe, sim, quem.
16. Carruagem, cavalheiro, selvagem

3. Sobre o vocbulo chuvinha, afirmamos:
1. Possui oito letras
2. Possui seis fonemas
3. Possui dois dgrafos
4. paroxtono.
a) Est correta apenas a primeira afirmativa.
b) Esto corretas s as duas primeiras afirmativas.
c) Nenhuma das afirmativas est correta.
d) As quatro afirmativas esto corretas.
e) Esto corretas s as duas primeiras afirmativas.

Tarefa Mnima #

1. (UFSC) A seguir, h palavras distribudas em
quatro colunas (A, B, C, D). Assinale a(s)
proposio(es) verdadeira(s) que tem(tm),
respectivamente, palavras com: dgrafo (sendo, ao
mesmo tempo, palavra derivada), hiato, oxtona e
paroxtona.


Coluna A Coluna B Coluna C Coluna D
01. Assassinato Aliviar Ter Correria
02. Senhora Agradvel Direo Lpis
04. Felizmente Dilogo Caf Acar
08. Chegou Pincis Est Tenso
16. Olhando Ateli Ingls Infantil
32. Passos Infncia J Rpido

2. Assinale a(s) alternativa(s) em que, na lngua culta,
h em todas as palavras o mesmo nmero de fonemas:
01. chave, quero, hbil, aguar.
02. Carro, quilo, gua, desa.
04. canhotos, txicos, extrair, prosseguir.
08. horrvel, velhaco, crescer, excessos.
16. classes, anexa, horrores, esquina.

3. Assinale a(s) proposio(es) verdadeira(s),
referente(s) tira que segue:

01. No primeiro quadrinho, nascido e trevas
apresentam encontros consonantais, sendo o
primeiro imperfeito e o segundo perfeito.
02. Em todos os quadrinhos da charge, existem sinais
de pontuao ao trmino das falas, sendo que nos
trs primeiros a pontuao indicativa de
indagao.
04. No ltimo quadrinho tem-se uma incorreo
gramatical, uma vez que a escrita correta seria h
mil anos .
08. As palavras falam e anos apresentam,
respectivamente, desinncia nmero-pessoal e
desinncia de nmero.
16. A afirmao As pessoas no esto seguras nem
nas suas prprias casas! significa que Os seres
humanos tm falta de segurana at nos prprios
lares.
32. Em As pessoas s falam de guerra, assassinato,
crime..., h dois monosslabos tonos e um
tnico, um ditongo nasal decrescente e um hiato.

Tarefa Complementar #

1. Leia o perodo abaixo e as afirmaes relacionadas
s expresses nele contidas:
O ceticismo constitui uma marca caracterstica do
conto machadiano, a qual vem sendo, amide,
assinalada pelos estudiosos da literatura brasileira,
notavelmente aqueles que se concentram na
chamada fase realista de sua obra.
I. A separao silbica das palavras machadiano e
assinalada , respectivamente, ma-cha-di-a-no e as-
si-na-la-da.
II. De acordo com as regras de acentuao grfica, o
verbo constituir escreve-se constitua em uma das
formas do passado.
III. Sem contrao de preposio com artigo, a
expresso pelos estudiosos deveria grafar-se pr
estudiosos.
IV. O advrbio derivado de notvel deveria estar
grafado no texto como notavelmente.
So corretas as afirmaes:
Please purchase PDF Split-Merge on www.verypdf.com to remove this watermark.
Incluso para a Vida Gramtica

Pr-Vestibular da UFSC

5
a) I, II e IV.
b) II e III.
c) I e II.
d) III e IV.
e) I e III.

2. Assinale a alternativa em que todas as palavras
estejam corretamente grafadas.
a) Empolgao, atravs, extrangeiro, despercebido,
excesso.
b) Eletricista, asterstico, celebral, frustado,
beneficiente.
c) Assessores, pretenso, losango, asterisco, alto-
falante.
d) Bandeija, vultosa, previlgio, entitular, prazeiroso.
e) Eletrecista, preteno, asceno, carangueijo,
prazeiroso.

3. Assinale a alternativa correta.
a) H encontros consonantais em varrido, milhes,
dinossauros.
b) H somente ditongos decrescentes nasais em
radiao, exploses, revelou, dinossauros.
c) O vocbulo h obedece mesma regra de
acentuao que atrs.
d) Em proveniente no encontramos dgrafo.
e) Em ideia, temos um ditongo seguido de um
hiato.

UNIDADE 4
ACENTUAO GRFICA
Regras Gerais de Acentuao Grfica
Acentuamos...
1. Monosslabos tnicos terminados em O(S), A(S) ou
E(S).
Ex.: p, l, d
2. Oxtonas que terminam em O(S), A(S), E(S), EM
ou ENS.
Ex.: sof, mant-lo, refns
Ateno com os derivados dos verbos TER e VIR!
Compare: ele mantm, eles mantm; ele intervm; eles
intervm
3. Paroxtonas terminadas em L, N, R, X, ONS, PS,
(S), O(S), I(S), US, UM, UNS ou em ditongo.
Ex.: bnus, eltrons, hfen (Mas: hfens), m, rgo,
jquei
4. todas as Proparoxtonas.
Ex.: rpido, faclimo

Casos Especiais de Acentuao Grfica
Acentuamos...
1. Os ditongos abertos I(S), U(S) e I(S), desde
que no formem paroxtona.
Ex.: heri, heroico, ideia, eu apoio, o apoio, cu,
papis
2. Pela Nova Ortografia, no acentuamos mais a
primeira vogal dos hiatos OO e EE.
Ex.: voo, eles veem
3. As letras I e U quando:
- forem tnicas;
- formarem hiato com a vogal anterior;
- estiverem sozinhas na slaba ou, no mximo,
acompanhadas de s;
- no forem seguidas de nh.
Ex.: ba, fasca, rainha, Lus, juzes, razes (Mas: juiz,
raiz, Luiz)
Fique atento: As palavras maiscula e veculo tm
sua acentuao grfica justificada por duas regras
simultaneamente!
4. Tambm pelas novas regras, mantm-se como
acentos diferenciais apenas os casos:
por z pr
pode z pde
forma z frma (acento facultativo)
ele tem z eles tm
ele vem z eles vm
5. Acentuamos Qu e Gu acompanhados de e ou i,
quando o u for tnico e pronunciado.
Ex.: argem, obliqe, obliqes, averige, averiges

Cuidado com as silabadas!!!





Aproveitando...






Exerccios de Sala #

1. Assinale a alternativa cujas palavras so acentuadas
graficamente pela mesma razo:
a) h, at, atrs
b) histria, geis, voc
c) est, at, ningum
d) ordinrio, prprio, defend-lo
e) mgoa, cone, jquei

2. Assinale a(s) alternativa(s) cujas palavras tm a
slaba tnica destacada corretamente. Observe que os
acentos grficos destas palavras, quando existentes,
foram excludos para efeito da questo.
01. Nobel, ruim, refem, erudito
02. Latex, pudico, rubrica, acrobata
04. pudico, nobel, rubrica, avaro
08. ruim, avaro, pudico, latex
16. sutil, avaro, mister, refem

3. Assinale a(s) proposio(es) correta(s).
01. Os acentos grficos em corrupio, l e baldeao
so justificados pela mesma regra.
02. So classificadas como oxtonas: saguo, poder e
conduzi-lo.
PORQUE
PORQU
POR QUE
PORQU
GRATUITO AVARO XROX XEROX
NTERIM ACROBATA RUBRICA
FILANTROPO MISANTROPO
Please purchase PDF Split-Merge on www.verypdf.com to remove this watermark.
Incluso para a Vida Gramtica

Pr-Vestibular da UFSC

6
04. A forma verbal destacada em Ainda tm
esperana acentuada por corresponder a um
monosslabo tnico, assim como ocorre na orao
Os relatrios contm alguns erros.
08. Os acentos grficos dos vocbulos voc, proteg-
los e contm seguem a mesma regra de acentuao.
16. Em idade, ainda e fluido temos o mesmo nmero
de slabas.
32. Os vocbulos gratuito, debaixo e implicou so
trisslabos.

Tarefa Mnima #

1. Assinale a alternativa em que todas as palavras
devem ser acentuadas:
a) pudico, pegada, rubrica
b) gratuito, avaro, pezinho
c) abdomen, itens, harem
d) magoo, perdoe, ecoa
e) contribuia, atribuimos, caiste

2. Considere o texto a seguir e indica a(s)
afirmao/afirmaes verdadeira(s):

EIROS
A leitora Elza Marques Marins me escreve uma
carta divertida estranhando que brasileiro seja o
nico adjetivo ptrio conhecido em eiro que,
segundo ela, um sufixo pouco nobre. Existem
suecos, ingleses e brasileiros, como existem mdicos,
terapeutas e curandeiros. (...)
a diferena entre jornalista e jornaleiro ou
entre msico ou musicista e roqueiro, timbaleiro ou
seresteiro. H o importador e h o muambeiro. Se
voc comeou como padeiro, aougueiro ou carvoeiro
escreve Elza as chances so mnimas de acabar
como advogado, empresrio, grande investidor ou
latifundirio, a no ser que se d o trabalho de ser
poltico antes. Alis, h polticos e politiqueiros. (...)
VERSSIMO, Lus Fernando. Jornal do Brasil, 7/10/95.

01. O morfema -eiro usado exclusivamente para
formar adjetivos a partir de substantivos.
02. O substantivo diferena apresenta um dgrafo
nasalizado.
04. Em chances encontramos um dgrafo
consonantal e um voclico.
08. Os termos Alis e h tm sua acentuao
grfica justificada pela regra das paroxtonas.
16. O paroxtono ptrio pode ser considerado,
eventualmente, como um proparoxtono.

3. A gente se acostuma a acordar de manh
sobressaltado, porque est na hora." Observe o uso de
porque na frase acima. Agora, analise as seguintes:
I. Porque deixar de lado uma causa porque lutamos h
tanto tempo?
II. Ningum sabe o porqu de nossa luta.
III. Ele vivia tranquilamente, porque tinha uma grande
herana.
IV. O governo no deve mudar, por qu?
V. No entendo por que voc to irresponsvel.
VI. Vivo feliz, porque amo minha esposa.

Assinale a nica afirmao verdadeira:
a) As frases I e III so as nicas corretas.
b) As frases I, III e V so corretas.
c) Na frase II, o porqu uma conjuno.
d) A frase V constitui uma pergunta indireta.
e) Em II, porqu formado por derivao regressiva.

Tarefa Complementar #

1. Assinale a alternativa que completa correta e
respectivamente os espaos constantes na frase a
seguir.
O ________ resulta da ______ entre a alga e o fungo.
a) lquen, simbiose
b) liquen, simbiose
c) liquem, simbiose
d) lquem, simbiose
e) liquem, simbise

2. Assinale a(s) frase(s) correta(s) quanto ao uso dos
termos sublinhados:
01. Ele ganhou o prmio porqu foi o melhor.
02. Vamos descobrir o por qu desta questo.
04. Voc no compareceu aula ontem por qu?
08. No sei por que brigamos.
16. Ele no o procurou por que estava doente.
32. Afinal, porque no procura sua amiga?

3. Considere o seguinte dilogo:
I. A: Por que voc est triste?
II. B: Porque ela me deixou.
III. A: E ela fez isso por qu?
IV. B: No sei o porqu. Tentei acabar com as causas
da crise por que passvamos.
V. A: Ah! Voc se perdeu nos porqus.
Do ponto de vista gramatical, os termos sublinhados
esto corretamente empregados:
a) somente em IV.
b) somente em I, III e V.
c) somente em II e IV.
d) I, II, III, IV e V.
e) somente em II e V.

4. Voc s precisa comprar a pipoca. O DVD
grtis.
Assinale a alternativa que apresenta a forma correta
para juntar os dois perodos da propaganda acima num
s.
a) Voc s precisa comprar a pipoca, entretanto o
DVD grtis.
b) Voc s precisa comprar a pipoca, j que o DVD
grtis.
c) Voc s precisa comprar a pipoca, inclusive o DVD
grtis.
d) Voc s precisa comprar a pipoca e o DVD grtis
e) Voc s precisa comprar a pipoca, cujo DVD
grtis.


Please purchase PDF Split-Merge on www.verypdf.com to remove this watermark.
Incluso para a Vida Gramtica

Pr-Vestibular da UFSC

7
UNIDADE 5

ESTUDO DE TEXTOS
Exerccios de Sala #















*arreliar-se = irritar-se; sentir despeito
**ditrio = zomberia

1. Explique, de modo denotativo, as expresses
transcritas a seguir:
a) de corao na goela
b) se dava mal
c) quatro pedras na mo

2. Em O amarelo devia saber isso. indicado
substituir a forma verbal devia pelo Futuro do
Pretrito? Por qu?

3. Na abertura das oraes No sabia. e apitara
foram omitidos conectivos. Indique que conjuno
pode ser empregada em cada um desses casos.

4. Na passagem Natural, xingar a me de uma pessoa
no vale nada, porque todo o mundo v logo que a
gente no tem a inteno de maltratar ningum. tem-
se a elipse do verbo ser e de uma conjuno.
Reescreva o perodo acrescentando tal verbo e o
conectivo indicado.

Tarefa Mnima #
























1. Assinale a(s) proposio(es) correta(s):
01. Em o senhor tambm, temos uma elipse.
02. No termo neles percebe-se a chamada
concordncia ideolgica elipse de nmero.
04. O ato de inspirar atribudo ao espelho ltimo
pargrafo um exemplo de Prosopopeia.
08. O termo se sublinhado no texto indica uma
ao reflexiva.

2. Reescreva a passagem Sou, porm, positivo, um
racional de modo a eliminar as vrgulas ali existentes.

3. Ao empregar o termo patas o narrador acaba por
se contradizer. Por qu?

4. O perodo Quem o Monstro? no nos favorece
uma compreenso imediata da leitura. Sugira como
reconstruir a passagem de modo a sanar esse
problema.

5. Explique, de modo denotativo, as expresses:
a) revivescncia de impresses atvicas
b) via de regra
c) fecundo ponto de partida

Tarefa Complementar #

H mais de meio sculo, continuou. Eu era
moleque, e eles uns curumins que j carregavam tudo,
iam dos barcos para o alto da praa, o dia todo assim.
Eu vendia tudo, de porta em porta. Entrei em centenas
de casas de Manaus, e quando no vendia nada, me
ofereciam guaran, banana frita, tapioquinha com caf.
Em vinte e poucos, por a, conheci o restaurante do
Galib e vi a Zana... Depois, a morte do Galib, o
nascimento dos irmos gmeos...
HATOUM, Milton. Dois irmos Texto Adaptado

1. Com relao ao texto acima assinale a(s)
proposio(es) correta(s).

01. Para que a passagem Eu era moleque, e eles uns
curumins fique mais coesa necessria a
repetio do verbo ser. Ou seja, tal trecho deveria
constar como Eu era moleque, e eles eram uns
curumins.
02. Em vinte e poucos, por a corresponde,
semanticamente, a Quando eu tinha vinte e
poucos anos.
Alguns minutos antes no pensava em nada,
mas agora suava frio e tinha lembranas
insuportveis. Era um sujeito violento, de corao
perto da goela. No, era um cabra que se arreliava*
algumas vezes e quando isso acontecia, sempre se
dava mal. (...) Impacientara-se e largara um
palavro. Natural, xingar a me de uma pessoa no
vale nada, porque todo o mundo v logo que a gente
no tem a inteno de maltratar ningum. Um
ditrio** sem importncia. O amarelo devia saber
isso. No sabia. Sara-se com quatro pedras na
mo, apitara. (...).
RAMOS, Graciliano Ramos. Vidas Secas, pg. 102


(...) Sou do interior, o senhor tambm; na
nossa terra, diz-se que nunca se deve olhar em
espelho s horas mortas da noite, estando-se
sozinho. Porque, neles, s vezes, em lugar de nossa
imagem, assombra-nos alguma outra e medonha
viso. Sou, porm, positivo, um racional, piso o
cho a ps e patas. Satisfazer-me com fantsticas
no-explicaes? jamais. Que me amedrontadora
viso seria ento aquela? Quem o Monstro?

Sendo talvez meu medo a revivescncia de
impresses atvicas? O espelho inspirava receio
supersticioso aos primitivos, aqueles povos com a
idia de que o reflexo de uma pessoa fosse a alma.
Via de regra, sabe-se o senhor, a superstio
fecundo ponto de partida para a pesquisa. (...)
ROSA, Guimares. Primeiras Estrias
O Espelho, pg. 115

Please purchase PDF Split-Merge on www.verypdf.com to remove this watermark.
Incluso para a Vida Gramtica

Pr-Vestibular da UFSC

8
04. Na frase Entrei em centenas de casas de
Manaus, pode-se substituir a forma verbal por
entrava, sem prejuzo do sentido.
08. No fechamento do texto encontramos um
pleonasmo.
16. A expresso por a faz referncia a um local.
32. O diminutivo tapioquinha foi empregado como
ironia, pois revela o desdm com que o narrador-
personagem era recebido nas casas as quais
visitava.
64. Por se tratar de uma fala coloquial, a vrgula
empregada em Depois, a morte do Galib pode
ser substituda pelo termo veio, mantendo-se o
sentido da sentena.

2. O programa Globo Esporte entrevistou, em
13/02/10, alguns rbitros de futebol de campo. E um
deles declarou J apitei partidas em dias de altssimas
temperaturas, tanto no frio quanto no calor.. Essa
frase foi mal estruturada. Assim, explique a
incoerncia nela existente.

TEXTO1
Meu Deus, que estais pendente em um madeiro,
Em cuja Lei protesto de viver,
Em cuja Santa Lei hei de morrer,
Animoso, constante, firme e inteiro.

Neste lance, por ser o derradeiro,
Pois vejo a minha vida anoitecer,
, meu Jesus, a hora de se ver
A brandura de um Pai, manso cordeiro.

Mui grande vosso amor e meu delito;
Porm pode ter fim todo o pecar,
E no o vosso amor, que infinito.

Essa razo me obriga a confiar
Que por mais que pequei neste conflito,
Espero em vosso amor de me salvar.

3. A partir dos versos de Gregrio de Matos citado
acima, assinale a(s) afirmao(es) correta(s):
01. Na expresso manso cordeiro (8 verso),
percebe-se ironia.
02. A passagem por ser o derradeiro (5 verso) pode
ser reescrita, mantendo-se o sentido do verso,
como porque o derradeiro.
04. O penltimo verso revela uma certa revolta do eu
lrico, j que apontado como um pecador.
08. O verbo anoitecer (6 verso) foi empregado no
sentido denotativo.
16. No fechamento do poema, o eu lrico revela a
esperana na sua salvao.
32. Em anoitecer (6 verso) tem-se um eufemismo.





















4. Sobre a passagem de Iracema, considere as
afirmaes:
I - Percebe-se, nesse trecho da obra Iracema, a
predominncia do discurso direto.
II - O termo seio (l. 2) empregado de modo
incoerente na passagem, j que se refere a um
personagem do sexo masculino.
III - O emprego dos termos vento e mar em
uma mesma sentena (l. 5) so um exemplo de
sinestesia, posto que indicam tato e viso,
respectivamente.
a) Apenas I est correta.
b) Apenas II est correta.
c) Apenas III est correta.
d) Apenas I e III esto corretas.
e) Nenhuma est correta.

5. Analise alguns versos da msica O paraso tem um
tempo bom, da banda Cidade Negra:

onda grande, onda pequena
na mar alta ou na mar vazia
surfei ondas de melodia
(...)
corre devagar

Agora, considere as sentenas:
I No primeiro verso mencionado, temos a presena
de anttese.
II O terceiro verso revela uma metfora.
III o verso corre devagar evidencia um paradoxo.

Quanto s trs classificaes enumeradas logo acima:
a) apenas a I est correta.
b) apenas a II est correta.
c) apenas a III est correta.
d) esto todas corretas.
e) nenhuma est correta.

UNIDADE 6

ANLISE SINTTICA DO PERODO
SIMPLES (Sujeito)

COMPARE:
a) A resposta do aluno foi satisfatria.
b) A resposta ao aluno foi satisfatria.

a) Era favorvel ao ru.
b) Deps favoravelmente ao ru.
que as margens do Jaguaribe para a taba dos
guerreiros de sua raa. Nestas guas as grandes igaras
que vm de longes terras, se esconderiam do vento e
do mar; daqui elas iriam ao Mearim destruir os
brancos tapuias, aliados dos tabajaras, inimigos de tua
nao.
O chefe pitiguara meditou e respondeu:
- Vai buscar teus guerreiros. Poti plantar sua
taba junto da mairi de seu irmo.
Aproximava-se Iracema. O cristo com um
gesto ordenou silncio ao chefe pitiguara.
ALENCAR, Jos de. Iracema, pg. 64
Veio Poti:
- O guerreiro branco pensa; o seio do irmo
est aberto para receber seu pensamento.
- Teu irmo pensa que este lugar o melhor do
Please purchase PDF Split-Merge on www.verypdf.com to remove this watermark.
Incluso para a Vida Gramtica

Pr-Vestibular da UFSC

9
Termos da orao

ESSENCIAIS
x Sujeito
x Predicado


INTEGRANTES
x Objeto Direto
x Objeto Indireto
x Complemento Nominal
x Agente da Passiva


ACESSRIOS
x Adjunto Adverbial
x Adjunto Adnominal
x Aposto
x Vocativo


SUJEITO
1. Simples
2. Composto
3. Oculto
4. Indeterminado
a) Ocorre com verbo na 3 pessoa do plural.
b) ou com a partcula SE indeterminando o sujeito.
5. Inexistente ou Orao sem Sujeito:
a) Com verbos que designam fenmenos
meteorolgicos.
b) HAVER no sentido de existir.
Cuidado!
Havamos partido.
c) FAZER, HAVER e IR indicando tempo.
d) SER, ESTAR e FAZER indicando tempo
meteorolgico.
e) SER indicando horas, datas e distncias.
f) PASSAR, CHEGAR e BASTAR + DE.


Exerccios de Sala #

1. Use os seguintes cdigos para os tipos de sujeito:
(1) Simples
(2) Oculto
(3) Composto
(4) Indeterminado
(5) Inexistente
( ) Chegaram, de manh, o mensageiro e o guia.
( ) Fala-se muito aqui.
( ) Aluga-se apartamento de dois quartos..
( ) No h flores no vaso.
( ) No existem flores no vaso.
( ) Houve uma manifestao diante da catedral.
( ) Precisa-se de servente.
( ) No inverno, amanhece tarde.
( ) Estou "pagando um mico" desgraado hoje.
( ) No dia seguinte, rezou Joo a Deus.
( ) "H uma gota de sangue em cada poema."
( ) J haviam sado do estdio os fotgrafos e os
jornalistas?

2. "Quando me procurar o desencanto, eu direi, sereno
e confiante, que minha vida no foi de todo intil."
O sujeito do verbo procurar :
a) indeterminado
b) eu (oculto)
c) o desencanto
d) me
e) inexistente

(UDESC PEDAGOGIA 2007.2) Leia com ateno a
tira abaixo.

No primeiro quadrinho h um erro em relao ao
emprego do sujeito.


3. Identifique esse erro.

4. Faa a correo e justifique-a.

Tarefa Mnima #

1. (UDESC) Assinale a alternativa incorreta em
relao a Tudo por estar cochilando!:
a) A passagem por estar cochilando tem valor
explicativo.
b) Tudo pronome indefinido.
c) Nessa orao o sujeito inexistente.
d) Em cochilando h dez letras e oito fonemas.
e) Estar cochilando uma locuo verbal, constituda
por infinitivo e gerndio.

2. A sentena correlata de Reviam-se os
manuscritos com cuidado na voz passiva analtica
:
a) Os manuscritos foram revistos com cuidado.
b) Os manuscritos eram revistos com cuidado.
c) Ns revamos com cuidado os manuscritos.
d) Revia-se os manuscritos com cuidado.
e) Reviram os manuscritos com cuidado.

3. (UDESC) Os primeiros versos do poema A Rosa
de Hiroxima, de VM, j retrata todo o horror da
guerra. Essa frase h um erro de:
a) regncia verbal.
b) concordncia verbal.
c) concordncia nominal.
d) regncia nominal.
e) colocao pronominal.

Texto 1

A. "A cabea inclinada, o espinhao curvo, agitava
os braos para a direita e para a esquerda. Esses
movimentos eram inteis, mas o vaqueiro, o pai do
vaqueiro, o av e outros antepassados mais antigos
haviam-se acostumado a percorrer veredas,
afastando o mato com as mos. E os filhos j
comeavam a reproduzir o gesto hereditrio." (p.17)
Please purchase PDF Split-Merge on www.verypdf.com to remove this watermark.
Incluso para a Vida Gramtica

Pr-Vestibular da UFSC

10

B. Pisou com firmeza o cho gretado, puxou a
faca de ponta, esgaravatou as unhas sujas. Tirou do
ai um pedao de fumo, picou-o, fez um cigarro
com palha de milho, acendeu-o ao binga, ps-se a
fumar, regalado.
Fabiano, voc um homem, exclamou em
voz alta.
Conteve-se, notou que os meninos estavam perto,
com certeza iam admirar-se ouvindo-o falar s.
E, pensando bem, ele no era um homem: era
apenas um cabra ocupado em guardar coisas dos
outros. Vermelho, queimado, tinha os olhos azuis, a
barba e os cabelos ruivos; mas como vivia em terra
alheia, cuidava de animais alheios, descobria-se,
encolhia-se na presena dos brancos e julgava-se
cabra.
Olhou em torno, com receio de que, fora os
meninos, algum tivesse percebido a frase
imprudente. Corrigiu-a, murmurando:
Voc um bicho, Fabiano." (p.18-19)

C. "Vivia longe dos homens, s se dava bem com
animais. Os seus ps duros quebravam espinhos e
no sentiam a quentura da terra. Montado,
confundia-se com o cavalo, grudava-se a ele. E
falava uma linguagem cantada, monossilbica e
gutural que o companheiro entendia. A p, no se
aguentava bem. Pendia para um lado, para o outro
lado, cambaio, torto e feio. s vezes utilizava nas
relaes com as pessoas a mesma lngua com que se
dirigia aos brutos exclamaes, onomatopeias.
Na verdade falava pouco. Admirava as palavras
compridas e difceis da gente da cidade, tentava
reproduzir algumas, em vo, mas sabia que elas
eram inteis e talvez perigosas." (p. 20)
RAMOS, Graciliano. Vidas Secas. Rio de Janeiro, So Paulo:
Editora Record, 2003.


4. Considerando os excertos de Vidas Secas, assinale
a(s) alternativa(s) correta(s).
01. No pargrafo Pisou com firmeza o cho gretado,
puxou a faca de ponta, esgaravatou as unhas sujas.
Tirou do ai um pedao de fumo, picou-o, fez um
cigarro com palha de milho, acendeu-o ao binga,
ps-se a fumar, regalado., que inicia o excerto B,
os verbos no pretrito perfeito do indicativo
representam uma sequncia cronolgica de aes.
02. As palavras firmeza e quentura (destacadas nos
excertos B e C, respectivamente) envolvem o
mesmo processo de formao: so adjetivos
derivados de outros adjetivos.
04. A sequncia com certeza iam admirar-se
ouvindo-o falar s (destacada no excerto B) pode
ser reescrita como com certeza iam admirar-se s
ouvindo-o falar, sem alterao do seu
significado.
08. No trecho como vivia em terra alheia, cuidava de
animais alheios, descobria-se, encolhia-se na
presena dos brancos e julgava-se cabra
(destacado no excerto B), o elemento como
estabelece uma relao semntica de comparao.
16. Em haviam-se acostumado a percorrer veredas,
afastando o mato com as mos (em destaque no
excerto A), o verbo no gerndio expressa uma
noo de sequencialidade temporal entre as aes
de afastar o mato e percorrer veredas.
32. Em: Montado, confundia-se com o cavalo,
grudava-se a ele. (excerto C) os termos
destacados indicam aes reflexivas.

Texto 2
O cego pedia suas esmolas rudemente. Xingava,
arrogava, desensofrido, dando com o bordo nas
portas das casas, no balco das vendas. Respeitavam-
no, mesmo por isso, jamais se viu que o
desatendessem, ou censurassem ou ralhassem,
repondo-o em seu nada.
ROSA, Joo Guimares.
Primeiras Estrias A Benfazeja, pg. 162
5. Assinale a(s) proposio(es) correta(s):
01. suas esmolas tem por funo complementar o
verbo pedir.
02. O pronome se destacado no texto indica uma
ao reflexiva.
04. Caso a vrgula empregada e o desatendessem, ou
censurassem fosse omitida, o sentido do perodo
no sofreria alterao.
08. A prclise facultativa na passagem se viu.
16. Os pronomes destacados em o desatendessem,
(...) repondo-o em seu nada. fazem aluso a
cego.

Tarefa Complementar #




























Achava-me, um dia, diante dos guichs do
London Bank espera de que o pagador gritasse a
minha chapa, quando vi a cochilar num banco ao fundo
certo corretor de negcios meu conhecido. Fui-me a ele,
alegre da oportunidade de iludir o fastio da espera com
uns dedos de prosa amiga.
- Esperando sua horinha, hein? disse-lhe com
um tapa amigvel no ombro, enquanto me sentava ao
seu lado.
- verdade. Espero pacientemente que me
cantem o nmero e, enquanto espero, filosofo sobre os
males que traz vida a desonestidade dos homens.
- ?
- Sim, _(1)_, se no fosse a desonestidade dos
homens, tudo se simplificaria grandemente. Esta demora
no pagamento do mais simples cheque, de donde _(2)_?
Da necessidade de controle em vista dos artifcios da
desonestidade. Fossem todos os homens srios, no
houvesse hiptese de falsificaes ou abusos, o
recebimento de um dinheiro far-se-ia instantneo. (...)
- O amigo tem razo concordei eu (...). A vida
complicada, existem leis, polcia, embaraos de toda
espcie, burocracia e mil peias (...).
LOBATO, Monteiro.
O Presidente Negro - Captulo I Trecho Adaptado

Please purchase PDF Split-Merge on www.verypdf.com to remove this watermark.
Incluso para a Vida Gramtica

Pr-Vestibular da UFSC

11
1. Tendo em conta o fragmento da obra de
Monteiro Lobato, indique o que correto:
01. Ao declarar Fui-me a ele, alegre da oportunidade
de iludir o fastio da espera (linhas 03-04), o
narrador-personagem passa a idia de que estava
satisfeito por haver encontrado uma forma de
amenizar o tdio causado pela espera.
02. Na inteno de se respeitar a pronncia culta,
necessrio reconhecer que o e presente em
espontneo corresponde a uma semivogal.
04. Por se tratar de uma proparoxtona, a palavra
sublinhada no texto deveria estar acentuada.
08. As lacunas _(1)_ e _(2)_ devem ser preenchidas,
respectivamente, com porque e provm.
16. As palavras negcios e srios, embora no
possuam o mesmo nmero de slabas, tm sua
acentuao grfica justificada pela mesma regra.
32. O termo complicada, empregado na ltima frase
do texto, tem como funo caracterizar o sujeito
vida.
64. A fim de concordar com o sujeito composto, o
verbo haver, empregado na linha 13, deveria
apresentar-se no plural.

2. Indique a(s) alternativa(s) em que se reescreve o
trecho transcrito em destaque a seguir, sem alterao
de sentido e respeitando-se as normas gramaticais:
- Esperando sua
horinha, hein?
disse-lhe com um tapa
amigvel no ombro,
enquanto me sentava
ao seu lado.
01. - Esperando sua horinha, hein? disse eu a ele,
dando um tapa de amigos no seu ombro, assim que
me sentei a seu lado.
02. - E a, esperando sua horinha? eu disse
enquanto me sentava a seu lado a ele, e lhe dava
um tapa amigvel no ombro.
04. - Est esperando sua horinha, hein? eu disse a
ele, dando um tapa de amigos no seu ombro, no
momento em que me sentava a seu lado.
08. Enquanto me sentava a seu lado e dava um tapa
amigvel em seu ombro, disse-lhe: - Esperando sua
horinha, hein?.
16. - Ests esperando sua horinha, hein? lhe disse
com tapa amistozo em seu ombro, no momento em
que sentava do lado dele.
32. No momento em que eu estava sentando-me do
seu lado dei um tapa de amigos no ombro e disse
ele: - Esperando sua horinha, hein?


1



5



Ao entardecer
A chuva bate nas costas desnudas dos
pescadores a puxarem os cabos da rede do
arrasto. Alguns veranistas abrigam-se sob
improvisados guarda-chuvas. As crianas entram
no mar, cercam a rede e recolhem os peixes que
escapam das malhas; misturam-se: crianas,
peixes e gua.
Os pescadores andam de costas, em gritos

10




15




20




25




30


e risos, num cdigo s deles, corpo arcado para
trs, calcanhares se firmando na areia, a cada
passada. Ignoram o vozerio dos espectadores que
se agrupam, em prvia disputa.
- Me reserva uma pescadinha, Z.
- Que vier de lula eu fico.
- Olha uma raia. Como d raia, hein? Diz
que tem quem come elas, que tu achas?
A rede na beira da praia, o pedido: Pr trs,
faz favor! Os pescadores se juntam, redobram
esforos. O tropeo dos veranistas, a disputa pela
minguada colheita, a bulha das crianas,
recolhendo sardinhas que lhes escapam das mos,
o ploc-ploc dos peixes se debatendo na areia.
- No esquece, Z! todos so Z.
At Onofre, durante dcadas vigia de pesca
ele preferia olheiro, estava mais de ajuste com
sua funo , o melhor das praias todas da ilha,
o que diziam. Ele no carecia subir no costo ou
se esticar na ponta dos ps, largando os olhos
inquietos pela extenso do mar, em busca das
manchas reveladoras.
(...)
(KRIEGER, Maria de Lourdes. Treze Cascaes)

3. A partir do conto transcrito acima, indique o que for
correto:
01. Os dois-pontos empregados no primeiro pargrafo
tm por finalidade destacar uma citao.
02. Coma a fala Que vier de lula eu fico., o
personagem revela uma preferncia sua.
04. O substantivo constante em redobram esforos
(6 pargrafo) apresenta variao de pronncia em
relao a seu singular.
08. Onomatopeias do veracidade a textos. As
expresses ploc-ploc e hein ambas empregadas
no texto lido so exemplos desse tipo de recurso.
16. Em Ignoram o vozerio dos espectadores que se
agrupam, em prvia disputa., o vocbulo se indica
uma ao reflexiva, tal como ocorre em Alguns
veranistas abrigam-se sob improvisados guarda-
chuvas..
32. Transcrevendo a passagem Diz que tem quem
come elas (5 pargrafo) para a linguagem formal,
teramos Dizem que h quem coma-as.
64. O pronome destacado em recolhendo sardinhas
que lhes escapam das mos (6 pargrafo)
indicativo de posse.

4. Assinale a(s) alternativa(s) que apresenta(m) uma
reformulao do trecho em destaque, mantendo-se seu
sentido e respeitando-se a Gramtica Normativa:
At Onofre, durante dcadas vigia de pesca ele
preferia olheiro,
estava mais de ajuste com sua funo ,
o melhor das praias todas da ilha, o que diziam.
01. Inclusive Onofre, o qual foi, segundo diziam, o
melhor vigia de pesca de todas as praias da ilha
durante dcadas. Ele preferia ser chamado de
olheiro, pois estava mais de acordo com sua
funo.
Please purchase PDF Split-Merge on www.verypdf.com to remove this watermark.
Incluso para a Vida Gramtica

Pr-Vestibular da UFSC

12
02. Onofre, inclusive, preferia ser chamado de olheiro
a ser denominado vigia de pesca, pois havia sido o
melhor na funo, segundo o que se conta.
04. At Onofre, o qual se destacou como vigia de
pesca o melhor de todas as praias da ilha
preferia ser olheiro, por estar mais de acordo com
sua funo.
08. At ele, Onofre, melhor de todos os olheiros da
ilha. Foi ele destaque em todas as praias da ilha por
dcadas, por ajustar-se funo.
16. Inclusive Onofre o melhor vigia pesca entre as
praias da ilha por dcadas , embora preferisse ser
chamado de olheiro do que ser identificado por
vigia.

UNIDADE 7

ANLISE SINTTICA DO PERODO
SIMPLES (Predicado)
Para Recordar:
VI verbo intransitivo
(no precisa de complemento)
VT verbo transitivo
(necessita de complemento, ou seja, de objeto)
VTD verbo transitivo direto
(necessita de objeto direto, ou seja, SEM preposio)
VTI verbo transitivo indireto
(necessita de objeto indireto, ou seja, COM
preposio)
VL verbo de ligao
(ser, estar, parecer, permanecer, ficar, andar,
continuar)

PREDICADO, INTEGRANTES E ACESSRIOS
PREDICADO
Nominal: Quando houver VL; obrigatrio o
PREDICATIVO DO SUJEITO.
Verbal: Quando se trata de verbo que NO SEJA DE
LIGAO. Apresenta como ncleo um verbo T.D.,
T.I., T.D.I ou I.
Verbo-nominal: Apresenta dois ncleos. Ou seja, um
V.T. ou V.I. e predicativo simultaneamente.
PREDICATIVO DO SUJEITO
Caracterizao no feita necessariamente por um
adjetivo.
OBJETO
Objeto Direto: Complemento verbal pedido por
VERBO TRANSITIVO DIRETO.
Objeto Indireto: Complemento verbal pedido por
VERBO TRANSITIVO INDIRETO.
COMPLEMENTO NOMINAL
Completa o sentido de um nome (substantivo, adjetivo e
advrbio). Sempre ter preposio.
AGENTE DA PASSIVA: equivale ao sujeito da voz
ativa.
Analise: Nosso pr-vestibular disponibilizou
um maior nmero de vagas.
Um maior nmero de vagas
foi disponibilizado por nosso vestibular.
(Voz Passiva Analtica)
No presente:
Nosso pr-vestibular disponibiliza
um maior nmero de vagas.
Um maior nmero de vagas
disponibilizado por nosso vestibular.
No futuro:
Nosso pr-vestibular disponibilizar
um maior nmero de vagas.
Um maior nmero de vagas
ser disponibilizado por nosso vestibular.
Contudo...
Disponibilizou-se um maior nmero de vagas.
(Voz Passiva Sinttica)


Ateno!
Em uma orao com um VTD e um VTI,
empregados simultaneamente, o complemento sem
preposio deve vir primeiro.
H o OBJETO DIRETO PREPOSICIONADO.
Um erro quanto ao uso de preposio pode alterar
o sentido da orao.
Ele gostou o suco.
Ele gostou do suco.
Ento, cuidado com a Regncia!

ADJUNTO ADNOMINAL: modifica a significao de
SUBSTANTIVO. Trata-se, sempre, de um ARTIGO, um
NUMERAL, um ADJETIVO, uma LOCUO ou um
PRONOME.
ADJUNTO ADVERBIAL: modifica o VERBO, o
ADJETIVO ou o ADVRBIO.
APOSTO: explica outro termo da orao. separado do
restante da orao por meio de vrgulas, travesses ou
dois pontos (quando no final do perodo).
VOCATIVO: indica chamamento.

Observe que, se no tomarmos cuidado com a
pontuao, poderemos estar alterando o sentido da
frase.
Otavio, um aluno no recebeu a apostila.
(VOCATIVO)
Otavio, um aluno, no recebeu a apostila. (APOSTO)

Exerccios de Sala #

1. Indique a funo sinttica de cada um dos termos
sublinhados na msica a seguir:
HOMEM PRIMATA
Desde os primrdios
At hoje em dia
O homem ainda faz
O que o macaco fazia
Eu no trabalhava, eu no sabia
Que o homem criava e tambm destrua.
Homem primata
Capitalismo selvagem
, ,
Eu aprendi:
Please purchase PDF Split-Merge on www.verypdf.com to remove this watermark.
Incluso para a Vida Gramtica

Pr-Vestibular da UFSC

13
A vida um jogo
Cada um por si
E Deus contra todos
Voc vai morrer e no vai pro cu
bom aprender, a vida cruel.
Homem primata
Capitalismo selvagem
, ,
Eu me perdi na selva de pedra
Eu me perdi, eu me perdi
(...)

BRITTO, Srgio, FROMER, Marcelo, REIS, Nando,
PESSOA, Ciro. Do CD Cabea de dinossauro.

2. Na msica Homem Primata, o paralelo estabelecido
entre o homem e macaco alude:
a) a uma das teorias sobre a origem da espcie
humana.
b) ao comportamento irracional do homem na
sociedade capitalista.
c) s semelhanas biolgicas entre os dois seres.
d) ao bom relacionamento entre homem e macaco.
e) ao capitalismo selvagem da sociedade
contempornea.

3. Use o cdigo adequado:

(1) sujeito
(2) predicado
(3) objeto direto
(4) objeto indireto
(5) predicativo do sujeito
(6) adjunto adnominal
(7) adjunto adverbial
(8) complemento nominal
(9) vocativo
(10) agente da passiva

( ) Em nosso pas, a vulgaridade um ttulo.
( ) Foram assistir ao batizado.
( ) As circunstncias mudam tudo.
( ) Aquela prova foi realizada pelos alunos.
( ) Todos os noivos parecem bons rapazes.
( ) Ele resistente ao frio.
( ) Aqueles alunos parecem bons.
( ) Jamais me esquecerei de voc.
( ) Meu colega caiu mal.
( ) Revelarei isto a ele.
( ) Os pais do comida aos filhos.
( ) Os pais do comida aos filhos.
( ) Minha redao no agradou.
( ) Oferecemos flores noiva.
( ) Ele disse tudo.
( ) Cultive boas amizades.
( ) Sade, amigos!
( ) desejo de todos um mundo mais justo.


4. (UDESC HISTRIA 2007.2) Comente a orao
Dominar a lngua, em qualquer de seus aspectos,
ferramenta para se avanar na vida.
5. (UDESC HISTRIA 2007.2) Leia com ateno a
tira abaixo.











No segundo quadrinho da tira h uma vrgula e o
terceiro quadrinho sugere a retirada do sinal de
pontuao.
a) Explique o sentido da orao com o sinal de
pontuao.
b) Explique o sentido da orao sem o sinal de
pontuao.

6. (UDESC PEDAGOGIA - Adaptada) Em relao ao
texto abaixo, comente a importncia da atividade
ldica no desenvolvimento para a aprendizagem.
A atividade ldica, inerente a toda criana,
vista como nova forma e possibilidade de ela entender
a realidade, por poder vivenciar situaes de seu dia-a-
dia. Isso acaba propiciando-lhe o desenvolvimento em
vrios sentidos.

7. A expresso em destaque no pargrafo acima
poderia ser retira do texto, mantendo-se a clareza do
exposto? Justifique sua resposta.

8. E o substantivo situaes imprescindvel
compreenso do perodo? Por qu?

9. Identifique a funo sinttica do pronome tono
empregado em Isso acaba propiciando-lhe o
desenvolvimento em vrios sentidos..

Tarefa Mnima #

1. Na orao "Eu sou, Senhor, a ovelha desgarrada."
(Gregrio de Matos), temos:
01. vocativo
02. aposto
04. trs adjuntos adnominais
08. predicativo do sujeito
16. predicado verbal.

2. Assinale a(s) alternativa(s) correta(s):
01. O Amazonas o maior rio brasileiro. - A palavra
sublinhada adjunto adverbial de intensidade.
02. Tem compaixo de ns, Cristo! - A expresso
sublinhada complemento nominal.
04. Pai, eu te agradeo o agasalho que me deste. - A
palavra sublinhada vocativo.
08. Lemos "Vidas Secas", a obra-prima de Graciliano
Ramos - A frase possui objeto direto e aposto.

Please purchase PDF Split-Merge on www.verypdf.com to remove this watermark.
Incluso para a Vida Gramtica

Pr-Vestibular da UFSC

14
16. As suas foras foram rapidamente recuperadas. - A
frase possui adjunto adverbial de tempo.

3. Dentre os enunciados a seguir, identifique o(s)
correto(s).
01. At conserva o mesmo valor semntico em
Afonso Ribeiro seguiu os tupiniquins at sua
aldeia (Eduardo Bueno, A viagem do
descobrimento) e em Doena associada elite, a
anorexia comea a fazer vtimas na periferia e at
no serto nordestino. (poca).
02. A mensagem: Movimento estudantil: pau,
pedra, mas no o fim do caminho., encontrada
em um panfleto direcionado a estudantes de
Comunicao Social, mantm relaes
intertextuais com conhecida cano da MPB.
04. O uso do futuro do pretrito em Os cientistas vo
ligar uma mquina que seria capaz de dissolver o
planeta? (Superinteressante) cria um efeito de
sentido de incerteza, de irrealidade.
08. Temendo o mau cheiro iminente, o proprietrio
do cabar ordenou ao empregado que pegasse o
cavalo para levar o morto. (Ablio Leite de
Barros, Campo Grande 100 anos de construo)
um perodo composto por subordinao e
coordenao.
16. No perodo A idia da teletaxa evoluiu a partir da
constatao de que a sociedade paulista estaria
disposta a contar com uma poltica de Primeiro
Mundo. (Folha de S. Paulo), h duas oraes que
exercem a funo sinttica de objeto indireto.
32. Em Foi ele que levou a Costa e Silva a notcia de
que seu governo estava extinto e seu cargo, vago.
(Manchete, 14/8/99, p. 61), a vrgula foi
empregada para indicar a omisso de um termo.

4. Levando em considerao a anlise sinttica, h
uma correta classificao do termo em destaque na(s)
alternativa(s):
01. Tambm no cantarei o mundo futuro. Objeto
Direto
02. Direi a todos aquilo que esperam ouvir. Verbo
Intransitivo
04. No serei o poeta de um mundo caduco. Verbo
Transitivo Direto
08. Estou preso vida e olho meus companheiros.
Adjunto Adnominal
16. Os verdadeiros vitoriosos sempre so humildes.
Sujeito Composto
32. Com muita destreza, atendeu a meu chamado.
Adjunto Adverbial

5. Quando me procurar o desencanto, ou a morte, eu
direi, sereno e confiante, que minha vida no foi de
todo intil.
Os termos sereno e distante so:
a) apostos.
b) predicativos do sujeito.
c) adjuntos adnominais.
d) sujeitos.
e) complementos nominais.

6. Analise as passagens a seguir.
(...) eu no sinto necessidade dos meus brinquedos
(...).
O seu destino fora cruel.
Gritava, dizia tanta coisa (...).
Quanto funo sinttica, os termos destacados so
respectivamente:
a) objeto direto, predicativo e objeto indireto.
b) complemento nominal, objeto direto e sujeito.
c) adjunto adnominal, sujeito e objeto indireto.
d) complemento nominal, predicativo e objeto direto.
e) Objeto direto, predicativo e complemento nominal.

Tarefa Complementar #
1. Em que alternativa(s) h objeto direto
preposicionado?
01. Passou aos filhos a herana recebida dos pais.
02. Amou a seu pai com a mais plena grandeza da
alma.
04. Naquele tempo, era muito fcil viajar para os
infernos.
08. Em dias ensolarados, gosto de ver nuvens
flutuarem nos cus de agosto.

2. Assinale a(s) proposio(es) correta(s):
01. Segure o garfo direito. (adjunto adverbial de
modo)
02. Passava-se isto na Rua da Lapa. (adjunto
adverbial causa)
04. Ficaram encantados com sua gentileza. (objeto
indireto)
08. Sentiu falta das pequenas coisas. (objeto indireto)
16. Pede-se silncio. (sujeito simples silncio)
32. Os viajantes chegaram cedo. (adjunto adverbial de
tempo)

3. Apenas em um dos perodos abaixo ocorre uma
orao sem sujeito. Assinale-o:
a) Havia comeado a festa, quando Bianca chegou.
b) Eles se houveram muito mal durante a defesa dos
trabalhos.
c) Faz dois anos que no vejo minha melhor amiga.
d) Andam espalhando boatos a respeito da queda do
ministro.
e) Vive-se muito bem em Florianpolis.

4. Quando soube que ele ia chegar, senti uma coisa
estranha, fiquei agitado. A imagem que faziam dele
era a de um ser perfeito ou de algum que buscava a
perfeio. Pensei nisso: se for ele o meu pai, ento sou
filho de um homem quase perfeito. A sabedoria dele
no me intimidava, nunca tinha sido uma ameaa para
mim. Eu o considerava um homem tenaz, respeitado
em casa, a ponto de ser elogiado pelo pai, que no
sabia at onde o filho queria chegar.

A respeito do pargrafo transcrito logo acima, o qual
faz parte da obra dois irmos, de Milton Hatoum,
correto afirmarmos que:
Please purchase PDF Split-Merge on www.verypdf.com to remove this watermark.
Incluso para a Vida Gramtica

Pr-Vestibular da UFSC

15
01. as passagens destacadas em Quando soube que
ele ia chegar, senti uma coisa estranha exercem
funo de objeto direto.
02. o vocbulo sublinhado em A imagem que faziam
dele era a de um ser perfeito classificado como
um verbo de ligao.
04. O verbo pensar, empregado na orao Pensei
nisso transitivo direto e indireto,
simultaneamente.
08. O adjetivo perfeito constante no trecho ento
sou filho de um homem quase perfeito
corresponde a um adjunto adnominal que tem
funo caracterizar o substantivo homem.
16. Tambm em relao ao trecho ento sou filho de
um homem quase perfeito podemos afirmar que
quase classificado como um adjunto adverbial.
32. Na orao Eu o considerava um homem tenaz o
termo sublinhado exerce funo de objeto direto.

1



5




10




15




20
(...) No sei como se chamaria o medo
de no ter o que ler. Existem as conhecidas
claustrofobias (medo de lugares fechados),
agorafobia (medo de espaos abertos), acrofobia
(medo de altura), collorfobia (medo do que ele
vai nos aprontar) e as menos conhecidas
ailurofobia (medo de gatos), iatrofobia (medo de
mdicos) e at treiskaidekafobia (medo do
nmero treze), mas o pnico de estar, por
exemplo, num quarto de hotel, com insnia, sem
nada para ler no sei que nome tem. uma das
minhas neuroses. O vcio que lhe d origem a
gutembergomania, uma dependncia patolgica
na palavra impressa. Na falta dela, qualquer
palavra serve. J sa de cama de hotel no meio da
noite e entrei no banheiro para ver se as torneiras
tinham Frio e Quente escritos por extenso,
para saciar minha sede de letras. J ajeitei o
travesseiro, ajustei a luz e abri a lista telefnica,
tentando me convencer que, pelo menos no
nmero de personagens, seria um razovel
substituto para um romance russo. J revirei
cobertores e lenis, procura de uma etiqueta,
qualquer coisa.
(...)
VERISSIMO, Luis Fernando. Comdias para se Ler na Escola.
Fobias, pg. 104

5. A partir do trecho da crnica Fobias, considere as
afirmaes a seguir e assinale a(s) correta(s):
01. A orao como se chamaria o medo (linha 1)
atua como complemento de sua antecedente.
02. Para revelar humor, Verssimo joga com a
formao de palavras, empregando vocbulos que
apresentam prefixos como claustofobias (linha 3) .
04. O termo collorfobia (linha 5) classificado como
um neologismo.
08. O pronome destacado em O vcio que lhe d
origem (linha 12) atua como complemento
indireto do verbo dar.
16. A passagem uma dependncia patolgica na
palavra impressa (linhas 1-14) constitui um
aposto, visto que esclarece o termo
gutembergomania (linha13).

UNIDADE 8

PONTUAO
Na lngua portuguesa, a ordem normal dos termos em
uma frase :
sujeito verbo complemento verbal
adjuntos adverbiais
Se os termos da orao se dispem assim, dizemos que
ocorre ordem direta.
Voc conquistar sua aprovao em breve
Suj. Verbo Comp. Verbal Adj. Adv.
Caso ocorra alguma alterao nessa sequncia,
teremos a ordem indireta.
Em breve, voc conquistar sua aprovao!
Termo
Deslocado
Quando a orao est na ordem direta, no se separam
seus termos imediatos por vrgulas. Ou seja, no
devemos separar:
- o sujeito do verbo
- o verbo de seu(s) complemento(s)
- o nome de seu(s) complemento(s) ou adjunto(s)

A VRGULA ser utilizada no interior da orao para
separar:
1 expresses de carter explicativo ou corretivo
Ex.: No teremos aula amanh, ou melhor, depois de
amanh.
2 conjunes coordenativas intercaladas
Ex.: Sua atitude, no entanto, pareceu-nos incoerente.
3 adjuntos adverbiais intercalados
Ex.: O ator, naquele dia, parecia um pouco nervoso.
4 aposto intercalado
Ex.: Obama, presidente dos EUA, ficou impressionado
com a receptividade dos brasileiros.
Importante: o aposto tambm pode ser intercalado
por parnteses ou por travesses. Quando est
posicionado no final da frase, pode vir precedido de
dois-pontos.

A VRGULA tambm empregada para indicar o
deslocamento de um termo na frase.
1 adjunto adverbial
Ex.: No ano passado, fiz a viagem de meus sonhos.
Importante: Se esse adjunto for um simples advrbio,
a vrgula dispensvel.
2 complemento pleonstico
Ex.: Esse livro, j o li quando cursava o Ensino Mdio.
3 o nome de lugar na indicao de datas
Ex.: Florianpolis, 21 de Setembro de 2003.
Ento... Fique de olho se lhe for solicitado que
produza uma carta neste vestiba!

A VRGULA tambm usada para:
1 separar vocativo
Ex.: Galera, fiquem de olho na pontuao.
Please purchase PDF Split-Merge on www.verypdf.com to remove this watermark.
Incluso para a Vida Gramtica

Pr-Vestibular da UFSC

16
Lembre: Quando se quer dar maior nfase ao vocativo,
pode-se usar ponto de exclamao.
2 separar termos coordenados assindticos, ou seja,
ligadas sem emprego de conjuno
Ex.: Seus conselhos me inspiraram tranquilidade,
segurana, paz.













A VRGULA entre oraes
As oraes que formam um perodo podem ser
separadas por vrgulas ou no, dependendo de sua
classificao.
1 oraes subordinadas adjetivas explicativas:
sempre separadas por vrgula.
2 oraes subordinadas adjetivas restritivas: via de
regra no so separadas por vrgula.
Contudo admite-se vrgula ao seu final (Mas nunca
antes delas!) quando:
a) tiverem certa extenso.
Ex.: O homem que foi sequestrado na semana passada
em So Jos, um empresrio muito conhecido na
regio.
b) houver sequncia de dois verbos.
Ex.: O homem que fuma, vive pouco.
3 oraes subordinadas adverbiais: sero separadas
por vrgula sempre que vierem antes da orao
principal.
4 oraes subordinadas substantivas: no devem ser
separadas da principal por meio de vrgula (Com
exceo das apositivas!).
5 oraes coordenadas: so separadas por vrgula
(Exceto as aditivas!)

Exerccios de Sala #

1. Empregue as vrgulas nas frases a seguir:
a. Os meninos atiravam pedras na vidraa; e as
meninas no carro.
b. Durante a longa noite de viglia Marlia esperou
Dirceu.
c. O quarto tinha uma cama uma mesa uma cadeira.
d. O av de Paulo um espanhol chamado Luciano
era muito valente.
e. Com muita rapidez e agilidade o piloto evitou um
choque de avies.
f. Os tamandus animais inofensivos sabem
defender-se.
g. V brigar em outro lugar meu senhor!
h. Aquele aluno por exemplo obteve o primeiro
lugar no concurso.
i. A garota tem medo de altura de recintos fechados
de ambientes escuros.
j. Aproxima-te rapaz.
k. Daniel gosta de louras; Laudelino de morenas.
l. Ela vai cantar ou melhor tentar cantar um samba.
m. A festa foi cancelada ou seja ficaremos sem baile.
n. Laguna cidade histrica de Santa Catarina possui
o marco do Tratado de Tordesilhas.
o. Importante plo industrial Joinvile a maior
cidade de Santa Catarina.
p. Marta e eu estamos te esperando pai.
q. O funcionrio chegou atrasado sem o uniforme
sem o crach de identificao.
r. Atrs de algumas rochas escondeu-se o menino.
s. Pediu a todos no comeo da apresentao um
pouco de silncio.
t. Eu quero a luz das estrelas; tu os raios de sol.
u. A laranja rica em vitamina C fruta para consumo
dirio.
v. Ali est Filomena uma bela e elegante mulher.
w. Caros colegas fazei do ideal uma ao concreta!
x. As tuas idias devem ser colocadas em prtica
Jos.
y. Pretendo estudar ingls ou melhor italiano.
z. As guerras por exemplo so responsveis por
grande parte do sofrimento humano.

Tarefa Mnima #

1. As frases "Antigamente, ia-se cidade" e "Hoje se
vai ao shopping center" so igualmente iniciadas por
advrbio de tempo. No entanto, separa-se por vrgula o
advrbio da primeira, mas no o da segunda. Tal fato
revela:
a) uma distrao por parte do emissor, pois o uso da
vrgula fundamental nessa situao.
b) Um erro grave, visto que no se deve usar vrgula
para separar o advrbio do restante da frase.
c) Que falta, ao autor do texto, o domnio dos sinais
de pontuao.
d) Obrigatoriedade de emprego da vrgula apenas na
primeira frase.
e) Uma escolha estilstica do emissor, uma vez que
nesse tipo de construo facultativo o emprego
da vrgula.

2. Considere os perodos I, II e III, pontuados de duas
maneiras diferentes:
I - Os jovens que amam so mais felizes.
Os jovens, que amam, so mais felizes.
II - Eu no fumo por opo.
Eu no fumo, por opo.
III - Os padres rezavam e o povo acompanhava a reza.
Os padres rezavam, e o povo acompanhava a reza.
Com pontuao diferente, ocorre alterao de sentido:
a) Em todos os perodos.
b) Em nenhum dos perodos.
c) Somente nos perodos I e II.
d) Somente nos perodos II e III.
e) Somente nos perodos I e III.

Termos coordenados ligados pelas conjunes E, OU,
NEM dispensam vrgula.
Seus conselhos me inspiraram tranquilidade,
segurana e paz.

Se, no entanto, essas conjunes vierem repetidas para
dar ideia de nfase, a vrgula dever ser empregada.
No fazia exerccios de Fsica, nem de Qumica, nem
de Matemtica, nem de Literatura, mas sonhava ser
aprovado na UFSC.

Please purchase PDF Split-Merge on www.verypdf.com to remove this watermark.
Incluso para a Vida Gramtica

Pr-Vestibular da UFSC

17
3. Observe as frases:
I - Ele foi, logo eu no fui.
II - O menino, disse ele, no vai.
III - Deus, que Pai, no nos abandona.
IV - Saindo ele e os demais, os meninos ficaro ss.

Assinale a afirmativa correta.
a) Em I h erro de pontuao.
b) Em II e III as vrgulas podem ser retiradas sem que
haja erro.
c) Na I, se se mudar a vrgula de posio, muda-se o
sentido da frase.
d) Na II, faltam dois pontos depois de disse.
e) N.d.a.

Tarefa Complementar #

1. "s vezes, tambm a gente tem o consolo de
saber..." A vrgula, na frase acima, foi usada:
a) para indicar elipse do verbo.
b) Por necessidade de clareza.
c) Para realar o aposto.
d) Para realar um adjunto adverbial.
e) Para separar termos da mesma funo.

2. Aponte a alternativa pontuada corretamente.
a) Entregarei quele rapaz, o filho do farmacutico, a
receita que, devia ser anexada ao envelope.
b) Entregarei quele rapaz, o filho do farmacutico - a
receita, que devia ser, anexada ao envelope.
c) Entregarei quele rapaz, o filho do farmacutico, a
receita que devia ser anexada ao envelope.
d) Entregarei quele rapaz o filho do farmacutico, a
receita que devia ser anexada ao envelope.
e) Entregarei quele - rapaz o filho do farmacutico - a
receita, que devia ser anexada ao envelope.

3. "Mensageira da idia, a palavra a mais bela
expresso da alma humana", emprega-se a vrgula
para:
a) Separar vocativo.
b) Indicar a elipse de um termo.
c) Separar elementos paralelos.
d) Separar aposto.
e) Separar uma orao adjetiva de valor restritivo.

UNIDADE 9

ESTUDO DE TEXTOS

Exerccios de Sala #

SEXA
- Pai..
- Hmmm?
- Como o feminino de sexo?
- O qu?
- O feminino de sexo.
- No tem.
- Sexo no tem feminino?
- No.
- S tem sexo masculino?
- . Quer dizer, no. Existem dois sexos. Masculino e
feminino.
- E como o feminino de sexo?
- No tem feminino. Sexo sempre masculino.
- Mas tu mesmo disse que tem sexo masculino e
feminino...
- O sexo pode ser masculino ou feminino. A palavra
"sexo" masculina. O sexo masculino, o sexo
feminino.
- No devia ser "a sexa"?
- No.
- Por que no?
- Porque no! Desculpe. Porque no. "Sexo" sempre
masculino.
- O sexo da mulher masculino?
- . No! O sexo da mulher feminino.
- E como o feminino?
- Sexo mesmo. Igual ao do homem.
- O sexo da mulher igual ao do homem?
- . Quer dizer... Olha aqui: tem sexo masculino e o
sexo feminino, certo?
- Certo.
- So duas coisas diferentes.
- Ento como o feminino de sexo?
- igual ao masculino.
- Mas no so diferentes?
- No. Ou, so! Mas a palavra a mesma. Muda o sexo,
mas no muda a palavra.
- Mas ento no muda o sexo. sempre masculino.
- A palavra masculina.
- No. "A palavra" feminino. Se fosse masculino
seria "o pal..."
- Chega! Vai brincar, vai...
O garoto sai e a me entra. O pai comenta:
- Temos que ficar de olho nesse guri...
- Por qu?
- Ele s pensa em gramtica...
(VERSSIMO, Lus Fernando.
Comdias para se Ler na Escola)

1. No texto de Verssimo, h algumas palavras
sublinhadas. Faa a anlise fonolgica de cada uma
delas.
2. Assinale a(s) proposio(es) correta(s):
01. A expresso ficar de olho tpica da linguagem
coloquial e, por isso, no foi empregada
adequadamente no texto Sexa.
02. O humor do texto se sustenta unicamente na
preocupao excessiva do menino com o gnero
da palavra sexo.
04. A ltima fala do pai obscura, pois, atravs dela,
pode-se deduzir tanto que o menino aficionado
pela gramtica de Lngua Portuguesa como que
no d a devida importncia sexualidade.
08. O pai tira todas as dvidas da criana em relao
palavra sexo.
16. Hmmm? (linha 2) tem o mesmo significado de
O que ?.
Please purchase PDF Split-Merge on www.verypdf.com to remove this watermark.
Incluso para a Vida Gramtica

Pr-Vestibular da UFSC

18
32. Ainda que de uma forma sutil, a me do garoto
demonstra interesse pelo que estaria acontecendo
com seu filho.

3. Assinale a(s) afirmao(es) correta(s) quanto
passagem destacada no texto:
01. Ao interromper a fala do pai, o filho revela-se
impaciente com a situao.
02. O conectivo se pode ser substitudo por caso
mantendo-se o sentido da frase.
04. Ao determinar que necessrio ficar de olho no
menino, o pai passa a ideia de que a me nunca
presta ateno nas atitudes do filho.
08. As reticncias indicam a agressividade do pai
dando ordens ao filho.
16. A ltima frase pode ser reescrita como Ele d
muito mais ateno Lngua Portuguesa. sem
qualquer alterao de sentido.

Tarefa Mnima #

1. Em relao aos excertos abaixo, das obras Riacho
Doce e Morte e Vida Severina, correto afirmar que:

Texto 1








Texto 2










01. No primeiro perodo do Texto 1 temos a presena
de verbos conjugado no pretrito mais-que-perfeito
do indicativo, enquanto que o segundo perodo do
mesmo texto apresenta um verbo no pretrito
imperfeito tambm do indicativo.
02. Nas formas verbais fizesse e cantava temos
como desinncia modo-temporal, respectivamente
sse e ava.
04. No Texto 1, as palavras verde e abandono esto
funcionando como substantivos.
08. Em deram ento de me chamar temos dois
ditongos nasais.
16. No Texto 2 as formas verbais e h tm sua
acentuao justificada pela mesma regra do verbo
pr, uma vez que os trs vocbulos so
monosslabos tnicos.
32. Em Severino e Maria temos i como vogal,
enquanto que na segunda slaba da forma verbal
fiquei esse se classifica como semivogal, j que
integra um tritongo.


2. Levando em considerao os versos de Vincius de
Moraes transcritos a seguir, assinale a(s) alternativa(s)
verdadeira(s).














01. No adjetivo maior, constante no primeiro verso
da primeira estrofe encontramos, temos um
ditongo oral decrescente seguido de um hiato.
02. Caso a passagem E se v descontente fosse
passada para o plural, teramos E se vm
descontentes.
04. No verso Fiel sua lei de cada instante, o
temos a presena de um complemento nominal.
08. A forma verbal sublinhada no texto pode ser
substituda, mantendo-se o sentido do verso e o
respeito pela norma culta, por se volta.

3. Observe o incio de uma das crnicas de Lus
Fernando Verssimo:

HBITO NACIONAL
Por uma destas coincidncias fatais, vrias
personalidades brasileiras, entre civis e militares, esto
no avio que comea cair. No h possibilidade de se
salvarem. (...) Nos poucos instantes que lhes restam de
vida, todos rezam, confessam seus pecados, em
verses resumidas e entregam suas almas
providncia Divina. O avio se espatifa no cho.
So Pedro os recebe de cara amarrada. O porta-
voz do grupo se adianta e, j esperando o pior, comea
a explicar quem so e de onde vm.
(...)

Agora, responda:
a) Caso o circunflexo fosse omitido na forma
verbal sublinhada no texto, teramos uma
variao de sentido na interpretao do
perodo? Justifique sua resposta:



O vento norte gelara o seu N, secara o
verde de suas folhas. Estava seco, frio, duro, ao
abandono, acabado para sempre. No havia
primavera ou sol de primavera que fizesse brotar
outra vez o N da beira do mar, o que cantava e
amava como um filho de Deus.
REGO, Jos Lins do.
Riacho Doce

O meu nome Severino, / No tenho
outro de pia. / Como h muitos Severinos, / que
santo de romaria, / deram ento de me chamar /
Severino de Maria; / como h muitos Severinos /
com mes chamadas Maria, / fiquei sendo o da
Maria / do finado Zacarias.
MELO NETO, Joo Cabral de.
Morte e vida Severina.

Maior amor nem mais estranho existe
Que o meu, que no sossega a coisa amada
E, quando a sente alegre, fica triste
E se v descontente, d risada.
(...)
Fiel sua lei de cada instante
Desassombrado, doido, delirante
Numa paixo de tudo e de si mesmo.

(Soneto do Amor Maior
Antologia Potica)

Please purchase PDF Split-Merge on www.verypdf.com to remove this watermark.
Incluso para a Vida Gramtica

Pr-Vestibular da UFSC

19
Tarefa Complementar #

TEXTO 1
1



5




10




15




20




A violncia social vem sendo praticada no
mundo inteiro, em todas as classes da sociedade.
uma luta pelo poder e pela sobrevivncia. Uns
matam e roubam para sobreviver, para conseguir um
pedao de po e ter com que se alimentarem. Outros
praticam a violncia apenas como objetivo de
enriquecerem ainda mais e dominarem a classe mais
fraca.
H rivalidades at em famlia. H filhos
matando pais para tomarem o que lhes pertence;
irmos brigando entre si. H brigas por posses de
terras, causando guerras entre pases.
Por qualquer motivo se pratica violncia, seja
uma simples discusso, cime, um lugar em uma
fila... Ningum respeita o prximo.
Qualquer coisa, por menor que seja, serve de
motivo para acabar em violncia. Basta andarmos uns
minutos pela rua para encontrarmos vrios exemplos
disso.
Mas o pior de tudo que no h apenas os que
praticam violncia pelo poder e pela sobrevivncia.
H tambm muitos que a praticam por prazer, por
quererem simplesmente mostrar que podem mais que
os outros.
(Redaes e Textualidade
Editora Martins Fontes)


1. No primeiro pargrafo, afirma-se que a violncia
praticada no mundo inteiro. Tal afirmao nos leva a
crer que o texto dar exemplos claros da realidade de
diferentes pases, citando, inclusive, seus nomes. Tais
exemplos:
a) so verificados no segundo pargrafo.
b) so verificados no terceiro pargrafo.
c) so verificados no quarto pargrafo.
d) so verificados em todos os pargrafos.
e) no so evidenciados em todo o decorrer do texto.

2. De modo geral, o autor procura explicar o
fenmeno da violncia, valendo-se de argumentos
sociais. E:
a) isso fica comprovado na concluso lida.
b) fica claro que o motivo apresentado pelo autor
para tanta violncia o desrespeito pelo prximo
c) j na introduo, fica claro que a maioria das
pessoas que roubam ou matam fazem isso por mero
prazer.
e) fica claro que a inveja o primeiro passo para
algum se tornar violento.
e) evidencia-se que a busca pela sobrevivncia o
motivo apresentado pelo autor para tanta violncia.

3. No pargrafo de concluso, o autor retoma a idia
central do texto, exposta nos primeiros pargrafos.
Qual seria essa idia central?
a) a busca por uma melhor posio econmica no tem
limites.
b) a tentativa de mudar de classe social enfrenta tudo e
todos.
c) a inconseqncia dos assaltantes faz mais vtimas
do que o esperado.
d) a falta de preparo dos policiais dificulta a ao dos
mesmos.
e) a violncia fruto da luta pelo poder e pela
sobrevivncia.

4. Alm de retomar a idia central do texto, a
concluso tambm apresenta uma nova idia, ou seja,
que a violncia:
a) pode ser praticada por simples prazer.
b) algo que jamais venceremos.
c) algo cujo incio no se conhece.
d) algo natural em todo e qualquer ser humano.
e) o que de pior se pode praticar.

5. Todos os versos a seguir so de Carlos Drummond
de Andrade. Assinale a(s) alternativa(s) em que o
poeta expressa uma ideia sobre o sentimento amoroso
com tom de humor:
01. Carlos, sossegue, o amor
isso que voc est vendo:
hoje beija, amanh no beija,
depois de amanh domingo
e segunda-feira ningum sabe
o que ser.

02. Amor primo da morte
E da morte vencedor
Por mais que o matem (e matam)
A cada instante de amor.

03. Porque o amor resultou intil.
E os olhos no choram.
E as mos tecem apenas o rude trabalho.
E o corao est seco.

08. A vontade de amar que me paralisa o trabalho,
vem de Itabira,
de suas noites brancas,
sem mulheres e sem horizontes.

16. Joo amava Teresa que amava Raimundo
que amava Maria que amava Joaquim
que amava Lili
que no amava ningum.

CLASSIFICAO VERBAL

Quanto flexo, os verbos classificam-se em:
a) Regulares
b) Irregulares
c) Anmalos
d) Defectivos
e) Abundantes

UNIDADE 10
Please purchase PDF Split-Merge on www.verypdf.com to remove this watermark.
Incluso para a Vida Gramtica

Pr-Vestibular da UFSC

20
Exerccios de Sala #

1. Complete a tabela abaixo com as formas do particpio.
Ateno: nem todos existem!!!
Infinitivo Particpio
Irregular
(V.L.)
Ex.: foi, est
Particpio
Regular
(ter / haver)
Ex.: havia, tinha
Aceitar
Acender
Benzer
Eleger
Envolver
Enxugar
Expulsar
Fritar
Ganhar
Gastar
Imprimir
Isentar
Limpar
Matar
Morrer
Pagar
Pegar
Prender
Salvar
Soltar
Escrever

2. Preencha as lacunas com as formas do imperativo.
a) ___________________ teu erro. (admitir)
b) No nos ___________________, rapazes!
(precipitar)
c) No ____________________ sem pensar em vossos
pais! (responder)
d) __________________ bons textos para o nosso
jornal. (produzir; ns)
e) Jamais __________________ tanto de ti mesma.
(exigir)
f) Quando fores cidade, ____________ a carta no
correio. (pr)
g) __________________, eu te espero! (vir)
h) _________________ o vosso dever e no
__________________ nos assuntos alheios. (cumprir /
intervir)
3. Assinale a opo cujo espao pode ser preenchido pela
palavra entre parnteses.
a) A criana havia _____ (pegado) o revlver da
cmoda.
b) O comerciante tinha _____ (pagado) sua dvida no
prazo.
c) O governo havia _____ (aceito) o pagamento em
moeda estrangeira.
d) Era _____ (matado) todo dia um marginal naquela
vila.
e) A mulher tem _____ (gastado) mais do que ganha,
apenas em suprfluos.
4. Quanto ao uso de tempos e modos verbais, identifique
a(s) proposio(es) correta(s).
01. Costumamos beijarmos nossos amigos.
02. Eu sempre expludo quando fico nervoso.
04. Tenho pegado alunos colando.
08. No faas o que lhe mandam fazer.
16. Quando eu pr a mo em voc, vou lhe dar um
conselho no ouvido.

5. Assinale a(s) alternativa(s) em que est incorreto o
uso do verbo no particpio.
01. Longa vela foi acendida para que se pudesse fazer a
prova com sucesso.
02. Todos ficaram isentos? Quem que o fiscal havia
isentado?
04. A vela era benta?
08. A prova est sendo muito bem aceita.
16. Os reprteres j haviam chego uma hora antes do
presidente se pronunciar.

Tarefa Mnima #

1. Observe as formas verbais destacadas.
1. Quando ele vir o presente, ficar emocionado.
2. Quando ele vier, receber o presente.
3. Ver o presente ser emocionante.
4. Vir e receber o presente ser emocionante.
Agora, assinale a alternativa correta quanto forma
verbal contida nas sentenas acima:
a) Na frase 1: infinitivo do verbo vir.
b) Na frase 1: futuro do subjuntivo do verbo ver.
c) Na frase 2: futuro do subjuntivo do verbo ver.
d) Na frase 3: infinitivo do verbo vir.
e) Na frase 4: infinitivo do verbo ver.

2. Observe os seguintes slogans e indique a(s)
alternativa(s) correta(s) quanto conjugao verbal:
01. Pega bem fumar Dallas. Pega o seu.
02. Taffman E. Se voc no gosta, continua no milk-
shake.
04. Dize-me com quem andas e te direi quem s.
(anncio da Antrctica)
08. Vem pr a Caixa voc tambm.

3. Assinale apenas as frases em que o emprego do
particpio esteja correto.
01. Sonhei que eu tinha morrido de tdio.
02. Eles tm gasto mais do que ganham.
04. A campanha da cidadania tem salvo muita gente.
08. A me havia frito os peixes que o menino pescara.

4. Observe:
Da Discrio
No te abras com teu amigo
Que ele um outro amigo tem.
E o amigo de teu amigo
Possui amigos tambm...
(Mrio Quintana)

Passando o primeiro verso para o imperativo afirmativo,
qual seria o correto (conservando a pessoa gramatical)?
a) Abre-te com seu amigo.
Please purchase PDF Split-Merge on www.verypdf.com to remove this watermark.
Incluso para a Vida Gramtica

Pr-Vestibular da UFSC

21
b) Abra-se com seu amigo.
c) Abri-vos com vosso amigo.
d) Abre-te com teu amigo.
e) Abra-te com teu amigo.

Tarefa Complementar #

1. "No tenhas cime de tua mulher, para que ela no
venha a enganar-te com a malcia que aprender de ti."
(Machado de Assis: Dom Casmurro)
Dispensando ao interlocutor o tratamento voc ao
interlocutor do pargrafo lido, teramos as seguintes
alteraes:
a) tenha, sua, enganar-lhe, de voc
b) te, sua, engan-lo, de si
c) tenha, tua, engan-lo, de voc
d) tenha, sua, engan-lo, de voc
e) tm, sua, engan-lo, de si

2. Assinale o item em que as formas dos verbos trazer,
ser, pr e ir correspondam ao seguinte exemplo:
Preferir, prefere!
a) Tragas! - Sejas! - Ponhas! - Vs!
b) Trazei! - Sede! - Pondes! - Ide!
c) Traze! - Se! - Ponha! - Vais!
d) Traz! - S! - Pe! - Vai!
e) Traga! - Seja! - Ponha! - V!

3. Assinale a alternativa na qual o infinitivo no deve
estar flexionado.
a) Aquele espertalho diz no existirem barreiras para
ele.
b) Para cumprires tua obrigao, precisas andar depressa.
c) Pareciam se queixarem apesar da vida cmoda que
levavam.
d) bom estudar para sermos bem sucedidos na vida.
e) Via sarem pequenas naves do fundo do lago.

4. Transpondo para a voz ativa a frase O processo deve
ser revisto pelos dois funcionrios, obtm-se a forma
verbal:
a) deve-se rever
b) ser revisto
c) devem rever
d) revero
e) rever-se-

5. Marque a(s) alternativa(s) correta(s), quanto ao
emprego dos tempos verbais:
01. Quando acharmos o garoto, passar nossa angstia.
02. Se Marta concordasse, eu vendo a casa.
04. Se ele fosse encontrado vivo, todos ficaramos
aliviados.
08. Que dor o destino nos impusera!
16. Talvez seje ele o responsvel por toda esta confuso.

6. Complete as lacunas com o particpio passado dos
verbos entre parnteses:
1. A cozinheira havia _____ peixe ( fritar)
2. O brinquedo estava _____ no lago.(imergir)
3. Minhas roupas esto ____, Anglica. (enxugar)
4. Anglia havia _____ minhas roupas a ferro. (enxugar)
5. A luz da vela tinha, aos poucos, _______ (extinguir-
se)
Agora, indique a(s) opo(es) que completa (m),
corretamente, as lacunas :
01. frito, imergido, enxugadas, enxugado, extinguida
02. frito, imergido, enxugadas, enxutas, extinto.
04. frito, imerso, enxutas, enxuto, extinto.
08. fritado, imerso, enxugadas, enxugado, extinta.
16. fritado, imerso, enxutas, enxugado, extinguido-se.
32. fritado, imerso, enxutas, enxugado, se extinguido-se.

UNIDADE 11

REGNCIA

Primeiro recorde as PREPOSIES:





Cuidado com o sentido atribudo ao verbo!
1. Aspirar = sorver
Aspirar a = desejar
2. Assistir = ajudar
Assistir a = presenciar
Assistir em = residir
3. Gostar = provar
Gostar de = aprovar
4. Querer = desejar
Querer a = estimar
5. Proceder = agir
Proceder = vir de
Proceder a = realizar
6. Agradar = fazer carinho
Agradar a = contentar
7. Visar = pr visto
Visar = mirar
Visar a = almejar
8. Implicar = acarretar
Implicar com = no simpatizar
Implicar-se em = envolver-se

Fique de olho!






Os verbos NAMORAR e CONVIDAR so
sempre VTD!
A, ANTE, AT, APS, COM, CONTRA, DE,
DESDE, EM, ENTRE, PARA, PER, PERANTE,
POR, SEM, SOB, SOBRE, TRS
Please purchase PDF Split-Merge on www.verypdf.com to remove this watermark.
Incluso para a Vida Gramtica

Pr-Vestibular da UFSC

22

Em compensao...
So VTI:




Tenha mais cuidado ainda com:







1. Preferir (VTDI)
Errado: Prefiro Exatas do que Humanas.
Certo: Prefiro Exatas a Humanas.
2. Lembrar (VTD)
Lembrar-se (VTI)
Compare: No lembro seu nome.
No me lembro de seu nome.
3. Recordar (VTD)
Recordar-se (VTI)
Compare: No recordo seu nome.
No me recordo de seu nome.
4. Esquecer (VTD)
Esquecer-se (VTI)
Compare: Nunca esqueci seu nome.
Nunca me esqueci de seu nome.
5. Pagar (VTDI)
Pense assim:
Pagamos algo a algum.
Ou...
Pagamos algum por algo.
6. Perdoar
Pense assim:
Perdoamos algum por algo.
Ou...
Perdoamos algo a algum.
7. Agradecer
Pense assim:
Agradecemos algo a algum.
Ou...
Agradecemos algum por algo que tenha feito.
8. Informar
Pense assim:
Informamos algo a algum.
Ou...
Informamos algum sobre algo.

Exerccios de Sala #

1. Reescreva as frases, substituindo os verbos
sublinhados pelos constantes entre parnteses,
obedecendo regncia e conjugao verbal.
1. Limpe a poeira de cima do mvel. (aspirar)
2. Ambicionam o ttulo. (aspirar)
3. Desejo a felicidade. (aspirar)
4. Vi o espetculo. (assistir)
5. Presenciei a briga. (assistir)
6. Ele segue as leis. (obedecer)
7. Visitem a Bahia. (Ir)
8. O gerente ps visto no cheque. (visar)
9. O atirador mirou o alvo. (visar)
10. Moro em Blumenau. (assistir)

Tarefa Mnima #

1. Indique a(s) proposio(es) em que as lacunas so
corretamente preenchidas na sequncia pelas
preposies colocadas entre parnteses.
01. Pareci muito aflita ______ a mudana inexplicvel,
porque no me encontrava apta ____ nova situao
nem sou imune _____ choques emocionais. (com, ,
a)
02. Confesso-te que estou ansiosa _____ ver-te ____
posio mais tranquila. (de, por)
04. Relativamente _____ essa questo, penso ____ que
ela no pertinente ___ pesca artesanal.
(, de, )
08. A causa da demora est apenas ____ averso que
sentimos ___ uma misso ____ qual pouco
acreditamos. (na, por, na )
16. O ministro h de convir ____ que essa medida
implicar _____ concurso de novos servidores
habilitados ____ tarefa. (de, no, )

2. Complete os espaos em branco da frase com
elementos de uma das alternativas dadas abaixo.
No nos referimos ___________ estava aqui, mas sim
__________ aluna __________ tu falaste ontem.
a) a que - - a quem
b) aquela que - a - quem
c) quela que - - sobre quem
d) que - a - de quem
e) sobre quem - a - a respeito de quem

3. Assinale a alternativa cuja frase foi escrita
respeitando-se a regncia:
a) A apostila que preciso a de Literatura.
b) Perdi o nibus e, por isso, tive que ir de p at o
centro.
c) Em minha redao, fiz uma referncia quilo que
lemos na crnica de Verssimo.
d) Tenho certeza que se sair bem na apresentao!
e) Infelizmente o apartamento que gostamos era muito
caro.

Tarefa Complementar #

1. H o respeito s regras de regncia verbal, exceto em:
01. Sempre aspirei a uma vida de luxo.
02. Talvez jamais nos esqueamos disso.
04. Obedec-la necessrio, ainda que voc no goste
disso.
08. Senhor Lus do Amaral, residente rua Pedro
Michel (...).
16. Que toda causa implica uma conseqncia, no
novidade nem para mim nem para voc.
32. Na minha infncia, sempre preferia brincadeiras
tradicionais do que brinquedos com controle remoto.

1. Responder a
2. Obedecer a
3. Desobedecer a
4. Chegar a
5. Ir a
6. Vir a
7. Referir-se a
8. Morar em
9. Residir em
10. Situar-se em

Please purchase PDF Split-Merge on www.verypdf.com to remove this watermark.
Incluso para a Vida Gramtica

Pr-Vestibular da UFSC

23
2. Indique a(s) alternativa(s) em que h o respeito s
regras de regncia verbal:
01. Talvez ele jamais tenha aspirado a uma ascenso
profissional; por isso nunca pde sentir o gosto de
uma vitria como a minha.
02. Hoje os jovens preferem relacionamentos breves a
terem que assumir um namoro mais srio.
04. Em um passado no muito longnquo, os pais eram a
autoridade mxima em casa. Os filhos jamais
ousavam desobedec-los.
08. Nossa nova filial est situada rua Getlio Vargas.
16. Talvez ele namore com Clara, mesmo no lhe
amando.
32. Eu jamais me esqueceria de que a nossa formatura foi
realizada aqui.

Texto 1
Um operrio desenrola o arame, o outro o endireita, um
terceiro corta, um quarto o afia nas pontas para a
colocao da cabea do alfinete; para fazer a cabea do
alfinete, requerem-se trs ou quatro operaes
diferentes.

Texto 2


3. A respeito do Texto 1 e do quadrinho /so correta(s)
a(s) afirmao(es):
01. Ambos retratam a intensa diviso do trabalho, qual
so submetidos os operrios.
02. O vocbulo montagem possui oito letras e seis
fonemas.
04. O termo o constante em o endireita e o afia
corresponde a um pronome e resgata o sujeito
operrio.
08. O Texto 1 refere-se produo informatizada; o
quadrinho, produo artesanal.
16. Ambos contm a ideia de que o produto da atividade
industrial no depende do conhecimento de todo o
processo por parte do operrio.
32. O numeral quatro apresenta, na ordem, um dgrafo
e um encontro consonantal perfeito.
64. Segundo o exposto na charge, pode-se dizer que o
operrio Aspira ao entendimento de algo..

4. Indique a(s) proposio(es) correta(s) quanto
regncia verbal:
01. Sabemos que tal atitude poder implicar em uma
manifestao de nossos empregados.
02. Dizer que prefiro ver jogo pela televiso do que ao
vivo fcil. O difcil convencer a mim mesmo de
que estou certo.
04. necessrio que obedeamos a todas as instrues
que nos foram passadas.
08. Talvez ele tenha se esquecido que nos prometeu
apoio.
16. Ao que parece, ele implicou-se em mais um acidente
de trnsito.
32. Fiquei assistindo quele documentrio durante uma
hora e meia.

UNIDADE 12

CRASE

NO posso usar crase
I - diante de:
a) palavras masculinas
1

b) verbos
c) artigo indefinido
d) pronomes
2

e) numerais cardinais
3


II - Entre palavras repetidas

III - Com a no singular seguido de palavra no plural






No posso me esquecer da crase FACULTATIVA
diante de:
1 - nomes prprios femininos
2 - pronomes possessivos femininos
3 - pronomes de tratamento femininos

Tambm devo me lembrar dos CASOS ESPECIAIS que
pedem crase...

I - especificando CASA, TERRA e DISTNCIA.

II - permitindo a troca:
quele(s) = a este (s)
quela(s) = a esta (s)
quele = a este

Exerccios de Sala #

1. H crase ou no?
a) Andei a p.
b) Estou apto a discutir.
c) Dirigiu-se a voc com aspereza.
d) Dirigiu-se a algum com aspereza.
e) Dirigiu-se a minha me.
f) Estavam cara a cara.
g) Fiz uma viagem a Roma.
h) Fiz uma viagem a Roma Antiga.
i) No se refere a pessoas estranhas.
j) No se refere as pessoas estranhas.
k) Cheguei a uma hora da tarde.
l) Cheguei a uma hora qualquer.
m) Usa ternos a Clodovil.
n) Chegou a tarde, o garoto.
Ateno com as excees!
1
2
3
Please purchase PDF Split-Merge on www.verypdf.com to remove this watermark.
Incluso para a Vida Gramtica

Pr-Vestibular da UFSC

24
o) A tarde chegou logo.
p) A medida que andvamos, sentamos um calafrio.
q) Estamos a beira do caos.
r) Voltei a casa tarde.
s) Voltei a casa de meus pais.
t) Jogou o disco a distncia de 10 metros.
u) Refiro-me aquele garoto.
v) Pegue aquele lpis ali no cho.
w) Dirigiu-se aquelas senhoras.
x) Comprou aquilo no Mercado Pblico.
y) Voc fez as compras a vista ou a prazo?
z) A garota cheirava a leo.

2. Que diferena de sentido existe entre as frases?
a) Saiu do salo francesa.
Saiu do salo a francesa.
b) Desenham chinesa.
Desenham a chinesa.
c) Ele cheira tinta.
Ele cheira a tinta.

Tarefa Mnima #

1. Assinale a alternativa correta quanto crase.
a) A atrao educa quem no teve acesso outras
informaes.
b) A atrao educa quem no teve acesso umas
informaes.
c) A atrao educa quem no teve acesso s informaes.
d) A atrao educa quem no teve acesso essas
informaes.
e) A atrao educa quem no teve acesso algumas
informaes.

2. Assinale a alternativa que preencha os espaos
corretamente na respectiva ordem.
Entreguei a carta .... homem .... quem voc
se referiu .... tempos.
a) aquele - -
b) quele - - h
c) aquele - a - a
d) quele - -
e) quele - a - h

3. Marque a(s) afirmativa(s) verdadeira(s) quanto ao
emprego da crase e d o valor total.
01. A atividade que ora realizas semelhante que
realizei em 1989.
02. Infelizmente, os preos continuam a subir.
04. tardinha, todos vo apreciar o pr-do-sol na Ilha da
Magia.
08. s vezes, no podemos ser honestos porque no
deixam.
16. O pintor cuja tela te referes fez uma exposio na
galeria de artes da UFSC.
32. O sujeito recorreu repartio e a ela se apegou
como uma tbua de salvao.

4. Assinale a alternativa correta quanto ao emprego da
crase.
a) Obedea sinalizao, o que pedem as placas nas
rodovias.
b) Telefonei ela, depois voc e a todos os nossos
amigos.
c) Vesti-me as pressas e sa procura de meus amigos.
d) Tenho um carro lcool e outro gasolina.
e) No me refiro a nenhuma dessas alternativas.

5. Assinale a alternativa correta quanto ao uso da crase.
a) Ela conhece lei.
b) Tu costumas andar p?
c) Dirigiu-se ela sem pensar.
d) O advogado se mostrou disposto rever o processo.
e) Foram Itlia.

6. Complete os espaos em branco da frase com
elementos de uma das alternativas dadas abaixo.
No nos referimos ___________ estava aqui,
mas sim __________ aluna __________ tu falaste
ontem.
a) a que - - a quem
b) aquela que - a - quem
c) quela que - - sobre quem
d) que - a - de quem
e) sobre quem - a - a respeito de quem

Tarefa Complementar #

1. Assinale a alternativa que completa a frase
corretamente, obedecendo ordem apresentada.
____ noitinha, chegava o bando ____ porta da casa-
grande.
Vinha Antnio Silvino na frente; os seus doze homens
____ distncia.
Subiu ____ calada como um chefe, apertou ____ mo
de meu av com um sorriso na boca.
a) A - - a - -
b) - - - a - a
c) - - a - - a
d) - a - - a -
e) A - a - a - - .

2. Em todas as frases deve ser utilizado o acento
indicativo de crase exceto em:
a) preciso resistir a violncia.
b) Nem sempre se sobrevive a violncia.
c) A dor do agredido sucede a violncia.
d) necessrio desaprovar a violncia.
e) n.d.a.

3. Assinale a alternativa que preencha, pela ordem, as
lacunas corretamente.
"____ seis horas da manh, j estvamos ____ esperar
o trem que nos levaria ____ cidadezinha, de onde
iramos, ____ cavalo, ____ fazenda do Sr. Juca."
a) As - - a - -
b) s - a - - - a
c) As - a - - a -
d) s - a - - a -
e) As - - - a - a.

4. Identifique a(s) proposio(es) correta(s):
01. No desobedea a ningum nem mesmo quele que voc no
admira.
Please purchase PDF Split-Merge on www.verypdf.com to remove this watermark.
Incluso para a Vida Gramtica

Pr-Vestibular da UFSC

25
02. No quero por isso no bolso, por isso, levem isso daqui.
04. Quero que eles dem tudo o que tm aos que nada tm.
08. Esta caneta semelhante que comprei ontem.
16. Foram a Braslia falar com os ministros.
32. Essa pea teatral destina-se a pessoas que tm o sentido do
humor.

5. Analise os versos a seguir:
de fugir de mim e me trazer de volta
casa a qualquer hora num fechar
de pginas?
Nos versos acima, a presena do acento grave revela
respeito s regras de:
a) colocao pronominal.
b) concordncia nominal.
c) regncia verbal.
d) regncia nominal.
e) concordncia verbal.

6. Leia o texto seguinte:
Antes de comear a aula matria e exerccios no
quadro, como muita gente entende , o
mestre sempre declamava um poema e fazia vibrar sua
alma de tanta empolgao e os alunos
ficavam admirados. Com a sutileza de um sbio foi nos
ensinando a linguagem potica mesclada
ao ritmo, melodia e prpria sensibilidade artstica.
Um verdadeiro deleite para o esprito, uma
sensao de paz, harmonia.
OSRIO, T. Meu querido professor. Jornal Vale Paraibano,
15/10/1999.
a) Qual a interpretao que pode ser dada ausncia da
crase no trecho a prpria sensibilidade artstica?
b) Qual seria a interpretao caso houvesse a crase?

UNIDADE 13

COLOCAO PRONOMINAL

PRONOMES E COLOCAO PRONOMINAL
Entre eu e tu ou entre mim e ti?
Para eu fazer ou para mim fazer?
Para ti decidir ou para tu decidires?









Colocao Pronominal
a) Prclise: posicionar o pronome antes do verbo.
Ex.: No te conheo muito bem.
b) Mesclise: o pronome se intromete no verbo...
Ex.: Algum dia, conhecer-te-ei por completo.
c) nclise: o pronome fica aps o verbo (Sempre com
hfen!).
Ex.: Conheo-te muito bem, meu amigo!

Regrinhas Bsicas
- Jamais devemos iniciar uma orao com esse tipo de
pronome.
- A mesclise s deve ocorrer em incio de oraes ou
aps vrgulas e com verbos no Futuro do Presente ou do
Pretrito.
- A prclise obrigatria com:
. vocbulos indicativos de negao (no, nunca, jamais,
nem, tampouco);
. a palavra que (independentemente de sua classe
gramatical);
. pronomes indefinidos (tudo, nada, ningum, algum...);
. conjunes subordinativas (se, caso, embora...);
. pronomes relativos (que, que, qual, cujo...);
. em + gerndio (Ex.: Em se tratando de...);
. oraes interrogativas, exclamativas ou optativas.
- Ento, nos demais casos... D-lhe nclise!!!
Mas fique atento ao pronome que ir empregar,
principalmente se estiver trabalhando com a terceira
pessoa!



VI (=Verbo Intransitivo)
no pede objeto
VTD (= Verbo Transitivo Direto)
pede OD
VTI ( Verbo Transitivo Indireto)
pede OI
VTDI (=Verbo Transitivo Direto e Indireto)
pede OD e OI simultaneamente
Pronomes tambm podem servir de complementos
aos verbos:
QUANDO A FORMA USAMOS O(S)
VERBAL TERMINA EM: PRONOME(S)

VOGAL o, a, os, as
R, S, Z lo, la, los, las
O, E, M no, na, nos, nas

- x - lhe, lhes



Exerccios de Sala #

1. A frase Eu queria encontrar ela ainda hoje. no
respeita as regrinhas de nossa amada gramtica de
Lngua Portuguesa. Por qu?

2. Como reescrever a frase citada na questo anterior, de
modo a seguir a norma culta?

3. E como inserir o pronome nos no perodo
Conhecemos naquela festa., respeitando a Gramtica
Compare:
Para mim, passar no vestiba moleza!
Para eu passar no vestiba, ser moleza!
ME TE SE O A LHE
NOS VOS OS AS LHES
Please purchase PDF Split-Merge on www.verypdf.com to remove this watermark.
Incluso para a Vida Gramtica

Pr-Vestibular da UFSC

26
Normativa? Vou convidar todos para minha
formatura!

4. Seria correto substituirmos o termo sublinhado pelo
pronome lhes? Justifique sua resposta:

5. Como reescrever tal perodo, fazendo a devida
colocao pronominal?

Exerccios de Sala #

1. A frase Eu queria encontrar ela ainda hoje. no
respeita as regrinhas de nossa amada gramtica de
Lngua Portuguesa. Por qu?

2. Como reescrever a frase citada na questo anterior, de
modo a seguir a norma culta?

3. E como inserir o pronome nos no perodo
Conhecemos naquela festa., respeitando a Gramtica
Normativa?Vou convidar todos para minha
formatura!

4. Seria correto substituirmos o termo sublinhado pelo
pronome lhes? Justifique sua resposta:

5. Como reescrever tal perodo, fazendo a devida
colocao pronominal?

Tarefa Mnima #



1. Observando os trs primeiros quadrinhos, pode-se
perceber que, no dilogo entre Calvin e sua me, uma das
formas verbais no condiz com as demais. Trata-se de:
a) Ides.
b) Pretendes.
c) Tenhais.
d) Segui.
e) Julgais.

2. Indique o que seja correto afirmar acerca da tira
acima:
01. O pronome empregado logo na abertura da segunda
fala da Calvin apresenta um erro de acentuao, j que,
sendo tono, no deveria receber acento grfico.
02. O termo onde (ltimo quadrinho) pode ser
substitudo por o qual, mantendo-se o sentido da
sentena.
04. O pronome destacado em Vs me julgais mal
exerce funo de objeto direto.
08. Reescrevendo o perodo Segui vosso caminho!, de
modo a introduzir nele um pronome tono, teramos
Segui-vos!.
16. Em Vs me julgais mal, o ltimo vocbulo
empregado indicativo de modo.

3. Nos quadrinhos, acontece um erro quanto norma
culta da Lngua. Identifique-o:


a) Falta vrgula depois do vocativo.
b) A personagem mistura, na sua fala, a 2 pessoa do
singular com a 3 pessoa do singular.
c) H erro de grafia ao reproduzir as falas coloquiais das
personagens. Exemplos: T, pra.
d) obrigatrio o uso de nclise no ltimo quadrinho.
e) H pontos de exclamao e interrogao demais nos
trechos.

Tarefa Complementar #

Texto 1
O homem disse que tinha de ir embora antes queria
me ensinar uma coisa muito importante:
- Voc quer conhecer o segredo de ser um menino
feliz para o resto da vida.
- Quero respondi.
O segredo se resumia em trs palavras, que ele
pronunciou com intensidade, mos nos meus ombros e
olhos nos meus olhos:
- Pense nos outros.
Na hora, achei esse segredo meio sem graa. S bem
mais tarde vim a entender o conselho que tantas vezes na
vida deixei de cumprir. Mas que sempre deu certo
quando me lembrei de segui-lo, fazendo-me feliz como
um menino. SABINO, Fernando. O menino no espelho.

1. Em relao ao Texto 1, correto afirmar que:
01. o pronome encltico empregado no trecho quando
me lembrei de segui-lo, exerce funo de objeto
direto e resgata o substantivo conselho.
02. o trecho quando me lembrei de segui-lo, pode ser
substitudo por quando lembrei de segui-lo, sem
desrespeitar a norma culta.
04. a palavra que, nas trs ocorrncias sublinhadas no
texto, est funcionando como pronome relativo, pois,
ao mesmo tempo em que liga oraes, tambm aponta
para um antecedente.
Please purchase PDF Split-Merge on www.verypdf.com to remove this watermark.
Incluso para a Vida Gramtica

Pr-Vestibular da UFSC

27
08. em como um menino temos o conectivo como
dando idia de conformidade.
16. a nclise constante em fazendo-me feliz como um
menino facultativa, uma vez que alterar a
colocao pronominal para qualquer outra posio,
tambm estaria de acordo com a norma culta.
32. Em Na hora, achei esse segredo meio sem graa.
temos meio como um vocbulo invarivel, por se
tratar de um advrbio.

2. (ENEM) A conversa entre Mafalda e seus amigos:












a) revela a real dificuldade de entendimento entre
posies que pareciam convergir.
b) desvaloriza a diversidade social e cultural e a
capacidade de entendimento e respeito entre as
pessoas.
c) expressa o predomnio de uma forma de pensar e a
possibilidade de entendimento entre posies
divergentes.
d) ilustra a possibilidade de entendimento e de respeito
entre as pessoas a partir do debate poltico de idias.
e) mostra a preponderncia do ponto de vista masculino
nas discusses polticas para superar divergncias.

3. (UDESC) Assinale a alternativa em que o pronome
oblquo lhe est no lugar do pronome oblquo o ou a, em
desacordo com as orientaes da norma culta.
a) Pediu a Rita que lhe explicasse tudo.
b) Ento ela declarou-lhe que no voltaria mais.
c) O cocheiro props-lhe voltar a primeira travessa, e ir
por outro caminho.
d) Ele, para lhe ser agradvel, estava sempre
discretamente afastado.
e) Vejamos o que lhe trouxe aqui.

4. Identifique a(s) alternativa(s) onde o trecho transcrito a
seguir foi reescrito, mantendo-se o sentido original:
Era a primeira vez que me encantavam assim
aquelas gradaes de verde, o verde-negro, de faiana,
luzente da hera, o verde flutuante mais claro dos
bambus.
01. Era a primeira vez que aquelas gradaes de verde
me deixavam assim encantado, o verde-negro, de
faiana, luzente da hera, o verde flutuante mais claro
dos bambus.
02. Pela derradeira vez, senti-me encantado por aquelas
gradaes de verde, o verde-negro, de faiana, luzente
da hera, o verde flutuante mais claro dos bambus.
04. Aquelas gradaes de verde, pela primeira vez, me
encantariam assim. Seria o verde-negro, o verde da
faiana, luzente da hera, assim como o verde flutuante
mais claro dos bambus.
08. Assim, aquelas gradaes de verde o verde-negro,
de faiana, luzente da hera, o verde flutuante mais
claro dos bambus me encantavam pela primeira vez.
16. Aquelas gradaes de verde me encantavam deste
modo pela primeira vez; o verde-negro, de faiana,
luzente da hera, o verde flutuante mais claro dos
bambus.
32. Era a primeira vez que aquelas gradaes de verde
me encantavam desta maneira, o verde-negro, de
faiana,luzente da hera, o flutuante mais claro dos
bambus.
64. E, assim, aquelas gradaes de verde o negro, de
faiana, luzente da era, o flutuante mais claro dos
bambus poderiam encantar-me.

UNIDADE 14
ESTUDO DE TEXTOS

Exerccios de Sala #

TEXTO1
SPLICA CEARENSE (O Rappa)
Oh! Deus,/perdoe esse pobre coitado,
que de joelhos rezou um bocado,
pedindo pra chuva cair,/cair sem parar.
Oh! Deus,/ser que o senhor se zangou,
e s por isso que o sol se arretirou,
fazendo cair toda chuva que h.
Oh! Senhor,/pedi para o sol se esconder um pouquinho,
pedi para chover,/mas chover de mansinho,
para ver se nascia uma planta,
uma planta no cho.
Oh! Meu Deus,/se eu no rezei direito,
a culpa do sujeito,
desse pobre que nem sabe fazer a orao.
Meu Deus,/perdoe encher meus olhos d'gua,
e ter-lhe pedido cheio de mgoa,
para o sol inclemente,/se arretirar, retirar.
Desculpe, pedir a toda hora,
para chegar o inverno e agora,
o inferno queima o meu humilde Cear.
Oh! Senhor,/pedi para o sol se esconder um pouquinho,
pedi para chover,/mas chover de mansinho,
para ver se nascia uma planta no cho,
planta no cho.
Violncia demais,/chuva no tem mais,
corrupto demais,/poltica demais,
tristeza demais.
O interesse tem demais!
Violncia demais,/fome demais,
falta demais,/promessa demais,
seca demais,/chuva no tem mais!
L no cu demais,/chuva tem,
tem, tem, no tem,/no pode tem,
demais.

Please purchase PDF Split-Merge on www.verypdf.com to remove this watermark.
Incluso para a Vida Gramtica

Pr-Vestibular da UFSC

28
Pobreza demais,
como tem demais!(Falta demais),
demais,/chuva no tem mais,
seca demais,/roubo demais,
povo sofre demais.
Oh! demais.
Oh! Deus./Oh! Deus.
S se tiver Deus.(/Oh! Deus.
Oh! fome./Oh! interesse demais,
falta demais...
1. O eu-lrico parece culpar-se por algo. Pelo qu?

2. O que o leva a sentir-se assim?

3. Com frequncia nos deparamos com vocbulos que
no fazem parte de nosso cotidiano. E, quando o contexto
no nos favorece e no temos um dicionrio mo,
recorremos anlise do radical de para compreendermos
o significado da palavra que nos chamou a ateno.
Assim, explique o sentido do termo destacado na msica
lida.

4. A composio faz um jogo com a palavra demais.
Explique o contraponto que esse advrbio indica.

5. Explique o paradoxo existente em pobre menina
rica.


Tarefa Mnima #

TEXTO 1


1. Assinale a afirmao correta:
a) A fim de estimular a escrita correta, a linguagem usada
na tira de Chico Bento deveria ser alterada para a
formal.
b) A forma de expresso dos personagens incompatvel
com sua faixa etria.
c) A linguagem usada coerente com o contexto
apresentado.
d) O modo caipira dos personagens se expressarem
dificulta muito a compreenso das falas.
e) A tira no apresenta total coerncia, j que foram feitas
quatros pergunta a Chico Bento e este apresentou
apenas uma resposta a seu amigo.

2. A atitude de Chico Bento revela:
a) incoerncia .
b) um pensamento utpico.
c) inconsequncia.
d) ingenuidade.
e) maturidade.

TEXTO 2
AULA DE PORTUGS
(Carlos Dummond de Andrade)
A linguagem/na ponta da lngua,
to fcil de falar/e de entender.

A linguagem
na superfcie estrelada de letras,
sabe l o que ela quer dizer?

Professor Carlos Gis, ele quem sabe,
e vai desmatando
o Amazonas de minha ignorncia.
Figuras de gramtica, esquipticas*,
atropelam-me, aturdem-me, sequestram-me.

J esqueci a lngua em que comia,
em que pedia para ir l fora,
em que levava e dava pontap,
a lngua, breve lngua entrecortada
do namoro com a prima.
O portugus so dois; o outro, mistrio.

*incomum; extravagante

3. A primeira estrofe revela um contraponto em relao
segunda. Explique que contraponto seria esse.
4. Reflita sobre os versos transcritos a seguir e explique
modo denotativo a mensagem passada por eles.
Figuras de gramtica, esquipticas
1
,
atropelam-me, aturdem-me, sequestram-me.

5. Drummond, em seu poema Aula de Portugus, faz
uma crtica. Produza um pargrafo (10 a 15 linhas),
revelando e defendendo seu ponto de vista quanto a essa
questo levantada pelo poeta.

Tarefa Complementar #

TEXTO 1

1. O efeito de humor foi um recurso utilizado para
mostrar que o pai de Mafalda:
01. revelou desinteresse pela leitura do dicionrio.
02. tentava ler o dicionrio, que uma obra muito extensa.
04. causou surpresa em sua filha, ao se dedicar leitura
de um livro to grande.
08. queria consultar o dicionrio para tirar uma dvida, e
no ler o livro, como sua filha pensava.
16. demonstrou que a leitura do dicionrio o desagradou
bastante, fato que decepcionou muito sua filha.
32. no sabia manusear um dicionrio.
Please purchase PDF Split-Merge on www.verypdf.com to remove this watermark.
Incluso para a Vida Gramtica

Pr-Vestibular da UFSC

29

TEXTO 2
SEM D
Eu sei que voc viu na TV
O assassinato de crianas de rua
Mas ser que voc viu na TV
As manchetes dos jornais do dia?
A moral de pases irmos
Que nos acusam de selvagens
Com suas competentes organizaes
Ser que voc viu na TV
Os nossos sonhos destrudos?
Juros altos, vistos negados
Portas fechadas para a civilizao
Que moral tem esses nossos irmos
Que nos acusam de selvagens
Com suas competentes organizaes
Ser que eles se preocupam com a gente?
Ser que eles realmente se preocupam?
Julgar e condenar parece fcil
Ser que eles realmente se preocupam?
Ou ser que somos
Um espetculo a mais
Nos seus telejornais

Num museu de horrores
A confirmar o seu lugar de civilizado
No primeiro mundo
Aquele que destri
Sem deixar vestgio
Onde o nico sinal
a riqueza cada vez maior
No sei se voc viu na TV
As conscincias de perfume francs
Ou se ao mudar de canal
Assistiu as imagens da guerra civil
No continente paira o humanismo
Pessoas tambm morrem todo dia
Assassinadas sem d
Por credo, nacionalidade e cor

(...)
Ser que eles se preocupam com a gente?
Ser que eles realmente se preocupam?
Julgar e condenar parece fcil
Ser que eles realmente se preocupam?
(Baro Vermelho)

2. A ideia que se encontra no texto :
a) O assassinato de crianas de rua no Brasil causa mal-
estar internacional.
b) As organizaes internacionais tm interferido,
frequentemente, nas questes sociais que dizem
respeito apenas a ns, brasileiros.
c) Os pases desenvolvidos ratificam sua superioridade
enquanto condenam as mazelas sociais no Brasil,
como se estas fossem exclusivamente do mundo
subdesenvolvido.
d) As questes religiosas e tnicas denunciam problemas
especficos de naes como o Brasil, em que
prevalece o colonialismo cultural.
e) Proliferam-se pelo globo um sem-nmero de conflitos
que preocupam as organizaes internacionais.

3. Assinale a passagem que melhor identifica o
sentimento de impotncia frente ao mundo desenvolvido:
a) A moral de pases irmos/Que nos acusam de
selvagens
b) Os nossos sonhos destrudos?/Juros altos, vistos
negados/Portas fechadas para a civilizao
c) Julgar e condenar parece fcil
d) Somos um espetculo a mais, nos seus telejornais
e) No continente do pai do humanismo/Pessoas tambm
morrem todo dia

4. O questionamento enfatizado em Ser que eles
realmente se preocupam? expressa:
a) ironia.
b) ceticismo.
c) possibilidade.
d) conformismo.
e) esperana.

UNIDADE 15

ANLISE SINTTICA DO PERODO
COMPOSTO - Parte I

ORAES COORDENADAS
As oraes coordenadas sindticas podem ser de cinco
tipos:
a. Aditiva (adio): e, nem, mas tambm, mas
ainda, bem como, como tambm.
b. Adversativa (oposio): mas, porm,
todavia, no entanto, entretanto, contudo.
c. Alternativa (alternncia): ou... ou, ora...
ora, quer... quer, seja... seja.
d. Conclusiva (concluso): por isso, logo,
portanto, pois (Nunca no incio da orao!),
ento, assim
e. Explicativa (explicao): pois (Sempre no
incio da orao!), porque, que, porquanto


Exerccios de Sala #

1. Classifique as oraes coordenadas a seguir:
a) A menina vai ficar ou vai embora?
b) Penso, logo existo.
c) Falou bem, mas ningum entendeu.
d) A doena vai a cavalo, e volta a p.
e) Vou passar no vestiba, pois estou estudando muito.
f) J aprendi as conjunes, portanto j sou capaz de
classificar as oraes sindticas.
g) A mulher tentou passar, porm sua passagem foi
barrada.
h) As folhas, no inverno, amarelecem e caem, ou ficam
inativas.
i) O acusado no criminoso, logo ser absolvido.

Tarefa Mnima #
1. "A nova bomba anunciava o rpido desfecho da guerra
em curso contra o Japo. Mas tambm prenunciava uma
nova era, cheia de inquietaes."
Please purchase PDF Split-Merge on www.verypdf.com to remove this watermark.
Incluso para a Vida Gramtica

Pr-Vestibular da UFSC

30
A expresso destacada exprime:
a) adio
b) alternncia
c) contraste
d) concluso
e) explicao

2. Leia atentamente as frases:
I - Mrio estudou muito e foi reprovado! (= mas)
II - Mrio estudou muito e foi aprovado! (= por isso)
Em I e II, a conjuno e tem, respectivamente, valor:
a) aditivo e conclusivo
b) adversativo e aditivo
c) aditivo e aditivo
d) adversativo e conclusivo
e) concessivo e causal

3. "Desde os cinco anos, merecera eu a alcunha de
menino-diabo; e verdadeiramente no era outra coisa; fui
dos mais malignos do meu tempo, arguto, indiscreto,
traquinas e voluntarioso."
(Machado de Assis)
No trecho acima h:
a) duas oraes, dois perodos
b) trs oraes, um perodo
c) trs oraes, trs perodos
d) quatro oraes, um perodo
e) trs perodos, trs oraes

4. A relao semntica implcita entre as oraes
coordenadas Os pais j podem escolher o tipo de filho
que querem, o filho continua no podendo escolher os
pais que o tero de:
a) adio.
b) condio.
c) causa.
d) oposio.
e) alternncia.

5. Muitos resultados so imprevisveis, mas os dados j
obtidos, diz a pesquisadora, sem dvida permitiro um
desenvolvimento extraordinrio, tanto na medicina e na
biotecnologia quanto na bioinformtica.
Os conectivos grifados podem ser substitudos, sem
alterao do significado, respectivamente, por:
a) porm no s mas tambm;
b) entretanto ora ora;
c) portanto no s mas tambm;
d) porque seja seja;
e) contudo ora ora.

1



5




10


A importncia do conto popular em nossa
cultura to forte que precisamos ter muito claro o
que se deve entender por popular, quando se trata
de estudar gneros literrios.
Geralmente se entende por popular um tipo de
criao rstica, caracterizada pela simplicidade e
pobreza expressiva. Talvez voc mesmo pense
assim. Mas, veja bem, se assim fosse, como se
justificaria a influncia que a tradio popular
exerceu e continua exercendo sobre a literatura e as
outras manifestaes artsticas e culturais, inclusive
aquelas de carter eminentemente tcnico?


15




20




25




30




Se este legado existe, porque a cultura
popular algo muito mais rico do que podemos
imaginar. Popular , portanto, uma manifestao
cultural de carter universal, nascida de modo
espontneo e totalmente indiferente a tudo que seja
imposto pela cultura oficial. Tambm no pode ser
entendido como sinnimo de regional, pois isto
eliminaria a tendncia universalizante das
manifestaes populares. Quer dizer, as criaes
populares no conhecem normas nem limites. Elas
esto acima de qualquer tipo de aprovao social.
O conto popular, embora tenha um carter
universal, seja uma criao coletiva e tenha vivido
muito tempo graas transmisso oral, apresenta um
modo narrativo que o singulariza diante de outros
tipos de narrativas. Com isso, possvel dizer que o
conto popular um gnero narrativo que desenvolve
traos que se repetem em histrias criadas nos mais
variados locais e pocas. Suas caractersticas
composicionais no conhecem fronteiras de tempo
nem de lugar.
MACHADO, Irene. Literatura e redao. So Paulo: Scipione,
1994. p. 28.
6. Conectores so elementos de ligao que estabelecem
diferentes relaes de sentido entre os segmentos de um
texto. Tendo em vista a definio acima, marque a(s)
alternativa(s) em que a relao estabelecida pelo(s)
elemento(s) em negrito est corretamente indicada.
01. to forte que precisamos ter muito claro
Relao de modo: expressa, numa das oraes, o
modo como se realiza o evento contido na outra.
02. quando se trata de estudar gneros literrios.
Ligao temporal: relaciona fatos ou eventos
simultneos.
04. Mas, veja bem, se assim fosse Relao de
oposio: introduz uma argumentao contrria ao
que foi dito anteriormente.
08. a cultura popular algo muito mais rico do que
podemos imaginar. Relao comparativa: estabelece
a superioridade de um elemento sobre outro(s).
16. Popular , portanto, uma manifestao cultural
Ligao conclusiva: introduz uma concluso
relativamente aos enunciados anteriores.
32. pois isto eliminaria a tendncia
universalizante Ligao explicativa: apresenta
uma justificativa ou explicao do que foi dito antes.
64. Quer dizer, as criaes populares Relao de
exemplificao: acrescenta um possvel exemplo a
uma declarao anterior, de ordem mais geral.

Tarefa Complementar #

1. Considere este perodo composto, extrado do livro
Macunama, de Mrio de Andrade, e indique a(s)
afirmao(es) incorreta(s):

Ento a velha apeou do tapir e montou num cavalo
gazeo sarar que nunca prestou nem prestar e
seguiu.
01. O perodo formado por cinco oraes, dentre as
quais, apenas duas so coordenadas sindticas
aditivas: a 2 e a 5, introduzidas pelo conectivo e.
Please purchase PDF Split-Merge on www.verypdf.com to remove this watermark.
Incluso para a Vida Gramtica

Pr-Vestibular da UFSC

31
02. A orao que nunca prestou inicia-se com um
pronome relativo e refere-se ao substantivo cavalo,
sendo ela classificada como subordinada adjetiva
explicativa.
04. A segunda orao coordenada sindtica aditiva
em relao primeira e principal
08. A ltima orao coordena-se aditivamente com a 2:
montou num cavalo gazeo sarar () e seguiu.

1



5




10




15




20


Qualquer inovao tecnolgica traz certo
desconforto queles que, apesar de conviverem com
ela, ainda no a entendem. As tecnologias no so
apenas produtos de mercado, mas produtos de
prticas sociais. Seus padres so arquitetados
simbolicamente como contedos sociais, para depois
haver uma adaptao mercadolgica.
As tecnologias da comunicao e informao
no podem ser reduzidas a mquinas; resultam de
processos sociais e negociaes que se tornam
concretas. Elas fazem parte da vida das pessoas; no
invadem a vida das pessoas. A organizao de seus
gneros, formatos e recursos procura reproduzir as
dimenses da vida no mundo moderno, o tempo, o
espao, o movimento: o mundo plural hoje vivido.
Novos modos de sentir, pensar, viver e ser,
construdos historicamente, se mostram nos
processos comunicativos derivados das necessidades
sociais.
Cabe escola o esclarecimento das relaes
existentes, a indagao de suas fontes, a conscincia
de sua existncia, o reconhecimento de suas
possibilidades, a democratizao de seus usos.

Trecho abaixo foi retirado dos Parmetros Curriculares
Nacionais Ensino Mdio (Braslia,
DF: Ministrio da Educao, 1999, p. 133-4)


2. Identifique o(s) item(ns) cuja proposio est correta.
01. O primeiro perodo do segundo pargrafo composto
por coordenao e subordinao.
02. No ltimo perodo/pargrafo do texto, o sujeito
composto o esclarecimento das relaes existentes, a
indagao de suas fontes, a conscincia de sua
existncia, o reconhecimento de suas possibilidades,
a democratizao de seus usos vem posposto ao
verbo (caber), o que justifica a concordncia no
singular.
04. A presena do pronome indefinido certo junto a
desconforto, tem por funo relativizar ou atenuar o
sentido atribudo a esse substantivo no texto.
08. O uso da crase facultativo diante de pronomes
demonstrativos. Portanto, a forma queles poderia ser
substituda, sem problemas, por aqueles.
16. Em As tecnologias no so apenas produtos de
mercado, mas produtos de prticas sociais, mas
indica uma oposio de idias.
32. Os pronomes que e elas remetem anaforicamente a
negociaes.





TRANSAMARGURA
(...) Idealizada como um dos maiores smbolos da
integrao nacional, a Transamaznica comeou a ser
aberta h trinta anos, na condio de carro-chefe do
projeto Brasil Grande, do regime militar. (...). Em
sintonia com o discurso ufanista da poca, o governo
prometia solenemente entregar terra sem homens para
homens sem terra. Mais de um milho de brasileiros
acabaram seduzidos pelas promessas redentoras daquela
obra grandiosa, mas a estrada jamais foi construda.
Isto, 11/10/2000.

3. A expresso ... mas a estrada jamais foi
construda:
a) estabelece uma relao de adio idia que a
antecede no perodo.
b) estabelece uma relao de oposio idia que a
antecede no perodo.
c) estabelece uma relao de concluso idia que a
antecede no perodo.
d) no estabelece relao alguma, pois independe da idia
que a antecede no perodo.
e) estabelece uma relao de explicao idia que a
antecede no perodo.

4. Analise os dois fragmentos abaixo.
nossos bisnetos vo passear ou, um dia, viver em
Marte.
quando teremos robs escravos, mquinas de orgasmos
ou naves para viajar no tempo
O vocbulo ou expressa, respectivamente, idia de:
a) adio e excluso.
b) alternncia e excluso.
c) excluso e adio.
d) adio e alternncia.
e) alternncia e adio.

5. Considerando aspectos sintticos dos perodos Todo
dia algum biruta apresentava uma nova mquina,
anunciava um plano mirabolante e desafiava a gravidade
e a prudncia e A tecnologia da aerodinmica, da
engenharia de estruturas, do desenho de motores e da
qumica de combustveis havia chegado a um estgio de
evoluo indito, aponte as alternativas corretas.
01. Ambos os perodos so compostos.
02. O primeiro perodo se compe de trs oraes
coordenadas.
04. Os verbos empregados nas oraes do primeiro
perodo tm em comum o fato de serem transitivos
indiretos.
08. Na nica orao que constitui o segundo perodo, o
termo tecnologia funciona como ncleo do sujeito
simples.
16. No primeiro perodo, a conjuno e, empregada
duas vezes, est separando oraes de igual valor e
tambm termos de valor idntico na mesma orao.

6. Observe os perodos a seguir:
L penso em me eleger deputado. J entrei em contato
com membros do Partido Verde que me apoiaro.
Assinale a alternativa que apresenta a expresso que
poderia ser usada para relacionar as duas oraes,
mantendo as relaes de sentido.
Please purchase PDF Split-Merge on www.verypdf.com to remove this watermark.
Incluso para a Vida Gramtica

Pr-Vestibular da UFSC

32
a) Por isso.
c) Mas.
b) Porque.
d) Como.
e) Nem.

7. No trecho Aqui, em Braslia, s misrias (). Mas
no s em Braslia., a palavra mas est sendo
empregada para indicar:
a) uma concluso sobre argumento j apresentado.
b) uma concluso contrria quela j sugerida.
c) uma justificativa sobre argumento j apresentado.
d) um argumento a favor de uma mesma concluso.
e) uma justificativa coerente.

UNIDADE 16

ANLISE SINTTICA DO PERODO
COMPOSTO - Parte II

ORAES SUBORDINADAS (Substantivas e
Adjetivas)
A orao que apresenta essas duas caractersticas chama-
se orao subordinada; a que se refere subordinada
chama-se orao principal.
De acordo com o valor que possuem, as oraes
subordinadas so classificadas como:
- Substantivas
- Adjetivas
- Adverbiais

I - SUBSTANTIVAS
So oraes que exercem funes de substantivo.
Podem exercer as seguintes funes sintticas:
a. Sujeito
b. Predicativo do sujeito
c. Objeto direto
d. Objeto Indireto
e. Complemento nominal
f. Aposto

As O.S.S. podem ser substitudas pelos pronomes ISTO,
ESTE, ESTA. Se for possvel substituir toda a orao por
um desses trs pronomes, ento ela substantiva.
Joo disse que prefere sua vovozinha.


(Joo disse ISTO.)


Orao Subordinada Substantiva

Ao substituir a orao substantiva por ISTO, ESTE,
ESTA, a funo do pronome ser a mesma da orao.
Joo disse ISTO.


Objeto Direto

Joo disse que prefere sua vovozinha.


O. S. Substantiva Objetiva Direta

A O.S.S introduzida pela chamada conjuno
integrante. As mais importantes so "que" e "se".
Tipos de Orao Substantiva
a. Subjetiva
Na O.P. falta o sujeito.
No introduzida por preposio.
O verbo da O.P. est sempre em 3 p. do singular.
Ex.: improvvel que a prova seja adiada.
b. Objetiva Direta
Na O.P. falta o O.D.
O verbo da O.P. T.D.
No introduzida por preposio.
Ex.: No percebeu que a isolavam do grupo.
c. Objetiva Indireta
Na O.P. falta o O.I.
O verbo da O.P. T.I.
introduzida por preposio.
Ex.: Impediu-os de que fizessem aquela maldade.
d. Predicativa
Na O.P. falta o Predicativo do Sujeito.
O verbo da O.P. sempre um verbo de ligao.
No introduzida por preposio.
Ex.: Meu desejo que eu ganhe um pirulito.
e. Completiva Nominal
Refere-se a um substantivo ou adjetivo da O.P.
iniciada por preposio.
Ex.: Sou favorvel a que jovens de 16 anos votem.
f. Apositiva
Exerce a funo de aposto de um substantivo.
No introduzida por preposio.
a nica substantiva que apresenta pontuao (os dois
pontos).
Ex.:Desejo uma coisa: que voc seja muito feliz.

II ADJETIVAS
So oraes que exercem funes de adjetivo.
So introduzidas por pronomes relativos (que,
o qual, onde, no qual, quem, cujo, etc.)
O melhor modo de perceber a orao adjetiva notar a
presena do pronome relativo. O mais comum o "que".
Sempre que for possvel substitu-lo por "o qual, a qual,
os quais, as quais", ele funciona como pronome relativo,
e a orao que ele introduz se classifica como
subordinada adjetiva.

Tipos de OraoSubordinada Adjetiva
Restritiva
Possui funo de limitar, restringir o sentido do
substantivo que vem antes da orao adjetiva.
Ex.: Eu j li o livro que voc trouxe.
Os homens que tm seu preo so fceis de corromper.

Explicativa
Funciona como se fosse um aposto.
Possui funo de explicar o sentido do substantivo que
vem antes da orao adjetiva.
marcada por vrgulas.
Ex.: Os homens, que tm seu preo, so fceis de
corromper.
Nossos pais, dos quais gostamos muito, j so idosos.

Please purchase PDF Split-Merge on www.verypdf.com to remove this watermark.
Incluso para a Vida Gramtica

Pr-Vestibular da UFSC

33
Exerccios de Sala #

1. Classifique as oraes subordinadas substantivas:
a) Era urgente que o plano fosse aprovado.
b) Convm que ele saiba de tudo.
c) Ele disse que houve muita briga.
d) O meu medo era que houvesse briga.
e) Agora, todos j sabem onde ele mora.
f) S desejo uma coisa: que no se esqueam de mim.

2. Classifique as oraes subordinadas adjetivas:
a)A me, que era surda, estava na sala com ela.
b)Ela reparou nas roupas curiosas que as crianas
usavam.
c) preciso gozarmos a vida, que breve.
d) Onde est a vela do saveiro que o mar engoliu?
e) O vulco, que parecia extinto, voltou a dar sinal de
vida.

3. Transforme o destacado em oraes adjetivas fazendo
as adaptaes necessrias:
a) O homem imaturo no assume suas responsabilidades.
b) Meu suco preferido o de abacaxi.
c) O assassino, flagrado pelo pai da vtima, ainda no foi
julgado.

Tarefa Mnima #

1. Em "No sei onde pegou meu p, na barriga
talvez...", a orao destacada classifica-se como
subordinada:
a) substantiva objetiva direta
b) adjetiva restritiva
c) substantiva predicativa
d) substantiva subjetiva
e) adjetiva explicativa

2. Assinale a(s) alternativa(s) correta(s) quanto
classificao posta entre parnteses.
01. possvel que comunicassem sobre poltica. (O.
Sub. Subjetiva)
02. Argumentei que no seria justo. (O. Sub. Objetiva
Indireta)
04. A notcia que a imprensa divulgou foi fantstica.
(O. Sub. Adjetiva Restritiva)
08. medida que as nuvens escureciam, a noite
parecia chegar. (O. Sub. Adverbial temporal)
16. A passarela foi feita para que os pedestres tivessem
segurana. (O. Sub. Adverbial final)
32. Os homens, que tm seu preo, so facilmente
corrompidos. (O. Sub. Adjetiva Explicativa)

Tarefa Complementar #

1. (UFSC) Assinale, dentre as proposies a seguir,
aquela(s) que apresenta(m) correspondncia entre o(s)
termo(s) sublinhado(s) e aquele que est entre parnteses.
01. A nova Lei de Trnsito impe aos motoristas novas
regras. (objeto indireto)
02. O processo foi-lhe favorvel. (complemento nominal)
04. A prova terminou muito cedo. (adjunto adverbial de
intensidade)
08. Dorme, cidade maldita, teu sono de escravido.
(aposto)
16. Loja com nome estrangeiro paga mais imposto.
(objeto direto)
32. Estou certo de que ela passar nos exames (orao
subordinada substantiva completiva nominal)

2. Observe o trecho () mas no vejo a f. E por que
no aparece a f nesta casa?
Os sujeitos dos verbos grifados so respectivamente:
a) sujeito inexistente e sujeito oculto.
b) sujeito composto e sujeito simples.
c) sujeito simples e sujeito composto.
d) sujeito simples e sujeito oculto.
e) sujeito oculto e sujeito simples.

UNIDADE 17

ANLISE SINTTICA DO PERODO
COMPOSTO - Parte III

ORAES SUBORDINADAS (Adverbiais)
A melhor maneira de classificar as oraes adverbiais
pensar no significado da orao dentro do contexto da
frase. No se habitue, portanto, a decorar conjunes.
a. Causal: d idia de causa, motivo.
Porque, visto que, j que, uma vez que, como
(equivalendo a porque)
b. Comparativa: d idia de comparao.
Como, que (precedido de mais ou de menos)
c. Consecutiva: d idia de consequncia.
Que (precedido de um termo intensivo: to, tal, tanto)
d. Concessiva: d idia de concesso, que o ato de
conceder, de permitir, de admitir uma idia contrria.
Embora, se bem que, ainda que, por mais que, por
menos que, conquanto
e. Condicional: d idia de condio.
Se, caso, contanto, que, desde que
f. Conformativa: d idia de conformidade, isto , de
acordo, de adequao, de no-contradio.
Como, conforme, consoante, segundo
g. Final: d idia de finalidade, entendida como o
objetivo, a destinao de um fato.
A fim de que, para que, que
h. Proporcional: d idia de proporo, entendida
como a relao existente entre duas coisas, de modo
que qualquer alterao em uma delas implique
alterao na outra.
Que, medida que, proporo que, quanto mais,
quanto menos
i. Temporal: d idia de tempo.
Quando, enquanto, logo que, desde que, assim que

Exerccios de Sala #

1. Classifique as oraes subordinadas adverbiais.
a) Minha mo tremia tanto que mal conseguia escrever.
b) Joel acompanhou a irm, embora estivesse cansado.
Please purchase PDF Split-Merge on www.verypdf.com to remove this watermark.
Incluso para a Vida Gramtica

Pr-Vestibular da UFSC

34
c) medida que subimos, o ar se rarefaz.
d) Fiz-lhe sinal para que no insistisse.
e) Os detentos fugiram da penitenciria porque eram
maltratados.
f) Envelheamos como as rvores fortes envelhecem!
g) Por que no foi l ontem como me tinha dito?
h) Ia escurecendo quando chegou em casa.
i) Se Deus no guarda a cidade, em vo a sentinela vigia.

Tarefa Mnima #

1. Assinale a opo em que a orao sublinhada uma
orao adverbial com valor de consequncia.
a) A psiquiatria tem repensado a noo de cura, sem que
obtenha resultados mais eficazes.
b) A psiquiatria tem repensado a noo de cura, para
obter resultados mais eficazes.
c) A psiquiatria tem repensado a noo de cura, obtendo,
assim, resultados mais eficazes.
d) Como a psiquiatria tem repensado a noo de cura,
tem obtido melhores resultados.
e) Sempre que a psiquiatria repensa a noo de cura,
obtm resultados mais eficazes.

2. No perodo Embora o porta-voz do Vaticano
tenha declarado que a viagem do Papa Terra Santa
uma peregrinao pelos caminhos de Cristo, impossvel
no identificar nela um ntido significado poltico.
est contida uma idia de:
a) fim.
b) oposio.
c) causa.
d) modo.
e) tempo.

3. Ao se enviar uma carta atravs de algum,
imagina-se a pobre pessoa, com uma carta atravessada no
corpo. Assinale a alternativa que melhor corresponde ao
trecho destacado acima.
a) Logo que se envia uma carta atravs de algum
b) Quando se envia uma carta atravs de algum
c) medida que se envia uma carta atravs de algum
d) Como se envia uma carta atravs de algum
e) Enquanto se envia uma carta atravs de algum...

Tarefa Complementar #

1. No perodo Vivemos ainda num ritmo de trabalho da
poca industrial, quando a sociedade est na era dos
servios, a orao introduzida por quando
estabelece com a orao anterior uma relao semntica
de:
a) causalidade.
b) concesso.
c) condicionalidade.
d) anterioridade.
e) simultaneidade.

2. Fomos e seremos assim, em nossa essncia, embora
as circunstncias mudem e ns mudemos com elas.
Assinale a opo em que, ao reescrever-se o fragmento
acima, substituiu-se o conectivo sublinhado por outro de
valor condicional, fazendo-se alteraes aceitveis.
a) Fomos e seremos assim em nossa essncia, porque as
circunstncias mudaram e ns mudamos com elas.
b) Fomos e seremos assim em nossa essncia, enquanto
as circunstncias mudarem e ns mudarmos com elas.
c) ramos e somos assim em nossa essncia, medida
que as circunstncias mudaram e ns mudamos com
elas.
d) Teramos sido e seramos assim em nossa essncia, se
as circunstncias mudassem e ns mudssemos com
elas.
e) Temos sido e somos assim em nossa essncia,
conforme as circunstncias tm mudado e ns temos
mudado com elas.

3. Como alguns moradores do vilarejo contam, muitos
forasteiros perderam suas vidas tentando encontrar
pedras preciosas.
Em que alternativa a palavra como expressa a mesma
relao de sentido que apresenta acima?
a) O grande pacificador morreu como heri.
b) Como era um garoto muito peralta, acabou
espatifando-se no cho.
c) Flix e o advogado encontraram-se ao amanhecer
como haviam combinado ontem.
d) Como o cu estivesse recoberto de nuvens escuras,
no fomos praia.
e) O garoto voltou para a cidade como quem vai para a
priso.

4. Assinale a alternativa em que se encontra a orao
subordinada grifada com o mesmo valor semntico de
ao despedir-se do sol frio e gasto, o derradeiro homem
h de ter um relgio na algibeira:
a) sentado entre dous sacos, o da vida e o da morte,
imaginava ento um velho diabo.
b) porque o relgio parava, eu dava-lhe corda, para que
ele no deixasse de bater nunca.
c) quando eu perdia o sono, o bater da pndula fazia-me
muito mal.
d) para que no deixasse de bater nunca, eu dava-lhe
corda.
e) invenes h, que se transformam ou acabam; o
relgio definitivo e perptuo.

UNIDADE 18

ANLISE SINTTICA DO PERODO
COMPOSTO - Parte IV

ORAES REDUZIDAS
Algumas vezes as oraes subordinadas podem aparecer
sob a forma de oraes reduzidas.
As oraes subordinadas reduzidas tm duas
caractersticas:

x Apresentam o verbo em uma das formas nominais:
gerndio, particpio, infinitivo.
x No vm introduzidas por conectivos (conjunes
subordinativas ou pronomes relativos).





Please purchase PDF Split-Merge on www.verypdf.com to remove this watermark.
Incluso para a Vida Gramtica

Pr-Vestibular da UFSC

35
CLASSIFICAO
x Subordinada reduzida de gerndio
x Subordinada reduzida de particpio
x Subordinada reduzida de infinitivo
DICAS
Para analisar uma O.S.Reduzida, basta fazer o seguinte:
x Desenvolv-la, ou seja, tir-la da forma reduzida,
fazendo aparecer o conectivo
x Analisar a orao desenvolvida
x Aplicar a anlise da orao desenvolvida reduzida,
acrescentando reduzida de (gerndio, particpio,
infinitivo).
Observe atentamente o exemplo que segue:
Penso estar doente.
Desenvolvendo:
Penso/ que estou doente.
Se quem pensa, pensa algo, diz-se, ento, que uma
orao subordinada substantiva objetiva direta.
Agora basta aplicar a classificao orao reduzida e
acrescentar as palavras reduzida de infinitivo.

Exerccios de Sala #

1. Desenvolva as oraes abaixo, introduzindo
conjunes.
a) Vi guardas conduzindo presos.
b) Terminado o baile, todos saram.
c) No participou do torneio por estar doente.
d) Viajando nas frias, no pde concluir o curso.

Tarefa Mnima #

1. Classifique as oraes reduzidas destacadas abaixo, de
acordo com as denominaes:
1 - coordenada sindtica aditiva
2 - subordinada substantiva subjetiva
3 - subordinada adverbial condicional
4 - subordinada adverbial causal
5 - subordinada adverbial temporal
a) No sabendo do resultado, preferiu desistir da
competio.
b) Urge eleger um bom candidato, a fim de que se
resolvam estes problemas.
c) O garoto caiu levantando-se rapidamente.
d) Antes de viajar, esteve aqui nos visitando.
e) Falando bem o ingls, ser mais fcil sua ida ao
exterior.

2. As oraes em destaque so substantivas reduzidas de
infinitivo. Classifique-as:
( 1 ) subjetiva
( 2 ) objetiva direta
( 3 ) objetiva indireta
( 4 ) completiva nominal
( 5 ) predicativa
( 6 ) apositiva
( ) Aconselharam-me a desfazer o noivado.
( ) Todos conheciam a mania de Laura: empenhar
jias.
( ) Depende de V. S libertar esses presos.
( ) Um de seus passatempos colecionar selos.
( ) Parti com a doce esperana de reencontrar meu
amor.
( ) Lamento ter perdido essa oportunidade.

3. Classifique as oraes adverbiais reduzidas de
infinitivo.
( 1 ) causal
( 2 ) concessiva
( 3 ) condicional
( 4 ) consecutiva
( 5 ) final
( 6 ) moda
( 7 ) temporal

( ) No podia demorar-me, sob pena de perder o
avio.
( ) Retirei-me discretamente, sem ser percebido.
( ) difcil curar um mal sem lhe conhecer as causas.
( ) Ao clarear o dia, descemos da montanha.
( ) No pude viajar por ter perdido o dinheiro.
( ) Tirou o cachimbo da boca a fim de poder falar.
( ) Apesar de ser mais fraco, Davi matou Golias.

Tarefa Complementar #

1. Classifique as oraes adverbiais reduzidas de
gerndio:
( 1 ) causal
( 2 ) condicional
( 3 ) modal
( 4 ) temporal

( ) Aumentando-se a produo, a exportao crescer.
( ) Vendo-se perdido, o toureiro gritou por socorro.
( ) Chegando ao alto da rvore, sacudiu-a fortemente.
( ) Matou as formigas esmagando-as com o
calcanhar.

2. Classifique as oraes adverbiais reduzidas de
particpio:
( 1 ) causal
( 2 ) concessiva
( 3 ) condicional
( 4 ) temporal

( ) Terminado o almoo, comentamos as notcias do
dia.
( ) Ofendido pelo empregado, o patro descontrolou-
se.
( ) Mesmo picado por uma jararaca, o novilho no
morreu.
( ) Instituda a pena de morte, o crime diminuiria?

3. Ponha o nmero correspondente orao reduzida em
destaque:
(1) subjetiva
(2) objetiva direta
(3) objetiva indireta
(4) completiva nominal

( ) Aconselhou-me a no o ler.
( ) Faz mal a Marcor ver me e av desunidas.
( ) Exortou-me a botar a mo na conscincia.
( ) Sou avesso a derramar sangue humano.
Please purchase PDF Split-Merge on www.verypdf.com to remove this watermark.
Incluso para a Vida Gramtica

Pr-Vestibular da UFSC

36
( ) Eu estava com sede e curioso de experimentar
aquela bebida.
( ) A FUNAI informou ter demarcado a reserva
indgena.
( ) uma obrigao pagar a dvida ao velho.
( ) Muitos preferem morrer lutando a viver sem
liberdade.
( ) O treinador do clube observava o interesse do rapaz
em melhorar o nado livre.

UNIDADE 19

QUE e SE

O termo "se" aparece na frase como:

1. Pronome Reflexivo
Os dois amam-se como
irmos.
Elas deram-se as mos.
4. Partcula Expletiva ou
de realce
As moas sorriram-se
agradecidas
O povo riu-se ao ouvir
tantas asneiras.
2. Partcula Apassivadora
Sabe-se que h pessoas
safadinhas.
Doam-se aulas de
Gramtica.
5. Parte integrante de
verbos
Ele queixou-se do
assunto.
Maria referiu-se ao pai.
3. ndice de
Indeterminao do Sujeito
Aqui vive-se bem.
Precisa-se de serventes.
6. Substantivo
O se a palavra que
estamos estudando.
7. Conjuno Subordinativa
Se voc no ficar quieto, no vai
aprender a lio.

O termo "que" aparece na frase como:
1. Advrbio
Que fria est sua sala!
5. Conjuno
Penso que est tudo ok.
2. Substantivo
H um qu de censura no
ar.
6. Pronome Interrogativo
Que voc quer comigo?
3. Preposio
Tinha que estudar.
7. Pronome Relativo
A candidata que treinei foi
aprovada
4. Interjeio
Qu! Ela acreditou nisso?
8. Partcula Expletiva ou
de Realce
Que nome que te deram....

Exerccios de Sala #

1. Classifique a palavra se nestas frases:
a) Ignorvamos se ele voltaria um dia nossa casa.
b) As pernas do velho encolheram-se debaixo do lenol.
c) Ele se fez doutor ainda jovem e orgulha-se disso.
d) Obtiveram-se timos resultados.
e) Ele convenceu-se disso muito cedo.
f) S faremos o acordo se houver segurana na transao.
g) L se vo os garotos.
h) Os jogadores abraaram-se felizes.


2. Identifique as funes do que nas frases abaixo:

a) A demora era tanta que desistimos de esperar.
b) As fotografias que vimos eram fantsticas!
c) Qu! Voc suspeita de mim?
d) Voc sabe que horas so agora no Japo?
e) Os pais tm que dialogar com os filhos.
f) A decorao da casa tinha um qu de extravagante.

Tarefa Mnima #

1. Em que orao a palavra "que" um pronome
relativo?

a) Observei um qu de desconfiana em seu olhar.
b) Que resultados voc espera com essa atitude?
c) A famlia que lhe pagava todos os gastos.
d) O itinerrio que seguimos era o mais prximo.
e) No te intimides que h outros menos capazes.

2. Identifique a frase em que a palavra "se" pronome
apassivador.


a) Viaja-se pelas praias brasileiras no vero.
b) Os namorados acomodaram-se no sof, em silncio.
c) Deixou-se abater com a notcia de sua partida.
d) Proibiam-se as queimadas por todo o territrio.
e) Vivia-se tranqilo naqueles confins da serra.

3. Em que frase a palavra "se" ndice de
indeterminao do sujeito?

a) Aqui se aceitam encomendas de doces e salgados.
b) O passante escondeu-se da chuva repentina.
c) Neste restaurante italiano come-se bastante bem.
d) Os dois garotos protegiam-se da forte ventania.
e) Liquidaram-se as dvidas em pouco tempo.

4. Em: "V-se embora, j, seno eu grito", a palavra se :
a) pronome apassivador b) conjuno integrante
c) conjuno condicional d) partcula expletiva
e) parte integrante do verbo

5. (ITA-SP) Considere as palavras destacadas em:
"Das grandes narinas escorriam gotas e pensei, por um
momento, que fossem lgrimas."
"Um de ns protesta que deviam servir-lhe a carne em
pedacinhos."
Elas classificam-se, respectivamente, como:
a) pronome relativo - conjuno explicativa
b) conjuno consecutiva - conjuno integrante
c) conjuno causal - partcula expletiva
d) pronome relativo - pronome relativo
e) conjuno integrante - conjuno integrante

Tarefa Complementar #

1. A classificao entre parnteses da palavra "que" est
errada em:
a) Tenho que comprar novos sapatos. (preposio)
b) Qu! Voc no quer sair daqui?!? (interjeio)
c) Que bela apresentao, meu filho! (advrbio)
d) Vamos para casa, que est chovendo. (preposio)
e) Vou descobrir o qu da questo. (substantivo)

Please purchase PDF Split-Merge on www.verypdf.com to remove this watermark.
Incluso para a Vida Gramtica

Pr-Vestibular da UFSC

37
2. Em: "E ento dormi to depressa que nem sei como
foi...", a palavra destacada uma conjuno:
a) explicativa
b) integrante
c) final
d) consecutiva
e) concessiva

3. Na frase: "No sei se ela aceitar o vale", a palavra
"se" classifica-se como:
a) conjuno subordinativa condicional
b) conjuno subordinativa integrante
c) pronome pessoal oblquo
d) pronome pessoal reflexivo
e) pronome apassivador

UNIDADE 20

CONCORDNCIA NOMINAL

Exerccios de Sala #

Dois ou mais substantivos seguidos de um adjetivo,
poder ficar no singular ou ir para o plural.
Ex.: Comprei cala e blusa clara (s).
Dois ou mais substantivos precedidos de um
adjetivo, dever concordar com o mais prximo.
Ex.: Havia intenso calor e poluio naquela regio.
Importante!!!
Se os gneros forem diferentes, prevalece a forma
masculina.
Ex.: Vestia terno e camisa clara / claros.
Adjetivo composto flexiona somente o ltimo
elemento.
Ex.: Vestia calcinhas verde-escuras. (azul-claras)
Adjetivo anteposto concorda com o substantivo
mais prximo.
Ex.: Velhas revistas e livros estavam no balco.
Dois ou mais adjetivos se referem ao mesmo
substantivo, ocorrero duas construes.
Ex.: Estudo as lnguas francesa e inglesa.
Estudo a lngua francesa e a inglesa.

Particularidades
Mesmo Elas mesmas resolvero o caso.
Ela no sabia disso mesmo.
Junto Juntos iremos longe.
Iremos junto com ele.
Anexo/Incluso

Prprio
Quite
Leso
Obrigado
As cartas seguem anexas.
As cartas seguem em anexo.
Eles prprios assaltaram o caixa.
Estou quite com voc.
Cometeu crimes de lesas-ptrias.
Muito obrigada, diz a menina.

Tal
O filho tal qual o pai.
Os filhos so tais quais os pais.
O filho tal quais os pais.
Os filhos so tais qual o pai.
Menos
Alerta
Ela estava menos calma que ontem.
Fiquemos alerta ao combate.
Pseudo
Monstro
As pseudo-atrizes foram presas.
Elas so um monstro.
S S as meninas foram embora.
As meninas ficaram ss.

Meio
Maria estava meio doente.
Os fins no justificam os meios.
Comi somente meia melancia.
Bastante Eles esto bastante apaixonados.
Comemos bastantes pras.
Proibido proibida a entrada de pessoas feias.
proibido entrada de pessoas feias.
Caro Compra caro os presentes. (adv.)
Compra os presentes caros. (adj.)
Longe Andei por longes terras. (adj.)
Estavam longe de ns. (adv.)

Exerccios de Sala #

1. Faa a pequena lista de exerccios, completando as
frases com o adjetivo entre parnteses e efetuando a
correta concordncia. Fique atento posio do adjetivo.
a) Escolheste ________ lugar e hora para fazer
chacotas. (inadequado)
b) Escolheste ____ hora e lugar para fazer chacotas.
(mau)
c) ________ rosas e cravos enfeitam o altar. (belo)
d) A formanda comprou vestido e meias _______.
(preto)
e) Comi po e pras ________. (maduro)
f) Coqueiro e palmeira ________. (alto)
g) O poeta apreciava chapus e gravatas ________.
(escuro)
h) Deodato trajava ________ cala e blusa. (claro).
Please purchase PDF Split-Merge on www.verypdf.com to remove this watermark.
Incluso para a Vida Gramtica

Pr-Vestibular da UFSC

38


Please purchase PDF Split-Merge on www.verypdf.com to remove this watermark.
Incluso para a Vida Histria A

Pr-Vestibular da UFSC
1
UNIDADE 1

ANTIGUIDADE ORIENTAL

As primeiras sociedades que surgiram no Oriente
Prximo se organizaram atravs do Modo de
Produo Asitico. Todas as terras pertenciam ao
Estado e a servido coletiva era a forma utilizada pela
populao como pagamento pelo usufruto dessas. O
estado se apropriava do excedente agrcola dos servos e
distribua-o entre a nobreza e a classe sacerdotal.
Nos perodos em que no estavam ocupados
com atividades agrcolas, os servos eram utilizados na
realizao de grandes obras pblicas, as quais iam de
pirmides e zigurates a dique, represas e canais de
irrigao. Tais obras no s serviam para exaltar seus
reis e faras, como garantiam a ampliao das reas
cultivveis, o que deu a tais povos a denominao de
Sociedades Hidrulicas.
Com exceo dos Hebreus, criadores da
primeira religio monotesta, as sociedades daquela
regio eram politestas e seus governos teocrticos. Os
governantes (reis, imperadores ou faras) eram
considerados verdadeiros deuses. Assim a populao
era levada a acreditar que esses eram responsveis pela
fertilidade da terra e pela prosperidade de suas
comunidades.

Egito
Entre 5.000 a.C. e 4.000 a.C., formaram-se os
primeiros Nomos, pequenos agrupamentos humanos
s margens do rio Nilo. Em 3.500 a.C., devido
insuficincia de mo-de-obra para construes
necessrias ampliao das reas cultivveis, esses
nomos uniram-se em dois reinos: Alto Egito, ao norte,
e Baixo Egito, ao sul. O responsvel pela unificao
desses reinos foi Mens que, em 3.200 a.C., deu incio
ao que chamamos Imprio Egpcio.
A sociedade egpcia estava dividida em
vrias camadas, sendo o fara a autoridade mxima,
chegando a ser considerado um deus na Terra. Abaixo
do fara encontravam-se sacerdotes e burocratas. As
camadas superiores eram sustentadas pelo trabalho e
impostos pagos por camponeses, artesos e pequenos
comerciantes. Embora a mo-de-obra comumente
utilizada fosse a servido coletiva, havia tambm
escravos.
A economia egpcia era baseada
principalmente na agricultura, realizada
principalmente nas margens frteis do rio Nilo. Os
egpcios tambm praticavam o comrcio de
mercadorias e o artesanato.
A religio egpcia era politesta e
antropozoomrfica. Seus deuses formavam um
complexo sistema mitolgico. Muitos animais
tambm eram considerados sagrados pelos egpcios,
de acordo com as caractersticas que apresentavam:
chacal (esperteza noturna), gato (agilidade), carneiro
(reproduo), jacar (agilidade nos rios e pntanos),
serpente (poder de ataque), guia (capacidade de
voar), escaravelho (ligado a ressurreio).
A civilizao egpcia destacou-se muito nas
reas de cincias. Desenvolveram conhecimentos
importantes na rea da matemtica, usados na
construo de pirmides e templos. Na medicina, os
procedimentos de mumificao, proporcionaram
importantes conhecimentos sobre o funcionamento do
corpo humano.

Mesopotmia
A Mesopotmia localiza-se entre os rios Tigre e
Eufrates no Oriente Mdio, onde atualmente situa-se o
Iraque. Entre os povos que habitaram essa regio
podemos destacar: sumrios, acdios, babilnicos
(amoritas), assrios e neobabilnicos (caldeus). Vale
dizer que os povos da antiguidade buscavam regies
frteis, prximas a rios, para desenvolverem suas
comunidades. Dentro desta perspectiva, a regio da
mesopotmia era uma excelente opo, pois garantia
populao: gua para consumo, rios para pescar e via
de transporte pelos rios. No geral, eram povos
politestas com economia baseada na agricultura e no
comrcio nmade de caravanas.

Sumrios
x Escrita cuneiforme
x Construes hidrulicas
x Zigurates
x Cidades-Estado

Acdios
x Primeiro Imprio da Mesopotmia (Rei Sargo)

Amoritas
x Cidade da Babilnia
x Cdigo de Hamurabi
x Astronomia

Assrios
x Cultura militar.
x Extremamente cruis com os povos inimigos
x Conquista do Egito

Caldeus
x Jardins Suspensos da Babilnia (Nabucodonosor)
x Torre de Babel.
x Hegemonia e esplendor
x Dominados pelos Persa (539 a.C.)

Exerccios de Sala #

1. Sobre o Egito antigo, correto afirmar que:
01. a construo das pirmides atendia s
necessidades da vida aps a morte dos faras.
Esse tipo de construo foi caracterstica da
arquitetura funerria durante todo o perodo do
Egito antigo e s foi possvel graas enorme mo-
de-obra escrava existente desde o Antigo Reino.
02. a despeito da influncia islmica, o Egito atual
mantm as mesmas crenas religiosas do Egito
antigo.
04. os egpcios antigos acreditavam em vrios deuses
que se relacionavam entre si e formavam seu
sistema mitolgico.
Please purchase PDF Split-Merge on www.verypdf.com to remove this watermark.
Incluso para a Vida Histria A

Pr-Vestibular da UFSC 2
08. o rio Nilo foi de suma importncia em vrios
aspectos da vida dos antigos egpcios. No s a
agricultura foi possvel devido ao seu ciclo de
cheias, como tambm a noo de tempo cclico,
base do pensamento egpcio, levou crena na vida
aps a morte.

2. Na Antiguidade Oriental, o Modo de Produo
Asitico caracterizou-se fundamentalmente pelo(a):
a) fracionamento da propriedade fundiria em partes
entregues a nobres da Casa Real;
b) concentrao do controle da produo num partido
poltico;
c) apropriao formal da terra pelo Estado e efetiva
pela comunidade camponesa, cujos membros
deveriam pagar impostos e prestar servios ao
Estado;
d) emprego da fora de trabalho escravo, com um
comrcio operoso, controlado por uma burguesia
ativa e numerosa.
e) industrializao acentuada, calcada sobre uma farta e
barata fora de trabalho servil, amplamente
dominada pela aristocracia fundiria.

Tarefa Mnima #

3. As sociedades que, na Antiguidade, habitavam os
vales dos rios Nilo, Tigre e Eufrates tinham em comum
o fato de:
01. terem desenvolvido um intenso comrcio martimo,
que favoreceu a construo de grandes civilizaes
hidrulicas.
02. serem povos orientais que formaram diversas
cidades-estado, as quais organizavam-se e
controlavam a produo de cereais.
04. haverem possibilitado a formao do Estado a
partir da produo de excedentes, da necessidade de
controle hidrulico e da diferenciao social.
08. possurem, baseados na prestao de servios dos
camponeses, imensos exrcitos que viabilizaram a
formao de grandes imprios.

4. Sobre as civilizaes da Antiguidade Oriental,
correto afirmar:
01. Entre os egpcios, embora a prtica de mumificar
cadveres tivesse contribudo para o estudo do
corpo humano, o respeito que essa civilizao tinha
pelos mortos proibia a dissecao de cadveres
unicamente para estudos.
02. Entre os hebreus, os escribas constituam-se num
grupo social que, aprendendo a ler e a escrever,
desempenhou importantes funes religiosas, na
converso de fiis ao monotesmo.
04. Os persas acreditavam que o bem e o mal viviam
em incessante luta at o dia do juzo final, quando
todos os homens seriam julgados por suas aes.
08. A inveno do alfabeto pela civilizao fencia
esteve ligada necessidade que seus mercadores
tinham de firmar contratos comerciais com povos
distantes.
16. Hamurabi foi um rei babilnico que se tornou
famoso por mandar elaborar o primeiro cdigo
jurdico com leis escritas.

5. Leia com ateno as afirmativas abaixo sobre as
condies sociais, polticas e econmicas da
Mesopotmia.
I. As condies ecolgicas explicam porque a
agricultura de irrigao era praticada atravs de
uma organizao individualista.
II. Na economia da baixa Mesopotmia, as fomes e
crises de subsistncia eram frequentes, causadas
pela irregularidade das cheias e tambm pelas
guerras.
III. Na Sumria, os templos e ziggurats foram
construdos graas riqueza que os sacerdotes
administravam custa do trabalho de grande parte
da populao.
IV. A presena dos rios Tigre e Eufrates possibilitou o
desenvolvimento da agricultura e da pecuria e
tambm a formao do primeiro reino unificado da
Histria.
Sobre as afirmativas acima, correto afirmar:
a) I e II so verdadeiras;
b) III e IV so verdadeiras;
c) I e IV so verdadeiras;
d) I e III so verdadeiras;
e) II e III so verdadeiras.

6. Em relao aos povos da Antiguidade, correto
afirmar que:
a) os assrios foram submetidos por Nabucodonosor,
originando o episdio conhecido como o Cativeiro
da Babilnia.
b) os fencios foram os criadores do alfabeto,
posteriormente aperfeioado pelos gregos e latinos.
c) os hebreus criaram um quadro religioso
caracterizado pelo politesmo e a mumificao.
d) os egpcios estabeleceram, em 300 a.C., o
importante Cdigo de Hamurabi, um dos primeiros
cdigos jurdicos escritos.
e) os persas, aps derrotarem as tropas de Alexandre,
conseguiram anexar o territrio grego ao seu
imprio.

7. Se tomarmos como referncia a antiguidade oriental,
correto afirmar-se sobre a sociedade egpcia que
01. os faras, apesar de estarem no topo da hierarquia
social, deviam obedincia aos sacerdotes, que
detinham o poder teocrtico.
02. a alta nobreza era constituda pelos parentes do
fara, pelos altos funcionrios do palcio, pelos
oficiais superiores do exrcito, pelos chefes locais
da administrao e pelos sacerdotes.
04. os escribas, homens letrados, eram considerados os
"olhos e ouvidos" do fara.
08. os camponeses e os artesos constituam uma
camada social dinmica que controlava a economia
agroindustrial egpcia.
16. os escravos no recebiam proteo dos seus
senhores e eram maltratados em todas as situaes.

Please purchase PDF Split-Merge on www.verypdf.com to remove this watermark.
Incluso para a Vida Histria A

Pr-Vestibular da UFSC
3
8. Bagd - O famoso tesouro de Nimrud,
desaparecido h dois meses em Bagd, foi encontrado
em boas condies em um cofre no Banco Central do
Iraque em Bagd, submerso em gua de esgoto,
segundo informaram autoridades do exrcito norte-
americano. Cerca de 50 itens, do Museu Nacional do
Iraque, estavam desaparecidos desde os saques que
seguiram invaso de Bagd pelas foras da coalizo
anglo-americana.
Os tesouros de Nimrud datam de aproximadamente
900 a.C. e foram descobertos por arquelogos
iraquianos nos anos 80, em quatro tmulos reais na
cidade de Nimrud, perto de Mosul, no norte do pas.
Os objetos, de ouro e pedras preciosas, foram
encontrados no cofre do Banco Central, em Bagd,
dentro de um outro cofre, submerso pela gua da rede
de esgoto.
Os tesouros, um dos achados arqueolgicos mais
significativos do sculo 20, no eram expostos ao
pblico desde a dcada de 90. Uma equipe de
pesquisadores do Museu Britnico chegar na prxima
semana a Bagd para estudar como proteger os
objetos.
(O ESTADO DE SO PAULO. Verso eletrnica. So Paulo: 07
jun. 2003. Disponvel em www.estadao.com.br.)

Assinale a(s) proposio(es) correta(s) em relao s
sociedades que se desenvolveram naquela regio na
Antiguidade.

01. A regio compreendida entre os rios Tigre e
Eufrates, onde hoje se localizam os territrios do
Iraque, do Kweite (Kwait) e parte da Sria, era
conhecida como Mesopotmia.
02. A religio teve notvel influncia na vida dos
povos da Mesopotmia. Entre eles surgiu a crena
em uma nica divindade (monotesmo).
04. Na Mesopotmia viveram diversos povos, entre os
quais podemos destacar os sumrios, acdios,
assrios e babilnios.
08. Os babilnios ergueram magnficas construes
feitas com blocos de pedra, das quais so exemplos
as pirmides de Gis.
16. Os povos da Mesopotmia, alm da significativa
contribuio no campo da Matemtica, destacaram-
se na Astronomia e entre eles surgiu um dos mais
famosos cdigos de leis da Antiguidade, o de
Hamurbi.
32. Muitos dos povos da Mesopotmia possuram
governos autocrticos. Entre os caldeus surgiu o
sistema democrtico de governo.
UNIDADE 2

ANTIGUIDADE OCIDENTAL

GRCIA
Idade Antiga: Os mais importantes aspectos da Grcia
antiga: o antropocentrismo, o escravismo e a
democracia. Considerada como a mais expressiva
civilizao da antiguidade e fonte da cultura ocidental,
devido ao seu rico legado artstico, sua Filosofia e
Poltica. As reformas de Clstenes institucionalizaram
a democracia em Atenas, aps diferentes formas de
governo (monarquia, oligarquia e tirania), atingindo
seu apogeu no governo de Pricles (sculo V a.C.). A
democracia ateniense era direta, enquanto a atual
representativa. A religio era politesta e
antropomrfica. Nas artes destaca-se a exaltao do
equilbrio, harmonia e proporo.

Perodo Pr-Homrico (XX XII a.C.)
x Civilizao Cretense
x Formao e povoamento da Pennsula
Balcnica(Drios, Jnios, Aqueus, Elios)

Perodo Homrico (XII VIII a.C.)
x Genos (unidade bsica de produo, sem
propriedade privada e sem classe social, membros
ligados por laos consanguneos)
x Desagregao da unidade gentlica

Perodo Arcaico (VII VI a.C.)
x Formao das Cidades-Estado
9 Atenas: Comrcio, Cultura e Democracia
9 Esparta: Militarismo, Autoritarismo, Aristocracia

Perodo Clssico (VI IV a.C.)
x Guerras Mdicas ou Grego-Prsicas
9 Ofensiva grega Liga militar de Delos (liderana
ateniense, com o fim das lutas Atenas transforma a
Liga em instrumento de poder)
x Guerra do Peloponeso
9 Oposio grega liderada por Esparta hegemonia
ateniense
9 Liga do Peloponeso X Atenas
9 Vitria de Esparta e incio de sua hegemonia
9 Nova guerra interna, devido oposio
hegemonia espartana
9 Os resultados das guerras internas e externas so o
enfraquecimento das cidades-Estado e a invaso
macednica.

ROMA

A FUNDAO
A verso lendria da fundao de Roma foi-nos legada
por Virglio, na clebre Eneida. Segundo o autor,
Roma teria sido obra dos irmos Rmulo e Remo, que
a criaram em 753 a.C. Mas h a verso histrica,
segundo a qual Roma teria sido fundada pelos latinos e
sabinos por volta de 1000 a.C., nas proximidades do rio
Tibre.

MONARQUIA (753 a.C 509 a.C.)
Seguindo os passos da tradio, Roma teve sete reis:
Rmulo, Numa Pomplio, Tlio Hostlio, Anco Mrcio,
Tarqunio Prisco, Srvio Tlio e Tarqunio, o Soberbo.
Na sociedade, havia uma diviso social
composta da seguinte maneira: Patrcios; Plebeus;
Clientes e Escravos.
Em 509 a.C., uma revolta patrcia, apoiada
pelos plebeus, deps o rei etrusco de Roma (Tarqunio,
o Soberbo).


Please purchase PDF Split-Merge on www.verypdf.com to remove this watermark.
Incluso para a Vida Histria A

Pr-Vestibular da UFSC 4
A REPBLICA (509 a.C. 27 a.C.)
O Senado era o rgo com maior poder, composto por
300 senadores vitalcios
A populao pobre da cidade (os
plebeus) no possua quaisquer direitos polticos,
econmicos ou sociais. Durante as guerras de
conquistas, somente os patrcios eram privilegiados.
Os patrcios, interessados em conquistar o Lcio,
precisavam dos plebeus, propondo, assim, a criao de
um pretrio que protegesse a plebe: o Tribunato da
Plebe. A Lei das Doze Tbuas, criada em 450 a.C.,
constituiu um dos fundamentos do direito romano.

A Crise da Repblica: Foi o perodo iniciado com as
lutas dos irmos Graco contra o Estado romano
(Tibrio, que props uma lei de Reforma Agrria, e
Caio, responsvel por criar a lei frumentria, para
vender trigo a preo mais baixo ao povo), estendendo-
se at o conflito entre os cnsules Mrio e Sila.
Primeiro Triunvirato Dividiu o poder poltico de
Roma entre Pompeu, Crasso e Jlio Csar. Pompeu
ficou responsvel pelos governos de Roma e do
Ocidente; Crasso, pelo Oriente; Jlio Csar, pelas
Glias. Assassinato de Jlio Csar (em 44 a.C.).
Segundo Triunvirato Foi formado por Otvio,
Marco Antnio e Lpido que, em seguida, dividiram
o mundo romano para melhor govern-lo: Otvio ficou
com Roma e o Ocidente; Marco Antnio recebeu o
Oriente (Egito / Clepatra); Lpido ficou com a frica,
porm foi deposto em 36 a.C.
Otvio assumiu sozinho o poder, tornando-se senhor
absoluto de Roma. Recebeu do Senado vrios ttulos,
entre eles o de Imperador, o de Csar (primeiro
general) e o de Augusto (sagrado). Iniciava-se, assim, o
Imprio Romano.

IMPRIO (27 a.C. 476 d.C.)
Otvio Augusto desenvolve a poltica do po e
circo e a Pax Romana.

Dinastias:
1. Dinastia Jlio-Claudiana (do ano 14 ao 68).
2. Dinastia dos Flvios (do ano 69 ao 96).
3. Dinastia do Antoninos (do ano 96 so 192).
4. Dinastia dos Severos (do ano 193 ao 235).

Crise e Decadncia do Imprio: O imprio Romano,
no sculo III, foi afetado pela crise geral do
escravismo, iniciada nos reinados dos ltimos
Antoninos.
Desses fatores, resultou a diminuio da arrecadao
de tributos, levando o Estado a dificuldades de manter
a mquina administrativa, principalmente o exrcito, o
que culminou com as invases brbaras.
Em 313, Constantino assumiu o poder e
restabeleceu a unidade imperial. Estabeleceu, em 330,
a capital do Imprio Romano na antiga colnia grega
de Bizncio, rebatizada com o nome de
Constantinopla. Alm disso, instituiu o Edito de
Milo, no qual reconheceu a religio crist,
transformando-a na mais importante de Roma. Ainda
no sculo IV, os brbaros iniciaram as invases em
busca de terras frteis.
Teodsio foi o ltimo imperador uno,
instituindo o Edito de Tessalnica, em 380, pelo qual
a religio crist tornava- se oficial do Imprio.
Por ocasio da morte de Teodsio (395), o
Imprio foi divido em Ocidente, governado por
Honrio, e Oriente, governado por Arcdio ambos
filhos do Imperador.
O Imprio Romano decaiu em 476, invadido pelos
hunos.

Exerccios de Sala #

1. Sobre a religio grega, correto afirmar que:
01. Baseava-se em dogmas rigorosos e seus fieis
deveriam crer em verdades absolutas.
02. Cada cidade-Estado tinha sua divindade protetora.
04. Heris como Perseu, Jaso, dipo e Hrcules eram
divinizados.
08. As oraes dirigidas aos deuses imploravam
principalmente a salvao da alma dos homens.
16. As aventuras dos deuses e heris so narradas em
um conjunto de mitos, o qual se denomina
"Mitologia Grega".

2. O Imprio Romano expandiu-se pelo Mar
Mediterrneo durante o perodo republicano; isso
gerou, no decorrer do sculo II d. C., vrias
repercusses, entre as quais no podemos destacar.
01. surgimento da classe mdia de pequenos
proprietrios rurais e desaparecimento dos
latifundirios.
02. aumento da populao rural na Itlia e consequente
declnio da populao urbana.
04. crescimento do nmero de escravos e grande fluxo
de riquezas.
08. criao de grande nmero de pequenas
propriedades e fortalecimento do sistema
assalariado.
16. difuso do Cristianismo e proscrio das
manifestaes culturais de outras regies.

Tarefa Mnima #

3. A navegao e o comrcio martimo foram
desenvolvidos pelos gregos. Dentre os vrios fatores
que os levaram a isso, podemos citar como causa
inicial:
a) a pobreza do solo grego e a necessidade de novas
terras para suprir suas necessidades.
b) o desejo de difundir a cultura grega.
c) o fato de serem constantemente molestados por
povos brbaros.
d) o seu amplo conhecimento geogrfico e martimo
que despertava em seu povo a busca do
desconhecido.
e) o fato de os mercadores gregos precisarem de novos
mercados consumidores.

4. "A plis se faz pela autonomia da palavra, no mais
a palavra mgica dos mitos, palavra dada pelos deuses
e, portanto, comum a todos, mas a palavra humana do
Please purchase PDF Split-Merge on www.verypdf.com to remove this watermark.
Incluso para a Vida Histria A

Pr-Vestibular da UFSC
5
conflito, da discusso, da argumentao. O saber deixa
de ser sagrado e passa a ser objeto de discusso."
(M. Lcia de Arruda Aranha e M. Helena Pires Martins)

A partir do texto anterior, correto afirmar que:

01. o advento da plis e, portanto, da vida poltica,
estabelece uma possibilidade de ruptura com o
universo herico-mtico de explicaes das coisas
mundanas;
02. o nascimento da plis (sc. VIII e VII a.C.) coloca
na ordem do dia as discusses sobre os destinos dos
homens por eles mesmos e no mais por desgnios
de carter mtico;
04. a experincia poltica exigiu que as explicaes
mticas fossem afastadas e que a causa/razo das
coisas mundanas tivesse preexistncia;
08. a experincia poltica instaura, entre os gregos, o
uso da argumentao/razo como instrumento de
soluo de conflitos;
16. o nascimento da plis possibilita a recuperao do
saber mtico pela argumentao e reinstaura o
sagrado em detrimento da razo.

5. "Usamos a riqueza mais como uma oportunidade
para agir que como um motivo de vanglria; entre ns
no h vergonha na pobreza, mas a maior vergonha
no fazer o possvel para evit-la... olhamos o homem
alheio s atividades pblicas no como algum que
cuida apenas de seus prprios interesses, mas como um
intil... decidimos as questes pblicas por ns
mesmos, ou pelo menos nos esforamos por
compreend-las claramente, na crena de que no o
debate que o empecilho ao, e sim o fato de no se
estar esclarecido pelo debate antes de chegar a hora da
ao".

Esta passagem de um discurso de Pricles, reproduzido
por Tucdides, expressa:

a) os valores tico-polticos que caracterizam a
democracia ateniense no perodo clssico.
b) os valores tico-militares que caracterizaram a vida
poltica espartana em toda a sua histria.
c) a admirao pela frugalidade e pela pobreza que
caracterizou Atenas durante a fase democrtica.
d) o desprezo que a aristocracia espartana devotou ao
luxo e riqueza ao longo de toda a sua histria.
e) os valores tico-polticos de todas as cidades gregas,
independentemente de sua forma de governo.

6. Sobre a civilizao grega, afirma-se:
01. A Grcia se organizava politicamente em cidades-
Estado, sendo as mais influentes Esparta e Atenas.
02. Em 560 a.C., em Atenas, Psstrato tomou o poder
apoiado pelos pequenos proprietrios, dando incio
ao perodo das tiranias.
04. Em 509 a.C., em Atenas, Clstenes organizou um
governo baseado nos princpios da igualdade
poltica dos cidados e da participao de todos nas
decises do governo.
08. Esparta e Atenas entraram em choque, devido s
suas rivalidades polticas, econmicas e sociais,
numa guerra que ficou conhecida como Guerra
Mdica, cabendo a vitria a Atenas, que passou a
dominar toda a Grcia.

7. A expanso de Roma durante a Repblica, com o
consequente domnio da bacia do Mediterrneo,
provocou sensveis transformaes sociais e
econmicas, dentre as quais no podemos incluir:

01. marcado processo de industrializao, xodo
urbano, endividamento do Estado.
02. fortalecimento da classe plebeia, expanso da
pequena propriedade, propagao do cristianismo.
04. crescimento da economia agro-pastoril,
intensificao das exportaes, aumento do
trabalho livre.
08. enriquecimento do Estado romano, aparecimento
de uma poderosa classe de comerciantes, aumento
do nmero de escravos.
16. diminuio da produo nos latifndios, acentuado
processo inflacionrio, escassez de mo-de-obra
escrava.

8. O Edito de Milo (313), no processo de
desenvolvimento histrico de Roma, reveste-se de
grande significado, tendo em vista que:
a) combateu a heresia ariana, acabando com a fora
poltica dos bispados de Alexandria e Antioquia.
b) tornou o cristianismo a religio oficial de todo
Imprio Romano, terminando com a concepo de
rei-deus.
c) acabou inteiramente com os cultos pagos que ento
dominavam a vida religiosa.
d) deu prosseguimento poltica de Deocleciano de
intenso combate expanso do cristianismo.
e) proclamou a liberdade do culto cristo passando
Constantino a ser o protetor da Igreja.

9. A religio romana era essencialmente politesta, e o
culto ao imperador era de grande significado pelo fator
da unidade que representava. Durante um perodo
determinado, teve incio o questionamento dessa ideia.
Esse grupo, que no reconhecia a divindade do
Imperador, era de:
a) brbaros invasores
b) primeiros cristos
c) bons espritos familiares
d) escravos e estrangeiros
e) judeus vindos da Palestina

10. Vrias razes explicam as perseguies sofridas
pelos cristos no Imprio Romano, entre elas
a) a oposio religio do Estado Romano e a negao
da origem divina do Imperador, pelos cristos.
b) a publicao do Edito de Milo que impediu a
legalizao do Cristianismo e alimentou a
represso.
c) a formao de heresias como a do Arianismo, de
autoria do bispo rio, que negava a natureza divina
de Cristo.
d) a organizao dos Conclios Ecumnicos, que
visavam promover a definio da doutrina crist.
Please purchase PDF Split-Merge on www.verypdf.com to remove this watermark.
Incluso para a Vida Histria A

Pr-Vestibular da UFSC 6
e) o fortalecimento do Paganismo sob o Imperador
Teodsio, que mandou martirizar milhares de
cristos.

UNIDADE 3

ALTA IDADE MDIA

costume dividir o perodo medieval em duas grandes
fases: a Alta Idade Mdia, que se estende do sculo
V ao sculo XI e a Baixa Idade Mdia, do sculo XII
ao sculo XV.

FEUDALISMO
A histria da Idade Mdia Ocidental basicamente a
histria dos Reinos Brbaros que se formaram a partir
do sculo V, com a desintegrao do Imprio Romano
do Ocidente.
Os povos brbaros germnicos, ao invadirem a
Europa, trouxeram consigo sua prpria maneira de
viver, na qual a economia tipicamente agrria era um
dos traos fundamentais. Outra caracterstica
germnica muito importante foi o Comitatus, que
expressava o comprometimento recproco entre lderes
de tribos. Essa realidade em contato com o processo
da ruralizao do Imprio Romano, caracterizado pelo
surgimento de uma nova forma de organizao
econmica e social conhecida como Colonato, que
teve incio ainda no sculo III, fez com que o
alinhamento de foras rumo ao Feudalismo se tornasse
bastante claro. Sendo assim, podemos afirmar que as
grande parte das origens do feudalismo encontram-se
na unio do comitatus germnico com o colonato
romano.
Ao invadirem o Imprio Romano, os povos
germnicos apresentavam uma organizao tribal, na
qual a guerra era a atividade fundamental de todos os
homens livres; as atividades produtivas da terra
(agricultura e criao de animais) eram de
responsabilidade das mulheres e dos escravos. Dentre
as atividades agrrias, o pastoreio era a principal, e a
existncia de boas pastagens era condio de fixao
de uma tribo em determinado espao. Em outras
palavras, os povos germnicos eram seminmades:
fixavam-se em um local enquanto l existissem boas
pastagens para o seu rebanho.
Dada a condio de seminomadismo, fcil
entender que a terra era considerada como propriedade
comunal. S a partir de sua penetrao no Imprio
Romano que comearam a surgir as primeiras formas
de propriedade privada do solo; mesmo assim, esta
coexistia com a propriedade comunal. O que
basicamente aconteceu foi que as reas de pastagem
passaram a ser consideradas como propriedade privada,
enquanto as de cultivo continuaram sendo propriedades
comunitrias.
Em funo das constantes guerras na ao de
ocupao das terras do Imprio Romano, verificamos a
formao, nas tribos germnicas, de verdadeiras
nobrezas guerreiras, s quais todos os demais
elementos da tribo tendiam a se sujeitar.
A estrutura familiar dos germnicos era
tipicamente patriarcal, sendo que uma tribo era um
agregado de famlias. As tribos eram unidades
politicamente independentes e s se uniam em funo
da necessidade gerada por uma guerra ou para um fim
especfico; logo, no existia entre eles a ideia de Estado
centralizado.
As unies temporrias entre tribos eram
fundamentadas em obrigaes recprocas entre os seus
chefes, que se uniam, sendo que para uma tribo, a
autoridade do chefe era incontestvel; logo, as
obrigaes por ele assumidas eram vlidas para todos
os membros da tribo.
O Feudalismo, como qualquer modo de
produo, no surgiu de forma rpida. o resultado de
um longo processo de formao, que se estende do
sculo IV ao sculo IX, sendo que s a partir de ento
passou a ser o modo de produo dominante nas
formaes sociais europeias, perdurando at o sculo
XII. Entre o sculo XII e o sculo XVIII, o modo de
produo feudal conheceu profundas transformaes,
que resultaram na formao de outro modo de
produo, o Capitalismo.
Podemos afirmar que o Feudalismo surgiu
atravs de um processo de integrao de uma srie de
instituies romanas com uma srie de instituies
brbaras germnicas, sendo que esse processo
estrutural foi catalisado pela ao conjuntural de
diversos fatores, tais como o expansionismo
muulmano pelo Mediterrneo e as invases dos
normandos, hngaros e eslavos.
A sociedade feudal deve ser classificada como
sendo uma Sociedade Estamental, ou seja, uma
sociedade na qual seus membros esto hierarquizados
em funo do seu status (posio na sociedade),
sendo que o status de cada um era fixado pelo fato de
dever ou receber determinadas obrigaes. Uma
sociedade estamental tem como uma de suas
caractersticas fundamentais a de apresentar reduzidos
veculos de mobilidade social.
Notvel exemplo desse fato o de a Igreja
haver proibido o lucro e a usura e, em consequncia
disso, os poucos comerciantes que existiam no eram
cristos, (na maioria, eram judeus), pois assim no
havia o peso da interdio da Igreja em relao quelas
atividades.

EXPANSO ISLMICA
Maom (570 - 632) nasceu em Meca, membro de uma
famlia pobre da tribo coraixita, e foi responsvel pelo
surgimento de uma nova religio, o islamismo, que
garantiu a unidade poltica Arbia.
Sua doutrina condenava o politesmo idlatra, fonte de
disputas entre os rabes, e defendia o monotesmo
fundado na submisso a Al e na leitura rigorosa do
Coro, livro sagrado dos muulmanos.
Maom foi perseguido e expulso de Meca em
622 (incio do calendrio islmico), dirigindo-se para a
cidade de Yatreb, episdio conhecido como Hgira.
Em pouco tempo, Maom conquistou uma legio de
adeptos que, em 630, se dirigiu e conquistou Meca. Em
Please purchase PDF Split-Merge on www.verypdf.com to remove this watermark.
Incluso para a Vida Histria A

Pr-Vestibular da UFSC
7
632, o profeta Maom morreu e foi sucedido pelos
Califas (seguidores do profeta)..
A Expanso Islmica: Segundo os preceitos
islmicos, todo seguidor de Maom deve ser um
soldado encarregado de levar a f a todos os
infiis(djihad = Guerra Santa). Tal motivao levou
os rabes, comandados pelos califas, expanso por
vastas reas do Mediterrneo.
Durante quase mil anos, os rabes-muulmanos
controlaram a navegao e o comrcio no
Mediterrneo, bloqueando o acesso dos europeus ao
comrcio com o Oriente.
A Cultura Islmica: Cincias: campo em que os
muulmanos mais se desenvolveram; na matemtica,
aprimoraram a lgebra e a Geometria; dedicaram-se
tambm Astronomia e Qumica (alquimia);
x Medicina: grande foi a importncia de Avicena
que, entre vrias descobertas, diagnosticou a
varola e o sarampo e descobriu a natureza
contagiosa da tuberculose;
x Literatura: contamos com vasta produo, com
destaque para a coletnea As mil e uma noites e o
poema Rubaiyat, de Omar Khayam.

Exerccios de Sala #

1. O Feudalismo europeu apresentava caractersticas
particulares de acordo com a localidade. Apesar das
diferenas regionais, podemos afirmar que sua origem
est relacionada com
01. o renascimento das cidades.
02. o ressurgimento do comrcio.
04. a ruralizao da sociedade.
08. o fortalecimento do poder imperial.
16. a descentralizao poltica.

2. Sobre a forma de pensar e de agir do homem da
Europa Ocidental durante o perodo medieval,
correto afirmar que:
01. Deus ocupava o centro de todas as coisas,
condicionando pensamento e ao dos homens.
02. A histria dos homens consistia numa marcha do
povo de Deus em direo a Ele, cabendo Igreja o
papel de guia.
04. A relao entre o senhor e seus vassalos era de
dependncia pessoal, ou seja, de homem a homem.
08. Nas atividades econmicas, a influncia da Igreja
imps princpios que condenavam a especulao e a
usura, bem como gerou a ideia de justo preo.
16. O dinamismo da economia feudal se fundamentava
na concepo de que o comrcio era o nico
gerador de riquezas.

Tarefa Mnima #

3. So caractersticas do perodo conhecido como Alta
Idade Mdia Europeia:
01. Surgimento dos Feudos.
02. Centralizao poltica.
04. Incio da ocupao da Pennsula Ibrica pelos
muulmanos.
08. Formao do sistema capitalista.

4. (UFSC) So caractersticas do perodo conhecido
como Alta Idade Mdia Europeia:
01. Surgimento dos Feudos.
02. Centralizao poltica.
04. Incio da ocupao da Pennsula Ibrica pelos
muulmanos.
08. Formao do sistema capitalista.

5. (UFBA) Em relao organizao social da Idade
Mdia, pode-se afirmar:
01. A utilizao em grande escala da mo-de-obra
escrava, no Imprio Romano, modificou as
condies de trabalho dos camponeses, motivando
a ampliao das obrigaes do Estado, em
decorrncia do crescimento demogrfico das reas
urbanas.
02. A pregao religiosa de Maom atraiu
significativamente os rabes pobres, que a
utilizaram como motivao para uma guerra civil
contra os mercadores judeus da Arbia.
04. A estrutura agrcola da sociedade feudal
fundamentou-se na grande concentrao de terras e
no trabalho servil.
08. A atividade urbana, na sociedade feudal, resultava
da livre iniciativa dos banqueiros, artesos e
comerciantes e era regulada por uma rgida diviso
social, nas relaes de trabalho, definida pelo
Estado.
16. A Igreja, no perodo medieval, tentou mostrar a
harmonia da sociedade, formulando o princpio das
"trs ordens", segundo o qual todos eram iguais,
porque recebiam uma misso de Deus e a
cumpriam de forma fraterna.

UNIDADE 4

BAIXA IDADE MDIA

Chamamos de Baixa Idade Mdia o perodo que se estende
do sculo XII ao sculo XV. Durante essa fase, as
estruturas socioeconmicas da Europa Ocidental e Central
passaram por profundas mudanas. Tais transformaes
foram consequncia da unio de dois elementos bsicos: a
crise do Feudalismo e o reaquecimento das atividades
comerciais que, sculos depois, resultaram na
caracterizao plena do Modo de Produo Capitalista.
De uma produo voltada para a auto-
suficincia, passamos a verificar uma produo cada vez
mais voltada para o mercado. As trocas monetrias
comeam a substituir as trocas em espcie. Comeam a
surgir a organizao empresarial, o esprito de lucro e o
racionalismo econmico. Em suma, num linguajar tcnico,
o Modo de Produo Feudal vai perdendo sua dominncia
nas formaes sociais europeias em favor dos modos de
produo pr-capitalistas.
Do sculo V ao sculo X, a Europa,
convulsionada por uma srie de invases, (germnicos,
muulmanos, normandos, magiares e eslavos) viveu em
permanente estado de belicosidade. Esta realidade
provocava uma significativa elevao nos ndices de
Please purchase PDF Split-Merge on www.verypdf.com to remove this watermark.
Incluso para a Vida Histria A

Pr-Vestibular da UFSC 8
mortalidade e, nessa medida, funcionava como elemento
importante do no-crescimento demogrfico significativo
na Europa. Por volta do ano 1000, as invases cessaram e
tendeu-se a uma acomodao poltica e militar da Europa
em torno da vida dos feudos. Com isso, as taxas de
mortalidade diminuram e, consequentemente, a populao
cresceu. O aumento demogrfico no era acompanhado
pelo aumento da produo. Em termos prticos: passou a
ser difcil para os feudos manter a auto-suficincia de seus
habitantes. Por outro lado, a volta da paz fez com que
fosse restabelecida a segurana nas vias de comunicao e,
consequentemente, pudessem ser retomadas as trocas
inter-regionais na Europa.
Muitos servos eram expulsos dos Domnios ao
cometerem as menores infraes e um grande nmero de
viles comeou a deixar os feudos espontaneamente em
busca de melhores oportunidades, j que no dependiam
mais da proteo dos senhores feudais. Esses contingentes
populacionais tenderam a emigrar, provocando o
povoamento de novas reas, principalmente no leste
Europeu.
Com a escassez se feudos, nobres saam pela
estrada em busca de alguma oportunidade. A belicosidade
era a marca desse tempo de crise, sendo evidenciada, por
exemplo, atravs da proliferao dos torneios de cavalaria,
torneios nos quais os senhores se enfrentavam em
verdadeiras batalhas campais que duravam vrios dias.
Para conter a belicosidade da nobreza, a Igreja proclamou
a Paz de Deus, isto , a proteo aos cultivadores da terra,
aos viajantes e s mulheres. Essa medida foi reforada pela
Trgua de Deus, que limitava a noventa o nmero de dias
do ano em que se podia combater.

AS CRUZADAS
O nome Cruzadas dado a um conjunto expedies
militares de cristos do Ocidente que se dirigiram ao
Oriente com o objetivo de libertar o Santo Sepulcro das
mos dos muulmanos. Todavia, outros interesses tambm
estavam em jogo: expanso territorial e reabertura das
rotas comerciais do Mar Mediterrneo.
O Papa Urbano II, ao receber do Imperador bizantino
Alexandre Clemont-Ferrand um pedido de ajuda militar
contra os muulmanos, convocou, para 1095, o Conclio
de Clermont, no qual exortou os fiis par uma guerra santa
contra o Islo.

Principais Cruzadas:
Em 1096, partiram oficialmente os cavaleiros da Primeira
Cruzada (1096-1099). Seus chefes foram Roberto da
Normandia, Godofredo de Bulho, Balduno de Flandres,
Roberto II de Flandres, Raimundo de Tolosa, Boemundo
de Tarento e Tancredo, este um chefe normando do sul da
Itlia. Como se v, era uma Cruzada da nobreza, sem a
participao de um rei sequer.
A Terceira Cruzada (1189-192) foi organizada em
consequncia da conquista de Jerusalm pelo Sulto
Saladino, fato ocorrido em 1187. Esta expedio
conhecida como A Cruzada dos Reis, pelo fato de ter
sido chefiada por Ricardo Corao de Leo (Rei da
Inglaterra), Felipe Augusto (Rei da Frana) e Frederico
Barba Ruiva (Imperador do Sacro Imprio). O Papa
Inocncio III foi o grande pregador desta Cruzada.
A Quarta Cruzada (1202-1204), organizada pori
Henrique VI, Imperador do Sacro Imprio, que contou
com o apoio de diversos nobres franceses, tais como
Bonifcio de Mont-Ferrat e Balduno de Flandres. Aps
desavenas com o imperador do Imprio Bizantino, os
cruzados tomaram Constantinopla, cujo governo foi
entregue a Balduno de Flandres, tutelado pelos
venezianos. Satisfeitos com o saque de Constantinopla e
com o monoplio comercial para Veneza, os cruzados
abandonaram seus objetivos e voltaram para a Itlia.
Em 1212 foi organizada a chamada Cruzada das
Crianas, que consistiu em um exrcito formado por
jovens, que teria o objetivo de retomar Jerusalm. Os
cristo acreditavam que os jovens, inocentes e sem
pecados, conseguiriam, com a ajuda de Deus, vencer os
muulmanos. Esse exrcito aportou em Alexandria e os
jovens foram todos aprisionados e vendidos como
escravos.
Principais Consequncias:
De um modo geral, a expanso europeia contribuiu para
dinamizar as relaes comerciais entre o Oriente e o
Ocidente. Aps sculos do bloqueio muulmano, os
cruzados reabriram parcialmente o Mediterrneo para o
comrcio europeu.
O desenvolvimento dessas atividades
comerciais mediterrneas deu vida a vrios portos do
Ocidente, dentre os quais destacaremos os seguintes:
Gnova, Pisa, Npoles, Amalfi, Bari, Veneza e Marselha.
Atravs desse comrcio as mercadorias do
Oriente se espalharam por todo o mundo ocidental. O
contato estreito com as civilizaes bizantinas e
muulmanas despertou nos cristos do Ocidente um gosto
mais apurado e um maior refinamento no modo de vida.
Esse fato fez com que o mercado consumidor para
produtos orientais se visse ampliado. Os cristos
aprenderam tambm novas tcnicas de irrigao, de
fabricao de tecidos e de produo de ao. Outros
elementos importantes foram as prticas financeiras como
a letra de cmbio, o cheque e a contabilidade.
O renascimento das atividades comerciais
provocou o crescimentos das cidades, o desenvolvimento
de uma classe de comerciantes, a difuso do esprito de
lucro e o racionalismo econmico.

RENASCIMENTO ARTSTICO
CULTURAL E CIENTFICO

CARACTERSTICAS GERAIS
O Renascimento exprime, sobretudo, os novos valores
e ideais da burguesia, classe ascendente na transio
para o capitalismo.

Principais Caractersticas:
x Antropocentrismo;
x Racionalismo;
x Individualismo;
x Universalismo;
x Humanismo.




Please purchase PDF Split-Merge on www.verypdf.com to remove this watermark.
Incluso para a Vida Histria A

Pr-Vestibular da UFSC
9
FORAM FATORES DO RENASCIMENTO
O Renascimento Comercial e Urbano da Baixa Idade
Mdia, que alterou os valores da poca feudal e
favoreceu um maior intercmbio intelectual. O
mecenato , isto , a proteo aos escritores e artistas,
que muito estimulou o movimento renascentista. Os
primeiros mecenas pertenciam burguesia, mas houve
tambm papas, reis e prncipes que praticaram o
mecenato. A burguesia fazia-o como forma de
investimento financeiro ou para adquirir status; os
governantes, porm, tornavam-se mecenas com o
objetivo de aumentar seu prestgio e,
consequentemente, legitimar o novo poder que
estavam implantando: o absolutismo.
A inveno da imprensa, que
permitiu uma maior divulgao das novas
ideias.

PRINCIPAIS RENASCENTISTAS
x na Pintura: Leonardo da Vinci, Michelangelo,
Rafael e Ticiano, na Itlia; El Greco, na Espanha.
x na Escultura: Michelangelo e Donatello, na Itlia.
x na Literatura: Cames, em Portugal; Cervantes, na
Espanha; Rabelais e Montaigne, na Frana;
Shakespeare, na Inglaterra.
x na Astronomia: Coprnico, na Polnia; Kepler, na
Alemanha; Galileu, na Itlia.

Exerccios de Sala #

1. (UFSC) Os relatos sobre o perodo histrico
conhecido como Idade Mdia revelam a ocorrncia de
conflitos blicos, pestes e fome. Sabe-se, porm, que
no mesmo perodo houve desenvolvimento econmico
e cultural. Assinale a(s) proposio(es) correta(s) nas
suas referncias cultura medieval.

01. O carter religioso predominou nas artes
medievais, pois um dos seus objetivos era a
glorificao de Deus.
02. Na literatura, alm de Dante Alighieri, destacaram-
se os trovadores, responsveis pela divulgao da
poesia popular e das canes de gesta.
04. O crescimento urbano e o comrcio foram
responsveis pela decadncia intelectual verificada
na Idade Mdia, por dificultarem a criao de novas
universidades.
08. Entre os pensadores medievais, destacou-se Santo
Toms de Aquino que, com a "Suma Teolgica",
tentou resolver a controvrsia entre f e razo.
16. Na arquitetura medieval predominaram dois estilos:
o romnico e o gtico.
32. Durante a Idade Mdia, as lnguas nacionais foram
denominadas "vulgares". O latim foi a lngua falada
pelos eruditos.

2. (UFSC) Numa sexta-feira, 8 de agosto de 1998, dois
atentados aterrorizaram o mundo. Bombas explodiram
nas embaixadas dos Estados Unidos em Nairobi e Dar
es-Salaan, deixando 248 mortos. Os atentados foram
reivindicados pelo grupo "Exrcito de Libertao dos
Santurios Islmicos".

Sobre o Islo e os grupos islmicos fundamentalistas
que aterrorizam o ocidente, assinale a(s)
proposio(es) verdadeira(s).

01. O Islo surgiu a partir das pregaes de Maom.
02. No "Alcoro", que segundo a tradio foi
transmitido a Maom, esto as leis e ensinamentos
da religio islmica.
04. Os fundamentalistas islmicos pretendem um
Estado dirigido pelas leis do Alcoro.
08. Um nmero expressivo de fundamentalistas
islmicos prega a guerra santa contra a sociedade
ocidental, principalmente contra os Estados Unidos.

Tarefa Mnima #

3. (UEPG) Sobre a sociedade feudal, assinale o que for
correto.

01 Os direitos de suserania e soberania eram
igualmente partilhados por toda a classe senhorial.
02. As monarquias feudais caracterizaram-se pela
ruptura dos laos feudo-vasslicos e a emergncia
de um poder pessoal e supremo do soberano.
04. Em algumas regies da Europa Medieval ocorreu
uma sntese equilibrada e espontnea entre
elementos romanos e germnicos.
08. Foi marcada pela predominncia da vida urbana
sobre a rural.
16. Havia uma estreita relao entre laos de
dependncia pessoal e uma hierarquia de direitos
sobre a terra.

4. Apesar de no terem alcanado seu objetivo -
reconquistar a Terra Santa -, as Cruzadas provocaram
amplas repercusses, porque:

a) favoreceram a formao de vrios reinos cristos no
Oriente, o que permitiu maior estabilidade poltica
regio.
b) consolidaram o feudalismo, em virtude da
unificao dos vrios reinos em torno de um
objetivo comum.
c) facilitaram a superao das rivalidades nacionais
graas influncia que a Igreja ento exercia.
d) uniram os esforos do mundo cristo europeu para
eliminar o domnio rabe na Pennsula Ibrica.
e) estimularam as relaes comerciais do Oriente com
o Ocidente, graas abertura do Mediterrneo a
navios europeus.

5. (UNIOESTE) Com relao ao Perodo Medieval,
pode-se afirmar que:
01. o comeo da Idade Mdia se caracteriza por uma
rpida urbanizao da Europa Ocidental.
02. o romanismo, o germanismo e o cristianismo
contriburam para a formao da Civilizao
Ocidental e da Europa Medieval.
04. o Feudalismo foi um sistema social, poltico e
econmico voltado para a produo e para o
consumo locais.
08. os servos eram homens livres que vendiam sua
fora de trabalho e habitavam, normalmente, os
Please purchase PDF Split-Merge on www.verypdf.com to remove this watermark.
Incluso para a Vida Histria A

Pr-Vestibular da UFSC 10
subrbios das cidades e os mestres de ofcio
cultivavam a terra.
16. a cultura do perodo clssico foi preservada nos
mosteiros medievais, donde emanavam princpios
da mentalidade crist.
32. as Cruzadas ocorreram no incio do Feudalismo e
propiciaram a invaso dos povos brbaros de terras
dos germanos.

UNIDADE 5

IDADE MODERNA

REFORMA RELIGIOSA

FATORES
A doutrina da Igreja, atravs da teoria do preo justo,
da condenao da usura, do menosprezo s atividades
comerciais e manufatureiras, impedia o
desenvolvimento do capital.
Havia ainda uma profunda contradio entre
as necessidades econmicas dos diversos grupos
sociais e o fiscalismo, a simonia (venda de cargos
eclesisticos) e a venda de indulgncias (perdo para
os pecados), que a Igreja realizava como um Estado
opressor.

REFORMA LUTERANA
Martinho Lutero (1483 - 1546) era monge agostiniano,
professor de Teologia na Universidade de Wittemberg,
quando o Papa Leo X renovou a indulgncia para a
obteno de fundos necessrios construo da
Baslica de So Pedro. O descontentamento geral com
o Papado aumentou na Alemanha quando o frade
Tetzel l chegou para pregar a indulgncia.
Em 1517, Lutero publicou suas 95 Teses
onde, entre outras coisas, pregava que a salvao se
dava exclusivamente pela f, condenando a venda de
indulgncias. Por ter negado o pedido papal de
retratao, Lutero foi excomungado pelo Papa e
declarado fora da lei por Carlos V e pelo dito de
Worms, sendo, no entanto protegido pelo duque
Frederico da Saxnia. Em 1522, retornou a
Wittemberg, onde permaneceu at a morte.

REFORMA CALVINISTA
O movimento reformista ocorrido na Sua, de maior
profundidade e maior influncia, foi aquele liderado
por Calvino. Joo Calvino (1509 - 1564) era francs e
fez estudos humansticos. Aps sua converso ao
luteranismo, foi obrigado a fugir de seu pas para a
Sua em consequncia das perseguies religiosas a
que foi submetido. Um dos principais pontos do
Calvinismo a Teoria da Predestinao.

REFORMA ANGLICANA

A insatisfao com a Igreja era muito grande na
Inglaterra desde o fim do sculo XIV, quando Wyclif
(tradutor da Bblia para o Ingls) apresentou uma das
doutrinas precursoras do protestantismo.
O fator que desencadeou a Reforma na Inglaterra foi a
negativa do Papado em atender ao pedido de divrcio
do rei Henrique VIII (1509 - 1547), que era casado
com Catarina de Arago, tia de Carlos V.

CONTRA-REFORMA CATLICA
As reformas protestantes provocaram um movimento
de reforma interna na Igreja Catlica que inicialmente
decorreu de iniciativas isoladas como a mudana das
Regras das ordens religiosas, bem como a formao de
novas ordens como a dos Capuchinhos, das Ursulinas,
dos Barnabistas e dos Jesutas.
Os Jesutas (a Companhia de Jesus) foram
organizados por Incio de Loyola, autor de uma obra
intitulada Exerccios Espirituais, antigo oficial do
exrcito espanhol, sendo que sua organizao foi
aprovada pela Papa Paulo III em 1540.
A Inquisio, criada no perodo feudal para o
combate s heresias, foi muito utilizada na Espanha,
desde o sculo XV, contra os mouriscos e os judeus.
Para o combate aos protestantes, ela foi restabelecida,
em 1542, como rgo oficial da Igreja, dirigida de
Roma pelo Santo Ofcio, que era um rgo presidido
pelo Grande Inquisidor.
Em 1543, a Igreja criou outro rgo, a
Congregao do Index, que recebeu a funo de
examinar todas as obras que viessem a ser publicadas,
editando uma relao peridica dos livros considerados
perigosos doutrina e moral dos fiis.
O Conclio de Trento (1545 - 1563) foi convocado pelo
Papa Paulo III para garantir a unidade da f catlica e
da Igreja.

Exerccios de Sala #

1. Sobre o Tratado de Tordesilhas, assinado em 7 de
junho de 1494, pode-se afirmar que no objetivava:

01. demarcar os direitos de explorao dos pases
ibricos, tendo como elemento propulsor o
desenvolvimento da expanso comercial martima.
02. estimular a consolidao do reino portugus, por
meio da explorao das especiarias africanas e da
formao do exrcito nacional.
04. impor a reserva de mercado metropolitano, por
meio da criao de um sistema de monoplios que
atingia todas as riquezas coloniais.
08. reconhecer a transferncia do eixo do comrcio
mundial do Mediterrneo para o Atlntico, depois
das expedies de Vasco da Gama s ndias.
16. reconhecer a hegemonia anglo-francesa sobre a
explorao colonial, aps a destruio da
Invencvel Armada de Felipe II, da Espanha.

2. Assinale as datas corretas.
01. 1212 - Bartolomeu Dias dobrou o Cabo da Boa
Esperana
02. 1492 - Colombo chegou ao continente americano
04. 1494 - Assinatura do Tratado de Tordesilhas entre
Portugal e Espanha
08. 1500 Pedro A. Cabral chegou na Amrica
16. 1532 - So Vicente foi fundada por Martin Afonso
de Sousa
Please purchase PDF Split-Merge on www.verypdf.com to remove this watermark.
Incluso para a Vida Histria A

Pr-Vestibular da UFSC
11
Tarefa Mnima #

3. A respeito da cultura grega, leia as snteses
filosficas abaixo.

I - A cincia, a moral e os credos religiosos eram
criaes humanas vlidas para determinados grupos
sociais em um determinado perodo.
II - Sua principal contribuio filosfica foi a Teoria
das Ideias, segundo a qual as ideias so a essncia
dos conceitos e das coisas e, portanto,
transcendentes ao homem, que delas tem apenas um
plido reflexo.
III - Defendia a existncia de um conhecimento estvel
e vlido para todos. Sua grande preocupao era o
autoconhecimento que poderia ser obtido atravs da
ironia e da maiutica.

As snteses que voc acabou de ler podem ser
associadas, respectivamente, a:

a) Plato, Aristteles e Scrates
b) Plato, Sofistas e Aristteles
c) Scrates, Sofistas e Plato
d) Sofistas, Plato e Scrates
e) Plato, Sofistas e Scrates

4. Assinale o que for correto a respeito da mulher na
sociedade democrtica ateniense, na Antiguidade.

a) Alm de cuidar da administrao interna das
residncias, cabia mulher fazer pessoalmente as
compras no mercado.
b) Naquela organizao social, a mulher estava
excluda da cidadania que era reservada aos
homens.
c) A mulher podia ser repudiada pelo marido desde que
este apresentasse motivo justo e devolvesse o dote
ao pai da esposa.
d) As mulheres passavam boa parte do tempo fora de
casa, nos locais pblicos com amigas e mesmo
estabelecendo relaes ntimas com outros homens.
e) No espao do lar a mulher exercia o poder, cabendo
a ela decidir pela rejeio ou no dos filhos recm-
nascidos.

5. A Civilizao Grega atingiu extraordinrio
desenvolvimento. Os ideais gregos de liberdade e a
crena na capacidade criadora do homem tm
permanente significado. Acerca do imenso e
diversificado legado cultural grego, correto afirmar
que:
a) a importncia dos jogos olmpicos limitava-se aos
esportes.
b) a democracia espartana era representativa.
c) a escultura helnica, embora desligada da religio,
valorizava o corpo humano.
d) os atenienses valorizavam o cio e desprezavam os
negcios.
e) poemas, com narraes sobre aventuras picas, so
importantes para a compreenso do perodo
homrico.

UNIDADE 6

EXPANSO MARTIMO-COMERCIAL
EUROPEIA

A necessidade de metais preciosos para a cunhagem de
moedas, indispensveis ao desenvolvimento comercial,
bem como de novas reas fornecedoras de mercadorias
que abastecessem o mercado europeu, determinaram a
expanso martima a partir do sculo XV.
Sua viabilizao foi favorecida por diversos
fatores, entre os quais se destacam: o avano
tecnolgico, responsvel pela melhoria das condies
de navegao (elaborao de mapas, aprimoramento de
instrumentos de orientao, construo de embarcaes
mais rpidas e seguras).
Nesse processo de expanso, Portugal
desempenhou papel pioneiro por ter, durante a Baixa
Idade Mdia, criado as condies necessrias sua
efetivao:

x privilegiada posio geogrfica;
x desenvolvimento das tcnicas de navegao,
sobretudo aps a fundao da Escola de Sagres;
x presena de uma burguesia forte e com
disponibilidade de capitais para a empresa
martima;
x paz interna e externa;
x centralizao poltica em mos do rei.
A conquista de Ceuta pelos portugueses, em 1415,
considerada o marco inicial da expanso ultramarina
europeia. O Tratado de Tordesilhas assegurou a
presena portuguesa no recm-descoberto continente
americano.
Com o objetivo de consolidar o domnio
lusitano sobre a rota das especiarias orientais, o rei D.
Manuel organizou uma poderosa esquadra que se
dirigiu s ndias, percorrendo a rota inaugurada por
Vasco da Gama. A esquadra contava com duas
caravelas, dez naus e 1500 homens e era comandada
pelo navegador Pedro lvares Cabral. A embarcao
em que se achava o comandante, porm, afastou-se da
costa africana em direo a oeste e, a 22 de abril de
1500, avistou terra. Aps rpido desembarque,
suficiente para oficializar a posse sobre o novo
territrio, Cabral seguiu viagem em direo ao Oriente.
Uma nau, no entanto, retornou a Portugal para dar a
notcia da descoberta ao rei.
Os navegadores das outras naes europeias,
que no Portugal e Espanha, tiveram que se contentar
em explorar o Atlntico Norte, sendo responsveis pela
explorao e ocupao da Amrica do Norte. A
pirataria foi tambm uma atividade desempenhada
pelos ingleses e franceses.

ABSOLUTISMO
As caractersticas do Estado absolutista foram:
x a grande centralizao representada pelo grande
poder do soberano, que no era controlado por outras
instituies polticas ou por leis limitativas de sua
autoridade.
Please purchase PDF Split-Merge on www.verypdf.com to remove this watermark.
Incluso para a Vida Histria A

Pr-Vestibular da UFSC 12
x a poltica econmica mercantilista, que intervinha
na estrutura econmica sob diversas formas, ampliou o
estabelecimento de relaes capitalistas de produo e
foi um dos aspectos da acumulao primitiva de
capital.

Os principais pensadores do poder absoluto foram:
x Nicolau Maquiavel (1469-1527) - em suas obras O
Prncipe e Discursos sobre a primeira dcada de Tito
Lvio, fundamentava a necessidade de um Estado
Nacional forte e independente da Igreja e encarnado na
pessoa do chefe do governo (o prncipe) para a
aplicao da razo do Estado, fortalecimento da nao
e o benefcio coletivo, considerando vlidos todos os
meios utilizados para o alcance desses objetivos.
x Jean Bodin (1530-1595) - em Da Repblica,
argumentava que a soberania do Estado personificada
no rei tinha origem divina, no havendo impedimento
autoridade real.
x Bossuet (1627-1704) - Poltica Tirada da Sagrada
Escritura reforou a doutrina do direito divino, que
legitima qualquer governo, justo e injusto;
x Thomas Hobbes (1588-1679) - no Leviat (1651),
abandonou a ideologia religiosa para justificar o
absolutismo.

Exerccios de Sala #

1. Sobre as caractersticas do Absolutismo na Idade
Moderna correto afirmar que

01. foi um tipo de regime republicano e democrtico.
02. procurou legitimar-se no "Direito Divino dos Reis".
04. foi a expresso do poder poltico descentralizado.
08. implementou o Estado burocrtico e nacional.
16. baseou-se no poder autocrtico do soberano.

2. Jacques Bossuet utilizou argumentos extrados da
Bblia para justificar o poder absoluto e de direito
divino da realeza, com o lema: "Um rei, uma lei, uma
f". So caractersticas do absolutismo na Frana:

01. A concentrao dos mecanismos de governo nas
mos do rei.
02. A identificao entre Nao e Coroa.
04. A influncia do racionalismo iluminista como
justificativa do poder absoluto e do "direito divino".
08. A criao de exrcito nacional permanente.
16. A ampla liberdade de expresso e de f.

Tarefa Mnima #

3. No conjunto de importantes viagens e expedies
martimas dos sculo XVI, as quais chamamos de
"Grandes Navegaes", nota-se clara preponderncia
dos pases Ibricos. A esse respeito, correto afirmar:

01. As navegaes do perodo se faziam com recurso
exclusivo bssola, uma vez que ainda no se
havia iniciado o estudo da navegao astronmica,
isto , orientada atravs da observao dos astros.
02. As embarcaes adotadas pelos portugueses e
espanhis - as galeras - eram semelhantes quelas
utilizadas pelos navegantes genoveses e
venezianos.
04. Por sua localizao geogrfica, Portugal tornava-se
particularmente indicado para promover
exploraes martimas: seu litoral se encontra a
meio caminho entre o Mediterrneo e o Mar do
Norte, e bastante prximo da costa africana e das
ilhas atlnticas.
08. Tanto Portugal quanto Espanha podiam contar com
o apoio financeiro de vrios comerciantes s
expedies, interessados em reatar relaes diretas
com o Oriente desde a queda de Constantinopla
(1453).
16. A Espanha entrou com relativo atraso na disputa
com os portugueses pela descoberta de novas terras,
em funo de sua luta contra os muulmanos pela
reconquista de territrios ibricos.
32. A precoce centralizao monrquica, a
consolidao do poder central e a aliana com uma
nova classe mercantil possibilitam a Portugal desde
o incio do sculo XV estimular a expanso
comercial e as expedies martimas.

4. O descobrimento do Brasil foi parte do plano
imperial da Coroa Portuguesa, no sculo XV. Embora
no houvesse interesse especfico de expanso para o
Ocidente:
a) a posse de terras no Atlntico ocidental consolidava
a hegemonia portuguesa neste Oceano.
b) o Brasil era uma alternativa mercantil ao comrcio
portugus no Oriente.
c) o desvio da esquadra de Cabral seguia a mesma
inspirao de Colombo para chegar s ndias.
d) a procura de terras no Ocidente foi uma reao de
Portugal ao Tratado de Tordesilhas, que o afastava
da Amrica.
e) essa descoberta foi mero acaso, provocado pelas
intempries que desviaram a esquadra da rota da
ndia.

5. Principalmente a partir do sculo XVI vrios autores
passam a desenvolver teorias, justificando o poder real.
So os legistas que, atravs de doutrinas leigas ou
religiosas, tentam legalizar o Absolutismo. Um deles
Maquiavel: afirma que a obrigao suprema do
governante manter o poder e a segurana do pas que
governa. Para isso deve usar de todos os meios
disponveis pois que "os fins justificam os meios."
Professou suas ideias na famosa obra:

a) "Leviat"
b) "Do Direito da Paz e da Guerra"
c) "Repblica"
d) "O Prncipe"
e) "Poltica Segundo as Sagradas Escrituras"








Please purchase PDF Split-Merge on www.verypdf.com to remove this watermark.
Incluso para a Vida Histria A

Pr-Vestibular da UFSC
13
UNIDADE 7

REVOLUO INDUSTRIAL

FATORES
A Revoluo Industrial um processo histrico de
radical transformao econmica e social, atravs do
qual o modo de produo capitalista assumiu a
dominncia de certas formaes sociais.
Para o desencadeamento da Revoluo Industrial,
certas pr-condies tiveram de ser preenchidas:
x a substituio do processo artesanal de
produo pelo processo mecnico exige a realizao de
um significativo investimento e uma considervel
imobilizao inicial de capital. Logo, necessria a
preexistncia desse capital acumulado.
x a Revoluo Industrial demanda um crescente
consumo de mo de obra urbana. Neste sentido, a
existncia de abundante disponibilidade de mo de
obra condio fundamental para a ocorrncia do
prprio processo.
Uma Balana Comercial altamente favorvel e
a abundncia dos metais preciosos eram os indcios da
prosperidade. Essa hegemonia martimo-comercial da
Inglaterra conferia-lhe uma condio singular em
termos de acumulao de capital. Por exemplo, a essa
hegemonia a Inglaterra deve o fato de haver podido
assinar com Portugal, em 1703, o Tratado de Methuen,
em funo do qual uma grande parte do ouro explorado
no Brasil, no sculo XVIII, foi acabar nos cofres
ingleses.

ASPECTOS TECNOLGICOS
O aparecimento das mquinas no significa apenas um
progresso tcnico, atravs do qual se verificou um
aumento da produtividade. A introduo das mquinas
na produo industrial significou uma substituio do
tipo de equipamento que era utilizado at ento, ou
seja, as ferramentas, e uma liberao da mo de obra.

ASPECTOS ECONMICOS E SOCIAIS
x a Burguesia Capitalista, que a classe dos
proprietrios dos meios de produo.
x o Proletariado, que a classe que rene os
trabalhadores diretos, cuja nica propriedade a sua
fora de trabalho, vendida Burguesia Capitalista em
troca de um salrio.

Exerccios de Sala #

1. Entre os efeitos da Revoluo Industrial ocorrida em
meados do sculo XVIII, pode-se incluir:

01. a afirmao do Estado liberal-burgus, triunfante
sobre o Antigo Regime.
02. a diviso tcnica do trabalho, permitindo celeridade
no processo produtivo.
04. a configurao da dicotomia bsica das sociedades
capitalistas: burguesia e proletariado.
08. o acirramento da luta de classes, com ecloso de
movimentos contestatrios ordem burgus-
capitalista.
16. a consolidao da concepo metalista,
estimulando a procura e a posse de metais preciosos
como fator de riqueza.
32. o incentivo ao protecionismo estatal, visando a
alcanar um supervit comercial necessrio
proteo da indstria nascente.
64. a acumulao de capitais na esfera da produo de
mercadorias e consequente hegemonia poltica e
social da burguesia.

2. A era da industrializao na Europa foi
acompanhada por transformaes no processo de
trabalho, entre as quais podemos citar:

01. Passagem do sistema domstico ao sistema fabril
de produo.
02. Concentrao de trabalhadores em unidades fabris,
desenvolvendo a diviso social do trabalho e a
especializao em determinados ramos de
produo.
04. Manuteno da estrutura corporativa de trabalho,
organizando os trabalhadores em corporaes de
ofcios.
08. Promoo de um novo modelo de trabalho, pronto
a aceitar a disciplina do trabalho fabril e
constituindo mo de obra assalariada.
16. Utilizao frequente de mo de obra feminina e
infantil, submetida ao mesmo regime de trabalho,
durante longas jornadas.

Tarefa Mnima #

3. Sobre a inovao tecnolgica no sistema fabril na
Inglaterra do sculo XVIII, correto afirmar que ela

a) foi adotada no somente para promover maior
eficcia da produo, como tambm para realizar a
dominao capitalista, na medida que as mquinas
submeteram os trabalhadores a formas autoritrias
de disciplina e a uma determinada hierarquia.
b) ocorreu graas ao investimento em pesquisa
tecnolgica de ponta, feito pelos industriais que
participaram da Revoluo Industrial.
c) nasceu do apoio dado pelo Estado pesquisa nas
universidades.
d) deu-se dentro das fbricas, cujos proprietrios
estimulavam os operrios a desenvolver novas
tecnologias.
e) foi nica e exclusivamente o produto da genialidade
de algumas geraes de inventores, tendo sido
adotada pelos industriais que estavam interessados
em aumentar a produo e, por conseguinte, os
lucros.

4. A Revoluo Industrial Inglesa s foi possvel pelo
processo histrico de acumulao primitiva criador
tanto do CAPITAL quanto do TRABALHO. A
liberao da mo de obra e formao do proletariado
ocorreu com:

a) os cercamentos dos campos e a expulso dos
camponeses das terras comuns.
b) o intenso cultivo de algodo nos campos ingleses.
c) o processo de reforma agrria na Inglaterra.
Please purchase PDF Split-Merge on www.verypdf.com to remove this watermark.
Incluso para a Vida Histria A

Pr-Vestibular da UFSC 14
d) o intenso processo de imigrao de trabalhadores de
outras naes europeias para as indstrias inglesas.
e) a produo agrcola organizada em tcnicas feudais.

5. Dentre as consequncias sociais forjadas pela
Revoluo Industrial, pode-se mencionar

a) o desenvolvimento de uma camada social de
trabalhadores, que destitudos dos meios de
produo, passaram a sobreviver apenas da venda
de sua fora de trabalho.
b) a melhoria das condies de habitao e
sobrevivncia para o operariado, proporcionada
pelo surto de desenvolvimento econmico.
c) a ascenso social dos artesos que reuniram seus
capitais e suas ferramentas em oficinas ou
domiclios rurais dispersos, aumentando os ncleos
domsticos de produo.
d) a criao do Banco da Inglaterra, com o objetivo de
financiar a monarquia e ser tambm, uma
instituio geradora de empregos.
e) o desenvolvimento de indstrias petroqumicas
favorecendo a organizao do mercado de trabalho,
de maneira a assegurar emprego a todos os
assalariados.

UNIDADE 8

REVOLUO FRANCESA 1789-1799

A Revoluo Francesa transformou a poltica do
mundo moderno. Os motivos que desencadearam seu
incio se originam na prpria estrutura social da Frana
durante a Idade Moderna. Nesse perodo a sociedade
dividia-se em trs estados:

x 1 Estado - Clero
x 2 Estado Nobreza
x 3 Estado Uma mistura heterognia onde eram
encontrados burgueses, sans-culottes, camponeses
entre outros.

Tanto o primeiro quanto o segundo estado eram
isentos do pagamento de impostos, cabendo ao terceiro
estado patrocinar a riqueza do clero e da nobreza.
O absolutismo monrquico mantinha o
terceiro estado completamente afastado das decises
polticas da Frana.
O aumento demogrfico no fora
acompanhado do aumento na produo de alimentos, o
que resultou em alta dos preos. Com isso os setores
mais empobrecidos da populao passaram enfrentar
um temido inimigo: a fome.
Diante das reclamaes constantes do terceiro
estado, o rei Lus XVI convocou a Assembleia dos
Estados Gerias.






ETAPAS DA REVOLUO

ASSEMBLEIA NACIONAL CONSTITUINTE
(1789/1791)

Por determinao do Rei, ao longo do processo
eleitoral para os Estados Gerais, foram elaborados os
chamados Cadernos de Queixas, nos quais os
representantes dos trs estados sociais registravam suas
pretenses e reivindicaes.
Diante desses fatos, o Terceiro Estado, sob a
liderana da burguesia, tomou a iniciativa: em 13 de
julho foi organizada uma milcia popular que recebeu o
nome de Guarda Nacional e foi organizado um Comit
Permanente de direo da insurreio (este comit
daria origem Comuna de Paris); em 14 de julho de
1789, aps intensas manifestaes de rua com forte
apoio popular, o Terceiro Estado, atravs de seus
deputados e na liderana de um movimento popular,
marchou sobre a priso da Bastilha que era um
verdadeiro smbolo do Absolutismo e de suas
arbitrariedades, sendo que aps vrias horas de stio, a
fortaleza capitulou.
Foi aprovada uma Declarao dos Direitos do
Homem e do Cidado, que proclamava a liberdade e
igualdade de todos diante da lei.

MONARQUIA CONSTITUCIONAL (1791/1792)
Os Jacobinos, que representavam a esquerda poltica e
eram partidrios de uma democracia burguesa; Diante
dessa estrutura poltica da Assembleia Legislativa, o rei
Lus XVI apoiava os Jacobinos de Brissot na esperana
de que a poltica extremista deles levasse a Frana
guerra e catstofre e, com isso, ficasse viabilizada a
contrarrevoluo.
Os sans-culottes, articulados em torno da
Comuna Insurreicional, pressionaram a Assembleia,
que se viu obrigada a votar a suspenso do Rei e a
convocar eleies, por sufrgio universal, para uma
nova Constituinte, que receberia a designao de
Conveno Nacional. Enquanto a Conveno no foi
instalada, o poder executivo foi exercido por um
Conselho Executivo Provisrio, onde se destacou a
figura de Danton.

A REPBLICA JACOBINA (1792/1795)
O primeiro ato importante da Conveno Nacional foi
tomado em 21 de setembro de 1792: a abolio da
Monarquia.
Os deputados da Conveno agregavam-se
nos seguintes partidos:
- os Girondinos (160 deputados), que eram partidrios
da legalidade e da liberdade econmica, pretendiam
limitar a influncia do povo de Paris, cuja ao
julgavam excessivamente radical.
- os Montanheses (140 deputados) que se apoiavam,
basicamente, nos sans-culottes (a maior parte desses
deputados havia sido eleita pelos votos de Paris),
defendiam uma forte radicalizao do processo
revolucionrio e seus principais lderes eram Carnot,
Saint-Just, Marat, Danton e Robespierre.
Please purchase PDF Split-Merge on www.verypdf.com to remove this watermark.
Incluso para a Vida Histria A

Pr-Vestibular da UFSC
15
- Centro (tambm conhecido como Plancie ou Pntano
e que agregava o restante dos deputados) inicialmente
apoiava os Girondinos, mas aos poucos, tendeu para os
Montanheses.
Por presso dos Montanheses, foi
desencadeado um processo contra o Rei que, apesar da
oposio dos Girondinos, acabou por condenar o
soberano morte. Lus XVI foi executado em 21 de
janeiro de 1793.
O pice das medidas de exceo foi atingido
em junho de 1794 com o chamado Grande Terror. Em
menos de trs meses, cerca de duas mil pessoas foram
executadas, dentre elas o poeta Andr Chenier e o
qumico Lavoisier.
Quando, em 8 do Termidor (06 de julho de
1794), Robespierre anunciou que faria uma nova
depurao da Conveno e nos Comits, Tallien e
Fouch, dois lderes moderados, conseguiram reunir
em torno de si a maioria dos deputados da Plancie e,
em 9 do Termidor (27 de julho), conseguiram fazer
com que a Conveno aprovasse a priso de
Robespierre, sendo executado no dia seguinte.

GOVERNO DO DIRETRIO (1795/1799)
Napoleo, a frente de trinta e oito mil soldados, seguiu
para o Egito em maio de 1798, tomou Alexandria e o
Cairo, mas, em agosto, o Almirante Nelson, da
Inglaterra, destruiu a frota francesa na Batalha de
Aboukir. Em agosto de 1799, Napoleo deixou o
comando das tropas de ocupao para Kleber e
embarcou secretamente para a Frana.
Os golpistas precisavam de um militar de
prestgio para que o golpe contasse com o apoio do
exrcito, encontraram-no na pessoa de Napoleo
Bonaparte.
Em 18 Brumrio (09 de novembro de 1799),
foi desfechado o golpe de Estado que conhecido pelo
nome de Golpe do 18 Brumrio: Bonaparte foi
nomeado comandante das tropas de Paris e os trs
diretores, que se mantinham fiis ao regime,
neutralizados. No dia seguinte, a resistncia do
Conselho dos Quinhentos foi quebrada graas ao
conjugada de seu presidente (Luciano Bonaparte,
irmo de Napoleo) e das tropas de Paris.
O Diretrio foi suprimido e substitudo por
trs cnsules provisrios: Bonaparte, Sieys e Roger
Ducos.

Exerccios de Sala #

1. A Revoluo Francesa representou um marco da
histria ocidental pelo carter de ruptura em relao ao
Antigo Regime. Dentre as caractersticas da crise do
Antigo Regime, na Frana, no podemos incluir:

01. a crescente mobilizao do Terceiro Estado,
liderado pela burguesia contra os privilgios do
clero e da nobreza.
02. o desequilbrio econmico da Frana, decorrente da
Revoluo Industrial.
04. a retomada da expanso comercial francesa,
liderada por Colbert.
08. o apoio da monarquia s sucessivas rebelies
camponesas contrrias nobreza.
16. o fortalecimento da monarquia dos Bourbons, aps
a participao vitoriosa na guerra de independncia
dos E.U.A.

2. A "Declarao dos Direitos do Homem e do
Cidado", da Revoluo Francesa, traz o seguinte
princpio: "Os homens nascem e se conservam livres e
iguais em direitos. As distines sociais s podem ter
por fundamento o proveito comum". Tal princpio no
decorrente:

01. da incorporao das reivindicaes da classe mdia
por maior participao na vida poltica.
02. do reconhecimento da necessidade de assegurar os
direitos dos vencidos, sem distino de classes.
04. da incorporao dos camponeses comunidade dos
cidados com direitos sociais e polticos
reconhecidos na lei.
08. da crena popular na perspectiva liberal burguesa
de que a Revoluo fora feita por todos e em
benefcio de todos.
16. da determinao burguesa de levar avante um
processo revolucionrio de distribuio da
propriedade privada.

Tarefa Mnima #

3. A Constituio da Frana de 1791, a partir dos
princpios preconizados por Montesquieu, consagrou,
como fundamento do novo regime,

a) a subordinao do Judicirio ao Legislativo, que
passou a exercer um poder fiscalizador sobre os
tribunais.
b) a identificao da figura do monarca, com a do
Estado, que a partir desse momento se tornou
inviolvel.
c) a supremacia do Poder Legislativo, deixando de ser
o rei investido de poder moderador.
d) o poder de veto monrquico, que se restringiu a
assuntos fiscais, limitando, assim, a soberania
popular.
e) a separao dos poderes at ento concentrados,
teoricamente, na pessoa do soberano.

4. Do ponto de vista social, pode-se afirmar, sobre a
Revoluo Francesa, que:

a) teve resultados efmeros, pois foi iniciada, dirigida e
apropriada por uma s classe social, a burguesia,
nica beneficiria da nova ordem.
b) fracassou, pois, apesar do terror e da violncia, no
conseguiu impedir o retorno das foras scio-
polticas do Antigo Regime.
c) nela coexistiram trs revolues sociais distintas:
uma revoluo burguesa, uma camponesa e uma
popular urbana, a dos chamados sans-culottes.
d) foi um fracasso, apesar do sucesso poltico, pois, ao
garantir as pequenas propriedades aos camponeses,
atrasou, em mais de um sculo, o progresso
econmico da Frana.
Please purchase PDF Split-Merge on www.verypdf.com to remove this watermark.
Incluso para a Vida Histria A

Pr-Vestibular da UFSC 16
e) abortou, pois a nobreza, sendo uma classe coesa,
tanto do ponto de vista da riqueza, quanto do ponto
de vista poltico, impediu que a burguesia a
conclusse.

5. Na Revoluo Francesa, foi uma das principais
reivindicaes do Terceiro Estado:

a) a manuteno da diviso da sociedade em classes
rigidamente definidas.
b) a concesso de poderes polticos para a nobreza,
preservando a riqueza dessa classe social.
c) a abolio dos privilgios da nobreza e instaurao
da igualdade civil.
d) a unio de poderes entre Igreja e Estado, com
fortalecimento do clero.
e) o impedimento do acesso dos burgueses s funes
polticas do Estado.

UNIDADE 9

IDADE CONTEMPORNEA

Imperialismo; o novo colonialismo partilha
frica e sia
A colonizao portuguesa e espanhola do sculo XVI
havia se limitado Amrica. Com raras excees, as
terras africanas e asiticas no foram ocupadas. Ali, os
europeus limitaram-se ao comrcio, principalmente o
de especiarias. Por isso, no sculo XIX, havia grandes
extenses de terras desconhecidas nos dois continentes.
Comeou ento nova corrida colonial de outras
potncias europeias, sobretudo as que haviam passado
por uma transformao industrial, como Inglaterra,
Blgica, Frana, Alemanha e Itlia.

Os motivos do neocolonialismo
No sculo XVI, o objetivo colonialista era encontrar
metais preciosos e mercados abastecedores de produtos
tropicais e consumidores de manufaturas europeias.
Todavia, os motivos que geraram o renascimento
colonialista do sculo XIX so mais. Havia, sobretudo,
interesses econmicos; mas a eles se juntaram outros,
sociais, polticos e at religiosos e culturais.
Nessa poca, vrios pases europeus passavam
pela Revoluo Industrial. Precisavam encontrar fontes
de matria-prima (carvo, ferro, petrleo) e de produtos
alimentcios que faltavam em suas terras. Tambm
precisavam de mercados consumidores para seus
excedentes industriais, alm de novas regies para
investir os capitais disponveis construindo ferrovias ou
explorando minas, por exemplo.
Tal mecanismo era indispensvel para aliviar
a Europa dos capitais excedentes. Se eles fossem
investidos na Europa, agravariam a Grande Depresso
e intensificariam a tendncia dos pases europeus
industrializados de adotar medidas protecionistas,
fechando seus mercados e tornando a situao ainda
mais difcil. Some-se a tudo isso o crescimento
acelerado da populao europeia, necessitada de novas
terras para estabelecer-se.
No plano poltico, cada Estado europeu estava
preocupado em aumentar seus contingentes militares,
para fortalecer sua posio entre as demais potncias.
Possuindo colnias, disporiam de mais recursos e mais
homens para seus exrcitos. Tal era a poltica de
prestgio, caracterstica da Frana, que buscava
compensar as perdas na Europa, especialmente a
Alscia-Lorena, para os alemes. Ter colnias
significava ter portos de escala e abastecimento de
carvo para os navios mercantes e militares
distribudos pelo planeta.
Os missionrios religiosos desejavam
converter africanos e asiticos. Havia gente que
considerava um dever dos europeus difundir sua
civilizao entre povos que julgavam primitivos e
atrasados. Tratava-se mais de um pretexto para
justificar a colonizao. importante notar o
desenvolvimento de ideologias racistas que, partindo
das teorias de Darwin, afirmavam a superioridade da
raa branca.

A Europa ocupa tudo
Em 1914, 60% das terras e 65 % da populao do
mundo dependiam da Europa. Suas potncias tinham
anexado 90% da frica, 99% da Oceania e 56% da
sia.

A partilha
Os continentes asitico e africano no foram
partilhados de forma igual entre as principais naes
europeias. Enquanto pases como Frana e Inglaterra
conquistaram muitas reas de influncias, Itlia e
Alemanha, que na segunda metade do sculo XIX
enfrentavam seus processos de unificao, foram
menos privilegiadas. Essa distribuio irregular
gerou diversos conflitos entre as naes. Assim foram
construdos os alicerces da Primeira Guerra Mundial.

PRIMEIRA GUERRA MUNDIAL
1914 - 1918

A Primeira Guerra Mundial foi um conflito militar
(1914-1918), iniciado por um confronto regional entre
o Imprio Austro-Hngaro e a Srvia, em 28 de julho
de 1914.
A causa imediata do incio das hostilidades
entre a ustria-Hungria e a Srvia foi o assassinato do
arquiduque Francisco Fernando de Habsburgo,
herdeiro do trono austro-hngaro, cometido, em
Sarajevo no dia 28 de junho de 1914, por um
nacionalista srvio. Entretanto, os verdadeiros fatores
determinantes do conflito foram: o esprito nacionalista
que crescia por toda a Europa durante o sculo XIX e
princpios do XX e a rivalidade econmica e poltica
entre as diferentes naes, o processo de militarizao
e a corrida armamentista que caracterizaram a
sociedade internacional dos ltimos anos do sculo
XIX, raiz da criao de dois sistemas de alianas que
se diziam defensivas: a Trplice Aliana e a Trplice
Entente. A primeira nasceu do pacto firmado entre a
Alemanha, ustria-Hungria e Itlia contra a ameaa de
ataque da Frana. A Trplice Entente era a aliana entre
Please purchase PDF Split-Merge on www.verypdf.com to remove this watermark.
Incluso para a Vida Histria A

Pr-Vestibular da UFSC
17
a Gr-Bretanha, Frana e Rssia para contrabalanar a
Trplice Aliana.

1914-1915: A GUERRA DE MOVIMENTO
As operaes militares na Europa se desenvolveram
em trs frentes: a ocidental ou franco-belga, a oriental
ou russa e a meridional ou srvia.

1915-1917 GUERRA DE TRINCHEIRAS

1917: ENTRADA DOS ESTADOS UNIDOS E O
ARMISTCIO COM A RSSIA

A poltica de neutralidade americana mudou quando a
Alemanha anunciou, em janeiro de 1917, que a partir
de fevereiro recorreria guerra submarina.
Vrias naes latino-americanas, entre elas o
Peru, o Brasil e a Bolvia apoiariam esta ao. O
afundamento de alguns navios levou o Brasil, em 26 de
outubro de 1917, a participar da guerra, enviando uma
diviso naval em apoio aos aliados. Aviadores
brasileiros participaram do patrulhamento do Atlntico,
navios do Lide Brasileiro transportaram tropas
americanas para a Europa e, para a Frana, foi enviada
uma misso mdica.
Representantes da Rssia, ustria e Alemanha
assinaram o armistcio em 15 de dezembro, cessando
assim a luta na frente oriental.

1918: ANO FINAL
Repblica de Weimar, cujo governo enviou uma
comisso para negociar com os aliados. Em 11 de
novembro foi assinado o armistcio entre a Alemanha e
os aliados, baseado em condies impostas pelos
vencedores.
O Tratado de Versalhes (1919), que ps fim
guerra, estipulava que todos os navios aprisionados
passassem a ser de propriedade dos aliados. Em
represlia a tais condies, em 21 de junho de 1919, os
alemes afundaram seus prprios navios em Scapa
Flow. As potncias vencedoras permitiram que
deixassem de ser cumpridos certos itens estabelecidos
nos tratados de paz de Versalhes, Saint-Germain-en-
Laye, Trianon, Neuilly-sur-le-Seine e Svres, o que
provocaria o ressurgimento do militarismo e de um
agressivo nacionalismo na Alemanha, alm de
agitaes sociais que se sucederiam em grande parte da
Europa.

Exerccios de Sala #

1. Tema recorrente da poltica contempornea, o
nacionalismo tem-se constitudo em foco permanente
de conflito. Sobre ele, correto afirmar que:
01. A Primeira Guerra foi precedida pelo confronto de
diferentes projetos expansionista. o caso da
Rssia, que pretendia avanar sobre territrios do
Imprio Austro-Hngaro e do Imprio Turco,
dizendo-se protetora dos povos eslavos. Ou da
Srvia, que, ao pretender unificar os eslavos do
Sudeste da Europa, formando a Grande Srvia,
tambm se chocava com os interesses desses
Imprios.
02. A Segunda Guerra, igualmente, foi precedida de
discursos nacionalistas, embora com novas feies.
o caso do fascismo italiano, embalado nos sonhos
de reconstruo das glrias do Imprio Romano, ou
do nazismo alemo, defensor da unificao dos
povos germnicos e da reconstruo do seu
Imprio, ento denominado III Reich.
04. A vitria do nacionalismo indiano (1947) e o
fracasso franco-britnico na guerra contra o Egito
(1956) desencadearam uma onda de nacionalismo
nas antigas colnias europeias na frica e na sia.
Aliando-se a outros pases de passado colonial,
como os latino-americanos, formaram uma terceira
fora internacional - Terceiro Mundo, situado entre
o Capitalismo e o Socialismo.
08. Nos anos 80-90, novamente os movimentos
nacionalistas abalam a poltica internacional. O j
frgil "imprio" sovitico v sua unidade desfazer-
se diante do separatismo das Repblicas Blticas -
Letnia, Estnia e Litunia. A partir de ento,
outras repblicas assumem o mesmo propsito,
pondo fim URSS. Entre os "eslavos do sul", as
disputas de croatas e srvios lanam a Iugoslvia
numa violenta guerra civil.

2. O clima de tenso oriundo da expanso imperialista
na sia e determinador do 1 Conflito Mundial no
pode ser avaliado pelas:
01. rivalidades entre franceses e ingleses na Indochina,
entre ingleses e russos na sia Central e entre
russos e japoneses na Mandchria e Coreia.
02. polticas de alianas entre russos e japoneses para
bloquear as pretenses inglesas e francesas no
sudeste asitico.
04. tenses entre o Imprio Ingls e o Imprio Chins
em torno da Coreia e da Mandchria com o apoio
da Frana Inglaterra.
08. rivalidades entre ingleses e franceses no sudeste
asitico, entre belgas e alemes em Port-Arthur e
entre russos e poloneses na sia Europeia.
16. tenses entre o Imprio Austro-Hngaro e a Grcia
na regio do sudeste asitico com o apoio da
Inglaterra aos gregos.

3. Os Estados Unidos emergiram como grande
potncia econmica mundial aps a Primeira Guerra
Mundial porque:

a) apoiou a Alemanha, com o objetivo de enfraquecer a
Inglaterra.
b) liderou a criao da ONU (Organizao das Naes
Unidas).
c) fortaleceu sua economia ao fornecer equipamentos e
suprimentos Entente, enquanto as potncias
europeias tiveram suas economias arrasadas aps o
conflito.
d) apresentou as propostas do Tratado de Versalhes,
para enfraquecer a Alemanha, a grande potncia
industrial do incio do sculo.
e) se manteve afastado do conflito direto com as
potncias europeias, concentrando seus esforos no
desenvolvimento interno.

Please purchase PDF Split-Merge on www.verypdf.com to remove this watermark.
Incluso para a Vida Histria A

Pr-Vestibular da UFSC 18


4. A respeito do envolvimento dos E.U.A. na Primeira
Grande Guerra incorreto afirmar que:

a) foi influenciado pela inteno germnica de atrair o
Mxico, prometendo-lhe ajuda na reconquista de
territrios perdidos para os E.U.A.
b) os E.U.A. financiaram diretamente a indstria blica
franco-inglesa e enviaram um grande contingente
de soldados ao fronte.
c) uma possvel derrota da Frana e Inglaterra colocaria
em risco os investimentos norte-americanos na
Europa.
d) contrariando o Congresso, o presidente dos E.U.A.
rompeu a neutralidade, declarando guerra s foras
do Eixo.
e) a adeso dos E.U.A. desequilibrou as foras em luta,
dando um novo alento Entente.

5. Ao trmino da Primeira Grande Guerra, as potncias
vencedoras responsabilizaram a Alemanha pela guerra
e foi-lhe imposto um tratado punitivo, o Tratado de
Versailles, que teve como consequncias:

a) degradao dos ideais liberais e democrticos,
agitaes polticas de esquerda - como o
movimento espartaquista, crise econmica e
desemprego.
b) enfraquecimento dos sentimentos nacionais,
militarizao do Estado Alemo, recuperao
econmica e incorporao de Gdansk.
c) anexao das colnias de Togo e Camares, a
afirmao dos ideais liberais e democrticos e a
valorizao do marco alemo.
d) prosperidade econmica, rearmamento alemo,
desmembramento da Alemanha e fortalecimento
dos partidos liberais.
e) surgimento da Repblica Democrtica Alem e da
Repblica Federal Alem, fortalecimento do
nazismo, militarismo e diminuio do desemprego.

UNIDADE 10

ENTRE GUERRAS

Revoluo Russa
A industrializao da Rssia ocorreu com atraso de
mais de um sculo em relao Inglaterra, mas foi
favorecida pelo fim da servido, que liberou mo de
obra, e pelos investimentos estrangeiros.
A Primeira Guerra Mundial enfraqueceu ainda
mais o governo Nicolau II. Em dois anos de luta, seu
exrcito sofreu 7 milhes de baixas, entre mortos,
feridos e desertores. A populao civil padeceu com a
fome e o desemprego.
Em fevereiro de 1917, o czar abdicou e foi
instalada uma repblica liderada por Kerensky. Em
novembro, os bolcheviques comandados por Lnin e
Trotsky tomam o poder. Logo aps, assinam a paz com
a Alemanha, efetuam uma radical reforma agrria e
nacionalizam bancos, indstrias e meios de transportes.
Depois de duas guerras, a economia russa
estava arrasada. Para recuper-la, Lnin adotou a NEP,
afrouxando alguns controles sobre os investimentos
estrangeiros, o excedente agrcola e as pequenas
empresas (menos de vinte funcionrios).
Em 1924, Lnin morreu e foi substitudo por
Stlin, que passou a perseguir todos os seus adversrios
(principalmente os adeptos de Trotsky), condenando
centenas de milhares morte e milhes ao exlio na
Sibria. Suprimiu A NEP e adotou os planos
quinquenais. A URSS tornou-se, assim, uma grande
potncia.

Crise de 1929
Ao final da Primeira Guerra, a indstria dos EUA era
responsvel por quase 50% da produo mundial.
O pas criou um novo estilo de vida: o american way
of life. Esse estilo de vida caracterizava-se pelo grande
aumento na aquisio de automveis, eletrodomsticos
e toda sorte de produtos industrializados.
Entretanto, os EUA sofreram grande abalo em
1929, quando mergulharam numa terrvel crise, de
repercusso mundial.
Por sua vez, Inglaterra, Frana e Alemanha
foram atualizando rapidamente seus mtodos
industriais. Isso colaborou para aumentar o
desequilbrio entre o excesso de mercadorias
produzidas e o escasso poder aquisitivo dos
consumidores. Configurava-se assim uma conjuntura
econmica de superproduo capitalista.

O crack da Bolsa de Valores de Nova York
A crise de superproduo teve como um de seus
grandes marcos o dia 29 de outubro de 1929, dia do
crack da Bolsa de Valores de Nova York, que
representava o grande termmetro econmico do
mundo capitalista.
O crack da Bolsa de Valores de Nova York
abalou o mundo inteiro. Os Estados Unidos no
podendo vender tambm deixaram de comprar e isso
afetou tambm o Brasil, que dependia das exportaes
de caf para os Estados Unidos.

New Deal: a reao crise
Nos primeiros anos do governo do presidente Franklin
Roosevelt, os Estados Unidos adotaram o New Deal,
um conjunto de medidas destinadas superao da
crise. O New Deal foi inspirado nas ideias do ingls
John Keynes. O Keynesianismo defende um Estado
mais interventor, que deveria evitar os riscos de
superproduo, alm de aumentar o poder de consumo,
mas preservando a economia de mercado. Dentre as
principais medidas adotadas pela poltica econmica do
New Deal, destacam-se:
x Controle governamental dos preos de diversos
produtos industriais e agrcolas.
x Concesso de emprstimos aos proprietrios
agrcolas.
x Realizao de um grande programa de obras
pblicas.
x Criao de um seguro-desemprego.
x Recuperao industrial.
Please purchase PDF Split-Merge on www.verypdf.com to remove this watermark.
Incluso para a Vida Histria A

Pr-Vestibular da UFSC
19

Com a Crise de 29 e a adoo do New Deal, o
liberalismo clssico de Adam Smith foi superado pelo
neocapitalismo.

Regimes Totalitrios
A Crise de 29 arruinou a economia americana, mas
teve reflexos em todo o mundo. Naes como a
Alemanha, que aps o final da Primeira Guerra
reconstrua-se com recursos oriundos principalmente
dos estados Unidos da Amrica, passaram a enfrentar
aguda crise econmica. Grandes banqueiros, temendo a
ameaa representada pelos partidos de orientao
marxista, passaram a apoiar a ascenso de regimes
totalitrios, que prometiam estabelecer a ordem e a
disciplina social. Exemplos marcantes desse processo
foram o desenvolvimento do fascismo na Itlia e do
nazismo na Alemanha.

Principais Caractersticas
x Totalitarismo: absoluta supremacia estatal
x Governo ditatorial
x Partido nico
x Nacionalismo
x Racismo
x Militarismo
x Expansionismo
x Culto ao lder
x Sociedade hierarquizada
x Corporativismo: estado mediando as relaes entre
patres e empregados.

Exerccios de Sala #

1. O perodo de 1919 a 1939, pelos componentes que o
constituram, marcados por esperanas e frustraes,
tido como um dos mais crticos da poca
contempornea. Dos esforos para superar a devastao
da Primeira Guerra Mundial, se encaminha para a
recuperao e logo em seguida para o novo conflito
mundial.
A respeito desse perodo correto afirmar que:
01. A frustrao e o inconformismo do alemes,
submetidos s clusulas do Tratado de Versalhes,
levaram - nos a chamar esse acordo de "Diktat".
02. A Liga das Naes (ou Sociedade das Naes),
criada aps a Primeira Guerra Mundial, recebeu
apoio de todas as potncias e teve atuao decisiva
para evitar todas as crises internacionais da dcada
de 1930.
04. A URSS participou ativamente da poltica
internacional europeia na dcada na dcada de
1920.
08. Nesse perodo houve a vitria das ditaduras do tipo
nazi - fascista na Itlia e na Alemanha, alm de
regimes autoritrios em diversos pases, como
Portugal e Espanha.
16. A crise de 1929 e a grande depresso econmica
que ela gerou, desencadearam tambm crises
polticas, reacenderam nacionalismos econmicos e
polticos, facilitaram a ascenso de ditaduras e
contriburam para o advento da Segunda Guerra
Mundial.

2. O perodo de entre guerras (1919-1939) foi
caracterizado pelo aparecimento de regimes
autoritrios na Europa. A esse respeito, correto
afirmar que:
01. Esses regimes podem ser entendidos como uma
alternativa tanto ordem liberal tradicional quanto
ao regime comunista.
02. No perodo em questo, acentuaram-se as
dificuldades dos regimes democrticos e acentuou-
se o fracionamento poltico, o que dificultava o
estabelecimento de maiorias parlamentares que
pudessem garantir a continuidade administrativa.
04. A incapacidade dos regimes de democracia liberal
de contornarem a crise econmica dos anos
1920/30, tambm contribuiu para abrir espaos para
a expanso dos regimes autoritrios.
08. Parte importante no projeto do nazismo de
unificao das vontades coletivas foi a nfase na
liberdade de expresso e na igualdade entre as
raas.
16. A expanso dos regimes autoritrios se fez com
base num acentuado internacionalismo e
cosmopolitismo, rejeitando-se qualquer nfase em
temas nacionalistas.
32. A tomada do poder pelos nazistas e fascistas teve
uma significativa participao popular, inclusive
com grandes manifestaes de massa.

Tarefa Mnima #

3. "... derrota na guerra, deseres, motins militares
contra os superiores, greves nas fbricas, falta de
gneros alimentcios e combustveis nas principais
cidades, queda na produo, aviltamento dos salrios,
incapacidade governamental e crescente misria das
massas."
O quadro descrito no texto conduziu
a) derrota dos franceses no Vietn em 1954.
b) descolonizao Afro-Asitica em 1945.
c) rebelio Boxer na China em 1900.
d) Segunda Guerra Mundial em 1939.
e) Revoluo Russa em 1917.

4. "A guerra atual , por parte de ambos os grupos
potncias beligerantes, uma guerra (...) conduzida pelos
capitalistas pela partilha das vantagens que provm
domnio sobre o mundo, pelos mercadores do capital
financeiro (bancrio), pela submisso dos povos fracos
etc."
("Resoluo sobre a Guerra", publicada no jornal PRAVDA em abril
de 1917.)
O texto oferece uma interpretao caracterstica dos
bolcheviques sobre a
a) Guerra Russo-Japonesa.
b) Guerra da Coreia.
c) Guerra da Crimeia.
d) Primeira Guerra Mundial.
e) Primeira Guerra Balcnica.


Please purchase PDF Split-Merge on www.verypdf.com to remove this watermark.
Incluso para a Vida Histria A

Pr-Vestibular da UFSC 20

5. Sobre fatos antecedentes Segunda Guerra
Mundial, assinale a alternativa incorreta.
a) Os E.U.A. cortaram o envio de ferro, ao, petrleo e
borracha e bloquearam capitais japoneses na
Amrica do Norte por causa da invaso da
Manchria pelo Japo.
b) Passando por cima das disposies dos tratados do
ps-guerra, em 1938, Hitler, com o apoio de
fascistas austracos, ordenou a ocupao da ustria.
e) Em 1936, um grupo de generais, chefiados por
Franco, iniciou uma revolta contra o governo de
esquerda, legalmente constitudo, na Espanha.
d) A euforia econmica decorrente da valorizao da
Bolsa de Nova Iorque em 1929 favoreceu a
recuperao econmica e a consolidao das
democracias na Europa.
e) Em 1939, Stlin conseguiu-se aproximar da
Alemanha atravs do Pacto Germano-Sovitico,
negociado por Ribbentrop e Molotov.

UNIDADE 11

SEGUNDA GUERRA MUNDIAL
1939-1945

A humilhao sofrida pela Alemanha com o Tratado de
Versalhes cria as condies ideais para a germinao
do nacional-socialismo - nazismo - alemo e a
ascenso de Hitler ao poder, em 1933. O nacional-
socialismo toma o poder pela violncia, elimina as
dissenses internas com mtodos violentos e combate a
diviso do mundo produzida pela 1
a
Guerra.

Reao mundial ao nazismo - As potncias ocidentais
tm uma posio dbia em relao ao nazismo.
Pressentem o perigo representado por Hitler, mas
permitem o crescimento da Alemanha nazista como
forma de bloquear a Unio Sovitica. A invaso da
Polnia, em 1
o
de setembro de 1939, por tropas e
avies alemes, no surpreende a Europa. Todos esto
espera da guerra.

Origens do Eixo - Itlia e Alemanha tm regimes
polticos semelhantes, mas o que mais as aproxima o
limitado espao territorial de que dispem e a acirrada
competio pelos mercados internacionais. Stalin
percebe que as anexaes alems caminham em
direo Unio Sovitica e firma com Hitler o Pacto
Germano-Sovitico, em 1939, pelo qual anexa a
Litunia, Letnia, Estnia e parte da Polnia e
Finlndia.

COMEA A GUERRA NA EUROPA
Em abril de 1939 Hitler exige a anexao de Dantzig, o
"corredor polons", e a concesso de uma rede
rodoviria e ferroviria que cruze a provncia polonesa
da Pomernia. A Polnia, sem condies de resistir,
invadida por tropas nazistas no dia 1
o
de setembro. O
Reino Unido, comprometido com a defesa da Polnia
em caso de agresso, declara guerra Alemanha. Horas
depois, seguida pela Frana. At junho de 1940,
quando a Itlia declara guerra Frana e ao Reino
Unido, o conflito est restrito aos trs pases. A
Alemanha invade e ocupa a Noruega, a Blgica, a
Holanda e a Frana.

Domnio alemo - O domnio alemo na Europa fica
patente com a expulso dos ingleses de Dunquerque e
os armistcios assinados pela Frana com a Itlia e
Alemanha, em junho de 1940, que dividem o territrio
francs em duas partes.
Na Batalha da Inglaterra, no vero de 1940, a aviao
inglesa, RAF (Royal Air Force), consegue rechaar os
ataques da Luftwaffe (aviao alem).

Defesa de Moscou - Em fins de 1941 a defesa de
Moscou marca uma das mais decisivas vitrias aliadas.

Ataque a Pearl Harbor - O ataque japons base
norte-americana de Pearl Harbor, no Hava, em 7 de
dezembro de 1941, leva os Estados Unidos a
declararem guerra ao Eixo e alastra o conflito a quase
todo o mundo. As duas faces beligerantes esto
definidas: os pases do Pacto Anticomintern (o Eixo) -
Alemanha, Itlia e Japo - contra os Aliados -
Inglaterra, Estados Unidos, Unio Sovitica e China. A
China j se encontra em guerra contra o Japo desde
1931.

Kamikazes - como so chamados os avies
japoneses carregados de explosivos e dirigidos por um
piloto suicida que com ele se atira sobre o alvo
inimigo.

SEGUNDA FASE DA GUERRA
quando o conflito se torna uma guerra de desgaste. O
Eixo tenta subjugar a Inglaterra, cortando suas linhas
de abastecimento no Atlntico e no Mediterrneo.

Contra-ofensiva na frica e Itlia - Em julho de
1943 os Aliados desembarcam na Siclia e, em
setembro, avanam at Npoles. Mussolini destitudo
em julho e a Itlia muda de lado.

Dia D - Em 6 de junho de 1944, chamado de "Dia D"
pelos Aliados, sob o comando do general Einsenhower,
feito o ataque estratgico que daria o golpe mortal
nas foras nazistas que ainda resistem na Europa.
Cinquenta e cinco mil soldados norte-americanos,
britnicos e canadenses desembarcam nas praias da
Normandia, noroeste da Frana, na maior operao
aeronaval da Histria, envolvendo mais de 5 mil navios
e mil avies.

Guerra no Pacfico - No Pacfico, a situao tambm
se inverte com a vitria das tropas norte-americanas na
batalha naval de Midway e em Guadalcanal, em 1942.

Ataque a Hiroshima e Nagasaki - Em 6 de agosto os
Estados Unidos lanam a primeira bomba atmica
sobre Hiroshima, deixando mais de 100 mil mortos e
100 mil feridos. A partir de 8 de agosto tropas
soviticas expulsam os japoneses da Mandchria e da
Coreia e ocupam as ilhas Kurilas e Sakalina. Em 9 de
agosto lanada a segunda bomba atmica, dessa vez
Please purchase PDF Split-Merge on www.verypdf.com to remove this watermark.
Incluso para a Vida Histria A

Pr-Vestibular da UFSC
21
sobre Nagasaki, com saldo de vtimas semelhante ao de
Hiroshima.

Final da guerra - Hitler suicida-se em 30 de abril,
com a chegada das tropas soviticas a Berlim, e o
almirante Doenitz forma novo governo e pede o fim
das hostilidades. A capital alem ocupada em 2 de
maio. A Alemanha se rende incondicionalmente em 7
de maio, em Reims. A capitulao do Japo acontece
em 2 de setembro, em Tquio. A 2
a
Guerra Mundial
deixa um saldo de 50 milhes de mortos e custa cerca
de US$ 1,40 trilho.

JULGAMENTO DE NUREMBERG
Terminado o conflito, os vitoriosos decidem julgar os
lderes nazistas num indito tribunal internacional de
crimes de guerra. A iniciativa contribui para a
descoberta dos campos de concentrao e extermnio.
A sede escolhida a cidade alem de Nuremberg, que
nos anos 30 havia sido palco dos maiores comcios
nazistas.

Exerccios de Sala #

1. A Segunda Guerra Mundial alterou a correlao de
foras no mundo. Entre as modificaes ocorridas,
destacam-se:

01. O declnio da influncia europeia cuja hegemonia
j havia sido comprometida desde a Primeira
Guerra Mundial.
02. A ascenso dos Estados Unidos e da Unio
Sovitica, liderando blocos de interesses
divergentes e originando a chamada "bipolarizao"
do mundo.
04. Aps a Segunda Guerra Mundial e at
recentemente, nenhuma potncia europeia ou os
Estados Unidos participaram de qualquer conflito
blico.
08. Aps a Guerra - e por causa dela -, houve
intensificao das manifestaes anticolonialistas,
acelerando-se o processo de descolonizao das
colnias europeias na frica e na sia.
16. O final da Segunda Guerra Mundial decretou o
desaparecimento dos Estados autoritrios,
reorganizando-se o mundo em bases inteiramente
democrticas.
32. Como tentativa de resolver os problemas
internacionais, criou-se em 1945 a Organizao das
Naes Unidas (ONU).

2. Sobre a Histria Contempornea, correto afirmar
que:
01. a Primeira Guerra Mundial (1914-18) resultou,
dentre outros motivos, da concorrncia comercial,
da disputa por colnias e da luta pela hegemonia
dos mares.
02. a grande vencedora da Primeira Guerra Mundial foi
a Alemanha, o que motivou a reao da Itlia e do
Japo no final dos anos 30, dando inicio Segunda
Guerra Mundial.
04. o Tratado de Versalhes foi imposto pela Alemanha
aos pases europeus, com o apoio dos Estados
Unidos.
08. a ideologia nazista enaltecia o nacionalismo e o
militarismo, visando conquistar as massas e o
exrcito, e pregava o anti-comunismo, visando
conquistar a alta burguesia.
16. apesar das guerras do sculo XX, a Europa
manteve sempre sua hegemonia econmica e
poltica sobre o mundo.

Tarefa Mnima #

3. Pensem nas crianas
Mudas telepticas
Pensem nas meninas
Cegas inexatas
Pensem nas mulheres
Rotas alteradas
Pensem nas feridas
Como rosas clidas
Mas oh! no se esqueam
Da rosa de Hiroshima
A rosa hereditria
A rosa radioativa
Estpida e invlida
A rosa com cirrose
A anti-rosa atmica
Sem cor, sem perfume
Sem rosa, sem nada

"Rosa de Hiroshima"
(Gerson Conrad e Vincius de Moraes)

Podemos considerar que o texto acima debate:

a) a herana terrvel das bombas atmicas atiradas em
Hiroshima e Nagasaky, no final da 2 Guerra
Mundial, levantando a necessidade de sua
lembrana para defendermos a paz.
b) a poesia no trata dos problemas relativos bomba
atmica, a guerra e a paz.
c) as armas atmicas nunca seriam usadas como forma
de poder entre as potncias mundiais.
d) a paz s ser garantida com a utilizao de armas
atmicas.
e) as armas atmicas deixaram poucas heranas
culturais e polticas durante o perodo da Guerra
Fria.

4. Na II Guerra Mundial, as bombas atmicas de
Hiroshima e Nagazaki foram consideradas crimes de
guerras, porque:
a) o conflito j tinha terminado em agosto de 1945 e a
resistncia japonesa era mnima em Pearl Harbor.
b) as bombas faziam parte de um esquema de testes
militares.
c) os E.U.A. queriam impor o seu domnio
Alemanha.
d) os E.U.A. pretendiam deter o avano dos soviticos
na sia.
e) as bombas tinham um nico alvo: os japoneses no
Pacfico.
Please purchase PDF Split-Merge on www.verypdf.com to remove this watermark.
Incluso para a Vida Histria A

Pr-Vestibular da UFSC 22
5. Foi o encontro do primeiro ministro ingls Winston
Churchill e dos presidentes Roosevelt, dos Estados
Unidos e Stlin, da Unio Sovitica onde confirmou-se
o desmembramento da Alemanha e da Coreia:
a) Conferncia do Cairo.
b) Conferncia de Teer.
c) Conferncia de Ialta.
d) Conferncia de Potsdam.
e) Conferncia de Bandung.

UNIDADE 12

IDADE CONTEMPORNEA: GUERRA
FRIA

Bipolarizao entre EUA (capitalismo) e URSS
(socialismo), estado de constante hostilidade entre as
duas superpotncias e seus aliados e corrida
armamentista e tecnolgica, mas sem conflito armado
direto, caracterizaram a Guerra Fria, que vigorou entre
o fim da Segunda Guerra Mundial e a crise do
socialismo real no final dos anos 1980.
Tradicionalmente as fases da Guerra Fria so a
Guerra Fria Clssica (1945 1953), o perodo do
Degelo ou da Coexistncia Pacfica (1953 1968) e a
poltica da Dtente ou da Distenso (1968 1989).

Marcos iniciais da Guerra Fria

Bloco Capitalista
Em 1946, Churchill discursa nos EUA e pede para que
estes se tornem os protetores do mundo livre, pois a
Gr-Bretanha, no poderia mais manter tal papel.

Doutrina Truman (1947) o presidente Truman
responde Churchill atravs de sua Doutrina e pede ao
Congresso Americano ajuda econmica e militar para
os governos europeus. Os EUA passaram a assumir a
tarefa de protetores da democracia, reagindo,
imediatamente, ao menor sinal de avano socialista. A
Doutrina Truman pode ser considerada marco inicial
da Guerra Fria.

Plano Marshall (1947) ajuda econmica para a
Europa e considerado brao econmico do Bloco
Capitalista no incio da Guerra Fria.

Organizao do Tratado do Atlntico Norte
(OTAN) 1949 fora poltico-militar com objetivos
defensivos, inicialmente composta por pases da
Europa Ocidental, EUA e Canad.

Bloco Socialista

Tratado de Assistncia Mtua da Europa Oriental
(Pacto de Varsvia) 1955 aliana militar do Bloco
Socialista e formada por pases da Europa Oriental e a
URSS.

Principais acontecimentos
Crise ou Bloqueio de Berlim (1948-49) com o fim
da Segunda Guerra, o Acordo de Potsdam (1945)
dividiu a derrotada Alemanha nazista e sua capital em
quatro zonas de ocupao entre URSS, EUA, Gr-
Bretanha e Frana.
Repblica Federal da Alemanha (capital Bonn), sob o
governo capitalista.
Repblica Democrtica Alem, inspirada no modelo
sovitico, com capital em Berlim Oriental.

Guerra da Coreia (1950-53) a derrota japonesa na
Segunda Guerra Mundial (1945) permitiu a libertao
da Coreia pelas tropas aliadas, mas o pas foi dividido
em dois setores de ocupao (norte-americano e
sovitico), pelo paralelo 38. A diviso do pas, as
tenses relacionadas com a Guerra Fria e a vitria
comunista de Mao Ts-tung na Revoluo Chinesa, em
1949, desencadearam a Guerra da Coreia, iniciada aps
a invaso do sul pelos norte-coreanos, em 1950. A
Coreia do Sul recebeu apoio militar dos Estados
Unidos, enquanto a do Norte da China e da URSS. O
conflito terminou com a assinatura do Armistcio de
Panmunjom (1953), que ratificou a diviso da Coreia
efetuada anteriormente.

Guerra do Vietn (1960-75) a luta pela
descolonizao da Indochina comeou no Vietn e foi
liderada por Ho Chi Minh - Guerra da Indochina
(1946-54). Na Conferncia de Genebra (1954), a
Frana reconheceu a independncia do Laos, Camboja
e Vietn este dividido em Norte (socialista) e Sul
(capitalista). As eleies que reunificariam o Vietn em
1956, no aconteceram, devido ao golpe de Estado de
Ngo Dinh Diem (Sul) que estabeleceu uma ditadura
militar em 1955, apoiado pelos Estados Unidos. A
resistncia ao governo golpista gerou a formao da
Frente de Libertao Nacional, cujo brao armado
eram os vietcongues, e a Guerra do Vietn. Nos anos
1960, auge da Guerra Fria, a guerra no Sudeste asitico
ampliou-se com a interveno militar norte-americana.
A sada dos Estados Unidos e o avano das tropas
comunistas levaram rendio do Sul, em 1975,
permitindo a reunificao do pas, em 1976, e o
nascimento da Repblica Socialista do Vietn.

Muro de Berlim (1961) o interesse sovitico em
bloquear fugas para o lado capitalista de Berlim e
enfraquecer as expectativas reunificadoras, levou a
Alemanha Oriental a erguer o muro em 1961. O muro
da vergonha dividiu famlias e ideologias,
transformando-se em um dos principais smbolos da
Guerra Fria.

Crise dos msseis (1962) em 1959, Fidel Castro e
alguns companheiros, contando com o apoio
campons, derrubaram o ditador cubano Fulgncio
Batista. O governo revolucionrio demonstrou, desde o
incio, uma grande preocupao com a justia social. A
reao americana foi imediata, merecendo destaque, o
embargo comercial e a presso para que Cuba fosse
expulsa da OEA. Isolada poltica e economicamente,
Cuba aproxima-se da URSS, ampliando o Bloco
Socialista. Mas, o momento de maior tenso aconteceu
em 1962, quando os EUA descobriram que msseis
soviticos estavam sendo instalados em Cuba Crise
dos Msseis. A conscientizao das graves
Please purchase PDF Split-Merge on www.verypdf.com to remove this watermark.
Incluso para a Vida Histria A

Pr-Vestibular da UFSC
23
consequncias de um confronto armado direto levou os
lderes das superpotncias ao entendimento. A URSS
concordou em retirar seus msseis e os EUA aceitaram
a perda do monoplio poltico-ideolgico no
continente.

Primavera de Praga (1968) as intenes de
modernizar a economia e o Estado, buscando uma via
independente e mais humanizada de socialismo, pelo
presidente, da ento Tchecoslovquia, Dubcek,
provocou a imediata reao sovitica, que a fim de
manter seu controle sobre a Europa Oriental, enviou
tropas do Pacto de Varsvia, reprimindo a Primavera
de Praga.

Maio de 1968 proibido proibir e paz e amor
foram palavras de ordem de uma gerao, nascida em
plena Guerra Fria, e que viveu os anos rebeldes a
dcada de 1960. Na Frana, em 1968, os estudantes
protestaram contra as reformas educacionais, alm de
pedirem maior liberdade, criticando abertamente o
conservadorismo.

Corrida espacial as pesquisas espaciais tiveram
destacado papel na tenso entre Estados Unidos e
URSS, transformando a eficincia tecnolgica em uma
importante arma poltico-ideolgica. A corrida espacial
comeou em 4 de outubro de 1957, quando foi lanado
o primeiro satlite em rbita da Terra pela Unio
Sovitica, o Sputnik-1. A competio aumentou
quando os soviticos, um ms depois do Sputnik-1,
enviaram o Sputnik-2. Em 1961 a URSS comemorou o
primeiro vo tripulado, transformando Iri Gagrin no
primeiro astronauta da histria. Apenas em 20 de julho
de 1969, os norte-americanos conseguiram impor-se,
enviando a Apolo-11, com os primeiros astronautas,
Lua.
Fim da Unio Sovitica
Em 1991, em meio a uma grave crise do que se passou
a chamar socialismo real, a Unio Sovitica deixava
oficialmente de existir. Era mais um fato de uma poca
de mudanas radicais queda do Muro de Berlim,
reunificao da Alemanha, queda dos regimes de
esquerda do Leste Europeu etc.
No final dos anos 80, o presidente sovitico Mikhail
Gorbachev estava ciente dos problemas que o pas
atravessava e decidiu adotar dois conjuntos de
reformas. A Perestroika, ou Reestruturao, visava a
mudar as condies econmicas do Estado na
realidade, permitia a volta da propriedade privada e do
capitalismo. J a Glasnost, ou Transparncia, tinha
como objetivo mudar a estrutura poltica e abrir
caminho para o surgimento de mecanismos de
expresso democrtica. Gorbachev queria diminuir a
burocracia e a corrupo, que eram alarmantes nos
rgos estatais naquela poca.

As guerras do Golfo Prsico
Guerra Ir-Iraque (1980-88): vitria militar no
conclusiva do Iraque. Amos os pases se debilitaram

Guerra do Kuwait (1990-91): o Iraque foi obrigado a
retirar-se do Kuwait pela Operao Tempestade no
Deserto liderada pelos EUA.

Guerra anglo-americana contra o Iraque (2003):
ocupao anglo-americana do Iraque, dissoluo do
regime de Saddam Hussein.

Exerccios de Sala #

1. Em 6 de junho de 1944, a Europa comeou a se
render aos Estados Unidos da Amrica, iniciando o
perodo da "Pax Americana" (sem os americanos, no
haveria Dia D). Apesar do nome "pax", os ltimos 50
anos foram uma poca recheada de "pequenas guerras"
[ ... ] o mesmo havia acontecido com a mais famosa
"Pax Romana". Roma, gestora da chamada Civilizao
Ocidental, soube resistir a invases brbaras, mantendo
as caractersticas bsicas da civilizao que chegou aos
brasileiros ps-1500.
(BONALUME NETO, p. 2)

Indique as proposies que confirmam o contedo do
texto anterior:
01. As alianas poltico-militares representadas pela
OTAN e pelo Pacto de Varsvia configuraram o
esforo americano no sentido da preservao da paz
mundial.
02. Assim como os romanos, durante a vigncia da sua
Pax, tiveram que enfrentar os ataques brbaros, os
americanos rechaaram os ataques de Hitler, o
"brbaro" moderno.
04. Aps a Segunda Grande Guerra, ocorreu a
redefinio da ordem mundial, com a hegemonia
dos Estados Unidos no bloco capitalista e o declnio
da influncia poltica, econmica e cultural da
Europa.
08. As "pequenas guerras" empreendidas pelos Estados
Unidos, na Coreia, no Vietn, no Afeganisto e no
Golfo Prsico, evidenciam o carter contraditrio
da Pax Americana.
16. A Pax Americana dependia da aquiescncia da
Alemanha, que passou a reivindicar reas coloniais
e a contestar a hegemonia internacional anglo-
americana.
32. Os acordos entre Estados Unidos e Unio Sovitica
do origem poltica de Coexistncia Pacfica, de
resultados mais aparentes que reais e permanente
competio militar.

2. No perodo imediatamente aps a Segunda Guerra
Mundial (1939-1945), ocorre o(a):
01. estabelecimento da bipolaridade nas relaes
internacionais, com os Estados Unidos e a Unio
Sovitica liderando os blocos capitalistas e
socialistas, respectivamente.
02. declnio da Europa como centro do poder mundial,
de que a descolonizao afro-asitica foi exemplo
marcante.
04. criao da Organizao das Naes Unidas, em
cujo Conselho de Segurana manifesta-se o
Please purchase PDF Split-Merge on www.verypdf.com to remove this watermark.
Incluso para a Vida Histria A

Pr-Vestibular da UFSC 24
princpio de absoluta igualdade entre os Estados
participantes.
08. refluxo no processo de expanso socialista, em
parte determinado pelo fracasso militar sovitico
durante a guerra.

Tarefa Mnima #

3. No incio da dcada de 60, o arsenal nuclear
disposio das grandes potncias era suficiente para
destruir a humanidade, caso fosse utilizado em uma
situao de confronto. Ao assumir o governo, o
Presidente Kennedy (1961-63) defendeu a substituio
da poltica externa norte-americana de confronto por
uma de entendimento com a URSS, cujo objetivo era o
desarmamento gradual das duas superpotncias. Esse
programa do governo Kennedy foi conhecido como:
a) Doutrina Drago.
b) Doutrina Monroe.
c) Corolrio Roosevelt
d) Nova Fronteira.
e) Poltica de Boa Vizinhana.

4. No caracterstica da Guerra Fria:
a) confronto ideolgico que pressupe equilbrio
nuclear entre as potncias.
b) polarizao do mundo em dois blocos poltico-
militares.
c) distenso poltica e alinhamento internacional entre
E.U.A. e U.R.S.S.
d) desconfiana entre americanos e soviticos e disputa
de reas de influncia.
e) criao das alianas militares O.T.A.N. e Pacto de
Varsvia.

5. As duas Guerras Mundiais, marcadas pelo
expansionismo europeu, deixaram consequncias
profundas. A imploso do Imprio Sovitico est
contribuindo para frear o perigoso confronto Leste-
Oeste. O cotidiano europeu, no entanto, ainda
apresenta cenas sombrias. A Guerra Civil na ex-
Iugoslvia, entremeada da brutalidade que gera
indignao, tem razes remotas e profundas porque:

a) expressa ressentimentos tnico-nacionalistas e
diferenas culturais nos Blcs.
b) o Pacto Nazista-Sovitico colocou os Estados do
Bltico sob domnio russo.
c) o colapso do comunismo abriu caminho para a
transio capitalista bem sucedida.
d) na federao multinacional iugoslava, o comunismo
foi edificado sobre base camponesa, e no operria.
e) o Tratado de Paz, que consagrou o desmembramento
do Imprio Austro-Hngaro, ps fim ao velho
antagonismo que dera origem Primeira Guerra
Mundial.

Please purchase PDF Split-Merge on www.verypdf.com to remove this watermark.
Incluso para a Vida Histria B

Pr-Vestibular da UFSC

1
UNIDADE 1

EXPANSO MARTIMA EUROPEIA DOS
SCULOS XV E XVI

Ao iniciar os estudos sobre o Brasil Colnia,
devemos ter em mente que essa parte de nossa histria esteve
intimamente vinculada aos interesses europeus. Dessa forma,
para compreendermos esse perodo faz-se necessrio o
conhecimento do contexto Europeu do incio da Idade
Moderna.

A nova ordem do dia
A chegada dos portugueses ao Brasil deve ser entendida
como consequncia da Expanso Ultramarina Europeia dos
sculos XV e XVI, que por sua vez foi resultado da
Revoluo Comercial ocorrida entre os sculos XV e XVIII.
Nesse perodo, o comrcio cresceu e passou a influenciar e
modificar toda estrutura social e econmica da Europa.
Um novo sistema econmico desenvolveu-se por
meio da acumulao do capital comercial, ou seja, do capital
oriundo da comercializao de mercadorias. As
transformaes desencadeadas a partir de ento substituram
os mecanismos econmicos feudais por outros mais
adaptados a nova realidade. A esse conjunto de novos
conceitos deu-se o nome de Mercantilismo, cujos principais
sustentculos foram:
x Metalismo: acumulao de metais para cunhagem de
moedas. A riqueza de uma nao passou a ter relao
direta com a quantidade de ouro e prata que esta possua.
x Protecionismo Alfandegrio: os pases passaram a
proteger sua burguesia atravs de pesadas taxas sobre
importaes.
x Balana Comercial Favorvel: exportao sempre
superior s importaes.
x Pacto Colonial: Exclusivismo comercial das colnias
para com suas metrpoles.
Uma das formas mais lucrativas de comercializao
na poca era trazer Europa especiarias vindas de outros
continentes. Produtos como o cravo, a pimenta e a canela
proporcionavam grandes lucros burguesia comercial. A
partir de ento o comrcio com o oriente passou a ser de
fundamental importncia.
Desde o fim das Cruzadas o mar Mediterrneo,
principal via de comercializao da poca, era controlado
pela Itlia. Coube a outras naes buscar rotas comerciais
para chegar ao oriente, que no passassem pelo
Mediterrneo. Tinha incio a Expanso Martima.

Navegar era preciso


mar salgado, quanto do teu sal
So lgrimas de Portugal!
Por te cruzarmos, quantas mes choraram,
Quantos filhos em vo rezaram!
Quantas noivas ficaram por casar
Para que fosses nosso, mar!

Valeu a pena? Tudo vale a pena
Se a alma no pequena.
Quem quer passar alm do Bojador
Tem que passar alm da dor.
Deus ao mar o perigo e o abismo deu,
Mas nele que espelhou o cu.
Fernando Pessoa

Portugal foi o grande pioneiro no processo de
Expanso Martima, isso devido a sua precoce centralizao
poltica, ocorrida com a Revoluo de Avis (1383 1385).
A Escola de Sagres, principal centro de estudos nuticos da
poca, criada por D. Henrique tambm contribuiu para o
sucesso portugus. Alm disso, os marinheiros portugueses
j tinham alguma experincia com a pesca no Atlntico.
Essas particularidades deram condio a Portugal lanar-se
no Mar Tenebroso no incio do sculo XV. A ideia era
chegar ndia contornando o continente africano.
Outra nao que tambm deu incio as navegaes
ultramarinas ainda no sculo XV foi Espanha. Todavia a
rota almejada pelos espanhis era outra: navegar para o
ocidente para chegar ao oriente.

Principais Datas:
x 1415-1469: Portugal faz colnias ao norte do Bojador;
x 1488: Bartolomeu Dias chega ao Cabo da Boa
Esperana;
x 1492: Cristvo Colombo chega s Amricas;
x 1498: Vasco da Gama chega ndia;
x 1500: Pedro lvares Cabral chega ao Brasil.

Em 1494, Portugal e Espanha assinaram o Tratado de
Tordesilhas, dividindo o mundo entre as naes ibricas. Tal
tratado acabou determinando que o Brasil, ou pelo menos boa
parte dele, pertencesse a Portugal.
A "descoberta" oficial ocorreu em 1500. Porm, segundo
alguns historiadores, na ocasio do Tratado de Tordesilhas j
existia uma razovel certeza quanto existncia de terras a
Ocidente. Descoberta ou acidente? Rejeitando-se tais
hipteses, qual seria a inteno da expedio de Cabral?
A colonizao veio como consequncia do descobrimento,
no tendo sido essa a finalidade.

Exerccios de Sala #

1. A expanso martima europeia dos sculos XV e XVI
permitiu:
a) A formao de domnios coloniais que dinamizaram o
comrcio europeu.
b) O crescimento do comrcio de especiarias pelas rotas do
Mediterrneo.
c) A implantao de imprios coloniais na sia, para
extrao de metais preciosos.
d) O fortalecimento do feudalismo e da servido na Europa
Ocidental.
Please purchase PDF Split-Merge on www.verypdf.com to remove this watermark.
Incluso para a Vida Histria B

Pr-Vestibular da UFSC

2
e) A colonizao do tipo mercantilista, sem a interferncia do
Estado e da Igreja.

2. (UFPE) Portugal e Espanha foram, no sculo XV, as
naes modernas da Europa, portanto, pioneiras nos grandes
descobrimentos martimos. Identifique as realizaes
portuguesas e as espanholas, no que diz respeito a esses
descobrimentos.
01. Os espanhis, navegando para o Ocidente, descobriam,
em 1492, as terras do Canad.
02. Os portugueses chegaram ao Cabo das Tormentas, na
frica, em 1488.
04. Os portugueses completaram o caminho para as ndias,
navegando para o Oriente, em 1498.
08. A coroa espanhola foi responsvel pela primeira
circunavegao da Terra iniciada em 1519, por Ferno
de Magalhes. Sebastio El Cano chegou de volta
Espanha em 1522.
16. Os portugueses chegaram s Antilhas em 1492,
confundindo o Continente Americano com as ndias.

Tarefa Mnima #

3. (Cesgranrio) O descobrimento do Brasil foi parte do plano
imperial da Coroa Portuguesa, no sculo XV. Embora no
houvesse interesse especfico de expanso para o Ocidente:
a) A posse de terras no Atlntico ocidental consolidava a
hegemonia portuguesa neste Oceano.
b) O Brasil era uma alternativa mercantil ao comrcio
portugus no Oriente.
c) O desvio da esquadra de Cabral seguia a mesma
inspirao de Colombo para chegar s ndias.
d) A procura de terras no Ocidente foi uma reao de
Portugal ao Tratado de Tordesilhas, que o afastava da
Amrica.
e) Essa descoberta foi mero acaso, provocado pelas
intempries que desviaram a esquadra da rota da ndia.

4. (Mackenzie) As razes do pioneirismo portugus na
Expanso Martima dos sculos XV e XVI foram:
a) A invaso da Pennsula Ibrica pelos rabes e a conquista
de Calicute pelos turcos.
b) A assinatura do Tratado de Tordesilhas por Portugal e
pelos demais pases europeus.
c) Um Estado Liberal centralizado, voltado para a
acumulao de novos mercados consumidores.
d) As guerras religiosas, a descentralizao poltica do
Estado e o fortalecimento dos laos servis.
e) Uma monarquia centralizada, interessada no comrcio de
especiarias.

Tarefa Complementar #

5. (CESGRANRIO) Foram inmeras as consequncias da
expanso ultramarina dos europeus, gerando uma radical
transformao no panorama da histria da humanidade.
Sobressai como UMA importante consequncia:
a) A constituio de imprios coloniais embasados pelo
esprito mercantilista.
b) A manuteno do eixo econmico do Mar Mediterrneo
com acesso fcil ao Oceano Atlntico.
c) A dependncia do comrcio com o Oriente, fornecedor de
produtos de luxo como sndalo, porcelanas e pedras
preciosas.
d) O pioneirismo de Portugal, explicado pela posio
geogrfica favorvel
e) a manuteno dos nveis de afluxo de metais preciosos
para a Europa.

6. (Fuvest) Sobre o Tratado de Tordesilhas, assinado em 7 de
junho de 1494, pode-se afirmar que objetivava
a) demarcar os direitos de explorao dos pases ibricos,
tendo como elemento propulsor o desenvolvimento da
expanso comercial martima.
b) estimular a consolidao do reino portugus, por meio da
explorao das especiarias africanas e da formao do
exrcito nacional.
c) impor a reserva de mercado metropolitano, por meio da
criao de um sistema de monoplios que atingia todas as
riquezas coloniais.
d) reconhecer a transferncia do eixo do comrcio mundial
do Mediterrneo para o Atlntico, depois das expedies
de Vasco da Gama s ndias.
e) reconhecer a hegemonia anglo-francesa sobre a explorao
colonial, aps a destruio da Invencvel Armada de
Felipe II, da Espanha.

UNIDADE 2

PERODO PR-COLONIAL E
ADMINISTRAO COLONIAL

Perodo Pr-Colonial (1500 1530)
Durante trs sculos, o Brasil ficou na condio de colnia
portuguesa. Havia uma grande dependncia da colnia em
relao metrpole, pois a primeira importava a cultura e
os comportamentos da ltima.
O primeiro produto a ser explorado foi o pau-brasil, retirado
do litoral brasileiro por indgenas, que negociavam com os
portugueses atravs do escambo. Para garantir a segurana
da madeira retirada pelos brasileiros, os portugueses
construram feitorias pela costa brasileira.
Nos primeiros tempos, os franceses mantinham bom
relacionamento com os ndios. Somente por volta de 1530
Portugal passou a se interessar mais pelo Brasil.

Primeiros Estgios da Administrao Colonial.
Para possibilitar uma melhor administrao, o Brasil foi
dividido em Capitanias Hereditrias. Em 1549, foi
nomeado o primeiro governador geral do Brasil, criado para
coordenao das capitanias.
O Governo Geral (1549/1572) pode ser definido como
primeiro esboo do poder pblico no Brasil. Os primeiros
Governadores Gerais foram encarregados de tarefas
administrativas e militares, sendo os primeiros marcos da
tarefa colonizadora:
- Tom de Souza (1549/1553): vinda dos primeiros jesutas,
Fundao de Salvador, Fundao do primeiro colgio,
incentivos agricultura e pecuria (primeiras cabeas de
gado);
- Duarte da Costa (1553/1558): Fundao do Colgio de
So Paulo de Piratininga (1554). Invaso francesa no Rio de
Janeiro (1555): fundaram a Frana Antrtica;
Please purchase PDF Split-Merge on www.verypdf.com to remove this watermark.
Incluso para a Vida Histria B

Pr-Vestibular da UFSC

3
- Mem de S (1558/1572): Confederao dos Tamoios,
Fundao de So Sabastio do Rio de Janeiro por Estcio de
S (1565) e expulso dos franceses (1567).
Com a morte de Mem de S, o Brasil foi dividido em dois: O
Governo do Norte e o Governo do Sul.
No sculo XVII ocorre outra diviso: Estado do
Maranho e o Estado do Brasil, pois em 1612, interessados
na colnia, os franceses tentaram apoderar-se do Maranho,
onde poderiam intervir no Caribe, por onde passavam navios
espanhis carregados de metais preciosos. Fundaram a cidade
de So Lus, e queriam fundar a Frana Equinocial. O
fracasso francs deu incio colonizao do Maranho, e sua
transformao em colnia separada do Brasil.

Exerccios de Sala #

1. (UFSC) "Pelo serto nos pareceu, vista do mar, muito
grande, porque, a estender olhos, no podamos ver seno
terra com arvoredos que nos parecia muito longa. Nela, at
agora, no pudemos saber que haja ouro, nem prata, nem
coisa alguma de metal ou ferro, nem lho vimos. Porm a terra
em si de muito bons ares, assim frios e temperados, como
os dentre Douro e Minho, porque neste tempo de agora assim
os achvamos como os de l. As guas so muitas; infindas.
E em tal maneira graciosa que, querendo-a aproveitar, dar-
se- nela tudo, por bem das guas que tem. Porm, o melhor
fruto, que dela se pode tirar, me parece que ser salvar esta
gente. E esta deve ser a principal semente que Vossa Alteza
nela deve lanar (...)." Pero Vaz de Caminha. Carta a el-rei
D. Manuel (1 de maio de 1500).

As informaes do texto apresentado permitem
afirmar que:
01. As terras avistadas despertaram o entusiasmo do cronista
pela extenso e pelas possibilidades que ofereciam da
existncia de metais preciosos.
02. As referncias ao clima, s guas, ao solo, natureza e s
possibilidades de evangelizao confirmam a certeza do
cronista que as terras eram habitadas.
04. A possibilidade de os nativos serem salvos apresentava-
se para o cronista como o principal investimento para os
portugueses.
08. Aos olhos do cronista de Cabral, as terras vislumbradas
da caravela ofereciam possibilidades promissoras ligadas
agricultura, pecuria e minerao.
16. As atitudes amistosas dos nativos da Amrica para com
os portugueses, a inocncia de sua nudez e o meio
ambiente descrito pelo cronista confirmavam a
localizao do paraso terrestre.

2. (UFPR) Em 1776, o primeiro ministro do reino portugus,
Marqus de Pombal, escrevia: "Para que prestem utilidade
desejada, as colnias no podem ter o necessrio para
subsistir por si sem dependncia da metrpole".
(LAPA, J.R.A. O ANTIGO SISTEMA COLONIAL.
So Paulo, Brasiliense, 1982.)
U
As palavras de Pombal denotam a "lgica
colonialista" que comandava as relaes Brasil-Portugal. A
respeito de tais relaes, correto afirmar:
01. O comrcio e a navegao da colnia independiam das
atividades comerciais da metrpole.
02. Cabia colnia o papel de suprir a metrpole de
matrias-primas necessrias ao seu enriquecimento.
04. A colnia deveria oferecer mercado consumidor s
manufaturas produzidas pela metrpole.
08. O sistema de monoplio comercial funcionava como um
dos eixos do mecanismo colonial.
16. Para a satisfao do mercado externo, a produo colonial
foi organizada com base na policultura e nas pequenas
propriedades.

Tarefa Mnima #

3. (UFBA) Sobre os fundamentos econmicos do Brasil
colonial, possvel afirmar:

01. A produo colonial orientava-se para o consumo interno
dos gneros tropicais, para a dinamizao do comrcio
entre as capitanias e para a capitalizao dos proprietrios
de terras.
02. A monocultura caracterizou a rea de produo do
acar, enquanto a policultura vigorou em todas as outras
reas econmicas coloniais.
04. A pequena propriedade agrcola, caracterstica da rea de
plantao do fumo, caracterizou tambm as regies de
plantao de algodo, caf e cacau.
08. A escravido indgena, substituda desde cedo em toda a
colnia pela escravido africana, foi condenada pela
Igreja, pelo governo portugus e pelos governos
coloniais.
16. O comrcio colonial era fator de transferncia de lucros
da colnia para a metrpole, j que, detendo legalmente a
exclusividade desse comrcio, Portugal controlava os
preos e as atividades de exportao e importao.
32. A criao de gado, considerada atividade de importncia
secundria, foi desenvolvida em pequenas propriedades
prximas das reas de cultivo do acar, no se
distanciando das zonas de povoamento do litoral.
64. O monoplio comercial estabelecido por Portugal sobre o
Brasil colonial foi derrubado a partir da Unio das Coroas
Ibricas em 1580.

4. (Mackenzie) A rvore de pau-brasil era frondosa, com
folhas de um verde acinzentado quase metlico e belas flores
amarelas. Havia exemplares extraordinrios, to grossos que
trs homens no poderiam abra-los. O tronco vermelho
ferruginoso chegava a ter, algumas vezes, 30 metros(...)
Nufragos, Degredados e Traficantes
(Eduardo Bueno)
Em 1550, segundo o pastor francs Jean de Lery, em um
nico depsito havia cem mil toras.

Sobre esta riqueza neste perodo da Histria do Brasil
podemos afirmar.
a) O extrativismo foi rigidamente controlado para evitar o
esgotamento da madeira.
b) Provocou intenso povoamento e colonizao, j que
demandava muita mo-de-obra.
c) Explorado com mo-de-obra indgena, atravs do
escambo, gerou feitorias ao longo da costa; seu intenso
extrativismo levou ao esgotamento da madeira.
d) O litoral brasileiro no era ainda alvo de traficantes e
corsrios franceses e de outras nacionalidades, j que a
madeira no tinha valor comercial.
e) Os choques violentos com as tribos foram inevitveis, j
que os portugueses arrendatrios escravizaram as tribos
litorneas para a explorao do pau-brasil.
Please purchase PDF Split-Merge on www.verypdf.com to remove this watermark.
Incluso para a Vida Histria B

Pr-Vestibular da UFSC

4
Tarefa Complementar #

5. (Mackenzie) Enquanto os portugueses escutavam a missa
com muito "prazer e devoo", a praia encheu-se de nativos.
Eles sentavam-se l surpresos com a complexidade do ritual
que observavam ao longe. Quando D. Henrique acabou a
pregao, os indgenas se ergueram e comearam a soprar
conchas e buzinas, saltando e danando (...)
Nufragos Degredados e Traficante (Eduardo Bueno)

Este contato amistoso entre brancos e ndios era preservado
a) pela Igreja, que sempre respeitou a cultura indgena no
decurso da catequese.
b) at o incio da colonizao quando o ndio, vitimado por
doenas, escravido e extermnio, passou a ser descrito
como sendo selvagem, indolente e canibal.
c) pelos colonos que escravizaram somente o africano na
atividade produtiva de exportao.
d) em todos os perodos da Histria Colonial Brasileira,
passando a figura do ndio para o imaginrio social como
"o bom selvagem e forte colaborador da colonizao".
e) sobretudo pelo governo colonial, que tomou vrias
medidas para impedir o genocdio e a escravido.

6. (Mackenzie) ...Esta terra, senhor, nela no podemos saber
que haja ouro nem prata, nem coisa alguma de metal ou ferro,
nem lho vimos (...) o melhor fruto que dela se pode tirar me
parece ser salvar esta gente. E esta deve ser a principal
semente que Vossa Alteza em ela deve lanar. (...), pois o
desejo que tinha de tudo vos dizer, mo fez por assim pelo
mido.
Beijo as mos de Vossa Alteza.
Deste Porto Seguro, da Vossa Ilha de Vera Cruz, hoje, Sexta-
feira, primeiro dia de maio de 1500.

Esses trechos da carta do escrivo Pero Vaz de Caminha
apresentam elementos que nos indicam alguns objetivos das
grandes navegaes. Dentre esses objetivos, podemos
destacar:
a) Acabar com a circulao de mercadorias baseada no
bulionismo, em decorrncia da escassez de metais
preciosos na Europa Ocidental.
b) A conquista de terras para a obteno de riquezas, atravs
da renda sobre a terra, defendida pelos tericos fisiocratas
da poca.
c) A obteno de novos mercados de matria-prima e a
poltica do laissez faire para a ampliao do fornecimento
de produtos manufaturados.
d) O processo de crescimento econmico, atravs da
conquista de novos mercados, a catequese e a consequente
afirmao dos Estados Nacionais.
e) A emigrao do excedente populacional europeu,
decorrente da descentralizao poltica e investimento de
capitais na periferia do sistema capitalista.

UNIDADE 3

O CICLO AUCAREIRO

Economia e Sociedade Aucareira
A base da colonizao foi o acar, riqueza trazida de fora;
Portugal j tinha experincia com plantio e a comercializao
do produto nas Ilhas Atlnticas.
Havia o predomnio do latifndio, tpico de
economia aucareira. Gerava altos lucros, ocorria a no-
diversificao de atividades e a monocultura (cultivo de um
s produto).
A mais significativa atividade propiciada pelo
acar foi a criao de gado, para a qual utilizava-se
brutalmente o brao escravo indgena e seu descendente
mestio.
A cana-de-acar exigiu muita mo de obra, e a
soluo inicial foi a escravido indgena, porm, o ndio se
mostrou um "mau trabalhador", pois os homens no
cultivavam a terra.
At os jesutas acabaram se opondo escravido dos
indgenas. Portugal precisou, ento, do brao africano.
O africano e seus descendentes foram a base do sistema
colonial do Brasil. Mais do que ps e mos do Engenho,
tambm foram ps e mos do Brasil.
A condio servil no estimulava ningum a
produzir, o escravo mostrou por todos os meios o quanto
quela situao no lhe servia. Reagiu sempre que pde, e
como pde, fugindo, assassinando e rebelando-se.
Os africanos eram trazidos nos navios "negreiros",
tambm chamados de "tumbeiros", devido quantidade de
pessoas que morriam durante a travessia do atlntico, por
causa das ms condies de higiene, fome, sede, doenas, e
superlotao dos pores dos navios.

Unio Ibrica (1580 1640)
Em 1580, com o objetivo de unificar a Pennsula Ibrica,
Felipe II, rei da Espanha, incorpora pacificamente o reino
Portugus, tornando-se o mais poderoso monarca europeu. O
conceito de "domnio espanhol" um tanto errado, pois
apenas o rei da Espanha passou a ser o mesmo de Portugal, as
naes se mantiveram separadas havendo apenas um vice-rei
em Lisboa.
A principal consequncia da unio ibrica para o
Brasil foi o incentivo penetrao pelo interior, pois o
Tratado de Tordesilhas, que dividia terras entre Portugal e
Espanha, foi suspenso, favorecendo a expanso da pecuria, e
as necessidades do bandeirismo. Gerou tambm novas e
intensas incurses europeias, baseadas nos conflitos entre
Espanha e o resto da Europa. A unio dinstica durou de
1580 a 1640, quando a aristocracia lusa rumou a uma tirania,
e com o apoio francs, tornou Portugal independente, com a
implantao da nova dinastia: a de Bragana, sustentada at a
proclamao da Repblica em 1910.

Invases Holandesas
As invases holandesas foram ocasionadas pelo conflito
entre o capitalismo comercial batavo em expanso, e a
monarquia espanhola aristocrtica e monopolista. Contra
isso, Felipe II rompeu ligaes luso-brasileiras com a
Holanda. Assim criou-se a Companhia do Comrcio
(holandesa), que invadiu a zona canavieira da colnia.
Tendo fracassado a invaso Bahia, os holandeses rumaram
a Pernambuco. A invaso teve como maior responsvel,
Maurcio de Nassau, hbil poltico de financiamentos e
reconstrutor de engenhos, agradando aos latifundirios. O fim
do governo Nassau, e as cobranas aos latifundirios, foram o
sinal para a ruptura. Os senhores, ameaados de perderem as
terras arrendadas, expulsaram os holandeses, caracterizando a
Insurreio Pernambucana, que no passou de uma luta
entre classes dominantes (latifundirios devedores X
Please purchase PDF Split-Merge on www.verypdf.com to remove this watermark.
Incluso para a Vida Histria B

Pr-Vestibular da UFSC

5
comerciantes credores). Aps a expulso dos holandeses,
o acar entra em declnio, pela perda do monoplio. A
segunda metade do sculo XVII foi tempo de crise. Passa-se
a estimular o bandeirismo para a busca do ouro nas Minas
Gerais, que marcaria a segunda fase da colonizao.

Exerccios de Sala #

1. (UFPR) No perodo compreendido entre os anos de 1624 e
1654, o Brasil-colnia foi alvo de duas tentativas de
conquista por parte da Companhia das ndias Ocidentais,
importante empresa mercantil dos Pases-Baixos (Holanda).
Sobre a conjuntura do domnio holands no Brasil, correto
afirmar que:
01. A ocupao holandesa se fez sem resistncia de qualquer
espcie.
02. A invaso foi decidida principalmente em funo dos
lucros que poderiam ser auferidos pela Companhia das
ndias Ocidentais com a explorao do acar, ento a
principal riqueza do Brasil.
04. O ataque colnia era uma tentativa dos Pases Baixos de
atingir a Espanha, pas com a qual travou uma guerra
prolongada, uma vez que, com a Unio Ibrica, o reino de
Portugal e todas as suas colnias haviam passado ao
domnio do Imperador espanhol Filipe II.
08. Com a sada dos holandeses do nordeste brasileiro, a
economia aucareira atingiu o apogeu no Brasil.

2. (UFPR) Nos sculos XVI e XVII, o Brasil foi alvo de
invases e de empreendimentos por parte de diversas naes
europeias. A este respeito correto afirmar que:
01. Os franceses fundaram a Frana Antrtica (1555) no Rio
de Janeiro e a Frana Equinocial (1612) no Maranho.
02. Os ingleses fizeram incurses, saqueando portos e suas
povoaes, bem como apresando cargas de navios.
04. Os holandeses se estabeleceram no nordeste brasileiro de
1630 a 1654, empenhando-se principalmente na produo
e explorao do acar.
08. As invases holandesas estavam ligadas Companhia das
ndias Ocidentais, criada para a explorao mercantil das
colnias na Amrica.

Tarefa Mnima #

3. (FUVEST) Foram, respectivamente, fatores importantes
na ocupao holandesa no Nordeste do Brasil e na sua
posterior expulso,
a) o envolvimento da Holanda no trfico de escravos e os
desentendimentos entre Maurcio de Nassau e a
Companhia das ndias Ocidentais.
b) a participao da Holanda na economia do acar e o
endividamento dos senhores de engenho com a
Companhia das ndias Ocidentais.
c) o interesse da Holanda na economia do ouro e a resistncia
e no aceitao do domnio estrangeiro pela populao.
d) a tentativa da Holanda em monopolizar o comrcio
colonial e o fim da dominao espanhola em Portugal.
e) a excluso da Holanda da economia aucareira e a
mudana de interesses da Companhia das ndias
Ocidentais.
4. A ocupao portuguesa do litoral Norte e Nordeste do
Brasil, em fins do sculo XVI e incio do sculo XVII, deu-se
em virtude dos ataques ingleses, franceses e holandeses a
esse territrio. Sobre estas invases e ocupaes, identifique
as proposies verdadeiras e falsas.
01. Os franceses invadiram Sergipe d'El Rei, a Paraba, o Rio
Grande do Norte, o Cear, o Maranho e o Gro-Par.
02. Os holandeses ocuparam, por longo tempo, os territrios
da Bahia, Pernambuco, Paraba e Rio Grande do Norte.
04. Os franceses, holandeses e ingleses conquistaram todo o
Norte e Nordeste, restando aos portugueses, no sculo
XVI, o domnio do territrio abaixo da Bahia.
08. De todas as invases do sculo XVII, a holandesa foi a
mais duradoura, no sentido da permanncia da ocupao.
Em Pernambuco, o domnio holands se estendeu de 1630
a 1654.

Tarefa Complementar #

5. (Cesgranrio) No sculo XVII, as invases do nordeste
brasileiro pelos holandeses estavam relacionadas s
mudanas do equilbrio comercial entre os pases europeus
porque
a) a Holanda apoiava a unio das monarquias ibricas.
b) a aproximao entre Portugal e Holanda era uma forma de
os lusos se liberarem da dependncia inglesa.
c) as Companhias das ndias Orientais e Ocidentais
monopolizavam o escambo do pau-brasil.
d) os holandeses tinham grandes interesses no comrcio do
acar.
e) Portugal era tradicionalmente rival dos holandeses nas
guerras europeias.

6. (Cesgranrio) A formao do territrio brasileiro no
perodo colonial resultou de vrios movimentos
expansionistas e foi consolidada por tratados no sculo
XVIII. Assinale a opo que relaciona corretamente os
movimentos de expanso com um dos Tratados de Limites.
a) A expanso da fronteira norte, impulsionada pela
descoberta das minas de ouro, foi consolidada nos
Tratados de Utrecht.
b) A regio missioneira no sul constituiu um caso parte, s
resolvido a favor de Portugal com a extino da
Companhia de Jesus.
c) O Tratado de Madri revogou o de Tordesilhas e deu ao
territrio brasileiro conformao semelhante atual.
d) O Tratado do Pardo garantiu a Portugal o controle da
regio das Misses e do rio da Prata.
e) Os Tratados de Santo Ildefonso e Badajs consolidaram o
domnio portugus no sul, passando a incluir a regio
platina.

UNIDADE 4

O CICLO DO OURO

AS BANDEIRAS
As bandeiras nasceram com dois objetivos principais:
aprisionar ndios e encontrar ouro. A busca por novas fontes
de riqueza levou os bandeirantes regio do atual estado de
Minas Gerais. A primeira jazida de ouro foi encontrada em
Caets, no ano e 1693.


Please purchase PDF Split-Merge on www.verypdf.com to remove this watermark.
Incluso para a Vida Histria B

Pr-Vestibular da UFSC

6
A MINERAO
O incio da extrao aurfera na colnia gerou uma srie de
mudanas sociais, polticas e econmicas.
Surgiu um novo tipo de sociedade, mais flexvel que a do
acar. Ocorreu grande aumento nas atividades comerciais,
que por sua vez acarretou significativa urbanizao e levou
ao surgimento de novas cidades como: Ouro Preto,
Sabar, Mariana, So Joo dEl Rey, etc. Novas capitanias
foram criadas (Minas Gerais, Gois e Mato Grosso).
Nas cidades surgiu um novo grupo social: a classe
mdia. Composta basicamente por comerciantes,
profissionais liberais, garimpeiros e funcionrios pblicos
(que tinham funes como cobrar impostos e coibir o
contrabando) a classe mdia deu nova dinmica as atividades
econmicas da colnia.
Deslocou-se do litoral para o sul o comrcio interno
de escravos, mercado de bovinos, sunos e caprinos e
manufatura de tecidos.
A expanso demogrfica durante o sculo XVIII foi
marcante, proporcionando o povoamento de novas regies.
Manifestaes artsticas e culturais, como o barroco,
surgiram na colnia.
A minerao incentivou a atividade pecuria no
extremo sul, necessria para abastecer a regio mineradora
com o charque para alimentar os escravos.
O ouro era monoplio real, a explorao era feita
atravs do arrendamento de lotes ou "datas de minas", que
eram sorteadas aos particulares. Seu tamanho variava
conforme o nmero de escravos do candidato contemplado
Entre outras consequncias do ciclo do ouro, tivemos
tambm a mudana da capital de Salvador para o Rio de
Janeiro (1763). Os Bragana (Reis D. Joo V e D. Jos I),
tornaram-se financeiramente independentes das cortes, graas
aos impostos cobrados no Brasil na poca faustosa (quinto) e
mesmo na decadente (derrama) da minerao.

Tratados de Limites:
- El Pardo (1761), com a Espanha. O oeste do Rio Grande
do Sul ficou para os Espanhis.
- Santo Ildefonso (1777), com a Espanha, que ficou com a
Colnia do Sacramento.
- Badajs (1801), com a Espanha, fixou os limites atuais no
sul.

Exerccios de Sala #

1. (UFSC) Assinale a nica proposio correta. Nos ciclos
sequenciais da economia do Brasil, a ordem cronolgica
01. pau-brasil, acar, ouro, caf.
02. pau-brasil, ouro, acar, caf.
04. pau-brasil, caf, ouro, acar.
08. pau-brasil, acar, caf, ouro.

2. E o pior que a maior parte do ouro, que se tira das minas,
passa em p e em moedas para os reinos estranhos... salvo o
que se gasta em cordes, arrecadas e outros brincos, dos
quais se vem hoje carregadas as mulatas de mau viver...
(Antonil apud MENDES JR., v. 1, p. 219)

A anlise do texto e os conhecimentos sobre o processo de
colonizao da Amrica Ibrica possibilitam afirmar:
01. Os "reinos estranhos" que mais receberam ouro extrado
do Brasil foram a Holanda e a Frana, cujas relaes
mercantis com Portugal lhes asseguravam direitos sobre
esse produto.
02. O jesuta, autor do texto, deixa transparecer o forte
preconceito racial e social que pesava sobre a mulher
no-branca, na sociedade colonial brasileira.
04. Portugal, consciente da importncia das minas do Brasil
para a sua poltica mercantilista, criou as Casas de
Fundio, para evitar a circulao de ouro em p.
08. O grande afluxo de populao para a regio mineradora
contribuiu para o povoamento de reas at ento
desconhecidas, resultando num conflito conhecido como
Guerra dos Emboabas.
16. Ao desembarcar nos atuais territrios do Mxico, Peru e
Colmbia, o colonizador espanhol encontrou uma
populao nativa dedicada predominantemente s
atividades agrcolas e pecurias, que foram estimuladas
pela metrpole, com o objetivo de atender aos interesses
mercantilistas.
32. O desenvolvimento da minerao no Brasil favoreceu o
crescimento de uma camada social intermediria, que se
instalou principalmente nos centros urbanos da zona
aurfera.
64. A produo agrcola e a explorao mineradora, no Brasil
colonial, apresentaram como caracterstica comum o fato
de favorecerem a estagnao do mercado interno, em
decorrncia do controle exercido pela metrpole sobre
suas produes.

Tarefa Mnima #

3. Sobre a pecuria na vida brasileira durante o Perodo
Colonial, correto afirmar que:
01. No sculo XVIII, a pecuria no Sul do Brasil no teve
qualquer relao com o surto minerador das Minas
Gerais.
02. A atividade pecuria foi muito importante para a
expanso do territrio brasileiro.
04. Era realizada desde o sculo XVI na costa nordestina,
coexistindo lado a lado com a produo aucareira.
08. Graas pecuria, a existncia de carne e leite no serto
nordestino amenizava a dura vida dos sertanejos, e o
couro lhes era matria-prima fundamental.
16. Cidades como Feira de Santana, na Bahia, e Sorocaba,
em So Paulo, eram importantes centros de
comercializao de gado.
32. No Rio Grande do Sul, o charque tornou-se grande fonte
de renda.

4. (PUC-SP) Personagem atuante no Brasil colnia, foi
"fruto social de uma regio marginalizada, de escassos
recursos materiais e de vida econmica restrita (...)", teve
suas aes orientadas "ou no sentido de tirar o mximo
proveito das brechas que a economia colonial eventualmente
oferecia para a efetivao de lucros rpidos e passageiros em
conjunturas favorveis - como no caso da caa ao ndio - ou
no sentido de buscar alternativas econmicas fora do quadro
da agricultura voltada para o mercado externo (...)".
Carlos Henrique Davidoff, 1982.
O personagem e a regio a que o texto se refere so,
respectivamente,
a) o jesuta e a provncia Cisplatina.
b) o tropeiro e o vale do Paraba.
c) o caipira e o interior paulista.
Please purchase PDF Split-Merge on www.verypdf.com to remove this watermark.
Incluso para a Vida Histria B

Pr-Vestibular da UFSC

7
d) o bandeirante e a provncia de So Paulo.
e) o caiara e o litoral baiano.

Tarefa Complementar #

5. (UNESP) A partir de 1750, com os Tratados de Limites,
fixou-se a rea territorial brasileira, com pequenas diferenas
em relao a configurao atual. A expanso geogrfica
havia rompido os limites impostos pelo Tratado de
Tordesilhas. No perodo colonial, os fatores que mais
contriburam para a referida expanso foram:
a) Criao de gado no vale de So Francisco e
desenvolvimento de uma slida rede urbana.
b) Apresamento do indgena e constante procura de riquezas
minerais.
c) Cultivo de cana-de-acar e expanso da pecuria no
Nordeste.
d) Ao dos donatrios das capitanias hereditrias e Guerra
dos Emboabas.
e) Incremento da cultura do algodo e penetrao dos
jesutas no Maranho.

6. (Unitau) O bandeirismo foi uma atividade paulista do
sculo XVI e XVII. Suas expedies podem ser divididas em
dois grandes ciclos:
a) O dos capites do mato e de prospeco.
b) O de expanso das fronteiras e de prospeco.
c) Da caa ao ndio e o de busca do ouro.
d) O dos capites do mato e de caa ao ndio.
e) O de expanso das fronteiras e o de busca do ouro.

UNIDADE 5

REVOLTAS DO PERODO COLONIAL

Movimentos nativistas:

Revolta no Maranho (Manuel Beckmann) - Contra a
oposio jesutica escravizao indgena, e contra o
monoplio extorsivo de uma Cia de Comrcio (a Revolta de
Beckmann).
Guerra dos Emboabas (S. Paulo, incio do sc XVIII
(1707) - Paulistas x forasteiros (baianos e portugueses),
devido concorrncia na procura do ouro.
Guerra dos Mascates (1710) - Latifundirios devedores
de Olinda x credores de Recife, de origem portuguesa.
Revolta de Filipe dos Santos (Ouro Preto, ou vila Rica -
1720) - Organizado pelos mineradores contra a instituio do
quinto e das casas de fundio para cobr-lo, impedindo
assim, a sonegao e o contrabando.

Movimentos Emancipacionistas :

- Inconfidncia Mineira(1798): Devido explorao
exagerada de Portugal, a influncia das ideias iluministas e a
independncia das Treze Colnias (EUA). Movimento da
elite, escritores, militares, que buscavam o regime
republicano e universidades. Priso dos principais lderes e
morte de Tiradentes (nico membro popular do movimento).
- Conjurao baiana ou dos alfaiates(1798): Camadas
populares, defendiam a igualdade social e racial, participando
diversos afro-descendentes e mulatos.
- A Revoluo Pernanbucana (1817): Queria uma
Repblica Federativa no Nordeste, com a adeso da Paraba e
do Rio Grande do Norte.

Exerccios de Sala #

1. (UFSC) "...Que estava plenamente provado o crime de
lesa-majestade [...] a que premeditadamente concorriam de se
subtrarem da sujeio em que nasceram e que como vassalos
deviam ter a dita senhora (Dona Maria I), para constiturem
uma Repblica, por meio de uma formal rebelio, pela qual
assentaram de assassinar ou depor General e Ministros, a
quem a mesma senhora tinha dado jurisdio e poder de reger
e governar os povos da Capitania [...]
Portanto, condenam o ru Joaquim Jos da Silva
Xavier, por alcunha Tiradentes, Alferes que foi da tropa paga
da Capitania de Minas, a que com barao e prego seja
conduzido pelas ruas pblicas ao lugar da forca e nela morra
morte natural, para sempre. E que depois de morto, lhe seja
cortada a cabea e levada a Vila Rica, onde em lugar mais
pblico dela, seja pregada em um poste alto, at que o tempo
a consuma e o seu corpo ser dividido em quatro quartos e
pregados em postes pelo caminho de Minas ..."
CASTRO, Therezinha de. "Histria documental do Brasil". Rio
de Janeiro, Record, 1968. p. 123-124.

Analisando o texto, o momento e as circunstncias em que
foi escrito, assinale a(s) proposio(es) verdadeira(s).

01. Trata-se da condenao de Joaquim Jos da Silva Xavier,
conhecido como Tiradentes, que com outros no citados
no trecho, foram julgados por terem participado de uma
insurreio contra o governo portugus.
02. Segundo o texto, entre outros objetivos do movimento
conspiratrio, estava o de proclamar uma Repblica.
04. A conspirao pretendia ainda a abolio da escravatura,
independncia das colnias americanas e a adoo dos
princpios da Declarao dos Direitos do Homem e do
Cidado, proclamada na Frana.
08. O movimento por cuja participao foi condenado
Tiradentes conhecido, na Histria do Brasil, como
"Revolta de Vila Rica"

2. (Cesgranrio) Durante as ltimas dcadas do sculo XVIII,
a colnia portuguesa na Amrica foi palco de movimentos
como a Inconfidncia Mineira (1789), a Conjurao do Rio
de Janeiro (1794) e a Conjurao Baiana (1798). A respeito
desses movimentos pode-se afirmar que:
01.Demonstravam a inteno das classes proprietrias,
adeptas das ideias liberais de seguirem o exemplo da
Revoluo Americana (1776) e proclamarem a
independncia, construindo uma sociedade democrtica
em que todos os homens seriam livres e iguais.
02. Expressavam a crise do Antigo Sistema Colonial atravs
da tomada de conscincia, por parte de diferentes setores
da sociedade colonial, de que a explorao exercida pela
Metrpole era contrria aos seus interesses e responsvel
pelo empobrecimento da Colnia.
04. Denunciavam a total adeso dos colonos s presses da
burguesia industrial britnica a favor da independncia e
da abolio do trfico negreiro para se constituir, no
Brasil, um mercado de consumo para os manufaturados.
08. Representavam uma forma de resistncia dos colonos s
tentativas de recolonizao empreendidas, depois da
Please purchase PDF Split-Merge on www.verypdf.com to remove this watermark.
Incluso para a Vida Histria B

Pr-Vestibular da UFSC

8
Revoluo do Porto, pelas Cortes de Lisboa, liberais em
Portugal, que queriam reaver o monoplio do comrcio
com o Brasil.

Tarefa Mnima #

3. (Cesgranrio) No perodo colonial surgiram vrias
rebelies e movimentos de libertao que questionaram a
dominao portuguesa sobre o Brasil. A respeito dessas
rebelies, podemos afirmar que:
I - Todos os Movimentos de contestao visavam separao
definitiva do Brasil de Portugal.
II - At a 1 metade do sculo XVIII, os movimentos
contestatrios exigiam mudanas, mas no o rompimento
do estatuto colonial.
III - Desde o final do sculo XVIII, os movimentos de
libertao sofreram a influncia do Iluminismo e
defendiam o fim do pacto colonial.
VI - A luta pela abolio da escravatura era uma das
propostas presentes em basicamente todas as rebelies.
V - Uma das razes de vrios movimentos contestatrios era
o abuso tributrio da Coroa Portuguesa em relao aos
colonos.

Esto corretas:

a) Alternativas I, II e III. b) Alternativas I, III e V.
c) Alternativas II, III e IV. d) Alternativas II, III e V.
e) Alternativas III, IV e V.

4. (Fatec) A Conjurao ou Inconfidncia Mineira foi o
primeiro movimento a manifestar de forma clara a inteno
de romper completamente com Portugal.
Dentre os muitos planos desses revolucionrios estava
a) fixar a capital em Sabar e implantar a Repblica, sendo o
primeiro presidente Alvarenga Peixoto.
b) fixar a capital em Mariana e criar uma bandeira com um
tringulo vermelho com a divisa "Libertas Quae Sera
Tamem".
c) fixar a capital em So Joo Del Rei e acabar com a
escravido negra.
d) fixar a capital em So Joo Del Rei e acabar com o
exrcito; em seu lugar atuariam as milcias.
e) fixar a capital em Sabar e premiar as mulheres brancas
que tivessem muitos filhos.

Tarefa Complementar #

5. (Fatec) Em 1798, surge na Bahia um movimento rebelde
conhecido como Conjurao Baiana ou Revolta dos
Alfaiates, que contou com a participao de pessoas das
camadas sociais mais humildes. Esse movimento
a) pretendia fundar uma Universidade, instalar manufaturas
de tecidos e aproveitar o ferro e o salitre da regio.
b) protestava contra os impostos, defendia a abolio da
escravatura e propunha aumento de soldo aos soldados.
c) defendia o fim do Pacto Colonial e o desenvolvimento de
manufaturas txteis e siderrgicas, alm do estmulo
produo agrcola.
d) foi o primeiro movimento de rebeldia a questionar o Pacto
Colonial.
e) no plano poltico contava com elementos adeptos da
repblica, enquanto outros pretendiam uma monarquia
constitucional.
6. (Fei) Foi consequncia da crise da minerao em Minas
Gerais no fim do sculo XVIII:
a) Uma maior interveno metropolitana nos assuntos
coloniais e o consequente aumento da extrao aurfera.
b) O aumento da populao na regio das Minas Gerais.
c) A maior tomada de conscincia por parte dos colonos da
explorao metropolitana, materializada na Inconfidncia
Mineira.
d) O deslocamento do interesse metropolitano para suas
colnias asiticas.
e) O fim da explorao portuguesa e o consequente
enriquecimento dos donos de minas coloniais.

UNIDADE 6

A CRISE NO SISTEMA COLONIAL E O
ESTADO PORTUGUS NO BRASIL

A DECADNCIA ECONMICA DE PORTUGAL:
Portugal desenvolveu ao mximo a ideia de que a colnia s
servia para enriquecer a metrpole. Em 1785, a Rainha D.
Maria I, assinou o famoso alvar que leva o seu nome,
proibindo as manufaturas no Brasil, afim de no desperdiar
os esforos que deveriam se concentrar na agricultura. O
colonialismo mercantilista e monopolista entrou em crise
quando as sociedades coloniais amadureceram. Combateram
impostos extorsivos e desejaram liberdade para comprar e
vender, enquanto o capitalismo em expanso no Velho
Mundo reclamou a expanso dos mercados, opondo-se aos
mercados fechados vigentes em defesa de seus negcios.
Em 1808, ocorreu a vinda da Famlia real portuguesa
para o Brasil, fugindo de Napoleo Bonaparte e da exigncia
do Bloqueio Continental decretado em 1806 contra a
Inglaterra.

Medidas adotadas: abertura dos portos s naes amigas
(Inglaterra); liberdade industrial; Tratado de 1810: taxas
preferenciais para a Inglaterra; Ilha de Santa Catarina: porto
franco aos ingleses; criao do Banco do Brasil; rgos
administrativos; Escola Real de Cincias, Artes e Ofcios e a
Academia e Escola Real de Artes; Imprensa Rgia (primeiro
jornal brasileiro, a Gazeta do Rio de Janeiro e a primeira
revista, Patriota).

Poltica Externa de D. Joo VI: Invaso e ocupao da
Guiana Francesa (1809); Conquista da Banda Oriental,
Uruguai (1816); - Brasil elevado a Reino Unido a Portugal e
Algarves (1815)

Exerccios de Sala #

1. (UFBA) A Independncia da Amrica espanhola
processou-se entre 1810 e 1825. , desta forma, um
acontecimento simultneo ao processado na Amrica
portuguesa. A simultaneidade no uma coincidncia
fortuita. Ao contrrio, componentes histricos em toda a
Amrica Latina tm relao comum entre si.
(RIBEIRO JNIOR, p.61)

Com base no texto anterior e no conhecimento sobre o
processo de independncia na Amrica Latina, indique a
proposio (ou proposies) correta(s).

Please purchase PDF Split-Merge on www.verypdf.com to remove this watermark.
Incluso para a Vida Histria B

Pr-Vestibular da UFSC

9
01. A "simultaneidade" referida no texto resultante da
presena de ideias as iluministas, componentes do quadro
geral de crise do antigo sistema colonial.
02. A independncia da Amrica Latina est diretamente
relacionada dominao napolenica na Pennsula
lbrica, a qual foi responsvel pela de desorganizao dos
laos de dominao metropolitana sobre as reas
coloniais.
04. A independncia do Brasil assume carter singular em
relao da Amrica espanhola, pelo fato de esse pas ter
se separado da metrpole, adotando a monarquia como
forma de governo e mantendo a sua frente um
representante legtimo da Casa de Bragana.
08. A unidade nacional brasileira, conseguida com a
independncia, era uma antiga reivindicao das camadas
populares e foi efetivada em decorrncia das lutas
travadas em todo o territrio nacional.

2. TEXTO I: A presso dos exrcitos de Napoleo e os
interesses ingleses em Portugal e suas colnias levaram o
prncipe regente, D. Joo, a se transferir para o Rio de Janeiro
(1808) com toda a sua famlia (inclusive a me, a rainha D.
Maria I) e sua Corte (...) [em] navios portugueses que saram
de Lisboa trazendo a Famlia Real, membros da Corte e
funcionrios (aproximadamente 15.000 pessoas, em 36
embarcaes).
(DARS, p. 10)
TEXTO II: Na realidade, "quase de sbito, e no maior
atropelo, tomaram-se providncias para o embarque da Corte,
quando as notcias da aproximao das tropas de Junot
traziam alarma a toda a populao. Foi um se salve quem
puder trgico, amargo, caracterstico do nvel de degradao
a que chegara o Reino de Portugal sob o governo bragantino
e de uma classe feudal inepta e corrupta."
(MENDES JR., p. 98)
Analisando os textos anteriores, pode-se concluir:
01. O texto I sugere que a fuga da Corte Portuguesa
aconteceu de forma organizada, endossando a verso
tradicional de que esse era um antigo plano da monarquia
lusitana.
02. O texto II trata a questo da fuga da Famlia Real
Portuguesa para o Brasil de forma alegrica, na medida
em que utiliza elementos satricos, ao analisar um fato
histrico.
04. O texto I enfoca o tema do ponto de vista da
historiografia romntico-oficial, transformando um fato
marcado at por elementos tragicmicos num ato de
racionalidade.
08. O texto II aborda o fato histrico segundo uma linha
crtico-interpretativa, ressaltando suas mltiplas
implicaes.

Tarefa Mnima #

3. A presena no Brasil da Corte e do Prncipe Regente, D.
Joo, criou condies concretas para que a separao do
Brasil em relao a Portugal se tornasse definitiva. A respeito
dessa conjuntura, correto afirmar que:
01. D. Joo manteve a proibio de se instalarem indstria no
Brasil.
02. A abertura dos portos brasileiros liquidou com o
elemento econmico essencial do sistema colonial
ibrico: o monoplio comercial.
04. A instalao da corte portuguesa no Rio de Janeiro
significou a transferncia das decises polticas do
Nordeste para o Sudeste.
08. Ao liberalismo comercial, que interessava aos ingleses e
s elites coloniais, corresponderia, no plano poltico, a
instalao de um Estado Nacional na antiga Colnia.

4. Sobre a crise do antigo sistema colonial, sabe-se:
01. O desenvolvimento do capital industrial e a crise do
Estado absolutista resultaram em contestaes ao sistema
colonial montado segundo os princpios da poltica
mercantilista.
02. De acordo com as teses livre-cambistas, defendidas a
partir da Revoluo Industrial, o sistema colonial era
espoliativo das metrpoles, sempre obrigadas a manter
despesas com suas colnias e a comprar produtos
inferiores por elas produzidos.
04.Os princpios do liberalismo econmico foram
veementemente defendidos pela burguesia e pelo Estado,
em Portugal, porque preconizavam os direitos naturais do
homem, a abolio do trabalho escravo e a soberania das
naes.
08.A Guerra dos Mascates se caracterizou como um
movimento entre colonos e metrpole, enquanto a
Inconfidncia Mineira e a Conjurao dos Alfaiates se
caracterizaram como movimentos anticoloniais.

Tarefa Complementar #

5. Com referncia singularidade do movimento de
emancipao poltica do Brasil, julgue os itens que se
seguem.
01. Ao contrrio da Amrica Espanhola, o Brasil teve um
processo de independncia liderado por foras polticas
renovadoras e ansiosas por uma profunda transformao
das estruturas coloniais.
02. A sociedade poltica colonial que Portugal criou no Brasil
permitiu uma independncia tranquila, sem movimentos
de contestao transio da colnia condio de pas
independente.
04. A unidade territorial mantida no Brasil durante as
negociaes da independncia foi resultado de vrios
fatores, tais como a presena da Corte portuguesa no Rio
de Janeiro e a manuteno do sistema escravista do norte
ao sul do pas.
08. A crise do sistema colonial no Brasil tem causas
econmicas e polticas profundas e bastante diversas
daquelas que conduziram a Amrica Espanhola
independncia.

6. Quanto aos mltiplos aspectos do processo de
independncia do Brasil, que se inicia em 1808 e culmina em
1822, julgue os seguintes itens.
01. A deciso portuguesa de transferncia da Corte para o
Brasil foi um ato de soberania poltica.
02. A permanncia de D. Pedro de Alcntara no Brasil,
coroado como imperador, foi a garantia da continuidade
dos interesses de Portugal com relao ao Brasil.
04. A Coroa britnica ocupou papel primordial nas
negociaes diplomticas que levaram, de forma
gradativa, entre 1808 e 1822, emancipao poltica do
Brasil.
Please purchase PDF Split-Merge on www.verypdf.com to remove this watermark.
Incluso para a Vida Histria B

Pr-Vestibular da UFSC

10
08. A partilha do imprio portugus, prevista no tratado de
Fontainebleau, era parte do intento napolenico de fazer
frente aos objetivos polticos e econmicos da Gr-
Bretanha na Europa Continental.

UNIDADE 5

A Revoluo do Porto e a Regncia de D. Pedro no Brasil:
Dia do Fico. No dia 7 de setembro de 1822, numa viagem a
So Paulo, Dom Pedro recebe as exigncias das Cortes.
Irritado, reage proclamando a Independncia do Brasil.

1 Reinado (1822-31): Regime poltico: Monarquia
Constitucional

A constituio de 1824: outorgada, monrquica,
hereditria, executivo forte e centralizador, poder
moderador, voto indireto e censitrio, catolicismo como
religio oficial do Brasil, regime do padroado.

Reao constituio: Confederao do Equador
(movimento republicano e separatista liderado por frei
Caneca no NE e que foi duramente reprimido)

Exerccios de Sala #

1. Finalmente, seguindo um plano j traado de antemo,
em 2 de julho de 1823, Madeira de Melo e seus homens
deixavam Salvador, pressionados tambm pela esquadra
inglesa comandada pelo Almirante Cochrane que veio
oficialmente em auxilio a Labatut.
(MENDES JR., p. 159)
Em relao ao processo histrico cujo desfecho est
descrito no texto anterior, pode-se dizer:
01. No Nordeste brasileiro, particularmente na Bahia, a
disposio do povo at pela luta armada foi decisiva para
a consolidao da Independncia.
02. As guerras pela Independncia configuram a luta dos
brasileiros contra os representantes do colonialismo
lusitano, ainda presentes em diversas provncias do
Brasil.
04. A luta travada pelo povo baiano buscou resgatar o ideal
da Conjurao dos Alfaiates de fazer da Bahia uma
repblica independente e democrtica.
08. A unio de algumas provncias do Norte e Nordeste em
torno da Republica do Equador foi decisiva para a vitria
dos revolucionrios.

2. A abdicao de D. Pedro I se traduziu na vitria das
tendncias liberais sobre as foras absolutistas
representadas pelo Imperador, completando tambm o
processo de emancipao poltica do Brasil em relao
metrpole portuguesa. O perodo regencial, que segue a
abdicao do Imperador, preparou o caminho para a
consolidao do Imprio. Sobre esse processo correto
afirmar que:
01. A iniciativa mais importante do incio do perodo
regencial foi desencadear vigoroso processo de
industrializao.
02. Foi consolidada a unidade poltica e territorial do Brasil,
apesar dos movimentos provinciais de autonomia.
04. O latifndio e a escravido permaneceram como bases
da sociedade brasileira naquele perodo.
08. A abdicao de D. Pedro I foi possvel porque havia
sido instalado formalmente o regime de
parlamentarismo.

Tarefa Mnima #

3. Brasileiros! Salta aos olhos a negra perfdia, so patentes
os reiterados perjrios do Imperador, e est conhecida nossa
iluso ou engano em adotarmos um sistema de governo
defeituoso em sua origem, e mais defeituoso em suas partes
componentes... O sistema americano deve ser idntico;
desprezemos instituies oligrquicas, s cabidas na
encanecida Europa. Manifesto de Proclamao da
Confederao do Equador, em 12 de julho de 1824.
(MENDES JR., v. 2, p. 169)
Com base no texto anterior e nos conhecimentos sobre o
processo de independncia do Brasil, pode-se afirmar:

01. A "negra perfdia" e os "perjrios do Imperador"
referidos no Manifesto demonstram o desagrado dos
brasileiros para com as atitudes autoritrias tomadas por
D. Pedro I, aps a dissoluo da Assembleia Constituinte.
02. O movimento revolucionrio pernambucano que criticou
o centralismo poltico imposto pela primeira Constituio
pretendia reunir as provncias do Nordeste num governo
republicano e federativo.
04. O "sistema de governo defeituoso em sua origem"
decorreu da participao dos deputados brasileiros nas
Cortes Constituintes de Lisboa e consequente aprovao
de uma nica constituio para o Reino Unido.
08. O sistema de governo mencionado no texto foi
considerado pelos manifestantes "mais defeituoso em
suas partes componentes", porque estabelecia eleies
baseadas no sufrgio universal e igualdade entre os trs
poderes.

4. (Cesgranrio) "Usando do direito que a Constituio me
concede, declaro que hei de muito voluntariamente abdicado
na pessoa de meu mui amado e prezado filho o Sr. D. Pedro
de Alcntara. Boa Vista - 7 de abril de 1831, dcimo da
Independncia e do Imprio - D. Pedro I."
Nesses termos, D. Pedro I abdicou ao trono brasileiro no
culminar de uma profunda crise, que NO se caracterizou
por
a) antagonismo entre o Imperador e parte da aristocracia rural
brasileira.
b) emprstimos externos para cobrir o dficit pblico gerado,
em grande parte, pelo aparelhamento das foras militares.
c) aumento do custo de vida, diminuio das exportaes e
aumento das importaes.
d) presso das elites coloniais que queriam o fim do Imprio
e a implantao de uma Repblica nos moldes dos
Estados Unidos.
e) a introduo do sufrgio universal permitiu a participao
poltica das camadas populares, provocando rebelies em
vrias partes do pas.

Tarefa Complementar #

5. (Cesgranrio) A Constituio imperial brasileira,
promulgada em 1824, estabeleceu linhas bsicas da estrutura
e do funcionamento do sistema poltico imperial tais como
o(a):
A INDEPENDNCIA E O I REINADO
Please purchase PDF Split-Merge on www.verypdf.com to remove this watermark.
Incluso para a Vida Histria B

Pr-Vestibular da UFSC

11

a) Equilbrio dos poderes com o controle constitucional do
Imperador e as ordens sociais privilegiadas.
b) Ampla participao poltica de todos os cidados, com
exceo dos escravos.
c) Laicizao do Estado por influncia das ideias liberais.
d) Predominncia do poder do imperador sobre todo o
sistema atravs do Poder Moderador.
e) Autonomia das Provncias e principalmente dos
Municpios, reconhecendo-se a formao regionalizada
do pas.

6. (Cesgranrio) Assinale a opo que apresenta um fato que
caracterizou o processo de reconhecimento da Independncia
do Brasil pelas principais potncias mundiais.
a) Reconhecimento pioneiro dos Estados Unidos, impedindo
a interveno da fora da Santa Aliana no Brasil.
b) Reconhecimento imediato da Inglaterra, interessada
exclusivamente no promissor mercado brasileiro.
c) Desconfiana dos brasileiros, reforada aps o falecimento
de D. Joo VI, de que o reconhecimento reunificaria os
dois reinos.
d) Reao das potncias europeias s ligaes privilegiadas
com a ustria, terra natal da Imperatriz.
e) Expectativa das potncias europeias que aguardavam o
reconhecimento de Portugal, fiis poltica internacional
traada a partir do Congresso de Viena.

UNIDADE 6



Principais grupos polticos: liberais exaltados ou
farroupilhas; restauradores ou caramurus; liberais moderados
ou chimangos. Com a morte de D. Pedro I acontece uma
rearticulao partidria: parte dos moderados se une com os
exaltados formando os LIBERAIS ou Luzias (Feij) e outra
parte se une com os restauradores formando os
CONSERVADORES ou Saquaremas (Arajo Lima).

Regncia Trina Provisria: para governar a nao, at que
se elegesse a regncia permanente.

A Regncia Trina Permanente: composta pelos deputados
Joo Brulio Muniz (poltico do nordeste) e Jos da Costa
Carvalho (poltico do sul) e pelo Brigadeiro Francisco de
Lima e Silva.

O ato adicional: reforma na constituio do imprio,
Regncia Una.
A regncia do Padre Feij: a exploso das rebelies: ala
progressista dos moderados, renncia.

A regncia de Arajo Lima: ministrio conservador,
violncia contra as revoltas polticas populares que
agitavam o pas.

Revoltas regenciais:
Guerra dos Farrapos - RS entre 1835 e 45 - luta dos
produtores de charque do sul por maior autonomia.
Proclamao da Repblica Riograndense e da Repblica
Juliana.
Sabinada - BA em 1837 - movimento autonomista de classe
mdia
Cabanagem - PA entre 1836 e 37 - movimento popular
contra o autoritarismo do governo central.
Revolta dos Mals - BA em 1835- revolta de escravos
muulmanos aproveitando-se de divergncias nas elites.
Balaiada - MA e PI em 1838-39 movimento popular contra
as arbitrariedades das elites. Aproveitam-se dos atritos entre
conservadores e liberais.

Exerccios de Sala #

1. No perodo compreendido entre a Independncia e 1849, o
Brasil foi marcado por agitaes sociais e polticas. Sobre
essas agitaes, correto afirmar que:
01. A "Cabanagem" no Par (1835-1840) foi um movimento
que teve forte participao das camadas populares.
02. Tambm no Maranho houve violncia social na rebelio
conhecida por "Balaiada" (1838 - 1841), com forte
participao popular.
04.Apenas na Bahia no houve agitao social ou
movimentos visando emancipao regional.
08. A revolta dos liberais em 1842, em So Paulo e em Minas
Gerais, contribuiu para que mais tarde fosse praticada a
alternncia no poder dos partidos Liberal e Conservador.

2. (Fatec) O Ato Adicional de 1834 foi de importncia
significativa para o Brasil porque:
01. Restaurou a paz no Imprio, tendo em vista o trmino das
rebelies no Nordeste do Pas.
02. Possibilitou a tomada do poder pelos conservadores que
formavam a aristocracia rural.
04. Antecipou a maioridade de D. Pedro I, evitando assim um
golpe de Estado dos conservadores.
08. Ampliou a autonomia das provncias, neutralizando a
tendncia centralizadora do Primeiro Reinado.

Tarefa Mnima #

3. (Fgv) Associe os fatos poltico-militares do Primeiro
Reinado e da Regncia brasileira a seguir, com suas
localizaes:
Coluna A
1 - Balaiada
2 - Cabanagem
3 - Ato Adicional
4 - Sabinada
5 - Confederao do Equador

Coluna B
I - Par
II - Bahia
III - Maranho
O PERODO REGENCIAL
Please purchase PDF Split-Merge on www.verypdf.com to remove this watermark.
Incluso para a Vida Histria B

Pr-Vestibular da UFSC

12
IV - Pernambuco
V - Rio de Janeiro

Escolha a alternativa que tem a associao correta:
a) 1 - III; 2 - I; 3 - V; 4 - II; 5 - IV
b) 1 - II; 2 - V; 3 - II; 4 - I; 5 - V
c) 1 - III; 2 - II; 3 - V; 4 - IV; 5 - I
d) 1 - IV; 2 - I; 3 - V; 4 - III; 5 - II
e) 1 - V; 2 - III; 3 - IV; 4 - II; 5 - I

4. (Faap) Lus Alves de Lima e Silva se inicia na tradio de
"O Pacificador" ao comandar as tropas que terminaram a
a) revolta dos liberais paulistas e mineiros em 1842.
b) Balaiada, no Maranho, 1838 1840.
c) Revoluo Farroupilha, no Rio Grande do Sul, 1835
1842.
d) Cabanada, no Par.
e) Sabinada, na Bahia, 1837.

Tarefa Complementar #

5. Em Pernambuco as terras e o poder poltico eram
controlados pela famlia Cavalcanti, enquanto o comrcio era
monopolizado pelos portugueses. Com esta situao, a
populao marginalizada das terras e do trabalho
desencadeou a que seria a ltima revolta popular no Segundo
Reinado.

A revolta popular a que se refere o texto foi
a) a Revoluo Praieira.
b) a Sabinada.
c) a Balaiada.
d) a Guerra dos Farrapos.
e) a Cabanagem.

6. (Mackenzie) Em 1848, os ventos revolucionrios europeus
chegavam a Pernambuco, onde a realidade social era marcada
pelo latifndio, opresso dos Cavalcanti, misria e
concentrao de poder poltico. Mobilizadas as massas
urbanas sob o comando de Pedro Ivo, explodia o ltimo grito
liberal do imprio.
O movimento descrito ficou conhecido como:
a) Sabinada.
b) Cabanagem.
c) Farroupilha.
d) Balaiada.
e) Praieira.

UNIDADE 7

O II REINADO

O Golpe da Maioridade (1840). Foram pacificados os
dois ltimos movimentos liberais (descentralizadores): as
revoltas de MG e SP em 1842 (elite) e a Praieira de 1848 em
PE (camadas populares). Parlamentarismo s avessas:
Ao contrrio do sistema ingls.
As disputas entre liberais e conservadores: Por volta de
1840, os polticos regressistas criaram o Partido
Conservador. E os progressistas constituram o Partido
Liberal. No decorrer do sculo XIX, principalmente no
perodo de 1850 a 1900, o Brasil viveu grande
transformao:
x O centro econmico do pas se deslocou das velhas reas
agrcolas do nordeste para o centro-sul.
x O caf se tornou o principal produto agrcola do pas,
superando todos os demais produtos como acar,
tabaco, algodo e cacau.
x Nas fazendas de caf de So Paulo o trabalho do escravo
foi sendo substitudo pelo trabalho assalariado do
imigrante europeu (italianos, alemes, etc.).
x O dinheiro obtido com a venda do caf foi aplicado na
industrializao do Brasil. Surgiram inicialmente
indstrias alimentares, de vesturio e de madeira.
x As cidades se desenvolveram e surgiram importantes
servios urbanos (iluminao das ruas, dos bondes, das
ferrovias, dos bancos, dos teatros, etc.).
O fim do trfico negreiro: Em 1850 foi extinto o comrcio
de escravos no Brasil, pela lei Eusbio de Queiros.
Baro de Mau: iniciativas como linhas frreas, iluminao
a gs, cabo submarino, investimento bancrio... Mau acabou
falido, pois no teve apoio para os seus projetos. O Brasil
continuava a ser um pas latifundirio, conservador,
aristocrtico e escravista.

Exerccios de Sala #

1. O processo de desenvolvimento da indstria brasileira no
foi acompanhado de uma efetiva poltica protecionista
aduaneira. Quais teriam sido as razes?
01. O Brasil como nao independente optou pelo liberalismo
econmico.
02. A partir da presena da famlia Real no Brasil e durante
todo o sculo XIX, a doutrina econmica que comandou a
industrializao foi o mercantilismo.
04. Os interesses britnicos criaram obstculos realizao
de uma poltica protecionista alfandegria.
08. A ideologia nacionalista encontrou grande ressonncia no
Imprio Brasileiro levando o Governo a praticar o
liberalismo econmico.

2. TEXTO I:
Eu no vejo salvao possvel para o estado desolador desta
provncia, seno quando variarmos de cultura e tratarmos de
proteger direta e indiretamente a indstria manufatureira.
Sem esta indstria no pode manter-se a riqueza pblica.
(Gordilho apud ALENCAR, p. 150)
TEXTO II:
No perteno ao nmero dos que se incomodam por existir
em nossa Provncia um s gnero de cultura: em regra geral
ningum vai explorar uma fonte de que lhe provenha receita
menor, quando pode ter outra mais abundante.
(Baro de Parnaba apud TEIXEIRA, p. 205)

Com base na anlise dos textos e nos conhecimentos sobre a
situao econmica do Brasil durante o Segundo Reinado,
pode-se afirmar:
01. Os autores dos dois textos discutem o mesmo tema,
usando argumentao diferente e concordando nas
concluses.
02. Segundo se depreende dos dois textos, a economia
brasileira na segunda metade do sculo XIX se manteve
como fornecedora de gneros alimentcios e matrias
primas para os pases industrializados.
04. O autor do texto II defende a monocultura do caf porque
os grandes lucros dela decorrentes conseguiram reintegrar
a economia agrcola brasileira no mercado mundial.
Please purchase PDF Split-Merge on www.verypdf.com to remove this watermark.
Incluso para a Vida Histria B

Pr-Vestibular da UFSC

13
08. Na dcada de sessenta, a crescente produo de algodo
no Brasil estava diretamente relacionada ao
desenvolvimento da indstria txtil, nas reas produtoras.

Tarefa Mnima #

3. Qual das alternativas a seguir contm as atividades
produtivas que mais utilizaram a mo de obra escrava nos
perodos Colonial e Imperial no Brasil?
a) Cultura de subsistncia nas colnias de parceria, na regio
Sul e criao de gado nas terras gachas.
b) Extrao de pau-brasil, culturas do fumo e do algodo.
c) Produo de acar, cultura do caf e da minerao.
d) Pecuria e minerao.
e) Comrcio, construo de estradas de ferro e produo de
acar.

4. (UFSC) Assinale a NICA proposio CORRETA. Nos
ciclos sequenciais da economia do Brasil, a ordem
cronolgica
01. pau-brasil, acar, ouro, caf.
02. pau-brasil, ouro, acar, caf.
04. pau-brasil, caf, ouro, acar.
08. pau-brasil, acar, caf, ouro.

Tarefa Complementar #

5. (Fuvest) A economia brasileira durante o perodo
monrquico se caracterizou fundamentalmente
a) pelo princpio da diversificao da produo agrria e pelo
incentivo ao setor de servios.
b) pelo estmulo imigrao italiana e espanhola e pelo
fomento incipiente indstria.
c) pela regionalizao econmica e pela revoluo no sistema
bancrio nacional.
d) pela produo destinada ao mercado externo e pela busca
de investimentos internacionais.
e) pela convivncia das mos de obra escrava e imigrante e
pelo controle do "dficit" pblico.

6. (Cesgranrio) A expanso da agricultura cafeeira no oeste
novo paulista aps 1880 introduziu uma srie de mudanas
na economia e nas relaes sociais da Regio Sudeste, entre
as quais se destaca
a) o reforo das relaes escravistas no interior das fazendas
cafeicultoras, pois os escravos transferidos das fazendas
aucareiras do Nordeste eram a maioria absoluta da mo
de obra nas plantaes do oeste paulista.
b) o desenvolvimento de uma poltica governamental de
distribuio de pequenas propriedades s famlias
imigrantes, que plantavam caf a baixo custo e o vendiam
a menores preos no mercado internacional.
c) a coexistncia de grandes propriedades escravistas e
monocultoras de caf para a exportao, e de pequenas
propriedades de famlias imigrantes, que produziam
gneros de subsistncia para os mercados urbanos.
d) o desenvolvimento de uma poltica governamental de
subveno imigrao, cujo objetivo era estimular o
investimento por parte dos imigrantes, de capitais na
construo de estradas de ferro e nas indstrias nascentes.
e) a substituio do trabalho escravo pelo trabalho livre de
imigrantes europeus no interior das fazendas
cafeicultoras, o que permitiu uma maior lucratividade do
capital cafeeiro e seu investimento em estradas de ferro,
no comrcio e em indstrias.

UNIDADE 8

A CRISE DO IMPRIO BRASILEIRO

Conflitos internacionais: Com a Inglaterra (Questo
Christie 1863-1865).Interveno contra Oribe e Rosas (1851-
1852), presidentes do Uruguai e Argentina, respectivamente.
Depois, a Guerra contra Aguirre (1864-1865), presidente do
Uruguai. Guerra do Paraguai (1865-1870). Brasil, Argentina
e Uruguai (Trplice Aliana) contra o Paraguai e deram incio
ao mais longo e sangrento conflito armado j ocorrido na
Amrica do Sul.


Questo abolicionista: Lei do Ventre Livre (1871); Lei dos
Sexagenrios (1885); 13 de maio de 1888: Lei urea
promulgada pela princesa Isabel: a escravido foi extinta no
Brasil.
Questo republicana: Partido Republicano Paulista,
fazendeiros de caf de So Paulo e contava com seguidores
no Rio de Janeiro, em Minas Gerais e no Rio Grande do Sul.
Questo religiosa: bispos de Olinda e de Belm contra
maons D. Pedro II, influenciado pela maonaria, decidiu
intervir na questo, solicitando aos bispos que suspendessem
as punies.
Questo militar: Depois da Guerra do Paraguai, o Exrcito
brasileiro foi adquirindo maior importncia na sociedade. Os
ideais republicanos contagiaram os oficiais, divulgados por
homens como o coronel Benjamin Constant, professor da
Escola Militar do Rio de Janeiro.
O fim do segundo imprio: A oposio de tantos setores da
sociedade monarquia tornou possvel o tranquilo sucesso do
golpe poltico que instaurou a repblica no Brasil. No dia 15
de novembro de 1889, o Marechal Deodoro da Fonseca
assumiu o comando das tropas revoltadas, ocupando o
quartel-general do Rio de Janeiro. Na noite do dia 15 se
constituiu o Governo Provisrio da Repblica dos Estados
Unidos do Brasil.

Exerccios de Sala #

1. Entre os fatores que atuaram para a extino do trabalho
escravo e o consequente avano do capitalismo no Brasil,
pode-se indicar
01. a reduo do fluxo imigratrio proveniente da Alemanha
e da Itlia, em virtude dos movimentos de unificao
poltica e da estabilidade econmica dessas reas.
Please purchase PDF Split-Merge on www.verypdf.com to remove this watermark.
Incluso para a Vida Histria B

Pr-Vestibular da UFSC

14
02. o desinteresse da burguesia cafeeira de So Paulo em
continuar utilizando o trabalho escravo, uma vez que
operava com o trabalho livre do imigrante europeu.
04. o propsito do governo brasileiro de atender s
solicitaes da aristocracia rural, no sentido de preservar
a produtividade das culturas tradicionais.
08. a luta desenvolvida pela campanha abolicionista, o que
contribuiu para uma maior conscientizao do problema.

2. A Revoluo Praieira, ocorrida em Pernambuco (1848-
1850), foi um dos principais levantes polticos durante o
Imprio brasileiro. Sobre este movimento poltico, podemos
afirmar corretamente:
01. Propiciou a unio de liberais, republicanos e socialistas.
02. Foi uma luta contra a oligarquia dos Cavalcanti-
Albuquerque.
04. Manifestou os ideais polticos do Partido da Ordem.
08. No contou com a participao das camadas populares.

Tarefa Mnima #

3. (UFPR) A economia cafeeira foi o principal meio de
acumulao de capital no Brasil durante o sculo XIX. na
regio do caf que o desenvolvimento das relaes
capitalistas mais acelerado e a que se encontra a maior
parte da industrializao nascente brasileira."
(SILVA, Srgio. EXPANSO CAFEEIRA E ORIGENS DA
INDSTRIA NO BRASIL. So Paulo, Alfa-Omega, 1976.)

A respeito dessas questes, correto afirmar que:
01. O incremento do consumo de caf na Europa e nos
Estados Unidos foi um dos fatores determinantes para a
expanso da lavoura cafeeira no Brasil.
02. Economicamente, A lavoura cafeeira transformou a
Regio Sudeste na mais importante do pas.
04. Ao se examinar o processo histrico brasileiro, nota-se
que h ligao entre expanso cafeeira, imigrao,
urbanizao e industrializao.
08. A burguesia agroexportadora foi responsvel pela
industrializao macia que antecedeu o grande impulso
da economia cafeeira.

4. (UFPR) Em 1850, o Segundo Imprio brasileiro atingiu
seu apogeu. E esse apogeu coincidiu, historicamente, com o
do primeiro ciclo do caf, as questes platinas, o
parlamentarismo e a arte neoclssica e romntica. No que diz
respeito ao sistema parlamentarista brasileiro do Imprio,
correto afirmar que:
01. O Imperador designava o presidente do Conselho de
Ministros.
02. O Presidente do Conselho de Ministros escolhia os
demais ministros.
04. Todos os ministros eram responsveis perante a Cmara
de Deputados.
08. O parlamentarismo de ento se pautava pelo modelo
ingls.

Tarefa Complementar #

5. (Cesgranrio) As Leis Abolicionistas, a partir de 1850,
podem ser consideradas como o nvel poltico da crise geral
da escravido no Brasil, porque

a) a Lei Euzbio de Queiroz (1850) proibiu o trfico quando
a necessidade de escravos j era declinante, face crise
da lavoura.
b) o sucesso das experincias de parceria acelerou a
emancipao dos escravos, crescendo um mercado de
mo de obra livre no pas.
c) a Lei do Ventre Livre (1871) representou uma vitria
expressiva do movimento abolicionista, tornando
irreversvel o fim da escravido.
d) as sucessivas leis emancipacionistas foram paralelas
progressiva substituio do trabalho escravo por homens
livres.
e) a Lei urea, iniciativa da prpria Coroa, visava a garantir
a estabilidade e o apoio dos setores rurais ao Imprio.

6. (Fuvest) A extino do trfico negreiro, em 1850
a) reativou a escravizao do ndio.
b) ocasionou a queda da produo cafeeira no Oeste Paulista.
c) acarretou uma crise na indstria naval.
d) acentuou a crise comercial da segunda metade do sculo
XIX.
e) liberou capitais para outros setores da economia.

UNIDADE 9

REPBLICA VELHA

REPBLICA DAS ESPADAS

Governo Provisrio (1889 - 1891)
Aps a Proclamao da Repblica foi institudo um Governo
Provisrio sob a Presidncia de Deodoro da Fonseca.
Realizaes importantes do Governo Provisrio Republicano:
Expulso da famlia imperial do Brasil;
A liberdade de culto, a separao da Igreja Catlica do
Estado; a instituio do casamento civil obrigatrio;
A criao da Bandeira republicana (19 de novembro) com o
lema "Ordem e Progresso";
As Provncias se tornaram Estados, formando o conjunto dos
"Estados Unidos do Brasil";
A crise econmica do "Encilhamento", ocasionada pela
poltica financeira de Rui Barbosa (Ministro da Fazenda.
A sede do governo passou a ser chamada de Distrito
Federal;

A Constituio de 1891
Promulgada pela primeira Constituinte republicana, foi
baseada na Constituio estadunidense. Extinguiu todas as
formas e instituies monrquicas como o Poder Moderador
e a unio da Igreja-Estado. Adotou a organizao do Estado
em trs Poderes - Executivo, Legislativo e Judicirio - e o
voto universal (no-obrigatrio) para maiores de 21 anos,
com exceo das mulheres, analfabetos, soldados e cabos. O
voto no era secreto e tinha de ser declarado em pblico e
assinalado em listas. Isto permitiu que se desenvolvesse por
todo pas, ento predominantemente rural, uma das maiores
foras polticas da poca: a dos coronis. Assim, as
oligarquias regionais conseguiam impor seus interesses locais
e descentralizadores.
A prtica do "voto de cabresto" marcou todo o perodo
da Repblica Velha. Eleio indireta de Deodoro e Floriano
para governar o Brasil at 1894.

Please purchase PDF Split-Merge on www.verypdf.com to remove this watermark.
Incluso para a Vida Histria B

Pr-Vestibular da UFSC

15
Mal. Deodoro Da Fonseca (1891)
Fechamento do Congresso Nacional com o apoio do
Exrcito e de todos os Estados, com exceo do Par,
governado por Lauro Sodr;
I Revolta da Armada, liderada pelo Almirante Custdio de
Melo.
Para evitar uma guerra civil, Deodoro renuncia sendo
substitudo pelo vice-presidente Marechal Floriano Peixoto.

Floriano Peixoto (1891 - 1894)
Enfrentou a II Revolta da Armada no Rio de Janeiro e a
Revoluo Federalista no Rio Grande do Sul, Santa
Catarina e no Paran.
Represso violenta aos revoltosos, chegando quase
ditadura.
Floriano foi chamado de "Marechal de Ferro" e
"Consolidador da Repblica".
Massacre de Anhatomirim: Desterrenses que apoiaram a
Revoluo Federalista foram executados na Ilha.
Recomendao do Professor Beto: Assistir ao curta-
metragem Desterro com a pessoa amada.

REPBLICA OLIGRQUICA

Prudentes De Morais (1894 - 1898)
Foi o primeiro Presidente civil do Brasil. Com ele iniciou-se,
na repblica o domnio poltico dos fazendeiros;
Foi solucionada a "Questo de Palmas ou misses" com a
Argentina, graas atuao do Baro do Rio Branco.
Cleveland, Presidente dos E.U.A, deu ganho de causa ao
Brasil;
Campanha de Canudos, nos sertes da Bahia, s margens do
rio Vaza Barris. O beato Antonio Conselheiro com sua
pregao de salvao para quem o seguisse, conseguiu
milhares de seguidores fanticos ("jagunos") entre as
populaes miserveis da Bahia, reunindo-os no Arraial de
Canudos. Nas palavras de Euclides da Cunha, autor do livro
"Os Sertes", que acompanhou o episdio como enviado
especial do jornal O Estado de So Paulo, "Canudos no se
rendeu... resistiu at o esmagamento completo, quando
caram seus ltimos defensores, quase todos morreram. Eram
quatro apenas: um velho, dois homens feitos e uma criana".

Campos Sales (1898 - 1902)
Inaugurou a poltica dos Governadores, ou seja, uma troca
de favores entre o Presidente e os Governadores dos Estados.


- A Poltica do Caf com Leite caracterizou-se pela
liderana poltica dos Estados de So Paulo e Minas Gerais
atravs do PRP (Partido Republicano Paulista) e do PRM
(Partido Republicano Mineiro).

Rodrigues Alves (1902 - 1906)
Rodrigues Alves teve a sorte de governar o Brasil na poca
do surto da borracha.
Entre os principais atos do governo de Rodrigues Alves
podemos citar:
Modernizao da cidade do Rio de Janeiro, graas ao
Prefeito Pereira Passos
Combate febre amarela e varola no Rio de Janeiro,
atravs de campanhas de higiene a vacinao, pelo cientista
Oswaldo Cruz, Diretor de Sade Pblica;
Levante da Escola Militar (Rio de Janeiro) devido
instituio da lei de obrigatoriedade da vacina;
-"Questo do Acre": Brasil x Bolvia

Governo de Afonso Pena (1906 - 1909)
Adotou como lema "Governar povoar". Por isso,
incentivou a imigrao, o que possibilitou a entrada de um
milho de estrangeiros no Brasil durante o seu governo;
Afonso Pena faleceu em 1909, sendo substitudo pelo vice-
presidente Nilo Peanha, que completou seu mandato.

Nilo Peanha (1909 - 1910)
Criao do Servio de Proteo aos ndios (SPI), graas
atuao do Marechal Cndido Mariano da Silva Rondon que
desenvolveu uma poltica de atrao pacfica junto aos
indgenas;

CRISE DA REPBLICA OLIGRQUICA

Hermes da Fonseca (1910 - 1914)
Gacho, foi o nico Presidente da Repblica Velha no
escolhido pela "Poltica dos Governadores";
Em seu governo ocorreu a "Poltica das Salvaes",
intervenes do governo federal no Estados, patrocinadas
pelo Senador Pinheiro Machado, para derrubar as oligarquias
que apoiavam a "Poltica dos Governadores"; Lutas
armadas: Cear, o Padre Ccero Romo Batista ("Padim
Cio"), aliado a outros coronis, liderou uma revolta de
sertanejos (Revolta do Juazeiro) No Rio de Janeiro, o
marinheiro negro Joo Cndido liderou a "Revolta da
Chibata" (1910). Iniciada a Guerra do Contestado. O
movimento foi liderado pelo beato Jos Maria.

Venceslau Brs (1914 - 1918)
Promulgao do Cdigo Civil (1916), elaborado por Clvis
Bevilcqua;
Participao do Brasil na 1 Guerra Mundial: declarao de
guerra aos Imprios Centrais (Alemanha e seus aliados) e
participao ao lado dos Estados Unidos, Inglaterra, Frana e
Itlia. O Brasil forneceu alimentos e matrias-primas, alm
de enviar um grupo de mdicos e aviadores para a Europa e
colaborou no policiamento do oceano Atlntico, com os
navios da Marinha de Guerra.
Expressivo crescimento industrial e aumento da
produo agrcola. Poltica de substituio das
importaes e aumento das exportaes.
-Fim da Guerra do Contestado: 1916 - morte de milhares de
sertanejos e soldados.

Epitcio Pessoa (1919 - 1922)
Revolta dos Dezoito do Forte Copacabana
(1922), Semana de Arte Moderna (1922)



Please purchase PDF Split-Merge on www.verypdf.com to remove this watermark.
Incluso para a Vida Histria B

Pr-Vestibular da UFSC

16
Artur Bernardes (1922 - 1926)
Movimentos operrios controlados pela polcia e a Lei da
Imprensa (censura); Coluna Prestes.

Washington Lus (1926 - 1930)
"Governar abrir estradas". Rodovias Rio-So Paulo e a Rio-
Petrpolis;
Queda nas exportaes de caf: Crise Econmica Mundial de
1929 ("Grande Depresso"), Bolsa de NY.
Eleio de 1930: a Aliana Liberal (Getlio Vargas-
Presidente e Joo Pessoa-Vice-presidente).
O movimento revolucionrio: Assassinato de Joo Pessoa.
No dia 03 de outubro, eclodiu a revolta no Rio Grande do
Sul, seguindo-se a do Nordeste, sob a chefia de Juarez
Tvora. No dia seguinte, participavam principalmente tropas
das milcias estaduais e foras arregimentadas por coronis,
que derrubaram Washington Lus e depois entregaram o
poder a Getlio Vargas.

Exerccios de Sala #

1. Os movimentos sociais rurais que caracterizam a Primeira
Repblica podem ser explicados por:
01.Crise na estrutura agrria brasileira, sobretudo no
Nordeste.
02.Pssimas condies naturais da regio nordestina.
04.Grande afluxo de imigrantes, sobretudo nas reas de caf.
08.Tendncia dos elementos mais pobres a sempre
procurarem uma soluo atravs da religio.
16.Disputa entre o governo federal e os posseiros nordestinos
pela terra.

2. (ITA) As razes da violenta represso ao movimento de
Canudos, no interior da Bahia, no incio da Repblica,
deveram-se principalmente
01. preocupao da Igreja com relao ao crescimento de
uma seita que lhe era hostil.
02. s tendncias esquerdistas manifestadas pelos seus
integrantes.
04. aos interesses prejudicados dos latifundirios, privados de
mo de obra farta e barata.
08. impossibilidade de controle fiscal do governo sobre
uma grande rea produtora de minrios.
16. recusa dos sertanejos em trabalhar nas terras dos
latifundirios e terem invadido terras do governo.

Tarefa Mnima #

3. ... Para que V. Excia. faa aos marinheiros brasileiros
possuirmos os direitos sagrados que as leis da Repblica nos
facilitam (...), reformar o cdigo imoral e vergonhoso que nos
rege a fim de que desaparea a chibata, o bolo e outros
castigos semelhantes; aumentar nosso soldo... Ultimato dos
Marinheiros revoltosos ao Presidente Hermes da Fonseca, em
1910.
As reivindicaes contidas no texto referem-se s
expectativas de alguns segmentos sociais quanto Repblica
que almejavam. Das alternativas abaixo, ilustrativas da
realidade poltica sob a Repblica Velha, qual contm um
sentido que se identifica com o ultimato transcrito?

01. A questo operria uma questo de polcia
(Washington Lus);
02. de l (dos Estados) que se governa a Repblica, por
cima das multides que tumultuam agitadas, as ruas da
capital da Unio (Campos Salles);
04. ... por isso, os que no quiserem render-se s persuases,
tero que render-se fora. (Aurelino Leal, Chefe de
Polcia);
08. O povo ficou reduzido a uma verdadeira situao de
impotncia, asfixiado em sua vontade pela ao
compressora dos que detm as posies polticas e
administrativas. (Manifesto dos Tenentes);
16. A 15 de novembro (de 1889) troca-se um trono vitalcio
pela cadeira quadrienal... O caboclo no d pela coisa.
Vem Floriano, estouram as granadas de Custdio na
Revolta da Armada... O caboclo continua de ccoras.
(Monteiro Lobato).

4. Assinale a(s) alternativa(s) incorreta(s):
01. A Revolta da Vacina ocorreu durante a presidncia de
Rodrigues Alves.
02. Podemos dizer que a ecloso da Revolta da Vacina
refletiu a falta de democracia no pas.
04. Os militares e a oposio aproveitaram o
descontentamento popular para se contrapor ao governo.
08. A Revolta da Vacina foi um fato inserido no processo de
reurbanizao feito pelo prefeito Pereira Passos.
16. A Revolta da Chibata foi feita por trabalhadores negros
nas fazendas de caf, tratados como escravos.

Tarefa Complementar #

5. (UFRN) A Poltica dos Governadores, iniciada, na
Repblica Velha, por Campos Sales, baseava-se no(a)
a) domnio das elites oligrquicas estaduais sobre as
populaes rurais, atravs da represso violenta s
constantes revoltas armadas.
b) controle exercido pelas oligarquias sobre os oficiais da
Guarda Nacional, os quais influenciavam fortemente a
conduo da poltica nacional.
c) elaborao de uma poltica de correo dos vcios do
sistema eleitoral, advinda de articulaes entre as
oligarquias e o governo federal.
d) teia de relaes polticas ligadas ao poder oligrquico, a
qual partia do presidente, passava pelos governadores e se
estendia at os eleitores nos municpios tutelados pelos
coronis.

6. (Mackenzie-SP)
Os vaqueiros e os pees do interior escutavam-no em
silncio, intrigados, atemorizados, comovidos Alguma vez,
algum o interrompia para tirar uma dvida. Terminaria o
sculo? Chegaria o mundo a 1900? Ele respondia () Em
1896, mil rebanhos correriam da praia para o serto e o mar
se tornaria serto e o serto mar (). Mario Vargas Llosa.
O carismtico Antonio Conselheiro, de que fala o texto
acima, liderou a Revolta de Canudos em 1897.
Dentre as causas dessa revolta, apontamos:
a) O isolamento do sertanejo, o coronelismo e a luta pela
posse da terra.
b) O apoio incondicional do sertanejo Monarquia.
c) A impossibilidade de adaptao do sertanejo aos valores
republicanos.
d) O crescimento e a modernizao da economia nordestina.
Please purchase PDF Split-Merge on www.verypdf.com to remove this watermark.
Incluso para a Vida Histria B

Pr-Vestibular da UFSC

17
e) A oposio contra a Igreja Catlica aliada dos
monarquistas.

UNIDADE 10

POPULISMO

GETLIO VARGAS (1930-1945)

Governo provisrio (1930 - 1934)

- Revoluo Constitucionalista de 1932;
- a criao dos Ministrios da Educao e Sade;
Trabalho, Indstria e Comrcio;
A Constituio de 1934
Promulgada; abria o direito de voto s mulheres e aos
maiores de 18 anos. Bases da legislao trabalhista: repouso
remunerado, previdncia social e a proteo ao trabalho
da mulher e do menor. Estabeleceu a justia eleitoral, a
justia do trabalho e a militar.

Governo Constitucional (1934 - 1937)

Ao Integralista Brasileira (AIB) e Aliana Nacional
Libertadora (ANL).
O movimento integralista de 1935, a descoberta do Plano
Cohen (forjado pelos integralistas, que continha um plano
comunista para a tomada do poder), a Intentona Comunista e
o apoio dos chefes militares: Golpe de Estado de 1937
(Estado Novo). Getlio decreta o "Estado de Guerra", fecha o
Congresso Nacional e outorga, no dia 10/11/1937, a nova
Constituio ("Polaca").

Estado Novo (1937 - 1945)






x Foi promulgada a CLT e criada a Previdncia Social.
Federalismo altamente centralizado, limitando-se a
autonomia dos Estados em favor do poder central.
Amplos poderes eram concedidos ao Presidente da
Repblica. Unificao do poder poltico, Extino do
cargo de Vice-presidente da Repblica;
x Restries Liberal Democracia, to defendida na
Constituio de 1891. Maior intervencionismo do Estado
Novo, que passou a tomar medidas de diversificao da
agricultura e incentivos industrializao;
x Criao do Ministrio da Aeronutica (Clvis Salgado foi
o primeiro titular desta pasta);
x Reuniu-se na CLT (Consolidao das Leis do Trabalho)
toda a legislao trabalhista (1943);
x Criao da Companhia Siderrgica (1941) (Usina de
Volta Redonda) e a Companhia Vale do Rio Doce (1942)
para extrair minrios;
x Surgimento de diversos territrios federais (Fernando de
Noronha, Amap, etc);
x Outorgou a Constituio de 1937, que instituiu um
federalismo centralizado;
x Enviou a FEB para os campos da Europa (Itlia);
Nomeou novos interventores para os Estados;
x Proibiu greves; Extinguiu todos os partidos polticos;
Regulamentou a pena de morte no Brasil;
x Criou o Departamento de Imprensa e Propaganda (DIP).
Institutos do Acar e do lcool (IAA), do Mate e do
Pinho;
x Plano Quinquenal que apresentava os seguintes itens:
usina de ao, fbrica de avies, usina hidreltrica em
Paulo Afonso, estradas-de-ferro e de rodagem;
x Participao do Brasil na Segunda Guerra. Em janeiro de
1942, o governo rompeu relaes diplomticas com os
pases do "Eixo" e permitiu a instalao de bases navais e
areas no Nordeste Brasileiro (Fernando de Noronha e
Natal); a Marinha de Guerra cooperou no patrulhamento
do Atlntico; DASP - Departamento de Administrao do
Servio Pblico;
x PTB (Partido Trabalhista Brasileiro), PSD (Partido Social
Democrtico) e UDN (Unio Democrtica Nacional).
Pela primeira vez o Brasil passou a ter partidos polticos
de atuao nacional.
x A reconstitucionalizao do pas (deposio de Getlio
Vargas pelos chefes militares em 29/10/1945).
x As Foras Armadas entregaram o governo ao Ministro do
Supremo Tribunal Federal, Jos Linhares, que realizou as
eleies (O General Eurico Gaspar Dutra saiu vitorioso).

EURICO GASPAR DUTRA (1946 - 1951)

Os principais fatos do governo Dutra foram:

- Promulgao da Constituio de 18 de outubro de 1946.

Principais caractersticas: Federalismo, Eleies diretas;
Presidencialismo (5 anos); Autonomia e harmonia dos trs
poderes; Cmara de Deputados e Senado Federal.

SEGUNDO GOVERNO DE GETLIO VARGAS
(1951 - 1954)
Criao da PETROBRS (Lei 2.004, de 03 de outubro de
1953), empresa que estabeleceu o monoplio estatal da
pesquisa e explorao de petrleo do Brasil;
Surgimento de sucessivas crises polticas devido s
acirradas crticas movidas pela oposio. Em 24 de agosto
de 1954 Getlio Vargas suicidou-se, o vice-presidente Caf
Filho assume o governo. Tendo convocado novas eleies, a
chapa Juscelino Kubistchek e Joo Goulart eleita.
Por motivo de doena, Caf Filho se afasta da
Presidncia, sendo substitudo por Carlos Luz (Presidente da
Cmara dos Deputados), que permaneceu no poder apenas 48
horas. Sob alegao da preparao de um golpe militar para
impedir a posse dos eleitos, o General Teixeira Lott (Ministro
da Guerra) afasta Carlos Luz da Presidncia. O Congresso
Nacional declara impedido Carlos Luz e nomeia Nereu
Ramos (Presidente do Senado) como presidente, que depois
passar a faixa presidencial para Juscelino Kubistchek.

JUSCELINO KUBISTCHEK (1956 - 1961)
O governo Juscelino foi marcado por um desenvolvimento
acelerado. Esse surto econmico foi denominado
nacionalismo desenvolvimentista. "Cinquenta anos em
cinco"):
- Construo de Braslia, o que possibilitou a transferncia
da capital do Rio de Janeiro para o Planalto Central (21 de
abril de 1960);
Represso + Concesses + Propaganda


P.E. CLT D.I.P.
Please purchase PDF Split-Merge on www.verypdf.com to remove this watermark.
Incluso para a Vida Histria B

Pr-Vestibular da UFSC

18
- Construo de Usinas Hidreltricas de Furnas e Trs
Marias; Implantao da indstria automobilstica e
construo naval; Execuo do "Plano de Metas"
(transporte, energia e alimentao); Construo da rodovia
Belm-Braslia; Envio do Batalho Suez, a pedido das
Naes Unidas, para atuar no Oriente Mdio (Faixa de Gaza,
entre Israel e Egito); Criao da SUDENE (Superintendncia
para o Desenvolvimento do Nordeste); No governo Juscelino
Kubistchek as empresas multinacionais penetraram
maciamente no Brasil e as grandes realizaes no setor
econmico significaram um agravamento da inflao,
aumentando enormemente o custo de vida.

JNIO QUADROS (GOVERNOU APENAS 7 MESES)
(1961)
- Procurou desenvolver uma poltica externa independente
(condecorao de Che Guevara);
- Eleito com grande nmero de votos, a maior votao de
toda histria republicana at ento, Jnio Quadros
renunciou ao cargo no dia 25 de agosto de 1961.

JOO GOULART (1961 - 1964)
Joo Goulart assumiu no dia 07 de setembro de 1961, com
um sistema Parlamentarista.
Foram escolhidos como Primeiro-Ministros, respectivamente:
Tancredo Neves, Brochado da Rocha e Hermes Lima.
Contando com o forte apoio popular, Joo Goulart convocou
um Plebiscito para 6 de janeiro de 1963, para decidir sobre
a manuteno ou no do Sistema Parlamentar. O resultado foi
a volta do Presidencialismo, que lhe devolvia os poderes
retirados em 1961. A tenso que reinaria durante o perodo de
governo de Joo Goulart atingiu o seu ponto mximo quando
o presidente criou uma lei implantando o 13 salrio. O
Congresso recusou-a e os trabalhadores entraram em greve.

Exerccios de Sala #

1. (Mackenzie-SP) So fatores que impulsionaram a
indstria durante o Estado Novo:
01. O governo democrtico de Vargas, que contava com forte
apoio da sociedade civil.
02. A regulamentao das relaes capital e trabalho,
marcadas pelo liberalismo, e a ausncia de interveno do
Estado Novo na rea trabalhista.
04. A reduo das importaes, fruto da desvalorizao da
moeda e a ecloso da II Guerra Mundial, fato que
contribuiu para ampliar o mercado de exportaes e
reduzir a concorrncia.
08. A captao de capitais externos, base do modelo
econmico adotado por Vargas no perodo.
16. O crescimento do mercado externo em funo da
participao brasileira na II Guerra Mundial ao lado das
potncias do Eixo.

2. O governo de Getlio Vargas, durante o qual foi institudo
o Estado Novo, terminou em outubro de 1945 por
01. fim de mandato.
02. renncia.
04. morte natural.
08. deposio por golpe militar.
16. suicdio.
Tarefa Mnima #

3. (UFPR) A respeito das tenses polticas e sociais descritas
na Carta Testamento de Vargas (24/08/1954), assinale as
alternativas verdadeiras:
01. Getlio Vargas identificava a si prprio como defensor
dos direitos dos trabalhadores brasileiros e arquiteto da
soberania nacional.
02. Getlio Vargas tentava minimizar a importncia do
desenvolvimento industrial, defendendo reiteradamente o
predomnio da cafeicultura.
04. Getlio Vargas havia optado pelo investimento dos
recursos do Estado nas indstrias de base como forma de
garantir a independncia econmica nacional.
08. Getlio Vargas reconhecia a sua responsabilidade na
instaurao da ditadura do Estado Novo (1937-1945),
fazendo uma autocrtica dos abusos perpetrados pela
represso policial naquele perodo.
16. Getlio Vargas criticava os setores empresariais que no
concordavam com sua poltica trabalhista.
32. Getlio Vargas defendia a privatizao das empresas
estatais, como a Eletrobrs e a Petrobrs, tidas por ele
como ineficientes e atrasadas, principais responsveis
pelo dficit pblico e pela inflao.

4. (UEPG-PR) Alm da reativao das relaes bilaterais
com os pases socialistas, em base de respeito mtuo e
visando ao incremento do comrcio, o Brasil considera
essencial diminuio da tenso mundial uma poltica de
fortalecimento das Naes Unidas. Para o governo brasileiro,
a ONU, sendo menos que um superestado, mais do que a
soma de seus estados-membros e no foi feita para ser
utilizada por eles, isoladamente ou em grupo, como
instrumento de sua poltica paroquial ou de seus interesses
mais imediatistas () Foram elas criadas () para
salvaguardar a paz e a segurana de todos (), permitir a
justa aplicao do princpio de autodeterminao dos povos,
vitalizar a cooperao internacional para o desenvolvimento
econmico () O Brasil apia todos esses objetivos ().
Afonso Arinos de Melo Franco, ministro das Relaes
Exteriores, 1950. De acordo com o texto, assinale o que for
correto.
01. O Brasil se posiciona favoravelmente organizao de
blocos econmicos e polticos, para a manuteno da
segurana mundial.
02. O Brasil defende o restabelecimento das relaes
diplomticas com os pases socialistas, pois estas
resultariam em desenvolvimento comercial e alvio da
tenso internacional.
04. A ONU no deve servir como instrumento de interesses
poltico-econmicos de pases-membros, isolados ou em
grupo.
08. A ONU um superestado que deve ser dotado de plenos
poderes para a resoluo de diferentes problemas
internacionais.
16. Os objetivos do Brasil e dos Estados Unidos em relao
aos pases socialistas so coincidentes.

Tarefa Complementar #

5. (UFPR) A nova democracia brasileira difere
radicalmente do modelo registrado na tradio. E a diferena
mais notvel est em que, nesta democracia de massas, o
Estado se apresenta de maneira direta a todos os cidados.
Com efeito, todas as organizaes importantes que se
Please purchase PDF Split-Merge on www.verypdf.com to remove this watermark.
Incluso para a Vida Histria B

Pr-Vestibular da UFSC

19
apresentam como mediao entre o Estado e o indivduo so,
em verdade, antes anexos do prprio Estado que rgos
efetivamente autnomos (...). O sistema partidrio, por outro
lado, tem base nos dois agrupamentos (PSD e PTB) criados
por Getlio e, em larga medida, dependentes do seu prestgio
pessoal (...). Neste quadro poltico em que o Estado, atravs
dos lderes populistas, se pe em contato direto com as
massas no h lugar de destaque para as ideologias. Os
aspectos decisivos da luta poltica as formas de aquisio e
preservao do poder esto vinculados a uma luta entre
personalidades. WEFFORT, F. O populismo na poltica brasileira.
Rio de Janeiro: Paz e Terra, 1980.

Considerando o perodo da histria do Brasil situado entre
1946 e 1964, assinale as alternativas verdadeiras:
01. A democracia brasileira, no perodo em questo, foi
sustentada mais por lideranas carismticas do que por
partidos fortes, amparados em ideologias claras.
02. No perodo histrico mencionado, as oligarquias agrrias
de So Paulo e Minas Gerais organizaram um sistema
partidrio que buscava, acima de tudo, a incluso da
classe trabalhadora na vida poltica.
04. No populismo, a estrutura poltica brasileira afastou-se do
modelo tradicional de democracia, na medida em que o
Estado procurava manter as massas sob seu controle.
08. Nos anos 1950, teve destaque a participao da Ao
Integralista Brasileira como a principal organizao
poltica mediadora das relaes entre Estado e
trabalhadores.
16. A tentativa de domnio poltico das massas pelo governo
tornou-se manifesta em 1947, quando o Partido
Comunista do Brasil foi novamente colocado na
ilegalidade.
32. A poca mencionada no texto refere-se complexa
conjuntura produzida pela crise do governo do General
Dutra, lder de um regime militar cujo fim proporcionou a
reorganizao partidria em torno de lideranas de
esquerda.

6. (UEPG-PR) Os anos que separam a queda de Vargas da
ascenso militar de 64 constituem um perodo de grande
diversificao e criatividade cultural. Sobre esse perodo,
assinale o que for correto.
01. As classes mdias nesse perodo rejeitavam as novidades
culturais e de bens de consumo.
02. Nele surgiu a bossa-nova, movimento de modernizao e
internacionalizao da msica popular brasileira.
04. Jnio Quadros, eleito presidente pela coligao
PTB/PSD, fortaleceu a hegemonia desses partidos em
nveis regional e nacional.
08. Desenvolveu-se o Cinema Novo, iniciado com os filmes
de Gluber Rocha, marcando a passagem do cinema de
arte para uma revoluo de linguagem e de temas.
16. Fundaram-se os Centros Populares de Cultura (CPCs),
envolvendo estudantes, artistas e intelectuais, para levar
arte e cultura aos trabalhadores.
UNIDADE 11

DITADURA MILITAR

MARECHAL CASTELO BRANCO (1964 - 1967)
Editou os Atos Institucionais nmeros 1, 2, 3 e 4 que
concedia amplos poderes ao Presidente da Repblica para
cassar mandatos, suspender direitos polticos, etc Extinguiu
os antigos partidos polticos (UDN, PSD, PTB, etc) atravs
de uma reformulao partidria, o que resultou no
bipartidarismo (ARENA e MDB); Promulgou a
Constituio de 24 de janeiro de 1967; O regime militar
instaurado e respaldado juridicamente por Atos Institucionais
dissolveu as organizaes consideradas "subversivas" como a
C.G.T. (Comando Geral dos Trabalhadores) e a U.N.E.
(Unio Nacional dos Estudantes).

MARECHAL COSTA E SILVA (1967 - 1969)
Editou o AI-5 (Ato Institucional n 5), concedendo ao
Presidente da Repblica poderes totais, inclusive de fechar o
Congresso Nacional. Perodo de governo marcado por muitas
agitaes polticas e atos de terrorismo em diversos pontos do
pas.
Devido enfermidade, foi substitudo por uma Junta Militar
composta pelos Ministros do Exrcito, Marinha e
Aeronutica, vindo a falecer em breve.
A Junta transmitiu o poder a Garrastazu Mdici, nome
indicado pelo Alto Comando das Foras Armadas e eleito
indiretamente pelo "Colgio Eleitoral".
Com a posse de Mdici entrou em vigor a Emenda
Constitucional de 1967 que ampliou o mandato presidencial
para cinco anos.

GENERAL EMILIO GARRASTAZU MDICI
(1969 - 1974)
Governo marcado pela criao de "Projetos de Impacto"
(Ponte Rio-Niteri), (Transamaznica), eliminao da
guerrilha urbana e rural, bem como pelos poderes
excepcionais que o Presidente dispunha. O "Milagre"
Econmico brasileiro.

GENERAL ERNESTO GEISEL (1974 - 1979)
O governo do Presidente Geisel conheceu uma crise
econmica marcada pelo fim do "milagre brasileiro" e pela
crise energtica mundial, quando ocorreu a elevao dos
preos do petrleo, que repercutiu internamente, baixando a
taxa de investimento pblico. Ministrio da Previdncia e
Assistncia Social (MPAS). Assinou um acordo de
cooperao nuclear com a Alemanha, para o Brasil assimilar
tecnologia avanada para fins pacficos. O mandato
presidencial passou para seis (06) anos; Iniciou a abertura
poltica (revogao dos Atos Institucionais e
Complementares). Construo da hidreltrica de Itaipu.
Criao do Pr-lcool (programa Nacional do lcool).

GENERAL JOO BATISTA FIGUEIREDO
(1979 - 1985)
"Fazer do Brasil uma Democracia", ou seja, dar continuidade
ao processo de abertura iniciado pelo governo Geisel.
Continuou o processo de "abertura poltica" e concedeu a
anistia, visando democratizao do pas. Estabeleceu uma
reforma partidria, que possibilitou o surgimento de novos
partidos polticos: PMDB, PT, PFL, PTB, etc. O
restabelecimento das eleies diretas para governadores de
Estados; Negociaes econmicas com o Fundo Monetrio
Internacional (F.M.I.)

Fim da Repblica Militar
Vigoroso movimento em favor das "Diretas J" eclodiu no
pas.


Please purchase PDF Split-Merge on www.verypdf.com to remove this watermark.
Incluso para a Vida Histria B

Pr-Vestibular da UFSC

20
Exerccios de Sala #

1. (UFMT) No dia 28 de agosto de 1999, o Brasil
comemorou os vinte anos da aprovao da lei da anistia aos
implicados em crimes polticos durante o regime militar no
pas. Sobre o assunto, marque os itens verdadeiros:
01. Durante a presidncia de Ernesto Geisel, o primeiro
presidente civil brasileiro, foi concedida tal prerrogativa
aos exilados polticos.
02. Desde 1974, entidades como a Ordem dos Advogados do
Brasil OAB vinham pressionando o governo brasileiro
no sentido da aprovao da lei da anistia integrando um
amplo movimento de luta contra a ditadura.
04. Alm das presses internas, como a manifestao pblica
que reuniu mais de 20.000 pessoas em 1979 no Rio de
Janeiro, presses externas, como a poltica de direitos
humanos dos Estados Unidos, contriburam para a
aprovao dessa lei.
08. O projeto aprovado pela ampla maioria do Congresso
Nacional previu anistia ampla, geral e irrestrita, porm
deixou de fora os torturadores e autores de fraude
eleitoral, exatamente como previa o projeto das
oposies.

2. (Unifor-CE) O atentado ao Riocentro, em 30 de abril de
1981, em que explodiu uma bomba num carro, matou um
sargento e feriu gravemente um capito do Exrcito, resultou
da reao
01. das entidades estudantis, proibidas de comemorar o Dia
do Trabalho pelo presidente Mdici.
02. da extrema direita civil e militar contra as medidas
redemocratizantes do presidente Figueiredo.
04. da esquerda radical operria e estudantil contra a poltica
antiinflacionria do presidente Castelo Branco
08. dos setores da esquerda contra as medidas tomadas pelo
presidente Ernesto Geisel que resultou no Pacote de
Abril.
16. da guerrilha urbana contra as medidas ditatoriais do
presidente Costa e Silva que colocou o Congresso em
recesso indefinido.

Tarefa Mnima #

3. (PUC-SP) O perodo militar brasileiro recente (1964-
1985):
a) Destacou-se pelo forte crescimento econmico nacional,
associado aplicao de vrios projetos voltados
diminuio das diferenas sociais e superao das
barreiras entre as classes.
b) Ocorreu simultaneamente presena de ditaduras militares
em outros pases latinoamericanos, como a Argentina, o
Chile e o Uruguai, o que caracteriza uma fase militarista
na histria latino-americana.
c) Caracterizou-se pela preservao da democracia, a
despeito a disposio autoritria de alguns grupos
militares, que desejavam suprimir direitos polticos de
membros da oposio.
d) Iniciou-se com o golpe militar que deps o presidente
Joo Goulart e encerrou-se com as eleies presidenciais
diretas e a convocao da Assembleia Constituinte ao
final do governo Mdici.
e) Contou com forte presena militar e poltica dos Estados
Unidos, que utilizaram o territrio brasileiro como base
para a instalao de msseis anticubanos, dentro do
cenrio da Guerra Fria.

4. (Univali-SC) O filme O que isso Companheiro?
identifica uma fase da Histria do Brasil na qual se pode
constatar muita contradio entre o que o governo pregava
e o que realmente acontecia. O governo pregava um
imbatvel crescimento econmico, glrias, ordem; nos
bastidores: censura, represso, perseguio aos opositores da
ordem. A partir da leitura do filme ou mesmo do prprio
enunciado, podemos constatar que se trata do perodo:
a) Da Repblica do caf-com-leite 1894-1930 perodo no
qual os coronis mandavam em tudo e em todos.
b) Da ditadura de Vargas (Estado Novo).
c) Especificamente do governo do General Costa e Silva.
d) Da vigncia das regncias, devido a pouca idade de D.
Pedro II.
e) Que corresponde ditadura militar de 1964-1985.

Tarefa Complementar #

5. (PUC-RS) A vitria de Fernando Henrique Cardoso nas
eleies presidenciais de 1994 teve como fator decisivo a:
a) Adoo de uma poltica eficaz de controle da natalidade,
visando a conscientizar uma parcela da populao menos
favorecida.
b) Reduo da criminalidade no campo, devido ao programa
de reforma agrria que prev tolerncia em relao
invaso de terras improdutivas no pas.
c) Poltica externa de importao de produtos do Mercosul,
com o objetivo de reduzir as taxas alfandegrias,
resultando em preos mais atrativos no mercado
brasileiro.
d) Implantao do Plano Real, que criou uma moeda estvel
no pas aps dcadas de inflao.
e) Queda do desemprego devido adoo do plano de
estatizao e interveno reguladora do Estado na
economia.

6. (UFRJ) Com 105 votos a mais do que os 336 necessrios,
a Cmara aprovou ontem o pedido de Impeachment do
Presidente Collor, (...) mesmo aps a concluso da CPI sobre
Paulo Csar Faria, o PC. (...) A votao foi acompanhada por
multides que ocuparam ruas e praas das principais cidades
do pas, festejando, voto a voto, o afastamento do Collor...
Jornal do Brasil. Rio: 30/09/92. p. 1.
O afastamento de Fernando Collor se deu entre outros
motivos pela:
a) Oposio externa sua poltica econmica.
b) Inexistncia de composio com a oposio.
c) Denncia de corrupo no interior de seu governo.
d) Alta inflao, apesar do confisco da poupana.
e) Adoo pelo presidente de uma poltica neoliberal.

UNIDADE 12

REDEMOCRATIZAO

NOVA REPBLICA (1985 - ...)
No final de junho de 1984, o PDMB lanou o nome de
Tancredo Neves disputa no Colgio Eleitoral. Em 15 de
janeiro, o Colgio Eleitoral deu 480 votos a Tancredo Neves
e 180 a Paulo Maluf, candidato governista. Entre fins de
Please purchase PDF Split-Merge on www.verypdf.com to remove this watermark.
Incluso para a Vida Histria B

Pr-Vestibular da UFSC

21
janeiro e princpios de fevereiro de 1985, Tancredo visitou os
Estados Unidos e vrios pases da Europa. Com o seu
regresso ao Brasil, tiveram incio as negociaes visando
formao do ministrio. Um dia antes da posse, marcada para
15 de maro de 1985, Tancredo Neves foi submetido a uma
cirurgia de emergncia. Jos Sarney tomou posse como
presidente na manh do dia 15. Tancredo Neves faleceu na
noite de 21 de abril, depois de ter sido submetido a sete
cirurgias. Na manh do 22, Sarney foi confirmado na
presidncia. No dia 23 o corpo de Tancredo Neves chegou ao
aeroporto de Belo Horizonte, para receber as homenagens de
cerca de 1,8 milho de pessoas. Finalmente, no dia 24, na
presena de 50 mil pessoas, foi enterrado no cemitrio de
So Joo del Rei. [Fonte: Dicionrio Histrico Biogrfico Brasileiro
ps 1930. 2 ed. Rio de Janeiro: Ed. FGV, 2001]

GOVERNO JOS SARNEY (1985 - 1990)
O pas atravessou sucessivas crises nesses cinco anos de
governo: crise econmica com altssimas taxas de inflao;
crises ministeriais (tivemos quatro Ministros da Fazenda
neste perodo); crise de credibilidade, pois o pas requereu a
moratria da dvida externa (suspenso temporria do
pagamento aos credores estrangeiros).
Alguns planos econmicos foram postos em prtica
na tentativa de combater a inflao, atravs de congelamentos
de preos e salrios:
- Plano Cruzado (1986), quando foi instituda uma nova
moeda chamada de "Cruzado";
- Plano Bresser (1987);
- Plano Vero (1989) com a instituio do Cruzado Novo.
A Constituio de 1988, em vigor, estabeleceu:
eleies diretas em todos os nveis (federal, estadual,
municipal); legalizao dos partidos de qualquer tendncia,
inclusive Comunista e Socialista; concesso do direito de
voto aos analfabetos, em carter optativo, aos menores de 18
e maiores de 16 anos e tambm de 70 anos; liberalizao das
atividades sindicais (direito de greve inclusive para os
funcionrios pblicos civis); uso irrestrito dos meios de
comunicaes nas campanhas eleitorais.

GOVERNO FERNANDO COLLOR (1990 - 1992)
Abertura do pas s importaes; privatizao das empresas
estatais; deu continuidade ao combate inflao ("Plano
Cruzado II"). "Esquema PC": afastamento da presidncia
pelo Congresso Nacional, em 25 de dezembro de 92. Foi
substitudo pelo Vice-presidente Itamar Franco. Plano Collor.

GOVERNO ITAMAR FRANCO (1992 - 1994)
Reforma monetria ("Plano Real") com a instituio de uma
nova moeda (Real); estabilizao da economia com reduo
significativa da inflao graas ao Ministro da Economia
Fernando Henrique Cardoso; continuidade do plano de
privatizao (Cia Siderrgica Nacional, Usiminas, etc).
No fundo, FHC e o plano Real foram apenas o "tempero"
para que o Brasil fosse servido para o mundo neoliberal.

GOVERNO FERNANDO HENRIQUE CARDOSO
(1995-2002)
Adotou uma poltica neoliberal segundo a atual tendncia da
globalizao da economia mundial; reduziram os gastos
pblicos e estatais; reforma constitucional (reeleio);
promoveu o ajuste fiscal; reformas
(adm/judicirio/tributrio); poltica de combate a corrupo.

GOVERNO LULA (2003-2010)
O ex-metalrgico Luiz Incio Lula da Silva foi eleito
presidente da repblica aps ter sido derrotado 3 vezes na
disputa eleitoral.
Sofre crtica forte da antiga oligarquia e tambm de
esquerdistas que desejam rupturas mais rpidas com o capital
externo.
- Misso Centenrio do astronauta Marcos Pontes;
- Realizao dos jogos Pan-americanos;
- Denncias de corrupo (escndalos como do mensalo e
dos cartes corporativos);
- Plano de Acelerao do Crescimento (PAC).

Exerccios de Sala #

1. (UFMT) O Brasil atual tem vivido entre a expectativa da
soluo definitiva dos problemas econmicos, o fantasma da
crise social e a consolidao de sua democracia. A respeito,
marque os itens verdadeiros:
01. Nos ltimos anos tem aumentado sensivelmente o
nmero de organizaes que atuam nos mais diversos
campos (sindical, poltico partidrio, ecolgico,
comunitrio), fortalecendo a ideia da defesa coletiva dos
interesses dos cidados.
02. Muitos dos princpios constitucionais aprovados na Carta
de 1988 tm sido alterados pelo atual governo. Dentre as
principais alteraes esto s relativas aposentadoria
dos trabalhadores e reeleio.
04. Graas s fortes presses populares verificadas no
movimento das diretas j, foi implantado, em 1996, o
Plano Real que deu incio ao controle inflacionrio e ao
pagamento da dvida externa brasileira.
08. Um dos elementos necessrios e fundamentais para a
consolidao da democracia o respeito opinio
pblica, elemento capaz de mobilizar a sociedade civil
contra ou a favor das mais diversas causas.

2. (UFRN) Segundo Cludio Vicentino, a globalizao
estimulou a formao de blocos econmicos regionais, com a
diminuio dos protecionismos e atrao de investimentos
internacionais. A isso, somou-se a preocupao com o limite
dos gastos governamentais, a prevalncia da economia de
mercado e a busca de um Estado mnimo. A crescente
fora privada e a crise do Estado intervencionista deram
impulso, por sua vez, s pregaes neoliberais. No Brasil,
essa situao se manifesta concretamente atravs do(a)
01. Programa Nacional de Reforma Agrria, com o qual o
governo pretende assentar famlias de sem-terra em reas
consideradas improdutivas.
02. Processo de reformulao constitucional que modifica a
organizao poltico-partidria, privilegiando os pequenos
partidos.
04. Privatizao de empresas estatais, provocando a reduo
da influncia e da ingerncia do Estado, principalmente
nos setores produtivos da economia.
08. Elevao da taxa de emprego, ocasionado pelo aumento
da atividade econmica do setor privado, especialmente
na rea industrial.

Tarefa Mnima #

3. (UFU-MG) Sobre os anos 90 no Brasil, assinale a
alternativa correta.
Please purchase PDF Split-Merge on www.verypdf.com to remove this watermark.
Incluso para a Vida Histria B

Pr-Vestibular da UFSC

22
a) Enquanto a presso do desemprego levou os sindicatos a
perderem fora nas negociaes salariais, o movimento
dos trabalhadores sem-terra (MST) foi se tornando um
dos principais movimentos sociais dos anos 90, lutando
pela ampliao da reforma agrria.
b) A poltica de moralizao da vida pblica foi a tnica do
primeiro governo de Fernando Henrique Cardoso,
levando-o a exigir que todos os escndalos financeiros e
as denncias de corrupo fossem investigados pelo
Congresso, por meio de CPIs, que conseguiram julgar e
prender todos os envolvidos.
c) A poltica de privatizaes, marca do governo Fernando
Henrique Cardoso, permitiu que o Estado investisse os
recursos obtidos em sade e educao, contribuindo para
diminuir significativamente as desigualdades sociais, de
acordo com os mais recentes indicadores do ndice de
Desenvolvimento Humano IDH.
d) A orientao poltica predominante nos anos 90, de cunho
neoliberal, foi responsvel pela diminuio acentuada dos
nveis de pobreza, pelo aumento da participao dos
salrios de renda nacional e pelo fortalecimento da
capacidade do setor pblico em atender as demandas
sociais.

4. (UTFPR) O governo de Fernando Henrique Cardoso
segue linhas acordadas pelo Consenso de Washington,
realizado em 1989. Tal consenso, de carter neoliberal,
adotou um conjunto de medidas para controlar a inflao e
modernizar o Estado. Entre estas medidas esto:
a) Ajuste fiscal, reduo do tamanho do Estado e
fortalecimento das empresas estatais.
b) Privatizao, abertura financeira e investimentos em infra-
estrutura bsica.
c) Fortalecimento do sistema previdencirio atual com
privatizao e investimento em infra-estrutura bsica.
d) Ajuste fiscal, restries econmicas s importaes de
gneros agrcolas e abertura financeira.
e) Abertura comercial, abertura financeira e regulamentao
do funcionamento da economia.
Tarefa Complementar #

5. (UFSE) Entre as caractersticas do modelo econmico que
passou a ser implementado no Brasil, especialmente a partir
do incio dos anos 90, esto a:
a) Privatizao e a abertura comercial.
b) Abertura comercial e a criao da CLT.
c) Estatizao e a desregulamentao econmica.
d) Interveno acentuada do Estado na economia e o rgido
controle de preos.
e) Privatizao e a criao de regras que dificultam a
importao.

6. (UFRJ) Desde o incio dos anos 90 o Brasil vem
experimentando os efeitos das polticas adotadas pelos
Governos Collor, Itamar Franco e Fernando Henrique
Cardoso. As principais caractersticas deste modelo poltico,
considerado por muitos como neoliberal, so:
a) O pleno emprego e o desenvolvimento econmico, com
base nos investimentos estatais e nas parcerias com o
setor financeiro.
b) O controle da inflao e da dvida pblica, a partir da
reduo dos impostos, da negociao da dvida externa e
da elevao salarial.
c) A reduo da interferncia do Estado na economia
(Estado-mnimo) e a abertura ao capital externo e s
privatizaes, alm da reduo de gastos do Estado,
atravs de reformas constitucionais.
d) Os investimentos exclusivos na poltica de bem-estar
social, expressos nos assentamentos dos Sem-Terra e na
Ao da Cidadania Contra a Fome, privilegiando a
redistribuio de renda e a permanncia do homem no
campo.
e) A valorizao das organizaes dos trabalhadores, visando
construir parcerias na luta contra o desemprego.

Please purchase PDF Split-Merge on www.verypdf.com to remove this watermark.
Incluso para a Vida Ingls

Pr-Vestibular da UFSC
1
UNIDADE 1

SIMPLE PRESENT TENSE

O Simple Present Tense usado para descrever aes
habituais ou uma verdade em geral. Voc no encontrar
dificuldade neste tempo verbal, porque para todas as
pessoas o verbo ficar igual, exceto na terceira pessoa do
singular (he , she , it), onde o verbo receber um s.

Exemplos:
Nancy plays chess every night.
Thieves always steal.

Pay Attention:
- Se o verbo terminar em: s, ss, ch, sh, x, z ou o,
acrescenta-se es.
George does his homework every night.
Leo watches TV every Saturday night.
He never brushes his teeth.

- Se o verbo terminar em y precedido de uma
consoante, tira-se o y e acrescenta-se ies.
Kate studies at night.
Karen cries at the end of every romantic film.

- Se o y vier aps uma vogal, acrescenta-se apenas o
s.

Patricia usually plays cards by herself.
Everybody says that I am too short.

- Expresses usadas com o Simple Present:
day
night
afternoon
Every week
month
year
Sunday

a day
Once a year


twice a week
a month

-Alguns advbios de frequncia:

Always, never, sometimes, usually, hardly, seldom,
rarely

Interrogative and Negative forms
(affirmative form)
Technology protects people.
They show some pictures to me.

(interrogative form)
Does technology protect people?
Do they show some pictures to me?

(negative form )
Technology doesnt protect people.
They dont show some pictures to me.
Assim observamos que o auxiliar do Simple Present
does para a terceira pessoa do singular (he, she, it) e
do para as demais pessoas (I , you , we , they).

Exerccios de Sala#

1. Choose the best form of the verbs to complete the
sentences:
A - Mr. Martin __________________ downtown on foot
every afternoon.
a) to go b) go c) gos d) goes

B - James usually________________ black clothes on
Saturday night.
a) wears b) is wearing c) wear d) to wear

C - We _________ exactly what we __________.
a) knows / wan c know / wants
b) know / want d) knows / wants
2. Complete the questions with do or does :
a)______________ Helen live in London ?
b)______________ those girls work hard ?
c)______________ Susan have two lovely sisters ?
d)______________ you like to read books ?

3. Choose the best option to complete the following
sentences:
My uncle ___________ his new car.
a) like b) likes c) liking d) is like

4. Teenagers __________ that new song.
a)love b) loves c) loved d) NRA

5. Put the sentences below in the Interrogative and
Negative forms:
a) I drink milk once a week.
I - __________________________________________
N ____________________________________________

b) She does the dishes three times a day.
I - ___________________________________________
N - ___________________________________________

c) She has breakfast early.
I - __________________________________________
N - __________________________________________

d) Marcelo studies Physics every Monday.
I - ___________________________________________
N - ___________________________________________

6. Complete the spaces with the correct form of the
Simple Present Tense:
a) He ______________ (to plan) the crime.
b) Susan ______________ (to Know) exactly what she
__________ (to want).
c) Mary ____________ (to spend) hours walking.
d) I ____________ (to drink) soft drinks.

Tarefa Mnima #

Text one
The tree is a beautiful machine that works for the
benefit of man. Most forms of life on Earth need oxigen.
We do not get oxygen from industrial machines. We get it
Please purchase PDF Split-Merge on www.verypdf.com to remove this watermark.
Incluso para a Vida Ingls

Pr-Vestibular da UFSC
2
from the tree machine. The tree machine makes the
oxygen that we need to take from the air.
Industrial machines pollute the air. Trees do not
pollute the air. On the contrary, they clean it. The
industrial machines make a lot of noise. The tree machine
does not make any noise. On the contrary, it filters noise.
The tree machine does not need coal, oil, gas or
electricity to work. Its fuel comes from water, sun light
and carbon dioxide. When a tree dies, it helps new plants.
The combination of water, insects, and micro-organisms
in the soil causes the decomposition of a tree. When a tree
dies, it gives nutrients to the soil for other trees use as
fuel. So, the tree does not die, really. It recycles its
substance for the benefit of all animal and plant life.
Please remember: the tree is your friend. Do not
destroy it.
Vocabulary:
Earth: Terra (planeta)
To need: precisar (de)
To clean: limpar
Noise: barulho
Coal: carvo
Fuel: combustvel
To die: morrer
To destroy: destruir
Soil: solo

7. According to the text, choose the correct alternative to
complete the sentence:
Trees dont cause pollution, they _______ the air.
a) take
b) come
c) pollute
d) clean
e) get

8. Choose the correct alternative, according to the text
above:
a) Trees make a lot of noise.
b) When a tree dies, it recycles its substances.
c) Trees need electricity to work.
d) Trees dont need sunlight
e) The soil is composed by coal, oil, and gas.

9. According to the text, the author compares a tree with:
a) A machine
b) The Earth
c) A man
d) Life
e) Micro-organisms

10. According to the text, select the correct answer for
the following question:

Who does the tree help when it dies?
a) The industries.
b) Insects and micro-organisms.
c) All animal and plant life.
d) The oxygen in the air.
e) The soil, the water, and the air.

11. Select the correct answer for the following question,
according to the text :
Does the tree machine need coal, gas or electricity to
work?
a) No, it doesnt.
b) Yes, it does.
c) No, they dont.
d) Yes, they do.
e) Yes, it needs

12. Select the correct English translation for the
following sentence:
Ns vivemos no planeta Terra.
a) You live on the planet Earth.
b) Live in the planet the Earth.
c) We can live on the planet Earth
d) We live on the planet Earth.
e) You lived in the planet Earth.

13. Identify the sentences in which the underlined verb
forms are in the Simple Present Tense:
1. The tree is a beautiful machine.
2. Trees dont pollute the air.
3. Industrial machines are polluting the air.
4. Water and insects caused decomposition.
5. The trees filter the noise.
Select the correct, sequence from the top to the
bottom:
a) 1 3 - 4
b) 2 3 4
c) 2 4 5
d) 1 2 5
e) 1 2 3

14. Select the alternative which presents the correct
affirmative form of the underlined verb in the sentence:
We dont get oxygen from industrial machines.
a) are getting
b) gets
c) get
d) have got
e) got

UNIDADE 2

Text two
A DAY IN THE LIFE OF FRED LEWIS

Fred Lewis, a fashion designer from New York,
talks to Margot Townsend:
I get up every day at eleven o'clock. I dont like
getting up early. I never eat breakfast I sometimes have
a cup of tea. I eat nothing during the day. I just dont feel
hungry at all.
I live about eight minutes from the office. I drive to
the office every morning in my car a black BMW. I love
driving. I always wear black or navy blue. I dont like
bright colors.
I work from about noon till seven in the evening. I
dont mind working late. Before a fashion show I usually
work until eleven or twelve at night. I eat out in
restaurants about six nights a week. I hate going to new
restaurants. I usually go to a few old favorites.
I live with my daughter, whos eighteen. The
apartment is always untidy. She hates cooking and
Please purchase PDF Split-Merge on www.verypdf.com to remove this watermark.
Incluso para a Vida Ingls

Pr-Vestibular da UFSC
3
cleaning the house and so do I! I dont spend much time
in my home.
I usually stay in New York at the weekend. I dont
mind that because I hate making plans. The traffic in New
York is always terrible but sometimes I just get in my car
and drive.
Vocabulary:
Matter: problema
Staircase: escadaria
Meat: carne
Noon: meio-dia
Almost: quase
Daughter: filha
Until: at
Can: poder
To stay: ficar, permanecer
To spend: passar
To hate: odiar
To mind: importar-se

Exerccios de Sala #

1. About Freds daughter is NOT true that...
a) Shes eighteen.
b) She hates cleaning the house.
c) She likes driving her fathers car.
d) She lives with her father.
e) She dislikes cooking.

2. Mark the correct proposition according to text two.
a) He dislikes driving.
b) He lives in a beautiful house.
c) He never works till late at night.
d) Fred does not eat during the day.
e) Fred hates going to old restaurants.

3. Mark the correct proposition according to text 2.
a) Fred gets up at twelve o'clock.
b) Fred eats out almost every day.
c) Fred is a website designer.
d) Fred lives far from his work.
e) Fred likes cleaning the house.

4. Answer the question according to text two.
Does Fred live in a flat?
a) No, he dont.
b) Yes, he does.
c) No, he doesnt.
d) Yes. he is.
e) No, he isnt.

5. Which question(s) CAN be answered according to text
2:
01. Who does Fred live with?
02. When was the fashion show?
04. What is the traffic in New York city like?
08. Where does Fred live?
16. How old is Freds daughter?
6. Text two contains information about Freds...
01. daily routine
02. daughter
04. wife
08. occupation
16. habits

Text Three
The Mountains
The Himalayas are the best-known mountain range
in the world and Mt Everest, with a height of 8,880
meters, is the highest mountain. Since Edmund Hillary
made the first ascent in 1953, mountaineers from many
countries have managed to climb to the peak. Normally
they need to take oxygen cylinders to help them breathe
and other special equipment including ropes to connect
themselves to each other. Its a dangerous sport and many
people have lost their lives, not just in the way up but
during the descent as well.
Vocabulary:
High: alto
Height: altura
Ascent: subida

Tarefa Complementar #

7. Mark the correct proposition according to the text.
a) The second person who climbed the Everest was Sir
Edmund Hillary.
b) Some climbers die when coming down the mountains.
c) It is an easy sport because the climbers use ropes and
other equipments.
d) The mountaineers never have to take oxygen
cylinders.
e) There are avalanches on the Everest.

8. Mark the correct proposition according to the text.
a) The Everest is the highest mountain in Brazil.
b) There is no need of equipment to climb the Everest.
c) People from different nationalities have climbed
Mount Everest.
d) Edmund Hillary is a famous peak.
e) People go snowboarding on the Everests slopes.

9. Mark the correct proposition according to the text.
a) During the climbing is not difficult to breathe because
there is much oxygen available.
b) Helmets are important equipment for the climbing.
c) In some parts of the climbing the mountaineers use
ropes to tie themselves to each other.
d) Mt Everest is almost eight hundred meters high.
e) Mountain-climbing is not a sport.

10. Which question cannot be answered, according to the
text?
a) When did Edmund Hillary climb the Everest?
b) Why do climbers need to take oxygen cylinders?
c) How long is the ascent of Mount Everest?
d) What are the Himalayas?
e) How high is the Everest?

11. Answer the following question according to the text:
Does a mountaineer need to take oxygen cylinders?
a) No, they dont.
b) Yes, he does.
c) Yes, they do.
d) No, they didnt.
e) Yes, they does.
Please purchase PDF Split-Merge on www.verypdf.com to remove this watermark.
Incluso para a Vida Ingls

Pr-Vestibular da UFSC
4
A song
LET IT BE (The beatles)
When I ___________(1)myself in time of trouble
Mother Mary ___________(2)to me
Speaking words of _____________(3, let it be...
And in my hours of darkness
She's standing right in front of me
Speaking words of wisdom, let it be...

And All the broken hearted people
Living in the world ____________(4)
There will be an answer, let it be...
For though they may be parted
There is still a chance that they ____(5)see
There will be an answer let it be...

Let it be, let it be, let it be, let it be
____________(6)words of wisdom, let it be...

And when the night ________(7)cloudy
There's still a light that _____________(8) on me
Shine until tomorrow, let it be...
I wake up to the sound of music
Mother Mary comes to me
Speaking words of wisdom, let it be...
Let it be, let it be, let it be, let it be
There will be an answer, let it be...
Let it be, let it be, let it be, let it be
Whisper words of wisdom, let it be...

UNIDADE 3

SIMPLE PAST TENSE

O Simple Past tense usado para descrever aes
acabadas num tempo determinado e para aes habituais
no passado.
Tambm no Simple Past tense, usamos uma
nica forma do verbo para todas as pessoas (exceto o
verbo to be).

Veja os exemplos:

I taught / You taught / He taught / We taught / etc.

-Expresses que geralmente aparecem com o Simple Past:

week
month
last night
year
Saturday


One hour
Two days
A week ago
Two minutes

Interrogative and Negative forms

(affirmative form) Helen wrote a letter to me yesterday.
(interrogative form) Did Helen write a letter to me
yesterday ?
(negative form) Helen didnt write to me yesterday.

Assim observamos que o auxiliar do Simple Past
o did e usamos nas formas Interrogativa e Negativa. O
verbo principal da orao fica no infinitivo sem o to.

REGULAR AND IRREGULAR VERBS

Regular Verbs
Os verbos regulares em Ingls so reconhecidos pela
terminao ed tanto no simple past quanto no past
participle.
Veja os exemplos:
Infinitive Simple past Past participle
To love (amar) loved loved
To walk (caminhar) walked walked
To work (trabalhar) worked worked

H algumas particularidades que voc precisa saber
com relao aos verbos regulares:

- Se o verbo terminar em e, acrescenta-se apenas o
d.
Infinitive Simple past Past participle
To dance (danar) danced danced
To agree (concordar) agreed agreed

- Se o verbo terminar em y e for precedido de vogal,
deixa-se o y e acrescenta-se ed.
Infinitive Simple past Past participle
To play (jogar, brincar) played played
To pray (rezar) prayed prayed

- Se o verbo terminar em y e for precedido de
consoante, tira-se o y e acrescenta-se ied.

Infinitive Simple past Past participle
To study (estudar) studied studied
To try (tentar) tried tried

- Se o verbo tiver uma slaba e terminar em consoante
vogal consoante (c-v-c), dobra-se a ltima
consoante e acrescenta-se ed .

Infinitive Simple past Past participle
To stop (parar) stopped stopped
To plan (planejar) planned planned

- Se o verbo tiver mais de uma slaba e a ltima for
tnica, dobra-se a ltima slaba e acrescenta-se ed.

Infinitive Simple past Past participle
To admit (admitir) admitted admitted
To omit (omitir) omitted omitted

Irregular Verbs
Os verbos irregulares no apresentam regras para a
formao do Simple past e Past participle ,ou seja, cada
um tem uma forma prpria de passado.
Veja os exemplos:

Infinitive Simple past Past participle
To tell (contar) told told
To keep (guardar) kept kept
To teach (ensinar) taught taught



Please purchase PDF Split-Merge on www.verypdf.com to remove this watermark.
Incluso para a Vida Ingls

Pr-Vestibular da UFSC
5
Exerccios de Sala #

1. (UEL):
I ____________ to school everyday when I was a child.
a) walk b) will walk c) have walked d) walked

2. (UFC):
Im terribly upset. My team _____ its third game this
season.
a) lose b) loses c) losing d) lost

3. Choose the best option to complete the following
sentences:
I __________ her six days ago.
a) leave b) have left c) left d) leaved

4. They ___________ to learn Biology last month.
a) begin b) began c) begun d)nra

5. Put the sentences below in the Interrogative and
Negative forms:

a)You bought a new car two years ago.
I - __________________________________________
N - __________________________________________

b) I drank milk when I was a child.
I - __________________________________________
N - __________________________________________

c) They watched the soccer game yesterday.
I - __________________________________________
N - ___________________________________________

Tarefa Mnima #

Text Four
An attractive young woman had just deplaned at
Los Angeles International Airport, and stood at the gate
waiting for someone in the crowd to greet her. After some
time had passed, she approached a man who had not yet
greeted anyone. Are you waiting for a girl named
Debbie? she asked cautiously.
The man hesitated for a moment, then smiled broadly
and gave her a bear-hug and a semi-passionate kiss.
Finally he stepped back and, a little sheepishly, replied,
No, Im not.

6. Select the incorrect alternative, according to text:
a) The man and Debbie didnt know each other.
b) The woman stood at the gate.
c) The girl asked the man if he was waiting for a person
called Debbie.
d) They left the airport together.
e) The girl did not know the person who would pick her
up at the airport.

7. Which question(s) below cannot be answered,
according to the text?
01. Who was the man waiting for?
02. Where did she land on?
04. Whats the girls name?
08. How did the man greet the girl?
16. When did she leave Los Angeles?

8. The text mentions:
01. The mans name;
02. The gate number Debbie was waiting;
04. The type of kiss she received from the man;
08. Where she went when she left the airport ;
16. The airports name.

UNIDADE 4

Text Five
When Charlie Chaplin was at the peak of his
popularity, there was a Charlie Chaplin Contest in an
American theater. They promised to give a silver cup to
the person who made himself look exactly like Charlie.
The great artist decided to enter the contest himself. He
came second.

Exerccios de Sala #

1. According to the text, mark the incorrect proposition:
a) Many people knew Charlie Chaplin.
b) The artist entered in the contest.
c) The contest was in the USA.
d) The prize was a silver cup.
e) Chaplin was American.

2. The text is in the...
a) Simple Past Tense;
b) Simple Present Tense;
c) Simple Future;
d) Present Continuous Tense;
e) Past Perfect Tense.

3. According to the text, the correct answer for the
following sentence is...
Which contest did Charlie Chaplin participate?
a) A contest for participating in a film.
b) A contest for participating in a theater play.
c) A sport contest.
d) An imitation contest.
e) A contest for the TV.

4. The interrogative form of the sentence below is ...
Charlie Chaplin was a great comedian.
a) Was a great Charlie Chaplin comedian?
b) Did he was a great comedian?
c) Does he be a great comedian?
d) Was he a great comedian?
e) He wasnt a great comedian?

A Song
NORWEGIAN WOOD (THE BEATLES)
Once _________ (1) a girl, or should I say,
She once had me...
She __________ (2) me her room, isn't it good,
Norwegian wood?

She asked me to stay and she __________ (3) me to sit
anywhere,
So I looked around and I __________ (4) there wasn't a
chair.
I __________ (5) on a rug, biding my time, drinking her
Please purchase PDF Split-Merge on www.verypdf.com to remove this watermark.
Incluso para a Vida Ingls

Pr-Vestibular da UFSC
6
wine.
We talked until two and then she __________ (6),
"It's time for bed"

She told me she __________ (7) in the morning and
started to laugh.
I told her I didn't and ____________ (8) off to sleep in the
bath

And when I ___________ (9), I was alone, this bird had
flown
So I ____________ (10) a fire, isn't it good, Norwegian
wood...

Text Six
Michael Schumacher

Michael Schumacher was born in Hurt-
Hermuhlheim, Germany, on 3 January 1969. When he
was only four years old he was given a toy kart, and later
a lawnmower engine was fitted to it. Schumachers
parents thought he was not safe driving his kart on the
pavements around their home so they arranged for him to
use the local kart track.
At the age of 15, he won his first major
championship the German Karting championship. When
he was nineteen, he was driving racing cars and
immediately began winning. At the age of 22, he took part
in his first Grand Prix, driving a Jordan, but shortly after
that he joined the Benneton team.
From: Reading Plus Mary underwood Macmillian
Publishers Lt. 1994.
Vocabulary
Toy: brinquedo
Lawnmower Engine: motor de mquina de cortar grama
To fit: adaptar
Safe: seguro
Pavements: caladas
To arrange; providenciar
Kart track: pista de Kart
To win (past- won): vencer
To turn on: voltar a ateno para
Racing cars: carros de corrida
To join: juntar-se
Lecture: palestra
Words: palavras

Tarefa Mnima #
5. According to text one, choose the correct alternative:
Michael Schumachers nationality is ...
a) Swiss
b) American
c) Swedish
d) Dutch
e) German

6. According to the text, which question CANNOT be
answered?
a) What kind of cars did he drive?
b) When was he born?
c) What did they fit to Schumachers toy kart?
d) Where was he born?
e) When did he win his first F-1 race?
7. Choose the alternative which contains a correct
English translation.
a) Ele comeou sua carreira na equipe Benneton.
He finished his career in the Benneton team.
b) Quem o piloto mais jovem?
Who is the newest pilot?
c) Ele tem quinze anos.
He has fifteen years old.
d) Ele foi ferido em um acidente.
He was hurt in an accident.
e) Ele um dos melhores pilotos do mundo.
He is one of the worst pilots in the world.

UNIDADE 5

SIMPLE FUTURE and CONDITIONAL

O Futuro Simples em Ingls usado para
expressar aes futuras, e quase sempre usado para
expressar pedidos, promessas e oferecimentos futuros.
O Futuro Simples formado pelo auxiliar will
(para todas as pessoas) e pelo verbo principal sem o to
do infinitivo.

Veja os exemplos:
Solar energy will be an alternative kind of energy.
We will be back soon.
She will tell us about the computer.

Voc poder abreviar o auxiliar do Futuro Simples da
seguinte maneira:
Ill go to the beach tomorrow.
Juliell travel to Europe.

A forma interrogativa do Futuro Simples feita
colocando-se o auxiliar will antes do sujeito , ou seja ,
no incio da orao.
Will solar energy be an alternative kind of energy ?
Will we be back soon?
Will she tell us about the computer?

A forma negativa do Futuro Simples feita
acrescentando-se o not ao auxiliar (will + not =
wont) .
Solar energy will not (wont) be an alternative kind of
energy.
We will not (wont) be back soon.
She will not (wont) tell us about the computer.

Expresses usadas no Simple Future
Tomorrow
The day after tomorrow
Tonight
Soon
week
Next Month
Year
Monday
Veja:
He will come soon.
She will be back next Saturday.



Please purchase PDF Split-Merge on www.verypdf.com to remove this watermark.
Incluso para a Vida Ingls

Pr-Vestibular da UFSC
7
CONDITIONAL

O condicional formado pelo auxiliar would
+ o infinitivodo verbo principal sem o to. Da mesma
forma que o auxiliar do Futuro Simples , would usado
para todas as pessoas e segue as mesmas regras para a
formao de Interrogaes e Negaes.

Veja exemplos:
(affirmative form) : Helen would listen to music.
Their friends would call them at
6:00.

(interrogative form) : Would Helen listen to music ?
Would their friends call them at
6:00?

(negative form): Helen would not (wouldnt) listen to
music.
Their friends would not (wouldnt) call them at
6:00

Lembrete: usamos would em oferecimentos.
Would you like some tea?
Would you like a piece of cake?
Would you like to go to the beach?

Exerccios de Sala #

1. Put the sentences below in the Interrogative and
Negative forms:
a) I will drink a lot of milk.
_________________________________________
_________________________________________

b) My friends will watch the game tonight.
_________________________________________
_________________________________________

c) She would talk to Susan.
_________________________________________
_________________________________________

A song: ALL MY LOVING (The Beatles)

Close your eyes and I'll _______ you
Tomorrow I'll ________ you
Remember I'll always be true.

And then while I'm _________
I'll write home every day
And I'll _________ all my loving to you.

I'll __________ that I'm kissing
The lips I am missing
And hope that my __________ will come true.

And then while I'm away
I'll ___________ home every day
And I'll send all my loving to you.

All my loving I will send to you.
All my loving, darling I'll be true.

Complete the gaps above with the words from the box.
Write kiss miss dreams pretend send - away

Tarefa Complementar #

2. (UFSC) This is the description of a vegetable called
onion. Make complete sentences by matching the part of
the sentence on the left with the part on the right.
1. This is one of the... ( ) several layers surrounding
each other.
2. The ancient Greeks and
Romans used...
( ) can be cooked or eaten.
3. They are round and
made up of...
( ) oldest vegetables in
history.
4. Their thin skin has to be
removed before they...
( ) flavor to soups, sauces, and
other dishes.
5. They have a very strong
smell and are often used to
give...
( ) to eat them raw for
breakfast, with a little salt.
The correct sequence(s) from the top to the bottom is
(are):
01. 5 3 1 2 4
02. 3 2 5 4 1
04. 1 5 3 4 2
08. 3 4 1 5 2
16. 2 5 4 1 3

3. (UFPR) A questo abaixo somatria, ou seja, a
resposta a ser dada a soma dos nmeros das alternativas
corretas.

According to the chart above:
01. The Northeast is the region which had the fewest
executions in the United States.
02. There is no death penalty in the Northeast.
04. There were more executions in the Midwest than in
the West.
08. Texas is the state with the lowest number of
executions.
16. There were executions in five different regions.
32. Excluding the estate of Texas, the South is still the
region where there were the most executions in the
United States.

Text Seven: (UFSC) Texto referente s questes de
nmeros 44, 45, 46 (questes somatrias).
Please purchase PDF Split-Merge on www.verypdf.com to remove this watermark.
Incluso para a Vida Ingls

Pr-Vestibular da UFSC
8
Fast Food
The American fast-food culture dates back to the
period after the Second World War. During the War, the
Americans had developed the production of dehydrated,
tinned and powdered food on an industrial scale; most of
this was sent abroad to feed troops at the front.
When the War ended, the USA found itself with a
massive production potential for this type of food but no
starving soldiers to eat it so the manufacturers began to
market it to the general public. Housewives started baking
cakes out of packets and made sure they kept up their
stock of powdered milk.
The present scenario is well known: American fast
food and fizzy drinks like Coca-Cola and Pepsi are being
successfully exported to countries around the world.
(Adapted from: Speak Up no 143. Abril 1999. Insert p. IV)

Read the following propositions. Some make sense and
some dont.

4. Select the one(s) that MAKES (MAKE) SENSE,
according to the text.
01. As soon as the Second World War ended, the
Americans sent their troops abroad.
02. Starving soldiers didnt eat fast food because they
were not sufficiently hungry.
04. Nowadays American fast food is sold all over the
world.
08. Housewives refused to use powdered food, therefore
they started baking cakes out of packets.
16. The producers of dehydrated, tinned and powdered
food decided to market their products to people in
general, when they realized that with the end of the
War there would be no starving soldiers to eat that
kind of food.

5. Identify the proposition(s) which contains (contain) the
correct explanation for the expressions from the text.
01. tinned food food that has been preserved by being
sealed in a can.
02. powdered milk a product from which water has been
eliminated through dehydration.
04. fizzy drinks they are full of little bubbles of gas and
make a hissing sound.
08. fast food food that is already prepared and so is
served quickly.
16. massive production the process of making goods in
small quantities.
32. industrial scale a system in which products are made on
a very restricted scale.

6. Read these short descriptions of certain kinds of food
and choose the proposition(s) in which the name of the
food corresponds to the description.

01. A food made from flour, water and usually yeast. The
mixture is baked in an oven. It is often cut into slices
and eaten with butter, jam, etc...
Food being described: corn.
02. They are eaten in many countries around the world.
They grow in the ground, are round and have a thin
skin. They can be cooked in many different ways
boiled, fried or baked.
Food being described: potatoes.
04. They are small, round and juicy, green or dark purple
in color. You can eat them raw or use them to make
wine.
Food being described: grapes.
08. This is the seed of a plant grown in warm, wet places.
You boil it in water and eat it usually with meat or
vegetables. It is eaten everywhere in the world, but
particularly in China, Japan, and other Asian
countries.
Food being described: rice.
16. They are really a fruit although many people regard
them as a vegetable. They are soft, juicy, red and
round. They have a lot of seeds and you can eat them
raw in salads, or cooked as a vegetable or in sauces.
Food being described: apples.

UNIDADE 6

IMMEDIATE FUTURE: GOING TO

A forma going to usada para expressar aes
futuras que esto prestes a acontecer ou que tenham
grandes chances de ocorrer. Neste tempo verbal o verbo
to be funciona como auxiliar , e seguido de um verbo no
infinitivo sem o to.

EXPRESSES USADAS
Next week in a week Tomorrow
NEXT YEAR IN A MONTH TONIGHT
Next month in a year

GOING TO: INTERROGATIVE AND NEGATIVE
FORMS
(affirmative form) : Shes going to buy a car.
They are going to wait for me.

(interrogative form) : Is she going to buy a car ?
Are they going to wait for me ?

(negative form): She is not (isnt) going to buy a car.
They are not (arent) going to wait for me.

Exerccios de Sala #

1. Supply the going to form of the verbs in parentheses.
a) The movie_____________(begin) at 8 oclock.
b) Mr. Brown_____________ (leave) the office at 6:OO
today.
c) What ________ they __________(do)with all those
books.
d) ________you _________ (send) her the messages ?
e) I think I _____________ (swim) tomorrow morning.
f) Helen_________ (ask) you to help her with her Math.

Mother-in-law is the most common suggestion, said
Charles Tweedie, who creates custom-made scarecrows
for English farmers, adding the face from any photo.

2. (UNIV. CAT. SALVADOR-BA) Segundo o texto:
a) Tweedie se recusa a fazer espantalhos com cara de
sogra, embora esse seja o pedido mais comum.
b) Os espantalhos de Tweedie costumam ser muito caros.
c) A maioria dos espantalhos que Charlie Tweedie cria
Please purchase PDF Split-Merge on www.verypdf.com to remove this watermark.
Incluso para a Vida Ingls

Pr-Vestibular da UFSC
9
tem a cara da sogra do cliente.
d) As sogras so as maiores freguesas de Tweedie.
e) As leis britnicas proibem espantalhos com cara de
me.

A song
Redemption Song (Bob Marley)

Old pirates, yes, they rob I;
Sold I to the merchant __________ (1),
Minutes after they took I
From the bottomless ____________ (2).
But my hand was made ___________ (3)
By the hand of the Almighty.
We forward in this ____________ (4)
Triumphantly.
Won't you help to sing
these songs of ____________ (5)
'Cause all I ever have:
Redemption songs
Redemption songs
Emancipate ____________ (6) from mental slavery;
None but ourselves can free our ______________ (7)
Have no fear for atomic _____________ (8),
'Cause none of them can stop the time
___________ (9) shall they kill our ____________ (10),
While we stand aside and look
Oh! Some say it's just a part of it:
We've got to fulfill the ____________ (11).
Won't you help to sing
these songs of ____________ (12)
'Cause all I ever have:
Redemption songs

UNIDADE 7

PRESENT PERFECT TENSE

- Formation: have/has + past participle
Examples:
1. I have been ready since 8 o clock.
2. She has lived in California for three years
3. Have you already talked to the teacher
4. MARY: The Blooms have bought a new house.
TOM: Really? When did they buy it?
MARY: They bought it last week.
TOM: Have they moved yet?
MARY: No, they havent had time for that yet.
But their new furniture has already arrived.
5. Have you ever eaten crocodile meat? / Has she ever
been to Europe?
6. The soccer match has just finished. / Weve just
arrived from our trip to Canada.

A song
I STILL HAVENT FOUND WHAT IM LOOKING
FOR - U2
I have climbed the highest ______________ (1)
I have run through the ______________ (2)
Only to be with you (x2)

I have ____________ (3) I have crawled
I have scaled
these city _________ (4) (x2)
Only to be with you

[Chorus]
But I still haven't ____________ (5)
What I'm looking for
But I still haven't found
What I'm looking for

I have __________ (6) honey lips
Felt the healing in her fingertips
It burned like _________ (7)
This burning desire

I have spoken with the tongue of angels
I have held the ___________ (8) of the devil
It was warm in the __________ (9)
I was cold as a stone

[Chorus]

I believe in the Kingdom Come
Then all the colours will
_________ (10) into one (x2)
But yes I'm still running

You __________ (11) the bonds and you loosed the
__________ (12)
You carried the cross
And my __________ (13) (x2)
You know I believe it

Exerccios de Sala #

1. Write the verbs from the parentheses in the Present
Perfect Tense.
a) I know that woman, but I _____________________
(forget) her name.
b) Heitor _________________ (be) to Florida many times
in his vacations.
c) John and Lucas __________________ (go) to Europe
in a business trip. Theyll be back soon.
d) Hellen ___________________ (work) for a
multinational company.
e) They _________________ (sell) all the tickets for the
show.
f) Sandra and Meg ________________ (live) in Canada
since 1998.

2. Complete these sentences with the SIMPLE PAST or
the PRESENT PERFECT Tense.
a) - John isnt here, is he?
- No, he _______________ (leave) to Rio de Janeiro.
b) Angela _________________ (buy) a nice dress at the
market place yesterday.
c) Can you lend me some money? Ill pay you back at
the end of the month.
- Sorry, I cant. I ________________ (already / spend)
all my money, and Im completely broke too.
d) My parents _______________ (arrive) from Europe
last week. They loved the trip.
e) Some of the students _______________ (study / not)
for the test. We can see it from their grades.
Please purchase PDF Split-Merge on www.verypdf.com to remove this watermark.
Incluso para a Vida Ingls

Pr-Vestibular da UFSC
10
f) My husband _______________ (send) me a bunch of
flowers on my birthday two days ago. It was so sweet
of him.
g) Tracy is so nervous because she ________________
(forget) to read the article about astronomy.

3. Complete the sentences with ALREADY or YET.
a) - Have you watched that film ____________?
- No, I havent watched it ________________.
b) They have ______________ sailed for all five
continents since January 2000.
c) Has Peter been to Paris ________________?
d) - Have you ____________ talked to Mary?
- No, not ___________. I will do it tonight.
e) The president has ______________been to many
countries this year.

4. Write since of before these expressions of time.
a) __________________Wednesday
b) __________________ five hours.
c) __________________ the last world cup.
d) __________________ Many years.
e) __________________ the moment I saw you.
f) __________________ months and months.

UNIDADE 8

MUCH / MANY / LITTLE / FEW

MUCH ( muito, muita ) - usamos a palavra much
antes de substantivos incontveis.
Veja os exemplos:
I dont drink much coffee.
She drinks much wine at home.

LITTLE ( pouco , pouca ) -usamos tambm a palavra
little antes de substantivos incontveis.
Veja os exemplos:
I have little money.
We drink little milk.
Pay Attention :
Little tambm poder significar pequeno (-inho). Veja:
My little boy meu pequeno garoto e / ou meu
garotinho.

MANY (muitos, muitas) usamos a palavra many
antes de substantivos contveis.

Veja os exemplos:
I drink many milk-shakes in summer.
Tom reads many books.

FEW (poucos, poucas) usamos tambm a palavra few
antes de substantivos contveis.
Veja os exemplos:
Robert wants few potatoes.
Jane makes few mistakes.
Veja as seguintes observaes:
x Em interrogaes e negaes usamos much / many sem
problemas;
x Em afirmaes much / many podem ser substitudos
por: a lot of , lots of ou plenty of : muito ( s ) , muita ( s).

Exerccios de Sala #

1. Rewrite the sentences using much or many:
a) Those families have lots of money.
_____________________________________________

b) These men lived a lot of years in prison.
_____________________________________________

2. Complete with much / many :
a) __________ books on the table.
b) __________ milk in the cartoon.
c) __________ women at the restaurant.
d) __________ wine in the botlle.
e) __________ sugar in the coffee.

3. (UEB) Assinale a alternativa correta:
There is ________ bread on the tray.
a) not many b) not much c) a few d) many

4. (OsecSP) When they were introduced, electronic
calculators were less powerful and cost _________ than
they do now.
a) very few c) very little
b) many more d) much more

5. (Vunesp) Do politicians work ________ and earn
________ money?
a) little many c) much few
b) very much d) little much

6. (ITA-SP) Please, can you give us ________ bit more
of that cake you baked yesterday?
a) several b) little c) many d) a little

7. (UFV MG) There are ________ dangerous drivers on
the streets nowadays.
a) a lot of c) very lot of
b) a very many d) very many of

GERUND or ING form

A) Usa-se o gerndio geralmente no incio de frases e
aps preposies
(without, for, in, before, etc) ;
Ex. Smoking is a bad habit.
Surfing is a beautiful sport.
They are tired of swimming.

B) Aps os verbos: admit, avoid, appreciate, consider,
delay, detest, deny, dislike, enjoy, escape, excuse,
finish, keep , mention, miss, practice, resist, suggest,
understand, quit.
Ex. You dislike driving at night.
John should avoid touching those things.

C) Aps os verbos Go e Come indicando atividade
fsica.
Ex. Go walking!
Go fishing!

USO do INFINITIVO
A) Usa-se o infinitivo sem to aps os verbos
anmalos e os verbos make e let.
Please purchase PDF Split-Merge on www.verypdf.com to remove this watermark.
Incluso para a Vida Ingls

Pr-Vestibular da UFSC
11
Ex. I can drive.
Let me dance.
Make him study.

B) Usa-se infinitivo sem to ou gerndio aps
os verbos de percepo:
Feel, hear, notice, observe, see, watch.
Ex. I hear her crying.
The man saw the queen writing.

C) Usa-se infinitivo com to ou gerndio aps os
verbos
Advice, attempt, begin, start, continue, forget,
hate, leave, like, love, mean, neglect, prefer, permit,
remember, stop, study, try.
Ex. He stopped to cry / crying

Exerccios de Sala #

8. (PUC) Britons are used to ___________ terrible
winters.
a) to have b) have c) had d) having

9. (UFRS) He was angry about ___________ failed the
test.
a) to have b) having c) have d) has

10. (F. Objetivo) SP Keep on _______________!
a) try b) to try c) trying d) had tried

11. (PUCRS) They avoided________to the festival
because of the crowds.
a) to go b) go c) going d) goes

12. (FCChagas SP) You cant make an omelette
without ________ eggs.
a) to break b) break c) breaking d) breaks

13. (ITA SP) He denied__________ seen her before .
a) having b) have c) to have d) has

14. (FMU-SP) She made me ______the bill
before________.
a) pay traveling
b) to pay to travel
c) to pay traveling
d) paying to travel

15. (FC Chagas-BA) After a long argument, we finally
decided__________ to Aparecida on Christmas day.
a) to go b) go c) going d) goes

A song
Mercedes Benz Janis Joplin

Oh Lord, won't you buy me a Mercedes Benz ?
My friends all drive Porsches, I must make amends.
Worked hard all my __________ (1), no help from my
___________(2),
So Lord, won't you buy me a Mercedes Benz ?

Oh Lord, won't you buy me a ____________ (3) ?
Dialing For Dollars is trying to ____________(4) me.
I wait for _____________ (5) each day until three,
So oh Lord, won't you buy me a ______________(6)?

Oh Lord, won't you buy me a _____________ (7) on the
town ?
I'm counting on you, Lord, please don't let me
____________(8).
Prove that you love me and _____________ (9) the next
round,
Oh Lord, won't you buy me a ____________(10) on the
town ?

Everybody!
Oh Lord, won't you buy me a Mercedes Benz ?
My friends all drive Porsches, I must make amends,
Worked hard all my lifetime, no help from my friends,
So oh Lord, won't you buy me a Mercedes Benz ?

That's it!

MODAL VERBS

CAN COULD SHOULD MAY MIGHT -
MUST -MUSTNT

UNIDADE 9

PLURAL FORM OF NOUNS

Para formarmos o plural dos substantivos em
Ingls precisamos conhecer as regras e saber us-las
corretamente.

Veja a seguir como devemos aplic-las:
1. Regra geral para a formao do plural o acrscimo
de um s ao substantivo:
Ex.: House houses
Egg eggs
Car cars
Cloud clouds

2. Usa-se es aos substantivos terminados em (s), (ss) ,
(ch), (sh), (x) e (z) :
Ex.: Bus - buses
Kiss kisses
Beach beaches
Topaz topazes
Dish dishes
Brush brushes

Mas ateno: h alguns substantivos terminados em ch,
com som de k, que recebero apenas s no plural.
Veja: Epoch epochs
Monarch monarchs
Stomach stomachs
Patriarch patriarchs

3. Aos substantivos terminados em (Y) , precedido de
vogal , usa-se apenas s.
Ex.: Day days
Boy boys
Key keys

Mas se o substantivo terminar em (Y) , precedido de
consoante, trocaremos o (Y) por ( ies ) :
Ex.: City cities
Please purchase PDF Split-Merge on www.verypdf.com to remove this watermark.
Incluso para a Vida Ingls

Pr-Vestibular da UFSC
12
Baby babies
Enemy enemies

Com nomes prprios : Kelly Kellys
Andy Andys

4. Aos substantivos terminados em (O) , precedido de
vogal , usa-se apenas s.
Ex.: Radio radios
Bamboo bamboos
Studio - studios

Mas se o substantivo terminar em (O) , precedido de
consoante, acrescentaremos es .
Ex.: Hero heroes
Potato potatoes
Volcano - volcanoes

Algumas palavras de origem grega e latina, recebem
apenas s
Ex.: Piano pianos
Photo photos
Kilo - kilos

5. Aos substantivos terminados em (F) ou (Fe) ,
trocamos por (ves) .
Ex. Leaf leaves
Loaf - loaves
Knife knives
Wife wives
Shelf - shelves

Mas h outros que recebero apenas (S) .
Ex.: Roof roofs
Proof proofs
Dwarf dwarfs
Cliff cliffs
Reef reefs
Belief beliefs
Safe safes

6. H alguns substantivos em Ingls que no possuem
regras de plural, ou seja, tm formas especiais de
plural .

Ex.: Man men
Child children
Ox oxen
Goose geese
Foot feet
Tooth teeth
Louse lice
Mouse mice
Die - dice

7. No caso das nacionalidades terminadas em man
trocam para men no plural.
Frenchman Frenchmen
Englishman Englishmen Irishman - Irishmen
Mas, no caso de:
German Germans
Roman Romans
Norman Normans

8. Para a formao do plural dos substantivos compostos,
forma-se pluralizando o principal elemento.
Ex.: Brother in law - brothers in- law
Mother in- law - mothers in- law

Mas no caso dos substantivos compostos que no so
separados por hfen, recebem s no plural.
Ex.: Armchair - armchairs
Bookcase bookcases

9. H alguns substantivos que no possuem plural em
Ingls, portanto, o verbo que os acompanha sempre
usado no singular.
Ex: Information, Progress, Furniture, Advice,
Baggage / Luggage, Knowledge.

E.g. This furniture is very old.
My luggage isnt heavy.

10. No caso dos substantivos news, mathematics,
optics, physics, so usados com o verbo no singular,
embora paream estar no plural.
e.g. There is a good news for you.
Physics is a very difficult subject.

11. H alguns substantivos que no possuem singular em
Ingls, assim o verbo sempre usado no plural.
Ex.: Shorts, Pants, Scissors, Trousers, etc.
My scissors are on the desk.
Your pants are nice.

12. H alguns substantivos em Ingls que so escritos no
singular, mas tem sentido plural, assim o verbo que os
acompanha fica sempre no plural. Ex: Cattle, Police,
People.
e.g. The police are coming.
There are more than thirty people here.

13. Os substantivos de nacionalidades terminados em
SE ou SS, permanecem na mesma forma do
singular.
Ex.: a Chinese - three Chinese
a Swiss - two Swiss

14. Os substantivos de origem Grega e Latina,
permanecem o plural de origem.

e.g. Agendum agenda
Crisis - crises
Phenomenon phenomena
Datum - data
Thesis theses

Exerccios de Sala #

1. (Cesgranrio) - Mark the item in which all the nouns
form their plural by adding an s , as year / years:
a) leaf world tooth pace
b) proof object problem key
c) roof goose man- day
d) life wife knife shelf

2. (UFBA) A palavra que forma o plural pelo simples
acrscimo de s :
a) man b) woman c) crisis d) night

3. (Mackenzie SP) Which of the following groups
consists of nouns which do NOT form their plural in
ves?
a) shelf calf thief c) chief reef proof
b) loaf wolf half d) life knife- leaf
Please purchase PDF Split-Merge on www.verypdf.com to remove this watermark.
Incluso para a Vida Ingls

Pr-Vestibular da UFSC
13

4. (Fuvest SP) Coloque no plural as palavras indicadas:
I bought some ________ , _________ , ________ this
morning.
( tomato ) ( cherry ) ( peach )
a) tomatos cherries peaches
b) tomatoes cherries peaches
c) tomatoes cherrys peachs
d) tomatos cherrys peaches
e)
5. (FMU SP) There are many _____ in those ____.
a) thieves citys c) thifs cities
b) thieves cities d) thiefs citys


Text Eight
Suspect Thief Drowns
Sun Aug 18, 7:35 AM ET
TULSA, Oklahoma (Reuters) - A suspected thief,
weighed down with more than 50 pounds of stolen
cameras and CDs, among other items, drowned as he
attempted to evade police by swimming across the
Arkansas River, officials said.
The man, identified as Edward McBride, 37, was
carrying a bag weighing 50 pounds that contained stolen
items and was found Friday with stolen goods also stuffed
in his pockets, said Tulsa police spokesman Lucky
Lamons.
He was being pursued by Tulsa police who
suspected him of robbing a Tulsa home when he jumped
into the muddy Arkansas River. "He got about 40 yards
out and yelled for help," Lamons said. "The officers took
off their shirts, shoes and belts and jumped into the river.
By the time they reached him, he had gone under."
Lamons said rescue workers retrieved McBride's
body about an hour later from about 8 feet to 10 feet of
water along with the bag containing stolen goods.

6. De acordo com o texto acima, correto afirmar:
01. Ao fugir da polcia, McBride caiu involuntariamente
no rio.
02. McBride tinha 37 anos e era suspeito de ter assaltado
uma casa.
04. O peso dos objetos roubados que carregava consigo
contribuiu para o afogamento de McBride.
08. McBride se especializava no roubo de jias e quadros.
16. McBride foi retirado do rio ainda com vida, mas
morreu logo em seguida.
32. Os fatos narrados ocorreram no ms de agosto, em
Tulsa, no estado americano de Oklahoma.
7. The text contains information on
01. how McBride died.
02. why McBride died.
04. who rescued McBride, preventing him from dying.
08. when McBride's body was found.
16. how much McBride weighed when he died.

8. According to the text, the police
01. tried to help McBride before he drowned.
02. was able to reach McBride in time to arrest him.
04. shot and killed McBride.
08. pursued McBride.
16. found some evidence that McBride was a thief.

Text Nine
The development of a baby

A fertilized egg is nourished and protected as it
develops from an embryo to a fetus during 40 weeks of
pregnancy. The placenta, a mass of blood vessels
implanted into the uterus lining, delivers nourishment and
oxygen, and removes waste through the umbilical cord.
Meanwhile, the fetus lies snugly in its amniotic sac, a bag
of fluid that protects it against any sudden jolts. In the last
weeks of the pregnancy, the growing fetus turns head
down: a baby ready to be born.

9. According to text 9 all propositions are correct, except:
a) The word development has a suffix.
b) The text is in the present tense.
c) One of the functions of the placenta is to feed the
fetus.
d) The fetus feels very uncomfortable inside the amniotic
sac.
e) There are three different words in the plural form.

10. According to text 9, which question cannot be
answered?
a) How long does pregnancy last?
b) What does the placenta do?
c) What are the nutrients of the fetus nourishment?
d) Whats nourished in order to become a fetus?
e) How does the waste leave the fetus?

11. According to text 9, mark the correct proposition(S).
01. The amniotic sac protects the baby against violent
movements.
02. Oxygen is removed through the umbilical cord.
04. When the baby is about to be born, its head must be
up.
08. First the fertilized egg turns into an embryo, then
into a fetus.
16. The placenta is located in the uterus lining.

12. According to text 9, which words are not verbs?
01. Turns;
02. Vessels;
04. Lies;
08. Jolts;
16. Removes;
32. Protects.

13. According to text 9, mark the WRONG proposition.
a) it (L. 1) refers to egg.
b) its refers to fetus (L. 4).
c) it (L. 4) refers to amniotic sac.
d) Removes refers to the placenta.
e) develops refers to fertilized egg.

Text Ten
EAT BETTER, FEEL BETTER
It is becoming more and more important to eat
the right food. Experts say that the type of food you
eat can damage your health. If you eat the right food,
you will have a better chance of living a long and
healthy life.
Please purchase PDF Split-Merge on www.verypdf.com to remove this watermark.
Incluso para a Vida Ingls

Pr-Vestibular da UFSC
14
If you eat a lot of processed food, you will be
more likely to have problems with your health. If
people eat food
with a lot of fat in it, they will have a greater risk of
getting heart disease.
In some countries, people eat lees fat in their diet.
Scientists have shown that fewer people get heart
disease in these countries. In Spain and Italy, for
example, most people have less fat in their diets than
people in England. And in England, the rate of heart
disease is double the rate in Spain or Italy.
So if you eat less fatty food, you will live longer.
Youll feel better, feel fitter, and have more energy if you
change to a simpler and healthier diet.
From: Break into English Michael Carrier & Siman Haines
Hodder & Stoughton, 1987.

14. According to the text, what sort of food is bad for
you?
a) Italian food.
b) Fruit and vegetables.
c) Fatty food.
d) Simple food.
e) Diet food.

15. According to the text, what happens in countries
where the diet is high in fat?
a) There are less sick people.
b) The population has a better life.
c) People are healthier.
d) The rate of heart disease is big.
e) People get fatter.

16. The text says that in Spain people eat _____ than
in England.
a) More food.
b) Much more fatty food.
c) More energetic food.
d) Less food.
e) less fatty food.

17. In the sentence ... you will have a better chance
of living a long and healthy life., the verb living is in
the...
a) Infinitive.
b) Present continuous tense.
c) Future tense.
d) Gerund.
e) Imperative form.

UNIDADE 10

1. Select the proposition(s) which contains(contain)
correct translations for the underlined words.

01. typical rural dishes, among many others, can be
enjoyed in the many different regions
podem ser divertidos
02. In typical environments like mountain resorts
gostam de hotis de montanha...
04. restaurants with beaten earthen floors along the
coast.
pisos de cho batido
08. There are food festivals
festivais gastronmicos
16. you can take part in cold climate activities, such as
picking apples and grapes
tais como plantar mas e uvas
32. the marine farms and restaurants along the shore
ao longo da costa

Text Forteen (UFSC 2001)

PLAY BALL!!!
It is hard to believe that a game as fast and
exciting as soccer had its origin in a religious ceremony
several thousand years ago in Egypt. At that time, the
contest was between teams of pretty girls who had first
taken part in a beauty contest and parade. After putting on
armor, they divided into teams and fought with sticks over
a round stone. The stone is believed to have represented
the sun, or possibly the head of the goddess of agriculture,
and this use of the round stone is thought to be the origin
of all ball games.
The custom of teams competing for control of a
round object, or ball, first spread across North Africa, the
Arabic countries, and Persia. Over the years, it also spread
to the whole world, and probably the most popular team
sports today are soccer, baseball, and basketball.
Perhaps sports, like music, could be called an
international language. Certainly they have done much to
bring people from different countries together and to
improve understanding among them. Through friendly
competition, players and spectators alike have learned that
people everywhere are very much the same, and that
everyone appreciates good sportsmanship and fair play.
From: Lets Learn English W.Bryce Van Syoc & Florence S. Van Syoc
- American Book Company. New York, 1971 (Adapted)

2. Considering the topic of the text, choose the
introduction(s) that can start it meaningfully.
01. Stop that kick!... Pass the ball!... Goal! Goal!
02. Known simply as The Boat Race, this test lasts
twenty minutes, but has captured the publics
attention for over 150 years.
04. The fans watch with attention as a player runs down
the soccer field trying to kick the ball towards the
goal.
08. Inspired by that idea, the fitness director at the Aspen
Club came to New York to organize a cross-training
program for Central Park.
16. There must be something that limits the number of
people that participate in extreme sports.
32. The spectators stand up and cheer enthusiastically as
the player nears the goal, then sit in disappointment
when an opponent suddenly gets the ball away from
him and starts towards the other end of the field with
it.

3. Select the correct proposition(s) according to the text.
01. Egyptian men worked hard to invent soccer.
02. Soccer players have a ritual of touching and hugging
each other after a goal.
04. It is difficult to accept the idea that soccer originated
in a religious ceremony.
08. The players and the spectators enjoyed the game and
decided to speak an international language.
Please purchase PDF Split-Merge on www.verypdf.com to remove this watermark.
Incluso para a Vida Ingls

Pr-Vestibular da UFSC
15
16. A round stone was used by the Egyptian girls during
the contest.
32. Sports promote better understanding among people
from different parts of the world.
64. Ancient people used to wear a protective covering
made of metal to kill their opponents.

4. Which of the questions below can be answered
according to the information contained in the text?
01. Which are possibly the most popular team sports
nowadays?
02. How many girls were there in each soccer team?
04. How long did a soccer game last in Egypt?
08. Where were the first ball games played?
16. Who used sticks to compete for a round stone in
Egypt?
32. What kind of music was appreciated by the
Egyptians?
64. What is the effect of friendly competition?

5. Read the four summaries below. Which one(s) contains
(contain) the same information found in the text?
01. Through the text we conclude that ball games had
their origin a few years ago in Egypt and that the
Egyptians practice spread all over the world. The text
also mentions the names of the most famous sports
nowadays and it associates music with sports, since
both activities contribute to make people rich.
02. From the text we learn about the origin of all ball
games and about the probable meanings of the round
stone used in the games in ancient times. It also refers
to the three team sports that can be possibly
considered the most popular ones nowadays. After
that, the text compares sports with music mentioning
the benefits they bring to people and it expresses
approval about friendly competition.
04. The text refers to the history of all sports. It expresses
the idea that music and sports have the same
importance when we think of bringing people
together.
08. The text tells us how soccer and all ball games
originated. Besides that, it mentions the names of
some team sports and makes a comparison between
sports and music. Finally, it refers to friendly
competition and its positive effects.

UNIDADE 11

TEXT 11

SOME EASY WAYS TO MAKE FRIENDS

1. Its hard to make friends if you stay home all the time.
Get out of the house and do things that will put you in
touch with other people. Join a club or play a sport.
Attend meetings of neighborhood associations or other
groups. Its easier to make friends when you have similar
interests.
2. Learn from people at school or work who seem to have
lots of friends. Observe how they make and keep friends.
Dont imitate all the things they do. But try to notice what
they do. Then try some of those things yourself.
3. Dont be afraid to show people what youre really good
at. Talk about the things you like and do best. Dont hide
your strong points. People will be interested in you if
there is something interesting about you.
4. Plan things to talk about with people. Find out whats
in the newspaper headlines, listen to the top CDs, learn
about whats new with your favorite TV or movie star.
The more you have to say, the more people will be
interested in having a conversation with you.
5. Be a good listener. Let people talk about themselves
before talking about me, me, me. Ask lots of questions.
Show an interest in their answers. This alone will make
people want to be your friend.
6. Once you start to get to know someone, dont be
friendly one day and then too shy to talk the next day. Be
consistent. Consistency is something people look for in
friends.
7. Have confidence in yourself. Dont be self-critical all
the time. This will only make the process more difficult.
Think of your good qualities. People are attracted to those
with self-confidence.
8. Try to make friends with the kind of people you really
like, respect, and admire not just with those who are
easy to meet. Be friendly with a lot of people.
From: RICHARDS, J. C. & ECKSTUT-DIDIER, S. Strategic Reading
1. CUP, 2003. (Adapted)
Exerccios de Sala #

1. Select the correct statement(s) about the text.
01. An important quality of friendship is consistency.
02. People who have many friends are more traditional.
04. Its better to have a few good friends than to have
many friends.
08. Friends can be important when we are shy.
16. It will be more difficult to make friends if you judge
yourself too hard.
32. Paying attention to what people say is a good way to
start a friendship.

2. According to the text, what advice can help these
people? Select the proposition(s) in which the advice is
correctly indicated.
01. Tony is never sure what to talk about when he meets
people. o Advice 4
02. Abby doesnt always say nice things about her new
friend James. o Advice 1
04. Rose wants to know why her classmate, Cindy, is so
good at making friends. o Advice 2
08. Benson is a terrific dancer, but he never tells anyone
about it. o Advice 3
16. Max talks to a girl in his history class on Monday, but
on Tuesday hes afraid to say Hi. o Advice 6
32. Jill always talks about herself. o Advice 7



3. According to the text, what are some things you can do
to make friends? Select the correct proposition(s).
01. Do exactly what people tell you to do, never disagree
with them.
02. Read newspapers, watch TV and listen to music in
order to be well-informed.
04. Be sociable, get involved in activities in the company
of other persons.
Please purchase PDF Split-Merge on www.verypdf.com to remove this watermark.
Incluso para a Vida Ingls

Pr-Vestibular da UFSC
16
08. Look for the most popular people in your group and
try to become their friend.
16. Talk mostly about yourself, your problems and what
is happening in your life.
32. Let people know the good things about you.

4. Considering the text, compare the meanings of each
pair of sentences and select the proposition(s) in which
the meanings of sentences A and B are similar.
01. A. People are attracted to those with self-confidence.
B. If you believe in yourself, youll have more
people near you.
02. A. A good idea is to imitate people who seem to have
lots of friends.
B. Maybe you can copy the behavior of people who
seem to have lots of friends.
04. A. Do things that will put you in touch with other
people.
B. Choose activities that will help other people.
08. A. You can learn the last facts about your favorite TV
star.
B. One possibility is to find out the news about your
favorite TV star.
16. A. This alone will make people want to be your friend.
B. When you are lonely, people will see that you need
a friend.

5. Select the proposition(s) that can correctly go at the
end of advice number 8.
01. For some people, friends have become more important
than their family.
02. There are many things you can ask a friend to do for
you.
04. That way, youll have a bigger group of people to
choose your friends from.
08. Good friends do not always have to tell each other the
truth.
16. Your chances of making friends will, therefore,
be greater.

TEXT TWELVE:

BEST FRIENDS
1. Men and women share the exact same view of a best
friend a person who is always there for you. Your best
friend is someone you can depend on to share your
happiness, suffer through your worries, or lessen your
sorrow.
2. A great variety of factors play into the birth of a best
friendship the age and circumstances under which
people meet, what first attracts them, why they remain
close, and how they fill each other's needs. Yet I found
the dominant themes that define a best friend were
remarkably similar across the broadest range of
experiences.
3. Safety was a word I heard over and over. A best friend
is a safe harbor, a guaranteed comfort zone. You never
have to explain yourself to best friends because they
really, really know who you are. With best friends, you
can be who you are. You can cry too hard or laugh too
loud and never worry what they'll think of you because
best friends are nonjudgmental. They will give you advice
if you want it and a kick in the pants if you need it, but
best friends will not judge you or make you ashamed of
your behavior. A best friend gives you what you expect
from a parent and don't always get: unconditional love.
4. Best friends are loyal and trustworthy. A best friend is
a person to whom you can tell your most embarrassing,
revealing, and damaging personal secrets with the full
confidence they will never be repeated. Best friends can
deliver brutally honest answers in the most gentle fashion.
5. Finally, best friends are the family you choose. They
love you because they want to, not because they have to.
And for many people, a best friend becomes the brother
or sister they'd always wanted, but never had.
From: RICHARDS, J. C. & ECKSTUT-DIDIER, S. Strategic Reading
1. CUP, 2003. (Adapted)

6. Identify the correct proposition(s) according to the
text.
01. When your personal secrets are told to a best friend
you can be sure he or she will not reveal them to
anyone else.
02. People always call a good friend when they have a
doubt.
04. To have best friends means to have people you can
trust and by whom you can be advised.
08. Men and women depend on each other to solve their
problems.
16. Friends, even best friends, can never replace your
family.
32. Age is one of the aspects that influence the
development of a friendship.

7. According to the text, select the correct answer(s) for
the question below.
How can a best friend be described?

01. As a place where you are not in danger.
02. Someone who is usually embarrassing.
04. A best friend follows the fashion most of the time.
08. He or she never tells you the truth.
16. A best friend doesn't ask you for explanations.
32. As a person who is always there to judge you.

8. Choose the proposition(s) in which the definitions of
the words below correspond to the meaning used in the
text.
01. lessen (Paragraph 1) o if something lessens or is
lessened, it becomes less strong.
02. sorrow (Paragraph 1) o a feeling of deep sadness or
regret.
04. range (Paragraph 2) o a group of hills or mountains.
08. advice (Paragraph 3) o an opinion which someone
offers you about what you should do in a particular
situation.
16. kick (Paragraph 3) o a new interest, especially
one that does not last long.
32. ashamed (Paragraph 3) o to feel embarrassed or
guilty because of something that you have done.
64. fashion (Paragraph 4) o a style that is popular at a
particular time, especially in clothes, hair, make-up,
etc.

9. Select the proposition(s) which contains (contain)
correct answers to the following questions, according to
the text.
01. What does a best friend do for you?
Please purchase PDF Split-Merge on www.verypdf.com to remove this watermark.
Incluso para a Vida Ingls

Pr-Vestibular da UFSC
17
Among many things, he or she shares our feelings.
02. Why does a best friend become the brother or sister
you had always wanted, but never had?
Because it is easier to help a friend.
04. How many times did the author of this text hear the
word "safety"?
A lot of times.
08. When do you have to give an explanation to a best
friend?
It becomes necessary when we are misunderstood.
16. What kind of judgment does a best friend make on
your behavior?
A best friend doesn't judge us.
32. Where can a best friend be found?
In secret places.

10. Select the proposition(s) which contains (contain)
correct references to the following words, underlined in
the text.
01. who (Paragraph 1) o men
02. them (Paragraph 2) o people
04. which (Paragraph 2) o a best friendship
08. it (Paragraph 3) o a kick in the pants
16. whom (Paragraph 4) o you
32. they (Paragraph 4) o personal secrets

UNIDADE 12

LINKING WORDS (Conectivos)
1. ADIO:
- and : e
- in addition, furthermore, besides, moreover: alm
disso
- as well as: assim como
- also: tambm
- apart from: com exceo de
- both....and: ambos; tanto como
- not only ... but also: no apenas...mas tambm

2. CONTRASTE/CONCESSO/ADVERSATIVA:
- but: mas
- however, neverthless: entretanto
- yet: entretanto, ainda
- although, even though, though: embora
- nonetheless, notwithstanding: no obstante
- despite that, in spite of: apesar de
- rather than, instead of: em vez de
- whereas: enquanto

3. PROPSITO: - in order to: a fim de
- so as to: de modo que

4. CONSEQUNCIA/CONCLUSO:
- therefore: portanto
- consequently, as a result: consequentemente
- accordingly: de acordo, adequadamente
- hence: pois, ento, da
- thus: assim
- thereby: assim, desse modo
- then: ento
- so: ento, pois
- finally: finalmente

5. ALTERNATIVA: - otherwise: por outro lado
- or: ou
- or else: ou ento, ou ainda
- either ...or: ou...ou
- while, whereas: enquanto

6. REITERAO:
- that is: isto
- in other words: em outras palavras
- in short, in brief: em resumo
- i.e. : (do latim) isto
- that is to say: quer dizer

7. COMPARAO:
- like, as: como
- than: do que

8. ILUSTRAO:
- e.g. (do latim) por exemplo
- for instance, for example: por exemplo
- such as: tal como
- namely: a saber
- viz: (do latim) quer dizer

9. CONDIO:
- if: se
- unless: se no, a menos que
- provided that: uma vez que
- on condition that: desde que
- as long as: uma vez que
- subject to: sujeito a
- whether: se

10. CAUSA:
- because: porque
- due to: devido a
- as: porque
- since: uma vez que

11. DVIDA OU HIPTESE:
- perhaps, maybe: talvez
- possibly: possivelmente

12. TEMPORAL: - when: quando
-while: enquanto
Nas questes 100 103, assinale a(s) alternativa(s) que
significa(m) o mesmo que o segmento em negrito
extrado do texto The Mayas.

Exerccios de Sala #

1. ...although the Mayas knew about the wheel, they
never used it.(lines 23-24)
01. The Mayas didnt know about the wheel, therefore
they never used it.
02. The Mayas knew about the wheel, but they never used
it.
04. The Mayas never used the wheel, however they knew
about it.
08. In spite of the knowing about the wheel, the Mayas
never used it.

Please purchase PDF Split-Merge on www.verypdf.com to remove this watermark.
Incluso para a Vida Ingls

Pr-Vestibular da UFSC
18
2. Catherwoods drawings revealed that the Mayas
had incredible cities with temples, pyramids, and other
buildings as impressive as those of their northern
neighbors, the Aztecs(lines 10-13)
01. Catherwoods drawings showed that the Mayas had
incredible cities with temples, pyramids, and other
buildings which made a deep impression exactly like
of the Aztecs, who lived in the north.
02. The cities with temples, pyramids, and other buildings
of the Mayan civilization did not cause a great
impression as the ones from the Aztecs.
04. The Aztecs lived south from the Mayas but both had
impressive cities with temples, pyramids, and other
buildings.
08. Catherwoods drawings revealed that the Mayas
incredible cities with temple, pyramids, and other
buildings made a deep impression on the Aztecs, who
inhabited another continent.

3. The Mayas had a highly developed system of
government as well as an incredibly accurate system of
measuring time.(lines 17-20)
01. The Mayas had a highly developed system of
government in addiction to an incredibly accurate
system of measuring time.
02. The Mayas had both a highly developed system of
government and an incredibly accurate system of
measuring time.
04. The Mayas had not only a highly developed system of
government but also an incredibly accurate system of
measuring time.
08. Besides having a highly developed system of
government, the Mayas also had an incredibly
accurate system of measuring time.

4. their civilization came to an end because the Mayas
never developed a proper resistance to local germs and
diseases.(lines 36-38)
01. Due to the fact that the Mayas never developed a
proper resistance to local microbes and illnesses, their
civilization disappeared.
02. As the Mayas were unable to resist local germs and
diseases in a proper way, their civilization ceased to
exist.
04. The Mayas never offered an adequate resistance to
local microbes and diseases; for that reason their
civilization came to an end.
08. The Mayas were able to develop a proper resistance to
local germs and diseases; thats why their civilization
never came to an end.

Os exerccios abaixo se baseiam no vocabulrio do
texto The Mayas. Assinale apenas uma opo que
equivalente (s) palavra(s) em negrito.

5. Mr Brown was on a business journey when he met his
future wife
a) deal.
b) Newspaper.
c) rush.
d) trip.

6. The architect was interested in the remains of an
ancient church
a) ruins.
b) rebuilding.
c) practical solutions.
d) drawings.

7. Catherwoods drawings revealed incredible cities.
a) Disrupted.
b) Changed the order of.
c) Showed.
d) Question the existence of.

8. In 1841 a book was published on the remarkable
civilization.
a) Extraordinary.
b) Lost.
c) Mischievous.
d) Menacing.

9. What gods do your people worship?
a) Add to
b) Adorn
c) Adore
d) Erode


































Please purchase PDF Split-Merge on www.verypdf.com to remove this watermark.
Incluso para a Vida Ingls

Pr-Vestibular da UFSC
19

PERSONAL PRONOUNS
Person Subject Object Possessive
Adjective
Possessive
pronoun
Reflexive
SINGULAR First I
e.g. I have
a book.
me
e.g. Give
me the
book.
mine
e.g. The book is
mine.
my
e.g. It is my
book.
myself
e.g. I did it myself.
Second you
e.g. You
have a
book.
you
e.g. I will
give you
the book.
yours
e.g. The book is
yours.
your
e.g. It is your
book.
yourself
e.g. You did it
yourself.
Third he / she /
it
e.g.
He/she/ it
has a
book.
him / her
/ it
e.g. I will
give
him/her/it
the book.
his / hers / its
e.g. The book is
his/hers.
his / her / its
e.g. It is his/her
book.
himself/herself
e.g. He did it
himself / She did it
herself.
PLURAL First we
e.g. We
have some
books.
us
e.g. Give
us the
books.
ours
e.g. The books
are ours.
our
e.g. They are
our books.
ourselves
e.g. We did it
ourselves.
Second you
e.g. You
have some
books.
you
e.g. I will
give you
the
books.
yours
e.g. The books
are yours.
your
e.g. They are
your books.
yourselves
e.g. You did it
yourselves.
Third they
e.g. They
have some
books.
them
e.g. I will
give them
the
books.
theirs
e.g. The books
are theirs.
their
e.g. They are
their books.
themselves
e.g. They did it
themselves.


Exerccios de Sala #

10. Replace the nouns using subject pronouns.

a)The girls: _____ b) an eggplant: _____
c) the employer: _____ d) Paul and I: ____
e) Iphones: _____ f) Your stepbrothers and you: ______
g) Mrs. Jones: _____

11. Complete the sentences using the subject and object
pronouns.

a)Do you know that man? Yes, I work with ________.
b) Where are the tickets? I cant find _______.
c) I cant find my keys. Where are ______?
d) Were going out. Do you want to come with _______?
e) Ive got a new computer. Do you want to see _______?
f) Maria likes music. ______ plays the piano.
g) Im talking to you. Please listen to _______.
h) Where is Anna? I need to talk to _______.
i) My nephew has a new job, but ______ doesnt like
______ very much.



12. Choose the right word.
a) Its their/ theirs problem, not our/ ours.
b) This is a nice camera. Is it your/ yours?
c) Thats not my/ mine umbrella. My/ Mine is black.
d) Whose books are these? Your/ Yours or my/ mine?
e) My/ Mine room is bigger than her/ hers.

13. Complete the sentences with REFLEXIVE
PRONOUNS.
a) The boy blame ____________ for the car accident.
b) I burned ____________ while I was cooking lunch.
c) Youre very noisy kids! Behave ___________ while
your grandparents are here.
d) The tigers hurt ___________ when it was hunting in
the fields.
e) The little girl smiled When she saw ___________ in
the mirror.
f) We enjoy _____________ at the party yesterday.







Please purchase PDF Split-Merge on www.verypdf.com to remove this watermark.
Incluso para a Vida Ingls

Pr-Vestibular da UFSC
20

UNIDADE 13

RELATIVE PRONOUNS
Pronoun Used for Usage Example
who person/people subject pronoun I liked the woman. She was so friendly.
(Replace she) I liked the woman who was so friendly.
that thing/things
(sometimes
people)
subject pronoun I wanted the dress. It was such a good price.
(Replace it) I wanted the dress that was such a good price.
I liked the woman. She was so friendly.
(Replace she) I liked the woman that was so friendly.
object (pronoun) The dress was a good price. I saw it at Baileys.
(Replace it) The dress that I saw at Baileys was a good price.
which thing/things subject pronoun The dress was a good price. I saw it at Baileys.
(Replace it) The dress, which I saw at Baileys, was a good price.
object pronoun I wanted the dress. It was such a good price.
(Replace it) The dress, which I wanted, was such a good price.
whose person/people possessive pronoun I have a friend. His parents are rich.
(Replace his) I have a friend whose parents are rich.
whom person/people object pronoun The girl is Susan. I like her a lot.
(Replace her) The girl whom I like a lot is Susan.
object after a
preposition
She is the girl. I told you about her.
(Replaces her) She is the girl about whom I told you / She is the
girl whom I told you about.
where place there/it The shop is downtown. We work there.
(Replace there) The shop where we work is downtown.


Exerccios de Sala #

1. Fill in the blanks with WHO, WHICH, and WHOSE.
a) This is the bank ________ was robbed yesterday.
b) A boy __________ sister is in my class was in the
bank at that time.
c) The man _________ robbed the bank had two pistols.
d) He wore a mask _________ made him look like
Mickey Mouse.
e) He came with a friend _________ waited outside in
the car.
f) The woman __________ gave him the money was
young.
g) The bag __________ contained the money was
yellow.
h) A man __________ mobile was ringing did not know
what to do.
i) He didn't wait at the traffic lights _________ were
red.
j) The police officer _________ car was parked at the
next corner stopped and arrested them.
2. Right or wrong? Correct the mistakes.

a) A thief is a person which steals things.
______________________________________________


b) An airplane is a machine that flies.
______________________________________________

c) Have you seen the money that was on the desk?
______________________________________________
d) I dont like people which never stop talking.
______________________________________________
e) I know somebody that can help you.
______________________________________________
f) Correct the sentences who are wrong.
______________________________________________

Choose the correct alternatives to fill in the blanks.

3. Is that the man _____ wants to buy your cottage?
a) whom c) that
b) who d) b and c are correct

4. We know the sculptor ______ works are displayed in
that art gallery.
a) whose c) who
b) whom d) which

5. Have you heard about the people and cars ____ are
involved in the accident?
a) whose c) who
b) whom d) that

Please purchase PDF Split-Merge on www.verypdf.com to remove this watermark.
Incluso para a Vida Ingls

Pr-Vestibular da UFSC
21
6. Who is the old lady about _____ health the doctors are
worried?
a) who c) whose
b) whom d) which
Interrogative Pronouns

Interrogative pronouns are used to begin or
introduce interrogative sentences.
They are who, whom, whose, what, which, why, how,
how often, how long, how many, how much, and other
derivatives.
Examples
- Whom did you invite to the party?
- What color do you prefer?
- Who did what to whom?
- Why didnt she come to school yesterday?
- How do you go to Johns party?
- How long have you been in California?

Exerccios de Sala #
7. Put in WHAT, WHERE, WHY, WHEN, HOW into the
gaps and form meaningful questions.

a) _______ often do you play volleyball?
b) _______ time does Bill get up in the morning?
c) _______ don't you go by bus, Max?
d) _______ hobbies does Andrew have?
e) _______ do they go to every week?
f) _______ old is Mike?
g) _______ is Susan's birthday?
h) _______ are you doing at the moment, Sally?
i) ________ deep is your the club swimming pool?
j) ________ do the Robinsons live?

8. A alternativa que completa corretamente a lacuna
abaixo :
- _______ are you doing?
- Im doing fine, thanks.
a) Why
b) How
c) Whose
d) What
e) Who

9. A alternativa que completa corretamente a lacuna
abaixo :
- _______ has a good sense of humor in your
family?
- Almost everybody does.
a) Why
b) When
c) Whose
d) Who
e) What

10._____ does he do in the laboratory?
He is the chief scientist here.
a) How long
b) Where
c) What
d) Why
e) Who


(UFSC 2008)
Text Thirteen
SANTA CATARINA
(1) [] Apart from indigenous inhabitants, only
adventurers, seamen and military personnel had
passed through these lands on their way to other
places. In the middle of the XVII century, explorers
from So Paulo founded three settlements So
Francisco do Sul, Desterro (today Florianpolis), and
Santo Antnio de Laguna. But it was a century later,
as of 1748 that the occupation of the territory
began, with the arrival of the Azorean immigrants.
(2) The Azoreans settled on the coast. The inland regions
were occupied in the XIX century by successive
migratory waves of different European origins.
Pioneering Germans, Italians, Poles, Ukrainians,
Norwegians, among others, had to face the natural
jungle-like surroundings, live from the land, and
develop enterprising spirits. During the XX century,
other Europeans, Asians and Arabs, as well as
Brazilians from other regions, came to plant their
dreams and hopes in Catarinense soil.
(3) The state has a wide variety of landscapes,
ecosystems, historic roots and economic activities.
Considering it occupies only 95.4 thousand km
2

equivalent to 1% of the Brazilian territory the state
offers a surprising variety of landscapes. The
coastline is 560km long and the beaches are
paradisiacal. The mountains with heights reaching
2,000 meters are covered extensively with preserved
forests, which mix and blend in with the rain forest
and the majesty of the pine trees. There are also
rivers, lakes and dozens of thermal-mineral springs.
Santa Catarina State Passport.
SANTUR Santa Catarina Tourist Agency (adapted).

11. Considering the topic of text 1, choose the
proposition(s) that can start it meaningfully.
01. The occupation of the land, however, would only
begin with the Vicentista settlement.
02. In the beginning, the land which was called Santa
Catarina by the navigator Sebastio Caboto, was
occupied only by the indians.
04. Although initially well implanted, Portugal saw no
growth in the nucleus of the three first settlements.
08. The king of Portugal, then, opened inscriptions for
anyone who would like to settle in the south of
Brazil.
16. During the first two centuries of Brazils history,
Santa Catarina was not part of the colonization
process.
32. With the possession of these open spaces, the
effective occupancy of the land began in Santa
Catarina.

12. Select the correct proposition(s) to answer the
following question.
What does the text say about the European immigrants
who came to Santa Catarina?
01. They had to deal with the difficulties of an unfriendly
environment.
02. The first settlements in Santa Catarina were founded
by them.
Please purchase PDF Split-Merge on www.verypdf.com to remove this watermark.
Incluso para a Vida Ingls

Pr-Vestibular da UFSC
22
04. The colonies they established were very similar to
their original countries.
08. They were mostly from Germany and established their
settlements in different parts of the state.
16. Their survival depended on the use of the natural
resources they found in the new land.
32. They arrived at different times and came from
different countries.

13. The following statements, about the third
paragraph of the text, are all false. Select the
proposition(s) that can be made TRUE by changing one
word or expression for the word or expression in
parenthesis.
01. The state occupies a considerable amount of the
Brazilian territory. (large percentage)
02. The forests which cover the mountains are in danger.
(in extension)
04. In some areas, pine trees cover the mountains.
(protect)
08. The coastal area extends for over six hundred
kilometers. (almost)
16. Geographic diversity is an unnoticeable feature of
Santa Catarinas territory. (outstanding)

14. Select the correct proposition(s) to fill in the gaps of
the following paragraph.
In Santa Catarina, ______ the special and ______
individual nature of the territorys occupation, the human
profile of the population is more ______ than in
______ Brazilian regions. The richness of the ______
mosaic is surprising!
01. because of very diversified other ethnic
02. just as most developed entire personal
04. resulting from truly varied many cultural
08. besides that proper settled several social
16. along with total organized each racial

15. Choose the proposition(s) that presents (present) the
correct punctuation.
01. In many cities rural, and agrestic land, some of it
untouched by human hand. Coexists in harmony,
near urban centers. The seasons are well defined:
tropical summers cold winters with snow in some
areas and warm autumns.
02. In many cities, rural and agrestic land, some of it
untouched by human hand, coexists in harmony near
urban centers. The seasons are well defined
tropical summers, cold winters, with snow in some
areas, and warm autumns.
04. In many cities, rural and agrestic land some of it,
untouched by human hand coexists in harmony.
Near urban centers the seasons, are well defined.
Tropical summers, cold winters with snow, in some
areas and warm autumns.
08. In many cities rural and agrestic land some of it,
untouched by the human hand coexists. In harmony,
near urban centers; the seasons are well defined,
tropical summers cold winters with snow in some
areas. And warm autumns.



Please purchase PDF Split-Merge on www.verypdf.com to remove this watermark.
Incluso para a Vida Literatura

Pr-Vestibular da UFSC
1

AUTORES CARACTERSTICAS
Q
U
I
N
H
E
N
T
I
S
M
O

1500
1 Documento escrito em terras
brasileiras: Carta a D. Manuel.
Gneros: poesia lrica e pica, teatro

Crnicas
9 Pero Vaz de Caminha.
Teatro
9 Jos de Anchieta.
Valorizao do homem (antropocentrismo); paganismo;
superioridade do homem sobre a natureza; objetividade;
racionalismo; universalidade; saber concreto dos valores
greco-romanos; rigor mtrico e rtmico: equilbrio e
harmonia.
B
A
R
R
O
C
O

1601
Bento Teixeira: Prosopopeia.

Padre Antnio Vieira
9 oratria (sermes)

Gregrio de Matos
(Boca do Inferno)
9 poesia satrica
9 poesia sacra (religiosa)
9 poesia amorosa
Arte dos contrastes; antinomia homem/cu; homem/terra;
visualizao e plasticidade; fugacidade; irracionalismo;
unidade e abertura (perspectivas mltiplas para o
observador); luta entre o profano e o sagrado. Culto a
elementos evanescentes (gua/vento). Movimento ligado ao
esprito da Contrarreforma; jogos de metforas; riqueza de
imagens; gosto pelo pormenor; malabarismo verbal uso de:
hiprbato, hiprbole, metforas e antteses.
A
R
C
A
D
I
S
M
O

1768
Cludio Manuel da Costa:
Obras Poticas.

Poesia lrica
9 Cludio Manuel da Costa
9 Toms Antnio Gonzaga
9 Silva Alvarenga
9 Alvarenga Peixoto
Poesia pica
9 Baslio da Gama
9 Santa Rita Duro
Arte do equilbrio e da harmonia; busca do racional, do
verdadeiro e da natureza (Bucolismo); retorno s concepes
de beleza do Renascimento; poesia objetiva e descritiva;
aurea mediocritas; o objetivo arcdico de uma vida serena e
buclica; pastoralismo; valorizao da mitologia; tcnica da
simplicidade.
Leitura linear e regrada: inutilia truncat (cortar o intil),
carpe diem (aproveitar o dia), fugere urbem (fugir da cidade).
R
O
M
A
N
T
I
S
M
O

1836
Gonalves de Magalhes
Suspiros Poticos e Saudades.

Poesia
1 Gerao
9 Gonalves Dias
2

Gerao
9 lvares de Azevedo
9 Casimiro de Abreu
9 Fagundes Varela
9 Junqueira Freire
3

Gerao
9 Castro Alves

Prosa
Urbano
9 Joaquim Manuel de Macedo
9 Manuel Antnio de Almeida
Regional
9 Bernardo Guimares
9 Visconde de Taunay
Indianista/Histrico/Regional/Urbano
9 Jos de Alencar.
1

Gerao: nacionalismo, ufanismo, natureza, religio


(cristianismo), indianismo/medievalismo.
2

Gerao: mal do sculo, evaso, solido, profundo


pessimismo, anseio de morte, Ultrarromantismo, Byronismo,
saudosismo.
3

Gerao: Condoreirismo, liberdade. Oratria


reivindicatria, literatura social e engajada. Hiprbole.
Geral: imaginao, fantasia, sonho, idealizao, sonoridade,
simplicidade, subjetivismo, sintaxe emotiva, liberdade
criadora, idealizao da mulher, cor local, fuga da realidade,
bem X mal, final feliz





Please purchase PDF Split-Merge on www.verypdf.com to remove this watermark.
Incluso para a Vida Literatura

Pr-Vestibular da UFSC
2
BARROCO

Pecado contrito aos ps do Cristo crucificado.

Ofendi-vos, meu Deus, bem verdade,
Verdade , meu Senhor, que hei delinquido,
Delinquido vos tenho, e ofendido,
Ofendido vos tem minha maldade.

Maldade, que encaminha a vaidade,
Vaidade, que todo me h vencido,
Vencido quero ver-me e arrependido,
Arrependido a tanta enormidade.

Arrependido estou de corao,
De corao vos busco, dai-me abraos,
Abraos, que me rendem vossa luz.

Luz, que claro me mostra a salvao,
A salvao pretendo em tais abraos
Misericrdia, amor, Jesus, Jesus!

Descreve o que era naquele tempo a cidade da Bahia.

A cada canto um grande conselheiro,
Que nos quer governar cabana e vinha;
No sabem governar sua cozinha,
E podem governar o mundo inteiro.

Em cada porta um bem frequente olheiro,
Que a vida do vizinho e da vizinha
Pesquisa, escuta, espreita e esquadrinha,
Para levar praa e ao terreiro.

Muitos mulatos desavergonhados,
Trazidos sob os ps os homens nobres,
Posta nas palmas toda a picardia,

Estupendas usuras nos mercados,
Todos os que no furtam muitos pobres:
E eis aqui a cidade da Bahia.

Moralidade sobre o dia de quarta-feira de cinzas.

Que s terra, homem, e em terra hs de tornar-te,
Te lembra hoje Deus por sua Igreja;
De p te faz espelho, em que se veja
A vil matria, de que quis formar-te.

Lembra-te Deus, que s p para humilhar-te,
E como o teu baixel sempre fraqueja
Nos mares da vaidade, onde peleja,
Te pe vista a terra, onde salvar-te.

Alerta, alerta, pois, que o vento berra
Se assopra a vaidade e incha o pano,
Na proa a terra tens, amaina e ferra.

Todo o lenho mortal, baixel humano;
Se busca a salvao, toma hoje terra,
Que terra de hoje porto soberano.

ARCADISMO

Soneto XLVI

No vs, Nise, brincar esse menino
Com aquela avezinha? Estende o brao;
Deixa-a fugir; mas apertando o lao
A condena outra vez ao seu destino.

Nessa mesma figura, eu imagino,
Tens minha liberdade; pois ao passo
Que cuido que estou livre do embarao,
Ento me prende mais meu desatino.

Em um contnuo giro o pensamento
Tanto a precipitar-me se encaminha,
Que no vejo onde para o meu tormento.

Mas se fora menos mal esta nsia minha,
Se me faltasse a mim o entendimento
Como falta razo a esta avezinha.

Texto I

Eu, Marlia, no sou algum vaqueiro,
Que viva de guardar alheio gado,
De tosco trato; de expresses grosseiro,
Dos frios gelos e dos sois queimado.
Tenho prprio casal e nele assisto;
D-me vinho, legume, fruta, azeite;
Das brancas ovelhinhas tiro o leite
E as mais finas ls, de que me visto.
Graas, Marlia bela,
Graas minha estrela!

Texto VII

Os seus compridos cabelos,
Que sobre as costas ondeiam,
So que os de Apolo mais belos,
Mas de outra cor no so.
Tm a cor da negra noite,
E com o brao do rosto
Fazem, Marlia, um composto
Da mais formosa unio.

Texto VIII

Os teus olhos espelham a luz divina,
A quem a luz do sol no se atreve;
Papoila ou rosa delicada e fina
Te cobre as faces, que so da cor da neve.
Os teus cabelos so uns fios douro;
Teu lindo corpo blsamo vapora.

ROMANTISMO

Canto da Morte

Meu canto de morte,
Guerreiros, ouvi:
Sou filhos das selvas,
Nas selvas cresci;
Guerreiros, descendo
Da tribo tupi.

Da tribo pujante,
Que agora anda errante
Por fado inconstante,
Guerreiros, nasci:
Sou bravo, sou forte,
Sou filho do Norte;
Meu canto de morte,
Guerreiros, ouvi. (...)



Please purchase PDF Split-Merge on www.verypdf.com to remove this watermark.
Incluso para a Vida Literatura

Pr-Vestibular da UFSC
3
A Maldio do Pai

Tu choraste em presena da morte?
Na presena de estranhos choraste?
No descende o cobarde do forte;
Pois choraste, meu filho no s!
Possas tu, descendente maldito
De uma tribo de nobres guerreiros,
Implorando cruis forasteiros,
Seres presa de vis Aimors.(...)

Cano do Exlio

Minha terra tem palmeiras,
Onde canta o Sabi;
As aves que aqui gorjeiam,
No gorjeiam como l.

Nosso cu tem mais estrelas,
Nossas vrzeas tm mais flores,
Nossos bosques tm mais vida,
Nossa vida mais amores.

Em cismar, sozinho, noite,
Mais prazer encontro eu l;
Minha vida tem palmeiras,
Onde canta o Sabi.

Minha terra tem primores,
Que tais no encontro eu c;
Em cismar - sozinho, noite -
Mais prazer encontro eu l;
Minha terra tem palmeiras,
Onde canta o Sabi.

No permita Deus que eu morra,
Sem que eu volte para l;
Sem que desfrute os primores
Que no encontro por c;
Sem quinda aviste as palmeiras,
Onde canta o Sabi. (Coimbra, julho 1843)

Se Eu Morresse Amanh

Se eu morresse amanh, viria ao menos
Fechar meus olhos minha triste irm;
Minha me de saudades morreria
Se eu morresse amanh!

Quanta glria pressinto em meu futuro,
Que aurora de porvir e que manh!
Eu perdera chorando essas coroas
Se eu morresse amanh!

Que sol! Que cu azul! Que doce nalva
Acorda a natureza mais lou!
No me batera tanto amor no peito
Se eu morresse amanh!

Mas essa dor da vida que devora
A nsia de glria, o dolorido af ...
A dor no peito emudecera ao menos
Se eu morresse amanh!

Lembrana de Morrer

Quando em meu peito rebentar-se a fibra,
Que o esprito enlaa dor vivente,
No derramem por mim nenhuma lgrima
Em plpebra demente.
E nem desfolhem na matria impura
A flor do vale que adormece ao vento:
No quero que uma nota de alegria
Se cale por meu triste pensamento.

Eu deixo a vida como deixa o tdio
Do deserto, o poento caminheiro
- Como as horas de um longo pesadelo
Que se desfaz ao dobre de um sineiro;

(...)Descansem o meu leito solitrio
Na floresta dos homens esquecida,
sombra de uma cruz, e escrevam nela:
Foi poeta - sonhou - e amou na vida.

Meus Oito Anos

Oh! Que saudades que tenho
Da aurora da minha vida,
Da minha infncia querida
Que os anos no trazem mais!
Que amor, que sonhos, que flores,
Naquelas tardes fagueiras
sombra das bananeiras,
Debaixo dos laranjais!

Como so belos os dias
Do despontar da existncia!
- Respira a alma inocncia
Como perfumes a flor;
O mar - lago sereno,

O cu - um manto azulado,
O mundo - um sonho dourado,
A vida - um hino damor!
Que auroras, que sol, que vida,
Que noites de melodia
Naquela doce alegria,
Naquele ingnuo folgar!
O cu bordado d estrelas,
A terra de aromas cheia,
As ondas beijando a areia
E a lua beijando o mar!

Oh! Dias da minha infncia!
Oh! Meu cu de primavera!
Que doce a vida no era
Nessa risonha manh!
Em vez de mgoas de agora,
Eu tinha nessas delcias
De minha me as carcias
E beijos de minha irm!

Livre filho das montanhas,
Eu ia bem satisfeito,
Da camisa aberto o peito,
- Ps descalos, braos nus -
Correndo pelas campinas
roda das cachoeiras,
Atrs das asas ligeiras
Das borboletas azuis!




Please purchase PDF Split-Merge on www.verypdf.com to remove this watermark.
Incluso para a Vida Literatura

Pr-Vestibular da UFSC
4
Cntico do Calvrio
memria de meu filho
morto a 11 de dezembro de 1863

Eras na vida a pomba predileta
Que sobre um mar de angstias conduzia
O ramo da esperana. - Eras a estrela
Que entre as nvoas do inverno cintilava
Apontando o caminho ao pegureiro.
Eras a messe de um dourado estio.
Eras o idlio de um amor sublime.
Eras a glria, - a inspirao, - a ptria,
O porvir de teu pai! - Ah! No entanto,
Pomba, - varou-te a flecha do destino!
Astro, engoliu-te o temporal no norte!
Teto, - caste! Crena, j no vives!(...)

Boa noite

Boa-noite, Maria! Eu vou-me embora.
A lua das janelas bate em cheio.
Boa-noite, Maria! tarde... tarde...
No me apertes assim contra teu seio.

Boa noite!... E tu dizes - Boa noite,
Mas no mo digas assim por entre beijos...
Mas no mo digas descobrindo o peito,
- Mar de amor onde vagam meus desejos.(...)

Vozes dfrica (trecho)

Deus! Deus! Onde ests que no respondes?
Em que mundo, em questrela tu tescondes
Embuado nos cus?
H dois mil anos te mandei meu grito,
Que embalde desde ento corre o infinito...
Onde ests, Senhor Deus?...

Qual Prometeu tu me amarraste um dia
Do deserto na rubra penedia
- Infinito: gal!...

Por abutre - me deste o sol cadente,
E a terra de Suez - foi a corrente

Que me ligaste ao p...
(...)
A Europa sempre Europa, a gloriosa!...
A mulher deslumbrante e caprichosa,
Rainha e cortes.
Artista - corta o mrmor de Carrara;
Poetisa - tange os hinos de Ferrara,
No glorioso af!...
(...)
Hoje em meu sangue a Amrica se nutre
- Condor que transformara-se em abutre,
Ave da escravido,
Ela juntou-se s mais ... irm traidora
Qual de Jos os vis irmos de outrora
Venderam seu irmo.

Basta, Senhor! De teu potente brao
Role atravs dos astros e do espao
Perdo pra os crimes meus!...
H dois mil anos... eu soluo um grito...
Escuta o brado meu l no infinito,
Meu Deus! Senhor, meu Deus!!...

Iracema (Jos de Alencar)

Pousando a criana nos braos paternos, a
desventurada me desfaleceu como a jetica, se lhe
arranca o bulbo. O esposo viu ento como a dor tinha
consumido o seu belo corpo; mas a formosura ainda
morava nela, como o perfume da flor cada do manac.
Iracema no se ergueu mais da rede onde a
pousaram os aflitos braos de Martim. O terno esposo,
em que o amor renascera com o jbilo paterno, acercou
de carcias que encheram sua alma de alegria, mas no a
puderam tornar vida: o estame de sua flor se rompera.
O doce lbio emudeceu para sempre; o ltimo
lampejo despediu-se dos olhos baos.

Poti amparou o irmo na grande dor, Martim
sentiu quanto um amigo verdadeiro precioso na
desventura: como o outeiro que abriga do vendaval o
tronco forte e robusto do Ubirat, quando o cupim lhe
broca o mago.
O camucim que recebeu o corpo de Iracema,
embebida em resinas odorferas, foi enterrado ao p do
coqueiro, borda do rio. Martim quebrou um ramo de
murta, a folha da tristeza, e deitou-o no jazigo de sua
esposa. A jandaia pousada no olho da palmeira repetia
tristemente:
Iracema!
Desde ento os guerreiros potiguaras que passavam
perto da cabana abandonada, e ouviam ressoar a voz
plangente da ave amiga, afastavam-se com a alma cheia
de tristeza do coqueiro onde cantava a jandaia. E foi
assim que um dia veio a chamar-se Cear o rio onde
crescia o coqueiro, e os campos onde serpeja o rio.

O Guarani
(Jos de Alencar)

Ceclia abriu os olhos e vendo seu amigo junto
dela, ouvindo ainda suas palavras, sentiu o enlevo que
deve ser o gozo da vida eterna.
Sim?...murmurou ela, viveremos!... l no cu,
no seio de Deus, junto daqueles que amamos!...

O anjo espanejava-se para remontar o bero.
Sobre aquele azul que tu vs, continuou ela,
Deus mora no seu trono, rodeado dos que O adoram.
Ns iremos l, Peri! Tu vivers com tua irm, sempre!...
Ela embebeu os olhos nos olhos do seu amigo,
e lnguida reclinou a loura fronte.
O hlito ardente de Peri bafejou-lhe a face.
Fez-se no semblante da virgem um ninho de
castos rubores e lnguidos sorrisos: os lbios abriram
como asas purpreas de um beijo soltando o voo.
A palmeira arrastada pela torrente impetuosa
fugia...
E sumiu-se no horizonte...

Tarefa Mnima

1. (PUC-RJ) Quais dessas afirmaes caracterizavam a
poesia realizada no Brasil no sculo XVIII?
01. Preocupa-se em descrever uma atmosfera
denominada locus amoenus.
02. A poesia seguia o tema de cortar o intil do texto.
Please purchase PDF Split-Merge on www.verypdf.com to remove this watermark.
Incluso para a Vida Literatura

Pr-Vestibular da UFSC
5
04. As amadas eram ninfas lembrando a mitologia
grega e romana.
08. Os poetas da poca no se expressaram no gnero
pico.

2. (UFPR)
Enquanto pasta, alegre, o manso gado,
minha bela Marlia, nos sentemos
sombra deste cedro levantado.
Um pouco meditemos
Na regular beleza,
Que em tudo quanto vive nos descobre
A sbia natureza.
Atente como aquela vaca preta
O novilhinho seu mais separa,
e o lambe, enquanto chupa a lisa teta.
Atente mais, cara,
Como a ruiva cadela
Suporta que lhe morda o filho o corpo,
e salte em cima dela.

Com relao ao fragmento acima, de uma lira de Toms
Antnio Gonzaga, podemos afirmar que:
01. Apresenta uma expresso potica altamente
subjetiva.
02. Apresenta uma ntida concepo de poesia de cunho
pedaggico, ao gosto barroco.
04. Apresenta a clareza, a simplicidade e a
verossimilhana como caractersticas evidentes.
08. Apresenta, como caracterstica pr-romntica, uma
preocupao idealizante em relao natureza.
16. Apresenta um clima pastoral, conveno potica
rcade que tematiza a naturalidade e o equilbrio
como formas ideais das relaes humanas.

3. (PUC-PR) Nos romances Senhora e Lucola, Jos de
Alencar apresenta como qualidade mais importante:
a) Exame psicolgico das personagens femininas;
b) Anlise do contexto social e poltico do Brasil do
segundo imprio;
c) Estudo das transformaes sociais e seu reflexo
sobre o comportamento das personagens;
d) Estudo do problema amor X dinheiro e sua crtica;
e) Verificao das diferenas entre o interior e a capital,
considerando que as personagens mais importantes
so originrias do interior do Brasil.

4. O padre Antnio Vieira celebrizou-se
a) no sculo XVII, pela sua extraordinria parentica
barroca.
b) no sculo XX, pela riqueza e beleza de seus ensaios.
c) no sculo XIX, por ser historiados e cronista dos
tempos coloniais.
d) no sculo XVIII, pelo jornalismo lcido e
equilibrado que fez na Colnia.
e) no sculo XVI, por suas atitudes vacilantes em
relao ao invasor holands protestante.

5. (FUVEST-SP)
Que s terra, homem, e em terra hs de tornar-te
Te lembra hoje Deus por sua Igreja;
De p te faz espelhos, em que se veja
A vil matria, de que quis formar-se.

Pelas caractersticas do quarteto acima podemos dizer
que ele se enquadra no:
a) Barroco.
b) Arcadismo.
c) Romantismo.
d) Parnasianismo.
e) Modernismo.

6. (FUVEST-SP)
Nasce o Sol, e no dura mais que um dia,
Depois da luz, se segue a noite escura,
Em tristes sombras morre a formosura,
Em contnuas tristezas a alegria.

Na estrofe acima, de um soneto de Gregrio de matos
Guerra, a principal caracterstica do Barroco
a) o culto da natureza.
b) a utilizao de rimas alternadas.
c) a forte presena de antteses.
d) o culto do amor corts.
e) o uso de aliteraes.

7. (PUC-SP)
Que falta nesta cidade? Verdade.
Que mais por sua desonra? Honra.
Falta mais que lhe ponha? Vergonha.
O demo a viver se exponha,
Por mais que a fama a exalta,
Numa cidade onde falta
Verdade, honra, vergonha.

Pode-se reconhecer nestes versos, de Gregrio de
Matos:
a) O carter de jogo verbal prprio do estilo Barroco, a
servio de uma crtica, em tom de stira, do perfil
moral da cidade da Bahia.
b) O carter de jogo verbal prprio da poesia religiosa
do sculo XVI, sustentando piedosa lamentao pela
falta de f do gentio.
c) O estilo pedaggico da poesia neoclssica, por meio
da qual o poeta se investe das funes de um
autntico moralizador.
d) O carter de jogo verbal prprio do estilo Barroco, a
servio da expresso lrica, do arrependimento do
poeta pecador.
e) O estilo pedaggico da poesia neoclssica,
sustentando em tom lrico as reflexes do poeta
sobre o perfil moral da cidade da Bahia.

8. (FAUS-SP) O indianismo de nossos poetas
romnticos
a) forma de apresentar o ndio em toda a sua realidade
objetiva. O ndio como elemento tnico da futura
raa brasileira.
b) meio de reconstruir o grave perigo que o ndio
representava durante a instalao da capital de So
Vicente.
c) modelo francs seguido no Brasil. Uma necessidade
de exotismo que em nada difere do modelo europeu.
d) meio de eternizar liricamente a aceitao, pelo ndio,
de nossa civilizao que se instalava.
e) forma de apresentar o ndio como motivo esttico.
Idealizao com simpatia e piedade. Exaltao da
Please purchase PDF Split-Merge on www.verypdf.com to remove this watermark.
Incluso para a Vida Literatura

Pr-Vestibular da UFSC
6
bravura, do herosmo e de todas as qualidades morais
superiores.

9. (UFJF-MG) Em relao ao Romantismo brasileiro,
quais afirmaes so verdadeiras?
01. Expresso do nacionalismo atravs da descrio dos
costumes e regies do Brasil.
02. Expresso potica de temas confessionais,
indianistas e humanistas.
04. Desenvolvimento do teatro nacional.
08. Anlise crtica e cientfica dos fenmenos da
sociedade brasileira.
16. Caracterizao do romance como forma de
entretenimento e moralizao.

10. (UFRGS) A produo de lvares de Azevedo , no
Brasil, a maior expresso
a) do culto natureza.
b) do cientificismo
c) da arte pela arte
d) do culto ao bom selvagem
e) do mal-do-sculo

11. (FUVEST-SP) Sobre o romance indianista de Jos
de Alencar, pode-se afirmar que:
a) Analisa as reaes psicolgicas da personagem como
um efeito das influncias sociais.
b) um composto resultante de formas originais do
conto.
c) D forma ao heri amalgamando-o vida da
natureza.
d) Representa contestao poltica ao domnio
portugus.
e) Mantm-se preso aos modelos legados pelos
clssicos.

12. (UFSC) Considere as afirmativas sobre o Barroco
e o Arcadismo e some as corretas sobre o Arcadismo.
01. Simplificao da lngua literria ordem direta
imitao dos antigos gregos e romanos.
02. Valorizao dos sentidos imaginao exaltada
emprego de vocbulos raros.
04. Vida campestre idealizada como verdadeiro estado
de poesia clareza harmonia.
08. Emprego frequente de trocadilhos e de perfrases
malabarismos verbais oratria.
16. Sugestes de luz, cor e som anttese entre vida e
morte esprito cristo antiterreno.
32. Bucolismo Epopeia Pseudnimos.

13. (UNIFOR-CE) Assinale a alternativa que preenche
corretamente as lacunas da frase apresentada.

TEXTO I
a vaidade, Fbio, nesta vida
Rosa que, da manh lisonjeada,
Prpuras mil, com ambio dourada,
Airosa rompe, arrasta presumida.

TEXTO II
Fatigado de calma se acolhia
Junto o rebanho sombra dos salgueiros,
E o sol, queimando speros oiteiros,
Com violncia maior no campo ardia.
A natureza, para os poetas ......., era a fonte de smbolos
(rosa, cristal, gua), que transcenderam do material para
o espiritual (Texto I); para os poetas ......., era sobretudo
o cenrio idealizado, dentro do qual se podia ser feliz.
(Texto II).

a) romnticos parnasianos
b) parnasianos simbolistas
c) rcades romnticos
d) simbolistas barrocos
e) barrocos rcades

14. Sobre o Arcadismo brasileiro pode-se afirmar que:
01. Tem suas fontes nos antigos grandes autores gregos
e latinos, dos quais imita os motivos e as formas.
02. Teve em Cludio Manuel da Costa o representante
que, de forma original, recusou a motivao buclica
e os modelos camonianos da lrica amorosa.
04. Nos legou os poemas de feio pica Caramuru (de
Frei Jos de Santa Rita Duro) e O Uraguai (de
Baslio da Gama) e na poesia satrica Gregrio de
Matos.
08. Norteou, em termos dos valores ticos bsicos, a
produo dos versos de Marlia de Dirceu, obra que
celebrizou Toms Antnio Gonzaga e que destaca a
originalidade de estilo e de tratamento local dos
temas pelo autor.
16. Apresentou uma corrente de conotao ideolgica,
envolvida com as questes sociais do seu tempo,
com a crtica aos abusos do poder da Coroa
Portuguesa.

15. (FMABC-SP) Assinale a alternativa em que se
encontram trs caractersticas do movimento literrio ao
qual se d o nome de Romantismo:
a) Predomnio da razo, perfeio da forma, imitao
dos antigos gregos e romanos.
b) Reao anticlssica, busca de temas nacionais,
sentimentalismo e imaginao.
c) Anseio de liberdade criadora, busca de verdades
absolutas e universais, arte pela arte.
d) Desejo de expressar a realidade objetiva, erotismo,
viso materialista do universo
e) Preferncia por temas medievais, rebuscamento de
contedo e de forma, tentativa de expressar a
realidade inconsciente.

16. (UCP-PR) Livros indianistas de Jos de Alencar:
a) Iracema, Ubirajara, Inocncia.
b) O Guarani, Iracema, A Escrava Isaura.
c) A Moreninha, Iracema, Lucola.
d) Memrias de um Sargento de Milcias, O Guarani, O
Tronco do Ip.
e) Ubirajara, O Guarani, Iracema.

17. (UFPR) Com relao ao Barroco, correto afirmar
que:
01. Os princpios da analogia, transformao e
contrastes so as principais matizes que subjazem
aos diversos processos estilsticos do movimento.
02. Como estado de esprito o movimento reflete
angstia e tenso existencial.
04. um movimento ps-romntico que se desenvolveu
principalmente nos pases da Pennsula Ibrica.
Please purchase PDF Split-Merge on www.verypdf.com to remove this watermark.
Incluso para a Vida Literatura

Pr-Vestibular da UFSC
7
08. Esteticamente o movimento apresenta duas
tendncias principais e o conceptismo que enfatiza o
contedo e a complexidade das ideias.
16. Historicamente o movimento est intimamente
relacionado contrarreforma que atuou de modo
marcante no campo da literatura, da arquitetura e da
msica.

18. (UCPelotas RS) Leia os textos seguintes, compare-
os e assinale a opo correta sobre eles:

Por morto, Marlia
Aqui me reputo:
Mil vezes escuto
O som do arrastado,
E duro grilho
Mas ah! que no treme,
No treme de susto
O meu corao.

Se penso que posso
Perder o gozar-te,
E a glria de dar-te
Abraos honestos,
E beijos na mo.
Marlia, j treme,
j treme de susto
O meu corao.
(Toms Antnio Gonzaga)

a) O primeiro corresponde a uma situao trgica
presente, e o segundo aos sucessos do passado.
b) O poeta e Marlia so personagens de um idlio,
distantes de um do outro por fatores de sade.
c) O medo um elemento comum nas duas estrofes,
sendo a causa do desespero do poeta.
d) A segunda estrofe a fundamental porque
identifica a amada do poeta.
e) O poeta no teme a violncia, mas, sim, a
possibilidade de ficar privado do amor.

19. Sobre Padre Anchieta incorreto afirmar que:
a) Cultivou especialmente os autos, buscando, na
alegoria, tornar mais acessveis s mentes indgenas
os conceitos e os dogmas do cristianismo.
b) No teatro, o "Auto de So Loureno" destaca-se
como obra catequtica de influncia medieval.
c) Na poesia lrica encontram-se suas mais belas
composies, expressivas de uma f profunda.
d) Apesar de pautada na lngua e na cultura do ndio, sua
produo literria no se caracteriza como literatura
j tipicamente brasileira.
e) Sua obra teatral, marcadamente alegrica e
antirreligiosa, moldou-se nos padres renascentistas.

20. Assinale a alternativa cujos termos preenchem
corretamente as lacunas do texto inicial. Como bom
barroco e oportunista que era, este poeta de um lado
lisonjeia a vaidade dos fidalgos e poderosos, de outro
investe contra os governadores, os "falsos fidalgos". O
fato que seus poemas satricos constituem um vasto
painel .................., que ............... comps com rancor e
engenho ainda hoje admirados pela expressividade.

a) do Brasil do sculo XIX - Gregrio de Matos
b) da sociedade mineira do sculo XVIII - Cludio
Manuel da Costa
c) da Bahia do sculo XVII - Gregrio de Matos
d) do ciclo da cana-de-acar - Antnio Vieira
e) da explorao do ouro em Minas - Cludio Manuel da
Costa.

21. SONETO

Ardor em firme corao nascido;
Pranto por belos olhos derramado;
Incndio em mares de guas disfarado;
Rio de neve em fogo convertido:

Tu, que em um peito abrasas escondido;
Tu, que em um rosto corres desatado;
Quando fogo, em cristais aprisionado;
Quando cristal em chamas derretido.

Se s fogo como passas bradamente,
Se s neve, como queimas com porfia?
Mas ai, que andou Amor em ti prudente!

Pois para temperar a tirania,
Como quis que aqui fosse a neve ardente,
Permitiu parecesse a chama fria.

Leia o poema de Gregrio de Matos:
I - O par fogo e gua, que figura amor e contentao,
passa por variaes contrastantes at evoluir para o
oxmoro.
II - O poema evidencia a "frmula da ordem barroca"
ditada por Grard Genette: diferena transforma-se
em oposio, oposio em simetria e simetria em
razo.
III - O poema inscreve, no mbito da linguagem, o
conflito vivido pelo homem do sculo XVII.

De acordo com o poema, pode-se concluir que esto
corretas:
a) I e II d) apenas uma delas
b) I e III e) todas elas
c) II e III

22. Assinale a alternativa em que os dois versos
indicados da 1
a
estrofe apresentam metforas de
lgrimas.
a) versos 1 e 2 d) versos 3 e 4
b) versos 2 e 4 e) versos 1 e 3
c) versos 2 e 3

23. Leia o soneto a seguir, de autoria de Gregrio de
Matos:

Pequei, Senhor, mas no porque hei pecado,
Da vossa piedade me despido,
Porque quanto mais tenho delinquido,
Vos tenho a perdoar mais empenhado.

Se basta a vos irar tanto pecado,
A abrandar-vos sobeja um s gemido,
Que a mesma culpa, que vos h ofendido,
Vos tem para o perdo lisonjeado.

Se uma ovelha perdida, e j cobrada
Glria tal e prazer to repentino
Please purchase PDF Split-Merge on www.verypdf.com to remove this watermark.
Incluso para a Vida Literatura

Pr-Vestibular da UFSC
8
vos deu, como afirmais na Sacra Histria:

Eu sou, Senhor, a ovelha desgarrada,
Cobrai-a, e no queiras, Pastor divino,
Perder na vossa ovelha a vossa glria.

Assinale a alternativa incorreta:
a) No jogo de antteses, o poeta v-se como culpado,
mas tambm ovelha indispensvel ao Pastor Divino.
b) O argumento do poeta, arrependido, constri-se pelo
jogo de ideias, ou seja, o cultismo.
c) O poeta recorre ao texto bblico para justificar,
perante Deus, a necessidade de ser perdoado.
d) Segundo o poeta, o perdo de sua culpa favorecia a
ambos: tanto ao culpado, quanto ao Pastor Divino.
e) O poeta busca, em sua linguagem dualista, conciliar,
poeticamente, f e razo.

24. Texto
"Alm do horizonte, deve ter
Algum lugar bonito para viver em paz
Onde eu possa encontrar a natureza
Alegria e felicidade com certeza.
L nesse lugar o amanhecer lindo
com flores festejando mais um dia que vem vindo
Onde a gente possa se deitar no campo
Se amar na selva, escutando o canto dos pssaros."

Roberto e Erasmo Carlos esto falando de um lugar
ideal, de um ambiente campestre, calmo.

Qual das afirmaes abaixo que faz referncia a essa
msica?

a) ROMANTISMO, para quem encontrar-se com a
natureza significava alargar a sensibilidade.
b) ARCADISMO, propondo um retorno ordem
natural, como na literatura clssica, na medida em
que a natureza adquire um sentido de simplicidade,
harmonia e verdade.
c) REALISMO, fugindo s exibies subjetivas e
mantendo a neutralidade diante daquilo que era
narrado; as referncias natureza eram feitas em
terceira pessoa.
d) BARROCO, movimento que valorizava a tenso de
elementos contrrios, celebrando Deus e a forma.
e) SIMBOLISMO quando estes escritores se mostravam
mais emotivos, transformando as palavras em
smbolos dos segredos da alma. A natureza era puro
mistrio.

25. Considere as seguintes afirmaes sobre a obra de
Jos de Alencar. Quais esto corretas?
I - Em IRACEMA, narram-se as aventuras e desventuras
de Martim Francisco, portugus, e Iracema, a
indgena dos lbios de mel, casal que simboliza a
unio dos dois povos nas matas brasileiras
inexploradas.
II - Em SENHORA, Aurlia herda uma fortuna que a
salva da pobreza e lhe permite comprar um marido,
Seixas, de quem j fora namorada e com quem
manter um casamento perturbado por conflitos e
acusaes mtuas.
III - Em O GUARANI, as aventuras de Peri, bravo
guerreiro indgena, so norteadas pela necessidade
de servir e proteger a jovem virgem loira Ceci, cuja
integridade fsica ameaada por malfeitores e
indgenas perigosos.

a) Apenas I d) Apenas II e III
b) Apenas III e) I, II e III
c) Apenas

AUTORES CARACTERSTICAS
R
E
A
L
I
S
M
O










N
A
T
U
R
A
L
I
S
M
O

P
A
R
N
A
S
I
A
N
I
S
M
O

1881
Machado de Assis
Memrias Pstumas de Brs Cubas/
Realismo

Alusio Azevedo
O Mulato/ Naturalismo.

Dcada de 80
Definio do iderio parnasiano.

Prosa
9 Machado de Assis
9 Alusio Azevedo
9 Raul Pompeia.


Poesia (Parnasianismo)
9 Olavo Bilac
9 Alberto de Oliveira
9 Raimundo Correia
9 Vicente de Carvalho
Realismo: preocupao com a verdade exata, observao
e anlise, personagens tipificadas, preferncia pelas
camadas mais altas da sociedade. Objetividade.
Descries pormenorizadas. Linguagem correta, no
entanto mais prxima da natural, maior interesse pela
caracterizao que pela ao tese documental.
Naturalismo: viso determinista do homem (animal,
presa de foras fatais e superiores meio, herana
gentica, fisiologia, momento). Tendncia para anlise
dos deslizes de personalidade. Deturpaes psquicas e
fsicas. Preferncia por camadas menos privilegiadas.
Patologia social: misria, adultrio, criminalidade, etc.
tese experimental.
Parnasianismo: arte pela arte, objetividade, poesia
descritiva, versos impassveis e perfeitos, exatido e
economia de imagens e metforas, poesia tcnica e
formal, retomada dos valores clssicos, apego
mitologia greco-romana.
Please purchase PDF Split-Merge on www.verypdf.com to remove this watermark.
Incluso para a Vida Literatura

Pr-Vestibular da UFSC
9
S
I
M
B
O
L
I
S
M
O

1893
Cruz e Souza
Missal (1

semestre)
Broqueis (2

semestre)

Poesia
9 Cruz e Souza
9 Alphonsus de Guimaraens
9 Pedro Kilkerry
9 Augusto dos Anjos
9 Emiliano Perneta
Simbolismo: reao contra o Positivismo, o Naturalismo
e o Parnasianismo; individualismo, subjetivismo, atitude
irracional e mstica, respeito pela msica, cor, luz;
procura das possibilidades do lxico, aliterao,
sinestesia, sugesto da palavra, musicalidade, eco, apuro
formal.
P
R

-
M
O
D
E
R
N
I
S
M
O

Primeiras dcadas do sc. XX Pr-
Modernismo.

Prosa
9 Monteiro Lobato
9 Euclides da Cunha
9 Lima Barreto
9 Graa Aranha.
Pr-Modernismo: tendncias das primeiras dcadas do
sculo XX, sentido mais crtico, fixando diferentes
facetas da realidade social, poltica ou alteraes na
paisagem e cor local.

Obs.: No considerado como escola literria.

PARNASIANISMO

A um poeta

Longe do estril turbilho da rua,
Beneditino, escreve! No aconchego
Do claustro, na pacincia e no sossego,
Trabalha, e teima, e lima, e sofre, e sua

Mas que na forma se disfarce o emprego
Do esforo; e a trama viva se construa
De tal modo, que a imagem fique nua,
Rica mas sbria, como um templo grego.

No se mostre na fbrica o suplcio
Do mestre. E, natural, o efeito agrade,
Sem lembrar os andaimes do edifcio:

Porque e Beleza, gmea da Verdade,
Arte pura, inimiga do artifcio,
a fora e a graa na simplicidade.

Vaso chins

Estranho mimo, aquele vaso! Vi-o
Casualmente, uma vez, de um perfumado
Contador sobre o mrmor luzidio,
Entre um leque e o comeo de um bordado.

Fino artista chins, enamorado,
Nele pusera o corao doentio
Em rubras flores de um sutil lavrado,
Na tina ardente, de um calor sombrio.

Mas, talvez por contraste desventura -
Quem o sabe? - de um velho mandarim
Tambm l estava a singular figura:

Que arte em pint-la! A gente acaso vendo-a
Sentia um no sei qu com aquele chim
De olhos cortados feio de amndoa.



Profisso de f (trechos)
(...)
Invejo o ourives quando escrevo:
Imito o amor
Com que ele, em ouro, o alto relevo
Faz de uma flor.

Imito-o . E, pois nem de Carrara
A pedra firo:
O alvo cristal, a pedra rara,
O nix prefiro.(...)

SIMBOLISMO

Antfona

Formas alvas, brancas, Formas claras
De luares, de neves, de neblinas!...
Formas vagas, fluidas, cristalinas...
Incensos dos turbulos das aras...

Formas do Amor, constelarmente puras,
De Virgens e de Santas vaporosas...
Brilhos errantes, mdidas frescuras
E dolncias de lrios e de rosas...

Indefinveis msicas supremas,
Harmonias da Cor e do Perfume...
Horas do Ocaso, trmulas, extremas,
Rquiem do Sol que a Dor da Luz resume...(...)

REALISMO

O Cortio
Alusio Azevedo

Eram cinco horas da manh e o cortio acordava,
abrindo, no os olhos, mas sua infinidade de portas e
janelas alinhadas.
Um acordar alegre e farto de quem dormiu de
uma assentada, sete horas de chumbo. Como que se
sentiam ainda na indolncia de neblina as derradeiras
notas da ltima guitarra da noite antecedente,
dissolvendo-se luz loura e tenra da aurora, que nem um
suspiro de saudade perdido na terra alheia.
Please purchase PDF Split-Merge on www.verypdf.com to remove this watermark.
Incluso para a Vida Literatura

Pr-Vestibular da UFSC
10
A roupa lavada, que ficara de vspera nos
coradouros, umedecia o ar e punha-lhe um fartum acre
de sabo ordinrio. As pedras do cho, esbranquiadas
no lugar da lavagem e em alguns pontos azulados pelo
anil, mostravam uma palidez grisalha e triste, feita de
acumulaes de espumas secas.

Quincas Borba
Machado de Assis

No h morte. O encontro de duas expanses,
ou a expanso de duas formas, pode determinar a
supresso de uma delas; mas, rigorosamente, no h
morte, h vida, porque a supresso de uma a condio
de sobrevivncia da outra, e a destruio no atinge o
princpio universal e comum. Da o carter conservador
e benfico da guerra. Supes tu um campo de batatas e
duas tribos famintas. As batatas apenas chegam para
alimentar uma das tribos, que assim adquire foras para
transpor a montanha e ir outra vertente, onde h
batatas em abundncia; mas, se as duas tribos dividirem
em paz as batatas do campo, no chegam a nutrir-se
suficientemente e morrem de inanio. A paz, nesse
caso, a destruio; a guerra a conservao. Uma das
tribos extermina a outra e recolhe os despojos. Da a
alegria da vitria, os hinos, aclamaes, recompensas
pblicas e todos os demais efeitos das aes blicas. Se
a guerra no fosse isso, tais demonstraes no
chegariam a dar-se pelo motivo real que o homem s
comemora e ama o que lhe aprazvel ou vantajoso, e
pelo motivo racional de que nenhuma pessoa canoniza
uma ao que virtualmente o destroi. Ao vencido, dio
ou compaixo; ao vencedor, as batatas.

Dom Casmurro
(Machado de Assis)

Enfim, chegou a hora da encomendao e da
partida. Sancha quis despedir-se do marido, e o desespero
daquele lance consternou a todos. Muitos homens
choravam tambm, as mulheres todas. S Capitu,
amparando a viva, parecia vencer-se a si mesma.
Consolava a outra, queria arranc-la dali. A confuso era
geral. No meio dela, Capitu olhou alguns instantes para o
cadver to fixa, to apaixonadamente fixa, que no admira
lhe saltassem algumas lgrimas poucas e caladas...
As minhas cessaram logo. Fiquei a ver a delas;
Capitu enxugou-as depressa, olhando de furto para a gente
que estava na sala. Redobrou de carcias para a amiga, e
quis lev-la; mas o cadver parece que a retinha tambm.
Momento houve em que os olhos de Capitu fitaram o
defunto, quais os da viva, sem o pranto nem palavras
desta, mas grandes e abertos, como a vaga do mar l fora,
como se quisesse tragar tambm o nadador da manh.

PR-MODERNISMO

Versos ntimos
Vs! Ningum assistiu ao formidvel
Enterro de tua ltima quimera.
Somente a Ingratido - esta pantera -
Foi tua companheira inseparvel!

Acostuma-te lama que te espera!
O homem, que, nesta terra miservel,
Mora, entre feras, sente inevitvel
Necessidade de tambm ser fera.

Toma um fsforo. Acende teu cigarro!
O beijo, amigo, a vspera do escarro,
A mo que afaga a mesma que apedreja.

Se algum causa inda pena a tua chaga,
Apedreja essa mo vil que te afaga,
Escarra nessa boca que te beija!

Tarefa Mnima

26. Em relao ao Realismo e ao Naturalismo, podemos
afirmar que:
01. Analisam o homem como objeto de investigao
religiosa.
02. So ambos anticlericais, antirromnticos e anti-
burgueses.
04. Preocupam-se em descrever ou narrar
minuciosamente personagens e cenas.
08. Ambos mantm pontos comuns, sendo que o
primeiro analisa o homem em seus aspectos
biopatolgicos, enquanto o segundo em seus
aspectos psicossociolgicos.
16. Ambos enfocam o real.

27. Afastando-se de uma concepo religiosa mstica e
transcendental da existncia, o naturalismo:
a) Criou uma literatura intimista, valorizando aquilo
que fortemente pessoal, em detrimento de
preocupaes sociais, que aborda de maneira mais
tnue.
b) Cultivou a proximidade da natureza e desenvolveu o
gosto buclico, pelo pastoril, pelas coisas do campo.
c) elegeu, alm de uma viso do mundo que valorizava as
leis cientficas, uma tica determinista para a explicao
do comportamento humano.
d) Fez a apologia de tudo que terreno e material,
apegando-se vida breve para usufruir todos os
prazeres possveis.
e) Criou uma torre de marfim em que se refugiavam
seus autores, valorizou as emoes fundas e ps em
relevo as peculiaridades que individualizam cada ser
humano.

28. (PUC PR)
Eu amo os gregos tipos de escultura:
Pags nuas no mrmore entalhadas;
No essas produes que a estufa escura
Das modas cria, tontas e enfezadas.
(Plena Nudez. Raimundo Correia)

um poema tipicamente parnasiano, porque revela
a) a concepo de arte pela arte , a sensibilidade
emotiva do poeta, a ruptura do discursivo.
b) o evasionismo, o fascnio pela Grcia antiga, o
engajamento histrico.
c) o fascnio pela Antiguidade Clssica, a concepo de
arte pela arte, o culto da liberdade.
d) o paganismo, o retorno aos modelos clssicos, o uso
acentuado de smbolos.
e) a concepo da arte pela arte, o fascnio da
Antiguidade Clssica, a descrio objetiva.

Please purchase PDF Split-Merge on www.verypdf.com to remove this watermark.
Incluso para a Vida Literatura

Pr-Vestibular da UFSC
11
29. Some as alternativas corretas com respeito ao
Parnasianismo.
01. Assimilao dos ideais das artes plsticas: arte pela
arte.
02. Gosto pelos poemas de forma fixa: balada, soneto,
etc.
04. Subjetivismo afastado do sentimentalismo
romntico.
08. Objetivismo.
16. Forte tendncia para o narrativo, desprezando-se os
outros tipos de composio.
32. Verbalismo intenso: culto das palavras, linguagem
trabalhada.

30. (UFPR) Sobre Dom Casmurro, romance de
Machado de Assis, correto afirmar que:
01. O narrador apresenta-se como um defunto-autor,
condio que lhe garante distanciamento e iseno
quanto aos fatos narrados.
02. A narrao de uma histria sobre o convencional
tringulo amoroso permite ao autor investigar
camadas profundas da psique humana.
04. As personagens secundrias no so delineadas com
preciso e no h referncias atmosfera cultural e
do cotidiano da poca.
08. A proposta de atar as duas pontas da vida que
ocasiona a construo da casa do Engenho Novo,
copiada da casa da Rua de Matacavalos, tambm se
materializa na elaborao do prprio livro.
16. Com o retorno do filho Ezequiel, j adulto, Bentinho
reconhece a falta de fundamento para as suas
suspeitas e reconcilia-se com o passado.
32. o agregado Jos Dias quem, inadvertidamente,
revela a Bentinho a sua paixo por Capitu. dele
tambm a definio que melhor a caracteriza: olhos
de cigana oblqua e dissimulada.

31. (UFVIOSA)
Eternas, imortais origens vivas
da Luz, do Aroma, segredantes vozes
do mar e luares contemplativas
vagas vises volpicas, velozes.

Aladas alegrias sugestivas
de asa radiante e branda de albornozes,
tribos gloriosas, flgidas, altivas,
de condores e de guias e albatrozes.

Espiritualizai nos Astros louros,
do Sol entre os clares imorredouros
toda esta dor que minhalma clama...

Quero v-la subir, ficar cantando,
nas chamas das estrelas, dardejando
nas luminosas sensaes da chama.
(Cruz e Souza)
Some as alternativas que correspondem a
caractersticas interpretativas do poema:
01. Valorizao da sinestesia, acentuando a
correspondncia entre imagens acsticas, visuais e
olfativas.
02. Preferncia por uma luminosidade que torna os
elementos nebulosos e imprecisos.
04. Viso objetiva da realidade, em que a tcnica se
sobrepem imaginao.
08. Sublimao, atravs dos astros, de toda a dor que a
alma clama.
16. Predomnio da sugesto e uso de smbolos para a
representao do mundo.
32. (UFPR) Com relao esttica simbolista no Brasil,
podemos afirmar que:
01. Entre seus poetas, destacam-se Cruz e Souza,
Alphonsus de Guimaraens e o paranaense Emiliano
Perneta.
02. Em seus versos, os poetas transmitiram toda a
homologia existente entre a vida social e a literatura.
04. Os poetas enfatizaram a percepo intuitiva da
realidade em vez de busc-la de maneira lgica.
08. Entre os recursos utilizados pelos poetas para
aproximar a poesia da msica destaca-se o uso de
aliteraes.
16. Foi um movimento literrio que primou por versos
descritivos e pela perfeio formal.

33. (PUC-RS)
Esta de ureos relevos trabalhada
De divas mos, brilhante copo, um dia,
J de aos deuses servir como cansada,
Vinda do Olimpo, a um novo deus servia.

A poesia que se concentra na reproduo de
objetos decorativos, como exemplifica a estrofe
de Alberto de Oliveira, assinala a tnica da:
a) Espiritualizao da vida.
b) Viso do real.
c) Arte pela arte.
d) Moral das coisas.
e) Nota do intimismo.

34. (UFV-MG) Assinale a alternativa em que todas as
caractersticas de estilo so do Simbolismo.
a) Impassibilidade, vida descrita objetivamente,
ecletismo.
b) Hermetismo intencional, alquimia verbal,
musicalidade.
c) Favor da forma, expresses ousadas, fidelidade nas
observaes.
d) Atmosfera de impreciso, realismo cru, religiosidade
e) Complexidade, ressurreio dos valores humanos,
materialismo pornogrfico.

35. (FUVEST-SP)
I
Longe do estril turbilho da rua,
Beneditino, escreve! No aconchego
Do claustro, na pacincia e no sossego,
Trabalha, e teima, e sofre, e sua!
II
formas alvas, brancas, Formas claras
De luares, de neves, de neblinas!
formas vagas, fludas, cristalinas...
Incensos dos turbulos das aras...

As estrofes acima so, respectivamente, dos poetas:
a) Manuel Bandeira e Olavo Bilac.
b) Vincius de Moraes e Fagundes Varela.
c) Olavo Bilac e Cruz e Souza.
d) Cruz e Souza e Castro Alves.
e) Castro Alves e Alphonsus de Guimaraens.
Please purchase PDF Split-Merge on www.verypdf.com to remove this watermark.
Incluso para a Vida Literatura

Pr-Vestibular da UFSC
12
36. (PUC-SP) Cruz e Souza e Alphonsus de
Guimaraens so poetas identificados com um
movimento artstico cujas caractersticas so
a) o jogo de contrastes, o tema da fugacidade da vida e
fortes inverses sintticas.
b) a busca da transcendncia, a preponderncia do
smbolo entre as figuras e o cultivo de um
vocabulrio ligado s sensaes
c) a espontaneidade coloquial, os temas do cotidiano e o
verso livre.
d) o perfeccionismo formalista, a recuperao dos ideais
clssicos e o vocabulrio precioso.
e) o jogo dos sentimentos exacerbados, o alargamento da
subjetividade e a nfase na adjetivao.

37. Na figura de........, Monteiro Lobato criou o smbolo
do brasileiro abandonado ao seu atraso e misria pelos
poderes pblicos.
a) Cabeleira d) Blau Nunes
b) Jeca Tatu e) Augusto Matraga
c) Joo Miramar

38. A Obra pr modernista de Euclides da Cunha situa-
se entre a ...... e a ......:
a) Histria e Psicologia
b) Geografia e Economia
c) Literatura e Sociologia
d) Arte e Filosofia
e) Teologia e Geologia

39. Triste Fim de Policarpo Quaresma, de Lima
Barreto,
a) um livro de memrias em que a personagem ttulo
atravs de um artifcio narrativo conta as
atribulaes de sua vida at a hora da morte.
b) a histria de um visionrio e nacionalista fantico
que busca ingenuamente resolver sozinho os males
sociais de seu tempo.
c) uma autobiografia em que o autor sob a capa do
personagem ttulo expe sua insatisfao em relao
burocracia carioca.
d) o relato das aventuras de um nacionalista ingnuo e
fantico que lidera um grupo de oposio no incio
dos tempos republicanos.
e) o retrato da vida e morte de um humilde burocrata
conformado a contragosto com a realidade social de
seu tempo.

40. (CEFET-PR) Considerando a esttica realista,
assinale a alternativa correta.
a) Ligada a teorias marxistas pretendeu transformar a
realidade a partir da palavra literria.
b) Presa a teorias positivistas, distorce a realidade,
idealizando a ptria e a natureza
c) Embasada em ideias nacionalistas, buscou
revalorizar o passado colonial brasileiro.
d) Objetivando democratizar a literatura, optou por
retratar a cultura popular de modo idealizador.
e) Condicionada por ideias cientficas, tentou
aproximar cincia e fico.





41. (UFPR) Ea de Queirs afirmava:
O Realismo a anatomia do carter. a crtica do
homem. a arte que nos pinta a nossos prprios olhos
para nos conhecermos, para que saibamos se somos
verdadeiros ou falsos, para condenar o que houve de
mau em nossa sociedade.
Para realizar esta proposta literria, quais os recursos
utilizados no discurso realista?
01. Preocupao revolucionria, atitude de crtica e de
combate.
02. Imaginao criadora.
04. Personagens frutos da observao, tipos concretos e
vivos.
08. Linguagem natural, sem rebuscamento.
16. Preocupao com mensagem que revela concepo
materialista do homem.
32. Retorno ao passado.
64. Determinismo biolgico e social.

42. (UFPG) Em O Alienista, Simo Bacamarte verifica
que quatro quintos da populao de Itagua estavam
internados na Casa Verde. Diante disso, reformula a sua
teoria das molstias cerebrais, nos seguintes termos:
a) Devia-se admitir como normal e exemplar o
desequilbrio das faculdades mentais.
b) O equilbrio ininterrupto das faculdades mentais
seria prova de normalidade.
c) Os comportamentos desequilibrados seriam anormais
e, portanto, desaconselhveis.
d) Todos os homens so loucos.
e) Quatro quintos da humanidade mentalmente
doente.

43. (Vunesp-SP) Assinale a alternativa em que se
caracteriza a esttica simbolista.
a) Culto ao contraste, que ope elementos como amor e
sofrimento, vida e morte, razo e f, numa tentativa
de conciliar plos antagnicos.
b) Busca do equilbrio e da simplicidade dos modelos
greco-romanos atravs, sobretudo, de uma
linguagem simples, porm nobre.
c) Culto do sentimento nativista, que faz do homem
primitivo e sua civilizao um smbolo de
independncia espiritual, poltica, social e literria.
d) Explorao de ecos, assonncias, aliteraes, numa
tentativa de valorizar a sonoridade da linguagem,
aproximando-a da msica.
e) Preocupao com a perfeio formal, sobretudo com
o vocabulrio carregado de termos cientficos, o que
revela a objetividade do poeta.

44. (UCP PR) Assinale a alternativa correta.
a) O Romantismo consequncia do surto de
cientificismo e da fadiga da repetio das frmulas
subjetivas.
b) O poeta parnasiano deixa-se arrebatar pelo conflito
entre o mundo real e o imaginrio, expresso num
sentimentalismo acentuado.
c) O Realismo consequncia do surto de cientificismo
e da fadiga da repetio das frmulas subjetivas.
d) No Romantismo, o escritor mergulha no interior das
personagens, mostrando ao leitor seus dramas e sua
agonia.
e) No Simbolismo, predominou a prosa.
Please purchase PDF Split-Merge on www.verypdf.com to remove this watermark.
Incluso para a Vida Literatura

Pr-Vestibular da UFSC
13
45. (UM-SP) Assinale a(s) alternativa(s) que se
aplica(m) esttica parnasiana.
01. Constante presena da temtica da morte.
02. Tentativa de superar o sentimento romntico.
04. Predomnio da forma sobre o contedo.
08. Correta linguagem, fundamentada nos princpios dos
clssicos.
16. Predileo pelos gneros fixos, valorizando o soneto.

46. (UFRGS) Uma atitude comum caracteriza a postura
literria de autores modernistas a exemplo de Lima
Barreto, Graa Aranha, Monteiro Lobato e Euclides da
Cunha. Pode ser ela definida como
a) a necessidade de superar em termos de um programa
definido as estticas romnticas e socialistas
b) a pretenso de dar um carter definitivamente
brasileiro nossa literatura, que julgavam por demais
europeizada.
c) uma preocupao com o estudo e com a observao
da realidade brasileira.
d) a necessidade de fazer crtica social, j que o
realismo havia sido ineficiente nessa matria.
e) o aproveitamento esttico do que havia de melhor na
herana literria brasileira, desde suas primeiras
manifestaes.

47. (UFPR) Enfim chegou a hora da encomendao e
da partida. Sancha quis despedir-se do marido, e o
desespero daquele lance consternou a todos. Muitos
homens choravam tambm, as mulheres todas. S
Capitu, amparando a viva, parecia vencer-se a si
mesma. Consolava a outra, queria arranc-la dali. A
confuso era geral. No meio dela, Capitu olhou alguns
instantes para o cadver, to fixa, to apaixonadamente
fixa, que no admira lhe saltassem algumas lgrimas
poucas e caladas.
A partir do fragmento acima, de Dom Casmurro, de
Machado de Assis, assim como de informaes gerais
sobre esse romance, correto afirmar que:
01. A cena descrita tipicamente romntica, em
consonncia com o estilo da obra, que tematiza os
infelizes amores de Bentinho e Capitu.
02. Como os fatos posteriores comprovaro, entre
Escobar e Capitu, no houve qualquer
relacionamento alm de uma slida amizade.
04. O momento descrito crucial para o
relacionamento de Bentinho e Capitu. Instalada a
dvida na mente do marido, o casamento se
deteriorar, encaminhando-se para a inevitvel
separao.
08. O fragmento acima comprova que Sancha uma
personagem trgica, pois aps a morte dos filhos, ela
perde o marido num naufrgio.
16. Para Bentinho, a irrefutvel prova de traio de
Capitu, ser a semelhana de Ezequiel com o amigo
morto.
32. Conforme observamos pela linguagem do
narrador Dom Casmurro, a atitude de Capitu diante
do cadver testemunha apenas a amizade que ela
sentia por Escobar.



48. (SUPRA) Adultrio ainda crime o ttulo de um
artigo publicado na revista VEJA (Cartas), no dia
03/05/2000. Nele, encontramos informaes como: ...o
ex-ministro Jos Carlos Dias defendeu a tese de que
crimes como seduo e adultrio deveriam sair do
Cdigo Penal diretamente para a rea do comportamento
pessoal. Mais adiante, o mesmo artigo traz: adultrio
ainda crime - cuja pena varia de quinze dias a seis
meses de deteno, segundo o artigo 240 do Cdigo
Penal Brasileiro, em vigor desde 1940.
O comportamento social sempre foi tema de romances
na literatura de todos os pases. Destaca-se o tema
adultrio que, na literatura brasileira, apresentado
em obras de diversos autores.

Assinale a alternativa que apresenta, respectivamente,
obras cuja trama envolve o adultrio e o seu autor:
a) Lucola e Encarnao - Jos de Alencar.
b) Lucola e Senhora - Jos de Alencar.
c) Memrias de um Sargento de Milcias e A Viuvinha
- Manuel Antnio de Almeida.
d) Dom Casmurro e Memrias Pstumas de Brs Cubas
- Machado de Assis.
e) Dom Casmurro e Memorial de Aires - Machado de
Assis.

49. (SUPRA). Leia o texto da Revista VEJA, de
08/03/2000, e responda questo proposta.

CRIME PRECOCE
"s 10 horas de tera-feira passada, um garoto de 6
anos chegou escola Buell Elementary, em Flint, no
Estado americano do Michigan, com um revlver calibre
32 escondido nas calas. Na frente de cinco coleguinhas
de classe, sacou a arma e fez um disparo. O tiro acertou
Kayla Renee Rolland, tambm de 6 anos, no pescoo. A
menina morreu meia hora depois. Mesmo para os
Estados Unidos, um pas aturdido por um surto de
crimes violentos e gratuitos nas escolas, o incidente
surpreende pela pouca idade dos envolvidos. O menino
nem sequer pode ir a julgamento, pois a
responsabilidade penal no Michigan comea aos 7 anos.
A polcia ainda no tem explicao para o crime. Ao que
parece, as crianas tiveram uma briga no dia anterior.
Filho de pais separados, o menino vive com um tio
materno, traficante de crack. O pai est preso por porte
de cocana. Desde 1993, 39 crianas foram mortas em
escolas americanas."

possvel constatar, no texto dado acima, o ntimo
relacionamento das influncias do ambiente em que vive
o garoto com os seus atos. Essa relao foi explorada
nos romances da escola literria denominada
___________ que, no Brasil, tem como principal
representante _____________.

A alternativa que completa as lacunas, em sequncia, :
a) Parnasianismo - Raimundo Correia
b) Realismo - Machado de Assis
c) Naturalismo - Alusio Azevedo
d) Pr-Modernismo - Monteiro Lobato
e) Romantismo - Visconde de Tauna





Please purchase PDF Split-Merge on www.verypdf.com to remove this watermark.
Incluso para a Vida Literatura

Pr-Vestibular da UFSC
14
50. (ACAFE) Texto:
Inicia-te, enfim, Alma imprevista, entra no seio dos
viciados. Esperam-te de luz maravilhados, os Dons
que vo te consagrar artista.
(Cruz e Souza)

Assinale a afirmativa que no corresponde ao texto:
a) O verbo no imperativo, no primeiro verso, usado
para conclamar a alma imprevista, que pode ser
um candidato a artista, a uma ao.
b) Alma imprevista (o candidato a artista) ter que se
iniciar, isto , instruir-se na arte simbolista, para
poder entrar no seio dos iniciados, fazendo parte do
grupo simbolista.
c) Quando a alma imprevista instruir-se na arte
simbolista, receber os Dons da arte.
d) O uso do verbo consagrar remete-nos sacralizao
da arte, achando-se que o poeta um ser diferente,
excepcional, cuja obra s pode ser entendida por
iniciados.
e) Os dons de luz maravilhados so atribudos aos que
iniciam a escrever.

51. (SUPRA) H meses, um programa dirio da Rede
Bandeirantes manda seus reprteres s ruas, tentando
descobrir algum que cante, corretamente, o Hino
Nacional. Embora o prmio seja tentador (j passa dos
dez mil reais!), at agora ningum conseguiu cumprir a
tarefa. A expectativa grande, mas o prmio continua
sendo aumentado diariamente e poucas chances
existem de que ele seja entregue to cedo. Enquanto
isso, cidades procuram criar seus hinos, como
aconteceu recentemente em Balnerio Cambori e
agora em Blumenau, para comemorar os 150 anos de
fundao da cidade.
Hinos parecem fazer parte da vida das pessoas e
identificam clubes de futebol, cidades, estados e
pases. No Brasil, alm do Hino Nacional, temos
outros hinos, que fazem parte do cotidiano das pessoas
ou so lembrados apenas em ocasies especiais. Um
deles, o Hino Bandeira, tem como autor de sua letra
Olavo Bilac, poeta que se destacou tambm pela
defesa do servio militar obrigatrio, ocasio em que
fez conferncias por todo o pas. Na poesia, destacou-
se como poeta parnasiano, formando a trade
parnasiana com os poetas:

a) Castro Alves e Augusto dos Anjos.
b) Alberto de Oliveira e Manuel Bandeira.
c) Raimundo Correia e Alphonsus de Guimaraens.
d) Cruz e Sousa e Alphonsus de Guimaraens.
e) Raimundo Correia e Alberto de Oliveira.









AUTORES CARACTERSTICAS
M
O
D
E
R
N
I
S
M
O


1


F
A
S
E

1922
SEMANA DE ARTE MODERNA
9 Mrio de Andrade
(Papa do Modernismo)
9 Oswald de Andrade
9 Manuel Bandeira
linguagem coloquial, cotidiano, negao da tradio cultural
antipurista, antiacademicista, antiparnasianismo, verso livre,
nacionalismo crtico, ironia, sarcasmo, irreverncia, poema-
piada, liberdade de criao, direito pesquisa folclore

M
O
D
E
R
N
I
S
M
O


2


F
A
S
E

1930
9 Jorge Amado
9 Rachel de Queiroz
9 Graciliano Ramos
9 Jos Lins do Rego
9 Jos Amrico de Almeida
9 Erico Verissimo
9 Carlos Drummond de Andrade
9 Ceclia Meireles
9 Vincius de Moraes
REGIONALISMO DE 30
engajamento religioso e social literatura de denncia das
condies humanas; tendncia neorrealista; coloquialismo;
nacionalismo; cotidiano; romance psicolgico
Temas apresentados por regio: Coronelismo, Cacau na
Bahia, Cana-de-acar, Seca no Cear e em Alagoas,
Misticismo e candombl, Cangao, Os pampas gachos
POESIA
Ampliao da temtica: Social nacional para o Universal,
Poemas de tendncia mstica, Revalorizao de estilos
anteriores, neossimbolismo
M
O
D
E
R
N
I
S
M
O


3


F
A
S
E

1945
9 Joo Cabral de Melo Neto
9 Joo Guimares Rosa
9 Clarice Lispector
Concretismo:
9 Dcio Pignatari
9 Haroldo de Campos
9 Augusto de Campos
poesia
Retomada dos aspectos formais da poesia.
Culto forma e linguagem (gerao de 45 e poetas
concretistas).
Prosa
o domnio do conto e da crnica, anlise e observao do
cotidiano; as linhas do fantstico, literatura psicolgica /
intimista, tendncias a reflexes metafsicas.
No campo regional = investigao de novas possibilidades
lingusticas (universalidade); Importante o desenvolvimento
do teatro psicolgico e social.

Please purchase PDF Split-Merge on www.verypdf.com to remove this watermark.
Incluso para a Vida Literatura

Pr-Vestibular da UFSC
15
52. O Romantismo e o Modernismo representam, no
panorama literrio brasileiro, momentos de renovao e
intensa criatividade. Assinale a opo que aponta
problemtica trabalhada anteriormente pelos
romnticos e que foi retomada pelos modernistas.
a) lrreverncia.
b) "Arte pela arte".
c) Nacionalismo idealizado.
d) Viso ufanista-irnica.
e) Criao de uma identidade brasileira.

53. Assinale a opo que preenche correta e
adequadamente os espaos do excerto a seguir.
" com o (.......) que se inicia (.......) da arte: a literatura
passa a ser divulgado pela imprensa; escolhem-se
temas e contedos acessveis a todos; substitui (em)-se
(........) pelos mitos nacionais; valoriza-se a cultura
popular; e a literatura utilizada como arma de ao
poltica e social mediante romances (........) e poesia
(.......)."

(1) Modernismo
(2) Realismo
(3) Romantismo
(4) a idealizao
(5) a revoluo
(6) a democratizao
(7) os smbolos cristos
(8) a mitologia clssica
(9) o escapismo
(10) indianista(s)
(11) regionalista(s)
(12) de teses sociais e de costumes
(13) saudosista(s)
(14) reformista(s)
(15) revolucionri(o)(a)

a) 3 - 6 - 8 - 12 - 15.
b) 2 - 5 - 7 - 10 - 14.
c) 3 - 5 - 7 - 11 - 9.
d) 1 - 6 - 4 - 11 - 13.
e) 1 - 4 - 8 - 13 - 10.

54. Esta questo versa sobre aspectos gerais da
Literatura brasileira.
01. Como qualquer outra arte, a Literatura, inclusive no
Brasil, um fenmeno social. a expresso
artstica, criativa e original (por meio da escrita) da
vida e do relacionamento com outros homens, com
o meio em que vive e com sua poca.
02. No Brasil, o Romantismo representou a importao
das idias do luxo, da pompa da vida social
francesa.
04. Com a vinda da Famlia Real para o Brasil, em
1808, houve modificaes profundas no ambiente
cultural e social, recebendo mais diretamente as
influncias europias. E isto repercutiu no
movimento romntico brasileiro.
08. No Brasil do final do Sculo XIX, atravs dos
escritores realistas, o objetivo no era mais divulgar
os padres burgueses importados da Europa. Era
transformar a literatura em um "instrumento de
luta" contra as distores provocadas pela mudana
na forma de produo de riqueza que atingia uma
industrializao acelerada na Europa.
16. O Modernismo brasileiro, segundo Mrio de
Andrade (o seu mais importante lder), derivaria de
trs princpios fundamentais: o direito permanente
pesquisa esttica, a atualizao da inteligncia
artstica brasileira e a estabilizao de uma
conscincia criadora nacional.

55. Assinale a(s) proposio(es) verdadeira(s)
quanto aos princpios bsicos divulgados pelos
participantes da Semana da Arte Moderna:
01. Desejo de expresso livre e a tendncia para
transmitir, sem os embelezamentos tradicionais do
academismo, a emoo e a realidade do pas;
02. Rejeio dos padres portugueses, buscando uma
expresso mais coloquial, prxima do falar
brasileiro, associado ao vocabulrio dicionarizado
04. Combate a tudo que indicasse o "statu quo", o
conhecido;
08. Manuteno da temtica simbolista e parnasiana;
16 Valorizao do prosaico e do humor, que, em todas
as suas gamas, lavou e purificou a atmosfera
sobrecarregada pelos acadmicos.

56. Um escritor classificou "Vidas secas como
"romance desmontvel", tendo em vista sua
composio descontnua, feita de episdios
relativamente independentes e sequncias parcialmente
truncadas. Essas caractersticas da composio do livro
a) constituem um trao de estilo tpico dos romances de
Graciliano Ramos e do Regionalismo nordestino.
b) indicam que ele pertence fase inicial de Graciliano
Ramos, quando este ainda seguia os ditames do
primeiro momento do Modernismo.
c) diminuem o seu alcance expressivo, na medida em
que dificultam uma viso adequada da realidade
sertaneja.
d) revelam, nele, a influncia da prosa seca e lacnica
de Euclides da Cunha, em "Os sertes".
e) relacionam-se viso limitada e fragmentria que as
prprias personagens tm do mundo.

57. Assinale a(s) proposio(es) verdadeira(s) sobre
o romance SO BERNARDO, de Graciliano Ramos.
01. D. Glria, mulher egosta como o filho Paulo
Honrio, vinga-se de Padilha e de Joo Nogueira,
proibindo a veiculao de seus artigos nos jornais
locais.
02. Nessa obra, o foco narrativo modela sua verdadeira
fora, porque projeta o nvel de conscincia da
personagem em primeira pessoa, exemplificando no
trecho a seguir: "Uma tarde subi torre da igreja e
fui ver Marciano procurar corujas. (...) Eu desejava
assistir extino daquelas aves amaldioadas".
04. uma obra cujo enredo trata da canonizao de
So Bernardo, padre nordestino, morto pelos
jagunos de Lampio.
08. Aps a morte de Madalena, Paulo Honrio tenta
retornar o ritmo de sua vida, na fazenda So
Bernardo, mas a lembrana da mulher morta tira-
lhe todo o entusiasmo.
Please purchase PDF Split-Merge on www.verypdf.com to remove this watermark.
Incluso para a Vida Literatura

Pr-Vestibular da UFSC
16
16. Entre Paulo Honrio, personagem principal, e sua
mulher Madalena h, praticamente, uma
impossibilidade de comunicao.

58. Assinale a afirmao correta sobre o Romance de
30.
a) Predominou, entre os autores, uma preocupao de
renovao esttica seguindo os padres da vanguarda
literria europia.
b) Na obra de Jos Lins do Rego, predomina a
narrativa curta na recriao do modo de vida dos
senhores de engenho.
c) Os autores, em suas obras, tematizaram os
problemas sociais com o intuito de denunciar as
agruras das populaes menos favorecidas.
d) O carter regionalista dos romances deste perodo
deve-se reproduo fiel do linguajar tpico de cada
regio.
e) A obra de Jorge Amado pode ser considerada uma
exceo, no conjunto da poca, porque seus romances
apresentam uma grande inovao na estrutura
narrativa.

59. Texto
Chega!
Meus olhos brasileiros se fecham saudosos.
Minha boca procura a "Cano do Exlio".
Como era mesmo a "Cano do Exlio"?
Eu to esquecido de minha terra...
Ai terra que tem palmeiras
onde canta o sabi!
(Carlos Drummond de Andrade, "Europa, Frana e Bahia",
ALGUMA POESIA)

Neste excerto, a citao e a presena de
trechos.............. constituem um caso de..............

Os espaos pontilhados da frase acima devero ser
preenchidos, respectivamente, com o que est em:
a) do famoso poema de lvares de Azevedo / discurso
indireto.
b) da conhecida cano de Noel Rosa / pardia.
c) do clebre poema de Gonalves Dias/
intertextualidade.
d) da clebre composio de Villa-Lobos/ ironia.
e) do famoso poema de Mrio de Andrade /
metalinguagem.

60. A segunda fase do nosso Modernismo (1930-1945)
mostrou o amadurecimento de vrias linhas temticas e
de processos artsticos. Firmou-se neste perodo a
poesia de
01. Carlos Drummond de Andrade e Murilo Mendes.
02. Jos Lins do Rego e Vinicius de Moraes.
04. Augusto dos Anjos e Cruz e Sousa.
08. Ceclia Meireles e Jorge de Lima.
16. Emiliano Perneta e Cassiano Ricardo.

61. Texto

1 De repente do riso fez-se o pranto
2 Silencioso e branco como a bruma
3 E das bocas unidas fez-se a espuma
4 E das mos espalmadas fez-se espanto.

5 De repente da calma fez-se o vento
6 Que dos olhos desfez a ltima chama
7 E da paixo fez-se o pressentimento
8 E do momento imvel fez-se o drama.

Vincius de Moraes, nos versos transcritos, mostra
como, na separao amorosa, uma coisa se transforma
em outra. Em que verso(s) NO h registro de tal
transformao?
a) v.2 e 6.
b) v.1.
c) v. 5.
d) v.3 e v.4.
e) v.7 e 8.

62. Texto
"Quando eu nasci, um anjo torto
desses que vivem na sombra
disse: Vai, Carlos! ser gauche na vida"

Assinale a alternativa que identifica, corretamente,
autor e caracterstica dos versos citados.
a) Joo Cabral - versos brancos
b) Carlos Nejar - insero do indivduo na sociedade
c) Drummond de Andrade - desajustamento do
indivduo no mundo
d) Ferreira Gullar - questionamento sobre o sentido da
vida
e) Mrio Quintana - aluso ordem sobrenatural

63. A respeito da poesia de Vinicius de Moraes,
assinale verdadeira (V) ou falsa (F) em cada afirmao
a seguir.
(___) Sob a forma de soneto, privilegia temticas
lricas, sobretudo o amor e sua mltiplas
manifestaes.
(___) Celebra a sensualidade com versos que
apresentam imagens a respeito do sexo e do corpo.
(___) O cunho ertico dos poemas exclui a experincia
mstica do amor.

A sequncia correta :
a) V - F - F.
b) V - V - F.
c) F - V - F.
d) V - F - V.
e) F - F - V.

64. Sobre Carlos Drummond de Andrade, correto
afirmar que:
a) o nome inquestionvel na consolidao da poesia
moderna no Brasil, pela sntese que logrou realizar
entre vrias contradies histricas do pas.
b) Predomina, em seus poemas, uma linguagem
erudita, com termos raros, compatvel com os
tpicos existenciais mas inadequada para os temas
do cotidiano.
c) Publicou uma srie de livros de poesia e de crnicas
que tratam predominantemente de temas
provincianos, o que localiza sua obra na vertente
regionalista da literatura brasileira.
d) Se dedicou construo de uma linguagem objetiva
e rigorosa, voltada para a expresso literria da
paisagem tropical.
Please purchase PDF Split-Merge on www.verypdf.com to remove this watermark.
Incluso para a Vida Literatura

Pr-Vestibular da UFSC
17
e) Sua poesia se caracteriza pelo lirismo de tom
irnico e melanclico, registro de um inconformismo
em relao escassez de leitores de poesia no Brasil.

65. O mais importante para a Literatura no provar o
que acontece, mas mostrar uma realidade sob o ponto
de vista do autor. Focalizada sob os mais diversos
ngulos, a MULHER inspira poetas e artistas:

01. "Sois anjo que me tenta, e no me guarda". - Verso
de Gregrio de Matos, delineando, pelo paradoxo, a
figura da mulher como apelo aos sentidos.
02. "Eu vi uma mulher verdadeiramente bela. Seus
cabelos so negros e luzidios como azeviche (...)
Sua fronte branca, elevada e lisa..." - Fragmento do
escritor romntico Manuel Antnio de Almeida,
traando um perfil psicolgico profundo do
personagem feminino, sem a idealizao prpria da
Escola Romntica.
04. "E estas, sem se desunharem, tinham j mordeduras
por todo o busto." - Fragmento de Alusio Azevedo,
mostrando que as personagens so de condio
inferior, vistas na sua condio animal.
08. "Porque a Beleza, gmea da verdade, / Arte pura,
inimiga do artifcio, / a fora e a graa na
simplicidade." - Versos de Olavo Bilac, contando
que mulher, mesmo sendo bela, compete viver um
vida que no escolheu e cujo destino lhe escapa.
16. "Como a senhora, comadre, / pode manter o seu
lar." - versos de Joo Cabral de Melo Neto,
voltando-se para a condio social da mulher
nordestina.

66. Assinale a proposio incorreta.
a) Monteiro Lobato faz retratos pitorescos, fortes e
irnicos do homem do campo.
b) Mrio de Andrade no foi alm dos limites
paulistanos.
c) Oswald de Andrade, poeta e prosador, quis destruir
para construir.
d) Vincius de Moraes escreveu desde o soneto
camoneano at letras de canes de sabor popular.
e) Clarice Lispector vai alm do regionalismo, alm do
realismo do mundo fsico.

67. Joo Guimares Rosa, em Sagarana, permite ao
leitor observar que:
a) Explora o folclrico do serto.
b) Em episdios muitas vezes palpitantes surpreende a
realidade nos mais leves pormenores e trabalha a
linguagem com esmero.
c) Limita-se ao quadro do regionalismo brasileiro.
d) muito sutil na apresentao do cotidiano banal do
jaguno.
e) intimista hermtico.

68. A respeito de Clarice Lispector, nos contos de
Laos de Famlia, seria correto afirmar que:
a) Parte frequentemente de acontecimentos
surpreendentes para banaliz-los.
b) Elabora o cotidiano em busca de seu significado
oculto.
c) altamente intimista, vasculhando o mago das
personagens com rara argcia.
d) regionalista hermtica.
e) Opera na rea da memria, da autoanlise e do
devaneio.

69. Em MORTE E VIDA SEVERINA, podem-se
reconhecer as seguintes caractersticas da poesia de
Joo Cabral de Melo Neto:
a) Stira aos coronis do Nordeste e versos inflamados.
b) Experimentalismo concretista e temtica urbana.
c) Memorialismo nostlgico e estilo oral.
d) Personagens da seca e linguagem disciplinada.
e) Descrio de paisagens e intenso subjetivismo.






Please purchase PDF Split-Merge on www.verypdf.com to remove this watermark.
Incluso para a vida Matemtica A

Pr-Vestibular da UFSC
1
UNIDADE 1

ARITMTICA BSICA

MLTIPLO DE UM NMERO
Sendo a, b e c nmeros naturais e a . b = c, diz-se que c
mltiplo de a e b.

Exemplo: Mltiplos de 3
M(3) = {0, 3, 6, 9, ....}

Observaes:
- O zero mltiplo de todos os nmeros.
- Todo nmero mltiplo de si mesmo.
- Os nmeros da forma 2k, k e N, so nmeros
mltiplos de 2 e esses so chamados nmeros pares.
- Os nmeros da forma 2k + 1, k e N, so nmeros
mpares.

DIVISOR DE UM NMERO
Sendo a, b e c nmeros naturais e a . b = c, diz-se que a e
b so divisores c.

Exemplo: Divisores de 12
D(12) = {1, 2, 3, 4, 6, 12}

Observaes:
- O menor divisor de um nmero 1.
- O maior divisor de um nmero ele prprio.

Quantidade de divisores de um nmero
Para determinar a quantidade de divisores de um nmero
procede-se assim:

a) Decompem-se em fatores primos o nmero
dado;
b) Toma-se os expoentes de cada um dos fatores e a
cada um desses expoentes adiciona-se uma
unidade.
c) Multiplica-se os resultados assim obtidos.

Exemplo: Determinar o nmero de divisores de 90
90 = 2
1
. 3
2
. 5
1


(1 + 1).(2+1).(1 +1) = 2.3.2 = 12

Logo, 90 possui 12 divisores

CRITRIOS DE DIVISIBILIDADE

Divisibilidade por 2
Um nmero divisvel por 2 se for par.
Exemplos: 28, 402, 5128.

Divisibilidade por 3
Um nmero divisvel por 3 se a soma dos valores
absolutos dos seus algarismos for divisvel por 3.
Exemplos: 18, 243, 3126.

Divisibilidade por 4
Um nmero divisvel por 4 se os dois ltimos algarismos
forem divisveis por 4 ou quando o nmero terminar em
00.
Exemplos: 5716, 8700, 198200.


Divisibilidade por 5
Um nmero divisvel por 5 se o ltimo algarismo for 0
ou 5.
Exemplos: 235, 4670, 87210.

Divisibilidade por 6
Um nmero divisvel por 6 se for simultaneamente
divisvel por 2 e 3.
Exemplos: 24, 288, 8460.

Divisibilidade por 7
Processo prtico: Veja o nmero 4137

1 Passo: separa-se o ltimo algarismo e dobra-se o seu
valor.
4137 7 2 x 7 = 14

2 Passo: subtrai-se o nmero assim obtido do nmero que
restou aps a separao do ltimo algarismo.
413 14 = 399

3 Passo: procede-se assim at se obter um nmero
mltiplo de 7.

399 9 2 x 9 = 18

39 18 = 21

21 1 2 x 1 = 2

2 2 = 0

Logo 4137 mltiplo de 7

Divisibilidade por 8
Um nmero divisvel por 8 se os trs ltimos algarismos
forem divisveis por 8 ou forem trs zeros.
Exemplos: 15320, 67000.

Divisibilidade por 9
Um nmero divisvel por 9 quando a soma dos seus
algarismos for um nmero divisvel por 9.
Exemplos: 8316, 35289.

Divisibilidade por 10
Um nmero divisvel por 10 se o ltimo algarismo for
zero.
Exemplos: 5480, 1200, 345160.

NMEROS PRIMOS
Um nmero p, p = 0 e p = 1, denominado nmero primo
se apresentar apenas dois divisores, 1 e p.

Exemplos: 2, 3, 5, 7, 11, 13,.....

Observao: Um nmero denominado composto se no
for primo.

MNIMO MLTIPLO COMUM
Denomina-se menor ou mnimo mltiplo comum (M.M.C)
de dois ou mais nmeros o nmero p diferente de zero, tal
que p seja o menor nmero divisvel pelos nmeros em
questo.

Exemplo: Determinar o M.M.C entre 6 e 8.

Processo 1: M(6) = {6, 12, 18, 24, 30, 36, ....}
M(8) = {8, 16, 24, 32, 40, 48, ...}
Logo o M.M.C. entre 6 e 8 24
Please purchase PDF Split-Merge on www.verypdf.com to remove this watermark.
Matemtica A Incluso para a Vida

Pr-Vestibular da UFSC
2
Processo 2:
6 8
3 4
3 2
3 1
1 1
2
2
2
3


Logo o M.M.C. entre 6 e 8 2
3
.3 = 24

MXIMO DIVISOR COMUM
Denomina-se mximo divisor comum (M.D.C) de dois ou
mais nmeros o maior dos seus divisores comuns.

Exemplo: Determinar o M.D.C. entre 36 e 42

Processo 1: D(36) = {1, 2, 3, 4, 6, 9, 12, 18, 36}
D(42) = {1, 2, 3, 6, 7, 21, 42}

Logo, o M.D.C. entre 36 e 42 6.

Processo 2: 36 = 2
2
.3
2
e 42 = 2.3.7
Os fatores comuns entre 36 e 42 so 2.3
Logo, o M.D.C. entre 36 e 42 6.

Exerccios de Sala #

1. (UFSC) Um pas lanou em 02/05/2000 os satlites
artificiais A, B e C com as tarefas de fiscalizar o
desmatamento em reas de preservao, as nascentes dos
rios e a pesca predatria no Oceano Atlntico. No dia
03/05/2000 podia-se observ-los alinhados, cada um em
uma rbita circular diferente, tendo a Terra como centro.
Se os satlites A, B e C levam, respectivamente, 6, 10 e 9
dias para darem uma volta completa em torno da Terra,
ento o nmero de dias para o prximo alinhamento :

2. Sejam x e y o m.d.c e o m.m.c de 12 e 20,
respectivamente. O valor de x. y :
a) 240 c) 100 e) 230
b) 120 d) 340

3. O nmero de divisores naturais de 72 :
a) 10 c) 12 e) 14
b) 11 d) 13

Tarefa Mnima #

4. Considere os nmeros A = 24, B = 60; C = 48.
Determine:
a) M.M.C entre A e B
b) M.D.C entre B e C
c) M.M.C entre A, B e C
d) M.D.C entre A, B e C

5. Sejam x e y o m.d.c e o m.m.c de 20 e 36,
respectivamente. O valor de x. y :
a) 240 c) 120 e) 230
b) 720 d) 340

6. Determine o nmero de divisores naturais dos nmeros
a) 80 b) 120

7. Um ciclista d uma volta em uma pista de corrida em 16
segundos e outro ciclista em 20 segundos. Se os dois
ciclistas partirem juntos, aps quanto tempo iro se
encontrar de novo no ponto de partida, levando em
considerao ambas as velocidades constantes?

8. Trs vizinhos tm por medidas de frente: 180m, 252m e
324m, respectivamente, e mesmas medidas para os fundos.
Queremos dividi-los em faixas que tenham me didas iguais
de frente e cujo tamanho seja o maior possvel. Ento cada
faixa medir na frente:
a) 12 m c) 24 m e) 36 m
b) 18 m d) 30 m

Tarefa Complementar#

9. Um alarme soa a cada 10 horas, um segundo alarme a
cada 8 horas, um terceiro a cada 9 horas e um quarto a
cada 5 horas. Soando em determinado instante os quatro
alarmes, depois de quanto tempo voltaro a soar juntos?
a) 240 horas c) 32 horas e) 320 horas
b) 120 horas d) 360 horas

10. Trs tbuas medindo respectivamente 24cm, 84cm e
90 cm sero cortadas em pedaos iguais, obtendo assim
tbuas do maior tamanho possvel. Ento cada tbua
medir:
a) 10 cm c) 8 cm e) 4 cm
b) 6 cm d) 12 cm

11. Sejam os nmeros
A = 2
3
.3
2
. 5 B = 2
2
. 3 . 5
2


Ento o M.M.C e o M.D.C entre A e B valem
respectivamente:
a) 180 e 60 d) 1800 e 60
b) 180 e 600 e) n.d.a.
c) 1800 e 600

12. (Santa Casa-SP) Seja o nmero 717171x, onde x
indica o algarismo das unidades. Sabendo que esse nmero
divisvel por 4, ento o valor mximo que x pode
assumir :
a) 0 c) 4 e) 8
b) 2 d) 6

13. (PUC-SP) Qual dos nmeros abaixo primo?
a) 121 c) 362 e) n.d.a.
b) 401 d) 201

14. (PUC-SP) Um lojista dispe de trs peas de um
mesmo tecido, cujos comprimentos so 48m, 60m e 80m.
Nas trs peas o tecido tem a mesma largura. Deseja
vender o tecido em retalhos iguais, cada um tendo a
largura das peas e o maior comprimento possvel, de
modo a utilizar todo o tecido das peas. Quantos retalhos
ele dever obter?

15. (UEL-PR) Seja p um nmero primo maior que 2.
verdade que o nmero p
2
1 divisvel por:
a) 3 c) 5 e) 7
b) 4 d) 6

16. Sejam A e B o mximo divisor comum (M.D.C) e o
mnimo mltiplo comum de 360 e 300, respectivamente.
O produto A.B dado por: 2
x
.3
y
.5
z
, ento x + y + z vale:

Please purchase PDF Split-Merge on www.verypdf.com to remove this watermark.
Incluso para a vida Matemtica A

Pr-Vestibular da UFSC
3
17. (Fuvest-SP) O menor nmero natural n, diferente de
zero, que torna o produto de 3 888 por n um cubo perfeito
:
a) 6 c) 15 e) 24
b) 12 d) 18

18. (ACAFE) Um carpinteiro quer dividir em partes iguais
trs vigas, cujos comprimentos so, respectivamente, 3m,
42dm, 0,0054 km, devendo a medida de cada um dos
pedaos ser a maior possvel. O total de pedaos obtidos
com as trs vigas :
a) 18 c) 210 e) 20
b) 21 d) 180

UNIDADE 2

CONJUNTOS NUMRICOS

CONJUNTOS NUMRICOS

Conjunto dos Nmeros Naturais

N = { 0, 1, 2, 3, 4, 5, ... }

Um subconjunto importante dos naturais (N) o conjunto
N
*
( naturais sem o zero )
N
*
= { 1, 2, 3, 4, 5, ... }

a, b e N, (a + b) e N e (a . b) e N


Conjunto dos Nmeros Inteiros
Os nmeros inteiros surgiram com a necessidade de
calcular a diferena entre dois nmeros naturais, em que o
primeiro fosse menor que o segundo.

Z = { ... -3, -2, -1, 0, 1, 2, 3, ... }

Podemos citar alguns subconjuntos dos inteiros
Z
*
= inteiros no nulos... { ... -3, -2, -1, 1, 2, 3, ... }
Z
+
= inteiros no negativos... { 0, 1, 2, 3, ... }
Z
*
+
= inteiros positivos... { 1, 2, 3, 4, ... }
Z
_
= inteiros no positivos... { ..., -3, -2, -1, 0}
Z
*
_
= inteiros negativos... { ... -3, -2, -1 }

a, b e Z, (a + b) e Z, (a . b) e Z e (a b) e Z


Conjunto dos Nmeros Racionais
Os nmeros Racionais surgiram com a necessidade de
dividir dois nmeros inteiros, onde o resultado era um
nmero no inteiro.

Q = { x | x =
a
b
, com a e Z, b e Z
*
}

Ou seja, todo nmero que pode ser colocado em forma de
frao um nmero racional.
So exemplos de nmeros racionais:
a) Naturais
b) Inteiros
c) decimais exatos ( 0,2 =
2
10
)
d) dzimas peridicas ( 0,333... =
1
3
)
As quatro operaes so definidas nos racionais.
Com a ressalva que a diviso por zero impossvel (exceto
quando o numerador for zero tambm).

Geratrizes de uma dzima peridica
Toda frao que d origem a uma dzima peridica se
chama GERATRIZ. Para determinarmos a GERATRIZ de
uma dzima peridica, procedemos assim:

a) Dzima Peridica Simples: um nmero fracionrio
cujo numerador o algarismo que representa a parte
peridica e o denominador um nmero formado por
tantos noves quantos forem os algarismos do perodo.

Exemplos:

a) 0777...=
9
7

b) 0,333....=
3
1
9
3
=
c) 0,434343... =
99
43


b) Dzima Peridica Composta: um nmero fracionrio
cujo numerador a diferena entre a parte no peridica
seguida de um perodo e a parte no peridica, e cujo o
denominador um nmero formado de tantos noves
quantos so os algarismos do perodo, seguido de tantos
zeros quantos so os algarismos da parte no peridica.

Exemplos:
a) 0,3777... =
45
17
90
34
90
3 37
= =



b) 0,32515151... =
3300
1073
9900
3219
9900
32 3251
= =



Conjunto dos Nmeros Irracionais
Apesar de que entre dois nmeros racionais existir sempre
um outro racional, isso no significa que os racionais
preencham toda reta. Veja o seguinte exemplo.
Dado o tringulo retngulo abaixo de catetos 1 e 1.
Calcular o valor da hipotenusa.

x
1

1

Aplicando o teorema de Pitgoras temos:
x
2
= 1
2
+ 1
2

x = 2

Extraindo a raiz de 2, teremos um nmero que no
natural, inteiro, nem racional, surge ento os nmeros
irracionais.
Please purchase PDF Split-Merge on www.verypdf.com to remove this watermark.
Matemtica A Incluso para a Vida

Pr-Vestibular da UFSC
4
Os nmeros irracionais so aqueles que no podem ser
colocados em forma de frao, como por exemplo:
a) t = 3,14...
b) e = 2, 71...
c) toda raiz no exata

Conjunto dos Nmeros Reais
Os nmeros reais surgem da unio dos nmeros racionais
com os irracionais.

QUADRO DE RESUMO
9

Q I
Z
N



Por enquanto, nosso conjunto universo ser o campo dos
reais. Porm, necessrio saber que existem nmeros que
no so reais, estes so chamados de complexos e sero
estudados mais detalhadamente adiante.

PROPRIEDADES EM 9

- Comutativa: a + b = b + a e a . b = b . a
- Associativa: (a + b) + c = a + (b + c) e (a.b).c = a.(b.c)
- Elemento neutro: a + 0 = a e a . 1 = a
- Simtrico: a + ( a) = 0
- Inverso: a .
a
1
= 1, a = 0

INTERVALOS NUMRICOS E MDULO DE
UM NMERO REAL

INTERVALOS NUMRICOS
Chamamos intervalo qualquer subconjunto contnuo de 9.
Sero caracterizados por desigualdades, conforme veremos
a seguir:
- {x e R| p s x s q} = [p, q]
- {x e R| p < x < q} = ]p, q[
- {x e R| p s x < q} = [p, q[
- {x e R| p < x s q} = ]p, q]
- {x e R| x > q} = [q, [
- {x e R| x > q} = ]q, [
- {x e R| x s q} = ] -, q]
- {x e R| x < q} = ] -, q[

Os nmeros reais p e q so denominados, respectivamente,
extremo inferior e extremo superior do intervalo.

Observaes
- O intervalo [x, x] representa um conjunto unitrio {x}
- O intervalo ]x, x[ representa um conjunto vazio { }
- O intervalo ( , + ) representa o conjunto dos
nmeros reais (R)
- (x, y) = ]x, y[
Pode-se representar um intervalo real de 3 maneiras:

Notao de conjunto. Exemplo: {x e R| 2 < x s 3}

Notao de intervalo. Exemplo: ]2, 3]

Representao Grfica.
Exemplo:

Veja outros exemplos:
1) {x e R| x > 2} = ]2, [



2) {x e R| x s 1} = ] -, 1]



3) {x e R| 3 s x < 4} = [3, 4[




MDULO DE UM NMERO REAL
Mdulo ou valor absoluto de um nmero real x a
distncia da origem ao ponto que representa o nmero x.
Indicamos o mdulo de x por | x |.

Definio

<
>
=
0 x se x, -
0 x se , x
x


Exemplos:
a) como 3 > 0, ento | 3 | = 3

b) como 3 < 0, ento |3| = (3) = 3

Propriedades
- | x | > 0
- | x |
2
= x
2

- | |
2
x x =
- |x y| = |y x|
- |x . y| = | x |. | y |
-
y
x
y
x
=

Equao Modular

Equao Modular a equao que possui a incgnita x
em mdulo.
Tipos de equaes modulares:

Exemplo 1: | x | = 3
x = 3 ou x = -3
S = {-3, 3}

Exemplo 2: Resolva a equao |x + 2|= 6
x + 2 = 6 ou x + 2= - 6
- | x | = k, com k > 0, ento: x = k ou x = k
Please purchase PDF Split-Merge on www.verypdf.com to remove this watermark.
Incluso para a vida Matemtica A

Pr-Vestibular da UFSC
5
x = 4 ou x = - 8
S = {-8, 4}


Exemplo 1: | x | = - 3
S = C

Exemplo 2: |x + 2| = -10
S = C

Inequao Modular

Sendo k > 0, as expresses do tipo | x | < k, | x | s k,
| x | > k, | x | > k denominam-se inequaes modulares.

Tipos de inequaes modulares:
-

Exemplos: | x | < 3 3 < x < 3
| x | < 10 10 < x < 10

-

Exemplos: | x | > 3 x < 3 ou x > 3
| x | > 10 x < 10 ou x > 10

Exerccios de Sala #

1. Calcule o valor das expresses abaixo:
a)
|
.
|

\
|
+ |
.
|

\
|

3
1
5
2
8
1
4
3


b)
|
.
|

\
|
+ |
.
|

\
|

3
4
1 :
5
3
2


2. (PUC-SP) Considere as seguintes equaes:
I - x
2
+ 4 = 0
II - x
2
4 = 0
III - 0,3x = 0,1

Sobre as solues dessas equaes verdade afirmar que:
a) II so nmeros irracionais.
b) III um nmero irracional.
c) I e II so nmeros reais.
d) I e III so nmeros no reais.
e) II e III so nmeros racionais.

3. Resolva em 9 as seguintes equaes:

a) | x | = 3 d) |x + 2| = 3

b) |2x 1| = 7 e) |x|
2
5|x| + 4 = 0

c) |x
2
5x | = 6
Tarefa Mnima #

4. Enumere os elementos dos conjuntos a seguir:
a) {x e N| x divisor de 12}
b) {x e N| x mltiplo de 3}
c) {x e N| 2 < x s 7}
d) {x e Z| - 1 s x < 3}
e) {x| x = 2k, k e N}
f) {x| x = 2k + 1, k e N}

5. As geratrizes das dzimas: 0,232323... e 0,2171717...
so respectivamente:

23 23 20 43 23 43
a) e b) e c) e
100 99 99 99 99 198
1 1 2 1
d) e e) e
3 10 10 5


6. (ACAFE) O valor da expresso ,
1
2
.

c
c b a
quando
a = 0,333...; b = 0,5 e c = - 2 igual a:

7. Resolva em 9 as seguintes equaes:

a) |x| = 10 c) |x 2| = -3

b) |x + 1| = 7 d) ,x

,
2
+ 3 ,x, - 4 = 0 :

8. A soluo da inequao
5 ) 1 2 (
2
s x

a) {x e 9| 2 s x s 3}
b) {x e 9| 1 s x s 6}
c) {x e 9| x s 3}
d) {x e 9| x s 7}
e) {x e 9| 3 s x s 2}

Tarefa Complementar #

9. (FATEC-SP) Se a = 0,666..., b = 1,333... e
c = 0,1414..., ento a.b
-1
+ c igual a:

10. (FGV-SP) Quaisquer que sejam o racional x e o
irracional y, pode-se dizer que:
a) x.y racional.
b) y.y irracional.
c) x + y racional.
d) x - y + 2 irracional.
e) x + 2y irracional.

11. (FUVEST) Na figura esto representados
geometricamente os nmeros reais 0, x, y e 1. Qual a
posio do nmero xy?

a) esquerda de 0 d) entre y e 1
b) entre zero e x e) direita de 1
c) entre x e y

12. Determine a soma dos nmeros associados s
proposies corretas:

01. possvel encontrar dois nmeros naturais, ambos
divisveis por 7 e tais que a diviso de um pelo outro
deixe resto 39.
02. Sejam a e b nmeros naturais. Sendo a = 1 + b
2
com b
sendo um nmero mpar, ento a par.
04. O nmero 2 5 7 + real.
- | x | = k, com k = 0, ento: x = 0
- | x | = k, com k < 0, ento: no h soluo
| x | < k, com k > 0, ento: k < x < k
| x | > k, com k > 0, ento: x < k ou x > k
Please purchase PDF Split-Merge on www.verypdf.com to remove this watermark.
Matemtica A Incluso para a Vida

Pr-Vestibular da UFSC
6
08. Existem 4 nmeros inteiros positivos e consecutivos
tais que o produto de 2 deles seja igual ao produto dos
outros dois.
16. o nmero 247 um nmero primo.

13. A expresso|2x 1| para x <
2
1
equivalente a:
a) 2x 1 d) 1 + 2x
b) 1 2x e) 1
c) 2x + 1

14. Assinale a alternativa correta:
a) Se x um nmero real, ento
2
x = |x |
b) Se x um nmero real, ento existe x, tal que
|x| < 0
c) Sejam a e b dois nmeros reais com sinais iguais,
ento |a + b| = |a| + |b|
d) Sejam a e b dois nmeros reais com sinais opostos,
ento |a + b| > |a| + |b|
e) | x | = x, para todo x real.
15. (UFGO) Os zeros da funo f(x) =
2 1
5
3
x
so:
a) 7 e 8 c) 7 e 8 e) n.d.a.
b) 7 e 8 d) 7 e 8

16. (FGV-SP) Qual dos seguintes conjuntos est contida
no conjunto soluo da inequao
1 ) 1 (
2
s + x
?
a) {x e R , - 5 s x s - 1}
b) {x e R , - 4 s x s 0}
c) {x e R , - 3 s x s 0}
d) {x e R , - 2 s x s 0}
e) Todos os conjuntos anteriores

17. (ITA-SP) Os valores de x e R para os quais a funo
real dada por f(x) =
| 6 | 1 2 || 5 x
est definida,
formam o conjunto:
a) [0, 1] d) (-, 0] [1, 6]
b) [-5, 6] e) [-5, 0] [1, 6]
c) [-5,0] [1, )

UNIDADE 3

EQUAES DO 1 GRAU INEQUAES

DEFINIO
Uma sentena numrica aberta dita equao do 1 grau
quando pode ser reduzida ao tipo ax + b = 0, com a
diferente de zero.

RESOLUO
Considere, como exemplo, a equao 2x + 1 = 9.
Nela o nmero 4 soluo, pois 2.4 + 1 = 9. O
nmero 4 nesse caso denominado RAIZ da equao
Duas equaes que tm o mesmo conjunto soluo
so chamadas equivalentes.


PRINCPIO ADITIVO E MULTIPLICATIVO DA
IGUALDADE

Se: a = b ento para m a + m = b + m
Se: a = b ento para m = 0 a . m = b . m


INEQUAES DO 1 GRAU
Inequaes so expresses abertas que exprimem uma
desigualdade entre as quantidades dadas.

Uma inequao dita do 1 grau quando pode ser escrita
na forma:
ax + b > 0 ax + b < 0
ax + b > 0 ax + b s 0

Nas inequaes do 1 grau valem tambm, os princpio
aditivo e multiplicativo com uma ressalva. Veja:

Se: a > b ento para m a + m > b + m
Se: a > b ento para m > 0 a . m > b . m
Se: a > b ento para m < 0 a . m < b . m

Exerccios de Sala #

1. Resolva em R as seguintes equaes e inequaes:

a) ax + b = 0, com a = 0

b) 4(x + 3) + 5 = 2(x + 7)

c)
10
4
3 2
3
1
=

+
+ x x

d) 502x = 500x

e) 0.x = 0

f) 0.x = 5

g)
8
3x
2
1 x
>



2. Obtenha m de modo que o nmero 6 seja raiz da
equao 5x + 2m = 20

3. Resolva em R, o seguinte sistema:

= +
=
2 3 2
1 3
y x
y x


Tarefa Mnima#

4. Resolver em R as equaes:
a) 6x 6 = 2(2x + 1)
b) 2(x + 1) = 5x + 3
c) (x + 1)(x + 2) = (x + 3)(x + 4) 3
d) 2(x 2) = 2x 4
e) 3(x 2) = 3x
f)
4
1
3 2
1
= +
x x

5. A soluo da equao
x
2
1 x
3
x
=

+
:

a) x = 2 c) x = 3 e) x = 1
b) x = 3 d) x = 2
Please purchase PDF Split-Merge on www.verypdf.com to remove this watermark.
Incluso para a vida Matemtica A

Pr-Vestibular da UFSC
7
6. (FGVSP) A raiz da equao
1
4
1 2x
3
1 x
=
+

:
a) Um nmero maior que 5.
b) Um nmero menor que 11.
c) Um nmero natural.
d) Um nmero irracional.
e) Um nmero real.

7. Determine a soluo de cada sistema abaixo:
a)

= +
=
3
3 2
y x
y x
b)

=
= +
1
5
y x
y x
c)

= +
= +
1 2 2
1 3
y x
y x


8. Resolva em R as inequaes:
a) 3(x + 1) > 2(x 2) c)
4
1
2
x
3
1
<
b)
2
3x
4
10 x
s
+

Tarefa Complementar#

9. O valor de x + y em

=
= +
1 4y 7x
21 3y 2x
:

10. Obtenha o maior de trs nmeros inteiros e
consecutivos, cuja soma o dobro do menor.

11. (UFSC) A soma dos quadrados dos extremos do
intervalo que satisfaz simultaneamente, as inequaes: x +
3 > 2 e 2x - 1 s 17; :

12. As tarifas cobradas por duas agncias de locadora de
automveis, para veculos idnticos, so:
- Agncia AGENOR: R$ 90,00 por dia, mais R$ 0,60
por quilmetro rodado.
- Agncia TEFILO: R$ 80,00 por dia, mais R$ 0,70
por quilmetro rodado.
Seja x o nmero de quilmetros percorridos durante um
dia. Determine o intervalo de variao de x de modo que
seja mais vantajosa a locao de um automvel na agncia
AGENOR do que na agncia TEFILO.

13. (UFSC) A soma dos dgitos do nmero inteiro m tal
que 5 m + 24 > 5500 e
5
8
m + 700 > 42 m, :
14. (UFSC) Para produzir um objeto, um arteso gasta R$
1,20 por unidade. Alm disso, ele tem uma despesa fixa de
123,50, independente da quantidade de objetos produzidos.
O preo de venda de R$ 2,50 por unidade. O nmero
mnimo de objetos que o arteso deve vender, para que
recupere o capital empregado na produo dos mesmos, :

15. (UFSC) A soma das idades de um pai e seu filho 38
anos. Daqui a 7 anos o pai ter o triplo da idade do
filho. A idade do pai ser:

16. (UFSC) Na partida final de um campeonato de
basquete, a equipe campe venceu o jogo com uma
diferena de 8 pontos. Quantos pontos assinalou a equipe
vencedora, sabendo que os pontos assinalados pelas duas
equipes esto na razo de 23 para 21?

17. (UNICAMP) Uma senhora comprou uma caixa de
bombons para seus dois filhos. Um deles tirou para si
metade dos bombons da caixa. Mais tarde, o outro
menino tambm tirou para si metade dos bombons que
encontrou na caixa. Restaram 10 bombons. Calcule
quantos bombons havia inicialmente na caixa.

18. (UEL-PR) Um trem, ao iniciar uma viagem, tinha em
um de seus vages um certo nmero de passageiros. Na
primeira parada no subiu ningum e desceram desse
vago 12 homens e 5 mulheres restando nele um nmero
de mulheres igual ao dobro do de homens. Na segunda
parada no desceu ningum, entretanto subiram, nesse
vago, 18 homens e 2 mulheres, ficando o nmero de
homens igual ao de mulheres. Qual o total de passageiros
no vago no incio da viagem?

UNIDADE 4

EQUAES DO 2 GRAU

Denomina-se equao do 2 grau a toda equao que pode
ser reduzida a forma:

ax
2
+ bx + c = 0 onde a, b e c so nmeros reais e a = 0.
RESOLUO

1 CASO: Se na equao ax
2
+ bx + c = 0, o coeficiente b
for igual a zero procede-se assim:

ax
2
+ c = 0
ax
2
= c
x
2
=
a
c

x =
a
c

S =
)
`


a
c
a
c
,

2 CASO: Se na equao ax
2
+ bx + c = 0, o coeficiente c
for igual a zero procede-se assim:

ax
2
+ bx = 0
x(ax + b) = 0
x = 0 ou ax + b = 0
S = {0,
a
b
}
3 CASO: Se na equao ax
2
+ bx + c = 0, a, b, c = 0
aplica-se a frmula de Bhskara

x =
2a
b
onde: A= b
2
4ac

Nessa frmula, A= b
2
4ac o discriminante da
equao, o que determina o nmero de solues
reais da equao. Pode-se ter as seguintes situaes:
Please purchase PDF Split-Merge on www.verypdf.com to remove this watermark.
Matemtica A Incluso para a Vida

Pr-Vestibular da UFSC
8

- A > 0. Existem duas razes reais e distintas
- A = 0. Existem duas razes reais e iguais
- A < 0. No h raiz real

RELAES DE GIRARD
Sendo x
1
e x
2
as razes da equao ax
2
+ bx + c, tem-se:

x
1
+ x
2
=
a
b
x
1
. x
2
=
a
c



Exerccios de Sala #

1. Resolva, em reais, as equaes:

a) 2x
2
32 = 0 c) 2x
2
5x 3 = 0

b) x
2
12x = 0

2. Considere a equao x
2
mx + m = 0 na incgnita x.
Para quais valores reais de m ela admite razes reais e
iguais?
a) 0 e 4 d) 1 e 3
b) 0 e 2 e) 1 e 4
c) 0 e 1

3. Sendo x
1
e x
2
as razes da equao 2x
2
6x + 1 = 0,
determine:
a) x
1
+ x
2
b) x
1
. x
2

c)
2
x
1
1
x
1
+


Tarefa Mnima #

4. Resolva em R, as equaes:
a) x
2
5x + 6 = 0
b) x
2
+ 6x 8 = 0
c) 3x
2
7x + 2 = 0
d) x
2
4x + 4 = 0
e) 2x
2
x + 1 = 0
f) 4x
2
100 = 0
g) x
2
5x = 0

5. Os nmeros 2 e 4 so razes da equao:
a) x
2
6x + 8 = 0 d) x
2
5x + 6 = 0
b) x
2
+ x 6 = 0 e) x
2
+ 6x 1 = 0
c) x
2
6x 6 = 0

6. (PUC-SP) Quantas razes reais tem a equao
2x
2
2x + 1 = 0?
a) 0 c) 2 e) 4
b) 1 d) 3

7. A soma e o produto das razes da equao
2x
2
6x + 9 = 0 so respectivamente:
a) 3 e 4,5 d) 4,5 e 5
b) 2 e 4 e) n.d.a.
c) 3 e 2

8. Sendo x
1
e x
2
as razes da equao 2x
2
5x 1 = 0.
Obtenha
2
x
1
1
x
1
+


Tarefa Complementar #
9. Resolver em R a equao 1
1 x
1
1
2
x
2
=
+
+



10. A maior soluo da equao 2x
4
5x
2
3 = 0 :
a) 3 b) 2 c) 3 d) 1 e) 2

11. Sendo x
1
e x
2
as razes da equao 2x
2
6x 3 = 0,
determine a soma dos nmeros associados s proposies
verdadeiras:
01. x
1
e x
2
so iguais
02. x
1
+ x
2
= 3
04. x
1
. x
2
=
2
3

08.
2
x
1
1
x
1
+ = 2
16. x
1
2
+ x
2
2
= 12
32. x
1
2
.x
2
+ x
1
.x
2
2
=
2
9

12. A soluo da equao x 3 = 3 + x :

13. (MACK-SP) Se x e y so nmeros reais positivos, tais
que x
2
+ y
2
+ 2xy + x + y 6 =0, ento x + y vale:
a) 2 b) 3 c) 4 d) 5 c) 6

14. Determine a soma dos nmeros associados s
proposies corretas:

01. Se a soma de um nmero qualquer com o seu
inverso 5, ento a soma dos quadrados desse
nmero com o seu inverso 23.

02. Se x
1
e x
2
so as razes da equao 2x
2
6x 3 = 0,
ento o valor de x
1
2
.x
2
+ x
1
.x
2
2
=
2
9

04. Se x e y so nmeros reais positivos, tais que
x
2
+ y
2
+ 2xy + x + y 6 =0, ento, x + y vale 2

08. Se x soluo da equao
x
2
3 + 3
2
x = 2, ento, o valor de x
4
= 16
16. O valor de
2
1
3
1
16 8 + 5

15. Considere a equao 2x
2
6x + 1 = 0. Sendo x
1
e x
2
,
razes dessa equao, pode-se afirmar:
01. x
1
= x
2

02. o produto das razes dessa equao 0,5
04. a soma das razes dessa equao 3
08. a soma dos inversos das razes 6
16. a equao no possui razes reais

16. A maior raiz da equao x
4
10x
2
+ 9 = 0 :
a) 3 b) 4 c) 8 d) 9 e) 1

17. Assinale a soma dos nmeros associados s
proposies corretas:
Please purchase PDF Split-Merge on www.verypdf.com to remove this watermark.
Incluso para a vida Matemtica A

Pr-Vestibular da UFSC
9
01. A maior raiz da equao x
6
x
3
2 = 0
3
2
02. A maior raiz da equao 3x
2
7x + 2 = 0 2
04. As razes da equao x
2
4x + 5 = 0 esto
compreendidas entre 1 e 3
08. A soma das razes da equao x
6
x
3
2 = 0 3
16. A equao x
2
4x + 2 = 0 no possui razes reais

18. Determine o valor de x que satisfaz as equaes:
a)
x x = + 3 1

b)
2 1 2
3
= + + x x


UNIDADE 5

ESTUDO DAS FUNES

Sejam A e B dois conjuntos no vazios e uma relao R de
A em B, essa relao ser chamada de funo quando todo
e qualquer elemento de A estiver associado a um nico
elemento em B.

Formalmente:

f funo de A em B (x e A, - y e B | (x, y) e f)

Numa funo podemos definir alguns elementos.

- Conjunto de Partida: A
- Domnio: Valores de x para os quais existe y.
- Contra Domnio: B
- Conjunto Imagem: Valores de y para os quais existe x.


Observaes:
- A imagem est sempre contida no Contra
Domnio (Im c C.D)
- Podemos reconhecer atravs do grfico de uma
relao, se essa relao ou no funo. Para
isso, deve-se traar paralelas ao eixo y. Se cada
paralela interceptar o grfico em apenas um
ponto, teremos uma funo.
- O domnio de uma funo o intervalo
representado pela projeo do grfico no eixo das
abscissas. E a imagem o intervalo representado
pela projeo do grfico no eixo y.


Domnio = [a, b] Imagem = [c, d]
Valor de uma Funo
Denomina-se valor numrico de uma funo f(x) o valor
que a varivel y assume quando a varivel x substituda
por um valor que lhe atribudo.
Por exemplo: considere a relao y = x
2
, onde cada valor
de x corresponde um nico valor de y.
Assim se x = 3, ento y = 9.
Podemos descrever essa situao como: f(3) = 9

Exemplo 1: Dada a funo f(x) = x + 2. Calcule o valor de
f(3)

Resoluo: f(x) = x + 2, devemos fazer x = 3
f(3) = 3 + 2
f(3) = 5

Exemplo 2: Dada a funo f(x) = x
2
- 5x + 6. Determine o
valor de f(-1).

Resoluo: f(x) = x
2
- 5x + 6, devemos
fazer x = -1
f(-1) = (-1)
2
- 5(-1) + 6
f(-1) = 1 + 5 + 6
f(-1) = 12

Exemplo 3: Dada a funo f(x 1) = x
2
. Determine f(5).

Resoluo: f(x 1) = x
2
, devemos fazer x = 6
f(6 1) = 6
2

f(5) = 36

Observe que se fizssemos x = 5, teramos f(4) e no f(5).

Exerccios de Sala #

1. Seja o grfico abaixo da funo f, determinar a soma
dos nmeros associados s proposies corretas:

01. O domnio da funo f {x e R | - 3 s x s 3}
02. A imagem da funo f {y e R | - 2 s y s 3}
04. para x = 3, tem-se y = 3
08. para x = 0, tem-se y = 2
16. para x = - 3, tem-se y = 0
32. A funo decrescente em todo seu domnio


2. Em cada caso abaixo, determine o domnio de cada
funo:
a) y = 2x + 1 b) y =
7 2
7
x

c) y = 2 3 x d) y =
2 2
3

+
x
x


Please purchase PDF Split-Merge on www.verypdf.com to remove this watermark.
Matemtica A Incluso para a Vida

Pr-Vestibular da UFSC
10
3.
( )
2x-1, se x 0
5, se 0 x 5
2
x 5x 6, se x 5
Seja f x

s
< s
+ >
.

Calcule o valor de:
) 6 (
) ( ) 3 (
f
f f t +


Tarefa Mnima #

4. (UNAERP-SP) Qual dos seguintes grficos no
representa uma funo f: R R ?
a)

b)

c)


d)

e)


5. Assinale a soma dos nmeros associados s proposies
corretas:


01. O domnio da funo f {x e R | - 2 s x s 2}
02. A imagem da funo f {y e R | - 1 s y s 2}
04. para x = -2 , tem-se y = -1
08. para x = 2, tem-se y = 2
16. A funo crescente em todo seu domnio

6. Determine o domnio das seguintes funes:
a) y =
9 3
2
x
b) y = 3 x

c) y =
2
6

+
x
x
d) y =
3
5 x

7. (UFSC) Considere as funes f: R R e g: R R
dadas por f(x) = x
2
x + 2 e g(x) = 6x +
5
3
. Calcule
f(
2
1
) +
4
5
g(1).

8. (UFPE) Dados os conjuntos A = {a, b, c, d} e
B = {1, 2, 3, 4, 5}, assinale a nica alternativa que define
uma funo de A em B.
a) {(a, 1), (b, 3), (c, 2)}
b) {(a, 3), (b, 1), (c, 5), (a, 1)}
c) {(a, 1), (b, 1), (c, 1), (d, 1)}
d) {(a, 1), (a, 2), (a, 3), (a, 4), (a, 5)}
e) {(1, a), (2, b), (3, c), (4, d), (5, a)}

Tarefa Complementar #

9. (UFC) O domnio da funo real y =
7
2

x
x
:
a) {x e R| x > 7}
b) {x e R| x s 2}
c) {x e R| 2 s x < 7}
d) {x e R| x s 2 ou x > 7}

10. Considere a funo f(x) = x
2
6x + 8. Determine:
a) f(3)
b) f(5)
c) os valores de x, tal que f(x) = 0

11. (USF-SP) O nmero S do sapato de uma pessoa est
relacionado com o comprimento p, em centmetros,do seu
p pela frmula S =
4
28 5 + p
. Qual o comprimento do
p de uma pessoa que cala sapatos de nmero 41?

a) 41 cm d) 29,5 cm
b) 35,2 cm e) 27,2 cm
c) 30,8 cm

12. (FUVEST) A funo que representa o valor a ser pago
aps um desconto de 3% sobre o valor x de uma
mercadoria :
a) f(x) = x 3 d) f(x) = - 3x
b) f(x) = 0,97x e) f(x) = 1,03x
c) f(x) = 1,3x

13. ( FCMSCSP ) Se f uma funo tal f(a + b) = (a).f(b),
quaisquer que sejam os nmeros reais a e b, ento f(3x)
igual a:
a) 3.f(x) d) [f(x)]
3

b) 3 + f(x) e) f(3) + f(x) c) f(x
3
)
Please purchase PDF Split-Merge on www.verypdf.com to remove this watermark.
Incluso para a vida Matemtica A

Pr-Vestibular da UFSC
11
14. (FGV-SP) Numa determinada localidade, o preo da
energia eltrica consumida a soma das seguintes
parcelas:
1 . Parcela fixa de R$ 10,00;
2 . Parcela varivel que depende do nmero de
quilowatt-hora (kWh) consumidos; cada kWh custa
R$ 0,30. Se num determinado ms, um consumidor
pagou R$ 31,00, ento ele consumiu:

a) 100,33 kWh d) entre 65 e 80 kWh
b) mais de 110 kWh e) entre 80 e 110 kWh
c) menos de 65 kWh

15. (PUC-Campinas) Em uma certa cidade, os taxmetros
marcam, nos percursos sem parada, uma quantia de 4UT
(unidade taximtrica) e mais 0,2 UT por quilmetro
rodado. Se, ao final de um percurso sem paradas, o
taxmetro registrava 8,2 UT, o total de quilmetros
corridos foi:

16. (UFSC) Dadas as funes f(x) = 3x + 5,
g(x) = x
2
+ 2x 1 e h(x) = 7 x, o valor em mdulo da
expresso:

( )
1
4 4
2
1
h g
f ( )
( | |

| (
\ .



17. (UFSC) Considere a funo f(x) real, definida por f(1)
= 43 e f(x + 1) = 2 f(x) 15. Determine o valor de f(0).
18. (UDESC) A funo f tal que f(2x + 3) = 3x + 2.
Nessas condies, f(3x + 2) igual a:

UNIDADE 6

FUNO POLINOMIAL DO 1 GRAU

FUNO POLINOMIAL DO 1 GRAU

Uma funo f de R em R do 1 grau se a cada x e R,
associa o elemento ax + b.

Forma: f(x) = ax + b com a = 0.

a o coeficiente angular e b o coeficiente linear.
Grfico

O grfico ser uma reta crescente se a for positivo e
decrescente se a for negativo.


Como o grfico de uma funo do 1 Grau uma reta, logo
necessrio definir apenas dois pontos para obter o
grfico.
Interceptos:
- Ponto que o Grfico corta o eixo y: deve-se fazer
x = 0. Logo, o ponto que o grfico corta o eixo y tem
coordenadas (0,b).
- Ponto que o Grfico corta o eixo x: deve-se fazer
y = 0. Logo, o ponto que o grfico corta o eixo x tem
coordenadas (

b
a
,0). O ponto que o grfico corta o
eixo x chamado raiz ou zero da funo.

RESUMO GRFICO

f(x) = ax + b, a > 0 f(x) = ax + b, a < 0



Funo crescente Funo decrescente

Exemplo: Esboar o grfico da funo da funo
f(x) = 3x + 1.

Resoluo: o grfico intercepta o eixo y em (0,b). Logo o
grfico da funo f(x) = 3x + 1 intercepta o
eixo y em (0,1).
Para determinar o ponto que o grfico corta o
eixo x deve-se fazer y = f(x) = 0.

3x + 1 = 0
x =
3
1

Logo, o ponto que o grfico corta o eixo x tem
coordenadas (
3
1
, 0)


D = 9 C.D. = 9 Im = 9

FUNO CONSTANTE
Uma funo f de R em R constante se, a cada x e R,
associa sempre o mesmo elemento k e R.
D(f) = R e Im (f) = k

Forma: f(x) = k

Grfico:
Exemplo: y = f(x) = 2
Please purchase PDF Split-Merge on www.verypdf.com to remove this watermark.
Matemtica A Incluso para a Vida

Pr-Vestibular da UFSC
12

D = 9 C.D. = 9 Im = {2}
Exerccios de Sala #

1. Considere as funes f(x) = 2x 6 definida em reais.
Determine a soma dos nmeros associados s
proposies corretas :

01. a reta que representa a funo f intercepta o eixo
das ordenadas em (0,- 6)
02. f(x) uma funo decrescente
04. a raiz da funo f(x) 3
08. f(-1) + f(4) = 0
16. a imagem da funo so os reais
32. A rea do tringulo formado pela reta que
representa f(x) e pelos eixos coordenados 18
unidades de rea.

2. (PUC-SP) Para que a funo do 1 grau dada por f(x) =
(2 - 3k)x + 2 seja crescente devemos ter:
) ) ) ) ) = < > < >
2 2 2 2 2
a k b k c k d k e k
3 3 3 3 3


3. (UFSC) Seja f(x) = ax + b uma funo linear. Sabe-se
que f(-1) = 4 e f(2) = 7. D o valor de f(8).

Tarefa Mnima #

4. Esboar o grfico das seguintes funes:
a) f(x) = x + 3
b) f(x) = 2x + 1

5. (FGV-SP) O grfico da funo f(x) = mx + n passa
pelos pontos A(1, 2) e B(4, 2). Podemos afirmar que m +
n vale em mdulo:

6. (UFMG) Sendo a < 0 e b > 0, a nica representao
grfica correta para a funo f(x) = ax + b :


7. (UFMA) O grfico da funo f(x) = ax + b intercepta o
eixo dos x no ponto de abscissa 4 e passa pelo ponto (1,
3), ento f(x) :
a) f(x) = x 3 d) f(x) = 2x 1
b) f(x) = x 4 e) f(x) = 3x 6
c) f(x) = 2x 5

8. Sendo f(x) = 2x + 5, obtenha o valor de
t
t

t
f t f ) ( ) (

com t = t.
Tarefa Complementar #

9. (UCS-RS) Para que 3 seja raiz da funo
f(x) = 2x + k, deve-se ter
a) k = 0 b) k = - 2 c) k = 6 d) k = -6 e) k = 2

10. (UFPA) A funo y = ax + b passa pelo ponto (1,2) e
intercepta o eixo y no ponto de ordenada 3. Ento, a 2b
igual a:
a) 12 b) 10 c) 9 d) 7 e) n.d.a.

11. (Fuvest-SP) A reta de equao 2x + 12y 3 = 0, em
relao a um sistema cartesiano ortogonal, forma com os
eixos do sistema um tringulo cuja rea :
a) 1/2 b) 1/4 c) 1/15 d) 3/8 e) 3/16

12. O grfico da funo f(x) est representado pela figura
abaixo:

Pode-se afirmar que f(4) igual a:

13. (Santo Andr-SP) O grfico mostra como o dinheiro
gasto ( y) por uma empresa de cosmticos, na produo de
perfume, varia com a quantidade de perfume produzida
(x). Assim, podemos afirmar:


a) Quando a empresa no produz, no gasta.
b) Para produzir 3 litros de perfume, a empresa gasta
R$ 76,00.
c) Para produzir 2 litros de perfume, a empresa gasta
R$ 54,00.
d) Se a empresa gastar R$ 170,00, ento ela produzir
5 litros de perfume.
e) Para fabricar o terceiro litro de perfume, a empresa
gasta menos do que para fabricar o quinto litro.

14. (UFSC) Sabendo que a funo: f(x) = mx + n admite 5
como raiz e f(-2) = -63, o valor de f(16) :

Please purchase PDF Split-Merge on www.verypdf.com to remove this watermark.
Incluso para a vida Matemtica A

Pr-Vestibular da UFSC
13
15. O valor de uma mquina decresce linearmente com o
tempo, devido ao desgaste. Sabendo-se que hoje ela vale
R$800,00, e que daqui a 5 anos valer R$160,00, o seu
valor, em reais, daqui a trs anos ser:
a) 480 b) 360 c) 380 d) 400 e) 416

16. (UFRGS) Considere o retngulo OPQR da figura
baixo. A rea do retngulo em funo da abscissa x do
ponto R

a) A = x
2
3x d) A = - 2x
2
+ 6x
b) A = - 3x
2
+ 9x e) A = 2x
2
6x
c) A = 3x
2
9x

17. (UFRGS) Dois carros partem de uma mesma cidade,
deslocando-se pela mesma estrada. O grfico abaixo
apresenta as distncias percorridas pelos carros em funo
do tempo.
Distncia (em km)
Tempo (em horas)

Analisando o grfico, verifica-se que o carro que partiu
primeiro foi alcanado pelo outro ao ter percorrido
exatamente:
a) 60km b) 85km c) 88km d) 90km e) 91km

Estudo do vrtice da parbola
A Parbola que representa a funo do 2 Grau dividida
em duas partes simtricas. Essa diviso feita por um eixo
chamado de eixo de simetria. A interseco desse eixo
com a parbola recebe o nome de vrtice da parbola.




- O vrtice o ponto de mximo da funo se a < 0.
- O vrtice o ponto de mnimo da funo se a > 0.

Coordenadas do vrtice

O vrtice um ponto de coordenadas V(x
v
, y
v
) onde

e
4a
= yv
2a
b
v
x
A
=


Imagem da funo quadrtica
- Se a > 0, ento Im = {y e R| y >
A
4a
}
- Se a < 0, ento Im = {y e R| y s
A
4a
}

Resumo grfico

- A > 0




- A = 0





Please purchase PDF Split-Merge on www.verypdf.com to remove this watermark.
Matemtica A Incluso para a Vida

Pr-Vestibular da UFSC
14
- A < 0




18. (UERJ) Considere a funo f, definida para todo x real
positivo, e seu respectivo grfico. Se a e b so dois meros
positivos (a < b), a rea do retngulo de vrtices (a, 0),
(b, 0) e (b, f(b) ) igual a 0,2. f(x) =
x
1


Calcule a rea do retngulo de vrtices (3a, 0), (3b, 0)
e (3b, f(3b))

UNIDADE 7

FUNO POLINOMIAL DO 2 GRAU

Uma funo f de R em R polinomial do 2 grau se a cada
x e R associa o elemento ax
2
+ bx + c, com a = 0

Forma: f(x) = ax
2
+ bx + c, com a = 0

Grfico
O grfico de uma funo polinomial do 2 Grau de R em R
uma parbola. A concavidade da parbola determinada
pelo sinal do coeficiente a (coeficiente de x
2
). Assim,
quando:
a > 0 tem-se a parbola com concavidade para cima

a < 0 tem-se parbola com concavidade para baixo

nterceptos
- O ponto que o grfico corta o eixo y possui
coordenadas (0,c)
- Para achar o(s) ponto(s) que o grfico corta o eixo x,
deve-se fazer y = 0. Tem-se ento uma equao do 2
grau ax
2
+ bx + c = 0, onde:

ac 4 b onde ,
2a
b
2
= A

= x

Se A > 0 Duas Razes Reais
Se A = 0 Uma Raiz Real
Se A < 0 No possui Razes Reais


Estudo do vrtice da parbola
A Parbola que representa a funo do 2 Grau dividida
em duas partes simtricas. Essa diviso feita por um eixo
chamado de eixo de simetria. A interseco desse eixo
com a parbola recebe o nome de vrtice da parbola



- O vrtice o ponto de mximo da funo se a < 0.
- O vrtice o ponto de mnimo da funo se a > 0.

Coordenadas do vrtice

O vrtice um ponto de coordenadas V(x
v
, y
v
) onde

e
4a
= yv
2a
b
v
x
A
=


Imagem da funo quadrtica
- Se a > 0, ento Im = {y e R| y >
A
4a
}
- Se a < 0, ento Im = {y e R| y s
A
4a
}


Resumo grfico

- A > 0





- A = 0





Please purchase PDF Split-Merge on www.verypdf.com to remove this watermark.
Incluso para a vida Matemtica A

Pr-Vestibular da UFSC
15
- A < 0



Exerccios de Sala #

1. Em relao a funo f(x) = x
2
6x + 8 definida de
9 9 correto afirmar:
01. 2 e 4 so os zeros da funo f
02. o vrtice da parbola possui coordenadas (3, -1)
04. O domnio da funo f(x) o conjunto dos
nmeros reais.
08. A imagem da funo : { y e R| y > 1}
16. A rea do tringulo cujos vrtices so o vrtice da
parbola e seus zeros, 4 unidades de rea.

2. Em cada caso abaixo, esboce o grfico de f e d seu
conjunto imagem.
a) f: 9 9, f(x) = x
2
2x

b) f: 9 9, f(x) = x
2
+ 4

c) f: [0, 3[ 9, f(x) = f(x) = x
2
2x

3. Considere f(x) = x
2
6x + m definida de 9 9.
Determine o valor de m para que o grfico de f(x):
a) tenha duas interseces com o eixo
b) tenha uma interseco com o eixo x
c) no intercepte o eixo x

Tarefa Mnima #

4. Determine as razes, o grfico, as coordenadas do
vrtice e a imagem de cada funo.
a) f: 9 9, f(x) = x
2
2x 3
b) f: 9 9, f(x) = (x + 2)(x 4)
c) f: 9 9, f(x) = x
2
+ 2x 1
d) f: 9 9, f(x) = x
2
3x

5. Dada a funo f(x) = x
2
- 8x + 12 de R em R. Assinale
as verdadeiras:
01. O grfico intercepta o eixo y no ponto de
coordenadas (0,12).
02. As razes de f so 2 e 6.
04. O domnio de f o conjunto dos nmeros reais.
08. O grfico no intercepta o eixo x.
16. A imagem da funo { y e R| y > 4 }
32. O vrtice da parbola possui coordenadas (4, 4)
64. A funo crescente em todo seu domnio.

6. (UFSC) Considere a parbola y = -x
2
+ 6x definida em
R x R. A rea do tringulo cujos vrtices so o vrtice da
parbola e seus zeros, :

7. (ACAFE-SC) Seja a funo f(x) = - x
2
2x + 3 de
domnio [-2, 2]. O conjunto imagem :
a) [0, 3] c) ]-, 4] e) [-5, 3]
b) [-5, 4] d) [-3, 1]

8. ( PUC-SP) Seja a funo f de R em R, definida por f( x)
= x
2
3x + 4. Num sistema de coordenadas cartesianas
ortogonais, o vrtice da parbola que representa f localiza-
se:
a) no primeiro quadrante.
b) no segundo quadrante.
c) no terceiro quadrante.
d) sobre o eixo das coordenadas.
e) sobre o eixo das abscissas.

Tarefa Complementar #

9. (UFSC) Seja f: R R, definida por: f(x) = - x
2
,
termine a soma dos nmeros associados s afirmativas
verdadeiras:
01. O grfico de f(x) tem vrtice na origem.
02. f(x) crescente em R.
04. As razes de f(x) so reais e iguais.
08. f(x) decrescente em [0, + )
16. Im(f) = { y e R , y s 0}
32. O grfico de f(x) simtrico em relao ao eixo x.

10. (ESAL-MG) A parabola abaixo o grfico da funo
f(x) = ax
2
+ bx + c. Assinale a alternativa correta:

a) a < 0, b = 0, c = 0 d) a < 0, b < 0, c > 0
b) a > 0, b = 0, c < 0 e) a > 0, b > 0, c > 0
c) a > 0, b < 0, c = 0

11. Considere a funo definida em x dada por
f(x) = x
2
mx + m. Para que valores de m o grfico de
f(x) ir interceptar o eixo x num s ponto?

12. (UFPA) As coordenadas do vrtice da funo
y = x
2
2x + 1 so:
a) (-1, 4) c) (-1, 1) e) (1, 0)
b) (1, 2) d) (0, 1)

13. (UFPA) O conjunto de valores de m para que o
grfico de y = x
2
mx + 7 tenha uma s interseco com o
eixo x :
a) { 7} c) { 2 }
b) { 0 } d) { 2 7 }

14. (Mack-SP) O vrtice da parbola y = x
2
+ kx + m o
ponto V(1, 4). O valor de k + m em mdulo :

Please purchase PDF Split-Merge on www.verypdf.com to remove this watermark.
Matemtica A Incluso para a Vida

Pr-Vestibular da UFSC
16
15. (UFSC) Dada a funo f: R R definida por f(x) =
ax
2
+ bx + c, sabe-se que f(1) = 4, f(2) = 7 e f(-1) = 10.
Determine o valor de a - 2b + 3c.

16. A equao do eixo de simetria da parbola de
equao y = 2x
2
- 10 + 7, :
a) 2x - 10 + 7 = 0 d) y = 3,5
b) y = 5x + 7 e) x = 1,8
c) x = 2,5

17. O grfico da funo f(x) = mx
2
(m
2
3)x + m
3

intercepta o eixo x em apenas um ponto e tem
concavidade voltada para baixo. O valor de m :

a) 3 c) 2 e) 1
b) 4 d) 2

18. (UFSC) Marque no carto a nica proposio
correta. A figura abaixo representa o grfico de uma
parbola cujo vrtice o ponto V. A equao da reta r :

01. y = -2x + 2
02. y = x + 2
04. y = 2x + 1
08. y = 2x + 2
16. y = -2x 2

UNIDADE 8

INEQUAES DO 2 GRAU
INEQUAES TIPO PRODUTO
INEQUAES TIPO QUOCIENTE

INEQUAES DO 2
O
GRAU

Inequao do 2 grau toda inequao da forma:

< + +
> + +
s + +
> + +
0
0
0
0
2
2
2
2
c bx ax
c bx ax
c bx ax
c bx ax
com a = 0

Para resolver a inequao do 2 grau se associa a expresso
a uma funo do 2 grau; assim, pode-se estudar a variao
de sinais em funo da varivel. Posteriormente,
selecionam-se os valores da varivel que tornam a
sentena verdadeira. Estes valores iro compor o conjunto-
soluo.

Exemplos:

a) resolver a inequao x
2
2x 3 > 0



S = {x e R | x s -1 ou x > 3} ou
S = ]-, -1] [3, +[

b) resolver a inequao x
2
7x + 10 s 0


S = { x e R | 2 s x s 5}
S = [2, 5]

c) resolver a inequao x
2
+ 5x 4 > 0

S = { x e R | 1 < x < 4}
S = [1, 4]

Inequaes Tipo Produto
Inequao Produto qualquer inequao da forma:
a) f(x).g(x) > 0 b) f(x).g(x) > 0
c) f(x).g(x) s 0 d) f(x).g(x) < 0

Para resolvermos inequaes deste tipo, faz-se necessrio
o estudo dos sinais de cada funo e em seguida aplicar a
regra da multiplicao.

Exemplo: Resolver a inequao (x
2
4x + 3) (x 2) < 0

S = { x e R | x < 1 ou 2 < x < 3}

Inequaes Tipo Quociente
Inequao quociente qualquer inequao da forma:
a)
f(x)
g(x)
0 b)
f(x)
g(x)
> 0 c)
f(x)
g(x)
0 d)
f(x)
g(x)
< 0 > s


Para resolvermos inequaes deste tipo necessrio que se
faa o estudo dos sinais de cada funo separadamente e,
em seguida, se aplique a regra de sinais da diviso.
necessrio lembrar que o denominador de uma frao no
pode ser nulo, ou seja, nos casos acima vamos considerar
g(x) = 0.
Please purchase PDF Split-Merge on www.verypdf.com to remove this watermark.
Incluso para a vida Matemtica A

Pr-Vestibular da UFSC
17
Exemplo: Resolver a inequao
0
2
3 4
2
>

+
x
x x


S = { x e R | 1 s x < 2 ou x > 3}

Exerccios de Sala #

1. Resolver em 9 as seguintes inequaes:
a) x
2
8x + 12 > 0
b) x
2
8x + 12 s 0
c) x
2
9x + 8 > 0

2. O domnio da funo definida por
f(x) =
x x
x
2
3 10
6

:
a) D = {x e R| x s 2 ou x > 5} {6}.
b) D = {x e R| x s - 2 ou x > 5} {6}.
c) D = {x e R| x s - 2 ou x > 5}
d) D = {x e R| x s - 2 ou x > 7} {6}.
e) n.d.a.

3. Determine o conjunto soluo das seguintes
inequaes:

a) (x 3)(2x 1)(x
2
4) < 0
b)
4
10 7
2

+
x
x x
> 0

Tarefa Mnima #

4. Resolver em 9 as seguintes inequaes:
a) x
2
6x + 8 > 0
b) x
2
6x + 8 s 0
c) x
2
+ 9 > 0
d) x
2
s 4
e) x
2
> 6x
f) x
2
> 1

5. (Osec-SP) O domnio da funo
f(x) = + + x x
2
2 3 , com valores reais, um dos conjuntos
seguintes. Assinale-o.
a) {x e R , -1 s x s 3 } d) { x e R , x > 3}
b) { x e R , -1 < x < 3 } e) n.d.a.
c) { }

6. Resolva, em R, as seguintes inequaes:
a) (x
2
2x 3).( x
2
3x + 4) > 0
b) (x
2
2x 3).( x
2
3x + 4) s 0
c) (x 3) (x
2
16) < 0
d) x
3
s x
e) x
3
3x
2
+ 4x 12 > 0

7. Resolva, em R, as seguintes inequaes:
a)
0
16
6 5
2
2
>

+
x
x x

b)
0
16
6 5
2
2
<

+
x
x x

c)
x
x
x
x +

>
1 1
0

d)
2
1 x
< 1

8. (ESAG) O domnio da funo y =
1 2
1
2

x
x
nos reais :
a) (-, -1 ) d) (-, -1) [1/2, 1)
b) (-1, ] e) { }
c) (-, ]

Tarefa Complementar #

9. Resolver em 9 as seguintes inequaes:
a) x
2
6x + 9 > 0 c) x
2
6x + 9 < 0
b) x
2
6x + 9 > 0 d) x
2
6x + 9 s 0

10. Resolver em 9 as seguintes inequaes:
a) x
2
4x + 5 > 0 c) x
2
4x + 5 < 0
b) x
2
4x + 5 > 0 d) x
2
4x + 5 s 0

11. (CESGRANRIO) Se x
2
6x + 4 s x
2
+ bx + c tem
como soluo o conjunto {x e 9| 0 s x s 3}, ento b e c
valem respectivamente:
a) 1 e 1 d) 0 e 1
b) 1 e 0 e) 0 e 4
c) 0 e 1

12. (UNIP) O conjunto verdade do sistema

s
< +
0 4 2
0 8 9
2
x
x x
:

a) ]1, 2] c) [2, 4[ e) [4, 8[
b) ]1, 4] d) [1, 8[

13. (PUC-RS) A soluo, em R, da inequao x
2
< 8 :
a) { 2 2 ; 2 2 } d) ( ; 2 2 )
b) [ 2 2 ; 2 2 ] e) ( ; 2 2 ]
c) ( 2 2 ; 2 2 )

14. (ACAFE) O lucro de uma empresa dado por L(x) =
100(8 x)(x 3), em que x a quantidade vendida. Neste
caso podemos afirmar que o lucro :
a) positivo para x entre 3 e 8
b) positivo para qualquer que seja x
c) positivo para x maior do que 8
d) mximo para x igual a 8
e) mximo para x igual a 3

15. (FATEC) A soluo real da inequao produto
(x
2
4).(x
2
4x) > 0 :
a) S = { x e R| - 2 s x s 0 ou 2 s x s 4}
b) S = { x e R| 0 s x s 4}
Please purchase PDF Split-Merge on www.verypdf.com to remove this watermark.
Matemtica A Incluso para a Vida

Pr-Vestibular da UFSC
18
c) S = { x e R| x s - 2 ou x > 4}
d) S = { x e R| x s - 2 ou 0 s x s 2 ou x > 4}
e) S = { }

16. (MACK-SP) O conjunto soluo de
5
3
6
<
+ x
x
:
a) { x e R , x > 15 e x < - 3}
b) { x e R , x < 15 e x = - 3}
c) { x e R , x > 0}
d) {x e R , - 3 < x < 15}
e) { x e R , - 15 < x < 15}

17. (Cescem-SP) Os valores de x que satisfazem a
inequao (x
2
2x + 8)(x
2
5x + 6)(x
2
16) < 0 so:
a) x < 2 ou x > 4 d) 4 < x < 2 ou 3 < x < 4
b) x < 2 ou 4 < x < 5 e) x < 4 ou 2 < x < 3 ou x > 4
c) 4 < x < 2 ou x > 4

18. (FUVEST) De x
4
x
3
< 0 pode-se concluir que:
a) 0 < x < 1 d) 2< x < 1
b) 1 < x < 2 e) x < 1 ou x > 1
c) 1< x < 0

UNIDADE 9

PARIDADE DE FUNES
FUNO COMPOSTA e FUNO INVERSA

Funo Par
Uma funo par quando para valores simtricos de x
temos imagens iguais, ou seja:

f(x) = f(x), x e D(f)

Uma consequncia da definio : Uma funo f
par se e somente se, o seu grfico simtrico em relao
ao eixo y.

FUNO MPAR
Uma funo mpar quando para valores simtricos de x
as imagens forem simtricas, ou seja:

f(x) = f(x), x e D(f)

Como consequncia da definio os grficos das funes
mpares so simtricos em relao origem do sistema
cartesiano.

FUNO COMPOSTA
Dadas as funes f: A B e g: B C, denomina-se
funo composta de g com f a funo gof: definida de
A C tal que gof(x) = g(f(x))


f: A B g: B C gof: A C

Condio de Existncia: Im(f) = D(g)

Alguns tipos de funes compostas so:

a) f(g(x)) b) g(f(x)) c) f(f(x)) d) g(g(x))

Exerccio resolvido:
Dadas as funes f(x) = x
2
- 5x + 6 e g(x) = x + 1, achar x
de modo que f(g(x)) = 0

Resoluo: Primeiramente vamos determinar
f(g(x)) e, em seguida, igualaremos a zero.

f(x) = x
2
- 5x + 6
f(g(x)) = (x + 1)
2
- 5(x + 1) + 6
Da vem que f(g(x)) = x
2
- 3x + 2.
Igualando a zero temos:
x
2
- 3x + 2 = 0

Onde x
1
= 1 e x
2
= 2

FUNO INJETORA, SOBREJETORA E
BIJETORA
Funo injetora: Uma funo f: A B injetora se e
somente se elementos distintos de A tm imagens distintas
em B. Em Smbolos:

f injetora x
1
, x
2
e A, x
1
= x
2
f(x
1
) = f(x
2
)


Funo sobrejetora: Uma funo f de A em B
sobrejetora, se todos os elementos de B forem imagem dos
elementos de A, ou seja: CD = Im


Funo bijetora: Uma funo bijetora se for ao mesmo
tempo injetora e sobrejetora.


DICA: De R R, a funo do 1 Grau bijetora, e a
funo do 2 Grau simples.

FUNO INVERSA
Seja f uma funo f de A em B. A funo f
1
de B em A
a inversa de f, se e somente se:
f
o
f
-1
(x) = x, x e A e f
-1
o
f (x) = x, x e B.
Observe que A = D(f) = CD(f
-1
) e B = D(f
-1
) = CD(f)

Please purchase PDF Split-Merge on www.verypdf.com to remove this watermark.
Incluso para a vida Matemtica A

Pr-Vestibular da UFSC
19
IMPORTANTE: f inversvel f bijetora

Para encontrar a inversa de uma funo, o processo
prtico trocar x por y e, em seguida, isolar y.

Os grficos de duas funes inversas f(x) e f
1
(x) so
simtricos em relao bissetriz dos quadrantes mpares.
(f(x) = x)

Exerccio Resolvido:
Dada a funo f(x) = 2x + 4 de R em R. determine a
sua inversa.

Resoluo: Como a funo f(x) bijetora, ento ela
admite inversa. Basta trocarmos x por y e
teremos:

f(x) = 2x + 4
x = 2y + 4
x - 4 = 2y
f
-1
(x) =
x 4
2


Exerccios de Sala #

1. Dadas as funes f(x) = 2x 1, g(x) = x
2
+ 2.
Determine:
a) f(g(x)) c) f(g(3))
b) g(f(x)) d) g(f(-2))

2. (UFSC) Considere as funes f, g: R R tais que
g(x) = 2x + 1 e g(f(x)) = 2x
2
+ 2x + 1. Calcule f(7).

3. Se x = 3, determine a inversa da funo
3
1 2
) (

+
=
x
x
x f


Tarefa Mnima #

1. Dadas as funes f(x) = x + 2 e g(x) = 2x
2
. Obter:

a) f(g(x)) e) f(g(3))
b) g(f(x)) f) g(f(1))
c) f(f(x)) g) f(f(f(2)))
d) g(g(x))

2. (UFU-MG) Dadas as funes reais definidas por f(x) =
2x - 6 e g(x) = x
2
+ 5x + 3, pode-se dizer que o domnio da
funo h(x) = ( )( ) fog x :
a) {x e R , x s -5 ou x > 0} b) {x e R , x > 0}
c) {x e R , x > -5} d) { }
e) n.d.a.
3. (UFSC) Sendo f(x) = 4x + 1 e f(g(x)) = x
2
+ 1, com f e
g definidas para todo x real, determine o valor numrico da
funo g no ponto x = 18, ou seja, g(18).

4. Determine a funo inversa de cada funo a seguir:

a) y = 2x 3 c) y =
4
1 2

+
x
x , x = 4
b) y =
4
2 + x

5. (UFSC) Seja a funo f(x) =

2
2
x
x
, com x = 2,
determine f
-1
(2).

Tarefa Complementar #

6. (UFSC) Sejam f e g funes de R em R definidas por:
f(x) = -x + 3 e g(x) = x
2
- 1.Determine a soma dos nmeros
associados (s) proposies verdadeiras.

01. A reta que representa a funo f intercepta o eixo
das ordenadas em (0,3).
02. f uma funo crescente.
04. -1 e +1 so os zeros da funo g.
08. Im(g) = { y e R , y > -1 }.
16. A funo inversa da f definida por
f
-1
(x) = -x + 3.
32. O valor de g(f(1)) 3.
64. O vrtice do grfico de g o ponto (0, 0).

7. Dadas as funes: f(x) = 5 x e g(x) = x
2
- 1, o
valor de gof(4) :

8. (UEL-PR) Sejam f e g funes reais definidas por f(x) =
2x
2
+ 1, g(x) = 2 - x. O valor de f(g(-5)) :

9. (Mack-SP) Sejam as funes reais definidas por f(x) = x
2 e f(g(x)) = 2x 3. Ento g(f(x)) definida por:
a) 2x 1 c) 2x 3 e) 2x 5
b) 2x 2 d) 2x 4

10. (F.C.Chagas-BA) A funo inversa da funo f(x) =
2 1
3
x
x

+
:
anteriores das nenhuma e)
x - 2
1 + 3x
= (x)
1 -
f d)
x - 3
2x - 1
= (x)
1 -
f c)
3 - x
1 + 2x
= (x)
1 -
f b)
1 - 2x
3 + x
= ) (
1 -
f a) x

11. Obtenha as sentenas que definem as funes inversas
de:
a) f: [ 3; 5] [1, 17] tal que f(x) = 2x + 7
b) g: [2, 5] [0,9] tal que g(x) = x
2
4x + 4
c) h: [3, 6] [1, 8]

tal que h(x) = x
2
6x + 8

12. (MACK-SP) Se f(g(x)) = 2x
2
4x + 4 e f(x 2) = x +
2, ento o valor de g(2) :
a) - 2 c) 0 e) 14
b) 2 d) 6
Please purchase PDF Split-Merge on www.verypdf.com to remove this watermark.
Matemtica A Incluso para a Vida

Pr-Vestibular da UFSC
20
13. (UFSC) Seja f uma funo polinomial do primeiro
grau, decrescente, tal que f(3) = 2 e f(f(1)) = 1. Determine
a abscissa do ponto onde o grfico de f corta o eixo x.

14. (UDESC) Se f(x) = ax
2
+ bx + 3, f(1) = 0 e f(2) = - 1.
Calcule f(f(a))

15. (IME-RJ) Sejam as funes g(x) e h(x) assim
definidas: g(x) = 3x 4; h(x) = f(g(x)) = 9x
2
6x + 1.
Determine a funo f(x).

UNIDADE 10

EXPONENCIAL

EQUAO EXPONENCIAL
Chama-se equao exponencial toda equao que pode ser
reduzida a forma a
x
= b, com 0 < a = 1.
Para resolver tais equaes necessrio transformar a
equao dada em:
- Igualdade de potncia de mesma base.
a
f(x)
= a
g(x)
f(x) =g(x)
- Potncias de expoentes iguais. a
f(x)
= b
f(x)
a = b
sendo a e b = 1 e a e b e R
*
+
.

Funo Exponencial f(x) = a
x


(a > 1) funo crescente


(0 < a < 1) funo decrescente



INEQUAO EXPONENCIAL
Para resolvermos uma inequao exponencial devemos
respeitar as seguintes propriedades:
- Quando as bases so maiores que 1 (a > 1), a relao
de desigualdade se mantm.
a
f(x)
> a
g(x)
f(x) > g(x)

- Quando as bases esto compreendidas entre 0 e 1 (0 <
< 1), a relao de desigualdade se inverte.

a
f(x)
> a
g(x)
f(x) < g(x)
Exerccios de Sala #

1. (UFSC) Dado o sistema
7 1
5 25
2
2
x y
x
y
+
+
=
=

, o valor de
y
x
|
\

|
.
|
4
:

2. (UFSC) O valor de x, que satisfaz a equao 2
2x + 1
-
3.2
x + 2
= 32, :

Tarefa Mnima #

1. Resolva, em R, as equaes a seguir:
a) 2
x
= 128 b) 2
x
=
1
16

c) 3
x 1
+ 3
x + 1
= 90 d) 25.3
x
= 15
x
:
e) 2
2x
2
x + 1
+ 1 = 0

2. (PUC-SP) O conjunto verdade da equao
3.9
x
26.3
x
9 = 0, :
3. Dadas f(x) =
1
2
|
\

|
.
|
x
e as proposies:
I - f(x) crescente
II - f(x) decrescente
III - f(3) = 8
IV- ( 0,1 ) e f(x)

podemos afirmar que:
a) todas as proposies so verdadeiras.
b) somente II falsa.
c) todas so falsas.
d) II e III so falsas.
e) somente III e IV so verdadeiras.

4. Resolva, em R, as inequaes a seguir:
a) 2
2x 1
> 2
x + 1

b) (0,1)
5x 1
< (0,1)
2x + 8

c)
3 1
4
7
4
7
2
|
.
|

\
|
<
|
.
|

\
|
x

d) 0,5
|x 2|
< 0,5
7

5. (OSEC-SP) O domnio da funo de definida por y =
1
1
3
243
|
\

|
.
|
x
, :
a) ( , 5 [ b) ] 5, + )
c) ( , 5 [ d) ] 5, + ) e) n.d.a.

Tarefa Complementar #

6. Resolvendo a equao 4
x
+ 4 = 5.2
x
, obtemos:

a) x
1
= 0 e x
2
= 1 c) x
1
= 0 e x
2
= 2
b) x
1
= 1 e x
2
= 4 d) x
1
= x
2
= 3

7. (Unesp-SP) Se x um nmero real positivo tal que
2 2
2
2 x x
=
+
, ento ( )
x
x
x
x
2
igual a:

8. A maior raiz da equao 4
|3x 1|
= 16

Please purchase PDF Split-Merge on www.verypdf.com to remove this watermark.
Incluso para a vida Matemtica A

Pr-Vestibular da UFSC
21
9. (ITA-SP) A soma das razes da equao
9
4
3
1
1
2
1
x
x

=
:

10. A soma das razes da equao

2
3
1
132
3
2 1
1
|
\

|
.
| + =

+
x x
x
.
:

11. (UFMG) Com relao funo f(x) = a
x
, sendo a e x
nmeros reais e 0 < a = 1, assinale as verdadeiras:
01. A curva representativa do grfico de f est toda
acima do eixo x.
02. Seu grfico intercepta o eixo y no ponto (0, 1).
04. A funo crescente se 0 < a < 1
08. Sendo a = 1/2, ento f(x) > 2 se x > 1.

12. Determine o domnio da funo abaixo:
7
5
) 4 , 1 ( ) (
5
2
=
x
x f


13. (UEPG-PR) Assinale o que for correto.
01. A funo f(x) = a
x
, 1 < a < 0 e x e R, intercepta o
eixo das abscissas no ponto (1,0)
02. A soluo da equao 2
x
.3
x
=
3
36 pertence ao
intervalo [0, 1]
04. Dada a funo f(x) = 4
x
, ento D = R e I
m
=
*
+
R
08. A funo f(x) = ( )
x
2 crescente
16.
b a
b a
< > |
.
|

\
|
|
.
|

\
|
2
1
2
1


14. Determine o valor de x no sistema abaixo:
1) y e 1 (x > >

=
=
3 5
y x
y x
x y


15. Resolver, em reais, as equaes abaixo:
a) 5
x
+ 0,2
x
= 5,2 b) 5.4
x
+ 2.5
2x
= 7.10
x

UNIDADE 11

LOGARITMOS

DEFINIO
Dado um nmero a, positivo e diferente de um, e um
nmero b positivo, chama-se logaritmo de b na base a ao
real x tal que a
x
= b.
(a > 0 e a = 1 e b > 0)

log
a
b = x a
x
= b

Em log
a
b = x temos que:
a = base do logaritmo
b = logaritmando ou antilogaritmo
x = logaritmo
Observe que a base muda de membro e carrega x como
expoente.

Exemplos:
1) log
6
36 = x 36 = 6
x
6
2
= 6
x
x = 2
2) log
5
625 = x 625 = 5
x
5
4
= 5
x
x = 4

Existe uma infinidade de sistemas de logaritmos. Porm,
dois deles se destacam:

Sistemas de Logaritmos Decimais:
o sistema de base 10, tambm chamado sistema de
logaritmos comuns ou vulgares, ou de Briggs (Henry
Briggs, matemtico ingls (1561-1630)).
Quando a base 10 costuma-se omitir a base na sua
representao.

Sistemas de Logaritmos Neperianos
o sistema de base e (e = 2, 718...), tambm chamado de
sistema de logaritmos naturais. O nome neperiano deve-se
a J. Neper (1550-1617).

Condio de Existncia
Para que os logaritmos existam necessrio que em: log
a
b
= x se tenha :
logaritmando positivo
base positiva
base diferente de 1
Resumindo
b > 0
a > 0 e a 1

=


Consequncias da Definio

Observe os exemplos:
1) log
2
1 = x 1 = 2
x
2
0
= 2
x
x = 0
2) log
3
1 = x 1 = 3
x
3
0
= 3
x
x = 0
3) log
6
1 = x 1 = 6
x
6
0
= 6
x
x = 0

log
a
1 = 0

4) log
2
2 = x 2 = 2
x
2
1
= 2
x
x = 1
5) log
5
5 = x 5 = 5
x
5
1
= 5
x
x = 1

log
a
a = 1

6) log
2
2
3
= x 2
3
= 2
x
x = 3
7) log
5
5
2
= x 5
2
= 5
x
x = 2

log
a
a
m
= m

8) 2 2 4
4 2 log
2
= = = x x x
9) 3 3 9
9 2 log
3
= = = x x x

b
b
a
log
a =


PROPRIEDADES OPERATRIAS

Logaritmo do Produto
O logaritmo do produto igual a soma dos logaritmos dos
fatores.

log
a
(b . c) = log
a
b + log
a
c

Exemplos:
a) log
3
7.2 = log
3
7 + log
3
2
b) log
2
5.3 = log
2
5 + log
2
3

Logaritmo do Quociente
O logaritmo do quociente o logaritmo do dividendo
menos o logaritmo do divisor.
Please purchase PDF Split-Merge on www.verypdf.com to remove this watermark.
Matemtica A Incluso para a Vida

Pr-Vestibular da UFSC
22

log
a
=
c
b
log
a
b log
a
c

Exemplos:
a) log
3
7/2 = log
3
7 - log
3
2
b) log
5
8/3 = log
5
8 - log
5
3

Logaritmo da Potncia
O logaritmo da potncia igual ao produto do expoente
pelo logaritmo da base da potncia.

log
a
x
m
= m . log
a
x


Exemplos: a) log
2
5
3
= 3. log
2
5
b) log
3
4
-5
= -5 log
3
4

Caso Particular
a
n
a a
b
n
b
n
b
log .
1
log log
1
= =


Exemplo: log
10
2
3
= log
10
2
1
3
=
1
3
log
10
2
Exerccio Resolvido:

Sabendo-se que log 2 = 0,30 e log 3 = 0,47. Calcule o
valor de log 18.

Resoluo: log 18 = log(2.3
2
)
log 18 = log 2 + log 3
2

log 18 = log 2 + 2log 3
log 18 = 0,30 + 2.0,47
log 18 = 1,24

Exerccios de Sala #

1. Com base na definio, calcule o valor dos seguintes
logaritmos:

a) log
2
1024
b) log 0,000001

c) log
2
0,25

d) log
4

13
128


2. Sabendo-se que log 2 = 0,30 e log 3 = 0,47, calcule o
valor de:
a) log 6 b) log 8

c) log 5 d)log 18

Tarefa Mnima #

1. Determine o valor dos logaritmos abaixo:

a) log
2
512 b)log
0,25
0,25

c) log
7
1 d)log
0,25

13
128


2. Determine o valor das expresses abaixo
a) 3 log
a
a
5
+ log
a
1 4 l g a
a
o , onde 0 < a = 1, :
b) 5
625
. 16
3
1
9
8
2
g l g l g l o o o + :
3. Sabendo-se que log 2 = 0,30 e log 3 = 0,47, calcule o
valor dos logaritmos abaixo:

a) log 12 b)log 54
c) log 1,5 d) log 512
5


4. (UFPR) Sendo log 2 = 0,301 e log 7 = 0,845, qual
ser o valor de log 28?
a) 1,146 b) 1,447
c) 1,690 d) 2,107 e) 1,107

5. (FEI-SP) A funo f(x) = log (50 5x x
2
) definida
para:
a) x > 10 b) 10 < x < 5
c) 5 < x < 10 d) x < 5 e) n.d.a.

Tarefa Complementar #

6. (PUC-SP) Se l g x o
2 2
512 = , ento x vale:

7. (PUC-SP) Sendo log
10
2 = 0,30 e log
10
3 = 0,47,
ento log
6 2
5
igual a:
a) 0,12 c) 0,32 e) 0,52
b) 0,22 d) 0,42

8. (ACAFE-SC) Os valores de m, com e R, para os quais
a equao x
2
2x + log
2
(m 1) = 0 admite razes (zeros)
reais e distintas so:
a) 2 < m < 4
b) m< 3
c) m s 3
d) 1 s m s 3
e) 1 < m < 3
9. Se log a = r, log b = s, log c = t e E =
3
3
c b
a , ento log E
igual a:

10. (ANGLO) Se log E = 2log a + 3log b log c log d,
Ento E igual a:

11. (UFSC) Se
3 125
14
l g x y l g
l gx l gy l g
o o
o o o
( ) =
+ =

, ento o valor de x
+ y

12. Se x = 360
3
, log
10
2 = 0,301 e log
10
3 = 0,477,
determine a parte inteira do valor de 20 log
10
x.

13. (UMC-SP) Sejam log x = a e log y = b. Ento o log
( ) y x. igual a:
a) a + b/2 b) 2a + b c )a + b d)a+2b e) a-b/2

14. Determine o domnio das seguintes funes:

a) y = log
x 1
(3 x) b) y = log
(5 x)
(x
2
4)

15. Se x a soluo da equao
7
...
=
x
x
x
x
, calcule o valor
da expresso 2x
7
+ log
7
x
7
1
Please purchase PDF Split-Merge on www.verypdf.com to remove this watermark.
Incluso para a vida Matemtica A

Pr-Vestibular da UFSC
23
UNIDADE 12

LOGARITMOS

MUDANA DE BASE
Ao aplicar as propriedades operatrias dos logaritmos
ficamos sujeitos a uma restrio: os logaritmos devem ser
de mesma base. Dado esse problema, apresentamos ento
um processo o qual nos permite reduzir logaritmos de
bases diferentes para bases iguais. Este processo
denominado mudana de base.

log
a
b =
a g l
b g l
c
c
o
o


Como consequncia, e com as condies de existncia
obedecidas, temos:

1) log
log
log log
B
A
A
A
k
A
B
B
k
B = =
1
2
1
)

EQUAO LOGARTMICA
So equaes que envolvem logaritmos, onde a incgnita
aparece no logaritmo, na base ou no logaritmando
(antilogaritmo).
Existem dois mtodos bsicos para resolver
equaes logartmicas. Em ambos os casos, faz-se
necessrio discutir as razes. Lembrando que no existem
logaritmos com base negativa e um, e no existem
logaritmos com logaritmando negativo.

1 Mtodo: log
a
X = log
a
Y X = Y

2 Mtodo: log
a
X = M X = a
M
Funo Logartmica f(x) = log
a
x

(a > 1) funo crescente


(0 < a < 1) funo decrescente



INEQUAO LOGARTMICA

a > 1

log
a
x
2
> log
a
x
1
x
2
> x
1


0 < a < 1

log
a
x
2
> log
a
x
1
x
2
< x
1


Exerccios de Sala #

1. Resolver as equaes abaixo:

a) log
x
(3x
2
- x) = 2

b) log
4
(x
2
+ 3x - 1) = log
4
(5x 1)

c) log
2
(x + 2) + log
2
(x 2) = 5

2. (UFSC) Determine a soma dos nmeros associados (s)
proposio(es) verdadeira(s).
01. O valor do 32 log
25 , 0
igual a
2
5
.
02. Se a, b e c so nmeros reais positivos e x =
c b
a
2
3
,
ento log x = 3log a 2log b
2
1
log c.

04. Se a, b e c so nmeros reais positivos com a e c
diferentes de um, ento tem-se
a l og
b l og
b l og
c
c
a
=
.

08. O valor de x que satisfaz equao 4
x
2
x
= 56 x =
3.




Tarefa Mnima #

1. (SUPRA) Se log
5
2 = a e log
5
3 = b ento log
2
6 :
a+b a b a+b
a) b) a+b c) d) e)
a b a 2


2. (ACAFE) O valor da expresso log
3
2. log
4
3 :
a) c) 4 e) 2
b) 3 d) 2/3


3
2,3
2
1,7
3
2

>
16.
Please purchase PDF Split-Merge on www.verypdf.com to remove this watermark.
Matemtica A Incluso para a Vida

Pr-Vestibular da UFSC
24
3. Resolver, em R as equaes:
a) log
5
(1 4x) = 2
b) log[x(x 1)] = log 2
c)
0 9 log 6 log
3
2
3
= + x x

d) log(log(x + 1)) = 0
e) log
2
(x - 8) log
2
(x + 6) = 3
f) log
5
(x 3) + log
5
(x 3) = 2

4. (UFSC) A soluo da equao:
log
2
(x + 4) + log
2
(x 3) = log
2
18, :

5. Resolver, em reais, as seguintes inequaes:
a) log
2
(x + 2) > log
2
8
b) log
1/2
(x 3) > log
1/2
4

Tarefa Complementar #

6. (UFSC) Dada a funo y = f(x) = log
a
x, com a > 0, a =
1, determine a soma dos nmeros associados s
afirmativas verdadeiras.

01. O domnio da funo f R.
02. A funo f crescente em seu domnio quando
a e (1, + )
04. Se a = 1/2 ento f(2) = 1
08. Se a = 3 e f(x) = 6 ento x = 27
16. O grfico de f passa pelo ponto P(1,0).
7. (ACAFE) Se log
3
K = M, ento log
9
K
2
:
a) 2M
2
c) M + 2 e) M

b) M
2
d) 2M
8. (UFSC) Se log
a
x = 2 e log
x
y = 3, ento, log
a xy
3
5

igual a:

9. (UFSC) Determine a soma dos nmeros associados s
proposies verdadeiras:
01. O valor do log
0,25
32 igual a
5
2
.
02. Se a, b e c so nmeros reais positivos e
x =
a
b c
3
2
ento log x = 3 log a 2log b 1/2 log c.
04. Se a, b e c so nmeros reais positivos com a e c
diferentes de um, ento tem-se log
a
b =
log
c
log
c
b
a

08. O valor de x que satisfaz equao 4
x
2
x
= 56
x = 3
16.
2
3
2
3
2 3 1 7
|
\

|
.
| >
|
\

|
.
|
, ,


10. (UFSC) O valor de x compatvel para a equao
log(x
2
1) - log(x 1) = 2 :

11. (UFSC) Assinale no carto-resposta a soma dos
nmeros associados (s) proposio(es) correta(s).
01. O conjunto soluo da inequao
log (x
2
9) > log (3 x) S = (, 4] [3, +).
02. Para todo x real diferente de zero vale ln |x| < e
x
.
04. A equao
2
x x
e e = no possui soluo inteira.
08. Considere as funes f(x) = a
x
e g(x) = log
a
x. Para
a > 1, temos f crescente e g decrescente e para
0 < a < 1, temos f decrescentes e g crescentes.

16. log 360 = 3 - log 2 + 2 - log 3 + log 5.
32. Se log N = 3,412 ento log N = 6,824.

12. Resolva a equao
l g x l g x o o
10 100
2 + =
. (divida o
resultado obtido por 4).

13. Assinale a soma dos nmeros associados s
proposies corretas:
01. A raiz da equao log(log(x + 1)) = 0 x = 9.
02. A soma das razes da equao.
1 + 2log
x
2 . log
4
(10 x) =
2
log
4
x
10.
04. A maior raiz da equao 9 . x
x log
3
= x
3
9.
08. O valor da expresso log
3
2. log
4
3 /2.
16. Se log
a
x = n e log
a
y = 6n, entol g x y
a
o
2
3

igual a 7n.
32. A soluo da equao 2
x
.3
x
=
3
36 pertence ao
intervalo [0, 1].

14. (UFPR) Com base na teoria dos logaritmos e
Exponenciais, correto afirmar que:
01. Se log
3
(5 y) = 2, ento y = - 4
02. Se x = log
e
3, ento e
x
+ e
-x
=
3
10

04. Se a e b so nmeros reais e 0 < a < b < 1, ento
|log
10
a| < |log
10
b|
08. Se z = 10
t
1, ento z > 0 para qualquer valor real
de t

15. (ITA - SP) O conjunto dos nmeros reais que
verificam a inequao 3log x + log (2x + 3)
3
s 3 log2
dado por:
a) { x e R| x > 3 }
b) { x e R| 1 s x s 3 }
c) { x e R| 0 < x s 1/2 }
d) { x e R| 1/2 < x < 1 }
e) n.d.a.
Please purchase PDF Split-Merge on www.verypdf.com to remove this watermark.
Incluso para a vida Matemtica B

Pr-Vestibular da UFSC
1
UNIDADE 1

POTENCIAO E RADICIAO

POTENCIAO

Definio
Potenciao uma multiplicao de fatores iguais.
Sendo a e R e a = 0 e m e Z. Tem-se que:

a
m
= a. a. a. a. a..... a.
m fatores

Casos Particulares

a
0
= 1 para a = 0
a
1
= a
a
-n
=
1
a
n


Propriedades

Se a e b so nmeros reais e m e n, nmeros inteiros,
tem-se:

- a
m
.a
n
= a
m + n

-
a
a
a
m
n
m n
=


- (a
m
)
n
= a
m.n

- (a.b)
n
= a
n
.b
n

-
a
b
a
b
n n
n
|
\

|
.
| =


Potncia de base 10

Sabe-se que: 10
0
= 1
10
1
= 10
10
2
= 100
10
3
= 1000

Ento 10
n
= 100...........00
n zeros

Observe ainda que: 10
-1
=
10
1
= 0,1
10
-2
=
2
10
1
= 0,01
10
-3
=
3
10
1
= 0,001

Ento 10
n
= 0,000.............001
n casas decimais


RADICIAO
Definio

b a raiz n-sima de a, se b
n
= a.
Representao

n
a = b b
n
= a
Nomenclatura

Em
n
a = b, temos:
- n o ndice
- a o radicando
- b a raiz

Condio de existncia

Em
n
a , se n for par, ento necessrio que a seja maior
ou igual a zero. Se n for mpar ento
n
a sempre existe.

Propriedades

( )
n.m
a
n m
a
n.p
m.p
a
n m
a
n m
a
m
n
a
n
b
a
n
b
n
a
n
a.b
n
b .
n
a
= -
= -
= -
= -
= -

-
n
m
n m
a a =

Racionalizao de denominadores
Dada uma frao com denominador contendo radical,
racionalizar o denominador um processo no qual se
obtm uma frao equivalente a primeira sem, no entanto,
com o radical no denominador.
1 CASO: O denominador do tipo
n m
a
Neste caso, multiplica-se numerador e denominador
pelo fator:
n m n
a

.
2 CASO: O denominador do tipo b a + Neste caso,
multiplica-se numerador e denominador. Pelo fator:
b a

Exerccios de Sala #

1. Calcule:

a) 2
4
d) 1
7
g) 3
-2
b)

2
4
e) 0
3
h)
4
3
2

|
.
|

\
|
c) ( 2)
4
f) 214
0



2. Transforme cada expresso em uma nica potncia de
base 3.
a) 3
7
. 3
-5
. 3
6
= c) (3
4
)
2
=
b)
3
3
5
3 .
2
3
= d)
2
4
3 =

3. Calcule:
a)
25 , 0
d)
3
64
b)
01 , 0
e) ( )
2
4
9
Please purchase PDF Split-Merge on www.verypdf.com to remove this watermark.
Matemtica B Incluso para a Vida

Pr-Vestibular da UFSC
2
c)
3
125 f)
242 2 2 32 50 + +


4. Racionalize:
a)
2
3
b)
5
5
c)
5
2
3
d)
3 5
2
+


Tarefa Mnima#

1. Determine o valor das expresses:

a) 3
4
g) 4
-2

b) 3
4
h)
3
2
5

|
.
|

\
|
c) ( 3)
4
i) 2
4
+ 1
201
+ 0
3
+ 4
0


d) 1
201
j)
4
2
3
)
2
(2
4
2) (

+
e) 0
80
k)
1 2
2
3
3
2

|
.
|

\
|
+ |
.
|

\
|

f) 500
0


2. Transforme cada expresso em uma nica potncia de
base 2.
a) 2
5
.2
3
.2
7
b)
4
2 . ) 2 (
2
3 2 3

3. Sendo A = 2
100
, obtenha:

a) sucessor de A d) quadrado de A
b) o dobro de A e) metade de A
c) qudruplo de A f) raiz quadrada de A

4. Usando a definio, calcule o valor de cada uma das
razes:
a)
4
625 c)
5
0 e)
16
81


b)
5
32 d)
3
1 f)
3
125 , 0


5. Racionalize:
a)
2
5
b)
3
6
c)
3
5
2
d)
2 3
5



Tarefa Complementar#

6. O valor da expresso
01 , 0
) 1 , 0 .( 100
3
equivalente a:

a) 10
2
b) 10
3
c) 10
4
d) 10
5
e) 10

7. Assinale a soma dos nmeros associados s proposies
corretas:
01. O nmero 573 equivalente a 5,73. 10
2

02. O valor da expresso 5.10
8
. 4.10
-2
2.10
7

04. Se n par, ento a expresso ( 1)
2n
+ ( 1)
2n + 1

zero.
08. A metade de 4
8
+ 8
4
17.2
11





8. (Fuvest-SP) Qual desses nmeros igual a 0,064?
2 2 3 2 3
1 1 2 1 8
a) b) c) d) e)
80 8 5 800 10
| | | | | | | | | |
| | | | |
\ . \ . \ . \ . \ .


9. (FGV-SP) Qual o valor da expresso
( ) ( )
( )( )
,
. . . . .
. . . . .
1 1 2 3
2
1
4
2 1 2
b a b a b a
b a b a b a


quando a = 10
3
e b = 10
2

a) 10
6
b) 10
2
c) 10
3
d) 10
9
e) 10
7


10. (FGV-SP) Simplificando a
expresso
1 2
1 2 4
2 2
2 2 2

+ +
+
+ +
n n
n n n
temos:

3
34
d
3
82
c
4
87
b
4
3
a ) ) ) )


11. (Cesgranrio) Se a
2
= 99
6
, b
3
= 99
7
e c
4
= 99
8
, ento
(abc)
12
vale:
a) 99
12
d) 99
88

b) 99
21/2
e) 99
99

c) 99
28


12. Determine a soma dos nmeros associados s
proposies corretas:

01. A expresso
5
80 20 45 + +

equivalente a
15 3

02. O valor de
4 2 2 2 2 + + + +
2
04. O valor de
2
1
3
1
16 8 + 4
08. Racionalizando
2
4
obtm-se 2 2
16. A expresso
3
5
5
3
+
igual a
15
15 8


13. Calculando
3 5
12 13
2 : 2
3 3 +
, acha-se:
a) 3
2
c) 3
6
e) n.d.a.

b) 3
4
d) 3
8



14. (UEL-PR) A expresso
1
2 2
1
2 2
1

equivalente
a) 1 d) 2 1
b) 2 2 e) 2 + 1

c) 2 + 2

15. (UEL-PR) Seja o nmero real
x =
1 5
5 2 2 20 3 500

+
. Escrevendo x na forma x = a
+ b c , tem-se que a + b + c igual a:

a) 5 c) 7 e) 9
b) 6 d) 8



Please purchase PDF Split-Merge on www.verypdf.com to remove this watermark.
Incluso para a vida Matemtica B

Pr-Vestibular da UFSC
3
UNIDADE 2

TRIGONOMETRIA NO TRINGULO
RETNGULO

Considere o tringulo retngulo ABC
Nesse tringulo podemos destacar os seguintes elementos:
-
___
AB e AC
____
so os catetos
-
___
BC a hipotenusa
- C e

B so os ngulos agudos

Pelo teorema angular de Thales prova-se que os ngulos
agudos so complementares, ou seja, C

+ B = 90

RELAES TRIGONOMTRICAS:
- SENO: seno de um ngulo agudo o quociente entre
o cateto oposto ao ngulo e a hipotenusa.
- CO-SENO: co-seno de um ngulo o quociente entre
o cateto adjacente ao ngulo e a hipotenusa.
- TANGENTE: tangente de um ngulo o quociente
entre o cateto oposto ao ngulo e o cateto adjacente.

Sendo assim, temos que:

sen o =
a
b
cos o =
a
c
tg o =
c
b


Observao:

Se o + | = 90 tem-se que sen o = cos |

Tabela de arcos notveis

Observe o tringulo equiltero. Traando uma de suas
alturas, dividimos o tringulo em dois tringulos
retngulos congruentes.

Observe, agora, o quadrado. Nele traamos a diagonal e
obtemos dois tringulos retngulos issceles.

Em resumo, temos:












Exerccios de Sala #

1. (FUVEST) Obter o valor de x na figura:

2. No tringulo ABC, o valor do ngulo o, em graus, :

a) 60 b) 45 c) 30 d) 90 e) n.d.a.

3. (UFSC) Dois pescadores P
1
e P
2
esto na beira de m
rio de margens paralelas e conseguem ver um bote B na
outra margem. Sabendo que P
1
P
2
= 63 m, os ngulos P
1
P
2

= o e BP
2
P
1
= | e que tg o = 2 e tg | = 4, a distncia
entre as margens (em metros) :


Please purchase PDF Split-Merge on www.verypdf.com to remove this watermark.
Matemtica B Incluso para a Vida

Pr-Vestibular da UFSC
4
Tarefa Mnima #

1. Nas figuras abaixo, determinar o valor de x
a)

30
X
12


b)

c)
60
X
6


d)


e)
45
x
5



2. Na cidade de pisa, situada na Itlia, est localizada a
Torre de Pisa, um dos monumentos mais famosos do
mundo. Atualmente a torre faz, na sua inclinao, um
ngulo de 74 com o solo. Quando o sol est bem em cima
da torre (a pino) ela projeta uma sombra de 15 m de
comprimento. A que distncia se encontra o ponto mais
alto da torre em relao ao solo?
(dados: sen 74 = 0,96 cos 74 = 0,28 tg74 = 3,4)

a) 55 metros d) 42 metros c) 45 metros
b) 15 metros e) 51 metros
3. (UFSC) Num vo entre duas paredes, deve-se construir
uma rampa que vai da parte inferior de uma parede at o
topo da outra. Sabendo-se que a altura das paredes de
4 3 m e o vo entre elas de 12m, determine o ngulo,
em graus, que a rampa formar com o solo.

4. Na figura abaixo, determinar o valor de x e y.

Tarefa Complementar #

5. Com base na figura abaixo correto afirmar:
01. h = 2 m
02. h = 3m
04. a = (1 + 3 ) m
08. O tringulo ACD issceles
16. O lado
____
AC mede 6m

6. Um barco navega seguindo uma trajetria retilnea e
paralela costa. Num certo momento, um coqueiro situado
na praia visto do barco segundo um ngulo de 20 com
sua trajetria. Navegando mais 500 m, o coqueiro fica
posicionado na linha perpendicular trajetria do barco.
Qual a distncia do barco costa? (sen 20 = 0,34; cos 20
= 0,93; tg 20 = 0,36)

7. Determine o valor de x e y na figura abaixo:

8. (Unicamp-SP) Uma pessoa de 1,65 m de altura observa
o topo de um edifcio conforme o esquema abaixo. Para
sabermos a altura do prdio, devemos somar 1,65m a:

a) b cos o c) a sen o e) b sen o
b) a cos o d) b tg o

9. (UEPG-PR) Na figura abaixo, em que o ponto B
localiza-se a leste de A, a distncia
___
AB = 5 km. Neste
momento, um barco passa pelo ponto C, a norte de B, e
leva meia hora para atingir o ponto D. A partir destes
dados, assinale o que for correto.
01.
___
AC = 10km
02.
___
AD = 2,5 km
04.
____
BC = 5 3 km
08. O ngulo D A B

mede 60
16. A velocidade mdia do barco de 15km/h

Please purchase PDF Split-Merge on www.verypdf.com to remove this watermark.
Incluso para a vida Matemtica B

Pr-Vestibular da UFSC
5
10. (UFSC) Na figura, abaixo, determine o valor de x

30
60
A
B
C
D

AD = x DC= x - 38 BD = y

UNIDADE 3

TEOREMA DOS CO-SENOS

Num tringulo qualquer, o quadrado da medida de um lado
igual soma dos quadrados das medidas dos outros dois
lados, menos duas vezes o produto das medidas destes
lados pelo co-seno do ngulo formado por eles.


TEOREMA DOS SENOS

Num tringulo qualquer, os lados so proporcionais aos
senos dos ngulos opostos. A razo de proporo o
dimetro da circunferncia circunscrita ao tringulo.



Exerccios de Sala #

1. Determine o valor de x na figura abaixo:
2. (FUVEST) Em um tringulo ABC, AB = 4 2 e o
ngulo C oposto ao lado AB mede 45. Determine o
raio da circunferncia que circunscreve o tringulo

3. Determine o valor de x na figura abaixo

4. Determine o valor da diagonal BD do paralelogramo
abaixo, :




Tarefa Mnima #

1. Determine o valor de x na figura abaixo:
2. (UFSC) Na figura, a medida do lado AC 75 2 cm. A
medida, em cm, do lado AB ser:
A
B C
45 30

3. O tringulo ABC est inscrito na circunferncia de
centro O e raio R. Dado que AC = 2 3 cm, determine a
soma dos nmeros associados s proposies verdadeiras:
75
60
O
A
B
C


01. O tringulo ABC equiltero
02. o raio da circunferncia vale 2cm
04.
___
AB = 2 2 cm
08. O comprimento da circunferncia 4t cm

4. (PUC-SP) Dois lados consecutivos de um
paralelogramo medem 3 2 cm e 5cm e formam um
ngulo de 45. Podemos afirmar que a diagonal menor, em
centmetros, mede:
a) 4 c) 3 e) 4 2
b) 11 d) 13

5. (FUVEST) Um tringulo T tem os lados iguais a 4, 5 e
6. O co-seno do maior ngulo de T :
a) 5/6 c) e) 1/8
b) 4/5 d) 2/3

Tarefa Complementar #

6. (CESGRANRIO) No tringulo ABC, os lados AC e BC
medem respectivamente 8cm e 6cm, respectivamente, e o
ngulo A vale 30. O seno do ngulo B vale:
a) c) e) 5/6
b) 2/3 d) 4/5

7. (FUVEST-SP) Numa circunferncia est inscrito um
tringulo ABC; seu lado
___
BC igual ao raio da
circunferncia. O ngulo B A

C mede:
a) 15 c) 36 e) 60
b) 30 d) 45
Please purchase PDF Split-Merge on www.verypdf.com to remove this watermark.
Matemtica B Incluso para a Vida

Pr-Vestibular da UFSC
6
8. (ITA-SP) Um navio, navegando em linha reta, passa
sucessivamente pelos pontos A, B e C. O comandante,
quando o navio est em A, observa o farol L e mede o
ngulo L A

C = 30. Aps navegar 4 milhas at B, verifica


o ngulo L B

C = 75. Quantas milhas separam o farol do


ponto B?
a) 2 2 c) 2 3
b) 3 d) 3 2 e) 4 2

9. Num tringulo ABC, AB = 5cm, AC = 7cm e BC =
cm. Calcule o comprimento da mediana relativa ao lado
BC.

10. (FUVEST) No quadriltero dado a seguir, BC = CD =
3cm, AB = 2cm, AD

C = 60 e AB

C = 90.
A
B
D
C

O permetro do quadriltero, em cm, :

a) 11 c) 13 e) 15
b) 12 d) 14

UNIDADE 4 e 5

INTRODUO CIRCUNFERNCIA
TRIGONOMTRICA

ARCO DE UMA CIRCUNFERNCIA

Arco de uma circunferncia cada uma das partes que
ficam divididas uma circunferncia por dois quaisquer de
seus pontos.
A cada arco corresponde um ngulo central (ngulo que
possui vrtice no centro da circunferncia).

Para medir arcos e ngulos usaremos o grau e o radiano.

- Graus: Um arco de um grau (1) aquele cujo
comprimento igual a
1
360
do comprimento da
circunferncia.
Logo, a circunferncia tem 360.
Os Submltiplos do Grau so os minutos e segundos:

1 = 60' 1'= 60''

- Radiano: Um radiano um arco cuja medida igual ao
raio da circunferncia onde est contido.
Uma circunferncia de raio unitrio possui 2t radianos.

Pode-se, ento, estabelecer uma relao entre graus e
radianos. Portanto:

360 2t rad
180 t rad

CICLO TRIGONOMTRICO

Quando numa circunferncia de raio unitrio se estabelece
um sentido de deslocamento, diz-se que se define o ciclo
trigonomtrico.

Os eixos x e y dividem o ciclo em quatro partes
denominadas quadrantes.

ORIENTAO

Negativo Horrio
Positivo Horrio Anti



ARCOS CNGRUOS
Dois ou mais arcos so cngruos quando a diferena entre
seus valores um mltiplo de 360.

Please purchase PDF Split-Merge on www.verypdf.com to remove this watermark.
Incluso para a vida Matemtica B

Pr-Vestibular da UFSC
7
Exemplo: 1) 30, 390, 750, 1110..........
Veja que esses arcos possuem a mesma extremidade e
diferem apenas no nmero de voltas.

A expresso x = 30 + 360 . k, com k e Z, denominada
expresso geral do arco de 30, onde 30 a primeira
determinao positiva.

A expresso geral dos arcos cngruos a ele dada por:

o + k . 360, com k e Z.

- Se um arco mede o radianos, a expresso geral dos
arcos cngruos a ele dada por:

o + k . 2t, com k e Z.

SENO e CO-SENO DE UM ARCO

DEFINIO

Considere o arco que possui extremidades na origem do
ciclo trigonomtrico e no ponto M o qual corresponde o
ngulo central o.


Denomina-se sen o a projeo do raio OM, pela
extremidade M do arco sobre o eixo y.
Denomina-se cos o a projeo do raio OM, sobre o eixo x.



2. Sinais








TABELA





Note que: 1 s sen o s 1 e 1 s cos o s 1

OBSERVAO: Com o auxlio da simetria de arcos
possvel determinar os valores de seno e co-seno de arcos
do 2, 3 e 4 quadrantes.

Equaes trigonomtricas num intervalo dado:

Equaes Trigonomtricas so aquelas que envolvem as
funes Trigonomtricas em seus membros.
So exemplos de equaes trigonomtricas:

1) sen x = 1

2) 2cos
2
x + 3cos x - 2 = 0

No possvel estabelecer um mtodo para resolver todas
as equaes trigonomtricas, pois, existe uma infinidade
delas. Para isso apresentaremos alguns tipos bsicos:

- sen x = sen a
x a k
x a k
= +
= +

2
2
t
t t
(congruos)
(suplementares)



Please purchase PDF Split-Merge on www.verypdf.com to remove this watermark.
Matemtica B Incluso para a Vida

Pr-Vestibular da UFSC
8
- cos x = cos a
x a k
a k
= +
= +

2
2
t
t
(congruos)
x (suplementares)


Exerccios de Sala #

1. Expresse em radianos os seguintes arcos:

a) 300 b) 60 c) 12

2. Um arco de 200 equivale em radianos a:

a)
3
2t
b)
2
5t
c) 4t d)
9
10t
e) 6t

3. Calcule a 1 determinao positiva e escreva a
expresso geral dos arcos cngruos a:

a) 930 b)
23
6
t
rad

4. Determine o valor de:

a) sen 150
b) cos 150
c) sen 210
d) cos 210
e) sen 330
f) cos 330

5. Para que valores de m a equao cos x = 2m 5
admite soluo.

a) - 1 s m s 1
b) - 2 s m s 5
c) 2 s m s 3
d) 2 < m < 3
e) 1 < m < 2

Tarefa Mnima #

1. Obter a medida em graus dos seguintes arcos:
a)
3
2t
b)
6
t


2. (UFMG) Transformando 730' em radianos, teremos:
a) t/24 c) t/30 e) 5t/32
b) t/25 d)3t/25

3. Determine o valor da expresso
$ $
$ $ $ $
180 cos 0 sen
270 sen . 180 cos 0 cos . 90 sen
2 2
+
+


4. Se sen x > 0 e cos x < 0, ento x um arco do:
a) 1 quadrante
b) 2 quadrante
c) 3 quadrante
d) 4 quadrante
e) n.d.a.

5. A equao sen x = 2m 5 admite soluo para:
a) 2 s m s 3
b) 1 s m s 4
c) -1 s m s 1
d) 2 < m < 3
e) 0 s m s 1

6. Resolver, no intervalo 0 s x < 2t, as seguintes
equaes:

a) sen x = 1
b) cos x = 0
c) sen x =
2
1


d) cos x =
2
2


7. Sabendo que 0 s x < 2t, o conjunto soluo da
equao: sen
2
x 3sen x 4 = 0 :

a) {90}
b) {-90}
c) {270}
d) {180}
e) {30}

Tarefa Complementar #

8. (Mack-SP) A menor determinao positiva de 4900 :

a) 100 c) 40 e) n.d.a.
b) 140 d) 80

9. (UFPA) Qual a 1 determinao positiva de um arco de
1000?

a) 270 c) 290 e) 310
b) 280 d) 300

10. (SANTO AMARO-SP) s 9 horas e 10 minutos, o
menor ngulo formado pelos ponteiros de um relgio :

a) 135 c) 145 e) n.d.a.
b) 140 d) 150

11. (UFPR) O maior ngulo formado entre os ponteiros de
um relgio, s 23h45min, vale:

a) 18930' c) 270 e) 27750'
b) 27730' d) 25445'

12. (UFSC) O maior valor numrico que y pode assumir
quando
y
37 2senx
3
=

, :

Please purchase PDF Split-Merge on www.verypdf.com to remove this watermark.
Incluso para a vida Matemtica B

Pr-Vestibular da UFSC
9
13. (UFPA) O menor valor positivo que satisfaz a equao
2 sen x = 1 :

a) t/6 c) t/3 e) n.d.a.
b) t/4 d) t/2

14. (UM-SP) O menor valor positivo de x para o qual
9
- cos x
=
1
3
:
2
6 4 3 2 3
t t t t t
a) b) c) d) e)


15. Determinar o nmero de solues da equao
2sen x cos x = sen x no intervalo 0 s x < 2t.

UNIDADE 6

RELAES FUNDAMENTAL DA
TRIGONOMETRIA

- sen
2
o + cos
2
o = 1 (Relao Fundamental)

A relao acima tambm vale para arcos com
extremidades fora do primeiro quadrante.

Exemplos: sen
2
30 + cos
2
30 = 1
sen
2
130 + cos
2
130 = 1

Convm lembrar que se o + | = 90, sen o = cos |.
Logo, vale tambm relaes do tipo:

sen
2
50 + sen
2
40 = 1
sen
2
10 + sen
2
80 = 1

TANGENTE DE UM ARCO

DEFINIO

Associa-se a circunferncia trigonomtrica mais um eixo,
a reta t, que tangencia a circunferncia no ponto P de
coordenadas (1,0). Define-se como tangente do arco PM
ao segmento PQ determinado sobre o eixo das tangentes.


SINAIS

TABELA



EQUAO TRIGONOMTRICA

- tg x = tg a {x a k = + 2 t

Exerccios de Sala#

1. Sabendo que sen x =
3
2
e que t
t
< < x
2
, calcule
cos x:

2. (FCChagas-BA) As sentenas sen x = a e cos x =
2 a 1 so verdadeiras para todo x real, se e somente
se:
a) a = 5 ou a = 1 d) a = 1
b) a = -5 ou a = -1 e) n.d.a.
c) a = 5 ou a = 1

3. Resolver no intervalo 0 s x < 2t, a equao
2cos
2
x = 3sen x

4. Determina o valor de:

a) tg 120 b) tg 210 c) tg 330
Please purchase PDF Split-Merge on www.verypdf.com to remove this watermark.
Matemtica B Incluso para a Vida

Pr-Vestibular da UFSC
10

5. Resolva no intervalo 0 s x < 2t as seguintes equaes:
a) tg x =
3
3
b) tg
2
x 1 = 0

Tarefa Mnima#

1. No intervalo
t
t
2
2
3
< < x
se sen x =
3
1
, calcule
cos x.
2. (UFSC) O valor, em graus, do arco x 0
2
s s x
t
na
equao: 1 cos
2
x + sen x = 0 :

3. O valor de tg 315 + tg 225

4. (UFSC) Considere o ngulo x = 1215. Determine |tg x |

5. Resolva as seguintes equaes no intervalo 0 s x < 2t

a) tg x = 3

b) tg
2
x + tg x = 0

Tarefa Complementar#

6. Determine m de modo que se obtenham
simultaneamente, sen x = m e cos x = m 3 3

7. No intervalo 0 s x < 2t, determine o nmero de
solues para a equao 2cos
2
x = 5 5sen x.

8. (FURG-RS) O valor numrico da funo f(x) = sen2x
tg x + 2cos 3x para x =
4
3t
:

9. (PUC-RS) O valor numrico de
x
x
tg
x
cos 3
4
3
2
2
sen +
para x =
3
t
:

a) 5/2 b) 5/3 c) 3/2 d) 2/5 e) 0

10. No intervalo 0 s x < 2t, a equao 3 tg
2
x + tg x = 0
possui quantas solues?

a) 1 c) 3 e) 5
b) 2 d) 4
UNIDADE 7

RELAES TRIGONOMTRICAS

- sen
2
x + cos
2
x = 1 (Relao Fundamental)

As demais Relaes Trigonomtricas com as condies de
existncia obedecidas so:

tg x =
sen x
cos x
cotg x =
1
tg x


sec x =
1
cos x
cossec x =
x sen
1


A partir da relao sen
2
x + cos
2
x = 1 podemos
estabelecer duas relaes derivadas.

Dividindo a Relao Fundamental por sen
2
x temos:

1 + cotg
2
x = cossec
2
x

E dividindo a Relao Fundamental por cos
2
x temos:

tg
2
x + 1 = sec
2
x

Sinais das Funes Trigonomtricas

1Q 2Q 3Q 4Q
seno e cossecante + +

cosseno e secante +

+
tangente e cotangente +

+


Exerccios de Sala #

1. Determine o valor de:

a) cossec 30 d) cossec 210
b) sec 30 e) sec 315
c) cotg 30 f) cotg 300

2. Sendo sen o =
5
4
e t o
t
2
2
3
< < , calcular:

a) cos o c) cotg o e) cosec o
b) tg o d) sec o

Tarefa Mnima #

1. Determine o valor de:

a) sec 60
o
b)

cossec 150
o
c) cotg 315
o


2. (Faap-SP)Se sen x = 3/5, com x e 4 quadrante,
ento tg x :

a) 3/4 d) 3/4
b) 1/2 e) 4/5
c) 4/5
3. (UFSC) Dados sen x =
3
5
e
t
t
2
< < x , determine o
valor de: 32 tg x + 1

4. (FGV-SP) Simplificando-se a expresso
sena tga coseca
cosa cotga seca
. .
. .
, obtm-se:
a) 0 d) 1
b) sec
2
a e) tg
2
a
c) sen
2
a


Please purchase PDF Split-Merge on www.verypdf.com to remove this watermark.
Incluso para a vida Matemtica B

Pr-Vestibular da UFSC
11
Tarefa Complementar #

5. (UFSC)Sabendo que cossec x = 5/4 e x do primeiro
quadrante, ento o valor da expresso 9.(sec
2
x + tg
2
x) :

6. (UFSC) Calcule o valor numrico da expresso:
( ).( )
( . )
sen30 cos120 cosec150 cotg330
sec300 tg60 cotg225
$ $ $ $
$ $ $

+


7. (UFCE) Para todo x e 1 quadrante, a expresso
(sec x - tg x)(sec x + tg x) sen
2
x igual a:
a) cos
2
x d) sec x + cos x
b) 1 + sen
2
x e) n.d.a.
c) cos x - sen x

8. Determine a soma dos nmeros associados (s)
proposio(es) correta(s).

01. A medida em radianos de um arco de 225
6
11
rad.
02. A menor determinao positiva de um arco de
1000 280.
04. Os valores de m, de modo que a expresso
sen x = 2m 5 exista, esto no intervalo [2,3].
08. sen x > cos x para
4 4
t t
s s x .
16. Se tg x =
4
3
e t < x <
2
3t
, ento o valor de
sen x cos x igual a
5
1
.
32. Se sen x > 0, ento cosec x < 0.
64. A soluo da equao 2sen
2
x + 3sen x = 2 para
0 s x s 2t x =
6
t
ou x =
6
5 t
.

9. (UFSC) Dado sen x =
3
5
e x e
0
2
,
t

(
, calcule o valor
numrico da expresso:
sec x cotgx cosecx tgx
6 senx cosec x
2
2
. .
. .

|
\

|
.
|
1


10. (FATEC) Se x e y so nmeros reais tais que
y =
x x tg x
x tg e e
x x
sec . sec
2
4

, ento:
a) y = e
x
d) y =
x
e
x
sec

b)

y = e
x
(1 + tg x) e) n.d.a.
c) y =
x
e
x
cos








UNIDADES 8 e 9

GEOMETRIA ANALTICA
ESTUDO DO PONTO

O sistema cartesiano ortogonal, como j vimos em
funes, composto por duas retas x e y perpendiculares
entre si, no ponto O (origem). A reta x denominada eixo
das abscissas, e a reta y denominada eixo das ordenadas.
Os dois eixos dividem o plano em quatro regies
denominadas quadrantes numerados no sentido anti-
horrio.


A cada ponto do plano cartesiano est associado um par
ordenado (x, y).

Dizemos que (x
p
, y
p
) so as coordenadas do ponto P, onde
o nmero real x
p
chamado abscissa do ponto e o nmero
real y
p
chamado ordenada do ponto.

OBSERVAES

- Se um ponto pertence ao eixo das abscissas, ento sua
ordenada nula.
P (x
p
, 0)
- Se um ponto pertence ao eixo das ordenadas, ento sua
abscissa nula.
P (0, y
p
)
- Se um ponto P pertence bissetriz dos quadrantes
mpares, ento suas coordenadas so iguais
x
p
= y
p
- Se um ponto P pertence bissetriz dos quadrantes
pares, ento suas coordenadas so simtricas.
x
p
= - y
p

DISTNCIA ENTRE DOIS PONTOS
Dados dois pontos A(x
A
, y
A
) e B(x
B
, y
B
) no plano
cartesiano, a distncia entre eles pode ser calculada em
funo de suas coordenadas. Observe a figura abaixo:


Please purchase PDF Split-Merge on www.verypdf.com to remove this watermark.
Matemtica B Incluso para a Vida

Pr-Vestibular da UFSC
12

O tringulo ABC retngulo em C, ento:
AB AC BC
2 2 2
= +

Da vem a frmula que calcula a distncia entre dois
pontos:

d x x y y
AB B A B A
= + ( ) ( )
2 2


PONTO MDIO DE UM SEGMENTO
Considere um segmento AB de extremidades A(x
A
, y
A
) e
B(x
B
, y
B
). Encontrar as coordenadas do ponto Mdio
M(x
M
, y
M
) encontrar a mdia aritmtica entre as
coordenadas de A e B.

Observe a figura:


Pelo teorema de Tales temos que AM = MB, logo,
no eixo x tem-se:
x
M
x
A
= x
B
x
M

x
x x
M
A B
=
+
2

no eixo y tem-se:
y
M
y
A
= y
B
y
M

y
y y
M
A B
=
+
2


Dessa forma as coordenadas do Ponto Mdio tero as
seguintes coordenadas:
M
x x y y
A B A B
+ + |
\

|
.
|
2 2
,



REA DE UM TRINGULO CONHECENDO AS
COORDENADAS DO VRTICE

Considere o tringulo abaixo:
y
x
y
C
x
A
B
y
A
x
B
A
y
B
x
C
C

Quando conhecemos as coordenadas dos vrtices A, B e C
podemos demonstrar que a rea desse tringulo dada por:

A =
1
1
1
.
2
1
C C
B B
A A
y x
y x
y x


OBSERVAES:

- O determinante
x y
x y
x y
A A
B B
C C
1
1
1
foi tomado em mdulo,
pois a rea indicada por um nmero positivo.
- Se o determinante
x y
x y
x y
A A
B B
C C
1
1
1
for nulo, dizemos
que os pontos esto alinhados.

Exerccios de Sala #

1. Dados os pontos A(3, 6) e B(8, 18), determine:

a) distncia entre A e B
b) Ponto Mdio do segmento AB

2. Sabe-se que o ponto P(a,2) equidistante dos pontos
A(3,1) e B(2,4). Calcule a abscissa a do ponto P.

3. Considere o tringulo de vrtices A(6,8); B(2,3);
C(4,5). O valor da medida da mediana AM do tringulo
ABC :
a) 3 c) 5 c) 7
b) 4 d) 6

4. Os pontos A(2, 4), B(-6, 2) e C(0, -2) so os vrtices de
um tringulo ABC. Calcule a rea desse tringulo.

Tarefa Mnima #

1. (Mack-SP) Identifique a sentena falsa:

a) o ponto (0,2) pertence ao eixo y.
b) o ponto (4,0) pertence ao eixo x.
c) o ponto (500,500) pertence bissetriz dos
quadrantes mpares.
d) o ponto (80,-80) pertence bissetriz dos quadrantes
pares.
e) o ponto ( 3 + 1, 3 + 1) pertence bissetriz dos
quadrantes pares.

2. (Cesgranrio) A distncia entre os pontos M(4,-5) e
N(-1,7) do plano x0y vale:

3. (UFRGS) A distncia entre os pontos A(-2,y) e B (6,7)
10. O valor de y :
a) -1 d) -1 ou 10
b) 0 e) 2 ou 12
c) 1 ou 13

Please purchase PDF Split-Merge on www.verypdf.com to remove this watermark.
Incluso para a vida Matemtica B

Pr-Vestibular da UFSC
13
4. (Cescea-SP) O ponto do eixo das abscissas,
equidistantes dos pontos P(-2,2) e Q(2,6), :
a) A(2,0) d) D(0,2)
b) B(5,0) e) E(4,0)
c) C(3,0)

5. Calcular a rea do tringulo ABC. Dados: A(8, 3); B(4,
7) e C(2, 1)

Tarefa Complementar #

6. (UFSC) Dados os pontos A(-1,-1); B(5,-7) e C(x,2),
determine x sabendo que o ponto C equidistante dos
pontos A e B.

7. (FCC-BA) O tringulo cujos vrtices so os pontos
(1,3), (-2,-1) e (1, -2) :
a) equiltero d) retngulo
b) escaleno e) n.d.a.
c) issceles

8. (PUC-SP) Dados A(4,5), B(1,1) e C(x,4), o valor em
mdulo de x para que o tringulo ABC seja retngulo em
B :

9. (UFJF-MG) Se (2,1), (3,3) e (6,2) so os pontos
mdios dos lados de um tringulo, quais so os seus
vrtices?
a) (-1,2), (5,0), (7,4)
b) (2,2), (2,0), (4,4)
c) (1,1), (3,1), (5,5)
d) (3,1), (1,1), (3,5)

10. (UCP-RJ) A distncia da origem do sistema
cartesiano ao ponto mdio do segmento de extremos
(-2,-7) e (-4,1) :
a) 3 b) 2 c) -3 d) 1 e) 3 2

11. (Mack-SP) A rea de um tringulo 25/2 e os seus
vrtices so (0,1), (2,4) e (-7,k). O valor de k pode ser:
a) 3 b) 2,5 c) 2 d) 4 e) 5

12. A rea do polgono, cujos vrtices consecutivos so:
A(10,4), B(9,7), C(6,10), D(-2,-4) e E(3,-5) em
unidades de rea, :

UNIDADE 10

ESTUDO DA RETA

Pode-se associar a cada reta no plano cartesiano uma
equao. Com tal equao podemos determinar se um
ponto pertence ou no a uma reta. Dois tipos de equao
merecem destaque:
- A Equao Geral
- A Equao Reduzida

EQUAO GERAL DA RETA
A Equao Geral da reta pode ser obtida pela condio de
alinhamento de 3 pontos.
Sejam A(x
A
, y
A
), B(x
B
, y
B
) e um ponto genrico P(x, y).

A, B e P esto alinhados se e s se:
x y
x y
x y
A A
B B
1
1
1
0 =
Desenvolvendo 0
1
1
1
=
B B
A A
y x
y x
y x
temos:

x . y
A
+ x
A
. y
B
+ y . x
B
y
A
. x
B
x . y
B
y . x
A
= 0

(y
A
y
B
) x + (x
B
x
A
) y + x
A
y
B
x
B
y
A
= 0
a b c

Logo: ax + by + c = 0 equao geral da reta.


2. Equao Reduzida da Reta
Pode-se obter a equao reduzida da reta se isolando na
equao geral y.
Veja: ax + by + c = 0
by = ax c

y
a
b
c
b
= substituindo
a
b
por m e
c
b
por n temos:

y = mx + n Equao Reduzida da Reta

No qual o coeficiente m denominado coeficiente angular
da reta, e n o coeficiente linear da reta.

3. Coeficiente Angular e Linear da Reta
Vamos considerar a equao y = mx + n. Sabemos que m
o coeficiente angular da reta e n, o coeficiente linear da
reta.
Vejamos, agora, o significado geomtrico deles.

COEFICIENTE LINEAR
O coeficiente linear vai indicar o ponto em que a reta corta
o eixo y.

COEFICIENTE ANGULAR
Define-se como coeficiente angular da reta a tangente do
ngulo o, onde o indica a inclinao da reta em relao ao
eixo x.
m = tg o ou
A
x
B
x
A
y
B
y
m

=


CASOS PARTICULARES
- Quando a reta paralela ao eixo x o ngulo o igual a
0, logo, o coeficiente angular ser nulo, pois tg 0 = 0.
Please purchase PDF Split-Merge on www.verypdf.com to remove this watermark.
Matemtica B Incluso para a Vida

Pr-Vestibular da UFSC
14


- Quando a reta paralela ao eixo y o ngulo o igual a
90, logo, o coeficiente angular no existe, pois tg 90
no definido.


4. Equao do Feixe de Retas
Pode-se conhecer a equao de uma reta r, quando dado
um ponto Q(x
o
, y
o
) e o coeficiente angular dessa reta. Para
isso, usa-se a relao: y y
o
= m(x x
o
)


Exerccios de Sala #

1. Em relao reta r que passa pelos pontos A(2, 5) e
B(4, 9), determine:

a) equao geral
b) equao reduzida
c) coeficiente angular e linear da reta



2. Determine o coeficiente angular das retas abaixo:

a) r: 2x + 3y + 1 = 0
b)

c)


3. Determine a equao da reta representada pela figura
abaixo:


Tarefa Mnima #

1. Em relao reta r que passa pelos pontos A(1, 2) e
B(2, - 3), determine:
a) equao geral
b) equao reduzida
c) coeficiente angular e linear da reta

2. Considere a reta r indicada pela figura abaixo


Assinale a soma dos nmeros associados s
proposies corretas:
01. A equao da reta r y = x 1
02. o coeficiente linear da reta r 1
04. o menor ngulo que a reta r determina no eixo x
45
o
08. a reta r passa pelo ponto de coordenadas (5, 3)
16. a reta r intercepta o eixo x no ponto de
coordenadas (1,0)

3. Determine a equao da reta r indicada abaixo


4. (FGV-SP) Os pontos A(-1, m) e B(n, 2) pertencem
reta 2x - 3y = 4. A distncia entre A e B :

a) 3 d) 2
b) 3,25 e) 9
c) 2 13

5. (Fac.Moema-SP) O coeficiente linear e angular da
reta 2x 3y + 1 = 0 so, respectivamente:

a) 2 e 3 d) 1/3 e 2/3
b) 2/3 e 1 e) n.d.a.
c) 2/3 e 1/3


Please purchase PDF Split-Merge on www.verypdf.com to remove this watermark.
Incluso para a vida Matemtica B

Pr-Vestibular da UFSC
15
Tarefa Complementar #

6. A equao da reta que passa pelo ponto (2, 4) e tem
coeficiente angular 3.

7. Considere as retas r e s indicadas abaixo:

Determine a soma dos nmeros associados s
proposies corretas:

01. A equao da reta r x + 2y 4 = 0
02. A equao da reta s x y 1 = 0
04. o ponto de interseco das retas r e s possui
coordenadas (2, 1)
08. A reta s passa pelo ponto de coordenadas (6,3)

8. (UFSC) As retas r, dada pela equao 3x - y + 7 = 0,
e s, dada pela equao 4x - y - 5 = 0, passam pelo
ponto P(a,b). O valor de a + b :

9. Calcular a rea da regio limitada pelas retas y = 5,
5x + 2y - 95 = 0, x = 0 e y = 0.

10. (UFPR) No plano cartesiano os pontos A(1, -1),
B(3,1), C(3,5) e D(-1, 5) so os vrtices de um
quadrado. correto afirmar que:

01. a origen do sistema de coordenadas est no interior
do quadrado.
02. a reta r que passa por A e B tem coeficiente
angular 1/2
04. a reta cuja equao x + y 4 = 0 contm a
diagonal BD do quadrado.
08. a reta r do item 04 intercepta o eixo y no ponto
(0, -4)
16. o centro do quadrado o ponto (1,3)

UNIDADE 11

ESTUDO DA RETA

POSIO RELATIVA ENTRE 2 RETAS

No plano cartesiano duas retas r e s podem ser:
- Concorrentes
- Paralelas
- Coincidentes
Considere as retas r e s de equaes:

r = m
1
x + n
1
e s = m
2
x + n
2


Assim, podemos ter as seguintes situaes:

- PARALELAS DISTINTAS:
m
1
= m
2


- PARALELAS COINCIDENTES:
m
1
= m
2
e n
1
= n
2

- CONCORRENTES
m
1
= m
2

- CONCORRENTES E PERPENDICULARES:
m
1
. m
2
= 1

DISTNCIA DE PONTO RETA
Considere um ponto P(x
0
, y
0
) e uma reta r: ax + by + c =
0, a distncia do ponto P a reta r pode ser calculada pela
expresso:



Exemplo: Calcular a distncia entre o ponto P(4, 3) e a reta
r de equao 5x + 2y 6 = 0.

Resoluo:
4
5
20
3 4
6 3 . 2 4 . 5
2 2
= =
+
+
= d d d


Portanto a distncia entre P e r de 4 unidades.

Exerccios de Sala #

1. Considere a reta r indicada pela figura abaixo:

Determinar:
a) a equao da reta s que passa pelo ponto P(3, 5) e
paralela reta r.
b) a equao da reta t que passa pelo ponto P(4, 3) e
perpendicular reta r.

2. Determine a distncia do ponto A(2, 3) reta r de
equao y = 2x + 5.

3. (UFSC) Considere as retas r: kx + 5y -7 = 0 e s: 4x + ky
-5 = 0. Determine a soma dos nmeros associados (s)
proposio(es) verdadeira(s).
01. O valor de k para que a reta r passe pelo ponto
(1, -2) 17.
02. O valor de k para que as retas r e s se interceptam
no ponto
0
7
5
,
|
\

|
.
|
25/7.
Please purchase PDF Split-Merge on www.verypdf.com to remove this watermark.
Matemtica B Incluso para a Vida

Pr-Vestibular da UFSC
16
04. As retas r e s so paralelas para k = 2 5 .
08. A equao geral da reta que perpendicular reta s
no ponto (2,1) 3x + 4y -10 = 0.
16. Sendo k = 0, ento a distncia do ponto (-1,3) reta
r 20.

Tarefa Mnima #

1. (UFRGS) As retas com equaes respectivas 4x + 2y -
4 = 0 e 4x - 3y + 12 = 0:
a) so paralelas
b) so coincidentes
c) so concorrentes mas no perpendiculares.
d) interceptam-se no 1 quadrante e so
perpendiculares.
e) interceptam-se no 4 quadrante e so
perpendiculares.

2. A equao da reta que passa pelo ponto P(-3, 5) e
paralela reta de equao 5x + y = 0 :
a) 5x + y + 10 = 0 d) 5x y 10 = 0
b) 5x + y + 10 = 0 e) 5x + y 10 = 0
c) 5x y + 10 = 0

3. (Cesgranrio-RJ) Se as retas (r) x + 2y + 3 = 0 e (s) ax +
3y + 2 = 0 so perpendiculares, ento o parmetro a vale:
a) 2 b) 2 c) 6 d) 6 e) 3

4. Considere o tringulo de vrtices A(0,0), B(1,4) e
C(4,1). A altura em relao base BC mede:

5. (UEL-PR) A distncia entre as retas de equaes x - y
+ 2 = 0 e 2x - 2y + k = 0 igual a 2 se, e somente se:
a) k = 0 c) k = 8 e) k = -4 ou k = 8
b) k = 4 d) k = 0 ou k = 8

Tarefa Complementar #

6. (UFSC) Dados os pontos A(1, 1), B(1, 3) e C(2, 7),
determine a medida da altura do tringulo ABC relativa ao
lado BC.

7. (UFSC) De acordo com o grfico abaixo, assinale
a(s) proposio(es) verdadeira(s).

01. A equao da reta s 3x 2y + 6 = 0.
02. A reta s e a reta r so perpendiculares.
04. As retas r e s se interceptam no ponto de
abscissa
5
4
.
08. A distncia da origem do sistema de coordenadas
cartesianas reta r de
2
2
unidades.
16. A rea da regio do plano limitada pelas retas r, s e
pelo eixo das abscissas igual a
10
3
unidades de rea.
8. (UFRGS) Os pontos A(-1,3) e B(5,-1) so
extremidades de uma das diagonais de um quadrado. A
equao da reta suporte da outra diagonal :

a) 2x - 3y - 1 = 0
b) 2x + 3y - 7 = 0
c) 3x + 2y - 8 = 0
d) 3x - 2y - 4 = 0\

9. A medida da altura do trapzio cujos vrtices so os
pontos A(1, 1), B(6, 1), C(2, 3) e D(4, 3) :

10. ( U. E. Maring-PR ) Considere as retas r, s e t, dadas
no grfico ao lado. Sabe-se que a equao de r 2y = x
3, que os pontos B e C so simtricos em relao ao eixo
das abscissas, que as retas r e s so paralelas e que t
perpendicular a r. Nessas condies, correto afirmar que:


01. o ponto A sobre o eixo x, interseo de r e t, (2,0).
02. o ponto C (0,
2
3
).
04. a distncia entre r e s 3.
08. os coeficientes angulares das retas r, s e t so,
respectivamente,
2
1
,
2
1
e 2.
16. a equao da reta t y = 2x + 6.
32. a equao da reta horizontal que passa por A
x = 0.
64. a equao da reta vertical que passa por A x = 3.

UNIDADE 12

GEOMETRIA ANALTICA
ESTUDO DA CIRCUNFERNCIA

DEFINIO

Recebe o nome de circunferncia o conjunto de pontos de
um plano o que se equidistam de um ponto C denominado
centro da circunferncia. Essa distncia denominada raio
da circunferncia.



R
C

o


Please purchase PDF Split-Merge on www.verypdf.com to remove this watermark.
Incluso para a vida Matemtica B

Pr-Vestibular da UFSC
17

EQUAO DA CIRCUNFERNCIA

Seja C(a, b) o centro da circunferncia e P(x, y) um ponto
genrico pertencente circunferncia, a distncia de C a P
o raio da circunferncia.
Pode-se escrever a equao da circunferncia das seguintes
formas:

Equao Reduzida:

(x a)
2
+ (y b)
2
= R
2



Exemplo: Determine equao da circunferncia de raio
3 e centro C(2, 5):

Resoluo: (x o)
2
+ (y |)
2
= R
2

(x 2)
2
+ (y 5)
2
= 3
2

Logo, a equao procurada : (x 2)
2
+ (y 5)
2
= 9

CASO PARTICULAR: Se a circunferncia possuir
centro na origem ento a equao
(x o)
2
+ (y |)
2
= R
2

fica reduzida a: x
2
+ y
2
= R
2

Equao Geral:

A Equao Geral da circunferncia obtida
desenvolvendo a equao reduzida. Veja:

(x a)
2
+ (y b)
2
= R
2

x
2
2ax + a
2
+ y
2
2by + b
2
= R
2

x
2
+ y
2
2ax 2by + a
2
+ b
2
R
2
= 0

x
2
+ y
2
+ Ax + By + C = 0

onde: A = 2a; B = 2b; C = a
2
+ b
2
R
2


Exemplo: Determinar a equao geral da circunferncia
de raio 3 e centro C(2, 5)

Resoluo: (x o)
2
+ (y |)
2
= R
2

(x 2)
2
+ (y 5)
2
= 3
2

(x 2)
2
+ (y 5)
2
= 9
x
2
4x + 4 + y
2
10y + 25 9 = 0
Logo, a equao geral x
2
+ y
2
4x 10y + 20 = 0

CONDIO DE EXISTNCIA
Vamos comparar a equao de uma circunferncia com
uma equao do 2 grau completa.
x
2
+ y
2
+ Kxy + Ax + By + C = 0

Sendo assim, essa equao s ir representar a equao de
uma circunferncia se e s se:

- Os coeficientes de x
2
e y
2
forem iguais e diferentes de
zero.
- No existir termo em xy, ou seja ter K = 0.
- A
2
+ B
2
4AC > 0

POSIES RELATIVAS DA CIRCUNFERNCIA

Ponto e Reta
Dado um ponto P(x
P
, y
P
) do plano e uma circunferncia
(x o)
2
+ (y |)
2
= R
2
. Em relao a circunferncia, o
ponto P pode assumir as seguintes posies:

Para determinar a posio do ponto P em relao a
circunferncia, substitui-se as coordenadas de P na
equao da circunferncia. Assim, podemos ter:

- (x
P
o)
2
+ (y
P
|)
2
R
2
< 0 P interior
circunferncia
- (x
P
o)
2
+ (y
P
|)
2
R
2
= 0 P pertence
circunferncia
- (x
P
o)
2
+ (y
P
|)
2
R
2
> 0 P exterior
circunferncia

Reta e Circunferncia
Dada uma reta ax + by + c = 0 do plano, e uma
circunferncia (x o)
2
+ (y |)
2
= R
2
. Em relao
circunferncia, a reta pode assumir as seguintes posies:


Para determinar a posio da reta r em relao
circunferncia, substitui-se a equao da reta na equao
da circunferncia. Assim, teremos uma equao do
2 Grau. Ento, se:

- A < 0 reta externa (no existe ponto de interseco)
- A = 0 reta tangente (existe um ponto de interseco)
- A > 0 reta secante (existe dois pontos de
interseco)

Caso exista o(s) ponto(s) de interseco, esse(s) so
obtidos por um sistema de equaes.

Exerccios de Sala #

1. Determinar a equao da circunferncia na forma
reduzida de centro C e raio R nos seguintes casos:
a) C(4, 7) e R = 2 d) C(0, 3) e R =
5

b) C(2, -3) e R = 5 e) C(0, 0) e R = 3
c) C(3, 0) e R =
5



Please purchase PDF Split-Merge on www.verypdf.com to remove this watermark.
Matemtica B Incluso para a Vida

Pr-Vestibular da UFSC
18
2. A soma das coordenadas do centro da circunferncia de
equao x
2
+ y
2
- 4x - 6y - 12 = 0, :
a) 4 c) 6 e) 8
b) 5 d) 7

3. (UFSC) Seja C uma circunferncia de equao x
2
+ y
2
-
2x -2y -6 = 0, e seja r a reta de equao x + y = 6.
Determine a soma dos nmeros associados (s)
proposio(es) verdadeira(s).

01. Em coordenadas cartesianas, o centro e o raio da
circunferncia C so (1,1) e 2 2 respectivamente.
02. A circunferncia C limita um crculo cuja rea
8t.
04. Com relao posio de C e r, pode-se afirmar
que C e r so secantes.
08. A circunferncia de centro no ponto (0,0) e raio
2 tangente externamente circunferncia C.
16. Com relao posio do ponto P(2,3) e C, pode-
se afirmar que o ponto P exterior C.
Tarefa Mnima #

1. A equao da circunferncia de centro C(-2,2) e
tangente aos eixos coordenados :
a) (x + 2)
2
+ (y 2)
2
= 4
b) (x 3)
2
+ (y 3)
2
= 4
c) (x + 2)
2
+ (y + 2)
2
= 2
d) (x 2)
2
+ (y 2)
2
= 4
e) (x + 2)
2
(y 2)
2
= 4

2. (ACAFE-SC) A circunferncia de equao x
2
+ y
2
+ 6x
4y q = 0 tem raio igual a 4. O valor de q :
a) 2 d) 2
b) 3 e) 1
c) 3

3. O centro da circunferncia x
2
+ y
2
8x 4y + 15 = 0
um ponto localizado no:
a) primeiro quadrante d) quarto quadrante
b) segundo quadrante e) eixo x
c) terceiro quadrante

4. (UECE) Sejam M(7,-2) e N(5,4). Se C
1
uma
circunferncia que tem o segmento MN como um
dimetro, ento a equao de C
1
:
a) x
2
+ y
2
- 12x - 2y + 27 = 0
b) x
2
+ y
2
+ 12x - 2y + 27 = 0
c) x
2
+ y
2
+ 12x + 2y + 27 = 0
d) x
2
+ y
2
- 12x + 2y + 27 = 0

5. (PUC-SP) Seja a circunferncia , de equao x
2
+ y
2
-
4x = 0. Determinar a rea da regio limitada por .
a) 4t c) 5t e) n.d.a.
b) 2t d) 3t


Tarefa Complementar #

6. (Mack-SP) O maior valor inteiro de k, para que a
equao x
2
+ y
2
+ 4x - 6y + k = 0 represente uma
circunferncia, :

a) 10 c) 13 e) 16
b) 12 d) 15

7. (UFRGS) O eixo das abscissas determina no crculo
x
2
+ y
2
- 6x + 4y 7 = 0 uma corda de comprimento

8. (FGV-SP) A reta 3x + 4y - 6 = 0 determina na
circunferncia x
2
+ y
2
- 2x - 4y + 1 = 0 uma corda de
comprimento igual a:

a) 3 c) 2 3 e) 2 2
b) 3 d) 6

9. Calcule a rea do crculo de centro (2, 5) sabendo que
a reta 3x + 4y - 6 = 0 tangente a circunferncia.

a) 16t c) 2t e) n.d.a.
b) 4t d) 32t

10. (UFSC) Considere a circunferncia C:
( ) ( ) 16 3 4
2 2
= + y x e a reta r: 4x + 3y 10 = 0.

Assinale no carto-resposta a soma dos nmeros
associados (s) proposio(es) correta(s).

01. r C = C.
02. O centro de C o ponto (3, 4).
04. A circunferncia C intercepta o eixo das abscissas
em 2 (dois) pontos e o das ordenadas em 1 (um)
ponto.
08. A distncia da reta r ao centro de C menor do
que 4.
16. A funo y dada pela equao da reta r
Decrescendo.

Please purchase PDF Split-Merge on www.verypdf.com to remove this watermark.
Incluso para a vida Matemtica C

Pr-Vestibular da UFSC 1
UNIDADE 1

NMEROS PROPORCIONAIS

RAZES E PROPORES
Razo a comparao obtida pela diviso entre as medidas
de duas grandezas na mesma unidade.
Ento, dados dois nmeros a e b , denomina-se razo ao
quociente de a por b e indica-se por
b
a

Obs.: a razo
b
a
usualmente lida assim: a est para b.
A igualdade entre duas razes uma proporo.

Representao:
d
c
b
a
=

onde: a, d = extremos b, c = meios
A expresso
d
c
b
a
=
l-se assim: a est para b, assim como
c est para d.

Observaes:

Considere os conjuntos A = {a, b, c} e B = {d, e, f} duas
sucesses numricas dadas nessa ordem.

- A e B so diretamente proporcionais se:

k
f
c
e
b
d
a
= = =

k a constante de proporo.

Propriedade:
f e d
c b a
f
c
e
b
d
a
+ +
+ +
= = =


- A e B so inversamente proporcionais se:
a . d = b . e = c . f = k
Propriedade: a . d = b . e = c . f =
f
1
c
e
1
b
d
1
a
= =

Exerccios de Sala #

1. Um automvel percorre 160km em 2 horas. A razo
entre a distncia percorrida e o tempo gasto para percorr-
la :

2. Determine dois nmeros, sabendo que a soma deles
42 e que a razo entre eles
4
3
.

3. a) Dividir 150 em partes diretamente proporcionais a 3,
5 e 7.

b) Dividir 14 em partes inversamente proporcionais a
3 e 4.





Tarefa Mnima #

4. Em uma universidade foram inscritos 3450 candidatos
para o curso de Odontologia. Sabendo que foram
fornecidas 100 vagas, qual a razo do nmero de
candidatos em relao ao nmero de vagas?

5. Determine dois nmeros, sabendo que a soma deles
60 e que a razo entre eles
3
2
.
6. Determine os valores de x e y sendo: x y = 10 e
3
1
x
y
=


7. Se (2, 3, x) e (8, y, 4) so duas sucesses de nmeros
diretamente proporcionais, ento:

a) x = 1 e y = 6
b) x = 2 e y = 12
c) x = 1 e y = 12
d) x = 4 e y = 2

8. Divida o nmero 360 em partes proporcionais aos
nmeros 2, 3, 4 e 6.

Tarefa Complementar#

9. Divida o nmero 220 em partes inversamente
proporcionais aos nmeros
7
4
4
3
,
3
2
e
.

10. A diferena entre as idades de duas pessoas 15 anos
e esto entre si como 7 para 4. Calcule as idades dessas
pessoas.

11. (PUC-SP) Se (9, x, 5) e (y, 8, 20) sejam diretamente
proporcionais, isto, , para que se verifique a igualdade
20
5
8
x
y
9
= = , os valores de x e y devem ser
respectivamente:
a) 2 e 36 d) 5 e 35
b)
5
1
e
4
1

e) n.d.a.
c) 2 e 5

12. (F.Carlos Chagas) Se as sequncias (a, 2, 5) e (3, 6, b)
so de nmeros inversamente proporcionais e a + mb =
10, ento m igual a:

a) 0,4 c) 2,0 e) 5,0
b) 1,0 d) 2,5

13. p inversamente proporcional a q + 2. Sabendo que
p = 1 quando q = 4, quanto vale p quando q = 1?

a) 2 c) 0,5 e) 3
b) 0 d) 2

Please purchase PDF Split-Merge on www.verypdf.com to remove this watermark.
Incluso para a vida Matemtica C

Pr-Vestibular da UFSC 2
14. (UFMG) Sabendo-se que x + y + z = 18 e que
4 3 2
z y x
= = , o valor de x :

15. (UFSC) O permetro de um terreno 72 m. As
medidas de seus lados so inversamente proporcionais
a 2, 3, 5 e 6. A medida, em metros, do menor lado
desse terreno, :

16. (UFBA) Sabe-se que das 520 galinhas de um avirio,
60 no foram vacinadas, e 92, vacinadas, morreram.
Entre as galinhas vacinadas, a razo do nmero de
mortas para o nmero de vivas :
1 1 4 4
a) b) c) d) e) n.d.a.
4 5 1 5


17. (FUVEST) Na tabela abaixo, y inversamente
proporcional ao quadrado de x. Calcule os valores de p e
m.
x Y
1 2
2 p
m 8

18. Num tanque de combustvel h 5 litros de leo e 25
litros de gasolina. Determinar as razes das medidas.
a) do leo para a gasolina
b) da gasolina para a mistura
c) do leo para a mistura

UNIDADE 2

GEOMETRIA PLANA

NGULOS

ngulo a regio formada por duas semi retas que tm a
mesma origem (vrtice).

O ngulo formado o ngulo AB no qual:
OA e OB so os lados do ngulo e O o vrtice

UNIDADES ANGULARES

Sistema Sexagesimal (Grau)

1 grau
360
1
da circunferncia.

Submltiplos do Grau: 1 = 60 e 1= 60

Os ngulos recebem nomes especiais de acordo com a sua
abertura.



ngulo Agudo

ngulo Reto

ngulo Obtuso


Dois ngulos o e | podem ser:

a) complementares: o + | = 90
b) suplementares: o + | = 180
c) replementares: o + | = 360

NGULOS OPOSTOS PELO VRTICE



Dois ngulos opostos pelo vrtice so congruentes.

NGULOS FORMADOS POR DUAS PARALELAS E
UMA TRANSVERSAL

Tringulos

Dados os pontos A, B e C no alinhados, chama-se
tringulo A, B, C (indicado por: AABC) reunio dos
segmentos AB, AC e BC.


Pode-se classificar um tringulo segundo dois critrios:

Quanto aos lados

Please purchase PDF Split-Merge on www.verypdf.com to remove this watermark.
Incluso para a vida Matemtica C

Pr-Vestibular da UFSC 3
Quanto aos ngulos

CRITRIOS: Sejam a, b e c lados de um tringulo e
considerando a, o lado maior temos:

- a
2
< b
2
+ c
2
tringulo acutngulo
- a
2
= b
2
+ c
2
tringulo retngulo
- a
2
> b
2
+ c
2
tringulo obtusngulo

NGULOS NUM TRINGULO


A + B + C = 180

Tringulo Equiltero
Se AB = BC = AC ento A = B = C = 60

Tringulo Retngulo


Exerccios de Sala #

1. (UFMA) Dois ngulos opostos pelo vrtice medem 3x
+ 10 e x + 50. Um deles mede:

2. Um ngulo mede a metade do seu complemento.
Ento, esse ngulo mede:
a) 30 c) 60 e) 15
b) 45 d) 80

3. Em cada figura abaixo, determine o valor de x.
a) r //s
b) ABCD um quadrado. ABE um tringulo
equiltero.

Tarefa Mnima #

4. (ACAFE) Dois ngulos opostos pelo vrtice medem 8x
40 e 6x 20. O valor do ngulo :

a) 80 b) 70 c) 40 d) 20 e) 10

5. Um ngulo mede o triplo do seu suplemento. Ento
esse ngulo mede:

a) 45 b) 135 c) 100 d) 175

6. Determine o valor de x na figura abaixo:
x
s
r s //
25
130


7. Nas figuras abaixo, o valor de x :

a)


b)

c)


d)



8. (FUVEST) Na figura, AB = BD = CD. Ento:



a) y = 3x c) x + y = 180 e) 3x = 2y
b) y = 2x d) x = y

Please purchase PDF Split-Merge on www.verypdf.com to remove this watermark.
Incluso para a vida Matemtica C

Pr-Vestibular da UFSC 4
Tarefa Complementar #

9. (UFSC) Na figura r e s so paralelas. O valor, em graus,
do arco x :

10. (UECE) O ngulo igual a 5/4 do seu suplemento
mede:

a) 100 c) 36 e) n.d.a.
b) 144 d) 80

11. (UFSC) Na figura abaixo, o valor em graus da
diferena x y :
23
o
y
x
112
o
r
s
t
r // s // t


12. (UFSC) Na figura abaixo as retas r e s so paralelas.
A medida do ngulo y, em graus, :

13. (Cesgranrio) Duas retas paralelas so cortadas por
uma transversal de modo que a soma de dois ngulos
agudos formados vale 72. Ento qualquer dos ngulos
obtusos formados mede:

a) 142 c) 148 e) 152
b) 144 d) 150

14. (Fuvest-SP) Na figura, as retas r e s so paralelas, o
ngulo 1 mede 45 e o ngulo 2 mede 55. A medida em
graus do ngulo 3 :



a) 50 c) 60 e) 100
b) 55 d) 80

15. Sabendo que o complemento de um ngulo est para o
seu suplemento assim com 2 est para 5, calcule em
graus a medida do ngulo:

16. Na figura a seguir, r//s. Determine o valor de y.

60
70
Y
r
s


17. Na figura , o valor de x :


UNIDADE 3

ESTUDO DOS POLGONOS

ELEMENTOS


CLASSIFICAO
Os polgonos podem ser classificados quanto o nmero de
lados. Os mais conhecidos so:

- Tringulos - 3 lados
- Quadrilteros - 4 lados
- Pentgono - 5 lados
- Hexgono - 6 lados
- Heptgono - 7 lados
- Octgono - 8 lados
Please purchase PDF Split-Merge on www.verypdf.com to remove this watermark.
Incluso para a vida Matemtica C

Pr-Vestibular da UFSC 5
- Enegono - 9 lados
- Decgono - 10 lados
- Undecgono 11 lados
- Dodecgono - 12 lados
- Pentadecgono 15 lados
- Icosgono - 20 lados

Observao: Um polgono dito regular se for equiltero
(lados iguais) e equingulo (ngulos iguais).

NMERO DE DIAGONAIS
O nmero de diagonais de um polgono de n lados dado
pela expresso:


SOMA DOS NGULOS INTERNOS
A soma dos ngulos internos de um polgono com n lados
(n > 3) dado pela expresso:



SOMA DOS NGULOS EXTERNOS
A soma dos ngulos externos de um polgono com n lados
(n > 3) sempre igual a 360

Observaes

- Para polgonos regulares, podemos calcular cada
ngulo interno ou externo atravs das seguintes
relaes:



- Sendo n o nmero de lados de um polgono, se n par,
ento n/2 o nmero de diagonais que passam pelo
centro.
- Se n mpar, no h diagonais que passam pelo centro.

POLGONOS REGULARES

Um polgono regular quando tem lados e ngulos
congruentes. Todo polgono regular inscritvel e
circunscritvel a uma circunferncia.

Nomenclatura

" o lado do polgono
R o raio da circunferncia circunscrita ao polgono
a o raio da circunferncia inscrita ou aptema





Tringulo Equiltero

h



Quadrado



Hexgono Regular



Exerccios de Sala #

1. (ACAFE) Diagonal de um polgono convexo o
segmento de reta que une dois vrtices no
consecutivos do polgono. Se um polgono convexo
tem 9 lados, qual o seu nmero total de diagonais?
a) 72 b) 63 c) 36 d) 27 e) 18

2. Em um icosgono regular ABCDE... calcule:
a) a soma dos ngulos internos.
b) a soma dos ngulos externos.
c) cada ngulo interno e externo

3. (UFSC) Considere um hexgono equingulo (ngulos
internos iguais) no qual quatro lados consecutivos medem
20 cm, 13 cm, 15 cm e 23 cm, conforme figura abaixo.
Calcule o permetro do hexgono.







E D
C
B A
F
20
13
15
23
Please purchase PDF Split-Merge on www.verypdf.com to remove this watermark.
Incluso para a vida Matemtica C

Pr-Vestibular da UFSC 6
4. Num quadrado de lado 10cm est circunscrita uma
circunferncia cujo raio, em centmetros, igual a:
a) 5 2 d) 20 2
b) 10 e) 3 2
c) 10 2

5. (VUNESP) A distncia entre dois lados paralelos de
um hexgono regular igual a 2 3 cm. A medida do
lado desse hexgono, em centmetros, :
a) 3 c) 4 e) 2,5
b) 2 d) 3

Tarefa Mnima #

6. O polgono que tem o nmero de lados igual ao
nmero de diagonais o:

a) hexgono d) heptgono
b) pentgono e) no existe
c) tringulo

7. Cada ngulo interno de um decgono regular mede:

a) 230 c) 144 e) 150
b) 130 d) 28
8. Qual o polgono regular cujo ngulo interno o triplo
do externo?

a) Dodecgono d) Heptgono
b) Pentgono e) Hexgono
c) Octgono

9. Dado uma crculo de raio 10cm. Determine:

a) o lado do tringulo equiltero inscrito nesse crculo



b) o lado do hexgono inscrito nesse crculo


c) o lado do quadrado inscrito nesse crculo


10. O lado de um tringulo equiltero inscrito numa
circunferncia mede 2 6 cm. Determine a medida da
altura do tringulo.
a) 2 2 b) 2 c) 3 2 d) 2 e) n.d.a.

11. (ACAFE) O dimetro mnimo de um tronco de
rvore, para que dele se possam fazer postes quadrados,
cujas arestas das bases meam 20cm, :
a) 10cm b) 40cm c) 30cm d) 20 2 cm e) 80 cm

Tarefa Complementar #

12. (UNICamp) O polgono convexo cuja soma dos
ngulos internos mede 1.440 tem exatamente:

a) 15 diagonais d) 30 diagonais
b) 20 diagonais e) 35 diagonais
c) 25 diagonais

13. (UNIFEI-MG) Achar dois polgonos regulares cuja
razo entre os ngulos internos 3/5 e a razo entre o
nmero de lados 1/3.

14. ( MACK-SP ) Os ngulos externos de um polgono
regular medem 20. Ento o nmero de diagonais desse
polgono :

a) 90 d) 135
b) 104 e) 152
c) 119

15. (PUC-SP) A figura mostra um hexgono regular de
lado a. A diagonal AB mede:
A
B
a) 2a c)
2
3 a
b) a 2 d) a 3 e)
3
2 a 2

16. (ACAFE) A razo entre os comprimentos das
circunferncias circunscrita e inscrita a um quadrado :
a) 2 c) 2 2

e )
2
3

b) 3 d) 2 3

17. (FUVEST) A, B, C e D so vrtices consecutivos de
um hexgono regular. A medida, em graus de um dos
ngulos formados pelas diagonais AC e BD :
a) 90 c) 110 e) 150
b) 100 d) 120

18. Calcule a medida do ngulo central de um enegono
Regular.

19. Qual a razo entre os raios dos crculos circunscrito e
inscrito de um tringulo equiltero de lado a?
Please purchase PDF Split-Merge on www.verypdf.com to remove this watermark.
Incluso para a vida Matemtica C

Pr-Vestibular da UFSC 7
20. Determinar em funo do raio R, o lado de um
decgono regular inscrito numa circunferncia de raio R.

UNIDADE 4

CIRCUNFERNCIA

ELEMENTOS


Raio: segmento CB.
Corda: segmento MN.
Dimetro: segmento AB.

NGULOS DA CIRCUNFERNCIA
ngulo Central: ngulo que tem vrtice no centro da
circunferncia.

ngulo Inscrito: ngulo que tem vrtice na
circunferncia.

Propriedade:

Consequncias

Se um tringulo inscrito numa semicircunferncia tem um
lado igual ao dimetro, ento ele um tringulo retngulo.









ngulo excntrico (fora do centro) interior



ngulo excntrico (fora do centro) exterior




Quadriltero Inscrito na circunferncia




SEGMENTOS TANGENTES



TEOREMA DE PITOT
Em todo quadriltero convexo circunscrito a uma
circunferncia a soma de dois lados opostos igual a soma
dos outros dois:





Please purchase PDF Split-Merge on www.verypdf.com to remove this watermark.
Incluso para a vida Matemtica C

Pr-Vestibular da UFSC 8
SEMELHANA DE TRINGULOS
TRINGULO RETNGULO

SEMELHANA DE TRINGULOS
Dois tringulos so semelhantes se e somente se os
ngulos internos forem congruentes e os lados
proporcionais. Assim temos:



=
= = = =
=
F

k
f
c
e
b
d
a
ento E

: Se


k a constante de proporo ou constante de semelhana.

Observao: As medidas dos permetros de dois
tringulos semelhantes so proporcionais s medidas de
dois lados homlogos quaisquer.

Tringulo Retngulo relaes mtricas

Considere o tringulo abaixo, retngulo em A.


Seus elementos so:
a: hipotenusa
b e c: catetos
h: altura relativa hipotenusa
n e m: projees ortogonais dos catetos sobre a
hipotenusa.

Relaes Mtricas

Atravs da semelhana de tringulos podemos estabelecer
as seguintes relaes:
a
2
= b
2
+ c
2
(teorema de Pitgoras)
a.h = b.c
b
2
= a.n
c
2
= a.m
h
2
= m.n










Exerccios de Sala #

1. Determine o valor de x em cada caso abaixo:

a)

b)

x 20
O

c)


2. Determine o valor do complemento do ngulo x
indicado na figura abaixo:
x
40


3. A circunferncia est inscrita no tringulo ABC
( AB=8, AC=9 e BC=7 ). Ento, x vale:
A
B P C
x
a) 1,5 c) 3,0 e)5,0
b) 2,8 d) 4,6







Please purchase PDF Split-Merge on www.verypdf.com to remove this watermark.
Incluso para a vida Matemtica C

Pr-Vestibular da UFSC 9
4. Na figura abaixo os ngulos CD e A B

D so
congruentes. Ento, o valor de x :

a) 42 c) 21 e) 10
b) 32 d) 60

Tarefa Mnima #

5. Nas figuras abaixo, determine o valor de x:


6. (ACAFE) Na figura a seguir, o valor de x :

3x 150
A
B
C
O


a) 25 c) 50 e) 100
b) 30 d) 75

7. (PUC-SP) Na figura, AB dimetro. O menor dos
arcos (AC) mede:
40
A B
C


8. (FUVEST) O valor de x na figura a seguir :

3
x
2
10






9. (UFSC) Na figura ao lado, AC paralelo a DE. Nessas
condies, determine o valor de x + y.
A y D 18 B
15
C
E
10
x
10


Tarefa Complementar #

10. (FUVEST) A medida do ngulo ADC inscrito na
circunferncia de centro O :


11. (FUVEST) Na figura abaixo, ABCDE um
pentgono regular. A medida em graus do ngulo o :


12. Na figura abaixo, o tringulo ABC retngulo em A,
e o ngulo ACB mede 20. Determine a medida do
ngulo agudo formado pela mediana AM e a altura AH do
tringulo.


13. Na figura, PA = 16 cm e A, B e C so pontos de
tangncia. Calcule o permetro do tringulo PRS.

14. Sendo O o centro da circunferncia circunscrita no
pentgono abaixo, calcule x + y.



Please purchase PDF Split-Merge on www.verypdf.com to remove this watermark.
Incluso para a vida Matemtica C

Pr-Vestibular da UFSC 10

15. Determine o permetro do quadriltero a seguir:
3x + 1
3x
2x
x+1


16. (ACAFE) Os lados de um tringulo medem 3cm, 7cm
e 9cm. Calcule os lados de um segundo tringulo
semelhante ao primeiro, cujo permetro mede 38cm.
a) 8cm, 14cm e 16cm d) 10cm, 13cm e 15cm
b) 6cm, 14cm e 18cm e) 5cm, 14cm e 19cm
c) 3cm, 7cm e 9cm

17. (UNICAMP) A figura mostra um segmento AD
dividido em trs partes: AB = 2cm, BC = 3cm e CD =
5cm. O segmento AD mede 13cm e as retas BBe CC
so paralelas a DD. Determine os comprimentos dos
segmentos AB, BC e CD


18. ( FUVEST ) No tringulo acutngulo ABC a base AB
mede 4cm, e a altura relativa a essa base mede 4cm.
MNPQ um retngulo cujos vrtices M e N pertencem ao
lado AB, P pertence ao lado BC e Q ao lado AC. O
permetro desse retngulo, em cm, :
A B
C
M N
Q P


a) 4 b) 8 c) 12 d) 14 e) 16

19. Na figura abaixo as circunferncias de centros A e B
tm raios 9cm e 6 cm, respectivamente, e a distncia
entre os centros 25cm. A reta t uma tangente interior as
circunferncias nos pontos C e D. Calcule, em centmetros,
a medida do segmento CD.










UNIDADE 5

REAS DE FIGURAS PLANAS

TRINGULOS QUAISQUER



TRINGULO EQUILTERO


QUADRILTEROS

Paralelogramo

A = a.h


Please purchase PDF Split-Merge on www.verypdf.com to remove this watermark.
Incluso para a vida Matemtica C

Pr-Vestibular da UFSC 11

Crculo e suas partes

Crculo
A = tR
2


Coroa Circular
A = t (R
2
r
2
)

Setor Circular
A =
$
360
R
2

Exerccios de Sala #

1. (FCC-SP) O retngulo ABCD tem rea 105 m
2
. O lado
do quadrado EFGD mede, em m:
A
B
C
D
E
F
10
2

a) 4 b) 5 c) 2 5

d) 5 2

e) 6

2. A rea da coroa limitada pelas circunferncias inscrita e
circunscrita a um quadrado de lado 3 :

a) 2,25t b) 5t c) 4t d) 2t e) 8t

3. (UFSC) Considere um tringulo equiltero cujo lado
mede 12cm de comprimento e um quadrado em que uma
das diagonais coincida com uma das alturas desse
tringulo. Nessas condies, determine a rea (em cm
2
)
do quadrado.

Tarefa Mnima #

4. A rea do tringulo ABC, conforme a figura, :
120
A
B
C
4
3

a) 3

b) 2 3

c) 3 d) 4 3 e) 6

5. (CEFET-PR) A rea do hexgono regular inscrito
numa circunferncia de raio 2 igual a:
a) 3 3 cm
2
d) 2 2 cm
2
b) 3 2 cm
2
e) n.d.a.
c) 2 3 cm
2

6. (UFSC) O tringulo ABC est inscrito em uma
circunferncia de centro O, cujo dimetro mede 10cm.
Se a corda AB mede 6cm, ento a rea sombreada, em
centmetros quadrados, :


7. (UFPR) Um retngulo de 6m por 12m est dividido
em trs retngulos, A, B e C, dispostos conforme a
figura abaixo, de modo que a rea de B a metade da
de A e um tero da de C.

A
B C


Com base nessas informaes, correto afirmar:

01. A soma das reas de A, B e C 72m
2
.
02. A rea de A 1/6 da rea de C.
04. A rea de A 24m
2
.
08. Um dos lados de A mede 2m.
16. Um dos lados de C mede 8m.

8. (UFSC) Na figura a seguir, a rea hachurada de
16 t cm
2
. Sabendo-se que a diferena entre os dois
raios de 2cm, determine o valor numrico do produto
desses raios.







Tarefa Complementar #

9. (FUVEST) No tringulo ABC, AB = 20cm, BC = 5cm
e o ngulo ABC obtuso. O quadriltero MBNP um
losango de rea 8cm
2

A
B C
M
N
P

Please purchase PDF Split-Merge on www.verypdf.com to remove this watermark.
Incluso para a vida Matemtica C

Pr-Vestibular da UFSC 12
A medida, em graus, do ngulo BNP :

a) 15 c) 45 e) 60
b) 30 d) 75

10. (CESGRANRIO) A base de um retngulo de rea S
aumentada de 20% e sua altura diminuda de 20%. A
rea do novo retngulo formado :

a) 1,04 S d) 0,98 S
b) 1,02 S e) 0,96 S
c) S

11. (CESCEM-SP) O quadriltero ABCD um
retngulo, e os pontos E, F, G dividem a base AB em
quatro partes iguais. A razo entre a rea do tringulo
CEF e a rea do retngulo :

A
B
C
E F G
D


a) 1/6 c) 1/8 e) 1/10
b) 1/7 d) 1/9

12. A rea da coroa limitada pelas circunferncias inscrita
e circunscrita a um tringulo equiltero ABC de lado 6cm
igual a:
A
B C
O


13. (MACK-SP) No crculo da figura, de centro O e raio
1, a rea do setor assinalado :


9
8
e)
9
5
d)
18
5
c)
18
7
b)
9
7
a)

14. (UEM) Considere o tringulo ABC, com base BC
medindo 6cm e com altura 5cm. Um retngulo inscrito
nesse tringulo tem o lado MN paralelo a BC, com x cm
de comprimento. Qual o valor de x, em cm, para que a
rea do retngulo seja mxima?

15. (VUNESP) Um cavalo se encontra preso num cercado
de pastagem, cuja forma um quadrado, com lado
medindo 50m. Ele est amarrado a uma corda de 40m que
est fixada num dos cantos do quadrado. Considerando t =
3,14, calcule a rea, em metros quadrados, da regio do
cercado que o cavalo no conseguir alcanar porque est
amarrado.

a) 1244 c) 1422 e) 1444
b) 1256 d) 1424

16. (UFRGS) Se o raio de um crculo cresce 20%, sua
rea cresce:

a) 14% c) 40% e) 144%
b) 14,4% d) 44%

17. (UFSC) Considere as circunferncias C1 de raio r e
C2 de raio R. A circunferncia C1 passa pelo centro de
C2 e lhe tangente. Se a rea do circulo, limitado pela
circunferncia C1, igual a 4 centmetros quadrados,
calcule em cm
2
a rea do crculo limitado pela
circunferncia C2.

18. (FUVEST) No trapzio ABCD, M o ponto
mdio do lado AD; N est sobre o lado BC e 2BN =
NC. Sabe-se que as reas dos quadrilteros ABNM e
CDMN so iguais e que DC = 10. Calcule AB.


UNIDADE 6

GEOMETRIA ESPACIAL POLIEDROS

Figuras tridimensionais limitadas por polgonos planos.


Relao de Euler: V + F = A + 2

Soma dos ngulos internos: Si = 360 (v 2)
onde v o nmero de vrtices.
Qual a quantidade de vrtices, arestas e faces de um
poliedro limitado por seis faces quadrangulares e duas
faces hexagonais?



Poliedros Regulares
Possuem todas as faces como polgonos regulares iguais e
ngulos formados pelas faces iguais.

Please purchase PDF Split-Merge on www.verypdf.com to remove this watermark.
Incluso para a vida Matemtica C

Pr-Vestibular da UFSC 13




Exerccios de Sala #

1. Um poliedro possui cinco faces triangulares, cinco faces
quadrangulares e uma pentagonal, determine as arestas,
faces e vrtices.

2. Um poliedro convexo possui 9 faces triangulares, 9
faces quadrangulares, 1 face pentagonal e 1 face
hexagonal. Determine o nmero de vrtices.

3. Calcule a rea total e o volume de um octaedro regular
de aresta l.

Tarefa Mnima #

4. (FISS-RJ) Um poliedro convexo formado por 20 faces
triangulares. O nmero de vrtices desse poliedro :
a) 12 c) 18 e) 24
b) 15 d) 20
5. (CEFET PR) Um poliedro convexo possui duas faces
triangulares, duas quadrangulares e quatro pentagonais.
Logo, a soma dos ngulos internos de todas as faces ser:
a) 3240 c) 3840 e) 4060
b) 3640 d) 4000

6. (PUCPR) Um poliedro convexo tem 3 faces
pentagonais e algumas faces triangulares. Qual o nmero
de faces desse polgono, sabendo-se que o nmero de
arestas o qudruplo do nmero de faces triangulares?
a) 6 c) 5 e) 8
b) 4 d) 3

7. (PUCPR) Um poliedro convexo de 10 vrtices possui
8 faces triangulares e x faces quadrangulares. Qual o
nmero total de faces desse poliedro?
a) 4 c) 8 e) 12
b) 6 d) 10

8. (PUCCAMPSP) Sobre as sentenas:
I - Um octaedro regular tem 8 faces quadradas.
II - Um dodecaedro regular tem 12 faces pentagonais.
III - Um icosaedro regular tem 20 faces triangulares.

correto afirmar que apenas:

a) I verdadeira b) II verdadeira
c) III verdadeira d) I e II so verdadeiras
e) II e III so verdadeiras.

Tarefa Complementar #

9. Some as alternativas corretas:
01. Um poliedro convexo que tem 7 faces e 15 arestas
possui 10 vrtices.
02. Um poliedro convexo que tem 6 faces triangulares e
somente faces triangulares possui 9 arestas.
04. Um poliedro que possui 10 vrtices tridricos possui
15 arestas.
08. Um poliedro que possui 6 vrtices tridricos e quatro
vrtices pentadricos possui 12 faces.
16. Todo poliedro convexo que tem o nmero de vrtices
igual ao nmero de faces possui um nmero par de
arestas.

10. (UFPR) Um poliedro convexo de 29 vrtices possui
somente faces triangulares e faces hexagonais. Quantas
faces tem o poliedro se o nmero de faces triangulares a
metade do nmero de faces hexagonais?

11. (CESGRANRIO) Considere o poliedro regular, de
faces triangulares, que no possui diagonais. A soma dos
ngulos das faces desse poliedro vale, em graus:
a) 180 c) 540 e) 900
b) 360 d) 720

12. (UFRGS) Um octaedro regular possui:
a) mais diagonais do que vrtices;
b) mais faces que arestas;
c) mais vrtices do que faces;
d) menos diagonais que faces;
e) igual nmero de vrtices e de arestas.

13. (PUCPR) Se a soma dos ngulos das faces de um
poliedro regular 1440, ento o nmero de arestas desse
poliedro :
a) 12 b) 8 c) 6 d) 20 e) 4









Please purchase PDF Split-Merge on www.verypdf.com to remove this watermark.
Incluso para a vida Matemtica C

Pr-Vestibular da UFSC 14
UNIDADE 7

PRISMAS

DEFINIO
Prismas so poliedros que possuem duas faces paralelas e
congruentes denominadas bases, e as demais faces em
forma de paralelogramos.


ELEMENTOS
BASES: so os polgonos ABCDE e ABCDE
FACES LATERAIS: So os paralelogramos ABAB;
BCBC; CDCD;
ARESTAS LATERAIS: so os segmentos AA; BB;
CC; DD e EE
ALTURA: A distncia EH entre as duas bases
denominada altura do Prisma.
ARESTAS DAS BASES: so os segmentos AB; BC;
CD ; DE e EA

NOMENCLATURA
O nome do prisma se d atravs da figura da base.

- Prisma Triangular: As bases so triangulares.
- Prima Quadrangular: As bases so quadrilteros.
- Prisma Hexagonal: As bases so hexgonos

Observao: Se o polgono da base for
regular, o prisma tambm ser chamados de Regular.










CLASSIFICAO
De acordo com sua inclinao um prisma pode ser:

Reto: quando as arestas
laterais so
perpendiculares aos planos
Oblquo: quando as arestas
laterais so oblquas aos
planos da base.
da base.



No prisma reto tem-se que as arestas laterais so iguais a
altura.
Frmulas
Considere um prisma reto regular com n lados da base.



Exerccios de Sala #

1. Dado um Prisma triangular regular com aresta lateral
igual a 7cm e aresta da base igual a 2cm. Determine:

a) a rea total do prisma
b) o volume do prisma

2. (UFSC) O volume de um prisma hexagonal regular
de 2cm de aresta da base 42 3 cm
3
. A medida, em
cm
2
, da rea lateral desse prisma :

Tarefa Mnima #

3. (ACAFE) Um prisma de 8dm de altura tem por base
um quadrado de 2dm de lado. O volume do prisma :

4. (UFSC) Um prisma triangular regular tem uma rea
total de ( 96 + 2 3 ) cm
2
. Sabe-se que a aresta da base
mede 2cm. A medida, em centmetros, da altura do
prisma :

5. (PUC-PR) O volume do prisma reto de 3 m de
altura, cuja base um hexgono de 2 m de lado, :
a) 3 m
3
d) 3 m
3

Please purchase PDF Split-Merge on www.verypdf.com to remove this watermark.
Incluso para a vida Matemtica C

Pr-Vestibular da UFSC 15
b) 3 3 m
3
e) 8 3 m
3

c) 9 m
3


6. (Mack-SP) Num prisma de base triangular, a altura
6 e os lados da base so 5, 6 e 7cm. O volume em
cm
3
:

Tarefa Complementar #

7. (PUC-SP) Se a rea da base de um prisma diminui
10% e a altura aumenta 20%, o seu volume:
a) aumenta 8% d) diminui 8%
b) aumenta 15% e) no se altera
c) aumenta 108%

8. (UFCE) Um prisma reto tem por base um losango
cujas diagonais medem 8 cm e 4cm, respectivamente.
Se a altura do prisma de 6cm, ento o volume desse
prisma, em cm
3
, :

9. (ITA-SP) Considere P um prisma reto de base
quadrada, cuja altura mede 3m e com rea total de
80m
2
. O lado dessa base quadrada mede:

10. ( FCC-SP ) Na figura abaixo, tem-se um prisma reto
de base triangular. Se AB = 17cm, AE = 8 cm e ED = 14
cm, a rea total desse prisma, em cm
2
, :


a) 1852 c) 926 e) 508
b) 1016 d) 680

11. (UFSC) Na figura a seguir, o segmento de reta AE
paralelo ao segmento BF e o segmento de reta CG
paralelo ao segmento DH; o trapzio ABDC tem os
lados medindo 2cm, 10cm, 5cm e 5cm, assim como o
trapzio EFHG; esses trapzios esto situados em planos
paralelos que distam 4cm um do outro. Calcule o volume
(em cm
3
) do slido limitado pelas faces ABFE, CDHG,
ACGE, BDHF e pelos dois trapzios.









UNIDADE 8

TIPOS ESPECIAIS DE PRISMAS

PARALELEPPEDO RETO RETNGULO
Paraleleppedo o prisma no qual as seis faces so
paralelogramos e as faces opostas so retngulos
congruentes.

Possui trs dimenses:
- comprimento (a)
- largura (b)
- altura (c)

Frmulas

rea Total: ST = 2(ab + ac + bc)

Volume: V = a.b.c

Diagonal: D
2
= a
2
+ b
2
+ c
2


RELAO AUXILIAR: (a + b +c)
2
= D
2
+ ST


Cubo Hexaedro Regular

Cubo um paraleleppedo com as dimenses iguais.


Todas as faces so quadrados

Frmulas

rea Total: ST = 6 "
2

Volume: V = "
3

Diagonais: d = " 2 D = " 3


Exerccios de Sala #

1. (UFSC) O volume de um paraleleppedo retngulo
24 m
3
. Sabendo-se que suas dimenses so
proporcionais aos nmeros 4, 3 e 2, calcule, em metros
quadrados, a rea total desse paraleleppedo.










Please purchase PDF Split-Merge on www.verypdf.com to remove this watermark.
Incluso para a vida Matemtica C

Pr-Vestibular da UFSC 16
2. No cubo da figura, rea da seco o ABCD 8 cm
2
.
Calcule o volume do cubo.


Tarefa Mnima #

3. (UFSC) Na figura abaixo, que representa um cubo, o
permetro do quadriltero ABCD mede 8(1 + 2 ) cm.
Calcule o volume do cubo em cm
3
.

4. (UFSC) Considerando que uma das dimenses de um
paraleleppedo retngulo mede 6dm, e as demais
dimenses so diretamente proporcionais aos nmeros 8 e
2, e que a soma de todas as arestas 44dm, calcule, em
dm
2
, a rea total desse paraleleppedo.

5. (FGV-SP) Um cubo tem 96m
2
de rea total. Em quanto
deve ser aumentada a sua aresta, em metros, para que seu
volume se torne igual a 216 m
3
?


6. ( UFSC ) Usando um pedao retangular de papelo, de
dimenses 12cm e 16cm, desejo construir uma caixa sem
tampa, cortando, em seus cantos, quadrados iguais de 2cm
de lado e dobrando, convenientemente, a parte restante. A
tera parte do volume da caixa, em cm
3
, :


7. (UFSC) Num paraleleppedo retngulo, as medidas das
arestas esto em progresso aritmtica de razo 3. A
medida, em CENTMETROS, da menor aresta desse
paraleleppedo, sabendo que a rea total mede 132 cm
2
, :


Tarefa Complementar #

8. (UFSC) A rea total de um paraleleppedo reto
retngulo de 376 m
2
e as suas dimenses so
proporcionais aos nmeros 3, 4 e 5. Determine a dcima
parte do volume desse paraleleppedo. Depois, passe o
resultado para o carto resposta.

9. (Fatec-SP) As medidas das arestas de um
paraleleppedo retngulo formam uma P.G. Se a menor das
arestas mede 1/2 cm e o volume de tal paraleleppedo
64cm
3
, ento a soma das reas de suas faces :
a) 292cm
2
c) 296cm
2
e) 290cm
2

b) 298cm
2
d) 294cm
2


10. (UEPG) Sobre trs cubos idnticos de aresta 1 dm
agrupados conforme mostra a figura abaixo, assinale o
que for correto.


01. A rea do tringulo ABC 2 dm
2
.
02. AD = 2 6 dm.
04. O tringulo ABC retngulo issceles.
08. O volume do slido formado pelos trs cubos de 3
dm
3
16. O permetro do tringulo BCD vale 4 2 dm.

11. (UFSC) Um tanque, em forma de paraleleppedo, tem
por base um retngulo de lados 0,50m e 1,20m. Uma
pedra, ao afundar completamente no tanque, faz o nvel da
gua subir 0,01m. Ento, o volume da pedra, em
decmetros cbicos, :

12. (UNICAMP) Ao serem retirados 128 litros de gua de
uma caixa dgua de forma cbica, o nvel da gua baixa
20 cm.
a) calcule o comprimento das arestas da referida caixa.
b) calcule sua capacidade em litros.

UNIDADE 9

PIRMIDES

DEFINIO

Pirmides so poliedros cuja base uma regio poligonal
ABCDEF e as faces so regies triangulares.
Uma pirmide se diz regular quando for reta
(projeo ortogonal do vrtice coincide com o centro da
base) e a figura da base for regular



NOMENCLATURA

D-se o nome da pirmide atravs do polgono da base.
Observe alguns exemplos.













Please purchase PDF Split-Merge on www.verypdf.com to remove this watermark.
Incluso para a vida Matemtica C

Pr-Vestibular da UFSC 17
- Pirmide Triangular a base um tringulo



- Pirmide quadrangular a base um quadrado



- Pirmide Pentagonal a base um pentgono



Pirmides Regulares
Se a base de uma pirmide reta for um polgono regular,
a pirmide regular.
Elementos e Formulrio



- aresta da base -
- aresta lateral -a
- altura h
- aptema da base ab
- aptema da pirmide ap
- Raio da circunferncia circunscrita R

Para uma pirmide regular com n lados da base vale as
seguintes relaes:

rea da Base: SB = a rea do Polgono que est na
base
rea Lateral : SL = n.
". ap
2


rea Total: ST = SB + SL

Volume V =
3
.h SB




Relaes Auxiliares na Pirmide

-
ap
2
= H
2
+ ab
2
- a "
2
= ap
2
+
"
2
2
|
\

|
.
|

- a "
2
= H
2
+ R
2


Exerccios de Sala #

1. Uma pirmide quadrangular regular tem 4 m de altura e
a aresta de sua base mede 6m. Determine a rea total dessa
pirmide.

2. Qual o volume de uma pirmide regular hexagonal, cuja
altura mede 3 3 m e o permetro da base mede 12 m?

3. (UFSC) A base quadrada de uma pirmide tem 144 m
2

de rea. A 4 m do vrtice traa-se um plano paralelo base
e a seco assim feita tem 64 m
2
de rea. Qual a altura da
pirmide?

Tarefa Mnima #

4. (UFSC) Uma pirmide regular, de base quadrada, tem
aresta da base 8cm e aptema da pirmide 5cm.
Determine, em cm
3
, o volume dessa pirmide.

5. (UFSC) A aresta da base de uma pirmide quadrangular
regular mede 4cm e sua altura mede 2 3 cm. Determine a
rea total, em cm
2
, dessa pirmide.

6. (UFSC) Em uma pirmide quadrangular regular a aresta
lateral mede 5cm e a altura mede 4cm. O volume, em cm
3
,
:
7. (Cescem-SP) Em uma pirmide com 12cm de altura,
tendo como base um quadrado de lado igual a 10 cm, a
rea lateral :
a) 240cm
2
c) 340cm
2
e) n.d.a.
b) 260cm
2
d) 400cm
2


8. (Osec-SP) Uma pirmide quadrada tem todas as arestas
medindo 2. Ento, a sua altura mede:
a) 1 c) 3 e) n.d.a.
b) 2 d) 4

Tarefa Complementar #

9. (UFPA) Uma pirmide triangular regular tem 9 cm
3
de
volume e 4 3 cm de altura. Qual a medida da aresta da
base?

10. (UECE) Se o volume de um cubo de 6cm de aresta
igual ao volume de uma pirmide regular que tem para
base de um quadrado de 6cm de lado, ento a altura da
pirmide, em cm, :

11. O aptema de uma pirmide regular igual ao
semipermetro da base, e esta um quadrado inscrito num
crculo de 8 metros de raio. Calcule a rea total da
pirmide. ( Divida o resultado obtido em m
2
por dez ).
Please purchase PDF Split-Merge on www.verypdf.com to remove this watermark.
Incluso para a vida Matemtica C

Pr-Vestibular da UFSC 18
12. (UEPG-PR) Calcule a rea total de um tetraedro
regular de aresta igual a 4 cm.
a) 4 3 cm
2
d) 16 3 cm
2

b) 8 3 cm
2
e) 24 3 cm
2

c) 12 3 cm
2


13. (ACAFE-SC) A figura abaixo mostra a planificao
de um slido. O volume desse slido de:

a) 1152cm
3
d) 1200cm
3

b) 1440cm
3
e) 240cm
3

c) 384cm
3


14. (VUNESP) Em cada um dos vrtices de um cubo de
madeira, recorta-se uma pirmide AMNP, em que M, N e
P so os pontos mdios das arestas, como se mostra na
ilustrao. Se V o volume do cubo, o volume do poliedro
que resta ao tirar as 8 pirmides igual a:

1 3 2 5 3
a) V b) V c) V d) V e) V
2 4 3 6 8


15. (UEPG-PR) Calcule a rea total de um tetraedro
regular de aresta igual a 4 cm.
a) 4 3 cm
2
d) 16 3 cm
2

b) 8 3 cm
2
e) 24 3 cm
2
c) 12 3 cm
2



16. (PUC-PR) A aresta da base de uma pirmide
hexagonal regular mede 3cm, e o aptema dessa pirmide,
4cm. A rea de uma das faces laterais desta pirmide
mede, em m
2
.
a) 6.10
-4
d) 12.10
-2

b) 6.10
-2
e) 15.10
-4

c) 12.10
-4


17. (EE Volta Redonda) A base de uma pirmide tem 225
cm
2
de rea. Uma seco paralela base, feita a 3cm do
vrtice, tem 36cm
2
de rea. A altura da pirmide :
a) 4,5 cm d) 9,5cm
b) 7,5 cm e) 3,5cm
c) 1,5 cm




UNIDADE 10

CILINDRO, CONE e ESFERA

CILINDRO DE REVOLUO

Cilindro de revoluo o slido obtido quando giramos
em torno de uma reta uma regio retangular. Tambm
chamado de cilindro circular.


Elementos


Se as geratrizes forem perpendiculares ao plano da base
dizemos que o cilindro reto, caso contrrio, dito
cilindro oblquo. No caso do cilindro reto, temos que g = h

Frmulas

Considere um cilindro reto.



rea da Base: SB = tr
2


rea Lateral: SL = 2trh

rea Total: ST = 2SB + SL

Volume: V = tr
2
h

Seco Meridiana:
A seco feita no cilindro reto por um plano que contm o
seu eixo denomina-se seco meridiana do cilindro. A
seco meridiana um retngulo de rea: 2r.h. Quando a
seco um quadrado temos um cilindro equiltero.
(g = h = 2r)
2R
h


CONE DE REVOLUO
Cone de revoluo o slido obtido quando giramos um
tringulo retngulo em torno de um de seus catetos. Este
cateto a altura do cone, e o outro seu raio. J a
hipotenusa a geratriz do mesmo.
Please purchase PDF Split-Merge on www.verypdf.com to remove this watermark.
Incluso para a vida Matemtica C

Pr-Vestibular da UFSC 19




Frmulas

rea da Base: SB = tr
2
rea Lateral: SL = trg
rea Total: ST = SB + SL Volume: V =
3
h r
2

Relao auxiliar: g
2
= h
2
+ r
2


Seco Meridiana
No cone reto temos a seco sendo um tringulo issceles.
Quando a seco meridiana for um tringulo equiltero
teremos um cone equiltero ( G = 2R )
h g
2R

ESFERA
Esfera o conjunto dos pontos do espao cujas distncias
ao ponto O so menores ou iguais a R. A esfera tambm
pode ser considerada um slido determinado pela rotao
de um crculo em torno de um de seus dimetros.


Seco de uma esfera
Qualquer plano o que secciona uma esfera de raio R
determina como seco plana um crculo de raio r.

d a distncia entre o plano o e o centro da esfera.
R o raio da esfera.
r o raio da seco.
Relao: R
2
= r
2
+ d
2

Frmulas da esfera
superfcie esfrica: As = 4tR
2
volume: V =
3
R
3
4

Exerccios de Sala #

1. (ACAFE) O volume de um cone circular reto de 27t
dm
3
e a altura de 9 dm. O raio da base :

a) 4dm b) 9dm c) 2dm d) 5dm e) 3dm

2. (UFSC) Determinar
1
t
do volume em m
3
de um cone
de revoluo cujo dimetro da base mede 8m e a rea
lateral, 20t m
2
.

3. (UFES) Enche-se um tubo cilndrico de altura h =
20cm e raio da base r = 2 cm com esferas tangentes ao
mesmo e tangentes entre si. O volume interior ao cilindro e
exterior s esferas vale:

a) 102
t
3
cm
3
b) 80
t
3
cm
3
e) 80 t cm
3

b) d) 160cm
3
d) 40 t cm
3


4. (UFSC) O volume, em cm
3
, de um cubo circunscrito a
uma esfera de t 16 cm
2
de superfcie :

Tarefa Mnima #

5. (UFSC) A rea lateral de um cilindro equiltero de
36tm
2
. O valor, em m
3
, de
1
t
do volume desse cilindro :

6. (UFSC) Derrete-se um bloco de ferro, de forma cbica,
de 9cm de aresta, para modelar outro bloco, de forma
cnica, de
15
t
cm de altura e 12 cm de raio da base. O
volume, em cm
3
, de ferro que sobrou aps a modelagem,
:

7. (UDESC) Uma caixa dgua de forma cilndrica tem
1,5 m de dimetro e capacidade de 7065 litros. A altura da
caixa :
a) 3,2 m c) 4,0 m
b) 3,6 m d) 4,8 m

8. (SUPRA) Um pedao de cano de 30cm de comprimento
e 10 cm de dimetro interno se encontra na posio
vertical e possui a parte interna vedada. Colocando-se dois
litros de gua em seu interior, a gua:

a) ultrapassa o meio do cano
b) transborda
c) no chega ao meio do cano
d) enche o cano at a borda
e) atinge exatamente o meio do cano

Please purchase PDF Split-Merge on www.verypdf.com to remove this watermark.
Incluso para a vida Matemtica C

Pr-Vestibular da UFSC 20
9. (FUVEST) Uma superfcie esfrica de raio 13cm
cortada por um plano situado a uma distncia de 12cm do
centro da superfcie esfrica, determinando uma
circunferncia, em cm, :
a) 1 b) 2 c) 3 d) 4 e) 5

Tarefa Complementar #

10. (UFSC) Um cilindro reto tem 63tcm
3
de volume.
Sabendo que o raio da base mede 3cm, determine, em
centmetros, a sua altura.

11. (UFCE) O raio de um cilindro circular reto
aumentado de 20% e sua altura diminuda de 25%. O
volume deste cilindro sofrer um aumento de:
a) 2% c) 6% e) n.d.a.
b) 4% d) 8%

12. (PUC-PR) Um tringulo retngulo issceles, de
hipotenusa 3 2 cm, gira em torno de um dos catetos.
Qual o volume do slido de revoluo gerado?
a) 3 2 cm
3
d) 27 t cm
3

b) 9 t cm
3
e) 1/3 t cm
3

c) 18 t cm
3



13. Uma esfera de raio 8cm seccionada por um plano
distante 5cm do seu centro. Calcule o raio (em cm) da
seco.
a) 39 c) 32 e) n.d.a.
b) 36 d) 65

14. (UFSC) A razo entre o volume de um cubo e sua rea
total 2. O valor de
1
3t
do volume da esfera, inscrita
nesse cubo, :

15. (UFSC) O volume, em cm
3
, de um cubo
circunscrito a uma esfera de 16t cm
2
de superfcie :

16. (F.Porto-Alegrense-RS) Se um cone e uma esfera tm
o mesmo volume, e o raio da base do cone o triplo do
raio da esfera, ento a razo entre o raio da esfera e a altura
do cone :
a) 9/4 c) 3/4 e) 1
b) 9/2 d) 2/3

17. (Santa Casa-SP) O raio da base de um cone equiltero
mede 6 3 cm. O volume da esfera inscrita nesse cone,
em cm
3
, :
a) 144t c) 192t e) 302t
b) 152t d) 288t

18. (UFRGS) Uma panela cilndrica de 20 cm de dimetro
est completamente cheia de massa para doce, sem
exceder a sua altura, que de 16cm. O nmero de doces
em formato de bolinhas de 2cm de raio que se podem obter
com toda a massa :
a) 300 c) 200 e) 100
b) 250 d) 150
19. (UFSC) A geratriz de um cone equiltero mede
3 2 cm. Calcule a rea da seo meridiana do cone, em
cm
2
, multiplique o resultado por 3 e assinale o valor
obtido no carto-resposta.

UNIDADE 11

PROGRESSO ARITMTICA

CONCEITOS INICIAIS
Vamos considerar a sequncia (a
n
) onde a
n
= 3n + 1,
sendo n inteiro positivo. Temos:
a
1
= 4, a
2
= 7, a
3
= 10, a
4
= 13 e assim por diante.

(4, 7, 10, 13, ...........)

Observe que a diferena entre cada termo e seu antecessor
se mantm igual a 3. Sequncias como esta so
denominadas progresses aritmticas.


DEFINIO
Chama-se progresso aritmtica uma sequncia em que, a
partir do segundo elemento, a diferena entre cada
elemento e seu antecessor constante. Essa constante
denominada razo da P.A. e indicada por r.

Veja que para a sequncia a
1
.a
2
.a
3
...a
n
ser uma P.A.
necessrio que:

a
2

a
1
= a
3

a
2
= ...... a
n
a
n1
= ..... = r

Veja os exemplos:

a) a sequncia (2, 5, 8, .......) uma P.A., pois,
5 2 = 8 5 = ..... Sua razo igual a 3.

b) a sequncia (1, 4, 5, .....) no P.A., pois,
4 1 = 5 4.

CLASSIFICAO DA P.A.
Uma P.A. pode ser classificada de acordo com valor da
razo. Observe o quadro abaixo:

r > 0 P.A. crescente (2, 4, 6, 8, 10) r = 2
r < 0 P.A. decrescente (10, 7, 4, 1, -2) r = 3
r = 0 P.A. constante (3, 3, 3, 3, 3) r = 0

FRMULA DO TERMO GERAL DA P.A.
Considere a sequncia (a
1
, a
2
, a
3
......a
n
). Partindo da
definio temos:
a
2
= a
1
+ r
a
3
= a
2
+ r = a
1
+ r + r = a
1
+ 2r
a
4
= a
3
+ r = a
1
+ 2r + r = a
1
+ 3r


a
n
= a
1
+ (n 1).r

Importante:
Se a
n
e a
k
so dois termos quaisquer de uma P.A. , da
frmula do termo geral temos:

a
n
= a
1
+ (n 1)r (1)
Please purchase PDF Split-Merge on www.verypdf.com to remove this watermark.
Incluso para a vida Matemtica C

Pr-Vestibular da UFSC 21
a
k
= a
1
+ (k 1)r (2)

Subtraindo-se (1) de (2) vem:

a
n
a
k
= (n 1)r (k 1)r
a
n
a
k
= (n 1 k + 1) r
a
n
= a
k
+ (n k)r
Logo, para dois termos quaisquer a
n
e a
k
, podemos
escrever:

a
n
= a
k
+ (n k).r

Exemplos: a
12
= a
3
+ 9r; a
20
= a
6
+ 14r; a
8
= a
2
+ 6r

Representaes Especiais

Para facilitar a resoluo de problemas em P.A. podemos
utilizar os seguintes artifcios:

- Trs termos em P.A.
: x r . x . x + r
- Quatro termos em P.A
: x 3r . x r . x + r . x + 3r
- Cinco termos em P.A.
: x 2r . x r . x . x + r . x + 2r

Propriedades da P.A.
Dada uma Progresso Aritmtica qualquer, de n termos e
razo r, podemos observar as seguintes propriedades:

- Um termo qualquer, excetuando os extremos a mdia
aritmtica entre o termo anterior e o posterior.

Q
D
Q
D
Q
D
+
+

=



Exemplo: (2, 5, 8, 11, 14, 17, 20, 23)

+
=

- Numa P.A. limitada, a soma dos termos extremos
igual soma dos termos equidistantes dos extremos.

Observao: Se dois termos a
p
e a
q
so equidistantes dos
extremos temos:
p + q = n + 1

Com essa igualdade possvel saber se dois termos
quaisquer so equidistantes dos extremos ou no.
Por exemplo, numa sequncia de 50 termos, a
16
e a
35
so
equidistantes dos extremos, pois
16 + 35 = 50 + 1.

INTERPOLAO ARITMTICA
Interpolar, inserir ou intercalar m meios aritmticos entre a
e b significa formar uma P.A. de extremos a e b com
m + 2 elementos.

Para determinarmos os meios aritmticos, devemos
calcular a razo da P.A.




SOMA DOS TERMOS DA P.A.

.n
2
n
a
1
a
n
S
|
|
.
|

\
| +
=



Exerccios de Sala #

1. A sequncia (19 6x, 2 + 4x, 1 + 6x) so termos
consecutivos de uma P.A. Logo, o valor de x :

2. Em uma P.A., a
5
= 30 e a
16
= 118. Calcule a razo da
P.A.

3. (UFSC) Marque no carto resposta a NICA
proposio correta. A soma dos mltiplos de 10,
compreendidos entre 1e1995,
01. 198.000
02. 19.950
04. 199.000
08. 1.991.010
16. 19.900

Tarefa Mnima #

4. Em cada caso abaixo, determine o valor de x para que
as sequncias representem trs nmeros consecutivos em
P.A.
a) (3x - 1, x + 3 e x + 9 )
b) (2x 3, 2x + 1, 3x + 1)
c) (x + 4)
2
, (x 1)
2
, (x + 2)
2

5. (FGV-SP) A sequncia ( 3m; m + 1; 5 ) uma
progresso aritmtica. Sua razo :

6. (PUC-SP) Se o quarto e o nono termos de uma P.A. so
respectivamente, 8 e 13, ento a razo da progresso :

7. Calcule a razo de uma P.A sabendo que a soma do
terceiro termo com o oitavo 74 e a soma do quinto com o
dcimo segundo 110.

8. (LONDRINA) Interpolando-se 7 termos aritmticos
entre os nmeros 10 e 98, obtm-se uma progresso
aritmtica cujo quinto termo vale:

9. (PUC-SP) Trs nmeros positivos esto em PA. A
soma deles 12 e o produto 18. O termo do meio :

10. (U.F OURO PRETO) A soma dos n primeiros
nmeros naturais mpares dada por:
a) n
2
b) 2n c) n/2 d) 2n 1 e) n
3


11. Numa cerimnia de formatura de uma faculdade, os
formandos foram dispostos em 20 filas de modo a formar
um tringulo; com 1 formando na primeira fila, 3
formandos na segunda, 5 formandos na terceira e assim
por diante, constituindo uma progresso aritmtica. O
nmero de formandos na cerimnia :
a) 400 b) 410 c) 420 d) 800 e) 840


Please purchase PDF Split-Merge on www.verypdf.com to remove this watermark.
Incluso para a vida Matemtica C

Pr-Vestibular da UFSC 22
Tarefa Complementar#

12. (UFSC) Numa P.A. decrescente de 7 termos, a soma
dos termos extremos 92, e a diferena entre os dois
primeiros termos 5. O valor do 1 termo :

13. O nmero de mltiplos de 5 compreendidos entre 21e
623 :

14. (U.CAXIAS DO SUL ) Sabendo que a sequncia (1
3x, x 2, 2x + 1) uma P.A, ento o dcimo termo da
P.A. (5 3x, x + 7, .) :
a) 62 b) 40 c) 25 d) 89 e) 56

15. (PUC) Os nmeros que exprimem o lado, a diagonal e
a rea de um quadrado esto em P.A, nessa ordem. O lado
do quadrado mede:
a) 2 b) 2 2 - 1 c) 1 + 2 d) 4 e) 2

16. (CEFET-PR) O nmero de inteiros compreendidos
entre 200 e 500, que so divisveis por 5 e no so
divisveis por 15, :
a) 100 b) 39 c) 41 d) 59 e) 80

17. (POLI) Inscrevendo nove meios aritmticos entre 15 e
45, qual o sexto termo da P.A.

18. (Unicamp-SP) Os lados de um tringulo retngulo
esto em progresso aritmtica. Sabendo que a rea do
tringulo 150, determine a soma dos lados do tringulo.

19. (UFSC) As medidas dos lados de um tringulo so
nmeros inteiros mpares consecutivos e seu permetro
mede 291 decmetros. Calcule em decmetros a medida do
maior lado desse tringulo.

20. Os lados de um tringulo retngulo esto em
progresso aritmtica. Sabendo que a rea do tringulo
150, determine o raio da circunferncia inscrita nesse
tringulo.

21. (UFSC) A Soma dos sete termos interpolados na P.A.
cujo primeiro termo e ltimo termos so respectivamente,
7 e 17 :

22. (UFSC) A soma dos 10 primeiros termos de uma P.A.,
na qual o primeiro termo igual a razo e a
3
+ a
8
= 18, :

23. (UFSC) Qual deve ser o nmero mnimo de termos da
sequncia (133, 126, 119, 112...) para que a soma de
seus termos seja positiva.

UNIDADE 12

PROGRESSO GEOMTRICA

DEFINIO
uma sequncia de nmeros no nulos em que cada termo
a partir do segundo, igual ao anterior multiplicado por
um nmero fixo chamado razo da PG.
Representao: :: a
1
: a
2
: a
3
: .... :a
n

onde
a
1
o primeiro termo
a
2
o segundo termo
a
3
o terceiro termo
an o ensimo ou ltimo termo
n o nmero de termos
q a razo da P.G.

q
a
a
a
a
a
a
a
a
n
n
= = = =

2
1
3
2
4
3 1


CLASSIFICAO DA P.G.

1 caso: a
1
> 0

Se q > 0 P.G. crescente ( 2, 6, 18, 54,...)
Se q = 1 P.G. constante ( 5, 5, 5, 5,...)
Se 0 < q < 1 P.G. decrescente ( 256, 64, 16,...)

2 caso: a
1
< 0

Se q > 0 P.G. decrescente (-2, -10, -50,..)
Se q = 1 P.G. constante ( -3, -3, -3,...)
Se 0 < q < 1 P.G. crescente ( -40, -20, -10,...)

Observao: So denominadas P.G. alternantes aquelas
em que cada termo tem sinal contrrio ao do termo
anterior. Isso ocorre quando q < 0.

TERMO GERAL
Considere a sequncia (a
1
, a
2
, a
3, .........
, a
n
). Partindo da
definio temos:
a
2
= a
1
.q
a
3
= a
2
.q = a
1
.q.q = a
1
.q
2

a
4
= a
3
.q = a
1
.q
2
.q = a
1
.q
3


an = a
1
.q
n - 1


Assim, como na P.A., podemos relacionar dois termos
quaisquer de uma P.G. Ou seja, dados dois termos de uma
P.G. a
m
e a
k
, podemos dizer que:

a
m
= a
k
.q
m - k


1. Representao de trs termos em
P.G.
x
x x q
q
, ,


2. Propriedades

1 Propriedade:
Dada uma P.G com trs termos consecutivos (a
1
, a
2
, a
3
),
podemos dizer que o termo central a mdia geomtrica
entre o anterior (a
1
) e o seu posterior (a
3
), ou seja:

a
2
2
= a
1
.a
3
ou a
n
2
= a
n - 1
.a
n + 1


2 Propriedade
Numa P.G. limitada o produto dos extremos igual ao
produto dos termos equidistantes dos extremos.
Veja a P.G. ( 2, 4, 8, 16, 32, 64 ).
Observe que: 2.64 = 4.32 = 8.16 = 128

3. Interpolao Geomtrica
Interpolar, inserir ou intercalar m meios geomtricos entre
a e b significa formar uma P.G. de extremos a e b com m
+ 2 elementos.
Please purchase PDF Split-Merge on www.verypdf.com to remove this watermark.
Incluso para a vida Matemtica C

Pr-Vestibular da UFSC 23
Para determinarmos os meios aritmticos, devemos
calcular a razo da P.G.

3. Soma dos termos de uma P.G. finita.
A soma dos "n" primeiros termos de uma P.G. finita
dada pela expresso:

1 1
1
1 1
n
n
a q a a q
Sn
q q
. ( )
= =


Observao: Se a razo da P.G. for igual a 1, temos
uma P.G. constante, e a soma dos
termos dessa P.G ser dada por:
S
n
= n. a
1

4. Soma dos termos de uma P.G. infinita.
Dada uma P.G. com: n e a
n
0, sua soma pode
ser calculada pela expresso:
q
a
S

=
1
1
0 < |q| < 1

5. Produto dos termos de uma P.G. finita
O produto dos termos de uma P.G. finita dado pela
expresso:
|P
n
| =
n
) .
Q
D D

Exerccios de Sala #

1. (UEL-PR) A sequncia (2x + 5, x + 1,
2
x
, ....) uma
progresso geomtrica de termos positivos. O dcimo
terceiro termo dessa sequncia :
a) 2 b) 3
-10
c) 3 d) 3
10
e) 3
12


2. (MACK-SP) Em uma progresso geomtrica o primeiro
termo 2 e o quarto 54. O quinto termo dessa P.G. :
a) 62 b) 68 c) 162 d) 168 e) 486

3. Numa P.G. de 10 termos, sabe-se que S
10
= 3069 e que
a razo vale 2, o valor do quinto termo :
a) 46 b) 47 c) 48 d) 24 e) 56

4. A soluo da equao:
x
x x x
+ + + +. =
3 9 27
15 ..
:

5. (UFSC) Determine a soma dos nmeros associados (s)
proposio(es) verdadeira(s).
01. A razo da P.A. em que a
1
= 8 e a
20
= 30 r=2.
02. A soma dos termos da P.A. (5, 8, ..., 41) 299.
04. O primeiro termo da P.G. em que a
3
= 3 e a
7
=
16
3

08. A soma dos termos da P.G.
|
.
|

\
|
,...
4
5
,
2
5
, 5
10.
Tarefa Mnima #

6. Em cada caso abaixo, determinar o valor de x para que
as sequncias representem trs nmeros consecutivos em
P.G.
a) (x + 1; x + 4; x + 10)
b) (4x, 2x + 1, x 1)

7. Numa P.G. de seis termos, o primeiro termo 2 e o
ltimo 486. Calcular a razo dessa P.G.

8. (Fuvest-SP) Numa P.G. de quatro termos positivos, a
soma dos dois primeiros vale 1 e a soma dos dois ltimos
vale 9. Calcule a razo da progresso.

9. (UFES) Qual a razo de uma P.G. de trs termos, onde
a soma de seus termos 14 e o produto 64?

a) 4 b) 2 c) 2 ou 1/2 d) 4 ou 1

10. (UFCE) A soluo da equao x
x x x
+ + + +. =
3 9 27
60 ..
:
a) 37 b) 40 c) 44 d) 50 e) 51

11. A soma dos termos da P.G. (2, 6, ......, 486) :
a) 567 b) 670 c) 728 d) 120 e) n.d.a.

Tarefa Complementar #

12. (UFPA) A sequncia (a, ab, 3a), com a = 0, uma
P.G. Ento, o nmero b :
a) o triplo de a. d) irracional
b) a tera parte de a. e) n.d.a.
c) racional

13. (UFPA) A razo da P.G. obtida ao somarmos um
mesmo nmero a 1,3 e 2, nessa ordem :
a) -1/2 b) 1/2 c) 2 d) - 2 e) -1/3

14. (FGV-SP) Em um tringulo, a medida da base, a
medida da altura e rea formam, nessa ordem, uma P.G. de
razo 8. Ento a medida da base vale:

15. (UFSC) Em uma progresso geomtrica o 3 termo
16/9 e o 7 termo 144. Determine o 5 termo.

16. ( UFSC ) Na progresso geomtrica (10, 2,
2
5
,
2
25
,
...), a posio do termo
2
625
:

17. Um artigo custa hoje R$ 100,00 e seu preo
aumentado, mensalmente, em 12% sobre o preo anterior.
Se fizermos uma tabela de preos desse artigo ms a ms,
obteremos uma progresso:
a) aritmtica de razo 12
b) aritmtica de razo 0,12
c) geomtrica de razo 12
d) geomtrica de razo 1,12
e) geomtrica de razo 0,12

18. (UFSC) Numa P.G. de 6 termos a razo 5. O produto
do 1 termo com o ltimo 12500. Determine o valor do
3 termo. Obs.: Considere a P.G. de termos positivos.

19. ( Santo Andr -SP ) Inserindo-se 5 meios geomtricos
entre 8 e 5832, obtm-se uma sequncia. Determine o 5
termo dessa sequncia.
a) 648 b) 78 c) 102 d) 354 e) 245

20. (UFSC) Sejam x, 6 e y uma progresso aritmtica
onde x e y so dois nmeros positivos. A sucesso x, 10 e
Please purchase PDF Split-Merge on www.verypdf.com to remove this watermark.
Incluso para a vida Matemtica C

Pr-Vestibular da UFSC 24
y + 40 uma progresso geomtrica. O valor numrico de
11y - 7x :

21. (UDESC) Um quadrado tem 4 cm de lado. Unem-se
os meios dos lados desse quadrado e obtm-se outro
quadrado. Unem-se os meios dos lados desse outro
quadrado e obtm-se um novo quadrado, e assim
sucessivamente. Determine a soma das reas de todos os
quadrados obtidos.

22. (IME) Uma bola lanada, na vertical, de encontro ao
solo, de uma altura de h metros. Cada vez que bate no
solo, ela sobe at a metade da altura que caiu. Determine a
distncia (em metros ) total percorrida pela bola em sua
trajetria at atingir o repouso.

23. (FGV-SP) O conjunto soluo da equao
2
1
...
27 9 3
2
=
x x x
x x
:
a) {
2
1
, 1} b) {
2
1
, 1} c) {1, 4}
d) {1, - 4} e) {1, 2}
24. Considere a expresso A =
...
81
4
27
3
9
2
3
1
+ + + +
em
que os numeradores formam uma P.A. e os denominadores
formam uma P.G. Determine o valor de 12A

25. (UFSC) Determine a soma dos nmeros associados
(s) proposio(es) verdadeira(s).
01. Existem 64 mltiplos de 7 entre 50 e 500.
02. O valor de x que satisfaz a equao
(x + 1) + (x + 4) + (x + 7) + ..... + (x + 28) = 155
x = 1
04. O oitavo termo da P.G. ( 2 , 2, ....) a
8
= 16.
08. A soma dos termos da P.G.
1
3
2
9
4
27
, , ,...
|
\

|
.
|
igual a 1.










Please purchase PDF Split-Merge on www.verypdf.com to remove this watermark.
Incluso para a vida Matemtica D

Pr-Vestibular da UFSC
1
UNIDADE 1

REGRA DE TRS

GRANDEZAS DIRETAMENTE PROPORCIONAIS
Duas grandezas so ditas diretamente proporcionais
quando o aumento uma delas implica no aumento da outra
na mesma razo.

Exemplo: 1 kg de alimento custa R$ 15,00
3 kg de alimento custam R$ 45,00
5kg de alimento custam R$ 75,00

GRANDEZAS INVERSAMENTE PROPORCIONAIS
Duas grandezas so ditas inversamente proporcionais
quando o aumento der uma delas implica na diminuio
da outra na mesma razo.

Exemplo: 2 pessoas constroem 1 obra em 18 dias
4 pessoas constroem a mesma obra em 9 dias
6 pessoas constroem a mesma obra em 6 dias

APLICAES REGRA DE TRS

Regra de Trs Simples
Regra de Trs Simples um processo matemtico
mediante o qual podemos resolver problemas do cotidiano
envolvendo duas grandezas, sejam elas direta ou
inversamente proporcionais. Este processo consiste no
seguinte:
- Identificar as grandezas envolvidas no problema.
- Nas situaes dadas (em relao s mesmas)
disp-las em colunas.
- Verificar se so GDP ou GIP.
- Montar a proporo correspondente.
- Resolver a proporo.

Regra de Trs Composta
Regra de trs composta um processo matemtico
mediante o qual podemos resolver problemas do cotidiano,
envolvendo trs ou mais grandezas. O processo
semelhante ao caso anterior (Regra de trs simples),
levando em considerao apenas o item da verificao
quanto a GDP ou GIP, que deve ser feito da seguinte
maneira: analisar as grandezas duas a duas, sempre em
relao que possui a varivel. A montagem e resoluo
da proporo seguem o mesmo roteiro do caso anterior
(Regra de Trs Simples).

PORCENTAGEM

As razes cujos denominadores so iguais a 100 so
chamadas razes centesimais.
Exemplo: ;
100
27
;
100
13
etc.

Noo Intuitiva
O ndice de analfabetismo da cidade x de 23% (l-se 23
por cento). Significa que, em mdia, 23 de cada 100
habitantes so analfabetos.

Clculo de uma porcentagem

Exemplo: 25% de R$ 80,00 R$ 20,00
pois 25% =
100
25
= 0,25
Logo 25% de R$ 80,00 = 0,25.80,00 = 20,00

Definio
Porcentagem uma razo centesimal que representada
pelo smbolo % que significa por cento.

Exerccios de Sala #

1. Se 12Kg de um certo produto custa R$ 600,00, qual o
preo de 25Kg do mesmo produto?

2. Sabendo que 36 operrios conseguem construir uma
casa em 30 dias, se dispomos apenas de 12 desses
operrios, em quanto tempo ser construda a mesma casa?

3. Calcular:
a) 60% de 30 d) 20% de 20%
b) 30% de 20 e) (20%)
2

c) 20% de 300 f) % 4

4. Numa cidade, 240 000 jovens representam 30% da
populao. Ento a populao da cidade de:

a) 500 000 habitantes d) 800 000 habitantes
b) 600 000 habitantes e) 900 000 habitantes
c) 700 000 habitantes

Tarefa Mnima #

5. Se trinta litros de um combustvel custam R$ 16,95,
quantos custaro oitenta litros do mesmo combustvel?

6. Se 14 pedreiros levam 180 dias para construir uma casa,
quanto tempo levaro para constru-la 10 pedreiros?

7. Um acampamento com 80 pessoas tem suprimento para
dez dias. Sabendo-se que chegaram mais vinte soldados,
pergunta-se: para quantos dias tero suprimentos,
considerando-os inalterveis?

8. Calcular as seguintes porcentagens:
a) 25% de 80 e) 20% de 30%
b) 4% de 50 f) (5%)
2

c) 120% de 200 g) % 49
d) 0,15% de 400

9. Numa sala de 80 alunos, 24 alunos foram aprovados. A
porcentagem de reprovao foi de:
a) 30% c) 50% e) 70%
b) 40% d) 60%

10. (UFSC) Ao vestibular de 1982 da UFSC, inscreveram-
se 15.325 candidatos, dos quais 14.099 concluram todas
as provas. O percentual de absteno foi:

11. Qual o preo de uma mercadoria que custava R$
80,00 e teve um aumento de 40%?
a) 110,00 c) 114,00 e) 98,00
b) 112,00 d) 116,00

12. (CESCEM-SP) 3% de 0,009 vale:
a) 0,00027 c) 0,00009 e) n.d.a.
b) 0,0027 d) 0,009
Please purchase PDF Split-Merge on www.verypdf.com to remove this watermark.
Matemtica D Incluso para a Vida

Pr-Vestibular da UFSC 2
Tarefa Complementar#

13. (UNIMEP-SP) Se dois gatos comem dois ratos em
dois minutos, para comer 60 ratos em 30 minutos so
necessrios:
a) 4 gatos c) 2 gatos e) 6 gatos
b) 3 gatos d) 5 gatos

14. Dezesseis operrios trabalhando seis horas por dia
constroem uma residncia em cento e oitenta dias. Quantos
operrios sero necessrios para fazer a mesma residncia,
trabalhando oito horas por dia durante cento e vinte dias?
a) 18 c) 19 e) 21
b) 10 d) 20

15. Durante 11 dias, 15 cavalos consomem 2200 kg de
alfafa. Retirando-se 7 cavalos, 1280 kg de alfafa sero
consumidos em quantos dias?
a) 12 c) 14 e) 16
b) 13 d) 15

16. (UFSC) Com uma lata de tinta possvel pintar 50 m
2

de parede. Para pintar uma parede de 72m
2
, gasta-se uma
lata e mais uma parte de uma segunda lata. A parte que se
gasta da segunda lata, em porcentagem, :

17. (UFSC) Pedro investiu R$ 1.500,00 em aes. Aps
algum tempo, vendeu essas aes por R$ 2.100,00.
Determine o percentual de aumento obtido em seu capital
inicial.

18. (UFSC) Um reservatrio contendo 120 litros de gua
apresentava um ndice de salinidade de 12%. Devido
evaporao, esse ndice subiu para 15%. Determinar, em
litros, o volume de gua evaporada.

19. (UFSC) Assinale a soma dos nmeros associados (s)
proposio(es) correta(s).
01. Um investidor tem seu dinheiro aplicado a 2% ao ms.
Deseja comprar um bem no valor de R$100.000,00,
que pode ser pago a vista ou em trs parcelas de R$
34.000,00, sendo a primeira de entrada e as outras em
30 e 60 dias. Ele sair lucrando se fizer a compra
parcelada.
02. Obter 7 acertos numa prova de 12 questes um
desempenho inferior a obter 6 acertos numa prova de
10 questes, porm superior a obter 5 acertos numa
prova de 9 questes.
04. Duplicando-se o lado de um tringulo equiltero, sua
rea fica tambm duplicada.
08. Se 2 impressoras trabalhando 10 horas por dia levam 5
dias para fazer determinado trabalho, ento 3
impressoras (com a mesma eficincia das anteriores)
trabalhando 8 horas por dia levaro 6 dias para fazer o
mesmo trabalho

UNIDADE 2

FATORIAL

Dado um nmero natural, denomina-se fatorial de n e
indica-se por n! a expresso:


n! = n.(n 1) . (n 2) . (n 3). ......... . 3 . 2 . 1

Assim temos:

5! = 5. 4. 3. 2. 1 = 120
4! = 4. 3. 2. 1 = 24
3! = 3. 2. 1 = 6
2! = 2. 1 = 2

1! = 1 e 0! = 1 (conceito primitivo)

Observao: Podemos desenvolver um fatorial at um
fator conveniente. Veja:

8! = 8. 7. 6. 5. 4. 3. 2. 1 = 8. 7. 6. 5. 4!

4!
6! = 6. 5. 4. 3. 2. 1 = 6. 5!

5!
n ! = n. (n 1).(n 2) !

PRINCPIO FUNDAMENTAL DA CONTAGEM
FRMULA DO ARRANJO

PRINCPIO FUNDAMENTAL DA CONTAGEM
O princpio fundamental da contagem, ou princpio
multiplicativo, estabelece um mtodo indireto de contagem
de um determinado evento, sem que haja a necessidade de
descrever todas as possibilidades. Pode ser enunciado
dessa forma:

Se um Evento E pode acontecer por n etapas sucessivas e
independentes de modo que:

E
1
o nmero de possibilidades da 1 Etapa
E
2
o nmero de possibilidades da 2 Etapa
E
n
o nmero de possibilidades da n-sima Etapa

Ento E
1
. E
2
. ......... .E
k
o nmero total de
possibilidades do evento ocorrer.

ARRANJO
Considere o conjunto K = {1, 2, 3, 4}. Vamos agora
montar os pares ordenados a partir do conjunto K.

(1, 2); (1, 3); (1, 4); (2, 3); (2, 4); (3; 4);
(2, 1); (3, 1); (4, 1); (3, 2); (4, 2); (4, 3)

Observe que esses agrupamentos diferem

- Pela natureza dos elementos componentes:
(2, 3) = (1,4)
- Pela ordem dos elementos:
(1, 3) = (3, 1)

A esses tipos de agrupamentos denomina-se ARRANJO
de n elementos tomados p a p, e indicado por
p n
A
,
.

Definio: Denomina-se arranjo de n elementos tomados p
a p cada grupo ordenado de p elementos escolhidos entre
n disponveis.




Please purchase PDF Split-Merge on www.verypdf.com to remove this watermark.
Incluso para a vida Matemtica D

Pr-Vestibular da UFSC
3
FRMULAS PARA O CLCULO DO ARRANJO


- ARRANJO COM REPETIO

A
*

n,p
= n
p

Exemplo: Considere o conjunto K = {2, 3, 4, 5, 6}.
Quantos nmeros de 3 algarismos podemos formar a partir
de K ?
Resoluo: A
*
5, 3
= 5
3
= 125
Logo, podemos formar 125 nmeros de 3 algarismos.

- ARRANJO SEM REPETIO (SIMPLES)
A
np
n
n p
,
!
( )!
=



Exemplo: Considerando o conjunto K = {1, 2, 3, 4, 5}.
Quantos nmeros de 3 algarismos sem repetio podem
ser formados?
Resoluo: A
5,3
=
5
5 3
5 4 3 2
2
60
!
( )!
. . . !
!
= =

Logo, podemos formar 60 nmeros de 3 algarismos
distintos.

Exerccios de Sala #

1. Calcular o valor de:
a)
10
8
!
!
b)
11!
11! 12!

2. Resolver as equaes:
a) (n 3) ! = 720 b)
( )!
( )!
n
n
+
+
=
3
1
20


3. Quatro selees de futebol (Brasil, Espanha, Portugal e
Uruguai) disputam um torneio. Quantas e quais so as
possibilidades de classificao para os dois primeiros
lugares?

4. Quantas placas para identificao de veculos podem
ser confeccionadas com 3 letras e 4 algarismos?
(Considere 26 letras, supondo que no h nenhuma
restrio.)

5. Considere o conjunto K = {1, 2, 3, 4, 5, 6, 7}. Quantos
nmeros com quatro algarismos distintos podemos formar
a partir do conjunto K?

Tarefa Mnima #

6. Calcular
5
3 2
!
! ! +
.

7. Resolver as equaes abaixo:
a) (n - 4)! = 120 c) (n - 2)! = 720
b) (4x - 6)! -120 = 600

8. Ache a soluo da equao
( )
12
)! 3 (
! 1
=

x
x


9. Dum ponto A a um ponto B existem 5 caminhos; de B a
um terceiro ponto C existem 6 caminhos; e de C a um
quarto ponto D existem tambm 6 caminhos. Quantos
caminhos existem para ir do ponto A ao ponto D?
a) 17 b) 30 c) 180 d) 680 e) 4080

10. Numa olimpada de Matemtica concorrem 100
participantes e sero atribudos dois prmios, um para o 1
lugar e outro para o 2 lugar. De quantas maneiras podero
ser distribudos esses prmios?
a) 199 c) 4.950 e) 10.000
b) 200 d) 9.900

11. Telefones de uma cidade possui 6 dgitos (1nunca
zero). Supondo que a cidade passe a ter 7 dgitos. Qual o
aumento no nmero de telefones?
a) 81.10
5
b) 8100 c) 90000 d) 90.10
3


Tarefa Complementar #

12. Qual o valor de n que satisfaz a equao
( ) n n
n
+ +
+
=
1
2
5
! !
( )!


13. Quantas solues possui a equao (x 2)! = 1

14. (UFPA) Simplificando
( )! !
( )!
n n
n
+ +
+
1
2
obtm-se:
a)
1
2 n +
d)
1
1 n +

b) n + 1 e) n
c) n+2
15. (FSBEF-DF) Sendo
( ) !
( )!
m m
m
+
+
=
1
2
1
10
e tendo em vista
que m > 0, o valor de m :

16. Se (n 6)! = 720, ento n igual a:

17. (F.Dom Bosco-DF) A expresso 3! 2! 2!
equivalente expresso:
a) 12! b) 7! c) 5! d) 5! e) 4!

18. Durante a Copa do Mundo, que foi disputada por 24
pases, as tampinhas de Coca-Cola traziam palpites sobre
os pases que se classificariam nos trs primeiros lugares
Se, em cada tampinha, os trs pases so distintos,
quantas tampinhas diferentes poderiam existir?
a) 69 c) 9.562
b) 2.024 d) 12.144 e) 13.824

19. (UECE) A quantidade de nmeros inteiros
compreendidos entre os nmeros 1000 e 4500 que
podemos formar utilizando somente os algarismos 1, 3, 4,
5 e 7, de modo que no figurem algarismos repetidos, :

20. (PUC-SP) Chamam-se palndromos os nmeros
inteiros que no se alteram quando invertida a ordem de
seus algarismos (por exemplo: 383, 4224, 74847). O
nmero total de palndromos com cinco algarismos :
a) 450 d) 2500
b) 1000 e) 5000
c) 900



Please purchase PDF Split-Merge on www.verypdf.com to remove this watermark.
Matemtica D Incluso para a Vida

Pr-Vestibular da UFSC 4
UNIDADE 3

TIPOS DE AGRUPAMENTOS PARTE II -
PERMUTAES

Quando fazemos arranjos de n elementos tomados n a n,
sem repetio, estamos montando grupos com todos os
elementos disponveis. Dizemos que esse tipo de
Agrupamento denominado PERMUTAO de n
elementos, e indicado por P
n
. Considere ento, o
conjunto K = {1, 2, 3}. As permutaes com esses
elementos so:

(1, 2, 3); (1, 3, 2); (2, 1, 3); (2, 3, 1); (3, 1, 2),
(3, 2, 1).

FRMULAS PARA O CLCULO DA PERMUTAO

- PERMUTAO SIMPLES

P
n
= n!

Exemplo 1: Quantos nmeros de 4 algarismos
distintos podemos formar com os nmeros usando os
algarismos { 2, 5, 6, 7}.

Resoluo: P
4
= 4! = 4.3.2.1 = 24
Logo, pode-se formar 24 nmeros com 4
algarismos distintos.

Exemplo 2: Calcule o nmero de anagramas da palavra
VASCO.

Resoluo: Cada anagrama uma permutao das letras V,
A, S, C e O. Como so 5 letras distintas, o nmero de
anagramas dado por:

P
5
= 5! = 5.4.3.2.1 = 120

Logo, pode-se formar 120 anagramas com as letras
que compem a palavra VASCO.


PERMUTAO COM REPETIO
Vamos considerar um conjunto com n elementos, dos
quais um dos deles repete o vezes, outro | vezes e assim
por diante, at que um elemento repita vezes. O nmero
de permutaes possveis dado pela expresso:


P
n
....
n
o|
o |
, ,
=
!
! !...... !


Exemplo: Quantos anagramas podemos formar com as
letras da palavra ARARA.

Resoluo: n = 5 o = 3 | = 2
P
5
3, 2
=
5
3 2
!
! !
=10

Logo, podemos formar 10 anagramas com as letras
que compem a palavra ARARA.

TIPOS DE AGRUPAMENTOS PARTE III -
COMBINAES

Considere o conjunto K = {1, 2, 3, 4}.
Vamos montar agora os subconjuntos com dois destes
elementos.
{1, 2}; {1, 3}; {1, 4}; {2, 3}; {2, 4}; {3, 4}.

Observe que esses agrupamentos diferem

- Apenas pela natureza dos elementos componentes: {1,
2} = {1, 4}
- Mas no diferem pela ordem: {1, 3} = {3, 1}

Esses tipos de agrupamentos so chamados de
COMBINAO de n elementos tomados p a p, e so
indicados por
C
np
ou C
n
p
,
.

Definio: Denomina-se combinao de n elementos p a p
todo subconjunto de p elementos.

FRMULA PARA O CLCULO DA COMBINAO

O nmero de combinaes simples dos n elementos
tomados p a p dado pela expresso:

! )! (
!
,
p p n
n
p n
C

=


Exemplo: Quantas comisses de 3 pessoas podemos
formar com um grupo de 10 pessoas.

Resoluo: As comisses so subconjuntos de 3 pessoas
escolhidas entre as 10, logo:

C
10,3
=
10
10 3 3
10 9 87
7 3 21
!
( )! !
. . . !
! . .
= = 120

Portanto, podemos formar 120 comisses de 3 pessoas
com um grupo de10 pessoas.

Exerccios de Sala #

1. Quantos so os anagramas das palavras:
a) ROMA
b) ESCOLA
c) BANANA.
d) MATEMATICA

2. Quantos so os anagramas da palavra MXICO em que
aparecem as letra E e X sempre juntas?

3. Quantas comisses de 2 pessoas podem ser formadas
com 5 alunos (A,B,C,D,E) de uma classe?

4. Marcam-se 8 pontos distintos numa circunferncia.
Quantos tringulos com vrtices nesses pontos podemos
obter?

Tarefa Mnima #

5. Quantos nmeros de 4 algarismos distintos podemos
formar com os nmeros utilizando os algarismos { 1, 3, 8,
9}.

Please purchase PDF Split-Merge on www.verypdf.com to remove this watermark.
Incluso para a vida Matemtica D

Pr-Vestibular da UFSC
5
6. Quantos nmeros diferentes obteremos permutando os
algarismos do nmero 336.223?

7. Quantos so os anagramas da palavra SAPO?

8. Determine os nmero de anagramas da palavra
CARCAR? (no considere o acento)

9. O valor de x em C
x,3
= 35, :
a) 12 c) 7 e) 9
b) 10 d) 8

10. Quantas comisses constitudas por 4 pessoas podem
ser formadas com 10 alunos de uma classe?
a) 210 c) 240 e) 200
b) 120 d) 100

11. Numa circunferncia so tomados 8 pontos distintos.
Ligando-se dois quaisquer desses pontos, obtm-se uma
corda. O nmero total de cordas assim formadas :

Tarefa Complementar #

12. Quanto aos anagramas da palavra ENIGMA, temos as
afirmaes:
I - O nmero total deles 720.
II - O nmero dos que terminam com a letra A 25.
III - O nmero dos que comeam com EN 24.
Ento apenas:
a) a afirmao I verdadeira.
b) a afirmao II verdadeira.
c) a afirmao III verdadeira.
d) as afirmaes I e II so verdadeiras.
e) as afirmaes I e III so verdadeiras.

13. (CEFET-PR) O nmero de anagramas da palavra
NMERO, em que nem as vogais nem as consoantes
fiquem juntas, :
a) 12 c) 48 e) 72
b) 36 d) 60

14. (PUC-SP) Alfredo, Armando, Ricardo, Renato e
Ernesto querem formar uma sigla com cinco smbolos,
onde cada smbolo a primeira letra de cada nome. O
nmero total de siglas possveis :

15. Considere um grupo de 3 moas e 4 rapazes. O
nmero de comisso de 4 membros, de modo que em cada
comisso figure pelo menos um rapaz, :

16. Os presentes a determinada reunio, ao final da
mesma, cumprimentam-se mutuamente, com aperto de
mo. Os cumprimentos foram em nmero de 66. O nmero
de pessoas presentes reunio :

17. (ACAFE) Diagonal de um polgono convexo o
segmento de reta que une dois vrtices no consecutivos
do polgono. Se um polgono convexo tem 9 lados, qual
o seu nmero total de diagonais?
a) 72 c) 36 e) 18
b) 63 d) 27

18. (UFRN) Se o nmero de combinaes de n + 2
elementos 4 a 4 est, para o nmero de combinaes de n
elementos 2 a 2, na razo de 14 para 3, ento n vale:
a) 6 b) 8 c) 10 d) 12 e) 14

UNIDADE 4

NMEROS BINOMIAIS

Dados dois nmeros naturais n e p, denomina-se nmero
binomial de n sobre p e indicado por
n
p
|
\

|
.
|
ao nmero
definido por:


|
|
.
|

\
|
p
n
=
p)! (n p!
n!

com n e N, p e N e n > p

Podemos concluir de imediato que:

a
n
0
1 b)
n
1
n c)
n
n
1 )
|
\

|
.
| =
|
\

|
.
| =
|
\

|
.
| =


NMEROS BINOMIAIS COMPLEMENTARES
Dois nmeros binomiais de mesmo numerador so
chamados complementares quando a soma dos
denominadores (classes) igual ao numerador.

Exemplos:
a)
n
p
e
n
n p
|
\

|
.
|

|
\

|
.
|
b)
5
2
e
5
3
|
\

|
.
|
|
\

|
.
|



PROPRIEDADES DOS NMEROS BINOMIAIS

1) Dois nmeros binomiais complementares so
iguais.
Ento se
n
k
n
p
k p
ou
k p n
|
\

|
.
| =
|
\

|
.
|
=
+ =



2 RELAO DE STIFFEL


n 1
p 1
n 1
p
n
p

|
\

|
.
|
+
|
\

|
.
|
=
|
\

|
.
|


Veja que
5
3
5
4
6
4
|
\

|
.
| +
|
\

|
.
| =
|
\

|
.
|


TRINGULO DE PASCAL

Vamos dispor agora os nmeros binomiais em um
tringulo, de forma que os binomiais de mesmo numerador
fiquem na mesma linha, e os binomiais de mesmo
denominador fiquem na mesma coluna.

Please purchase PDF Split-Merge on www.verypdf.com to remove this watermark.
Matemtica D Incluso para a Vida

Pr-Vestibular da UFSC 6
col 0 col 1 col 2 col 3 col 4 col 5 col 6
linha 0
0
0


1
0

1
1
linha 2
2
0

2
1

2
2
linha 3
3
0

3
1

3
2

3
3
linha 4
4
0

4
1

4
2

4
3

4
4

5
0
|
\

|
.
|
|
\

|
.
|
|
\

|
.
|
|
\

|
.
|
|
\

|
.
|
|
\

|
.
|
|
\

|
.
|
|
\

|
.
|
|
\

|
.
|
|
\

|
.
|
|
\

|
.
|
|
\

|
.
|
|
\

|
.
|
|
\

|
.
|
|
\

|
.
|
|
linha
linha 5
1
\

|
.
|
|
\

|
.
|
|
\

|
.
|
|
\

|
.
|
|
\

|
.
|
|
\

|
.
|
|
\

|
.
|
|
\

|
.
|
|
\

|
.
|
|
\

|
.
|
|
\

|
.
|
|
\

|
.
|
|
\

|
.
|

5
1

5
2

5
3

5
4

5
5
linha 6
6
0
6
1
6
2
6
3
6
4
6
5
6
6

. . . . . . . . . . . . . . . . . . . . . . . . . . . . . . . . . . . . . . . . . . . . . . . . .

Substituindo cada binomial pelo respectivo valor, temos:

PROPRIEDADES DO TRINGULO DE PASCAL

- PRIMEIRA PROPRIEDADE
Todos os elementos da 1 coluna so iguais a 1.

- SEGUNDA PROPRIEDADE
O ltimo elemento de cada linha igual a 1.

- TERCEIRA PROPRIEDADE
Numa linha qualquer dois binomiais equidistantes dos
extremos so iguais. (binomiais complementares)

- QUARTA PROPRIEDADE
Cada binomial
n
p
|
\

|
.
|
da linha n igual soma de dois
binomiais da linha (n - 1); aquele que est na coluna p com
aquele que est na coluna (p - 1).

|
|
.
|

\
|
|
|
.
|

\
|
|
|
.
|

\
|
=

p
n
p
1 n
1 p
1 n



- QUINTA PROPRIEDADE
A soma dos elementos da linha do numerador n igual
a 2
n
.

Linha 0 1 = 2
0

Linha 1 1 + 1 = 2
1

Linha 2 1 + 2 + 1 = 2
2

Linha 3 1 + 3 + 3 + 1 = 2
3


Exerccios de Sala #

1. Calcule A, sendo A =
4
0
8
2
9
7
10
1
|
\

|
.
| +
|
\

|
.
| +
|
\

|
.
| +
|
\

|
.
|


2. Ache o conjunto soluo da equao
n |
\

|
.
| =
3
2
21


3. Calcule o valor de:
a)
_
=
|
|
.
|

\
|
7
0
7
p
p
b)
_
=
|
|
.
|

\
|
10
0
10
p
p
c)
_
=
|
|
.
|

\
|
8
3
8
p
p


4. Resolva a equao:
|
|
.
|

\
|
=
|
|
.
|

\
|
+
|
|
.
|

\
|
x
15
5
14
4
14


Tarefa Mnima#

5. Calcule E, sendo E =
5
2
3
3
5
0
7
1
|
\

|
.
|
+
|
\

|
.
|
+
|
\

|
.
|
+
|
\

|
.
|
.
6. (UECE) A soma das solues da equao
18
6
18
4 1
|
\

|
.
| =

|
\

|
.
|

x
:
a) 8 b) 5 c) 6 d) 7

7. (PUC-SP) A soma dos valores que m pode assumir na
igualdade:
17
m 1
17
2m 6
|
\

|
.
| =

|
\

|
.
|

8. Calcule
5
0
5
p
p
|
\

|
.
|
=
_

9. Resolva a equao:
8
6
8
7
9
3
|
\

|
.
| +
|
\

|
.
| =
+
|
\

|
.
|
x

10. (Mack-SP) O valor de
7
2
7
3
7
4
7
5
7
6
7
7
|
\

|
.
| +
|
\

|
.
| +
|
\

|
.
| +
|
\

|
.
| +
|
\

|
.
| +
|
\

|
.
|
:
a) 128 b) 124 c) 120 d) 116 e) 112

Tarefa Complementar #

11. (Mack-SP) Considere a sequncia de afirmaes:

. . .
| | | | | | | | | | | |
= = =
| | | | | |
\ . \ . \ . \ . \ . \ .
15 15 15 15 15 15
I II III
1 3 2 13 3x 6


Associando V ou F a cada afirmao, conforme seja
verdadeira ou falsa, tem-se:
a) F, F, V c) F, V, F
b) F, V, V d) F, F, F e) V, V, V
12. (Fatec-SP) Calcule E de modo que
E
p 1
n 1
n 1
p 1
=
+
+
+
+
|
\

|
.
|

onde p, n e N
*
e p < n

ou
n
0 = p
n n
p
n
n
n n n
o
n
2 2 .......
2 1
=
|
|
.
|

\
|
=
|
|
.
|

\
|
+ +
|
|
.
|

\
|
+
|
|
.
|

\
|
+
|
|
.
|

\
|
_

13. (U.C.-MG) O resultado de
8
2
6
p
p
|
\

|
.
|
=
_
igual a:
a) 216 b) 238 c) 240 d) 247 e) 256
Please purchase PDF Split-Merge on www.verypdf.com to remove this watermark.
Incluso para a vida Matemtica D

Pr-Vestibular da UFSC
7
14. (UNESP) Seja num nmero natural tal que
10
4
10
1
11
4
|
\

|
.
| +
+
|
\

|
.
| =
|
\

|
.
|
n
. Ento:
a) n = 5 b) n = 4 c) n = 3 d) n = 2

15. (FGV-SP) Sabendo-se que
m
p
x e y
|
\

|
.
| =
|
\

|
.
| =
|
\

|
.
|
m+1
p +1
entao
m
p +1
,
:
a) x + y b) x - y c) y - x d) x - p e) y p

UNIDADE 5

BINMIO DE NEWTON

Observe abaixo os desenvolvimentos:
- (a + b)
0
= 1
- (a + b)
1
= 1a + 1b
- (a + b)
2
= 1a
2
+ 2ab + 1b
2

- (a + b)
3
= 1a
3
+ 3a
2
b + 3ab
2
+ 1b
3

- (a + b)
4
= 1a
4
+ 4a
3
b + 6a
2
b
2
+ 4ab
3
+ 1b
4

- (a + b)
5
= 1a
5
+ 5a
4
b + 10a
3
b
2
+ 10a
2
b
3
+ 5ab
4
+ 1b
5


Observe que:
O nmero de termos do desenvolvimento de (a + b)
n

n + 1.
Os coeficientes dos termos do desenvolvimento de (a + b)
n

formam o tringulo de Pascal.
Os expoentes de a decrescem de n a 0, e os expoentes de b
crescem de 0 a n.
A soma dos expoentes de a e b sempre igual a n

Com base nessas observaes podemos generalizar o
desenvolvimento de (a + b)
n
. Veja:

( ) ...... a b
n
b
n
b
n
b
n
n
b
n n n
+ =
|
\

|
.
| +
|
\

|
.
| +
|
\

|
.
| +. +
|
\

|
.
|

0 1 2
0 1 2 2 0
a a a a
n n-1

Um termo qualquer do desenvolvimento de (a + b)
n
dado
pela expresso:

T
p 1
n
p
a
n p
b
p
+
=

|
\

|
.
|. .


Exerccios de Sala #

1. Desenvolver o binmio (x + 2)
4


2. Determinar o 5 termo do desenvolvimento de (x + 2)
6
.

3. Determinar o termo independente no desenvolvimento
de (2x + 3)
4
.

4. A soma dos coeficientes do desenvolvimento do
binmio (4x 3y)
6

Tarefa Mnima#

5. Determinar o coeficiente numrico do 4 termo no
desenvolvimento de (x + 2)
7
.

6. Achar o termo independente de x no desenvolvimento
de (2x 1)
6
.
7. Se a soma dos coeficientes do binmio ( ) a b
m
+
1

64, ento o valor de m :

8. (UEL-PR) Para qualquer valor natural de n, o nmero
de termos do binmio (x + a)
n
:
a) n + 1 b) n c) n - 1 d) par e) mpar

9. (UFRN) A soma dos coeficientes dos termos do
desenvolvimento do binmio (x + a)
n
:

a) 2n b) n/2 c) n + 2 d) n
2
e) 2
n


Tarefa Complementar #

10. (UDESC) Sendo 125 a soma dos coeficientes do
desenvolvimento de (2x + 3y)
m
. O valor de m! :
a) 6 b) 24 c) 120 d) 2 e) 3

11. (CEFET-PR) O 4 termo do desenvolvimento de
(x + 2)
6
:
a) 80x
3
b) 80x
4
c) 40x
5
d) 320x
3
e) 160x
3


12. (MACK-SP) Qual a soma dos coeficientes numricos
do desenvolvimento de
3
2
2
8
x
x

|
\

|
.
|
?
13. (FAAP-SP) O sexto termo do desenvolvimento de
(x + 2 )
8
pelo binmio de Newton :
a) 48x
3
b)10752x
3
c) 1792x
3
d) 3584x
3


14. (Mack-SP) O coeficiente x
3
do desenvolvimento de
3
1
5
x
x

|
\

|
.
|
:
a) -405 b) -90 c) -243 d) -27 e) -81

UNIDADE 6

POLINMIOS

DEFINIO
Dados os nmeros reais a
n
, a
n - 1
, ....., a
2
, a
1
e a
0
,
chamamos de polinmio na varivel x toda expresso da
forma:

P(x) = a
n
x
n
+ a
n - 1
x
n - 1
+ ..... + a
2
x
2
+ a
1
x + a
0


Nomenclatura
COEFICIENTES: a
n
, a
n - 1
, .........a
2
, a
1
, a
0
.
TERMOS: a
n
x
n
, a
n - 1
x
n - 1
, ..... a
2
x
2
, a
1
x, a
0

TERMO INDEPENDENTE: a
0
n um nmero natural e indica o grau do polinmio se a
n

for diferente de zero.

Observao: Se P(x) = 0, no definido o grau do
polinmio.

VALOR NUMRICO
Valor Numrico de um polinmio P(x), o valor que se
obtm substituindo a varivel x por um nmero o e
efetuando as operaes indicadas.

Observao: Quando P(o) = 0 dizemos que o a raiz do
polinmio.
Observe que os nmeros 2 e 3 so razes do polinmio
P(x) = x
2
- 5x + 6, pois P(2) = 0 e P(3) = 0.
Please purchase PDF Split-Merge on www.verypdf.com to remove this watermark.
Matemtica D Incluso para a Vida

Pr-Vestibular da UFSC 8
POLINMIOS IDNTICOS

Dados os polinmios:
P
1
(x) = a
n
x
n
+ a
n - 1
x
n - 1
+ ..... + a
2
x
2
+ a
1
x + a
0
e
P
2
(x) = b
n
x
n
+ b
n - 1
x
n - 1
+ ..... + b
2
x
2
+ b
1
x + b
0


A condio para que P
1
e P
2
sejam idnticos que os
coeficientes dos termos de mesmo grau sejam iguais.
Indicamos por P
1
(x) P
2
(x)

Assim: a
n
= b
n
; a
n - 1
= b
n - 1
; a
2
= b
2
; a
1
= b
1
;
a
0
= b
0


Vale ressaltar que, se P
1
e P
2
so idnticos, para qualquer
valor de x eles assumem o mesmo valor numrico.

Em smbolos: P
1
(x) P
2
(x) P
1
(x) = P
2
(x)

Exerccios de Sala #

1. Encontre o valor numrico do polinmio P(x) = 5x
4
+
2x
3
x
2
+ 3x 3 para x = 3.

2. Dado o polinmio P(x) = (a
2
4)x
2
+ (a + 2)x + 3.
Determine o valor de a de modo que P(x) seja do 1 grau.

3. Seja P(x) = ax
2
+ bx + c, em que a, b, e c so nmeros
reais. Sabendo que P(0) = 9, P(1) = 10 e P(2) = 7, calcule
P(3).

Tarefa Mnima #

4. Dado P(x) = 2x
3
+ 3x
2
5, calcule:

a) P(0) b) P(1) c) P(2)

5. Considere o polinmio P(x) = mx
2
5x + 2. Sabendo
que P(-2) = - 4, determine o valor de m.

6. Sabendo-se que P
1
(x) = ax
2
+ (b + c)x - 2a - 3x
2
+ 3cx +
3b + 1 e P
2
(x) = 10x
2
+ 158x + 29 so polinmios
idnticos, determine o valor da expresso: a + b + c.

7. O polinmio p(x) = (a - 3)x
3
+ (b + 2a)x
2
+ (6b + c)x
identicamente nulo. Calcule o valor de 2(a + b + c).

8. Se
x
x x
A
x
B
x
+
+

+
+
1
2 24 4 6
2
, ento 2A + B igual a:
a) -3/2 b) 1/2 c) 1 d) 3/2 e) -1

Tarefa Complementar #

9. (UEM-PR) Seja P(x) = ax
2
+ bx + c, em que a, b, e c
so nmeros reais. Sabendo que P(0) = 9, P(1) = 10 e
P(2) = 7, calcule P(3).

10. (PUC-SP) Efetuando a soma de
ax b
x
e
c
x
+
+
2
1 1
,
obtemos a expresso
x
x x

+
3
1 1
2
( )( )
. Os valores de a, b e c
so respectivamente:
a) 0, 1, -3 c) -1, 1, 1
b) 1, -1, -3 d) 1, 2, -1 e) 2, 1, -2

11. (ABC-SP) Num polinmio P(x) de 3 grau, o
coeficiente de x
3
1. Se P(1) = P(2) = 0 e P(3) = 30, o
valor de P(1) :

12. (UFRGS) O polinmio do 2 grau p(x), que tem zero
como raiz e tal que p(x) - p(x - 1) = 6x - 2,
a) 2x
2
+ 3x 6 c) 6x
2
- x
b) 6x - 2 d) 3x
2
+ x e) x
2
+ 3x

13. (Londrina-PR) Sendo F, G e H polinmios de graus 4,
6 e 3, respectivamente, o grau de (F + G).H ser:
a) 9 b) 10 c) 12 d) 18 e) 30

UNIDADE 7

DIVISO DE POLINMIOS

Dados os polinmios P(x) e D(x), com D(x) no
identicamente nulos, dividir P(x) por D(x) equivale obter
os polinmios Q(x) (quociente) e R(x) (resto), tais que:

P(x) D(x)
R(x) Q(x)

- P(x) D(x) . Q(x) + R(x)
- gr(R) < gr(D) ou R(x) 0

Onde:
P(x) o dividendo
D(x) o divisor
Q(x) o quociente
R(x) o resto

OBSERVAES:
- O grau de Q(x) a diferena entre os graus de P(x) e
de D(x), ou seja, gr(Q) = gr(P) gr(D)
- Se R(x) for um polinmio nulo, apontamos que P(x)
divisvel por D(x), dizemos ento, que a diviso
exata.
MTODO DA CHAVE
(ALGORITMO DE EUCLIDES)
O mtodo das chaves um dos quais podemos obter o
quociente entre dois polinmios. Para isso, devemos seguir
os seguintes procedimentos:
- Ordenamos os polinmios P(x) e D(x) segundo as
potncias decrescentes de x.
- Dividi-se o primeiro termo de P(x) pelo primeiro de
D(x), obtendo o primeiro termo de Q(x).
- Multiplica-se o termo obtido pelo divisor D(x) e
subtrai-se de P(x)
- Continua-se o processo at que haja um resto de grau
inferior que o de D(x).

Exemplo: Determinar o quociente e o resto da diviso de
P(x) = 4x
3
2x
2
+ 6x 10 por D(x) = 2x
2
+ 3x + 2

Resoluo:




Please purchase PDF Split-Merge on www.verypdf.com to remove this watermark.
Incluso para a vida Matemtica D

Pr-Vestibular da UFSC
9
Observe que:

4x
3
2x
2
+ 6x 10 = (2x
2
+ 3x + 2) . (2x 4) + (14x 2)

Dividendo Divisor Quociente Resto

MTODO DE DESCARTES
Mtodo de Descartes ou Mtodo dos Coeficientes a
determinar um Mtodo que consiste na obteno dos
coeficientes do quociente e do resto com o auxlio da
seguinte identidade de Polinmios:
P(x) D(x) . Q(x) + R(x)

onde gr(Q) = gr(P) gr(D) e gr(R) < gr(D)

Exemplo: Obter o quociente e o resto da diviso do
polinmio P(x) = x
4
x
3
2x
2
x + 3 por
D(x) = x
3
3x
2
+ 2
Resoluo: O grau do resto no mximo 2, pois
gr(R) < gr(D) e gr(Q) = gr(P) gr(D)
gr(Q) = 4 3 = 1

Isso nos permite escrever:

R(x) = cx
2
+ dx + e e Q(x) = ax + b

Aplicando a identidade, temos:

P(x D(x) . Q(x) + R(x)

x
4
x
3
2x
2
x + 3 (x
3
3x
2
+ 2) . (ax + b) + cx
2
+ dx
+ e

x
4
x
3
2x
2
x + 3 ax
4
+ (b 3a)x
3
+ (c 3b)x
2
+ (2a +
d)x + (2b + e)

Da vem:

= +
= +
=
=
=
3 e 2b
1 d 2a
2 3b c
1 3a b
1 a
resolvendo o sistema, temos:
a = 1, b = 2, c = 4, d = 3, e = 1

Logo: Q(x) = x + 2 e R(x) = 2x
2
3x 1

TEOREMA DO RESTO
O resto da diviso de um polinmio P(x) por um binmio
do tipo ax + b o valor numrico de P(x) para
x =
b
a
, ou seja P(
b
a
).
Observe que

b
a
a raiz do divisor.

Esse teorema nos permite achar o resto de uma diviso
sem que haja a necessidade de aplicar o mtodo das chaves
ou o mtodo de Descartes.
Exemplo: Determinar o resto da diviso do polinmio
P(x) = 2x
2
+ 3x + 1 pelo polinmio D(x) = x 3

Resoluo: A raiz do divisor 3, logo, para determinarmos
o resto da diviso de P(x) por D(x), basta
calcular P(3). Da vem:
P(x) = 2x
2
+ 3x + 1
P(3) = 2(3)
2
+ 3(3) + 1
P(3) = 28
TEOREMA DE D'ALEMBERT

Um polinmio P(x) divisvel por D(x) = ax + b se, e
somente se, P(
b
a
) = 0.
Veja por exemplo que o polinmio P(x) = x
3
3x + 2
divisvel por (x + 2) pois P(2) = 0.

Exemplo: Determinar o valor de m de modo que o
polinmio P(x) = x
3
x
2
+ mx 12 seja
divisvel por x 3

Resoluo: Para que P(x) seja divisvel por x 3, deve-se
ter P(3) = 0. Ento

P(x) = x
3
x
2
+ mx 12
P(3) = (3)
3
(3)
2
+ m(3) 12
0 = 27 9 + 3m 12
6 = 3m
2 = m

Logo, para a diviso ser exata devemos ter m = 2

TEOREMA DAS DIVISES SUCESSIVAS
Se um polinmio P(x) divisvel por (x a) e por (x b),
ento P(x) divisvel por (x a).(x b).
Observe que o polinmio P(x) = x
4
+ 2x
3
6x
2
5x + 2
divisvel por (x + 1).(x 2), uma vez que ele divisvel
separadamente por (x + 1) e (x 2).

DISPOSITIVO DE BRIOT-RUFFINI
O dispositivo de Briot-Ruffini, tambm conhecido como
algoritmo de Briot-Ruffini, um modo prtico para dividir
um polinmio P(x) por um binmio da forma
ax + b. Vamos apresentar esse processo atravs de um
exemplo.

Determine o quociente e o resto da diviso da diviso de
P(x) = 2x
3
x
2
+ 4x 1 por (x 3)

Resoluo:

1 Passo
Dispem-se todos os coeficientes de P(x) de forma
ordenada e segundo os expoentes decrescentes de x na
chave.

2 1 4 1



2 Passo

Coloca-se esquerda a raiz do divisor.

3 2 1 4 1


3 Passo
Abaixa-se o primeiro coeficiente de P(x)

3 2 1 4 1

2




Please purchase PDF Split-Merge on www.verypdf.com to remove this watermark.
Matemtica D Incluso para a Vida

Pr-Vestibular da UFSC 10
4 Passo
Multiplica-se o coeficiente baixado pela raiz, somando o
resultado com o prximo coeficiente de P(x) e o resultado
abaixo desse ltimo.
+

3 2 1 4 1
x 2 5


5 Passo
Multiplica-se o esse ltimo resultado pela raiz e soma o
resultado com o prximo coeficiente de P(x) de forma
anloga ao ltimo passo, e assim sucessivamente.
+

3 2 1 4 1

x 2 5 19


+

3 2 1 4 1

x 2 5 19 56



Terminando assim o processo, temos:

raiz coeficientes de P(x)

2 5 19 56

coeficientes de Q(x) R(x)

Como gr(Q) = 2 [gr(P) gr(D)] temos que Q(x) = 2x
2
+ 5x
+ 19 e resto R(x) = 56

Exerccios de Sala #

1. (FUVEST) O quociente de 2x
4
5x
3
10x 1 por x
3 :
a) 2x
3
11x
2
+ 23x 68
b) 2x
3
11x
2
+ 33x + 109
c) 2x
3
11x
2
+ 33x 109
d) 2x
2
+ x 7
e) 2x
3
+ x
2
+ 3x 1

2. Qual o valor de "a" para que o polinmio x
5
+ 2x
4
+
3x
3
+ ax
2
4x + 12 seja divisvel por x
3
+ 2x
2
x + 3?

3. ( UFSM ) O resto da diviso de x
142
1 por x + 1 :
a) 0 b) 1 c) 2 d) 141 e) n.d.a.

Tarefa Mnima #

4. (UFSC) Determine o resto da diviso do polinmio 3x
3

+ 8x
2
+ 32 por x + 3.

5. (UECE) Se na diviso do polinmio 12x
4
+ 5x
3
+ 5x +
12 por 3x
2
+ 2x - 1 o quociente Q(x), ento o valor de
Q(3) :

6. (UFMG) O quociente da diviso de P(x) = 4x
4
- 4x
3
+ x
- 1 por Q(x) = 4x
3
+ 1 :
a) x 5 c) x + 5 e) 4x + 8
b) x - 1 d) 4x - 5

7. (UFSC) Qual o valor de "a" para que o polinmio x
5
+
2x
4
+ 3x
3
+ ax
2
- 4x + 12 seja divisvel por x
3
+ 2x
2
- x +
3?

8. (UFSC) Determine o valor de m, para que o resto da
diviso do polinmio P(x) = x
3
+ mx
2
- 2x + 1 por x + 3
seja 43.

Tarefa Complementar #

9. (UFSC) Se o polinmio 2x
3
- ax
2
+ bx + 2 divisvel
por 2x
2
+ 5x - 2, ento o valor de a - b :

10. (Mack-SP) Um polinmio desconhecido ao ser
dividido por x - 1 deixa resto 2 e ao ser dividido por x - 2
deixa resto 1. Ento, o resto da diviso desse polinmio
por (x - 1) (x - 2) :

a) x 3 c) x + 3 e) -x + 5
b) -x + 3 d) x - 5

11. (UFBA) O resto da diviso de P(x) = 3x
5
+ 2x
4
+ 3px
3

+ x - 1 por (x + 1) 4, se p igual a:

a) 5/3 b) -2 c) -3 d) -10 e) -7/3

12. (FGV-SP) O resto da diviso do polinmio 2x
5
- 15x
3

+ 12x
2
+ 7x - 6 por (x - 1)(x - 2)(x + 3) :

a) x
2
- 2x + 5 c) x - 4 e) 0
b) -6 d) 1

13. (PUC-MG) Os valores de a e b que tornam o
polinmio P(x) = x
3
+ 4x
2
+ ax + b divisvel por (x + 1)
2

so respectivamente:

a) 1 e 2 b) 3 e 2 c) 4 e 5 d) 5 e 2 e) n.d.a.

UNIDADE 8

EQUAES POLINOMIAIS

DEFINIO
Denomina-se Equao Polinomial toda sentena do tipo
P(x) = 0, ou

a
n
x
n
+ a
n - 1
x
n - 1
+ ..... + a
2
x
2
+ a
1
x + a
0
= 0

- onde a
n
, a
n - 1
, .........a
2
, a
1
, a
0
so nmeros complexos
- n um nmero natural
- x a varivel
- O expoente da equao o expoente do polinmio P(x)

Denomina-se raiz de uma equao polinomial todo nmero
o, tal que P(o) = 0



Please purchase PDF Split-Merge on www.verypdf.com to remove this watermark.
Incluso para a vida Matemtica D

Pr-Vestibular da UFSC
11
TEOREMA FUNDAMENTAL DA LGEBRA
Toda equao polinomial de grau n (n > 1) tem pelo
menos uma raiz complexa.
Esse teorema foi demonstrado por Gauss em
1799.

DECOMPOSIO DE UM POLINMIO EM
UM PRODUTO DE FATORES DO 1 GRAU
Como uma consequncia do Teorema Fundamental
pode-se afirmar que todo polinmio de grau n pode ser
escrito na forma:

P(x) = a
n
(x o
1
).(x o
2
)(x o
3
)....... .(x o
n
)

onde o
1
, o
2
, o
3
, ..... o
n
so razes de P(x).

MULTIPLICIDADE DE UMA RAIZ
Denomina-se multiplicidade de uma raiz ao nmero de
vezes que a mesma se repete no conjunto soluo.
Genericamente, pode-se dizer que o nmero o
raiz de multiplicidade n da equao polinomial P(x) = 0 se
e somente se, P(x) = (x o)
n
. Q(x), com Q(o) = 0.

TEOREMA DAS RAZES COMPLEXAS
Se um nmero complexo z = a + bi raiz de uma equao
polinomial de coeficientes reais, ento seu conjugado z =
a bi tambm raiz dessa equao.

Consequncias:
- Se a raiz (a + bi) de multiplicidade k, ento seu
conjugado (a bi) ter tambm multiplicidade k.
- Toda equao polinomial de grau mpar admite pelo
menos uma raiz real, pois o nmero de razes no reais
sempre par.

RELAES DE GIRARD
So relaes estabelecidas entre os coeficientes e razes de
uma equao polinomial.

Sejam x
1
e x
2
as razes da equao ax
2
+ bx + c = 0. Valem
as seguintes relaes:

x
1
x
2
b
a

x
1
x
2
c
a
+ =
=

.


Sejam x
1
, x
2
e x
3
as razes da equao
ax
3
+ bx
2
+ cx + d = 0. Valem as seguintes relaes:

x
1
x
2
x
3
b
a

x
1
x
2
x
3
d
a
x
1
x
2
x
1
x
3
x
2
x
3
c
a
+ + =
=
+ + =

. .
. . .



EQUAO DE GRAU n

Sendo o
1
, o
2
,........... o
n
as razes da equao
a
n
x
n
+ a
n - 1
x
n - 1
+ ..... + a
1
x + a
0
= 0, valem as seguintes
relaes:

( )
a a a
n
a
n
a
n
a a a a a a
n
a a a
n
a
n
a
n
a
n
a a a a
n
a
n
a
n
a
n
a
n
a a a a
n
n
a
a
n
1 2
1
1 2 1 3 1 2 3 1
2
1 2 3 2 1
3
1 2 3
1
0
+ + + =

+ + + + + +

+ +

=

..........
..... ...... .
..... . .
. . .......


Exerccios de Sala #

1. O polinmio P(x) = x
3
+ 4x
2
+ 3x pode ser escrito
como:
a) P(x) = x(x 1)(x 3) d) P(x) = x(x 2)(x +4)
b) P(x) = x(x + 1)(x + 2) e) (x) = x(x 1)(x + 5)
c) P(x) = x(x + 1)(x + 3)

2. Resolver a equao x
3
12x
2
+ 41x - 42 = 0, sabendo
que x = 2 uma das razes.

3. Determine a menor raiz da equao x
3
15x
2
+ 66x
80 = 0, sabendo que suas razes esto em P.A.

Tarefa Mnima #

4. (ACAFE) A equao polinomial cujas razes so 2, 1
e 1 :
a) x
3
+ 4x + x 2 = 0 d) x
3
+ 2x
2
x 2 = 0
b) x
3
x 2 = 0 e) x
3
+ 2x + 1 = 0
c) x
3
+ 2x
2
3x 2 = 0

5. (FGV-SP) A equao 2x
3
5x
2
x + 6 admite uma raiz
igual a 2. Ento, as outras duas razes so:
a) 3/2 e 1 c) 3 e 1 e) 3/2 e 2
b) 2 e 1 d) 3/2 e 1

6. (UFSC) Sabendo-se que uma das trs razes da equao
2x
3
- 17x
2
+ 32x - 12 = 0 igual a 1/2 determine a soma
das outras duas razes.

7. (UDESC) As razes do polinmio x
3
6x
2
x + 30:

a) somadas do 6 e multiplicadas do 30
b) somadas do -6 e multiplicadas do 30
c) somadas do 6 e multiplicadas do -30
d) somadas do -6 e multiplicadas do 30
e) so 5, -2 e 3

Tarefa Complementar #

8. (Med ABC-SP) As razes da equao
x
3
- 9x
2
+ 23x -15 = 0 esto em progresso aritmtica. Suas
razes so:
a) 1, 2, 3 c) 1, 3, 5 e) 3, 6, 9
b) 2, 3, 4 d) 2, 4, 6

9. (Mackenzie) Uma raiz da equao x
3
4x
2
+ x + 6 = 0
igual a soma das outras duas. As razes so:
a) 2, 2 e 1 d) 1, 1 e 2
b) 3, 2 e 1 e) 1, 2 e 3
c) 2, 1 e 3
Please purchase PDF Split-Merge on www.verypdf.com to remove this watermark.
Matemtica D Incluso para a Vida

Pr-Vestibular da UFSC 12

10. (MACK-SP) O determinante da matriz
a a c
b c

(
(
(
0
1 0 1
,
onde a, b, e c so razes da equao x
3
5x
2
+ 4 = 0, :

11. (SANTA CASA) Sabe-se que a equao: 4x
3
12x
2

x + k = 0, onde k e 9, admite duas razes opostas. O
produto das razes dessa equao :
a) 12 b) 3/4 c) 1/4 d) 3/4 e) 12

12. (ITA-SP) Considere a equao x
3
+ px
2
+ qx + r = 0
de coeficientes reais, cujas as razes esto em P.G. Qual
das relaes verdadeira?
a) p
2
= r.q d) p
3
= r.q
3

b) 2p + r = q e) q
3
= r.p
3

c) 3p
2
= r
2
. q

13. (UFSC) Assinale no carto-resposta a soma dos
nmeros associados (s) proposio(es) correta(s).
01. A equao polinomial x
3
2x
2
4x + 1 = 0 possui as
razes a, b e c. Logo, a soma a
2
+ b
2
+ c
2
igual a 12.
02. O resto da diviso do polinmio x
6
x
4
+ x
2
por x + 2
52.
04. Dado o polinmio p(x) = x
4
+ 8x
3
+ 23x
2
+ 28x + 12
correto afirmar que 2 raiz de multiplicidade 3 para
p(x).
08. Para que o polinmio p(x) = (a + b) x
2
+ (a b + c) x
+ (b + 2c 6) seja identicamente nulo, o valor de c
4.

UNIDADE 9

MATRIZES

DEFINIO
Uma matriz do tipo m x n (l-se: m por n), m, n > 1, uma
disposio tabular formada por m.n elementos dispostos
em m linhas e n colunas.
As matrizes so representadas atravs de parnteses ( ),
colchetes [ ] ou atravs de barras duplas || ||
Exemplos.:

A =
2 0 3
6 9 5
|
\

|
.
|
A
2 x 3
(l-se: A dois por trs)

A =
3 2 8 7
6 1 0 3

(
A
2 x 4
(l-se: A dois por quatro)

A =
6 0
6 1
1 2
A
3 x 2
(l-se: A trs por dois)

NOTAES

Notao Explcita
Uma matriz genericamente representada por letras
maisculas e seus elementos por letras minsculas.
Sendo assim, uma matriz A
m x n
algebricamente pode ser
representada assim:
A =
a a a a
a a a a
a a a a
a a a a
n
n
n
m m m mn
11 12 13 1
21 22 23 2
31 32 33 3
1 2 3

(
(
(
(
(
(
com m e n e N
*


Notao Condensada
Podemos tambm, abreviar essa representao da seguinte
forma:
A = [aij]
m x n


Os elementos da matriz A so indicados por aij de forma
que:
i e {1, 2, 3,......m} (indicador da linha)
j e {1, 2, 3, .....n} (indicador da coluna)

CLASSIFICAO DE MATRIZES
Seja a matriz A = (aij)
mxn
, lembrando que m e n so
respectivamente a quantidade de linhas e colunas da matriz
A, temos:

a) MATRIZ LINHA se m = 1

Exemplo: A
1x3
( ) 2 1 3

b) MATRIZ COLUNA se n = 1

Exemplo: A
4x1
=
|
|
|
|
|
.
|

\
|

0
5
2
1

c) RETANGULAR se m = n

Exemplo: A
2 x 3
=
|
|
.
|

\
|
0 4 9
1 3 2


d) QUADRADA se m = n

Exemplo: A
2x2

(

8 5
6 3


Definio: Diz-se que uma matriz quadrada se a
quantidade de linhas for igual a quantidade de colunas.
Pode-se dizer ento que ela n x n ou simplesmente de
ordem n.

Possui duas diagonais:

- diagonal principal (quando i = j para todo aij)
- diagonal secundria (quando i + j = n + 1) , onde n a
ordem da matriz.

TIPOLOGIA

Matriz Transposta
Seja A uma matriz de ordem m x n, denomina-se
transposta de A a matriz de ordem n x m obtida quando
trocamos de forma ordenada as linhas pelas colunas.
Representa-se por: A
t
ou A'
Exemplo A
2 x 3
=
|
|
.
|

\
|
0 4 9
1 3 2
A
t
3 x 2
=
2 9
3 4
1 0
|
\

|
.
|
|
|


Please purchase PDF Split-Merge on www.verypdf.com to remove this watermark.
Incluso para a vida Matemtica D

Pr-Vestibular da UFSC
13
OBSERVAO: Seja uma matriz A de ordem n.

- Se A = A
t
, ento A dita SIMTRICA

Exemplo: A =
|
|
|
.
|

\
|
0 8 5
8 1 3
5 3 2


- Se A = A
t
, ento A dita ANTISIMTRICA
(A indica matriz oposta de A que se obtm
trocando o sinal dos seus elementos)
Exemplo: A =
|
|
|
.
|

\
|

0 4 3
4 0 1
3 1 0


Matriz Identidade
Uma matriz A de ordem n dita identidade ou unidade se
os elementos da diagonal principal forem iguais a 1 e os
demais elementos iguais a zero.
Exemplos: I
2
=
1 0
0 1

(
I
3
=
1 0 0
0 1 0
0 0 1
|
\

|
.
|
|
|

Pode se indicar a matriz identidade por:
I
n
= [aij] , aij =
1, para i = i
0, para i j =



Importante: A matriz identidade neutra na
multiplicao de matrizes.

Matriz Nula

Uma matriz dita nula quando todos seus elementos forem
iguais a zero. A matriz Nula neutra na soma de matrizes.

Matriz Diagonal
toda matriz de ordem n tal que aij = 0 para i = j.
Exemplo: A =
1 0 0
0 4 0
0 0 3
|
\

|
.
|
|


Matriz Triangular
toda matriz quadrada onde a
ij
= 0 para i > j ou/e para i <
j.
Exemplos:
|
|
|
.
|

\
|
|
|
|
.
|

\
|

8 1 9
0 2 1
0 0 4

1 0 0
7 4 0
5 1 3



IGUALDADE DE MATRIZES
Duas matrizes A
mxn
e B
mxn
so iguais se os elementos
correspondentes (elementos de mesmo ndice) forem
iguais.

ADIO E SUBTRAO DE MATRIZES
efetuada somando ou subtraindo os elementos
correspondentes das matrizes. (vlido para matrizes de
mesma ordem).



Propriedades:

1) A + B = B + A (propriedade comutativa)
2) A + (B + C) = (A + B) + C (propriedade
associativa)
3) A + O = A (elemento neutro)
4) (A + B)
t
= A
t
+ B
t

PRODUTO DE UM NMERO POR MATRIZ
Dado um nmero real K e uma matriz A
m x n
, denomina-se
produto de K por A e se indica por k.A, matriz que se
obtm multiplicando-se todo elemento de A por k.

Propriedades:

Sendo x e y dois nmeros reais e A e B duas matrizes de
mesma ordem, valem as seguintes propriedades:
1) x . (yA) = (xy) . A
2) x . (A + B) = xA + xB
3) (x + y) . A = xA + yA

Exerccios de Sala #

1. A uma matriz 3 por 2, definida pela lei
aij =

=
= +
j i se
j i se j 2i
, 3
,
Ento, A se escreve:

2. (UFSC) Dadas as matrizes:
A =
2 1 3 1
0 4
x y
x z
+
+
|
\

|
.
|
e B =
x 0
12 4
1 6
|
\

|
.
|
|
|

Se A = B
t
, o valor de x.y.z :

3. O valor de x.y de modo que a matriz A seja simtrica,
:
A =
|
|
|
.
|

\
|

6 2 5
2 0 1
1 2 5 2
x
y


a) 6 b) 12 c) 15 d) 14 e) 0

Tarefa Mnima #

4. Escreva, na forma explcita, cada matriz abaixo:

a) A = (a
ij
)
2x2
, com a
ij
= i + j
b) A = (a
ij
)
3x2
, com a
ij
= 3i j
2

c) A = (a
ij
)
3x2
, com a
ij
=
1 se i j
i
2
se i j
,
,
=
=


d) A = (a
ij
)
2x3
, com a
ij
=
2, se i = j
2 + j, se i j =



5. (UFSC) Dada a matriz A = [aij]
2 x 3
definida por aij =

> +
=
< +
j i se j, i
j i se 7,
j i se j, 3i
2
o valor da expresso 2a
23
+ 3a
22
- a
21
:
6. (UFOP-MG) Observe a matriz
|
|
.
|

\
|
y
x
0 0
4 0
3 2 1
.
Determine x e y de tal forma que seu trao valha 9 e x seja
o triplo de y.
Please purchase PDF Split-Merge on www.verypdf.com to remove this watermark.
Matemtica D Incluso para a Vida

Pr-Vestibular da UFSC 14

7. Considere as matrizes A =
|
|
.
|

\
|
+ 7
2
log 3
2 1
5 2
x
y

e B =
|
.
|

\
|
7 16 5
8 1 2
. Determine o valor de x + y de
modo que A = B
t

8. Considere as matrizes A =
|
.
|

\
|
0 3
1 2
e B =
|
.
|

\
|
2 1
3 0

a) Obter a matriz X tal que A + X = B
b) Obter as matrizes X e Y tal que:

=
= +
B Y X
A Y X 3


Tarefa Complementar #

9. Calcule 5x + 2y, de modo que se tenha:

|
|
.
|

\
|

=
|
|
.
|

\
|

|
|
.
|

\
|
1 5
3 1
1 2
2 6
0 3
1 2 5
y y
x


10. (FCMSCSP) Se A uma matriz quadrada, define-se o
TRAO de A como a soma dos elementos da diagonal
principal de A. Nestas condies, o trao da matriz A =
(aij)
3 x 3
, onde aij = 2i - 3j igual a:
a) 6 b) 4 c) -2 d) -4 e) -6

11. Determine a soma dos elementos da diagonal principal
da matriz A = ( aij )
3 X 3
, onde aij = i + j se i > j ou aij = i
j se i < j.

12. Uma matriz se diz anti-simtrica se A
t
= A. Nessas
condies, se a matriz A anti-simtrica, ento, x + y + z
igual a:
A =
(
(
(

0 3 1
3 0 2
z y x

a) 3 b) 1 c) 0 d) 1 e) 3

13. (LONDRINA-PR) Uma matriz quadrada A diz-se
simtrica se A = A
t
. Assim, se a matriz
A =
|
|
|
.
|

\
|

2 3 4
1 0
2 1 2
z x
y
simtrica, ento x + y + z igual a:

a) 2 b) 1 c) 1 d) 3 e) 5
14. (U.Catlica de Salvador - BA) Uma matriz quadrada
A, de ordem n, se diz anti-simtrica se A = -A
t
, onde A
t
a
matriz transposta de A. Nessas condies, qual das
matrizes seguintes anti-simtrica?


0 3 - 2
3 0 1 -
2 - 1 0
b
4 1 3
1 0 2 -
3 2 - 1
a
(

) )

(

0 3 1
3 0 2
1 2 0
e

3 2 3
2 2 0
3 0 1
d
1 0 1 -
0 1 1 -
1 1 - 1
c

)
) )


15. Se a matriz quadrada A tal que A
t
= A, ela
chamada matriz anti-simtrica. Sabe-se que M anti-
simtrica e:
M =
4
2
2 8
12 13
23
+
+

|
\

|
.
|
|
|
a a a
a b a
b c c
.
Os termos a
12
, a
13
e a
23
valem respectivamente:

a) 4, 2 e 4 d) 2, 4 e 2
b) 4, 2 e 4 e) n.d.a.
c) 4, 2 e 4

16. Sendo A =

(
1 7
2 4
e B =
3 1
4 0

(
, ento a matriz X,
tal que
X A X B
=
+
2
2
3
, igual a:
17. Dadas as matrizes: A =
3 1
2 4

|
\

|
.
|
|
\

|
.
| e B =
2 2
0 4
, o
produto dos elementos da segunda linha de
1
4
B
1
2
A
:
a) 1 b) 1 c) 0 d) 2 e) 2

18. Dadas as matrizes
A
x y
z w
B =
x 6
- 1 2w
C =
4 x y
z + w 3
=
+
|
\

|
.
|
|
\

|
.
|
|
\

|
.
|
e sendo 3A = B +
C, ento:
a) x + y + z + w = 11 d) x + y z w = 1
b) x + y + z + w = 10 e) x + y + z + w > 11
c) x + y z w = 0

UNIDADE 10

MULTIPLICAO DE MATRIZES

Considere as matrizes A = [a
ij
]
m x n
e a matriz B = [b
jk
]
n x p
.
O produto de A por B a matriz C = [c
ik
]
m x p
, de tal forma
que os elementos cik so obtidos assim:

c
ik
= a
i1
. b
1k
+ a
i2
. b
2k
+ ai
3
. b
3k
+ .... + a
in
. b
nk

ou seja:
_
=
n
j
jk ij
b a
1
para todo i e {1, 2, ........, m} e todo k e
{1, 2,...,p}.

Exemplo: Considere as matrizes
A =
3 0
2 1
|
\

|
.
| e B =

|
\

|
.
|
1 3
9 2
. Determine A.B
Resoluo: O produto AxB uma matriz obtida da
seguinte forma:
A.B =
3 1 0 9 3 3 0 2
2 1 19 2 3 12
.( ) . .( ) .
.( ) . .( ) .
+ +
+ +
|
\

|
.
|
Please purchase PDF Split-Merge on www.verypdf.com to remove this watermark.
Incluso para a vida Matemtica D

Pr-Vestibular da UFSC
15
A.B =

|
\

|
.
|
3 9
7 4

PROPRIEDADES

1) A.(B.C) = (A.B).C 2) A.(B + C) = A.B + A.C
3) (B + C).A = B.A + C.A 4) A.I = I.A = A

Observaes:

1) Na multiplicao de matrizes geralmente
A.B = B.A. Se A.B = B.A dizemos que A e B se
comutam.

2) Na multiplicao de matrizes no vale a lei do
anulamento, ou seja, podemos ter A.B = 0 mesmo
com A = 0 B = 0.

DETERMINANTES

DEFINIO

Dada uma matriz quadrada de ordem n, podemos associar
ela, atravs de certas operaes, um nmero real
chamado determinante da matriz.



Podemos simbolizar o determinante de uma matriz por
duas barras verticais. Assim, se
a a
a a
11 12
21 22

(
a matriz A,
indicamos o determinante de A por det A =
a a
a a
11 12
21 22


CLCULO

- 1 ORDEM
Seja a matriz A = [a
11
] , denomina-se o determinante de
A o prprio elemento a
11
e se indica por:

det A = |a
11
| = a
11


- 2 ORDEM



- 3 ORDEM



Exerccios de Sala #

1. Dadas as matrizes A =
|
|
.
|

\
|
|
|
.
|

\
|
0
3
3 4
1 2
1 -
5
= B e
.
Determine:
a) A.B c) A
t
.B
t
e) A.I
2

b) B.A d) B
t
.A
t
f) a matriz X, tal que A.X = B

2. (UFSC) Sejam A = (aij )
4 x 3
e B = (bij)
3 x 4
duas
matrizes definidas por aij = i + j e bij = 2i + j,
respectivamente. Se A.B = C, ento o elemento C
32
da
matriz C, :

3. Calcule os determinantes:
a)
5 2
4 3
b)
4 2
1 3



4. Calcule o determinante:
1 6 3
3 4 1
2 0 2


Tarefa Mnima #

5. (UEL-PR) Sobre as sentenas:
I - O produto de matrizes A
3x2
. B
2x1
uma matriz 3x1.
II - O produto de matrizes A
5x4
. B
5x2
uma matriz 4x2.
III - O produto de matrizes A
2x3
. B
3x2
uma matriz
quadrada 2 x 2.

verdade que

a) somente I falsa
b) somente II falsa
c) somente III falsa
d) somente I e III so falsas.
e) I, II e III so falsas

6. Se
3 2
1 4
|
\

|
.
|
a
b
1
2
|
\

|
.
|
=
5 7
5 9
|
\

|
.
|
, ento a + b igual a:

7. Dadas as matrizes A =
1 1
0 0
|
\

|
.
|
e B =
0 1
0 1
|
\

|
.
|
, para
A.B temos a matriz:

8. (UCMG) O valor de x, para que o produto das matrizes:
A =

(
2
3 1
x
e B =
1 1
0 1

(
seja uma matriz simtrica, :

9. (UFSC) Dada a equao matricial:

4 2
1 3 0
4 2
3
1
4
2
3
x
y
z x
y

|
\

|
.
|
|
|

|
\

|
.
|
|
|
=
|
\

|
.
|
|
|
.
O valor da expresso 5x +
4y + z :

10. Calcule os seguintes determinantes:

a)
1 6
3 4

b)
1 3
2 5


c)
4 3 2
3 1 4
5 2 3


11. (MACK-SP) Sendo A = ( aij ) uma matriz quadrada de
ordem 2 e a
ij
= j - i
2
, o determinante da matriz A :


12. (UFSC) Obtenha o valor do determinante da matriz
A = (aij)
2 x 2
, onde aij =

= +
=
j i se j, i
j i se 0,



Please purchase PDF Split-Merge on www.verypdf.com to remove this watermark.
Matemtica D Incluso para a Vida

Pr-Vestibular da UFSC 16
13. O valor de x na equao
15
1 0 2
1
1 3 2
= x x
:

Tarefa Complementar #
14. (CESCEM) O produto M.N da matriz M =
1
1
1
|
\

|
.
|
|
|
pela
matriz N =
( ) 1 1 1 :

a) no se define
b) a matriz identidade de ordem 3
c) uma matriz de uma linha e uma coluna
d) uma matriz quadrada de ordem 3
e) no uma matriz quadrada

15. (FEI-SP) As matrizes abaixo se comutam.
a a
a 2

(
e
0 3
3 3

(
O valor de a :


16. (UFSC) Determine o produto dos valores de x e y que
satisfaam a equao matricial
4 3
5 4
1
2
4 2
7 3

|
\

|
.
|
|
\

|
.
| =
|
\

|
.
| .
x
y

17. (UFSC) Dadas as matrizes: A =
1 0 2
0 1 3
4 1 2

|
\

|
.
|
|
|
;
B =
2 1 1
0 3 0
4 2 1
|
\

|
.
|
|
|
; C =
1 0 0
0 1 0
0 0 1
|
\

|
.
|
|
|
e seja P = (2A - C).B.
Determine a soma dos elementos da diagonal principal da
matriz P.

18. (UFSC) Considere as matrizes A =
1 0
2 1
1 2
|
\

|
.
|
|
|

B =
2 0 1
1 1 3
|
\

|
.
|
Sejam M = ( A + B
t
).(A
t
B ), onde A
t

e B
t
so matrizes transpostas de A e B, respectivamente. O
produto dos elementos m
ij
com i = j da matriz M :

19. Se A =
1 2
4 3
|
\

|
.
| , ento A
2
+ 2A 11 I, onde I a
matriz identidade de ordem 2, igual a:

20. (UFSC) Determine o valor de x para que o
determinante da matriz C = A x B
t
seja igual a 602, onde:
A =
1 2 3
4 1 2

(
, B =
x

(
1 8 5
2 7 4
e B
t
a matriz
transposta de B.
21. (UFSC) Em R,a soluo da equao
2 3
2 4
1 3
x
x
x

= 175 :
22. (MACK) O conjunto soluo de
1
1 1
1 1
1
1 1
1
x
x
x
=
:
a) { x e R| x = 1} c) { 1 } e) { 0 }
b) { 0,1 } d) { -1}

23. (MACK-SP) Sejam as matrizes A =
1 2
3 4
e B =
3 4
1 2
|
\

|
.
|
|
\

|
.
|
,
e seja X uma matriz tal que X.A = B. Ento, det X vale:
a) -2 b) -1 c) 0 d) 1 e) 2

UNIDADE 11

PROPRIEDADES DE DETERMINANTES

1 PROPRIEDADE

Casos onde o determinante nulo
1 Se uma matriz possui uma fila de elementos iguais a
zero.
Exemplo:
0 3 9
0 8 3
0 4 1
0 =


2 Se uma matriz possui duas filas iguais.
Exemplo:
2 8 2
3 5 3
1 6 1
0

=


3 Se uma matriz possui duas filas proporcionais.
Exemplo:
2 3 5
4 6 10
7 0 3
0

=


4 Se uma fila de uma matriz for uma combinao linear
de duas outras.
Exemplo:
3 5 1
0 4 2
3 9 3
0 =


2 PROPRIEDADE
Se multiplicarmos uma fila de uma matriz por um nmero
k, o determinante da nova matriz fica multiplicado por k.

Exemplo:
2 4
1 3
2
2 4
1 3
2 10 = = =
5 5
5
. .
.

CONSEQUNCIAS
- No clculo dos determinantes, possvel colocar o
fator comum em evidncia.
-216 = 3.(-72)
1 4 3
0 5 1
4 2 6
3
1 4 3
0 5 1
4 3 2 3 6 3
1 4 3
0 5 1
12 6 18
=

.
. . .

(72)

- Se multiplicarmos uma matriz quadrada de ordem n
por um nmero k o determinante fica multiplicado pelo
nmero k
n
.
det(k.A) = k
n
.detA
Please purchase PDF Split-Merge on www.verypdf.com to remove this watermark.
Incluso para a vida Matemtica D

Pr-Vestibular da UFSC
17

3 PROPRIEDADE
Se trocarmos duas filas paralelas de uma matriz o
determinante muda de sinal.

4 PROPRIEDADE
O determinante de uma matriz triangular o produto dos
elementos da diagonal principal.
Exemplo:
3 9 8
0 4 5
0 0 1
12 =

5 PROPRIEDADE
(TEOREMA DE BINET)
Se A e B so duas matrizes de ordem n o determinante do
produto de A por B o produto dos determinantes da
matriz A pelo determinante da matriz B, ou seja:

det(A.B) = det(A).det(B)

6 PROPRIEDADE
O determinante de uma matriz igual ao determinante de
sua transposta.

7 PROPRIEDADE
(TEOREMA DE JACOBI)
Se somarmos a uma fila de A uma outra fila previamente
multiplicada por um nmero real, obtemos uma matriz A',
tal que det A' = det A

Exemplo: A =
(
(
(


1 2 2
1 5 1
2 1 4
det A = 15
Multiplicando a terceira linha por 2 e adicionando

primeira, obtemos A': A' =
0 3 0
1 3 2
2 2 1

(
(
(
det A = 15

INVERSO DE MATRIZES

Sejam A e B duas matrizes quadradas.
Se A.B = B.A = I, dizemos que B a matriz inversa de A.
e indicamos por A
-1
.

Logo: A . A
-1
= A . A
-1
= I
n


PROPRIEDADES DA INVERSA:

- (A
-1
)
-1
= A
- (A.B)
-1
= B
-1
. A
-1

- det A
-1
=
1
det A


OBSERVAES:

- Uma matriz s possui inversa se o seu determinante for
diferente de zero, sendo assim, chamada de inversvel.
- Uma matriz que no admite inversa chamada de
singular.
- Se a matriz A inversvel, ento, ela quadrada.
- Se a matriz A inversvel, ento, a sua inversa nica.

OBSERVAO:

O processo de se obter a inversa de uma matriz muitas
vezes trabalhoso, pois recai na resoluo de n sistemas
de n equaes e n incgnitas.
Vamos agora apresentar um processo que simplifica esse
clculo.

Teorema
Se A uma matriz quadrada de ordem n e det A = 0, ento
a inversa de A :

A
1
= .
det
1
A

A

Onde A representa a matriz adjunta.

Matriz Adjunta: a matriz transposta da matriz dos
cofatores de A.


Consequncia
Para calcular um elemento b
ij
da matriz inversa de A,
pode-se aplicar:

b
ij
= .
det
1
A

C
ji

onde C
ji
o cofator do elemento aij

Exerccios de Sala #

1. Sabe-se que
2 =
i f c
h e b
g d a
. Determine o valor de
i f c
h e b
g d a
4 3 2
4 3 2
4 3 2



2. Uma matriz A quadrada de ordem 4 e seu
determinante igual a 3. Calcule o valor do determinante
da matriz 2A.

3. Determine a inversa das seguintes matrizes:
a)
1 5
2 0
|
\

|
.
| b)
3 1
5 2
|
\

|
.
|

4. Determine o valor de x de modo que a matriz
|
|
.
|

\
|
9
3 2
x

seja singular

Tarefa Mnima #

5. Sabendo que
2 =
i f c
h e b
g d a
, calcule
i f c
h e b
g d a

3 2
3 2
3 2


Please purchase PDF Split-Merge on www.verypdf.com to remove this watermark.
Matemtica D Incluso para a Vida

Pr-Vestibular da UFSC 18
6. (UFRN) O determinante
1 72 81
0 2 200
0 0 3
igual a:

7. (UFRGS) Considere as seguintes afirmaes.

I - O determinante de uma matriz no se altera, quando so
trocadas, ordenadamente, as linhas pelas colunas.
II - O determinante de uma matriz com linhas
proporcionais nulo.
III - Multiplicando-se uma linha de uma matriz por um
nmero real p,no nulo,o determinante da nova matriz
fica dividido por p.

Quais so as verdadeiras?
a) I
b) II
c) I e II
d) II e III
e) todas so verdadeiras

8. (UDESC) A partir da matriz A = |aij|
2 x 2
onde
aij =
>
+ s

1. . . . . . . . . .
. . . . . . . . . .
se i j
i j se i j
calcular o determinante
do produto da matriz A pela sua transposta, ou seja: det(
A
t
.A ), onde A
t
a matriz transposta de A.

9. (Unisinos-RS) O valor de um determinante 48.
Dividimos a 2 linha por 8 e multiplicamos a 3 coluna por
6, ento o novo determinante valer:
10. (UFRGS) A inversa da matriz A =
|
.
|

\
|
2 5
1 3
:
|
.
|

\
|
|
.
|

\
|
|
.
|

\
|
|
.
|

\
|
|
.
|

\
|

2 5
1 3
e)
3 5
0 2
d)
3 1
5 2
c)
2 5
1 3
b)
3 5
1 2
a)
11. O maior elemento da inversa da matriz A =
|
.
|

\
|
5 1
4 2
:
a) 2 c) 1/5 e) 1/3
b) 5/6 d) 1/6

12. (UFVIOSA) Sejam as matrizes A =
|
.
|

\
|
6 2
2 1
e M =
|
.
|

\
|

y
x
1
1
, onde x e y so nmeros reais e M a matriz
inversa de A. Ento o produto x.y :

a) 3/2 b) 2/3 c) 1/2 d) 3/4 e) 1/4

13. (UCSal-BA) A matriz
1
1
x
x
|
\

|
.
| , na qual x um
nmero real, inversvel se, e somente se:

a) x = 0 b) x = 1 c) x = -1 d) x = 1

14. Considere a matriz A =
(

+ 2 1
3
x
x
. Sabendo que det
A
- 1
= 0,25, ento x :

a) 0 b) 2 c) 2 d) 4 e) 1

Tarefa Complementar #

15. (UECE) Sabe-se que M uma matriz quadrada de
ordem 3 e que det(M) = 2. Ento det (3M) igual a:

a) 2 b) 6 c) 18 d) 54 e) 27

16. (UFSM) Sejam as matrizes A, de ordem 3 e B =
2 1 4
1 0 2
0 1 6

|
\

|
.
|
|
|
. Se o det A = 6 e C = A.B, o det C vale:

a) 24 b) 12 c) -6 d) -12 e) -24

17. (SANTA CASA) Dadas as matrizes A e B tais que:

1 5 1 3 0 0 0
0 2 2 4 3 4 0 0
0 0 3 1 1 2 1 0
0 0 0 4 2 1 3 2
A
( (
( (

( (
=
( (

( (
( (

-1
e B=


O valor do determinante de A.B :

a) 192
b) 32
c) -16
d) 0
e) n.d.a.
18. (F.M.Santos-SP) O determinante
1 0 0 0 0
2 2 0 0 0
3 2 1 0 0
4 2 3 2 0
5 1 2 3 3

:

a) -12 b) 10 c) 9 d) 0 e) n.d.a.

19. (MACK-SP) Seja A uma matriz quadrada de ordem 2
e I =
|
|
.
|

\
|
1 0
0 1
. Chamam-se auto valores de A as razes
da equao det (A xI) = 0. Obtenha os autovalores de
A =
|
|
.
|

\
|
3 2
4 1

20. (FGV-SP) Considere as matrizes A =
(

p c
n b
m a
4
4
4

e B =
(

3
3
3
c p
b n
a m
. Se o determinante da matriz A
igual a 2, ento o determinante da matriz B igual a:

a) 3/2 b) 2/3 c) 3 d) 3/2 e) 2/3
Please purchase PDF Split-Merge on www.verypdf.com to remove this watermark.
Incluso para a vida Matemtica D

Pr-Vestibular da UFSC
19
21. (UEPG-PR) Dada a matriz A = (a
ij
)
3x3
, onde aij =

<
>
j i se 0,
j i se 4,
. Ento correto afirmar:

01. det (A) = 64
02. (A).(A
t
) uma matriz quadrada de ordem 6
04. det(2A) = 8 det(A)
08. det(A) = det(A
t
)
16. A
2
=
(
(
(

16 16 16
0 16 16
0 0 16

22. Os valores de k para que a matriz A =
|
|
.
|

\
|
3 1
3 1
1 0 1
k
k
no
admita inversa so:

a) 0 e 3 c) 1 e 2 e) 3 e 1
b) 1 e 1 d) 1 e 3

23. (UFPB) Se a matriz
2 5
5
x x
x
+

|
\

|
.
|
no invertvel,
ento, o valor de x em mdulo :

24. (UDESC) Seja a matriz A = ( aij )
3 x 3
definida por
aij =
( ) /
, /
=
=

+
1
0
i j
para i j
para i j
o determinante de A
-1
:

UNIDADES 12

SISTEMAS LINEARES

DEFINIO
Denomina-se Sistema Linear todo conjunto de m equaes
lineares com n incgnitas.

a x a x a x b
a x a x a x b
a x a x a x b
n n
n n
m m mn n n
11 1 12 2 1 1
21 1 22 2 2 2
1 1 2 2
+ +. + =
+ +. + =
+ +. + =

...........
..........
.........



Se b
1
, b
2
, ......, b
n
= 0 dizemos que o sistema homogneo.

Soluo de um Sistema Linear
Denomina-se soluo de um sistema a sequncia de
nmeros reais (o
1
, o
2
,..........., o
n
) que satisfaz
simultaneamente todas as equaes do sistema.

Sistemas Equivalentes
Dois Sistemas so ditos equivalentes se e somente se:
- So Possveis e admitem as mesmas solues, ou
- So Impossveis.

Classificao de um Sistema Linear
Um Sistema Linear pode ser classificado de acordo com o
nmero de solues que ele apresenta. Sendo assim ele
pode ser:
DETERMINADO
(1 soluo)
- POSSVEL
INDETERMINADO
(infinitas solues)
- IMPOSSVEL No Admite Soluo

REGRA DE CRAMER
A Regra de Cramer consiste num mtodo para resolvermos
sistemas Lineares de n equaes e n incgnitas.

Seja o sistema

a x a x a x b
a x a x a x b
a x a x a x b
n n
n n
n n nn n n
11 1 12 2 1 1
21 1 22 2 2 2
1 1 2 2
+ +. + =
+ +. + =
+ +. + =

...........
..........
..........



Para obtermos a soluo para esse sistema vamos fazer
alguns clculos. Acompanhe:

- det S
Determinante associado matriz formada pelos
coeficientes das incgnitas.

det S =
a a a
a a a
a a a
n
n
n n nn
11 12 1
21 22 2
1 2



- det X
i

Determinante associado matriz obtida a partir de S,
trocando a coluna dos coeficientes de X
i
, pela coluna dos
termos independentes do sistema.

det X
1
=
b a a
b a a
b a a
n
n
n n nn
1 12 1
2 22 2
2

det X
2
=
a b a
a b a
a b a
n
n
n n nn
11 2 1
21 2 2
1



det X
n
=
a a b
a a b
a a b
n n n
11 12 1
21 22 2
1 2



A soluo do Sistema dada por:

x
1
= = =
det X
det S
x
det X
det S
x
det X
det S
1
2
2
n
n


Veja que s possvel aplicar a Regra de Cramer em
sistemas n x n em que det S = 0. Esses sistemas so
denominados normais.

3. Discusso com base na regra de Cramer (2x2)
1) Quando det S = 0, o sistema possvel e determinado.
2) Quando det S = det X
1
= det X
2
= ...= 0, o sistema
possvel e indeterminado
3) Quando det S = 0 e pelo menos um dos demais
determinantes for diferente de zero, os sistema
impossvel.

O sistema homogneo sempre possvel.

Please purchase PDF Split-Merge on www.verypdf.com to remove this watermark.
Matemtica D Incluso para a Vida

Pr-Vestibular da UFSC 20
Exerccios de Sala #

1. Usando a regra de Cramer, resolva os seguintes
sistemas:

a)

= +
=
1 5 2
11 3 4
y x
y x


b)

=
=
6 2 2
3
y x
y x


c)

= +
=
2 3 3
1
y x
y x

2. Dado o sistema de equaes lineares
x y z
x y z
x y z
+ + =
+ =
=

|
o 1
1

com o, | e R, ento o sistema determinado se:

a) se o = -1 d) se o = -1 e | = 1
b) se o = -1 e | = 1 e) se o = -1 e
| = -1
c) se o = 1

3. (FGV-SP) O sistema linear

=
= + +
= +
0
0
0 2
z y x
z y x
z y x o
admite
soluo trivial, se:

a) o = - 2 c) o = 2 e) o e9
b) o = - 2 d) o = 2

Tarefa Mnima#

4. (USF-SP) Resolvendo o sistema
x y z
x y z
x y z
+ + =
+ + =
+ =

9
2 11
1
, obtm-
se y igual a:

5. (UFRGS) Dado o sistema de equaes lineares sobre
R
2 4
3 2 4
4 0
x y z
x y z
x y z
+ =
+ =
+ + =

os valores de x, y e z que constituem


sua soluo:
a) formam uma progresso geomtrica
b) formam uma progresso aritmtica
c) so iguais entre si
d) no existem
e) tm uma soma nula

6. (FGV-SP) O sistema de equaes
2 5 10
2 3
x y
x y
+ =
=


equivalente a:
2 5 10 10
) . ) .
1 2 3 3
10 10
) . )
3 3
x x
a b
y y
x x
c d
y y
| | | | | | | | | | | |
= =
| | | | | |

\ . \ . \ . \ . \ . \ .
| | | | | | | || | | |
= =
| | | | | |

\ . \ . \ . \ .\ . \ .
-2 -5

1 2
2 -1 -2 1

5 -2 -5 2



7. (UFSC)Para que o sistema abaixo seja impossvel, o
valor de a :
x y z
x y az
x y z
+ + =
+ + =
+ + =

3 4 1
2
2 3


8. (UFSC)Determine o valor de m para que o sistema,
abaixo admita infinitas solues:
mx y z
x my z
x y
=
=
=

2 0
2 0
3 2 0


Tarefa Complementar #

9. (UEPG-PR) O sistema linear

= + +
= + +
= + +
b 4z 2y 3x
2 z y x
3 3z y ax
:
01. impossvel para a= 2 e b = 5
02. impossvel para a = 2 e b = 5
04. possvel e determinado para a = 2 b e R
08. possvel e indeterminado para a = 2 e b = 5
16. possvel e determinado para a = 2

10. (UFSCar-SP) Dado o sistema linear
x ay z
ax y az
x ay z
+ + =
+ + =
+ + =

0
0
0
assinale a alternativa correta:

a) O sistema admite uma infinidade de solues
para qualquer a real.
b) O sistema no admite soluo de a = 1.
c) O sistema admite uma nica soluo se a = 3.
d) O sistema admite somente a soluo trivial.
e) O sistema admite uma nica soluo se a = 1.

11. (FEI-SP) Se o sistema
3 2 1 0
4 2 2 0
2 3 2 0
x y z
mx y z
x my z
+ + =
+ + =
+ + =


admite uma nica soluo, ento:

a) m = 6 d) m = 4
b) m = 2 e) m = 3
c) m = 8
12. (UFSC) Considere o sistema S
1
:

=
= +
0 6y - 2x -
0 3y x

determine a soma dos nmeros associados (s)
proposio(es) verdadeira(s).

01. O par ordenado (15,5) uma soluo do
sistema S
1
.
02. O sistema S
1
possvel e determinado.
04. A soluo do sistema S
1
uma reta que no passa
pela origem.
08. O sistema S
2
:

=
= +
0 30y - 10x -
0 6y 2x
equivalente ao
sistema S
1
.
Please purchase PDF Split-Merge on www.verypdf.com to remove this watermark.
Incluso para a vida Matemtica D

Pr-Vestibular da UFSC
21
13. (UFSC) Assinale a soma dos nmeros associados s
proposies verdadeiras:

01. O nmero de elementos de uma matriz quadrada de
ordem 12 48.
02. Somente podemos multiplicar matrizes de mesma
ordem.
04. A soma das razes da equao
x 4 4
x x 4
x x x
= 0 8.
08. Uma matriz quadrada pode ter diversas matrizes
inversas.
16. O sistema

= +
=
0 y x
0 2y 3x
indeterminado.
14. (UFSC) Assinale a soma dos nmeros associados s
proposies verdadeiras.
01. A matriz
(
(
(
(

0 2 1 3
1 8 4 5
1 5 2 4
0 3 2 1
no possui inversa.
02. Se um sistema de equaes indeterminado, ento no
se pode encontrar soluo para ele.
04. Uma pequena indstria produz trs tipos de produto
que indicamos por x, y, z. As unidades vendidas de
cada produto e o faturamento bruto da empresa em
trs meses consecutivos so os dados na tabela
abaixo. Ento, os preos dos produtos x, y e z s
podem ser, respectivamente, R$ 1.000,00, R$
5.000,00 e R$ 3.000,00.


Ms
Unidades
de x
vendidas
Unidades
de y
vendidas
Unidades
de z
vendidas
Faturamento
bruto
1 1 5 3 R$
35.000,00
2 4 1 2 R$
15.000,00
3 5 6 5 R$
50.000,00

08. A soluo da equao
0
2 1 3
4 2
1 4 2
= x
x = 1

15. (UFSC) Assinale as proposies corretas.

01. O par ordenado (x, y) = (5, 2) a nica soluo do
sistema

= +
= +
27 6y 3x
9 2y x

02. A matriz A = (a
ij
)
13
, tal que a
ij
= i 3j
A = | | 8 5 2 .
04. A soma dos elementos da inversa da matriz

(

1 0
1 1
igual a 2.
08. Uma matriz quadrada A se diz anti-simtrica se
t
A = -
A, sendo
t
A a transposta da matriz A. Nessas
condies, pode-se afirmar que a matriz

(
(
(

0 0 1
0 0 0
1 0 0
anti-simtrica.
16. Se as matrizes P, Q e R so escolhidas entre as listadas
a seguir, para que PQ R seja uma matriz nula, o
valor de x deve ser 2.

(
(
(

2
1
3
, | | 5 3x ,
(


x 2 0
1 1 6
,
(

6
19

32. A e B so matrizes quadradas de ordem 2 tais que A =
5B. Nestas condies, pode-se afirmar que det(A) =
5det(B), sendo que det(A) e det(B) designam,
respectivamente, os determinantes das matrizes A e
B.

16. (UFSC) Marque a(s) proposio(es) correta(s).

01. Dada uma matriz A, de ordem m x n, e uma matriz B
de ordem n x p, a matriz produto A.B existe e de
ordem m x p.
02. Se um sistema de equaes possui mais equaes do
que incgnitas, ento ele incompatvel (impossvel).
04. A terna (2, 1, 0) soluo do sistema

x y z
x y z
x y z
x y z
+ + =
=
+ + =
+ + =

2 3 4
2 2 3
3 7
6 2 2 14

08. Trs pessoas foram a uma lanchonete. A primeira
tomou 2 (dois) guarans e comeu 1 (um) pastel e pagou
R$ 4,00. A segunda tomou 1 (um) guaran e comeu
2(dois) pastis e pagou R$ 5,00. A terceira tomou 2
(dois) guarans e comeu 2(dois) pastis e
pagou R$ 7,00. Ento, pelo menos, uma das pessoas
no pagou o preo correto.

17. (FUVEST) O sistema linear

= +
= +
a y x
a y x
9 log 4 log
3 log 2 log
a) tem soluo nica se a = 0
b) tem infinitas solues se a = 2
c) no tem soluo se a = 3
d) tem infinitas solues se a = 4
e) tem soluo nica se a = 9

Please purchase PDF Split-Merge on www.verypdf.com to remove this watermark.
Incluso para a Vida Qumica A


1
UNIDADE 1

A Qumica pode ser definida como sendo a cincia que
estuda a matria e suas transformaes.
Matria: Tudo que tem massa e ocupa lugar no espao.
Massa: quantidade de matria de um corpo.
Volume: espao ocupado pela matria.
Corpo: uma poro finita de matria.
Objeto: um corpo produzido para utilizao humana.

Ex.: Matria Corpo Objeto

Granito Pedra Esttua

rvore Madeira Cadeira

Unidades de medida:
De massa: kg (SI) 1 kg = 1000g
1g = 1000mg
De volume: m
3
(SI) 1m
3
= 1000L
1mL = 1cm
3

Transformaes da Matria
So chamadas de fenmenos as transformaes ocorridas
em um sistema. Sistema toda regio do espao, com ou
sem matria, que esteja sendo estudada.
- Fenmenos qumicos: So aquelas onde h rearranjo da
estrutura molecular, ou seja, as molculas das substncias
envolvidas so desmanchadas e seus tomos
reagrupados de maneira diferente, dando origem a outras
substncias com propriedades diferentes. Dizemos, ento,
que ocorreu uma reao qumica. Exemplo: formao de
ferrugem, combusto da matria.
- Fenmenos fsicos: so transformaes que no alteram
a composio da matria. H, somente, uma mudana no
estado de agregao. Exemplo: a formao de gelo a partir
da gua.

Estados Fsicos da Matria
SLIDO: forma e volume constantes.
LQUIDO: forma varivel e volume constante.
GASOSO: forma e volume variveis.
Densidade: quantidade de matria contida em uma unidade
de volume. (d = m/v) g/mL ou g/cm
3
ou kg/L
_____________ _______________


SLIDO LQUIDO GASOSO


_____________ _______________



Exerccios de Sala #

1. O que faz a matria ser slida, lquida ou gasosa :
01. Densidade das substncias
02. Tamanho de suas molculas
04. Maior ou menor coeso entre suas molculas
08. Distribuio dos tomos na formao molecular
2. Considere os seguintes experimentos:
I - gotas de limo adicionadas a um copo com leite
II - fotografia amarelada pelo tempo ou pela exposio ao
sol
III - ter derramado sobre a pele
IV - cozimento de um ovo
V - aquecimento de um pedao de solda

H ocorrncia de reaes qumicas em:
a) I e II. c) I, II e III e) II, III e V
b) II e III. d) I, II e IV

Tarefa Mnima #

3. (ETFSP) Em um acampamento, todo o sal de cozinha
caiu e misturou com a areia. Para recuperarmos o sal:
I - Juntamos a mistura num recipiente e colocamos gua
para solubilizar o sal (a areia no se dissolve em gua).
II - Filtramos a mistura (a areia fica retida no filtro e a
mistura gua e sal so recolhidos num recipiente).
III - Evaporamos a gua recuperando o sal.

Dos fenmenos:
a) Apenas I qumico. d) I e II so fsicos.
b) I e II so qumicos. e) I, II e III so fsicos.
c) II e III so qumicos.

4. (ACAFE) fenmeno qumico a alternativa:
A pedra de naftalina deixada no armrio
a) A fuso do chumbo
b) A produo de luz por uma lmpada comum de
filamento
c) A atrao de uma pea de m
d) A combusto da gasolina

5. Considerando-se as foras de coeso e repulso que
atuam entre as molculas de determinada matria,
podemos afirmar que ao passar do estado lquido para o
estado slido um corpo ter o seu (a sua):
01. Volume aumentado
02. Densidade diminuda
04. Massa aumentada
08. Volume diminudo
16. Massa diminuda

6. (EEAR) A superfcie da gua em contato com uma
outra superfcie muito quente se vaporiza e, assim, a
poro superior da gua, ainda lquida, flutua sobre o
colcho de vapor de gua. Este fenmeno denominado:
a)Saturao c)Decantao
b) Calefao d) Destilao

Tarefa Complementar #

7. (UFSC) Fenmeno qumico aquele que altera a
natureza da matria, isto , aquele no qual ocorre uma
reao qumica. Baseado nessa informao, analise a(s)
proposio(es) abaixo e escolha aquela(s) que
corresponde(m) a um fenmeno qumico.
01. A combusto de lcool ou gasolina nos motores de
automveis.
02. A precipitao das chuvas.
04. A queima do gs de cozinha.
08. A formao do gelo dentro do refrigerador.
16. A formao da ferrugem sobre uma pea de ferro
deixada ao relento.
32. A respirao animal.


Please purchase PDF Split-Merge on www.verypdf.com to remove this watermark.
Qumica A Incluso para a Vida

Pr-Vestibular da UFSC
2
8. (UFSC) O(s) fenmeno(s) abaixo, que envolve(m)
reao(es) qumica(s), (so):
01. digesto dos alimentos
02. enferrujamento de uma calha
04. exploso da dinamite
08. fuso do gelo
16. queda da neve
32. combusto do lcool de um automvel
64. sublimao da naftalina

9. (UFSC) Assinale a nica proposio correta que contm
o melhor mtodo para separar os trs componentes de uma
mistura de areia, gua e sal, na seqncia adequada.
01. destilar e decantar
02. filtrar e decantar
04. decantar e filtrar
08. destilar e filtrar
16. filtrar e destilar

10. (UFSC) As transformaes que ocorrem em um
sistema podem ou no ocasionar alterao na constituio
da matria envolvida. De acordo com o enunciado,
est(o) correta(s) a(s) associao(es):

01. Digesto de um alimento (Fenmeno fsico)
02. gua oxigenada
luz
gua + oxignio gasoso
(Reao qumica)
04. Queima de fogos de artifcio (Fenmeno fsico)
08. Transformao do gelo em gua (Fenmeno fsico)
16. Sublimao do iodo slido (Reao qumica)

11. (UFSC) Considere os seguintes experimentos, e
assinale qual(is) da(s) transformao(es) (so)
considerada(s) fenmeno(s) qumico(s).
01. gua
eltrica corrente
gs hidrognio + gs
oxignio.
02. Gelo
calor
gua lquida.
04. Naftalina
(slida)
naftalina
(lquida)
.
08. Papel
fogo
CO
2
+ gua + cinzas.
16. Vinho vinagre.
32. Barra de ferro ferrugem.

UNIDADE 2

SUBSTNCIAS QUMICAS
Cada molcula representa uma substncia qumica bem.
Cada substncia, por sua vez, representada por sua
frmula.

Substncia: um tipo especfico de matria com
propriedades caractersticas.

Simples
Pura Composta
Subst. Homognea
Impura Heterognea
(Mistura) Euttica
Azeotrpica

Substncias Puras: So formadas por molculas
quimicamente iguais. Uma de suas caracterstica ter os
pontos de fuso e ebulio constante.

Substncia simples: combinao entre partculas (tomos)
de um mesmo elemento qumico.
Alotropia: o fenmeno qumico que ocorre quando um
elemento qumico forma duas ou mais substncias
qumicas diferentes.
Substncia composta: combinao entre tomos de
elementos qumicos diferentes.
Misturas: so formadas por molculas quimicamente
diferentes. Na realidade, so sistemas formados pela
reunio de diversos tipos de substncias. Diferente das
substncias puras, as misturas no tm temperaturas de
fuso e ebulio constantes.

Misturas homogneas (solues): misturas uniformes,
que apresentam as mesmas composies, propriedades e
aparncias ao longo de todo o volume, como o sal
dissolvido na gua ou o ar que respiramos.
Misturas heterogneas: no tm as mesmas composies,
propriedades e aparncia ao longo de todo o volume, como
a areia, as rochas e a madeira.
Misturas eutticas: mantm a fuso a temperatura
constante.


Misturas azeotrpica: mantm a ebulio a temperatura
constante.




V
t (s)
T
o
C
L+V
L
S+L
S
V
t (s)
T
o
C
L+V
L
S+L
S
V
t (s)
T
o
C
L+V
L
S+L
S
V
t (s)
T
o
C
L+V
L
S+L
S
Please purchase PDF Split-Merge on www.verypdf.com to remove this watermark.
Incluso para a Vida Qumica A


3
Classificao dos sistemas quanto capacidade de
trocar matria e energia com o meio ambiente:

Sistema Aberto: Troca tanto matria quanto energia. Ex.:
vidro de acetona aberto.
Sistema Fechado: Troca energia, mas no troca matria.
Ex.: garrafa de refrigerante fechada.
Sistema Isolado: No troca nem matria e no troca
energia. Ex.: garrafa trmica.
Fases de um sistema heterogneo: so as diferentes
partes homogneas da mistura.
Componentes de uma mistura: so as diferentes
substncias presentes na mistura.
OBS.:
- Todo sistema com mais de um componente uma
mistura.
- Todo sistema como mais de uma fase heterogneo.
- Num sistema com fase slida, os vrios pedaos da
mesma constituem uma nica fase. Ex.: 3 cubos de gelo.
- Toda mistura gasosa , por conveno, homognea.

CUIDADO: O sistema gua + gelo um sistema
heterogneo, mas no uma mistura.

Exerccios de Sala #

1. (EEAR) Um sistema cujo componente apenas gua,
constitudo por gua no estado lquido e trs cubos de gelo.
O sistema do tipo:
a) Homogneo
b) Heterogneo com uma fase
c) Heterogneo com duas fases
d) Heterogneo com quatro fases

2. (ACAFE) (modificada) O leite usado na alimentao
contm gua, sais minerais, protenas insolveis e
gorduras. Podemos afirmar que o leite :
a) Substncia pura simples
b) Mistura homognea
c) Mistura heterognea
d) Mistura azeotrpica
e) Substncia pura composta

Tarefa Mnima #

3. (ACAFE) A camada de oznio absorve os raios
ultravioletas mais energticos que chegam atmosfera,
protegendo-nos contra essas radiaes. A frmula
molecular do oznio :
a) O
2
b) O
3
c) O
4
d) CO
2
e) SO
2

4. (EEAR) Ao, Etanol (C
2
H
5
OH) e Oznio representam,
respectivamente:
a) substncia composta, mistura e substncia simples.
b) substncia simples, substncia composta e mistura.
c) substncia composta, elemento qumico e mistura.
d) mistura, substncia composta e substncia simples.

5. Assinale a(s) alternativa(s) que contm(m) sistemas
que renem somente substncias puras:
01. gua, areia e ar.
02. sal, grafite e limalha de ferro.
04. glicose, granito e gs carbnico.
08. lcool etlico, gua mineral e glicerina.

Tarefa Complementar #

6. (UFSC) Um dos critrios utilizados pelos qumicos para
classificar as substncias est no tipo de tomos que as
constitui. Assim, uma substncia formada por um nico
tipo de tomo dita simples e aquela formada por mais de
um tipo de tomo dita composta. Baseado nisso, escolha
a(s) proposio (es) que contm (m) somente
substncias simples:
01. H
2
O, H
2
CO
3
, CaO, MgS.
02. H
2
, CO
2
, O
2
, Au.
04. Na
2
CO
3
, Na
2
O, KCO, MgBr
2.

08. Ag, Fe, Na, K.
16. CH
4
, C
2
H
6
, H
2
, O
2.

32. H
2
O, H
2
, Na
2
CO
3
, O
2.


7. (UFSC) Tendo como base o diagrama abaixo, que
representa as curvas de aquecimento de duas substncias
puras slidas, A e B, ao longo do tempo, correto afirmar
que:

01. a temperatura de fuso de A maior que a de B.
02. admitindo-se que uma mistura de A e B, numa
proporo determinada, formaria uma soluo euttica,
isto , uma soluo que funde como se fosse um
composto puro, a mesma apresentaria ponto de fuso
igual a 70
o
C.
04. no intervalo de tempo, representado no grfico, a
substncia B no apresenta fuso.
08. nas temperaturas de fuso de A ou de B temos uma
mistura de slido e lquido, com muito slido no incio,
tendendo a lquido no final do processo.
16. Sob presso de 1 atm, o composto B poderia
representar a gua pura.

8. (UFSC) O gs carbnico (CO
2
) um(a) :
01. substncia simples.
02. composto binrio.
04. elemento qumico.
08. mistura homognea.
16. composto qumico.
32. variedade alotrpica do carbono.
64. substncia composta

9. (UFSC) Observe os recipientes A, B e C e seus
respectivos contedos.

Aps mistura e agitao do contedo dos trs recipientes
em um s, observa-se que apenas parte do acar e parte
do gelo permanecem insolveis. Assinale o nmero de
fases e o nmero de componentes do sistema resultante.
01. trifsico
02. 4 componentes
Please purchase PDF Split-Merge on www.verypdf.com to remove this watermark.
Qumica A Incluso para a Vida

Pr-Vestibular da UFSC
4
04. bifsico
08. 5 componentes
16. 2 componentes
32. monofsico
64. 3 componentes

UNIDADE 3

MODELOS ATMICOS

Modelo Atmico de Dalton
Todo e qualquer tipo de matria formado por partculas
indivisveis, macias, indestrutveis e eletricamente neutras
chamadas tomos. Cada tipo de tomo tem a sua massa e o
seu volume prprio.
Para Dalton, uma reao qumica uma
recombinao entre elementos de uma ou mais substncias
qumicas, formando conjuntos (substncias) diferentes dos
iniciais (reagentes).

Modelo Atmico de Thomson
O modelo do pudim de passas, segundo o qual os
tomos seriam bolhas positivas recheadas por eltrons, de
carga negativa, serviu para explicar a natureza eltrica da
matria, que pode ser comprovada atravs de fenmenos
como a condutividade eltrica de certas solues aquosas e
de metais.

Modelo Atmico de Rutherford

Experincia de Rutherford (1911)

Na figura temos:
1. Invlucro de chumbo.
2. Material Radiativo.
3. Partculas a emitidas.
4. Placa de ouro.
5. Tela fluorescente.
6. Partculas que retornam.
7. Partculas desviadas.
8. Partculas sem desvio mensurvel.
9. Cintilaes produzidas pelos choques.
Rutherford, ento, imaginou que o tomo teria uma
regio central densa (positivamente carregada), onde as
partculas o
refletiriam; regio a
qual ele chamou de
ncleo, e concluiu que
a mesma cercada por
um grande vazio de
massa desprezvel,
incapaz de refletir as
partculas, chamada
eletrosfera. Esse modelo tambm denominado modelo
planetrio.
A experincia mostrou ainda outro resultado
surpreendente: o dimetro do tomo 10.000 vezes maior
que o do ncleo, e a massa do tomo est praticamente
toda nos prtons e nutrons confinados na pequena regio
do ncleo.

Modelo Atmico de Bohr
Os eltrons nos tomos se movimentam ao redor do ncleo
em trajetrias circulares, chamadas de camadas ou nveis
de energia. Hoje so conhecidas pelo menos sete camadas,
denominadas K, L, M, N, O, P e Q.
Cada um desses nveis possui um valor
determinado de energia; os eltrons que esto mais
afastados do ncleo apresentaro valores mais altos.
Um eltron pode passar de um nvel para outro de
maior energia, desde que absorva energia externa (energia
eltrica, luz, calor etc.). Quando isso acontece, dizemos
que o eltron foi excitado.
O retorno do eltron ao nvel inicial se faz
acompanhar de liberao de energia na forma de ondas
eletromagnticas, que pode ser luz visvel, ultravioleta,
calor e etc. Cada salto quntico (retorno a um nvel menor)
libera uma quantidade fixa de energia.

Exerccios de Sala #

1. Considerando a experincia quanto ao modelo atmico
de Rutheford, assinale a alternativa incorreta.
a) Observando a disperso das partculas o (alfa) ao se
chocarem com a lmina de ouro, o cientista conclui que
o tomo tem o ncleo muito grande em relao sua
eletrosfera.
b) Rutheford sabia antecipadamente que as partculas o
(alfa) eram carregadas positivamente.
c) Algumas partculas alfa foram desviadas do seu trajeto
devido repulso exercida pelo ncleo atmico e seu
tamanho em relao eletrosfera do tomo.
d) Essa experincia permitiu descobrir o ncleo atmico e
seu tamanho em relao eletrosfera do tomo.
e) A experincia consistiu em bombardear lminas
finssimas de ouro com partculas o (alfa).

Tarefa Mnima #

2. (UFSC) Na famosa experincia de Rutherford, no
incio do sculo XX, com a lmina de ouro, o(s) fato(s)
que (isoladamente ou em conjunto) indicava(m) que o
tomo possui um ncleo pequeno e positivo foi (foram):

01. A maioria das partculas alfa atravessaria os tomos da
lmina sem sofrer desvio de sua trajetria.
02. Ao atravessar a lmina, uma maioria de partculas alfa
sofreria desvio de sua trajetria.
04. Um pequeno nmero de partculas alfa atravessando a
lmina sofreria desvio de sua trajetria.
08. Um grande nmero de partculas alfa no atravessaria
a lmina.
16. As partculas alfa teriam cargas negativas.

3. (UFSC) Uma das principais partculas atmicas o
eltron. Sua descoberta foi efetuada por J. J. Thomson em
uma sala do Laboratrio Cavendish, na Inglaterra, ao
provocar descargas de elevada voltagem em gases bastante
rarefeitos, contidos no interior de um tubo de vidro.
Please purchase PDF Split-Merge on www.verypdf.com to remove this watermark.
Incluso para a Vida Qumica A


5
ctodo
nodo
TUBOA


ctodo
nodo
TUBOB

ctodo
nodo
TUBOC

No tubo de vidro A, observa-se que o fluxo de eltrons
(raios catdicos) colide com um anteparo e projeta sua
sombra na parede oposta do tubo.
No tubo de vidro B, observa-se que o fluxo de eltrons
(raios catdicos) movimenta um catavento de mica.
No tubo de vidro C, observa-se que o fluxo de eltrons
(raios catdicos) sofre uma deflexo para o lado onde foi
colocada uma placa carregada positivamente.

Observando os fenmenos que ocorrem nos tubos,
podemos afirmar corretamente que:
01. gases so bons condutores da corrente eltrica.
02. os eltrons possuem massa so corpusculares.
04. os eltrons possuem carga eltrica negativa.
08. os eltrons partem do ctodo.
16. os eltrons se propagam em linha reta.
32. o catavento entrou em rotao devido ao impacto dos
eltrons na sua superfcie.

UNIDADE 4

ESTUDO DO TOMO
O tomo, Hoje:

Prtons
Ncleo Nutrons
tomo
Eletrosfera Eltrons

Partcula
Massa
Real (g)
Massa
Relativa
Carga
Real (C)
Carga
Relativa
Prton 1,67x10
-24
1 +1,6x10
-19
+1
Nutron 1,68x10
-24
1 0 0
Eltron 9,11x10
-28

1840
1
-1,6x10
-19
-1

- Os prtons e os nutrons tm massas praticamente
iguais.
- A massa do eltron desprezvel, se comparada do
prton.
- A carga do prton igual do eltron, porm de sinal
contrrio.
- A quantidade de prtons de um tomo sempre igual
de eltrons (p=e
-
), por isso, todo tomo eletricamente
neutro.

Nmero atmico (Z): o nmero de prtons presentes no
ncleo do tomo. Z=p

Nmero de massa (A): a soma do nmero de prtons e
de nutrons existente no ncleo de um tomo. A=p+n

ons: so partculas em desequilbrio eltrico, ou seja, tm
quantidade de prtons diferente da quantidade de eltrons.
Quando um tomo perde ou recebe eltrons ele se
transforma em on.

ons Ction = on (+) = perdeu e
-
.
nion = on () = ganhou e
-
.

nion (on negativo): forma-se quando um tomo ou
grupo de tomos recebe eltrons, ficando com um excesso
de carga negativa. O nmero de cargas negativas igual ao
nmero de eltrons recebidos.
Ction (on positivo): forma-se quando um tomo ou
grupo de tomos perde eltrons, ficando com um excesso
de carga positiva. O nmero de cargas positivas igual ao
nmero de eltrons perdidos.

Elemento Qumico: o conjunto de tomos com o
mesmo nmero atmico (Z). Um elemento qumico
representado por
Z
X
A
.

IstoPos: So tomos diferentes que tm a mesma
quantidade de prtons. Todos os istopos pertencem a um
mesmo elemento qumico.
IstoNos: So tomos diferentes que tm a mesma
quantidade de nutrons.
IsbAros: So tomos diferentes que tm o mesmo nmero
de massa.
ons isstEros ou isoeletrnicos: So formados a partir de
tomos de diferentes elementos qumicos e tm a mesma
estrutura eletrnica (mesma quantidade de eltrons).

Atividade interativa: Dados os tomos representados
abaixo, diga quais so (se existirem) istopos, istonos e
isbaros.
A
38
18
B
36
18
C
33
15
D
30
15
E
36
16


Exerccios de Sala #
1. As partculas que compem o nion
2 32
16
S so:
a) 16 prtons, 18 eltrons e 32 nutrons
b) 16 prtons, 16 eltrons e 16 nutrons
c) 18 prtons, 18 eltrons e 18 nutrons
d) 16 prtons, 32 eltrons e 16 nutrons
e) 16 prtons, 18 eltrons e 16 nutrons

2. Podemos afirmar que o nmero de massa de um on :
a) Menor que o nmero de massa do tomo que lhe deu
origem.
b) A soma do nmero de eltrons do tomo que lhe deu
origem.
Please purchase PDF Split-Merge on www.verypdf.com to remove this watermark.
Qumica A Incluso para a Vida

Pr-Vestibular da UFSC
6
c) Maior que o nmero de massa do tomo que lhe deu
origem.
d) A soma do nmero de prtons do tomo que lhe deu
origem.
e) Igual ao nmero de massa do tomo que lhe deu origem.

Tarefa Mnima #

3. Considerando o sistema abaixo, onde os tomos esto
representados por esferas, pode-se afirmar que neles
existem:

a) 2 elementos, 2 substncias simples e 2 substncias
compostas
b) 4 elementos, 1 substncias simples e 3 substncias
compostas
c) 4 elementos, 2 substncias simples e 3 substncias
compostas
d) 3 elementos, 1 substncias simples e 4 substncias
compostas
e) 1 elementos, 2 substncias simples e 3 substncias
compostas

4. (ACAFE) Uma das seguintes espcies qumicas
no isoeletrnica das demais: P
3+
, S
2-
, Cl
-
, Ar, Ca
2+
. Qual
, dentre as seguintes alternativas, a que contm esta
espcie qumica?
a) Cl
-
b) S
2-
c) P
3+
d) Ar e) Ca
2+


5. (ESA) O tomo eletricamente neutro porque:
a) nmero de prtons = nmero de nutrons;
b) nmero de prtons > nmero de eltrons
c) nmero de nutrons = nmero de eltrons
d) nmero de eltrons = nmero de prtons

6. A partcula formada por 30 prtons, 33 nutrons e 28
eltrons constitui um:
a) ction bivalente d) nion monovalente
b) nion bivalente e) tomo neutro
c) ction monovalente

Tarefa Complementar #

7. (UFSC) A respeito da estrutura da matria, correto
afirmar que:
01. os tomos formam a matria
02. os tomos possuem partculas chamadas de prtons e
eltrons
04. os eltrons possuem carga eltrica positiva
08. os nutrons possuem carga eltrica negativa
16. os nutrons tambm so partculas do tomo

8. Assinale a(s) alternativa(s) correta(s).
01. O nmero de massa (A) dado pela quantidade total de
partculas do tomo.
02. Ction um on positivo que se forma quando um
tomo ganha eltrons.
04. Todo tomo tem a quantidade de prtons igual
quantidade de nutrons.
08. Todo tomo tem a quantidade de prtons igual
quantidade de eltrons.
16. nion um on negativo que se forma quando um
tomo perde eltrons.

9. (UFSC) A palavra tomo originria do grego e
significa indivisvel, ou seja, segundo os filsofos
gregos, o tomo seria a menor partcula da matria que no
poderia ser mais dividida. Atualmente essa idia no
mais aceita. A respeito dos tomos, verdadeiro afirmar
que:
01. no podem ser desintegrados
02. so formados por, pelo menos, trs partculas
fundamentais.
04. possuem partculas positivas denominadas eltrons
08. apresentam duas regies distintas o ncleo e a
eletrosfera
16. apresentam eltrons, cuja carga eltrica negativa.
32. contm partculas sem carga eltrica, os nutrons.

10. (UFPR) Para interpretar a grande maioria dos
processos qumicos, suficiente considerar o tomo como
sendo constitudo por apenas trs partculas: o prton, o
nutron e o eltron. Essas trs partculas no so
distribudas ao acaso; elas interagem entre si e essa
interao produz o conjunto organizado, que o tomo. A
respeito do tomo, correto considerar:
01. Prtons e nutrons so encontrados no ncleo, que a
parte do tomo com carga eltrica positiva e que
contm praticamente toda a massa do tomo.
02. Os eltrons, partculas de carga eltrica negativa,
distribuem-se em torno do ncleo em diversos nveis e
subnveis energticos (camadas e subcamadas).
04. Se o nmero de eltrons de um tomo for igual ao
nmero de prtons, o tomo ser neutro; se for maior
ser um nion; se for menor, ser um ction.
08. O nmero de prtons de um tomo denominado
nmero atmico e representado pela letra Z.
16. O ncleo de tomos ser sempre formado por igual
nmero de prtons e nutrons.
32. A soma de prtons e nutrons de um tomo
conhecida como nmero de massa, que representado
pela letra A.

UNIDADE 5

A ELETROSFERA

A eletrosfera toda a regio que envolve o ncleo e
composta por eltrons. uma espcie de nuvem
eletrnica.
Sommerfield percebeu que os nveis de energia de Bohr
estariam divididos em regies ainda menores,
denominadas de subnveis, denominados: s, p, d, f.
DISTRIBUIO ELETRNICA:
Existem algumas regras que devem ser observadas para o
correto preenchimento dos orbitais:

Regra de Hund: O eltron ocupa inicialmente o orbital
vazio de um subnvel.

Princpio de Pauli: Dentro de um orbital podemos
encontrar no mximo 2 eltrons, sempre com spins
contrrios.

A ordem crescente de energia dos subnveis definida
pelo diagrama de Linus Pauling.

Please purchase PDF Split-Merge on www.verypdf.com to remove this watermark.
Incluso para a Vida Qumica A


7
DIAGRAMA DE LINUS-PAULING:

Para um on: faz-se a distribuio dos eltrons para o
tomo neutro e, s depois que ser feita a retirada ou o
acrscimo dos eltrons na camada mais externa do tomo.

Camada de valncia: a camada mais externa do tomo,
e dela que se retira ou acrescenta eltrons.

Orbitais: Orbital a regio da eletrosfera onde mxima
a probabilidade de se encontrar determinado eltron. Os
orbitais do subnvel s tm o formato esfrico e os do
subnvel p tm o formato alongado.




orbital s orbital p

Exerccios de Sala #

1. (ACAFE) A configurao eletrnica do on frrico
(Fe
3+
) :
a) 1s
2
2s
2
2p
6
3s
2
3p
6
4s
2
3d
3

b) 1s
2
2s
2
2p
6
3s
2
3p
6
4s
2
3d
6

c) 1s
2
2s
2
2p
6
3s
2
3p
6
3d
4

d) 1s
2
2s
2
2p
6
3s
2
3p
6
3d
5

e) 1s
2
2s
2
2p
6
3s
2
3p
6
4s
2
3d
9


Tarefa Mnima #
2. Os tomos
32
16
31
15
S e P so:
a) istopos c) istonos
b) isbaros d) ismeros

3. Sejam dois elementos isbaros A e B. Sabendo que o
nmero atmico de A 64 e o nmero de massa de B
154, ento o nmero de nutrons no ncleo dos tomos de
A ser igual a:
a) 64 b) 90 c) 154 d) 218

4. (ACAFE) A vitamina B
12
, antianmica, contm um
on Co
2+
. A configurao eletrnica, nos orbitais 4s e 3d
do Co
2+
:
a) 4s
2
3d
7
c) 4s 3d
5
e) 4s
0
3d
6

b) 4s
0
3d
7
d) 4s 3d
6


5. (EEAR) O nmero mximo de eltrons de um tomo,
distribudos em cinco nveis de energia, :

6. Sobre o elemento qumico de nmero atmico 23,
correto afirmar que:
a) Tem sete nveis de energia.
b) Seu subnvel mais energtico o 4d.
c) Tem dois eltrons em sua camada de valncia.
d) A sua camada de valncia a camada M.
e) Todos os seus orbitais esto completos.

Tarefa Complementar #

7. (UFSC) Considerando as relaes entre os tomos,
indicadas no esquema abaixo, pode-se afirmar que o(s)
nmero(s)
20
38
i s b a r o s
i

s


t

o

n

o

s
i

s


t

o

p

o

s
X
Z
Y
40
20

01. de massa de Y 40.
02. de massa de Z 20.
04. de prtons de Y 22.
08. de nutrons de X 20.
16. de prtons de Z 22.
32. de nutrons de Y 20.
64. de nutrons de Z 20.

8. (UFSC) A(s) distribuio(es) eletrnica(s), em ordem
crescente de energia, no on Mn
2+
(so):
01. 1s
2
2s
2
2p
6
3s
2
3p
6
4s
2
3d
5

02. 1s
2
2s
2
2p
6
3s
2
3p
6
3d
5
4s
2

04. 1s
2
2s
2
2p
6
3s
2
3p
6
3d
5

08. 1s
2
2s
2
2p
6
3s
2
3p
6
4s
1
3d
4

16. 1s
2
2s
2
2p
6
3s
2
3p
6
3d
3
4s
2

32. 1s
2
2s
2
2p
6
3s
2
3p
6
3d
7

64. 1s
2
2s
2
2p
6
3s
2
3p
6
3d
4
4s
1


9. (UFSC) Sobre o tomo de potssio K
40
19
, no estado
fundamental, correto afirmar que:
01. Possui 19 prtons, 21 nutrons e 19 eltrons.
02. Sua ltima camada possui 8 eltrons.
04. Possui 6 eltrons no subnvel 3p.
08. Sua penltima camada possui 8 eltrons.
16. Seus eltrons esto distribudos em trs nveis de
energia.
32. um metal alcalino.

10. (UFSC) Analise as duas afirmaes:
A luz emitida nos luminosos a base de gs
nenio, ( ) Ne
20
10
, so originadas em tubos de baixa presso
com descarga eltrica de alta voltagem. Os chineses, desde
o sculo X, utilizavam efeitos luminosos pela queima de
fogos de artifcio.
Assinale a(s) proposio(es) verdadeira(s):
01. A ionizao do tomo de nenio acontece com a perda
de eltrons do subnvel 2p.
02. A luz emitida tanto pelo gs nenio, quanto pelos
fogos de artifcio pode ser explicada atravs do salto
dos eltrons para nveis mais energticos. Esta luz ser
liberada quando da volta do eltron sua camada de
origem.
04. A luz emitida pelo gs nenio ocorre pela reao
qumica entre todos os tomos presentes no tubo.
08. O nenio um gs nobre com a seguinte configurao
eletrnica: 1s
2
2s
2
2p
6
.
Please purchase PDF Split-Merge on www.verypdf.com to remove this watermark.
Qumica A Incluso para a Vida

Pr-Vestibular da UFSC
8
11. (UFSC) Assinale o(s) elemento(s) qumico(s) que
apresenta(m) seu tomo com todos os orbitais atmicos
completos em sua distribuio eletrnica fundamental.
01. cloro
02. nquel
04. nitrognio
08. nenio
16. ltio
32. zinco

UNIDADE 6

NMEROS QUNTICOS

So em nmero de 4 e servem para identificar ou localizar
um determinado eltron na eletrosfera. So eles:

n
o
quntico principal (n): indica a camada em que se
encontra determinado eltron. Varia de 1 a 7.
K L M N O P Q
N 1 2 3 4 5 6 7
n
o
quntico secundrio (l): Indica o subnvel em que se
encontra determinado eltron. Varia de 0 a 3:
s p d F
l 0 1 2 3

n
o
quntico magntico (m ou ml): Indica o orbital em que
se encontra determinado eltron. Varia de -l a + l.
Subnvel s: 1 orbital, 2 eltrons.

0

Subnvel p: 3 orbitais, 6 eltrons

-1 0 +1

Subnvel d: 5 orbitais, 10 eltrons

-2 -1 0 +1 +2

Subnvel f : 7 orbitais, 14 eltrons

-3 -2 -1 0 +1 +2 +3

n
o
quntico spin (m ou m
s
): Indica o sentido de rotao do
eltron dentro do orbital. Pode ser 1/2 ou +1/2.
Convencionou-se adotar o valor negativo para o primeiro
eltron a ocupar um determinado orbital, que
representado, tambm por conveno, por uma seta
apontada para cima.

Eltrons de diferenciao ou diferenciador: o eltron
de maior nvel energtico da eletrosfera, ou seja, o ltimo
da distribuio eletrnica.

Exerccios de Sala #

1. (UFSC) Qual o nmero atmico (Z) do tomo cujo
eltron de diferenciao (3, 2, +1, +1/2)?

2. Assinale a alternativa incorreta.
a) O nmero quntico principal indica a camada em que
se encontra o eltron.
b) O nmero quntico secundrio indica a forma do
orbital.
c) O nmero quntico magntico indica qual o nvel
energtico onde se encontra o eltron.
d) O nmero quntico spin indica qual o sentido de
rotao do eltron.
e) O nmero quntico magntico varia de -l a +l.

Tarefa Mnima #

3. Assinale a(s) alternativa(s) correta(s).
01. impossvel um eltron ter n=3 e l =3.
02. impossvel um eltron ter l =1 e m=+2.
04. Se um tomo tem quatro camadas eletrnicas, o seu
eltron de diferenciao deve estar necessariamente na
camada N.
08. O eltron de diferenciao est sempre na camada de
valncia.
16. O eltron de diferenciao est sempre no seu subnvel
mais energtico.

4. (UFSC) Considere um tomo representado pelo seu
nmero atmico Z = 58 e em seu estado normal. correto
afirmar que:
01. o mesmo possui um total de 20 eltrons em subnvel f.
02. o primeiro nvel de energia com eltrons em orbitais d
o n = 4.
04. se um de seus istopos tiver nmero de massa 142, o
nmero de nutrons desse istopo 82.
08. os subnveis 5s 4d 5p 6s 4f no esto escritos na sua
ordem crescente de energia.
16. sua ltima camada contm 2 eltrons no total.
32. um de seus eltrons pode apresentar o seguinte
conjunto de nmeros qunticos: n=2, " =0, m =+1, s
=+.

UNIDADE 7

TABELA PERIDICA

uma forma de organizar os resultados obtidos
experimentalmente para cada elemento, de tal maneira que
as semelhanas, diferenas e tendncias se tornassem mais
evidentes.
Os elementos qumicos conhecidos esto organizados em
ordem crescente de nmeros atmicos e, agrupados em
linhas (perodos) e em colunas (grupos ou famlias).
Perodos: So agrupamentos horizontais, as linhas da
tabela peridicas, nas quais esto agrupados os elementos
que tm em comum a quantidade de camadas eletrnicas.
So em nmero de 7.
Grupos ou Famlias: So agrupamentos verticais, as
colunas da tabela peridica, nas quais esto agrupados os
elementos que apresentarem o mesmo nmero de eltrons
no ltimo subnvel da distribuio eletrnica; o que
determina a semelhana entre as propriedades qumicas
dos mesmos. Existem 18 grupos ou famlias na tabela
peridica.





Please purchase PDF Split-Merge on www.verypdf.com to remove this watermark.
Incluso para a Vida Qumica A


9
CLASSIFICAO DOS ELEMENTOS

Representativos
de Transio

Elementos Metais
Qumicos No-metais
Semimetais
Gases nobres
Hidrognio

Gases Nobres: so elementos que possuem estabilidade
eletrnica, ou seja, no possuem tendncia em perder nem
em receber eltrons.

Metais: possuem tendncia em perder eltrons para
possuir a estabilidade eletrnica dos gases nobres.

No-Metais: possuem a tendncia em receber eltrons
para possuir a estabilidade eletrnica dos gases nobres.
H





G
a
s
e
s

N
o
b
r
e
s




Ametais

Metais





Metais


Elementos Representativos: so os elementos que tm o
subnvel mais energtico na ltima camada e cujas
distribuies eletrnicas terminam em s ou p
elementos das famlias A.
Famlia IA (grupo 1) = metais alcalinos:
Famlia II A (grupo 2): metais alcalinos terrosos:
Famlia VI A (grupo 16): calcognios
Famlia VII A (grupo 17): halognios
Famlia 0 (zero) (grupo 18): gases nobres

Elementos de Transio (externa) so os elementos que
tem o subnvel mais energtico na penltima camada e
cujas distribuies eletrnicas terminam em d.
Elementos de Transio (interna) so os elementos que
tem o subnvel mais energtico na antepenltima camada e
cujas distribuies eletrnicas terminam em f.

s

R
e





p
r
e
s
e
n
t
a
t
i
v
o


Representativo
p
Transio Externad


Transio Interna f


Exerccios de Sala #

1. (UFSC) Considerando a classificao peridica dos
elementos, assinale as opes corretas:
01. Os metais alcalinos e alcalino-terrosos ocupam a parte
central dessa classificao.
02. Os tomos de cloro e bromo possuem o mesmo
nmero de eltrons na ltima camada.
04. Os elementos qumicos de nmeros atmicos 44 e 82
so, respectivamente, a prata e o ouro.
08. O tomo de fsforo, no seu estado fundamental, possui
2 eltrons no subnvel 3s e 3 eltrons no subnvel 3p.

2. Um tomo apresenta a seguinte configurao eletrnica:
1s
2
2s
2
2p
6
3s
2
3p
6
4s
1
. Podemos afirmar que este tomo
pertence ao grupo dos:
a) Metais alcalino-terrosos.
b) Halognios.
c) Gases nobres.
d) Metais alcalinos
e) Calcognios.

Tarefa Mnima #

3. (ACAFE) Um elemento X apresenta como ltimo
subnvel de sua distribuio eletrnica 4s
1
. O elemento X
:

4. (UFSC) A partir do grfico dos nveis de energia do
tomo neutro e com auxlio da tabela peridica, assinale
a(s) opo(es) correta(s):
E
N
E
R
G
I
A
1s
2s
2p
3s
3p
3d
4p
4s

01. Trata-se de um elemento representativo
02. Sua posio, na tabela peridica, na coluna 7B e
quarto perodo.
04. Apresenta propriedades qumicas semelhantes s dos
elementos com nmeros atmicos entre 5 e 10
08. O subnvel de maior energia o 4s
16. Possui 8 (oito) eltrons em subnveis s
32. Todos os orbitais esto completos
64. O elemento apresenta cinco eltrons desemparelhados
no subnvel d

5. (ACAFE) Os trs elementos x, y e z tm as seguintes
estruturas eletrnicas no estado fundamental:
x 1s
2
, 2s
2
, 2p
6
, 3s
2
, 3p
6
, 4s
2
, 3d
5.

y 1s
2
, 2s
2
, 2p
6
, 3s
2
, 3p
6.

z 1s
2
, 2s
2
, 2p
6
, 3s
2
, 3p
6
, 4s
2
, 3d
10
, 4p
4.


De acordo com tais estruturas, os trs elementos podem ser
classificados, respectivamente, como:

a) elemento de transio gs nobre elemento
representativo.
b) elemento de transio elemento representativo gs
nobre.
c) elemento representativo gs nobre elemento de
transio.
Please purchase PDF Split-Merge on www.verypdf.com to remove this watermark.
Qumica A Incluso para a Vida

Pr-Vestibular da UFSC
10
d) elemento representativo elemento de transio gs
nobre.
e) gs nobre elemento de transio elemento
representativo.

Tarefa Complementar #

6. (UFSC) Assinale as correspondncias corretas entre
smbolos e elementos.
01. W - tungstnio
02. Zn - zinco
04. Pb - chumbo
08. Fr - frmio
16. Ra - rdio
32. Mg - mangans
64. Co - cobre

7. (UFSC) Cada elemento qumico tem, associado ao seu
nome, um smbolo que o representa. Escolha a(s)
opo(es) que associa(m) corretamente nomes e
smbolos:
01. S = enxofre, F = flor.
02. Ag = prata, O = oxignio.
04. Pb = chumbo, Po = potssio.
08. C = carbono, Au = ouro.
16. N = nitrognio, H = hlio.
32. Fe = ferro, Na = sdio.
64. Hg = mercrio, P = polnio.

8. (UFSC) Assinale a(s) correspondncia(s) correta(s)
entre smbolo, elemento e classificao peridica.
01. S enxofre metal alcalino
02. Ca clcio metal alcalino
04. Nb nibio metal de transio
08. Si silcio semi-metal
16. Li ltio metal alcalino terroso
32. Mg mangans no metal

9. (UFPR) Entre as alternativas abaixo, indique aquela(s)
que contm afirmaes incorretas sobre os elementos
cujos nmeros atmicos so dados a seguir: A (Z=11), B
(Z=8), C(Z=20), D(Z=18), E(Z=17).
01. O elemento C um gs nobre e o elemento B um
halognio.
02. A e C esto em um mesmo grupo, mas esto em
perodos diferentes.
04. O elemento E um calcognio e se situa no quinto
perodo.
08. O elemento B um halognio do terceiro perodo da
tabela peridica.
16. A, B, C, D e E so todos metais de transio.
32. O elemento C se encontra no quinto perodo da tabela
peridica.

10. (UFSC) Recentemente foi divulgada pela imprensa a
seguinte notcia: Uma equipe de cientistas americanos e
europeus acaba de acrescentar dois novos componentes da
matria tabela peridica de elementos qumicos,
anunciou o laboratrio nacional Lawrence Berkeley
(Califrnia). Estes dois recm-chegados, batizados
elementos 118 e 116, foram criados em abril num
acelerador de partculas, atravs do bombardeamento de
objetos de chumbo com projteis de criptnio, precisou o
comunicado do laboratrio, do Departamento Americano
de Energia. A equipe que criou os dois novos elementos
composta de cientistas europeus e americanos.
DIRIO CATARINENSE-13/06/99.

Com base neste texto, assinale a(s) proposio(es)
verdadeira(s) de acordo com a classificao peridica
atual.
01. O elemento de nmero 118 ser classificado como um
gs nobre.
02. O elemento de nmero 116 ser classificado como
pertencente famlia dos halognios.
04. Os dois novos elementos pertencero ao perodo
nmero 7.
08. O elemento chumbo utilizado na experincia
representado pelo smbolo Pb.
16. O novo elemento de numero 118 tem 8 eltrons no
ltimo nvel, quando na sua configurao fundamental.
32. Esses dois novos elementos so caracterizados como
elementos artificiais, uma vez que no existem na
natureza.

UNIDADE 8

PROPRIEDADES PERIDICAS
E APERIDICAS

PROPRIEDADES APERIDICAS:
So aquelas cujos valores numricos crescem ou
decrescem com o aumento do nmero atmico.
Ex.: Massa atmica: cresce com o aumento do nmero
atmico.
Calor especfico: decresce com o aumento do nmero
atmico.

PROPRIEDADES PERIDICAS:
So aquelas cujos valores numricos crescem e decrescem
com o aumento do nmero atmico. Estas propriedades
dependem da posio dos elementos qumicos na tabela
peridica. As principais funes peridicas so:

Raio Atmico:
a distncia entre o centro do ncleo e a ltima camada
eletrnica. O raio atmico possui relao direta com o
tamanho do tomo.
Numa mesma famlia, aumenta de cima para baixo,
conforme o aumento do nmero de camadas ocupadas por
eltrons. Num mesmo perodo, aumenta da direita para a
esquerda porque, para um mesmo nmero de camadas
ocupadas, os elementos situados esquerda possuem carga
nuclear menor, o que implica numa menor atrao por
eltrons.




Eletronegatividade (carter no metlico):
a capacidade que um tomo tem de atrair para si o par
eletrnico que ele compartilha com outro em uma ligao
covalente.
Quanto menor o raio atmico, maior a
eletronegatividade. Por isso, nas famlias, ela aumenta de
baixo para cima. Nos perodos, devido maior atrao
exercida pela carga nuclear de elementos com maior
nmero atmico, aumenta da esquerda para a direita.
Portanto, os elementos mais eletronegativos so os
Please purchase PDF Split-Merge on www.verypdf.com to remove this watermark.
Incluso para a Vida Qumica A


11
2 4 6 8 10 12 14 16 18 20 22 24 26 28 30 32
5
10
15
20
25
E
n
e
r
g
i
a

d
e

i
o
n
i
z
a

o

(
e
V
)
Nmero atmico (Z)
halognios (especialmente o flor), o oxignio e o
nitrognio.




Potencial de Ionizao (energia de ionizao):
a energia mnima necessria para arrancar um eltron de
um tomo isolado no estado gasoso. a medida da
facilidade que um tomo possui em perder eltrons.
Quanto menor o potencial de ionizao, maior a
tendncia do tomo em perder eltrons.
Est diretamente relacionada com o raio atmico:
quanto maior for o raio atmico, menor ser a atrao do
ncleo (+) pelos eltrons e maior ser a tendncia do
tomo em perder eltrons. Os potenciais de ionizao mais
altos so os dos gases nobres.
Pode ser representado pelas seguintes equaes:
E + energia
1
E
+
+ e
-
(1
o
Pot. De Ioniz.)
E
+
+ energia
2
E
++
+ e
-
(2
o
Pot. De Ioniz.)
E
++
+ energia
3
E
+++
+ e
-
(3
o
Pot. De Ioniz.)
Onde E um tomo de um elemento qumico qualquer.
energia
1
< energia
2
< energia
3





Afinidade Eletrnica (eletroafinidade):
a quantidade de energia liberada quando um tomo
isolado no estado gasoso recebe um eltron. a medida da
atrao por eltrons. Est diretamente relacionada com a
eletronegatividade. Por isso, nas famlias aumenta de baixo
para cima e nos perodos da esquerda para a direita. Pode
ser representado pelas seguintes equaes:
E + e
-
E
-
+ energia
1
(1
a
Eletroafinidade)
E
-
+ e
-
E
2-
+ energia
2
(2
a
Eletroafinidade)
E
2-
+ e
-
E
3-
+ energia
3
(3
a
Eletroafinidade)
Onde E um tomo de um elemento qumico qualquer.
energia
1
> energia
2
> energia
3





Exerccios de Sala #

1. (ACAFE) (modif.) Com relao eletronegatividade,
verdade afirmar:
a) Quanto mais eletronegativo um elemento qumico,
maior ser o seu carter metlico.
b) Quanto maior a eletronegatividade, maior a tendncia
do tomo neutro doar eltrons.
c) A eletronegatividade aumenta nos perodos da
esquerda para a direita, pois os gases nobres so
elementos qumicos mais eletronegativos.
d) Quanto maior a eletronegatividade, maior ser a
energia liberada por um tomo neutro no estado gasoso
quanto recebe um eltron.
e) A eletronegatividade uma propriedade peridica
que se comporta de maneira semelhante densidade
nos perodos.
2. (UFSC) Dados os seguintes elementos qumicos,
assinale a(s) proposio(es) correta(s).

Li Fr He F
C"

A"
01. O flor mais eletronegativo que o frncio.
02. O elemento ltio um metal alcalino.
04. O elemento hlio um gs nobre.
08. O elemento cloro liga-se ao elemento alumnio na
proporo de um tomo de cloro para um tomo de
alumnio.
16. O raio atmico do ltio menor que o raio do on Li
1+
.
32. O elemento alumnio um semimetal.

Tarefa Mnima #

3. (ACAFE) Considerando os elementos do grupo 7A, na
tabela peridica, incorreto afirmar:
a) Entre eles esto o flor e o iodo.
b) Tm facilidade em receber eltrons.
c) Podem se combinar com o hidrognio na proporo
atmica 1:1.
d) Possuem 7 eltrons na camada de valncia.
e) So eletropositivos.

4. (ACAFE) A alternativa verdadeira que apresenta os
ons, em ordem crescente de seus raios, :
a) K
+
< Ca
2+
< Se
2-
< Br
-

b) K
+
< Ca
2+
< Br
-
< Se
2-

c) Ca
2+
< K
+
< Br
-
< Se
2-

d) Br
-
< Se
2-
< Ca
2+
< K
+

e) Se
2-
< Br
-
< K
+
< Ca
2+

Tarefa Complementar#

5. (ACAFE) Assinale a alternativa que apresenta o
comportamento da eletronegatividade dos elementos nos
perodos da tabela peridica.
a) aumenta da direita para a esquerda
b) aumenta da esquerda para a direita
c) no varia
d) diminui de acordo com o aumento do nmero atmico
e) aumenta com o aumento do carter metlico

6. (UFSC) A energia de ionizao dos elementos qumicos
uma propriedade peridica, isto , varia regularmente
quando os mesmos esto dispostos num sistema em ordem
crescente de seus nmeros atmicos. O grfico a seguir,
mostra a variao da energia de ionizao do 1
o
eltron,
em e.V, para diferentes tomos. Com base na ilustrao,
assinale a(s) proposio(es) correta(s).












Please purchase PDF Split-Merge on www.verypdf.com to remove this watermark.
Qumica A Incluso para a Vida

Pr-Vestibular da UFSC
12
01. A carga nuclear o nico fator determinante da
energia de ionizao.
02. Selecionando-se trs tomos com maior dificuldade
para formarem ctions monovalentes, teramos os
tomos de He, Li e Na.
04. O potssio o metal que apresenta o menor potencial
de ionizao, entre os elementos representados.
08. No intervalo Z = 3 a Z = 10, observa-se que o
aumento da carga nuclear tende a aumentar a fora
de atrao do eltron pelo ncleo.
16. Os elevados valores da energia de ionizao para os
gases He, Ne e Ar so evidncias de que camadas
eletrnicas completas so um arranjo estvel.
32. Considerando os elementos que formam um perodo
da tabela peridica, a tendncia da energia de
ionizao diminuir com o aumento do nmero
atmico.
64. As menores energias de ionizao correspondem aos
metais alcalinos.

UNIDADE 9

LIGAES QUMICAS
Na natureza os tomos, de uma forma geral, buscam
adquirir maior estabilidade. Para isto tendem a se unir uns
aos outros, ganhando, perdendo ou compartilhando
eltrons.
Um tomo adquire estabilidade quando possui oito
eltrons na u. c. e., ou dois, se tiver apenas a primeira
camada, ou seja, quando assume a configurao de um gs
nobre, que so os nicos tomos que so estveis quando
isolados.
Ao se unirem uns aos outros os tomos do origem a
foras de atrao denominadas ligaes qumicas, que
podem ser classificadas em 3 tipos, dependendo dos
tomos envolvidos.

Inica (M-N)
Ligao Qumica Metlica (M-M)
Covalente (N-N)

Onde M um metal e N pode ser um no-metal, um
semimetal ou o Hidrognio.

Ligao Inica
Acontece entre metal e ametal, ou entre um metal alcalino
ou alcalino terroso e o hidrognio.


Metal Perde e
-
Fica (+) Ction
No-metal Ganha e
-
Fica (-) nion
Baseando-se no esquema acima, podemos concluir
que a ligao inica resulta de uma atrao eletrosttica
entre ctions e nions.

Compostos Inicos:
So aqueles que possuem pelo menos uma ligao inica.
Por ser uma atrao muito forte, as substncias inicas
apresentam elevados pontos de fuso e ebulio (P.F. do
NaCl: 800
o
C) e, conseqentemente, so slidas
temperatura ambiente.
So formadas por cristais, no interior dos quais h um
aglomerado de ons positivos e negativos, que se alternam
nas trs dimenses; a estrutura se mantm rgida pela ao
das foras de atrao eltrica entre os ctions e nions.
No h formao de molculas e sim de retculos
cristalinos.
As substncias inicas no conduzam corrente
eltrica no estado slido, mas o fazem quando fundidas ou
em soluo aquosa, pois nos ltimos dois casos seus ons
tm liberdade de movimento.

Frmula de um Composto Inico
[Ction
X+
][nion
Y-
] = Ction
Y
Anion
X

LIGAO METLICA
Os metais tm como principal caracterstica a grande
tendncia em perder eltrons. Devido a isso, os eltrons da
ltima camada escapam facilmente do tomo (o que
pode ser comprovado pelo baixo potencial de ionizao
dos metais) e ficam passeando pelo metal. Podemos
dizer que, segundo esta teoria, o metal seria um
aglomerado de ctions mergulhados em um mar de
eltrons livres que estariam atraindo-se mutuamente.
Ligas metlicas so misturas de diferentes metais
que, por terem tomos de diferentes tamanhos, se
caracterizam por maior dureza e menor maleabilidade.

Exerccios de Sala #

1. (UFSC) Considerando a ligao qumica entre o
oxignio e o alumnio, sob a luz da teoria do octeto, para a
formao do xido de alumnio, correto afirmar:
01. Cada tomo de alumnio perder trs eltrons.
02. O oxignio ser o nion, com carga negativa igual a
trs para cada tomo.
04. Sero envolvidos dois tomos de alumnio na ligao.
08. Cada tomo de oxignio receber dois eltrons.
16. O nmero total de cargas positivas, por frmula, ser 6.
32. A configurao eletrnica do Al
+3
ser 1s
2
2s
2
2p
6.

64. A frmula mnima do xido de alumnio conter
quatro tomos no total.

Tarefa Mnima #

2. As alternativas abaixo apresentam vrias afirmaes
sobre os elementos situados no primeiro grupo da
classificao peridica. Assinale a nica falsa.
a) So designados metais alcalinos.
b) So mais eletronegativos do que os elementos
correspondentes do quarto grupo.
c) Seus xidos reagem com a gua formando bases fortes,
os hidrxidos alcalinos.
d) So bons condutores de eletricidade no estado slido.
e) Apresentam um eltron na camada de valncia.

3. (UFSC) Sobre um elemento qumico que possui a
configurao eletrnica 1s
2
2s
2
2p
6
3s
2
, verdadeiro
afirmar que:
01. Ao se unir com o oxignio, poder faz-lo na
proporo de dois tomos dele para um de oxignio
02. Ter nmero de oxidao 2 nos seus compostos
04. Ao se unir com o cloro, poder faz-lo na proporo de
um tomo dele para trs de cloro
08. Ao se unir com os outros elementos para formar
Please purchase PDF Split-Merge on www.verypdf.com to remove this watermark.
Incluso para a Vida Qumica A


13
compostos, poder faz-lo por ligao inica
16. Nas suas ligaes qumicas, doar dois eltrons por
tomo

4. (UFSC) De modo geral, os compostos que possuem
ligaes inicas:
01. So solveis em solventes apolares
02. So encontrados na natureza no estado slido
04. Apresentam pontos de ebulio elevados e pontos de
fuso baixos
08. So duros e quebradios
16. Apresentam alta condutividade eltrica em soluo
aquosa

5. (UFPel-RS) Em abril de 1997, foi comemorado o
centenrio de descoberta do eltrons. Considerando-se que
a interao mais ntima entre os tomos que se ligam, na
formao de compostos, feita pelos seus eltrons mais
externos, caracterizando, assim, o seu comportamento, a
frmula qumica provvel, resultante da combinao entre
tomos dos elementos K (Z = 19) e S (Z = 16), e o tipo de
ligao que ocorre entre eles so, respectivamente:
a) KS
2
; ligao inica.
b) K
2
S; ligao covalente polar.
c) K
2
S; ligao covalente apolar.
d) K
2
S; ligao inica.
e) KS
2
; ligao covalente polar.

Tarefa Complementar#

6. (UEL-PR) A frmula do hidreto de um metal alcalino-
terroso (M) :
a) M
2
H b) MH
3
c) MH d) M
2
H
2
e) MH
2


7. (ACAFE) Um metal alcalino terroso (M) apresenta 2
eltrons na sua camada de valncia. A alternativa que
indica a frmula de um xido e de um cloreto desse metal,
respectivamente, :
a) M
2
O - M
2
Cl
b) M
2
O - MCl
c) MO
2
- MCl
2

d) MO - MCl
2

e) MO - MCl
4


8. O elemento qumico alumnio (Al) pode se ligar a um
elemento qumico para formar um composto inico na
proporo de 2:3. Este elemento um:
a) Metal alcalino.
b) Metal alcalino terroso.
c) Calcognio.
d) Halognio.
e) Gs nobre.
UNIDADE 10

LIGAO COVALENTE

o tipo de ligao qumica que ocorre com
emparelhamento ou compartilhamento de eltrons. As
substncias que possuem somente ligaes deste tipo
formam molculas.
Como os dois elementos que se ligam possuem
tendncia em receber eltrons, eles compartilham um ou
mais pares de eltrons de forma que os dois passem a
possuir o mesmo nmero de eltrons na ltima camada que
um gs nobre. Estas ligaes Acontecem entre:
- NO-METAIS + NO-METAIS ou
- HIDROGNIO + NO-METAIS.
As ligaes covalentes podem ser classificadas de
vrias formas:

Simples
Dupla
Tripla
Ligao Normal
Covalente Dativa
Polar
Apolar
o (sigma)
t (pi)

Ligao Simples: os tomos compartilham apenas um par
de eltrons ( representada por um trao). Ex: HH;
Ligao Dupla: os tomos compartilham dois pares de
eltrons ( representada por dois traos); Ex: O=O;
Ligao Tripla: os tomos compartilham trs pares de
eltrons ( representada por trs traos). Ex: NN;
Ligao Covalente Normal: o par de eltrons
compartilhado00 formado por um eltron de cada um dos
tomos, ou seja, cada tomo participou com 1 eltron. Ex:
Todas as acima.
Ligao Covalente Dativa: Neste tipo de ligao, os dois
eltrons do par so fornecidos por um nico tomo, e
ocorre quando um dos tomos j completou o octeto e o
outro no. Apesar de o tomo j estar estvel ele possui
um par de eltrons livre sem fazer nenhuma ligao e,
portanto, pode utiliz-lo para fazer outra ligao.
Obs: importante ressaltar que no h transferncia de
eltrons, como na ligao inica, e sim compartilhamento,
afinal, um tipo particular de ligao covalente. (
representada por uma seta). Ex: O=OO;
Ligao Polar: Ocorre quando h diferena de
eletronegatividade entre os tomos que se ligam. Para isto,
basta que estes tomos sejam de diferentes elementos
qumicos. Neste caso o tomo do elemento mais
eletronegativo formar o plo negativo da ligao, e o
mais eletropositivo formar o plo positivo. Quanto maior
a diferena de eletronegatividade entre os dois, maior a
polaridade da ligao (dizemos que h um maior carter
inico da ligao covalente).
Ligao Apolar: Ocorre quando no h diferena de
eletronegatividade entre os tomos que se ligam. Para isto,
basta que estes tomos sejam do mesmo elemento
qumico. Neste caso, dizemos que a ligao 100%
covalente (carter inico igual a zero).

Exerccios de Sala #

1. (UFSC) Assinale a(s) proposio(es) correta(s). Os
compostos formados a partir dos elementos oxignio,
cloro, sdio e clcio devem apresentar frmulas, ligaes
qumicas predominantes e estados fsicos, em condies
ambientes, respectivamente:
01. CaCl
2
, inica, slido.
02. NaCl, inica, lquido.
04. Cl
2
, covalente, gs.
08. Na
2
O, covalente, lquido.
16. O
2
, inica, gs.
Please purchase PDF Split-Merge on www.verypdf.com to remove this watermark.
Qumica A Incluso para a Vida

Pr-Vestibular da UFSC
14

2. (UFSC) Os smbolos X e Y representam elementos
qumicos quaisquer, diferenciados por seus nmeros
atmicos (Z). Sobre eles, correto afirmar que:
01. Quando X (Z=19) se combina com Y (Z=17), o
composto resultante tem frmula molecular XY e a
ligao covalente apolar.
02. Se X e Y esto bem afastados na fila de reatividade
qumica, e X e Y tm 1 e 6 eltrons, respectivamente,
na ltima camada eletrnica, o composto resultante da
combinao entre eles ser molecular e com frmula
XY
2
.
04. tomos do elemento X (Z=20) e do elemento Y (Z=7)
unem-se por ligaes covalentes e o composto tem
frmula molecular X
3
Y
4
.
08. Pela configurao eletrnica da ltima camada dos
elementos X e Y,
X =
1s
|

Y =
2p 2p 2p 2s
| | |! |!

O composto formado entre X e Y tem frmula X
2
Y.
16. Se X (Z=1) e Y (Z=11, 37 ou 55), os compostos
formados sero todos hidretos metlicos.

Tarefa Mnima #

3. (ACAFE) Temos uma substncia A que:
- nas condies ambientais slida;
- possui alto ponto de fuso;
- no estado slido no conduz eletricidade;
- dissolve-se em gua;
- conduz eletricidade em soluo aquosa
ou quando fundida.
Considerando as caractersticas dadas, o mais provvel
que A seja um:
a) composto molecular polar que se ioniza em gua.
b) composto inico que se dissocia em gua.
c) composto molecular polar que se dissocia em gua.
d) composto apolar que se dissocia em gua.
e) metal que reage com a gua.

4. (ACAFE) Na formao da molcula de cloro, Cl
2
, o
orbital atmico dos tomos de Cl que se entrelaam, para
formar um orbital molecular, :
a) f b) s c) d d) p e) sp
3



Tarefa Complementar#

5. (ACAFE) A ligao qumica entre tomos iguais para
formar molculas diatmicas sempre uma ligao:
a) polar
b) inica
c) metlica
d) eletrovalente
e) covalente apolar

6. O elemento de nmero atmico 16 usado pela
natureza para ligar cadeias de protenas. A alternativa que
mostra a configurao eletrnica do tomo desse elemento
e prediz o tipo de ligao que ele forma com tomos de
carbono :
a) 1s
2
2s
2
2p
6
3s
2
3p
4
: ligao covalente
b) 1s
2
2s
2
2p
6
3s
1
3p
5
: ligao covalente
c) 1s
2
2s
2
2p
6
3s
2
3p
4:
ligao inica
d) 1s
2
2s
2
2p
6
3s
2
3p
2
4s
2
: ligao covalente
e) 1s
2
2s
2
2p
6
3s
1
3p
5
: ligao inica

7. (MACK-SP) A molcula que apresenta somente uma
ligao covalente normal :
a) O
2
b) F
2
c) CO d) O
3
e) H
2
O


UNIDADE 11

GEOMETRIA E POLARIDADE
DAS MOLCULAS

As molculas apresentam uma forma geomtrica definida,
fundamental para o estudo da polaridade molecular. Para
prever a geometria de uma molcula basta sabermos o
nmero de tomos ao redor do tomo central e se
existe(m) par(es) de eltrons no compartilhado(s) pelo
tomo central.
Para entender a polaridade de uma molcula faz-se
necessrio lembrar da polaridade de uma ligao. Ligao
polar aquela em que um tomo concentra cargas
positivas e o outro concentra as cargas negativas. Da
mesma forma, molcula polar aquela que tem um lado
positivo e outro negativo.
A polaridade de uma molcula depende da
eletronegatividade dos elementos qumicos envolvidos e
da geometria molecular. Para saber se uma molcula
polar ou apolar devemos lembrar a regra abaixo:
Ligaes Apolares Molcula Apolar
Ligaes Polares Molcula Polar
Ligaes Apolares Molcula Polar (os vetores se
anulam)
Para que os vetores se anulem, so necessrias duas
condies:
Os eltrons da u.c.e. do tomo central devem estar
todos sendo compartilhados (geometria linear, trigonal
ou tetradrica);
Todos os seus ligantes devem ser iguais.

Modelo Exemplo Geometria Polaridade Obs.:
X
2
H
2
Linear Apolar
XY HCl Linear Polar
XY
2

CO
2
Linear Apolar
H
2
O Angular Polar
XY
3

BF
3
Trigonal Apolar
NH
3
Piramidal Polar
XY
4
CH
4
Tetradrica Apolar

Exerccios de Sala #

1. (UFSC) Considere a tabela abaixo e selecione a(s)
proposio(es) que relaciona(m) corretamente a forma
geomtrica e a polaridade das substncias citadas:


Please purchase PDF Split-Merge on www.verypdf.com to remove this watermark.
Incluso para a Vida Qumica A


15
FRMULA CO
2
H
2
O NH
3
CCl
4

MOMENTO
DIPOLAR
RESULTANTE (
R
)

R
= 0
R
= 0
R
= 0
R
= 0
01. H
2
O: angular e polar.
02. CO
2
: linear e apolar.
04. CCl
4
: trigonal e polar.
08. NH
3
: piramidal e polar.
16. CCl
4
: tetradrica e apolar.

2. (ACAFE) O gs dixido de carbono (CO
2
), principal
responsvel pelo efeito estufa, apresenta quatro ligaes
covalentes _________e . Assinale a alternativa
que completa corretamente o enunciado acima.
a) polares ; molcula polar
b) polares ; molcula apolar
c) apolares ; molcula polar
d) apolares ; molcula apolar
e) coordenadas ; molcula apolar

Tarefa Mnima #

3. (UFES) A molcula que apresenta momento dipolar
diferente de zero (molcula polar) :
a) CS
2
b) BCl
3
c) NH
3
d) CBr
4
e) BeH
2

4. (ACAFE) Em relao polaridade das molculas no
suficiente conhecer a eletronegatividade dos elementos
constituintes da mesma, pois ela depende tambm da sua
geometria. A alternativa que apresenta as molculas, H
2
S,
CO
2
e H
2
O, em ordem decrescente de polaridade, :
a) H
2
O - H
2
S - CO
2

b) H
2
S - CO
2
- H
2
O
c) CO
2
- H
2
O - H
2
S
d) CO
2
- H
2
S - H
2
O
e) H
2
S - H
2
O - CO
2


UNIDADE 12

FORAS INTERMOLECULARES

So foras de atrao existentes entre as molculas de uma
mesma substncia, podendo ser intensas, moderadas ou
fracas.

Tipos de Interaes Intermoleculares:

Foras de Van der Waals (dipolo induzido): so foras
de atrao de fraqussima intensidade e ocorrem apenas
entre molculas apolares,
Dipolo-dipolo: so foras de atrao de fraca intensidade
e ocorrem entre molculas polares. Como as molculas
polares possuem plos (+) e (), ocorre uma atrao entre
os mesmos fazendo com que as molculas fiquem unidas.
Pontes de Hidrognio: ou Ligaes de Hidrognio. So
foras de atrao de forte intensidade e ocorrem entre
molculas polares que tenham tomo(s) de hidrognio
ligado(s) a tomo(s) dos elementos flor, oxignio ou
nitrognio.

Foras Intermoleculares e as Propriedades Fsicas

SOLUBILIDADE:
Substncias que possuem o mesmo tipo de atrao
intermolecular so, geralmente, miscveis (solveis).
Lembre-se que semelhante dissolve semelhante:
Substncia polar dissolve substncia polar.
Substncia apolar dissolve substncia apolar.
A gua considerada um solvente universal somente
para compostos inorgnicos, pois poucos compostos
orgnicos so solveis em gua.

PONTOS DE FUSO E EBULIO:
Dependem do tamanho da molcula (quanto maior, mais
altos PF e PE) e do tipo de atrao intermolecular (quanto
mais forte, mais altos PF e PE).

Exerccios de Sala #

1. (ACAFE) Do ponto de vista fsico, as pontes de
hidrognio influem sobre o comportamento das
substncias. Com relao a este fato, analise o grfico
abaixo e assinale a alternativa verdadeira.
0
-50
-100
50
100
H O
2
H
2
Se
H
2
HF
HC"
H
2
S
HBr
HI
Te
Temperatura de ebulio (C)
Nmero atmico
crescente

a) A ponte de hidrognio torna a substncia mais voltil.
b) Ponte de hidrognio facilita a vaporizao ou a fervura
da substncia.
c) A gua possui maior temperatura de ebulio e mais
voltil devido ponte de hidrognio
d) Entre os halogenetos (HF, HCl, HBr e HI), o HF possui
maior temperatura de ebulio devido ponte de
hidrognio
e) O gs sulfdrico no forma ponte de hidrognio, por isso
menos voltil que a gua.

2. (ACAFE) A gua uma substncia que permitiu a
criao e a manuteno da vida no planeta Terra. Analise
as seguintes afirmaes sobre a gua.
I - capaz de formar pontes de hidrognio.
II - Sua molcula tem forma geomtrica no-linear.
III - Sua molcula tem forma geomtrica linear.
IV - Solubiliza substncias de baixa polaridade, como
hidrocarbonetos.
A alternativa, que contm todas as afirmaes que esto
corretas, :
a) II III c) I - II e) III IV
b) I - II IV d) I - II - III IV

t
e
m
p
e
r
a
t
u
r
a

o

C

Please purchase PDF Split-Merge on www.verypdf.com to remove this watermark.
Qumica A Incluso para a Vida

Pr-Vestibular da UFSC
16
Tarefa Mnima #

3. (UFSC) O gelo seco corresponde ao CO
2
solidificado,
cuja frmula estrutural O=C=O. O estado slido
explicado por uma nica proposio. Assinale-a:
01. Foras de Van der Waals entre molculas fortemente
polares do CO
2

02. Pontes de hidrognio entre molculas do CO
2

04. Pontes de hidrognio entre a gua e o CO
2

08. Foras de Van der Waals entre as molculas apolares
do CO
2

16. Interaes fortes entre os dipolos na molcula do CO
2


4. (UFSC) O ponto de ebulio das substncias qumicas
pode ser utilizado para se estimar a fora de atrao entre
as suas molculas. O grfico abaixo relaciona as
temperaturas de ebulio, na presso de 1atm,
considerando o equilbrio lquido-vapor dos hidretos das
famlias 6A e 7A da tabela peridica, em funo do
perodo do elemento que se liga ao hidrognio. Com base
na mesma, assinale (a)s proposio(es) verdadeira(s):





















01. A partir do 3
o
perodo, as molculas dos hidretos se
tornam maiores e os seus pontos de ebulio
aumentam.
02. A gua e o fluoreto de hidrognio tm pontos de
ebulio mais altos do que os previsveis em relao ao
tamanho de suas molculas.
04. O HF e a H
2
O apresentam foras de atrao
intermoleculares, caractersticas de molculas polares,
contendo tomos de hidrognio ligados a tomos muito
eletronegativos.

08. A 25
o
C e 1 atm, todas as substncias representadas
esto no estado fsico gasoso, com exceo da gua.
16. A -50
o
C e 1 atm, o H
2
Se est no estado fsico lquido.

5. (ACAFE) As interaes na hlice dupla do DNA esto
exemplificadas abaixo.
Essas interaes esto identificadas na opo:
a) 1 ponte de hidrognio
b) 3 pontes de hidrognio
c) 1 ligao trplice coordenada
d) 3 ligaes de Wan der Walls
e) ponte etilnica

UNIDADE 13

NMERO DE OXIDAO (NOX)

o nmero que designa a carga eltrica real ou aparente
de um tomo em funo da diferena de eletronegatividade
entre ele e seus ligantes.
a carga positiva ou negativa que indica o n
o
de
eltrons que se afasta ou aproxima do tomo, e depende da
eletronegatividade dos tomos envolvidos na ligao.
- Nos compostos inicos o Nox equivale carga do on;
- Nos compostos moleculares no h perda ou ganho de
e

. Por isso, neste caso, devemos entender o Nox como


sendo a carga que o tomo teria se as suas ligaes
fossem rompidas, ficando os e

com os tomos mais


eletronegativos.


Como o cloro mais eletronegativo que o hidrognio, ele
puxa os eltrons com mais fora ficando parcialmente
negativo.
Nox do H = +1 (perde e

)
Nox do Cl = 1 (ganha e

)


Como o oxignio mais eletronegativo que o carbono, ele
puxa os eltrons com mais fora ficando parcialmente
negativo.
Nox do C = +4 (perde 4e

)
Nox do O = 2 (ganha 2e

) cada tomo

Nmero de Oxidao Mdio:
a mdia aritmtica dos Nox de todos os tomos de cada
elemento qumico em um determinado composto.

Regras Prticas para Calcular o Nox
1A e Ag: Nox = +1
2A e Zn: Nox = +2
Al: Nox = +3
H: Nox = +1 (nos hidretos metlicos = -1)
F: Nox = 1
O: Nox = 2 (exceto binrio c/ F, H, 1A e 2A)
7A: Nox = 1 (se estiver direita da frmula)
6A: Nox = 2 (se estiver direita da frmula)
Subst. Simples: Nox = zero
H2O
HF
HCl
H2S
H2Se
H2Te
HBr
HI
100
75
50
25
0
-25
-50
-75
-100
2
o
3
o
4
o
5
o
perodo
H
C
C
N
H
C
C
N
O
N
H
H O
H
H
N
N
C
C
H
C
N
N
C N
C H
Guanina
Citosina
Please purchase PDF Split-Merge on www.verypdf.com to remove this watermark.
Incluso para a Vida Qumica A


17
Nos compostos, a soma proporcional de todos os Nox
igual a zero.
A soma dos Nox dos tomos de um on composto igual
carga do on.

VARIAO DO NOX (Nox mximo e mnimo):
O nmero de oxidao de cada elemento qumico pode
variar, dependendo do nmero de eltrons na sua camada
de valncia.
Ex.:
S (6 e

na u.c.e.): pode ganhar at dois eltrons e pode


perder at seis: 2 s Nox s +6;
C (4 e

na u.c.e.): pode ganhar at quatro eltrons e


pode perder at quatro: 4 s Nox s +4;
Br (7 e

na u.c.e.): pode ganhar um eltron e pode


perder at sete: 1 s Nox s +7.

Atividade interativa:
Determinar o nox de cada um dos elementos das
substncias a seguir:

a) Al
2
O
3
b) KNO
3
c) KMnO
4


d) HClO
4
e) Al
2
(SO
4
)
3
e) O
2
f) Cu

g) NO
3
-
h) NH
4
+


Cuidado com elemento de Nox varivel (principalmente
metais de transio).

g) Fe
2
(SO
4
)
3
h) FeSO
4
i) CuSO
4


Exerccios de Sala #
1. Assinale a opo onde temos um metal de transio
com o nox +4:
a) FePO
4
b)

SO
2
c) CaO d) PbO
2
e) PbO

2. Assinale a opo onde o elemento central possui o nox
+2:
a) NaHSO
4
b) NaHCO
3
c) H
2
S
2
O
3
d) KMnO
4


Tarefa Mnima #

3. Indique a opo onde o nmero de oxidao do
nitrognio +4:
a) HNO
2
c) NO
2
e) NH
4
+

b) NH
3
d) N
2


4. O hidrognio possui Nox 1 no composto:
a) CH
4
c) NaHSO
4
e) RbH
b) NH
3
d) H
2
O

Tarefa Complementar #

5. (ACAFE) Determinando o nmero de oxidao do
elemento central do cido sulfuroso (H
2
SO
3
), cido
carbnico (H
2
CO
3
), cido silcico (H
2
SiO
3
), cido
pirofosfrico (H
4
P
2
O
7
) e cido perclrico (HClO
4
), os
valores so respectivamente:
a) +2, +4, +5, +5, +7.
b) +1, +1, +1, +2, +3.
c) +4, +4, +4, +5, +7.
d) +3, +3, +3, +7, +4.
e) 2, +4, +5, -5, +7.

UNIDADE 14

FUNES QUMICAS
(SEGUNDO ARRHENIUS)

Funes qumicas so grupos de
substncias que se assemelham
propriedades. So quatro as
funes qumicas estudadas:
cidos, bases, sais e xidos.

CIDOS:
cidos so substncias moleculares que, quando em
soluo aquosa, sofrem ionizao formando como nico
ction o on H
3
O
+
(H
2
O + H
+
), denominado on hidrnio.

Ionizao: o processo em que ons so criados quando
certas substncias moleculares polares so misturadas em
gua.
Dissociao: a separao dos ons que ocorre quando
uma substncia inica misturada em gua.

Frmula geral: H
a
Y onde a a carga de Y.
Note que a frmula de todo cido comea com H.

CLASSIFICAO DOS CIDOS

Quanto ao n
o
de
elementos
Binrios: HCl, H
2
S, HI.
Ternrios: H
3
PO
4
, H
2
SO
4
, HCN.
Quaternrios: H
4
Fe(CN)
6

Quanto ao n
o
de
hidrognios
ionizveis
Monocidos: HCl, HCN, H
3
PO
2.

Dicidos: H
2
SO
4
, H
3
PO
3.

Tricidos: H
3
Fe(CN)
6
, H
3
PO
4.

Policidos: H
4
Fe(CN)
6

Quanto presena
de oxignio
Oxicidos: H
2
SO
4
, H
3
PO
4
, H
4
SiO
4.

Hidrcidos: HCl, HCN, H
4
Fe(CN)
6.

FORA DE UM CIDO
A fora de um cido dada por:
s dissolvida molculas
ionizadas molculas
_
_
= o
a porcentagem de molculas que se ionizaram.
Quando o<5%, o cido fraco;
Quando 5<o<50%, o cido moderado;
Quando o>50%, o cido forte;
Para saber em qual categoria se enquadra cada
cido apenas pela frmula, utilizamos as seguintes regras:

Para os hidrcidos: Fortes: HI, HBr, HCl.
Moderado: HF
Fracos: todos os demais

Para os oxicidos: Temos que calcular A= m n,
sendo H
n
XO
m

cidos Fortes: A=2 ou 3
cidos Moderados: A=1
Ction nion
cido H
+
Y
a

Base X
b+
(OH)


Sal X
b+
Y
a

Please purchase PDF Split-Merge on www.verypdf.com to remove this watermark.
Qumica A Incluso para a Vida

Pr-Vestibular da UFSC
18
cidos Fracos: A = 0

Uma exceo regra o H
2
CO
3
, que um cido fraco.
Cuidado com os cidos derivados do fsforo, H
3
PO
2
,
H
3
PO
3
, H
3
PO
4
, todos moderados, pois nem todos os H da
molcula so ionizveis.

Exerccios de Sala #

1. O cido sulfrico (H
2
SO
4
) considerado um cido forte
porque:
a) Possui hidrognios ionizveis.
b) corrosivo.
c) Possui densidade elevada.
d) Possui elevado grau de ionizao.
e) N.d.a.

2. Quando um cido reage com um metal, h formao de:
a) um sal c) um xido
b) uma base d) outro cido

Tarefa Mnima #

3. As caractersticas abaixo so dos cidos, exceto:
a) Apresentam H na molcula.
b) Bons condutores de eletricidade em soluo aquosa.
c) Coram em vermelho o tornassol azul.
d) Tornam incolor a fenolftalena vermelha.
e) Sabor custico.

4. (UFSC) Assinale a(s) equao(es) que representa(m),
isoladamente, cada etapa da dissociao do cido
fosforoso, cuja frmula estrutural :
O
||
H O P O H
|
H
01. H
3
PO
3
+ H
2
O H
2
PO
3

+ H
3
O
+

02. H
3
PO
3
+ 3H
2
O H
2
PO
3
3
+ 3H
3
O
+

04. H
3
PO
3
+ H
2
O H
3
PO
2
+ H
2
O
2

08. H
2
PO
3

+ H
2
O HPO
3
2
+ H
3
O
+

16. H
3
PO
3
+ H
2
O H
3
PO
4
+ H
2

32. HPO
3
2
+ H
2
O PO
3
3
+ H
3
O
+


5. As substncias que tm sabor azedo, reagindo com as
bases, formando sal e gua, so:
a) sais c) cidos
b) bases d) xidos

6. (UFSC) Considerando-se, exclusivamente, a diferena
entre o nmero de oxignios e o nmero de hidrognios
ionizveis, em cada cido, assinale o(s) par(es) abaixo em
que o cido esquerda mais forte que o cido direita.
01. H
3
BO
3
e HNO
3
08. H
3
PO
4
e HNO
3

02. HClO
4
e H
2
SO
4
16. H
3
PO
2
e HBrO
4
04. HClO
4
e HClO 32. H
2
SO
4
e HClO






UNIDADE 15

NOMENCLATURA DOS CIDOS

P/ os hidrcidos: cido _________drico elemento

A terminao dos oxicidos depende do Nox do
elemento central. A tabela abaixo mostra a terminao
para os principais oxicidos.

Per_______ico +7
_______ico
+6 ou +5
_______oso +4 ou +3
hipo______oso +2 ou +1

Exceto com B, C e Si, sempre com terminao ico.
- Abaixo esto os nomes dos principais cidos que todos
devem saber, lembrando que, ao retirar um Oxignio da
frmula, desce-se uma linha na tabela de Nox acima;

Propriedades dos cidos:
a) Sabor azedo;
b) Conduzem eletricidade quando em soluo;
c) So, na grande maioria, solveis em gua;
d) Reagem com base formando sal e gua;
e) Descoram a fenolftalena vermelha.

Exerccios de Sala #

1. Os cidos fosfrico, nitroso, sulfrico e clordrico so
representados, respectivamente, pelas seguintes frmulas
moleculares:
a) H
3
PO
4
, HNO
2
, H
2
SO
4
e HCl.
b) H
3
PO
3
, HNO
3
, H
2
SO
3
e HCl.
c) H
3
PO
4
, HNO
2
, H
2
SO
4
e HClO
3
.
d) H
3
PO
3
, HNO
2
, H
2
SO
3
e HClO
3
.
e) H
3
PO
4
, HNO
3
, H
2
SO
4
e HClO.

Tarefa Mnima #

2. O cido sulfuroso apresenta a seguinte frmula:
a) HSO
3
c) H
2
SO
4

b) HSO
4
d) H
2
SO
3


UNIDADE 16

BASES

Bases so substncias inicas* que, quando misturadas
gua, sofrem dissociao liberando como on negativo
somente o nion OH
-
, denominado nion hidrxido
H
2
SO
4
cido Sulfrico
H
3
PO
4
cido Fosfrico
HNO
3
cido Ntrico
H
2
CO
3
cido Carbnico
HClO
4
cido Perclrico
HCl cido Clordrico
HCN cido Ciandrico
Please purchase PDF Split-Merge on www.verypdf.com to remove this watermark.
Incluso para a Vida Qumica A


19

Frmula geral: X(OH)
b
onde b a carga de X.
Note que a frmula de toda base termina com OH.

CLASSIFICAO DAS BASES

Quanto ao n
o

de OH
Monobases: NaOH, KOH, NH
4
OH.
Dibases: Ca(OH)
2
, Pb(OH)
2.

Tribases: Au(OH)
3
, Fe(OH)
3.

Tetrabases: Pb(OH)
4
,
Quanto
volatilidade
Voltil: NH
4
OH (nica molecular).
Fixas: Todas as demais (inicas).
Quanto
solubilidade
em gua
Muito solveis: Todas as que tm metal do
grupo 1A e NH
4
OH.
Pouco solveis: Todas as que tm metal
do grupo 2A.
Praticamente insolveis: Todas as demais.

FORA DE UMA BASE
- Bases fortes: Todas as que tm metal do grupo 1A e
2A, exceto Mg.
- Bases fracas: Todas as demais bases.
NOMENCLATURA DAS BASES
- Quando o metal tem nox fixo:
hidrxido de .
nome do elemento
- Quando o metal tem nox varivel:
hidrxido de Nox em n
o
romano
nome do elemento

hidrxido term. ico(>) ou oso(<)
nome do elemento

Ex.: NaOH: hidrxido de sdio
Ca(OH)
2
: hidrxido de clcio
Al(OH)
3
: hidrxido de alumnio
NH
4
OH: hidrxido de amnio
CuOH: hidrxido de cobre I ou hidrxido cuproso
Cu(OH)
2
: hidrxido de cobre II ou hidrxido cprico
Fe(OH)
2
: hidrxido de ferro II ou hidrxido ferroso
Fe(OH)
3
: hidrxido de ferro III ou hidrxido frrico
Pb(OH)
2
: hidrxido de chumbo II ou plumboso
Pb(OH)
4
: hidrxido de chumbo IV ou plmbico

Propriedades das bases:
a) Sabor adstringente (prende a lngua);
b) Conduzem eletricidade quando fundidas ou em soluo
aquosa;
c) A maior parte insolvel em gua;
d) Reagem com cido formando sal e gua;
e) Tornam azul o papel tornassol vermelho;

Exerccios de Sala #

1. Assinale a alternativa na qual o nome da base no
corresponde frmula dada.
a) LiOH = hidrxido de ltio.
b) Fe(OH)
3
= hidrxido ferroso.
c) Al (OH)
3
= hidrxido de alumnio.
d) Fe(OH)
2
= hidrxido de ferro II.
e) N.d.a.

2. (UFSC) Assinale qual(is) das substncias abaixo
reage(m) com a gua lquida, produzindo hidrognio
gasoso (H
2
).
01. Sdio metlico.
02. Hidrxido de potssio.
04. cido clordrico.
08. Potssio metlico.
16. Cloro gasoso.
32. Gs carbnico.

Tarefa Mnima #

3. (ACAFE) O gs carbnico, CO
2
, absorvido por
solues bsicas. Para remover o CO
2
de uma mistura
gasosa, borbulha-se a mistura em uma soluo de:
a) H
2
SO
4
c) KOH e) C
2
H
5
OH
b) HCl d) NaCl

4. O hidrxido de clcio a cal apagada ou extinta est na
opo:
a) HCl c) NaCl
b) Ca(OH)
2
d) CaO

UNIDADE 17

SAIS

Sais so compostos que podem ser obtidos a partir da
reao de um cido com uma base. Este tipo de reao
chamado de reao de neutralizao ou salificao.
Frmula geral: X
a
Y
b
onde a a carga de Y e
b a carga de X.
cido + base sal + gua
H
a
Y + X(OH)
b
X
a
Y
b
+ (ab)H
2
O
HCl + NaOH NaCl + H
2
O
HBr + Ca(OH)
2
CaBr
2
+ 2H
2
O
H
2
SO
4
+ KOH K
2
SO
4
+ 2H
2
O
H
2
SO
4
+ Fe(OH)
3
Fe
2
(SO
4
)
3
+ 6H
2
O

CLASSIFICAO DOS SAIS

Quanto ao n
o
de
elementos
Binrio: NaCl , Na
2
S, KI.
Ternrio: CaCO
3
, MgSO
4
, Fe(NO
3
)
3.

Quaternrio: Na
3
Fe(CN)
6
, KOCN.
Quanto presena
de oxignio
Sal oxigenado ou oxissal: NaNO
3
,
Li
3
PO
4
, CaSO
4.

Sal no oxigenado ou halide: CaCl
2
,
LiBr, K
3
Fe(CN)
6
.
Quanto ao grau de
hidratao
Sal anidro: CaCl
2
, NaCl, Na
2
SO
4
.
Sal hidratado: CaCl
2
.3H
2
O, NaCl.H
2
O,
Na
2
SO
4
.5H
2
O
Quanto natureza
dos ons
Sal neutro: NaNO
3
, CaCl
2
, MgSO
4
.
Sal cido: NaHCO
3
, K
2
HPO
4
, KHS.
Sal bsico: Al(OH)
2
Cl , FeOHSO
4
.
Sal duplo ou misto: CaBrCl , NaKSO
4
.
Please purchase PDF Split-Merge on www.verypdf.com to remove this watermark.
Qumica A Incluso para a Vida

Pr-Vestibular da UFSC
20
NOMENCLATURA DOS SAIS
O nome do sal deriva do nome do cido e da base que lhe
do origem, alterando apenas a terminao do nome do
cido:
cido...............................sal
drico...............................eto
ico...................................ato
oso...................................ito

Ex:
NaNO
3
Nitrato de sdio
CaCl
2
Cloreto de clcio
FeSO
4
Sulfato ferroso ou Sulfato de ferro II
NaHCO
3
Carbonato cido de sdio ou
Hifrogenocarbonato de sdio ou
Bicarbonato de sdio
Al(OH)
2
Cl Cloreto dibsico de alumnio
Dihidroxicloretode alumnio
KAl(SO
4
)
2
Sulfato de alumnio e potssio
CaBrCl Cloreto e brometo de clcio

Principais nions:
SO
4
2
: sulfato SiO
4
2
: silicato
CO
3
2
: carbonato BO
3
3
: borato
PO
4
3
: fosfato Cr
2
O
7
2
: dicromato
NO
3

: nitrato MnO
4

: permanganato

Exerccios de Sala #

1. (ACAFE) O carter bsico do bicarbonato de sdio
(NaHCO
3
), que torna eficaz no tratamento da acidez
estomacal, deve-se:
a) presena de hidrognio.
b) Ao fato do Na
+
ser proveniente de uma base forte e o
HCO
3

ser proveniente de um cido fraco.


c) Ao fato do NaHCO
3
ser um hidrogenossal,
d) total dissoluo do bicarbonato de sdio na gua.
e) Ao NaHCO
3
no reagir com o HCl, cido causador da
acidez estomacal.

Exerccios de Sala #

2. Clorato de potssio, sulfato de amnio, iodeto de sdio
e nitrato de brio so representados, respectivamente, pelas
seguintes frmulas:
a) KClO
4
, (NH
4
)
2
SO
4
, NaI e Ba(NO
2
)
2.

b) KClO
3
, (NH
4
)
2
SO
4
, NaI e Ba(NO
3
)
2.

c) KClO
3
, (NH
4
)
2
SO
3
, NaI e Ba(NO
3
)
2.

d) KClO
3
, (NH
4
)
2
SO
3
, NaIO
3
e Ba(NO
2
)
2.

e) KClO
3
, (NH
4
)
2
SO
3
, NaI e Ba(NO
2
)
2.


3. (ACAFE) O sulfato de alumnio utilizado no
tratamento de guas para o abastecimento pblico, no
processo denominado floculao. Na floculao, as
partculas coloidais presentes na gua so absorvidas na
superfcie do hidrxido de alumnio formado,
precipitando. A alternativa que mostra a frmula emprica
do sulfato de alumnio :
a) AlSO
4
c) Al
2
S
3
e) H
2
SO
4

b) Al
3
(SO
4
)
2
d) Al
2
(SO
4
)
3



Tarefa Mnima #

4. (ACAFE) A frmula do bicarbonato de clcio :
a) NaHCO
3
c) Ca(CO
3
)
2
e) Ca(HCO
3
)
2

b) CaCO
3
d) CaHCO
3


5. Bromato de potssio, sulfito de amnio, iodeto de sdio
e nitrito de brio so representados, respectivamente, pelas
seguintes frmulas:
a) KBrO
3
, (NH
4
)
2
SO
4
, NaI e Ba(NO
2
)
2.

b) KBrO
4
, (NH
4
)
2
SO
3
, NaI e Ba(NO
2
)
2.

c) KBrO
3
, (NH
4
)
2
SO
3
, NaI e Ba(NO
3
)
2.

d) KBrO
3
, (NH
4
)
2
SO
3
, NaIO
3
e Ba(NO
2
)
2.

e) KBrO
3
, (NH
4
)
2
SO
3
, NaI e Ba(NO
2
)
2.


6. (ACAFE) Escreva a equao balanceada para a
preparao do sulfato de magnsio a partir de solues de
um cido e uma base. A alternativa que mostra os
reagentes dessa reao :
a) H
2
SO
4(aq)
+ KOH
(aq)

b) H
2
S
(aq)
+ Mg(OH)
2(aq)

c) H
2
CO
3(aq)
+ Mn(OH)
2(aq)

d) H
2
SO
3(aq)
+ Ca(OH)
2(aq)

e) H
2
SO
4(aq)
+ Mg(OH)
2(aq)


UNIDADE 18

XIDOS

xidos so compostos binrios, onde o oxignio o
elemento mais eletronegativo. Podem ser inicos ou
moleculares, dependendo do elemento que estiver ligado
ao oxignio.

XIDOS MOLECULARES
So compostos que possuem um ametal ligado ao
oxignio. A nomenclatura procura descrever a proporo
de tomos existente de cada elemento em cada frmula:
Mono-, di-, tri-...xido de mono-, di-, tri-...elemento
Ex.: SO
2
: dixido de enxofre
SO
3
: trixido de enxofre
CO
2
: dixido de carbono
CO: monxido de carbono
NO: monxido de nitrognio (xido ntrico)
N
2
O: monxido de dinitrognio (xido nitroso)

Os xidos moleculares podem ser:

xidos cidos ou Anidridos: So aqueles que reagem
com gua formando um cido ou reagem com base
produzindo sal e gua. Maioria absoluta dos xidos
moleculares, os anidridos posem ter seus nomes da mesma
forma que os cidos, apenas trocando a palavra cido
pela palavra anidrido.
Ex: SO
3
+ H
2
O H
2
SO
4

SO
3
+ 2NaOH Na
2
SO
4
+ H
2
O

xidos Neutros ou Indiferentes: So os xidos que no
reagem nem com gua, nem com cidos e nem com bases.
So formados por ametais com Nox pequenos.
Ex: CO, NO, N
2
O, H
2
O.



Please purchase PDF Split-Merge on www.verypdf.com to remove this watermark.
Incluso para a Vida Qumica A


21
XIDOS INICOS
So compostos que possuem metal ligado ao oxignio. A
nomenclatura semelhante nomenclatura das bases
trocando a palavra hidrxido por xido.
Ex: Na
2
O: xido de sdio
CaO: xido de clcio
Fe
2
O
3
: xido de ferro III ou xido frrico
FeO: xido de ferro II ou xido ferroso

Os xidos inicos podem ser:

xidos Bsicos: So aqueles que reagem com gua
formando uma base ou reagem com cido produzindo sal e
gua. Estes xidos so formados por metais, especialmente
alcalinos e alcalinos terrosos, e possuem Nox menor ou
igual a +3.
Ex: Na
2
O + H
2
O 2 NaOH
Na
2
O + 2HCl 2NaCl + H
2
O

xidos cidos ou Anidridos: Alguns metais tambm
formam xidos cidos, desde que seu Nox seja igual a +6
ou +7.
Ex: Mn
2
O
7
+ H
2
O 2HMnO
4

Mn
2
O
7
+ 2KOH 2KMnO
4
+ H
2
O
xidos Anfteros: So aqueles que podem se comportar,
ora como xido cido, ora como xido bsico. So
formados por alguns metais com Nox entre +2 e +4.
Ex: ZnC
2
+ H
2
O
ZnO + 2NaOH Na
2
ZnO
2
+ H
2
O

xidos Duplos, Mistos ou Salinos: So formados por
dois outros xidos do mesmo elemento. So formados por
metais com Nox fracionrio de frmula geral X
3
O
4
.
Ex: Fe
3
O
4
, Pb
3
O
4.


Perxidos: So xidos que contm o on perxido (O
2
)
2-
.
Nestes compostos o Nox do oxignio ser -1. Eles reagem
com a gua ou com cidos em meio aquoso produzindo
gua oxigenada (H
2
O
2
). Estes xidos so formados por
metais alcalinos e alcalino-terrosos e pelo hidrognio e em
seu nome utiliza-se a palavra perxido.
Ex: Na
2
O
2
+ 2H
2
O 2NaOH + H
2
O
2

Na
2
O
2
+ 2H
2
SO
4
2Na
2
SO
4
+ H
2
O
2


Superxidos ou Polixidos: So xidos que contm o on
perxido (O
4
)
2-
. Nestes compostos o Nox do oxignio ser
. Eles reagem com a gua ou com cidos em meio
aquoso produzindo gua oxigenada (H
2
O
2
) e oxignio
gasoso (O
2
). Estes xidos so formados por metais
alcalinos e alcalino-terrosos e seu nome utiliza a palavra
superxido.
Ex: Na
2
O
4
+ 2H
2
O 2NaOH + H
2
O
2
+ O
2

Na
2
O
4
+ 2H
2
SO
4
2Na
2
SO
4
+ H
2
O
2
+ O
2


Exerccios de Sala #

1. A alternativa que apresenta, respectivamente, xido
cido, xido bsico, xido neutro e xido duplo, :
a) CO; CaO; SO
3
; Na
2
O
4.

b) N
2
O
5
; BaO; NO; Pb
3
O
4.

c) CO
2
; Al
2
O
3
; Fe
3
O
4
; Cl
2
O
d) N
2
O; MgO; CO
2;
Mn
3
O
4.

e) SO
2
; K
2
O; CO; K
2
O
2.




2. Dos compostos abaixo o xido :
a) OF
2
c) HClO
2
e) O
2

b) O
2
F
2
d) H
2
O


Tarefa Mnima #

3. (ACAFE) A alternativa que apresenta os anidridos
correspondentes aos cidos H
2
SO
3
, H
2
CO
3
, H
2
SiO
3
,
HClO
4
, :
a) SO
2
, CO
2
, SiO
2
, ClO
4

b) SO
3
, CO
2
, SiO
2
, ClO
4

c) SO
3
, CO
2
, SiO
2
, Cl
2
O
5

d) SO
2
, CO, SiO
2
, Cl
2
O
3

e) SO
2
, CO
2
, SiO
2
, Cl
2
O
7


4. (UFSC) Os automveis modernos possuem os
chamados conversores catalticos ou catalisadores
que, neste caso, so capazes de transformar uma
substncia poluente em outra no poluente. Observe a
ilustrao e assinale a(s) afirmao(es) correta(s).

01. O CO produto da queima incompleta do
combustvel.
02. O NO
x
sofre reduo do nitrognio que passa a N
2
.
04. O CO sofre oxidao e passa a CO
2
.
08. O combustvel no totalmente queimado no motor
deve ser convertido em CO
2
.
16. O CO
2
um xido neutro e no reage com a gua.
32. O NO
x
que representa diversos xidos de nitrognio,
entre eles, o N
2
O e NO
2
, pode produzir o efeito de
chuva cida quando em contato com o ar
atmosfrico.
5. (ACAFE) Um dos componentes da chuva cida, o cido
sulfuroso (H
2
SO
3
), formado pela reao do dixido de
enxofre, SO
2
, com a umidade na atmosfera. A alternativa
que melhor define a procedncia do SO
2
, responsvel pelas
chuvas cidas, :
a) A origem do SO
2
est relacionada com o tratamento da
gua para caldeiras nas indstrias.
b) O SO
2

provm da combusto do enxofre presente nos
combustveis.
c) O SO
2
provm da decomposio da matria orgnica
nos mangues.
d) O SO
2
forma-se na reao do CO
2
presente nas guas.
e) O SO
2
tem sua origem na Antrtica.

6. A nica alternativa incorreta :
a) Fe
3
O
4
um xido salino.
b) H
2
O um xido neutro.
c) CaO um xido anftero.
d) Cl
2
O
7
um xido cido.
e) H
2
O
2
um perxido.






Please purchase PDF Split-Merge on www.verypdf.com to remove this watermark.
Qumica A Incluso para a Vida

Pr-Vestibular da UFSC
22
F H
| |
F B :N H
| |
F H
cido Base
Tarefa Complementar #

7. (UFSC) Escolha a(s) proposio(es), cujo nome est
corretamente associado frmula qumica.
01. CO
2
, monxido de carbono
02. NaCl, cloreto de sdio
04. KOH, xido de potssio
08. Ca(OH)
2
, hidrxido de clcio
16. HCl, cido cloroso
32. Al
2
O
3
, xido de alumnio
64. MgBr
2
, brometo de mangans

8. (UFSC) Selecione, entre as opes abaixo, aquelas que
apresentam somente sais e as que apresentam somente
xidos e some os nmeros.
01. H
2
O, NaCl, HCl
02. KF, CaCl
2
, HCN
04. HNO
3
, NaOH, BaCO
3

08. CaCO
3
, AgBr, NaCl
16. H
2
SO
4
, KNO
3
, PbS
32. FeO, CuO, CO
2

9. (UFSC) Assinale a(s) seqncia(s) que apresenta(m)
respectivamente, um cido, uma base, um sal e um xido.
01. AlCl
3
, H
2
O, NaCl, Fe
2
O
3
02. P
2
O
5
, KNO
3
, H
2
SO
4
, HCl
04. HClO
3
, Al(OH)
3
, (NH
4
)
2
SO
4
, Na
2
O
08. Al
2
(SO
4
)
3
, NAOH, H
2
SO
4
, H
2
O
16. KNO
3
, BaO, HClO
4
, Al
2
O
3

32. HNO
3,
LiOH, Na
2
S, Li
2
O.
64. HCl

, Ba(OH)
2
, Al
2
O
3
, H
2
S

10. (UNIOESTE) Sobre as funes inorgnicas, assinale o
que for correto:
01. A substncia HClO
4
um tricido, ternrio e hidrcido.
02. A substncia NaHCO
3
um sal oxigenado, quaternrio
e denominado de bicarbonato de sdio.
04. A frmula do sulfato de ferro (III) Fe
2
(SO
4
)
3
.
08. A substncia NaOH uma base fraca.
16. Os sais so, geralmente, slidos que fundem e fervem a
baixas temperaturas.
32. Os xidos so compostos binrios onde o oxignio o
elemento mais eletronegativo.

11. (UFSC) O cobre encontrado livre, na natureza, na
forma de pepitas. Conhecido desde a antiguidade, foi o
primeiro metal utilizado pelo ser humano e tornou-se o
substituto ideal da pedra na fabricao de vrios utenslios,
tanto puro como na forma de ligas. um metal de cor
avermelhada, dctil, malevel e tenaz. tambm excelente
condutor de eletricidade, sendo empregado principalmente
em fios e cabos eltricos. Considerando o enunciado e as
propriedades do cobre, assinale a(s) proposio(es)
correta(s).
01. Devido a sua baixa eletropositividade, de se esperar
que no reaja espontaneamente com o hidrognio.
02. O nmero de eltrons contido na espcie qumica Cu
2+

igual a 31.
04. O cobre conduz corrente eltrica na fase slida ou em
soluo aquosa.
08. As frmulas possveis para os xidos desse elemento
so Cu
2
O
3
e CuO.
16. Os hidrxidos desse metal so solveis em gua.
32. O bronze uma liga metlica, obtida pela combinao
de cobre e estanho, formando uma soluo slida.

UNIDADE 19

OUTROS CONCEITOS DE CIDO E BASE

BRNSTED-LOWRY:
- cido toda espcie qumica (molcula ou on) capaz
de doar prtons (H
+
).
- Base toda espcie qumica (molcula ou on) capaz
de receber prtons (H
+
).

Ex: HCl + H
2
O Cl

+ H
3
O
+


Esta teoria conhecida como teoria protnica por
ocorrer com transferncia de prtons de uma molcula
para outra.
Na reao acima verificamos a formao de dois
cidos e duas bases, que se diferenciam entre si por
nmeros de H
+
. Por esse motivo so conhecidos como par
(cido-base) conjugado.
Par Conjugado: So aqueles que diferem entre si quanto
ao n
o
de hidrognio (H
+
). O para conjugado sempre
formado por um componente forte e um fraco, assim
temos:
cido forte Base fraca
cido fraco Base forte

Substncias Anfteras: So aquelas que se comportam,
ora como cido, ora como base, como a gua.

HCl + H
2
O Cl

+ H
3
O
+

NH
3
+ H
2
O OH

+ NH
4
+


CIDOS E BASES DE LEWIS
- cido toda espcie qumica (molcula ou on) capaz
de receber eltrons.
- Base toda espcie qumica (molcula ou on) capaz
de ceder eltrons.

Ex:






Obs: Tanto para Lewis quanto para Brnsted-Lowry, a
espcie positiva provavelmente ser o cido, e a
espcie negativa, a base.

Exerccios de Sala#

1. (UFSC) Nas reaes:
I - HCl + H
2
O H
3
O
+
+ Cl


II - NH
3
+ H
2
O NH
4
+
+ OH
-

De acordo com a conceituao de Brnsted e Lowry, a
gua :

01. um cido na reao I e uma base na reao II.
02. uma doadora de prton na reao II.
04. a base conjugada do cido H
3
O
+
na reao I.
08. receptora de prton na reao I.



Please purchase PDF Split-Merge on www.verypdf.com to remove this watermark.
Incluso para a Vida Qumica A


23
2. (ACAFE) Analise a estrutura de Lewis dos reagentes e
dos produtos de cada uma das equaes abaixo e
identifique o cido de Lewis em cada uma.
I - O
2-
+ SO
3
SO
4
2-

II - AlCl
3
+ Cl
-
AlCl
4
-

III - B(OH)
3
+ OH
-
B(OH)
4
-

IV - CO
2
+ OH
-
HCO
3
-

A alternativa que contm, em seqncia, os cidos de
Lewis identificados, :

a) O
2-
- AlCl
3
- B(OH)
3
- CO
2
b) SO
3
- Cl
-
- B(OH)
3
- CO
2

c) O
2-
- Cl
-
- OH
-
- CO
2

d) O
2-
- Cl
-
- B(OH)
3
- CO
2

e) SO
3
- AlCl
3
- B(OH)
3
- CO
2


UNIDADE 20

REAES QUMICAS

So transformaes da matria que ocorrem com formao
de novas substncias. As molculas ou retculos so
desmontados e seus tomos reagrupados de maneira
diferente.
Equao qumica a representao grfica (por
escrito) de uma reao.

Ex.: 2 NaCl + CaBr
2
CaCl
2
+ 2 NaBr

Reagentes Produtos
Coeficiente ndice

As reaes podem ser classificadas em:
Sntese (Adio)
Anlise (Decomposio)
Deslocamento (Simples Troca)
Dupla Troca
Endotrmica
Exotrmica
Reversvel
Irreversvel
Oxirreduo (Redox)

- Reao de Sntese ou Adio: (A+B AB)
o tipo de reao onde duas ou mais substncias
reagem formando um nico produto. Pode ser de doi tipos:
- Sntese Total: Quando todos os reagentes so
substncias simples.
Ex.: H
2
+ O
2
H
2
O
- Sntese Parcial: Quando houver entre os reagentes
pelo menos uma substncia composta.
Ex.: H
2
O + SO
2
H
2
SO
3


- Reao de Anlise ou Decomposio: (AB A+B)
o tipo de reao onde um nico reagente se divide
em dois ou mais produtos. Dependendo do fator que
provoca a decomposio podemos classificar a reao em:
- Fotlise: Decomposio provocada pela luz.
Ex.: H
2
O
2

luz
H
2
+ O
2

- Pirlise: Decomposio provocada pelo calor
Ex.: CaCO
3

A
CaO + CO
2

- Eletrlise: Decomposio provocada pela eletricidade
Ex.: H
2
O
de eletricida
H
2
+ O
2

- Reao de Simples Troca ou Deslocamento
(AB+C AC+B ou AB+C A+CB)
o tipo de reao onde uma substncia simples reage
com uma substncia composta deslocando o ction ou o
nion, dependendo da sua natureza, formando uma nova
substncia.
Para que este tipo de reao ocorra necessrio que o
elemento da substncia simples seja mais reativo que o do
tomo a ser deslocado (mais eletropositivo se for um metal
e mais eletronegativo se for um ametal).

Srie de Reatividade dos Metais:

Ordem Decrescente
Cs > Rb > K > Na > Ba > Li > Sr > Ca > Mg > Al > Mn >
Zn > Cr > Fe > Co > Ni > Sn > Pb > H > Sb > Bi > Cu >
Hg > Ag > Pd > Pt > Au
Ou, simplificadamente: 1A > 2A > M* > H > MN
Al + MgSO
4
no ocorre
Zn + FeSO
4
ZnSO
4
+ Fe

Srie de Reatividade dos Ametais:

Ordem Decrescente
F > O > N > Cl > Br > I > S > C > P > H
Ex.: Zn + 2HCl ZnCl
2
+ H
2

Cu + MgCl
2
no ocorre
Cl
2
+ 2NaBr 2NaBr + Br
2

I
2
+ NaF no ocorre

- Reao de dupla troca:
o tipo de reao onde duas substncias compostas
reagem trocando o ction (e o nion) de uma com o da
outra.
Estas reaes no obedecem escala de reatividade e,
geralmente, ocorrem em soluo aquosa. Para que ocorra,
um dos produtos, em relao aos reagentes, deve ser:
a) Mais voltil;
b) Menos solvel;
c) Mais fraco (no caso de eletrlitos).
Um caso particular de reao de dupla troca a reao
de salificao ou neutralizao, onde um cido reage com
uma base formando sal e gua.

Ex.: H
2
SO
4
+ KI HI + K
2
SO
4

AgNO
3
+ NaCl AgCl + NaNO
3


Exerccios de Sala #

1. Relacione a reao com o tipo:
1 Simples troca ( ) 2AgCl 2Ag + Cl
2

2 Dupla troca ( ) H
2
S + Ca(OH)
2
CaS + 2H
2
O
3 Anlise ( ) 2AgBr + Cl
2
2AgCl + Br
2

4 Sntese ( ) N
2
+ 3H
2
2NH
3

5 Neutralizao ( ) 2HCl + FeS FeCl
2
+ H
2
S
Assinale a opo que fornece a relao correta de cima
para baixo:
a) 4; 5; 1; 3; 2.
b) 3; 5; 2; 4; 1.
c) 5; 3; 1; 4; 2.
d) 3; 1; 5; 4; 2.
e) 3; 5; 1; 4; 2.

2. (UFSC) Assinale a(s) reao(es) de dupla troca:
01. NaOH + HBr NaBr + H
2
O
02. 2Al + 3H
2
SO
4
Al
2
(SO
4
)
3
+ 3H
2

Please purchase PDF Split-Merge on www.verypdf.com to remove this watermark.
Qumica A Incluso para a Vida

Pr-Vestibular da UFSC
24
04. Na
2
S + FeCl FeS + 2NaCl
08. 3NH
4
OH + Al(NO
3
)
3
3NH
4
NO
3
+ Al(OH)
3

16. C + H
2
O CO + H
2

32. Cl
2
+ 2KI 2KCl + I
2

64. CuCl
2
+ Zn ZnCl
2
+ Cu

Tarefa Mnima #

3. (UDESC) Na formao de hidrxido de alumnio,
representada pela equao abaixo, usado no tratamento de
gua, ocorre uma reao de:
Al
2
(SO
4
)
3
+ 6 NaOH 2Al(OH)
3
+ 3Na
2
SO
4

a) Decomposio.
b) Sntese.
c) Simples troca.
d) Dupla troca.
e) Oxirreduo.

4. H
2
CO
3
H
2
+ CO
2
uma reao do tipo:
a) dupla troca
b) simples troca
c) sntese ou combinao
d) anlise ou decomposio
e) n.d.a.

5. 2H
2
+ O
2
2H
2
O uma reao de:
a) anlise
b) calcinao
c) dupla troca
d) sntese
e) simples troca

6. Na reao CaCO
3
CaO + CO
2
o carbonato de
clcio, sob a ao do calor, se decompe em xido de
clcio e gs carbnico. Tem-se uma reao de:
a) Anlise
b) Sntese
c) Dupla troca
d) Substituio
e) n.d.a.

Tarefa Complementar #

7. (UFSC) Assinale qual(is) das substncias abaixo
reage(m) com a gua lquida, produzindo hidrognio
gasoso (H
2
).
01. Sdio metlico.
02. Hidrxido de potssio.
04. cido clordrico.
08. Potssio metlico.
16. Cloro gasoso.
32. Gs carbnico.

8. (UFSC) O carbonato de clcio praticamente insolvel
em gua, mas dissolve-se de modo aprecivel em gua
com o dixido de carbono existente da atmosfera. Essa
uma das causas das formaes de carbonato de clcio
conhecidas como ESTALACTITES (superiores) e
ESTALAGMITES (inferiores).
Assinale a(s) afirmativa(s) correta(s).
01. O aumento da solubilidade do carbonato pode ser
explicado pela reao:
CaCO
3(S)
+ H
2
O
(l)
+ CO
2(g)
Ca
2+
(aq)
+ HCO
3

(aq)
.
02. O desprendimento do CO
2
do meio aquoso provoca a
precipitao do carbonato de clcio.
04. Estalactites e estalagmites so resistentes ao de
cidos.
08. O carbonato de clcio pode ser formado a partir da cal
extinta, Ca(OH)
2
, em reao com o gs carbnico.
16. A decomposio trmica do carbonato de clcio,
tambm encontrado no mrmore, nas conchas e no
calcrio, ir produzir a cal viva (CaO) e gs
carbnico.
32. O carbonato de clcio reage com cidos formando sal,
gua e CO
2
.

UNIDADE 21

REAES DE OXIRREDUO

So as reaes nas quais h tranferncia de eltrons. Por
causa dessa tranferncia de eltrons, h variao do
nmero de oxidao.
Oxidao: Perde eltrons = Aumenta o NOX
Reduo: Recebe eltrons = Diminui o NOX
Agente Oxidante a substncia que possui o elemento
que sofreu reduo.
Agente Redutor a substncia que possui o elemento
que sofreu oxidao.
Oxidante provoca oxidao porque sofre reduo
Redutor provoca reduo porque sofre oxidao

Exerccios de Sala #

1. (UFSC) Na reao representada abaixo, diga qual o
elemento que sofre oxidao, qual o elemento que sofre
reduo, qual o agente oxidante e qual o agente redutor
e assinale as opes corretas:

HI + H
2
SO
4
I
2
+ H
2
O + H
2
S

01. Sofre Oxidao: I
02. Agente Oxidante: H
2
SO
4

04. Sofre Reduo: S
08. Agente redutor: H
2
SO
4
16. Agente Redutor: HI

2. (ACAFE) Observe a reao:
4Fe + O
2

A
2Fe
2
O
3

a) O ferro sofre reduo
b) O gs oxignio o agente oxidante
c) O oxignio sofre oxidao
d) O ferro o agente oxidante
e) n.d.a

Tarefa Mnima #

3. (ACAFE) A equao abaixo representa o processo de
uma reduo do ferro nos altos fornos.
Fe
2
O
3
+ 3CO
A
2Fe + 3CO
2

A alternativa que mostra o agente redutor nesse processo :
a) CO
2
b) Fe
2
O
3
c) Fe d) C e) CO



Please purchase PDF Split-Merge on www.verypdf.com to remove this watermark.
Incluso para a Vida Qumica A


25
4. (UFSC) Sobre o fenmeno de oxirreduo, correto
afirmar que:
01. Oxidante a espcie qumica que recebe eltrons.
02. A oxidao o fenmeno de perda de eltrons.
04. Na reao Ag
+
+ e
-
Ag, o on prata reduz-se.
08. A reduo o fenmeno de ganho de eltrons.
16. O redutor a espcie qumica que recebe eltrons.
32. Na reao Ni Ni
2+
, 2e
-
ocorre oxidao do nquel.

Tarefa Complementar #

5. Dada a reao abaixo, podemos afirmar que:
H
2
C
2
O
4
+ KMnO
4
CO
2
+ MnO + K
2
O + H
2
O
01. O elemento que se oxida o Mn.
02. O agente redutor o H
2
C
2
O
4
.
04. O agente oxidante o KMnO
4
.
08. O elemento que se reduz o Mn.
16. O agente oxidante o H
2
C
2
O
4
.
32. O agente redutor o KMnO
4
.

UNIDADE 22

BALANCEAMENTO DE EQUAES

Balancear uma equao qumica significa atribuir
coeficientes s substncias de tal maneira que a quantidade
de tomos de cada elemento seja igual no reagente e no
produto. Pode ser feito de duas maneiras:

- Mtodo das Tentativas
- Mtodo da Oxirreduo

Mtodo das Tentativas:
Indicado nas equaes mais simples.
1. Selecione os elementos que s aparecem uma vez entre
os reagentes e uma vez entre os produtos;
2. Escolha aquele que aparece em maior quantidade
(maiores ndices);
3. Transforme o ndice do produto em coeficiente do
reagente, e vice-versa.
4. Acerte as quantidades dos outros elementos, dando
preferncia queles que s tenham uma substncia por
acertar.
Ex.: Ca + AlCl
3
CaCl
2
+ Al

Ca + 2 AlCl
3
3 CaCl
2
+ Al
3 Ca + 2 AlCl
3
3 CaCl
2
+ 2 Al

Mn
3
O
4
+ Al Al
2
O
3
+ Mn

Mtodo da Oxirreduo (Redox):

Indicado em equaes mais complexas que tenham algum
elemento variando o Nox.
1. Determinar o Nox de todos os elementos;
2. Achar os elementos que sofreram variao de Nox;
3. Calcular a variao (A) de cada um;
4. Escolher a substncia que tem este elemento em maior
quantidade (maior ndice), desde que seu nox no repita;
5. Calcular o A
T
(A x ndice);
6. Se for possvel, simplifique os valores encontrados;
7. Inverta os valores encontrados para um como coeficiente
do outro;
8. Continue o balanceamento por tentativas.

Atividade interativa!

HI + H
2
SO
4
I
2
+ H
2
O + H
2
S

Exerccios de Sala #

Da questo 1 a 18, acerte os coeficientes das equaes
qumicas abaixo e some os valores encontrados:

1. N
2
+ H
2
NH
3


2. Cr + O
2
Cr
2
O
3


3. P + O
2
P
2
O
5


4. Fe + O
2

A
Fe
2
O
3


5. Ca + AlCl
3
CaCl
2
+ Al

6. Mn
3
O
4
+ Al Al
2
O
3
+ Mn

Tarefa Mnima #

7. Al + HCl AlCl
3
+ H
2


8. NaNO
3
NaNO
2
+ O
2


9. Al
2
(CO
3
)
3
CO
2
+ Al
2
O
3


10. Cu(OH)
2
+ H
4
P
2
O
7
H
2
O + Cu
2
P
2
O
7


11. Al(OH)
3
+ H
4
SiO
4
Al
4
(SiO
4
)
3
+ H
2
O

12. K
2
Cr
2
O
7
+ KOH K
2
CrO
4
+ H
2
O

13. NH
3
+ O
2
H
2
O + N
2


14. C
3
H
8
+ O
2
CO
2
+ H
2
O

15. HIO
3
+ HI I
2
+ H
2
O

16. HI + H
2
SO
4
I
2
+ H
2
O + H
2
S

17. NaOH + Al + H
2
O NaAlO
2
+ H
2


18. MnO
2
+ HCl MnCl
2
+ H
2
O + Cl
2


Tarefa Complementar #

19. (UFSC) Dada a reao qumica expressa pela
equao: a H
3
PO
4
+ b Na
2
O c Na
3
PO
4
+ d H
2
O e,
sendo: (A) + (B) (C) + (D), respectivamente, os
reagentes e produtos, assinale a(s) proposio(es)
correta(s). (Obs: Os coeficientes a, b, c e d devem estar
na sua proporo mnima de nmeros inteiros).
01. A dissociao inica do cido (A) feita em trs
etapas.
02. O Na
3
PO
4
um sal neutro.
Please purchase PDF Split-Merge on www.verypdf.com to remove this watermark.
Qumica A Incluso para a Vida

Pr-Vestibular da UFSC
26
04. O coeficiente do reagente (B) 3.
08. A soma de todos os coeficientes da equao (a+b+c+d)
10.
16. As substncias reagentes (A) e (B) so,
respectivamente, um cido triprotnico e uma
monobase.
32. O produto (D) um xido cido.
64. A soma dos coeficientes dos reagentes da equao
(a+b) 4.

20. (UFSC) Na seguinte equao qumica no balanceada
Fe + CuSO
4
Fe
2
(SO
4
)
3
+ Cu, assinale no carto-resposta
a soma da(s) proposio(es) verdadeira(s).
01. Temos uma reao de oxirreduo.
02. O nmero de oxidao do cobre no CuSO
4
+2.
04. O tomo de ferro perde 2 eltrons.
08. Houve oxidao do ferro.
16. O cobre sofre oxidao.
32. Aps balanceamento, a soma dos menores nmeros
inteiros possveis para os coeficientes 9.
64. O ferro o agente oxidante.

UNIDADE 23

CLCULOS QUMICOS

CONCEITOS BSICOS:

Unidade de Massa Atmica (u.m.a.):
Para medir uma grandeza qualquer utilizamos uma
unidade padro. Por exemplo: para medir comprimento,
normalmente utiliza-se o metro (m); para medir massa, as
unidades mais usadas so o grama (g) e o quilograma (kg).
Para determinar a massa de um tomo foi criada a
unidade de massa atmica (u.m.a. ou ), que equivale a
1/12 da massa do istopo 12 do carbono (A=12, Z=6, 6
prtons e 6 nutrons), ou seja, aproximadamente a mdia
entre a massa de um prton e a de um nutron.

Massa Atmica de um Elemento Qumico:
a mdia das massas de todos os tomos de um
determinado elemento, dadas as propores em que os
diversos istopos se encontram na natureza.
Ex: Os istopos do cloro so:
35
17
Cl (75%) e
37
17
Cl (25%)
M
Cl
= 35
-
0,75 + 37
-
0,25 = 35,5

tomo-grama (at.g):
a massa atmica expressa em gramas.

Massa Molecular ou Peso Molecular:
a soma das massas de todos os tomos que
compe uma determinada substncia.

Molcula-grama (mol):
a massa molecular expressa em gramas.

Nmero de Mols (n):
a relao entre a massa da amostra da substncia
e a sua molcula-grama.
mol
m
n =
Nmero de Avogadro:
o nmero de tomos existentes em um tomo-
grama de um elemento ou o nmero de molculas
existentes em uma molcula-grama de uma substncia e
igual a 6,02x10
23
.

MOL
Atualmente o termo mol no mais utilizado apenas para
molcula-grama, e sim para uma determinada quantidade
(o nmero de Avogadro) de qualquer coisa. Ex.:
1 dzia = 12
1 centena = 100
1 mol = 6,02x10
23
.

Volume Molar
o volume ocupado por um mol de qualquer substncia
gasosa. Nas CNTP, o volume molar de qualquer
substncia igual a 22,4L.

CNTP (Condies Normais de Temperatura e Presso):
Temperatura = 0
o
C = 273K
Presso = 1atm = 760mmHg

Se a substncia se encontrar em outras condies de
temperatura e presso, devemos recorrer Equao de
Clayperon. PV = nRT .

Exerccios de Sala #

1. Quantos mols h em 360g de glicose (C
6
H
12
O
6
)?

2. Quantos gramas h em 0,5 mol de K
2
Cr
2
O
7
?

3. Qual o volume, em litros, ocupado por 880g de
oxignio (CO
2
) nas CNTP?

Tarefa Mnima #

4. Quantos mols h em 360g de grafite (C
(s)
)?

5. Quantos gramas h em 4,5 mols de H
2
O?

6. Qual o volume ocupado por 640g de oxignio (O
2
) nas
CNTP?
a) 44,8. b) 224 c) 22,4 d) 448 e) n. d. a.

Tarefa Complementar #

7. Quantos mols h em 360g de gua?

8. Quantos gramas h em 7 mols de H
2
SO
4
?

9. Qual , aproximadamente, a massa em gramas de um
tomo de carbono?
a) 2x10
23
.
b) 2x10
23
.
c) 6x10
23
.
d) 6x10
23
.
e) n.d.a.

10. Qual o volume, em litros, de um cilindro que
armazena 800g de Argnio (Ar) a uma presso de 4,1atm
temperatura de 27
o
C?
Dado: R=0,082 atm.L.K
-1
.mol
-1




Please purchase PDF Split-Merge on www.verypdf.com to remove this watermark.
Incluso para a Vida Qumica A


27
UNIDADE 24

CLCULO ESTEQUIOMTRICO

LEIS DAS REAES QUMICAS
As bases para os clculos de substncias que participam de
uma reao qumica surgiram no sculo XVIII, com as leis
de Lavoisier e de Proust:
- Lei de Lavoisier (conservao das massas): Numa
reao qumica, em um sistema fechado, a massa dos
reagentes igual dos produtos.
- Lei de Proust (lei das propores constantes): As
substncias participantes de uma reao qumica esto
em uma proporo, em massa, constante e definida.

Clculo Estequiomtrico
o clculo das quantidades de reagentes e/ou produtos das
reaes qumicas (massas, volumes, nmero de mols e o
nmero de molculas), feito com base nas leis das reaes
e executado, em geral, com o auxlio das equaes
qumicas correspondentes. Esse tipo de clculo segue, em
geral, as seguintes regras:

1- Escrever a equao qumica balanceada;
2- Estabelecer uma regra de trs entre o dado e a pergunta
do problema, que poder ser em massa, n
o
de mol,
volume, etc., conforme o problema exigir.

Ex.: Qual a massa de Na
2
SO
4
formada a partir da
neutralizao de 16g de NaOH?

2NaOH + H
2
SO
4
Na
2
SO
4
+ 2H
2
O
2mol 1mol
2x40g/mol 1x142g/mol

80g 142g

16g x = 28,4g .
Rendimento:
definido pela relao entre a massa efetivamente
formada de um determinado produto e a massa prevista
pelos clculos. dado por:
% 100 x
m
m
r
esperada
obtida
=



Ex.: Sabendo que, na reao descrita no exerccio anterior,
verificou-se a formao de 20g de Na
2
SO
4
, qual foi o
rendimento da reao?
% 100 x
m
m
r
esperada
obtida
= , sendo m
real
= 20g e m
esperada
= 28,4g
% 100
4 , 28
20
x r = r = 70,4%

Ou, por regra de trs:
28,4g 100%
20,0g x = 70,4%



Reagente Limitante e Reagente em Excesso:
Quando for dada a massa de dois (ou mais) reagentes,
deve-se descobrir qual dos dois est (se tiver um deles) em
excesso. O que estiver em quantidade proporcionalmente
menor ser chamado de reagente limitante e todos os
clculos devero ser feitos a partir dele. Pode-se, inclusive
calcular a quantidade em excesso do outro reagente.

Ex.: Qual o reagente limitante e qual a massa em excesso
do outro reagente quando reagem 49g de H
2
SO
4
com
45g de NaOH?

2NaOH + H
2
SO
4
Na
2
SO
4
+ 2H
2
O
2mol 1mol
80g 98g

45g 49g
x = 40g (massa utilizada)

Reagente limitante: H
2
SO
4

Massa em excesso: 5g de NaOH

Exerccios de Sala #

1. A equao abaixo descreve uma reao na qual se
misturou 72g de NaH com 72g de H
2
O, obtendo como
resultado a formao de 90g de NaOH.
NaH + H
2
O NaOH + H
2

Assinale as opes corretas:
01. O NaH o reagente limitante.
02. O excesso em gramas de gua 18.
04. A reao apresenta um rendimento de 75%.
08. A gua o reagente limitante.

Tarefa Mnima #

2. (UFPR) Que quantidade de cal (xido de clcio) ser
produzida por calcinao de uma tonelada de calcrio puro
(carbonato de clcio)?
Ca = 40; C = 12; O = 16.
Dados: CaCO
3

A
CaO + CO
2

a) 840 kg c) 240 kg e) 560 kg
b) 790 kg d) 700 kg

3. Quantos gramas de gua devem ser decompostos para
produzirem 11,2 litros de oxignio, medidos nas CNTP?

4. Dada a reao 2H
2
+ O
2
2H
2
O, se reagirmos 3
gramas de hidrognio e 30 gramas de oxignio, qual a
massa em excesso?

5. Qual o rendimento aproximado de uma reao que
forma 168g de CaO a partir de 400g de CaCO
3
?
CaCO
3
CaO + CO
2


a) 42%. c) 75% e) 100%.
b) 56% d) 84%.

Tarefa Complementar #

6. Sabendo que o ouro 18 quilates uma mistura que
contm 75% de ouro, 12,5% de prata e 12,5% de cobre,
diga quantos gramas de ouro h em 80g de ouro 18?
Please purchase PDF Split-Merge on www.verypdf.com to remove this watermark.
Qumica A Incluso para a Vida

Pr-Vestibular da UFSC
28

7. Qual a massa de CaO obtida a partir da pirlise de 300g
de CaCO
3
.
CaCO
3
CaO + CO
2

a) 56. c) 168 e) n.d.a.
b)112 d) 224.

8. A combusto completa do metanol pode ser
representada pela equao balanceada:
2 CH
3
OH + 3 O
2
2 CO
2
+ 4 H
2
O
Quando se utiliza 320g de metanol (CH
3
OH), so
formados:
01. 44g de CO
2
.
02. 440g de CO
2
.
04. 448L de CO
2
nas CNTP.
08. 224L de CO
2
nas CNTP.
16. 44,8L de CO
2
nas CNTP.
32. 10 mol de CO
2
.
64. 2 mol de CO
2
.

9. A respeito da equao abaixo, calcule a massa de Fe
formada a partir de 464g de Fe
3
O
4
.
Fe
3
O
4
+ H
2
H
2
O + Fe

10. Qual a massa de cido actico (CH
3
COOH) obtida a
partir da oxidao de 230g de etanol (C
2
H
5
OH).
C
2
H
5
OH + O
2
CH
3
COOH + H
2
O

11. Numa reao qumica, em um sistema fechado, a
massa dos reagentes igual dos produtos. Esta
afirmao atribuda a:
a) Dalton. c) Lavoisier. e) Thomson.
b) Proust. d) Rutheford.

12. Assinale a(s) alternativa(s) correta(s):
01. Composio centesimal indica a porcentagem em
massa de cada elemento qumico presente na
composio da substncia.
02. Frmula mnima indica a quantidade de tomos de
cada elemento qumico presente em uma molcula.
04. Frmula molecular indica a proporo mnima entre
os elementos constituintes de uma substncia.
08. Nenhuma das afirmaes anteriores est correta.

13. Qual o rendimento aproximado de uma reao que
forma 42g de CaO a partir de 100g de CaCO
3
?
a) 42%. c) 75%. e) 100%.
b) 56%. d) 84%.

14. Assinale a(s) alternativa(s) correta(s):
01. Para se fazer clculos sobre uma reao qumica,
necessrio que a sua equao qumica esteja
devidamente balanceada.
02. O volume molar de uma substncia gasosa qualquer
depende apenas da presso e da temperatura, qualquer
que seja a substncia.
04. Massa molecular a soma das massas de todos os
tomos que compem esta substncia.
08. O nmero de Avogadro (6,02x10
23
) indica a
quantidade de molculas existentes em um mol de
qualquer substncia.

Please purchase PDF Split-Merge on www.verypdf.com to remove this watermark.
Incluso para a Vida Qumica B

Pr-Vestibular da UFSC 1
UNIDADE 1

SOLUES

qualquer mistura homognea onde um componente
denominado soluto e o outro solvente, este ltimo
representado geralmente pela gua.

Tipos de Soluo
De acordo com a quantidade de soluto, podemos
classificar as solues em:

Saturadas: Tm a concentrao igual ao limite de
saturao.
Insaturadas: Tm a concentrao menor que o limite de
saturao.
Supersaturadas: Tm a concentrao maior que o limite
de saturao.

Limite de Saturao: Quantidade mxima de soluto que
pode ser dissolvida em uma certa quantidade de solvente.

SOLUBILIDADE DAS SOLUES

Solubilidade a mxima quantidade possvel de um
determinado soluto que pode ser dissolvida em uma certa
quantidade padro de solvente a uma dada temperatura.

Influncia da Temperatura na Solubilidade: A maioria
das substncias tem sua solubilidade aumentada com a
temperatura.

Curvas de Solubilidade



Podemos observar que alguns sais tm sua solubilidade
aumentada significativamente com o aumento da
temperatura, j em outros a temperatura tem pouca
influncia.

Exerccios de Sala #

1. O grfico acima fornece as curvas de solubilidade de
diversas substncias em funo da temperatura e de uma
mesma quantidade de solvente.
Com base neste grfico, a(s) concluso(es) correta(s)
(so):
01. A 10C a substncia mais solvel o sulfato de crio.
02. A 10C a substncia menos solvel a KNO
3.

04. A aproximadamente 20C, KNO
3
e Ce
2
(SO
4
)
3
tm a
mesma solubilidade.
08. A 20C, KNO
3
mais solvel que o NaCl.
16. A ordem crescente de solubilidade destas substncias
, a 40C, Ce
2
(SO
4
)
3
, NaCl, KNO
3
, H
4
Cl, NaNO
3
.
32. A 50C, todas as substncias tm a mesma
solubilidade.

UNIDADE 2

CONCENTRAO DAS SOLUES

Chama-se concentrao de uma soluo a toda e qualquer
maneira de expressar a proporo existente entre as
quantidades de soluto e de solvente, ou ento, as
quantidades de soluto e da soluo. As definies mais
comuns so as que mencionamos a seguir. Nelas,
usaremos as seguintes convenes:
ndice 1: quantidade relativa ao soluto;
ndice 2: quantidade relativa ao solvente;
s/ ndice: quantidade relativa soluo.

Concentrao Comum:
o quociente entre a massa do soluto (em
gramas) e o volume da soluo (em litros),
ou seja, quantos gramas de soluto h em
cada litro de soluo.
Unidade: ___ g/L

Densidade:
o quociente entre a massa da soluo
(em gramas) e o volume da soluo (em
mL ou cm
3
) ou seja, quantos gramas de
soluo h em cada mL ou cm
3
de soluo.
Unidade: ___ g/mL ou g/cm
3


Ttulo ou Porcentagem em Massa
o quociente entre a massa do
soluto e a massa da soluo (em
gramas), que pode ser expresso
como nmero puro (0 < t < 1) ou
em porcentagem (0 < P
%
< 100%).

Molaridade ou Concentrao Molar
o quociente entre o nmero de
mols do soluto e o volume da
soluo (em litros), ou seja,
quantos mol de soluto h em cada
litro de soluo.
Unidade: ___mol/L___M ou Molar

Normalidade ou Concentrao Normal
o quociente entre o nmero de
equivalentes do soluto e o volume da
soluo (em litros), ou seja, quantos
equivalentes de soluto h em cada litro de
soluo.
Onde x o nmero de cargas geradas na ionizao ou
dissociao de um mol do eletrlito.
Unidade: ___Normal ou N



T = m
1
= m
1
.
m m
1
+ m
2


C = m
1
.
V
d = m .
V

M = n
1
= m
1
.
V mol .V


N = M.x

Please purchase PDF Split-Merge on www.verypdf.com to remove this watermark.
Qumica B Incluso para a Vida

Pr-Vestibular da UFSC 2
Exerccios de Sala #

1. Foi preparado uma soluo com 18 g de
glicose(C
6
H
12
O
6
) em 182 g de gua encerrando um volume
de 200 mL. Sobre a mesma correto afirmar:

01. Apresenta densidade igual a 2,25 g/mL.
02. O ttulo de 9 % de soluto.
04. A concentrao comum de 90 g/L
08. A massa molar do soluto de 180 g/mol
16. O nmero de mols de soluto 0,2 mol.
32. A concentrao molar de 0,5 mol/L.

Tarefa Mnima #

1. Qual o ttulo de uma soluo que contm 20g de soluto
e 80g de solvente?

2. Que massa de H
2
SO
4
deve ser dissolvida em 800ml de
gua para se obter uma soluo de ttulo igual a 0,6?

3. Que massa de gua deve ser usada para se preparar
400g de soluo de NaCl a 8%?

4. Calcule a concentrao em g/l de uma soluo de
nitrato de potssio, sabendo que ela encerra 60g de sal em
300cm
3
de soluo.

5. Calcule a massa de cido ntrico necessria para a
preparao de 150ml de uma soluo de concentrao
50g/l.

6. Em um balo volumtrico so adicionados 20g de KBr
e gua sulficiente para 250ml de soluo. Calcule a
concentrao da soluo em g/l.

7. (UEMS) Sabendo que a densidade de uma soluo
0,789 g/ml, qual a massa aproximada, em gramas,
contida em 75 ml desta soluo?
a) 7,8 x 102 g d) 592 g
b) 75 g e) 59,2 g
c) 0,789 g

8. (UFF) Dissolveu-se 4,6 g de NaCl em 500 g de gua
pura, fervida e isenta de bactrias.
A soluo resultante foi usada como soro fisiolgico na
assepsia de lentes de contacto.
Assinale a opo que indica o valor aproximado da
percentagem, em peso, de NaCl existente nesta soluo.
a) 0,16 % c) 0,46 % e) 2,30 %
b) 0,32 % d) 0,91 %

9. (FEI-SP) No rtulo de uma garrafa de gua mineral l-
se, entre outras coisas:
Contedo: 1,5L
Bicarbonato de clcio: 20 ppm
Sabendo que ppm = mg soluto/L soluo aquosa, qual a
massa de bicarbonato de clcio,
no contedo da garrafa:
a) 0,03g d) 0,06g
b) 0,02g e) 150mg
c) 0,01g
10. Uma soluo apresenta 3 mols de HCl dissolvidos em
17mols de gua. Qual a frao molar do soluto?

11. Calcule a concentrao molar de uma soluo que
apresenta 0,4 mol de KNO
3
em 500ml de soluo.

12. Foram dissolvidos 9,8g de H
2
SO
4
em gua sulficiente
para 400ml de uma soluo. Calcule a concentrao molar
dessa.

13. (ACAFE) O leite de vaca contm, em mdia, 4,5g de
lactose, C
12
H
22
O
11
, por 0,100L. A concentrao molar :
a) 0,26M. c) 4,5M e) 0,45M
b) 0,39M. d) 0,13M

14. (ACAFE) Uria, NH
2
CONH
2
, um produto do
metabolismo de protenas. Que massa de uria necessria
para preparar 500mL de uma soluo 0,20M?
a) 5,1g c) 18,0g e) 6,0g
b) 12,0g d) 24,0g

15. (UFSC) Determine a massa (em gramas) de hidrxido
de sdio NaOH, existente em 500 ml de sua soluo 0,2
molar.

Tarefa Complementar #

16. (UFMA) O dixido de enxofre considerado um dos
maiores poluentes industriais, e adicionado
freqentemente em sucos de frutas naturais, com a
finalidade de eliminar microorganismos e prevenir
oxidaes. Assumindo que uma garrafa comum contm
500 mL de suco com um teor de 2,0 x 10
3
mol/L de SO
2
,
qual a massa de dixido de enxofre no suco?
Dados: O = 16 u; S = 32 u
a) 64 mg c) 1,0 mg e) 4,0 mg
b) 1,0 g d) 4,0 g

17. (UFRS) Solues de uria, (NH
2
)
2
CO, podem ser
utilizadas como fertilizantes. Uma soluo foi obtida pela
mistura de 210 g de uria e 1.000 g de gua. A densidade
da soluo final 1,05 g/mL. A concentrao da soluo
em percentual de massa de uria e em mol/ L,
respectivamente :



18. (Unifor-CE) Uma bebida alcolica contem 20,0% em
massa de etanol e o resto praticamente gua.
temperatura de 20C sua densidade de 0,970 g/mL. A
concentrao dessa soluo em mol/L, :
a) 0,24 b) 0,42 c) 2,4 d) 4,2 e) 6,0
Dado: Massa molar do etanol: 46 g/mol


Please purchase PDF Split-Merge on www.verypdf.com to remove this watermark.
Incluso para a Vida Qumica B

Pr-Vestibular da UFSC 3
19.(FEI-SP) O gs sulfdrico (H
2
S), produto da
fermentao do esgoto chegou a atingir o elevado ndice
de 0,4 mg/L, no rio Tiet. Tal ndice expresso em
molaridade, seria aproximadamente:
Dados: H = 1 e S = 32
a) 1,17 105 c) 2,35 105 e) 1,7 104
b) 1,2 104 d) 3,4 104

20.(UAlfenas-MG) O cido acetilsaliclico um
analgsico que pode ser encontrado em comprimidos ou
em soluo. Um comprimido analgsico tem massa de 500
mg, sendo cerca de 90% constitudo desse cido.Sendo
assim, qual o volume de uma soluo do cido em questo
a 2,5 mols/L que apresenta a mesma massa de cido que
esta presente em dois comprimidos de analgsico?
a) 4,0 mL b) 8,0 mL c) 2,0 mL d) 1,0 mL e) 6,0 mL

Frmula molecular do cido acetilsaliclico:
C
8
O
2
H
7
COOH
Massas molares (g/mol): C = 12; H = 1; O = 16

UNIDADE 3

DILUIO E MISTURA DE SOLUES


Diluir uma soluo significa diminuir a sua concentrao
atravs da adio de mais solvente, sem alterar a
quantidade de soluto.
Molaridade: M
1
.V
1
= M
2
.V
2

Mistura de Solues de mesmo Soluto
Neste caso, tanto a quantidade de soluto quanto o volume
da nova soluo equivalem soma das solues iniciais.

Exerccios de Sala #

1. (UFSC) Qual a massa de Na
2
SO
4
, em gramas,
necessria para preparar 100mL de uma soluo 3,50
molar? Qual o volume de gua, em mL, necessrio para
diluir 10mL desta soluo, transformando-a em 1,75
molar?

2. Qual a concentrao final (em mol/L) da soluo
resultante da mistura de 200mL de uma soluo 0,5M com
100mL de uma soluo 2,0M ?

Tarefa Mnima #
1. (UFRJ) Diluio uma operao muito empregada no
nosso dia-a-dia, quando, por exemplo, preparamos um
refresco a partir de um suco concentrado. Considere 100
mL de determinado suco em que a concentrao de soluto
seja 0,4 mol.L
1
. O volume de gua, em mL, que dever
ser acrescentado para que a concentrao do soluto caia
para 0,04 mol.L
1
, ser de:
a) 1.000 b) 500 c) 900 d) 400

2. (UFRJ) Misturou-se 15 mL de uma soluo KClO
3
0,25
M com 35 mL de gua. A concentrao final da soluo
em molaridade :
a) 0,75 M c) 0,25 M e) 0,0075 M
b) 0,075 M d) 0,025 M

3. (UCS-RS) Um processo de evaporao de uma soluo
aquosa AB 0,05 molar foi interrompido aps trs horas,
quando restavam 100 mL de uma soluo aquosa 1,2
molar. O volume da soluo inicial e o volume de gua
evaporada so respectivamente:
a) 1,5 L e 0,1 L d) 2,0 L e 2,4 L
b) 2,1 L e 2,2 L e) 2,5 L e 0,1 L
c) 2,4 L e 2,3 L

4. (ACAFE) Foram misturados 400 mililitros de soluo
0,25 molar de cido sulfrico com 600 mililitros 1,5 molar
do mesmo cido. A molaridade da soluo final :
a) 1,5 b) 0,5 c) 2,0 d) 1,0 e) 3,0

5. Tm-se trs solues de H
2
SO
4
designadas por A, B e
C.
Soluo A: V = 300mL, N = 0,4N
Soluo B: V = 200mL, N = 0,1N
Soluo C: V = 500mL, N = 0,6N
A soluo resultante da mistura das solues A, B e C
ser:
a) 0,44N b) 0,92N c) 0,23N d) 0,46N

6. (UFSC) O uso de flor na gua para consumo
domstico uma das medidas que renem eficcia e baixo
custo na preveno da crie dental. Quando na
concentrao 5,0 x 10
-5
mol . L
-1
de ons fluoreto, qual o
volume de soluo, em litros, que se deve ingerir para
consumir uma massa de 2,85 miligramas de ons fluoreto?
(on-grama do fluoreto = 19g)

7. Que volume de uma soluo de hidrxido de sdio
1,5mol/L deve ser misturado a 300mL de uma soluo
2mol/L da mesma base a fim de torn-la 1,8mol/L.

UNIDADE 4

TITULAO

a determinao da concentrao de uma soluo
fazendo-a reagir com outra de concentrao conhecida.
Numa titulao cido-base, utilizamos uma soluo cida
para neutralizar uma soluo bsica.
M = n .
.
. n = M.V
V

Ex: H
2
SO
4
+ 2NaOH Na
2
SO
4
+ 2H
2
O

M
A
.V
A
.x = M
B
.V
B
.y
Onde x= nmero de hidrognios ionizveis.
e y= nmero de hidroxilas da base.


C = m .
.
. m = C.V .
.
. C
1
.V
1
= C
2
.V
2

V

Please purchase PDF Split-Merge on www.verypdf.com to remove this watermark.
Qumica B Incluso para a Vida

Pr-Vestibular da UFSC 4
PROPRIEDADES COLIGATIVAS
So propriedades relacionadas ao nmero de partculas de
soluto dissolvidas em uma soluo.

Ao adicionarmos um determinado soluto no voltil a um
lquido puro, verificamos diversas alteraes neste lquido
tais como:

1. Abaixamento da presso de vapor (Tonoscopia)
2. Aumento da temperatura de ebulio (Ebulioscopia).
3. Abaixamento da temperatura de Congelamento
(Crioscopia)
4. Presso Osmtica (Osmometria)

Classificao das Membranas
- Membrana permevel: Permite a passagem do soluto e
do solvente
- Membrana Impermevel: No permite a passagem do
soluto nem do solvente
- Membrana Semipermevel: Permite a passagem
apenas do solvente.

OBS.: quanto maior o nmero de espcie adicionada,
maior o efeito.

Exerccios de Sala #

1. Foram titulados 20 mL de soluo de H
2
SO
4
com 20
mL de soluo 0,4 mol/L de NaOH. Qual a molaridade do
cido titulado?

2. (UFSC) Assinale as proposies corretas.
01. A gua do mar ferve a uma temperatura mais baixa
que a gua pura a uma mesma altitude em relao ao
nvel do mar.
02. A gua do mar congela a uma temperatura mais baixa
que a gua pura ou a uma mesma altitude em relao
ao nvel do mar.
04. Uma soluo aquosa de sacarose ferve a uma
temperatura mais alta que a gua pura a uma mesma
altitude em relao ao nvel do mar.
08. Uma soluo aquosa de sacarose congela a uma
temperatura mais alta que a gua pura a uma mesma
altitude em relao ao nvel do mar.
16. Entre a gua e o lcool, o lcool apresenta a maior
presso de vapor porque mais voltil que a que a
gua.
32. A adio de um soluto no voltil provocar um
aumento da presso de vapor solvente

Tarefa Mnima #

1. (Unifor-CE) Quando se comparam solues aquosas de
mesma concentrao, em mol/L, de cloreto de sdio e
cloreto de potssio pode-se afirmar que possuem idnticas
propriedades
I. coligativas;
II. qumicas;
III. fsicas, sob a mesma temperatura.

Dessas afirmaes somente:
a) I correta. d) I e II so corretas.
b) II correta. e) II e III so corretas.
c) III correta.
2. (UFPE) O grfico abaixo representa a presso de vapor
(eixo das ordenadas), em atm, em funo da temperatura
(eixo das abcissas), em C, de trs amostras, I, II e III. Se
uma destas amostras for de gua pura e as outras duas de
gua salgada, podemos afirmar que:

a) a amostra I a amostra de gua salgada;
b) a amostra I a mais voltil;
c) a amostra II mais concentrada que a amostra III;
d) a amostra I a menos voltil;
e) na temperatura TIII e 1 atm a amostra II ainda no
entrou em ebulio.

3. A uma dada temperatura, possui a menor presso de
vapor a soluo aquosa:
a) 0,1 mol/L de sacarose.
b) 0,2 mol/L de sacarose.
c) 0,1 mol/L de cido clordrico.
d) 0,2 mol/L de cido clordrico.
e) 0,1 mol/L de hidrxido de sdio.

4. (Fei) Aquecendo gua destilada, numa panela aberta e
num local onde a presso ambiente 0,92atm, a
temperatura de ebulio da gua:
a) ser inferior a 100C
b) depende da rapidez do aquecimento
c) ser igual a 100C
d) alcanada quando a presso mxima de vapor
saturante for 1atm.
e) ser superior a 100C

5. (UFPE) Foi observado que o cozimento de meio quilo
de batatas em 1 litro de gua mais rpido se
adicionarmos 200 gramas de sal gua de cozimento.
Considere as seguintes possveis explicaes para o fato:
1- a adio de sal provoca um aumento da temperatura de
ebulio da gua;
2- a adio de sal provoca um aumento da presso de
vapor da gua;
3- o sal adicionado no altera a temperatura de ebulio da
gua, mas reage com o amido das batatas.
Est(o) correta(s) a(s) explicao(es):
a) 1 apenas
b) 2 apenas
c) 3 apenas
d) 1 e 2 apenas
e) 1, 2 e 3

6. Na panela de presso, os alimentos cozinham em menos
tempo, porque a presso exercida sobre a gua torna-se
maior que a presso atmosfrica.
Em conseqncia desse fato, podemos afirmar que o
tempo de cozimento do alimento menor porque:
a) a gua passa a "ferver" abaixo de 100C.
b) a gua passa a "ferver" acima de 100C.
c) a gua passa a "ferver" a 100C.
Please purchase PDF Split-Merge on www.verypdf.com to remove this watermark.
Incluso para a Vida Qumica B

Pr-Vestibular da UFSC 5
d) no h mudana na temperatura de ebulio da gua.
e) sob presso maior a temperatura de ebulio da gua
deve ser menor.

7. Considere o grfico a seguir que representa as variaes
das presses mximas de vapor da gua pura (A.P.) e duas
amostras lquidas A e B, em funo da temperatura.

Pode-se concluir que, em temperaturas iguais,
a) a amostra A se constitui de um lquido menos voltil
que a gua pura.
b) a amostra B pode ser constituda de uma soluo aquosa
de cloreto de sdio.
c) a amostra B constitui-se de um lquido que evapora
mais rapidamente que a gua pura.
d) a amostra A pode ser constituda de soluo aquosa de
sacarose.
e) as amostras A e B se constituem de solues aquosas
preparadas com solutos diferentes.

8. Considere os sistemas I e II, constitudos,
respectivamente, por:
I- 50mL de gua pura.
II- 50mL de soluo 0,1M de cloreto de sdio.
Submetidos s mesmas condies apropriadas, verifica-se
que:
a) no sistema I, a presso de vapor da gua menor do que
no sistema II.
b) no sistema II, a temperatura de solidificao da soluo
maior do que no sistema I.
c) no sistema II, a temperatura de ebulio da soluo
maior do que no sistema I.
d) os dois sistemas apresentam a mesma temperatura de
congelamento.
e) nos dois sistemas, a presso de vapor a mesma.

9. Sejam dadas as seguintes solues aquosas:
I - 0,1 mol/L de glicose (C
6
H
12
O
6
)
II - 0,2 mol/L sacarose (C
12
H
22
O
11
)
III - 0,1 mol/L de hidrxido de sdio (NaOH)
IV - 0,2 mol/L de cloreto de clcio (CaCl
2
)
V - 0,2 mol/L de nitrato de potssio (KNO
3
)
A que apresenta maior temperatura de ebulio :
a) I b) II c) III d) IV e)V

10. Um aluno, interessado em estudar as propriedades de
solues colocou em uma caixa dois copos contendo
volumes iguais de solues aquosas de um mesmo soluto
no-voltil, fechando-a hermeticamente, conforme ilustra a
figura a seguir:

A soluo contida no copo I era mais concentrada que a
contida no copo II. A temperatura externa caixa
permaneceu constante durante o experimento. Acerca das
observaes que poderiam ser feitas a respeito desse
experimento, podemos afirmar.
01. Aps alguns dias, o volume da soluo contida no
copo I diminuir.
02. As concentraes das solues nos dois copos no se
alteraro com o tempo porque o soluto no voltil.
04. O ar dentro da caixa ficar saturado de vapor d'gua.
08. Aps alguns dias, as duas solues ficaro com a
mesma presso de vapor.

11. (UFSC) Verifica-se, experimentalmente, que a presso
de vapor de um lquido aumenta com a elevao da
temperatura e que, na temperatura de ebulio, seu valor
mximo. A 100
o
C a presso mxima de vapor da gua
pura de 1 atmosfera, e nessa temperatura a gua pura
entra em ebulio, conforme ilustrao a seguir:
0 100
t (C)
p (mm Hg)
50
200
400
600
800
760

Numa cidade, cuja altitude superior do nvel do mar, a
temperatura de ebulio da gua pura :
01. menor que 100
o
C, porque a presso atmosfrica
menor.
02. maior que 100
o
C, porque a presso atmosfrica
menor.
04. menor que 100
o
C, porque a presso atmosfrica
maior.
08. maior que 100
o
C, porque a presso atmosfrica
maior.
16. igual a 100
o
C, porque a frmula da gua no se altera,
seja qual for a temperatura ou presso.

UNIDADE 5

TERMOQUMICA

a parte da qumica que estuda o calor envolvido nas
reaes qumicas .

Unidades de Energia

- Caloria (cal): a quantidade de calor necessria para
aquecer 1 grama de gua em 1C.
- Joule (J): a energia necessria para deslocar o ponto
de aplicao de uma fora constante de 1 newton em
uma distncia de 1 metro, na direo do movimento.

1 cal = 4,18J

Libera CalorExotrmica
Reao A + B C+calor
qmc Absorve CalorEndotrmica
A + B + Calor C
Please purchase PDF Split-Merge on www.verypdf.com to remove this watermark.
Qumica B Incluso para a Vida

Pr-Vestibular da UFSC 6
ENTALPIA (H): o contedo global de energia de um
sistema.

Em uma reao qumica temos:

Reagentes Produtos
H
r
H
p


- Quando H
r
maior que H
p
a reao ocorrer com
liberao de energia e denominada reao exotrmica
(Hp < Hr).
- Quando H
r
menor que H
p
a reao ocorrer com
absoro de energia e denominada reao
endotrmica(Hp > Hr).


Variao de Entalpia AH

a diferena de energia entre os produtos (H
p
) e os
reagentes (H
r
) em uma dada reao qumica.

AH = H
p
- H
r


Hp = estado final (produto)
Hr = estado inicial (reagente)

DIAGRAMAS DE ENERGIA

Diagrama de reao Exotrmica (Hp < Hr)












AH < 0
AH = -(negativo) liberao de calor
Reagentes Produto + calor
Reagentes Produto AH = -(negativo)

Diagrama da reao Endotrmica (Hp > Hr)












AH > 0
AH = +(positivo) absoro de calor
Reag. + calor Prod.
Reag. Prod. AH = +
TIPOS DE CALORES DE REAO

Calor de Formao
o AH que ocorre na sntese total de 1mol de uma
substncia a partir de seus elementos no estado padro.
tambm conhecido como entalpia de formao.
Exemplo:

H
2 (g)
+ O
2 (g)
H
2
O
(l)
AH = -68,3 Kcal

(25C e 1 atm)

Lembre-se que neste caso as substncias simples possuem
H = 0

Calor de Combusto
o calor (AH) que ocorre quando 1mol de uma substncia
qualquer sofre combusto completa.
A reao de combusto ocorre quando uma substncia
reage com o oxignio tendo, em geral, como produtos
finais gs carbnico e gua (combusto completa). O calor
de combusto sempre possuir AH negativo (liberao de
calor nas reaes de combusto).
Exemplo:

CH
4 (g)
+ 2O

2 (g)
CO
2 (g)
+ 2 H
2
O
(l)


AH = -211,5 Kcal mol de CH
4
(25C, 1 atm)

Complemento: estados fisicos e a variao de entalpia
( S L G)

Exerccios de Sala #

1. (UFSC) As reaes:

I - A + B C + 30 kcal

II - A + B 20kcal C

III - A + B + 60kcal C

IV - CaCO
3
CaO + CaO AH > 0

01. So todas endotrmicas
02. So todas exotrmicas
04. I e II so endotrmicas
08. II exotrmica
16. III endotrmica

UNIDADE 6

MTODOS PARA CLCULOS DE AH

Experimentalmente, o calor absorvido ou liberado durante
uma reao qumica pode ser determinado atravs de um
calormetro. Teoricamente, existem vrias maneiras de se
calcular a variao de entalpia de uma reao qumica.
Esse clculo pode ser feito de trs maneiras diferentes,
dependendo dos dados do problema:

(Entalpia)
Curso da Reao
Reagentes
Produtos

Hr


Hp

AH
H
Curso da Reao
Reagentes
Produtos
Hp


Hr
AH
H
Please purchase PDF Split-Merge on www.verypdf.com to remove this watermark.
Incluso para a Vida Qumica B

Pr-Vestibular da UFSC 7
1 MTODO (a partir dos calores de formao):

Determinando o AH de uma reao a partir das entalpias
de formao utilizando a expresso:
AH = E Hp - E Hr

2 MTODO (a partir dos calores de ligao):

Aplicao do conceito de energia de ligao:
AH = EAH Rompidas + EAH Formadas

REAGENTES PRODUTOS
Rompem
AH > O (+)
Formam
AH < O ()

Reagentes: quebra de ligaes
Produtos: formao de ligaes

3 MTODO: Lei de Hess (a partir dos calores de
combusto):
As equaes qumicas para os passos individuais de uma
reao podem ser combinadas para obter a equao
termoqumica da reao global. A Lei de Hess tambm
conhecida por princpio da aditividade.

De acordo com Hess a variao da entalpia de uma reao
qumica s depende do estado inicial e final do processo.
AH = AH
1
+ AH
2
+ ___

APLICAO NA RESOLUO DE PROBLEMAS
Observaes:

* a inverso de uma equao termoqumica implica na
inverso do sinal da variao da entalpia dessa reao;
* a multiplicao ou diviso dos coeficientes de uma
equao termoqumica por um dado valor implica na
multiplicao ou diviso da variao da entalpia dessa
reao por esse valor.

Exerccios de Sala #

1. Dados
AH CO
2(g)
= -94,1 Kcal/mol
AH H
2
O
(l)
= -68,3 Kcal/mol
AH CH
4(g)
= -17,9 Kcal/mol

Calcular a variao da entalpia da reao:

CH
4(g)
+ 2O
2(g)
CO
2 (g)
+ 2H
2
O
(l)

2. So dadas as seguintes energias de ligao:

LIGAO
ENERGIA
(KJ/mol de ligao )
H Cl
H - H
Cl Cl

431,8
102,45
242,6


Com os dados fornecidos possvel prever que a reao:

2 HCl
(g)
H
2(g)
+ Cl
2 (g)

tenha AH, em kJ, da ordem de:

3. Dadas as equaes termoqumicas:

NO
(g)
+ O
2(g)
NO
2(g)
AH
1
= -13,5 Kcal
Reag

N
2(g)
+ O
2(g)
NO
2(g)
AH
2
= +8,13 Kcal
Reag.

Calcular o AH da reao:
N
2(g)
+ O
2(g)
NO
(g)

Reag. Prod.

Tarefa Mnima #

1. (UPotiguar-RN) Quais das seguintes afirmativas so
verdadeiras para uma reao endotrmica?
I - O .AH positivo.
II - O calor transferido ao meio ambiente.
III - A entalpia dos produtos maior que a entalpia dos
reagentes.
IV - O AH negativo
a) I e II b) II e IV c) I e III d) III e IV

2. (UFSC) Dadas as variaes de entalpia de formao
para as substncias:
,
SUBSTNCIA AH
f
(Kcal/mol)
CH
4 (g)
-17,9
CO
2

(g)
-94,0
H
2
O
(g)
-68,3

Calcule a entalpia (em Kcal/mol) da reao de combusto
do metano.
CH
4 (g)
+ 2 O
2 (g)
1 CO
2
+ 2H
2
O
(l)

Divida o resultado por 10 e assinale no carto resposta o
mdulo do nmero inteiro mais prximo.

3. (UP) O gs amnia (NH
3
) tem um odor muito
irritante. Pode ser sintetizado atravs da reao assim
equacionada:
N
2 (g)
+ 3 H
2 (g)
2NH
3 (g)
AH = -92,2KJ
Analisando o texto acima, correto afirmar:
a) A sntese do gs amnia endotrmica.
b) O gs amnia uma substncia pura simples.
c) O gs nitrognio uma substncia pura composta.
d) O gs hidrognio produto nessa equao.
e) Na sntese da amnia ocorre liberao de calor.

4. (ACAFE) Dada a reao de combusto de 2 mols de
benzeno (C
6
H
6 (l)
), o seu AH
0
de combusto em kcal/mol :
2 C
6
H
6(l)
+ 15 O
2 (g)
12 CO
2 (g)
+ 6 H
2
O
(l)
+ 800kcal
a) + 400
b) 800
c) + 800
d) 1.600
e) 400

5. (UFSC) Observe as equaes que representam a
formao da gua, a partir de seus elementos. Assinale a(s)
proposio(es) falsa(s).
Please purchase PDF Split-Merge on www.verypdf.com to remove this watermark.
Qumica B Incluso para a Vida

Pr-Vestibular da UFSC 8

H
2(g)
+ O
2 (g)
H
2
O
(s)
AH
1
= 96kcal/mol
H
2(g)
+ O
2 (g)
H
2
O
(l)
AH
2
= 68,3kcal/mol
H
2(g)
+ O
2 (g)
H
2
O
(v)
AH
3
= 57,8kcal/mol

01. O sinal negativo indica que as reaes so
exotrmicas
02. A transformao H
2
O
(v)
H
2
O
(l)
libera
10,5kcal/mol
04. O calor de solidificao da gua vale
12,2kcal/mol.
08. 1 mol de H
2
O
(v)
contm mais energia que 1 mol de
H
2
O
(l)

16. A formao de gua a partir do hidrognio libera calor.

6. (PUC-Campinas) So dadas as seguintes energias de
ligao:

LIGAO ENERGIA
(KJ/mol de ligao formada)
H Cl
H F
Cl Cl
F F
-431,8
-563,2
-242,6
-153,1

Com os dados fornecidos possvel prever que a reao:
2 HCl
(g)
+ F
2 (g)
2 HF
(g)
+ Cl
2 (g)
, tenha AH, em kJ,
da ordem de:
a) 584,9, sendo endotrmica
b) 352,3, sendo exotrmica
c) 220,9, sendo endotrmica
d) +220,9, sendo endotrmica
e) +352,3, sendo endotrmica
7. (MED. POUSO ALEGRE MG) Observe o grfico a
seguir e assinale a alternativa correta.:

A variao de entalpia da reao Y
2
+ X
2
2YX, :
a) A c) 2 A e) B + A
b) B d) B A

8. (UFSC) As reaes:
I - A + B C + 30 kcal
II - A + B 20kcal C
III - A + B C 60kcal

01. So todas endotrmicas
02. So todas exotrmicas
04. I e II so endotrmicas
08. II exotrmica
16. III endotrmica

Tarefa Complementar #

9. (FUVEST) Na reao representada por:
CH
4 (g)
+ 4 Cl
2 (g)
CCl
4 (l)
+ 4HCl
(g)
H liberao de 108 kj de energia trmica por mol de
HCl
(g)
formado. Nas mesmas condies, qual ser a
energia trmica liberada na formao de 73,0g de HCl
(g)
?
Dados: H = 1; Cl = 35,5

a) 54 kj d) 216 kj
b) 108 kj e) 432 kj
c) 162 kj

10. (MOJI SP) Dada a tabela:

LIGAO
Cl Cl
H Cl
C H
C Cl
C C
ENERGIA DE LIGAO
58 kcal/mol
103 kcal/mol
99 kcal/mol
79 kcal/mol
83 kcal/mol

Calcular a variao de entalpia da reao
C
2
H
6 (g)
+ Cl
2 (g)
C
2
H
5
Cl
(g)
+ HCl
(g)

a) zero
b) + 25kcal/mol
c) 25 kcal/mol
d) + 83 kcal/mol
e) 83 kcal/mol

11. (UFPel-RS) O flor um gs amarelado que,
temperatura ambiente, extremamente reativo. Forma com
o hidrognio uma mistura explosiva, sintetizando o
fluoreto de hidrognio (em soluo aquosa, o HF difere
dos outros hidrcidos halogenados por formar um cido
fraco e por ser capaz de dissolver o vidro, formando flor-
silicatos). Observe a reao, nas condies padro, e
marque a alternativa que responde corretamente pergunta
abaixo.

H
2
(g) + F
2
(g) 2 HF(g) ; AH = 5,4 kcal

Qual o calor de formao do HF e o tipo de reao
representada acima?

a) +5,4 kcal/mol; reao endotrmica
b) 2,7 kcal/mol; reao exotrmica
c) +2,7 kcal/mol; reao exotrmica
d) 5,4 kcal/mol; reao endotrmica
e) +7,0 kcal/mol; reao exotrmica

12. (UEL-PR) Considere as seguintes entalpias de
formao em kJ/mol:

Al
2
O
3
(s) ............. 1.670
MgO(s) ................. 604

Com essas informaes, pode-se calcular a variao da
entalpia da reao representada por:

3 MgO(s) + 2 A l (s) 3 Mg(s) + Al
2
O
3
(s)

Seu valor igual a:
a) 1.066 Kj d) + 1.066 kJ
b) 142 kJ e) + 2.274 kJ
c) +142 kJ

2 YX
Y
2 +
X
2

Y X
B

0
-A
Please purchase PDF Split-Merge on www.verypdf.com to remove this watermark.
Incluso para a Vida Qumica B

Pr-Vestibular da UFSC 9
13. (UFRN) Considere as seguintes equaes
termoqumicas hipotticas:
A + B C AH = 20,5 Kcal
D + B C AH = 25,5 Kcal
A variao de entalpia da transformao de A em D ser:
a) 5,0 Kcal c) + 46,0 Kcal
b) + 5,0 Kcal d) 46,0 Kcal

14. (UFRJ) Para a equao
HNO
3
(aq) + KOH(aq) KNO
3
(aq) + H
2
O(l), que
apresenta valor de .AH = 13,8 Kcal/mol, o calor de
reao envolvido nessa transformao de:
a) combusto; c) formao; e) soluo.
b) dissoluo; d) neutralizao;

15. (Univali) Uma das etapas envolvidas na produo do
lcool combustvel a fermentao.
A equao que apresenta esta transformao :
Enzima.

C
6
H
12
O
6
2 C
2
H
5
OH + 2 CO
2


Conhecendo-se os calores de formao da glicose, do gs
carbnico e do lcool, respectivamente, 302, 94 e 66
kcal/mol, podemos afirmar que a fermentao ocorre com:
a) liberao de 18 kcal/mol;
b) absoro de 18 kcal/mol;
c) liberao de 142 kcal/mol;
d) absoro de 142 kcal/mol;
e) variao energtica nula.

UNIDADE 7

CINTICA QUMICA

Cintica qumica a parte da qumica que estuda a
velocidade das reaes.
Supondo a reao:
A + B C + D

Quando colocamos os reagentes A e B em contato, eles
reagem para produzir C e D.

A medida que o tempo passa os reagentes A e B so
consumidos e os produtos so formados.

De acordo com o tempo que esse processo ocorre as
reaes podem ser classificadas em:

1) Reaes Lentas: Os produtos so formados
lentamente. Ex.: A oxidao de uma barra de ferro.
2) Reaes Rpidas: Os produtos so formados
rapidamente. Ex.: A exploso de uma dinamite.

Podemos definir velocidade de reao como sendo a
relao entre a quantidade de reagente ou produto,
consumidos ou formados e o intervalo de tempo para isso
ocorrer.

VELOCIDADE MDIA DE UMA
REAO QUMICA

t
n
Vm
A
A
= ou Vm = A[ ]
At
Vm Velocidade mdia da reao (relativa)
An Variao do nmero de mols de um componente
At Variao do tempo da reao
A[ ] variao da concentrao molar.

Exemplo:
Considere a reao:
CaCO
3 (s)
CaO
(s)
+ CO
2 (g)


Certa massa de carbonato de clcio foi aquecida e o
volume de CO
2
formado foi observado, em funo do
tempo.

Observe os dados da tabela:

MOLS DE CO
2
TEMPO EM MIN.
0
20
35
45
50
52
0
10
20
30
40
50

Qual a velocidade mdia dessa reao no intervalo de 0 a
20 minutos?
Vm =
t
n
A
A

V
0 20
= 75 , 1
0 20
0 35
=



V
CO2
= 1,75 mols/min.

VELOCIDADE MDIA ABSOLUTA

aA + bB cC + dD
V
mdia
= V
A
= V
B
= V
C
= V
D
a b c d

Para concentrao molares:
Vm = -A[ A] = - A[ B] = A[ C] = A[ D]
a.At b.At c.At d.At

CONDIES PARA UMA
REAO OCORRER

- Afinidade qumica entre os reagentes.
- Contacto entre os reagentes.

Exemplo:

H
2
+ Cl
2
2 HCl

1) COLISES EFETIVAS;
Para que a reao ocorra os reagentes devem possuir
energia suficiente para haver choques entre suas
molculas.

Exemplo:

reagentes complexo ativado produtos

A
2
B
2
#
Please purchase PDF Split-Merge on www.verypdf.com to remove this watermark.
Qumica B Incluso para a Vida

Pr-Vestibular da UFSC 10
2) ENERGIA DEATIVAO: (Eat)
Energia mnima necessria para uma reao qumica
ocorrer.

Exerccios de Sala #

1. (UFSC) Na reao 2 HI H
2
+ I
2
, observou-se a
seguinte variao na quantidade de HI em funo do
tempo.

TEMPO (min) MILIGRAMAS DE HI
0 200
5 125
10 75
15 40
20 24

A velocidade mdia desta reao, em relao ao HI, no
intervalo de 10 a 15 min., ser:
01. 7 mg/min.
02. 0,7 mg/min.
04. 14 mg/min.
08. 2,0 mg/min.
16. 3,5 mg/min.

UNIDADE 8

CINTICA QUMICA

FATORES QUE ALTERAM A VELOCIDADE DA
REAO:

TEMPERATURA
A temperatura geralmente aumenta a velocidade de uma
reao qumica. Algumas reaes tem sua velocidade
diminuda com o aumento de temperatura (as reaes
exotrmicas).

Regra de Vant Hoff

A cada aumento de temperatura de 10C a velocidade de
uma reao duplica.

Exemplo:

30C 0,1mol/min.
40C 0,2 mol/min.

CATALISADORES
Substncias que diminuem a energia de ativao, e por
conseqncia aumentam a velocidade da reao.

Os catalisadores so substncias que participam de uma
das etapas da reao formando um sub-produto que reage
mais facilmente com o reagente.

Os catalisadores no participam da formao do produto
final da reao e so recuperados exatamente da forma que
iniciaram na reao.






INFLUNCIA DA CONCENTRAO
O aumento da concentrao dos reagentes implica no
aumento do nmero de colises entre as molculas,
aumentando com isso a velocidade da reao

SUPERFCIE DE CONTATO
Quanto maior a superfcie de contato, maior ser a
velocidade da reao.

Exemplo:
Se voc colocar em gua dois comprimidos efervecentes,
um inteiro e o outro triturado, voc observar que o
comprimido que foi triturado reage primeiro, denunciando
um contato maior com a gua.















Lei da Ao das Massas de Guldberg e Waage

Para reaes elementares (que ocorrem em uma etapa).
aA + bB cC + dD

| | | |
b a
B A K V =


[ ] = concentrao molar = mol/l (apenas para gases e
solues).

Para reaes que ocorram em vrias etapas, a lei se aplica
a etapa mais lenta (no elementares).

ORDEM DE UMA REAO QUMICA
A ordem de uma reao dada pela soma dos expoentes
dos reagentes na equao da velocidade.

Exemplo:

2A + 1 B C

V = K . [A]
2
. [B]
1


caminho da reao

E (kcal/mol)
complexo ativado (sem catalisador)
Ecat
E

Ecat




Er

Ep

energia do comp. ativ.
complexo ativado (c/ catalisador)
energia do comp. ativ.
Please purchase PDF Split-Merge on www.verypdf.com to remove this watermark.
Incluso para a Vida Qumica B

Pr-Vestibular da UFSC 11
A reao de segunda ordem em relao ao reagente A, e
de primeira ordem em relao ao reagente B.

A reao total de terceira ordem.

Exerccios de Sala #

1. Assinale as alternativas corretas:
01. Em geral, a velocidade de reao diminui com uma
diminuio de temperatura.
02. A velocidade de uma reao pode ser aumentada,
aumentando-se as concentraes dos reagentes.
04. A velocidade de uma reao determinada pela
velocidade da etapa mais rpida do mecanismo.
08. Os slidos, quando reagentes, tornam as reaes
extremamente rpidas.
16. Em geral, uma reao rpida tem energia de ativao
alta.

2. (ACAFE) Abaixo temos o grfico energtico da reao
A + B C + D. A energia de ativao dessa reao :


a) 10kJ
b) 20kJ
c) 40kJ
d) 50kJ
e) 30kJ

Tarefa Mnima #

1. (FPMPR) Consomem-se 5mols de NH
3
, em 50
minutos na seguinte reao de anlise em um recipiente de
2 litros:
2NH
3
N
2
+ 3 H
2

Calcule a velocidade de consumo de NH
3
em mols/l.h

2. Assinale as afirmativas corretas.
01. Todas as colises intermoleculares resultam em reo
qumica.
02. O aumento da energia da coliso favorece a reao.
04. Em geral, uma reao rpida tem energia de ativao
baixa.
08. O catalisador, que aumenta a velocidade de uma
reao, deve criar um novo caminho para a reao,
com energia de ativao menor.
16. A pulverizao de um slido influi na velocidade de
suas reaes.
32. A oxidao de uma barra de ferro em contato com o ar
atmosfrico uma reao instantnea.

3. (Carlos Chagas) Um catalisador age sobre uma reao
qumica:
a) aumentando a energia de ativao da reao.
b) diminuindo a energia de ativao da reao.
c) diminuindo a variao de entalpia da reao.
d) aumentando o nvel energtico dos procutos.
e) Diminuindo o nvel energtico dos reagentes.

4. (FAC. MED. SANTA CASASP modificada) Se o
diagrama abaixo se refere a uma reao exotrmica,

01. na posio I esto os reagentes;
02. na posio II esto os produtos;
04. na posio III est o complexo ativado;
08. a energia de ativao da reao direta menor do que
na reao inversa;
16. na posio III esto os reagentes da reao direta;
32. na posio II est o complexo ativo.

5. (ACAFE) A reao do monxido de carbono com
oxignio, formado dixido de carbono, representada no
grfico abaixo.

A alternativa falsa :
a) O monxido de carbono e o oxignio esto em A.
b) A reao endotrmica.
c) Em D est o dixido de carbono.
d) O calor de reao representado por C.
e) B a energia de ativao.

6. Assinale as alternativas corretas:
01. Em geral, a velocidade de reao diminui com uma
diminuio de temperatura.
02. A velocidade de uma reao pode ser aumentada,
aumentando-se as concentraes dos reagentes.
04. A velocidade de uma reao determinada pela
velocidade da etapa mais rpida do mecanismo.
08 Os slidos, quando reagentes, tornam as reaes
extremamente rpidas.
16. Em geral, uma reao rpida tem energia de ativao
alta.
32. A subdiviso de um slido aumenta a velocidade de
suas reaes.

7. (PUCMG) A reao 2NO
(g)
+ 2H
2(g)
N
2(g)
+ 2H
2
O
(g)
,
realiza-se em duas etapas:
2NO + H
2
N
2
O + H
2
O (lenta).
N
2
O + H
2
N
2
+ H
2
O (rpida).
Please purchase PDF Split-Merge on www.verypdf.com to remove this watermark.
Qumica B Incluso para a Vida

Pr-Vestibular da UFSC 12
Triplicando-se a presso parcial do NO e mantendo
constante a do H
2
, a velocidade da reao aumentar:
a) 6 vezes;
b) 9 vezes;
c) 8 bezes;
d) 12 vezes;
e) 18 vezes.

8. (UFMA) Considere a reao:
NO
(g)
+ O
2(g)
NO
2(g)

Supondo que o oxignio no influencie na velocidade da
reao, a expresso de velocidade correta para essa
equao ser:
a) v = k [NO
2
] [O
2
]
b) v = k [NO]
1/2

c) v = k [NO] [O
2
]
2

d) v = k [NO]
n

e) v = k [O
2
]
1/2


9. (SUPRA) Dona Salete sempre procura aplicar novas
tcnicas baseadas em seus conhecimentos de qumica
prtica de preparar po caseiro. Por exemplo: deixar a
massa descansar em um lugar mais aquecido para:
a) aumentar o processo de decomposio das gorduras
que deixam ranosa a massa.
b) evitar a fermentao que intensifica o sabor azedo na
massa
c) favorecer a fermentao que produz CO
2
e faz a massa
crescer e ficar macia.
d) diminuir a degradao dos steres para que a massa
no fique pesada.
e) dificultar a ao dos microorganismos para no
estragar a massa.

10. Na reao 2 HI H
2
+ I
2
, observou-se a seguinte
variao na quantidade de HI em funo do tempo.

Tempo (min) Mols de HI
0 0,200
5 0,125
10 0,075
15 0,040
20 0,024

A velocidade mdia desta reao, no intervalo de 10 a 15
min., ser:
a) 0,007 mols/min. d) 2,0 mols/min
b) 0,7 mols/min. e) nda
c) 1,4 mols/min.

11. (ACAFE) Dada a reao 4NH
3 (g)
+ 3O
2 (g)
2N
2 (g)
+
6H
2
O
(g)
e sabendo que o N
2
formado a uma velocidade
de 5 moles/L . s, calcule a velocidade de formao da
gua, em moles/L.s.
a) 3 b) 6 c) 1,5 d) 2 e) 15





12. (UDESC) Com base no grfico abaixo, podemos
afirmar que:

a) V representa a energia dos produtos formados na
reao.
b) IV representa a energia de ativao de uma reao
endotrmica com catalisador.
c) III representa a energia de ativao de uma reao
exotrmica com catalisador.
d) II representa a energia de ativao de uma reao
endotrmica sem catalisador.
e) I representa a variao de entalpia de uma reao
exotrmica sem catalisador.

UNIDADE 9

EQUILBRIO QUMICO

Algumas reaes qumicas se processam simultaneamente
em dois sentidos: direto e inverso. Estas reaes so
denominadas reversveis.

Vamos supor uma reao reversvel:
V1
xA + yB zC + wD

V2
Se a velocidade da reao direta for igual a velocidade da
reao inversa dizemos que a reao se encontra em
equilbrio qumico.
Segundo Guldberg-Waage, a velocidade de uma reao
qumica proporcional concentrao dos reagentes.

V
1
= k
1
[A]
x
. [B]
y

V
2
= k
2
[C]
z
. [D]
w


Como no equibrio V
1
= V
2
, teremos:

k
1
[A]
x
. [B]
y
= k
2
[C]
z
. [D]
w

| | | |
| | | |
y x
w z
B A
D C
K
K

=
2
1


Como resultado da razo entre duas constantes, temos uma
nova constante, Kc, denominada constante de equilbrio
em termos de concentrao.

Kc = k
1
k
2



Please purchase PDF Split-Merge on www.verypdf.com to remove this watermark.
Incluso para a Vida Qumica B

Pr-Vestibular da UFSC 13
temos:
| | | |
| | | |
y x
w z
B A
D C
Kc

=

Para sistemas gasosos, a constante de equilbrio ser
expressa em termos de presses parciais:

( ) ( )
( ) ( )
y x
w z
pB pA
pD pC
Kp

=

DESLOCAMENTO DO EQUILBRIO QUMICO

Princpios de Le Chatelier
Se em um sistema em equilbrio ocorrer alguma alterao,
haver um deslocamento do equilbrio no sentido de
minimizar ou anular a ao desta mudana.

Fatores que influenciam no deslocamento do Equilbrio
Qumico.

CONCENTRAO
Se aumentarmos a concentrao de uma substncia o
equilbrio se deslocar no sentido de consum-la. O
contrrio tambm verificado quando diminumos a
concentrao.

Exemplo:

Aumento da concentrao de H
2
ou Cl
2



Diminuio da concentrao de H
2
ou Cl
2



PRESSO
O aumento de presso desloca o equilbrio no sentido de
menor volume de molculas no estado gasoso.

Exemplo:
Aumento da Presso
N
2(g)
+ 3H
2(g)
2NH
3(g)

22,4 L + 67,2L 44,8L
Diminuio da Presso


TEMPERATURA
Aumentando a temperatura de um sistema em equilbrio, o
equilbrio se desloca no sentido em que h absoro de
calor (endotrmico). Se diminurmos a temperatura, o
equilbrio se desloca no sentido em que h liberao de
calor (exotrmico).
Exemplo:
Diminuio da Temperatura

2H
2 (g)
+ O
2 (g)
H
2
O
(g)
+ calor

Aumento da Temperatura


CATALISADOR
Os catalisadores no alteram o equilbrio qumico. O
catalisador tem papel de diminuir a energia de ativao de
uma reao para favorec-la. Portanto, apenas aumentam a
velocidade da reao.

Exerccios de Sala #

1. Os xidos de nitrognio desempenham um papel chave
na formao de "smog fotoqumico". A queima de
combustveis a alta temperatura a principal fonte de
xidos de nitrognio. Quantidades detectveis de xido
ntrico so produzidas pela reao em equilbrio:

N
2(g)
+ 0
2(g)
2NO
(g)
H = + 180,8 KJ

Supondo o sistema em equilbrio e que numa determinada
temperatura as presses parciais dos gases em equilbrio
so iguais a: pNO=0,1atm; pN
2
= 0,2atm; pO
2
= 0,01atm,
indique o valor correto da constante de equilbrio (Kp).

a) 0,2 b) 4 c) 5 d) 40 e) 50

2. (UFSC) Considere o sistema em equilbrio:

2 NO
(g)
+ 2 CO
(g)
N
2 (g)
+ 2 CO
2 (g)
AH = -747 kJ

Assinale a(s) proposio(es) verdadeira(s).

01. A formao de N
2 (g)
ser favorecida se aumentarmos
a presso total sobre o sistema
02. Aumentando a presso total sobre o sistema, o
equilbrio no ser deslocado.
04. A adio de um catalisador favorece a formao dos
produtos
08. A diminuio da temperatura desloca o equilbrio para
a direita
16. Aumentando a presso parcial do CO
2
, desloca o
equilbrio para a direita
32. A constante de equilbrio Kp da reao, em termos de
presses parcial, dada pela expresso:
| | | |
| | | |
2 2
2
2 2
CO NO
CO N
P P
P P
Kp

=


Tarefa Mnima #

1. (FACITOL-PR) Consideremos o equilbrio:
2A
(g)
+ 3B
(g)
C
(g)
+ 4D
(g)
A expresso da lei da ao das massas para esse equilbrio
ser:
a) k =
| || |
| || | B A
D C
2 2

b) k =
| | | |
| || |
4
2
D C
B A

c) k =
| || |
| | | |
3 2
4
B A
D C






Please purchase PDF Split-Merge on www.verypdf.com to remove this watermark.
Qumica B Incluso para a Vida

Pr-Vestibular da UFSC 14
d) k =
| | | |
| | | |
4 3
4
B A
D C

e) k =
| || |
| || |
3
4
2 B A
D C


2. (PUCSP) Considerando o sistema em equilbrio:
CO
(g)
+ H
2
O
(g)
CO
2(g)
+ H
2(g)
A = -10 kcal
Iremos aumentar a concentrao de equilbrio do
hidrognio se:
a) diminuirmos a presso total sobre o sistema;
b) aumentarmos a preso total sobre o sistema;
c) diminuirmos a temperatura;
d) aumentarmos a temperatura;
e) introduzirmos um gs inerte no sistema.

3. (UFSC) Dada a reao:
2 NO
2 (g)
N
2
O
4 (g)
AH = 14,1 kcal
Qual das alteraes abaixo aumentaria a concentrao
molecular do produto?

a) aumento de temperatura
b) diminuio da concentrao de NO
2

c) diminuio da temperatura
d) diminuio da presso
e) adio de um catalisador

4. (ACAFE) Para a reao:

2 NO
2 (g)
N
2
O
4 (g)
,

cujo K = 171 L/mol a 25C e [NO
2
] no equlibro 0,0250
moles/L. Assinale a alternativa que contm o valor de
[N
2
O
4
] em moles/L.

a) 0,107
b) 0,250
c) 0,344
d) 0,625
e) 4,28

5. (PUC-SP) Considere o processo em equilbrio
2 X
(g)
X
2(g)


Se a concentrao inicial de X for 0,2 M e passar a 0,04M
aps o estabelecimento do equilbrio, porque a constante
de equilbrio vale:
a) 50
b) 100
c) 200
d) 400
e) 800

6. (UFSC) Considere as reaes em equilbrio:
(a) H
2
O H
+
+ OH
-

(b) H H
+
+ A
-

(c) BOH B
+
+ OH
-

Quando se adicionam ons H
+
a esses sistemas: (assinale
a(s) opo(es) correta(s):
01. O equilbrio se desloca para a esqueda em (a).
02. O equilbrio se desloca para a direita em (b).
04. O equilbrio no afetado em (c).
08. O grau de dissociao aumenta em (c).
16. Produz-se oxignio em (b).
32. Aumenta o produto inico [H
+
] [OH
-
] em (a).

7. (UFSC) As reaes representadas abaixo esto na fase
gasosa e em equilbrio. Assinale a nica proposio
correta em que o equilbrio no fica alterado quando se
varia a presso total da mistura.

01. O
3 (g)
3 O
(g)


02. 2 CO
2 (g)
2 CO
(g)
+ O
2

04. H
2 (g)
+ I
2 (g)
2 HI
(g)


08. N
2 (g)
+ 3 H
2 (g)
2 NH
3 (g)


8. (UFSC) Considere o sistema em equilbrio:
2NO
(g)
+ 2CO
(g)
N
2 (g)
+ 2CO
2 (g)
AH = -747 kJ
Assinale a(s) proposio(es) verdadeira(s).
01. A formao de N
2 (g)
ser favorecida se aumentarmos a
presso total sobre o sistema
02. Aumentando a presso total sobre o sistema, o
equilbrio no ser deslocado.
04. A adio de um catalisador favorece a formao dos
produtos
08. A diminuio da temperatura desloca o equilbrio para
a direita
16. Aumentando a presso parcial do CO
2
, o equilbrio
para a direita
32. A constante de equilbrio Kp da reao, em termos de
presses parcial, dada pela expresso:
| | | |
| | | |
2 2
2
2 2
CO NO
CO N
P P
P P
Kp

=

9. Considerando o equilbrio:
3Fe
(s)
+ 4H
2
O
(g)
Fe
3
O
4 (s)
+ 4H
2 (g)

Verificando que a constante de equilbrio desta reao
qumica varia quando se altera:
a) a presso.
b) a temperatura.
c) o volume.
d) a concentrao de Fe
(s).

e) a concentrao de Fe
2
O
4 (s).


10. (ACAFE) Em relao ao equilbrio:
PCl
3 (g)
+ Cl
2 (g)
PCl
5 (g)
+ 165,11 kj
A alternativa falsa :
a) a reao endotrmica.
b) aumentando a presso, o equilbrio ser deslocado
para os produtos.
c) aumentando a temperatura, o equilbrio ser deslocado
para os reagentes.
d) uma possvel equao da velocidade ser V = k [Cl
2
] .
[PCl
3
].






Please purchase PDF Split-Merge on www.verypdf.com to remove this watermark.
Incluso para a Vida Qumica B

Pr-Vestibular da UFSC 15
K
a
=
[HA]
] [A ] [H
+

e) a expresso matemtica da constante de equilbrio
| |
| | | |
2 3
5
Cl PCl
PCl
Kc

=
.
11. (CESCEA-SP) Quais das seguintes reaes so
favorecidas no sentido indicado quando se eleva a presso,
mantendo-se a temperatura constante?
I - N
2
+ O
2
2 NO
II - Br
2
+ H
2
2 HBr
III - N
2
+ 3 H
2
2 NH
3

IV - 2 H
2
+ O
2
2 H
2
O

a) I e II c) I e IV e) III e IV
b) I e III d) II e III

UNIDADE 10

EQUILBRIO INICO

AB A
+
+ B
-

K
i
=
[AB]
] [B ] [A
+

O equilbrio inico aquele que se estabelece entre uma
substncia (eletrlito) e seus ons em soluo aquosa.
As regras usadas no equilbrio inico so as mesmas do
equilbrio molecular.
K
i
K
a
para cidos
K
i
K
b
para bases
K
i
K
w
para gua

Grau de Ionizao (o)
A fora de um eletrlito determinada pelo seu grau de
ionizao.
Dada a ionizao de um cido HA
HA H
+
+ A


Temos: o =
molculas de total nmero
ionizadas molculas de nmero



Quando a ionizao do cido ocorre em vrias etapas
(cido poliprtico), haver uma constante (K) e um grau
de ionizao (o) para cada etapa, onde:

K
1
> K
2
> K
3

Para eletrlitos fracos: Ka = Mo
2 .

Onde M a molaridade da soluo.

Produto Inico da gua
A gua se ioniza em pequena escala e podemos representar
sua ionizao por:
H
2
O
(l)
H
+
(aq)
+ OH
-
(aq)
Kw = [H
+
].[OH

]
O valor de Kw foi determinado experimentalmente a 25C
e possui o valor de 10
14
.
Portanto, Kw = [H
+
].[OH

] =10
14
. Para a gua pura,
temos: [H
+
]=[OH

] = 10
7
mol/l
Se adicionarmos um cido gua, a concentrao dos ons
H
+
aumenta e [OH

] diminui.

POTENCIAL HIDROGENINICO (pH)
logaritmo negativo da concentrao de ons H
+
:
pH = | |

OH log

POTENCIAL HIDROXILINICO (pOH)
o logaritmo negativo da concentrao dos ons OH
-

pOH = | |

OH log
[H
+
] . [OH
-
] = 10
-14
pH + pOH = 14

Para gua pura temos:

[H
+
] = [OH
-
] = 10
-7
pH = 7 e pOH=7

Em solues cidas: Em solues cidas:

[H
+
] > 10
-7
[H
+
] < 10
-7

[OH
-
] < 10
-7
[OH
-
] > 10
-7
pH < 7 e pOH > 7 pH > 7 e pOH < 7

Exerccios de Sala #

1. (UFSC) Assinale a(s) proposio(es) verdadeira(s) e
d o valor total como resposta.

01. O vinagre (pH = 3) cido.
02. A gua do mar (pH = 8,3) cida.
04. O vinho (pH = 2,8 a 3,8) cido.
08. Uma soluo aquosa de HCl 0,001 molar tem pH = 3.
16. Uma soluo aquosa de NaOH 0,001 molar tem
pH=11.
32. A cerveja (pH = 4,5) bsica.
64. O suco de tomate (pH = 3,0) bsico.

2. (UFSC) So dadas as duas solues aquosas:









[H
+
] = 1.10
4
Molar [H
+
] = 1.10
8
Molar

Com base nas afirmaes acima, podemos concluir
corretamente que:
01. A soluo A apresenta pH = 4, portanto, com carter
cido.
02. A soluo B apresenta carter bsico e pH = 8.
04. A concentrao de ons OH

,

presentes na soluo A,
10
10
mol/L.
08. A concentrao de ons OH

, presentes na soluo B,
10
6
mol/L.
Soluo
A
Soluo
B

Please purchase PDF Split-Merge on www.verypdf.com to remove this watermark.
Qumica B Incluso para a Vida

Pr-Vestibular da UFSC 16
16. Adicionando 100 mL de gua a 100 mL da soluo
A, a nova concentrao ser [H
+
] = 1.10
2
mol/L.
32. Ao adicionarmos 100 mL de gua a 100 mL da soluo
A, a nova soluo ficar mais cida.

Tarefa Mnima #

1. (MARING-PR) Uma soluo 0,05M de um cido
fraco 1% ionizado. Qual , aproximadamente, a sua
constante de ionizao?
a) 5 x 10
-8
d) 2 x 10
-3

b) 5 x 10
-6
e) nda
c) 5 x 10
-5


2. (ACAFE) Assinale a alternativa que corresponde ao
grau de ionizao (%) do cido ciandrico, HCN, numa
soluo 0,01 molar, sabendo que a sua constante de
ionizao de 4 . 10
-10
(considerar 1 - o ~ 1).
a) 0,02 d) 4 . 10
-2

b) 2 . 10
4
e) 4 . 10
-4

c) 2 . 10
-4


3. (UFViosa-MG) Em relao a uma soluo de pH = 5 a
25C e 1atm, podemos afirmar que:
I - [H
+
] = 10
-5
(mols/litro)
II - O meio cido
III - pH = log [H
+
]
IV - 14 = [H
+
] + [OH
-
]
So verdadeiras as afirmativas:
a) II e III c) I e II e) I, II, III e IV
b) I e III d) II, III e IV
4. (UFPR) Uma soluo 0,001M de HCl acusar um pH
prximo de:
a) 2,2 c) 5,5 e) nda
b) 3,0 d) 6,2

5. (FMPOUSO ALEGREMG) O valor de concentrao
do on hidroxila em uma soluo 0,001 M de HCl :
a) 10
-11
M c) 10
-3
M e) 10
-2
M
b) 10
-10
M d) 10
-7
M

6. (PUCPELOTAS-RS) Acrescentou-se gua a 0,20L de
uma soluo de cido ntrico de pH = 2,0, a 25C, at
completar o volume de 2,0 L. O pH da soluo resultante
:
a) 0,10 c) 1,0 e) 3,0
b) 0,20 d) 2,0

7. (FPMPR) 999 litros de gua so adicionadas a um litro
de soluo de NaOH de pH = 12,5. O pH, aps a diluio
ser:
a) 12,5 c) 10,5 e) nda
b) 11,5 d) 9,5

8. (UFSC) Assinale a(s) proposio(es) verdadeira(s) e
d o valor total como resposta.
01. O vinagre (pH = 3) cido.
02. A gua do mar (pH = 8,3) cida.
04. O vinho (pH = 2,8 a 3,8) cido.
08. Uma soluo aquosa de HCl 0,001 molar tem pH = 3.
16. Uma soluo aquosa de NaOH 0,001 molar tem pH =
11.
32. A cerveja (pH = 4,5) bsica.
64. O suco de tomate (pH = 3,0) bsico.

9. (ACAFE) Marque a alternativa que indica a substncia
que devemos dissolver em gua pura para obter uma soluo
aquosa com pH menor que 7.
a) Cloreto de sdio.
b) cido clordrico.
c) Acetona.
d) Hidrxido de sdio.
e) Bicarbonato de sdio.

10. (ACAFE) O peixe cru, preparado com suco de limo
ou vinagre, consumido em diversos pases. Esse prato
de fcil digesto, porque o suco de limo ou o vinagre:
a) Forma soluo bsica e no hidrolisa as protenas do
peixe.
b) Forma soluo cida e no hidrolisa as protenas do
peixe.
c) soluo bsica e hidrolisa as protenas do peixe.
d) soluo neutra e hidrolisa as protenas do peixe.
e) Forma soluo cida e hidrolisa as protenas do peixe.

11. Disolvem-se 3,65g de HCl e 4,08 de NaOH em gua
sulficiente para um litro de soluo. Calcule o pH da
soluo resultante a 25C (log2 = 0,3)

12. Calcule o pH de uma soluo 0,020 molar de HCl.
(Dado log 2 = 0,3)

13. (ACAFE) Com relao ao produto inico da gua:
Kw = [H
3
O
+
] [OH
-
]
Pode-se afirmar que, com o aumento da concentrao do
on OH:
a) a soluo resultante ser cida.
b) a concentrao do on H
+
diminuir.
c) as concentraes dos ons H
+
e OH
-
no sofrero
alteraes.
d) o produto inico (Kw) aumentar.
e) o produto inico (Kw) diminuir
.
UNIDADE 11

ELETROQUMICA

Potencial de Oxidao
a capacidade dos metais de dar eltrons.

Al Al
3+
+ 3 e E = + 1,66V
Cu Cu
2+
+ 2 e E = 0 ,34V

Assim, o alumnio tem maior tendncia para ceder eltrons
que o cobre.

Clculo da Voltagem
Dados os potenciais de oxidao:


Please purchase PDF Split-Merge on www.verypdf.com to remove this watermark.
Incluso para a Vida Qumica B

Pr-Vestibular da UFSC 17
Zn Zn
2+
+ 2 e E = 0,76V
Cu Cu
2+
+ 2 e E = -0,34V

V A = E
oxi
+ E
red

V A = 0,76 + 0,34
V A = 1,10V

Pilha de Daniell
So sistemas que produzem corrente contnuas e se
baseiam nas diferentes tendncias para ceder e receber
eltrons das espcies qumicas.

Sentido dos eltrons
Os eltrons circulam do eletrodo de maior potencial de
oxidao para o de menor potencial de oxidao.

Plos da Pilha
Positivo: ctodo ocorre reduo massa aumenta
Negativo: nodo ocorre oxidao massa diminui

Eletrlise
Decomposio de uma substncia pela corrente eltrica.
Caractersticas

nions vo para o nodo sofrer Oxidao
Ctions vo para o Ctodo sofrer Reduo

Eletrlise gnea: NaCl
(s)

A
Na
+
(l)
+ Cl
-
(l)

Ctodo: Na
+
+ 1 e Na
(s)

nodo: Cl
-
- 1 e Cl
2

(g)
Na
+
(l)
+ Cl
-
(l)
Na
(s)
+ Cl
2(g)
Exerccios de Sala #

1. (UFSC) Com base no diagrama de pilha Pb | Pb
2+

(1,0M) | | Ag
+
(1,0M) | Ag e nos potenciais padres
(reduo), a 25C, das semi-reaes:
Ag
+
+ e Ag, E = + 0,80 V
Pb
2+
+ 2 e Pb, E = - 0,13 V

correto afirmar que:
01. O eletrodo de chumbo o nodo e o de prata o
ctodo.
02. O sentido da reao 2Ag + Pb
2+
2Ag
+
+ Pb.
04. A diferena de potencial padro entre os eletrodos
+0,93V.
08. A massa do eletrodo de chumbo aumenta com o
tempo.
16. Os ons de prata sofrero reduo
32. A diferena de potencial padro entre os eletrodos
de +1,73V.
64. Os eltrons se deslocam, no circuito externo, do
eletrodo de prata para o eletrodo de chumbo.

2. (UFSC) Na pilha esquematizada a seguir, fornecido o
sentido do fluxo de eltrons. Dados os valores dos
potenciais-padro de reduo (a 25C e 1 atm) do eletrodo
de cobre (E
red
= 0,34 V) e do eletrodo de prata (E
red
= 0,80
V), indique a(s) proposio(es) correta(s):


01. No eletrodo de cobre ocorre a reduo.
02. Os eltrons fluem do eletrodo de cobre para o eletrodo
de prata.
04. O cobre o agente redutor.
08. A reao global da pilha : Cu
(s)
+ 2Ag
1+
(aq)


Cu
2+
(aq)
+ 2Ag
(s)
.
16. A diferena de potencial da pilha 0,46 V, nas
condies indicadas.
32. A representao correta da pilha : Ag
1+
(aq)
| Ag
(s)
||
Cu
(s)
| Cu
2+
(aq)
.

Tarefa Mnima #

1. (UFMT) Os potenciais-padro dos eletrodos de cobre e
de prata so dados abaixo:

A diferena de potencial da pilha (d.d.p) :
a) 1,14V b) 1,26V c) 1,94V d) 0,46 V

2. (UEPG-PR) Sobre a pilha esquematizada abaixo,
assinale o que for correto:

a) Seu funcionamento diminuiu a concentrao de ons
B
3+
.
b) O eletrodo B sofre oxidao.
c) O eletrodo A denominado ctodo.
d) A equao global dada por
2B
(s)
+ 3A
2+
(aq)
2B
3+
(aq)
+ 3A
(s).

e) O eletrodo B sofre corroso.

3. (UFSM-RS) Existem pilhas, constitudas de um
eletrodo de ltio e outro de iodo, que so utilizadas em
marca-passos cardacos. Seu funcionamento se baseia nas
seguintes semirreaes:


Please purchase PDF Split-Merge on www.verypdf.com to remove this watermark.
Qumica B Incluso para a Vida

Pr-Vestibular da UFSC 18
Considerando esse tipo de pilha, assinale, no quadro a
seguir, a alternativa correta.

4. (UFRJ) Considere uma pilha de prata/magnsio e as
semi-reaes representadas abaixo, com seus respectivos
potenciais de reduo.

O oxidante, o redutor e a diferena de potencial da pilha
esto indicados. respectivamente, em:

a) Mg, Ag+, + 3,17 d) Mg+2, Ag, 3,17
b) Mg, Ag+, + 3,97 e) Ag+, Mg, + 3,17
c) Ag+, Mg, + 1,57

5. (PUC-PR) Dados os potenciais:


o agente redutor mais forte presente na tabela o:

a) Na
0
d) Ni
2+
b) Ag
0
e) Co
2+

c) Fe
2+

6. (UFMS) Um qumico queria saber se uma amostra de
gua estava contaminada com um sal de prata. Ag+ e para
isso, mergulhou um fio de cobre, Cu, na amostra. Com
relao a essa anlise, correto afirmar que:


01. A amostra se torna azulada e isso foi atribudo
presena de ons Cu
+2
;
02. A amostra doa eltrons para o fio de cobre;
04. O fio de cobre torna-se prateado devido ao depsito de
prata metlica;
08. O fio de cobre doa eltrons para a amostra;
16. Ag
+
o agente oxidante da reao.

Please purchase PDF Split-Merge on www.verypdf.com to remove this watermark.
Incluso para a Vida Qumica C

Pr-Vestibular da UFSC 1
UNIDADE 1

QUMICA ORGNICA

a parte da qumica que estuda os compostos formados
por carbono. importante saber, porm, que nem todo
composto que tenha carbono um composto orgnico,
mas todo composto orgnico tem carbono.

Propriedades do Carbono ( Postulados de Kekul)
1- Tetravalncia: Apresenta 4 e

na camada de
valncia podendo, com isso, fazer quatro
ligaes covalentes.


C C C C


2- Igualdade das valncias: As quatro valncias do
carbono so iguais entre si.

H H H Cl

H C H H C Cl Cl C H H C H

Cl H H H

x Carter Anftero: Combina-se tanto com tomos
eletronegativos como com elementos eletropositivos.
x Nmero de Oxidao (Nox) varivel:
Valor mnimo: 4
Valor mximo: + 4
3- Encadeamento: Os tomos de carbono tm a
propriedade de se ligarem entre si, ou com outros
tomos formando cadeias carbnicas com
diversas disposies.

HIBRIDAO DO CARBONO

uma mistura de orbitais de um mesmo tomo, formando
novos orbitais chamados orbitais hbridos. Este fenmeno
explica, por exemplo, a tetravalncia do carbono.
Analisando a sua distribuio eletrnica (1s
2
2s
2
2p
2
)
vemos que h somente dois eltrons desemparelhados.
Para que um tomo de carbono possa fazer as quatro
ligaes, um dos eltrons do subnvel s transferido
para o orbital vazio do subnvel p. O tipo de hibridao
dependendo das ligaes que o tomo faz.
x Hibridao sp: resulta da mistura de um orbital s
com um orbital p. H formao de dois orbitais
hbridos.
x Hibridao sp
2
: resulta da mistura de um orbital s
com dois orbitais p. H formao de trs orbitais
hbridos.
x Hibridao sp
3
: resulta da mistura de um orbital s
com trs orbitais p. H formao de quatro orbitais
hbridos.

Obs.: Os orbitais hbridos so responsveis pelas ligaes
sigma e os orbitais p puros, pelas ligaes pi.

1s 2s 2p

Fundamental

sp
2




Ativado (excitado)

sp

sp
3

EM RESUMO TEMOS:

C


C


C


C

sp
3
sp
2
sp sp

Classificao do Carbono numa cadeia carbnica.
Primrio: Est ligado a pelo menos um tomo de carbono
ou isolado.
Secundrio: Est ligado a dois tomos de carbono.
Tercirio: Est ligado a trs tomos de carbono.
Quaternrio: Est ligado a quatro tomos de carbono.

Observao:
x Hidrognio primrio est ligado a C primrio.
x Hidrognio secundrio est ligado a C secundrio.
x Hidrognio tercirio est ligado a C tercirio.

Saturado: Quando apresenta somente ligaes simples
(somente ligaes sigma).
Insaturado: Quando apresenta ligao dupla ou tripla
(tem ligao pi).
Assimtrico: Quando est ligado a quatro grupos
diferentes.

Exerccios de Sala #

1. (UNIOESTE) Sobre os compostos orgnicos correto
afirmar que:
01. Obrigatoriamente apresentam carbono em sua
constituio.
02. So formados principalmente por ligao covalente.
04. S podem ser obtidos de seres vivos.
08. Podem ser obtidos em laboratrio, sendo que o
primeiro composto obtido foi a ureia.
16. S podem ser obtidos de seres vivos.
32 So formados principalmente por CHONS.

2. (FEMPAR) No composto abaixo existem:
01. 3 carbonos primrios, 1 secundrio e 1 tercirio;
02. 3 carbonos primrios e 2 secundrios;
04. 2 carbonos primrios e 2 secundrios;
08. 2 carbonos primrios e 3 secundrios;
H H

H
3
C C C CH
3


CH
3
H

Please purchase PDF Split-Merge on www.verypdf.com to remove this watermark.
Qumica C Incluso para a Vida

Pr-Vestibular da UFSC
2
16. apenas carbonos secundrios;
32. apenas carbonos saturados;
64. carbonos hbridos sp
2
e sp
3.


UNIDADE 2

CADEIA CARBNICA

Sequncia de tomos de carbono:
Cadeia
aliftica
Cadeia
cclica
Mista
a cadeia
carbnica que
no forma
ciclos
a cadeia
carbnica que
forma ciclos
a cadeia carbnica
onde parte ciclo e
parte no
possui ciclo.
C C
~ ~
C C C = C
~
C
C = C
/ \
C C
\ /
C C
C
~
C C C C
~ ~ ~
C C C

CADEIA HOMOGNEA
Tambm pode ser chamada de homocclica no caso de
cadeia cclica. a cadeia que possui s carbonos entre
carbonos.
C C
~ ~
C C C C C N
~ ~
C C

CADEIA HETEROGNEA
Tambm pode ser chamada de heterocclica no caso de
cadeia cclica. a cadeia que possui pelo menos um
tomo diferente de carbono entre carbonos.
C C
~ ~
C C O C C

CADEIA SATURADA
a cadeia que possui somente simples ligaes entre os
carbonos.

C = C C C C

C O C C C C

CADEIA INSATURADA
a cadeia que possui pelo menos uma dupla e/ou tripla
ligao entre carbonos.


CADEIA NORMAL
a cadeia que constituda de tomos de carbonos
primrios e/ou secundrios.

C C N C C C
CADEIA RAMIFICADA
a cadeia constituda de pelo menos um carbono tercirio
e/ou quaternrio.
C C
~ ~
C C C C

CADEIA ALICCLICA
a cadeia carbnica cclica que no possui anel
aromtico.


CADEIA AROMTICA
a cadeia carbnica que possui anel aromtico. O ncleo
benznio, principal anel aromtico, uma sequncia
cclica de seis tomos de carbono, fazendo simples e
duplas ligaes alternadamente.


CADEIA AROMTICA MONONUCLEADA
a cadeia carbnica que possui somente um nico anel
aromtico.

CADEIA AROMTICA POLINUCLEADA
a cadeia carbnica que possui pelo menos dois anis
aromticos.


CADEIA CARBNICA MISTA
a cadeia carbnica que possui uma miscelnia das
outras classificaes.



Please purchase PDF Split-Merge on www.verypdf.com to remove this watermark.
Incluso para a Vida Qumica C

Pr-Vestibular da UFSC 3
Exerccios de Sala #

1. Sobre a frmula estrutural abaixo, verdadeiro
afirmar que:
H H H H H
~ ~ ~ ~ ~

H  C  C  C  C  C  OH
~ ~ ~ ~ ~

H CH
3
CH
3
H CH
3


01. Apresenta 7 carbonos sp.
3

02. Apresenta cadeia saturada e homognea.
04. Apresenta 4 carbonos primrios.
08. Sua cadeia carbnica heterognea e ramificada.

2. A cadeia
H
3
C  C = CH  C { C  H
~
CH
3

a) aberta, heterognea, saturada e normal.
b) acclica, homognea, insaturada e normal.
c) acclica, homognea, etennica e ramificada.
d) aliftica, heterognea, ramificada e insaturada.
e) acclica, heterognea, etennica e ramificada.

Tarefa Mnima #

3. Observe a frmula:

As quantidades totais de tomos de carbono primrio,
secundrio e tercirio so respectivamente:
a) 5, 2 e 2. c) 3, 3 e 2. e) 5, 1 e 3.
b) 3, 2 e 2. d) 2, 3 e 4.

4. Na frmula H
2
C...x...CH-CH
2
-C...y...N

x e y representam, respectivamente, ligaes
a) simples e dupla.
b) dupla e dupla.
c) tripla e simples.
d) tripla e tripla.
e) dupla e tripla.

5. A frmula molecular de um hidrocarboneto com cadeia
carbnica
:
a) C
9
H
8

b) C
9
H
7

c) C
9
H
10

d) C
9
H
11

e) C
9
H
12


6. (UPotiguar-RN) A cadeia carbnica abaixo:
(CH
3
)
2
CH CH = CH (CH
2
)
3
CH
2
OH
classificada como:
a) insaturada, ramificada e homognea;
c) saturada, ramificada e heterognea;
b) saturada, normal e homognea;
d) insaturada, ramificada e heterognea.

Tarefa Complementar #

7. (Cefet-RJ) O pireno, hidrocarboneto de ncleos
condensados,obtido do alcatro de hulha, solvel em ter
e insolvel em gua, apresenta a frmula estrutural plana.

A frmula molecular do pireno :
a) C
14
H
8
c) C
16
H
10
e) C
18
H
12

b) C
15
H
10
d) C
17
H
12


8. (UFSM-RS) O mirceno, responsvel pelo gosto azedo
da cerveja, representado pela estrutura:

Considerando o composto indicado, assinale a alternativa
correta quanto classificao da cadeia.
a) acclica, homognea, saturada;
b) acclica, heterognea, insaturada;
c) cclica, heterognea, insaturada;
d) aberta, homognea, saturada;
e) aberta, homognea, insaturada.

9. (UFJF) Qual dos compostos relacionados abaixo
apresenta simultaneamente uma cadeia carbnica aberta,
ramificada, heterognea e insaturada?

10. (Uniderp-MS) Quem exerce atividade em laboratrio
qumico necessita de algumas informaes sobre um
produto que vai ser utilizado. Assim sendo,
recomendvel que o recipiente que contm o produto seja
rotulado no apenas com a indicao de frmulas. Por
exemplo, a frmula C3H9N pode indicar as molculas I
ou II com estruturas e pontos de ebulio diferentes, isto
,
Please purchase PDF Split-Merge on www.verypdf.com to remove this watermark.
Qumica C Incluso para a Vida

Pr-Vestibular da UFSC
4


As cadeias orgnicas apresentadas pelas molculas I e II
so, respectivamente, do tipo:

a) Saturada e mista. d) Heterognea e normal.
b) Insaturada e normal. e)Heterognea e insaturada.
c) Homognea e saturada.

11. (Mackenzie-SP) Das frmulas abaixo, a nica que
possui cadeia carbnica heterognea, saturada e normal, :
a) H
3
CCH
2
CH
2
MgI d) H
3
CCH
2
OCH
3


12. (Univali-SC) No ms de junho deste ano, 10 milhes
de belgas tiveram de fazer malabarismos para atender
uma necessidade bsica do dia a dia: comer.
Acontecimento impensvel em um pas rico, desenvolvido
e altamente organizado como a Blgica. Frango, ovos,
carne de boi e de porco, leite, manteiga e at chocolates
foram retirados de circulao pelo governo porque
estavam contaminados, em escala ainda indeterminada,
por dioxinas. Dioxinas so substncias altamente txicas
que podem causar uma srie de doenas, do diabetes ao
cncer. A contaminao aconteceu pela rao fornecida
aos animais nas granjas e fazendas; ela pertence ao grupo
de substncias chamadas poluentes persistentes;
compostos txicos gerados a partir de processos
industriais, como a produo de plsticos PVC e a
incinerao do lixo. Est presente no meio ambiente de
qualquer pas medianamente industrializado. Por meio da
cadeia alimentar chega aos seres humanos.
Veja, 16/6/99
Sua cadeia :

a) alicclica d) homocclica
b) aromtica e) saturada
c) aliftica

13. O cido acetilsaliclico de frmula:


Um analgsico de diversos nomes comerciais (AAS,
Aspirina, Buferin e outros), apresenta cadeia carbnica:

a) Acclica, heterognea, saturada, ramificada.
b) Mista, heterognea, insaturada, aromtica.
c) Mista, homognea, saturada, alicclica.
d) Aberta, heterognea, saturada, aromtica.
e) Mista, homognea, insaturada, aromtica.

14. O 1, 2- benzopireno, mostrado na figura adiante,
agente cancergeno encontrado na fumaa do cigarro um
hidrocarboneto:
a) Aromtico polinuclear.
b) Aromtico mononuclear.
c) Aliftico saturado.
d) Aliftico insaturado.
e) Alicclico polinuclear.


UNIDADE 3

NOMENCLATURA OFICIAL DOS
COMPOSTOS ORGNICOS

RAIZ DESINNCIA TERMINAO
Indica o n
o
de
carbonos
Indica o tipo de
ligao
Indica a funo do
composto
RAIZ:
1 C = met 4 C = but 7 C = hept 10 C = dec
2 C = et 5 C = pent 8 C = oct 11 C = undec
3 C = prop 6 C = hex 9 C = non 12 C = dodec
DESINNCIA:
AN = S ligaes simples
EN = Ligao dupla
IN = Ligao tripla
DIEN = Duas duplas

RADICAIS ORGNICOS
So derivados de hidrocarbonetos com uma ou mais
valncias livres.
OBS.: So pr-requisitos para a nomenclatura dos
compostos orgnicos ramificados.


H H
~ ~
H C H H C x + x H
~ ~
H H


















Please purchase PDF Split-Merge on www.verypdf.com to remove this watermark.
Incluso para a Vida Qumica C

Pr-Vestibular da UFSC 5
NOMENCLATURA DOS PRINCIPAIS
RADICAIS ORGNICOS (terminao IL)

CH
3
(metil) CH
3
CH
2
(etil)

CH
3
CH
2
CH
2
(n-propil)

~
CH
3
CH CH
3
(isopropil)

CH
3

~
CH
3
C (tercbutil)
~
CH
3

~
CH
3
C CH
2
(isobutil)
~
CH
3


H
2
C = CH (vinil)

H
2
C = CH CH
2
(alil)

HC { C CH
2
(propargil)


CH
2

(benzil) (fenil)

Exerccios de Sala #

1. CH
3


H
3
C CH CH
2

*
CH CH
3


CH
2


CH
3

O nome dos radicais ligados ao carbono assinalado esto
nas opes
01. metil. 04. isopropil. 16. terc-butil.
02.etil. 08. isobutil.

2. O nome dos radicais circulados esto nas opes
8 7 6 5 4 3 2 1
H
3
C CH
2
CH CH
2
CH CH CH
2
CH
3


CH
3
CH
2
CH
3


CH
2


CH
3

01. metil;
02. etil;
04. isopropil;
08. isobutil;
16. terc-butil;
32. n-propil.

UNIDADE 4

NOMENCLATURA DE COMPOSTOS ORGNICOS

Nomenclatura Oficial (IUPAC):
As regras abaixo valem para todas as funes orgnicas:

1
o
) Definir a cadeia principal que deve:
a) Conter o grupo funcional;
b) Conter a(s) insaturao(es);
c) Conter o maior n
o
possvel de tomos de carbono;
d) Ter o maior n
o
possvel de ramificaes.

2
o
) Numerar a cadeia principal a partir da extremidade
mais prxima:
a) Do grupo funcional;
b) Da(s) insaturao(es);
c) De uma ramificao;
d) Se houver vrias ramificaes, considere a regra dos
menores nmeros.

3
o
) Dar o nome:
a) Da(s) ramificao(es) precedido(s) pelo n
o
do tomo
de carbono pelo qual est(ao) ligada(s) cadeia
principal;
b) Os nomes das ramificaes devem estar em ordem
alfabtica;
c) Por ltimo, o nome da cadeia principal (alcanos tm
terminao ano) precedido do n
o
do tomo de
carbono onde est(o) localizada(s) a(s)
insaturao(es) ou o grupo funcional.

Funes Orgnicas
So compostos que apresentam propriedades qumicas
semelhantes devido presena de um mesmo grupo
funcional. As principais funes orgnicas esto
representadas abaixo:

Hidrocarbonetos
Haletos
lcoois
Enis
Fenis
teres
Funes Cetonas
Orgnicas Aldedos
cidos carboxlicos
Anidridos
steres
Sais orgnicos
Aminas
Amidas

Hidrocarbonetos
So compostos orgnicos formados, exclusivamente por
tomos de carbono e hidrognio. Os hidrocarbonetos (HC)
esto divididos em classes ou subfunes, representadas
no esquema abaixo:
Alcanos, Alcenos, Alcinos
Hidrocarbonetos Alcadienos, Ciclanos
Ciclenos
HC aromticos

Please purchase PDF Split-Merge on www.verypdf.com to remove this watermark.
Qumica C Incluso para a Vida

Pr-Vestibular da UFSC
6
Alcanos (Parafinas)
So hidrocarbonetos alifticos (de cadeia aberta) saturados
(apenas ligaes simples).

Frmula Geral C
n
H
2n+2


Nomenclatura Oficial: terminao ano.

Ex.:
8 7 6 5 4 3 2 1
H
3
C CH
2
CH CH
2
CH CH CH
2
CH
3


CH
3
CH
2
CH
3


metil no C n
o
6 CH
2


CH
3

metil no C n
o
3

propil no C n
o
4

Nome do composto:
3,6-dimetil-4-propiloctano
Alcenos (Olefinas)
So hidrocarbonetos alifticos (de cadeia aberta)
insaturados pela presena de uma dupla ligao na cadeia.
Frmula Geral C
n
H
2n


Nomenclatura Oficial: terminao eno.

Ex.:
5 4 3 2 1
H
3
C CH
2
CH
2
C = CH
2


CH
2


CH
2


CH
3



propil no C n
o
2

Nome do composto:
2-propil-1-penteno

Alcinos
So hidrocarbonetos alifticos (de cadeia aberta)
insaturados pela presena de uma tripla ligao na
cadeia.

Frmula Geral C
n
H
2n-2


Nomenclatura Oficial: terminao ino.

Ex.:
CH
3


H
3
C C C { C CH CH
3


CH
3
CH
3


2,2,4-trimetil-3-hexino

Classificao:

x Alcinos verdadeiros: possuem a tripla ligao na
extremidade da cadeia;

Ex.: H
3
C C{CH

x Alcinos falsos: possuem a tripla ligao no meio da
cadeia;
Ex.: H
3
C C{CCH
3


Alcadienos
So hidrocarbonetos alifticos (de cadeia aberta)
insaturados pela presena de duas duplas na cadeia.

Frmula Geral C
n
H
2n-2


Nomenclatura Oficial: terminao dieno.

Ex.:
1 2 3 4
H
2
C = C C = CH
2


CH
2
CH
3


CH
3

2-etil-3-metil-1,3-butadieno

Classificao:
x Alcadienos c/ duplas acumuladas(alnicos):
No apresentam nenhuma ligao simples entre as duas
duplas, que esto em um nico tomo de carbono;

Ex.: H
2
C = C = CH CH
3


x Alcadienos c/ duplas conjugadas(eritrnicos):
Apresentam uma ligao simples entre as duas duplas;

Ex.: H
2
C = CH CH = CH
2


x Alcadienos c/ duplas isoladas (allicos):
Apresentam duas ou mais ligaes simples entre as duas
duplas;

Ex.: H
2
C = CH CH
2
CH = CH
2


Ciclanos (Cicloalcanos)
So hidrocarbonetos de cadeia cclica saturada.

Frmula Geral C
n
H
2n


Nomenclatura Oficial: terminao ano com prefixo
ciclo.
Havendo apenas uma ramificao, no necessrio
numerar a cadeia principal, que o anel. Do contrrio,
com duas ou mais ramificaes, deve-se numerar a partir
de uma das ramificaes no sentido das outras, de forma
que se tenham os menores n
os
possveis.
Ex.:
CH
2


CH
2
CH
2
=
Ciclopropano


3
H
3
C
4 2
CH
3



5 1

CH
3
CH
2
CH
3

Metilciclobutano 1-etil-2,4-dimetilciclopentano

Please purchase PDF Split-Merge on www.verypdf.com to remove this watermark.
Incluso para a Vida Qumica C

Pr-Vestibular da UFSC 7
Ciclenos (Cicloalcenos)
So hidrocarbonetos de cadeia cclica insaturada pela
presena de uma dupla ligao no anel.

Frmula Geral C
n
H
2n-2


Nomenclatura Oficial: terminao ano com prefixo
ciclo.
Ex.:

4

5 3
CH
3



1 2


ciclobuteno 3-metilciclopenteno

Hidrocarbonetos Aromticos
So aqueles que apresentam pelo menos um anel
aromtico, que se caracteriza pela ressonncia das
ligaes S (pi).

O HC aromtico mais simples o benzeno. A
nomenclatura de aromticos mononucleares como a dos
derivados do benzeno.
Ex.:

CH
3




Benzeno Tolueno (usual)
Metilbenzeno

Quando o anel tiver duas ramificaes, podem-se usar os
prefixos orto (posio 1,2), meta (posio 1,3) ou
para (posio 1,4).

D D

E E

E E
D D
Naftaleno

D J D

E E

E E
D J D
Antraceno




Fenantreno



Exerccios de Sala #

1. Faa a unio dos radicais isobutil e terc-butil e assinale
a(s) opo(es) correta(s):
01. A cadeia resultante aberta, homognea, saturada e
ramificada.
02. A cadeia apresenta dois carbonos tercirios e cinco
primrios.
04. O composto formado um hidrocarboneto do tipo
alcano.
08. O nome do composto 2,4-dimetil-pentano.
16. A frmula molecular do composto resultante C
8
H
18

2. Assinale as alternativas corretas:
01. Alcanos so hidrocarbonetos de cadeia carbnica
aberta e saturada.
02. Todos os alcenos apresentam carbono sp.
2

04. O acetileno (etino) apresenta apenas carbonos sp.
08. Todos os alcenos apresentam cadeia insaturada por
dupla.
16. O propino um alcino verdadeiro.
32. O 2-butino um alcino falso.

3. O nome oficial do composto:

H
3
C CH CH = C CH
2
CH CH
2
CH
2
CH
3

_
CH
3
CH
3
CH
2


CH CH
3


CH
3

est na opo
01. dimetil 2,3 terc-butil 6 noneno 3.
02. trimetil 2,4,8 n-propil 6 noneno 3.
04. trimetil 2,5,8 n-propil 6 octeno 3.
08. trimetil 2,5,8 n-propil 5 octeno 4.
16. dimetil 2,4 isobutil 6 noneno 3.

Tarefa Mnima #

4. (Mackenzie-SP) As frmulas do etano, do eteno e do
propino so, respectivamente,

Ento as frmulas do propano, do propadieno e do etino,
naordem mencionada, so:


Please purchase PDF Split-Merge on www.verypdf.com to remove this watermark.
Qumica C Incluso para a Vida

Pr-Vestibular da UFSC
8
5. (Unifor-CE) A frmula geral C
n
H
2n+2
inclui muitos
compostos orgnicos. Refere-se srie dos
hidrocarbonetos cujo primeiro membro o
a) etino. d) etano.
b) eteno. e) carbono.
c) metano.

6. (Mackenzie-SP) Relativamente ao composto de
frmula estrutural H
3-
CH
2-
CH
2-
CH
3
, considere as
afirmaes:
I - um alcano.
II - Apresenta somente carbonos primrios em sua
estrutura.
III - Apresenta uma cadeia carbnica normal.
IV - Tem frmula molecular C
4
H
10
.

So corretas somente:
a) I, III e IV b) II, III e IV c) I e II d) I e IV
e) I e III

7. (UFSCarlos) Considere as afirmaes seguintes sobre
hidrocarbonetos.
I - Hidrocarbonetos so compostos orgnicos constitudos
somente de carbono e hidrognio.
II - So chamados de alcenos somente os hidrocarbonetos
saturados de cadeia linear.
III - Cicloalcanos so hidrocarbonetos, saturados de
frmula geral C
n
H
2n
.
IV - So hidrocarbonetos aromticos: bromobenzeno,
pnitrotolueno e naftaleno.

So corretas as afirmaes:
a) I e III, apenas. d) III e IV, apenas.
b) I, III e IV, apenas. e) I e II e IV, apenas.
c) II e III, apenas.

Tarefa Complementar #

8. (PUC-PR) Alcinos so hidrocarbonetos
a) alifticos insaturados com dupla ligao.
b) alicclicos insaturados com tripla ligao.
c) alifticos insaturados com tripla ligao.
d) alifticos saturados.
e) alicclicos saturados.

9. (Mackenzie-SP) Relativamente ao composto de
frmula H
2
C=CHCH=CH, incorreto afirmar que
a) um hidrocarboneto.
b) possui dois carbonos secundrios em sua estrutura.
c) um alceno.
d) sua frmula molecular C
4
H
6
.
e) ismero do ciclobuteno.

10. (Univali-SC) A partir de novembro do prximo ano,
chegar ao estado de Santa Catarina gs natural
proveniente da Bolvia, via Mato Grosso do Sul, passando
por So Paulo, Paran, Santa Catarina e Rio Grande do
Sul. O gs natural utilizado com xito nos pases
desenvolvidos e estar disponvel para uso industrial,
comercial e residencial. A mdio prazo, trar economia
aos seus usurios substituindo o emprego de leo diesel
nas indstrias. As vantagens ecolgicas so as primeiras
destacadas por quem conhece os resultados do uso do gs
natural. O gs no poluente, porque no emite cinzas e
tem queima de 97%, no necessita de tratamento de
efluentes gasosos e no interfere na colorao dos
produtos fabricados (especialmente a cermica).
Registros da Petrobrs responsvel pelo gasoduto Bolvia
Brasil.
Este texto se refere principalmente ao gs
a) etano. c) benzeno. e) acetileno.
b) propano. d) metano.

11. (Univali-SC) Nos vazamentos de GLP (Gs
Liquefeito de Petrleo, ou seja, o gs de cozinha), alguns
procedimentos devem ser seguidos, como: no inalar o
gs, no acender fsforos ou isqueiros, no acionar o
interruptor de luz, procurar levar o botijo para local
ventilado e chamar a assistncia tcnica. Isto porque seu
contedo formado por uma mistura de Propano e
Butano, dois hidrocarbonetos txicos e inflamveis,
classificados como
a) alcadienos. b) ciclanos. c) alcinos.
d) alcenos. e) alcanos.

12. (UFRJ) Uma mistura de hidrocarbonetos e aditivos
compe o combustvel denominado gasolina. Estudos
revelaram que quanto maior o nmero de hidrocarbonetos
ramificados, melhor a performance da gasolina e o
rendimento do motor.
Observe as estruturas dos hidrocarbonetos abaixo:
O hidrocarboneto mais ramificado o de nmero:
a) IV b) III c) II d) I

13. (FEI-SP) O nome do composto formado pela unio
dos radicais etil e terc-butil
a) 2,2-dimetilbutano d) n-hexano
b) 2-metilpentano e) 2-etilpropano
c) 3-metilpentano

14. (U.Catlica de Salvador-BA)
O composto representado o isso-octano, um
hidrocarboneto com excelente capacidade antidetonante e,
por isso, associado qualidade da gasolina.

Sobre esse composto, pode-se afirmar:
a) o dimetil-isopentano.
b) o 2,2,4-trimetilpentano.
c) hidrocarboneto aromtico.
d) Apresenta cadeia normal insaturada.
e) Possui grande reatividade qumica em relao aos
demais hidrocarbonetos.

Please purchase PDF Split-Merge on www.verypdf.com to remove this watermark.
Incluso para a Vida Qumica C

Pr-Vestibular da UFSC 9
15. (PUC-PR) Dada a estrutura

A funo a qual pertence este composto e seu nome oficial
esto corretamente indicados na alternativa:
a) Alcano 4-fenil-3-hepteno.
b) Alceno 4-benzil-3-hepteno.
c) Hidrocarboneto 1-metil-3-fenil-2-hexeno.
d) Hidrocarboneto 4-fenil-3-hepteno.
e) Hidrocarboneto 4-fenil-4-hepteno.

16. A borracha natural um lquido branco e leitoso,
extrado da seringueira, conhecido como ltex. O
monmero que origina a borracha natural o metil-1, 3-
butadieno,

o qual correto afirmar que:

a) um hidrocarboneto de cadeia saturada e ramificada.
b) um hidrocarboneto aromtico.
c) Tem frmula molecular C
4
H
5

d) Apresenta dois carbonos tercirios, um carbono
secundrio e dois carbonos primrios.
e) um hidrocarboneto insaturado de frmula molecular
C
5
H
8

17. As afirmaes a seguir se referem substncia de
frmula estrutural mostrada na figura adiante:

I - 4-etil-3-isopropil-2,4-dimetil-2-hexeno o seu nome
oficial (I.U.P.A.C.).
II - Sua cadeia carbnica aberta, insaturada, homognea
e ramificada.
III - um alcino (ou alquino).
IV - Trata-se de um alcano ramificado.

Das afirmaes feitas, so corretas apenas:
a) I e II. b) II e III. c) I e III.
d) II e IV. e) I, II e III.

18. Faa a unio dos radicais isobutil e terc-butil e
assinale a(s) opo(es) correta(s):
01. A cadeia resultante aberta, homognea, saturada e
ramificada.
02. A cadeia apresenta dois carbonos tercirios e cinco
primrios.
04. O composto formado um hidrocarboneto do tipo
alcano.
08. O nome do composto 2,4-dimetil-pentano.
16. A frmula molecular do composto resultante C
8
H
18.

19. Analise os compostos a seguir e assinale a(s)
proposio(es) verdadeira(s):
I) Tolueno. II) Benzeno. III) 1,2-dimetil-benzeno.

01. Todos so aromticos ramificados.
02. O composto III pode ser chamado de orto-xileno.
04. I e II so compostos no ramificados.
08. O composto I apresenta um carbono tercirio e cinco
carbonos secundrios, alm de um primrio.
16. O composto III, apresenta dois carbonos tercirios e
quatro secundrios, alm de um primrio.
32. I e III so aromticos ramificados sendo que o
composto I pode ser chamado de metil-benzeno.

20. Sobre o metil-ciclo-hexano correto afirmar:
01. um alcano ramificado.
02. Sua frmula molecular C6H12.
04. Apresenta cadeia mista, homognea e saturada.
08. um ciclano ramificado.

21. A respeito dos hidrocarbonetos a seguir, assinale a
opo falsa:

a) So todos aromticos.
b) Pertencem todos mesma srie homloga.
c) Suas propriedades fsicas variam de forma previsvel.
d) So chamados respectivamente: benzeno; metil-
benzeno; 1,2-dimetil benzeno; 1,3,5-trimetil bezeno;
1,2,3,4-tetrametil benzeno.
e) Possuem a mesma frmula mnima (CH)n

22. Nas indstrias de couro, encontram-se presentes
alguns agentes qumicos potencialmente nocivos; dentre
eles, os representados a seguir:

Os nomes oficiais desses compostos, na ordem dada, so
a) benzeno, xileno e tolueno.
b) benzeno, tolueno e p-xileno.
c) toluol, benzil e fenol.
d) benzil, toluol e m-xileno.
e) benzeno, tolueno e o-xileno.

23. A alternativa que contm o nome correto do
composto a seguir :

a) 3 - etil - 1 - metil benzeno.
b) 2 - metil - 5 - etil benzeno.
c) 1 - etil - 5 - metil benzeno.
Please purchase PDF Split-Merge on www.verypdf.com to remove this watermark.
Qumica C Incluso para a Vida

Pr-Vestibular da UFSC
10
d) 1 - metil - 2 - etil benzeno.
e) 2 - metil - 3 - etil - benzeno.

24. Indique a alternativa em que, no composto a seguir,
os grupos so ligados ao N:

a) -naftil - o-toluil - m-toluil.
b) -naftil - o-toluil - p-toluil.
c) -naftil - o-toluil - m-toluil.
d) -naftil - m-toluil - p-toluil.
e) -naftil - m-toluil - p-toluil.

25. No rtulo de um solvente comercial h indicao de
que contm apenas hidrocarbonetos alifticos. A partir
dessa informao se conclui que esse solvente no dever
conter, como um de seus componentes principais, o:
a) n-hexano. c) heptano. e) pentano.
b) tolueno. d) 2-hexeno.

UNIDADE 5
FUNES

Para outras funes de cadeia homognea se utiliza a
mesma estrutura de nomenclatura dos hidrocarbonetos,
trocando apenas a terminao.

LCOOIS: Possuem o grupo -OH (hidroxila) preso a um
carbono saturado.

Nomenclatura: Raiz + tipo de ligao + ol

(nomenclatura
usual)



lcoois podem ser classificados em primrios,
secundrios ou tercirios se a hidroxila estiver presa a
carbono primrio, secundrio e tercirio, respectivamente.
Nos exemplos acima, so primrios o etanol e o 1-
propanol, enquanto o 2-propanol e o cicloexanol so
secundrios.
ALDEDOS: Possuem o grupo carbonila (C = O)
em carbono primrio.
Nomenclatura: Raiz + tipo de lig. + al
(nomenclatura usual)
H
2
CBr CHBr CHO 2, 3-dibromopropanal

CETONAS: Possuem o grupo carbonila (C = O) em
carbono secundrio.
Nomenclatura: Raiz + tipo de lig. + ona:


CIDOS CARBOXLICOS: Possuem o grupo carboxila

Nomenclatura: cido + Raiz + tipo de lig. +ico


(nomenclatura usual)

ac. 2-vinil-butanodiico


FENOIS: Possuem o grupo -OH (hidroxila) preso a anel
aromtico.
Please purchase PDF Split-Merge on www.verypdf.com to remove this watermark.
Incluso para a Vida Qumica C

Pr-Vestibular da UFSC 11
Nomenclatura: hidroxi + nome do hidrocarboneto
correspondente:

O grupo funcional fica sempre com o menor nmero

Exerccios de Sala #

1. (UFSC) A seguir, temos compostos pertencentes a
quatro funes orgnicas distintas:


01. O composto I chama-se Butanal e o composto II,
Butanona.
02. O composto III chama-se cido Butanico e o
composto IV, Butano.
04. O composto III encontrado na manteiga ranosa.
08. A cadeia do composto IV acclica ou aberta, normal,
homognea e saturada.
16. A cadeia do composto II acclica ou aberta,
ramificada, heterognea e insaturada.

2. Os compostos mostrados na figura adiante:

01. Possuem a mesma frmula estrutural.
02. Possuem a mesma frmula molecular.
04. Pertencem mesma funo orgnica.
08. Possuem cadeia carbnica ramificada.
16. O primeiro composto recebe o nome comercial de
acetona.
32. O segundo composto recebe o nome oficial de
propanal.

UNIDADE 6
FUNES

Para funes de cadeia heterognea d-se nome a cada
grupo preso ao heterotomo, independentemente.

TERES: Podem ser considerados derivados dos lcoois
pela substituio do hidrognio da hidroxila (R-OH) por
um radical (-R) : R-O-R.
A nomenclatura mais utilizada d nome aos radicais
presos ao oxignio, precedido da palavra ter.



STERES: So derivados dos cidos carboxlicos pela
substituio do hidrognio da carboxila (R-COOH) por
um radical R: R-COO-R.
(((

D-se a terminao oato parte que provm do cido e ila
ao radical que substitui o hidrognio.












Please purchase PDF Split-Merge on www.verypdf.com to remove this watermark.
Qumica C Incluso para a Vida

Pr-Vestibular da UFSC
12
Exerccios de Sala #

1. Na reao de esterificao abaixo, os nomes dos
compostos I, II e III so, respectivamente:


a) cido etanico, propanal e metanoato de isopropila.
b) Etanal, propanol-1 e propanoato de etila.
c) cido etanico, propanol-1, etanoato de n-propila.
d) Etanal, cido propanico, metanoato de n-propila.
e) cido metanico, propanal e etanoato de n-propila.

2. Identifique a(s) associao(es) correta(s) das frmulas
estruturais com os nomes:


UNIDADE 7
FUNES NITROGENADAS

AMINAS: So derivados da amnia (NH
3
) obtidos pela
substituio de um, dois ou dos trs hidrognios da
amnia por radicais. Obtm-se, dessa maneira, aminas
primrias, secundrias e tercirias, respectivamente.
Nomeia-se cada radical preso ao nitrognio, em ordem de
tamanho e ramificao crescente, seguido da terminao
amina:



Nos exemplos acima, metilamina e fenilamina so aminas
primrias, propil-isopropilamina secundria e dimetil-
etilamina, terciria.
AMIDA: Derivadas dos cidos carboxlicos pela
substituio do grupo -OH por -NH
2


Nomenclatura: Nome do hidrocarboneto correspondente +
amida:


NITRILA: Possuem o grupo nitrila ou nitrilo

Nomenclatura: Nome do hidrocarboneto correspondente +
nitrila:

2-fenil-propanonitrila

NITROCOMPOSTO: R NO
2
Nomenclatura: a palavra Nitro + Nome do hidrocarboneto
correspondente.
CH
3
NO
2
Nitro-metano

OUTRAS FUNES:

Haletos
So compostos que apresentam tomos de halognios
ligados cadeia carbnica. A nomenclatura oficial feita
tratando o halognio como uma ramificao e a usual
considerando o radical ligado ao halognio.

CH
3
Cl clorometano

CH
3
CH
2
Cl cloroetano

CH
3
CH CH
3

2-bromopropano
Br

CH
3


CH
3
C CH
2
CH
2
CH
3
2-bromo-2-metilpentano

Br

H
2
C = CH Cl cloroeteno

CH
2
Cl
2
diclorometano

CHCl
3
triclorometano




Please purchase PDF Split-Merge on www.verypdf.com to remove this watermark.
Incluso para a Vida Qumica C

Pr-Vestibular da UFSC 13
Exerccios de Sala #

1. (UFSC) Apresente a(s) associao(es) correta(s)
entre a frmula e o nome dos compostos orgnicos
abaixo:


Tarefa Mnima #

2. As funes orgnicas presentes na substncia
epinefrina, so:

a) lcool, amina e aldedo.
b) hidrocarboneto, lcool e nitrila.
c) fenol, lcool e amina.
d) fenol, amina e hidrocarboneto.
e) lcool, fenol e amida.

3. Alm de ser utilizada na preparao do formol, a
substncia cuja formula mostrada na figura a seguir tem
aplicao industrial na fabricao de baquelite.

A funo qumica e o nome oficial desse composto so,
respectivamente:
a) cido carboxlico e metanico.
b) cetona e metanal.
c) lcool e metanol.
d) aldedo e metanal.
e) ter e metoximetano.

4. A seguir apresentada uma reao qumica onde
compostos de diferentes funes orgnicas tomam parte:

Os compostos orgnicos I, II e III pertencem,
respectivamente, s seguintes funes orgnicas:
a) aldedo, lcool, ter.
b) aldedo, cido carboxlico, ster.
c) cido carboxlico, aldedo, ster.
d) cido carboxlico, aldedo, ter.
e) cido carboxlico, lcool, ster.

5. A nomenclatura correta do composto da frmula
seguinte :

a) 3-4 dimetil hexanona-5
b) 3 metil 4 etil pentanona-2
c) 3-4 dimetil 2-hexanona
d) 3 metil 2 etil pentanona-4
e) 3 sec butil butanona-2

6. Os ces conhecem seus donos pelo cheiro. Isso se deve
ao fato de os seres humanos apresentarem junto pele
glndulas que produzem e liberam cidos carboxlicos. A
mistura desses cidos varia de pessoa para pessoa, o que
permite a animais de faro bem desenvolvido conseguir
discrimin-la. Com o objetivo de testar tal discriminao,
um pesquisador elaborou uma mistura de substncias
semelhantes produzida pelo dono de um co.
Para isso, ele usou substncias genericamente
representadas por:
a) RCHO c) RCH
2
OH
b) RCOOH d) RCOOCH
3



7. O composto abaixo responsvel pelo sabor de banana
nos alimentos

Dentre as afirmaes, est(o) correta(s):
01. Possui quatro carbonos secundrios e dois tercirios.
02. Possui uma ligao pi e as outras ligaes so sigma.
04. Possui funes cetona e ter.
08. Apresenta cadeia mista, normal, homognea e
saturada.
16. Apresenta cadeia aberta, ramificada, heterognea e
insaturada.
32. Possui funo ster.
64. Todas as ligaes entre os tomos de carbono e
hidrognio so ligaes sigma.

Tarefa Complementar #

8. Na estrutura da tirosina,

h radicais caractersticos das funes:
a) amina, lcool e fenol.
b) nitrila, fenol e aldedo.
c) lcool, ter e cido carboxlico.
d) amida, lcool e cetona.
e) amina, fenol e cido carboxlico.

Please purchase PDF Split-Merge on www.verypdf.com to remove this watermark.
Qumica C Incluso para a Vida

Pr-Vestibular da UFSC
14
9. Um composto, com frmula estrutural a seguir, absorve
radiaes ultravioleta, sendo usado em preparaes de
bronzeadores.

Nesta frmula temos os grupos funcionais:
a) amina e um cido.
b) amina e um lcool.
c) amida e um cido.
d) amida e um lcool.

10. Na adrenalina, cuja frmula estrutural dada a seguir,
podemos identificar os seguintes grupos funcionais:

a) fenol, lcool e amina.
b) lcool, nitrocomposto e aldedo.
c) lcool, aldedo e fenol.
d) enol, lcool e cetona.
e) cetona, lcool e fenol.

11. O nome do composto a seguir, que pode ser usado
para dar o sabor "morango" balas e refrescos :

a) etanoato de butila.
b) butanoato de etila.
c) cido etil-n-propil-metanico.
d) propanoato de metila.
e) butanoato de metila.

12. Na soluo contida num frasco com picles, entre
outras substncias, encontra-se o cido actico, cuja
frmula estrutural plana :


13. Dentre os compostos:

So cetonas:
a) apenas I, III e IV
b) apenas I, IV e V
c) apenas II, III e IV
d) apenas II, IV e V
e) apenas II, V e VI

14. A substncia com frmula a seguir um

a) diol.
b) dialdedo.
c) cido dicarboxlico.
d) anidrido duplo.
e) hidrxi-cido.

15. " O nome oficial da glicerina, representada na figura a
seguir, -___X___, tratando-se de um ___Y___"

Completa-se corretamente a afirmao acima quando X e
Y so substitudos, respectivamente por
a) 1, 2, 3- propanotriol e trilcool.
b) lcool proplico e trilcool.
c) propanotrial e trialdedo.
d) ter proplico e politer.
e) 1, 2, 3- tripropanol e trialdedo.

16. Com base nos compostos orgnicos representados a
seguir e nos conhecimentos sobre cidos carboxlicos e
teres, pode-se afirmar:

01. Os compostos A, B e D so cidos carboxlicos.
02. O composto E um ster.
04. O composto B o benzoato de metila.
08. O composto C pode ser obtido pela reao do etanol
com o cido etanico.
16. O composto D o cido butanico.
32. Os cidos carboxlicos, em gua, so cidos fortes.

UNIDADE 8

ISOMERIA

Ismeros so compostos que apresentam a mesma frmula
molecular, mas tm diferentes frmulas estruturais. Em
outras palavras, tm os mesmos elementos qumicos
constituintes na mesma quantidade, mas distribudos de
forma diferente.
Please purchase PDF Split-Merge on www.verypdf.com to remove this watermark.
Incluso para a Vida Qumica C

Pr-Vestibular da UFSC 15
Existem vrios tipos de isomeria, sendo
classificada sempre em funo da diferena entre os
compostos e nunca da semelhana.

Tipos de Isomeria:

ptica
trans cis ou geomtrica
meria estereoiso ou espacial
dinmica ou tautomeria
o compensa ou metameria
funo de
posio de
cadeia de
plana


ISOMERIA PLANA

a) Isomeria de cadeia: os ismeros diferem pelo tipo de
cadeia.
Exemplos:



b) Isomeria de posio: os ismeros diferem pela
posio de um grupo funcional, de uma insaturao
ou de uma ramificao.
Exemplos:





c) Isomeria de funo: os ismeros diferem pela
funo.
Exemplos:


d) Isomeria de compensao ou metameria: os
ismeros diferem pela posio de um heterotomo na
cadeia
Exemplos:


e) Isomeria dinmica ou tautomeria: um caso
particular de isomeria, onde os ismeros coexistem
em equilbrio. Geralmente ocorre entre:
aldedo e enol equilbrio aldo-enlico
cetona e enol equilbrio ceto-enlico

Please purchase PDF Split-Merge on www.verypdf.com to remove this watermark.
Qumica C Incluso para a Vida

Pr-Vestibular da UFSC
16
Exerccios de Sala #

1. (CESCEA-SP) A isomeria que ocorre entre o 1-
propanol e o 2-propanol chamada
a) de cadeia.
b) funcional.
c) de compensao.
d) de posio.
e) isomorfismo.

2. (UFAL) So ismeros de cadeia:
a) propino e ciclopropeno.
b) cloroetano e bromoetano.
c) 1 - buteno e 2 - butino.
d) propano e propeno.
e) benzeno e tolueno.

UNIDADE 9

ISOMERIA ESPACIAL

Neste tipo de isomeria a diferena entre os compostos est
no arranjo espacial dos tomos.

ISOMERIA GEOMTRICA OU CIS-TRANS

Pode ocorrer de duas formas:

x Em Cadeia Fechada: Ocorre quando a molcula tem
pelo menos dois ligantes diferentes dos demais. Caso estes
dois ligantes estejam do mesmo lado do plano formado
pelo anel, dizemos que o ismero CIS, do contrrio, com
os ligantes um em cada lado do plano, dizemos que o
ismero trans.

x TRANS.
Ex.:
H

OH H OH
Ismero CIS
H H

H

OH H H
Ismero TRANS
H OH

x Em Compostos com uma dupla ligao: Para que este
tipo de molcula forme ismeros geomtricos,
necessrio que os dois ligantes de cada tomo de carbono
que faz a dupla sejam diferentes um do outro. O ligante de
um carbono pode ser igual ao do outro. Caso estes dois
ligantes estejam do mesmo lado do plano formado pela
ligao dupla, dizemos que o ismero CIS, do contrrio,
com os ligantes um em cada lado do plano, dizemos que
o ismero TRANS. Se no houver nenhum ligante igual
ao outro, considere os dois mais simples
Ex.:
H H
C = C Ismero CIS
Cl OH

H OH
C = C Ismero TRANS
Cl H

ISOMERIA PTICA
Ocorre em compostos cujos ismeros diferem entre si
apenas no desvio da luz polarizada.
Um composto orgnico, para apresentar isomeria
ptica, deve ter molcula assimtrica (no tem plano de
simetria); nesse caso ele recebe a seguinte classificao:
Dextrgiro direita a para polarizada luz a Desvia


Levgiro esquerda a para polarizada luz a Desvia


Se o composto orgnico no for assimtrico, no
haver desvio da luz polarizada, consequentemente no
apresentar isomeria ptica.

Mistura Racmica: formada por 50% de dextrgiro e
50% de levgiro. Portanto, opticamente inativa e no
desvia a luz polarizada.
Enantiomorfos ou Antpodas pticos: So ismeros
pticos em que um a imagem especular do outro.
Diasterioismeros: So ismeros pticos em que um no
a imagem especular do outro.

Carbono assimtrico (quiral)
aquele que apresenta quatro ligantes diferentes.

Essa molcula possui quatro carbonos assimtricos.
Quando um composto apresenta mais de um carbono
assimtrico, o nmero de ismeros opticamente ativos
pode ser determinado pela regra de VantHoff:
I = 2
n

I: ismeros opticamente ativos
n: nmero de C* diferentes
O nmero de misturas racmicas a metade do nmero de
ismeros opticamente ativos.

Quando um composto apresenta tomos de carbono
assimtricos iguais, os C* desviam a luz num mesmo
ngulo (D).
Se os carbonos desviam a luz polarizada em sentidos
opostos, o composto se torna opticamente inativo por
compensao interna (meso).








Please purchase PDF Split-Merge on www.verypdf.com to remove this watermark.
Incluso para a Vida Qumica C

Pr-Vestibular da UFSC 17
Exerccios de Sala #

1. (UFSCAR) Apresenta isomeria ptica:


2. (UFSC) Observe a estrutura do composto abaixo:


Assinale a(s) proposio(es) correta(s).
01. Possuem os grupos funcionais amina, amida, cido
carboxlico e ster.
02. Sua frmula apresenta trs carbonos assimtricos.
04. Sua cadeia carbnica classificada como aberta,
ramificada, insaturada e heterognea.
08. A anlise da frmula estrutural acima permite
identificar cinco tomos de carbono com hibridizao
sp3e trs com hibridizao sp
2
.
16. Apresenta isomeria ptica, com 4 ismeros pticos
ativos e 2 inativos.

Tarefa Mnima #

3. O cido benzlico, o cresol e o anizol,
respectivamente:

so ismeros
a) de posio. d) de cadeia.
b) de funo. e) dinmicos.
c) de compensao.

4. Compare as frmulas a seguir:

Nelas verificamos um par de ismeros
a) cis-trans. d) de funo.
b) de cadeia. e) de posio.
c) de compensao.

5. O ismero plano de etanol (H
3
C CH
2
- OH) tem
frmula:


6. Assinale a alternativa que indica um par de ismeros:



7. O nmero de compostos ismeros formados durante a
monoclorao de 2 metil butano :
a) 1 b) 2 c) 3 d) 4 e) 5

8. Considere os seguintes pares de compostos
1) propanal e propanona.
2) 1-buteno e 2-buteno.
3) metoxi-metano e etanol.
4) n-pentano e neopentano.
5) metil-n-propilamina e di-etilamina.

So, respectivamente, ismeros de funo e cadeia:
a) 1 e 2 b) 2 e 3 c) 3 e 4
d) 4 e 5 e) 5 e 2

9. (ITA) Considere as afirmaes:
I - Propanal um ismero da propanona.
II - Etil-metil-ter um ismero do 2-propanol.
III - 1-Propanol um ismero do 2-propanol.
IV - Propilamina um ismero da trimetilamina.
Esto corretas:
a) Todas.
b) Apenas I, II e III.
c) Apenas I e II.
d) Apenas II e IV.
e) Apenas III e IV.

10. (PUC-MG) A 4-metil-2-pentanona usada como
solvente na produo de tintas, ataca o sistema nervoso
central, irrita os olhos e provoca dor de cabea"
(Globo Cincia, maio de 1997)
O composto considerado ismero funcional de:
a) 1-hexanol
b) hexanal
c) 4-metil-butanal
d) 4-metil-1-pentanol

Please purchase PDF Split-Merge on www.verypdf.com to remove this watermark.
Qumica C Incluso para a Vida

Pr-Vestibular da UFSC
18
11. A gasolina uma mistura de hidrocarbonetos diversos
que apresenta, dentre outros, os seguintes componentes:


Os pares de componentes I - II e I - III apresentam
isomeria plana, respectivamente, do tipo
a) cadeia e cadeia.
b) cadeia e posio.
c) posio e cadeia.
d) posio e posio.

12. (PUC-MG)Numere a segunda coluna relacionando os
pares de compostos com o tipo de isomeria na primeira
coluna.

lsomeria
1. de cadeia
2. de funo
3. de posio
4. de compensao
5. tautomeria

Pares
( ) etoxi-propano e metoxi-butano
( ) etenol e etanal
( ) etanoato de metila e cido propanico
( ) 1-propanol e 2-propanol
( ) n-pentano e neopentano

A numerao correta encontrada, de cima para baixo, :
a) 5 - 4 - 2 - 3 - 1
b) 3 - 1 - 2 - 4 - 5
c) 5 - 2 - 4 - 3 - 1
d) 3 - 5 - 1 - 2 - 4
e) 4 - 5 - 2 - 3 - 1

13. (PUC-Campinas) Considere as frmulas estruturais
seguintes:
I - CH
2
(OH) CH
2
(OH)
II - CH
3
- CH(OH) CH
2
CH
3

III - CH
2
(OH) - CH = CH CH
3

IV - CH
2
(OH) - CH = CH
2


H ismeros pticos e ismeros geomtricos,
respectivamente, nos compostos representados por:
a) I e II
b) I e IV
c) II e III
d) II e IV
e) III e IV


14. (Unioeste) Sobre a isomeria, assinale a(s)
afirmativa(s) correta(s):

01. (CH
3
)
2
C=CH
2
apresenta ismeros "cis" e "trans".
02. O 1,2-dicloro-1-penteno apresenta isomeria
geomtrica.
04. Na molcula do 3-metilpentanodiol-2,4 h dois
carbonos assimtricos.
08. O composto apresenta dois ismeros ativos e um
ismero inativo.
16. Os hidrocarbonetos podem apresentar isomeria de
posio e cadeia.
32. O butanal e a butanona so ismeros funcionais.
64. O ciclopropano e o propeno so ismeros de posio.

15. (ITA) Considere os seguintes compostos orgnicos:
I - 2-Cloro-butano.
II - Bromo-cloro-metano.
III - 3, 4-Dicloro-pentano.
IV - 1, 2, 4-Tricloro-pentano.

Assinale a opo que apresenta as quantidades corretas
de carbonos quirais nos retrospectivos compostos
anteriores:

a) 0 em I; 1 em II; 2 em III; 3 em IV.
b) 1 em I; 0 em II; 2 em III; 2 em IV.
c) 0 em I; 0 em II; 1 em III; 3 em IV.
d) 1 em I; 1 em II; 1 em III; 2 em IV.
e) 1 em I; 0 em II; 1 em III; 2 em IV.

Tarefa Complementar #

16. A partir das estruturas dos compostos de I a IV
abaixo, assinale a alternativa correta:


a) I e II no possuem ismero geomtrico.
b) I e II so ismeros de funo.
c) II e III possuem tautmeros.
d) III possui um ismero tico.
e) III e IV so ismeros de cadeia.

17. (UFRS) Uma reao tpica dos alcenos a adio de
halognios ligao dupla, formando compostos
dihalogenados vicinais, conforme exemplificado a seguir:
Please purchase PDF Split-Merge on www.verypdf.com to remove this watermark.
Incluso para a Vida Qumica C

Pr-Vestibular da UFSC 19
AlCl3
H
+


Em relao a essa equao, podemos afirmar que:
a) O composto II apresenta dois carbonos assimtricos.
b) O nome do produto formado 2,3-dibromo-3-metil-
butano.
c) O nome do composto I 2-metil-buteno-2.
d) O alceno pode apresentar isomeria geomtrica.
e) O nome do produto formado 2,3-dibromo-2-metil-
propano.

18. (Vunesp) O adoante artificial aspartame tem frmula
estrutural:

Sobre o aspartame, so feitas as seguintes afirmaes:
I - apresenta as funes ster e amida;
II - no apresenta isomeria ptica;
III - sua frmula molecular C
14
H
13
N
2
O
5


Das afirmativas apresentadas,
a) apenas I verdadeira.
b) apenas I e II so verdadeiras.
c) apenas I e III so verdadeiras.
d) apenas II e III so verdadeiras.
e) I, II e III so verdadeiras.

19. Assinale, entre os compostos a seguir, aquele que
possui um carbono assimtrico:


20. Considere os compostos:
I - Buteno -2
II - Penteno -1
III - 1,2 - Dimetil Ciclopropano
IV - Ciclobutano

Em relao possibilidade de isomeria cis-trans, pode-se
afirmar que:
a) aparece apenas no composto I.
b) ocorre em todos.
c) ocorre somente nos compostos II e IV.
d) aparece somente nos compostos I e III.
e) s no ocorre no composto I.

21. Dentre as seguintes substncias, qual apresenta
isomeria espacial?


22. Apresenta isomeria geomtrica:
a) propeno.
b) 1,2-butadieno.
c) 2-penteno.
d) tetrabromoetileno
e) 1,2-dimetilbenzeno

UNIDADE 10

REAES ORGNICAS

CLASSIFICAO DAS REAES ORGNICAS

Reaes de Substituio
Reaes em que ocorre troca de tomos ou radicais.
CH
3
CH
2
CH
3
+ Cl
2
CH
3
CHCH
3
+ HCl
~
Cl

Observe que houve substituio de um hidrognio do
propano por cloro.

CH
3
CH
2
OH + HCl CH
3
CH
2
OH + H
2
O

Observe que neste caso houve substituio da hidroxila
por um tomo de cloro.

HC Aromticos

x PROPRIEDADES QUMICAS:
Os hidrocarbonetos aromticos so muito estveis,
sofrendo basicamente reaes de substituio eletroflica.
Halogenao: Necessita de catalisador:

+ Cl Cl o Cl + HCl


Nitrao: Em meio cido concentrado:

+ NO
2
+
o NO
2
+ H
+




Obs.: O tolueno pode sofrer ataque eletroflico no anel
(ausncia de luz) ou substituio radicalar no grupo metila
(em presena de luz).

Reaes de Adio
Reaes em que h um aumento de tomos na molcula
resultante. O reagente deve ser insaturado (ou um ciclo de
3 ou 4 C)
Please purchase PDF Split-Merge on www.verypdf.com to remove this watermark.
Qumica C Incluso para a Vida

Pr-Vestibular da UFSC
20
H
+

K2CrO7
H
+

KmnO4
H
+

K2Cr2O7
H
+

KmnO4
H
+

KmnO4
H
+

Exemplo:
CH
2
= CH
2
+ HCl CH
3
CH
2
Cl

Observe que o HCl foi adicionado molcula de eteno.

+ HCl CH
3
CH
2
Cl

Formao de lcoois: Reao com gua em meio
cido. Lembre-se de Markownikoff:
O H entra no carbono mais hidrogenado.
OH

CH
3

CH

= CH
2
+ H
2
O o CH
3

CH

CH
3


Formao de Haletos: Reao com hidrcidos de
halognios. Obedece a regra de Markownikoff (exceto em
presena de perxido).
Cl

CH
3

CH

= CH
2
+ HCl o CH
3

CH

CH
3

Reaes de Eliminao

Ao contrrio da reao de adio, nesse tipo de reao
ocorre reduo no nmero de tomos na molcula
resultante, levando formao de duplas e triplas
ligaes.
Exemplo:

Exerccios de Sala #

1. (UFSC) Assinale as proposies corretas. Encontre, na
relao a seguir, os produtos da adio de 1 mol de HBr a
1 mol de cada um dos seguintes reagentes:
a) propeno
b) propino

01. 1,2 - dibromopropano.
02. 2 - bromopropano.
04. 1,2 - dibromopropeno.
08. 2 - bromopropeno.
16. 1 - cloro - 2 - bromopropano.
32. 1,2,3 - tribromopropano.
64. 1,1,1 tribromopropano

2. Observe as reaes de adio dadas aos seguintes
alcenos:


Assinale a opo que contm apenas a(s) reao(es) que
obedece(m) Regra de Markovnikov:
a) II.
b) V.
c) I e III.
d) II e IV.
e) IV e V.

UNIDADE 11

OXIDAO

Oxidao: Pode ser oxidao branda ou enrgica:
Oxidao Branda: Com KmnO
4
diludo ou K
2
Cr
2
O
7
,
sempre em meio cido. Forma um diol.

OH OH

CH
3

CH

= CH
2
o CH
3

CH

CH
2


Oxidao Enrgica: Com KmnO
4
em meio cido. A
dupla se quebra formando dois cidos.
O

CH
3

CH

= CH
2
o CH
3

C OH + H
2
CO
3


Oxidao de alcoois

Oxidao: Depende do tipo de lcool:
lcool primrio: Pode formar um aldedo
(oxidao branda) ou um cido carboxlico
(oxidao enrgica).
O

CH
3

CH
2

OH o CH
3

C

H

O

CH
3

CH
2

OH o CH
3

C OH

lcool secundrio: Forma uma cetona.

OH O

CH
3

CH

CH
3
o CH
3

C CH
3


lcool tercirio: no sofre oxidao.

Please purchase PDF Split-Merge on www.verypdf.com to remove this watermark.
Incluso para a Vida Qumica C

Pr-Vestibular da UFSC 21
[O]parcial

COMPLEMENTO:

OUTRAS REAES

Esterificao: quando reage com cidos.
O

CH
3

C OH + HO CH
2

CH
3


O p

CH
3

C O CH
2

CH
3
+ H
2
O

REAES DE POLIMERIZAO
Reaes em que duas ou mais molculas unem-se
formando uma molcula maior.
Por exemplo:
n H
2
C = CH
2
( CH
2
CH
2
) n
Etileno (monmero) Polietileno (polmero)

Se duas molculas se unirem, teremos um dmero;
se forem trs, um trmero, e se forem muitas, um polmero
(poli = muitos; meros = partes).

Exerccios de Sala #

1. (UFSC) Em relao aos compostos orgnicos e suas
reaes caractersticas, assinale a(s) proposio(es)
verdadeira(s):
01. Os lcoois secundrios produzem, por oxidao,
aldedos.
02. A queima de um hidrocarboneto produz gs carbnico
e gua.
04. Os aldedos no se oxidam to facilmente como as
cetonas.
08. A polaridade do grupo carbonila faz os aldedos serem
reativos.
16. Os lcoois primrios produzem, por oxidao, cetonas.

Tarefa Mnima #

2. (UFSC) Em relao aos compostos orgnicos e suas
reaes caractersticas, assinale a(s) proposio(es)
verdadeira(s):
01. Os lcoois secundrios produzem, por oxidao,
aldedos.
02. A queima de um hidrocarboneto produz gs carbnico
e gua.
04. Os aldedos no se oxidam to facilmente como as
cetonas.
08. A polaridade do grupo carbonila faz os aldedos serem
reativos.
16. Os lcoois primrios produzem, por oxidao, cetonas.

3. Em relao aos compostos orgnicos e suas reaes
caractersticas, assinale a(s) proposio(es)
verdadeira(s):
01. Um dos mtodos de obteno de alcanos a
hidrogenao cataltica de alcenos.
02. lcoois tercirios podem sofrer oxidao enrgica e
formar cidos carboxlicos.
04. Cetonas no sofrem oxidao.
08. Compostos cclicos com mais de cinco tomos de
carbono tendem a sofrer reaes de adio com
abertura da cadeia.
16. Um dos mtodos de obteno de lcoois a hidratao
de alcenos.

4. (UFSC) lcoois primrios, secundrios e tercirios,
quando tratados com agentes oxidantes, comportam-se de
maneiras diferentes. Veja os esquemas abaixo e
identifique os X, Y e Z em cada processo:

I H
3
C

CH
2

OH



II

OH

III H
3
C

CH

CH
3


01. X o cido actico.
02. Y e o etanal.
04. Z a propanona.
08. Y um cido carboxlico.
16. X um aldedo.
32. X igual a Z.
64. Y igual a Z.

5. (UFSC) Assinale a(s) proposio(es) verdadeira(s):

01 um lcool.


02. 2-Propanol um lcool secundrio.

04. tem carter bsico.


08. CH
3
CH
2
NH
2
tem carter bsico.

16. um cido carboxlico.


32. H
3
C N H uma amina terciria.

CH
3


6. (UFSC) Na reao de hidrlise do acetato de etila, em
meio cido, obtm-se como produto(s):
01. Acetato de sdio.
02. Acetato de metila.
04. cido actico
08. Metanol.
16. gua.

7. (FUVEST-SP) O composto obtido quando se faz
reagir 1 molcula de acetileno com 1 molcula de H
2
:
a) Etanol c)Eteno e) Etanico
b) Etanal d) Etino

X
[O]

Y X
Z [O]

Please purchase PDF Split-Merge on www.verypdf.com to remove this watermark.
Qumica C Incluso para a Vida

Pr-Vestibular da UFSC
22
H2SO4

140
o
C
KMnO4

excesso
KMnO4

8. (FEI) Um alceno de frmula molecular C
5
H
10
ao ser
oxidado com soluo cida de permanganato de potssio,
deu origem acetona e cido etanico em proporo
equimolar. O nome do alceno :

a) 1-penteno d) 2-metil-2-buteno
b) 2-penteno e) 2-etil propeno
c) 2-metil-1-buteno

Tarefa Complementar #

9. (UFSC) Complete as reaes:
01-CH
3

CH
2

Cl + NaOH o A + NaCl

CH
3


02- CH
3

C

Br + NaOH o B + NaBr

CH
3

04- CH
3

CH
2

CH
2

OH o C + H
2
O


08. CH
3

CH
2

OH o D + H
2
O


16. CH
3

CH

CH
2

CH
3

o E + H
2
O

OH
Assinale a(s) proposio(es) na(s) qual(is) os compostos
foram corretamente indicados:

01. A igual a: CH
3

CH
2

OH

02. B igual a: CH
3

Br

04. C : CH
3

CH
2

CH
2

O CH
2

CH
2

CH
3


O

08. D igual a: CH
3

C

H

O

16. E igual a: CH
3

C

CH
2

CH
3


10. (UFSC) Complete a sequncia de reaes:
3 HC{CH o X
X + Cl
2
o Y + HCl
Y + OH
-
o Z + Cl
-


Sabendo que temos, no necessariamente nesta ordem,
duas substituies e uma trimerizao, assinale a(s)
proposio(es) correta(s).


01. = X


02. HC{CC=CC{CH = Y
| |
H H


04. = Y
Cl

Cl Cl
| |
08. HC{CCCC{CH = X
| |
H H

16. HC{CCHCHC{CH = Z
| |
HO OH
OH


32. = Z


11. (UFSC) Assinale abaixo o(s) reagente(s) estritamente
necessrio(s) para, numa sequncia de etapas reacionais
envolvendo hidrogenao e oxidao, obter-se a
cicloexanona.

01.

02. H
2
, CrO
3

04. CH
3
COOH
08. CH
3
Br
16. CHBr
3

32. CH
3
CH
2
-OH
64. NaOH

12. (FUVEST-SP) A hidrogenao das duplas ligaes
de 1mol de benzeno produz:
a) meio mol de naftaleno.
b) 1 mol de fenol.
c) 1 mol de cicloexano.
d) 2 mols de ciclopropano.
e) 3 mols de etileno.


O
H
Please purchase PDF Split-Merge on www.verypdf.com to remove this watermark.
Incluso para a Vida Redao

Pr-Vestibular da UFSC
1
:Caro professor, este material foi confeccionado a fim
de suporte terico para as aulas de Produo textual.
Contudo sugerimos que os temas trabalhados em sala
de aula sejam atualizados ano a ano.

TEXTO 1

A REGREO DA REDASSO
Semana passada, recebi um telefonema de uma senhora
que me deixou surpreso. Pedia encarecidamente que
ensinasse seu filho a escrever.
- Mas, minha senhora - desculpei-me -, eu no sou
professor.
- Eu sei. Por isso mesmo. Os professores no tm
conseguido muito.
- A culpa no deles. A falha do ensino.
- Pode ser, mas gostaria que o senhor ensinasse o
menino. O senhor escreve muito bem.
- Obrigado - agradeci -, mas no acredite muito nisso.
No coloco vrgulas e nunca sei onde botar os acentos.
A senhora precisa ver o trabalho que dou ao revisor.
- No faz mal - insistiu -, o senhor vem e traz um
revisor.
- No d, minha senhora - tornei a me desculpar -, eu
no tenho o menor jeito com crianas.
- E quem falou em crianas? Meu filho tem 17 anos.
Comentei o fato com um professor, meu amigo, que me
respondeu: "Voc no deve se assustar, o estudante
brasileiro no sabe escrever". No dia seguinte, ouvi de
outro educador: "O estudante brasileiro no sabe
escrever". Depois li no jornal as declaraes de um
diretor da faculdade: "O estudante brasileiro escreve
muito mal". Impressionado, sa a procura de outros
educadores. Todos me disseram: acredite, o estudante
brasileiro no sabe escrever. Passei a observar e notei
que j no se escreve mais como antigamente.
Ningum mais faz dirio, ningum escreve em portas
de banheiros, em muros, em paredes.
No tenho visto nem aquelas inscries, geralmente
acompanhadas de um corao, feitas em casca de
rvore. Bem, verdade que no tenho visto nem
rvore.
- Quer dizer - disse a um amigo enquanto amos pela
rua - que o estudante brasileiro no sabe escrever? Isto
timo para mim. Pelo menos diminui a concorrncia
e me garante emprego por mais dez anos.
- Engano seu - disse ele. - A continuar assim, dentro de
cinco anos voc ter que mudar de profisso.
- Por qu? - espantei-me. - Quanto menos gente
sabendo escrever, mais chance eu tenho de sobreviver.
- E voc sabe por que essa gerao no sabe escrever?
- Sei l - dei com os ombros -, vai ver que porque no
pega direito no lpis.
- No senhor. No sabe escrever porque est perdendo
o hbito da leitura. E quando o perder completamente,
voc vai escrever para quem?
Ta um dado novo que eu no havia considerado.
Imediatamente pensei quais as utilidades que teria um
jornal no futuro: embrulhar carne? Ento vou trabalhar
num aougue. Serviria para fazer barquinhos, para
fazer fogueira nas arquibancadas do Maracan, para
forrar sapato furado ou para quebrar um galho em
banheiro de estrada? Imaginei-me com uns textos na
mo, correndo pelas ruas para oferecer s pessoas,
assim como quem oferece hoje bilhete de loteria:
- Por favor, amigo, leia - disse, puxando um cidado
pelo palet.
- No, obrigado. No estou interessado. Nos ltimos
cinco anos a nica coisa que leio a bula de remdio.
- E a senhorita no quer ler? - perguntei,
acompanhando os passos de uma universitria. A
senhorita vai gostar. um texto muito curioso.
- O senhor s tem escrito? Ento no quero. Por que o
senhor no grava o texto? Fica mais fcil ouvi-lo no
meu gravador.
- E o senhor, no est interessado nuns textos?
- sobre o qu? Ensina como ganhar dinheiro?
- E o senhor, vai? Leva trs e paga um.
- Deixa eu ver o tamanho - pediu ele.
Assustou-se com o tamanho do texto:
- O qu? Tudo isso? O senhor est pensando que sou
vagabundo? Que tenho tempo para ler tudo isso? No
d para resumir tudo em cinco linhas?
A seguir assim, logo o verbo escrever mudar sua
grafia para ex-crever.

(Carlos Eduardo Novaes, 1976 Texto Adaptado)
Considerando o texto acima, responda s questes a
seguir, prezando sempre pela clareza, pela coerncia,
pela coeso e pelo respeito s normas gramaticais.

Tarefa Mnima #

1. Explique a relao existente entre o ttulo e o
contedo do texto lido.

2. Qual foi a reao inicial do narrador ao se
conscientizar de que os alunos no escrevem como o
esperado?

3. Em determinada passagem do texto, foi explicado ao
narrador-personagem que no lhe era vantajoso os
alunos escreverem mal. Explique que prejuzo essa
deficincia dos estudantes traria ao dito personagem.

4. O que Carlos Eduardo Neves quis sugerir com o
termo ex-crever?

5. O texto que voc acaba de ler levanta uma
problemtica da atualidade. Produza um pargrafo
(entre 10 e 15 linhas), discutindo-a.

DENOTAO E CONOTAO
Uma palavra tomada no sentido denotativo quando
entendida no seu sentido literal, na sua significao
bsica, sem que se leve em conta o contexto em que
est inserida. Toma-se o sentido conotativo quando se
faz uma anlise contextual da palavra, admitindo-se a
associao valorativa a outros conceitos possveis.

FIGURAS DE LINGUAGEM

Podem ser classificadas em quatro tipos: figuras de
som, figuras de palavra, figuras de construo e figuras
Please purchase PDF Split-Merge on www.verypdf.com to remove this watermark.
Incluso para a Vida Redao

Pr-Vestibular da UFSC
2
de pensamento. A seguir, veremos as principais figuras
de cada um dos tipos:

ALITERAO a repetio ordenada de
consoantes de mesmo som, podendo sugerir, alm do
prprio jogo de som, a ideia de movimento.
Ex.: A brisa do Brasil beija e balana (Castro
Alves, O Navio Negreiro).

ONOMATOPEIA a imitao de determinado som
a partir da criao de uma palavra.
Ex.: Um forr de p-de-serra
Fogueira, milho e balo,
Um tum-tum-tum de pilo,
Um cabritinho que berra(...)
(Jessier Quirino, Paisagem de Interior)

METFORA Consiste na utilizao de uma palavra
para designar outra, com base em traos de
similaridade entre os seus conceitos. a realizao de
uma comparao implcita, sem o emprego de um
termo comparativo.
Ex.: O amor pedra no abismo, a meio passo
entre o mal e o bem (Zeca Baleiro, Cigarro)

CATACRESE o emprego de palavras com um
sentido diferente do real, mas cujo uso reiterado torna
imperceptvel o sentido figurado.
Ex.: O p da mesa, A perna da cala,
Embarcar no avio, etc.

METONMIA Assim como na metfora, emprega-
se um termo para designar outro, cujo conceito guarda
uma relao lgica com o termo empregado. Substitui-
se, por exemplo, o autor pela obra, a parte pelo todo, a
marca pelo produto, etc.
Ex.: Devolva o Neruda que voc me tomou, e
nunca leu (...)
(Chico Buarque, Trocando em Midos)
Aqui, refere-se ao autor Pablo Neruda em lugar da
obra.

ANTONOMSIA ou PERFRASE - a utilizao
de uma expresso caracterizadora para designar um
nome prprio.
Ex.: O poeta dos escravos, referindo-se a
Castro Alves;
A cidade maravilhosa, em lugar de Rio de
Janeiro.

SINESTESIA - uma derivao da metfora, que
ocorre quando numa mesma expresso misturam-se
sensaes percebidas por diferentes rgos de sentidos.
Ex.: Voc te uma voz macia (a voz percebida
pela audio e a maciez, pelo tato)
Sinto o cheiro doce da paixo (o cheiro
olfativo, enquanto o doce percebido pelo
paladar)

ELIPSE ocorre quando da omisso proposital de um
termo facilmente identificvel atravs do contexto ou
de elementos presentes na prpria orao.
Ex.: No jardim, flores secas e cores mortas (h
omisso de havia)
EUFEMISMO a suavizao de uma expresso por
meio da sua substituio por outra mais polida e sutil.
Ex.: Joo bateu as botas. (em vez de : Joo
morreu.)
Ela uma dama da noite. (em vez de: Ela
uma prostituta.)

HIPRBOLE o exagero enftico de uma ideia.
Ex.: Estou morrendo de sede.
Ele ganha rios de dinheiro.

PROSOPOPEIA ou PERSONIFICAO a
atribuio de predicados prprios dos seres vivos a
seres inanimados.
Ex.: A noite chora a sua ausncia.
Esta cama me convida ao sono.

ANTTESE a oposio de frases, oraes ou
palavras de sentido contrrio.
Ex.: Onde queres prazer sou o que di (...)
(Caetano Veloso, Quereres)
s velho na idade e jovem na alma.

IRONIA - o uso de um termo com a finalidade de
expressar o oposto do que este significa, dando um
efeito humorstico ou crtico mensagem.
Ex.: Moa linda,, bem tratada, trs sculos de
famlia, burra como uma porta: um amor
(Mrio de Andrade)

GRADAO a apresentao de ideias numa
sequncia ascendente (clmax) ou descendente
(anticlmax).
Ex.: Um corao chagado de desejos
Latejando, batendo, restrugindo.
(Vicente de Carvalho).

Exerccios de Sala #

1. A prosopopeia, figura que se observa no verso
"Sinto o canto da noite na boca do vento", ocorre em:
a) "A vida uma pera e uma grande pera."
b) "Ao cabo to bem chamado, por Cames, de
Tormentrio, os portugueses apelidaram-no de Boa
Esperana."
c) "Uma talhada de melancia, com seus alegres
caroos."
d) "Oh! eu quero viver, beber perfumes.
Na flor silvestre, que embalsama os ares."
e) "A felicidade como a pluma..."

2. Assinalar a alternativa que contm as figuras de
linguagem correspondentes aos perodos a seguir:
I. "Est provado, quem espera nunca alcana".
II. "Onde queres o lobo sou o irmo".
III. Ele foi discriminado por sofrer de uma doena
contagiosa muito falada atualmente.
IV. Ela quase morreu de tanto estudar para o vestibular.
a) ironia - anttese - eufemismo - hiprbole
b) eufemismo - ironia - hiprbole - anttese
c) anttese - hiprbole - ironia - eufemismo
d) hiprbole - eufemismo - anttese - ironia
e) ironia - hiprbole - eufemismo anttese
Please purchase PDF Split-Merge on www.verypdf.com to remove this watermark.
Incluso para a Vida Redao

Pr-Vestibular da UFSC
3
3. Assinale a alternativa em que a CONOTAO
esteja presente.
a) Diante da exploso da aniversariante, todos
engoliram o sorriso.
b) A mesa estava imunda e as mes enervadas com o
barulho que os filhos faziam.
c) O vendedor insistira muito e ela, sempre to tmida
quando a constrangiam, acabou por comprar as
rosas.
d) Quando recolheu do cho o caderno aberto, viu a
letra redonda e grada que era a sua.
e) Todas eram vaidosas e de pernas finas, com aqueles
colares falsificados e com as orelhas cheias de
brincos.

DEFEITOS E QUALIDADES DE UM TEXTO

Para que um texto seja considerado bem-sucedido,
necessrio prezar por:

clareza;
coerncia;
coeso;
correo gramatical;
elegncia.

Assim, sabe-se que os ditos defeitos de um texto
devem ser evitados! Os mais comuns so:

A. Ambiguidade
Uma frase ambgua aquela que apresenta mais de um
sentido.
Veja: Em poca de pleito* comum ouvirmos
candidatos dizendo ao povo que se preocupam com o
seu bem-estar.
*Ortografia/Semntica: pleito = eleio; preito =
homenagem
Analisando: bem-estar de quem? Do prprio
candidato? Do povo?
Percebe-se que o pronome possessivo (seu) no foi
bem-empregado.

B. Obscuridade
A obscuridade ainda pior que a ambiguidade, pois,
quando se ambguo, pode-se ter duas interpretaes,
ao passo que na obscuridade, muitas vezes, no
podemos nem mesmo imaginar do que se trata. uma
falta total de clareza. Normalmente a obscuridade
causada por frases longas, m pontuao, desrespeito
s normas gramaticais e linguagem rebuscada.
Analise um trecho retirado da Folha de So Paulo j h
algum tempo:
As videolocadoras de So Carlos esto escondendo
suas fitas de sexo explcito. A deciso atende a uma
portaria de Dezembro de 2001, do Juizado de Menores,
que probe que as casas de vdeo aluguem, exponham e
vendam fitas pornogrficas a menores de dezoito anos.
A portaria probe ainda os menores de dezoito anos de
irem a motis e rodeios sem a acompanhia ou
autorizao dos pais.
Fica a pergunta: ento, se estiverem acompanhados dos
pais, os menores podero ir a motis?


C. Pleonasmo
uma espcie de redundncia.
No Ensino Mdio estudamos que Pleonasmo faz parte
dos Vcios de Linguagem e nos so passados exemplos
de expresses que devemos evitar, tais como subir
para cima, descer para baixo, entrar para dentro e
sair para fora. Contudo no so apenas esses os
pleonasmos existentes. Veja outros casos: matinal da
manh, noturno noite, hemorragia de sangue, dois
irmos gmeos, ganhou de graa, recinto fechado,
certeza absoluta, grande maioria, dois iros gmeos e
outros casos.
Ateno: quando usado de maneira potica, o
Pleonasmo no constitui um erro.
Exemplos:
E rir meu riso e derramar meu pranto. (Vincius de
Moraes)
Os sonhos mais lindos, sonhei... (F. D. Marchetti e M. de
Feraudy, veerso de Armando Louzada).

H casos de Pleonasmo mais comuns do que se
imagina!
Acesso restrito somente a funcionrios do setor.
H cinco anos atrs(...).
Este produto de boa qualidade.
Isso um pequeno detalhe.
Estas so propriedades prprias do sdio.
Que surpresa inesperada!

D. Cacofonia
um som desagradvel (s vezes obsceno), resultante
da proximidade de determinadas slabas.
Por exemplo: l tinha; da vez passada; mande-me j; ,
dor horrvel!

E. Eco
a repetio de terminaes (mesmo som).
Observe: A deciso causou comoo na populao.
Corrigindo: A deciso fez com que o povo se
comovesse.

F. Prolixidade
A prolixidade o oposto da conciso. Consiste,
portanto, em utilizarmos mais palavras do que o
necessrio, tornando a leitura cansativa e de difcil
compreeenso.
O uso de cacaoetes lingusticos (expresses que no
acrescentam nada ao texto, as quais s falamos porque
estamos acostumados a ouvir) deve ser evitado.
Por exemplo, no devemos introduzir um texto com
expresses do tipo antes de mais nada ou
inicialmente, pois, se estamos iniciando, bvio que
inicialmente.
Outros casos: pelo contrrio, por outro lado, por sua
vez.
Na prolixidade (lembre: ser prolixo enrolar; no
ser objetivo), tambm encontramos o uso dos chaves.
Os chaves so frases ou expresses feitas, as quais
s empobrecem o texto.
Veja alguns exemplos: inflao galopante, caloroso
abrao, caixinha de surpresas, vitria esmagadora e
outros.

Please purchase PDF Split-Merge on www.verypdf.com to remove this watermark.
Incluso para a Vida Redao

Pr-Vestibular da UFSC
4
G. Incoerncia
Ser incoerente se contradizer. Por isso, tome muito
cuidado e releia mais de uma vez seu texto, analisando-
o, principalmente para que a concluso no destoe do
restante do texto.
O desrespeito s regras gramaticais pode causar
incoerncia. Portanto, tenha muita ateno com as
regras de concordncia, de regncia, de acentuao e
at mesmo com a ortografia.
Veja:
RETIFICAR = corrigir
RATIFICAR = confirmar
IR DE ENCONTRO = chocar-se; ideias contrrias
IR AO ENCONTRO = lado a lado; ideias afins,
semelhantes

Tarefa Mnima #

1. Leia com ateno o texto a seguir e identifique os
problemas existentes nas passagens numeradas:
A explorao do trabalho infantil, ela algo1
alarmante e intrigante2. Seu ndice tem diminudo3 nos
ltimos anos, talvez pelas medidas tomadas pelo
governo. E isso4 de responsabilidade do governo
federal e no do estadual. Na poca de nossos avs,
isso no acontecia4, pois viviam na tranquilidade do
interior. L tinha5 tudo que precisavam6. Os
governantes valorizavam o povo e mantinham seu
conforto7.

2. H uma ambiguidade no trecho:
a) Esse tipo de crime vem sendo praticado no mundo
inteiro.
b) H quem se sirva at mesmo de violncia na guerra
por bens materiais.
c) H filhos matando pais para tomarem o que lhes
pertence.
d) Basta andarmos uns minutos observando as atitudes
do ser humano e perceberemos do que ele capaz.
e) Por quererem mostrar que podem mais que os
outros, acabaram conquistando a antipatia alheia.

3. A expresso m qualidade constitui:
a) pleonasmo.
b) ambiguidade.
c) incoerncia.
d) obscuridade.
e) cacofonia.

4. Certo dia, houve, na Inglaterra, um jogo entre Brasil
e Argentina. Pouco antes do incio da partida, um
comentarista esportivo da Rede Globo afirmou: O
estdio um espetculo! Mas o gramado um tapete: a
bola rola com a maior perfeio.
a) Para fazer aluso qualidade do gramado, usou-se
uma metfora. Sublinhe-a.
b) Explique o sentido dessa metfora.
c) Aps responder s questes a e b, percebemos
que o comentarista no formulou bem seu
comentrio. Explique a incoerncia cometida.

5. Agora analise um trecho de uma fala de um
apresentador de telejornal, feita logo aps um
comentrio sobre a fauna brasileira:

(...) a caa e a extrao ilegais de palmito (...).
a) Tal passagem apresenta uma incoerncia. Qual?
b) Reescreva a fala do jornalista, de modo a torn-la
coerente.

6. Todas as frases listadas a seguir apresentam
ambiguidade. Indique quais so as interpretaes
possveis em cada caso:
a) O juiz declarou ter julgado o ru errado.
b) O piloto enjoado levantou voo.
c) Comprou um carro rpido.
d) Deixou a sala vazia.
e) Confessou os erros que cometeu com franqueza.
f) O jornal criticou a pea exibida com falta de talento.
g) Trata-se de um estudo a respeito de Machado de
Assis cuja leitura recomendo.
h) A professora deixou a turma entusiasmada.

Durante a sua carreira de goleiro, iniciada no
Comercial de Ribeiro Preto, na sua terra natal, Leo,
de 51 anos, sempre imps seu estilo ao mesmo tempo
arredio e disciplinado. Por outro lado, costumava ficar
horas aprimorando seus defeitos aps os treinos. Ao
chegar seleo brasileira em 1970, quando fez parte
do grupo que consquistou o tricampeonato mundial,
Leo no dava um passo em falso. Cada atitude e cada
declarao eram pensadas com um racionalismo tpico
de sua famlia, j que seus outros dois irmos,
Edmlson, 53 anos, e dson, 58, so mdicos.
Correio Popular, edio de 20/Out./2000.

7. O que aconteceria com Leo se ele, efetivamente,
ficasse aprimorando seus defeitos?

8. A expresso Por outro lado contribui para tornar o
trecho incoerente. Por qu?

9. A charge a seguir apresenta coerncia incontestvel.
O que tornou coerente a fala de Hagar?


10. Analise, agora, a segunda tira de Chris Browne: o
que torna o terceiro quadrinho incoerente em relao
ao primeiro?

Please purchase PDF Split-Merge on www.verypdf.com to remove this watermark.
Incluso para a Vida Redao

Pr-Vestibular da UFSC
5
Coeso textual
Quando conspiravam para derrubar a Monarquia, os
lderes republicanos j sabiam como seria a bandeira da
Repblica. Ela j estava desenhada, inspirada pelo
exterior, especificamente pela Frana. Seguia o
pensamento de Auguste Comte (1798-1857), que
defendia que cada sociedade deveria ter por divisa o
amor por princpio, a ordem por base e o progresso
como objetivo. As bandeiras nacionais deveriam ter a
inscrio: Amor, Ordem e Progresso. Na nossa
bandeira, no coube a palavra Amor. Ficaram apenas
Ordem e Progresso.
Mas muitas outras palavras ficaram ausentes.

No texto acima, os pronomes: ela e que retomam
expresses ditas anteriormente, so elementos
anafricos, responsveis, portanto pela coeso textual,
ou seja, por uma melhor organizao do texto.
Observe o prximo pargrafo:
O resultado de tudo isso que a populao brasileira
ficou mais velha. No entanto, no houve mudanas
significativas na qualidade de vida, na distribuio da
riqueza e, principalmente, das terras, que no Brasil
sempre foram o privilgio histrico da minoria. A
populao ficou mais velha e mais pobre tambm. A
distncia entre o ricos e pobres s fez aumentar nestas
dcadas e, at hoje, esse processo no foi revertido.
Aqui se cruzam a luta pela cidadania e a terceira
idade.
As palavras destacadas acima so anafricos, pois
retomam algo que foi dito antes. Entre elas, que e no
entanto so conjunes; principalmente, sempre,
tambm e aqui so advrbios.

Ateno! Esse ou este?

1) Para localizao de seres no espao usamos:
- esta, isto se nos referimos a algo prximo de
quem est falando. Como em: Pedro, esta minha saia
nova.
- essa, isso se nos referimos a algo prximo da
pessoa com quem se est falando. Como em: Pedro,
voc vai festa com essa gravata?

2) Para localizao no tempo usamos:
- este, esta, isto em relao a um espao de
tempo presente. Como em: Este ano quero paz/ No
meu corao/ Quem quiser ter um amigo/Que me d a
mo...
- esse em relao a um espao de tempo passado.
Como em: No ano passado, tive muitos contratempos,
foi um ano difcil; no gosto de me lembrar desse ano.

3) Nas cartas:
- este, esta referem-se ao local em que se encontra
o redator.
- esse, essa referem-se ao local em que se
encontra o destinatrio.
Exemplo: Esta capital, Rio de Janeiro, sediar os Jogos
Olmpicos; sentir-se- honrada em contar com os
atletas dessa cidade.


4) Nos textos tcnicos (dissertativos):
- isto um catafrico, refere-se a algo que ser dito.
Como em: No esquea isto: disciplina importante.
- esse um anafrico, refere-se a algo que j foi dito.
Como em: Disciplina importante; isso no devemos
esquecer.

Exerccios de Sala #

1. Leia, com ateno, a proposta de redao da
UNICAMP de 2006, o exemplo de redao abaixo da
mdia e a anlise da banca corretora.

PROPOSTA DE REDAO
Com o auxlio de elementos presentes na coletnea,
trabalhe sua dissertao a partir do seguinte recorte
temtico:

Diferentes so os meios de transporte, assim como as
polticas adotadas pelo Estado para viabiliz-los.
O Estado pode atuar de forma mais direta, por meio de
financiamentos, concesses, isenes e privilgios
fiscais, ou apenas exercer um papel regulador dos
diversos setores envolvidos.

Instrues

1) Discuta que meio(s) de transporte deve(m) ser
priorizado(s) para atender s necessidades da
realidade brasileira atual.
2) Trabalhe seus argumentos no sentido de explicitar
como esse(s) meio(s) pode(m) ser viabilizado(s) e
qual poderia ser o papel do Estado nesse processo.
3) Explore tais argumentos de modo a justificar seu
ponto de vista.

Tipos redacionais:

- Dissertao;
- Descrio;
- Narrao;
- Carta argumentativa;

TEXTO DESCRITIVO

A Descrio um retrato verbal. Ento para que esse
tipo de texto seja bem-sucedido, ele deve ser capaz de
fazer com que o leitor visualize o que ou em est sendo
descrito.
Analise:














Iracema saiu do banho: o alfajor
dgua ainda a roreja, como doce magaba
que corou em manh de chuva.
Please purchase PDF Split-Merge on www.verypdf.com to remove this watermark.
Incluso para a Vida Redao

Pr-Vestibular da UFSC
6
Produzir um texto exclusivamente descritivo no ser
uma boa opo, pois a leitura pode tornar-se cansativa.
Contudo excelente servir-se da descrio para
enriquecer outros textos: d veracidade ao exposto,
mas sempre se tomando muito cuidado a fim de no se
tornar prolixo.

DESCRIO OBJETIVA E DESCRIO
SUBJETIVA
Compare:

a) A casa pequena. Nela, h somente um dormitrio,
um banheiro, sala, cozinha e uma rea de servio. O
jardim, embora tambm seja pequeno, bonito e
repleto de flores.

b) A casa pequena. Nela, h somente meu
aconchegante cantinho de dormir, um banheiro, sala
para acolher meus amigos quando me presenteiam com
sua visita. H tambm o meu terror: a cozinha. nela
que fica o fantasma das colorias a geladeira! A rea
de servio o beco do trabalho! O jardim um
encanto. Lindo e colorido!

I - Descrio Objetiva
aquela em que o observador apresenta o tema-ncleo
de maneira impessoal, empregando a representao fiel
ao aspecto exterior.

I - Descrio Objetiva
aquela em que o observador apresenta o tema-ncleo
de maneira pessoal, empregando a imaginao e
externando suas impresses pessoais.

Observao:
Para que um texto descritivo tenha xito, uma boa dica
explorar a base sensorial.

a) Viso
A dona era uma idosa franzina, de cabelos mais
negros que a asa da grana. Vestia um pijama
desbotado, de seda japonesa. Tinha as unhas bem
curtas, recobertas por uma crosta de esmalte vermelho
escuro, descascado nas pontas.
(Lygia Fagundes Telles, As formigas)
b) Audio
De uma mesa distante, a nica ocupada ainda, vinha o
rudo de vozes de homens. Uma gargalhada sonora em
meio de vozes exaltadas. E a palavra cabrito saltou
dentre outras que se arrastavam pastosas. Num rdio da
vizinhana, ligado ao volume mximo, havia uma
cano que contava a histria de uma jovem a qual
vendia violetas na porta do teatro. A voz da cantora era
plana e um pouco fanhosa.
(Lygia Fagundes Teles, A ceia)
c) Olfato
L, os armazns tresandavam* a lixo e peixe podre, a
latas vazias de leo como cheiro de homens
esfarrapados.
(Autran Dourado, A barca dos homens)
*tresandar = 1. fazer andar para trs; 2. exalar (mau
cheiro); 3. cheirar mal.

d) Tato
O pai comprou o sapado dois nmeros maiores (...).
Enfiou o sapato branco, um rgido como s o couro
pode ser, no p frio e trmulo do garoto. Ao pentear o
loiro e sedoso cabelo do caula, a cabea ainda em
fogo. (Dalton Trevisan, Pedrinho)

e) Paladar
Deitado, ele beliscou dois ou trs gomos. Chupou o
sumo azedo, deixou cair a casca no prato. Apanhou
outro bago, desta vez mais doce.

TEXTO NARRATIVO
O texto narrativo tem sua base em fatos, aes as quais
fazem com que o enredo se desenvolva.
Para que a narrativa tenha sucesso,
imprescindvel que haja o conhecimento do tema e se
defina o enredo, para s ento escolher os personagens.
Por qu? Porque a escolha prvia dos personagens
um convite prolixidade, pois acabamos
preocupando-nos com a participao de cada um deles
e nos esquecemos de enriquecer nosso
desenvolvimento.

ELEMENTOS DA NARRATIVA
I Discurso
II Narrador
III Personagem
IV Tempo

PRODUO TEXTUAL

PROPOSTA 1
Na obra Inocncia, temos um narrador de 3 pessoa.
Mas como ficaria esse enredo sendo contado por um
dos personagens ou at mesmo pela protagonista?
Assim, produza um texto narrativo, sintetizando a dita
obra de Visconde Taunay, e fazendo com que algum
desses seja o narrador.
Mas sem esquecer: cada um tem seu ponto de vista, seu
modo de perceber os acontecimentos e de relat-los.

TEXTO DISSERTATIVO

RELAO DE SENTIDOS
- O encadeamento de ideias -

Uma boa forma de se buscar um encadeamento de
ideias capaz de deixar seu texto claro trabalhar com
premissas. Veja:

PREMISSA
Um dos meios mais simples de argumentar a premissa: a
apresentao de duas frases, uma das quais concluso da
outra.

INFERNCIAS
Quando observamos um fato, tiramos algumas
concluses (inferncias). Imaginemos, por exemplo,
que leiamos a seguinte manchete em um jornal: Brasil
importa automveis da Frana. Poderamos, ento,
tirar como possveis concluses:
1 o Brasil no est produzindo automveis em
nmero suficiente;
2 a economia brasileira est cada vez mais
dependente da estrangeira;
Please purchase PDF Split-Merge on www.verypdf.com to remove this watermark.
Incluso para a Vida Redao

Pr-Vestibular da UFSC
7
3 os automveis importados so os preferidos dos
brasileiros.
Contudo nem sempre as inferncias so
verdadeiras. Por isso necessrio analisar uma a uma
das levantadas antes de integr-las argumentao.

Para praticar:
1. Indique uma inferncia de carter positivo e outra de
negativo para cada uma das afirmaes manchetes a
seguir:
a) Florianpolis no acolher a prxima Copa.
b) Aulas de Ensino Religioso voltaro a ser
obrigatrias no Ensino Fundamental.
c) O Carnaval se aproxima.
d) As questes discursivas so cobradas em muitos
vestibulares.

A favor ou contra?
1. Em cada item h uma afirmao. Indique dois
argumentos capazes de defend-la e outros dois que se
oponham a ela:
a) As provas de mltipla escolha devem ser proibidas.
b) Mulheres j tm tantos cargos de chefia quanto os
homens.
c) Maiores de 15 anos j deveriam receber penas
semelhantes s destinadas a adultos.
d) Hoje est mais fcil comprar um carro.
e) Ministrio da Sade levanta a hiptese de alguns
medicamentos, hoje s vendidos com receita,
passarem a no necessitar de prescrio mdica.

CARTA ARGUMENTATIVA
Como o prprio nome j diz, a correspondncia em
que se apresenta uma argumentao.
Nela, no podem faltar os itens:
Local e data;
Saudao;
Despedida;
Assinatura (no caso de vestibular, falsa, a fim de que
no haja identificao do candidato).

OLHA A TAL CARTA A, GENTE!

PROPOSTA 3 (UFSC 2007)
A partir da leitura dos trechos de poemas transcritos
abaixo, o que voc escreveria ao presidente da
Organizao das Naes Unidas (ONU)?

POEMA A:
[...]
Mas oh no se esqueam
Da rosa da rosa
Da rosa de Hiroxima
A rosa hereditria
A rosa radioativa
Estpida e invlida
A rosa com cirrose
A anti-rosa atmica
Sem cor sem perfume
Sem rosa sem nada.
(MORAES, Vincius de. A Rosa de Hiroxima. In: Nova Antologia
Potica. So Paulo: Companhia das Letras, 2004).

POEMA B:
Ns merecemos a morte,
porque somos humanos,
e a guerra feita pelas nossas mos,
pela nossa cabea embrulhada em sculos de sombra,
por nosso sangue estranho e instvel, pelas ordens
que trazemos por dentro, e ficam sem explicao.
(MEIRELLES, Ceclia. Lamento do Oficial por seu Cavalo Morto.
In: Obra Potica. 1 ed. Rio de Janeiro: Aguilar, 1958 ).

POEMA C:
Este tempo de partido,
tempo de homens partidos.
[...]
O poeta
declina de toda responsabilidade
na marcha do mundo capitalista
e com suas palavras, intuies, smbolos e outras armas
promete ajudar
a destru-lo
como uma pedreira, uma floresta,
um verme.
(DRUMMOND DE ANDRADE, C. Nosso Tempo. In: A Rosa do
Povo. Rio de Janeiro: Record, 2004).

(UFSC 2009.1)
Observe o quadro Cena de famlia, do pintor paulista
Almeida Jnior (1850-1899).











(UFSC 2009.2)
A partir de sua compreenso do poema transcrito
abaixo, escreva uma carta a um amigo sobre seus
sentimentos, hoje, a respeito do pas em que voc
nasceu. Assine Fulano de Tal.

UMA CANO
Minha terra no tem palmeiras...
E em vez de um mero sabi,
Cantam aves invisveis
Nas palmeiras que no h.

Minha terra tem relgios,
Cada qual com sua hora
Nos mais diversos instantes...
Mas onde o instante de agora?

Mas onde a palavra onde?
Terra ingrata, ingrato filho,
Sob os cus da minha terra
Eu canto a Cano do Exlio!

QUINTANA, Mrio. Poesias.
Porto Alegre: Globo/SERS. 1962.

Proposio 3: Redija uma carta

Please purchase PDF Split-Merge on www.verypdf.com to remove this watermark.
Incluso para a Vida Tabela Peridica

Pr-Vestibular da UFSC

1

Please purchase PDF Split-Merge on www.verypdf.com to remove this watermark.

You might also like